instruction,input,output,text "This is a real-world medical entrance exam question, please give the true answer based on the question and selection. Topic name: None, Subject name: Physiology","Which of the following is not true for myelinated nerve fibers: Select and explain. A. Impulse through myelinated fibers is slower than non-myelinated fibers, B. Membrane currents are generated at nodes of Ranvier, C. Saltatory conduction of impulses is seen, D. Local anesthesia is effective only when the nerve is not covered by myelin sheath",A: Impulse through myelinated fibers is slower than non-myelinated fibers,"Answer: A: Impulse through myelinated fibers is slower than non-myelinated fibers Explanation: None" "This is a real-world medical entrance exam question, please give the true answer based on the question and selection. Topic name: None, Subject name: Physiology","Which of the following is not true about glomerular capillaries') Select and explain. A. The oncotic pressure of the fluid leaving the capillaries is less than that of fluid entering it, B. Glucose concentration in the capillaries is the same as that in glomerular filtrate, C. Constriction of afferent aeriole decreases the blood flow to the glomerulas, D. Hematocrit of the fluid leaving the capillaries is less than that of the fluid entering it",A: The oncotic pressure of the fluid leaving the capillaries is less than that of fluid entering it,"Answer: A: The oncotic pressure of the fluid leaving the capillaries is less than that of fluid entering it Explanation: Ans-a. The oncotic pressure of the fluid leaving the capillaries is less than that of fluid entering it Guyton I LpJ1 4-.;anong 23/e p653-6_)Glomerular oncotic pressure (due to plasma protein content) is higher than that of filtrate oncotic pressure in Bowman's capsule""Since glucose is freely filtered and the fluid in the Bowman's capsule is isotonic with plasma, the concentration of glucose in the filtrate is the same as in the capillaries" "This is a real-world medical entrance exam question, please give the true answer based on the question and selection. Topic name: None, Subject name: Medicine","A 29 yrs old woman with a pregnancy of 17 week has a 10 years old boy with down syndrome. She does not want another down syndrome kid; best advice to her is Select and explain. A. No test is required now as her age is below 35 years, B. Ultra sound at this point of time will definitely tell her that next baby will be down syndromic or not, C. Amniotic fluid samples plus chromosomal analysis will definitely tell her that next baby will be down syndromic or not, D. blood screening at this point of time will clear the exact picture",C: Amniotic fluid samples plus chromosomal analysis will definitely tell her that next baby will be down syndromic or not,"Answer: C: Amniotic fluid samples plus chromosomal analysis will definitely tell her that next baby will be down syndromic or not Explanation: None" "This is a real-world medical entrance exam question, please give the true answer based on the question and selection. Topic name: None, Subject name: Physiology","Axonal transport is: Select and explain. A. Antegrade, B. Retrograde, C. Antegrade and retrograde, D. None",C: Antegrade and retrograde,"Answer: C: Antegrade and retrograde Explanation: Fast anterograde (400 mm/day) transport occurs by kinesin molecular motor and retrograde transport (200 mm/day) occurs by dynein molecular motor." "This is a real-world medical entrance exam question, please give the true answer based on the question and selection. Topic name: None, Subject name: Biochemistry","Low insulin to glucagon ratio is seen in all of these except: Select and explain. A. Glycogen synthesis, B. Glycogen breakdown, C. Gluconeogenesis, D. Ketogenesis",A: Glycogen synthesis,"Answer: A: Glycogen synthesis Explanation: Answer- A. Glycogen synthesisLow insulin to glucagon ratio implies a catabolic state, i.e. usage of body stores to form energy, typically seen in fasting state ancl diabetes mellitus. Hence, this state will promote glycogen breakdown, gluconeogenesis as well as ketone body formation, while at the same time inhibiting glycogen synthesis and storage." "This is a real-world medical entrance exam question, please give the true answer based on the question and selection. Topic name: None, Subject name: Ophthalmology","Concentration of tropicamide: Select and explain. A. 0.01, B. 0.02, C. 0.03, D. 0.04",A: 0.01,"Answer: A: 0.01 Explanation: Answer- A. 0.01Tropicamide is the shoest acting mydriatic, concentration used is 0.5-l% drops (0.005-0.010).Tropicamide:Concentration used: 0.5-l% drops (0.005-0.010)Duration of action: Though effective for upto 3 hours, maximum effect appear 30 minutes after the last drop and lasts for only 10-15 minutesUsed in Adults." "This is a real-world medical entrance exam question, please give the true answer based on the question and selection. Topic name: None, Subject name: Medicine","Which of the following statements is true regarding H I N1 Influenza? Select and explain. A. Pregnant woman with sore throat can be staed immediately on oseltamivir without diagnostic testing under category B, B. People on long-term steroids cannot receive Os-eltam ivir, C. Category B concerns with low risk cases, D. Category B patients have to undergo immediate testing",A: Pregnant woman with sore throat can be staed immediately on oseltamivir without diagnostic testing under category B,"Answer: A: Pregnant woman with sore throat can be staed immediately on oseltamivir without diagnostic testing under category B Explanation: Ans: A. Pregnant woman with sore throat can be staed immediately on oseltamivir without diagnostic testing under category B(Ref. http..ilviww.molifir.nic.in/WriteReadData/1892s/ 804456402Categorisation.pcifi)Pregnant woman with sore throat can be staed immediately on oseltamivir without diagnostic testing under category B.Drug of choice - Oseltamivir (given for 10 days post-exposure). Priority groups (in order) for Influenza vaccinesPregnant womenHealthy young childrenAge >6 months with chronic medical conditionsHealthy adults 49-65 years15-49 years healthy young adultsHealthy adults > 65 years" "This is a real-world medical entrance exam question, please give the true answer based on the question and selection. Topic name: AIIMS 2017, Subject name: Anatomy","Which of the following are not a branch of external carotid Aery in Kiesselbach's plexus. Select and explain. A. Sphenopalatine aery, B. Anterior ethmoidal aery, C. Greater palatine aery, D. Septal branch of superior labial aery",B: Anterior ethmoidal aery,"Answer: B: Anterior ethmoidal aery Explanation: *Kiesselbach's plexus: Antero superior pa is supplied by ANTERIOR & POSTERIOR ETHMOIDAL AERIES which are branches of ophthalmic aery, branch of INTERNAL CAROTID AERY. Antero inferior pa is supplied by SUPERIOR LABIAL AERY - branch of facial aery, which is branch of EXTERNAL CAROTID AERY. Postero superior pa is supplied by SPHENO-PALATINE AERY - branch of MAXILLARY aery, which is branch of ECA. POSTERO INFERIOR pa is supplied by branches of GREATER PALATINE AERY - branch of ECA Antero inferior pa/vestibule of septum contain anastomosis b/w septal ramus of superior labial branch of facial aery & branches of sphenopalatine, greater palatine & anterior ethmoidal aeries. These form a large capillary network called KIESSELBACH'S PLEXUS If dryness persists, bleeding will occur Therefore, in given options, Anterior ethmoidal aery is a branch of ICA not ECA" "This is a real-world medical entrance exam question, please give the true answer based on the question and selection. Topic name: AIIMS 2017, Subject name: Medicine","Diagnosis of the following ECG- Select and explain. A. Ventricular bigeminy, B. Electrical alternans, C. P pulmonale, D. Left ventricular failure",B: Electrical alternans,"Answer: B: Electrical alternans Explanation: Option A- Broad QRS complex with normal sinus rhythm Digoxine toxicity Option B- P wave with fluctuating QRS complexes is seen. showing ELECTRICAL ALTERANS seen in large pericardial effusion in which hea is swinging. Option C- N Height in P wave: 2.5mm But in P-pulmonale: || by 1.5mm" "This is a real-world medical entrance exam question, please give the true answer based on the question and selection. Topic name: None, Subject name: Pediatrics","A blue new born presents with cyanosis. The X–ray chest reveal oligaemic lung field and normal sized heart. Most likely diagnosis is – Select and explain. A. Ebstein's anomaly, B. Pulmonary atresia, C. Transposition of great arteries, D. Tetralogy of fallot",B: Pulmonary atresia,"Answer: B: Pulmonary atresia Explanation: The findings in this newborn are Cyanosis at birth Oligaemic lung .fields Normal sized heart Lets see each option one by one Ebstein anomaly - It can be easily ruled out as there is marked cardiomegaly in it. Transposition of great vessels- There is cardiomegaly and plethoric lung fields see in it. So ruled out. Now the real confusion - between PA and TOF. TOF - in TOF there is oligaemic lung fields and normal sized heart but usually cyanosis is not present at birth.   Nelson states -  ""Often cyanosis is not present at birth. but with increasing hypertrophy of the right ventricular infimdibulum and patient growth, cyanosis occurs later in the 1st year of life. But some cases may present with cyanosis at birth."" a Pulmonary atresia - The child is cyanotic at birth There is pulmonary oligaemia But the heart is of variable size (from small to enlarge)" "This is a real-world medical entrance exam question, please give the true answer based on the question and selection. Topic name: None, Subject name: Pathology","27. The lateral spread of dental caries is facilitated mostly by the Select and explain. A. Enamel spindles, B. Dentinoenamel junction, C. Enamel lamellae, D. Striae of Retzius",B: Dentinoenamel junction,"Answer: B: Dentinoenamel junction Explanation: None" "This is a real-world medical entrance exam question, please give the true answer based on the question and selection. Topic name: None, Subject name: Medicine","A second-year PG resident tells you to perform an ABG of a patient. All of the following are true about performing an ABG except: Select and explain. A. Before performing the ABG, syringe should be loaded with 0.3 cc of heparin, B. Normal pH, HCO. and PCO, levels may not indicate absence of an acid-base imbalance, C. A different site should be tried i f modified Allen's test is negative, D. Radial aery is the preferred site","A: Before performing the ABG, syringe should be loaded with 0.3 cc of heparin","Answer: A: Before performing the ABG, syringe should be loaded with 0.3 cc of heparin Explanation: Ans: A. Before performing the ABG, syringe should be loaded with 0.3 cc of heparin(Ref: Harrison 18/e p364; http:// emedicine.medscape.comlaiclell 902703-overview).Care should be taken when measuring blood gases to obtain the aerial blood sample without using excessive heparin.Heparin should be expelled from the syringe after loading as it may lead to false pCO2 readings.Precautions:Most syringes come pre-packaged & contain a small amount of heparin, to prevent coagulation.Other syringes may need to be heparinized, by drawing up a small amount of liquid heparin & squiing it out again to remove air bubbles.Once the sample is obtained, care is taken to eliminate visible gas bubbles, as these bubbles can dissolve into the sample & cause inaccurate results." "This is a real-world medical entrance exam question, please give the true answer based on the question and selection. Topic name: None, Subject name: Physiology","Respiratory rhythm generation center is located at: Select and explain. A. Dorsal respiratory group, B. Pre-Botzinger complex, C. Ventral respiratory neurons, D. Pneumotaxic center",B: Pre-Botzinger complex,"Answer: B: Pre-Botzinger complex Explanation: Ans: B. Pre-Botzinger complexRef: Ganong 25Ie p656, 24Ie p658)Rhythmic respiration is initiated by a small group of synaptically coupled pacemaker cells in the pre-Botzinger complex (pre-BOTC) on either side of the medulla between the nucleus ambiguus and the lateral reticular nucleus.The main components of the respiratory control pattern generator responsible for automatic respiration are located in the medulla. Rhythmic respiration is initiated by a small group of synaptically coupled pacemaker cells in the pre-Botzinger complex (pre-BOTC) on either side of the medulla between the nucleus ambiguus and the lateral reticular nucleus." "This is a real-world medical entrance exam question, please give the true answer based on the question and selection. Topic name: None, Subject name: Medicine","Gait apraxia is seen in thromboembolic episode involving: Select and explain. A. ACA, B. MCA, C. PCA, D. Posterior choroidal aery",A: ACA,"Answer: A: ACA Explanation: Ans. a. ACA (Ref: Harrison 19/e p163, 18/e p193)Gait apraxia is seen in thromboembolic episode involving anterior cerebral aery.Frontal lobe is mainly supplied by anterior cerebral aery; its occlusion leads to gait apraxia" "This is a real-world medical entrance exam question, please give the true answer based on the question and selection. Topic name: None, Subject name: Dental","Heavy forces on periodontal ligament causes: Select and explain. A. Hyalinization, B. Osteoclastic activity around tooth, C. Osteoblastic activity around tooth, D. Crest bone resorption",A: Hyalinization,"Answer: A: Hyalinization Explanation: None" "This is a real-world medical entrance exam question, please give the true answer based on the question and selection. Topic name: None, Subject name: Microbiology","About N. gonorrhoea, all are true except Select and explain. A. Most common cause of urethritis in male, B. All stains are highly susceptible to penicillin, C. Wide spread axoregnic stains cause disseminated gonococcal infection, D. Found exclusively in human beings",B: All stains are highly susceptible to penicillin,"Answer: B: All stains are highly susceptible to penicillin Explanation: None" "This is a real-world medical entrance exam question, please give the true answer based on the question and selection. Topic name: None, Subject name: Anaesthesia","A patient who was on ventilator and being ventilated for past few days, suddenly pulls out the endotracheal tube. What is the next step of management? Select and explain. A. Assess the patient, give bag and mask ventilation and look for spontaneous breathing, B. Sta bag and mask ventilation and reintubate, C. Sedate and reintubate, D. Make him sit and do physiotherapy","A: Assess the patient, give bag and mask ventilation and look for spontaneous breathing","Answer: A: Assess the patient, give bag and mask ventilation and look for spontaneous breathing Explanation: Ans. a. Assess the patient, give bag and mask ventilation and look for spontaneous breathingIn self-extubation, assess the patient, give bag and mask ventilation and look for spontaneous breathing.Unplanned ExtubationUnplanned extubation of mechanically ventilated patients is relatively commonSelf-extubation refers to the patient's action, who deliberately removes the endotracheal tube (MC type of unplanned extubation, typically occur at night)Accidental extubation is attributed either to personnel's inappropriate manipulation of the tube during patient care or to a non-purposeful patient's action, e.g. coughing (mostly occur in the morning) Risk Factors for Unplanned ExtubationPatient factorsStaff factorsMaleDeliriumLight sedationDifficulty in securing tube (e g. facial swelling, facial burns)Previous unplanned extubationJunior staffNurse-to-patient ratioInadequately secured endotracheal tubeand/or checks" "This is a real-world medical entrance exam question, please give the true answer based on the question and selection. Topic name: None, Subject name: Dental","In a 6-month-old child, thick curd like white patch appears on the buccal mucosa. On rubbing it leaves an erythematous patch. Most likely diagnosis is: Select and explain. A. Tuberculosis, B. Lichen planus, C. Lupus erythematous, D. Candidiasis",D: Candidiasis,"Answer: D: Candidiasis Explanation: None" "This is a real-world medical entrance exam question, please give the true answer based on the question and selection. Topic name: AIIMS 2018, Subject name: Radiology","Characteristic of venous blood flow of lower limb in duplex Doppler is? Select and explain. A. Monophasic, B. Biphasic, C. Triphasic, D. Non phasic",A: Monophasic,"Answer: A: Monophasic Explanation: * Doppler is based on frequency shift and color coding depends on the direction of blood flow* Direction of blood flow:- Flow towards the probe- Red- Flow away from probe- Blue Intensity of color depends on velocity of flow Doppler effect: change in frequency because of relative motion between moving source of sound and observer. Direction of flow Color doppler showing CCA bifurcation Spectral doppler: Cursor is placed into the area of blood vessel we want to trace and its velocity is measured.An angle has to be maintained to obtain a proper image- Ideal angle = 0o (practically not possible)- Optimal angle = 45o-60o (never 90o) Spectral doppler of aeries Low resistance High resistance Example:* Brain aeries* ICA* CCA* VA- Renal aeries| Monophasic with pattern pulsatility Example: - Extremity aeries* ECA | Triphasic pattern Normal Peripheral Aerial Waveform Triphasic waveform Initial high velocity forward flow component Early diastolic reverse flow component Late diastolic forward flow component Narrow systolic window * Venous doppler:- It is Monophasic- during free breathing variations in amplitude due to cardiac and respiratory phasicity- In breath hold- pressure changes of . Atrium are reflected back in the major veins" "This is a real-world medical entrance exam question, please give the true answer based on the question and selection. Topic name: None, Subject name: Dental","The source of calcium ions of a calcific bridge in a tooth in which calcium hydroxide pulpotomy has been performed is: Select and explain. A. Blood vessel borne, B. Derided cells, C. Dentin, D. From the calcium hydroxide",A: Blood vessel borne,"Answer: A: Blood vessel borne Explanation: None" "This is a real-world medical entrance exam question, please give the true answer based on the question and selection. Topic name: None, Subject name: Biochemistry","2, 3-BPG binds to sites of haemoglobin and the affinity for oxygen Select and explain. A. 4, decreases, B. 1, decreases, C. 4, increases, D. 1, increases","B: 1, decreases","Answer: B: 1, decreases Explanation: Answer- B. 1, decreases2,3-BPG binds to 1 site of haemoglobin and decreases the affinity for oxygen.2, 3 Bisphosphoglycerate (2,3- BPG) or 2,3 Diphosphoglycerate 2,3- DPG)It is most abundant organic phosphate in RBC." "This is a real-world medical entrance exam question, please give the true answer based on the question and selection. Topic name: AIIMS 2017, Subject name: Anaesthesia","Highest concentration of oxygen is delivered through? Select and explain. A. Nasal cannula, B. Venturi mask, C. Bag and mask, D. Mask with reservoir",C: Bag and mask,"Answer: C: Bag and mask Explanation: Bag and mask-100% O2 Venturi mask- 60% O2 Nasal cannula - 40% O2 Mask with reservoir- 80-90% O2" "This is a real-world medical entrance exam question, please give the true answer based on the question and selection. Topic name: None, Subject name: Dental","Sharpest cusp is seen in Select and explain. A. Upper Canine, B. Lower Canine, C. Upper Premolar, D. Lower premolar",A: Upper Canine,"Answer: A: Upper Canine Explanation: None" "This is a real-world medical entrance exam question, please give the true answer based on the question and selection. Topic name: None, Subject name: Gynaecology & Obstetrics","A mother is Hbs Ag positive and anti Hbe Ag positive. Risk of transmission of Hep. B in child is: Select and explain. A. 20%, B. 50%, C. 0%, D. 90%",A: 20%,"Answer: A: 20% Explanation: Before answering these questions lets first discuss hepatitis in brief. Maternal infection: The acute infection is manifested by flu like illness as malaise, anorexia, nausea and vomiting. In majority, it remains asymptomatic. Jaundice is rare and fever is uncommon. Clinical course (HBV): Nearly 90–95% of patients clear the infection and have full recovery. 1% develop fulminant hepatitis resulting massive hepatic necrosis. 10-15% become chronic and 10% of these chronic cases suffer from chronic active hepatitis, cirrhosis and hepatocellular carcinoma. Diagnosis is confirmed by serological detection of HBsAg, HBeAg (denote high infectivity) and antibody to hepatitis B core antigen (HBcAg) and HBV DNA titer (107–1011). Screening: All pregnant women should be screened for HBV infection at first antenatal visit and it should be repeated during the third trimester for ‘high risk’ groups (intravenous drug abusers, sexual promiscuity, hemophilics, patients on hemodialysis or having multiple sex partners). Hepatitis C (HCV): It is recognized as the major cause of non-A, non-B hepatitis worldwide and is the leading cause of transfusion associated hepatitis. Transmission is mainly blood borne and to a lesser extent by faecal-oral route. It is responsible for chronic active hepatitis and hepatic failure. Perinatal transmission (10–40%) is high when coinfected with HIV and HBV. Detection is by antibody to HCV by EIA, which develops usually late in the infection. Confirmation is done by recombinant immunoblot assay (RIBA-3). Chronic carrier state is present. Breastfeeding is not contraindicated. Hepatitis D (HDV): It is seen in patients infected with HBV either as a co-infection or super infection. Perinatal transmission is known. Hepatitis E (HEV): Hepatitis E is the most important cause of non-A, non-B hepatitis in developing countries like India. Chronic carrier state is present. Perinatal transmission is uncommon. Maternal mortality is very high (15–20%)." "This is a real-world medical entrance exam question, please give the true answer based on the question and selection. Topic name: None, Subject name: Dental","In a patient with competent lips together at rest, the lip line is opposite the tips of the upper incisors. The lip line is then described as Select and explain. A. Average, B. High, C. Incomplete, D. Low",D: Low,"Answer: D: Low Explanation: None" "This is a real-world medical entrance exam question, please give the true answer based on the question and selection. Topic name: AIIMS 2019, Subject name: Pharmacology","A patient presented to emergency with overdose some drug. There were increased salivation and increased bronchial secretions. On examination, blood pressure was 88/60 mmHg. RBC esterase level is 50. What should be the treatment of this person? Select and explain. A. Neostigmine, B. Atropine, C. Flumazenil, D. Physostigmine",B: Atropine,"Answer: B: Atropine Explanation: Low esterase levels are suggestive of overdose of some drug which acts by inhibiting acetyl choline esterase like organophosphate poisoning. Inhibition of choline esterase results In increased amount of Ach at synapse, so poisoning or overdose results in cholinergic symptoms like salivation, lacrimation, urination, emesis, increased bronchial secretions, reduction in blood pressure.So treatment Of choice is Atropine. Atropine -It is highly effective in counter-acting the muscarinic symptoms, but higher doses are required to antagonize the central effects. It does not reverse peripheral muscular paralysis which is a nicotinic action. All cases of anti-ChE (carbamate or organophosphate) poisoning must be promptly given atropine2 mg i.v. repeated every 10 min till dryness of mouth or other signs of atropinization appear (upto 200 mg has been administered in a day). Continued treatment with maintenance doses may be required for 1-2 weeks. Neostigmine and physostigmine worsens the signs and symptoms as these also act by inhibiting choline esterase. Physostigmine and flumazenil are respectively used for Atropine and benzodiazepines poisoning." "This is a real-world medical entrance exam question, please give the true answer based on the question and selection. Topic name: None, Subject name: Dental","The current bar clasp design used is: Select and explain. A. Mesio-occlusal rest, B. Disto-occlusal rest, C. Mesial or distal depending on the situation, D. Choice of the dentist",A: Mesio-occlusal rest,"Answer: A: Mesio-occlusal rest Explanation: None" "This is a real-world medical entrance exam question, please give the true answer based on the question and selection. Topic name: None, Subject name: Dental","All of the following are true regarding Class-II inlay gingival margins preparation except? Select and explain. A. Removal of week enamel rods, B. Increased metal burnishability, C. Lap sliding fit of metal margin, D. Margins of restoration in self-cleansable area",D: Margins of restoration in self-cleansable area,"Answer: D: Margins of restoration in self-cleansable area Explanation: None" "This is a real-world medical entrance exam question, please give the true answer based on the question and selection. Topic name: None, Subject name: Dental","Breakdown of periodontal fibres in periodontitis is due to bacterial enzyme: (OR) Which of the following bacterial products have been implicated in initiation of inflammatory periodontal disease? Select and explain. A. Collagenase, B. Hyaluronidase, C. Coagulase, D. None of the above",A: Collagenase,"Answer: A: Collagenase Explanation: None" "This is a real-world medical entrance exam question, please give the true answer based on the question and selection. Topic name: None, Subject name: Social & Preventive Medicine","In a study, two groups of newborns are checked for their weights based on whether their mothers received food supplements or not. The appropriate test which can be used for comparing the data is: Select and explain. A. Chi square test, B. Paired T-test, C. Student's T-test, D. Fischer exact test",C: Student's T-test,"Answer: C: Student's T-test Explanation: Ans: C. Student's T-test(Ref Park 22/e p795; Biostatistics by Mahajan 7/e p134)Here, comparing a parametric quantitative variable (weight) in two unrelated group of people.Hence, ""Student's t-test"" used.Student t-test:Used when outcome variable is normally distributed in population (for quantitative data).Ex: BP, blood glucose." "This is a real-world medical entrance exam question, please give the true answer based on the question and selection. Topic name: AIIMS 2017, Subject name: Biochemistry","Which vitamin is required for glycogen Phosphorylase? Select and explain. A. PLP, B. TPP, C. Riboflavin, D. Lipoic acid",A: PLP,"Answer: A: PLP Explanation: Glycogen phosphorylase is the rate limiting enzyme of glycogenolysis. And it requires PLP. The active form of vitamin B6 is the coenzyme pyridoxal phosphate (PLP) PLP can be synthesized from the three compounds pyridoxine, pyridoxal and pyridoxamine. This PLP for this enzyme Glycogen phosphorylase is not required as co-enzyme, but it act as a phosphate donor. Enzyme glycogen phosphorylase will cut glycogen a (1-4) bond apa and the glucose released are transferred in Glucose-1-phosphate and that phosphate is taken from PLP." "This is a real-world medical entrance exam question, please give the true answer based on the question and selection. Topic name: None, Subject name: Dental","A child's behaviour is traced to have dental difficulties from his mother. The most satisfactory method of handling the situation is to: Select and explain. A. Introduce another child as a good example, B. Modify his fear by familiarization, C. Use small amounts of barbiturates, D. Use firmness",B: Modify his fear by familiarization,"Answer: B: Modify his fear by familiarization Explanation: None" "This is a real-world medical entrance exam question, please give the true answer based on the question and selection. Topic name: None, Subject name: Anatomy","Which of the following is the only complete cailage ring in the respiratory tree? Select and explain. A. Cricoid cailage, B. Thyroid cailage, C. Cunieform cailage, D. Epiglottis",A: Cricoid cailage,"Answer: A: Cricoid cailage Explanation: Answer- A (Cricoid cailage)(Ref: BDC 5/e Vol lII/p238)Complete cailage ring is seen in Cricoid cailage.Cricoid cailage is shaped like a ring. It encircles the larynx below the thyroid cailage. It is thicker and stronger than the thyroid cailage." "This is a real-world medical entrance exam question, please give the true answer based on the question and selection. Topic name: AIIMS 2019, Subject name: Anaesthesia","Steps of intubation - arrange in sequence:- a. Head extension and flexion of neck b. Introduction of laryngoscope c. Inflation of cuff d. Check breath sounds with stethoscope e. fixation of the tube to prevent dislodgement Select and explain. A. ABCDE, B. DBCEA, C. ACBED, D. CBAED",A: ABCDE,"Answer: A: ABCDE Explanation: For intubation: 1st step: Morning sniffing position: extension of atlanto-occipital joint and flexion of neck. 2nd step: Introduction of laryngoscope 3rd step: Push the tongue away, visualize the epiglottis, aryepiglottic fold, hinge the tip of laryngoscope with aryepiglottic fold and lift the laryngoscope to visualize the glottic opening. 4th step: Introduce ETT and inflate the cuff 5th check for bilateral breath sound FIXATION of the ETT." "This is a real-world medical entrance exam question, please give the true answer based on the question and selection. Topic name: None, Subject name: Pediatrics","In Erythroblastosis fetalis not involved is – Select and explain. A. Anti C, B. Anti D, C. Anti E, D. Anti Lewis",D: Anti Lewis,"Answer: D: Anti Lewis Explanation: Erythroblastosis fetalis is caused by the transplacental passage of maternal antibody active against paternal RBC antigens of the infant and is characterized by an increased rate of RBC destruction. Although more than 60 different RBC antigens are capable of eliciting an antibody response, significant disease is associated primarily D antigen of Rh group and with A 0 incompatibility. Other rare antigens involved are - C or E antigen of Rh group RBC antigens - Cw, Cx, Cu, K (kell), M, Duffy, S, P, MNS, Xg, Lutheran, Diego and Kidd. Anti-Lewis antibodies do not cause disease." "This is a real-world medical entrance exam question, please give the true answer based on the question and selection. Topic name: None, Subject name: Microbiology","A dentist suffered from Hepatitis B infection 3 months back. His laboratory tests are normal, but he is not allowed by the medical board to do surgical practice. He is: Select and explain. A. Inactive carrier, B. Healthy carrier, C. Convalescent carrier, D. Paradoxical carrier",C: Convalescent carrier,"Answer: C: Convalescent carrier Explanation: Convalescent carrier is the host who continues to harbor the microorganism even after recovering from the clinical disease caused by the same pathogen. Key Points  HBsAg: The antigen appears in blood during incubation period and is detectable in most patients during the prodrome and acute phase of the disease. Persistent presence of HBsAg in blood for at least 6 months indicates the carrier state and also indicates the risk of chronic hepatitis and hepatic carcinoma. It is not detectable in the serum during convalescent stage. The presence of HBsAg alone does not necessarily indicate the replication of complete virion, and the patients may not have symptoms of liver damage.  Parija SC. Textbook of Microbiology & Immunology. Elsevier Health Sciences; 2014. Page:74,553" "This is a real-world medical entrance exam question, please give the true answer based on the question and selection. Topic name: None, Subject name: Pediatrics","A pediatrician in a district hospital with specialization neonatal care unit calls an ophthalmologist for consultation for which of the following? Select and explain. A. A newborn with respiratory distress, B. A baby born at 28 weeks of gestation, C. Newborn with jaundice, D. A newborn with bih weight 2300 grams",B: A baby born at 28 weeks of gestation,"Answer: B: A baby born at 28 weeks of gestation Explanation: Answer- B. A baby born at 28 weeks of gestationPremature baby needs to be screenedfor Retinopathy of prematurity.All babies weighing <1500 gm or having a gestational period <32 weeks should be screened with indirect ophthalmoscopybetween 32-36 weeks postconception." "This is a real-world medical entrance exam question, please give the true answer based on the question and selection. Topic name: None, Subject name: Pediatrics","In a child with active liver failure, the most important prognosis factor for death is – Select and explain. A. Increasing transaminases, B. Increasing bilirubin, C. Increasing prothrombin time, D. Gram (–)ve sepsis",D: Gram (–)ve sepsis,"Answer: D: Gram (–)ve sepsis Explanation: Prognosis of hepatic failure in children Children with hepatic failure have somewhat better prognosis. The prognosis varies considerably with - i) Cause of liver failure ii) Stage of hepatic encephalopathy Poor prognostic factors for mortality If cause of liver failure is         —>       Wilson disease, idiopathic Stage IV coma —> Most common cause of death is brain stem herniation Age < I year Sepsis Severe hemorrhage Renal failure , Need for dialysis before transplantation For option b & c - ""Pretransplant serum bilirubin concentration and the INR (prothrombin time) of coagulation are not predictive of post transplant survival""." "This is a real-world medical entrance exam question, please give the true answer based on the question and selection. Topic name: None, Subject name: Gynaecology & Obstetrics","A patient went into shock immediately after normal delivery, likely cause: Select and explain. A. Amniotic fluid embolism, B. PPH, C. Uterine inversion, D. Eclampsia",C: Uterine inversion,"Answer: C: Uterine inversion Explanation: Friends this is one of those questions where we can derive the answer by excluding other options as very little information has been provided to us. Sudden post partum collapse – may be seen in all the four cases viz – amniotic fluid embolism, PPH, uterine inversion and eclampsia. But in case of PPH antecedent H/O excessive blood loss, in eclampsia – H/O antecedent convulsions and in amniotic fluid embolism – H/O abrupt onset of respiratory distress before collapse should be present, which is not given in the question so these options are being excluded. The clinical picture of acute inversion occurring in the third stage of labour is characterised by shock and haemorrhage, the shock being out of proportion to the bleeding. Since this a problem which occurs due to mismanaged third stage of labour, patient doesnot have any complain in the antenatal period or during labour. Uterine inversion – “It should be suspected whenever a woman has unexplained postpartum collapse.” Textbook of Obs, Sheila Balakrishnan, p 489" "This is a real-world medical entrance exam question, please give the true answer based on the question and selection. Topic name: None, Subject name: ENT","A patient presented with flat topped violaceous papules, thinning of nails and onycholysis. What is the microscopic finding observed in this patient? Select and explain. A. Prominent necrotic cell, B. Suprabasal split, C. Basal cell degeneration, D. Acantholysis",C: Basal cell degeneration,"Answer: C: Basal cell degeneration Explanation: Ans. c. Basal degeneration" "This is a real-world medical entrance exam question, please give the true answer based on the question and selection. Topic name: None, Subject name: Medicine","Which of the following is not. true regarding myelopathy? Select and explain. A. Sensory loss of facial area, B. Brisk jaw jerk, C. Brisk pectoral jerk, D. Urgency and incontinence of micturition",B: Brisk jaw jerk,"Answer: B: Brisk jaw jerk Explanation: Ans. b. Brisk jaw jerk(Ref: De Jongs Neurological examination/ p194, 201, 474.'Jaw jerk is exaggerated in supranuclear lesions that are above the mid pons." "This is a real-world medical entrance exam question, please give the true answer based on the question and selection. Topic name: None, Subject name: Social & Preventive Medicine","Which of the following committees has recommended a 3-year bachelor degree in medical education for rural health service? Select and explain. A. High level expe group of universal health program for india, B. Central bureau of health investigation, C. Srivastava committee, D. Sundar committee",A: High level expe group of universal health program for india,"Answer: A: High level expe group of universal health program for india Explanation: Answer- A. High level expe group of universal health program for indiaConcept of 3 years bachelor para-medical course for rural health service was proposed by Ministry of health and family welfare and received backing of Planning Commission of India's High level expe group of universal health program." "This is a real-world medical entrance exam question, please give the true answer based on the question and selection. Topic name: None, Subject name: Pediatrics","Sequential arrangement of fetal scans - Select and explain. A. Growth scan, B. Triple marker, C. Anomalous Scan and NT scan, D. All",D: All,"Answer: D: All Explanation: Ans: D. AllThe NT scan must be done between 11 and 14 weeks pregnant, because this is when the base of baby's neck is still transparent. (The last day for scan is 13 weeks and 6 days pregnant.)Triple marker test is performed in pregnant women at the end of first trimester and the beginning of the second trimester.The anomaly scan, also sometimes called the anatomy scan, 20 week ultrasound, or level 2 ultrasound, is a pregnancy ultrasound performed between 18-22 weeksA growth scan is an ultrasound scan that determines whether your baby's growth is normal. Doctors typically recommend it for women during the third trimester of pregnancy; one of the reasons it is also a fetal growth scan between 28 weeks and 32 weeks of pregnancy." "This is a real-world medical entrance exam question, please give the true answer based on the question and selection. Topic name: None, Subject name: Surgery","Which one of the following is a muscle splitting incision? Select and explain. A. Kocher's incision, B. Rutherford-Morrison incision, C. Pfannenstiel incision, D. Lanz incision",D: Lanz incision,"Answer: D: Lanz incision Explanation: Ans: D. Lanz incisionLanz incision:An oblique (transverse skin crease), muscle splitting incision used for an appendectomy.A modification of McBurney's (Grid Iron) incision.Advantages:Considered cosmetically better.Exposure is better.The extension is easier.Measurements:Incision, appropriate in length to size & obesity of patient - Approximately 2 cm below umbilicus centered on the midclavicular-mid inguinal line.It can be extended medially - With retraction or suitable division of rectus abdominis muscle." "This is a real-world medical entrance exam question, please give the true answer based on the question and selection. Topic name: None, Subject name: Dental","Multiple canals in mandibular premolars are seen in? Select and explain. A. Africas, B. Caucians, C. Not Recalled, D. Not Recalled",A: Africas,"Answer: A: Africas Explanation: None" "This is a real-world medical entrance exam question, please give the true answer based on the question and selection. Topic name: None, Subject name: Dental","Which of the following blade angle is appropriate for scaling and root planing Select and explain. A. A, B. B, C. C, D. D",B: B,"Answer: B: B Explanation: Blade angulation. (A) 0 degrees: correct angulation for blade insertion. (B) 45 to 90 degrees: correct angulation for scaling and root planing. (C) less than 45 degrees: incorrect angulation for scaling and root planing. (d) More than 90 degrees: incorrect angulation for scaling and root planing, but correct angulation for gingival curettage." "This is a real-world medical entrance exam question, please give the true answer based on the question and selection. Topic name: None, Subject name: Anatomy","Which pa of brachial plexus do not give branches Select and explain. A. Root, B. Division, C. Cord, D. Trunk",B: Division,"Answer: B: Division Explanation: Answer: B. DivisionDivision does not give branches" "This is a real-world medical entrance exam question, please give the true answer based on the question and selection. Topic name: None, Subject name: Pathology","If the Rb gene phosphorylation is defective, which of the following will happen? Select and explain. A. Cell cycle will stop at GI phase, B. Cell cycle will stop at G2 phase, C. The cell cycle will progress and the cell will divide, D. There will be no effect on cell cycle as for Rb gene phosphorylation is not needed",A: Cell cycle will stop at GI phase,"Answer: A: Cell cycle will stop at GI phase Explanation: Answer- A. Cell cycle will stop at GI phaseRb is a tumor suppressor gene. It normally arrests cell division at G1-S phase. Phosphorylation of Rb gene allows the cell to divide, hence inhibition of phosphorylation (which is the constitutive scenario for Rb gene) arrests the cell in GI phase.RB (Retinoblastoma) geneLocated on chromosome on 13q14Tumor suppressive pocket protein that binds E2F transcription factors in hypophophorylated stateKey negative regulator of G1/S cell cycle transition . Tumors associated: Retinoblastoma, osteosarcoma, Glioblastoma, small cell carcinoma of lung, CA breast & CA bladderSequence: Go- G1- S- G2 - M" "This is a real-world medical entrance exam question, please give the true answer based on the question and selection. Topic name: None, Subject name: Dental","Cumulative index is: Select and explain. A. Russet's periodontal index, B. Ramjford's periodontal index, C. PMA (Massler and Schlour), D. Gingival index (Loe and Silness)",C: PMA (Massler and Schlour),"Answer: C: PMA (Massler and Schlour) Explanation: None" "This is a real-world medical entrance exam question, please give the true answer based on the question and selection. Topic name: None, Subject name: Anatomy","The cells which will proliferate from top to bottom of villi are: Select and explain. A. Chief cells, B. Goblet cells, C. Paneth cells, D. Parietal cells",C: Paneth cells,"Answer: C: Paneth cells Explanation: Paneth cells or zymogen cells are found only in the deeper parts of the intestinal crypts.  They contain prominent eosinophilic secretory granules.  They also contain high amount of zinc.  They are known to produce lysozyme, defensins and TNF alpha that destroy bacteria.  They exceptionally migrate towards the base of the crypts of Lieberkuhn instead of going toward the villi" "This is a real-world medical entrance exam question, please give the true answer based on the question and selection. Topic name: None, Subject name: Ophthalmology","What is the usual weight of rabbit used in ophthalmological experiments? Select and explain. A. 0.5-1 kg, B. 1.5-2.5 kg, C. 5-7 kg, D. 10-12 kg",B: 1.5-2.5 kg,"Answer: B: 1.5-2.5 kg Explanation: Ans: B. 1.5-2.5 kg(Ref: Animal Models in Eve Research/ p188).The usual weight of rabbit used in ophthalmological experiments is between 1.5-2.5 Kg. Laboratory Animals:Laboratory AnimalsAnimalWeightRat180-200 gmGuinea Pig400-600 gmMouse20-25 gmRabbit1.5-2.5 KgdegHamster80-90 gm" "This is a real-world medical entrance exam question, please give the true answer based on the question and selection. Topic name: None, Subject name: Medicine","In plasmodium vivax malaria, relapse is caused by:September 2012 Select and explain. A. Sporozoite, B. Schizont, C. Hypnozoite, D. Gamteocyte",C: Hypnozoite,"Answer: C: Hypnozoite Explanation: Ans: C i.e. HypnoziteMalariaIn malaria, size of RBC is increased in: VivaxInfective agent of malaria is: SporozoiteFalciparum malariaGametocytes are seen in peripheral blood smearParasitemia is highestMost virulent plasmodium speciesExo-erythrocytic stage is absentMultiple infections of RBC'sSplenic rupture is common" "This is a real-world medical entrance exam question, please give the true answer based on the question and selection. Topic name: None, Subject name: Anatomy","Retraction of mandible is achieved by: Select and explain. A. Lateral pterygoid, B. Temporalis, C. Medial pterygoid, D. Masseter",B: Temporalis,"Answer: B: Temporalis Explanation: None" "This is a real-world medical entrance exam question, please give the true answer based on the question and selection. Topic name: AIIMS 2017, Subject name: Medicine","How is modified shock index represented as? Select and explain. A. HR/MAP, B. MAP/HR, C. HR/SBP, D. HR/DBP",A: HR/MAP,"Answer: A: HR/MAP Explanation: HR normal: 60-100 MAP normal: 95" "This is a real-world medical entrance exam question, please give the true answer based on the question and selection. Topic name: AIIMS 2019, Subject name: Pediatrics","A 3 week neonate with ambiguous genitalia presented with Na+ 127 meq/L, K+ 7.2 meq/L wit BP 52/24 mm Hg and he was managed with IV fluids. What is the next step of management? Select and explain. A. Spironolactone, B. Hydrocoisone administration, C. Broad spectrum antibiotics, D. Calcium gluconate",B: Hydrocoisone administration,"Answer: B: Hydrocoisone administration Explanation: Karyotype 46 XX Physical appearance - male pattern Precocious pubey at 2-4 yrs. of age. Uterus, cervix and fallopian tubes are present (bcz 44 XX) - Mullerian duct development is normal. Metabolic problems: Hyponatremia - aldosterone deficiency Hyperkalemia Hypotension Diagnosis: 1) USG Normal 2) Barr body present 3) Biochemistry - 17 hydroxyprogesterone || > 800 ngm/dl. Management: DOC - Dexamethasone - to pregnant women and fetus both having CAH. Hydrocoisone - pregnant women alone having CAH but fetus is normal. Neonate with CAH. Labioscrotal folds may be separated to folds may be fused at the midline, giving an appearance of a scrotum Labioscrotal folds with increased pigmentation suggest the possibility of increased coicotropin levels as pa of adrenogenital syndrome" "This is a real-world medical entrance exam question, please give the true answer based on the question and selection. Topic name: None, Subject name: Pathology","Base pairs in DNA Select and explain. A. 1.5 billion, B. 46 billions, C. 3.2 billion, D. 100 billion",C: 3.2 billion,"Answer: C: 3.2 billion Explanation: None" "This is a real-world medical entrance exam question, please give the true answer based on the question and selection. Topic name: None, Subject name: Dental","Which of the following is a solid solution? Select and explain. A. Amalgam, B. Cobalt-chromium, C. Silver-palladium, D. Gallium-silver",C: Silver-palladium,"Answer: C: Silver-palladium Explanation: The structure of noble alloys can consist of solid solutions, in which the elements are completely soluble in one another at all temperatures and compositions. The majority of noble metal casting alloys for dental restorations are based on solid solutions. An important example is the palladium-silver (Pd-Ag) alloy system. Silver forms a series of solid solutions with palladium and gold, and is therefore common in gold- and palladium-based dental alloys. Ref: Phillip’s 12th edition page 79. Craig’s 14th edition page 182,184" "This is a real-world medical entrance exam question, please give the true answer based on the question and selection. Topic name: None, Subject name: Anaesthesia","Tracheal secretions should be suctioned for: Select and explain. A. 10-15 seconds, B. 60 seconds, C. 30 seconds, D. 3 minutes",A: 10-15 seconds,"Answer: A: 10-15 seconds Explanation: Ans. a. 10-15 seconds(Ref Current DiMMOCIC and Treatment Critical Care 3/e p255)Tracheal secretions should he suctioned limiting the time to less than 10-15 seconds. The patient should be preoxygenated with 100% oxygen for at least a minute, and the total suction time should be limited to no more than 10-15 seconds on each attempt." "This is a real-world medical entrance exam question, please give the true answer based on the question and selection. Topic name: None, Subject name: ENT","A patient presents with carcinoma of the larynx involving the left false cords, left arytenoid and the left aryepiglottic folds with bilateral mobile true cords. Treatment of choice is - Select and explain. A. Vertical hemilaryngectomy, B. Horizontal partial hemilaryngectomy, C. Total laryngectomy, D. Radiotherapy followed by chemotherapy",A: Vertical hemilaryngectomy,"Answer: A: Vertical hemilaryngectomy Explanation: In the Patient Involvement of unilateral false cord, aryepiglottic folds and arytenoids with mobile cord suggest supraglottic cancer in T2 stage (morem than one subsites of supraglottis are involved). For T2 stage radiotherpy is best. But it is not given in options. Hence we will go for voice conserving surgery-vertical hemilaryngectomy. Vertical hemilaryngectomy means excision of one half of larynx, one half of supraglottis, glottis and subglottis." "This is a real-world medical entrance exam question, please give the true answer based on the question and selection. Topic name: None, Subject name: Biochemistry","Which of the following lipoproteins does not move towards charged end in electrophoresis? Select and explain. A. VLDL, B. LDL, C. HDL, D. Chylomicrons",D: Chylomicrons,"Answer: D: Chylomicrons Explanation: Based on electrophoretic separation from cathode to anode, the order of lipoprotein in an electrophoretogram is  Chylomicron LDL (β Lipoprotein) VLDL (Pre β Lipoprotein) IDL (Broad β Lipoprotein) HDL (α Lipoprotein)" "This is a real-world medical entrance exam question, please give the true answer based on the question and selection. Topic name: None, Subject name: Surgery","A 60 years old male presented with fever, chills and dysuria. Patient was hospitalized in emergency for 5 days. PSA level was 7.4. Next best step in this patient: Select and explain. A. Repeat PSA, B. TURP, C. TRUS guided biopsy, D. Antibiotics and admit",D: Antibiotics and admit,"Answer: D: Antibiotics and admit Explanation: Answer- D. Antibiotics and admitTreatment for acute bacterial prostatitis:MC used antibiotics are: TMP-SMX and Ciprofloxacin (Both are having better concentration in prostatic tissue)Around 4-6 weeks of antibiotic therapy is used to ave chronic bacterial prostatitis." "This is a real-world medical entrance exam question, please give the true answer based on the question and selection. Topic name: AIIMS 2018, Subject name: Physiology","Name the structure marked with arrow Select and explain. A. Mitochondria, B. Golgi bodies, C. Secretory vesicles, D. Rough Endoplasmic reticulum",C: Secretory vesicles,"Answer: C: Secretory vesicles Explanation: The structure marked with arrow- Secretory vesicles." "This is a real-world medical entrance exam question, please give the true answer based on the question and selection. Topic name: None, Subject name: Pharmacology","A patient from,nqh-eastern states was diagnosed to have infection with P. falciparum malaria. What is the most appropriate drug for this patient? Select and explain. A. Aemether plus lumefantrine, B. Sulfadoxine plus pyrimethamine, C. Chloroquine, D. Mefloquine",A: Aemether plus lumefantrine,"Answer: A: Aemether plus lumefantrine Explanation: Ans: A. Aemether plus lumefantrineRef: The ACT recommended in the National Program all over India except noheastern states is aesunate (AS) daily for3 days and Sulfadoxine-pyrimethamine (SP) on Day 0.Noheastern states presently recommended ACT in national drug policy is fixed dose combination of Aemether-lumefaritrine.Hence, the clear-cut answer of this question is Aemether plus lumefantrine." "This is a real-world medical entrance exam question, please give the true answer based on the question and selection. Topic name: None, Subject name: Biochemistry","Iron is present in all, EXCEPT Select and explain. A. Myoglobin, B. Cytochrome, C. Catalase, D. Pyruvate kinase",D: Pyruvate kinase,"Answer: D: Pyruvate kinase Explanation: None" "This is a real-world medical entrance exam question, please give the true answer based on the question and selection. Topic name: None, Subject name: Pathology","The most common bone tumor that occurs in children is: Select and explain. A. Osteosarcoma, B. Ewing's sarcoma, C. Metastatic carcinoma, D. Multiple myeloma",A: Osteosarcoma,"Answer: A: Osteosarcoma Explanation: None" "This is a real-world medical entrance exam question, please give the true answer based on the question and selection. Topic name: None, Subject name: Surgery","In a couple for treatment of infeility from the last four years, female paner is normal. Male paner has 0.8 ml semen volume per ejaculate on two repeated samples and absent fructose, with no sperms on examination under microscope. What is the next line of management? Select and explain. A. Per-rectal examination to check ejaculatory duct obstruction, B. Give antioxidants, C. Testicular biopsy, D. Transrectal ultrasound to detect duct obstruction",D: Transrectal ultrasound to detect duct obstruction,"Answer: D: Transrectal ultrasound to detect duct obstruction Explanation: Answer- D. Transrectal ultrasound to detect duct obstructionAbsent fructose with no sperms in ejaculate is suggestive of obstruction of vas deferens with seminal vesicle agenesis or obstruction. Next line of management in this patient would be transrectal ultrasound to detect duct obstruction.Transrectal Ultrasound-High-frequency (5-7) mHz transrectal ultrasound (TRUS) offers superb imaging of the prostate, seminal vesicles, and ejaculatory ducts.Due to both accuracy and convenience, TRUS has replaced surgical vasography in the diagnosis of obstructive lesions that cause infeility." "This is a real-world medical entrance exam question, please give the true answer based on the question and selection. Topic name: None, Subject name: Dental","The zygomatic bone does not articulate with: Select and explain. A. Frontal bone, B. Maxillary bone, C. Nasal bone, D. Temporal bone",C: Nasal bone,"Answer: C: Nasal bone Explanation: None" "This is a real-world medical entrance exam question, please give the true answer based on the question and selection. Topic name: None, Subject name: Gynaecology & Obstetrics","Which of the following statements is true regarding medical aboion? Select and explain. A. Ultrasound should be done in all cases, B. If the patient has an IUCD in-situ, it doesn't need to be removed, C. Can only be done up to 72 days, D. Only a person ceified under MTP act can perform medical termination of pregnancy",D: Only a person ceified under MTP act can perform medical termination of pregnancy,"Answer: D: Only a person ceified under MTP act can perform medical termination of pregnancy Explanation: Ans: D. Only a person ceified under MTP act can perform medical termination of pregnancy(Ref Shaw 16/e 28(, 15/e p244-245, William) 24/e p568).MTP can be performed up to 20 weeks according to MTP act.Ultrasound is not needed in all cases.Only ceified person for MTP act can perform medical termination of pregnancy." "This is a real-world medical entrance exam question, please give the true answer based on the question and selection. Topic name: None, Subject name: Physiology","Calcium ions triggers muscle contraction by binding to: Select and explain. A. Actin, B. Myosin, C. Troponin, D. Tropomyosin",C: Troponin,"Answer: C: Troponin Explanation: None" "This is a real-world medical entrance exam question, please give the true answer based on the question and selection. Topic name: AIIMS 2018, Subject name: Microbiology","SD plasma destroys lipid enveloped virus. On SD plasma transfusion, which of the following infection is the likely possibility? Select and explain. A. HIV, B. HAV, C. HBV, D. HCV",B: HAV,"Answer: B: HAV Explanation: Since SD plasma destroys lipid enveloped viruses the viruses which are not enveloped are most likely to cause infection. The impoant non enveloped viruses are viruses belonging to family PICORNAVIRIDAE, REOVIRIDAE, ASTROVIRIDAE and CALCIVIRIDAE and one of the impoant member of PICORNAVIRIDAE is hepatitis a virus which is a non enveloped RNA virus." "This is a real-world medical entrance exam question, please give the true answer based on the question and selection. Topic name: None, Subject name: Dental","According to the intelligence quotient classification, a score of 90-110 is: Select and explain. A. Low average., B. Average, C. Superior, D. Mentally retarded.",B: Average,"Answer: B: Average Explanation: None" "This is a real-world medical entrance exam question, please give the true answer based on the question and selection. Topic name: AIIMS 2018, Subject name: Biochemistry","Due to which of the following enzyme deficiency, vitamin C cannot be synthesised in humans? Select and explain. A. L-Glucuronic acid oxidase, B. L-Gulonic acid reductase, C. L-Gulonolactone oxidase, D. L-Gulonolactone reductase",C: L-Gulonolactone oxidase,"Answer: C: L-Gulonolactone oxidase Explanation: L-Gulonolactone oxidase produces Vitamin C Uronic acid pathway, but only in plants & most animals. Humans cannot make vitamin C due to absence of L-Gulonolactone oxidase enzyme.(See fig) Uronic acid pathway- glucose is conveed to glucuronic acid, pentoses and, in some animals, to ascorbic acid (not in man). ADDITIONAL EDGE: Uronic Acid Pathway synthesize Glucuronic Acid, Pentoses and Vit C Uses of Glucuronic acid: Incorporated into proteoglycans (Glucuronate used) Acts as a conjugating agent (Phase II conjugation reactions like bilirubin conjugation)" "This is a real-world medical entrance exam question, please give the true answer based on the question and selection. Topic name: None, Subject name: Medicine","In a patient of heart disease antibiotic prophylaxis for dental extraction is: Select and explain. A. Amoxicillin., B. Imipenem., C. Gentamicin., D. Erythromycin.",A: Amoxicillin.,"Answer: A: Amoxicillin. Explanation: None" "This is a real-world medical entrance exam question, please give the true answer based on the question and selection. Topic name: None, Subject name: Dental","Which of the following type of failure may occur, if the coefficient of thermal expansion of ceramic is much lower than that of metal? Select and explain. A. Cohesive failure of ceramic, B. Adhesive failure of metal ceramic bond, C. Cohesive failure of metal, D. Cohesive failure of metal ceramic bond",B: Adhesive failure of metal ceramic bond,"Answer: B: Adhesive failure of metal ceramic bond Explanation: A thermal contraction mismatch that results from a higher coefficient of contraction of porcelain (αP is greater than αM) will introduce residual tangential and hoop tensile stresses. The additive effect of tangential tensile stress induced in the porcelain by the intraoral force can exceed the tensile strength of the porcelain and causes crack propagation in the porcelain veneer. When the contraction coefficient of the porcelain is much lower than that of the metal (αP much lower than αM), porcelain cracking or metal-ceramic bond failure can occur near the metal-porcelain interface. This incompatibility failure is likely caused by the development of radial tensile stresses that exceed the tensile strength of porcelain. Contraction coefficient of the porcelain - αP   Contraction coefficient of the metal - αM   Phillip’s 12th edition page 423" "This is a real-world medical entrance exam question, please give the true answer based on the question and selection. Topic name: None, Subject name: Dental","Anti monsoon curve seen in : Select and explain. A. Premolar, B. Molar, C. Anterior, D. All of the above.",D: All of the above.,"Answer: D: All of the above. Explanation: None" "This is a real-world medical entrance exam question, please give the true answer based on the question and selection. Topic name: None, Subject name: Pediatrics","A 10 years old child has lytic lesions in upper femur, the differential diagnosis can be all except : Select and explain. A. Plasmacytoma, B. Browns tumour, C. Metastasis, D. Histiocytosis",A: Plasmacytoma,"Answer: A: Plasmacytoma Explanation: Plasmocytoma can be easily ruled out. It is a localized form of multiple myeloma which usually occurs in 5th to 7th decade. Metastasis, histiocytosis and brown tumor can cause lytic bone lesions in childhood." "This is a real-world medical entrance exam question, please give the true answer based on the question and selection. Topic name: None, Subject name: Dental","During periradicular surgery bone is removed with? Select and explain. A. Round burr, B. Double inverted cone burr, C. Fissure burr, D. Taper Burr",A: Round burr,"Answer: A: Round burr Explanation: None" "This is a real-world medical entrance exam question, please give the true answer based on the question and selection. Topic name: None, Subject name: Dental","The primary function of the dental pulp is: Select and explain. A. Nutritive, B. Production of dentin, C. Production of enamel, D. Vascular supply to the teeth",B: Production of dentin,"Answer: B: Production of dentin Explanation: None" "This is a real-world medical entrance exam question, please give the true answer based on the question and selection. Topic name: None, Subject name: Gynaecology & Obstetrics","Which of the following increases callus formation: Select and explain. A. Rigid immobilization, B. Movement at fracture site, C. Compression plating, D. Intraosseous nailing",B: Movement at fracture site,"Answer: B: Movement at fracture site Explanation: Ans: B. Movement at fracture site (Ref Apley 9/e p689)Micro movements at fracture site encourages vascular proliferation -Increases callus formation." "This is a real-world medical entrance exam question, please give the true answer based on the question and selection. Topic name: None, Subject name: Pediatrics","Child of Vasanthi was weaned from breast milk on the 5th day and was given sugarcane juice the child developed hypoglycemia and hepatomegaly biochemical examination showed hypophosphatemia and enzyme deficiencies–reducing substances in urine. The child is probably suffering from which of the following enzyme deficiencies – Select and explain. A. Fructokinase, B. Aldolase B, C. Glucose 6 Phosphatase, D. Beta galactosidase",B: Aldolase B,"Answer: B: Aldolase B Explanation: HEREDITARY FRUCTOSE INTOLERANCE Symptoms of hypoglycemia, hepatomegaly develping after injection of excessive fructose intake (sugar cane juice contains the major dietary source of fructose-Sucrose suggest the diagnosis of hereditary fructose intolerance. Clinical disorders related to fructose metabolism can result from excessive fructose consumption that exceed's the body's ability to efficiently convert the sugar into metabolic intermediates. In normal persons fructose is metabolized in the following way" "This is a real-world medical entrance exam question, please give the true answer based on the question and selection. Topic name: None, Subject name: Physiology","Skeletal muscles Select and explain. A. Contracts when calcium is taken up by sarcoplasmic reticulum, B. Contracts when actin and myosin filaments shorten, C. Contraction is initiated by calcium binding to troponin, D. Contraction is initiated by calcium binding to tropomyosin",C: Contraction is initiated by calcium binding to troponin,"Answer: C: Contraction is initiated by calcium binding to troponin Explanation: None" "This is a real-world medical entrance exam question, please give the true answer based on the question and selection. Topic name: None, Subject name: Biochemistry","Which of the following is most effective for gluconeogenesis in starvation? Select and explain. A. Acetyl Co-A stimulation of pyruvate carboxylase, B. Fructose-1, 6-biphosphate stimulation of phosphofructokinase-1, C. Citrate stimulation of acetyl carboxylase, D. Fructose-2, 6-biphosphate stimulation of phosphofructokinase-2",A: Acetyl Co-A stimulation of pyruvate carboxylase,"Answer: A: Acetyl Co-A stimulation of pyruvate carboxylase Explanation: Ans: A(Ref: Harper 30/c 1)1880)Acetyl Co-A stimulation of pyruvate carboxylase is most effective for gluconeogenesis.In Gluconeogenesis, pyruvate carboxylase catalyzes oxaloacetate synthesis from pyruvate." "This is a real-world medical entrance exam question, please give the true answer based on the question and selection. Topic name: AIIMS 2017, Subject name: Biochemistry","Which is not involved in iron metabolism? Select and explain. A. Transthyretin, B. Ceruloplasmin, C. Hepcidin, D. Ferropoin",A: Transthyretin,"Answer: A: Transthyretin Explanation: Transferrin is used for transpoing iron throughout the body BUT Transthyretin (Option a) - transpos Thyroxine & Retinol binding protein. So, it is NOT involved in iron metabolism WITH RESPECT TO OTHER OPTIONS: (Option b) Ceruloplasmin is a Cu containing enzyme also having Ferroxidase activity. Ferroxidase conves Fe +2 to Fe +3 (i.e. Ferrous to ferric). Ferric form of iron is required for transpo by transferrin in plasma. (Option c) Hepcidin regulates iron transpo in circulation. (Option d) Ferritin & Haemosiderin are proteins for iron storage. Haemosiderin has higher iron content than Ferritin." "This is a real-world medical entrance exam question, please give the true answer based on the question and selection. Topic name: None, Subject name: Gynaecology & Obstetrics","A 14 year girl presented with absent thelarche.On examination uterus was present. Investigations showed high FSH. karyotype is XY.What is the probable diagnosis? Select and explain. A. Gonadal dysgenesis, B. Kallman syndrome, C. Androgen insensitivity syndrome, D. Adrenal hyperplasia",A: Gonadal dysgenesis,"Answer: A: Gonadal dysgenesis Explanation: Ans. A. Gonadal dysgenesisRef: Clinical Gynecologic Endocrinoktgt Infeility, 8"" ed.Gonadal dysgenesis (Swyer Syndrome):Uncommon form of gonadal dysgenesis, characterized by a 46,XY karyotype." "This is a real-world medical entrance exam question, please give the true answer based on the question and selection. Topic name: None, Subject name: Dental","Reciprocal arm taper in Select and explain. A. 1 dimension, B. 2 dimension, C. 3 dimension, D. Not tapered",A: 1 dimension,"Answer: A: 1 dimension Explanation: None" "This is a real-world medical entrance exam question, please give the true answer based on the question and selection. Topic name: None, Subject name: Dental","Bluegrass appliance is used to treat: Select and explain. A. Thumb sucking., B. Tongue thrusting., C. Both, D. Mouth breathing.",C: Both,"Answer: C: Both Explanation: None" "This is a real-world medical entrance exam question, please give the true answer based on the question and selection. Topic name: None, Subject name: Dental","On a primary 2nd molar caries occur most commonly on Select and explain. A. Occlusal pit and fissures, B. Proximal caries below contact point, C. Buccal surface, D. Promixal caries above contact point",A: Occlusal pit and fissures,"Answer: A: Occlusal pit and fissures Explanation: None" "This is a real-world medical entrance exam question, please give the true answer based on the question and selection. Topic name: None, Subject name: Anatomy","Waldeyer's lymphatic chain is formed by all except Select and explain. A. Palatine tonsils, B. Pharyngeal tonsils, C. Tubal and lingual tonsils, D. Post auricular nodes",D: Post auricular nodes,"Answer: D: Post auricular nodes Explanation: None" "This is a real-world medical entrance exam question, please give the true answer based on the question and selection. Topic name: None, Subject name: Pathology","Polydactyly, craniosynostosis, Late closure of fontanelles is a feature of: Select and explain. A. Apert's syndrome, B. Crouton's syndrome, C. Pierre robin syndrome, D. Down' syndrome",A: Apert's syndrome,"Answer: A: Apert's syndrome Explanation: None" "This is a real-world medical entrance exam question, please give the true answer based on the question and selection. Topic name: None, Subject name: Surgery","Physiological changes seen in laparoscopy include all except: Select and explain. A. Increased ICP, B. Decreased FRC, C. Increased CVP, D. Increased pH",D: Increased pH,"Answer: D: Increased pH Explanation: Ans: D. Increased pH(Ref Bailey 27/e p87, 26/e p94,. http://www.laparoscopyhospitaLcom/physiological-changes!laparasconyhtm1)Metabolic acidosis (decrease pH) from CO, absorption is the primary derangement with laparoscopy.Laparoscopy:In laparoscopic surgeries, rigid endoscope introduced through a sleeve into peritoneal cavity.Needle used for pneumoperitoneum: Veress needle.Most commonly used gas: CO2.Physiological Effects of LaparoscopyCardiovascularIncreased intra-abdominal pressure a | CVP, | PCWP, | SVR & | MAP a | Preload &| afterload a decreasing cardiac output.PulmonaryCephalad shift of diaphragm decreases FRC, chest wall compliance and tidal volume increasing the work of breathing.RenalIncreased IAP decreases renal flow, decreasing GFR & reduced urine output.Raised pCO2 leads to RAAS stimulation. No long-term change in GFR/UO.GastrointestinalDecreased perfusion to intestines and stomach (as a result of increase IAP) decreases pHDecreased poal and hepatic flow leads to elevation of LFTs.Peripheral vascularIncidence of DVT, PE is generally lower post-laparoscopic procedures probably secondary to improved prophylaxisRisk is increased with longer procedures and reverse Trendelenberg position." "This is a real-world medical entrance exam question, please give the true answer based on the question and selection. Topic name: AIIMS 2017, Subject name: Anaesthesia","Which of the following nerve fibre types is least susceptible to LA blockade? Select and explain. A. A beta, B. A alpha, C. B fibers, D. C fibers",B: A alpha,"Answer: B: A alpha Explanation: Sensitivity of Local anesthetic To nerve fibers B > A 8 = C > Ag > Ab > Aa" "This is a real-world medical entrance exam question, please give the true answer based on the question and selection. Topic name: None, Subject name: Gynaecology & Obstetrics","A child born with multiple congenital defect including cleft palate, neural tube defect, atrial septal defect and microcephaly which of the following drug is used by mother during pregnancy: Select and explain. A. Erythromycin, B. Isotretinoin, C. Ibuprofen, D. Metronidazole",B: Isotretinoin,"Answer: B: Isotretinoin Explanation: Friends, Remember CNS anomaly (NTD) + CVS anomaly + facial defects are seen with the use of isotretinoin." "This is a real-world medical entrance exam question, please give the true answer based on the question and selection. Topic name: None, Subject name: Dental","Incorporation of peripheral seal in an impression is necessary to obtain; Select and explain. A. Stability, B. Functionally moulded periphery, C. Harmonious occlusion, D. Posterior palatal seal",B: Functionally moulded periphery,"Answer: B: Functionally moulded periphery Explanation: None" "This is a real-world medical entrance exam question, please give the true answer based on the question and selection. Topic name: None, Subject name: Pathology","Increase in height of mandible with increase in interdental spaces in elderly man Select and explain. A. Hyperpituitarism, B. Hyperthyroidism, C. Hypopituitarism, D. Hyperparathyroidism",A: Hyperpituitarism,"Answer: A: Hyperpituitarism Explanation: None" "This is a real-world medical entrance exam question, please give the true answer based on the question and selection. Topic name: None, Subject name: Pharmacology","A patient has seasonal allergic rhinitis and is on terfenadine. He develops cardiac arrhythmias after a course of antibiotics. Which of the following antibiotic could have been given: Select and explain. A. Ampicillin, B. Tetracycline, C. Erythromycin, D. Co-trimoxazole",C: Erythromycin,"Answer: C: Erythromycin Explanation: None" "This is a real-world medical entrance exam question, please give the true answer based on the question and selection. Topic name: None, Subject name: Dental","Modulus of elasticity means: Select and explain. A. Rigidity or stiffness of the material, B. Ability to be stretched with permanent deformation, C. Ductility of a material, D. Malleability of the metal",A: Rigidity or stiffness of the material,"Answer: A: Rigidity or stiffness of the material Explanation: ELASTIC MODULUS (YOUNG’S MODULUS OR MODULUS OF ELASTICITY) The word stiffness should come to mind upon reading one of these three terms in the dental literature. Elastic modulus describes the relative stiffness or rigidity of a material, which is measured by the slope of the elastic region of the stress-strain graph. Reference: PHILLIPS’ SCIENCE OF DENTAL MATERIALS, 12th ed page no 53" "This is a real-world medical entrance exam question, please give the true answer based on the question and selection. Topic name: None, Subject name: Pathology","Vwf factor is produced by: Select and explain. A. Platelets, B. Endothelial cells, C. Neutrophils, D. Monocytes",B: Endothelial cells,"Answer: B: Endothelial cells Explanation: Ans: B. Endothelial cellsVon willebrand factor (vWF)* This is the larger component.* It is produced by endothelial cells (main source) and megakaryocytes.It has two major functions ?* Helps in platelets adhesion by interacting with platelet membrane glycoprotein ib-IX.* Stabilizes factor VIllc." "This is a real-world medical entrance exam question, please give the true answer based on the question and selection. Topic name: None, Subject name: Forensic Medicine","Which of the following statement is false statement about snake-bites? Select and explain. A. Anti-venom is not effective in humpnosed pit viper bite, B. Cobra venom is neurotoxic, C. Atropine premedication should be used before administering Neostigmine, D. Neostigmine has a role in krait bite.",D: Neostigmine has a role in krait bite.,"Answer: D: Neostigmine has a role in krait bite. Explanation: Ans: D. Neostigmine has a role in krait bite.(Ref: Parikh 6/e p9.47: Harrison 19/e p2736; Snake Bite: Indian Guidelines and Protocol p 425)Role of neostigmine in snake-bite:An anticholinesterase.Paicularly effective in postsynaptic neurotoxins (cobra).Not useful against presynaptic neurotoxin (common krait & Russell's viper).Polyvalent Anti-snake venom - Ineffective against humpnosed pit viper (Hypnale)." "This is a real-world medical entrance exam question, please give the true answer based on the question and selection. Topic name: None, Subject name: Surgery","A prosthetic appliance given to cover the palatal defect in patients with cleft palate is called Select and explain. A. Artificial velum, B. Obturator, C. Removable prosthetic appliance, D. None of the above",B: Obturator,"Answer: B: Obturator Explanation: None" "This is a real-world medical entrance exam question, please give the true answer based on the question and selection. Topic name: None, Subject name: Microbiology","Biofilm forming bacteria causes antimicrobial resist-ance by all of the following except: Select and explain. A. Mechanical barrier, B. Increased excretion of antibiotics, C. Altered metabolism, D. Adherence",D: Adherence,"Answer: D: Adherence Explanation: Ans. d. AdherenceBiofilm in the bacteria leads antimicrobial resistance by acting as mechanical barrier, increased excretion of antibiotics and altered metabolism inside the biofilms.Growth in biofilms leads to altered microbial metabolism, production of extra cellular virulence factors, and decreased susceptibility to biocides, antimicrobial agents, and host defense molecules and cells. P aeruginosa growing on the bronchial mucosa during chronic infection, staphylococci and other pathogens growing on implanted medical devices, and dental pathogens growing on tooth surfaces to form plaque are several examples of microbial biofilm growth associated with human disease." "This is a real-world medical entrance exam question, please give the true answer based on the question and selection. Topic name: None, Subject name: Pharmacology","Which of these drugs is an antidote for fibrinolytic therapy? Select and explain. A. Epsilon aminocaproic acid, B. Protamine, C. Heparin, D. Streptokinase",A: Epsilon aminocaproic acid,"Answer: A: Epsilon aminocaproic acid Explanation: Answer- A. Epsilon aminocaproic acidEpsilon aminocaproic acid is an antidote for fibrinolytic therapy. Epsilon-aminocaproic qcid is a synthetic inhibitor of theplasmin-plasminogen system. It is the only potent antifibrinolylic agent, which is commercially available.""Aminocaproic acid is a lysine analog that competes for lysine binding sites on plasminogen and plasmin, blocking theinteraction of plasmin with fibrin.Fibrinolytic overdose - Epsilon Amino Caproic Acid (EACA)" "This is a real-world medical entrance exam question, please give the true answer based on the question and selection. Topic name: None, Subject name: Surgery","Which facial bone is most prone to fracture? Select and explain. A. Mandible, B. Nasal, C. Zygoma, D. Frontal",B: Nasal,"Answer: B: Nasal Explanation: None" "This is a real-world medical entrance exam question, please give the true answer based on the question and selection. Topic name: None, Subject name: Physiology","To check objective pain response, which is best used method Select and explain. A. Facial pain scale, B. Knee jerk reflex, C. H - Reflex, D. R III Reflex",A: Facial pain scale,"Answer: A: Facial pain scale Explanation: None" "This is a real-world medical entrance exam question, please give the true answer based on the question and selection. Topic name: None, Subject name: Pathology","A one year old child, who is earlier normal, is suffering from copious nasal discharge, coarse facial features, large tongue, bulging head and flattened nose. He is also having enlarged spleen and liver. What may be the cause? Select and explain. A. Hypothyroidism, B. Beckwith widman syndrome, C. Proteus syndrome, D. Hurler syndrome",D: Hurler syndrome,"Answer: D: Hurler syndrome Explanation: None" "This is a real-world medical entrance exam question, please give the true answer based on the question and selection. Topic name: None, Subject name: Dental","The resolution of IOPA is about: Select and explain. A. 10 Lp/mm, B. 16 Lp/mm, C. 28 Lp/mm, D. 30 Lp/mm",B: 16 Lp/mm,"Answer: B: 16 Lp/mm Explanation: The resolution of IOPA is 16 lines pair/millimeter." "This is a real-world medical entrance exam question, please give the true answer based on the question and selection. Topic name: None, Subject name: Medicine","A patient diagnosed with carcinoma of lung presented with a serum calcium level of 16.4 mmol/L. What will be the first step in management? Select and explain. A. IV fluids and furosemide, B. Immediate hemodialysis, C. Bisphosphonates, D. Chemotherapy with gemcitabine and carboplatin",A: IV fluids and furosemide,"Answer: A: IV fluids and furosemide Explanation: Ans: A. IV fluids and furosemide(Ref Harrison 19/e p314; 18/e p361)Hypercalcemia - Initial therapy:Begins with volume expansion to prevent dehydration.4-6 L of intravenous saline required over 1st 24 hr.Use of loop diuretics:Enhances sodium & calcium excretion.Treats underlying co-morbidities (e.g., congestive hea failure).Used only when volume is stable & normal." "This is a real-world medical entrance exam question, please give the true answer based on the question and selection. Topic name: None, Subject name: Gynaecology & Obstetrics","The following test may be abnormal in disseminated intravascular coagulation except: Select and explain. A. Prothrombin, B. Activated partial thromboplastin time, C. D-timer levels, D. Clot solubility.",D: Clot solubility.,"Answer: D: Clot solubility. Explanation: Laboratory findings in case of DIC are: The platelet count is low. Blood film shows the features of microangiopathic hemolytic anaemia. There is presence of schistocytes and fragmented red cells (helmet shaped) due to damage caused by trapping and passage through the fibrin thrombi. Prothrombin time, thrombin time and activated partial thromboplastin time, are all prolonged. Plasma fibrinogen levels are reduced due to consumption in microvascular coagulation. Fibrin degradation products (FDPs) are raised due to secondary fibrinolysis. D-dimer levels are raised in DIC. Clot observation test (Weiner)—It is an useful bed side test. It can be repeated at 2–4 hours intervals. 5 ml of venous blood is placed in a 15 ml dry test tube and kept at 37°C. Usually, blood clot forms within 6-12 minutes. This test provides a rough idea of blood fibrinogen level. If the clotting time is less than 6 minutes, fibrinogen level is more than 150 mg percent. If no clot forms within 30 minutes, the fibrinogen level is probably less than 100 mg percent." "This is a real-world medical entrance exam question, please give the true answer based on the question and selection. Topic name: None, Subject name: Pathology","In Xerostomia the salivary pH is: Select and explain. A. Unaffected, B. Low, C. High, D. increased in morning & decreases in day",B: Low,"Answer: B: Low Explanation: None" "This is a real-world medical entrance exam question, please give the true answer based on the question and selection. Topic name: None, Subject name: Dental","Which of the following is not true about stamp cusp? Select and explain. A. Maintain vertical height of face, B. Facial incline might contact in MI, C. Sharp cusps and prominent ridges are present, D. Not recalled",C: Sharp cusps and prominent ridges are present,"Answer: C: Sharp cusps and prominent ridges are present Explanation: None" "This is a real-world medical entrance exam question, please give the true answer based on the question and selection. Topic name: None, Subject name: Dental","Sugar restricted to diet was beneficial in presence of unfavorable hygiene was from which study? Select and explain. A. Hopewood, B. Experimental, C. Vipeholm, D. Turku",A: Hopewood,"Answer: A: Hopewood Explanation: HOPEWOOD HOUSE STUDY  (Sullivan and Harris- 1958, Harris- 1963 )  The dental status Of children between 7 to 14 years of age residing at Hopewood House, Bowral New South Wales, was studied longitudinally for 10 years. Almost all these children had lived from early infancy at Hopewood House.  All lived on a strictly natural diet, that, with the exception of on occasional serving of egg yolk, was entirely vegetable in nature and largely raw. The absence of meat and restriction of refined carbohydrate were the principal features of the Hopewood House diet.  The meals were supplemented by vitamin concentrates and an occasional serving of nuts and a sweetening agent such as honey. The fluoride content of water and food was insignificant and no tea was consumed.  TURKU SUGAR STUDY  (Scheinin and Makinen in 975)  The study was carried outin Turku, Finland.  Aim Of the study  To compare the cariogenicity of sucrose, fructose and Xylitol.  Basis of the study  XylitoI is a sweet substance not metabolized by plaque micro organisms. Key Concept: In Hopewood house study : the children's oral hygiene was poor, calculus is uncommon,  but  gingivitis  was  prevalent  in 75%  of  children. This work  showed  that,  in institutionalized  children  at  least,  dental caries  can  be  reduced  by  a  spartan  diet, without the beneficial effects of fluoride  and in the presence of unfavorable oral hygiene. Ref: Soben Peter , 4th ed page no 96" "This is a real-world medical entrance exam question, please give the true answer based on the question and selection. Topic name: None, Subject name: Dental","Alginate at 20 degrees, the gel is formed in Select and explain. A. 3-4 mins., B. 6-8 mins., C. 15-16 mins., D. 1-2 min.",A: 3-4 mins.,"Answer: A: 3-4 mins. Explanation: None" "This is a real-world medical entrance exam question, please give the true answer based on the question and selection. Topic name: None, Subject name: Ophthalmology","Most common tumor of lacrimal gland: Select and explain. A. Trans-scaphoid perilunate fracture, B. Scaphoid fracture, C. Distal radius fracture, D. Hamate fracture",D: Hamate fracture,"Answer: D: Hamate fracture Explanation: Ans. d. Hamate fractureMost common tumor of lacrimal gland is Non-Hodgkin's lymphoma (37%)> Pleomorphic adenoma (25%). Most commonmalignant epithelial tumor of the lacrimal gland is adenoid cystic carcinoma.Lacrimal Gland TumorMC tumor of lacrimal glandNon-Hodgkin's lymphomadegMC epithelial tumor of lacrimal glandPleomorphic adenomadegMC malignant epithelial tumor of lacrimal glandAdenoid cystic carcinomadeg" "This is a real-world medical entrance exam question, please give the true answer based on the question and selection. Topic name: None, Subject name: Dental","Percentage of carbon in carbon steel hand instruments is Select and explain. A. 1-1.2%, B. 10-12%, C. 0.6 to 0.8%, D. 6 to 8%",A: 1-1.2%,"Answer: A: 1-1.2% Explanation: MATERIALS USED FOR MANUFACTURING CUTTING INSTRUMENTS  Carbon Steel Carbon steel alloy contains 0.5 to 1.5 percent carbon in iron. Instruments made from carbon steel are known for their hardness and sharpness. Disadvantages with these instruments are their susceptibility to corrosion and fracture. They are of two types: 1. Soft steel: It contains <0.5% carbon 2. Hard steel: It contains 0.5 to 1.5% carbon Nisha Garg, Amit Garg. Textbook of Operative Dentistry.  Edition 3. Page: 80" "This is a real-world medical entrance exam question, please give the true answer based on the question and selection. Topic name: None, Subject name: Ophthalmology","Topical administration of I% Pilocarpine failed to produce pupillary constriction in a patient who had a large, dilated pupil What should be the most probable reason? Select and explain. A. Adie's tonic pupil, B. Diabetic III nerve palsy, C. Pharmacological blockade, D. Uncal herniation",C: Pharmacological blockade,"Answer: C: Pharmacological blockade Explanation: Ans. c. Pharmacological blockade (Ref-YANNOFF 3/E PG,1052-1055)Pilocarpine I% is a sufficient miotic dose for any eye, but a sphincter with all its cholinergic receptor blockade by atropine or tropicamide (i.e. eye with pharmacological blockade) does not constrict with pilocarpine 1%." "This is a real-world medical entrance exam question, please give the true answer based on the question and selection. Topic name: None, Subject name: Dental","Pregnancy gingivitis is caused by: Select and explain. A. Bacteriodes melaninogenicus, B. Actinobacillus actinomycetemcomitans, C. Streptococcus sanguis, D. Fusobacterium",A: Bacteriodes melaninogenicus,"Answer: A: Bacteriodes melaninogenicus Explanation: None" "This is a real-world medical entrance exam question, please give the true answer based on the question and selection. Topic name: None, Subject name: Anatomy","Artery palpable at the anterior border of the masseter is : Select and explain. A. Superficial temporal artery, B. Facial artery, C. Lingual artery, D. Maxillary artery",B: Facial artery,"Answer: B: Facial artery Explanation: Facial artery enters the face by winding around the base of the mandible, and by piercing the deep cervical fascia, at the anteroinferior angle of the masseter muscle. It can be palpated here and is called ‘anaesthetist’s artery’." "This is a real-world medical entrance exam question, please give the true answer based on the question and selection. Topic name: None, Subject name: Medicine","In DIC following are seen except Select and explain. A. Fibrinogen decreased, B. Thrombocytopenia, C. Normal APTT, D. PT elevation",C: Normal APTT,"Answer: C: Normal APTT Explanation: None" "This is a real-world medical entrance exam question, please give the true answer based on the question and selection. Topic name: None, Subject name: Pharmacology","Absorption of which of the following drugs is increased after a fatty meal? Select and explain. A. Amphotericin B, B. Griseofulvin, C. Ampicillin, D. Aspirin",B: Griseofulvin,"Answer: B: Griseofulvin Explanation: Ans: B. Griseofulvin (Ref: Goodman Gilman I2/e p1585; Katzung 13/e p632, 12/e p855,. KUT 7/e p790, 6/e p760)The oral administration of a 0.5 g dose of griseofulvin produces peak plasma concentrations of 1ug/mL in 4 hours.Improved drug absorption along with a fatty meal.Drug Absorption Reduced/Delayed by FoodDrug Absorption increased by foodAmpicillindegAspirindegAtenololdegAzithromycindegCaptoprildegCefaclorCephalexinCiprofloxacinDidanosineIndinavirdegIsoniazidaLoratidineNaficillinPenicillin G or VdegPhenobarbitaldegPhenytoindegRifampindegSucralfatedegTetracyclinedegDoxycyclinedegAtovaquonedegCarbamazepinedegChlohiazidedegCefuroximedegClofaziminedegDiazepamdegErythromycin GanciclovirdegHydrochlorothiazidedegItraconazoleaLithiumdegLovastatinMethylphenidateMetoprololdegNelfinavirNitrofurantoinPropranololdegPropoxypheneRitonavirdegSaquinavirdegSpironolactonedeg*Hydralazinedeg" "This is a real-world medical entrance exam question, please give the true answer based on the question and selection. Topic name: None, Subject name: Dental","What is the basis of behavior shaping: Select and explain. A. Stimulus response theory, B. Social learning principle., C. Hierarchy of needs, D. None of the above.",A: Stimulus response theory,"Answer: A: Stimulus response theory Explanation: None" "This is a real-world medical entrance exam question, please give the true answer based on the question and selection. Topic name: None, Subject name: Dental","Shortest root of maxillary 1st molar Select and explain. A. Distobuccal, B. Mesiobuccal, C. Palatal, D. All are equal",A: Distobuccal,"Answer: A: Distobuccal Explanation: Distobuccal < Mesiobuccal Most commonly capillary malformation. Bony capillary malformation and soft tissue hypertrophy. Venous abnormalities —> Deep venous system may be absent or hypoplastic. The anomaly present at birth and usually involves lower limb. When there is associated Arterio - venous malformation, it is known as Klippel - Trenaunay - Weber syndrome. Thick walled venous varicocities typically become apparent ipsilateral to the vascular malformation —> May be pulsatile because ofAV malformation." "This is a real-world medical entrance exam question, please give the true answer based on the question and selection. Topic name: None, Subject name: Dental","The number of dental caries cases reported during last 10 years are given below 250, 320, 190, 300, 5000, 100, 260, 350, 320 and 160. The epidemiologist wants to find out average number of caries cases reported in that town for last 10 yrs. The most appropriate average per this data will be Select and explain. A. Arithmetic mean, B. Mode, C. Median, D. Geometric mean",C: Median,"Answer: C: Median Explanation: None" "This is a real-world medical entrance exam question, please give the true answer based on the question and selection. Topic name: None, Subject name: Pathology","The most common bone cancer is: Select and explain. A. Osteosarcoma, B. Metastatic bone cancer, C. Multiple myelonia, D. Squamous cell carcinoma",B: Metastatic bone cancer,"Answer: B: Metastatic bone cancer Explanation: None" "This is a real-world medical entrance exam question, please give the true answer based on the question and selection. Topic name: None, Subject name: Biochemistry","Which is the ligand for receptors present in liver for uptake of LDL? Select and explain. A. Apo B-100, B. Apo B-48, C. Apo E and Apo B-100, D. Apo A-I",A: Apo B-100,"Answer: A: Apo B-100 Explanation: None" "This is a real-world medical entrance exam question, please give the true answer based on the question and selection. Topic name: None, Subject name: Pathology","Amyloid protein in Hemodialysis associated with amyloidosis is- Select and explain. A. AA, B. A b, C. b- 2 microglobulin, D. Transthyretin",C: b- 2 microglobulin,"Answer: C: b- 2 microglobulin Explanation: Answer-C. b- 2 microglobulinHemodialysis-associated amyloidosis-Associated disease is Chronic renal failureMajor fibril protein is A" "This is a real-world medical entrance exam question, please give the true answer based on the question and selection. Topic name: None, Subject name: Dental","Teeth that erupt in 30 days of birth Select and explain. A. Neonatal teeth, B. Natal teeth, C. Premature teeth, D. None",A: Neonatal teeth,"Answer: A: Neonatal teeth Explanation: None" "This is a real-world medical entrance exam question, please give the true answer based on the question and selection. Topic name: None, Subject name: Pediatrics","Arrange the following in the sequence of closure : Select and explain. A. Umbilical Aery, B. Umbilical Vein, C. Ductus venosus and Foramen ovale, D. All",D: All,"Answer: D: All Explanation: Ans. D. All" "This is a real-world medical entrance exam question, please give the true answer based on the question and selection. Topic name: None, Subject name: Pharmacology","Drug adveisement letter is a necessary component of each drug formulation and contains various information about the drug like drug dosing, frequency and half-life. Which of the following information need not be given in the drug adveisement letter? Select and explain. A. Research papers and other aicles proving efficacy of the drug, B. Date of expiry of the drug, C. Rare, but serious life threatening adverse-effects, D. Common, not so serious adverse-effects",B: Date of expiry of the drug,"Answer: B: Date of expiry of the drug Explanation: Ans: B. Date of expiry of the drug(Ref Goodman Gilman 12/e p1883; Manual of Experimental and Clinical Pharmacology/p 345).Date of drug expiry need not to be given in drug adveisement letter.Regulated according to The Federal Food, Drug, and Cosmetic Act.Amended (Food and Drug Administration Modernization Act of 1997).Permits use of print & television adveising for prescription drugs.Drug adveisements:Contain summary information relating to side effects, contraindications & effectiveness.Current adveising regulations:Specify that this information disclosure needs to include all risk information in a product's approved labeling or must direct consumers to healthcare professionals to obtain this information.Print adveisements:Include reprinting of risk-related sections of product's approved labeling (package inse).Television adveising:Not includes reprinting of risk-related.Note:Adveisements cannot be false or misleading or omit material facts.Must present fair balance between effectiveness & risk information." "This is a real-world medical entrance exam question, please give the true answer based on the question and selection. Topic name: None, Subject name: Surgery","In a patient with fresh blow out fracture of the orbit, best immediate management is Select and explain. A. Wait & watch, B. Antral pack, C. Titanium Mesh, D. Glass bead mesh",A: Wait & watch,"Answer: A: Wait & watch Explanation: None" "This is a real-world medical entrance exam question, please give the true answer based on the question and selection. Topic name: None, Subject name: Pharmacology","Which of the following drugs can be stopped abruptly without any withdrawal symptoms? Select and explain. A. Esctilaopram, B. Fluoxetine, C. Fluvoxamine, D. Seraline",B: Fluoxetine,"Answer: B: Fluoxetine Explanation: Answer- B. FluoxetineFluoxetine has the lowest incidence of discontinuation syndrome among several antidepressants including paroxetin and venlafaxine. The longer half life of fluoxetine will avoid any withdrawal symptoms because this medication effectively tapers itself from the patient's system over a few days." "This is a real-world medical entrance exam question, please give the true answer based on the question and selection. Topic name: None, Subject name: Ophthalmology","Keyhole-shaped visual field defect is seen in lesion involving which of the following regions? Select and explain. A. Optic disk, B. Optic chiasma, C. Lateral geniculate body, D. Occipital lobe",C: Lateral geniculate body,"Answer: C: Lateral geniculate body Explanation: Ans: C. Lateral geniculate body(Ref Walsh and Hoyt; Clinical Neuro-Ophthalmology 6/e p122)Key-hole shaped visual field defects are typically seen in the lesions involving lateral geniculate body but keyhole shaped defect (not visual field defect) is seen in the coloboma of Iris." "This is a real-world medical entrance exam question, please give the true answer based on the question and selection. Topic name: None, Subject name: Dental","The soft tissue tooth interface that forms after flap surgery in a previously denuded area is Select and explain. A. Long junctional epithelium, B. Collagen adhesion, C. Scar tissue attachment, D. Connective tissue attachment",A: Long junctional epithelium,"Answer: A: Long junctional epithelium Explanation: None" "This is a real-world medical entrance exam question, please give the true answer based on the question and selection. Topic name: None, Subject name: Dental","The test done for a statistically significant change in cholesterol values in a group of patients following an interventional programme is: Select and explain. A. Chi square test, B. Paired ""t"" test, C. Unpaired ""t"" test, D. Fisher test","B: Paired ""t"" test","Answer: B: Paired ""t"" test Explanation: None" "This is a real-world medical entrance exam question, please give the true answer based on the question and selection. Topic name: AIIMS 2019, Subject name: Microbiology","Mw vaccine is prepared from: Select and explain. A. Mycobacterium bovis, B. Mycobacterium indicus pranii, C. Mycobacterium welchii, D. Mycobacterium tuberculosis",B: Mycobacterium indicus pranii,"Answer: B: Mycobacterium indicus pranii Explanation: Mw vaccine is a killed leprosy vaccine developed in India in 2018 using Mycobacterium indicus pranii." "This is a real-world medical entrance exam question, please give the true answer based on the question and selection. Topic name: None, Subject name: Dental","Guiding planes are: Select and explain. A. Located adjacent to the edentulous area, B. Located far anterior to the edentulous area, C. Helps in the stability of the denture, D. Provides a different path of insertion and removal",A: Located adjacent to the edentulous area,"Answer: A: Located adjacent to the edentulous area Explanation: None" "This is a real-world medical entrance exam question, please give the true answer based on the question and selection. Topic name: None, Subject name: Dental","Reflex which never disappear: Select and explain. A. Walking and stepping reflex, B. Limb placement reflex, C. Parachute reflex, D. All of the above",D: All of the above,"Answer: D: All of the above Explanation: None" "This is a real-world medical entrance exam question, please give the true answer based on the question and selection. Topic name: None, Subject name: Pharmacology","Vitamin D toxicity is treated with Select and explain. A. Chloroquine, B. Hydroxycholoquine, C. Dexamethasone, D. Ketoconazole",C: Dexamethasone,"Answer: C: Dexamethasone Explanation: Ans. C. Dexamethasone* Dexamethasone is used in the treatment of hypercalcemia due to vitamin D toxicity. Glucocoicoid (Dexamethasone, prednisolone etc) therapy will decrease plasma calcium levels by blocking the action of vitamin D(1,25 D) which results in reduced intestinal calcium absorption and increased urinary excretion of calcium. * Other drugs like chloroquine and hydroxychloroquine are less effective in treating hypercalcemia due to vitamin D toxicity." "This is a real-world medical entrance exam question, please give the true answer based on the question and selection. Topic name: AIIMS 2019, Subject name: Surgery","Which of the following is not the component of qSOFA? Select and explain. A. Respiratory rate >22/min, B. Systolic BP<100 mmHg, C. Altered mental status, D. Unequally dilated pupils",D: Unequally dilated pupils,"Answer: D: Unequally dilated pupils Explanation: qSOFA: Quick - SOFA Sequential 0rgan Failure Assessment Score. QRS - MS Score*Mn: Q- Quick SofaR- RR > 22/min - 1S- SBP < 100 mmHg -1MS= Altered Mental Status. - 1 Score Moality0 - < 1%1 - 2-3%2 - > 10%" "This is a real-world medical entrance exam question, please give the true answer based on the question and selection. Topic name: None, Subject name: Surgery","How much hone can be stretched at a time before it fractures: Select and explain. A. 1%, B. 2%, C. 6%, D. 8%",B: 2%,"Answer: B: 2% Explanation: None" "This is a real-world medical entrance exam question, please give the true answer based on the question and selection. Topic name: None, Subject name: Anatomy","Tonsils developed from: Select and explain. A. Ventral part of 3rd pouch., B. Ventral part of 2nd pouch., C. Dorsal part of 2nd pouch., D. Ventral part of 3rd pouch.",B: Ventral part of 2nd pouch.,"Answer: B: Ventral part of 2nd pouch. Explanation: None" "This is a real-world medical entrance exam question, please give the true answer based on the question and selection. Topic name: None, Subject name: Dental","Gutta Percha can effectively be sterilized by? Select and explain. A. Hot salt sterilizer, B. Chemical solutions, C. Autoclaving, D. Dry heat",B: Chemical solutions,"Answer: B: Chemical solutions Explanation: None" "This is a real-world medical entrance exam question, please give the true answer based on the question and selection. Topic name: None, Subject name: Surgery","Which of these conditions does not require SABE prophylaxis Select and explain. A. MR, B. ASD, C. MS, D. CABG",D: CABG,"Answer: D: CABG Explanation: None" "This is a real-world medical entrance exam question, please give the true answer based on the question and selection. Topic name: None, Subject name: Anatomy","Motor supply of infrahyoid muscle is: Select and explain. A. Branches of cervical plexus, B. Vagus nerve, C. Glossopharyngeal nerve, D. Mandibular nerve",A: Branches of cervical plexus,"Answer: A: Branches of cervical plexus Explanation: None" "This is a real-world medical entrance exam question, please give the true answer based on the question and selection. Topic name: None, Subject name: Dental","This direction of force can cause: Select and explain. A. Reduce overbite, B. Worsen open bite, C. Both, D. None",C: Both,"Answer: C: Both Explanation: A labially directed force against a maxillary incisor (from a removable or fixed appliance) will tip the tooth and cause an apparent intrusion of the crown, which reduces the overbite (or makes anterior open bite worse)." "This is a real-world medical entrance exam question, please give the true answer based on the question and selection. Topic name: None, Subject name: Pediatrics","Cystinuria is characterised by – Select and explain. A. Generalised aminoaciduria, B. Systemic acidosis, C. Deposition of cystine crystals in Renal tubular cells, D. Recurrent urinary caliculi",D: Recurrent urinary caliculi,"Answer: D: Recurrent urinary caliculi Explanation: Cystinuria It is an autosomal recessive disorder. The disease is caused by a defective high affinity transporter for L-cystine and diabasic amioacids present in the proximal tubules. It is characterize by recurrent kidney stone formation." "This is a real-world medical entrance exam question, please give the true answer based on the question and selection. Topic name: None, Subject name: Medicine","Patient with PPF transfusion complaint of breathing difficulty, tachycardic, tachypnia, Had Batwing sign present in X-ray. What will be the possible reason? Select and explain. A. Transfusion related lung injury, B. Due to overload of PPF, C. Acute renal tubular acidosis, D. Hemoglobinuria",B: Due to overload of PPF,"Answer: B: Due to overload of PPF Explanation: Answer- B. Due to overload of PPFTransfusion associated circulatory overload (TACO) is an established, but grossly under diagnosed and underrepoed complication of blood transfusion." "This is a real-world medical entrance exam question, please give the true answer based on the question and selection. Topic name: None, Subject name: Surgery","Which of these is not a theory of TMJ ankylosis? Select and explain. A. Calcification in the soft tissues around the joint space, B. Synovial fluid attracting calcium ions, C. Lateral displacement of fracture condyle with fusion with zygomatic bone, D. Condylar burst",B: Synovial fluid attracting calcium ions,"Answer: B: Synovial fluid attracting calcium ions Explanation: None" "This is a real-world medical entrance exam question, please give the true answer based on the question and selection. Topic name: None, Subject name: Forensic Medicine","Fingerprinting (FINDER) involves recording prints of 8 fingers. Which finger pair is excluded? Select and explain. A. Ring finger, B. Thumb, C. Little finger, D. Middle finger",C: Little finger,"Answer: C: Little finger Explanation: Ans: C. Little finger(Ref: Reddy 34Ie p84, 33Ie p87; Parikh 6Ie p2.15).Fingerprint reader (FINDER):Computerized automatic fingerprint reading system which can record each fingerprint data in half second. Prints of eight fingers are recorded excluding little fingers.The light reflected from a fingerprint can be measured and conveed to digital data which is classified, codified and stored in the computer." "This is a real-world medical entrance exam question, please give the true answer based on the question and selection. Topic name: None, Subject name: Dental","Impression compound is characterized by all the following except: Select and explain. A. Warps at room temperature, B. Is a thermoset material, C. Shows increased flow when kneaded with water, D. Low coefficient of thermal conductivity",B: Is a thermoset material,"Answer: B: Is a thermoset material Explanation: Impression compound is not a thermoset material." "This is a real-world medical entrance exam question, please give the true answer based on the question and selection. Topic name: None, Subject name: Gynaecology & Obstetrics","In Galactorrhoea--amenorrhea syndromes, which is the investigation you should advise (apa from serum prolactin)? Select and explain. A. TSH, B. LH, C. hCG, D. Urinary ketosteroids",A: TSH,"Answer: A: TSH Explanation: Ans: A. TSH(Ref Harrison I9/e p2 p2267)Galactorrhea - amenorrhea syndromes:Serum prolactin &TSH advised.Laboratory diagnosis:Measure basal, fasting morning PRL levels - To assess hypersecretion.Normal PRL levels <20 ,ug/L.Markedly elevated PRL levels (>1000 ,ug/L) -Falsely lowered.Due to assay aifacts & aggregated circulating PRL forms.Sample dilution required for accurate measurement.Usually biologically inactive (macroprolactinemia).Measuring TSH and T4 levels - To exclude hypothyroidism." "This is a real-world medical entrance exam question, please give the true answer based on the question and selection. Topic name: None, Subject name: Dental","What is common in class I cavity preparation for amalgam and gold inlay? Select and explain. A. Buccolingually divergent walls, B. Mesiodistally divergent walls, C. Maximum depth is needed in both cases to provide sufficient thickness, D. All of the above",B: Mesiodistally divergent walls,"Answer: B: Mesiodistally divergent walls Explanation: None" "This is a real-world medical entrance exam question, please give the true answer based on the question and selection. Topic name: None, Subject name: Dental","Periodontal surgery is best performed Select and explain. A. 4 weeks after completion of the hygienic phase, B. 8 weeks after the completion of the restorative phase, C. 4 weeks before completion of occlusal adjustments, D. Immediately after the completion of the hygienic phase",A: 4 weeks after completion of the hygienic phase,"Answer: A: 4 weeks after completion of the hygienic phase Explanation: None" "This is a real-world medical entrance exam question, please give the true answer based on the question and selection. Topic name: None, Subject name: Physiology","Endocrine glands that are not influenced by the pituitary gland include the: Select and explain. A. Thyroid gland, testes, and adrenal medulla, B. Adrenal medulla, parathyroids, and the islets of langerhans, C. Adrenal cortex, parathyroids and ovaries, D. Pancreas, adrenal medulla and thyroid gland","B: Adrenal medulla, parathyroids, and the islets of langerhans","Answer: B: Adrenal medulla, parathyroids, and the islets of langerhans Explanation: None" "This is a real-world medical entrance exam question, please give the true answer based on the question and selection. Topic name: None, Subject name: Dental","Rooting reflex also known as search reflex disappears: Select and explain. A. 2 month, B. 3 month, C. 4 month, D. 5 month",C: 4 month,"Answer: C: 4 month Explanation: Rooting reflex disappears in 3-4 months but if both are in option then go for 4 months." "This is a real-world medical entrance exam question, please give the true answer based on the question and selection. Topic name: None, Subject name: Physiology","Migratory motor complexes in the gut reappear after intervals of: Select and explain. A. 60 minutes, B. 90 minutes, C. 120 minutes, D. 150 minutes",B: 90 minutes,"Answer: B: 90 minutes Explanation: Ans: B. 90 minutes(Ref Ganong 25/e p496, 24/e p498)Migrating motor complexes in the gut reappear after intervals of 90 minutes.MMCs:Initiated by motilin.Circulating motilin increases at intervals of 100 min in interdigestive state.Coordinated with contractile phases of MMC.Contractions migrate aborally at a rate of about 5 cm/ min." "This is a real-world medical entrance exam question, please give the true answer based on the question and selection. Topic name: None, Subject name: Pharmacology","IV ultrashort acting barbiturate: Select and explain. A. Meprobamate, B. Pentobarbitone, C. Thiopentone, D. Phenobarbitone",C: Thiopentone,"Answer: C: Thiopentone Explanation: None" "This is a real-world medical entrance exam question, please give the true answer based on the question and selection. Topic name: None, Subject name: Microbiology","The component that sensitizes bacteria and virus to UV irradiation Select and explain. A. Lipids, B. Carbohydrates, C. Inorganic salts, D. Nucleic acids",D: Nucleic acids,"Answer: D: Nucleic acids Explanation: None" "This is a real-world medical entrance exam question, please give the true answer based on the question and selection. Topic name: None, Subject name: Dental","All these are Included In active tissue engineering, except Select and explain. A. Antigens fibroblasts, B. Bilayered cell therapy, C. Enamel matrix derivative, D. Acellular dermal matrix",D: Acellular dermal matrix,"Answer: D: Acellular dermal matrix Explanation: None" "This is a real-world medical entrance exam question, please give the true answer based on the question and selection. Topic name: AIIMS 2019, Subject name: Microbiology","For which of the following is PPV-23 is most beneficial: Select and explain. A. Cystic fibrosis patient, B. Sickle cell anemia patient, C. Patient with recurrent rhinitis and sinusitis, D. Child less than 2 years",B: Sickle cell anemia patient,"Answer: B: Sickle cell anemia patient Explanation: Repeated microinfarction can destroy tissues having micro vascular beds prone to sickling. Thus splenic function is frequently lost within 18-36 months of life causing susceptibility to infection paicularly by pneumococci. So ( PPV-23) polysaccharide pneumococcal vaccine is beneficial in sickle cell anemia." "This is a real-world medical entrance exam question, please give the true answer based on the question and selection. Topic name: None, Subject name: Pathology","Characteristic histopathological feature of basal cell car cinoma is Select and explain. A. Nuclear palisading, B. Keratin pearls, C. Psammoma bodies, D. Foam cells",A: Nuclear palisading,"Answer: A: Nuclear palisading Explanation: None" "This is a real-world medical entrance exam question, please give the true answer based on the question and selection. Topic name: AIIMS 2019, Subject name: Surgery","Most common mediastinal tumor: Select and explain. A. Neurogenic tumor, B. Thymoma, C. Lymphoma, D. Bronchogenic cyst",A: Neurogenic tumor,"Answer: A: Neurogenic tumor Explanation: M/C Mediastinal tumor - Neurogenic tumor M/C anterior mediastinal tumor - Thymoma Mediastinal Tumors in Adults Tumor Type Percentage of Total Location Neurogenic tumors 21 Posterior Cysts 20 All Thymomas 19 Anterior Tumor Type Percentage of Total Location Lymphomas 13 Anterior/middle Germ cell tumors 11 Anterior Mesenchymal tumors 7 All Endocrine tumors 6 Anterior/middle Mediastinal Tumors in Children Tumor Type Percentage of Total Location Neurogenic tumors 400 Posterior Lymphomas 180 Anterior/middle Cysts 18 All Germ cell tumors 11 Anterior Mesenchymal tumors 9 All Thymomas Rare Anterior MEDIASTINAL MASSES (MM) MC anterior MM: Thymoma Q MC middle MM: Cyst (Pericardial cyst is MC) MC posterior MM: Neurogenic tumors MC MM (overall): Neurogenic tumors MM seen in all three compaments of mediastinum: Lymphoma, bronchogenic cyst & mesenchymal tumors IOC for diagnosis of MM (except neurogenic tumors): CT IOC for diagnosis of neurogenic tumors: MRI" "This is a real-world medical entrance exam question, please give the true answer based on the question and selection. Topic name: AIIMS 2019, Subject name: ENT","All are the branches of ECA that supply nasal septum except: Select and explain. A. Anterior ethmoidal aery, B. Sphenopalatine aery, C. Facial aery, D. Superior labial aery",A: Anterior ethmoidal aery,"Answer: A: Anterior ethmoidal aery Explanation: Internal Carotid System 1. Anterior ethmoidal aery 2. Posterior ethmoidal aery Branches of ophthalmic aery External Carotid System Sphenopalatine Aery Branches of Internal maxillary Aery Greater palatine Aery Superior labial Aery Branch of Facial Aery" "This is a real-world medical entrance exam question, please give the true answer based on the question and selection. Topic name: None, Subject name: Social & Preventive Medicine","All of these are continuous variables except: Select and explain. A. Height in cms, B. Weight in kg, C. Blood groups A. B. ABO, D. Age in years and months",C: Blood groups A. B. ABO,"Answer: C: Blood groups A. B. ABO Explanation: Ans: C. Blood groups A. B. ABORef High Yield Statistics/p13).Blood groups (A, B, ABO) is a discrete variable." "This is a real-world medical entrance exam question, please give the true answer based on the question and selection. Topic name: None, Subject name: Dental","Which of the following type of tongue thrusting has the worst prognosis? Select and explain. A. Simple, B. Complex, C. Retained infantile swallow, D. All of the above",C: Retained infantile swallow,"Answer: C: Retained infantile swallow Explanation: Tongue thrusting has been variously described as a deviate swallow, visceral swallow, retained infantile swallow or reverse swallow. Prognosis Simple tongue thrusting - Excellent Complex tongue thrusting - Good Retained infantile swallow - Poor Pediatric Dentistry, Principles and Practice / M.S. Muthu, N. Sivakumar - 2nd Ed" "This is a real-world medical entrance exam question, please give the true answer based on the question and selection. Topic name: None, Subject name: Pharmacology","CYP50 inhibition is least by: Select and explain. A. Pantoprazole, B. Rabeprazole, C. Lansoprazole, D. Omeprazole",B: Rabeprazole,"Answer: B: Rabeprazole Explanation: Ans: B. Rabeprazole(Ref: Goodman Gilman 12/e p1990; Katzung131e p1060, 12/e p1089; KDT 7/e p653).Rabeprazole:Exclusively metabolized by CYP2C19 and does not cause any inhibition of CYP450.Highest efficacy.Lansoprazole - Most potent.Rabeprazole & pantoprazole have no significant drug interactions.The FDA has issued a warning about a potentially impoant adverse interaction between clopidogrel and proton pump inhibitors.Proton pump inhibitors should be prescribed to patients taking clopidogrel only if they have an increased risk of gastrointestinal bleeding or require them for chronic gastro-esophageal reflux or peptic ulcer disease, in which case agents with minimal CYP2C19 inhibition (pantoprazole or rabeprazole) are preferred" "This is a real-world medical entrance exam question, please give the true answer based on the question and selection. Topic name: None, Subject name: Pediatrics","Which of the following is a component of Pentalogy of Fallot – Select and explain. A. Atrial septal Defect (ASD), B. Patent Ductus Arteriosus (PDA), C. Coarctation of Aorta (COA), D. Left Ventricular Hypertrophy (LVH)",A: Atrial septal Defect (ASD),"Answer: A: Atrial septal Defect (ASD) Explanation: Peutalogy of fallot consists : - Tetralogy of Faint --> VSD, Pulmonic stenosis, Overriding of aorta,Right ventricular hypertrophy Atrial septal defect" "This is a real-world medical entrance exam question, please give the true answer based on the question and selection. Topic name: None, Subject name: Physiology","The contractile element in a skeletal muscle is present in: Select and explain. A. Sarcolemma, B. Sarcoplasm, C. Myofibril, D. Endomysium",C: Myofibril,"Answer: C: Myofibril Explanation: None" "This is a real-world medical entrance exam question, please give the true answer based on the question and selection. Topic name: None, Subject name: Ophthalmology","Cherry red spot and Hollenhorst plaque are seen in: Select and explain. A. CRAO, B. CRVO, C. Branch RAO, D. Branch RVO",A: CRAO,"Answer: A: CRAO Explanation: Ans: A. CRAO(licr Parson's 22/e p320-32l, 21 /e p313,314: Kanski 7/e p559-562: Yanoff and Duker 4/e p522)Cherry red spot & Hollenhorst plaque:Seen in both Central Retinal Aery Occlusion (CRAO) & BRAG.CRAO incidence is more common than BRAG." "This is a real-world medical entrance exam question, please give the true answer based on the question and selection. Topic name: None, Subject name: Dental","Optimal location for intra osseous anesthesia for mandibular second molars is Select and explain. A. Distal to the molar, B. Mesial to the molar, C. In the furcation area, D. Apical perforation and injection",B: Mesial to the molar,"Answer: B: Mesial to the molar Explanation: None" "This is a real-world medical entrance exam question, please give the true answer based on the question and selection. Topic name: None, Subject name: Gynaecology & Obstetrics","Which statements is true regarding VENTOUSE : Select and explain. A. Minor scalp abrasions and subgaleal hematomas in new born are more frequent than forceps, B. Can be applied when foetal head is above the level of ischial spine, C. Maternal trauma is more frequent than forceps, D. Can not be used when fetal head is not fully rotated",A: Minor scalp abrasions and subgaleal hematomas in new born are more frequent than forceps,"Answer: A: Minor scalp abrasions and subgaleal hematomas in new born are more frequent than forceps Explanation: None" "This is a real-world medical entrance exam question, please give the true answer based on the question and selection. Topic name: None, Subject name: Dental","After cleaning and pumicing the tooth surface, plaque formation takes place within Select and explain. A. A few minutes, B. 1/2 to 1 hour, C. 2 to 4 hour, D. After 1 hour",B: 1/2 to 1 hour,"Answer: B: 1/2 to 1 hour Explanation: None" "This is a real-world medical entrance exam question, please give the true answer based on the question and selection. Topic name: None, Subject name: Anatomy","Which of the following structure is not present on floor of third ventricle? Select and explain. A. Optic stalk, B. Third nerve, C. Infundibulum, D. Mammillary body",B: Third nerve,"Answer: B: Third nerve Explanation: Answer- B (Third nerve)The third nerve (Oculomotor) does not form the floor of the third ventricle although it lies in close relation to the floor.The anterior wall of the third ventricle is formed by Lamina terminalis, Anterior commissure, Anterior columns of the fornixThe posterior wall is formed by the Pineal body, Posterior commissure, Cerebral aqueduct.The roof is formed by Body of fornix and the ependyma lining the undersurface of the tela choroidea of the third ventricle.The floor of the third ventricle is formed by Optic chiasma, Tuber cinereum, Infundibulum, the Mammillary bodies, Posterior perforated substance, and Tegmentum of the midbrain. The lateral wall is formed by the Medial surface of the thalamus, Hypothalamus, and the Hypothalamic sulcus." "This is a real-world medical entrance exam question, please give the true answer based on the question and selection. Topic name: None, Subject name: Surgery","Speech problems in cleft palate patients are due to Select and explain. A. Lisping of tongue, B. Inability of soft palate to stop air to go into nasopharynx, C. Inability of learning process, D. All of the above",B: Inability of soft palate to stop air to go into nasopharynx,"Answer: B: Inability of soft palate to stop air to go into nasopharynx Explanation: None" "This is a real-world medical entrance exam question, please give the true answer based on the question and selection. Topic name: None, Subject name: ENT","According to the WHO definition of hearing loss, what is the value to classify as profound hearing loss? Select and explain. A. 61-71 dB, B. >81 dB, C. >91 dB, D. >101 dB",B: >81 dB,"Answer: B: >81 dB Explanation: Ans: B. >81 dB(Ref: http://wwwwlio.intipbaideulne.ssilieuring_impairment_gracies/en/)According to the WHO (2008) definition of hearing loss, hearing threshold in better ear >81 dB is classified as profound hearing loss.WHO (2008) Classification of Degree of Hearing LossHearing threshold in better ear (average of 500, 1000, Grade of Impairment0-25No impairment26-40Milddeg impairment41-60Moderatedeg impairment61-80SevereQ impairment>81Profound impairment including" "This is a real-world medical entrance exam question, please give the true answer based on the question and selection. Topic name: None, Subject name: Dental","What we will define it ""ratio of no. of death under 1 year of age to total live birth per 1000 live births per year""? Select and explain. A. Infant mortality rate, B. Child mortality rate, C. Child morbidity rate, D. Life expectancy rate",A: Infant mortality rate,"Answer: A: Infant mortality rate Explanation: None" "This is a real-world medical entrance exam question, please give the true answer based on the question and selection. Topic name: None, Subject name: Dental","The necrotising pressure areas, undergoing bone resorption and endosteal bone formation Select and explain. A. Juvenile periodontitis, B. Occlusal trauma, C. Chronic suppurative periodontitis, D. ANUG",B: Occlusal trauma,"Answer: B: Occlusal trauma Explanation: None" "This is a real-world medical entrance exam question, please give the true answer based on the question and selection. Topic name: None, Subject name: Microbiology","CLED is better medium than MacConkey medium for processing of urine samples Select and explain. A. It differentiate LF from NLF Bacteria, B. Prevents swarming of proteus, C. Identifies pseudomonas, D. Allows staphylococcus,Streptococcus and candida to grow","D: Allows staphylococcus,Streptococcus and candida to grow","Answer: D: Allows staphylococcus,Streptococcus and candida to grow Explanation: Answer- D. Allows staphylococcus,Streptococcus and candida to grow* CLED agar (cystine-lactose-electrolyte-deficient agar or medium) is a valuable non-inhibitorygrowth medium used in the isolation and differentiation ofurinary microbes* It suppos the growth of urinary pathogens and contaminants but prevents undue swarming of Proteus species due to its lack of electrolytes." "This is a real-world medical entrance exam question, please give the true answer based on the question and selection. Topic name: None, Subject name: Dental","Pre-maturity in edge to edge position is relieved by reducing: Select and explain. A. Mandibular incisal edge., B. Maxillary lingual fossa., C. Both AB, D. Either AB",A: Mandibular incisal edge.,"Answer: A: Mandibular incisal edge. Explanation: None" "This is a real-world medical entrance exam question, please give the true answer based on the question and selection. Topic name: None, Subject name: Physiology","Most effective method of assessing breathing is by measuring: Select and explain. A. Tidal volume, B. Respiratory rate, C. Alveolar ventilation, D. FEV 1",D: FEV 1,"Answer: D: FEV 1 Explanation: None" "This is a real-world medical entrance exam question, please give the true answer based on the question and selection. Topic name: AIIMS 2018, Subject name: Pharmacology","Major aim of phase 1 clinical trials is: Select and explain. A. Safety, B. Efficacy, C. Dose, D. Pharmacokinetics",C: Dose,"Answer: C: Dose Explanation: Phase 1 clinical trial : It is first phase in humans. It is conducted mainly on healthy volunteers. In drugs with serious adverse effects and where alternative treatment is not available, phase 1 can be conducted on patients also. Major aim is to know the maximum tolerable dose (MTD). The aim of phase 1 trials should be marked as MTD > Dose > Pharmacokinetics > Safety" "This is a real-world medical entrance exam question, please give the true answer based on the question and selection. Topic name: None, Subject name: Social & Preventive Medicine","The blood pressure data of 200 persons were collected. The first quaile BP of the data was 94 mm Hg and third quaile was 110 mm. How many patients have blood pressures between the 3rd and 4th quaile? Select and explain. A. 25, B. 50, C. 100, D. 200",B: 50,"Answer: B: 50 Explanation: Ans: B. 50Quaile:Type of quantile.In descriptive statistics - Quailes are ranked set of data values.Are 3 points dividing data set into four equal groups.Each group comprising quaer of data.1stquaile (Q1):25' percentile (splits off lowest 25% of data from highest 75%).2ndquaile (Q2):50' percentile (cuts data set in half).3rdquaile (Q3):75"" percentile (splits off highest 25% of data from lowest 75%).Interquaile range (IQR):Difference between upper & lower quailes.IQR = Q3 - Q1.Per question:Between every two quailes one/ fouh of total number of patients present.I.e. 200/4 = 50 patients." "This is a real-world medical entrance exam question, please give the true answer based on the question and selection. Topic name: None, Subject name: Physiology","Which of the following defines the pressure in the vascular system in the absence of blood flow? (Asked twice) Select and explain. A. Pulse pressure, B. Critical closing pressure, C. Mean circulatory filling pressure, D. Perfusion pressure",C: Mean circulatory filling pressure,"Answer: C: Mean circulatory filling pressure Explanation: Answer- C. Mean circulatory filling pressureMean Circulatory Filling pressure (MCFP) is equilibrium pressure that is reached throughout the cardiovascular system when cardiac output is stopped completely. MCFP can be measured by stopping blood flow (e.g., by stopping hea pumping by giving shock to hea with electricity) and allowing the pressure throughout the circulatory system to reach equilibrium. MCFP is equilibrium pressure everywhere in circulation." "This is a real-world medical entrance exam question, please give the true answer based on the question and selection. Topic name: None, Subject name: Gynaecology & Obstetrics","Best parameter for ultrasound evaluation of IUGR is: Select and explain. A. Placental membrane, B. Length of femur, C. Abdominal circumference, D. BPD",C: Abdominal circumference,"Answer: C: Abdominal circumference Explanation: “Abdominal circumfernece (AC) is the single most sensitive parameter to detect IUGR. Serial measurements of AC and estimations of fetal weight are more diagnostic to fetal growth restriction.” Dutta 7/e, p 463 “The biometric parameters, AC is most affected by fetal growth” Williams 24/e, p 199" "This is a real-world medical entrance exam question, please give the true answer based on the question and selection. Topic name: None, Subject name: Dental","According to WHO classification of periapical pathology, sinus to oral cavity comes under: Select and explain. A. 4.6, B. 4.6, C. 4.61, D. 4.62",D: 4.62,"Answer: D: 4.62 Explanation: None" "This is a real-world medical entrance exam question, please give the true answer based on the question and selection. Topic name: None, Subject name: Pediatrics","In a child, surgery was done for biliary stricture with hepatojejunal anastomosis. Postoperative bilirubin level after 2 weeks was 6 mg/dL from a preoperative level 12mg/dL. The reason for this could be: Select and explain. A. Normal lowering of bilirubin takes time, B. Anastomotic stricture, C. Delta bilirubin, D. Mistake in lab technique",C: Delta bilirubin,"Answer: C: Delta bilirubin Explanation: Ans: C. Delta bilirubin(Ref Harrison 19/e p280, 18/e p325)Delta Bilirubin:Albumin-linked bilirubin fraction (delta fraction/biliprotein)Is pa of direct-reacting bilirubin fraction including conjugated bilirubin, covalently linked to albumin.Represents an impoant fraction of total serum bilirubin in patients with cholestasis & hepatobiliary disorders.Clearance rate of albumin-bound bilirubin from serum approximates half-life of albumin (12-14 days) rather than sho half-life of bilirubin (4 hours).Due to tight albumin bonding." "This is a real-world medical entrance exam question, please give the true answer based on the question and selection. Topic name: None, Subject name: Pediatrics","A 2 month old girl has failure to thrive, polyuria and medullary nephrocalcinosis affecting both kidneys. Investigations show blood pH 7.48, bicarbonate 25 mEq/l, potassium 2 mEq/l, sodium 126 mEq/l and chloride 88 mEq/l. The most likely diagnosis is - Select and explain. A. Distal renal tubular acidosis, B. Primary hyperaldosteronism, C. Bartter syndrome, D. Pseudohypoaldosteronism",C: Bartter syndrome,"Answer: C: Bartter syndrome Explanation: Clinical features of the infant --> Polyuria, Growth retardation, Medullary Nephrocalcinosis Electrolyte abnormalities: - Decreased potassium (Normal 3.5 — 5.0 meq/L)                 o Normal Bicarbonate (Normal 21-30 meq/L) Decreased sodium (Normal 136 — 145 ineq/L)                    o Increased pH (Normal 7.38 — 7.44 meq/L) Decreased Chloride (Normal 98 — 106 meq/L) C/F and electrolyte abnormalities in Bartter's syndrome a Hypokalemia                                o Polyurea and Nocturea (d/t hypokalemia) Metabolic alkalosis                    ❑ Increased urinary chloride (cause Hypochloremia) Normal to low blood pressure o Hypomagnesemia (seen in minority of patient) Growth retardation                    o Hypercalciurea (causes nephrocalcinosis which is visible on ultrasound) Other options Distal renal tubular acidosis      - It can be easily ruled out as it causes acidosis not alkalosis Primary hyperaldosteronism - In primary hyperaldosteronism, Sodium level is always increased and the B.F. is high. Pseudo hypoaldosteronism             - Aldosterone level is normal or increased but there are features of aldosterone deficiency such as decreased sodium and increased potassium." "This is a real-world medical entrance exam question, please give the true answer based on the question and selection. Topic name: None, Subject name: Dental","Resin teeth: Select and explain. A. Show a high compressive strength, B. Contraindicated in patients with decreased inter occlusal distance, C. Show crazing, D. Show a high fracture strength",D: Show a high fracture strength,"Answer: D: Show a high fracture strength Explanation: None" "This is a real-world medical entrance exam question, please give the true answer based on the question and selection. Topic name: None, Subject name: Medicine","In alpha-thalassemia trait,electrophoresis shows:March 2005 Select and explain. A. Increased HbF and normal HbA2, B. Normal HbF and normal HbA2, C. Normal HbF and decreased HbA2, D. Decreased HbF and normal HbA2",B: Normal HbF and normal HbA2,"Answer: B: Normal HbF and normal HbA2 Explanation: Ans. B: Normal HbF and normal HbA2Alpha (0) thalassemia -Individuals with this disorder are not able to produce any functional alpha-globin and thus are unable to make any functional hemoglobin A, F, or A2. This leads to the development of hydrops fetalis, also known as hemoglobin Ba, a condition that is incompatible with extra uterine life.Alpha (+) thalassemia - genetic mutations that result in decreased production of alpha-globin usually due to the functional deletion of 1 of the 4 alpha globin genes. Based on the number of inherited alpha genes, it is subclassified into 3 general forms:Thalassemia (-u/ a a) is characterized by inheritance of 3 normal a-genes. These patients are referred to clinically as silent carrier of alpha thalassemia. Other names for this condition are alpha thalassemia minima, alpha thalassemia-2 trait, and heterozygosity for alpha (+) thalassemia minor. The affected individuals exhibit no abnormality clinically and may be hematologically normal or have mild reductions in red cell mean corpuscular volume (MCV) and mean corpuscular hemoglobin (MCH).Inheritance of 2 normal alpha genes due to either heterozygosity for alpha (0) thalassemia (u a/ --) or homozygosity for alpha (+) thalassemia (-u/-a) results in the development of alpha thalassemia minor or alpha thalassemia-1 trait. The affected individuals are clinically normal but frequently have minimal anemia and reduced mean corpuscular volume (MCV) and mean corpuscular hemoglobin (MCH).Inheritance of one normal alpha gene (-a/ --) results in abundant formation of hemoglobin H. This condition is known as ElbH disease. The affected individuals have moderate to severe lifelong hemolytic anemia, modest degrees of ineffective erythropoiesis, splenomegaly and variable bony changesPersons with alpha-thalassemia traits have normal HbA2 and HbF levels whereas beta- thalassemia patients have elevated HbA2" "This is a real-world medical entrance exam question, please give the true answer based on the question and selection. Topic name: None, Subject name: Dental","Zone of stimulation contains all except: Select and explain. A. Fibroblasts, B. Osteoclasts, C. Osteoblasts, D. New bone formation",B: Osteoclasts,"Answer: B: Osteoclasts Explanation: The reaction of the periradicular tissues to noxious products of tissue necrosis, bacterial products and antigenic agents from the root canal has been described by Fish. The four zones  Zone of infection Zone of contamination Zone of irritation Zone of stimulation Zone of Infection: This is present in the center of the lesion. It is characterized by PMNL's. Zone of Contamination: It is characterized by round cell infiltration. It demonstrates cellular destruction due to toxins from the central zone or zone of infection. Because of autolysis and death of bone cells, the lacunae were empty. Lymphocytes are prevalent everywhere. Zone of Irritation: This zone is characterized by macrophages and osteoclasts. The collagen framework was digested by phagocytic cells, the macrophages, while osteoclasts attacked the bone tissue. This opens a gap in the bone all around the center of lesion. That space becomes filled with PMNL's. This zone demonstrates much activity preparatory to repair. Zone of Stimulation: (Peripheral zone) This zone is characterized by fibroblasts and osteoblasts. In this zone, the effects of toxins were mild enough to be stimulant which results in laying down of collagen fibers around the zone of irritation that acts both as a wall of defense and as a scaffolding on which the osteoblasts built new bone. This new bone was built in an irregular fashion." "This is a real-world medical entrance exam question, please give the true answer based on the question and selection. Topic name: None, Subject name: Dental","Minimum number of lobes require to form a tooth Select and explain. A. 1, B. 2, C. 3, D. 4",A: 1,"Answer: A: 1 Explanation: None" "This is a real-world medical entrance exam question, please give the true answer based on the question and selection. Topic name: None, Subject name: Dental","In the odontoblastic layer the following connections is/are found Select and explain. A. Zona adherens, B. Gap junctions, C. Zonula occludens, D. All of the above",D: All of the above,"Answer: D: All of the above Explanation: None" "This is a real-world medical entrance exam question, please give the true answer based on the question and selection. Topic name: None, Subject name: Pediatrics","All are features of Neonatal Tetanus except – Select and explain. A. Refusal to feed is common initial symptom, B. Caused by clostridium tetani, C. Usually occurs in 1st 2 days of life, D. Mortality is 50–75%.",C: Usually occurs in 1st 2 days of life,"Answer: C: Usually occurs in 1st 2 days of life Explanation: Neonatal Tetanus It is caused by clostridium tetani Clinical features Symptoms of neonatal tetanus starts by 5-15 days after birth, never in the first 2 days of life and rarely after the age of 2 weeks. (ft is commonly k/a as 811! day disease) Common initial symptoms are Excessive unexplained crying, Refusal of feeds, Apathy Prognosis —> Mortality rate varies from 50-75%." "This is a real-world medical entrance exam question, please give the true answer based on the question and selection. Topic name: None, Subject name: Forensic Medicine","In assessing infant deaths, Ploucquet's test involves: Select and explain. A. Change in specific gravity of lungs, B. Presence of air in stomach and duodenum, C. Change in paial weight of lungs, D. Air in middle ear",C: Change in paial weight of lungs,"Answer: C: Change in paial weight of lungs Explanation: Ans: C. Change in paial weight of lungs(Ref: Reddy 34/e p411, 33/e p439).In assessing infant deaths, Ploucquet's test involves change in paial weight of lungs.Ploucquet's test:The blood flow in the lung beds is so increased after breathing that their weight is almost doubled from 1/70 of the body weight before respiration to 1/35 after respiration.This increase in weight is not constant and is not a reliable indication of breathing." "This is a real-world medical entrance exam question, please give the true answer based on the question and selection. Topic name: None, Subject name: Dental","Scammon's growth curve. False is Select and explain. A. Neural tissues-Most of the growth is completed by 6 years, B. Lymphoid tissues — growth reaches 200% by age 13 and regresses afterwards, C. Genital tissues — most of the growth is completed by the age of puberty, D. None of the above",C: Genital tissues — most of the growth is completed by the age of puberty,"Answer: C: Genital tissues — most of the growth is completed by the age of puberty Explanation: None" "This is a real-world medical entrance exam question, please give the true answer based on the question and selection. Topic name: None, Subject name: Physiology","The transport of CO is diffusion limited because Select and explain. A. Binds avidly with Hb, B. Partial pressure of CO is less, C. CO is less absorbed in water, D. Alveolar membrane is less permeable",A: Binds avidly with Hb,"Answer: A: Binds avidly with Hb Explanation: None" "This is a real-world medical entrance exam question, please give the true answer based on the question and selection. Topic name: AIIMS 2018, Subject name: Pharmacology","Acetazolamide is given to a patient of angle closure glaucoma. It is a non-competitive inhibitor of carbonic anhydrase enzyme. Which of the following should be the effect of this drug? Select and explain. A. Decrease in Vmax, B. A.Decrease in Km, C. Increase in both Km and Vmax, D. No change in Vmax",A: Decrease in Vmax,"Answer: A: Decrease in Vmax Explanation: * Enzyme inhibition is of 3 types: Type of inhibition Km Vmax Competitive inhibition Increases No change Non competitive inhibition No change Decreases Uncompetitive inhibition Decreases Decreases * In question non competitive inhibitor is given, so answer is decrease in Vmax." "This is a real-world medical entrance exam question, please give the true answer based on the question and selection. Topic name: None, Subject name: Anaesthesia","All are true about rapid sequence induction done in a cardiac patient in emergency except: Select and explain. A. Inducing agent and neuromuscular relaxant are administered together, B. The patient is pre-oxygenated for 3 minutes before the procedure, C. Cricoid pressure has to be applied till the endotracheal tube has been secured with a cuff, D. Induction should be done with thiopentone sodium and succinylcholine for muscle relaxation",D: Induction should be done with thiopentone sodium and succinylcholine for muscle relaxation,"Answer: D: Induction should be done with thiopentone sodium and succinylcholine for muscle relaxation Explanation: Ans: D. Induction should be done with thiopentone sodium and succinylcholine for muscle relaxation(Ref Miller 7/e p34, 887)Thiopentone induction is contraindicated in cardiac patients because of direct negative inotropic effects on the hea and depressant effects on systemic BR.Etomidate is the preferred induction agent in such a patient." "This is a real-world medical entrance exam question, please give the true answer based on the question and selection. Topic name: None, Subject name: Ophthalmology","A 25-year-old lady presents with sudden severe bilateral loss of vision, more so on the right side, with no perception of light. Rest of the examination including pupillary reflexes, fundus and optokinetic nystagmus are normal. She was able to touch the tips of her fingers with her right eye closed but not with her left eye closed. The most like1 tgnosis is: Select and explain. A. Optic neuritis, B. Anterior ischemic optic neuropathy, C. CMV retinitis, D. Functional vision loss",D: Functional vision loss,"Answer: D: Functional vision loss Explanation: Ans. d. Functional vision loss (Ref Kanski 6/e p 477, 790, 792-793)The most likely diagnosis in this young lady who presents with sudden severe bilateral loss of vision, more so on the right side, with no perception of light and normal examination findings including pupillary reflexes, fundus and optokinetic nystagmus, who is able to touch tips of her finger with right eye closed but not with left eye closed, is functional vision loss." "This is a real-world medical entrance exam question, please give the true answer based on the question and selection. Topic name: None, Subject name: ENT","A 60-year old male presents with painful, 141 oupet. vesicles over erythematous plaques in T3 dermatome region of trunk. Which of the following would be the most likely causative organism? Select and explain. A. Varicella zoster, B. Herpes simplex, C. Pox virus, D. Papilloma virus",A: Varicella zoster,"Answer: A: Varicella zoster Explanation: Ans. a. Varicella zosterA 60-year-old male presents with painful, grouped vesicles over erythematous plaques in T dermatome region of trunk. Varicella zoster is the most likely causative organism." "This is a real-world medical entrance exam question, please give the true answer based on the question and selection. Topic name: None, Subject name: Pharmacology","A patient is administered 200 mg of a drug. 75 mg of the drug is eliminated from the body in 90 minutes. If the drug follows first order kinetics, how much drug will remain after 6 hours? Select and explain. A. 12.5 mg, B. 25 mg, C. 30 mg, D. 50 mg",C: 30 mg,"Answer: C: 30 mg Explanation: Answer- C. 30 mgDrug remains in the body at the end of 4th 90 minutes (6 hours) : 48.7 -18.3 = 30.4 (Approx. 30 mg)" "This is a real-world medical entrance exam question, please give the true answer based on the question and selection. Topic name: None, Subject name: Pharmacology","The most common side effect of the oral administration of ampicillin is Select and explain. A. Anaphylactic shock, B. Diarrhoea, C. Oral candidiasis, D. Renal failure",B: Diarrhoea,"Answer: B: Diarrhoea Explanation: None" "This is a real-world medical entrance exam question, please give the true answer based on the question and selection. Topic name: AIIMS 2018, Subject name: Pharmacology","Adverse effects of salbutamol are all except Select and explain. A. Tremors, B. Tachycardia, C. Hypoglycemia, D. Hypokalemia",C: Hypoglycemia,"Answer: C: Hypoglycemia Explanation: Salbutamol is a beta 2 agonist. Adverse effects of beta 2 agonists: Tremors Tachycardia Tolerance Hypokalemia (by enhancing potassium uptake into muscles) Hypoxia Hyperglycemia (by stimulating glycogenolysis and gluconeogenesis) Note: Tolerance occurs to only long acting beta 2 agonists ." "This is a real-world medical entrance exam question, please give the true answer based on the question and selection. Topic name: None, Subject name: Dental","Most common extraradicular microorganism is/ are: Select and explain. A. Actinomyces species., B. Propioni bacterium., C. Propionicum., D. All of the above.",D: All of the above.,"Answer: D: All of the above. Explanation: None" "This is a real-world medical entrance exam question, please give the true answer based on the question and selection. Topic name: None, Subject name: Surgery","Ludwig's angina is characterized by Select and explain. A. Raised tongue, B. Elevation of ear lobe, C. Trismus, D. Unilateral swelling",A: Raised tongue,"Answer: A: Raised tongue Explanation: None" "This is a real-world medical entrance exam question, please give the true answer based on the question and selection. Topic name: None, Subject name: Dental","Least condensation pressure is required in: Select and explain. A. Spherical amalgam alloy, high copper, B. Admixed amalgam alloy, high copper, C. Admixed amalgam alloy, low copper, D. Lathe cut alloy","A: Spherical amalgam alloy, high copper","Answer: A: Spherical amalgam alloy, high copper Explanation: A spherical amalgam contains small, round alloy particles that are mixed with mercury to form the mass that is placed into the tooth preparation. Because of the shape of the particles, the material is condensed into the tooth preparation with little condensation pressure. This advantage is combined with its high early strength to provide a material that is well suited for very large amalgam restorations, such as complex amalgams or foundations. Sturdevant ed 4 pg 654" "This is a real-world medical entrance exam question, please give the true answer based on the question and selection. Topic name: None, Subject name: Social & Preventive Medicine","Denominator in perinatal moality rate Select and explain. A. Total bihs, B. Total live bihs, C. Live bihs + Still bih, D. Total number of newborns",A: Total bihs,"Answer: A: Total bihs Explanation: Ans: A. Total bihs* PERINATAL MOALITY RATE is the sum of the number of resident fetal deaths of 28 or more weeks gestation plus the number of resident newborns dying under 7 days of age in a specified geographic area (country, state, county, etc.) divided by TOTAL BIHS includes the sum of the number of resident live bihs plus the number of resident fetal deaths of 28 or more weeks gestation for the same geographic area (for a specified time period, usually a calendar year) and multiplied by 1,000." "This is a real-world medical entrance exam question, please give the true answer based on the question and selection. Topic name: None, Subject name: Dental","A child spends 8 years of life in a temperate climate where the drinking water had fluoride of 3 ppm. The teeth that are effected are: Select and explain. A. All primary teeth, B. All permanent teeth, except third molars, C. Incisors, canines, premolars and third molars, D. Only molars","B: All permanent teeth, except third molars","Answer: B: All permanent teeth, except third molars Explanation: Enamel fluorosis is a developmental phenomenon due to excessive fluoride ingestion during amelogenesis. Once the crowns of the teeth are formed, no further fluorosis can be induced by additional intake of fluoride or by posteruptive topical applications of fluoride." "This is a real-world medical entrance exam question, please give the true answer based on the question and selection. Topic name: None, Subject name: Dental","The ideal instrument that is used to widen the canal is Select and explain. A. Broach, B. File, C. Fissure bur, D. Reamer",B: File,"Answer: B: File Explanation: None" "This is a real-world medical entrance exam question, please give the true answer based on the question and selection. Topic name: None, Subject name: Dental","According to instrument formula given by G.V. Black, primary cutting edge angle should be: Select and explain. A. More than 50 degree, B. More than 90 degree, C. Less than 50 degree, D. Less than 90 degree",A: More than 50 degree,"Answer: A: More than 50 degree Explanation: The second number of a four-number code indicates the primary cutting edge angle, measured from a line parallel to the long axis of the instrument handle in clockwise centigrade. The centigrade angle is expressed as a percent of 360 degrees (e.g., 85 = 85% × 360 degrees = 306 degrees). The instrument is positioned so that this number always exceeds 50. Sturdevants operative dentistry 7th edition page e2" "This is a real-world medical entrance exam question, please give the true answer based on the question and selection. Topic name: None, Subject name: Dental","Which of the following is not true for incubation period? Select and explain. A. For determining isolation period of a disease, B. To prevent infection among contacts, C. For quarantine of disease, D. To differentiate among secondary cases and co-primary cases",A: For determining isolation period of a disease,"Answer: A: For determining isolation period of a disease Explanation: None" "This is a real-world medical entrance exam question, please give the true answer based on the question and selection. Topic name: None, Subject name: Dental","Treatment of ANUG utilizes all except: Select and explain. A. Debridement, B. Oral hygiene, C. Penicillin, D. Topical steroid",D: Topical steroid,"Answer: D: Topical steroid Explanation: None" "This is a real-world medical entrance exam question, please give the true answer based on the question and selection. Topic name: None, Subject name: Pathology","Median rhomboid glossitis is associated with: Select and explain. A. Burning mouth syndrome, B. Fungal infection, C. Bacterial infection, D. Absence of filiform papillae",D: Absence of filiform papillae,"Answer: D: Absence of filiform papillae Explanation: None" "This is a real-world medical entrance exam question, please give the true answer based on the question and selection. Topic name: None, Subject name: Microbiology","A young male present with loose motions and intermittent abdominal pain over the past 1 year. Wet mount stool specimen showed the presence of multiple ova which are more than 100u in diameter. The causative organisms for the disease shall not include? Select and explain. A. Fasciola gigantica, B. Gastrodiscoides hominis, C. Echinostoma ilocanum, D. Opisthorchis viverrini",D: Opisthorchis viverrini,"Answer: D: Opisthorchis viverrini Explanation: None" "This is a real-world medical entrance exam question, please give the true answer based on the question and selection. Topic name: None, Subject name: Forensic Medicine","Concentric teeth bite mark on forearm, what to do next? Select and explain. A. 2 swab technique for saliva collection, B. Keep scale for measuring below the mark and take photo in the plane of bite, C. Complete description of injury as seen, D. Prepare cast using vinyl polysiloxane",C: Complete description of injury as seen,"Answer: C: Complete description of injury as seen Explanation: Ans: C. Complete description of injury as seenRef KSN Reddy's The Essentials of Forensic Medicine and Toxicology 33' edn; Page no. 97Impoant: The order of Bite mark investigation: C > B > A> DBite Mark InvestigationComplete description of injury as seenPhotograph: Keep scale for measuring below the mark and take photo from different angle.Swabbing of saliva: To identify or exclude assailant from secretor status who exude blood group substance in the saliva.Impression of bite mark: Plastic substance (rubber or silicone based) or plaster of Paris is laid over the bite mark that hardens and produces permanent negative cast of the lesion.Skin carrying the bite is removed and preserved in formalin during autopsy." "This is a real-world medical entrance exam question, please give the true answer based on the question and selection. Topic name: None, Subject name: Psychiatry","A patient believes he is the most impoant person in the world than anyone so his neighbors and family is trying to harm him as they are jealous of him. His wife says otherwise and says he behaves like this recently only before he was working as a school-teacher peacefully and brought to OPD. He is suffering from: Select and explain. A. Delusion of grandiosity, B. Delusion of persecution, C. Delusion of grandiosity and persecution, D. Delusion of grandiosity, persecution and reference","D: Delusion of grandiosity, persecution and reference","Answer: D: Delusion of grandiosity, persecution and reference Explanation: Ans. d. Delusion of grandiosity, persecution and reference The given description suggests that the patient is having delusion of grandiosity, persecution and reference.Delusion of grandeurExaggerated conception of one's impoance, power, or identity.Delusion of persecutionFalse belief of being harassed or persecuted; often found in litigious patients who have a pathological tendency to take legal action because of imagined mistreatment.MC delusion.Delusion of referenceFalse belief that behavior of others refers to oneself or that events, objects, or other people have a paicular & unusual significance, usually of a negative natureDerived from idea of reference, in which persons falsely feel that others are talking about them (e.g., belief that people on television or radio are talking to or about the person)." "This is a real-world medical entrance exam question, please give the true answer based on the question and selection. Topic name: None, Subject name: Social & Preventive Medicine","Measuring variation between two different units is done through Select and explain. A. Variance, B. Standard deviation, C. Coefficient of variation, D. Range",C: Coefficient of variation,"Answer: C: Coefficient of variation Explanation: None" "This is a real-world medical entrance exam question, please give the true answer based on the question and selection. Topic name: AIIMS 2019, Subject name: Pathology","Which of the following diseases is diagnosed with the help of the flowcytometry pattern? Select and explain. A. Bruton disease, B. Bare lymphocyte syndrome, C. Hyper IgM syndrome, D. Severe combined immunodeficiency disease",C: Hyper IgM syndrome,"Answer: C: Hyper IgM syndrome Explanation: Flow cytometry can rapidly and quantitatively measure several individual cell characteristics, but is mainly used to identify cellular antigens expressed by liquidtumors , those that arise from blood forming tissues. Hyper IgM syndrome - In this disorder the affected patients make IgM antibodies but are deficient in their ability to produce IgG, IgA and IgE antibodies. There is problem in Ig class switching. Approximately 70%of individuals with hyper lgM syndrome have the X linked form of the disease , caused by mutations in the gene coding CD40L located on Xq26. In remaining patients,the disease is inherited in an autosomal recessive pattern. Most of these patients have loss of function mutation involving either CD40 or enzyme called AID. So, B-cells can be detected by flowcytometryhaving low expression of CD40. Bruton's disease -In this disease, there is defect in B-cell tyrosine kinase enzyme. This leads to hypogammaglobulinemia. Bare lymphocyte syndrome- In this disorder, There is defect in MHC-2 molecule which is required for CD4 T-cell formation. SCID - This is both X-linked and autosomal recessive disorder." "This is a real-world medical entrance exam question, please give the true answer based on the question and selection. Topic name: None, Subject name: Medicine","Best indicator of developing cardiovascular disease is Select and explain. A. LDL cholesterol level more than 120 mg/dl, B. History of salt intake of more than 6 gm/day, C. LDL/HDL ratio, D. All of the above",C: LDL/HDL ratio,"Answer: C: LDL/HDL ratio Explanation: None" "This is a real-world medical entrance exam question, please give the true answer based on the question and selection. Topic name: None, Subject name: Dental","Pulp proper contains Select and explain. A. Large Blood vessels, B. Nerve fibers, C. Fibroblasts, D. All of the above",D: All of the above,"Answer: D: All of the above Explanation: None" "This is a real-world medical entrance exam question, please give the true answer based on the question and selection. Topic name: None, Subject name: Dental","Which of the following is used as a bio-degradable membrane in ""guided tissue regeneration"" process? Select and explain. A. Polylactic acid, B. Poly tetrafluoroethylene, C. Millipore, D. Core membrane",A: Polylactic acid,"Answer: A: Polylactic acid Explanation: None" "This is a real-world medical entrance exam question, please give the true answer based on the question and selection. Topic name: None, Subject name: Medicine","Pulmonary-renal syndromes include- Select and explain. A. Goodpasture, B. Leptospirosis, C. Hantavirus and Wegner's, D. All",D: All,"Answer: D: All Explanation: Answer-D. AllPulmonary-renal syndromes are most commonly caused by an underlying autoimmune disease. PRS is most commonly due to ANCA-associated vasculitides (e.g., granulomatosis with polyangiitis) or due to anti-basement membrane diseases (e.g., Goodpasture's syndrome). Other etiologies include toxic injury such as paraquat poisoning, infection with hantavirus, leptospirosis, or legionella, or vascular, as seen in nephrotic syndrome when a renal vein thrombosis embolism to the lungs." "This is a real-world medical entrance exam question, please give the true answer based on the question and selection. Topic name: None, Subject name: Dental","Bone sounding done in modern times is performed by which method? Select and explain. A. RVG, B. Probing, C. CBCT, D. Radiovisiography",C: CBCT,"Answer: C: CBCT Explanation: None" "This is a real-world medical entrance exam question, please give the true answer based on the question and selection. Topic name: None, Subject name: Pathology","Oil red '0' stain is used for: Select and explain. A. Glutaraldehyde fixed specimen, B. Alcohol fixed specimen, C. Formalin fixed specimen, D. Frozen specimen",D: Frozen specimen,"Answer: D: Frozen specimen Explanation: Ans: D. Frozen specimenFor Connective Tissue and LipidsName of stainElements stainedTrichrome StainCollagenVerhoeff-Van Gieson stain (Best for Elastin)Elastic fibersLuna stainElastin & Mast cellsSilver Methenamine stainReticulinOil red '0' stain (on Fresh specimen)Sudan black (on fixed specimen)FatMallory's PTAH stainMuscle striationsMaius scarlet blue (MSB)FibrinPAS, Silver Methenamine stainBasement membraneBielschowsky (silver stain)Neurofibrillary tangles senile plaquesLuxol fast blueMyelin(Ref Netter :s* Essential Histology 2/e p479)" "This is a real-world medical entrance exam question, please give the true answer based on the question and selection. Topic name: None, Subject name: Dental","Jaw or tooth apart swallow is: Select and explain. A. Infantile swallow., B. Mature swallow., C. Both, D. None.",A: Infantile swallow.,"Answer: A: Infantile swallow. Explanation: None" "This is a real-world medical entrance exam question, please give the true answer based on the question and selection. Topic name: None, Subject name: Surgery","A female with suspected child abuse was brought to the casualty with severe bleeding from the perineum. What should be the first line of management? Select and explain. A. Airway maintenance, B. Internal iliac aery ligation, C. Whole blood transfusion, D. Inform police before staing the treatment",A: Airway maintenance,"Answer: A: Airway maintenance Explanation: Ans. a. Airway maintenanceThe primary survey aims to identify and manage the most immediately life-threatening pathologies first and follows ABCDE.c- exsanguinating external hemorrhageA- Airway maintenanceB- Breathing & ventilationC- Circulation & hemorrhageD- DisabilityE- Exposure" "This is a real-world medical entrance exam question, please give the true answer based on the question and selection. Topic name: None, Subject name: Dental","Classify the class of recession: Select and explain. A. Class 1, B. Class 2, C. Class 3, D. Class 4",C: Class 3,"Answer: C: Class 3 Explanation: None" "This is a real-world medical entrance exam question, please give the true answer based on the question and selection. Topic name: None, Subject name: Pediatrics","Method of choice for a New born child not passing urine for 36 hours : Select and explain. A. Ultrasound of kidney & bladder, B. CT Scan, C. Cystoscopy, D. X–ray pelvis",A: Ultrasound of kidney & bladder,"Answer: A: Ultrasound of kidney & bladder Explanation: A history of Anuria suggests Acute Renal failure. Ultrasonography is the ideal imaging tool in Renal failure because of its non dependence on Renal function. As it allows visualization of : Pelvicalyceal system ofAssessment of Renal Size o Structural anomalies and calculi . Note - In this question DTPA scan has not been provided as an option. So, the best answer is USG." "This is a real-world medical entrance exam question, please give the true answer based on the question and selection. Topic name: None, Subject name: Pathology","Bacteria free zone of dentinal caries is Select and explain. A. a)Zone of decomposed dentin, B. Zone of bacterial invasion, C. Zone of decalcification, D. Zone of dentinal sclerosis",D: Zone of dentinal sclerosis,"Answer: D: Zone of dentinal sclerosis Explanation: None" "This is a real-world medical entrance exam question, please give the true answer based on the question and selection. Topic name: None, Subject name: Dental","Which is not a type of Gracey curvettes? Select and explain. A. # 1-2, B. # 11-12, C. # 13-14, D. # 5-16",D: # 5-16,"Answer: D: # 5-16 Explanation: None" "This is a real-world medical entrance exam question, please give the true answer based on the question and selection. Topic name: None, Subject name: Dental","Which ion of hydroxyapatite crystals is substituted by Fluoride imparts more strength? Select and explain. A. Hydroxyl ion, B. Calcium ion, C. Phosphate ion, D. None",A: Hydroxyl ion,"Answer: A: Hydroxyl ion Explanation: None" "This is a real-world medical entrance exam question, please give the true answer based on the question and selection. Topic name: None, Subject name: Dental","Cleaning of base metal alloy is done by Select and explain. A. Warm HCl, B. Cold HF, C. Cold H2 So4, D. Sand blasting with aluminium oxide",D: Sand blasting with aluminium oxide,"Answer: D: Sand blasting with aluminium oxide Explanation: None" "This is a real-world medical entrance exam question, please give the true answer based on the question and selection. Topic name: None, Subject name: Pediatrics","Estimation of the blood sugar is relevant in all except – Select and explain. A. Birth asphyxia, B. Large for date baby, C. Baby of hypothyroid mother, D. Rh Incompatibility",C: Baby of hypothyroid mother,"Answer: C: Baby of hypothyroid mother Explanation: ARDS (Birth asphyxia), Rh incompatibility (if causes erythroblastosis fetalis) and large for date baby of diabetic mother are prone for hypoglycemia." "This is a real-world medical entrance exam question, please give the true answer based on the question and selection. Topic name: None, Subject name: Medicine","All are true about isolated aoic stenoses except: Select and explain. A. Pulsus bisferiens, B. Cardiac apex is displaced laterally to left, C. Thrill in carotid aery, D. Blood pressure maintained in initial phase",A: Pulsus bisferiens,"Answer: A: Pulsus bisferiens Explanation: Ans: A. Pulsus bisferiens(Ref Harrison 19/e p15.31, 1536, 18/e p1939)Pulsus bisferiens is seen in a patient of aoic stenosis (AS) along with aoic regurgitation (AR) and is rare in isolated AS.The cardiac apex is usually displaced to left because of left ventricular hyperophy.In some patients with AR or with combined AS and AR, the carotid aerial pulse may be bisferiens, i.e., with two systolic waves separated by a trough." "This is a real-world medical entrance exam question, please give the true answer based on the question and selection. Topic name: None, Subject name: Pathology","Which of the following is the most common site for the occurrence of a basal cell carcinoma? Select and explain. A. Buccal mucosa, B. Hard palate, C. Skin of the lower lip, D. Dorsum of the tongue",C: Skin of the lower lip,"Answer: C: Skin of the lower lip Explanation: None" "This is a real-world medical entrance exam question, please give the true answer based on the question and selection. Topic name: None, Subject name: Dental","Who was associated with second amalgam war? Select and explain. A. G V Black, B. Charles bell, C. Huggins, D. Alfred stock",D: Alfred stock,"Answer: D: Alfred stock Explanation: None" "This is a real-world medical entrance exam question, please give the true answer based on the question and selection. Topic name: None, Subject name: Anatomy","Reticular fibers of collagen tissues are present in all of the following except: Select and explain. A. Thymus, B. Spleen, C. Bone marrow, D. Lymph node",A: Thymus,"Answer: A: Thymus Explanation: Ans: A. ThymusReticular fibers of collagen tissues are present in Spleen, Bone marrow & Lymph node but not in thymus.Reticulin:Type of fiber in connective tissue.Composed of type III collagen.Secreted by reticular cells.Reticular fibers crosslink to form a fine meshwork.Acts as a suppoing mesh in soft tissues such as liver, bone marrow & tissues and organs of lymphatic system." "This is a real-world medical entrance exam question, please give the true answer based on the question and selection. Topic name: AIIMS 2019, Subject name: Ophthalmology","Which of the following muscle is not supplied by the nerve marked in the diagram? Select and explain. A. Superior oblique, B. Medial rectus, C. Inferior rectus, D. Inferior oblique",A: Superior oblique,"Answer: A: Superior oblique Explanation: The nerve marked in the diagram is oculomotor nerve. It supplies superior rectus, inferior rectus, medial rectus and inferior oblique. Superior oblique is supplied by Trochlear nerve." "This is a real-world medical entrance exam question, please give the true answer based on the question and selection. Topic name: None, Subject name: Anatomy","Which is a direct content of cavernous sinus? Select and explain. A. Ophthalmic division of trigeminal nerve, B. Trochlear nerve, C. Abducent nerve, D. Oculomotor nerve",C: Abducent nerve,"Answer: C: Abducent nerve Explanation: Abducent nerve: Only cranial nerve passing through the centre of cavernous sinus." "This is a real-world medical entrance exam question, please give the true answer based on the question and selection. Topic name: None, Subject name: Dental","In Mount and Hume classification, what will be the site size of the following lesion? Select and explain. A. 2.1, B. 2.2, C. 3.1, D. 3.2",C: 3.1,"Answer: C: 3.1 Explanation: MOUNT AND HUME CLASSIFICATION 1998" "This is a real-world medical entrance exam question, please give the true answer based on the question and selection. Topic name: None, Subject name: Forensic Medicine","Keeping in Jack-knife position for long leads to death by: Select and explain. A. Wedging, B. Burking, C. Positional asphyxia, D. Traumatic asphyxia",C: Positional asphyxia,"Answer: C: Positional asphyxia Explanation: Answer- C. Positional asphyxiaKeeping in Jack-knife position for long leads to death by positional asphyxia.""Postural or Positional Asphyxia"":Occasionally, it results from indirect compression, when the body is subjected to force in such a manner that his thighs and the knees are driven against his chest, the so-called ""Jack-Knife"" position. There is usually marked congestion, cyanosis, and petechiae in the face and neck.This occurs when the individual acquires a ceain body position in which breathing is impaired, often because of neck twisting with kinking or compression of trachea and/or elevation of the tongue into the posterior hypopharynx. Normal venous return to the hea may be impaired.The body is typically inveed (upside-down) & weight of abdominal contents press against the diaphragm pushing it upwards, thus compressing the thoracic organs, which combined with decreased respiratory movements, leads to cardiorespiratory failure & death.It is always accidental" "This is a real-world medical entrance exam question, please give the true answer based on the question and selection. Topic name: None, Subject name: Dental","Extraction of 3rd molar tooth bud in 7-9 yr old child: Select and explain. A. Improves growth of maxilla, B. Causes excessive damage to the mandible, C. Results of future orthodontic treatment are improved anterior teeth, D. Results in less crowding",D: Results in less crowding,"Answer: D: Results in less crowding Explanation: None" "This is a real-world medical entrance exam question, please give the true answer based on the question and selection. Topic name: None, Subject name: Pathology","Which of the following statements is false in relation to myofacial pain dysfunction syndrome? Select and explain. A. Mainly affects young females, B. Is caused by muscle fatigue due to chronic oral habits as grinding and clenching, C. Treatment involves construction of occlusal guard and stress free emotional condition, D. The perioral musculature becomes hypotonic",D: The perioral musculature becomes hypotonic,"Answer: D: The perioral musculature becomes hypotonic Explanation: None" "This is a real-world medical entrance exam question, please give the true answer based on the question and selection. Topic name: None, Subject name: Pathology","Which of the following features is not seen in Crouzon syndrome? Select and explain. A. Midface hypoplasia, B. Syndactyly, C. Beak shaped nose, D. Ocular hypertelorism",B: Syndactyly,"Answer: B: Syndactyly Explanation: Crouzon syndrome can have following appearances/conditions" "This is a real-world medical entrance exam question, please give the true answer based on the question and selection. Topic name: None, Subject name: Dental","The best finish line for anterior metal ceramic crown is Select and explain. A. Chamfer with bevel, B. Heavy chamfer, C. Shoulder, D. Shoulder with bevel",D: Shoulder with bevel,"Answer: D: Shoulder with bevel Explanation: None" "This is a real-world medical entrance exam question, please give the true answer based on the question and selection. Topic name: None, Subject name: Gynaecology & Obstetrics","Kamlesh, a 2 year old girl, has Down's syndrome. Her karyotype is 21/21 translocation. What is the risk ofrecurrence in subsequent pregnancies if the father is a balanced translocation carrier : Select and explain. A. 100%, B. 50%, C. 25%, D. 0%",A: 100%,"Answer: A: 100% Explanation: Down Syndrome– (Trisomy 21) Seen in 11n 800 to 1000 newborns M/C Nonlethal trisomy Risk of Down syndrome increases with increase in maternal age At maternal age of 35 years, the risk of having a baby with Down syndrome is 1:365. to 1:400. Recurrent Risk of Down’s syndrome" "This is a real-world medical entrance exam question, please give the true answer based on the question and selection. Topic name: None, Subject name: Dental","Which of the following are the predominant connective tissue cells of the periodontal Ligament? Select and explain. A. Cementoblasts, B. Fibroblasts, C. Osteoblasts, D. Rests of Malassez",B: Fibroblasts,"Answer: B: Fibroblasts Explanation: None" "This is a real-world medical entrance exam question, please give the true answer based on the question and selection. Topic name: None, Subject name: Medicine","Which of the following is characterized by Apple Jelly Nodules? Select and explain. A. Scrofula, B. Lupus vulgaris, C. Ghon’s focus, D. Ghon's complex",B: Lupus vulgaris,"Answer: B: Lupus vulgaris Explanation: Granulomatous skin diseases may have a characteristic appearance under diascopy, such as in lupus vulgaris (cutaneous tuberculosis), in which ‘apple jelly nodules’ are typically seen on diascopy. Ref: Davidson Ed 23 Pg 1214" "This is a real-world medical entrance exam question, please give the true answer based on the question and selection. Topic name: None, Subject name: Medicine","A 50 years okl smoker and hypeensive was diagnose to have non-small cell lung carcinoma with brain metastases. He is on enalapril and hydrochlorothiazide for hypeension. On investigation, he had a serum Sodium 120 mg/dl|, Urinary Sodium ll0 mg/dl, Serum creatinine 0.8 mg/dL, Serum osmolarity 285 mOsm/L, Urinary osmolarity 35l mOsm/L, Urinary K+ 9 mg/dl, Blood sugar ll2 mgldL and BP of 150/90 mm Hg. Which of the following is the mnst probable cause for his hyponatremia? Select and explain. A. Cerebral salt wasting, B. Diuretic induced, C. SIADH, D. Pseudohyponatremia",C: SIADH,"Answer: C: SIADH Explanation: Answer-C. SIADHHyponatremia is a common electrolyte abnormality in cancer patients, and SIADH is the most common cause among patients with cancer.The term SIADH is applied to conditions with vasopressin excess.Vasopressin excess is termed inappropriate as this increase occurs despite decreased plasma osmolality.Increased vasopressin acts on renal tubules, resulting in increased absorption of water (increased total body water), concentrated urinec and decreasing serum osmolalityEUR and hyponatremiao.Edema does not occur despite increased total body waterg (due to unknown reasons)-Clinical euvolemia" "This is a real-world medical entrance exam question, please give the true answer based on the question and selection. Topic name: AIIMS 2019, Subject name: Medicine","A 40 year male complaining of hot flushes each time he baths. Hb: 20%gm, Platelet: 89,000/mL, WBC: 30,000/mL, Investigation revealed JAK2 mutation. What is the most likely diagnosis? Select and explain. A. PMF, B. CML, C. PCV, D. Essential thrombocytosis",C: PCV,"Answer: C: PCV Explanation: diagnosis Polycythemia Vera Polycythemia | platelet | WBL | | RBC | JAK2 mutation Polycythemia Vera | WBC | Basohils | histamine release - contribute to hot flushes or itching Every time patient takes bath (aquagenic pruritis) impoant diagnostic feature: Serum erythropoietin level below normal. Impoant diagnostic feature: Serum erythropoietin level below normal. Rx=Hydroxyurea Cells count control Option A- PMF Should show SplenoHepatomegaly Pancytopenia No TLC RBC drop mentioned Option B- CML ||| WBL upto 1 lakh Jak2 must along with bcr-abl fusion transcription present. Option D- Essential thrombocytosis Leads to |only platelet count initially other counts will increase Later significant increase in platelet count upto 5-6 lakh approx.: less in our of answer." "This is a real-world medical entrance exam question, please give the true answer based on the question and selection. Topic name: None, Subject name: Medicine","A 55- year old diabetic patient presents with transient obscuration in vision for 2-3 days followed by sudden loss of vision. Which of the following would be the best test to evaluate the symptoms? Select and explain. A. Serum ACE levels, B. Quantiferon-Gold TB test, C. Elevated homocysteine levels, D. Serum creatinine levels",D: Serum creatinine levels,"Answer: D: Serum creatinine levels Explanation: Answer- D. Serum creatinine levelsDM can lead to both retinopathy and nephropathy, Serum creatinine levels would be the best for kidney function and evaluation of patient symptoms." "This is a real-world medical entrance exam question, please give the true answer based on the question and selection. Topic name: None, Subject name: Dental","One of the most common side effects of phenytoin is Select and explain. A. Gingival Hyperplasia, B. Gingival Necrosis, C. Dental stains, D. Hypertension",A: Gingival Hyperplasia,"Answer: A: Gingival Hyperplasia Explanation: None" "This is a real-world medical entrance exam question, please give the true answer based on the question and selection. Topic name: AIIMS 2018, Subject name: Gynaecology & Obstetrics","A 38-years old patient who had a total abdominal hysterectomy presents 2 months postop complaining of leakage of urine from the vagina. On examination, no fistula can be identified. Dilute methylene blue (100mL) is injected into the bladder through a Foley catheter and a tampon is placed in the vagina. The patient is asked to walk for about 15 minutes, after which the patient is found to be wet, but there is no blue stain anywhere on the tampon. Which of the following is true:- Select and explain. A. 100mL was probably not enough methylene blue, and the test should be repeated with a higher volume, B. The patient probably has stress incontinence not a fistula, C. The patient probably has a ureterovaginal fistula, D. The patient most likely has small apical vesicovaginal fistula that was too small to leak the dye in 15 minutes",C: The patient probably has a ureterovaginal fistula,"Answer: C: The patient probably has a ureterovaginal fistula Explanation: Methylene blue Swab Test-- A catheter is introduced into the bladder through the urethra, The vaginal cavity is packed with three sterile swabs, 50-100 mL of dilute methylene blue dye is injected into the bladder through the catheter. If there is a VVF present, the methylene blue dye stains the uppermost swab.If the lowermost swab gets stained, the leak is from the urethra--> urethrovaginal fistula Swabs do not take up the stain, but get wet with urine, the leak is from the ureter-->ureterovaginal fistula" "This is a real-world medical entrance exam question, please give the true answer based on the question and selection. Topic name: None, Subject name: Dental","Freedom of centric relation correlates with Select and explain. A. 0.15 - 1.5 mm, B. 0.20 - 0.40 mm, C. 0.5 to 1.5 mm, D. 1 to 2 mmm",C: 0.5 to 1.5 mm,"Answer: C: 0.5 to 1.5 mm Explanation: None" "This is a real-world medical entrance exam question, please give the true answer based on the question and selection. Topic name: None, Subject name: Pharmacology","Ganglionic transmission is mediated by: Select and explain. A. Presynaptic alpha-receptors, B. Postsynaptic beta-receptors, C. Postsynaptic dopaminergic receptors, D. Postsynaptic nicotinic receptors",D: Postsynaptic nicotinic receptors,"Answer: D: Postsynaptic nicotinic receptors Explanation: Ans: D .Postsynaptic nicotinic receptors(Ref: Goodman Gilman 13/e p108, 12Ie p255; Katzung 13Ie p108, 12/e p98).Ganglionic transmission is mediated by nicotinic receptors present post-synaptically.Nicotinic acetylcholine (ACh) receptor mediates neurotransmission post-synaptically at the neuromuscular junction and peripheral autonomic ganglia.In CNS, controls release of neurotransmitters from presynaptic sites.The receptor is called the nicotinic acetylcholine receptor - Due to both alkaloid nicotine & neurotransmitter ACh can stimulate the receptor." "This is a real-world medical entrance exam question, please give the true answer based on the question and selection. Topic name: None, Subject name: Pediatrics","Which of the following is not an example of a syndrome caused by uniparental disomy? Select and explain. A. Prader-Willi syndrome, B. Angelman syndrome, C. Russell-Silver syndrome, D. Bloom syndrome",D: Bloom syndrome,"Answer: D: Bloom syndrome Explanation: Answer- D. Bloom syndromeUniparental disomy is the term used when both chromosome of a pair of chromosome is a person with normal number of chromosome, have been inherited from only one parent (Normally one chromosome of a pair is inherited from only one parent).The two chromosomes inherited from the same parent maybe identical (Uniparental isodisomy) or different (Uniparental heterodisomy).In Uniparental isodisomy, both chromosomes in the pair are identical; consequently the genes on both chromosomes are also identical.Angelman syndrome, Prader-Willi syndrome and Russell-Silver syndrome are ossociated with uniparental disomy." "This is a real-world medical entrance exam question, please give the true answer based on the question and selection. Topic name: None, Subject name: Pharmacology","One of the many mechanisms of adverse events is its increased binding to secondary targets, usually proteins. With respect to the primary target, the secondary target: Select and explain. A. Should be present in the same tissue as the primary target, B. Should have similar binding sites, C. Should have absolutely identical secondary structure, D. Should have a similar primary structure",B: Should have similar binding sites,"Answer: B: Should have similar binding sites Explanation: Ans. b. Should have similar binding sites (Ref: vvww'plosone'org)Secondary site compared to the primary site has the same sequence of binding site." "This is a real-world medical entrance exam question, please give the true answer based on the question and selection. Topic name: None, Subject name: Dental","School based dental health care for whole country is adopted by which of the following country? Select and explain. A. USA, B. NewZealand, C. Sweden, D. Australia",B: NewZealand,"Answer: B: NewZealand Explanation: None" "This is a real-world medical entrance exam question, please give the true answer based on the question and selection. Topic name: None, Subject name: Microbiology","Postrenal transplant patient presents with diarrhoea after 3 months; it was showing organism measuring 2-6 micron meter and kinoyn stain positive. Most likely it is caused by: Select and explain. A. Balantidium coli, B. Clostridium difficile, C. Cystoisospora belli, D. Cryptosporidium hominis",D: Cryptosporidium hominis,"Answer: D: Cryptosporidium hominis Explanation: Ans. d. Cryptosporidium hominisOppounistic infections are extremely common post-transplant in the period staing from 1 month post-transplant to 6 months post-transplant.Clostridium difficle is usually seen within 1 month post-transplant because of profound antibiotic use.Kinoyn stain (Modified acid fast) positivity rules out Balantidium.Now it is between cryptosporidium vs cycloisospora. Size (smaller i.e., 2-6 microns) tells us that the organism is undoubtedly cryptosporidium hominis (previously parvum)." "This is a real-world medical entrance exam question, please give the true answer based on the question and selection. Topic name: AIIMS 2019, Subject name: Pathology","Iron entry into enterocytes occurs which of the following? Select and explain. A. DMT-1, B. Ferropoin, C. Transferrin, D. Hepcidin",A: DMT-1,"Answer: A: DMT-1 Explanation: GI Luminal non-heme iron is mostly in the Fe3+ (ferric) state and must first be reduced to Fe2+ ferrous) iron by ferrireductases, such as b cytochromes and STEAP3. Fe2+ iron is then transpoed across the apical membrane of enterocytes by divalent metal transpoer-1 (DMT1)." "This is a real-world medical entrance exam question, please give the true answer based on the question and selection. Topic name: None, Subject name: Microbiology","An elderly male patient presented with fever, chest pain, and dry coughp; sputum culture showed growth on Charcoal Yeast Extract Medium, the organism is Select and explain. A. H. influenza, B. Moraxella catarrhalis, C. Legionella, D. Burkholderia cepacia",C: Legionella,"Answer: C: Legionella Explanation: None" "This is a real-world medical entrance exam question, please give the true answer based on the question and selection. Topic name: None, Subject name: Dental","Ashley-Howe model analysis is used to predict: Select and explain. A. Tooth material excess, B. Maxillo-mandibular relationships, C. Basal bone-transverse relationship, D. Growth prediction",C: Basal bone-transverse relationship,"Answer: C: Basal bone-transverse relationship Explanation: None" "This is a real-world medical entrance exam question, please give the true answer based on the question and selection. Topic name: None, Subject name: Pediatrics","A neonate on routine examination at bih was found to have hepatomegaly. Rest of the examination was essentially unremarkable. On investigations, Anti-HCMV antibodies were found to be positive. What sequelae in later life is the child at risk of? Select and explain. A. Renal failure, B. Mental retardation, C. Hepatic fibrosis, D. Sensorineural hearing loss",D: Sensorineural hearing loss,"Answer: D: Sensorineural hearing loss Explanation: Ans: D. Sensorineural hearing loss (Ref: (Ghai 8/e p272 ; Nelson 20/e p 592-1594)Positive human cytomegalovirus (HCMV) antibodies at bih - Suggestive of congenital asymptomatic CMV infection.Symptomatic child at increased risk a Develops mental retardation.Asymptomatic child is at as high as 7% risk a Develop sensorineural hearing loss.Congenital CMV infection:Characteristic signs & symptoms:Clinically manifested infections - Intrauterine growth restriction, prematurity, hepatosplenomegaly, jaundice, blueberry muffin--like rash, thrombocytopenia and purpura, microcephaly & intracranial calcifications.Neurologic problems:Chorioretinitis, sensorineural hearing loss & mild increases in cerebrospinal fluid protein." "This is a real-world medical entrance exam question, please give the true answer based on the question and selection. Topic name: None, Subject name: Pharmacology","Patient is a known case of epilepsy, taking levetiracetam 1 gm BD. He is now seizure free from 2 years but he developed agitation and anger issues interfering with day to day activities as a result of the drug intake. What should be the next best step? Select and explain. A. Stop levetiracetam and sta on a different antiepileptic, B. Discontinue the drug as he is seizure free, C. Slowly taper the drug over next 6 months, D. Continue levetiracetam since a 5-year seizure free interval is needed",C: Slowly taper the drug over next 6 months,"Answer: C: Slowly taper the drug over next 6 months Explanation: Answer- C. Slowly taper the drug over next 6 monthsStopping or switching an antiepileptic is based on seizure free period & compliance or adverse effects of the drug.Withdrawal of therapy should be gradual over 2-3 monlhs, in the question 6 months. Anti-epileptic drug therapy shouldnever he stopped abruptly. Even if new drug is to be added/replaced, the previous drug should be gradually stoppedotherwise it can lead to breakthrough seizures." "This is a real-world medical entrance exam question, please give the true answer based on the question and selection. Topic name: None, Subject name: Social & Preventive Medicine","Which of the following statements regarding the National Programme for Non-communicable diseases-Cancer, Diabetes, Cardiovascular diseases (CVD) and stroke (NPCDCS) is true? Select and explain. A. The plan is to implement the programme in 10 districts across the 5 states during 2010-2012, B. The diagnosis or treatment of non-communicable diseases to be carried out at the subcentre level, C. Coronary care unit and cancer care facility to be established at the district level hospital, D. All the components of the programme to be addressed separately- cancer, diabetes, CVD and stroke",C: Coronary care unit and cancer care facility to be established at the district level hospital,"Answer: C: Coronary care unit and cancer care facility to be established at the district level hospital Explanation: Ans. c. Coronary care unit and cancer care facility to be established at the district level hospital (Ref: NPCDCS Operational Guidelines, DGHS, GOI, Page 6)The NPCDCS program has two components viz. (i) Cancer and (ii) Diabetes, CVDs and Stroke." "This is a real-world medical entrance exam question, please give the true answer based on the question and selection. Topic name: None, Subject name: Physiology","Contractile element in myofibril is: Select and explain. A. H band, B. Sarcoplasm, C. Sarcomere, D. A line",C: Sarcomere,"Answer: C: Sarcomere Explanation: None" "This is a real-world medical entrance exam question, please give the true answer based on the question and selection. Topic name: None, Subject name: Surgery","All of the following complications are more common in ventouse assisted delivery than forceps except: Select and explain. A. Subgaleal hemorrhage, B. Cephalhematoma, C. Intracranial hemorrhage, D. Transient lateral rectus palsy",D: Transient lateral rectus palsy,"Answer: D: Transient lateral rectus palsy Explanation: Answer- D. Transient lateral rectus palsyTransient lateral rectus palsy is more common inforceps assisted delivery.Subgaleal hemonhage, cephalhematoma and intracranial hemonhage are more common in ventouse assisted delivery." "This is a real-world medical entrance exam question, please give the true answer based on the question and selection. Topic name: None, Subject name: Medicine","All of the following are true about aspiration pneumonia except: Select and explain. A. Aspiration of 20-30 mL of contents with pH < 2.5 is required, B. Fungal infection is the common cause of pneumonia, C. Posterior segment of the right upper lobe is most commonly affected in the recumbent position, D. Aspiration responsible for 5-15% of community acquired pneumonia",B: Fungal infection is the common cause of pneumonia,"Answer: B: Fungal infection is the common cause of pneumonia Explanation: Answer- B.Fungal infection is the common cause of pneumoniaMixed polymicrobial infection involving both aerobes and anaerobes are the common cause of aspiration pneumonia, not the fungal infection.Incidence: It is common and may account for up to 15% of patients with community-acquired pneumonia.Gastric pH of 2.5 or less with a gastric contents volume greater than 25 ml are critical values for causing aspiration pneumonia.The posterior segments of the upper lobe and apical segments of lower lobes are most commonly involved when aspiration occurs in a supine position.The basal segments of the lower lobes are usually affected in patients who aspirate in an upright or semirecumbent position." "This is a real-world medical entrance exam question, please give the true answer based on the question and selection. Topic name: None, Subject name: Pathology","Which of the following is not associated with haemorrhage? Select and explain. A. Ecchymosis, B. Petechiae, C. Melanosis, D. Purpura",C: Melanosis,"Answer: C: Melanosis Explanation: None" "This is a real-world medical entrance exam question, please give the true answer based on the question and selection. Topic name: AIIMS 2018, Subject name: Pharmacology","Dapsone is used for treatment of bacterial and fungal infections as well as for immunomodulatory actions. What is mechanism of dapsone for these indication? Select and explain. A. Inhibition of cell wall synthesis, B. Inhibition of ergosterol in cell membranes, C. Inhibition of protein synthesis, D. Competition with PABA in folic acid synthesis",D: Competition with PABA in folic acid synthesis,"Answer: D: Competition with PABA in folic acid synthesis Explanation: Sulfonamides (including Dapsone and PAS) : Competitively inhibit folic acid synthase enzyme as these are PABA analogues. Trimethoprim, Pyrimethamine : Inhibit Dihydro folate reductase enzyme (DHFRase). Inhibition of ergosterol in cell membrane (in fungus and leishmania): Amphotericin B." "This is a real-world medical entrance exam question, please give the true answer based on the question and selection. Topic name: None, Subject name: Dental","In processing methyl methacrylate, the resin may show porosity if the flask is placed too soon. Porosity most likely occur Select and explain. A. Throughout the denture, B. Near the borders, C. In the thickest part, D. On the denture surface",C: In the thickest part,"Answer: C: In the thickest part Explanation: None" "This is a real-world medical entrance exam question, please give the true answer based on the question and selection. Topic name: None, Subject name: Anatomy","Pre-maxillary hard palate is supplied by which nerve Select and explain. A. Nasopalatine nerve, B. Pharyngeal branch of glossopharyngeal nerve, C. Greater palatine nerve, D. Tensor palatine nerve",A: Nasopalatine nerve,"Answer: A: Nasopalatine nerve Explanation: None" "This is a real-world medical entrance exam question, please give the true answer based on the question and selection. Topic name: None, Subject name: Microbiology","23 serotypes pneumococcal vaccine Most useful in Select and explain. A. Cystic fibrosis, B. Recurrent otitis media & sinusitis, C. Child less than 2 years, D. Sickle cell anaemia",D: Sickle cell anaemia,"Answer: D: Sickle cell anaemia Explanation: Ans-D. Sickle cell anaemia* PPSV23 (Pneumovax or Pnu-Immune) is the most widely available formulation and contains 23 pneumococcal polysaccharides* A single dose of the 23-valent pneumococcal polysaccharide vaccine (PPSV23) is indicated for adults (ages 19 to 64 years)* Vaccination with both PCV13 and PPSV23 is indicated in individuals with impaired splenic function for eg- Sickle cell disease or other hemoglobinopathy" "This is a real-world medical entrance exam question, please give the true answer based on the question and selection. Topic name: None, Subject name: Dental","Fourth generation apex locater based on Select and explain. A. Resistance, B. lmpedence, C. Frequency, D. Multiple frequency ratio",D: Multiple frequency ratio,"Answer: D: Multiple frequency ratio Explanation: None" "This is a real-world medical entrance exam question, please give the true answer based on the question and selection. Topic name: None, Subject name: Surgery","Biopsy specimens removed for examinations are immediately placed in: Select and explain. A. 10% ethanol, B. 10% formalin, C. Hydrogen peroxide, D. 1% formalin",B: 10% formalin,"Answer: B: 10% formalin Explanation: None" "This is a real-world medical entrance exam question, please give the true answer based on the question and selection. Topic name: None, Subject name: Surgery","True open bite is caused by Select and explain. A. Horizontal fracture of the maxilla, B. Unilateral fracture of mandibular angle, C. Fracture of the coronoid process of left side of mandible, D. Fracture of mandibular symphysis",A: Horizontal fracture of the maxilla,"Answer: A: Horizontal fracture of the maxilla Explanation: None" "This is a real-world medical entrance exam question, please give the true answer based on the question and selection. Topic name: None, Subject name: Dental","Greatest potential hazard of mercury toxicity occurs due to: Select and explain. A. Skin contact with mercury, B. Inhalation of mercury vapours, C. During amalgam restoration, D. Ingestion of amalgam scrap during removal",B: Inhalation of mercury vapours,"Answer: B: Inhalation of mercury vapours Explanation: None" "This is a real-world medical entrance exam question, please give the true answer based on the question and selection. Topic name: None, Subject name: Pediatrics","Which of the following is an indiction for tonsillectomy –a) Rheumatic feverb) Glomerulonephritisc) Recurrent upper respiratory infectiond) Persistent carrier of diptheria bacilli Select and explain. A. ab, B. cd, C. bd, D. ac",B: cd,"Answer: B: cd Explanation: Indications of tonsillectomy Recurrent sore throat —› If more than six attacks of tonsillitis in a year for two consecutive years. Tonsillar or peritonsillar abscess              o Retention cyst of tonsil                  o Diphtheria carriers Tonsillolith                                                   o Suspicious malignancy                  o Obustructive sleep apnea" "This is a real-world medical entrance exam question, please give the true answer based on the question and selection. Topic name: AIIMS 2017, Subject name: Physiology","Which of the following is not an effect of efferent aeriole constriction : Select and explain. A. Decreased GFR, B. Increased glomerular hydrostatic pressure, C. Decreased blood flow in peritubular vessels, D. Increased oncotic pressure in peritubular vessels",A: Decreased GFR,"Answer: A: Decreased GFR Explanation: Constriction of efferent aeriole has biphasic effect on GFR. At moderate levels of constriction, It increases glomerular hydrostatic pressure resulting in increasing GFR If efferent aeriolar constriction is severe; It decreases renal blood flow; filtration fraction & glomerular colloid osmotic pressure increases as efferent aeriolar resistance increases resulting in colloid osmotic pressure exceeds increase in glomerular capillary hydrostatic pressure resulting in decrease in GFR. Blood from efferent aerioles enter peritubular capillaries, so if efferent aeriole is constricted, less blood enters peritubular capillaries, which results in increase in oncotic pressure in peri tubular capillaries." "This is a real-world medical entrance exam question, please give the true answer based on the question and selection. Topic name: None, Subject name: Surgery","Cleft lip is due to non-union of Select and explain. A. Maxillary process with MNP, B. MNP - LNP, C. MNP - MNP, D. All of the above",A: Maxillary process with MNP,"Answer: A: Maxillary process with MNP Explanation: None" "This is a real-world medical entrance exam question, please give the true answer based on the question and selection. Topic name: None, Subject name: Biochemistry","Sickle cell anemia is a - Select and explain. A. Gene deletion, B. Gene modification, C. Point mutation, D. Frame shift mutation",C: Point mutation,"Answer: C: Point mutation Explanation: Answer- c- point mutation.Sickle-cell anemia is caused by a point mutation in the b-globin chain of hemoglobin, causing the hydrophilic amino acid glutamic acid to be replaced with the hydrophobic amino acid valine at the sixth position.It is caused by a point mutation (Base substitution mutation) at the sixth position of the b-globin chain leading to subsitution of a valine residue for a glutamic acid residue resulting in sickle hemoglobin (HbS).Sickle cell anemia is an autosomal recessive disorder." "This is a real-world medical entrance exam question, please give the true answer based on the question and selection. Topic name: None, Subject name: Microbiology","All are true about widal test except Select and explain. A. Base lines differ depending on the endemicity of the disease, B. O antibodies last longer and hence is not indicative of recent infection, C. H antigen cannot differentiate between subtypes, D. High titre value in a single widal test is not confirmative",B: O antibodies last longer and hence is not indicative of recent infection,"Answer: B: O antibodies last longer and hence is not indicative of recent infection Explanation: None" "This is a real-world medical entrance exam question, please give the true answer based on the question and selection. Topic name: AIIMS 2018, Subject name: Surgery","What is the name of the given instrument, used for harvesting the graft from healthy area in split thickness skin graft? Select and explain. A. Dermatome, B. Silver's knife, C. Catlin amputating knife, D. Humby knife",D: Humby knife,"Answer: D: Humby knife Explanation: Humby Knife A knife with a roller attached, used for cutting skin grafts of varying thickness The distance between the roller and blade of knife can be varied by means of a calibration device." "This is a real-world medical entrance exam question, please give the true answer based on the question and selection. Topic name: None, Subject name: Microbiology","A person working in an abattoir presented with a papule on hand, which turned in to an ulcer. Which will best help in diagnosis Select and explain. A. Polychrome Methylene blue, B. Carbol Fuschin, C. Acid Fast Stain, D. Calcoflour White",A: Polychrome Methylene blue,"Answer: A: Polychrome Methylene blue Explanation: None" "This is a real-world medical entrance exam question, please give the true answer based on the question and selection. Topic name: None, Subject name: Gynaecology & Obstetrics","A midwife at a PHC did per vaginal examination of a women in labor with 8 cm cervical dilation and 70% cervical effacement with the fetal head at +1 station. This +1 station implies the position of fetal head is- Select and explain. A. 1 cm above the ischial spine, B. 1 cm below the ischial spine, C. At the level of ischial spine, D. 1 cm below the cervical os",B: 1 cm below the ischial spine,"Answer: B: 1 cm below the ischial spine Explanation: Answer-B. 1 cm below the ischial spineStation describes descent of the fetal biparietal diameter in relation to a line drawn between maternal ischial spines. Thus+1 station implies fetal head is 1 cm below the ischial spine." "This is a real-world medical entrance exam question, please give the true answer based on the question and selection. Topic name: None, Subject name: Biochemistry","High content of cholesterol is present in: Select and explain. A. Coconut oil, B. Egg yolk, C. Hydrogenated fat, D. Ghee (unsaturated)",B: Egg yolk,"Answer: B: Egg yolk Explanation: None" "This is a real-world medical entrance exam question, please give the true answer based on the question and selection. Topic name: None, Subject name: Dental","Tooth with smallest root Select and explain. A. Mandibular central incisor, B. Mandibular lateral incisor, C. Maxillary lateral incisor, D. Maxillary 1st premolar",A: Mandibular central incisor,"Answer: A: Mandibular central incisor Explanation: None" "This is a real-world medical entrance exam question, please give the true answer based on the question and selection. Topic name: None, Subject name: Dental","The function of a sprue is to Select and explain. A. Form an opening for molten metal to enter the mold, B. Help polish cast restorations, C. Eliminate air bubbles on the wax pattern, D. Reproduce fine detail",A: Form an opening for molten metal to enter the mold,"Answer: A: Form an opening for molten metal to enter the mold Explanation: None" "This is a real-world medical entrance exam question, please give the true answer based on the question and selection. Topic name: None, Subject name: Dental","Access cavity of mandibular 1st molar is: Select and explain. A. Oval, B. Rounded triangle, C. Rhomboid, D. None of the above",B: Rounded triangle,"Answer: B: Rounded triangle Explanation: 1st choice for access cavity preparation in mandibular 1st molar is trapezoidal. 2nd choice is rounded triangle." "This is a real-world medical entrance exam question, please give the true answer based on the question and selection. Topic name: None, Subject name: Dental","A pier abutment is: Select and explain. A. Periodontally weak abutment, B. With an edentulous space on mesial and distal sides of the abutment, C. Edentulous space on one side of the abutment, D. Abutment adjacent to edentulous space",B: With an edentulous space on mesial and distal sides of the abutment,"Answer: B: With an edentulous space on mesial and distal sides of the abutment Explanation: None" "This is a real-world medical entrance exam question, please give the true answer based on the question and selection. Topic name: None, Subject name: Social & Preventive Medicine","According to the new RNTCP guidelines, the following is not a suspect of tuberculosis Select and explain. A. Confirmed extra-pulmonary tuberculosis patient with cough of 2 weeks or more, B. HIV-positive patient with cough of any duration, C. Contacts of sputum positive tuberculosis patient with cough of any duration, D. Any individual having cough of duration 2 weeks or more",A: Confirmed extra-pulmonary tuberculosis patient with cough of 2 weeks or more,"Answer: A: Confirmed extra-pulmonary tuberculosis patient with cough of 2 weeks or more Explanation: Ans: A. Confirmed extra-pulmonary tuberculosis patient with cough of 2 weeks or more(Ref: Park 24/e p188-189, 23/e p178: 22/e p 168; htip://tbcindia.nic.in)A contact of confirmed extra-pulmonary tuberculosis patient - TB suspect.If persistent cough of any duration (2 weeks is not required).TB suspects:Pulmonary TB Suspect:Defined as:Any individual having cough of >2 weeks.Contacts of smear positive TB patients having cough of any duration.Suspected/confirmed extra-pulmonary TB patient having cough of any duration.HIV-positive patient having cough of any duration.MDR-TB Suspect:Any TB patient who fails an RNTCP Category failing I or III treatment regimen.Any RNTCP Category II patient with sputum smear positive at end of 4th month of treatment or later.Close contacts of MDR-TB patients with smear positive pulmonary TB." "This is a real-world medical entrance exam question, please give the true answer based on the question and selection. Topic name: None, Subject name: Pharmacology","All are topical hemostatic agents except? Select and explain. A. Bone wax & patty, B. HemCon bandage, C. Quikclot, D. Fixclot",D: Fixclot,"Answer: D: Fixclot Explanation: Ans. D. Fixclot Topical hemostatic agents are used when surgical hemostasis is inadequate or impractical. The majority of routine, elective operations are performed in patients with normal hemostasis and with minimal blood loss.The two main categories of topical hemostatic agents are physical agents, which promote hemostasis using a passive substrate, and biologically active agents, which enhance coagulation at the bleeding site" "This is a real-world medical entrance exam question, please give the true answer based on the question and selection. Topic name: None, Subject name: Dental","Which of the following pulp horn of primary molar is commonly exposed during restorative procedures: Select and explain. A. Mesiobuccal puplhorn of 1st molars., B. Distobuccal pulp horn of 1st molars., C. Mesiobuccal pulp horn of 2nd molars., D. Distobuccal pulp horn of 2nd molars.",A: Mesiobuccal puplhorn of 1st molars.,"Answer: A: Mesiobuccal puplhorn of 1st molars. Explanation: The mesiobuccal pulphorn of permanent maxillary 1st molar is commonly exposed during cavity preparation. Mesiolingual pulphorn of permanent mandibular 1st molar is commonly exposed during cavity preparation. Mesiobuccal pulphorn of primary 1st molars is commonly exposed during restorative procedures on primary molars." "This is a real-world medical entrance exam question, please give the true answer based on the question and selection. Topic name: None, Subject name: Dental","Rate of Dentinal fluid permeability depends on all except? Select and explain. A. Temperature variation, B. Pressure gradient across dentinal tubule, C. Increase viscosity of dentinal fluid, D. Rate of removal of substances by blood vessels",A: Temperature variation,"Answer: A: Temperature variation Explanation: None" "This is a real-world medical entrance exam question, please give the true answer based on the question and selection. Topic name: None, Subject name: Surgery","Extraction of disto-angular impaction of mandibular 3rd molar can cause: Select and explain. A. Slippage in lingual pouch, B. # of ramus of mandible, C. Excessive Haemorrhage, D. Dry socket",B: # of ramus of mandible,"Answer: B: # of ramus of mandible Explanation: None" "This is a real-world medical entrance exam question, please give the true answer based on the question and selection. Topic name: AIIMS 2018, Subject name: Pharmacology","Acetazolamide is given to a patient of angle closure glaucoma. It is a non- competitive inhibitor of carbonic anhydrase enzyme. Which of the following should be the effect of this drug? Select and explain. A. Decrease in Vmax, B. Decrease in Km, C. Decrease in both Km and Vmax, D. No change in Vmax",A: Decrease in Vmax,"Answer: A: Decrease in Vmax Explanation: Enzyme inhibition is of 3 types: Type of inhibition Km Vmax Competitive inhibition Increases No change Non competitive inhibition No change Decreases Uncompetitive inhibition Decreases Decreases In question non competitive inhibitor is given, so answer is decrease in Vmax." "This is a real-world medical entrance exam question, please give the true answer based on the question and selection. Topic name: None, Subject name: Biochemistry","Which of the following does not require 5'capping? Select and explain. A. tRNA of alanine, B. mRNA for histone, C. U6 snRNA, D. siRNA",A: tRNA of alanine,"Answer: A: tRNA of alanine Explanation: Ans: A. tRNA of alanineSmall nuclear RNAs contain a unique 5'-caps. Sm-class snRNAs are found with 5'-trimethylguanosine caps, while Lsm-class snRNAs are found with S'-monomethyl phosphate caps.mRNAs do have a 7-methylguanylate cap, abbreted m7G.tRNAs and rRNAs don't require 5' capping. They have other modifications." "This is a real-world medical entrance exam question, please give the true answer based on the question and selection. Topic name: None, Subject name: Surgery","Peau d'orange in carcinoma breast is due to: Select and explain. A. Obstruction of sub-dermal lymphatics, B. Infiltration of Cooper's ligament, C. Hematogenous dissemination, D. Hematogenous dissemination",A: Obstruction of sub-dermal lymphatics,"Answer: A: Obstruction of sub-dermal lymphatics Explanation: Ans: A. Obstruction of sub-dermal lymphaticsPeau-d-orange is due to cutaneous lymphatic edema.Caused by obstruction of subdermal lymphatics. As the disease advances locally there may be skin involvement with peau d'orange or frank ulceration and fixation to the chest wall. This is described as cancer-en-cuirasse when the disease progresses around the chest wall." "This is a real-world medical entrance exam question, please give the true answer based on the question and selection. Topic name: None, Subject name: Pathology","True about adult polycystic kidney disease is all, except: Select and explain. A. Autosomal dominant inheritance, B. Hypertension is rare, C. Can be associated with cysts in liver, lungs and pancreas, D. Pyelonephritis is common",B: Hypertension is rare,"Answer: B: Hypertension is rare Explanation: Hypertension is common in patients with autosomal dominant polycystic kidney disease. It is present in 75% of adult patients and 25% of children." "This is a real-world medical entrance exam question, please give the true answer based on the question and selection. Topic name: None, Subject name: Pharmacology","Propranolol is drug of choice for - Select and explain. A. Ulcerated infantile hemangioma, B. Lymphangioma, C. Pyogenic granuloma, D. Capillary malformation",A: Ulcerated infantile hemangioma,"Answer: A: Ulcerated infantile hemangioma Explanation: Ans. A. Ulcerated infantile hemangiomaRef: Ulceration--the most frequent complication of IH--tends to heal poorly and is associated with pain, bleeding, infection, and scarring.Mainstay treatment modalities include propranolol (b-blocker) and coicosteroids, whose effectiveness is countered by a need for long-term medication and risk of systemic adverse effects and ulcer recurrence." "This is a real-world medical entrance exam question, please give the true answer based on the question and selection. Topic name: None, Subject name: Gynaecology & Obstetrics","During pregnancy baby can be affected in utero in all except: Select and explain. A. Candida, B. Syphilis, C. Toxoplasmosis, D. Polio",D: Polio,"Answer: D: Polio Explanation: In case of polio only feco-oral transmission is known, no placental transmission has been reported yet All other disease mentioned may be transmitted to fetus by mother I am in doubt about candida, because although congenital candidiasis can occur, the mode of transmission is not transplacental but ascending infection from external genitals of mother or during parturition. But as far as the answer is concerned it is undoubtedly “POLIO”." "This is a real-world medical entrance exam question, please give the true answer based on the question and selection. Topic name: None, Subject name: Medicine","Long term assessment of glucose control can be made by: Select and explain. A. Estimation of post prandial blood sugar, B. Estimation of fasting blood sugar, C. Estimation of urine sugar level, D. Estimation of blood level of glycosylated hemoglobin",D: Estimation of blood level of glycosylated hemoglobin,"Answer: D: Estimation of blood level of glycosylated hemoglobin Explanation: None" "This is a real-world medical entrance exam question, please give the true answer based on the question and selection. Topic name: None, Subject name: Pathology","Mosaic pattern of bone is seen in radiographic features of: Select and explain. A. Fibrous dysplasia, B. Paget's disease, C. Osteopetrosis, D. Osteogenesis imperfecta",B: Paget's disease,"Answer: B: Paget's disease Explanation: None" "This is a real-world medical entrance exam question, please give the true answer based on the question and selection. Topic name: None, Subject name: Dental","Fourth generation meth-acrylate based resin sealer include? Select and explain. A. Endorez, B. Resilon, C. Hydron, D. Real Seal E",D: Real Seal E,"Answer: D: Real Seal E Explanation: None" "This is a real-world medical entrance exam question, please give the true answer based on the question and selection. Topic name: AIIMS 2018, Subject name: Gynaecology & Obstetrics","Maternal moality ratio expressed in:- Select and explain. A. Maternal death per 10,000 lives, B. Maternal death per 100,000 lives, C. Maternal death per 1,000,000 lives, D. Maternal death per 100 lives","B: Maternal death per 100,000 lives","Answer: B: Maternal death per 100,000 lives Explanation: None" "This is a real-world medical entrance exam question, please give the true answer based on the question and selection. Topic name: None, Subject name: Pharmacology","Which induction anaesthetic agent has antiemetic effect? Select and explain. A. etomidate, B. propofol, C. thiopentone, D. ketamine",B: propofol,"Answer: B: propofol Explanation: None" "This is a real-world medical entrance exam question, please give the true answer based on the question and selection. Topic name: None, Subject name: Microbiology","A 10 year old boy following a road traffic accident presents to the casualty with contaminated wound over the left leg. He has received his com- plete primary immunization before preschool age and received a booster of DT at school entry age. All of following can be done except : Select and explain. A. Injection of TT, B. Injection of human antiserum, C. Broad spectrum antibiotics, D. Wound debridement and cleaning",B: Injection of human antiserum,"Answer: B: Injection of human antiserum Explanation: None" "This is a real-world medical entrance exam question, please give the true answer based on the question and selection. Topic name: None, Subject name: Medicine","In Myocardial infarction, the major pathology is Select and explain. A. Ischemia caused by dynamic obstruction of a coronary artery., B. Myocardial necrosis caused by acute occlusion of a coronary artery., C. Ischemia due to fixed atheromatous stenosis of one or more coronal artery., D. Altered conduction due to ischemia or infarction.",B: Myocardial necrosis caused by acute occlusion of a coronary artery.,"Answer: B: Myocardial necrosis caused by acute occlusion of a coronary artery. Explanation: None" "This is a real-world medical entrance exam question, please give the true answer based on the question and selection. Topic name: None, Subject name: Dental","Localized Juvenile Periodontitis- primary pathogen is Select and explain. A. Acinobacillus Actinomycetemcomitans, B. Prevotella intermedia, C. Mycoplasma, D. all the above",D: all the above,"Answer: D: all the above Explanation: None" "This is a real-world medical entrance exam question, please give the true answer based on the question and selection. Topic name: None, Subject name: Pediatrics","A male child with Fanconi syndrome with nephrocalcinosis has a variant of dent disease. All are true except : Select and explain. A. Hypercalciuria, B. Proteinuria, C. Similar presentation in father, D. Rickets",C: Similar presentation in father,"Answer: C: Similar presentation in father Explanation: Dent's disease : Dent's disease, a familial proximal tubular syndrome, is an X-linked recessive disorder of proximal tubules characterized by : Hypercalciuria and nephrocalcinosis and nephrolithiasis. Low-molecular-weight proteinuria. Metabolic bone disease/Rickets. Progressive renal failure. Marked inale predominance. In addition to above features, other renal proximal tubular defect can cause: (i) Fanconi syndrome (Aminoaciduria, proteinuria, phosphaturia). (i) Glycosuria, uricosuria and kalliuresis. (iii) Impaired urinary acidification. Dent's disease occurs due to mutations that inactivate a voltage-gated chloride transporter, CLC-.5. In some cases, it is associated with mutations in the OCRL-1 gene that is also mutated in the oculocerebral syndrome of Lowe. Coming back to question. Dent's disease is an X-linked renal tubular disorder. X-linked disease cannot be transmitted from male to male i.e., from father to son." "This is a real-world medical entrance exam question, please give the true answer based on the question and selection. Topic name: None, Subject name: Physiology","Which of the following is not involved in blood clotting? Select and explain. A. Calcium, B. Prothrombin, C. Fibrinogen, D. Fe+2",D: Fe+2,"Answer: D: Fe+2 Explanation: None" "This is a real-world medical entrance exam question, please give the true answer based on the question and selection. Topic name: AIIMS 2020, Subject name: Biochemistry","DNA Methylation is not related with? Select and explain. A. Mismatch repair, B. DNA Replication, C. Gene silencing, D. Capping",D: Capping,"Answer: D: Capping Explanation: Capping of mRNA in eukaryotes: 7-Methyl Guanosine cap is added at the 5' end of mRNA by enzyme Guanylyl Transferase. So, this is RNA methylation, not DNA. DNA methylation Epigenetic mechanism Transfer of methyl group to C5 position of cytosine to form 5-methyl cytosine. DNA replication Dnmt (DNA methyl transferases) during DNA replication to copy DNA methylation pattern from parental strand to daughter strand Mismatch repair -Identify parent strand with help of CH3" "This is a real-world medical entrance exam question, please give the true answer based on the question and selection. Topic name: None, Subject name: Dental","While preparing a Class II cavity on a maxillary first molar which of the pulp horns are likely to get exposed: Select and explain. A. Mesiolingual and mesiobuccal, B. Distolingual and distobuccal, C. Mesiolingual and distobuccal, D. Distolingual and mesiobuccal",A: Mesiolingual and mesiobuccal,"Answer: A: Mesiolingual and mesiobuccal Explanation: None" "This is a real-world medical entrance exam question, please give the true answer based on the question and selection. Topic name: None, Subject name: Dental","Additional period required for enamel calcification after eruption into oral cavity Select and explain. A. 1 yr, B. 2 yr, C. 3 yr, D. 4 yr",B: 2 yr,"Answer: B: 2 yr Explanation: None" "This is a real-world medical entrance exam question, please give the true answer based on the question and selection. Topic name: None, Subject name: Pharmacology","All are Gp lIbilIla inhibitors except: Select and explain. A. Prasugrel, B. Abciximab, C. Tirofiban, D. Eptifibatide",A: Prasugrel,"Answer: A: Prasugrel Explanation: Ans: A. Prasugrel(Ref Goodman Gilman 13/e p595, 12/e p870; Katzung 13/e p595, 12Ie p612; KDT 7/e p631)Prasugrel reduce platelet aggregation by inhibiting the ADP pathway of platelets. It irreversibly blocks the ADP receptor on platelets.Targets for platelet inhibitory drugs:Inhibition of prostaglandin synthesis (aspirin).Inhibition of ADP- induced platelet aggregation (clopidogrel, prasugrel, ticlopidine).Blockade of glycoprotein Ilb/IIIa receptors on platelets (abciximab, tirofiban, and eptifibatide). Dipyridamole & cilostazol - Additional antiplatelet drugs." "This is a real-world medical entrance exam question, please give the true answer based on the question and selection. Topic name: None, Subject name: Forensic Medicine","The active principle of white oleander is: Select and explain. A. Nicotine, B. Nerin, C. Abrine, D. Pilocarpine",B: Nerin,"Answer: B: Nerin Explanation: Ans. b. NerinNerium odorum (white oleander, kanero) grows wild in India.. Seed pod is slim, cylindrical, ribbed, up to l5 cm. long, tums brown, dries and splits, releasing small seeds tipped withbrown hair.. All pas of the plant including nectar are poisonousq.. It contains several cardiac glycosides, primarily:- Oleandrosidee (Oleandrin) and Nerioside (Nerin), both of which resembles digitalis in action- Folinerin- Rosagenin" "This is a real-world medical entrance exam question, please give the true answer based on the question and selection. Topic name: None, Subject name: Dental","The yellowish white soft sticky deposit loosely adherent present on tooth surface is Select and explain. A. Materia alba, B. Food debris, C. Plaque, D. Calculus",A: Materia alba,"Answer: A: Materia alba Explanation: None" "This is a real-world medical entrance exam question, please give the true answer based on the question and selection. Topic name: None, Subject name: Pediatrics","A term gestation newborn developed respiratory distress. Which of the following would favour Respiratory distress syndrome (HIVID) ? Select and explain. A. History of receiving antenatal corticosteroids, B. Air bronchogram on chest x–ray, C. Onset of distress after 6 hours of birth, D. Term birth",B: Air bronchogram on chest x–ray,"Answer: B: Air bronchogram on chest x–ray Explanation: chest x-ray of an infant with RDS is characterized by atelectasis, air bronchograms, and a diffuse reticular-granular pattern commonly referred to as ""ground glass"". The chest x-ray may progress to a complete ""white out"" with severe disease. About other options Antenatal corticosteroids are given in pre-term (pre-mature) pregnancies not in term pregnancies. Further, antenatal corticosteroids are given to prevent RDS (HMD) --> after antenatal corticosteroids administration, risk of HMD is reduced. Respiratory distress occurs within first 6 hours. HMD occurs in pre-term neonate." "This is a real-world medical entrance exam question, please give the true answer based on the question and selection. Topic name: None, Subject name: Pathology","‘Fleur -de- lys’ term is used for: Select and explain. A. Dens invaginatus, B. Denticle, C. Odontome, D. Supernumerary tooth",A: Dens invaginatus,"Answer: A: Dens invaginatus Explanation: None" "This is a real-world medical entrance exam question, please give the true answer based on the question and selection. Topic name: None, Subject name: Medicine","Renal feed for CKD patients- Select and explain. A. Low calorie low volume, B. Low calorie high volume, C. High calorie low volume, D. High calorie high volume",C: High calorie low volume,"Answer: C: High calorie low volume Explanation: Answer- C. High calorie low volumeLimiting fluids, eating a low-protein diet, limiting salt, potassium, phosphorous, and other electrolytes, and getting enough calories if you are losing weight." "This is a real-world medical entrance exam question, please give the true answer based on the question and selection. Topic name: None, Subject name: Dental","Point used as termination of Root canal procedure is Select and explain. A. Apical constriction, B. Apical foramen, C. Both, D. None",A: Apical constriction,"Answer: A: Apical constriction Explanation: None" "This is a real-world medical entrance exam question, please give the true answer based on the question and selection. Topic name: None, Subject name: Biochemistry","Kwashiorkar , true is Select and explain. A. Deficiency of protein with sufficient calorie intake, B. Occurs in children between 6 months and 3 years of age, C. Subcutaneous fat preserved but atrophy of different tissues, D. All of the above",D: All of the above,"Answer: D: All of the above Explanation: None" "This is a real-world medical entrance exam question, please give the true answer based on the question and selection. Topic name: None, Subject name: Dental","Lenolin is added in ZOE paste to: Select and explain. A. Decrease flow, B. Increase flow, C. Accelerate reaction, D. Decrease irritation due to eugenol",B: Increase flow,"Answer: B: Increase flow Explanation: None" "This is a real-world medical entrance exam question, please give the true answer based on the question and selection. Topic name: None, Subject name: Dental","Which of the following is not transmitted by lice Select and explain. A. Q fever, B. Trench fever, C. Relapsing fever, D. Epidemic typhus",A: Q fever,"Answer: A: Q fever Explanation: None" "This is a real-world medical entrance exam question, please give the true answer based on the question and selection. Topic name: None, Subject name: Physiology","What will be the oxygen carrying capacity of an 18-year-old patient with a hemoglobin of 14 g/dL? Select and explain. A. 7, B. 14, C. 18, D. 28",C: 18,"Answer: C: 18 Explanation: Answer- C. 18Each gram of hemoglobin can bind with a maximum of 1.34 milliliters of oxygen (1.39 milliliters when the hemoglobin is chemically pure, but impurities such as methemoglobin reduce this).Here, Hb = 14.Hence, oxygen carrying capacity: l.34 x 14 = 18 mLO2/dL" "This is a real-world medical entrance exam question, please give the true answer based on the question and selection. Topic name: None, Subject name: Anatomy","All the following muscles of larynx are supplied by the recurrent laryngeal nerve except: Select and explain. A. Cricohyoid, B. Cricothyroid, C. Arytenoid, D. Aryepiglotticus",B: Cricothyroid,"Answer: B: Cricothyroid Explanation: None" "This is a real-world medical entrance exam question, please give the true answer based on the question and selection. Topic name: None, Subject name: Biochemistry","Which of the following techniques is based on RNA? Select and explain. A. PCR, B. Sanger's technique, C. Next generation sequencing, D. Western blot",A: PCR,"Answer: A: PCR Explanation: Ans: A. PCRReverse transcription-polymerase chain reaction (-PCR) It is a laboratory technique combining reverse transcription of RNA into DNA (in this context called complementary DNA or cDNA) and amplification of specific DNA targets using polymerase chain reaction (PCR).It is primarily used to measure the amount of a specific RNA. This is achieved by monitoring the amplification reaction using fluorescence, a technique called real-time PCR or quantitative PCR (qPCR). Combined -PCR and qPCR are routinely used for the analysis of gene expression and quantification of viral RNA in research and clinical settings.A method used to quantitate mRNA levels that rely upon the first step of cDNA copying of mRNAs catalyzed by reverse transcriptase before PCR amplification and quantitation. Sanger sequencing,Also known as the chain termination method, is a technique for DNA sequencing-based upon the selective incorporation of chain-terminating dideoxynucleotides (ddNTPs) by DNA polymerase during in vitro DNA replication.Next-generation sequencing:Modified methods of genome sequencing. It has significantly reduced the cost and time taken for whole-genome sequencing.It is a technique similar to Sanger sequencing, done on DNA" "This is a real-world medical entrance exam question, please give the true answer based on the question and selection. Topic name: None, Subject name: Pharmacology","Minimum concentration of oxygen needed in NO analgesia Select and explain. A. 10%, B. 20%, C. 30%, D. 40%",C: 30%,"Answer: C: 30% Explanation: None" "This is a real-world medical entrance exam question, please give the true answer based on the question and selection. Topic name: None, Subject name: Medicine","In rheumatoid arthritis there is inflammation of: Select and explain. A. Cartilage, B. Synovial membrane, C. Sclerosis of joints, D. Articular bone",B: Synovial membrane,"Answer: B: Synovial membrane Explanation: None" "This is a real-world medical entrance exam question, please give the true answer based on the question and selection. Topic name: None, Subject name: Dental","Winged rubber dam retainer in endodontics advantages are all except Select and explain. A. Radiographs are good showing full length of canals, B. Stability, C. Provide extra bucco-lingual retraction, D. Dam, clamp and frame placed in one operation",A: Radiographs are good showing full length of canals,"Answer: A: Radiographs are good showing full length of canals Explanation: None" "This is a real-world medical entrance exam question, please give the true answer based on the question and selection. Topic name: None, Subject name: Dental","A 11-year-old child comes to the dental office one hour after injury to a maxillary central incisor. The tooth is vital and slightly mobile. Radiographic examination reveals a fracture at the apical third of the root. What is the best treatment at this point of time: Select and explain. A. Render palliative therapy, B. Extract the tooth, C. Relieve the occlusion and splint the tooth, D. Perform immediate root canal treatment and splint",C: Relieve the occlusion and splint the tooth,"Answer: C: Relieve the occlusion and splint the tooth Explanation: None" "This is a real-world medical entrance exam question, please give the true answer based on the question and selection. Topic name: None, Subject name: Dental","Resistance form of endodontics is: Select and explain. A. Resists movement of gutta-percha in apical area, B. To allow use of spreader in lateral condensation, C. Fracture of root while vertical condensation, D. None of the above",A: Resists movement of gutta-percha in apical area,"Answer: A: Resists movement of gutta-percha in apical area Explanation: None" "This is a real-world medical entrance exam question, please give the true answer based on the question and selection. Topic name: None, Subject name: Microbiology","Antibody in chronic allergy ? Select and explain. A. IgM, B. IgA, C. IgG, D. IgE",D: IgE,"Answer: D: IgE Explanation: Answer- D- IgEWhen someone has allergies, their immune system makes an antibody called immunoglobulin E (IgE). These antibodies respond to allergens. The symptoms that result are an allergic reaction.IgE also has an essential role in type I hypersensitivity, which manifests in various allergic diseases, such as allergic asthma, most types of sinusitis, allergic rhinitis, food allergies, and specific types of chronic uicaria and atopic dermatitis." "This is a real-world medical entrance exam question, please give the true answer based on the question and selection. Topic name: None, Subject name: Pharmacology","While assessing the efficacy of a newly developed drug in comparison to placebo, the 95% confidence interval in clinical trials is used to check for: Select and explain. A. Efficacy of the drug, B. Non-efficacy of the drug, C. Both efficacy and non-efficacy of the drug, D. Either efficacy or non-efficacy of the drug",A: Efficacy of the drug,"Answer: A: Efficacy of the drug Explanation: Answer- a (efficacy of the drug)" "This is a real-world medical entrance exam question, please give the true answer based on the question and selection. Topic name: None, Subject name: Dental","Which material is the most difficult to remove from the patient's mouth? Select and explain. A. Metallic oxide paste, B. Silicone impression material, C. Reversible hydrocolloid, D. Impression plaster",A: Metallic oxide paste,"Answer: A: Metallic oxide paste Explanation: In dentistry, zinc oxide eugenol is popular as an impression material for making impressions of edentulous arches for the construction of complete dentures. It is classified as a rigid, irreversible impression material. It cannot be used for recording impressions of dentate arches and in areas of severe undercuts. Mannapalli ed 3 page no 166" "This is a real-world medical entrance exam question, please give the true answer based on the question and selection. Topic name: None, Subject name: Dental","Main cause of RPD failure is: Select and explain. A. Improper clasp design, B. Insufficient contact of teeth, C. Insufficient contact of teeth and improper clasp design, D. None of the above",C: Insufficient contact of teeth and improper clasp design,"Answer: C: Insufficient contact of teeth and improper clasp design Explanation: None" "This is a real-world medical entrance exam question, please give the true answer based on the question and selection. Topic name: None, Subject name: Medicine","A 40 years old female presented with acute onset shoness of breath. She has a history of nephrotic syndrome 1 year back and recent prolonged air travel. She has a BP of 90/60 mm Hg, hea rate of 115 per minute and sinus tachycardia on ECG. A 2-D echocardiogram revealed dilation of right ventricle with bulging of the interventricular septum to the left. What will be the primary treatment modality? Select and explain. A. Thrombectomy, B. Intravenous tissue plasminogen activator, C. Unfractionated heparin, D. d. IVC filter",B: Intravenous tissue plasminogen activator,"Answer: B: Intravenous tissue plasminogen activator Explanation: Ans: B. Intravenous tissue plasminogen activator (Ref: Harrison 19/e p1634-1636, 18/2170-2177)Findings indicate patient most probably suffering from massive pulmonary embolism.Management of Massive Pulmonary Embolism:For patients with massive PE & hypotension, replete volume with 500 mL of normal saline.Additional fluid should be infused with extreme caution-Due to excessive fluid administration exacerbates RV wall stress a more profound RV ischemia & worsens LV compliance and filling.Dopamine & dobutamine - 1st line inotropic agents for PE-related shock treatment.Fibrinolysis in Massive Pulmonary Embolism:Preferred fibrinolytic regimen: Recombinant tissue plasminogen activator (tPA).Contraindications: Intracranial disease, recent surgery & trauma." "This is a real-world medical entrance exam question, please give the true answer based on the question and selection. Topic name: None, Subject name: Dental","In multirooted teeth, formation of multiple roots is due to presence of: Select and explain. A. Epithelial diaphragm, B. Cell rests of Malassez, C. Toot bud division or bell stage, D. None of the above",A: Epithelial diaphragm,"Answer: A: Epithelial diaphragm Explanation: Prior to beginning of root formation, the HERS forms epithelial diaphragm. The outer and the inner enamel epithelia bend at the future cementoenamel junction into a horizontal plane, narrowing the wide cervical opening of the tooth germ. Differential growth of the epithelial diaphragm in the multi rooted teeth causes the root trunk into two or three roots." "This is a real-world medical entrance exam question, please give the true answer based on the question and selection. Topic name: None, Subject name: Dental","In Down syndrome patient, which teeth have severe periodontitis Select and explain. A. Lower Anterior, B. Upper Anterior, C. Upper Molar, D. Lower Molar",A: Lower Anterior,"Answer: A: Lower Anterior Explanation: None" "This is a real-world medical entrance exam question, please give the true answer based on the question and selection. Topic name: None, Subject name: Pharmacology","Folic acid deficiency occurs in: Select and explain. A. Aspirin, B. Phenytoin, C. Chloromphenicol, D. Cyclosporine",B: Phenytoin,"Answer: B: Phenytoin Explanation: None" "This is a real-world medical entrance exam question, please give the true answer based on the question and selection. Topic name: None, Subject name: Gynaecology & Obstetrics","Which of the following will not decrease mother to child transmission of HIV? Select and explain. A. Avoid breastfeeding, B. Vaginal delivery, C. Zidovudine given to mother antenataly and to neonate after bih, D. Vitamin A supplementation given to mother",B: Vaginal delivery,"Answer: B: Vaginal delivery Explanation: Answer- B (Vaginal delievery)Vaginal delivery increases risk of mother to childhood transmission of HIV.'HIV can be transmitted through breast milk, avoiding breast feeding decreases the mother to child transmission of HIV.'" "This is a real-world medical entrance exam question, please give the true answer based on the question and selection. Topic name: None, Subject name: Dental","On the day following a formocresol pulpotomy of a mandibular molar, a three year old patient returned with a large ulcer near the midline of the lower lip. What is the probable cause of the ulcer? Select and explain. A. Lip biting by the patient, B. Allergy to the anaesthetic solution, C. Pressure by the rubber dam frame, D. Leakage of formocresol on the lip",A: Lip biting by the patient,"Answer: A: Lip biting by the patient Explanation: None" "This is a real-world medical entrance exam question, please give the true answer based on the question and selection. Topic name: None, Subject name: Gynaecology & Obstetrics","Which of the following conditions is associated with polyhy dramnios? Select and explain. A. Posterior urethral valve, B. Cleft palate, C. Congenital diaphragmatic hernia, D. Bladder exostrophy",B: Cleft palate,"Answer: B: Cleft palate Explanation: None" "This is a real-world medical entrance exam question, please give the true answer based on the question and selection. Topic name: None, Subject name: Microbiology","Chemotactic cytokine is Select and explain. A. IL-1, B. IL-6, C. IL-8, D. TNF",C: IL-8,"Answer: C: IL-8 Explanation: None" "This is a real-world medical entrance exam question, please give the true answer based on the question and selection. Topic name: None, Subject name: Forensic Medicine","A 38 years old female presented to the emergency with extensive burns. The patient had grade 3 burns on the face, back, upper arms and forearms along with singeing of hairs. Which of the following is not a proof of inhalation burns? Select and explain. A. Yellow colored sputum, B. Blackish soot deposit on posterior pa of tongue, C. Hoarseness & stridor of voice, D. Singeing of eyebrows and facial hair",A: Yellow colored sputum,"Answer: A: Yellow colored sputum Explanation: Ans: A. Yellow colored sputum(Ref Reddy 33/e p325-326: Parikh 6/e p4. /60)Yellow sputum/ phlegm = Mostly a sign of bacterial infection.In burns takes time to develop.Grade 3 burns:Sputum with carbonaceous colored.Singeing of facial hair, burns on face, soot marks & singed eyebrows or facial hair - Indicative of acute burns inhalational injury.Features of inhalation injury:Characteristic:History of fire in an enclosed space.Burns of face.Singed nasal & facial hair.Inflamed pharyngeal mucosa.Carbonaceous sputum.Evidence of edematous glottis (e.g., hoarseness)." "This is a real-world medical entrance exam question, please give the true answer based on the question and selection. Topic name: None, Subject name: Surgery","After spleenectomy patient becomes more prone to Select and explain. A. Gram +ve & anaerobic, B. Gram -ve & anaerobic, C. Nonencapsulated, D. Encapsulated organisms",D: Encapsulated organisms,"Answer: D: Encapsulated organisms Explanation: None" "This is a real-world medical entrance exam question, please give the true answer based on the question and selection. Topic name: None, Subject name: Anatomy","All of the following are developed from Meckel's cartilage except: Select and explain. A. Zygomatic bone, B. Malleus, C. Incus, D. Stapes",D: Stapes,"Answer: D: Stapes Explanation: None" "This is a real-world medical entrance exam question, please give the true answer based on the question and selection. Topic name: None, Subject name: Forensic Medicine","In judicial hanging, the knot is placed at: Select and explain. A. Below the chin, B. Angle of the jaw, C. The back of the neck, D. Choice of hangman",B: Angle of the jaw,"Answer: B: Angle of the jaw Explanation: Ans: B. Angle of the jawJudicial Hanging:Placement of knot beneath the chin (submental position) - Most effective.Ensures quicker death.In India & UK, left sub-aural (below the angle of the jaw) knot - Preferred for hanging.Process:The rope is looped around the neck & person is allowed to drop for 5-7 meters - According to weight, age & build of the person.A sudden stop causes fracture-dislocation usually at the level of C2-C3 or C3-C4 veebra & rupture of brain stem between pons & medulla.Death is instantaneous.Hea-beat may continue for 15-20 minutes & spasmodic muscle jerking may occur for a considerable time." "This is a real-world medical entrance exam question, please give the true answer based on the question and selection. Topic name: None, Subject name: Gynaecology & Obstetrics","Non-pulsatile dose of GnRH agonist is used in all the following conditions except- Select and explain. A. Endometriosis, B. infeility, C. Precocious pubey, D. Prostate cancer",B: infeility,"Answer: B: infeility Explanation: Ans.B.Male infeilityGnRH is used in all those conditions where there is increased estrogen as GnRH decrease estrogen secretion.Mnemonic :* A - F and HIP* A = Adenomyosis* B = Irritable Bowel Syndrome (under Trial)* C = Ca Breast (Tamoxifen + GnRH agonist give good result)* D = DUB* E = Endometriosis* F = Fibromyoma uterus.* H = Hirsutism* I = Infeility* P = Precocious Pubey." "This is a real-world medical entrance exam question, please give the true answer based on the question and selection. Topic name: None, Subject name: Pediatrics","Eisenmenger syndrome–True are A/E – Select and explain. A. Pulmonary veins are not distended, B. RV & LV walls come back to normal size, C. Dilatation of central pulmonary artery, D. Peripheral pruning of pulmonary arteries",B: RV & LV walls come back to normal size,"Answer: B: RV & LV walls come back to normal size Explanation: The characteristic pathological pulmonary vascular changes that develop in a patients with Eisenmenger syndrome are confined to arteries only and involve small pulmonary arterioles and muscular arteries (veins are not involved). There is right ventricular hypertrophy (RV walls do not come back to normal size). On x-ray there are dilated and prominent central arteries with rapid tapering (pruning) of the peripheral vasculature." "This is a real-world medical entrance exam question, please give the true answer based on the question and selection. Topic name: None, Subject name: Surgery","Gap healing, a type of primary healing having gap of 0.8 mm is healed as: Select and explain. A. Direct lamellar bone formation., B. Woven bone formation replaced by lamellar bone., C. Woven bone formation only., D. Any of the above.",B: Woven bone formation replaced by lamellar bone.,"Answer: B: Woven bone formation replaced by lamellar bone. Explanation: None" "This is a real-world medical entrance exam question, please give the true answer based on the question and selection. Topic name: None, Subject name: Dental","The radiographic examination of a traumatized tooth is essential - Select and explain. A. To establish baseline data, B. To find associated root fracture, C. To determine stage of root development, D. All of the above",D: All of the above,"Answer: D: All of the above Explanation: None" "This is a real-world medical entrance exam question, please give the true answer based on the question and selection. Topic name: None, Subject name: Anatomy","The fifth nerve innervates the following: Select and explain. A. Mylohyoid, anterior and posterior belly of digastric, B. Mylohyoid, anterior belly of digastric, and tensor tympani, C. Mylohyoid, posterior belly of digastric and tensor tympani, D. Mylohyoid, posterior belly of digastric, Stapedius and tensor tympani","B: Mylohyoid, anterior belly of digastric, and tensor tympani","Answer: B: Mylohyoid, anterior belly of digastric, and tensor tympani Explanation: Trigeminal nerve is the 5th cranial nerve. It is a mixed nerve containing both the motor and sensory fibres but predominantly it is sensory. It is a motor nerve to the muscles of mastication and several small muscles and the principal sensory nerve of the head and face. It consists of three large nerves: ophthalmic, maxillary, and mandibular, hence the name trigeminal nerve (L. trigeminus = triplet). Key concept: Mandibular nerve supplies muscles of mastication mylohyoid, anterior belly of digastric, and tensor tympani. Posterior belly of digastric and stapedius are supplied by Facial nerve." "This is a real-world medical entrance exam question, please give the true answer based on the question and selection. Topic name: None, Subject name: Social & Preventive Medicine","A study was conducted to find average intra-ocular pressure. IOP was measured in 400 people and the mean was found to be 25 mm Hg with a standard detion of 10 mm Hg. What is the range in which 10P of 95% of the population would be lying? Select and explain. A. 22-28 mm Hg, B. 20-30 mm Hg, C. 24-26 mm Hg, D. 23-27 mm Hg",C: 24-26 mm Hg,"Answer: C: 24-26 mm Hg Explanation: Ans: C. 24-26 mm HgStandard error, SE = {standard detion / n)}n = sample sizeC= Confidence coefficient = 1.96 for 95% confidence interval = 2.58 for 99% confidence interval= 3.29 for 99.9% confidence intervalHence in this question, n = 400SD = 10SE = 10/400 = 10/20 = 0.5Mean statistic = 25 mm Hg.Now for 95% confidence interval,C = 2 approximately 95%CI = 25 - 2x0.5 to 25 + 2x0.5 = 24-26 mm HgHence, 95% CI of IOP = 24 to 26 mm Hg." "This is a real-world medical entrance exam question, please give the true answer based on the question and selection. Topic name: None, Subject name: Dental","The lag phase of tooth movement usually lasts for: Select and explain. A. 2-3 mins, B. 2-3 hrs, C. 2-3 day, D. 2-3 weeks",D: 2-3 weeks,"Answer: D: 2-3 weeks Explanation: None" "This is a real-world medical entrance exam question, please give the true answer based on the question and selection. Topic name: None, Subject name: Dental","Which of the following is a common osseous lesion in periodontitis is Select and explain. A. Exostosis, B. crater, C. buttressing bone, D. hemiseptum",B: crater,"Answer: B: crater Explanation: None" "This is a real-world medical entrance exam question, please give the true answer based on the question and selection. Topic name: None, Subject name: Dental","Which of the following groups of fibres are not attached to alveolar bone? Select and explain. A. Transseptal, B. Horizontal, C. Oblique, D. Apical",A: Transseptal,"Answer: A: Transseptal Explanation: None" "This is a real-world medical entrance exam question, please give the true answer based on the question and selection. Topic name: None, Subject name: Dental","First nerve fiber present in vicinity of dental papilla in Select and explain. A. Bud stage, B. Cap stage, C. Bell stage, D. Advanced Bell stage",A: Bud stage,"Answer: A: Bud stage Explanation: None" "This is a real-world medical entrance exam question, please give the true answer based on the question and selection. Topic name: None, Subject name: Gynaecology & Obstetrics","A 32 years old female came for routine PAP smear testing. The repo came as carcinoma in situ. What is the next step? Select and explain. A. HPV-DNA testing, B. Hysterectomy, C. Conization, D. Colposcopy and biopsy",D: Colposcopy and biopsy,"Answer: D: Colposcopy and biopsy Explanation: Answer- D. Colposcopy and biopsyRegressions typically occur within a 2-year follow-up with cytology & colposcopy.Observation - Biopsy diagnoses of CIN 1 with satisfactory colposcopy and who agree to the evaluation every 6 monthsAblative treatment - lf the lesions progress during follow-up or persist at 2 yearsTreatment options: LEEP & Cryosurgery" "This is a real-world medical entrance exam question, please give the true answer based on the question and selection. Topic name: None, Subject name: Anatomy","The frontal paranasal sinus drains into the: Select and explain. A. Superior meatus, B. Middle meatus, C. Inferior meatus, D. Spheno-ethmoidal recess",B: Middle meatus,"Answer: B: Middle meatus Explanation: None" "This is a real-world medical entrance exam question, please give the true answer based on the question and selection. Topic name: None, Subject name: Biochemistry","In the mammalian genome, maximum number of genes code for the receptors of: Select and explain. A. Immunoglobulin receptors, B. Interleukins, C. Growth factors, D. Odorants",D: Odorants,"Answer: D: Odorants Explanation: Ans: D. OdorantsIn the mammalian genome, maximum number of genes code for the receptors of odorants.""The olfactory receptor (OR) genes constitute the largest gene family in mammalian genomes.Humans have >1,000 OR genes, of which only -40% have an intact coding region and are therefore putatively functional.Odorant receptor genes form the largest gene family in the genome of many animals: for example, the mouse genome contains approximately 1200 of these genes." "This is a real-world medical entrance exam question, please give the true answer based on the question and selection. Topic name: AIIMS 2019, Subject name: Pharmacology","Pegloticase is used in Select and explain. A. Chronic gout, B. Paralytic ileus, C. Psoriatic ahritis, D. Rheumatoid ahritis",A: Chronic gout,"Answer: A: Chronic gout Explanation: Pegloticase - It is recombinant uricase, an enzyme which oxidises uric acid to highly soluble allantoin, that is easily excreted by kidney. Humans lack this enzyme. In this preparation, the enzyme has been coupled with methoxy polyethylene glycol (mPEG) which serves to prolong its sojourn in the body, permitting i.v. infusion of the drug to be given every 2 weeks. It is indicated only in rare cases of refractory symptomatic gout because it is immunogenic and carries high risk of infusion reactions, including anaphylaxis." "This is a real-world medical entrance exam question, please give the true answer based on the question and selection. Topic name: None, Subject name: Physiology","Wave patterns of EEF, ECG and EMG are depicted below. The B pattern belongs to (Figure was not provided in the exam): Select and explain. A. NREM sleep, B. REM sleep, C. Wakefulness, D. Quiet wakefulness",C: Wakefulness,"Answer: C: Wakefulness Explanation: Answer- C. WakefulnessBeta (B) Wave- Parietal and frontal region. Patients awake, at rest with eyes open." "This is a real-world medical entrance exam question, please give the true answer based on the question and selection. Topic name: None, Subject name: Medicine","Staphylococcus aureus and pneumoniae are responsible for: Select and explain. A. Subacute endocarditis., B. Acute endocarditis., C. Post operative endocarditis., D. None.",B: Acute endocarditis.,"Answer: B: Acute endocarditis. Explanation: None" "This is a real-world medical entrance exam question, please give the true answer based on the question and selection. Topic name: None, Subject name: ENT","First paranasal sinus to develop at bih is: Select and explain. A. Maxillary, B. Ethmoidal, C. Frontal, D. Sphenoidal",A: Maxillary,"Answer: A: Maxillary Explanation: Development of SinusesSinusGestational Month WhenDevelopment StasPresent in ClinicallySignificant SizeFully DevelopedMaxillary2degBihdeg12 yearsdegEthmoid30Bihdeg12 yearsdegFrontal4deg3 yearsdeg18-20 yearsdegSphenoid3deg8 yearsdeg12-15 yearsdeg" "This is a real-world medical entrance exam question, please give the true answer based on the question and selection. Topic name: None, Subject name: Medicine","Which of the following is the best test for assessment of intestinal malabsorption? Select and explain. A. Fecal fat estimation, B. Serum lactose levels, C. Serum amylase levels, D. NBT-PABA test",A: Fecal fat estimation,"Answer: A: Fecal fat estimation Explanation: Ans: A(Ref: Harrison 19/c p193(S)Fecal fat estimation - Gold standard test for diagnosis of malabsorption.Intestinal malabsorption diagnosis:D-xylose test.Tests for xylose absorption in intestinal villi.Distinguishs intestinal from pancreatic malabsorption.Pancreatic malabsorption diagnosis:NBT-PABA & serum amylase." "This is a real-world medical entrance exam question, please give the true answer based on the question and selection. Topic name: None, Subject name: Dental","Primary Colonization of plaque is dominated by Select and explain. A. Facultative Aerobic Gram + ve rods, B. Facultative Anaerobic Gram - ve rods, C. Facultative Aerobic Gram - ve cocci, D. Facultative Anaerobic Gram + ve cocci",D: Facultative Anaerobic Gram + ve cocci,"Answer: D: Facultative Anaerobic Gram + ve cocci Explanation: None" "This is a real-world medical entrance exam question, please give the true answer based on the question and selection. Topic name: None, Subject name: Dental","Types of pocket respectively: Select and explain. A. Simple pocket, compound pocket, complex pocket., B. Simple pocket, complex pocket, compound pocket., C. Compound pocket, simple pocket, complex pocket., D. Simple pocket, compound pocket, tertiary pocket.","A: Simple pocket, compound pocket, complex pocket.","Answer: A: Simple pocket, compound pocket, complex pocket. Explanation: None" "This is a real-world medical entrance exam question, please give the true answer based on the question and selection. Topic name: None, Subject name: Dental","Depression of mandible is achieved by: Select and explain. A. Digastric, B. Lateral pterygoid, C. Geniohyoid, D. All of the above",D: All of the above,"Answer: D: All of the above Explanation: None" "This is a real-world medical entrance exam question, please give the true answer based on the question and selection. Topic name: AIIMS 2017, Subject name: Gynaecology & Obstetrics","18 year old female presents with an ovarian mass, her serum bio marker are found to be normal except for LDH, which is found to be elevated. The most likely diagnosis is: Select and explain. A. Dysgerminoma, B. Endodermal sinus tumor, C. Malignant terratoma, D. Mucinous cystadeno carcinoma",A: Dysgerminoma,"Answer: A: Dysgerminoma Explanation: NOTE :- * Young girls with - Germ cell tumor Ovarian mass * Old women with - Epithelial serous tumor Ovarian mass Biomarkers Dysgerminoma | LDH, | placental alkaline Po4 Endodermal sinus tumor a feto protein and antitrypsin" "This is a real-world medical entrance exam question, please give the true answer based on the question and selection. Topic name: AIIMS 2019, Subject name: Microbiology","Most common infection post solid organ transplantation Select and explain. A. CMV, B. HSV, C. EBV, D. HPV",A: CMV,"Answer: A: CMV Explanation: Cytomegalovirus (CMV) is transmitted post solid organ transplantation whereas herpes simplex virus (HSV) is transmitted post solid organ transplantation only within a month. Epstein-Barr virus (EBV) is a glandular disease so transmitted by close contact where as human papilloma virus (HPV) is transmitted through sexual contact since it is a sexually transmitted disease. So first option is a better option. Infection in Children: Primary infections in older children and adults are usually asymptomatic. However, a heterophile, antibody-negative, infectious mononucleosis may been. This is more common following transfusion of CMV-infected blood (post - transfusion mononucleosis)." "This is a real-world medical entrance exam question, please give the true answer based on the question and selection. Topic name: None, Subject name: ENT","Tuberculous otitis media is characterized by all except - Select and explain. A. Multiple perforations, B. Pale granulations, C. Pain, D. Thin odourless fluid",C: Pain,"Answer: C: Pain Explanation: Tubercular Otitis Media Important points: ––Seen mainly in children and young adult Q ––It is secondary to pulmonary tuberculosis. Q ––Route of spread - Mainly through eustachian tube (not blood borne). Q Symptoms: 1. Patients often present with chronic painless otorrhoea (usually foul smelling) which is resistant to antibiotic treatment Q 3. Severe conductive type hearing loss Q. (sometimes due to involvement of labyrinth may be SNHL) 4. Facial nerve palsy may be the presenting symptom in childrenQ 5. Cough; fever and night sweats may be present in patients with tuberculous infection in other organ system. O/E –– Multiple perforationsQ in tympanic membrane (This feature was once considered characteristic of TB but now is seldom seen). –– Middle ear and mastoid are filled with pale granulation tissueQ (It is a characteristic of tuberculous otitis media) Complications: (Early onset of these symptoms is seen) ––Mastoiditis ––Osteomyelitis ––Postauricular fistula ––Facial nerve palsy" "This is a real-world medical entrance exam question, please give the true answer based on the question and selection. Topic name: AIIMS 2017, Subject name: Social & Preventive Medicine","Child Pugh score put patients into three categories of Score A (<7), Score B (7-9) and Score C (10-15). The following is a type of which scale? Select and explain. A. Nominal, B. Ordinal, C. Quantitative, D. Continuous",B: Ordinal,"Answer: B: Ordinal Explanation: Child Pugh Score' divides the scores into 3 CATEGORIES, hence it is a CATEGORICAL scale. The variables are qualitative and arranged in an order of increasing severity, hence it is an example of ORDINAL scale. Other options: Nominal scale is also a type of categorical scale in which the variables are not arranged in any order. Metric scale is used for quantitative variables. Continuous is a type of variable and not a type of scale." "This is a real-world medical entrance exam question, please give the true answer based on the question and selection. Topic name: None, Subject name: Anatomy","Skin of the entire upper lip is supplied by: Select and explain. A. Maxillary nerve., B. Facial nerve., C. Mandibular nerve., D. None.",A: Maxillary nerve.,"Answer: A: Maxillary nerve. Explanation: None" "This is a real-world medical entrance exam question, please give the true answer based on the question and selection. Topic name: None, Subject name: Medicine","Microangiopathic hemolytic anemia Select and explain. A. TTP, B. ITP, C. Senile purpura, D. CML",B: ITP,"Answer: B: ITP Explanation: None" "This is a real-world medical entrance exam question, please give the true answer based on the question and selection. Topic name: AIIMS 2017, Subject name: Pharmacology","Treatment of choice for anaphylactic shock is: Select and explain. A. Adrenaline 0.5 mL of 1:1000 solution by intramuscular route, B. Adrenaline 1 mL of 1:10000 by intravenous route, C. Atropine 3 mg intravenously, D. Adenosine 12 mg intravenously",A: Adrenaline 0.5 mL of 1:1000 solution by intramuscular route,"Answer: A: Adrenaline 0.5 mL of 1:1000 solution by intramuscular route Explanation: DOC is adrenaline.Route is intramuscular > subcutaneous.Dose: 0.5 ml of 1:1000(1mg/ml solution).This dose is repeated once gain within 10 minutes.If it's still not improving; then intravenous 1:10,000 dose is given." "This is a real-world medical entrance exam question, please give the true answer based on the question and selection. Topic name: None, Subject name: Dental","The cotton applied to the pulpal stumps in the formocresol pulpotomy technique should be Select and explain. A. Slightly dampened with formocresol, B. Saturated with formocresol, C. Left for 2 hours in the cavity, D. Sealed until the next appointment.",A: Slightly dampened with formocresol,"Answer: A: Slightly dampened with formocresol Explanation: None" "This is a real-world medical entrance exam question, please give the true answer based on the question and selection. Topic name: None, Subject name: Pathology","Which of following is not a tumor suppressor gene? Select and explain. A. pRb, B. FHIT, C. PTCH, D. c-erbB1",D: c-erbB1,"Answer: D: c-erbB1 Explanation: None" "This is a real-world medical entrance exam question, please give the true answer based on the question and selection. Topic name: None, Subject name: ENT","All the following instruments are required for tonsillectomy except: Select and explain. A. Coblation wand, B. Bipolar cautery, C. Microdebrider, D. Harmonic scalpel",C: Microdebrider,"Answer: C: Microdebrider Explanation: Ans: C. Microdebrider(Ref: Scott-Brown's 7Ie p3576; Cummings Pediatric Otolaryngology By Marci M. Lesperance, Paul W. Flint/p51; Dhingra 6/e p430)All these four instruments are used in different techniques of tonsillectomy.Microdebrider is less commonly used than others as it leaves behind a small amount of tissue covering theconstrictor muscle.Hence, preferred answer would be microdebrider." "This is a real-world medical entrance exam question, please give the true answer based on the question and selection. Topic name: AIIMS 2017, Subject name: Anatomy","Which marker shows holocrine gland? Select and explain. A. A, B. B, C. C, D. D",A: A,"Answer: A: A Explanation: Marker (A) shows the Holocrine gland i.e, Sebaceous gland related with hair follicle sending sebum to skin surface. Marker (C) - Hair follicle Marker (D) - Sub cutaneous fat (Adipocytes) - empty looking cells with fat & peripheral nucleus - Usually skin & sebaceous glands have - stratified squamous epithelium" "This is a real-world medical entrance exam question, please give the true answer based on the question and selection. Topic name: None, Subject name: Dental","Landmark for pup chamber opening is : Select and explain. A. CEJ, B. Cervical third of crown, C. Level of alveolar bone, D. Enamel",A: CEJ,"Answer: A: CEJ Explanation: Krasner and Rankow determined that the CEJ is the most important anatomic landmark for determining the location of pulp chambers and root canal orifices. They demonstrated  that  specific  and  consistent  pulp  chamber floor and wall anatomy exist and proposed laws for assisting  clinicians  to  identify  canal  morphology. Ref: Grossman endodotic practice 13th ed page no 248" "This is a real-world medical entrance exam question, please give the true answer based on the question and selection. Topic name: None, Subject name: Physiology","Bezold-Jarisch reflex is mediated by: Select and explain. A. Serotonin, B. Angiotensin, C. Prostaglandin, D. Histamine",A: Serotonin,"Answer: A: Serotonin Explanation: Answer- A. SerotoninThe Bezold-Jarisch reflex involves a variety of cardiovascular and neurological processes which cause hypopnea (excessively shallow breathing or an abnormally low respiratory rate) and bradycardia (abnormally low resting hea rate). Serotonin can elicit Bezold-Jarisch reflex.Bezold-Jarisch reflex responds to noxious ventricular stimuli sensed by chemoreceptors & mechanoreceptors within the LV wall by inducing the triad of hypotension bradycardia & coronary aery dilatation.Stimulation of aerial baroreceptors or ventricular baroreceptors by any of a host of chemicals-veratrum alkaloids, nicotine capsaicin, anti-histamine serotonin snake and insect venoms--can also trigger the Bezold- Jarisch reflex." "This is a real-world medical entrance exam question, please give the true answer based on the question and selection. Topic name: None, Subject name: Pharmacology","Action of Nitroglycerin is Select and explain. A. Direct action on smooth muscle, B. Slows SA node conductvity, C. Blocks arrhythmia, D. Increases perfusion to heart",A: Direct action on smooth muscle,"Answer: A: Direct action on smooth muscle Explanation: None" "This is a real-world medical entrance exam question, please give the true answer based on the question and selection. Topic name: None, Subject name: Social & Preventive Medicine","ICD-10 stands for Select and explain. A. International Classification of Drugs, 10th revision, B. International Classification of Disabilities, 10th revision, C. International Classification of Diseases, 10th revision, D. International Classification of Disasters, 10th revision","C: International Classification of Diseases, 10th revision","Answer: C: International Classification of Diseases, 10th revision Explanation: None" "This is a real-world medical entrance exam question, please give the true answer based on the question and selection. Topic name: None, Subject name: Pathology","A 40 year old woman has arneloblastoma, the histomorphologic features will be Select and explain. A. Peripheral palisading cellular strand with central loose stellate reticulum, B. Peripheral palisading with central stromal retraction artefact, C. Peripheral palisading cellular strand with peripheral loo se stellate reticulum, D. Central Loose stellate reticulum shows marked nuclear atypia and numerous mitotic",A: Peripheral palisading cellular strand with central loose stellate reticulum,"Answer: A: Peripheral palisading cellular strand with central loose stellate reticulum Explanation: None" "This is a real-world medical entrance exam question, please give the true answer based on the question and selection. Topic name: AIIMS 2018, Subject name: Biochemistry","Method of transpo of glucose in the intestine is: Select and explain. A. Primary active transpo, B. Secondary active transpo, C. Simple diffusion, D. Counter transpo",B: Secondary active transpo,"Answer: B: Secondary active transpo Explanation: The method of transpo of glucose in the intestine is Secondary active transpo. Both glucose and Na+enter inside the cell (in same direction) i.e. Sympo. No energy used when glucose and Na+ enters the cell. Energy is used indirectly for glucose transpo Na+/ K+ ATPase pump. Sodium-glucose sympoer carries 2Na+for each glucose SGLT-1- Small intestine and kidneys -for Glucose and Galactose SGLT-2- in kidneys -only for glucose In intestine on luminal surface there is SGLT-1 (secondary active transpo) while on basal surface is GLUT-2 (facultative transpo)" "This is a real-world medical entrance exam question, please give the true answer based on the question and selection. Topic name: None, Subject name: Pathology","A 28 year old man has lenticonus and end stage renal disease now. His maternal uncle also died of the same illness. What is the most likely diagnosis? Select and explain. A. Autosomal dominant polycystic kidney disease, B. Autosomal recessive polycystic kidney disease, C. Oxalosis, D. Alport syndrome",D: Alport syndrome,"Answer: D: Alport syndrome Explanation: Presentation of male patient with lenticonus and end stage renal disease with a family history of renal disease is highly suggestive of Alport syndrome. AR polycystic kidney is ruled out because the age of presentation in ARPKD is childhood and most of the affected children do not survive beyond their childhood. AD polycystic kidney is ruled out because there is no association of ADPKD with lenticonus as is mentioned in our question. –– Alport syndrome is manifest by hematuria with progression to chronic renal failure, accompanied by nerve deafness and various eye disorders, including lens dislocation, posterior cataracts, and corneal dystrophy. –– In about 85% cases, it is inherited as an X-linked trait. So, males express the full syndrome, and females are carriers in whom manifestations of disease are typically limited to hematuria. –– Autosomal recessive and autosomal dominant forms also exist in which both the sexes are equally susceptible. –– In Alport syndrome, Hematuria is the earliest manifestation and the sensorineural deafness is the commonest extra renal abnormality." "This is a real-world medical entrance exam question, please give the true answer based on the question and selection. Topic name: None, Subject name: Microbiology","A patient comes after a dog bite with hydrophobia, tearing and altered sensorium. You suspect rabies in this patient. Corneal impression has been taken. What test will you do on it for most accurate diagnosis? Select and explain. A. Indirect immunofluorescence, B. -PCR for virus, C. Histopathological examination for Negri bodies, D. Antibodies against Rabies virus",B: -PCR for virus,"Answer: B: -PCR for virus Explanation: Answer- B. -PCR for virusDetection of rabies virus RNA by - PCR is highly sensitive and specific. This technique can detect virus infresh saliva samples, skin, CSF, and brain tissues.Detection of rabies virus RNA by -PCR is highly sensitive and specific. This technique can detect virus in fresh saliva samples, skin, CSF, and brain tissues. In addition, -PCR with genetic sequencing can distinguish among rabies virus variants, permitting identification of the probable source of an infection.""-Harrison 19/e p1302.Reverse transcription-polymerase chain reaction testing can be used to amplify pas of a rabies virus genome from fixed or unfixed brain tissue or saliva. Sequencing of amplified products can allow identification of the infecting virus strain." "This is a real-world medical entrance exam question, please give the true answer based on the question and selection. Topic name: None, Subject name: Biochemistry","True about LDL receptor: Select and explain. A. It recognizes apo B100 and apo E, B. Involved in endocytosis process, C. Present in both hepatic and extrahepatic cells, D. All of the above",D: All of the above,"Answer: D: All of the above Explanation: LDL receptor: It recognizes apo B-100 and apo-E. Has clathrin coating. Involved in endocytosis process. Present in both hepatic and extra-hepatic cells." "This is a real-world medical entrance exam question, please give the true answer based on the question and selection. Topic name: None, Subject name: Surgery","Pterygomandibular space contains all, except: Select and explain. A. Nerve to mylohyoid muscle, B. Long buccal nerve, C. Loose areolar tissue, D. Nerve to medial pterygoid muscle",B: Long buccal nerve,"Answer: B: Long buccal nerve Explanation: None" "This is a real-world medical entrance exam question, please give the true answer based on the question and selection. Topic name: None, Subject name: Gynaecology & Obstetrics","A 32-year-old P2L2 lady comes five days after unpro!tected sexual intercourse. What will be your advice for contraception in this lady? Select and explain. A. Copper IUCD, B. Levonorgestrel 0.75 mg, C. Two tablets of high dose OCP, repeated after 24 hours, D. Laparoscopic tubectomy",A: Copper IUCD,"Answer: A: Copper IUCD Explanation: Ans: A. Copper IUCD(Ref Dutta 8/e p615, 7/e p551)Best contraceptive for 32-year old P2L2 lady coming after 5 days after unprotected sexual intercourse - Copper-containing IUCD.Copper IUCD:Inseion within maximum period of 5-7 days after accidental unprotected exposure.Prevents implantation.Unsuitable for women with multiple sex paners & rape victims." "This is a real-world medical entrance exam question, please give the true answer based on the question and selection. Topic name: None, Subject name: Social & Preventive Medicine","Data about recent trends of immunization in the community can be found by: Select and explain. A. Sample registration system, B. District level health survey, C. Rural survey, D. Census data",B: District level health survey,"Answer: B: District level health survey Explanation: Ans: B. District level health survey(Ref Park's 22/e p786)District Level Health Survey - Objectives:Coverage of ante-natal, natal and post-natal checkups and child immunization.Propoion of institutional/safe deliveries.JSY beneficiaries.Contraceptive prevalence rates.Unmet need for contraceptives - Spacing & limiting.Awareness about I/ STI and HIV/AIDS.Ministry of Health and Family Welfare (MOHFW), Government of India, has included the Clinical, Anthropometric and Biochemical (CAB) component for data collection in the District Level Household Survey (DLHS)-4." "This is a real-world medical entrance exam question, please give the true answer based on the question and selection. Topic name: None, Subject name: Pharmacology","Which of the following diuretic can be given in mild to moderate hypeension? Select and explain. A. Potassium sparing diuretic, B. Osmotic diuretic, C. Thiazide diuretic, D. Loop diuretic",C: Thiazide diuretic,"Answer: C: Thiazide diuretic Explanation: Ans: C. Thiazide diureticThiazide diuretics :Given in mild to moderate hypeension.Low-dose thiazide diuretics used alone or in combination with other antihypeensive drugs.Safe, efficacious, inexpensive & reduce clinical events. MOA:Inhibits Na/Cl- pump in DCT a Increases sodium excretion.In long term - Acts as vasodilators.Provide additive blood pressure-lowering effects (On combination with beta blockers, ACE-I, ARBs).Drug of Choicein HypeensionConditionDrug of ChoiceHypeensionThiazidesHypeension with BPHPrazosinHypeension with diabetes mellitusACE inhibitorsHypeension with ischemic hea disease (angina)Beta-blockersHypeension with chronic kidney diseaseACE inhibitorsHypeension in pregnancyAlpha-methyldopaHypeensive emergenciesNicardipine + EsmololHypeensive emergencies in cheese reactionPhentolamineHypeensive emergencies in clonidine withdrawalPhentolamineHypeensive emergencies in aoic dissectionNitroprusside + EsmololHypeensive emergencies in pregnancyLabetalol(Ref: Harrison 19/e p1623)." "This is a real-world medical entrance exam question, please give the true answer based on the question and selection. Topic name: None, Subject name: Dental","Which of the following is used for etching porcelain? Select and explain. A. 1.23% HF, B. 4% HF, C. 9.6% HF, D. 37% HF",C: 9.6% HF,"Answer: C: 9.6% HF Explanation: Chemical preparation of glazed ceramic may be done with 9.6% hydrofluoric acid (HFA) or acidulated phosphate fluoride and then treated with silane coupling agent to increase the bond strength. Zachrisson recommends the following techniques for bonding to porcelain surfaces: Isolation is very critical in this technique, not just to prevent contamination with saliva and water, but also to prevent soft tissues contact with HFA. Barrier gel such as Kool-Dam prevents flowing of etchant onto the gingival tissue. Deglaze the area slightly larger than bracket base by sandblasting with 50 µm aluminium oxide for 3 s. Etch porcelain with 9.6% HF acid for 2 min. Carefully remove gel with cotton roll and then rinse using high volume suction. Dry with air, apply silane coupling agent and bond bracket conventionally." "This is a real-world medical entrance exam question, please give the true answer based on the question and selection. Topic name: None, Subject name: Biochemistry","Ceramide is: Select and explain. A. Simple lipid, B. Compound lipid, C. Precursor lipid, D. Derived lipid",A: Simple lipid,"Answer: A: Simple lipid Explanation: 1. Simple lipid: TAG (glycerol + 3 FA) Ceramide ( Sphingosine + FA)  2. Compound lipid: Phospholipids Glycolipid / Sphingoglycolipid Aminolipids Sulfolipids Lipoproteins 3. Miscellaneous: Precursor lipid - Compounds that can give rise to another lipids. Eg: Fatty Acids Derived lipid - Eg: Bile acids Precursor derived lipid - Eg: Cholesterol" "This is a real-world medical entrance exam question, please give the true answer based on the question and selection. Topic name: None, Subject name: Skin","Which type of oral candidiasis does not presents with white patch? Select and explain. A. Chronic atrophic candidiasis, B. Chronic hyperplastic candidiasis, C. Chronic mucocutaneous candidiasis, D. Pseudomembranous candidiasis",A: Chronic atrophic candidiasis,"Answer: A: Chronic atrophic candidiasis Explanation: Ans: A. Chronic atrophic candidiasisChronic erythematous (atrophic) candidiasis appears as a red, raw-looking lesion instead of a white patch seen in all other types.Erythematous (atrophic) candidiasis:Appears as a red, raw-looking lesion.Subtypes of erythematous candidiasis:Denture-related stomatitis, angular stomatitis, median rhomboid glossitis & antibiotic-induced stomatitis.Since they are commonly erythematous/atrophic.Precede pseudomembrane formation,.Left when membrane is removed, or arise de novo.Tongue:Loss of lingual papillae, leaving a smooth area on tongue.Occurs on dorsum of tongue in long-term coicosteroids or antibiotic patient.But occasionally it can occur after only a few days of using a topical antibiotic.This is usually termed 'antibiotic sore mouth/stomatitis' because it is commonly painful as well as red." "This is a real-world medical entrance exam question, please give the true answer based on the question and selection. Topic name: AIIMS 2018, Subject name: Pathology","Which of the following statements is false about apoptosis? Select and explain. A. No inflammation, B. Intact plasma membrane, C. Swelling of organelles, D. Affected by dedicated genes",C: Swelling of organelles,"Answer: C: Swelling of organelles Explanation: Apoptosis is genetically regulated, hence apoptosis is sometimes referred to as programmed cell death. Plasma membrane remains intact in apoptosis hence there is no leakage of enzymes and inflammation. Apoptosis cause activation of caspase and proteases which cause cells to shrink. Hence organelle swelling is not a feature of apoptosis. Characteristic feature of apoptosis microscopically is chromatin condensation." "This is a real-world medical entrance exam question, please give the true answer based on the question and selection. Topic name: AIIMS 2019, Subject name: Orthopaedics","A 55 yr old patient presented to the opd with complaint of pain in back. On examination his limbs seemed to be bent with overlying warm and thick skin. Patient also complain of decreased hearing over a few days. On biochemical analysis his serum ALP was raised. X-ray obtained showed the following features. Most probable diagnosis? Select and explain. A. Osteoporosis, B. Osteopetrosis, C. Pagets disease, D. Renal osteodystrophy",C: Pagets disease,"Answer: C: Pagets disease Explanation: Given scenario suggests diagnosis of Paget's disease. Image shows- a) Ivory veebra, b) Picture frame veebra Osteoporosis- Cod fish veebra Osteopetrosis- bone within a bone appearance Renal osteodystrophy- Rugger jersey spine" "This is a real-world medical entrance exam question, please give the true answer based on the question and selection. Topic name: None, Subject name: Dental","Halo effect surrounding the root of tooth on IOPA X-Ray is seen with? Select and explain. A. Horizontal root fracture, B. Root caries, C. Widening of periodontal ligament space, D. Periapical-periostitis",D: Periapical-periostitis,"Answer: D: Periapical-periostitis Explanation: None" "This is a real-world medical entrance exam question, please give the true answer based on the question and selection. Topic name: None, Subject name: Dental","A temporary bridge constructed from a synthetic resin Select and explain. A. Is not required when the missing tooth is provided on a partial denture, B. Should be cemented with a polycarboxylate cement to ensure retention between the preparation and fitting stages, C. Should have the same buccal and lingual contours as the permanent bridge, D. Should restore the occlusion to the same extent as the permanent bridge",D: Should restore the occlusion to the same extent as the permanent bridge,"Answer: D: Should restore the occlusion to the same extent as the permanent bridge Explanation: None" "This is a real-world medical entrance exam question, please give the true answer based on the question and selection. Topic name: None, Subject name: Physiology","After resection of posterior pituitary all of the following hormones is given except? Select and explain. A. Glucagon, B. Thyroxine, C. Estradiol, D. Mineralocoicoid",D: Mineralocoicoid,"Answer: D: Mineralocoicoid Explanation: Ans. D. Mineralocoicoid" "This is a real-world medical entrance exam question, please give the true answer based on the question and selection. Topic name: None, Subject name: Dental","What is the thickness of cement-dentinal junction? Select and explain. A. 1-2 μm, B. 2-3 μm, C. 3-4 μm, D. 4-5 μm",B: 2-3 μm,"Answer: B: 2-3 μm Explanation: None" "This is a real-world medical entrance exam question, please give the true answer based on the question and selection. Topic name: None, Subject name: Dental","Which of the following appliances is not used for slow maxillary expansion? Select and explain. A. Jack screw, B. Coffin spring, C. Quad helix appliance, D. Hyrax appliance",D: Hyrax appliance,"Answer: D: Hyrax appliance Explanation: Hyrax appliance  This type of expander uses the Hyrax (hygienic rapid expansion) screw. It has heavy wires that are adapted, welded and soldered to the palatal aspects of the bands in the permanent molars. It is used for Rapid palatal expansion. Examples of slow expansion:  Jack screw Coffin spring Quad helix appliance Textbook of ORTHODONTICS Sridhar Premkumar" "This is a real-world medical entrance exam question, please give the true answer based on the question and selection. Topic name: None, Subject name: Dental","For how long should NiTi files be submerged in NaOCl at 122-degree F to prevent damage to file corrosion Select and explain. A. 20 mins, B. 45 mins, C. 60 mins, D. 2 hours",C: 60 mins,"Answer: C: 60 mins Explanation: Direct lines from book ""warming of NaOCI to 122° F (50' C) leads to increase in collagen dissolution as potential to disinfecting, but it can leads to detrimental corrosion effects on NiTi instruments immersion for 1 hr""." "This is a real-world medical entrance exam question, please give the true answer based on the question and selection. Topic name: None, Subject name: Anatomy","Mandibular process of each side fuse to form: Select and explain. A. Lower lip., B. Lower jaw., C. Both., D. None.",C: Both.,"Answer: C: Both. Explanation: None" "This is a real-world medical entrance exam question, please give the true answer based on the question and selection. Topic name: None, Subject name: Dental","True about meningococcal vaccine is - Select and explain. A. Capsule polysaccharide act as a virulent factor for the production of antibody, B. Conjugate vaccine is not given in outbreaks, C. Polysaccharide vaccine should be given to children below 2 years of age, D. Vaccine with polysaccharide B is immunogenic",A: Capsule polysaccharide act as a virulent factor for the production of antibody,"Answer: A: Capsule polysaccharide act as a virulent factor for the production of antibody Explanation: None" "This is a real-world medical entrance exam question, please give the true answer based on the question and selection. Topic name: None, Subject name: Surgery","Self injury most likely caused after extraction in children is Select and explain. A. Inadvertent lip biting after anesthesia, B. Chemical burn, C. Permanent palsy, D. Trismus",A: Inadvertent lip biting after anesthesia,"Answer: A: Inadvertent lip biting after anesthesia Explanation: Inadvertent lip biting after inferior alveolar nerve block is most commonly seen in children after extraction. This self-injury mostly occurs due to dentist fail to mention/provide important instructions to child/ their parents that he/she should not eat anything till effect of anesthesia completely wears off as it leads to unknowing/ Inadvertent lip biting during eating. Sometimes this also happens if child/parents fail to follow the instructions mentioned by dentist." "This is a real-world medical entrance exam question, please give the true answer based on the question and selection. Topic name: None, Subject name: Dental","Antempasterior relation of jaws is measured by: Select and explain. A. ANB angle, B. Angle between SN & FH Plane, C. Angle between SN & mandibular plane, D. Facial angle",A: ANB angle,"Answer: A: ANB angle Explanation: None" "This is a real-world medical entrance exam question, please give the true answer based on the question and selection. Topic name: None, Subject name: Radiology","Intensifying screen is used in extraoral radiograph to: Select and explain. A. Decrease radiation to patient, B. Increase contrast, C. Decrease contrast, D. Collimation",A: Decrease radiation to patient,"Answer: A: Decrease radiation to patient Explanation: None" "This is a real-world medical entrance exam question, please give the true answer based on the question and selection. Topic name: None, Subject name: Dental","Gold Standard for evaluation of any obstruction in the nasal pathway: Select and explain. A. Mirror test., B. Butterfly test., C. Rhinomanometry., D. To check the size of nostril.",C: Rhinomanometry.,"Answer: C: Rhinomanometry. Explanation: None" "This is a real-world medical entrance exam question, please give the true answer based on the question and selection. Topic name: AIIMS 2019, Subject name: Gynaecology & Obstetrics","The following is used in which procedure in gynecology? Select and explain. A. Pap smear, B. Endometrial Biopsy, C. Cervical Biopsy, D. Vulval Biopsy",A: Pap smear,"Answer: A: Pap smear Explanation: The image shown in question is of Ayre's spatula which is used in papsmear Sensitivity of the cervical cytology for the detection of CIN 2 or 3 ranged from 47% to 62% Errors occurred because of - Poor fixation on the glass slide, leading to air drying - Smear were thick and obscured by vaginal discharge, blood, or mucus. Liquid - based Cytology Liquid samples are processed to provide a uniform, thin layer of cervical cells without debris on a glass slide & this eliminates air drying. The cell sample is collected with an endocervical brush used in combination with a plastic spatula or with a plastic broom. The sample is rinsed in a l containing liquid alcohol-based preservative. Hence 80% to 90% of the cells are retrieved The cells are retrieved from the l by passing the liquid through a filter. Recommendation : 2020 Novaks For women 21 to 29 years Screening with cytology every 3 year From 30 to 65 years Co - testing with conventional cytology and high - risk HPV testing every 5 years Or Cytology alone every 3 years are appropriate alternatives After the age of 65 it is Discontinue screening when 3 negative cytology results or 2 negative co test in the previous 10 years" "This is a real-world medical entrance exam question, please give the true answer based on the question and selection. Topic name: None, Subject name: Pediatrics","A 5 years old child presented with continuous fever and features of sepsis with a BP of 90/60 mm Hg, Pulse rate 144/min and respiratory rate of 30/min. What is the initial fluid of choice for management? Select and explain. A. 10 mL/kg of 10% dextrose, B. 10 mL/kg of hydroxyethyl starch, C. 20 mL/kg of 0.45% normal saline, D. 20 mL/kg of 0.9% normal saline",D: 20 mL/kg of 0.9% normal saline,"Answer: D: 20 mL/kg of 0.9% normal saline Explanation: Ans: D : 20 mL/kg of 0.9% normal salineExplanation:(Ref: Ghai 8Ie p718)In hypovolemic or septicemic shock, replacement of intravascular volume by isotonic fluids is the main stay of treatment.Hence, normal saline resuscitation with 20 ml/kg boluses." "This is a real-world medical entrance exam question, please give the true answer based on the question and selection. Topic name: None, Subject name: Pathology","Patient with increased PT, APTT & TT and normal fibrinogen and platelet counts, diagnosis is? Select and explain. A. DIC, B. Fac. VIII def., C. Liver disease, D. Vitamin K deficiency",D: Vitamin K deficiency,"Answer: D: Vitamin K deficiency Explanation: None" "This is a real-world medical entrance exam question, please give the true answer based on the question and selection. Topic name: AIIMS 2017, Subject name: Pediatrics","A child presented with abdominal distension, hepatomegaly, doll like facies and recurrent episodes of hypoglycemia. Which of the following is the most likely diagnosis? Select and explain. A. Menkes disease, B. Down syndrome, C. Von gierke disease, D. Lesch nyhan syndrome",C: Von gierke disease,"Answer: C: Von gierke disease Explanation: Menkes disease- aka kinky hair disease - Defect in copper binding ATPase - Involves CNS- intellectual disability, seizures - Characterized by sparse hypopigmented kinky hairs. - Trichorrhexis nodosa and pili toi seen Von Gierke disease is glycogen storage disease type I, in which the child has doll like facies, abdominal distension, hepatomegaly and recurrent episodes of hypoglycemia. On investigation, there is hyperlipidemia, hyperuricemia and lactic acidosis. Lesch nyhan syndrome- mental retardation, microcephaly, seizures and gout." "This is a real-world medical entrance exam question, please give the true answer based on the question and selection. Topic name: None, Subject name: Gynaecology & Obstetrics","Estrogen and progesterone in the first 2 months pregnancy are produced by: Select and explain. A. Fetal ovaries, B. Fetal adrenal, C. Placenta, D. Corpus luteum",D: Corpus luteum,"Answer: D: Corpus luteum Explanation: Ans: D. Corpus luteum(Ref Williams 24/e p169; Ganong 25/e p412, 24/e p414).Estrogen and progesterone in first 2 months of pregnancy - Produced by Corpus luteum.Functions of corpus luteum:Enlarged corpus luteum of pregnancy secretes estrogens, progesterone & relaxin.Progesterone & relaxin -Helps maintain pregnancy.By inhibiting myometrial contractions.Progesterone prevents prostaglandin production by uterus - Stops contractions.Corpus luteum function begins to decline after 8 weeks of pregnancy.Yet persists throughout pregnancy.In humans placenta produces sufficient estrogen & progesterone from maternal & fetal precursor taking over corpus luteum function after 6th week of pregnancy.Ovariectomy before 6th week a Hence cause aboion.Ovariectomy after 6th week doesn't affect pregnancy.Note:hCG secretion decreases after initial marked rise.Estrogen & progesterone secretion increase until just before paurition." "This is a real-world medical entrance exam question, please give the true answer based on the question and selection. Topic name: None, Subject name: Dental","Which of the following is the common factor for the initiation of both dental caries and periodontal disease Select and explain. A. bacterial plaque, B. lactic acid, C. calculus, D. no common factor",A: bacterial plaque,"Answer: A: bacterial plaque Explanation: None" "This is a real-world medical entrance exam question, please give the true answer based on the question and selection. Topic name: None, Subject name: Dental","Hidden X suture proves to be a novel suturing technique for: Select and explain. A. Alveolar ridge preservation, B. Apical repositioning flap, C. Coronal repositioning flap, D. Papillae repositioning flap",A: Alveolar ridge preservation,"Answer: A: Alveolar ridge preservation Explanation: None" "This is a real-world medical entrance exam question, please give the true answer based on the question and selection. Topic name: None, Subject name: Surgery","Tracheostomy will: Select and explain. A. Increase dead air space, B. Increase respiratory resistance, C. Decrease respiratory work, D. Cause obstruction in respiration",C: Decrease respiratory work,"Answer: C: Decrease respiratory work Explanation: None" "This is a real-world medical entrance exam question, please give the true answer based on the question and selection. Topic name: None, Subject name: Pathology","All are diseases of skin except Select and explain. A. Erythema multiforme, B. Keratosis folticularis, C. Erythema migrans, D. Psoriasis form lesion",C: Erythema migrans,"Answer: C: Erythema migrans Explanation: Option 'C' erythema migrans is other name of ectopic geographic tongue / annulus migrans in which lesions are not always restricted to the tongue, and similar irregular or circinate lesions occurring elsewhere in the oral cavity." "This is a real-world medical entrance exam question, please give the true answer based on the question and selection. Topic name: None, Subject name: Social & Preventive Medicine","Case control study was done regarding breast cancer & risk factors & odds ratio was obtained. Which of the following are causative factors ?ParameterCaseControlORAge at menarche>12171423291.04Menopause177124081.53OCP use93511051.02Smoking932140.97Family history5135021.10BMI>27.51654930.53Breastfeeding94225140.56Multiparity(2)277833661.05 Select and explain. A. 6, B. 5, C. 3, D. 2",B: 5,"Answer: B: 5 Explanation: Ans. (B) 5An odds ratio (OR) is a statistic that quantifies the strength of the association between two events, A and B.Two events are independent if and only if the OR equals 1: the odds of one event are the same in either the presence or absence of the other event.If the OR is greater than 1, then A and B are associated (correlated) in the sense that, compared to the absence of B, the presence of B raises the odds of A, and symmetrically the presence of A raises the odds of B.Conversely, if the OR is less than 1, then A and B are negatively correlated, and the presence of one event reduces the odds of the other event.Most Impoantly Odds ratio does not demonstrate causality .But we can say that they are positively correlated. Since 5 Factors have odds ratio more than 1 so the answer will be B" "This is a real-world medical entrance exam question, please give the true answer based on the question and selection. Topic name: None, Subject name: Microbiology","Compared with autoclave, use of a dry heat oven for sterilization has the following disadvantage: Select and explain. A. Time-consuming, B. Less effective, C. Causes instruments to rust, D. Causes sharp instruments to dull",A: Time-consuming,"Answer: A: Time-consuming Explanation: None" "This is a real-world medical entrance exam question, please give the true answer based on the question and selection. Topic name: None, Subject name: Social & Preventive Medicine","A screening test is used in same way in two similar populations, but the proportion of false positive results among those who test positive in population A is lower than among those who test positive in population B. What is the likely explanation for this finding? Select and explain. A. The prevalence of disease is higher in population A, B. The prevalence of disease is lower in population A, C. The specificity of the test is lower in population A, D. The specificity of the test is higher in population A",A: The prevalence of disease is higher in population A,"Answer: A: The prevalence of disease is higher in population A Explanation: Total population having the disease : ‘a + c’ (TP + FN) Total population not having the disease : ‘b + d’ (FP + TN) Total population: a + b + c + d = TP + FP + FN + TN PPV depends on sensitivity, specificity and prevalence of disease in the population. Now in this question, a screening test is used in same way in two similar populations; thereby the screening test will have similar sensitivity and specificity in both populations. PPV = a/ (a + (b) X 100, thus b (False Positive rate) is inversely proportional to PPV; and PPV is directly proportional to Prevalence of disease in a population. So, False Positive rate (FP rate) is inversely proportional to the prevalence of disease in the population. Therefore, if the same screening test is having lower FP rate in population A (as compared to a similar population B), then this could be explained by higher prevalence of disease in population A Also," "This is a real-world medical entrance exam question, please give the true answer based on the question and selection. Topic name: AIIMS 2018, Subject name: Pathology","Anti-centromere antibodies seen in which of the followings conditions: Select and explain. A. Drug induced lupus, B. SLE, C. Sjogren syndrome, D. Scleroderma",D: Scleroderma,"Answer: D: Scleroderma Explanation: It is seen in scleroderma. Limited form of scleroderma known as CREST has anti centromere antibody positive and generalized form has anti topoisomerase positive antibody Anti histone antibodies are positive in drug induced lupus Anti SS-A (RO) antibodies and anti SS-B(LA) antibodies are positive in Sjogren's Syndrome Most specific antibody in SLE is anti nuclear antibody and most specific antibody is anti ds-DNA and Anti smith antibody Presence of anti P antibody is associated with development if psychosis /CNS manifestation in SLE patients Presence of anti RO antibody is associated with development of cutaneous lupus and congenital hea block Presence of b2 glycoprotein is associated with development of Antiphospholipid antibody" "This is a real-world medical entrance exam question, please give the true answer based on the question and selection. Topic name: None, Subject name: Dental","Denture cleaning tablets include all, EXCEPT Select and explain. A. Hydrogen peroxide, B. Alkaline material, C. Sodium perborate, D. Detergent",A: Hydrogen peroxide,"Answer: A: Hydrogen peroxide Explanation: None" "This is a real-world medical entrance exam question, please give the true answer based on the question and selection. Topic name: None, Subject name: Gynaecology & Obstetrics","26 years old lady with H/o recurrent abortion which of the following investigations you will do to confirm the diagnosis ? Select and explain. A. PT, B. BT, C. Anti Russel viper venom antibodies, D. Clot solubility test",C: Anti Russel viper venom antibodies,"Answer: C: Anti Russel viper venom antibodies Explanation: None" "This is a real-world medical entrance exam question, please give the true answer based on the question and selection. Topic name: None, Subject name: Pharmacology","Shelf life of colourless, aqueous solution of suxamethonium is Select and explain. A. 6 months, B. 1 year, C. 2 yr, D. 3 yr",C: 2 yr,"Answer: C: 2 yr Explanation: None" "This is a real-world medical entrance exam question, please give the true answer based on the question and selection. Topic name: None, Subject name: Medicine","Infective endocarditis is least common in : Select and explain. A. Mitral stenosis., B. Aortic stenosis., C. VSD., D. ASD.",D: ASD.,"Answer: D: ASD. Explanation: None" "This is a real-world medical entrance exam question, please give the true answer based on the question and selection. Topic name: None, Subject name: Surgery","Facial nerve lies: Select and explain. A. Below SMAS & above parotidomasseteric fascia, B. Above SMAS & below parotidomasseteric fascia, C. Above SMAS & above parotidomasseteric fascia, D. Below SMAS & below parotidomasseteric fascia",D: Below SMAS & below parotidomasseteric fascia,"Answer: D: Below SMAS & below parotidomasseteric fascia Explanation: None" "This is a real-world medical entrance exam question, please give the true answer based on the question and selection. Topic name: None, Subject name: Dental","Which of the following is common to both zinc eugenol cement and polycarboxylate cement? Select and explain. A. Polyacrylic acid is liquid, B. Chemical bond to tooth structure, C. Chelation, D. Substitution of eugenol by EBA to increase strength of cement",C: Chelation,"Answer: C: Chelation Explanation: None" "This is a real-world medical entrance exam question, please give the true answer based on the question and selection. Topic name: AIIMS 2019, Subject name: Gynaecology & Obstetrics","What is the drug of choice of Bacterial vaginosis in pregnancy Select and explain. A. Metronidazole, B. Clindamycin, C. Erythromycin, D. Rovamycin",A: Metronidazole,"Answer: A: Metronidazole Explanation: BV is a maldistribution of normal vaginal flora Lactobacilli are decreased, and Anaerobic bacteria species are overrepresented. Anaerobes include Gardnerella, Prevotella, Mobiluncus, and Bacteroides species; Atopobium vaginae; BV-associated bacteria, provisionally named BVAB1, BVAB2, and BVAB3. Pregnancy -DOC - Metronidazole 500 mg twice daily orally for 7 days; 0.75-percent gel, intra-vaginally, daily for 5 days Clindamycin 2% One applicator intra-vaginally nightly for 7 days. 300 mg orally twice daily for 7 days 100-mg clindamycin ovules Vaginally nightly for 3 days" "This is a real-world medical entrance exam question, please give the true answer based on the question and selection. Topic name: None, Subject name: Biochemistry","Which of the following is having Maximum buffering capacity Select and explain. A. Histidine, B. Cysteine, C. Tyrosine, D. Arginine",A: Histidine,"Answer: A: Histidine Explanation: Answer-A- HistidineMaximum buffering capacity occurs at a pH equal to the pKa, So amino acid which has pKa range near physiologic pH can act as an effective buffer * lmidazole group of histidine - 6.5- 7.4 pKa range * SH group of cysteine- 8.5-9.0 pKa range * OH group of tyrosine - 9.5-10.5 pKa range* Arginine - > 12" "This is a real-world medical entrance exam question, please give the true answer based on the question and selection. Topic name: None, Subject name: Surgery","For >10 mm setback of mandible, which of these surgeries is most suitable: Select and explain. A. Sagittal split ramus osteotomy, B. Vertical ramus osteotomy, C. Subapical osteotomy, D. Body osteotomy with extraction of premolars",B: Vertical ramus osteotomy,"Answer: B: Vertical ramus osteotomy Explanation: None" "This is a real-world medical entrance exam question, please give the true answer based on the question and selection. Topic name: None, Subject name: Dental","In full tooth borne dentures occlusal rests transmits _________ percentage of occlusal forces to teeth: Select and explain. A. 50%, B. 70%, C. 80%, D. 100%",D: 100%,"Answer: D: 100% Explanation: None" "This is a real-world medical entrance exam question, please give the true answer based on the question and selection. Topic name: None, Subject name: Medicine","The definition of pyrexia of unknown origin includes all except: Select and explain. A. Diagnosis requires fever persisting for 3 weeks, B. Fever undiagnosed after 1 week of in-patient workup, C. Absence of immunological compromise, D. Temperature of 38.3degC or more",B: Fever undiagnosed after 1 week of in-patient workup,"Answer: B: Fever undiagnosed after 1 week of in-patient workup Explanation: Answer- B. Fever undiagnosed after 1 week of in-patient workupAccording to new definition of fever of unknown origin,fever undiagnosed after I week of in-patient work up is not included." "This is a real-world medical entrance exam question, please give the true answer based on the question and selection. Topic name: None, Subject name: Dental","Subantimicrobial dose of doxycycline is Select and explain. A. 20 mg doxycycline, B. 60 mg doxycycline, C. 80 mg doxycycline, D. 150 mg doxycycline",A: 20 mg doxycycline,"Answer: A: 20 mg doxycycline Explanation: None" "This is a real-world medical entrance exam question, please give the true answer based on the question and selection. Topic name: None, Subject name: Pediatrics","A child has bilirubin of 4 mg. Conjugated bilirubin and alkaline phosphatase are normal, bile salts and bile in urine are absent. However urobilinogen in urine is raised. What is the likely diagnosis – Select and explain. A. Obstructive jaundice, B. Rotor's syndrome, C. Biliary cholestasis, D. Hemolytic jaundice",D: Hemolytic jaundice,"Answer: D: Hemolytic jaundice Explanation: Important clues provided in question are - Increased total bilirubin I So, T unconjugated bilirubin Normal conjugated bilirubin Amongst the given options, only hemolytic jaundice causes increased unconjugated bilirubin. Remaining three cause conjugated hyperbilirubinemia." "This is a real-world medical entrance exam question, please give the true answer based on the question and selection. Topic name: None, Subject name: Dental","In mandibular primary second molar true statement is Select and explain. A. ML is largest cusp and distobuccal is smallest, B. All buccal cusp are fo same size> all lingual cusp are of same size, C. DB is largest cusp, D. All of the above",B: All buccal cusp are fo same size> all lingual cusp are of same size,"Answer: B: All buccal cusp are fo same size> all lingual cusp are of same size Explanation: None" "This is a real-world medical entrance exam question, please give the true answer based on the question and selection. Topic name: None, Subject name: Dental","The advantage of ultra speed hand piece is: Select and explain. A. Low frequency and high amplitude make the patient comfortable, B. High frequency and low amplitude do not let the patient perceive pain, C. Less trauma, D. None of the above",B: High frequency and low amplitude do not let the patient perceive pain,"Answer: B: High frequency and low amplitude do not let the patient perceive pain Explanation: None" "This is a real-world medical entrance exam question, please give the true answer based on the question and selection. Topic name: None, Subject name: Dental","Highest incidence of atypical root canal or root among all teeth is seen in Select and explain. A. Mandibular 1st premolar, B. Mandibular canine, C. Maxillary 1stpremolar, D. Mandibular 2nd premolar",A: Mandibular 1st premolar,"Answer: A: Mandibular 1st premolar Explanation: None" "This is a real-world medical entrance exam question, please give the true answer based on the question and selection. Topic name: None, Subject name: Medicine","Which Hodgkin’s disease is associated with best prognosis? Select and explain. A. Lymphocytic predominance, B. Lymphocyte depletion, C. Mixed cellularity, D. Nodular sclerosis",A: Lymphocytic predominance,"Answer: A: Lymphocytic predominance Explanation: None" "This is a real-world medical entrance exam question, please give the true answer based on the question and selection. Topic name: None, Subject name: Medicine","________occurs whenever there is an imbalance between myocardial oxygen supply and demand. Select and explain. A. Sudden Death, B. Stable angina, C. Unstable angina, D. Myocardial infarction",B: Stable angina,"Answer: B: Stable angina Explanation: None" "This is a real-world medical entrance exam question, please give the true answer based on the question and selection. Topic name: AIIMS 2019, Subject name: Biochemistry","Mutation seen in sickle cell anemia: Select and explain. A. Point, B. Inseion, C. Deletion, D. Frame shift",A: Point,"Answer: A: Point Explanation: Sickle-cell anemia Due to a single base alteration Point mutation Glutamate at the 6th position of b-chain of haemoglobin is replaced by Valine. UCA (serine) UCU (Serine) CCA (Proline) UAA (Stop codon) Silent mutation Mis-sense mutation Non sense Mutation" "This is a real-world medical entrance exam question, please give the true answer based on the question and selection. Topic name: AIIMS 2018, Subject name: Microbiology","Isolation period of measles:- Select and explain. A. Onset of prodromal stage until 7th day of rash, B. Eruptive stage until 2 days of rash, C. Onset of prodromal phase until 3rd day of rash, D. Eruptive stage until 7th day of rash",C: Onset of prodromal phase until 3rd day of rash,"Answer: C: Onset of prodromal phase until 3rd day of rash Explanation: Isolation period of measles is Onset of prodromal phase until 3rd day of rash. Maximum communicability occurs from onset of prodrome through the first 3-4 days of rash.Prodromal phase-stage last for 4 days(from 10th to 14th day of infection) characterized by Fever, Koplik spots , coryza, nasal discharge, redness .Incubation period of measles is 10 daysEruptive phase- Maculopapular rash appear after 4 days of fever (14day after infection)" "This is a real-world medical entrance exam question, please give the true answer based on the question and selection. Topic name: None, Subject name: Medicine","Endocarditis culture sampling is done as: Select and explain. A. 2 culture 12 hrly, B. 1 culture 12 hrly, C. 2 culture 24 hrly, D. 3 culture separated by 1 hr over 24 hour",D: 3 culture separated by 1 hr over 24 hour,"Answer: D: 3 culture separated by 1 hr over 24 hour Explanation: None" "This is a real-world medical entrance exam question, please give the true answer based on the question and selection. Topic name: None, Subject name: Pediatrics","A 6-year old girl presents with fever tor the past 5 days, generalized erythematous rash, strawberry tongue and cervical lymphadenopathy. The most likely diagnosis is: (Asked twice) Select and explain. A. Kimura disease, B. Kawasaki disease, C. Scarlet fever, D. Rosie-Dorfman syndrome",B: Kawasaki disease,"Answer: B: Kawasaki disease Explanation: Answer- B. Kawasaki diseaseThe most likely diagnosis is Kawasaki disease." "This is a real-world medical entrance exam question, please give the true answer based on the question and selection. Topic name: None, Subject name: Gynaecology & Obstetrics","Hematuria during labour in previous LSCS is sign Select and explain. A. Impending rupture of scar, B. Urethral trauma, C. Prolong labour, D. Sepsis",A: Impending rupture of scar,"Answer: A: Impending rupture of scar Explanation: Now friends - here in the question it is asked specifically that hematuria is seen in a patient with previous LSCS during labour - which indicates impending rupture of scar. ""There are no reliable signs of impending uterine rupture that occurs before labor, although the sudden appearance of gross hematuria is suggestive.” COGDT 10/e, p 340 Here in this questions obstructed labor is not given in the options, but even if it was given, I would have still opted for impending scar rupture as the question is specifically asking, in a case of previous LSCS." "This is a real-world medical entrance exam question, please give the true answer based on the question and selection. Topic name: None, Subject name: Surgery","Delayed haemolytic transfusion involves all except? Select and explain. A. Positive DAT, B. Spherocytes positive, C. Haemoglobinuria, D. Reduced haemoglobin",B: Spherocytes positive,"Answer: B: Spherocytes positive Explanation: None" "This is a real-world medical entrance exam question, please give the true answer based on the question and selection. Topic name: None, Subject name: Surgery","All the following can lead to damage of the axillary nerve except: Select and explain. A. Fracture of surgical neck of humerus, B. Intramuscular injection, C. Improper use of crutches, D. Shoulder dislocation",C: Improper use of crutches,"Answer: C: Improper use of crutches Explanation: Ans: C. Improper use of crutches(Ref Apley's* 9/c p282)Improper use of crutches causes radial nerve palsy mostly rather than axillaty nerve injury.Radial Nerve Injury:Very high lesions may be caused by trauma or operations around the shoulder.Due to chronic compression in the axilla.Seen in drink & drug addicts who fall into a stupor with the arm dangling over the back of a chair - Saturday night palsy.In thin elderly patients using crutches - Crutch palsy.Weakness of wrist & hand.Paralysed triceps.Absent triceps reflex." "This is a real-world medical entrance exam question, please give the true answer based on the question and selection. Topic name: AIIMS 2018, Subject name: Biochemistry","An infant presented to the OPD with a history of vomiting and malnutrition. Patient has blue eyes, blonde hair & fair skin. On investigation, Guthrie test was found to be positive. All are true regarding this disease EXCEPT: Select and explain. A. Due to PAH enzyme defect, B. White patch of hair due to tryptophan deficiency, C. Phenyl acetate positive in urine, D. Mental retardation is present",B: White patch of hair due to tryptophan deficiency,"Answer: B: White patch of hair due to tryptophan deficiency Explanation: This is Phenyl-ketonuria. Guthrie's bacterial inhibition test is a screening test for phenylketonuria (PKU), used to detect the abnormal presence of phenylalanine metabolites in blood i.e. Phenyl pyruvate, Phenyl alanine & Phenyl-lactate. FeCl3 detects phenyl alanine levels in urine. WITH REGARDS TO OTHER OPTIONS In Phenylketonuria, Phenylalanine is accumulated due to deficiency of enzyme Phenylalanine Hydroxylase. As a result Phenylalanine is not conveed to Tyrosine and is responsible for mental retardation in patients. Tyrosine becomes essential in these patients. CILINICAL MANIFESTATION CNS - Mental retardation, failure to walk and talk, growth retardation, seizures, tremors, microcephaly. Pigmentation - melanin is synthesized from tyrosine. so in PKU melanin is less result in hypopigmentation that cause light skin colour, blue eyes, fair hair Treatment - low phenylalanine content diet" "This is a real-world medical entrance exam question, please give the true answer based on the question and selection. Topic name: None, Subject name: Forensic Medicine","Which acid does not show coagulation necrosis on contact? Select and explain. A. HC1, B. H,SO4, C. HF, D. HNO3",C: HF,"Answer: C: HF Explanation: Ans: C. HF(Ref Reddy 34/e p493, 33/e p530; Principles of Clinical Toxicology 3/e p220; Forensic Pathology 3/ep241, 110).HF does not show coagulation necrosis on contact.Hydrofluoric acid causes liquefaction necrosis." "This is a real-world medical entrance exam question, please give the true answer based on the question and selection. Topic name: AIIMS 2018, Subject name: Physiology","Hormones which is/are under inhibitory control of hypothalamus: Select and explain. A. Prolactin, B. Prolactin only, C. GH, D. Both GH and prolactin",D: Both GH and prolactin,"Answer: D: Both GH and prolactin Explanation: Prolactin - predominantly under inhibitory control by the hypothalamus GH - hypothalamus exes inhibitory as well as stimulatory effect on GH secretion (GHIH = somatostatin has inhibitory effect on GH) (GHRH = stimulates GH secretion) Almost all secretions by the pituitary are controlled by either hormonal (anterior pituitary) or nervous (posterior pituitary) from the hypothalamus." "This is a real-world medical entrance exam question, please give the true answer based on the question and selection. Topic name: None, Subject name: Biochemistry","In myocardial infarction the first enzyme to rise: Select and explain. A. CPK, B. LDH, C. SGOT, D. Amylase",A: CPK,"Answer: A: CPK Explanation: None" "This is a real-world medical entrance exam question, please give the true answer based on the question and selection. Topic name: None, Subject name: Physiology","In a blood sample antiserum A and antiserum B and Rh +ve factor is added. No agglutination is seen. This is: Select and explain. A. O group and Rh+ve, B. O group and Rh-ve, C. AB group and Rh+ve, D. AB group and Rh-ve",B: O group and Rh-ve,"Answer: B: O group and Rh-ve Explanation: None" "This is a real-world medical entrance exam question, please give the true answer based on the question and selection. Topic name: None, Subject name: Microbiology","A 22 years old male presented with history of fever, sore throat and enlarged neck lymph nodes. He was ordered a Paul-Bunnell test with a suspicion of Infectious mononucleosis. What is the immunological basis behind the use of this test? Select and explain. A. Heterophile antibody test, B. Complement mediated agglutination reaction, C. Homophile antibody test, D. Latex agglutination test",A: Heterophile antibody test,"Answer: A: Heterophile antibody test Explanation: Answer- A. Heterophile antibody testEpstein-Bat virus (EBV) infection includes specific antibodies to EBV and various unrelated non-EBV heterophile antibodies. These heterophile antibodies reuct to antigens from animal RBCs. Sheep RBCs agglutinate in the presence of heterophile antibodies and are the basis for the Paul-Butrnell test." "This is a real-world medical entrance exam question, please give the true answer based on the question and selection. Topic name: None, Subject name: Pathology","Histopathologic study of lichen planus shows: Select and explain. A. Mixed cellular inflammatory infiltrate, B. Presence of T-Lymphocytes predominantly, C. Antiepithelial antibodies, D. Scattered infiltrate with ill-defined lower border",B: Presence of T-Lymphocytes predominantly,"Answer: B: Presence of T-Lymphocytes predominantly Explanation: Histologic features of lichen planus include: Saw, tooth appearance of retepegs Liquefaction degeneration of basal layer which is replaced by thin band of eosinophilic coagulum Presence of civatte bodies Characteristic band Like subepithelial mononuclear infiltrate consisting of T- lymphocytes and histiocytes sharply limited to papillary and most superficial portion of the reticular layers of connective tissue Hyperparakeratosis or hyperorthokeratosis Thickening of granular layer" "This is a real-world medical entrance exam question, please give the true answer based on the question and selection. Topic name: None, Subject name: Biochemistry","Markers of acute kidney injury inculde all of the following except: Select and explain. A. Micro RNA-122, B. Cystatin C, C. N-gal, D. Kim-1",A: Micro RNA-122,"Answer: A: Micro RNA-122 Explanation: None" "This is a real-world medical entrance exam question, please give the true answer based on the question and selection. Topic name: None, Subject name: Dental","The microwave method of processing resin is better than the conventional in that it provides: Select and explain. A. Better shine, B. Less porosity, C. Less shrinkage, D. Color stability",B: Less porosity,"Answer: B: Less porosity Explanation: None" "This is a real-world medical entrance exam question, please give the true answer based on the question and selection. Topic name: None, Subject name: Forensic Medicine","A 19-year-old woman presented with primary amenorrhea, sho stature, webbed neck and widely spaced nipples. Examination showed weak pulses in lower extremity and streak ovaries, raised FSH, no oocyte in histology of ovary. Karyotype most likely to be present:KCET 12; JIPMER 12; AIIMS 13; PGI 13 Select and explain. A. 45XO, B. 47XXY, C. 46XY, D. 46XX",A: 45XO,"Answer: A: 45XO Explanation: Ans. 45XOTurner's syndrome is a genetic disease with a karyotype of 45, X or 46, XX/45, X (mosaicism) or other structural abnormalities of X chromosomes.With conventional chromosomal studies, about 50% of Turner syndrome patients show a 45,X pattern.* Mosaicism of 45,X with other cell lines such as 46,XX, 46,XY, or 47,XXX are common.* Structural abnormalities of an X chromosome (deletions, rings, or translocations), either isolated or mosaic with a 45,X or 46,XX cell line, are also seen.* With modern cytogenetic techniques, mosaicism is increasingly being detected.* The sho stature in Turner syndrome appears to be caused by the absence of one copy of the SHOX gene, which is located on the sho arm of the X chromosome." "This is a real-world medical entrance exam question, please give the true answer based on the question and selection. Topic name: None, Subject name: Dental","Example of excavator are all except: Select and explain. A. Hoe, B. Ordinary hatchet, C. Enamel hatchet, D. Angle former",C: Enamel hatchet,"Answer: C: Enamel hatchet Explanation: The four subdivisions of excavators are Ordinary hatchets Hoes Angle formers Spoons Chisels are intended primarily for cutting enamel and may be grouped as Straight, slightly curved, or bin-angle Enamel hatchets Gingival margin trimmers Sturdevants operative dentistry 7th edition page e4" "This is a real-world medical entrance exam question, please give the true answer based on the question and selection. Topic name: None, Subject name: Anatomy","The special visceral afferent fibres of the facial nerve are located in which nuclei: Select and explain. A. Motor nucleus, B. Nucleus ambiguous, C. Nucleus of tractus solitarius, D. Lacrimatory nucleus",C: Nucleus of tractus solitarius,"Answer: C: Nucleus of tractus solitarius Explanation: None" "This is a real-world medical entrance exam question, please give the true answer based on the question and selection. Topic name: AIIMS 2018, Subject name: Biochemistry","Best investigation for metabolic disorders is? Select and explain. A. Western blot, B. Tandem mass spectrometry, C. PCR, D. Gel electrophoresis",B: Tandem mass spectrometry,"Answer: B: Tandem mass spectrometry Explanation: Most Powerful technique of screening of inborn error of metabolism is Tandem Mass Spectrometer. *TM Spectrometer combines two mass spectrometers. The first one is used to select a single (precursor) mass, which is characteristic of given analyte in the mixture. These selected ions are collided with neutral gas for activation. The second Mass Spectrometer is used to separate the fragment ions according to mass resulting in ""MS/MS"" or MS2 spectrum. Advantage is that large number of samples can be dealt with in a sho time.. *HPLC is used as a confirmatory test of the primary test by TMS. HPLC has limited capacity, so not a good screening method. * An alternative good screening method is Microfluorometry (MFL)." "This is a real-world medical entrance exam question, please give the true answer based on the question and selection. Topic name: None, Subject name: Dental","Strain is defined as: Select and explain. A. An applied load or force, B. A deformation resulting from an applied load, C. An external force opposing an applied load, D. An internal force opposing an applied Load",B: A deformation resulting from an applied load,"Answer: B: A deformation resulting from an applied load Explanation: Strain—Change in dimension per unit initial dimension. For tensile and compressive strain, a change in length is measured relative to the initial reference length. Strain, or the change in length per unit length, is the relative deformation of an object subjected to a stress. Strain may be either elastic, plastic, elastic and plastic, or viscoelastic. Elastic strain is reversible. The object fully recovers its original shape when the force is removed.  Phillips dental materials 12th edition page no 48, 51" "This is a real-world medical entrance exam question, please give the true answer based on the question and selection. Topic name: None, Subject name: Dental","Which cement is irritating to the pulp? Select and explain. A. Carboxylate cement, B. Zinc oxide-eugenol cement, C. Zinc phosphate, D. Ethoxybenzoic acid",C: Zinc phosphate,"Answer: C: Zinc phosphate Explanation: The phosphoric acid in the liquid makes the mixture quite acidic and therefore cytotoxic, when a prosthesis is luted with this cement on a prepared tooth. As setting occurs, the acidity is partially neutralized, but the cement remains acidic after 24 hours, with the pH rising from about 3 to about 6. For thin layers of dentin, a cavity liner (i.e.,Ca2) is recommended to prevent an adverse pulpal response from the pressure of luting, which forces acid into the pulp tissue. Younger patients are especially susceptible, because they have a more open dentin tubule area, whereas older patients with sclerotic dentin will have a more tortuous path that restricts the penetration of acid toward the pulp. Key concept​​ : pH of the cement The acidity is high at the time of insertion due to phosphoric acid. At the time of cementation, the pH is 2 (approx.). As time passes, the acidity reduces. By the end of 24 hours, the pH is 5.5, which is still in the acidic range (neutral value is 7). Pulpal response: The pulp response may be classified as moderate." "This is a real-world medical entrance exam question, please give the true answer based on the question and selection. Topic name: None, Subject name: Social & Preventive Medicine","APGAR scores of 30 children are recorded in a hospital and most of the readings are found to be 7 or above. What can you make out about this data distribution? Select and explain. A. Positively skewed data, B. Negatively skewed data, C. Normal distribution, D. Symmetrically skewed data",B: Negatively skewed data,"Answer: B: Negatively skewed data Explanation: Ans: B. Negatively skewed data(Ref Park 24Ie p886, 23Ie p847, 22Ie p786; High Yield Statistics/p67)Since more number of children has a higher APGAR score, the tail of the distribution curve will be towards the right.Hence, the data will be negatively skewed.Skewed Data Negative skewPositive skewLeft tail is longerdegMass of distribution is concentrated on the right of figuredegDistribution is said to be left-skewed, left-tailed, or skewed to the leftdeg.Right tail is longerdegMass of distribution is concentrated on the left of figuredeg.Distribution is said to be right-skewed, right-tailed, or skewed to the rightdeg." "This is a real-world medical entrance exam question, please give the true answer based on the question and selection. Topic name: None, Subject name: Dental","Is it possible to remove centric interference and working interference by reducing supporting cusps in natural dentition: Select and explain. A. Yes, B. No, C. Yes but we should avoid it, D. It is not possible to remove both the interferences simultaneously.",C: Yes but we should avoid it,"Answer: C: Yes but we should avoid it Explanation: None" "This is a real-world medical entrance exam question, please give the true answer based on the question and selection. Topic name: None, Subject name: Surgery","Incision given within hairline, 45° to zygoma is: Select and explain. A. Gillis temporal, B. Alkat Bramley, C. Risdon, D. Morey",A: Gillis temporal,"Answer: A: Gillis temporal Explanation: None" "This is a real-world medical entrance exam question, please give the true answer based on the question and selection. Topic name: None, Subject name: Biochemistry","Which of the following has least density? Select and explain. A. VLDL, B. LDL, C. HDL, D. Chylomicrons",D: Chylomicrons,"Answer: D: Chylomicrons Explanation: Major Classes of Lipoproteins: Based on ultracentrifugation, in the ascending order of density is Chylomicrons (Least density) Very Low Density Lipoproteins (VLDL) Low Density Lipoproteins (LDL) Caps-Intermediate density Lipoproteins (IDL) High Density Lipoproteins (HDL)" "This is a real-world medical entrance exam question, please give the true answer based on the question and selection. Topic name: None, Subject name: Medicine","A 42 years old male patient presented with jaundice. His AST was 48 U, ALT was 51 U, ALP, GGTP were normal. Ultrasound of liver was suggestive of cirrhosis. Viral markers were done and the following results were obtained.TestResultAnti-HAVNegativeAnti-HBsAgNegativeAnti-HBeAgNegativeAnti-HBcAg IgGPositiveAnti-HBcAg IgMNegativeHBsAgNegativeAnti-HCVPositiveAnti-HEVNegativeWhat is the next best step in management of this patient? Select and explain. A. Sta interferon therapy, B. Liver biopsy, C. -PCR for hepatitis C virus, D. PCR for HBV-DNA",C: -PCR for hepatitis C virus,"Answer: C: -PCR for hepatitis C virus Explanation: Ans: C. -PCR for hepatitis C virus(Ref Harrison 19/e p2017, 2018: 18/e p2551)Positive Anti-HBcAg IgG:Mere indicator of hepatitis B infection in past.Positive Anti-HCV antibody:Most sensitive indicator of HCV infection - Presence of HCV.Presence of Anti-HCV antibody points to hepatitis C infection a Hence HCV-RNA levels determined to establish chronic hepatitis C diagnosis before staing Interferon therapy.Requires molecular amplification by PCR.Testing for HCV RNA or repeat anti-HCV testing later stages - Necessary to establish diagnosis." "This is a real-world medical entrance exam question, please give the true answer based on the question and selection. Topic name: AIIMS 2018, Subject name: Anatomy","Action of the muscle marked (with arrow) on mandible is Select and explain. A. Elevation, B. Depression, C. Retraction, D. Protraction",A: Elevation,"Answer: A: Elevation Explanation: Muscle marked - temporalis Elevators of mandible: MTM Masseter Elevation, protraction Temporalis Elevation retraction Medial pterygoid Elevation, protraction Depressors of mandible Lateral pterygoid Depression, protraction Mylohyoid Depression Ant. belly of digastric Depression Muscle of mastication develop from 1st pharyngeal arch Nerve - mandibular branch of trigeminal." "This is a real-world medical entrance exam question, please give the true answer based on the question and selection. Topic name: None, Subject name: Pathology","Which is associated with defect in DNA repair Select and explain. A. Xeroderma pigmentosum, B. Albinism, C. Icthyosis, D. Sickle cell anaemia",A: Xeroderma pigmentosum,"Answer: A: Xeroderma pigmentosum Explanation: None" "This is a real-world medical entrance exam question, please give the true answer based on the question and selection. Topic name: None, Subject name: Biochemistry","A child to emergency with accidental ingestion of cyanide. It blocks citric acid cycle by blocking: Select and explain. A. Aconitase, B. Acetyl-CoA production, C. NAO, D. Citrate",C: NAO,"Answer: C: NAO Explanation: Ans: C. NAORef: Harper 30Ie p132)Cyanide blocks citric acid cycle by blocking NAD+" "This is a real-world medical entrance exam question, please give the true answer based on the question and selection. Topic name: None, Subject name: Dental","When a dentist says that "" I cannot fix your teeth if you do not open your mouth wide:"" He is employing: Select and explain. A. Problem ownership., B. Voice control, C. Tolerance., D. Flexibility.",A: Problem ownership.,"Answer: A: Problem ownership. Explanation: Problem ownership: Negative messages like 'You must sit still'; undermine the rapport between child and dentist. ""You"" messages carry the implication that the child is wrong. Instead ""I"" messages increase the flow of information. Eg: I cannot fix your teeth if you do not open your mouth wide. This is called problem ownership." "This is a real-world medical entrance exam question, please give the true answer based on the question and selection. Topic name: None, Subject name: Surgery","A 55-year-old male presents with severe backache for 10 days and urinary incontinence with a H/o Interveebral lumbar disc prolapse. There is no H/o fever or weight loss. What is the likely diagnosis? Select and explain. A. Potts spine, B. Multiple myeloma, C. Cauda equine syndrome, D. Bone metastasis",C: Cauda equine syndrome,"Answer: C: Cauda equine syndrome Explanation: Ans: C: Cauda equine syndromeRef: Apley!c system of ohopaedics and fracture 9"" ed., pg. 246,480Cauda equina is tuft of fibres which begins at the end of spinal cord.Compression over this pa may cause cauda equina syndrome.Causes of Cauda equina syndrome are:Lumbar disc herniation, Spinal canal stenosis, Trauma, Abscess etc." "This is a real-world medical entrance exam question, please give the true answer based on the question and selection. Topic name: None, Subject name: Dental","Metallic taste is due to SnF in saliva is Select and explain. A. Stannous triphosphate, B. Stannic triphosphate, C. Tin hydroxyl phosphate, D. Stannous hydroxide",C: Tin hydroxyl phosphate,"Answer: C: Tin hydroxyl phosphate Explanation: None" "This is a real-world medical entrance exam question, please give the true answer based on the question and selection. Topic name: None, Subject name: Surgery","Pediatric patient planned for costochondral graft for TMJ ankylosis is intubated best by Select and explain. A. Topical anesthesia plus sedation, B. General anaesthesia, C. Awake fibreoptic intubation, D. Tracheostomy",C: Awake fibreoptic intubation,"Answer: C: Awake fibreoptic intubation Explanation: None" "This is a real-world medical entrance exam question, please give the true answer based on the question and selection. Topic name: None, Subject name: Surgery","Thyroglossal fistula is: Select and explain. A. Lined by squamous epithelium, B. Causes a bulge in neck, C. Inflammatory lesion, D. Precancerous lesion",B: Causes a bulge in neck,"Answer: B: Causes a bulge in neck Explanation: None" "This is a real-world medical entrance exam question, please give the true answer based on the question and selection. Topic name: None, Subject name: Dental","Fluoride pit and fissure sealants belong to which generation? Select and explain. A. I, B. II, C. III, D. IV",D: IV,"Answer: D: IV Explanation: TYPES OF PIT AND FISSURE SEALANTS 1. Based on curing method: First generation: Polymerized with ultraviolet light of 350 nm wavelength.  Absorbs UV light excessively and prevents complete polymerization of the sealant.  Light intensity varies from lamp to lamp. Second generation: Self-cured or chemically cured.  Most of them were unfilled.  Can be transparent, opaque or tinted.  Filled show increased wear and abrasion resistance than unfilled. Third generation - Visible light cured of 480-490 nm wavelength.  Fourth generation - With addition of fluoride for added benefit. Soben Peter 5th edition Page no. 440" "This is a real-world medical entrance exam question, please give the true answer based on the question and selection. Topic name: None, Subject name: Dental","Which of the following is true about Catlan's appliance? Select and explain. A. It is used to treat cross-bite of maxillary posterior teeth, B. It is constructed on the lower anterior teeth with an inclined plane, C. If used for a long period it results in anterior deep bite, D. Of constructed on the upper anterior teeth it has a 45 degree angulation",B: It is constructed on the lower anterior teeth with an inclined plane,"Answer: B: It is constructed on the lower anterior teeth with an inclined plane Explanation: None" "This is a real-world medical entrance exam question, please give the true answer based on the question and selection. Topic name: None, Subject name: Dental","Which teeth are least involved in periodontitis? Select and explain. A. Lower incisor and lower molar, B. Lower premolar and upper canine, C. Upper molars and upper incisors, D. Lower incisors and upper molars",B: Lower premolar and upper canine,"Answer: B: Lower premolar and upper canine Explanation: None" "This is a real-world medical entrance exam question, please give the true answer based on the question and selection. Topic name: None, Subject name: Pharmacology","Prucalopride drug is ? Select and explain. A. 5HT4 agonist, B. 5HT2b agonist, C. 5HT2b antagonist, D. 5HT2a agonist",A: 5HT4 agonist,"Answer: A: 5HT4 agonist Explanation: Ans. A. 5HT4 agonist* Prucalopride is a drug acting as a selective, high affinity 5-HT4 receptor agonist which targets the impaired motility associated with chronic constipation, thus normalizing bowel movements.* Approved for use in Europe in 2009.* Prucalopride, a first in class dihydro-benzofuran-carboxamide, is a selective, high affinity serotonin (5-HT4) receptor agonist with enterokinetic activities.* Prucalopride alters colonic motility patterns serotonin 5-HT4 receptor stimulation: it stimulates colonic mass movements, which provide the main propulsive force for defecation.* The observed effects are exeed highly selective action on 5-HT4 receptor.* Prucalopride has >150-fold higher affinity for 5-HT4 receptors than for other receptors." "This is a real-world medical entrance exam question, please give the true answer based on the question and selection. Topic name: None, Subject name: Physiology","During voluntary movements, Golgi tendon organ has an impoant role to play because it continuously relays to the efferent neurons: Select and explain. A. Length of the muscle at rest, B. Change in angle of joint during motion, C. Change in length of muscle before and after the movement, D. Tension in the muscle",D: Tension in the muscle,"Answer: D: Tension in the muscle Explanation: Ans: D. Tension in the muscle(Ref.: Ganong 25/e p232; Guyton 13/e p697, 701).Golgi tendon organ senses muscle tension.""The Golgi organ (also called Golgi tendon organ, GTO, tendon organ, neurotendinous organ or neurotendinous spindle) senses changes in muscle tension.It is a proprioceptive sensory receptor organ that is at the origins and inseion of skeletal muscle fibers into the tendons of skeletal muscle. It provides the sensory component of the Golgi tendon reflex." "This is a real-world medical entrance exam question, please give the true answer based on the question and selection. Topic name: None, Subject name: Physiology","The hormone, which stimulates uterus contraction and lets down milk, is: Select and explain. A. Progesterone, B. Prolactin, C. Prostaglandin, D. Oxytocin",D: Oxytocin,"Answer: D: Oxytocin Explanation: None" "This is a real-world medical entrance exam question, please give the true answer based on the question and selection. Topic name: None, Subject name: Dental","Production and citation impact of published work of a scientist or scholar Select and explain. A. Impact factor, B. Citation, C. H index, D. Z index",B: Citation,"Answer: B: Citation Explanation: None" "This is a real-world medical entrance exam question, please give the true answer based on the question and selection. Topic name: None, Subject name: Dental","Ig in GCF? Select and explain. A. Ig A, B. Ig M, C. Ig G, D. Ig D",C: Ig G,"Answer: C: Ig G Explanation: None" "This is a real-world medical entrance exam question, please give the true answer based on the question and selection. Topic name: None, Subject name: Dental","The function of compensating curve is Select and explain. A. To provide balanced occlusion in complete dentures when mandible is protruded, B. To aid in establishing an incisal guide, C. Same as function of curve of spee, D. None of the above",A: To provide balanced occlusion in complete dentures when mandible is protruded,"Answer: A: To provide balanced occlusion in complete dentures when mandible is protruded Explanation: None" "This is a real-world medical entrance exam question, please give the true answer based on the question and selection. Topic name: None, Subject name: Anaesthesia","Induction of inhalational agent is faster. Select and explain. A. Agent with high blood gas solubility, B. Combined with nitrous oxide, C. Person with increased residual volume, D. Right to left shunt",B: Combined with nitrous oxide,"Answer: B: Combined with nitrous oxide Explanation: Answer- B. Combined with nitrous oxideInduction of inhalational agent is faster, if it is combined with nitrous oxide.'The blood:gas paition coelficient is the mainfactor that determines the rate of induction and recovery of an inhalationanaesthesic, and the lower the blood: gas paition coefficient, the faster is induction and recovery.The second gos effect usually refers to nitrous oxide combined with an inhalation agent. Because nitrous oxide is notsoluble in blood, its rapid absorption from alveoli causes an abrupt rise in the alveolar concentration of the otherinhalation anesthetic leading to faster induction." "This is a real-world medical entrance exam question, please give the true answer based on the question and selection. Topic name: None, Subject name: Dental","Hammock ligament is present: Select and explain. A. Between temporal and sphenoid bone, B. Between hamular notch and mandible, C. In apical area of a tooth, D. As a part of deep cervical fascia",C: In apical area of a tooth,"Answer: C: In apical area of a tooth Explanation: None" "This is a real-world medical entrance exam question, please give the true answer based on the question and selection. Topic name: None, Subject name: Dental","Centric holding cusp is? Select and explain. A. Mesiobuccal cusp of maxillary 1st molar, B. Mesiolingual cusp of mandibular first molar, C. Distolingual cusp of mandibular 1st molar, D. Mesiopalatal cusp of maxillary 1st molar",D: Mesiopalatal cusp of maxillary 1st molar,"Answer: D: Mesiopalatal cusp of maxillary 1st molar Explanation: None" "This is a real-world medical entrance exam question, please give the true answer based on the question and selection. Topic name: None, Subject name: Gynaecology & Obstetrics","What is the Ilekt step in management of a 32 years old woman with a 5 years history of primary infeil-ity with bilateral tubal block seen at cornu on hys-terosalpingogram? Select and explain. A. In vitro feilization, B. Laparoscopy and hysteroscopy, C. Intracytoplasmic sperm injection, D. Tuboplasty",B: Laparoscopy and hysteroscopy,"Answer: B: Laparoscopy and hysteroscopy Explanation: Answer- B. Laparoscopy and hysteroscopyBilateral tubal block at cornu should be confirmed using Laparoscopy and hysteroscopy (Chromopeubation test), which is the gold standard. Trestment of choice will be tuboplosty, but other causes like spasm shoukl be ruled out ss hysterosalpingogram is not a very reliable test." "This is a real-world medical entrance exam question, please give the true answer based on the question and selection. Topic name: None, Subject name: Dental","Pickling is done: Select and explain. A. To remove oxide film from casting, B. Polish the casting, C. Improve the strength of casting, D. Avoid casting defects",A: To remove oxide film from casting,"Answer: A: To remove oxide film from casting Explanation: None" "This is a real-world medical entrance exam question, please give the true answer based on the question and selection. Topic name: None, Subject name: Microbiology","MHC Class II proteins are expressed by: Select and explain. A. B-cells, dendritic cells and macrophages, B. Platelets, C. T-cells, D. All nucleated cells","A: B-cells, dendritic cells and macrophages","Answer: A: B-cells, dendritic cells and macrophages Explanation: Answer- A. B-cells, dendritic cells and macrophagesMHC Class II proteins are expressed by all antigen-presenting cells, which include B-cells, follicular dendritic cells and macrophages. MHC Class I proteins are expressed by all nucleated cells, which excludes platelets and RBCsHLA class I antigens (A, B and C) are found on the surface of viually all nucleated cells- They ere the principal antigens involved in graft rejection and cell-mediated cytolysis.Class, I molecules may function as components of hormonal receptors. HLA class II antigens are more restricted in distribution, being found only on cells of the immune system macrophages, dendritic cells, activated T cells, and paicularly on B cells.""" "This is a real-world medical entrance exam question, please give the true answer based on the question and selection. Topic name: None, Subject name: Dental","Soft tissue curettage is used for: Select and explain. A. Shallow pockets with gingivitis, B. Deep pockets with gingivitis, C. Infrabony pockets, D. Oedematous gingiva",A: Shallow pockets with gingivitis,"Answer: A: Shallow pockets with gingivitis Explanation: None" "This is a real-world medical entrance exam question, please give the true answer based on the question and selection. Topic name: None, Subject name: Dental","All are true about RVG except: Select and explain. A. 80% reduction of patient exposure, B. Instant imaging, C. Easy to storage and retrieval., D. Image is sharper than caused by silver halide.",D: Image is sharper than caused by silver halide.,"Answer: D: Image is sharper than caused by silver halide. Explanation: The digital radiographic resolution is lower than produced with silver halide emulsions." "This is a real-world medical entrance exam question, please give the true answer based on the question and selection. Topic name: None, Subject name: Biochemistry","HCO3/H2CO3 is the best buffer because it is: Select and explain. A. pKa near physiological pH, B. Its components can be increased or decreased in the body as needed, C. Good acceptor and donor of H+ ions, D. Combination of a weak acid and weak base",B: Its components can be increased or decreased in the body as needed,"Answer: B: Its components can be increased or decreased in the body as needed Explanation: Ans: B. Its components can be increased or decreased in the body as needed(Ref:Harper 30/e p11; Gaizung 25/e p6).HCO/H2CO3:Best buffer.As components can be increased or decreased in body as needed.Bicarbonate buffer system:Most powerful extracellular buffer in body.Since both elements of buffer system (HCO 3- & CO).Regulated respectively by kidneys & lungs.pH of extracellular fluid precisely controlled by HCO removal & addition by kidneys & CO 2 removal by lungs." "This is a real-world medical entrance exam question, please give the true answer based on the question and selection. Topic name: None, Subject name: Biochemistry","Which of the following is an essential fatty acid? Select and explain. A. Linoleic acid, B. Alpha linolenic acid, C. Both of the above, D. None of the above",C: Both of the above,"Answer: C: Both of the above Explanation: The fatty acids that are required by humans, but are not synthesized in the body, hence need to be supplied in the diet are known as essential fatty acid (EFA). Humans lack the enzymes that can introduce double bond beyond 9th Carbon. They are Polyunsaturated Fatty Acid namely: Linoleic acid. Alpha linolenic acid. Arachidonic acid is considered as semi-essential fatty acid as, it can be synthesized from linoleic acid." "This is a real-world medical entrance exam question, please give the true answer based on the question and selection. Topic name: None, Subject name: Physiology","In female adrenal gland secretes which hormone? Select and explain. A. Progesterone, B. Testosterone, C. Estrogen, D. DHEA",D: DHEA,"Answer: D: DHEA Explanation: Ans. D. DHEADHEA is an endogenous steroid hormone. This means it is naturally made by the body, and it spurs specific tissues or cells into action.It is also known as androstenolone, 3b-hydroxyandrost-5-en-17-one and 5-androsten-3b-ol-17-one.DHEA is one of the most abundant steroid hormones in the human body. It is produced by the adrenal glands, the gonads, and the brain.It is normally found in the form of dehydroepiandrosterone sulfate (DHEAS).The body holds DHEAS in reserve and conves it to specific hormones when needed.It is impoant for creating estrogen and androgen sex hormones and contributes to the development of so-called androgenic effects, or masculinization.These changes include the production of oilier skin, changes in body odor, and the growth of armpit and pubic hair." "This is a real-world medical entrance exam question, please give the true answer based on the question and selection. Topic name: None, Subject name: Physiology","Which of the following is the function of Hyperpolarizing Cyclic Nucleotide (HCN) gated channels? Select and explain. A. Cardiac rhythm generation, B. Generation of mitochondrial action potential, C. Myocardial muscle contraction, D. Memory formation",A: Cardiac rhythm generation,"Answer: A: Cardiac rhythm generation Explanation: Ans: A. Cardiac rhythm generationHyperpolarizing Cycling Nucleotide (HCN) gated channels:Present in SA & AV nodes.Channel opens in hyperpolarization phase.Essential for generating pacemaker potential in SA Node (cardiac rhythm generation).HCN-Nervous systemHCN-Cardiovascular systemControls neuronal excitability, synaptic transmission & rhythmic oscillatory activity in individual neurons & neuronal networks.Play an impoant role in synaptic plasticity & memory, thalamocoical rhythms & somatic sensation.Some evidence indicates they also play a role in mechanisms of epilepsy & pain.HCN4 is the main isoform expressed in the SA node, but low levels of HCN1 & HCN2 are also seen.Current through HCN channels, called ""funny current or pacemaker current"", plays a key role in generation & modulation of cardiac rhythmicity." "This is a real-world medical entrance exam question, please give the true answer based on the question and selection. Topic name: None, Subject name: Dental","A 4 year old child sustained a fracture in central incisor one month ago. On examination, a necrotic pulp was seen with no other pathological findings. The treatment of choice is: Select and explain. A. Watchful observation, B. Extraction followed by space maintainer, C. Pulpectomy and root canal filling with gutta percha, D. Endodontic treatment and root canal filling with ZOE",D: Endodontic treatment and root canal filling with ZOE,"Answer: D: Endodontic treatment and root canal filling with ZOE Explanation: A 4 year old child sustained a fracture in central incisor one month ago. On examination, a necrotic pulp was seen with no other pathological findings. The treatment of choice is endodontic treatment and root canal filling with ZOE." "This is a real-world medical entrance exam question, please give the true answer based on the question and selection. Topic name: None, Subject name: Biochemistry","Both ketogenic and glucogenic amino acids as Select and explain. A. Isoleucine, B. Leucine, C. Arginine, D. Glycine",A: Isoleucine,"Answer: A: Isoleucine Explanation: None" "This is a real-world medical entrance exam question, please give the true answer based on the question and selection. Topic name: None, Subject name: Pediatrics","An un-immunized 13 months old child comes to you in OPD, according to the latest immunizations schedule, what vaccines will you advise?? Select and explain. A. OPV 3 doses, I IPV 3 Pentavalent and I measles, B. BCG, OPV 3 doses, 3 lPV, 3 Pentavalent and I measles, C. OPV 3 doses, I IPV 3 Pentavalent and 2 measles, D. OPV 3 doses, 3 IPV 3 DPI : Hep-B","D: OPV 3 doses, 3 IPV 3 DPI : Hep-B","Answer: D: OPV 3 doses, 3 IPV 3 DPI : Hep-B Explanation: Ans: D. OPV 3 doses, 3 IPV 3 DPI : Hep-BRef: ip. o rg/file s/I A P-imm unizution-sc he du le- 2 0 I 6- I P- 2 0 I 6-E p u b. p df* The latest schedule of immunization under NIS is as follows:* vNational Immunization Schedule (NIS) for Infants, children and Presnant women - - OPV 3 doses, 3 IPV 3 DPI : Hep-B" "This is a real-world medical entrance exam question, please give the true answer based on the question and selection. Topic name: AIIMS 2018, Subject name: Gynaecology & Obstetrics","A 32 weeks pregnant female presented with labor pains and minimal vaginal discharge, on analysis of the cervicovaginal discharge showed presence of fetal fibronectin. What is the probable diagnosis? Select and explain. A. Preterm labour, B. IUGR, C. IUD, D. Cervical infection",A: Preterm labour,"Answer: A: Preterm labour Explanation: Fibronectin is a glycoprotein that binds amnion and chorion to the decidua of uterus. Normally present in the cervicovaginal secretions before 22 weeks and after 37 weeks of pregnancy. Presence of Fibronectin (>50ng/ml) in between these weeks is suggestive of preterm labor. When the test is negative it reassures that delivery will not occur within next 7 days." "This is a real-world medical entrance exam question, please give the true answer based on the question and selection. Topic name: None, Subject name: Medicine","A65-year-old male adult presents with chronic sinusitis, nasopharyngeal ulcers, cavitatory lung nodules and renal failure. What will be the appropriate next diagnostic step? Select and explain. A. Lung biopsy, B. Sputum AFB and PCR for TB, C. ANCA and evaluation for vasculitis, D. ESR",C: ANCA and evaluation for vasculitis,"Answer: C: ANCA and evaluation for vasculitis Explanation: Ans: C. ANCA and evaluation for vasculitis(Ref Harrison 19/e p2182-2184, 18/e p2786, 2789)Suggestive of Wegener's Granulomatosis.Wegener's Granulomatosis:c-ANCA positive small vessel vasculitis. levels are required for diagnosis.Tests:Specificity of a positive antiproteinase-3 ANCA for granulomatosis with polyangiitis (Wegener's) is very high, especially if active glomerulonephritis is present.However, the presence of ANCA should be adjunctive and, with rare exceptons, should not substitute for a tissue diagnosis." "This is a real-world medical entrance exam question, please give the true answer based on the question and selection. Topic name: None, Subject name: Medicine","All of the following are true about COPD except: Select and explain. A. Decreased FEV1, B. Decreased MEFR, C. Increased RV, D. Decreased diffusion capacity",D: Decreased diffusion capacity,"Answer: D: Decreased diffusion capacity Explanation: None" "This is a real-world medical entrance exam question, please give the true answer based on the question and selection. Topic name: None, Subject name: Dental","Bias that arises from evaluating data on patients and hospital records only such that probability of exposure to a particular factor is increased is known as: Select and explain. A. Berkesonian bias, B. Confound bias, C. Memory or recall bias, D. Selection bias",A: Berkesonian bias,"Answer: A: Berkesonian bias Explanation: None" "This is a real-world medical entrance exam question, please give the true answer based on the question and selection. Topic name: None, Subject name: Gynaecology & Obstetrics","A midwife at a PI-IC is monitoring pregnancy and maintaining the paograph of pregnancy progression. At how much cervical dilation should the paograph plotting be staed? Select and explain. A. 4 cm, B. 5 cm, C. 6 cm, D. 8 cm",A: 4 cm,"Answer: A: 4 cm Explanation: Answer- A. 4 cmPaograph recording is usually staed after a cervical dilation of 3 cm (not the 4 cm), i.e. the active stage of labor As 3 cm is not given in the option, we have to choose 4 cm." "This is a real-world medical entrance exam question, please give the true answer based on the question and selection. Topic name: None, Subject name: Pathology","Geniculate neuralgia is caused in the nerve Select and explain. A. VII, B. IX, C. X, D. II",A: VII,"Answer: A: VII Explanation: Geniculate neuralgia (Nervus intermedius neuralgia) results from herpes zoster infection of geniculate ganglion and nervus intermedius branch of seventh cranial nerve." "This is a real-world medical entrance exam question, please give the true answer based on the question and selection. Topic name: AIIMS 2018, Subject name: Radiology","Empty Thecal sac sign in: Select and explain. A. Arachnoiditis, B. Tethered Cord syndrome, C. Veebral osteomyelitis, D. Discitis",A: Arachnoiditis,"Answer: A: Arachnoiditis Explanation: * Arachnoiditis is chronic inflammation of meninges and can occur after intrathecal injection of contrast agent, infections, drugs leading to nerve root getting adhered into peripheral meninges giving the appearance of EMPTY THECAL SAC sign on T2 weighted MRI * Tethered Cord Syndrome is when the spinal cord extends below the elbow level due to lipoma of Filum terminal or post op scarring/ adhesions that prevent spinal cord from ascending upward and it remains below L2 level." "This is a real-world medical entrance exam question, please give the true answer based on the question and selection. Topic name: None, Subject name: Gynaecology & Obstetrics","An absolute indication for LSCS in case of a Heart disease is: Select and explain. A. Co-arctation of Aorta, B. Eisenmenger syndrome, C. Ebsteins anomaly, D. Pulmonary stenosis",A: Co-arctation of Aorta,"Answer: A: Co-arctation of Aorta Explanation: Heart disease during pregnancy, in itself is not an indication for cesarean section. Cesarean section in heart disease is done in specific cases. “In coarctation of aorta, elective cesarean section is indicated to prevent rupture of the aorta or mycotic cerebral aneurysm.” Dutta Obs. 7/e, p 278" "This is a real-world medical entrance exam question, please give the true answer based on the question and selection. Topic name: None, Subject name: Microbiology","Which is not true about vibrio cholera Select and explain. A. It is non-halophilic, B. Grows on simple media, C. Man is the only natural host, D. Cannot survive in extracellular environment",D: Cannot survive in extracellular environment,"Answer: D: Cannot survive in extracellular environment Explanation: None" "This is a real-world medical entrance exam question, please give the true answer based on the question and selection. Topic name: None, Subject name: Ophthalmology","Gene commonly indicated in congenital cataract: Select and explain. A. PAX-6, B. CRYGS-3, C. LMX- IB, D. PITX-3",B: CRYGS-3,"Answer: B: CRYGS-3 Explanation: Answer- B. CRYGS-3'Gene-S crystalline gene (CRYGS) mutalion causes dominant progressive coical cataract in humans." "This is a real-world medical entrance exam question, please give the true answer based on the question and selection. Topic name: None, Subject name: Pediatrics","A case of jaundice with 50% direct bilirubin, other LFTs normal. Diagnosis is – Select and explain. A. Rotor syndrome, B. Gilbert syndrome, C. Glucuronyl transferase deficiency, D. Primary biliary cirrhosis",A: Rotor syndrome,"Answer: A: Rotor syndrome Explanation: 50% direct bilirubin means conjugated hyperbilirubinemia. Normally the direct (conjugated) bilirubin is less than 15-20% of total bilirubin. Normal Values of bilirubin Total bilirubin       >             0.2 - 1.9 mg/d1.     Direct bilirubin >             0 - 0.3 mg/dl. So, this child has : - Conjugated hyperbilirubinemia Other LFTs normal Amongst the given options, Rotor syndrome and Primary biliary cirrhosis cause conjugated hyperbilirubinemia. In Primary biliary cirrhosis, other LFTs are also abnormal, e.g., SGOT and SGPT are raised. Now we are left with Rotor syndrome which causes conjugated hyperbilirubinemia. Other LFTs are normal." "This is a real-world medical entrance exam question, please give the true answer based on the question and selection. Topic name: None, Subject name: Dental","Pickling: Select and explain. A. Is accomplished by soaking the casting in baking soda, B. Causes porosity in gold, C. Removes surface oxides from gold castings, D. Remove investment from gold casting",C: Removes surface oxides from gold castings,"Answer: C: Removes surface oxides from gold castings Explanation: None" "This is a real-world medical entrance exam question, please give the true answer based on the question and selection. Topic name: None, Subject name: Surgery","Easiest 3rd molar surgical extraction is: Select and explain. A. Mesioangular, B. Horizontal, C. Vertical, D. Distoangular",A: Mesioangular,"Answer: A: Mesioangular Explanation: None" "This is a real-world medical entrance exam question, please give the true answer based on the question and selection. Topic name: None, Subject name: Dental","In examining the edentulous mouth of an aged patient wearing complete maxillary denture against six mandibular teeth, the dentist will see: Select and explain. A. Cystic degeneration of the foramina of the anterior palatine nerve, B. Loss of osseous structure in the anterior maxilla, C. Flabby tissue in the posterior region, D. Decrease interocclusal distance",B: Loss of osseous structure in the anterior maxilla,"Answer: B: Loss of osseous structure in the anterior maxilla Explanation: None" "This is a real-world medical entrance exam question, please give the true answer based on the question and selection. Topic name: None, Subject name: Surgery","Obstructive sleep apnoea caused by the following Select and explain. A. Mandibular ameloblasma, B. Dentigerous cyst, C. Orbital fracture, D. Bilateral TMJ ankylosis",D: Bilateral TMJ ankylosis,"Answer: D: Bilateral TMJ ankylosis Explanation: None" "This is a real-world medical entrance exam question, please give the true answer based on the question and selection. Topic name: AIIMS 2019, Subject name: Pathology","Von Willebrand factor is secreted by which of the following? Select and explain. A. Macrophages, B. Endothelial cells, C. Platelets, D. Fibroblast",B: Endothelial cells,"Answer: B: Endothelial cells Explanation: Endothelial cells contain intracellular stores known as WeibelPallad bodies which contain Von-willebrand factor and P-selectin." "This is a real-world medical entrance exam question, please give the true answer based on the question and selection. Topic name: None, Subject name: Medicine","Factors affecting the treatment of MI Select and explain. A. Regional wall motion abnormality, B. Troponin level, C. Both, D. None",C: Both,"Answer: C: Both Explanation: None" "This is a real-world medical entrance exam question, please give the true answer based on the question and selection. Topic name: None, Subject name: Pediatrics","All of the following are features of systemic Juvenile Rheumatoid Arthritis except – Select and explain. A. Uveitis, B. Rash, C. Fever, D. Hepatosplenomegaly",A: Uveitis,"Answer: A: Uveitis Explanation: The eye manifestation are seen in Pauciarticular and Polyarticular JRA but not in systemic JRA. Juvenile Rheumatoid Arthritis can be divided in 3 major clinical types" "This is a real-world medical entrance exam question, please give the true answer based on the question and selection. Topic name: None, Subject name: Medicine","An 80 kg male patient presented to the emergency with hypotension and you have been instructed to sta him on an inotrope at a dose of 10 mcg/kg/min. Each 5 mL amp of the drug contains 200 mg drug. You choose 2 ampules of the drug and decide to mix it with saline to make a 250 mL solution. What should be the flow rate of the drug solution to maintain the BP of the patient (assuming 16 drops = 1 mL)? Select and explain. A. 4 drops/min, B. 8 drops/min, C. 10 drops/min, D. 16 drops/min",B: 8 drops/min,"Answer: B: 8 drops/min Explanation: Ans: B. 8 drops/minA dosing of 10 mgm/kg/min of drug is requiredWeight = 80 kgTotal dose required = 10x80 = 800 mgm/min = 0.8 mg/minNow two 5 mL ls each containing 200 mg is diluted to a 250 mL solution.i.e. 400 mg is mixed in 250 mLConcentration of solution: 1 mL = 400/250 = 1.6 mg/mLNow, 1 mL 16 drops = 1.6 mg i.e. 16 drops contain 1.6 mgHence, 0.8 mg/min = 8 drops/min = 0.5 mL/min" "This is a real-world medical entrance exam question, please give the true answer based on the question and selection. Topic name: None, Subject name: Pediatrics","A two year old girl child is brought to the out patient with features of hand wringing stereotype movements, impaired language and communication development, breath holding spells, poor social skills and deceleration of head growth after 6 months of age. The most likely diagnosis is – Select and explain. A. Asperger's syndrome, B. Rett's syndrome, C. Fragile x–syndrome, D. Colarad syndrome",B: Rett's syndrome,"Answer: B: Rett's syndrome Explanation: This 2 years child has following problems. Hand wringing movements. Impaired language and communication development. Breath holding spells Poor social skills Decleration of hand growth after 6 months. Now see the clinical features of Rett syndrome. a. Age of onset is around 5 months. Development may proceed normally until 1 yr of age, when regression of language and motor milestones become apparent. a. Acquired microcephaly (Decleration of head growth due to significantly reduced brain weight). * Most children develop peculiar sighing respirations with intermittent periods of apnea that may be associated with cyanosis --> Breath holding spells. The hallmark of Rett syndrome is repetitive hand wringing movements and a loss ofpurposeful and spontaneous use of the hands. Autistic behavior is a typical finding in all patients. Generalized tonic-clonic convulsions occur in the majority. Feeding disorder and poor weight gain are common. Death occurs in adolescence or in the 3rd decade. Cardiac arrhythmias may result in sudden, unexpected death. Clinical features given in question prefectly match with Rett syndrome Asperger syndrome Qualitative impairment in the development of reciprocal social interaction. More common in males Normal intelligence. Eccentric interests. No language impairments that characterize autism. Children with Asperger syndrome appear to be at high risk for other psychiatric disorder" "This is a real-world medical entrance exam question, please give the true answer based on the question and selection. Topic name: None, Subject name: Dental","How many scores are used in modified Dean's fluorosis index? Select and explain. A. 4, B. 6, C. 8, D. 5",B: 6,"Answer: B: 6 Explanation: DEAN'S FLUOROSIS INDEX It was introduced by Trendley H. Dean in 1934. It is also known as 'Dean's Classification System For Dental Fluorosis'. The criteria for Dean's fluorosis index was based on a 7-point ordinal scale: normal, questionable, very mild, mild, moderate, moderately severe, and severe. Dean's Fluorosis Index – Modified Criteria (1942) 6 points – Normal, questionable, very mild, mild, moderate, severe. Essentials of preventive and community dentistry 5th edition Soben Peter" "This is a real-world medical entrance exam question, please give the true answer based on the question and selection. Topic name: AIIMS 2019, Subject name: Microbiology","Beta 3 glucan assay testing not done for Select and explain. A. Invasive candidiasis, B. Aspergillosis, C. Pneumocystis carnii, D. Mucormycosis",D: Mucormycosis,"Answer: D: Mucormycosis Explanation: Beta 3 glucan assay is used to detect the presence of beta 3 glucan in the fungal cell wall. It is used for Invasive candidiasis, Aspergillosis, Pneumocystis cranii but not for Mucormycosis." "This is a real-world medical entrance exam question, please give the true answer based on the question and selection. Topic name: None, Subject name: ENT","Topical treatment for recurrent respiratory papillomatosis includes: Select and explain. A. Acyclovir, B. Cidofovir, C. Ranitidine, D. Zinc",B: Cidofovir,"Answer: B: Cidofovir Explanation: Ans: B. Cidofovir(Ref Dhingra 74, p346, 6/c, p305).Cidofovir:Topical treatment for recurrent respiratory papillomatosis (Incomplete in stopping tumor growth).Mostly injected to control frequency of tumor growth." "This is a real-world medical entrance exam question, please give the true answer based on the question and selection. Topic name: None, Subject name: Dental","More than 80% cephalic index of a patient indicates which of the following Select and explain. A. Brachycephalic, B. Mesocephatic, C. Dolicocephalic, D. Depends on age",A: Brachycephalic,"Answer: A: Brachycephalic Explanation: None" "This is a real-world medical entrance exam question, please give the true answer based on the question and selection. Topic name: AIIMS 2018, Subject name: Pharmacology","Major determinant of loading dose of a drug is Select and explain. A. Half life, B. Clearance, C. Volume of distribution, D. Bioavailability",C: Volume of distribution,"Answer: C: Volume of distribution Explanation: LOADING DOSE = Vd * target plasma concentration MAINTENANCE DOSE = Clearance * target plasma concentration" "This is a real-world medical entrance exam question, please give the true answer based on the question and selection. Topic name: None, Subject name: Dental","Divergence from contact area in proximal surfaces causes embrasures: Select and explain. A. Facially, B. Lingually, C. Cervically, D. Facially, lingually, cervically & occlusally","D: Facially, lingually, cervically & occlusally","Answer: D: Facially, lingually, cervically & occlusally Explanation: None" "This is a real-world medical entrance exam question, please give the true answer based on the question and selection. Topic name: None, Subject name: Radiology","Yellowish-brown staining of the IOPA dental X-ray film is due to: Select and explain. A. Immersion in fixer without washing, B. Increased temperature of developer, C. Increased exposure time, D. Drying away of the developer solution",A: Immersion in fixer without washing,"Answer: A: Immersion in fixer without washing Explanation: None" "This is a real-world medical entrance exam question, please give the true answer based on the question and selection. Topic name: None, Subject name: Surgery","Which of the following is not ture about increased intracraniaL pressure Select and explain. A. Headache, B. Nausea / vomiting, C. Muscle twitching, D. Somnolence",C: Muscle twitching,"Answer: C: Muscle twitching Explanation: None" "This is a real-world medical entrance exam question, please give the true answer based on the question and selection. Topic name: None, Subject name: Dental","Retromolar pad: Select and explain. A. Should not be covered by Low denture, B. Should be covered by lower denture, C. Has tendon of temporal muscle attached to it, D. Disappears on eruption of mandibular last molars",B: Should be covered by lower denture,"Answer: B: Should be covered by lower denture Explanation: None" "This is a real-world medical entrance exam question, please give the true answer based on the question and selection. Topic name: None, Subject name: Biochemistry","The process of transfer of information from the RNA to the proteins is called: Select and explain. A. Mutation, B. Translation, C. Transcription, D. Conjugation",B: Translation,"Answer: B: Translation Explanation: None" "This is a real-world medical entrance exam question, please give the true answer based on the question and selection. Topic name: None, Subject name: Pharmacology","All of the following are potentially serious side effects of thioamide group of antithyroid drugs except: Select and explain. A. Hepatic dysfunction, B. Severe rash, C. Agranulocytosis, D. Anaphylaxis",D: Anaphylaxis,"Answer: D: Anaphylaxis Explanation: Ans: D. Anaphylaxis(Ref. Goodman Gilman 12/e p1149; Katzung 13/e p671, 12/e p688)Adverse effects of anti-thyroid drugs:Most serious reaction: AgranulocytosisMC reaction: Mild, purpuric, uicarial papular rash.Less frequent complications: Pain & stiffness in joints, paresthesias, headache, skin pigmentation & loss of hair.ANCAs in -50% of patients receiving propylthiouracil.Cholestatic jaundice - More common with methimazole than propylthiouracil.Propylthiouracil-associated hepatic failure." "This is a real-world medical entrance exam question, please give the true answer based on the question and selection. Topic name: None, Subject name: Pathology","In which stage of neurocysticercosis, there is no edema? Select and explain. A. Vesicular, B. Vesicular colloidal, C. Granular nodular, D. Nodular calcified",D: Nodular calcified,"Answer: D: Nodular calcified Explanation: Ans. d. Nodular calcified (Ref Robbins 9/e1)395, SA, 1)392-393)Surrounding edema is seen in the colloidal vesicular and granular nodular stages.StageCyst wallScolexCommentVesicularNon-enhancingWall defined membraneOnly one ble scolexEccentric hyperdense hole-with-dot"" appearanceSuggestive of ble larvaColloidalRing Enhancing with perilesional edemaDegenerating scolexFluid becomes more turbidEarliest stage in the cyst involution - larval degenerationGranularFocal nodular enhancing necrotic lesions with perilesional edemaDegenerating scolexEosinophilic structure Bladder and scolex are in various stages of disintegationCalcifiedSmall hyperdense nodules without perilesional edema" "This is a real-world medical entrance exam question, please give the true answer based on the question and selection. Topic name: AIIMS 2018, Subject name: Pathology","Which of the following is not a pa of the quadruple test for antenatal detection of Down syndrome? Select and explain. A. AFP, B. Estriol, C. Beta HCG, D. Inhibin B",D: Inhibin B,"Answer: D: Inhibin B Explanation: TRIPLE TEST b Hcg || a feto protein || Estriol || QUADRAPLE TEST BHcg || A feto protein || Estriol || Inhibin A ||" "This is a real-world medical entrance exam question, please give the true answer based on the question and selection. Topic name: None, Subject name: Microbiology","Antibody-dependent enhancement is implicated in the immunopathogenesis of which disease? Select and explain. A. Influenza, B. Staphylococcal toxic shock syndrome, C. Waterhouse-Friderichsen syndrome, D. Dengue hemorrhagic fever",D: Dengue hemorrhagic fever,"Answer: D: Dengue hemorrhagic fever Explanation: Ans: D. Dengue hemorrhagic feverAntiborly-dependent enhancement is implicated in the immunopathogenesis of Dengue hemorrhagic fever." "This is a real-world medical entrance exam question, please give the true answer based on the question and selection. Topic name: None, Subject name: Pharmacology","The drug not belonging to amide group Select and explain. A. Procaine, B. Xylocaine, C. Lignocaine, D. Bupivacaine",A: Procaine,"Answer: A: Procaine Explanation: None" "This is a real-world medical entrance exam question, please give the true answer based on the question and selection. Topic name: None, Subject name: Dental","Extra retention in abutment teeth is obtained with: Select and explain. A. Dovetail, B. Slots, pins and grooves, C. Outline form, D. Increasing tooth reduction","B: Slots, pins and grooves","Answer: B: Slots, pins and grooves Explanation: None" "This is a real-world medical entrance exam question, please give the true answer based on the question and selection. Topic name: None, Subject name: Dental","Sonic frequency range is: Select and explain. A. 1000-2000 Hz, B. 1500-6000 Hz, C. 20,000-30,000 Hz, D. 10,000 Hz",B: 1500-6000 Hz,"Answer: B: 1500-6000 Hz Explanation: None" "This is a real-world medical entrance exam question, please give the true answer based on the question and selection. Topic name: None, Subject name: Dental","Cause of apical periodontitis is/are: Select and explain. A. Sequel of pulpal diseases, B. Wedging of foreign object between the teeth, C. High points in restoration, D. All of the above",D: All of the above,"Answer: D: All of the above Explanation: Apical periodontitis (AP) may occur in a vital or nonvital tooth. Causes of AP in a vital tooth: Abnormal occlusal contacts High points in restoration Wedging of foreign object between the teeth Traumatic blow to teeth (Eg: Concussion). Causes of AP in a nonvital tooth: Sequelae of pulpal diseases   Iatrogenic Apical extrusion of debris through apical foramen during RCT. Pushing irrigants or medicaments or obturating material through apical foramen." "This is a real-world medical entrance exam question, please give the true answer based on the question and selection. Topic name: None, Subject name: Dental","Which of the following series act as the best space maintainer in a child's mouth? Select and explain. A. Restored deciduous tooth, B. Acrylic partial denture, C. Distal shoe space maintainer, D. Band and loop space maintainer",A: Restored deciduous tooth,"Answer: A: Restored deciduous tooth Explanation: None" "This is a real-world medical entrance exam question, please give the true answer based on the question and selection. Topic name: None, Subject name: Pediatrics","Most common cause of postauricular lymphadenopathy in children : Select and explain. A. Sore throat, B. Pediculosis capitis, C. Pulmonary Koch's, D. Chronic suppurative otitis media",B: Pediculosis capitis,"Answer: B: Pediculosis capitis Explanation: Posterior or Suboccipital cervical lymphadenopathy is caused by Roseola, Rubella, Scalp infections. The lymphatic drainage of posterior scalp is to the occipital region. Posterior or Suboccipital cervical lymphadenopathy without obvious disease or secondary infection is characteristic of lice. Secondary pyodenna due to scratching may result in matting together of the hair and cervical and occipital lymphadenopathy." "This is a real-world medical entrance exam question, please give the true answer based on the question and selection. Topic name: None, Subject name: Surgery","Infected mandibular angle fracture is treated by? Select and explain. A. Mini plates, B. Reconstruction plates, C. Champy plate at upper border, D. IMF + ID",D: IMF + ID,"Answer: D: IMF + ID Explanation: We can never place a foreign object(implant mini plates or reconstruction plates) at infection site." "This is a real-world medical entrance exam question, please give the true answer based on the question and selection. Topic name: None, Subject name: Dental","Of all the solvent used to dissolve Gutta Percha, safest & efficacious G.P. solvent is? Select and explain. A. Halothane, B. Chloroform, C. Methylchloroform, D. Xylene",B: Chloroform,"Answer: B: Chloroform Explanation: None" "This is a real-world medical entrance exam question, please give the true answer based on the question and selection. Topic name: None, Subject name: Physiology","The following enzyme is responsible for thirst mechanism during dehydration? Select and explain. A. ADH, B. Noradrenaline, C. Epinephrine, D. Dopamine",A: ADH,"Answer: A: ADH Explanation: Ans. A. ADH* Because, arginine vasopressin (AVP) also known as antidiuretic hormone, cannot reduce water loss below a ceain minimum level obligated by urinary solute load and evaporation from skin and lungs, a mechanism for ensuring adequate intake is essential for preventing dehydration.* This vital function is performed by the thirst mechanism.* Like AVP, thirst is regulated primarily by an osmostat that is situated in the anteromedial hypothalamus and is able to detect very small changes in the plasma concentration of sodium and ceain other effective solutes." "This is a real-world medical entrance exam question, please give the true answer based on the question and selection. Topic name: None, Subject name: Medicine","Ptosis may be caused by a lesion of the: Select and explain. A. Oculomotor nerve, B. Superior oblique, C. Trigeminal nerve, D. Trochlear nerve",A: Oculomotor nerve,"Answer: A: Oculomotor nerve Explanation: None" "This is a real-world medical entrance exam question, please give the true answer based on the question and selection. Topic name: None, Subject name: Pediatrics","The treatment of choice for primary grade V vesicoureteric reflux involving both kidneys in a 6 month old boy is – Select and explain. A. Antibiotic prophylaxis, B. Ureteric reimplantation, C. Cystoscopy followed by subureteric injection of teflon, D. Bilateral ureterostomies.",A: Antibiotic prophylaxis,"Answer: A: Antibiotic prophylaxis Explanation: Since the age of the boy is less than 1 year we will try medical management. See the below given tables from Nelson and the answer will be clear. Treatment Recommendation for VUR diagnosed following a UTI" "This is a real-world medical entrance exam question, please give the true answer based on the question and selection. Topic name: None, Subject name: Pharmacology","Idiosyncratic side-effects of carbamazepine are all except: Select and explain. A. Steven-Johnson syndrome, B. Agranulocytosis, C. Rash, D. Blurred vision",D: Blurred vision,"Answer: D: Blurred vision Explanation: Answer- D. Blurred visionDiplopia (not the blurring of vision) is a dose related side effect of carbamazepine, while others are idiosyncratic, reactions.""The most common dose-related adverse effects of carbamazepine are diplopia and ataxia. The diplopia often occursfirst and may last less than an hour during a paicular time of day" "This is a real-world medical entrance exam question, please give the true answer based on the question and selection. Topic name: None, Subject name: Pathology","Neoplasia of blood vessel is called Select and explain. A. Angioma, B. Hematoma, C. Lymphosarcoma, D. Papilloma",A: Angioma,"Answer: A: Angioma Explanation: None" "This is a real-world medical entrance exam question, please give the true answer based on the question and selection. Topic name: None, Subject name: Anatomy","Which of the following is innervated by the vagus nerve? Select and explain. A. The levator veli palatine (levator palatini), B. The posterior belly of digastric, C. Mylohyoid, D. The tensor veli palatine (tensor palatini)",A: The levator veli palatine (levator palatini),"Answer: A: The levator veli palatine (levator palatini) Explanation: None" "This is a real-world medical entrance exam question, please give the true answer based on the question and selection. Topic name: AIIMS 2017, Subject name: Pathology","A voluntary donor underwent apheresis for platelet donation for the first time after which he developed perioral tingling and numbness. This is seen because Select and explain. A. His platelet count was low for donation, B. He underwent apheresis for the first time, C. Due to fluid depletion, D. Due to citrate based anticoagulant",D: Due to citrate based anticoagulant,"Answer: D: Due to citrate based anticoagulant Explanation: SDP-Single Donor Platelet are preffered. -In this process whole bloodis drawn from one arm into a sterile kit inside a cell separating machine. -The machine separates the bloodso that onlyplateletsand plasma are collected. -The other bloodcomponents (red cells and white cells) are returned to thedonor the same arm. -Because we want the blood to flow in the proper way, so we will do addition of anti-coagulant solution like citrate based anti-coagulant. -This anti-coagulant solution will not only prevent clot formation but will also cause decrease in the concentration of ionized Ca2+ - causing transient hypocalcemia which will lead to Perioral tingling and numbness. To prevent this condition from happening we prescribe oral calcium supplementation to the respective patient or we reduce the flow of device through which the blood is supposed to pass." "This is a real-world medical entrance exam question, please give the true answer based on the question and selection. Topic name: None, Subject name: Gynaecology & Obstetrics","P3L3 came to opd with postcoital bleeding and pap positive p/v cervix hyperophied bleed on touch diagnosis - Select and explain. A. CA cervix, B. Fibroid, C. Cervicitis, D. Cervical polyp",A: CA cervix,"Answer: A: CA cervix Explanation: Ans. A. CA cervixSYMPTOMS:Arises from: Squamo-columnar junctionEarliest symptom: Post-coital bleedingAs the cancer progresses, symptoms may include:Unusual vaginal dischargeVaginal bleeding between periodsBleeding after menopausePyometraBleeding or pain during sexMC site: EctocervixLymph nodes affected: Obturator, hypogastric and external iliacTime taken for conversion of CIN to invasive Ca: 10 yearsMC type: Squamous cell Ca100% cure rates are seen in: CISUremia: altered sensorium and is having hiccupsMC cause of death: Renal failureCa cervix can be prevented by screeningBoth positive Pap smear and test is suggestive of CA cervix" "This is a real-world medical entrance exam question, please give the true answer based on the question and selection. Topic name: AIIMS 2019, Subject name: Surgery","Which of the following PPE (Personal Protective Equipment) is removed first? Select and explain. A. Gloves, B. Face shield, C. Gown, D. Mask",A: Gloves,"Answer: A: Gloves Explanation: Personal protective equipment: PPE - protect the user against health or safety risks at work. Can include items such as safety helmets, gloves, eye- protection, high visibility clothing, safety footwear and safety harness. It can also include respiratory protective equipment (RPE) Order of removing PPE: Gloves Face shield Gown Mask" "This is a real-world medical entrance exam question, please give the true answer based on the question and selection. Topic name: None, Subject name: Pathology","A cyst occurs under the tongue, caused by obstruction of a salivary gland. Such a cyst is called: Select and explain. A. Mucocele, B. Ranula, C. Derrnoid cyst, D. Dentigerous cyst",B: Ranula,"Answer: B: Ranula Explanation: None" "This is a real-world medical entrance exam question, please give the true answer based on the question and selection. Topic name: None, Subject name: Gynaecology & Obstetrics","What is the maximum capacity of Bakri balloon which is used in post paum hemorrhage? Select and explain. A. 200 mL, B. 300 in L, C. 500 mL, D. 1000 m L",C: 500 mL,"Answer: C: 500 mL Explanation: Ans: C. 500 mLBakri balloon:Inseed & inflated to tamponade endometrial cavity to stop bleeding.Initially staed by rapidly infusing at least 150 mL followed by fuher instillation over a few minutes for a total of 500 mL to arrest hemorrhage." "This is a real-world medical entrance exam question, please give the true answer based on the question and selection. Topic name: None, Subject name: Pharmacology","All of these are G2 phase blockers except: Select and explain. A. Etoposide, B. Topotecan, C. Paclitaxel, D. Daunorubicin",C: Paclitaxel,"Answer: C: Paclitaxel Explanation: Ans: C. Paclitaxel(Ref: Goodman Gilman 121e p1708; Katzung 13/e p932, 12Ie p963; KDT 7/e p865)Paclitaxel:Taxane, which act by causing disruption of the cell's microtubule function by stabilizing microtubule formation.Acts on the M-phase of cell division.Binds specifically to the P-tubulin subunit of microtubules and antagonizes the disassembly of this key cvtoskeletal protein, with the result that bundles of micro-tubules and aberrant structures derived from microtubules appear in the mitotic phase of the cell cycle.Arrest in mitosis follows." "This is a real-world medical entrance exam question, please give the true answer based on the question and selection. Topic name: None, Subject name: Dental","The primary action of topical fluoride is a: Select and explain. A. Conversion of hydroxyapatite to fluoroapatite, B. Decrease in the plaque bacteria, C. Form a reservoir in saliva, D. Improve morphology of teeth",A: Conversion of hydroxyapatite to fluoroapatite,"Answer: A: Conversion of hydroxyapatite to fluoroapatite Explanation: None" "This is a real-world medical entrance exam question, please give the true answer based on the question and selection. Topic name: None, Subject name: Pediatrics","A child is admitted on 7 days of life with severe respiratory distress and shock. He was discharged 2 days back healthy. What could be the probable diagnosis – Select and explain. A. VSD large, B. Hypoplastic left heart syndrome, C. Ebstein anomaly, D. AP window defect",B: Hypoplastic left heart syndrome,"Answer: B: Hypoplastic left heart syndrome Explanation: livpolastic left heart syndrome Hypoplastic left heart syndrome occurs when parts of the left side of the heart (mitral valve, left ventricle, aortic valve, and aorta) do not develop completely. The condition is present at birth (congenital). Hypoplastic left heart is a rare type of congenital heart disease. It is more common in males than in females. As with most congenital heart defects, there is no known cause. About 10 % of patients with hypoplastic left heart syndrome also have other birth defects. The problem develops before birth when the left ventricle and other structures do not grow properly, including the: (i) Aorta-the blood vessel that carries oxygen-rich blood from the left ventricle to the entire body Entrace and exit of the ventricle Mitral and aortic valves This causes the left ventricle and aorta to be poorly developed, or hypoplastic. In most cases, the left ventricle and aorta are much smaller than normal. In patients with this condition, the left side of the heart is unable to send enough blood to the body. As a result, the right side of the heart must maintain the circulation for both the lungs and the body. The right ventricle can support the circulation to both the lungs and the body for awhile, but this extra workoad eventually causes the right side of the heart to fail. The only possibility of survival is a connection between the right and left side of the heart, or between the systemic arteries and pumonary arteries (the blood vessels that carry blodd to the lungs). Babies are normally born with two of these connections: (i) Foramen ovale (a hole between the right and left atrium) Ductus arteriosus (a small vessel that connects the aorta to the pulmonary artery) Both of these connections normally close on their own a few days after birth. In babies with hypoplastic left heart syndrome, blood from the right side of the heart travels through the ductus arteriosus. This is the only way for blood to get to the body. if the ductus arteriosus is allowed to close in a baby with hypoplastic left heart syndrome, the patient may quickly die because no blood will be pumped to the body. Babies with known hypoplatic left heart syndrome are usually started on a medicine to keep the ductus arteriosusopen. Because there is little or no flow out of the left heart, blood reaming to the heart from the lungs needs to pass through the foramen ovale or an atrial septal defect (a hole connecting the collecting chambers on the left and right sides of the heart) back to the right side of the heart. If there is no foramen ovale, or if it is too small, the baby could die. Patients with this problem have the hole between their atria opened, either with surgery or using a thin, flexible tube (heart catheterization). Symptoms At first, a newborn with hypoplastic left heart may apper normal. Symptoms usually occur in the first few hours of life, although it may take up to a few days to develop symptoms. These symptoms may include: Bluish (cyanosis) or poor skin color Cold hands and feed (extremities) Lethargy Poor pulse Poor suckling and feeding Pounding heart Rapid breathing Shortness of breath. Since the systemic circulation is dependent on the patent ductus arteriosus the closure of ductus arterious leads to shock. Signs of heart failure usually appears within the first few days or weeks of life and include dyspnoea, hepatomegaly and low cardiac output. When PDA closes suddenly shock occurs all the peripheral pulses may be weak or absent. Ventricular septal defect These patients with VSD's become symptomatic around 6-10 weeks of age. They usually present with congestive cardiac failure. Ebstein's anomaly Ebstein's anomaly consists of downward displacement of an abnormal tricuspid value into the right ventricle. These patients usually present in teenage/adolescent years. They may also present in infancy but they usually do not present with shock or severe hypoperfusion. Aorticopulmonary window defect An Aorticopulmanry window defect consists of a communication between the ascending Aorta and the main pulmonary artery. In these cases minimal cyanosis is present and they may present with heart failure in infancy." "This is a real-world medical entrance exam question, please give the true answer based on the question and selection. Topic name: None, Subject name: Pathology","A 20 years old boy presented with persistent cervical lymphadenopathy for the past 1 year. Histopathology of lymph node shows Reed-Sternberg cells with focal nodularity and background of T reactive lymphocytes. The cells were positive for CD20, LCA, EMA and negative for CD15 and CD30 and EBV negative. Diagnosis is: Select and explain. A. Nodular lymphocyte predominant Hodgkin's lymphoma, B. Lymphocyte rich Hodgkin's lymphoma, C. Diffuse large B-cell lymphoma, D. Small cell lymphoma",A: Nodular lymphocyte predominant Hodgkin's lymphoma,"Answer: A: Nodular lymphocyte predominant Hodgkin's lymphoma Explanation: Answer- A. Nodular lymphocyte predominant Hodgkin's lymphomaThis tumor contains so-called L&H (lymphocytic and histiocytic) variants, which have a multilobed nucleus resembling a popcorn kernel (""popcorn cell""). Eosinophils and plasma cells are usually scant or absent. In contrast to the Reed-Sternberg cells found in classical forms of HL, L&E variants express B-cell markers typical of germinal-cenler B cells, sach as CD20 and BCL6, and are usually negative for CD15 and CD30.Histological diagnosis is established by presence of Reed-Sterntrerg cells along with background of mixed inflammation consisting of neutrophils, plasma cells, eosinophils & histiocytes.Reed-Sternberg cells are large and are either multinucleated or have a bilobed nucleus (thus resembling an ""owl's eye"" appearance) with prominent eosinophilic inclusion-like nucleoli.Reed-Sternberg cells are CD30 & CD15 positive, usually negative for CD20 & CD45.Nodular sclerosisMC subtype; usually stage I or ll disease frequent mediastinal involvementMore common in females, most patients young adults" "This is a real-world medical entrance exam question, please give the true answer based on the question and selection. Topic name: None, Subject name: Anatomy","Among all of the following foramens in the base of skull, which is, the most posteriorly present: Select and explain. A. Foramen spinosum, B. Foramen rotundum, C. Foramen ovale, D. All at same level",A: Foramen spinosum,"Answer: A: Foramen spinosum Explanation: None" "This is a real-world medical entrance exam question, please give the true answer based on the question and selection. Topic name: None, Subject name: Surgery","Most common pathognomonic sign of mandibular fracture: Select and explain. A. Sublingual hematoma., B. Malocclusion, C. Tenderness, D. Buccal hematoma.",A: Sublingual hematoma.,"Answer: A: Sublingual hematoma. Explanation: Sublingual hematoma is the best pathognomonic sign." "This is a real-world medical entrance exam question, please give the true answer based on the question and selection. Topic name: None, Subject name: Surgery","Open reduction of condylar fracture is strongly indicated in (Or) Absolute indication for open reduction in condylar fractures Select and explain. A. Condyle fracture along with body fracture, B. Lateral displacement of the condyle (or) Lateral fracture dislocation condyle, C. 200 angulations, D. Condyle is separated from the stump",B: Lateral displacement of the condyle (or) Lateral fracture dislocation condyle,"Answer: B: Lateral displacement of the condyle (or) Lateral fracture dislocation condyle Explanation: None" "This is a real-world medical entrance exam question, please give the true answer based on the question and selection. Topic name: AIIMS 2018, Subject name: Pediatrics","BCG is maximally protective against:- Select and explain. A. Pulmonary TB, B. Pulmonary and CNS TB, C. CNS and Disseminated TB, D. Extra pulmonary TB",C: CNS and Disseminated TB,"Answer: C: CNS and Disseminated TB Explanation: BCG VACCINE: - Vaccine used in India- Copenhagen strain - Primarily includes cell-mediated immunity - It has low protective efficacy against primary infection & pulmonary infection & all forms of TB - But enables satisfactory protection against severe forms of TB like Miliary TB, Tubercular meningitis & reduces moality" "This is a real-world medical entrance exam question, please give the true answer based on the question and selection. Topic name: None, Subject name: Surgery","A patient underwent laparoscopic cholecystectomy and was discharged on the same day. On postoperative day 3, he presented to the hospital with fever. Ultra-sonography showed a 5 x 5 cm collection in the right sub diaphragmatic region. What will be the manage!ment? Select and explain. A. Observe with antibiotic cover, B. Re-explore the wound with T-tube inseion, C. Pigtail inseion and drainage, D. ERCP and proceed",C: Pigtail inseion and drainage,"Answer: C: Pigtail inseion and drainage Explanation: Answer- C. Pigtail inseion and drainageThis patienl is most likely having a biliary leak from the cystic duct stump, which has formed a large 5 x 5 cm collection.Such a patient needs to be managed with intravenous antibiotics as well as drainage of the collections (as it acts as source of injection). Ultrasound guided pigtail inseion is the easiest method to obtain adequate drainage of the bile leak." "This is a real-world medical entrance exam question, please give the true answer based on the question and selection. Topic name: None, Subject name: Medicine","About Transfusion Related Acute Lung Injury (TRALI), all of the following are true except: Select and explain. A. Signs and symptoms usually subsides within 2-3 weeks of onset, B. Suppoive care is the mainstay of treatment, C. Steroids have a doubtful role in management, D. Moality is less than 10%",A: Signs and symptoms usually subsides within 2-3 weeks of onset,"Answer: A: Signs and symptoms usually subsides within 2-3 weeks of onset Explanation: Ans: A. Signs and symptoms usually subsides within 2-3 weeks of onset(Ref Harrison 19/e p138e-5, 18/e pg 1217 Wintrobe's 14/e p 575)TRALI - Treatment & recovery:Recovery within 2-3 days (instead of 2-3 weeks).No definitive treatment.Suppoive care:Patient's oxygenation improves & return to normal in 2-3 days." "This is a real-world medical entrance exam question, please give the true answer based on the question and selection. Topic name: None, Subject name: Gynaecology & Obstetrics","A pregnant lady is diagnosed to be HBs Ag positive. Which of the following is the best way to prevent infection to the child: Select and explain. A. Hepatitis vaccine to the child, B. Full course of Hepatitis B vaccine and immunoglobulin to the child, C. Hepatitis B immunoglobulin to the mother, D. Hepatitis B immunization to mother",B: Full course of Hepatitis B vaccine and immunoglobulin to the child,"Answer: B: Full course of Hepatitis B vaccine and immunoglobulin to the child Explanation: The best way to prevent infection in a child born to HBsAg positive mother is to give both active and passive immunization. Infants born to HBsAg positive mothers should be given hepatitis immunoglobulin (0.5 m 1/m) within 12 hours after birth. Along with this the first dose of hepatitis B recombinant vaccine is given. This is followed by hepatitis B vaccine at 1 and 6 months. Hepatitis B is not a contraindication for breastfeeding." "This is a real-world medical entrance exam question, please give the true answer based on the question and selection. Topic name: None, Subject name: Gynaecology & Obstetrics","Which of the following statements concerning hepatitis infection in pregnancy is true?: Select and explain. A. Hepatitis B core antigen status is the most sensitive indicator of positive vertical transmission of disease, B. Hepatitis B is the most common form of hepatitis after blood transfusion, C. The proper treatment of infants born to infected mothers includes the administration of hepatitis B immune globulin as well as vaccine, D. Patients who develop chronic active hepatitis should undergo MTP",C: The proper treatment of infants born to infected mothers includes the administration of hepatitis B immune globulin as well as vaccine,"Answer: C: The proper treatment of infants born to infected mothers includes the administration of hepatitis B immune globulin as well as vaccine Explanation: Persons at increased risk of hepatitis B infection include homosexuals, abusers of intravenous drugs, healthcare personnel, and people who have received blood or blood products. However, because of intensive screening of blood for type B hepatitis, hepatitis C has become the major form of hepatitis after blood transfusion. (i.e. option b. incorrect). The most sensitive indicator of positive vertical transmission of disease is HBe antigen. (i.e. option a. incorrect). The proper treatment of infants born to infected mothers include administration of hepatitis B immune globulin as well as vaccine. Chronic acute hepatitis does not necessarily warrant therapeutic abortion (i.e. option d. incorrect). Fertility is decreased, but pregnancy may proceed on a normal course as long as steroid therapy is continued. Prematurity and fetal loss are increased, but there is no increase in malformations." "This is a real-world medical entrance exam question, please give the true answer based on the question and selection. Topic name: None, Subject name: Medicine","Endocarditis is most commonly seen in: Select and explain. A. Aortic stenosis., B. Mitral regurgitation., C. Patent ductus arteriosus., D. Venous bypass graft.",A: Aortic stenosis.,"Answer: A: Aortic stenosis. Explanation: None" "This is a real-world medical entrance exam question, please give the true answer based on the question and selection. Topic name: None, Subject name: Medicine","A patient with multiple loose teeth requires extraction and has mitral stenosis with mild cardiac insufficiency. He is on enalapril, digoxin, and furosemide. The antibiotic of choice to prevent bacterial endocarditis is: Select and explain. A. Amoxicillin, B. Doxycycline, C. Cotrimoxazole, D. Gentamicin",A: Amoxicillin,"Answer: A: Amoxicillin Explanation: None" "This is a real-world medical entrance exam question, please give the true answer based on the question and selection. Topic name: None, Subject name: Ophthalmology","A patient with a history of diabetes for one year with no other complications should have an ophthalmic examination? Select and explain. A. As early as feasible, B. After 5 years, C. After 10 years, D. Only after visual symptoms level",A: As early as feasible,"Answer: A: As early as feasible Explanation: Answer- A i.e. As early as feasibleAll diabetic (IDDM & NIDDM both) aged over 12 years and/or entering pubey should be screened (visual activity measurement and fundus examination by ophthalmoscopy)For retinopathy. and those with risk for visual loss referred to an ophthalmologist.Type I DM (IDDM) require ophthalmoscopic examination within 3 years of diagnosis and annual review. (If lt is diagnosed before the age of pubey).Type II DM (NIDDM) require ophthalmoscopic examination at the time of diagnosis (because it is usually diagnosed after the age of 12 years) and annual review." "This is a real-world medical entrance exam question, please give the true answer based on the question and selection. Topic name: None, Subject name: Medicine","An I.V. drug abuser presents with fever for 10 days. CXR shows B/L lower lobe consolidation with necrosis and right sided pyopneumothorax. Probable diagnosis is: Select and explain. A. MV endocarditis due to viridans Streptococci, B. TV endocarditis due to Staph. aureus, C. Tuberculosis, D. Pneumocystis jirovecii infection",B: TV endocarditis due to Staph. aureus,"Answer: B: TV endocarditis due to Staph. aureus Explanation: Infective endocarditis typically occurs at sites of pre-existing endocardial damage, but infection with particularly virulent or aggressive organisms such as Staphylococcus aureus can cause endocarditis in a previously normal heart. Staphylococcal endocarditis of the tricuspid valve is a common complication of intravenous drug use. Many acquired and congenital cardiac lesions are vulnerable, particularly areas of endocardial damage caused by a high-pressure jet of blood, such as ventricular septal defect, mitral regurgitation and aortic regurgitation, many of which are haemodynamically insignificant. In contrast, the risk of endocarditis at the site of haemodynamically important low-pressure lesions, such as a large atrial septal defect, is minimal. Reference: : Davidson 23rd ed page no 527" "This is a real-world medical entrance exam question, please give the true answer based on the question and selection. Topic name: None, Subject name: Pediatrics","A breast fed child presents with hypernatremia (Serum sodium > 170m Eq/L). His urine sodium is 70 mEq/L. Which of the following is the most likely cause – Select and explain. A. Diabetes insipidus, B. Acute necrosis, C. Severe dehydration, D. Excessive intake of sodium",D: Excessive intake of sodium,"Answer: D: Excessive intake of sodium Explanation: The child is having hypernatremia. Serum sodium >170 rnEq/L (Normal level is 135-145 mEq/L). Urine sodium is also very high, >70mEq/L (Normal urine sodium level in <20 mEq/L) This combination can be seen with excessive intake of sodium. With excessive intake of sodium there will be increase in serum sodium and excessive excretion of sodium in urine (Kidney tries to compensate for increase in serum sodium by excreting large amount of sodium). Diabetes Insipidus      ->    Serum sodium conc. will be high but urine sodium conc. will be very low (Lack of ADH leads to defect in concentration of urine). Acute necrosis            ->    Urine sodium conc. will be high but serum sodium conc. will be low.  Severe dehydration    ->   Urine sodium concentration will be low." "This is a real-world medical entrance exam question, please give the true answer based on the question and selection. Topic name: None, Subject name: Dental","Gingival massage increases blood supply in: Select and explain. A. Epidermis, B. Basal layer, C. Lamina propria, D. All of the above",C: Lamina propria,"Answer: C: Lamina propria Explanation: None" "This is a real-world medical entrance exam question, please give the true answer based on the question and selection. Topic name: None, Subject name: Dental","Initial calcification of crown represents Which Nolla's stage: Select and explain. A. Stage 1, B. Stage 2, C. Stage 3, D. Stage 4",B: Stage 2,"Answer: B: Stage 2 Explanation: Stages 0 : Absence of crypt 1 : Presence of crypt 2 : Initial calcification of crown 3 : 1/3rd calcification of crown – completed 4 : 2/3rd of crown completed 5 : Crown almost completed 6 : Complete calcification of crown. Tooth shows eruptive movement 7 : 1/3rd of root completed 8 : 2/3rd of Root completed 9 : Root almost completed with open apex 10 : Apical end of Root completed" "This is a real-world medical entrance exam question, please give the true answer based on the question and selection. Topic name: None, Subject name: Dental","Plastic carrier size of 40-90 in carrier base Gutta percha are made from? Select and explain. A. Poly propylene, B. Poly Sulphone, C. Poly Sulphide, D. Liquid crystal plastic",B: Poly Sulphone,"Answer: B: Poly Sulphone Explanation: None" "This is a real-world medical entrance exam question, please give the true answer based on the question and selection. Topic name: None, Subject name: Dental","All of the following are true for light cure composite except: Select and explain. A. Held at distance of 1 mm from tooth, B. Use of orange glass shield for eye protection, C. Polymerization reaction continues for a period of 72 hours, D. Adding increments of 1-2 mm",C: Polymerization reaction continues for a period of 72 hours,"Answer: C: Polymerization reaction continues for a period of 72 hours Explanation: In case of light cure composites, application of light source to the composite material is held at a distance of 1 mm from the tooth surface and the 1-2 mm of increments are added, bulk addition of composite is contraindicated.  Usually, 70% of polymerization takes place during the first 10 minutes, though the polymerization reaction continues for a period of 24 hours.  The operator is asked to wear an orange glass shield for the protection of the eyes. Ref: Textbook of Operative Dentistry, Nisha and Amit Garg, 3rd edition, Pg no:264" "This is a real-world medical entrance exam question, please give the true answer based on the question and selection. Topic name: None, Subject name: Surgery","The joint which histologically & morphologically best simulate the TMJ is Select and explain. A. 5th costochondral graft, B. 3rd metatarsal graft, C. Sternoclavicular graft, D. None of the above",C: Sternoclavicular graft,"Answer: C: Sternoclavicular graft Explanation: None" "This is a real-world medical entrance exam question, please give the true answer based on the question and selection. Topic name: None, Subject name: Surgery","A patient sustained A and endotracheal intubation was done. Most likely GCS score of such a patient would be:March 2013 (b, c, d) Select and explain. A. 8, B. 10, C. 12, D. 15",A: 8,"Answer: A: 8 Explanation: Ans. A i.e. 8Patients with severe head injury, an altered level of consciousness, or a Glasgow Coma Scale (GCS) score of 8 or less usually require placement of a definitive airway.Orotracheal or nasotracheal intubation can be attempted with cervical spine precautions if a second person maintains axial immobilization of the head to prevent destabilization of the spine.In this patient, GCS score was 8, so endotracheal intubation was done to secure airway.You don't have to calculate the GCS score of this patient after intubation." "This is a real-world medical entrance exam question, please give the true answer based on the question and selection. Topic name: None, Subject name: Surgery","Rapid onset of action seen by local anesthesia in small nerve endings is due to Select and explain. A. Increased threshold of small nerves due to depolarization, B. Low pH of small nerve fibres, C. High ratio of surface area to the volume of small nerve fibres, D. Increased resting potential of small nerve fibres",C: High ratio of surface area to the volume of small nerve fibres,"Answer: C: High ratio of surface area to the volume of small nerve fibres Explanation: None" "This is a real-world medical entrance exam question, please give the true answer based on the question and selection. Topic name: None, Subject name: Pediatrics","Which of the following intrauterine infections is associated with limb reduction defects and scarring of skin – Select and explain. A. Varicella virus, B. Herpes virus, C. Rubella, D. Parvovirus",A: Varicella virus,"Answer: A: Varicella virus Explanation: Congenital varicella syndrome is characteristically associated with scarring of skin and limb reduction defect (limb hypoplasia). Perinatal chicken Pox (Perinatal varicella infection) The baby may develop two types of complications depending on the period of gestation." "This is a real-world medical entrance exam question, please give the true answer based on the question and selection. Topic name: None, Subject name: Dental","Acromegaly is associated with Select and explain. A. Class. I malocclusion, B. Class. I cross bite, C. Class. II malocclusion, D. Class. III malocclusion",D: Class. III malocclusion,"Answer: D: Class. III malocclusion Explanation: None" "This is a real-world medical entrance exam question, please give the true answer based on the question and selection. Topic name: None, Subject name: Dental","The appropriate force required to tip a tooth is: Select and explain. A. 10 - 20mg, B. 35 - 60mg, C. 50 - 70mg, D. 70 - 100mg",B: 35 - 60mg,"Answer: B: 35 - 60mg Explanation: None" "This is a real-world medical entrance exam question, please give the true answer based on the question and selection. Topic name: None, Subject name: Physiology","Function of Merkel's cells is: Select and explain. A. Tactile sensation, B. Melanophage, C. Chemoreceptor, D. Proprioception",A: Tactile sensation,"Answer: A: Tactile sensation Explanation: None" "This is a real-world medical entrance exam question, please give the true answer based on the question and selection. Topic name: None, Subject name: Dental","Which of the following is function of fluoride flux in soldering process? Select and explain. A. It prevents the formation of copper oxide, B. It prevents the formation of chromium oxide or dissolves chromic oxide layer, C. It prevents the flow of material in undesirable area, D. Increases the strength of solder",B: It prevents the formation of chromium oxide or dissolves chromic oxide layer,"Answer: B: It prevents the formation of chromium oxide or dissolves chromic oxide layer Explanation: None" "This is a real-world medical entrance exam question, please give the true answer based on the question and selection. Topic name: None, Subject name: Medicine","Hyperparathyroidism is associated with: Select and explain. A. Renal stones, B. Bone resorption, C. Increased level of serum calcium, D. All of the above",D: All of the above,"Answer: D: All of the above Explanation: None" "This is a real-world medical entrance exam question, please give the true answer based on the question and selection. Topic name: None, Subject name: Surgery","Surgery on the hard palate of a 3 years old cleft patient may inhibit growth, causing the facial profile to become: Select and explain. A. Straight, B. Elongated, C. Shortened, D. More concave",D: More concave,"Answer: D: More concave Explanation: None" "This is a real-world medical entrance exam question, please give the true answer based on the question and selection. Topic name: None, Subject name: Gynaecology & Obstetrics","A 16-year old girl was brought with primary amenorrhea. Her mother mentioned that she staed developing breast at the age of 12. She was prescribed OCPs 2 years back by a doctor with no effect. She was having normal stature and was a football player. On examination, breasts were well developed (Tanner's stage 5) and pubic hair was minimal (Tanner's stage 1). What is the most probable diagnosis? Select and explain. A. Premature ovarian failure, B. Turner's syndrome, C. Miillerian agenesis, D. Androgen insensitivity",D: Androgen insensitivity,"Answer: D: Androgen insensitivity Explanation: Ans: D. Androgen insensitivity(Ref: Shawl 16/e p141, 15/e p111-112; Novaks 14/1037-1038; Dutta Gvnae 6/e p424)Most likely diagnosis here is androgen insensitivity syndrome.Androgen Insensitivity Syndrome:Also referred as ""Testicular feminization"".An X-linked recessive condition.Results in failure of normal masculinization of external genitalia in chromosomally male individuals.Testes produce normal amounts of mullerian-inhibiting factor (MIF), also known as mullerian-inhibiting substance (MIS) or anti-mullerian hormone/factor (AMH/AMF),Features:Absence of fallopian tubes, a uterus, or proximal (upper) vagina.Identified in newborn period - By presence of inguinal masses & later identified as testes during surgery.Sometimes diagnosed in teenage years during primary amenorrhea evaluation.Absence of pubic & axillary hair in adolescent patients.Scanty body hair & lack of acne.Yet normal breast - Due to testosterone to estradiol conversion." "This is a real-world medical entrance exam question, please give the true answer based on the question and selection. Topic name: AIIMS 2017, Subject name: Pediatrics","Absence of which of the given milestones in a 3 year old child should be called delayed development? Select and explain. A. Hopping on one leg, B. Drawing square, C. Feeding by spoon, D. Catching a ball reliably",C: Feeding by spoon,"Answer: C: Feeding by spoon Explanation: 18 Months Walks up/down stairs with help Throws a ball overhand Jumps in place Builds 3-4 blocks tower Turns 2-3 book pages Scribbles Uses cup and spoon 10+ word vocabulary Identifies common objects Has temper tantrums Understands ownership (""mine"") Imitates others 3-4 years old Movements milestones Hops and stands on one foot up to five seconds Goes upstairs and downstairs without suppo Kicks ball forward Throws b all overhand Catches bounced ball most of the time Moves forward and backward with agility Milestones in hand and finger skills Copies square shapes Draws a person with two to four body pas Uses scissors Draws circles and squares Begins to copy some capital letters" "This is a real-world medical entrance exam question, please give the true answer based on the question and selection. Topic name: None, Subject name: Pathology","Which of the following tooth structure during formation is most effected due to Vit -A deficiency Select and explain. A. Enamel, B. Dentin, C. Cementom, D. Periodontal Ligament",A: Enamel,"Answer: A: Enamel Explanation: None" "This is a real-world medical entrance exam question, please give the true answer based on the question and selection. Topic name: None, Subject name: Gynaecology & Obstetrics","Indications for caesarean section in pregnancy are all except: Select and explain. A. Eisenmenger's syndrome, B. Aortic stenosis, C. MR, D. Aortic regurgitation",A: Eisenmenger's syndrome,"Answer: A: Eisenmenger's syndrome Explanation: Cardiac indications for cesarean section: Aortic aneurysm or dilated aortic root ≥ 4 cm Marfans syndrome with aortic involvement Severe symptomatic aortic stenosis Acute severe congestive heart failure Recent MI Need for emergency valve replacement immediately after delievery A patient who is fully anticoagulated with warfarin at the time of labor needs to be counseled for cesarean section because the baby is also anticoagulated and vaginal delivery carries increase risk to the fetus of intracranial hemorrhage." "This is a real-world medical entrance exam question, please give the true answer based on the question and selection. Topic name: None, Subject name: Pediatrics","Brachycephaly is due to fusion of – Select and explain. A. Parietal suture, B. Sagittal suture, C. Lambdoid suture, D. Coronal suture",D: Coronal suture,"Answer: D: Coronal suture Explanation: None" "This is a real-world medical entrance exam question, please give the true answer based on the question and selection. Topic name: None, Subject name: Dental","The forces acting through a fixed partial denture on the abutment tooth should be directed: Select and explain. A. As far as possible at right angles to the long axis of the teeth, B. Parallel to the long axis of the teeth, C. By decreasing the mesio-distal dimension, D. None of the above",B: Parallel to the long axis of the teeth,"Answer: B: Parallel to the long axis of the teeth Explanation: None" "This is a real-world medical entrance exam question, please give the true answer based on the question and selection. Topic name: AIIMS 2018, Subject name: Pharmacology","All of the following drugs require dose reduction in renal failure except? Select and explain. A. Amphotericin B, B. Vancomycin, C. Gentamicin, D. Doxycycline",D: Doxycycline,"Answer: D: Doxycycline Explanation: Those drugs which are secreted in bile do not require dose reduction in renal disease. Doxycycline is secreted in bile, so it is safe in renal failure Drugs (commonly asked) safe in renal diseases include: Safe (Cef): Cefoperazone, Ceftriaxone In The: Tigecycline R: Rifampicin E: Erythromycin N: Nafcillin A: Ampicillin L: Lincosamides (Clindamycin) Disease: Doxycycline" "This is a real-world medical entrance exam question, please give the true answer based on the question and selection. Topic name: None, Subject name: Pharmacology","Which of the following is not used in osteoporosis? Select and explain. A. Milnacipran, B. PTH, C. Strontium ranelate, D. Denosumab",A: Milnacipran,"Answer: A: Milnacipran Explanation: Ans: A. Milnacipran(Ref. Goodman Gilman 12/e p1299; Katzung 13/e p761-762, 12/e p775; Harrison 19/e p2493, 18/c' p3120; Apley 9/e p /3 I-133)Milnacipran:Serotonin-norepinephrine reuptake inhibitor (SNRI).Approved for treatment of pain in fibromyalgia, not in osteoporosis.Drugs useful in OsteoporosisInhibit Bone ResorptionStimulates Bone Both actionBisphosphonates: Alendronate, risedronate & etidronateCalcium receptor agonist: CinacalcetCalcitoninSERMs: Tamoxifen, raloxifeneGallium nitrateRANKL inhibitors: DonesumabTeriparatideCalciumCalcitriolFluorideStrontium ranelate" "This is a real-world medical entrance exam question, please give the true answer based on the question and selection. Topic name: None, Subject name: Radiology","Consistent radiographs require: Select and explain. A. Long cone technique, B. Fixed kvp & mA, C. Automatic processing, D. Proper developing",B: Fixed kvp & mA,"Answer: B: Fixed kvp & mA Explanation: None" "This is a real-world medical entrance exam question, please give the true answer based on the question and selection. Topic name: None, Subject name: Gynaecology & Obstetrics","All of the following are true about augmentation of labor except: Select and explain. A. Twin pregnancy precludes the use of oxytocin, B. Amniotomy decreases the need for oxytocin use, C. Methods of augmentation does not increase the risk of operational management, D. Associated with a risk of uterine hyper stimulation",A: Twin pregnancy precludes the use of oxytocin,"Answer: A: Twin pregnancy precludes the use of oxytocin Explanation: Answer- A. Twin pregnancy precludes the use of oxytocin'Augmentation of labour is the process of stimulating the uterus to increase the frequency, duration and intensity of contractions after the onset of spontaneous labour. It has commonly been used to treat delayed labour when poor uterine contractions are assessed to be the underlying cause. The traditional methods of labour augmentation have been with the use of intravenous ocytocin infusion and aificial rupture of the membranes (amniotomy)." "This is a real-world medical entrance exam question, please give the true answer based on the question and selection. Topic name: None, Subject name: Anatomy","Nucleus in brain common to IX, X and XI cranial nerves: Select and explain. A. Nucleus solitarius, B. Nucleus ambiguus, C. Dentate nucleus, D. Red nucleus",B: Nucleus ambiguus,"Answer: B: Nucleus ambiguus Explanation: The nucleus ambiguus is a group of large motor neurons, situated deep in the medullary reticular formation. The nucleus ambiguus contains the cell bodies of nerves that innervate the muscles of the soft palate, pharynx, and larynx which are strongly associated with speech and swallowing. Nucleus in brain common to IX, X and XI cranial nerves is  nucleus ambiguous." "This is a real-world medical entrance exam question, please give the true answer based on the question and selection. Topic name: None, Subject name: Medicine","Acromegaly occurs: Select and explain. A. Before closure of epiphyses of long bones, B. After closure of epiphyses of long bones, C. Not related to closure of epiphyses of long bones, D. Depending on functioning on posterior Pituitary gland",B: After closure of epiphyses of long bones,"Answer: B: After closure of epiphyses of long bones Explanation: None" "This is a real-world medical entrance exam question, please give the true answer based on the question and selection. Topic name: None, Subject name: Pathology","Fordyce's spots are: Select and explain. A. Fat tissue embedded in buccal mucosa, B. Red spots, C. Present on the cheek mucosa lateral to angle of the mouth, D. All of the above",C: Present on the cheek mucosa lateral to angle of the mouth,"Answer: C: Present on the cheek mucosa lateral to angle of the mouth Explanation: None" "This is a real-world medical entrance exam question, please give the true answer based on the question and selection. Topic name: None, Subject name: Dental","The tipping of a tooth results in the fibers of the PDL to be: Select and explain. A. All compressed, B. All stretched, C. Half compressed half stretched, D. No effect",C: Half compressed half stretched,"Answer: C: Half compressed half stretched Explanation: None" "This is a real-world medical entrance exam question, please give the true answer based on the question and selection. Topic name: None, Subject name: Microbiology","A 6-month child presented with diarrhea and vomiting for three days. Which of the following enterotoxin is most likely responsible for the condition? Select and explain. A. NSP4, B. NSP6, C. VP3, D. VP7",A: NSP4,"Answer: A: NSP4 Explanation: Ans. A. NSP4Rotavirus nonstractural protein 4 (NSP4) is known to function as an intracellalar receptor at the endoplssmic reticalam (ER) critical to virul morphogenesis and is the first characterized virul enterotoxin.The associntion of NSP4 snd cuveolin-I contributes to NSP4 intrqcellular trfficking from the ER to the cell surfuce and speculate that exogenously added NSP4 stimulutes sigaaling molecules located in caveola microdomains." "This is a real-world medical entrance exam question, please give the true answer based on the question and selection. Topic name: None, Subject name: Surgery","Most common site of CSF Rhinorrhoea is Select and explain. A. Ethmoidal sinus, B. Frontal sinus, C. Petrous bone, D. Cribriform plate",D: Cribriform plate,"Answer: D: Cribriform plate Explanation: None" "This is a real-world medical entrance exam question, please give the true answer based on the question and selection. Topic name: None, Subject name: Anatomy","Lymph from tongue not drained by following vessels Select and explain. A. Central, B. Ventral, C. Posterior, D. Marginal",B: Ventral,"Answer: B: Ventral Explanation: None" "This is a real-world medical entrance exam question, please give the true answer based on the question and selection. Topic name: None, Subject name: Biochemistry","Vitamin K: Select and explain. A. Helps in formation of prothrombin, B. Inhibition of antithrombin, C. Prevention of capillary fragility, D. Stimulation of hematopoiesis in red bone marrow",A: Helps in formation of prothrombin,"Answer: A: Helps in formation of prothrombin Explanation: None" "This is a real-world medical entrance exam question, please give the true answer based on the question and selection. Topic name: None, Subject name: Physiology","The primary site of action of antidiuretic hormone is on the: Select and explain. A. Distal tubules and collecting ducts in the kidney, B. Afferent arterioles of the glomeruli, C. Thirst center in the hypothalamus, D. Osmoreceptors in the hypothalamus",A: Distal tubules and collecting ducts in the kidney,"Answer: A: Distal tubules and collecting ducts in the kidney Explanation: None" "This is a real-world medical entrance exam question, please give the true answer based on the question and selection. Topic name: None, Subject name: Anatomy","The part of the mandible which ossify in cartilage is: Select and explain. A. Coronoid process, B. Condylar process, C. Upper half of ramus above the level of mandibular foramen, D. All of the above",D: All of the above,"Answer: D: All of the above Explanation: The mandible is the second bone, next to the clavicle, to ossify in the body. Its greater part ossifies in membrane. The parts ossifying in cartilage include the incisive part below the incisor teeth, the coronoid and condyloid processes, and the upper half of the ramus above the level of the mandibular foramen." "This is a real-world medical entrance exam question, please give the true answer based on the question and selection. Topic name: AIIMS 2018, Subject name: Biochemistry","In a cerebrohepatorenal syndrome, which of the following accumulate in brain? Select and explain. A. Pyruvate, B. Sho-chain fatty acid, C. Very long-chain fatty acid, D. Acetyl CoA",C: Very long-chain fatty acid,"Answer: C: Very long-chain fatty acid Explanation: Zellweger syndrome Cerebro-Hepato-Renal Syndrome. Impaired neuronal migration, hypomyelination, hepatomegaly, renal cysts Autosomal recessive Rare Absence of Peroxisomes in almost all tissues, peroxisomes are responsible of oxidation of very long chain fatty acid which contain more than 22 carbon Accumulation of polyenic acids in brain, with carbon > 22 Severe neurological symptoms Most patients die within 1st year of life." "This is a real-world medical entrance exam question, please give the true answer based on the question and selection. Topic name: None, Subject name: Radiology","Patient's cross-infection with sensor, in digital radiography/RVG is prevented by Select and explain. A. Cover with autoclaved cloth with each use, B. Wipe with ethyl alcohol in each patient, C. Clean with 5.25% sodium hypochlorite in each patient, D. Cover with impervious barrier",D: Cover with impervious barrier,"Answer: D: Cover with impervious barrier Explanation: None" "This is a real-world medical entrance exam question, please give the true answer based on the question and selection. Topic name: None, Subject name: Dental","Term ""Clean Tooth Never Decays"" given by Select and explain. A. Perrie Fauchard, B. Leon J Williams, C. Dean, D. MJ Rihanna",B: Leon J Williams,"Answer: B: Leon J Williams Explanation: None" "This is a real-world medical entrance exam question, please give the true answer based on the question and selection. Topic name: None, Subject name: Biochemistry","intermediate of kreb cycle used in heme synthesis; Select and explain. A. succinyl-CoA, B. Alpha ketoglutarate, C. Citrate, D. Aspaate",A: succinyl-CoA,"Answer: A: succinyl-CoA Explanation: Answer (A) succinyl-CoAAll the carbon and nitrogen atoms of the porphyrin molecule are provided by glycine(a nonessential amino acid) and succinyl coenzyme A (an intermediate in the citric acid cycle) that condense to form ALA in a reaction catalyzed by ALA synthase(ALAS)Heme synthesis also requires a functional tricarboxylic acid cycle and an oxygen supply.Heme synthesis stas in mitochondria with the condensation of succinyl-CoA with the amino acid glycine, activated by pyridoxal phosphate.ALA synthase is the rate-limiting enzyme of heme synthesis. ALA molecules enter the cytoplasm, where their union in the presence of ALA dehydratase yields porphobilinogen(PBG) and water molecules." "This is a real-world medical entrance exam question, please give the true answer based on the question and selection. Topic name: AIIMS 2018, Subject name: Biochemistry","Patient with Type I diabetes mellitus, with complains of polyuria. Which of the following will occur normally in his body? Select and explain. A. Glycogenesis in muscle, B. Increased protein synthesis, C. Increased conversion of fatty acid to acetyl CoA, D. Decreased cholesterol synthesis",C: Increased conversion of fatty acid to acetyl CoA,"Answer: C: Increased conversion of fatty acid to acetyl CoA Explanation: The diabetic situation is same as fasting i.e. breakdown or catabolism will be increased & anabolism or synthesis is decreased. So in Diabetes, fats (TG) are broken down in adipose tissue to give fatty acids. These fatty acids go in blood & then to the liver. In liver, these fatty acids are broken down by beta-oxidation of fatty acids to give Acetyl CoA. This excess acetyl CoA obtained from fatty acids breakdown is used for the formation of fats e.g. fatty acids, endogenous TGs, VLDL & cholesterol. This is the only anabolic thing occurring in the liver of diabetic patients. Options A & B - wrong as the body is mainly in catabolism, not anabolism. Option C is correct i.e. increased conversion of fatty acid to Acetyl CoA. Option D - wrong- There is increased cholesterol synthesis." "This is a real-world medical entrance exam question, please give the true answer based on the question and selection. Topic name: None, Subject name: Dental","Reactive oxygen metabolite in lysosomes is released by Select and explain. A. Peroxidase, B. NADPH oxidase, C. Superoxide dismutase, D. Catalase",B: NADPH oxidase,"Answer: B: NADPH oxidase Explanation: None" "This is a real-world medical entrance exam question, please give the true answer based on the question and selection. Topic name: None, Subject name: Dental","Square root of pq/n indicates: Select and explain. A. Standard error of mean, B. Standard error of difference of means, C. Standard error of proportions, D. Standard error of difference in proportions",C: Standard error of proportions,"Answer: C: Standard error of proportions Explanation: None" "This is a real-world medical entrance exam question, please give the true answer based on the question and selection. Topic name: None, Subject name: Dental","Which of the following should be treated by using a tissue conditioner material? Select and explain. A. Tissue symptoms related to diabetes mellitus, B. Generalized ridge hyperplasia, C. Traumatized mucosa caused by ill fitting dentures, D. Sharp residual ridge in molar area",C: Traumatized mucosa caused by ill fitting dentures,"Answer: C: Traumatized mucosa caused by ill fitting dentures Explanation: None" "This is a real-world medical entrance exam question, please give the true answer based on the question and selection. Topic name: None, Subject name: Ophthalmology","Which of the following is not an indication for evisceration? Select and explain. A. Malignancy, B. Panophthalmitis, C. Severe globe trauma, D. Expulsive hemorrhage",A: Malignancy,"Answer: A: Malignancy Explanation: Ans: A. Malignancy(Ref Yanoff and Duker 4/e p1339)Evisceration of eye is not done in malignancy.Evisceration:Surgical technique removing entire intraocular eye contents.Simpler procedure than enucleation surgery.Advantages:Leaves scleral shell & extraocular muscle attachments intact.Offers better orbital anatomy preservation & natural motility of ophthalmic socket tissues.Contraindications:Documented or suspected intraocular malignant tumors." "This is a real-world medical entrance exam question, please give the true answer based on the question and selection. Topic name: None, Subject name: Gynaecology & Obstetrics","The one measurement of fetal maturity that is not affected by a 'bloody tap' during amniocentesis is: Select and explain. A. L/S ratio, B. Phosphatidyl glycerol, C. α-fetoprotein, D. Bilirubin as a measured by DOD 450",B: Phosphatidyl glycerol,"Answer: B: Phosphatidyl glycerol Explanation: The best test to detect fetal lung maturity in diabetic mothers is presence of phophatidyl glycerol (PG) in amniotic fluid. If PG is present in amniotic fluid fetal lungs are considered mature and vice versa." "This is a real-world medical entrance exam question, please give the true answer based on the question and selection. Topic name: None, Subject name: Dental","Sensitivity in gingival area of class II light cured composite restoration after 2 months is due to: Select and explain. A. Improper cavity preparation at gingival margin, B. Curing from occlusal area only, C. Addition of resin in multiple increments, D. Curing from buccal and lingual aspect",A: Improper cavity preparation at gingival margin,"Answer: A: Improper cavity preparation at gingival margin Explanation: None" "This is a real-world medical entrance exam question, please give the true answer based on the question and selection. Topic name: None, Subject name: Dental","Which dentifrices should be used for periodontal patients? Select and explain. A. Tooth powders with high abrasive content, B. Tooth pastes with high abrasive content, C. Tooth pastes with minimum abrasive content, D. All of the above",C: Tooth pastes with minimum abrasive content,"Answer: C: Tooth pastes with minimum abrasive content Explanation: None" "This is a real-world medical entrance exam question, please give the true answer based on the question and selection. Topic name: None, Subject name: Surgery","A young patient is diagnosed to have irreparable tear of the rotator cuff. Treatment of choice will be: Select and explain. A. Tendon transfer, B. Total shoulder replacement, C. Reverse c shoulder replacement, D. Acromioplasty",A: Tendon transfer,"Answer: A: Tendon transfer Explanation: Ans: A. Tendon transfer(Ref Apley 9/e p347,348)Treatment of choice for irreparable tear of rotator cuff in young patient = Tendon transfer.Subacromial decompression and debridement of degenerate cuff tissue:Useful for irreparable massive full thickness tears.Pain relief allows reasonable shoulder abduction by remaining muscles.Other methods:Supraspinatus advancement.Latissimus dorsi transfer.Rotator cuff transposition.Fascia Iota autogruit.Synthetic tendon graft." "This is a real-world medical entrance exam question, please give the true answer based on the question and selection. Topic name: None, Subject name: Medicine","A 22 year old female in emergency presents with sore throat from 3 days, headache and vomiting, blood pressure 90/50, tiny red spots distal to sphygomomanometer cuff Select and explain. A. Brucella species, B. Neisseria Meningitidis, C. P.falciparum, D. Salmonella species",B: Neisseria Meningitidis,"Answer: B: Neisseria Meningitidis Explanation: None" "This is a real-world medical entrance exam question, please give the true answer based on the question and selection. Topic name: AIIMS 2019, Subject name: Microbiology","A young male complains of burning sensation upon urination and a purulent urethral discharge. Gram stain of the specimen shows pus cells with Gram negative diplococci. What is not true about the organism? Select and explain. A. Pili is a virulence factor, B. Intracellular obligate, C. Show twitching motility, D. It is both catalase and oxidase positive",B: Intracellular obligate,"Answer: B: Intracellular obligate Explanation: The picture depicts Neisseria gonorrhea which have pili as a virulence factor and are both catalase and oxidase positive and show slow twitching (false kind of motility) But are not strict intracellular aerobes.it could be extracellular." "This is a real-world medical entrance exam question, please give the true answer based on the question and selection. Topic name: None, Subject name: Pathology","Hypogonadism, developmental delay, loss of taste and smell is due to deficiency of: Select and explain. A. Cu, B. Zn, C. K, D. Cr",B: Zn,"Answer: B: Zn Explanation: None" "This is a real-world medical entrance exam question, please give the true answer based on the question and selection. Topic name: None, Subject name: Pediatrics","Characteristics radiological feature of transient tachypnoea of newborn is – Select and explain. A. Reticulogranular appearance, B. Low volume lungs, C. Prominent horizontal fissure, D. Air bronchogram",C: Prominent horizontal fissure,"Answer: C: Prominent horizontal fissure Explanation: The lungs are generally clear without rales or rhonchi and chest roentgenogram shows Prominent pulmonary vascular markings            ❑ Overaeration Fluid lines in the fissure                                          ❑ Flat diaphragms Prominent inter lobar fissure                                   ❑ Occasionally pleural fluid." "This is a real-world medical entrance exam question, please give the true answer based on the question and selection. Topic name: AIIMS 2019, Subject name: Gynaecology & Obstetrics","16 year girl, Not sexually active, came for vaccination against cervical cancer. Which vaccine to be given Select and explain. A. Gardasil, B. Rubavac, C. Biovac, D. Tdap",A: Gardasil,"Answer: A: Gardasil Explanation: Quadrivalent Gardasil and the bivalent Cervarix against HPV subtypes 16 and 18. 97% to 100% efficacy for preventing cervical intraepithelial neoplasia (C1N) grades 2 and 3 caused by HPV 16 or 18 in females who were not previously infected However, efficacy was only 44% in those who were infected prior to vaccination. A third HPV vaccine, Gardasil-9, an additional five high-risk subtypes (HPV 31/33/45/52/58)" "This is a real-world medical entrance exam question, please give the true answer based on the question and selection. Topic name: None, Subject name: Physiology","plate endings are features of Select and explain. A. Nuclear chain fibres, B. Nuclear bag fibres, C. Golgi tendon, D. Extra fusal",B: Nuclear bag fibres,"Answer: B: Nuclear bag fibres Explanation: None" "This is a real-world medical entrance exam question, please give the true answer based on the question and selection. Topic name: None, Subject name: Social & Preventive Medicine","Tuberculin test is a cheap and easily available test. In which of the following situations there is high failure in the interpretation of the test? Select and explain. A. High percentage of immunized people, B. HIV cases are less, C. High prevalence of disease, D. Environmental mycobacterium infections are less",A: High percentage of immunized people,"Answer: A: High percentage of immunized people Explanation: Ans: A. High percentage of immunized people(Ref Park 24/e p195-196, 23/e p186)High failure of tuberculin test:High percentage of immunized people - Due to higher BCG coverage producing tuberculin hypersensitivity.Exaggerated responses:In presence of atypical mycobacteria infection & 'boosting effect' of second tuberculin dose producing larger reaction than 1st.Tuberculin test lost its sensitivity as an indicator of true prevalence of infection." "This is a real-world medical entrance exam question, please give the true answer based on the question and selection. Topic name: None, Subject name: Social & Preventive Medicine","Which of the following parameters is used to determine the sensitivity of vaccine due to heat? Select and explain. A. VVM, B. VMV, C. VCM, D. VMM",A: VVM,"Answer: A: VVM Explanation: Ans: A. VVMRef: K Park, 23'd ed, pg. 110VVM (Vaccine Vial Monitor)VVM is a label containing a heat-sensitive material which is placed on a vaccine l to register cumulative heat exposure over time.It has outer blue circle and inner white square." "This is a real-world medical entrance exam question, please give the true answer based on the question and selection. Topic name: None, Subject name: Medicine","A young man was on high protein diet. After 3 days he developed weakness. Blood investigation revealed hypoglycemia. Hypoglycemia is due to inhibition of which of the following enzymes Select and explain. A. Glucose 6 phosphatase, B. Glycogen phosphorylase, C. Phosphoglucomutase, D. PEP carboxylase",D: PEP carboxylase,"Answer: D: PEP carboxylase Explanation: Answer- D. PEP carboxylase" "This is a real-world medical entrance exam question, please give the true answer based on the question and selection. Topic name: AIIMS 2019, Subject name: Radiology","A 8 year old child presented with cough and fever since 15 days. On auscultation, decreased air entry was found on right side. A CXR was done and shown below. What is the most likely diagnosis? Select and explain. A. Consolidation, B. Abscess, C. Hydatid Cyst, D. Congenital. AV Malformation",A: Consolidation,"Answer: A: Consolidation Explanation: Rt. upper lobe Consolidation Ill-defined radio-opacity in the right upper lobe-Obscuration/Silhouetting the right hea border No tracheal detion NOTE:* Silhouetting of right hea border - Rt. Middle lobe pathology* Silhouetting of left hea border - Lingular pathology Normal CXR RML Consolidation Lung abscess: Hydatid cyst Presence of Air fluid levels Shows cystic lesion with multiple floating membranes" "This is a real-world medical entrance exam question, please give the true answer based on the question and selection. Topic name: AIIMS 2019, Subject name: Social & Preventive Medicine","Malaria control means Select and explain. A. To reduce Malaria moality to Zero, B. To prevent local transmission for 3 years, C. To reduce Malaria disease so that its no longer a Public Health Problem, D. To reduce Incidence to Zero",C: To reduce Malaria disease so that its no longer a Public Health Problem,"Answer: C: To reduce Malaria disease so that its no longer a Public Health Problem Explanation: - Disease control is determined by, 1. Reduction of Incidence 2. Reduction of duration of disease 3. Reduction of complications 4. Reduction of financial burden to the community. - Reducing incidence to zero Localized - Elimination Worldwide - Eradication - If the chain of transmission has been reasonably interrupted for minimum 3 years, ceificate of Elevation of malaria from world Health Organisation is given." "This is a real-world medical entrance exam question, please give the true answer based on the question and selection. Topic name: None, Subject name: Dental","Which of the following bacteria is responsible for progression of dental caries: Select and explain. A. Actinomyces spp., B. Proteolytic bacteria, C. Acidogenic bacteria, D. Gram positive bacteria",B: Proteolytic bacteria,"Answer: B: Proteolytic bacteria Explanation: None" "This is a real-world medical entrance exam question, please give the true answer based on the question and selection. Topic name: None, Subject name: Pediatrics","4yr boy absence of right testes, diagnostic laparoscopy done, a blind end vessel seen what to do next? Select and explain. A. Open laprotomy, B. Nothing to be done, C. Scrotal approach, D. Inguinal approach",D: Inguinal approach,"Answer: D: Inguinal approach Explanation: Ans. D. Inguinal approachCryptorchidism is the most common congenital abnormality of the genitourinary tract.Cryptorchidism means hidden testis.An absent testis may be due to agenesis or atrophy secondary to intrauterine vascular compromise also known as the ""vanishing testis syndrome"".Bilaterally absent testes is anorchia which is 10% cases.More common on Right Side.Complications of undescended testesTorsion can be seen in incomplete testicular descentSterility is seen in bilateral cases (especially intra-abdominal testes)Incomplete testicular descent predisposes to malignant disease; cancer is more common in an incompletely descended testes-orchidopexy may or may not diminish the risk.Atrophy of an inguinal testes before pubey may possibly be caused by recurrent minor trauma.In patients with a unilateral nonpalpable testis, a descended testis that is larger than expected suggests an atrophic undescended testis; confirmation requires surgical intervention typically diagnostic laparoscopy to seek an intra-abdominal testis or confirm testicular agenesis. However, scrotal or inguinal exploration is sometimes done if a testicular remnant distal to the internal inguinal ring is suspected." "This is a real-world medical entrance exam question, please give the true answer based on the question and selection. Topic name: None, Subject name: Dental","If multiple firing is done to opaque layer of dental porcelain than It become Select and explain. A. Too Smooth, B. Too glazed, C. Become more opaque, D. Crack",B: Too glazed,"Answer: B: Too glazed Explanation: None" "This is a real-world medical entrance exam question, please give the true answer based on the question and selection. Topic name: None, Subject name: Dental","Distolingual extension of the mandibular impression for a complete denture is limited by the action of: Select and explain. A. Stylohyoid muscle, B. Medial pterygoid muscle, C. Lateral pterygoid muscle, D. Superior constrictor muscle",D: Superior constrictor muscle,"Answer: D: Superior constrictor muscle Explanation: None" "This is a real-world medical entrance exam question, please give the true answer based on the question and selection. Topic name: None, Subject name: Pharmacology","Among the following propeies of dopamine, which of them is not helpful in acute shock? Select and explain. A. Alpha-1 agonist action leading to peripheral vasoconstriction, B. Increase in renal perfusion due to agonist action on DI receptors, C. Releases noradrenaline and causes positive inotropic effect, D. Direct action on hea beta-1 receptors",B: Increase in renal perfusion due to agonist action on DI receptors,"Answer: B: Increase in renal perfusion due to agonist action on DI receptors Explanation: Ans: B. Increase in renal perfusion due to agonist action on DI receptors(Ref: Goodman Gilman 13Ie p1062, 12/e p355; KDT 71e p134, 6Ie p507)Dopamine at low concentrations (2 to 5 mug/kg per minute) primarily acts on vascular D, receptors, especially in the renal, mesenteric, and coronary beds.Increased renal perfusion caused by dopamine due to DI agonism (at low doses) doesn't improves survival in acute shock." "This is a real-world medical entrance exam question, please give the true answer based on the question and selection. Topic name: None, Subject name: Pediatrics","3.5 kg term male baby, born of uncomplicated pregnancy, developed, respiratory distress at birth, not responded to surfactant, ECHO finding revealed nothing abnormal, X–ray showed ground glass appearance and culture negative. Apgars 4 and 5 at 1 and 5 minutes. History of one month female sibling died before. What is the diagnosis ? Select and explain. A. TAPVC, B. Meconium aspiration, C. Neonatal pulmonary alveolar proteinosis, D. Diffuse herpes simplex infection",C: Neonatal pulmonary alveolar proteinosis,"Answer: C: Neonatal pulmonary alveolar proteinosis Explanation: ""Respiratory distress in an infant along with a positive family history of similarly affected newborn infants strongly suggests, pulmonary alveolar proteinosis"". Pulmonary alveolar proteinosis Pulmonary alveolar proteinosis is a disorder characterized by the intraalveolar accumulation of pulmonary surfactant. Two clinically distinct forms of pulmonary alveolar proteinosis are seen - Fatal form                                -->          Presenting shortly after birth (congenital PAP) u Gradually progressive form -->           Presenting in older infants and children Pathology Although the mechanisms that lead to alveolar proteinosis are undefined, histological findings suggests that they result in a disruption of pulmonary surfactant metabolism. The main surface tension lowering agent in surfactant is phospholipids le. primary dipalmitol phosphatidyl choline. However DPCC needs surfactant protein (in endogenous and natural surfactants) for efficient dispersion which enables the formation of a phospholipid monolayer on the alveolar surface. There are two surfactant proteins present in the body i.e., protein A and protein B. In pulmonary alveolar proteinosis, there is absence of protein B. In the absence of protein B, the rapid spread and absorption of the phospholipid (DPCC) does not take place so they cannot form a phospholipid monolayer on the alveolar surface. This in turn leads to failure of expansion of alveoli leading to poor cardiorespiratory adaptation at birth. Clinical manifestation - This disorder is immediately apparent in the newborn period and rapidly leads to respiratory failure. Congenital pulmonary alveolar proteinosis is clinically and radiographically indistinguishable from more common disorders of the newborn that lead to respiratory failure including pneumonia, generalized bacterial infection, respiratory distress syndrome and total anomalous pulmonary venous return with obstruction. Diagnosis Histopathological examination of lung biopsy specimen is the gold standard for diagnosis. On histopathological examination distal air spaces are filled with a granular, eosinophillic material that stains positively with periodic - acid schiff reagent and is diastase resistant. Treatment Untreated, alveolar proteinosis in newborns is rapidly fatal and no successful medical therapy has been developed. Repeated bronchoalveolar lavage is a temporizing measure. Lung transplantation is the only therapeutic option but its use is limited by concerns about disease recurrence." "This is a real-world medical entrance exam question, please give the true answer based on the question and selection. Topic name: None, Subject name: Anatomy","Upper lip is formed by the fusion of: Select and explain. A. Maxillary and lateral nasal process., B. Lateral nasal and median nasal process., C. Maxillary, lateral nasal, median nasal process., D. Maxillary, mandibular, lateral nasal, median nasal process.","C: Maxillary, lateral nasal, median nasal process.","Answer: C: Maxillary, lateral nasal, median nasal process. Explanation: None" "This is a real-world medical entrance exam question, please give the true answer based on the question and selection. Topic name: None, Subject name: Medicine","Increased Monocytic count is seen in Typhoid and which of the following conditions? Select and explain. A. Parasitic infections, B. Sub-Acute Bacterial Endocarditis, C. Hodgkin's Lymphoma, D. None of the above",B: Sub-Acute Bacterial Endocarditis,"Answer: B: Sub-Acute Bacterial Endocarditis Explanation: None" "This is a real-world medical entrance exam question, please give the true answer based on the question and selection. Topic name: None, Subject name: Dental","If 2 Implants of size 4 mm are to be placed in a ridge, what should be the minimum width required in the ridge Select and explain. A. 14 mm, B. 15 mm, C. 17 mm, D. 18 mm",A: 14 mm,"Answer: A: 14 mm Explanation: None" "This is a real-world medical entrance exam question, please give the true answer based on the question and selection. Topic name: AIIMS 2019, Subject name: Orthopaedics","Elderly female present with pain and swelling in the thigh. Xray shows multiple lytic lesions in distal femur with no periosteal reaction. Aspirate from the swelling shows homogenous serosanguinous fluid with RBCs. Which investigation can confirm the diagnosis? Select and explain. A. S. Calcium levels, B. S. PTH levels, C. MRI, D. CD1a IHC",B: S. PTH levels,"Answer: B: S. PTH levels Explanation: History is suggestive of blood filled bone tumor seen in Hyperparathyroidism. HYPERPARATHYROIDISM May be primary (due to adenoma or hyperplasia), secondary (due to persistent hypocalcemia) or tertiary (when secondary hyperplasia leads to autonomous overactivity). Fig. X-ray hand--hyperparathyroidism Fig. Brown tumor Figs. A and B: (A) Lamina dura and (B) Resorption of lamina dura Salt pepper skull Parathyroid adenoma Most common cause of primary hyperparathyroidism. Presents with subperiosteal resorption, and replacement of endosteal cavitation marrow by vascular granulation and fibrous tissue (osteitis fibrosa cystica). Classical and pathognomic feature of hyperparathyroidism is subperiosteal coical resorption of middle phalanges on radial aspect Clinical features- Abdominal groans (dyspepsia), psychic moans, renal stones and weak bones (Groans, moans, stones and Bones). Ix- Parathyroid hormone and S. alkaline phosphate are raised. Calcium is high, serum phosphate is low." "This is a real-world medical entrance exam question, please give the true answer based on the question and selection. Topic name: None, Subject name: Pathology","Radiation affects all three germ layers (ectoderm mesoderm and endoderm) in which of the following Select and explain. A. Teratoid cyst, B. Dermoid cyst, C. Thyroglossal cyst, D. Branchial cyst",A: Teratoid cyst,"Answer: A: Teratoid cyst Explanation: None" "This is a real-world medical entrance exam question, please give the true answer based on the question and selection. Topic name: AIIMS 2019, Subject name: Anaesthesia","Post op pulmonary complications are seen/expected in all except:- Select and explain. A. BMI>30, B. Upper abdominal surgery, C. Patient with 7 pack years of smoking, D. Age >70",A: BMI>30,"Answer: A: BMI>30 Explanation: General anesthesia produces changes in respiratory system and can cause post op pulmonary complication. Risk factor: Thoracic surgery/Upper abdomen surgery Cigarette smoker Chronic respiratory disease Emergency surgery Anesthesia time duration if more than 180 minutes Age >70 years/> 65 years Renal failure patient Poor nutritional status Significant Blood loss or blood transfusion" "This is a real-world medical entrance exam question, please give the true answer based on the question and selection. Topic name: None, Subject name: Pharmacology","The antidote for heparin is: Select and explain. A. Penicillinase, B. Protamine sulphate, C. Pryosulphate, D. Potassium sulphate",B: Protamine sulphate,"Answer: B: Protamine sulphate Explanation: None" "This is a real-world medical entrance exam question, please give the true answer based on the question and selection. Topic name: None, Subject name: Gynaecology & Obstetrics","26 years old female suffers from PPH on her second postnatal day. Her APTT and PTT are prolonged while BT, PT and platelet counts are normal. Likely diagnosis is: Select and explain. A. Acquired hemophilia, B. Lupus anticoagulant, C. DIC, D. Inherited congenital hemophilia.",A: Acquired hemophilia,"Answer: A: Acquired hemophilia Explanation: None" "This is a real-world medical entrance exam question, please give the true answer based on the question and selection. Topic name: None, Subject name: Pathology","Which of the following is a tool used in gene editing? Select and explain. A. CRISPR, B. Gene Xpe, C. Big Data, D. HealthCare App",A: CRISPR,"Answer: A: CRISPR Explanation: Answer- A. CRISPRA type of genetic engineering in which DNA is inseed, deleted or replaced in the genome of a living organism using engineered nucleases, or ""molecular scissors.Four Families of Engineered Nucleases being usedMeganucleasesTranscription Activator-Like Effector-based Nucleases (TALEN)Zinc finger nucleases (ZFNs)Clustered Regularly Interspaced Sho Palindromic Repeats (CRISPR)-Cas systemForm the basis of a genome editing technology known as CRISPR/Cas9 that allows permanent modification of genes within organisms." "This is a real-world medical entrance exam question, please give the true answer based on the question and selection. Topic name: None, Subject name: Anatomy","Which of the following muscles causes the forward movement of the condyle in the glenoid fossa? Select and explain. A. Masseter, B. Temporalis, C. Medial pterygoid, D. Lateral pterygoid",D: Lateral pterygoid,"Answer: D: Lateral pterygoid Explanation: The lateral pterygoid muscle has 2 heads, one arising from the outer part of the lateral pterygoid plate and other from the base of sphenoid bone. The two heads come together and are inserted into the neck of the condyle. The main function of this muscle is to draw both the head of the condyle and the disc forward during opening movement. Lateral pterygoid muscles cause the forward movement of the condyle into the glenoid fossa." "This is a real-world medical entrance exam question, please give the true answer based on the question and selection. Topic name: None, Subject name: ENT","Which of the following is characteristic of T.B. otitis media - Select and explain. A. Marginal perforation, B. Attic perforation, C. Large central perforation, D. Multiple perforation",D: Multiple perforation,"Answer: D: Multiple perforation Explanation: None" "This is a real-world medical entrance exam question, please give the true answer based on the question and selection. Topic name: None, Subject name: Medicine","All of the following features are suggestive of asbestosis except: Select and explain. A. Occurs within five years of exposure, B. The disease progresses even after removal of contact, C. Can lead to pleural mesothelioma, D. Sputum contains asbestos bodies",A: Occurs within five years of exposure,"Answer: A: Occurs within five years of exposure Explanation: None" "This is a real-world medical entrance exam question, please give the true answer based on the question and selection. Topic name: None, Subject name: Ophthalmology","High molecular weight proteins in cataractous lens seen only in humans: Select and explain. A. HM 1 and 2, B. HM 2 and 3, C. HM 2 and 4, D. HM 3 and 4",D: HM 3 and 4,"Answer: D: HM 3 and 4 Explanation: Ans: D. HM 3 and 4(Ref The Eve 3/e p305-307: Biochemistry of Eve by David R/p28)High molecular weight proteins in cataractous lens only in humans = HM 3 & 4.HM3 & HM4:Insoluble high molecular weight proteins.Found in cataractous lens." "This is a real-world medical entrance exam question, please give the true answer based on the question and selection. Topic name: None, Subject name: Pathology","Which of following in not true about SLE? Select and explain. A. IgA. IgU against basement of mucosa or increased collagen formation, B. ROR-2 gene mutation, C. Complement c3 activation and fibroblast formation, D. Positive antinuclear antibody test",B: ROR-2 gene mutation,"Answer: B: ROR-2 gene mutation Explanation: None" "This is a real-world medical entrance exam question, please give the true answer based on the question and selection. Topic name: None, Subject name: Pathology","The surgical registrar successfully performs a testicu!lar biopsy and hands over the specimen to the attend!ing nurse. The sister asks you how to send the speci!men to the pathologist. What fluid will you tell the sister to put the specimen in? Select and explain. A. 95% ethanol, B. Zenker's solution, C. Bouin's solution, D. 10% formalin",C: Bouin's solution,"Answer: C: Bouin's solution Explanation: Ans: C. Bouin's solution(Ref: Surgical Pathology by 'Rosai and Ackerman 9Ie p27; Upper Urinary tract Urothelial Carcinoma by Michael Grasso (2015)1p11)Testicular biopsy specimen should be put into Boitin's solution.""Bouin's is a noncoagulate picrate solution which is routinely utilized to fix testicular biopsies because it preserves nuclear detail.""" "This is a real-world medical entrance exam question, please give the true answer based on the question and selection. Topic name: None, Subject name: Dental","Most common phobia in chilhood: Select and explain. A. Zoophobia, B. Nyclophobia, C. Xenophobia., D. Claustrophobia",A: Zoophobia,"Answer: A: Zoophobia Explanation: Zoophobia is found in children of age group 2-4 yrs and it disaapears by 10 yrs of age." "This is a real-world medical entrance exam question, please give the true answer based on the question and selection. Topic name: None, Subject name: Dental","The finish line used in complete metal cast crown is: Select and explain. A. Chamfer, B. Chafer with bevel, C. Feather edge, D. Shoulder",A: Chamfer,"Answer: A: Chamfer Explanation: None" "This is a real-world medical entrance exam question, please give the true answer based on the question and selection. Topic name: None, Subject name: Dental","In carrier base gutta percha the plastic carrier size of 40- 90 are made by? Select and explain. A. Plastic liquid crystal, B. Poly propylene, C. Poly sulphide, D. Poly sulphone",D: Poly sulphone,"Answer: D: Poly sulphone Explanation: None" "This is a real-world medical entrance exam question, please give the true answer based on the question and selection. Topic name: None, Subject name: Medicine","A 4-year old girl presents with severe vomiting after a viral fever of 5 days. She is hospitalized and develops cerebral edema. Liver biopsy is most likely to demonstrate w hich of the following? Select and explain. A. Centrizonal hemorrhagic necrosis, B. Non-alcoholic steatohepatitis, C. Ring granulomas, D. Marked microvesicular steatosis",D: Marked microvesicular steatosis,"Answer: D: Marked microvesicular steatosis Explanation: Ans. d. Marked microvesicular steatosisReye!s Syndrome (Fatty Liver with Encephalopathy):Acute illness, encountered exclusively in childrenOnset usually follows an upper respiratory tract infection, especially influenza or chickenpoxdegCharacterized clinically by: Vomitingdeg, signs of progressive CNS damagedeg, signs of hepatic injurydeg, hypoglycemiadegCause is unknown, although viral agents and salicylates have been implicate&Morphologically, extensive vacuolization of the liver and renal tubulesdeg.Extensive accumulation of fat droplets within hepatocytes (microvesicular steatosis)" "This is a real-world medical entrance exam question, please give the true answer based on the question and selection. Topic name: None, Subject name: Dental","The carat for Au foil used for restorations is: Select and explain. A. 18, B. 20, C. 22, D. 24",D: 24,"Answer: D: 24 Explanation: Direct filling gold, also called gold foil, is a metallic restorative material that relies on its ductility and malleability to adapt to a cavity preparation. The restorative process requires condensation that makes the final restoration practically a wrought metal. KARAT It refers to the parts of pure gold present in 24 parts of alloy, e.g., — 24 karat gold is pure gold. — 22 karat gold is 22 parts pure gold and 2 parts of other metal." "This is a real-world medical entrance exam question, please give the true answer based on the question and selection. Topic name: AIIMS 2020, Subject name: Pathology","APAF-1 is involved in the activation of which of the following casapses? Select and explain. A. Caspase 8, B. Caspase 9, C. Caspase 3, D. Caspase 10",B: Caspase 9,"Answer: B: Caspase 9 Explanation: Cytochrome c is released from mitochondria during apoptosis. Once released into the cytosol, cytochrome c binds to a protein called APAF-1 (apoptosis-activating factor-1), forming a multimeric structure called the apoptosome. This complex binds to caspase-9, the critical initiator caspase of the mitochondrial pathway, and promotes its autocatalytic cleavage, generating catalytically active forms of the enzyme." "This is a real-world medical entrance exam question, please give the true answer based on the question and selection. Topic name: None, Subject name: Forensic Medicine","All of the following are suggestive of domestic violence in a child except: Select and explain. A. Wormian bones, B. Microfractures in the sub-ep physeal region, C. Corner fractures, D. Bucket handle fractures of metaphyses",A: Wormian bones,"Answer: A: Wormian bones Explanation: Ans: A. Wormian bones(Ref: Reddy 34/e p417; Parikh 6/e p4. 187).Wormian bones are seen in Osteogenesis imperfecta and not child abuse.Wormian bones, also known as intrasutural bones or sutural bones, are extra bone pieces that can occur within a suture (joint) in the skull. These are irregular isolated bones that can appear in addition to the usual centres of ossification of the skull and, although unusual, are not rare'Battered baby' syndrome:Suspected due to severe & varied changes in metaphyses, periosteal new bone formation or epiphyseal separation.Also referred as ""Caffey's Syndrome/Infantile Whiplash Syndrome"".Injuries Type & Pattern:Arms & legs:Move about in a whiplash movement resulting in typical 'corner'or 'bucket-handle'-fractures in metaphyseal region.Classical metaphyseal corner or bucket handle fracture"" - Viually pathognomonic for abuse.Fractures of acromion, sternum & spinous processes - High specificity for abuse." "This is a real-world medical entrance exam question, please give the true answer based on the question and selection. Topic name: None, Subject name: Dental","What should be ideal depth of MTA for root end filling material? Select and explain. A. 1 mm, B. 3 mm, C. 5 mm, D. 7 mm",B: 3 mm,"Answer: B: 3 mm Explanation: None" "This is a real-world medical entrance exam question, please give the true answer based on the question and selection. Topic name: None, Subject name: Forensic Medicine","The high cou has the power to stay the execution of a pregnant woman according to which section of Criminal Procedure Code? Select and explain. A. 416 CrPC, B. 417 CrPC, C. 418 CrPC, D. 419 CrPC",A: 416 CrPC,"Answer: A: 416 CrPC Explanation: Ans: A. 416 CrPC(Ref Reddy 33/e p392; Textbook on the Indian Penal Code by Krishna Deo Gaur 4/e p594, the-indian-penal-code-pdf:d74214920)Section 416 of Criminal Procedure Code, 1973:Postponement of capital sentence pregnant woman.Ie., The power of high cou to stay execution of a pregnant woman.Sometimes if fit, commutes the sentence to life imprisonment." "This is a real-world medical entrance exam question, please give the true answer based on the question and selection. Topic name: None, Subject name: Anatomy","Which of the following muscle divides the sub-mandibular gland into a superficial and deep part? Select and explain. A. Genioglossus, B. Mylohyoid, C. Sternohyoid, D. Digastric",B: Mylohyoid,"Answer: B: Mylohyoid Explanation: None" "This is a real-world medical entrance exam question, please give the true answer based on the question and selection. Topic name: None, Subject name: Medicine","Renal osteodystrophy differs from nutritional and genetic form of osteomaLacia in having: Select and explain. A. Hypocalcemia, B. Hypercalcemia, C. Hypophosphatemia, D. Hyperphosphatemia",D: Hyperphosphatemia,"Answer: D: Hyperphosphatemia Explanation: None" "This is a real-world medical entrance exam question, please give the true answer based on the question and selection. Topic name: None, Subject name: Physiology","Skeletal muscles: Select and explain. A. Contracts when calcium is taken up by sacroplasmic reticulum, B. Contracts when actin and myosin filaments shorten, C. Contraction is initiated by calcium binding to troponin, D. Contraction is initiated by calcium binding to tropomyosin",C: Contraction is initiated by calcium binding to troponin,"Answer: C: Contraction is initiated by calcium binding to troponin Explanation: None" "This is a real-world medical entrance exam question, please give the true answer based on the question and selection. Topic name: None, Subject name: Dental","The advantage of glass ionomer cement reinforced with metal particles is: Select and explain. A. Decreased strength but biocompatibility, B. Increased resistance to abrasion, C. Better colour, D. Ease of manipulation",B: Increased resistance to abrasion,"Answer: B: Increased resistance to abrasion Explanation: None" "This is a real-world medical entrance exam question, please give the true answer based on the question and selection. Topic name: AIIMS 2017, Subject name: Surgery","Best guide for the management of Resuscitation is: Select and explain. A. CVP, B. Urine output, C. Blood pressure, D. Saturation of Oxygen",B: Urine output,"Answer: B: Urine output Explanation: The best measure of organ perfusion and the best monitor of the adequacy of shock therapy remains the urine output"". Minimum urine output in ADULTS - 1ml/min Minimum urine output in CHILDRENS - (0.5 - 1) ml/min" "This is a real-world medical entrance exam question, please give the true answer based on the question and selection. Topic name: None, Subject name: Dental","The disturbances occurred during ""Calcification"" stage of tooth development is seen in: Select and explain. A. Peg laterals, B. Microdontia, C. Supernumerary tooth, D. Interglobular dentin",D: Interglobular dentin,"Answer: D: Interglobular dentin Explanation: None" "This is a real-world medical entrance exam question, please give the true answer based on the question and selection. Topic name: None, Subject name: Dental","All of the following affects bur efficiency except: Select and explain. A. Spiral angle, B. Head length and diameter, C. Neck diameter, D. Crosscut",B: Head length and diameter,"Answer: B: Head length and diameter Explanation: Taper angle and head length and diameter has no effect on bur efficiency." "This is a real-world medical entrance exam question, please give the true answer based on the question and selection. Topic name: None, Subject name: Anatomy","All are structures lying deep to the hyoglossus muscle except: Select and explain. A. Hypoglossal nerve, B. Lingual artery, C. Stylohyoid muscle, D. Geniohyoid muscle",A: Hypoglossal nerve,"Answer: A: Hypoglossal nerve Explanation: The hyoglossus, a quadrilateral sheet of muscle, is the key muscle of the suprahyoid region because it serves as a landmark for neighboring structures in the region.  Superficial relations:  Hypoglossal nerve, crosses the lower part of muscle from behind forwards. Lingual nerve, crosses the upper part of muscle from behind forwards. Deep part of the submandibular gland and submandibular duct. The gland lies in the middle of hyoglossus muscle and the duct lies between the gland and the muscle. Submandibular ganglion lies between the lingual nerve and deep part of the submandibular gland. Styloglossus muscle—interdigitates with hyoglossus. Mylohyoid muscle overlaps the hyoglossus anterosuperiorly. Deep relations:  Inferior longitudinal muscle of tongue. Genioglossus muscle anteriorly. Middle constrictor of pharynx posteriorly. Glossopharyngeal nerve. Stylohyoid ligament. Lingual artery. Structures passing deep to posterior border of hyoglossus: Glossopharyngeal nerve. Stylohyoid ligament. Lingual artery." "This is a real-world medical entrance exam question, please give the true answer based on the question and selection. Topic name: None, Subject name: Pharmacology","Which of the following disease modifying anti-rheumatoid drugs acts by increasing extracellular adenosine? Select and explain. A. Leflunomide, B. Hydroxychloroquine, C. Azathioprine, D. Methotrexate",D: Methotrexate,"Answer: D: Methotrexate Explanation: Answer-D. Methotrexate""Methotrexate is the DMARD of choicefor the treatment of RA and is the anchor drug for most combination therapies.At the dosages ased for lhe treatment of RA, methotrexale has been shown to stimulate adenosine release from cells, producing an anti-inflammatory effect.Methotrexate (MTX) is a folic acid analog that binds with high affinity to the active catalytic site of dihydrofolate reductase (DHFR)." "This is a real-world medical entrance exam question, please give the true answer based on the question and selection. Topic name: None, Subject name: Dental","Oral prophylaxis is contraindicated in: Select and explain. A. Prepubertal gingivitis, B. Pregnancy gingivitis, C. Ulcerative refractory gingivitis, D. Leukemic gingivitis",D: Leukemic gingivitis,"Answer: D: Leukemic gingivitis Explanation: None" "This is a real-world medical entrance exam question, please give the true answer based on the question and selection. Topic name: None, Subject name: Anatomy","Injury to the right mandibular nerve as it passes through foramen ovale produces the following effects: Select and explain. A. Loss of taste from the anterior 2/3 of tongue, B. Deviation of the tongue to the left, C. Deviation of chin to left when mouth is opened, D. Loss of general sensation to anterior 2/3 of tongue",D: Loss of general sensation to anterior 2/3 of tongue,"Answer: D: Loss of general sensation to anterior 2/3 of tongue Explanation: {This is due to damage of lingual branch of mandibular nerve.}" "This is a real-world medical entrance exam question, please give the true answer based on the question and selection. Topic name: None, Subject name: Dental","The cross section of the root most prone to vertical root fracture is Select and explain. A. Round, B. Oval, C. Kidney shape, D. Elliptical",B: Oval,"Answer: B: Oval Explanation: One of the common anatomic features shared by teeth that typically develop VRFs is an oval cross section of the root, with a buccolingual diameter being larger than the mesiodistal diameter.These teeth include the maxillary and mandibular premolars, the mesial roots of the mandibular molars, and the mandibular incisors. Such anatomy is easily observed in the axial plane of a CBCT scan.The fracture in these teeth typically starts in the buccolingual plane, specifically at the highest convexity of the oval roots. Cohen's Pathways of the pulp ed 11 pg 809" "This is a real-world medical entrance exam question, please give the true answer based on the question and selection. Topic name: None, Subject name: Dental","In which of the following conditions oral screen should not be used: Select and explain. A. Nail biting habit, B. Tongue thrusting, C. Acute infection of tonsils and adenoids, D. Thumb sucking",C: Acute infection of tonsils and adenoids,"Answer: C: Acute infection of tonsils and adenoids Explanation: Oral screen was first introduced by NEWELL. It works on the principles of both force application and force elimination." "This is a real-world medical entrance exam question, please give the true answer based on the question and selection. Topic name: None, Subject name: Surgery","A patient who is on a regimen of steroid therapy and has need for extraction of chronically infected teeth requires premedication with Select and explain. A. Atropine to reduce the hazard of vagal stimulation and cardiac arrest., B. Antihypertensive to combat tendency towards shock, C. Antihistaminics to prevent allergic reaction, D. Antibiotics",D: Antibiotics,"Answer: D: Antibiotics Explanation: None" "This is a real-world medical entrance exam question, please give the true answer based on the question and selection. Topic name: None, Subject name: Pediatrics","A 6–month old boy weighing 3.2 kg presents with recurrent vomiting and polyuria. Investigations show blood area 60 mg/dl creatinine 0.7 mg/dL, calcium 12.8 mg/dL, phosphate 3 mg/dL, pH 7.45, bicarbonate 25 mEq/L and PTH 140 pg ml (normal <60 pg/ml). Daily urinary calcium excretion is reduced. Ultrasound abdomen show bilateral nephrocalcinosis. The most likely diagnosis is – Select and explain. A. Bartter syndrome, B. Mutation of the calcium sensing receptor, C. Ps eudo–ps eudohypoparathyroidism, D. Parathyroid adenoma",B: Mutation of the calcium sensing receptor,"Answer: B: Mutation of the calcium sensing receptor Explanation: None" "This is a real-world medical entrance exam question, please give the true answer based on the question and selection. Topic name: None, Subject name: Dental","Type of mouth breathing can be: Select and explain. A. Obstructive., B. Anatomic., C. Habitual., D. All of the above.",D: All of the above.,"Answer: D: All of the above. Explanation: None" "This is a real-world medical entrance exam question, please give the true answer based on the question and selection. Topic name: None, Subject name: Dental","The order of caries susceptibility of primary teeth is: Select and explain. A. 2nd molar>1st molar>canine, B. 1st molar>2st molar>canine, C. Canine>1st molar>2nd molar, D. Canine>2nd molar>1st molar",A: 2nd molar>1st molar>canine,"Answer: A: 2nd molar>1st molar>canine Explanation: Caries susceptibility of primary teeth (in descending order) 2nd molar > 1st molar > canine > lateral incisor > central incisor Caries susceptibility of permanent teeth (in descending order) 1st molar > 2nd molar > Max. 2nd PM > Max. 1st PM & Mand. 2nd PM > Max. central & lateral incisor > Max. canine & Mand. 1st PM > Mand. Central & lateral incisor > Mand. canine Surfaces involved in descending order (in primary teeth)  Occlusal > molar interproximal > incisor interproximal. (Proximal surfaces were affected more in first than in second primary molars)." "This is a real-world medical entrance exam question, please give the true answer based on the question and selection. Topic name: AIIMS 2017, Subject name: Radiology","26 year old, 6 feet 4 inch male presented to the emergency depament with complaints of acute onset dysnea. On examination, he was tachypneic, other vitals were stable. A chest xray was done which shown the following pathology: Select and explain. A. Pneumothorax, B. Hydropnemo thorax, C. Massive pleural effusion, D. Pulmonary embolism",A: Pneumothorax,"Answer: A: Pneumothorax Explanation: - The given image shows * Absent of Bronchovascular markings on the left lung compared to right lung * Left lung shifted or pushed toward the hilum- Hence, the given image is a case of Pneumothorax- If patient has pneumothorax with mediastinal shift, it is Tension pneumothorax and should be immediately managed with needle inseion in 5th ICS in mid-axillary line." "This is a real-world medical entrance exam question, please give the true answer based on the question and selection. Topic name: AIIMS 2019, Subject name: Physiology","Acetyl choline release can be increased from presynaptic membrane by: Select and explain. A. Blocking voltage gated Na+ channels on presynaptic membrane, B. Blocking voltage gated Ca+ channels on presynaptic membrane, C. Blocking voltage gated K+ channels on presynaptic membrane, D. Blocking voltage gated CI- channels on presynaptic membrane",C: Blocking voltage gated K+ channels on presynaptic membrane,"Answer: C: Blocking voltage gated K+ channels on presynaptic membrane Explanation: Blocking voltage gated K+ channels on presynaptic membrane | K+ exit stops | Membrane does not get hyperpolarized |Continued Ach release When an AP depolarizes the presynaptic membrane=Ca++ channel open=allow release of neurotransmitter=from terminal=into synaptic cleft=forming bridge for the vesicles and release sites of a depolarized membrane. Blocking voltage gated Na++ channels will block the AP propagation Chloride channels are present=on the post synaptic membrane." "This is a real-world medical entrance exam question, please give the true answer based on the question and selection. Topic name: None, Subject name: Social & Preventive Medicine","In a JubLeitter arca pith crude bih rate of 211, what would be the expected number of ANC registrations? Select and explain. A. 60, B. 80, C. 100, D. 120",A: 60,"Answer: A: 60 Explanation: Ans. a. 60In a subcenter area with crude bih rate of 20, the expected number of ANC registrations should be approximately 55.Number of Expected Pregnancies per Year Expected no. of live-bihs (Y)/year= Bih rate (per 1000 population) x Population of the area/1000 Number of Expected Pregnancies per YearAs some pregnancies may not result in a live bih (i.e., aboions & stillbih may occur), the expected number of live bihs would be an under-estimation of the total number of pregnancies. Hence, a correction factor of 10% is required, i.e., add 10% to the figure obtained above.Total number of Expected Pregnancies Z = Y + 10% of YPopulation under the subcentre = 5000Bih rate = 20Expected no. of live-bihs (Y)/year = 20 x 5000/1000 = 100Total number of Expected Pregnancies Z = Y + 10% of Y =100 + 10% of 100 = 110Expected number of ANC registrations will be half of yearly calculation = 55" "This is a real-world medical entrance exam question, please give the true answer based on the question and selection. Topic name: None, Subject name: Gynaecology & Obstetrics","A woman comes with obstructed labour and is grossly dehydrated. Investigations reveal fetal demise. What will be the management ? Select and explain. A. Craniotomy, B. Decapitation, C. Cesarean section, D. Forceps extraction",C: Cesarean section,"Answer: C: Cesarean section Explanation: None" "This is a real-world medical entrance exam question, please give the true answer based on the question and selection. Topic name: None, Subject name: Anatomy","When a patient is asked to say 'ah' , if the uvula is drawn upwards to the left, the cranial nerve likely to be damaged is Select and explain. A. Vagus, B. Rt accessory, C. Lt accessory, D. Hypoglossal",B: Rt accessory,"Answer: B: Rt accessory Explanation: {All the muscles of soft palate are supplied by pharyngeal plexus derived from the cranial part of accessory nerve except tensor veli palatine which is supplied by the mandibular nerve. The action of musculus uvula is it pulls up the uvula. As the Right accessory is paralysed the uvula is drawn to the left side due to the normal action of left accessory.}" "This is a real-world medical entrance exam question, please give the true answer based on the question and selection. Topic name: None, Subject name: Dental","Which of the following event has little or no significance? Select and explain. A. Mandibular incisor erupting before mandibular molar, B. Mandibular canine erupting before mandibular premolar, C. Mandibular 2nd premolar erupting before mandibular 2nd molar, D. Maxillary premolar and canine erupting before maxillary 2nd molar",A: Mandibular incisor erupting before mandibular molar,"Answer: A: Mandibular incisor erupting before mandibular molar Explanation: None" "This is a real-world medical entrance exam question, please give the true answer based on the question and selection. Topic name: None, Subject name: Dental","On rubber dam application gingiva is seen protruding out of margins, it can be seen due to? Select and explain. A. Edematous gingival tissues, B. Holes paced too far apart, C. Using a light-weight dam, D. Holes placed too close together",D: Holes placed too close together,"Answer: D: Holes placed too close together Explanation: None" "This is a real-world medical entrance exam question, please give the true answer based on the question and selection. Topic name: None, Subject name: Pharmacology","Which of these statements depicts vasomotor reversal of Dale? Select and explain. A. Propranolol followed by adrenaline, B. Propranolol followed by noradrenaline, C. Decrease in hea rate on adrenaline administration after phentolamine has been given, D. Noradrenaline followed by propranolol",C: Decrease in hea rate on adrenaline administration after phentolamine has been given,"Answer: C: Decrease in hea rate on adrenaline administration after phentolamine has been given Explanation: Ans: C. Decrease in hea rate on adrenaline administration after phentolamine has been given (Ref: KDT 7/e e p131, 140).This was first demonstrated by Sir HH Dale (1913) and is called vasomotor reversal of Dale.Blockade of vasoconstrictor alpha-1 (also alpha-2) receptors reduces peripheral resistance and causes pooling of blood in capacitance vessels a venous return and cardiac output are reduced a Fall in BP.Postural reflex is interfered with a marked hypotension occurs on standing a dizziness and syncope. Hypovolemia accentuates the hypotension.Decrease in hea rate on adrenaline administration after phentolamine has been given depicts vasomotor reversal of Dale.The alpha-blocker abolishes the pressor action of adrenaline (injected IV in animals), which then produces only fall in BP due to beta-2 mediated vasodilatation." "This is a real-world medical entrance exam question, please give the true answer based on the question and selection. Topic name: None, Subject name: Dental","Which of the following best describes the resistance form in endodontic root canal preparation? Select and explain. A. The establishment of apical tug back resistance to prevent the apical root fracture during obturation, B. The cavity form which is obtained for application of condenser in vertical compaction, C. The cavity form which is designed for application of condenser in lateral compaction, D. Cavity form which is designed to obtain for entry of condenser during obturation",A: The establishment of apical tug back resistance to prevent the apical root fracture during obturation,"Answer: A: The establishment of apical tug back resistance to prevent the apical root fracture during obturation Explanation: None" "This is a real-world medical entrance exam question, please give the true answer based on the question and selection. Topic name: None, Subject name: Dental","Which of the following statements about high copper silver alloy compared to conventional alloy is not true? Select and explain. A. It has increased tensile and compressive strength, B. It has poor tarnish and corrosion resistance, C. Its' edge strength is greater, D. Mercury content in the final restoration is less",B: It has poor tarnish and corrosion resistance,"Answer: B: It has poor tarnish and corrosion resistance Explanation: None" "This is a real-world medical entrance exam question, please give the true answer based on the question and selection. Topic name: None, Subject name: Surgery","Nimodipine is approved for use in: Select and explain. A. Subdural hemorrhage, B. Extradural hemorrhage, C. Intracerebral hemorrhage, D. Subarachnoid hemorrhage",D: Subarachnoid hemorrhage,"Answer: D: Subarachnoid hemorrhage Explanation: Ans: D. Subarachnoid hemorrhage(Ref: Harrison 19/e p1787, 1735; Ratting 13/e p202, 12/e p204).Nimodipine is approved for use in subarachnoid hemorrhage.Subarachnoid hemorrhage:Typically treated with the calcium channel blocker nimodipine, aggressive IV fluid administration, and therapy aimed at increasing blood pressure, typically with vasoactive drugs such as phenylephrine.The IV fluids and vasoactive drugs (hypeensive hypervolemic therapy) are used to overcome the cerebral vasospasm.Early surgical clipping or endovascular coiling of aneurysms is advocated to prevent complications related to re-bleeding.Hydrocephalus, typically heralded by a decreased level of consciousness, may require ventriculostomy drain." "This is a real-world medical entrance exam question, please give the true answer based on the question and selection. Topic name: None, Subject name: Dental","The first instrument used on the facial surfaces of gingiva while gingivectomy is: Select and explain. A. Orban's knife, B. Kirkland's knife, C. BP blade with No. 11 blade, D. BP blade with No. 12 blade",B: Kirkland's knife,"Answer: B: Kirkland's knife Explanation: None" "This is a real-world medical entrance exam question, please give the true answer based on the question and selection. Topic name: None, Subject name: Dental","Internal occlusal rest provided in partial denture does not provide: Select and explain. A. Horizontal stabilization, B. Retention, C. Vertical stop, D. Occlusal support",B: Retention,"Answer: B: Retention Explanation: None" "This is a real-world medical entrance exam question, please give the true answer based on the question and selection. Topic name: None, Subject name: Pediatrics","Hyperkalemia in children may occur in all except – Select and explain. A. Insulin deficiency, B. Metabolic acidosis, C. Acute renal failure, D. Cushing's syndrome",D: Cushing's syndrome,"Answer: D: Cushing's syndrome Explanation: Hyperkalemia Hyperkalemia defined as serum K+ concentration > 5.5 meq/L." "This is a real-world medical entrance exam question, please give the true answer based on the question and selection. Topic name: None, Subject name: Pathology","The feature that distinguishes herpes zoster from other vesiculo butlous eruptions is Select and explain. A. Unilateral occurrence, B. Severe burning pain, C. Prominent crusting vesicles, D. Sub epidermal bulious formation",A: Unilateral occurrence,"Answer: A: Unilateral occurrence Explanation: None" "This is a real-world medical entrance exam question, please give the true answer based on the question and selection. Topic name: None, Subject name: Biochemistry","Which of the following is not a technique for protein precipitation? Select and explain. A. Trichloroacetic acid, B. Heat precipitation, C. Isoelectric point method, D. Titration with reducing sugar",D: Titration with reducing sugar,"Answer: D: Titration with reducing sugar Explanation: Ans: D. Titration with reducing sugar(Ref Harper 30/e p26)Titration with reducing sugar is not used for protein precipitation.Protein precipitation:Widely used to concentrate proteins & purify them from various contaminants.Used for isolating specific protein in quantities sufficient for analysis.Requires multiple successive purification techniques.Classic approaches:Based on,Isoelectric precipitation - Differences in relative solubility of individual proteins as pH function.Precipitation with ethanol or acetone - Polarity.Salting out with ammonium sulfate.Paper chromatography including TLCD." "This is a real-world medical entrance exam question, please give the true answer based on the question and selection. Topic name: None, Subject name: Dental","Resistance to corrosion in a cobalt-chrome casting is due to presence of: Select and explain. A. High quality iron, B. Chrome, C. Cobalt, D. Nickel",B: Chrome,"Answer: B: Chrome Explanation: None" "This is a real-world medical entrance exam question, please give the true answer based on the question and selection. Topic name: None, Subject name: Surgery","A 60-year-old chronic smoker presented with progres-sive jaundice, pruritus and clay colored stools for 2 months. History of waxing and waning of jaundice was present. A CT scan revealed dilated main pancre-atic duct and common bile duct. What is the likely di-agnosis? Select and explain. A. Carcinoma head of pancreas, B. Periampullary carcinoma, C. Chronic pancreatitis, D. H lar cholangiocarcinoma",B: Periampullary carcinoma,"Answer: B: Periampullary carcinoma Explanation: Ans: B. Periampullary carcinoma(Ref Sabiston 201e p1544, 19/e p1535-1544; Schwaz 10/e p1408, 9/e p1220-1225 Bailey 27/e p1234, 26/e p1138, Blumga 5/e p919-925; Shackelford 7/e p1190-1196)Suggestive diagnosis is periampullary carcinoma.Since history of waxing and waning episode is seen.The waxing and waning nature of jaundice is due to sloughing of ampullaty cancer, resulting in transient resolution of the jaundice." "This is a real-world medical entrance exam question, please give the true answer based on the question and selection. Topic name: None, Subject name: Surgery","Patellar cluck syndrome is interposition of soft tissue at or hyperophied scar: Select and explain. A. Superior pole of patella and impinging on femoral component during flexion, B. Superior pole of patella and impinging on femoral component during extension, C. Inferior pole of patella and impinging on femoral component during flexion, D. Inferior pole of patella and impinging on femoral component during extension",B: Superior pole of patella and impinging on femoral component during extension,"Answer: B: Superior pole of patella and impinging on femoral component during extension Explanation: Ans: B. Superior pole of patella and impinging on femoral component during extension Patellar Clunk Syndrome is a painful condition associated with a mechanical catching or clunking during active extension following total knee ahroplasty (TKA).The syndrome is caused by growth of interposing soft tissue usually at the superior pole of the patella.This interposed soft tissue cannot be visualized on plain radiographs." "This is a real-world medical entrance exam question, please give the true answer based on the question and selection. Topic name: None, Subject name: Dental","For statistical test output of p value is interpreted as Select and explain. A. Rejective Ho, p <0.05, B. Rejective Ho, p>0.05, C. Rejective H1, p<0.05, D. Accepting Ho, p<0.05","A: Rejective Ho, p <0.05","Answer: A: Rejective Ho, p <0.05 Explanation: None" "This is a real-world medical entrance exam question, please give the true answer based on the question and selection. Topic name: None, Subject name: Dental","What is the most frequently used restorative material? Select and explain. A. Silicate, B. Amalgam, C. Composite, D. Gold",B: Amalgam,"Answer: B: Amalgam Explanation: None" "This is a real-world medical entrance exam question, please give the true answer based on the question and selection. Topic name: None, Subject name: Biochemistry","Magnesium is needed for the activity of which enzyme? Select and explain. A. Phosphatase, B. Aldolase, C. Dismutase, D. ATPase",D: ATPase,"Answer: D: ATPase Explanation: None" "This is a real-world medical entrance exam question, please give the true answer based on the question and selection. Topic name: None, Subject name: Dental","Anterior 2/3rd of tongue is supplied by Select and explain. A. Facial nerve, B. X nerve, C. IX nerve, D. XI nerve",A: Facial nerve,"Answer: A: Facial nerve Explanation: None" "This is a real-world medical entrance exam question, please give the true answer based on the question and selection. Topic name: None, Subject name: Dental","What percentage of cases of mandibular incisors have single apical foramen Select and explain. A. 97.80%, B. 82.50%, C. 60%, D. 60%- 78%",A: 97.80%,"Answer: A: 97.80% Explanation: None" "This is a real-world medical entrance exam question, please give the true answer based on the question and selection. Topic name: None, Subject name: Dental","Condensation shrinkage of porcelain during firing depends on: Select and explain. A. Rate of arriving at firing temperature., B. Uniformity of particle size., C. Shape and size of particle., D. Type of investment used",C: Shape and size of particle.,"Answer: C: Shape and size of particle. Explanation: Firing shrinkage occurs due to loss of water and densification through sintering. It is about 40% by volume. This can be reduced by proper condensation and firing technique. The higher the firing temperature the greater is the firing shrinkage." "This is a real-world medical entrance exam question, please give the true answer based on the question and selection. Topic name: None, Subject name: Social & Preventive Medicine","The prevalence of disease in a population is expected to be 50%. An investigator is interested in the range of 45 to 55% with 95% confidence. The minimum sample size required for the study is: Select and explain. A. 100, B. 200, C. 300, D. 400",D: 400,"Answer: D: 400 Explanation: Answer- D (400)In this question, use Standard Error of propoions.The 95% confidence limits for a propoion are p +- 1.96 s.e.p, where s.e.p is the standard error of a propoion.p = sample propoion = propoion of diseased persons = 0.5 q = 1-p = 0.5 s.e.p .NI(p x q / n) Now here, the range of propoions with 95% confidence is mentioned as? 45-55%Hence, 2 s.e.p = 0.55-0.50 = 0.05 s.e.p = 0.0250.025= J(pxq/n)n = 400Thus, the sample size required will be 400." "This is a real-world medical entrance exam question, please give the true answer based on the question and selection. Topic name: None, Subject name: Anatomy","Inferior alveolar nerve runs: Select and explain. A. Medial to lateral pterygoid, B. From Infratemporal fossa, C. Lateral to medial pterygoid, D. Lateral to lateral pterygoid",C: Lateral to medial pterygoid,"Answer: C: Lateral to medial pterygoid Explanation: None" "This is a real-world medical entrance exam question, please give the true answer based on the question and selection. Topic name: None, Subject name: Surgery","A 12-years-old boy presents with a symmetric, expansile cystic lesion in the proximal humerus. All of the following can be done for his treatment except Select and explain. A. Curettage, B. Intralesional steroids, C. Intralesional sclerosing agents, D. Radiotherapy",B: Intralesional steroids,"Answer: B: Intralesional steroids Explanation: Ans. BSolitary bone cyst is the diagnosis of a 12 years old boy, who presents with a symmetric, expansile cystic lesion in the proximal humerus.Solitary Bone Cyst:Common in first two decades, primarily between 5-15 yearsMore common in malesMC site: Proximal humerus (50-60%) >Femur (25-30%)Cyst appears as centrally radioluscent lesions, on the metaphyseal side of the growth plate of a long boneTreatment:CurettageMethylprednisolone acetate injection into lesionRadiographic contrast injection into lesionIntralesional injection of sclerosing agentsSolitary Bone Cyst (Unicameral Bone Cyst)Common in first two decades, primarily between 5-15 yearsQMore common in malesQMC site: Proximal humerusQ (50-60%) >Femur (25-30%)It is not a true cystdeg because not lined by endothelial cells but lined by fibrous tissue and blood vesselsClinical Features:Asymptomaticdeg unless presents as a fractureRadiological Investigations:Cyst appears as centrally radioluscent lesionsQ, on the metaphyseal side of the growth plate of a long boneCoex is thinned but intactQFallen fragment sign"" may present indication that the lesion is fluid filled rather than solidQOther tests are usually not required except in unusual locations such as pelvis. In such locations MRI or CT is indicatedMRI can document extent of the lesion and its cystic natureQMRI helps in distinguishing unicameral cyst (SBC) from aneurysmal bone cyst (ABC), giant celltumor (GCT) and fibrous dysplasiaQTreatment:CurettageQMethylprednisolone acetateQ injection into lesionRadiographic contrast injectionQ into lesionIntralesional injection of sclerosing agentsQ" "This is a real-world medical entrance exam question, please give the true answer based on the question and selection. Topic name: None, Subject name: Pathology","Iron is taken up into enterocytes through which of the following? Select and explain. A. DMT-2, B. DMT-1, C. Ferroprotein 1, D. GLUT1",B: DMT-1,"Answer: B: DMT-1 Explanation: Ans: B. DMT-1Iron is brought into the cell through an active transpo processProtein DMT-1 (Divalent Metal Transpoer-1),Expressed on the apical surface of enterocytes in initial pa of duodenum.DMT-1 is not specific to iron & transpos other substances like zinc, copper, cobalt & manganese." "This is a real-world medical entrance exam question, please give the true answer based on the question and selection. Topic name: None, Subject name: Surgery","Which is not associated with GDM (Gestational DM)? Select and explain. A. Past history of fetal abnormality, B. Obesity, C. Hypeension, D. Macrosomia",C: Hypeension,"Answer: C: Hypeension Explanation: Answer- C. HypeensionA previous diagnosis of gestational diabetes or prediabetes, impaired glucose tolerance, or impaired fasting glycaemia.Being overweight, obeseA previous pregnancy which resulted in a child with a macrosomia." "This is a real-world medical entrance exam question, please give the true answer based on the question and selection. Topic name: None, Subject name: Biochemistry","A person on fat free carbohydrate rich diet continues to grow obese. Which of the following lipoproteins is likely to be elevated in his blood? Select and explain. A. Chylomicron, B. VLDL, C. HDL, D. LDL",B: VLDL,"Answer: B: VLDL Explanation: As VLDL is the main transporter of endogenous triglycerides; all the conditions causing increased endogenous triglyceride synthesis will also increase VLDL secretion. And these are:- Well fed state (not starved). High level of circulating free fatty acids. Carbohydrate rich diet (especially sucrose and fructose) Present of high level of insulin and low level of glycogen. Ethanol (alcohol) ingestion. Key Concept:" "This is a real-world medical entrance exam question, please give the true answer based on the question and selection. Topic name: None, Subject name: Radiology","Which of the following is the least useful for diagnosis of Spondylolisthesis? Select and explain. A. CT, B. MR, C. X-ray lumbar spine- AP view, D. X-ray lumbar spine- Lateral view",C: X-ray lumbar spine- AP view,"Answer: C: X-ray lumbar spine- AP view Explanation: Ans. c. X-ray lumbar spine- AP view (Ref Tureks 6/e p503; Maheshwarl 3e/p 237; Wolfgang 7/e p 228-29, 206; Sutton 7/e p/66'Although for imaging of spondylolisthesis usually all are done, AP view appears to be least useful of these choices. On AP view, spondylolisthesis can show a sign called Napolean Hat sign. The lateral view is useful in detecting spondylolisthesis; it may demonstrate the pars defect.Spondylolisthesis is forward slip of one veebrae upon another; so it is best viewed (or seen earliest) in sagittal images of spine i.e. lateral and oblique X-ray of spine and saggital and axial views of CT and MRIQ" "This is a real-world medical entrance exam question, please give the true answer based on the question and selection. Topic name: None, Subject name: Medicine","Which of the following is false about Transfusion-Related Acute Lung Injury? Select and explain. A. Develops within 24 hours, B. Mostly seen after sepsis and cardiac surgeries, C. It's a cause of non-cardiogenic pulmonary edema, D. Plasma is more likely to cause it than whole blood",A: Develops within 24 hours,"Answer: A: Develops within 24 hours Explanation: Ans: A. Develops within 24 hours(Ref Harrison 19/e 138e-5, 18/e p1217)Transfusion-Related Acute Lung Injury (TRALI):MC cause of transfusion related fatalities.Cause:Usually results from transfusion of donor plasma.Risk factors:Smoking.Chronic alcohol useShockLiver surgery (transplantation).Symptoms development:During or within 6 hrs of transfusion.Develops symptoms of hypoxia (PaO2/FIO2 <300 mmHg).Non-cardiogenic pulmonary edema signs including bilateral interstitial infiltrates on chest X-ray." "This is a real-world medical entrance exam question, please give the true answer based on the question and selection. Topic name: AIIMS 2018, Subject name: Anatomy","All of the following bones are involves in wrist joint formation EXCEPT? Select and explain. A. Scaphoid, B. Ulna, C. Radius, D. Triquetral",B: Ulna,"Answer: B: Ulna Explanation: Wrist joint aka radiocarpal joint is a synol, ellipsoid joint formed by: - Ulna: - Forms inferior radioulnar joint Function: pronation & supination Separated from wrist joint by an aicular disc. Wrist joint formed by: - Upper: Inferior surface of lower end of radius Aicular disc of the inferior radioulnar joint Lower: Scaphoid Lunate Triquetral (ulnar detion)" "This is a real-world medical entrance exam question, please give the true answer based on the question and selection. Topic name: None, Subject name: Dental","The non-invasive method to measure the blood flow is: Select and explain. A. Electric pulp test, B. Percussion, C. Radiograph, D. Laser Doppler flowmetry",D: Laser Doppler flowmetry,"Answer: D: Laser Doppler flowmetry Explanation: None" "This is a real-world medical entrance exam question, please give the true answer based on the question and selection. Topic name: None, Subject name: Forensic Medicine","All are true about Doctrine ofborrowed servant except? Select and explain. A. Hospital administration is responsible for the actions of interns and trainees, B. A nurse assisting a surgery is a borrowed servant ofthe hospital, C. Doctor assisting a surgeon for fee in OT, D. Physician is responsible for the acts ofthe orderlies and nursing staff done by his orders",C: Doctor assisting a surgeon for fee in OT,"Answer: C: Doctor assisting a surgeon for fee in OT Explanation: Ans: C. Doctor assisting a surgeon for fee in OTBorrowed servant doctrine:The common law principle that the employer of a borrowed employee' rather than the emp|oyee's regular employer is liable for employees actions that occur while the emptoyee is under the control of the temporary employer.Sometimes referred to as borrowed employee doctrine." "This is a real-world medical entrance exam question, please give the true answer based on the question and selection. Topic name: None, Subject name: Pharmacology","Which antibiotic is effective against pseudomonas and indole +ve proteus? Select and explain. A. Methicillin, B. Carbenicillin, C. Lincomycin, D. Oxacillin",B: Carbenicillin,"Answer: B: Carbenicillin Explanation: None" "This is a real-world medical entrance exam question, please give the true answer based on the question and selection. Topic name: None, Subject name: Dental","Which of the following analysis helps in determining the disproportion in the size between maxillary and mandibular teeth? Select and explain. A. Pont's analysis, B. Bolton's analysis, C. Peck and Peck analysis, D. Carey's analysis",B: Bolton's analysis,"Answer: B: Bolton's analysis Explanation: None" "This is a real-world medical entrance exam question, please give the true answer based on the question and selection. Topic name: None, Subject name: Surgery","Trismus is due to inflammation of: Select and explain. A. Medial pterygoid, B. Facial nerve irritation, C. Superior constrictor, D. Buccinator",A: Medial pterygoid,"Answer: A: Medial pterygoid Explanation: None" "This is a real-world medical entrance exam question, please give the true answer based on the question and selection. Topic name: None, Subject name: Dental","All of the following are disadvantages of composite restoration except: Select and explain. A. Greater localized occlusal wear, B. Recurrent secondary caries, C. Technique sensitive, D. More time in replacement",D: More time in replacement,"Answer: D: More time in replacement Explanation: None" "This is a real-world medical entrance exam question, please give the true answer based on the question and selection. Topic name: None, Subject name: Dental","Which of the following is common to both zinc oxide eugenol cement and polycarboxylate cement? Select and explain. A. Polyacrylic acid is liquid, B. Chemical bond to tooth structure, C. Chelation, D. Substitution of eugenol by EBA to increase strength of cement",C: Chelation,"Answer: C: Chelation Explanation: Glass ionomers bond to tooth structure by chelation of the carboxyl groups of the polyacrylic acids with the calcium in the apatite of the enamel and dentin, in a manner similar to polycarboxylate cement. The setting reaction of ZOE starts with water in the eugenol solution that hydrolyzes the zinc oxide to form zinc hydroxide. The zinc hydroxide and eugenol chelate and solidify. Ref: Phillips Ed-12th P: 331" "This is a real-world medical entrance exam question, please give the true answer based on the question and selection. Topic name: None, Subject name: Pediatrics","A 5 year-old child presents with non-utatiening macules and pappules on the skin. Skin biopsy revealed a perivascular IgA deposition. Which of the following is the most likely diagnosis? Select and explain. A. Henoch Shonlein purpura, B. Wegener's granulomatosis, C. Kawasaki disease, D. Drug-induced vasculitis",A: Henoch Shonlein purpura,"Answer: A: Henoch Shonlein purpura Explanation: Ans. a. Henoch-Shonlein purpura (Ref Harrison 19/e p2100, 18/e p2797)Presence of non-blanching palpable purpura (vasculitis of skin) and colicky abdominal pain (vasculitis of gastrointestinal tract) in a five-year-old child together with evidence of IgA deposition in immune complexes suggests the diagnosis of Henoch Scholein Purpura" "This is a real-world medical entrance exam question, please give the true answer based on the question and selection. Topic name: None, Subject name: Pediatrics","Which of the following statements about cystic fibrosis (CF) is not true – Select and explain. A. Autosomal recessive disorder, B. Abnormality in CFTR which leads to defective calcium transport, C. Mutation in cystic fibrosis transport regulator, D. All of the above",B: Abnormality in CFTR which leads to defective calcium transport,"Answer: B: Abnormality in CFTR which leads to defective calcium transport Explanation: Cystic fibrosis is an autosomal recessive disorder of ion transport in epithelial cells that affect fluid secretion in exocrine glands and the epithelial lining of the respiratory, gastrointestinal and reproductive tracts. Cystic fibrosis occurs due to mutation in the CFTR gene. CFTR was initially recognized as chloride conductance channel but it is now recognized that CFTR can regulate multiple ion channels and cellular processes. These include : Chloride channels Potassium channels Gap junction channels Bicarbonate ions" "This is a real-world medical entrance exam question, please give the true answer based on the question and selection. Topic name: None, Subject name: Pediatrics","Treatment of choice in childhood thyrotoxicosis : Select and explain. A. Radio Iodine, B. Lugols Iodine, C. Carbimazole, D. Surgery",C: Carbimazole,"Answer: C: Carbimazole Explanation: Most pediatric endocrinologists recommend initial medical therapy using antithyroid drugs rather than radioiodine or subtotal thyroidectomy. The 2 antithyroid drugs in widest use are methimazole and propylthiouracil." "This is a real-world medical entrance exam question, please give the true answer based on the question and selection. Topic name: AIIMS 2018, Subject name: Microbiology","A child with 10 days abdominal pain presented to OPD. Stool microscopy was done which showed the given findings. What is the DOC for the disease caused by the given organism? Select and explain. A. Albendazole, B. Mebandazole, C. Praziquintal, D. Pyrantelpamoate",C: Praziquintal,"Answer: C: Praziquintal Explanation: In the given picture the egg contains polar filaments and hooklets and the egg is non bile stained so this is the egg of H.nanan D.O.C- praziquintal" "This is a real-world medical entrance exam question, please give the true answer based on the question and selection. Topic name: None, Subject name: Radiology","The skin of a patient undergoing radiotherapy will show after 2 weeks? Select and explain. A. Erythema, B. Depigmentation, C. Radiation induced carcinomas, D. No change",A: Erythema,"Answer: A: Erythema Explanation: None" "This is a real-world medical entrance exam question, please give the true answer based on the question and selection. Topic name: None, Subject name: Gynaecology & Obstetrics","Which of the following histories is not an indication to perform oral glucose tolerance test to diagnose gestational diabetes mellitus? Select and explain. A. Previous Eclampsia, B. Previous Congenital anomalies in the fetus, C. Previous Unexplained fetal loss, D. Polyhydramnios",A: Previous Eclampsia,"Answer: A: Previous Eclampsia Explanation: Indications for performing GCT: All those conditions in which there is risk of having diabetes. On the basis of risk factors females are categorised into 3 category: Low risk All of the folowing: Member of an ethnic group with a low prevalence of GDM No known diabetes in first degree relatives Age <25 years Weight normal before pregnancy Weight of previous baby normal at birth No history of abnormal glucose metabolism No H/o poor obstetrical outcome Average risk One or more of the following: Member of an ethnic group with a high prevalence of GDM Diabetes in a first degree relative Age > 25 years Overweight before pregnancy Weight high at birth (previous baby) High risk Marked obesity Strong family history of type II DM Previous history of GDM impaired glucose metabolism or glucosouria Unexplained stillbirth H/o previous congenitally malformed baby" "This is a real-world medical entrance exam question, please give the true answer based on the question and selection. Topic name: None, Subject name: Dental","For intrusive luxation injuries, fixation period is about: Select and explain. A. 2 weeks only, B. 4 weeks, C. 4 weeks with semi rigid and 2 week with rigid, D. 6 to 8 weeks",B: 4 weeks,"Answer: B: 4 weeks Explanation: Intrusion Most severe form of luxation injuries. Displacement is seen. TOP is positive. Mobility is not seen.  High pitched metallic sound on percussion is heard.  Splinting should be done for 4 weeks." "This is a real-world medical entrance exam question, please give the true answer based on the question and selection. Topic name: None, Subject name: Ophthalmology","Interruption of the optic chiasm will lead to: Select and explain. A. Bitemporal hemianopia, B. Binasal hemianopia, C. Homonymous hemianopia, D. Normal vision",A: Bitemporal hemianopia,"Answer: A: Bitemporal hemianopia Explanation: Answer- A. Bitemporal hemianopiaInterruption of the optic chiasma will lead to bitemporal homonymous hemianopia because optic chiasma contains crossed over meclial fibers from both optic nerves, which are responsible for temporal Jield of vision.""Hemianopia denotes loss of half of the field of vision. The commonest clinical form is homonymous hemianopia, in which the right or left half of the binocular field of vision is lost, owing to loss of the temporal half of one field and lhe nasal half of the other. This condition is due to a lesion situated in any pa of the visual paths from the chiasma to the occipital lobe." "This is a real-world medical entrance exam question, please give the true answer based on the question and selection. Topic name: None, Subject name: Dental","The function of posterior palatal seal is: Select and explain. A. To aid in balanced occlusion, B. To aid in insertion and removal of complete denture, C. To ensure a complete seal thus helping in retention of a denture, D. Retention of mandibular denture by sealing its posterior margin.",C: To ensure a complete seal thus helping in retention of a denture,"Answer: C: To ensure a complete seal thus helping in retention of a denture Explanation: None" "This is a real-world medical entrance exam question, please give the true answer based on the question and selection. Topic name: None, Subject name: Dental","All of the following are true about property of polymethyl methacrylate except: Select and explain. A. KHN 18-20, B. chemical stable if heated till 260 degree F, C. Modulous of elasticity is 2.4 Gpa., D. Tensile strength 29Mpa.",D: Tensile strength 29Mpa.,"Answer: D: Tensile strength 29Mpa. Explanation: It is a hard resin with a Knoop hardness number of la to 20 KHN. It has a tensile strength approximately 60 MPa a density of 1.19 g/ cm3, and a modulus of elasticity of approximately 2.4 GPa. It is chemically stable to heat below 125°C (About 260°F), softens at 125°C, and can be molded as a thermoplastic material." "This is a real-world medical entrance exam question, please give the true answer based on the question and selection. Topic name: None, Subject name: Dental","A lateral sliding flap done on a tooth with roots denuded for 6 years will show Select and explain. A. PDL fibres attached to cementum, B. PDL fibres attached to dentin, C. Epithelium adjacent to previously denuded root surface, D. Co adaptation to the remaining fibres",C: Epithelium adjacent to previously denuded root surface,"Answer: C: Epithelium adjacent to previously denuded root surface Explanation: None" "This is a real-world medical entrance exam question, please give the true answer based on the question and selection. Topic name: AIIMS 2017, Subject name: Forensic Medicine","A person is criminally not responsible for his actions if at the time of doing it by reason of unsoundness of mind, in incapable of knowing the nature of the act or that he is doing what is either wrong or contrary to the law. Select and explain. A. McNaughten rule, B. Curren's rule, C. Durham's rule, D. Irresistible impulse test",A: McNaughten rule,"Answer: A: McNaughten rule Explanation: Mc Naughten rule states that person is criminally not responsible for his actions if he is unsound by mind and incapable of knowing the nature of the act. Curren's rule states the person do not have any capacity to regulate the conduct to the requirements of the law as a result of mental disease. Durham's rule is person's unlawful act is the product of mental disease or mental defect. Irresistsble impulse test states that even if the person knows that the nature and quality of the act and he Is wrong because the free agency of his will has been destroyed by mental disease." "This is a real-world medical entrance exam question, please give the true answer based on the question and selection. Topic name: None, Subject name: Dental","Core built up on a molar tooth should preferably be done with a chemically cured composite as: Select and explain. A. Light cure resin lacks sufficient strength for a core, B. Core made with a light cure resin would be too opaque and esthetically unacceptable, C. core made with a light cure resin would be too brittle and would fracture easily, D. Its lower viscosity allows free flow of resin also around pins and post",D: Its lower viscosity allows free flow of resin also around pins and post,"Answer: D: Its lower viscosity allows free flow of resin also around pins and post Explanation: None" "This is a real-world medical entrance exam question, please give the true answer based on the question and selection. Topic name: None, Subject name: Surgery","In dislocation of the jaw, displacement of the articular disc beyond the articular tubercle of the temporomandibular joint results from spasm or excessive contraction of the following muscle? Select and explain. A. Buccinator, B. Lateral pterygoid, C. Masseter, D. Temporalis",B: Lateral pterygoid,"Answer: B: Lateral pterygoid Explanation: None" "This is a real-world medical entrance exam question, please give the true answer based on the question and selection. Topic name: None, Subject name: Medicine","40-year male patient who was a known smoker presents in AIIMS OPD with fever, latigue, jaundice, yellow colored urine with clay colored stools. For the past few days he had developed aversion to cigarette smoking. On examination scleral icterus was present. LFT are:Total Bill18.5Direct Bill7.5SGOT723SGPT812What investigations will you send for ruling out acute viral hepatitis? Select and explain. A. HBsAg, IgM antiHBC, AntiHCV AntiHEV, B. AntiHAV, HBsAg, IgM antiHBc, AntiHCV, C. HBsAg, IgM antiHBC,AntiHDV AntiHCV AntiHEV, D. AntiHAV IgMantiHBC, AntiHCV, AntiHEV.","B: AntiHAV, HBsAg, IgM antiHBc, AntiHCV","Answer: B: AntiHAV, HBsAg, IgM antiHBc, AntiHCV Explanation: Ans: B. AntiHAV, HBsAg, IgM antiHBc, AntiHCVRef: Harrison, I8't' ed., pg. 328, 2549-2552, 2550t, 2551tHB- sAgIgM Anti- HAVIgM Anti- HBcAnti-HCVDiagnostic Interpretation+ + Acute hepatitis B Chronic hepatitis B++ Acute hepatitis A superim? posed on chronic hepatitis B+++ Acute hepatitis A and B--+ Acute hepatitis A--++ Acute hepatitis A and B (HBsAg below detection threshold) + Acute hepatitis B (HBsAg below detection threshold) Acute hepatitis C" "This is a real-world medical entrance exam question, please give the true answer based on the question and selection. Topic name: None, Subject name: Radiology","Which of these is not a pa of catatonia? Select and explain. A. Akathisia, B. Ambivalence, C. Ambitendency, D. Akinesia",A: Akathisia,"Answer: A: Akathisia Explanation: Ans: A. Akathisia(Ref Kaplan and Sadock 11/e p329, 871; Niraj Ahuja 7/e p59).Akathisia:Not a pa of catatonia.Characterized by a subjective & objective sense of restlessness, anxiety & agitation.Ambitendency:Example of negativism.Considered an ambivalence form.Ambivalence:Sign of Schizophrenia." "This is a real-world medical entrance exam question, please give the true answer based on the question and selection. Topic name: None, Subject name: Medicine","Thirty-eight children consumed eatables procured from a single source at a picnic party. Twenty children developed abdominal cramps followed by vomiting and watery diarrhea 6-10 hours after the party. The most likely etiology for the outbreak is: Select and explain. A. Rotavirus infection, B. Enterotoxigenic E.coli infection, C. Staphylococcal toxin, D. Clostridium perfringens infection",D: Clostridium perfringens infection,"Answer: D: Clostridium perfringens infection Explanation: None" "This is a real-world medical entrance exam question, please give the true answer based on the question and selection. Topic name: None, Subject name: Skin","A young female presented with vaginal itching and green frothy genital discharge. Strawberry vagina is seen on examination. What w ill be the drug of choice? Select and explain. A. Doxycycline, B. Oral fluconazole, C. Metronidazole, D. Amoxicillin",C: Metronidazole,"Answer: C: Metronidazole Explanation: Answer- C. MetronidazoleHistory of vaginal itching and green frothy genital discharge in a young female with strawberry vagina on examinstion ishighly suggestive of Trichomoniasis. The drag of choice for Trichomoniasis is metronidazole.Treatment:DOC: Metronidazolea (tinidazole is also effective)All sexual paners must be treated concurently to prevent reinfection" "This is a real-world medical entrance exam question, please give the true answer based on the question and selection. Topic name: None, Subject name: Dental","A patient who has recently received a pair of complete denture complains of difficulty in swallowing. The reason for this is most likely to be: Select and explain. A. Excessive vertical dimension of occlusion with resulting decrease in interacclusal distance, B. InteroccLusal distance of more than normal, C. Alveolar ridge with bony spicules, D. Candida albicans colonizing the mouth causing patient discomfort",A: Excessive vertical dimension of occlusion with resulting decrease in interacclusal distance,"Answer: A: Excessive vertical dimension of occlusion with resulting decrease in interacclusal distance Explanation: None" "This is a real-world medical entrance exam question, please give the true answer based on the question and selection. Topic name: None, Subject name: Dental","Hardness of stone surface is affected by? Select and explain. A. Impression wax, B. Impression compound, C. Hydrocolloid impression material, D. Rubber based material",C: Hydrocolloid impression material,"Answer: C: Hydrocolloid impression material Explanation: Colloidal systems such as agar and alginate retard the setting of gypsum products. If these materials are in contact with CaSO4·½H2O during setting, a soft, easily abraded surface is obtained. Accelerators such as potassium sulfate are added to improve the surface quality of the set CaSO4·2H2O against agar or alginate. These colloids do not retard the setting by altering the solubility ratio of the hemihydrate and dihydrate forms, but rather by being adsorbed on the hemihydrate and dihydrate nucleation sites, thus interfering in the hydration reaction. The adsorption of these materials on the nucleating sites retards the setting reaction more effectively than adsorption on the calcium sulfate hemihydrate. Craig’s RESTORATIVE DENTAL MATERIALS E 14th P: 255" "This is a real-world medical entrance exam question, please give the true answer based on the question and selection. Topic name: None, Subject name: Surgery","Cleft in facial region; Select and explain. A. Non fusion of ectodermal process, B. Absence or deficiency of mesodermal mass or other failure to penetrate ectodermal grooves, C. Covering of mesoderm on both sides by ectodermal process, D. Failure of endodermal structures to fuse in midline",B: Absence or deficiency of mesodermal mass or other failure to penetrate ectodermal grooves,"Answer: B: Absence or deficiency of mesodermal mass or other failure to penetrate ectodermal grooves Explanation: None" "This is a real-world medical entrance exam question, please give the true answer based on the question and selection. Topic name: None, Subject name: Dental","The upper denture falls when the patient opens his mouth wide. This is due to: Select and explain. A. Thick labial flange, B. Over-extended borders, C. Thick disto-buccal flange, D. Poor peripheral seal",C: Thick disto-buccal flange,"Answer: C: Thick disto-buccal flange Explanation: None" "This is a real-world medical entrance exam question, please give the true answer based on the question and selection. Topic name: None, Subject name: Dental","Overload of denture base is caused by: Select and explain. A. Small bases, B. Too large bases, C. Small teeth, D. Large teeth",A: Small bases,"Answer: A: Small bases Explanation: None" "This is a real-world medical entrance exam question, please give the true answer based on the question and selection. Topic name: None, Subject name: Dental","If during access opening of a maxillary incisor perforation occurs on labial surface just above CEJ, reason is? Select and explain. A. Inability to assess the lingual inclination of crown to root, B. Inability to assess the facial inclination of crown to root, C. All of the above, D. None of the above",A: Inability to assess the lingual inclination of crown to root,"Answer: A: Inability to assess the lingual inclination of crown to root Explanation: None" "This is a real-world medical entrance exam question, please give the true answer based on the question and selection. Topic name: None, Subject name: Dental","For periodontal patients, the most frequently recommended tooth brushing technique is: Select and explain. A. Scrub technique, B. Roll technique, C. Sulcular technique, D. Fones technique",C: Sulcular technique,"Answer: C: Sulcular technique Explanation: None" "This is a real-world medical entrance exam question, please give the true answer based on the question and selection. Topic name: AIIMS 2018, Subject name: Pathology","Which of the following is true regarding blood transfusion of packed RBC? Select and explain. A. Should be staed within 4 hours of receiving it from blood bank, B. Should be completed within 4 hours of receiving from blood bank, C. Wait till the patient is stable then transfuse, irrespective of any timing, D. Should be completed within 6 hours of receiving from blood bank",B: Should be completed within 4 hours of receiving from blood bank,"Answer: B: Should be completed within 4 hours of receiving from blood bank Explanation: 18-19 G needle is used for blood transfusion 170-180 u pore filter is used Packed RBC's are stored at a temperature of 2-6? C. Hence, they should be rewarmed before use. Rewarming should be done within 30 minutes of collecting blood from blood bank Transfusion should be completed within 4 hrs of taking the blood from the blood bank FFP and cryoprecipitate are stored at -18 to -30? C. Transfusion with these factors should be staed ASAP as the factors are labile to heat and completed in 20 minutes" "This is a real-world medical entrance exam question, please give the true answer based on the question and selection. Topic name: None, Subject name: Dental","Endontically treated posterior teeth are more susceptible to fracture than untreated one because: Select and explain. A. Loss of moisture., B. Hollow canals., C. Loss of coronal structure., D. All of the above.",C: Loss of coronal structure.,"Answer: C: Loss of coronal structure. Explanation: None" "This is a real-world medical entrance exam question, please give the true answer based on the question and selection. Topic name: None, Subject name: Forensic Medicine","The following system of depiction of 32 teeth is in accordance with which system?Permanent TeethUpper RightUpper Left1234567891011121314151632313029282726252423222120191817Lower RightLower Left Select and explain. A. Universal system, B. Palmer's system, C. Haderup system, D. Diagrammatic depiction",A: Universal system,"Answer: A: Universal system Explanation: Ans: A. Universal system(Ref: Reddy 341e p96-97, 33/e p99-100).The given system of depiction of the 32 teeth is in accordance with Universal system.Universal system:Teeth are numbered 1 to 16 from upper right to upper left, and 17 to 32 from lower left to lower right.This follows the plan advocated by American and International Society of Forensic Odontology." "This is a real-world medical entrance exam question, please give the true answer based on the question and selection. Topic name: None, Subject name: Medicine","Which of the following has intracellular receptor - Select and explain. A. Glucagon, B. Insulin, C. Epinephrine, D. Thyroxine",D: Thyroxine,"Answer: D: Thyroxine Explanation: Answer- D. ThyroxineClassic hormones that use intracellular receptors include thyroid and steroid hormones." "This is a real-world medical entrance exam question, please give the true answer based on the question and selection. Topic name: None, Subject name: Surgery","Lidocaine is used more commonly in dentistry because : Select and explain. A. Causes less depression of CNS., B. Causes less cardiovascular collapse., C. Causes lesser incidence of allergic reactions., D. Is 50 times more potent then procaine.",C: Causes lesser incidence of allergic reactions.,"Answer: C: Causes lesser incidence of allergic reactions. Explanation: None" "This is a real-world medical entrance exam question, please give the true answer based on the question and selection. Topic name: None, Subject name: Dental","Negative rake angle means: Select and explain. A. Rake face is ahead of radial line., B. Rake face coincides with radial line., C. Rake face is behind the radial line., D. None.",A: Rake face is ahead of radial line.,"Answer: A: Rake face is ahead of radial line. Explanation: None" "This is a real-world medical entrance exam question, please give the true answer based on the question and selection. Topic name: None, Subject name: Dental","Lateral pedicle flap is used for Select and explain. A. Increasing the width of attached gingiva, B. Recession coverage, C. Curettage, D. Vestibular deepening",B: Recession coverage,"Answer: B: Recession coverage Explanation: None" "This is a real-world medical entrance exam question, please give the true answer based on the question and selection. Topic name: None, Subject name: Anatomy","Anterior division of mandibular nerve has: Select and explain. A. One sensory and all motor branches, B. One motor and all sensory branches, C. All motor branches, D. All sensory branches",A: One sensory and all motor branches,"Answer: A: One sensory and all motor branches Explanation: Key Concept: All the branches of anterior division of the mandibular nerve are motor except buccal nerve which is sensory. Reference: TEXTBOOK OF ANATOMY HEAD, NECK AND Brain, Volume III, second edition page no 140" "This is a real-world medical entrance exam question, please give the true answer based on the question and selection. Topic name: None, Subject name: Microbiology","Transfusion-associated malaria has a shoer incubation period because of the presence in blood of: Select and explain. A. Trophozoites, B. Sporozoites, C. Female gametocyte, D. Merozoites",A: Trophozoites,"Answer: A: Trophozoites Explanation: Ans: A. Trophozoites(Ref: Paniker's 7/e p77, 6/e p69; Harrison 19/e p1374)Transfusion associated malaria has a shoer incubation period because of presence of trophozoites in blood.Malaria can be transmitted by blood transfusion, needle-stick injury, sharing of needles by infected injection drug users, or organ transplantation.The incubation period in these settings is often sho because there is no pre-erythrocytic stage of development.Parasites may remain ble in blood bank for 1-2 weeks.As this condition is induced by direct infection of red cells by the merozoites, pre-erythrocytic schizogony and hypnozoits are absent.Relapse does not occur and incubation period is sho." "This is a real-world medical entrance exam question, please give the true answer based on the question and selection. Topic name: None, Subject name: Medicine","A boy presented with a history of fever, headache and vomiting since 3 days and disorientation since I day. On examination, neck rigidity was positive. He also had 1 episode of generalized tonic clonic seizures (GTC) and then he became unconscious. CECT was found to be normal. CSF examination reveals a cell count of 300 cells/mm3 (polymorphs: 50-70%), protein 70 mg/dL, sugar 50 mg/dL (blood: 95 mg/dL). Which of the following is the most likely diagnosis? Select and explain. A. Pyogenic meningitis, B. Tubercular meningitis, C. Herpes encephalitis, D. Cerebral malaria",A: Pyogenic meningitis,"Answer: A: Pyogenic meningitis Explanation: Ans. a. Pyogenic meningitis (ReJ: Harrison l9/e p887, 891, llll, l8/e p1029, 1692, 3414)The classic CSF abnormalities in bacterial meningitis are:Elevated opening pressure (> 180 mm H2O in 90%)Polymorphonuclear leakocytosis (>100 cells/ul in 90%)Decreased glucose concentration (<2.2 mmol/L or <40 mg/dl and or CSF/serum glucose ration <0.4 in 60%)Elevated protein concentration (>45 mg/dl)" "This is a real-world medical entrance exam question, please give the true answer based on the question and selection. Topic name: None, Subject name: Dental","Tooth most frequently requiring RCT is: Select and explain. A. Mandibular 1st molar, B. Maxillary 1st molar, C. Mandibular 2nd premolar, D. Maxillary 2nd premolar",A: Mandibular 1st molar,"Answer: A: Mandibular 1st molar Explanation: The most common tooth to be affected is mandibular first molar. Teeth most frequently requiring RCT is mandibular first molar." "This is a real-world medical entrance exam question, please give the true answer based on the question and selection. Topic name: None, Subject name: Social & Preventive Medicine","A study finds no significant association between two variables but truly there exists a difference. What type of error is this? Select and explain. A. Type I error, B. Type II error, C. Random error, D. Systematic error",B: Type II error,"Answer: B: Type II error Explanation: Ans: B. Type II error(Ref: High Yield Biostatistics/p46)Type I ErrorType II ErrorThe null hypothesis is true but rejected (False positive)degProbability of type I error is given by p valuedegSignificance (alpha) level is the maximum tolerable probability of type I errorsdegKeep type I error to be minimum; then results are declared to be statistically significantNull hypothesis is not false but is not rejected/accepted (false negative)degProbability of type II error is given by betadeg" "This is a real-world medical entrance exam question, please give the true answer based on the question and selection. Topic name: None, Subject name: Dental","To prepare a cavity on a mandibular first molar in a child we should give: Select and explain. A. Mental nerve block, B. Lingual nerve block, C. Inferior alveolar nerve block and long buccal nerve block, D. Long buccal nerve block",C: Inferior alveolar nerve block and long buccal nerve block,"Answer: C: Inferior alveolar nerve block and long buccal nerve block Explanation: None" "This is a real-world medical entrance exam question, please give the true answer based on the question and selection. Topic name: None, Subject name: Gynaecology & Obstetrics","While performing Burch operation there was significant bleeding and pooting of blood in the space of Retzius. The source of bleeding cannot be visualized. What is the next step in the management? Select and explain. A. Call vascular stlrgeon, B. Give a generalized suture in bleeding area, C. Lift endopelvic fascia by putting fingers in vagina, D. Placing surgical drain",C: Lift endopelvic fascia by putting fingers in vagina,"Answer: C: Lift endopelvic fascia by putting fingers in vagina Explanation: Ans. C. Lift endopelvic fascia by putting fingers in vaginaBurch colposuspension (retropubic urethropexy) involves the attachment of the fascia at the level of the bladder neck to the iliopectineal ligament (Cooper's ligantent).lt is an abdorninally perfbrmed surgery for stress urinary incontinence ." "This is a real-world medical entrance exam question, please give the true answer based on the question and selection. Topic name: None, Subject name: Radiology","Radiation causes cell death by: Select and explain. A. Charring of nucleoproteins, B. Destroying their mitochondria, C. Ionization, D. Disruption of cytosol",C: Ionization,"Answer: C: Ionization Explanation: None" "This is a real-world medical entrance exam question, please give the true answer based on the question and selection. Topic name: None, Subject name: Anaesthesia","A patient of head injury is intubated and ventilated. The ideal mode of ventilation in him would be: Select and explain. A. CMV, B. CPAP, C. AMV, D. SIMV",A: CMV,"Answer: A: CMV Explanation: Ans. a. CMVControlled Mechanical Ventilation (CMV)The ventilator delivers a present number of breathes/min of a preset volume." "This is a real-world medical entrance exam question, please give the true answer based on the question and selection. Topic name: AIIMS 2018, Subject name: Medicine","All of the following statements about insulin Afreeza are true except Select and explain. A. Inhalational preparation of insulin, B. Not a substitute of injectable insulin, C. Needs to be given in multiple doses, D. Used in combination with sho acting insulin",D: Used in combination with sho acting insulin,"Answer: D: Used in combination with sho acting insulin Explanation: AFREEZA insulin : Inhalational preparation of insulin. Sho and fast acting insulin Used incombination with long acting insulin. Needs to be given in multiple dose. Should be used in combination with a long-acting insulin. S/E- Cough and throat irritation. It should not be used in individuals who smoke and COPD" "This is a real-world medical entrance exam question, please give the true answer based on the question and selection. Topic name: None, Subject name: Anatomy","The lateral lingual swellings and tuberculum impar give rise to: Select and explain. A. Anterior 1/3 of tongue, B. Anterior 2/3 of tongue, C. Posterior 1/3 of tongue, D. Posterior 2/3 of tongue",B: Anterior 2/3 of tongue,"Answer: B: Anterior 2/3 of tongue Explanation: None" "This is a real-world medical entrance exam question, please give the true answer based on the question and selection. Topic name: None, Subject name: Dental","Impression compound has which of the following characteristic property? Select and explain. A. Low thermal conductivity, B. High flow property, C. Degradation in presence of moisture, D. Remain distortion free up to 72 hrs. pouring can be safely delayed",A: Low thermal conductivity,"Answer: A: Low thermal conductivity Explanation: None" "This is a real-world medical entrance exam question, please give the true answer based on the question and selection. Topic name: None, Subject name: Dental","Craters in interseptal bone are best eliminated by Select and explain. A. Gingivectomy, B. Supragingival curettage, C. Interseptal massage with stimulators, D. Osseous recontouring",D: Osseous recontouring,"Answer: D: Osseous recontouring Explanation: None" "This is a real-world medical entrance exam question, please give the true answer based on the question and selection. Topic name: None, Subject name: Gynaecology & Obstetrics","In pregnancy, the most common cause of transient- diabetes insipidus is: Select and explain. A. Severe preeclampsia, B. Hydramnios, C. Multiple pregnancy, D. IUGR",A: Severe preeclampsia,"Answer: A: Severe preeclampsia Explanation: Diabetes insipidus can be caused by: Deficiency of antiduretic hormone               Resistance of ADH action                     ↓                                                                       ↓ Central diabetes insipidus                           Nephrogenic diabetes insipidus. A transient form of DI occurs during pregnancy due to: – Excessive placental production of vasopressinase – Decreased hepatic clearance due to abnormal liver function there in case of: Preeclampsia Fatty liver Hepatitis. Approximately 60% of women with previously known DI worsen, 20% improve and 20% do not change during pregnancy. Worsening is attributed to excessive placental vasopressinase production. Some females with DI who develop placental insufficiency show DI improvement, which is attributed to decreased vasopressinase production by the damaged placenta. Symptoms: Polyuria (4-15 liters/day) Intense thirst particularly for ice cold fluids. Diagnosis: is confirmed by water deprivation test. Treatment: of choice intranasal L-deamino 8D arginine vasopressin (DDAVP) which is a synthetic analogue of ADH and is resistant to vasopressinase." "This is a real-world medical entrance exam question, please give the true answer based on the question and selection. Topic name: None, Subject name: Dental","A 12-years-old child suffers from an avulsed tooth following trauma. The best medium which to carry in the tooth to the dental clinic is: Select and explain. A. Buccal vestibule, B. Milk, C. Water, D. Cotton",B: Milk,"Answer: B: Milk Explanation: None" "This is a real-world medical entrance exam question, please give the true answer based on the question and selection. Topic name: None, Subject name: Surgery","Which of the following is used in the management of haemophillic patient? Select and explain. A. Tranxemic acid, B. Acetic acid, C. Ascorbic acid, D. Palmitonic acid",A: Tranxemic acid,"Answer: A: Tranxemic acid Explanation: None" "This is a real-world medical entrance exam question, please give the true answer based on the question and selection. Topic name: None, Subject name: Dental","After a class V amalgam restoration, patient experiences pain, which may be due to? Select and explain. A. Dentin (Hyper) sensitivity, B. Irreversible pulpal damage, C. Injury to adjacent gingiva, D. Any of the above",D: Any of the above,"Answer: D: Any of the above Explanation: None" "This is a real-world medical entrance exam question, please give the true answer based on the question and selection. Topic name: None, Subject name: Dental","All of the following influence the efficiency of bur, except: Select and explain. A. Neck diameter, B. Length and diameter of bur, C. Height of taper of bur, D. Spiral angle and cross-cuts of bur",C: Height of taper of bur,"Answer: C: Height of taper of bur Explanation: None" "This is a real-world medical entrance exam question, please give the true answer based on the question and selection. Topic name: None, Subject name: Medicine","A man comes with aphasia. He is unable to name things and repetition is poor. However comprehension, fluency and understanding written words is unaffected. He is probably suffering from: Select and explain. A. Anomic aphasia, B. Broca's aphasia, C. Transcoical sensory aphasia, D. Conduction aphasia",D: Conduction aphasia,"Answer: D: Conduction aphasia Explanation: Ans: D. Conduction aphasia(Ref Harrison 19/e p 1 76, 18/e p302)Typical case of conduction aphasia with normal comprehension & fluency, but impaired naming & repetition.Conduction aphasia:Due to damaged arcuate fasciculus.Arcuate fasciculus:Connection between Wernicke's (language comprehension) & Broca's (Language production) areas.Hence, connection lost.Clinical features:Hallmark finding - Inability to repeat words.Comprehension & language expression intact.Inability to transfer understood word to Broca 's area to be expressed.Results in meaningful fluent speech & relatively good comprehension but very poor repetition." "This is a real-world medical entrance exam question, please give the true answer based on the question and selection. Topic name: None, Subject name: ENT","All of the following are true regarding tonsillectomy in children except: Select and explain. A. Extracapsular approach is best for cold approach, B. Sleep apnea is an indication, C. Adenoids should also be removed if significantly involved, D. Cricothyroid region is high and anterior in children than adults",A: Extracapsular approach is best for cold approach,"Answer: A: Extracapsular approach is best for cold approach Explanation: Answer- A. Extracapsular approach is best for cold approachIntracapsular approach is prefened over exlracapsular approach in tonsillectomyfor cold opproach.'Intracapsular tonsillectomy with bipolar scissors results in less postoperative pain than extracapsular tonsillectomywith bipolar scissors in children age 5 to 19 years.Obstructive sleep apnea is the most common indicationfor T and A.'The most common indication for Tonsillectomy and Adenoidectomy (T and A) was for infectious etiology." "This is a real-world medical entrance exam question, please give the true answer based on the question and selection. Topic name: None, Subject name: Dental","Korffs fibres are seen in: Select and explain. A. Mantle dentin, B. Secondary dentin, C. Reactionary dentin, D. Indeed dermal tubules",A: Mantle dentin,"Answer: A: Mantle dentin Explanation: None" "This is a real-world medical entrance exam question, please give the true answer based on the question and selection. Topic name: None, Subject name: Surgery","ABCDE mnemonic is used for which disease? Select and explain. A. SCC, B. Melanoma, C. Basal Cell carcinoma, D. Verrucous carcinoma",B: Melanoma,"Answer: B: Melanoma Explanation: Answer- B. MelanomaABCDE rule: Asymmetry (one half of the mole doesn't match the other), Border irregularity, Color that is not uniform, Diameter greater than 6 mm (about the size of a pencil eraser), and Evolving size, shape or color" "This is a real-world medical entrance exam question, please give the true answer based on the question and selection. Topic name: None, Subject name: Dental","At what age root resorption start Select and explain. A. 5 years, B. 3 years, C. 6.5 years, D. 7 years",A: 5 years,"Answer: A: 5 years Explanation: 1st to resorb is Mandibular central incisor start resorbing at 5 years of age" "This is a real-world medical entrance exam question, please give the true answer based on the question and selection. Topic name: None, Subject name: Forensic Medicine","Boiled lobster syndrome is seen in poisoning of: Select and explain. A. Boric acid, B. HNO,, C. H,SO4, D. Phenol",A: Boric acid,"Answer: A: Boric acid Explanation: Ans: A. Boric acid(Ref: Principles of Clinical Toxicology 3/e p221).Boiled lobster syndrome is seen in poisoning of Boric acid.Features:Major symptom is erythema, desquamation and exfoliation.The skin of the patient looks like a 'boiled lobster'." "This is a real-world medical entrance exam question, please give the true answer based on the question and selection. Topic name: None, Subject name: Physiology","Sodium iodide sympoer is not present in Select and explain. A. Pituitary gland, B. Placenta, C. Parotid, D. Thyroid",A: Pituitary gland,"Answer: A: Pituitary gland Explanation: Ans: A. Pituitary gland Sodium iodide sympoer - Not present in the pituitary gland.Sodium-Iodide Sympoer (NIS) is present inThyroiddegSalivary glandsdegGastric mucosadegPlacentaoCiliary body of the eyedegChoroid plexusdegMammary glandsdegCeain cancers derived from these" "This is a real-world medical entrance exam question, please give the true answer based on the question and selection. Topic name: None, Subject name: Biochemistry","Fate of IDL(Intermediate Density Lipoprotein) in plasma Select and explain. A. 50% is taken up by liver, B. 50% is converted to LDL particle, C. Both 1 and 2, D. None of the above",C: Both 1 and 2,"Answer: C: Both 1 and 2 Explanation: Fate of IDL in plasma: 50% IDL particles → taken up by liver. Remaining 50% will convert to → LDL particle." "This is a real-world medical entrance exam question, please give the true answer based on the question and selection. Topic name: None, Subject name: Medicine","Nowadays, swine flu is in news, it is considered as Select and explain. A. Pandemic, B. Epidemic, C. Endemic, D. Seasonal variation",A: Pandemic,"Answer: A: Pandemic Explanation: None" "This is a real-world medical entrance exam question, please give the true answer based on the question and selection. Topic name: None, Subject name: Medicine","A 55-year-old male presents with renal failure. He gives a history of mild bone pains for the last 7 years. X-ray pelvis shows osteolytic lesions. Serum electrophoresis reveals a M spike. Peripheral blood evaluation showed rouleaux formation of RBCs with 35% plasma cells. Bone marrow examination showed a increase in plasma cells with aberrant antigen expression. The most likely diagnosis is: Select and explain. A. Multiple myeloma, B. Monoclonal gammopathy with unknown significance, C. Smouldering myeloma, D. Plasma cell leukemia",A: Multiple myeloma,"Answer: A: Multiple myeloma Explanation: Answer- A (Multiple myeloma)Malignant proliferation of plasma cells in the bone marrow results in the production of large number of complete and incomplete immunoglobulins." "This is a real-world medical entrance exam question, please give the true answer based on the question and selection. Topic name: None, Subject name: Anatomy","Anterior limit of infratemporal fossa is: Select and explain. A. Lateral pterygoid plate, B. Maxillary posterior wall, C. Pterygomaxillary fissure, D. Mastoid process",B: Maxillary posterior wall,"Answer: B: Maxillary posterior wall Explanation: Infratemporal fossa is an irregular space found below zygomatic arch. Boundaries: Anteriorly : Posterior surface of maxilla. Medially : Lateral pterygoid plate and pyramidal process of palatine bone. Laterally : Ramus of mandible. Superiorly : Infratemporal surface of greater wing of sphenoid." "This is a real-world medical entrance exam question, please give the true answer based on the question and selection. Topic name: AIIMS 2018, Subject name: Physiology","Which of the following is the diluting segment of kidney? Select and explain. A. PCT, B. Collecting duct, C. Ascending thick loop Henle, D. Descending loop of Henle",C: Ascending thick loop Henle,"Answer: C: Ascending thick loop Henle Explanation: Structure Function Glomerulus PCA Descending limb of loop of henle Ascending limb of loop of henle DCT Collecting duct Isotonic ultrafiltration Reabsorption of Na+ H2O, amino acids, HCO3,K+, Ca2+, Cl- Reabsorbs H2O into interstitium, impermeable to solute. Impermeable to H2O,loop diuretics acts here. Resorption of Na, K, Cl, k/a the diluting segment. Thiazide diuretics act here, removes 10% of Na+& Cl-, some H2O also removed. Aka concentrating segment, absorbs 10 - 12% of H2O under ADH, principle cells reabsorbs Na+& secretes K+ , intercalated cells reabsorbs K+ & secretes HCO-3." "This is a real-world medical entrance exam question, please give the true answer based on the question and selection. Topic name: None, Subject name: Medicine","Vitamin K dependent factor is Select and explain. A. II, B. III, C. IV, D. VI",A: II,"Answer: A: II Explanation: None" "This is a real-world medical entrance exam question, please give the true answer based on the question and selection. Topic name: None, Subject name: Radiology","The principal reason for placing an aluminium filter in the primary beam of radiation is to: Select and explain. A. Reduce exposure time, B. Decrease development time of the films, C. Reduce radiation to the skin of the patient, D. Obtain greater definition of the images of teeth",C: Reduce radiation to the skin of the patient,"Answer: C: Reduce radiation to the skin of the patient Explanation: Although an x-ray beam consists of a continuous spectrum of x-ray photon energies, only photons with sufficient energy to penetrate through anatomic structures and reach the image receptor (digital or film) are useful for diagnostic radiology. Low-energy photons that cannot reach the receptor contribute to patient risk but do no offer any benefit. Consequently, it is desirable to remove these low-energy photons from the beam. This removal can be accomplished in part by placing a metallic disk (filter) in the beam path.  A filter preferentially removes low-energy photons from the beam, while allowing high energy photons that are able to contribute to making an image to pass through.  Ref: Oral Radiology, Principles and Interpretation / Stuart C. White, Michael J. Pharoah - 7th ed - pg - 10" "This is a real-world medical entrance exam question, please give the true answer based on the question and selection. Topic name: None, Subject name: Surgery","Myelinated nerve fibres have all of the following properties except Select and explain. A. Conduction is slower in myelinated than in non-myelinated fibres, B. Current discharges at nodes of Ranvier, C. Outer layer is of lipids, D. Depolarisation occurs only at nodes of ranvier",A: Conduction is slower in myelinated than in non-myelinated fibres,"Answer: A: Conduction is slower in myelinated than in non-myelinated fibres Explanation: None" "This is a real-world medical entrance exam question, please give the true answer based on the question and selection. Topic name: None, Subject name: Physiology","Central wrapping is done by: Select and explain. A. Golgi tendon, B. Type 2 fibre, C. Flower spray ending, D. Annulospiral ending",D: Annulospiral ending,"Answer: D: Annulospiral ending Explanation: Primary sensory nerve fiber belongs to type Iα (Aα) nerve fiber. Each sensory (afferent) nerve fiber has two branches. One of the branches supplies the central portion of nuclear bag  fiber. The other branch ends in central portion of the nuclear chain fiber. These branches end in the form of rings around central portion of nuclear bag and nuclear chain fibers. Therefore, these nerve endings are called annulospiral endings." "This is a real-world medical entrance exam question, please give the true answer based on the question and selection. Topic name: None, Subject name: Anatomy","Golgi tendon organs used to detect ? Select and explain. A. Dynamic, B. Static, C. Tension of muscle, D. All",C: Tension of muscle,"Answer: C: Tension of muscle Explanation: Ans. C. Tension of muscleThe Golgi tendon organ (GTO) (also called Golgi organ, tendon organ, neurotendinous organ or neurotendinous spindle) is a proprioceptive sensory receptor organ that senses changes in muscle tension. It lies at the origins and inseion of skeletal muscle fibers into the tendons of skeletal muscle." "This is a real-world medical entrance exam question, please give the true answer based on the question and selection. Topic name: None, Subject name: Dental","Function of WHO collaborating centers: Select and explain. A. Collection, collation and dissemination of information, B. Functioning and forming new guidelines/definitions for diseases, C. Contact with grass root workers, D. Collaborate information mainly","A: Collection, collation and dissemination of information","Answer: A: Collection, collation and dissemination of information Explanation: None" "This is a real-world medical entrance exam question, please give the true answer based on the question and selection. Topic name: None, Subject name: Pediatrics","Vesicoureteric reflux is more common in – Select and explain. A. Newborn females, B. Older girls, C. Older boys, D. Only during pregnancy",A: Newborn females,"Answer: A: Newborn females Explanation: Clinical manifestations of VUR In primary reflux the average age ofpresentation is 2-3 years. Reflux is more common in females and it is usually detected earlier in females so we can say that reflux is present from birth but it is usually detected at 2-3 years of age but in feinales the age of detection is earlier. Features are - In primary reflux the patient usually presents with symptoms of pyelonephritis or cystitis. In secondary reflux manifestation of primary diseases are usually the presenting symptoms Reflux is the most common cause of pyelonephritis. It is seen in 30-50% of patients with pyelonephritis. It is present in over 75% of patients with radiological evidence of chronic pyelonephritis Diagnosis of Vesicoureteric reflux - The most useful process for conclusive diagnosis of reflux is micturating cystourethrogram. This will demonstrate the grade of reflux as well as urethral anatomy. The other useful technique for detecting vesicoureteric reflux is radionuclide imaging. It is extremaly sensitive at detecting reflux but do not demonstrate the anatomic detail seen with voiding cystourethro gram. So they are used for follow up after micturating cystourethrogram because they offer less radiation exposure." "This is a real-world medical entrance exam question, please give the true answer based on the question and selection. Topic name: None, Subject name: Biochemistry","A young man was on high protein diet and raw eggs. After 3 days he developed weakness. Blood investigation revealed hypoglycemia. Hypoglycemia is due to inhibition of which of the following enzymes Select and explain. A. Glucose 6 phosphatase, B. Glycogen phosphorylase, C. Pyruvate Carboxylase, D. Glucokinase",C: Pyruvate Carboxylase,"Answer: C: Pyruvate Carboxylase Explanation: Ans- c- Pyruvate Carboxylase.When gluconeogenesis is elevated due to low blood glucose levels, the activation by acetyl-CoA of pyruvate carboxylase, which catalyzes the conversion of pyruvate to oxaloacetate, paially alletes this problem, but in conditions such as starvation and untreated diabetes mellitus, ketone bodies are overproduced and cause ketosis." "This is a real-world medical entrance exam question, please give the true answer based on the question and selection. Topic name: AIIMS 2017, Subject name: Orthopaedics","A 20 year old man fell on an outstretched hand and later had pain in the anatomic snuff box area of hand. What is the deformity in this person. The x - ray after the fall is given below: Select and explain. A. Scaphoid - lunate fracture, B. Distal radius fracture, C. Ulnar fracture, D. Scaphoid fracture",D: Scaphoid fracture,"Answer: D: Scaphoid fracture Explanation: * This is a case of FOOSH (fall on out-stretched hand). X - ray image shows fracture of scaphoid.* Overall most common of FOOSH is colle's fracture. It is common in elderly.* Common in FOOSH occurring in young adult is scaphoid fracture * Common in FOOSH occurring in children is supracondylar of humerus * Pain on anatomical snuffbox signifies scaphoid fracture" "This is a real-world medical entrance exam question, please give the true answer based on the question and selection. Topic name: None, Subject name: Surgery","A 70 years old patient presents with a lesion on the cheek with raised, pearly borders and with telangiectasias on surface of the lesion. What is the likely diagnosis? Select and explain. A. Basal cell carcinoma, B. Squamous cell carcinoma, C. Was, D. Actinic keratosis",A: Basal cell carcinoma,"Answer: A: Basal cell carcinoma Explanation: Ans: A. Basal cell carcinoma(Ref: Harrison 19/c p500; Robbins 9/e p1157)Lesion on the cheek with raised, pearly borders and with telangiectasia on surface of the lesion is highly suggestive of basal cell carcinoma.Basal cell carcinomas:Present as pearly papules containing prominent dilated subepidermal blood vessels (telangiectasias).Arises from epidermal basal cells.Present as a small, slowly growing pearly nodule, often with touous telangiectatic vessels on its surface, rolled borders, and a central crust (nodular BCC).The occasional presence of melanin in this variant of nodular BCC (pigmented BCC) may lead to confusion with melanoma.Morpheaform (fibrosing), infiltrative, and micronodular BCC, the most invasive and potentially aggressive subtypes, manifest as solitary flat or slightly depressed, indurated whitish, yellowish, or pink scar-like plaques." "This is a real-world medical entrance exam question, please give the true answer based on the question and selection. Topic name: AIIMS 2017, Subject name: Social & Preventive Medicine","Crude Bih rate for a sub-center is 20. What the number of pregnant woman registered in sub-center? Select and explain. A. 80, B. 100, C. 110, D. 60",D: 60,"Answer: D: 60 Explanation: Population of a sub-centre =5000 Crude bih rate= 20 per 1000 mid-year population Therefore, no. of bihs= 20/1000 x 5000=100 Pregnancy wastage factor = 10% (i.e. not all pregnancies result in a live bih, and roughly 10% ANC women have a ""wastage"" of their pregnancy. Therefore, total ANC= 100+ =110 As a thumb rule, Minimum number of ANC that should be registered at a time= 50%. - Min. No. registered in the sub-centre= 110 x 50/100=55 ~ ~60" "This is a real-world medical entrance exam question, please give the true answer based on the question and selection. Topic name: AIIMS 2018, Subject name: Pathology","A 12 - year - old boy had a cut in his forearm 4 days ago. Now the bleeding has been stopped due to granulation tissue formation. While taking a skin biopsy a pa of the granulation tissue was also included in the specimen. The histology of granulation tissue is shown below. Which type of collagen is found in this granulation tissue? Select and explain. A. Type 1, B. Type 2, C. Type 3, D. Type 4",C: Type 3,"Answer: C: Type 3 Explanation: In wound healing, granulation tissue formation takes place 2 weeks - Predominantly, it is collagen type 3 >2 weeks - Collagen type 1" "This is a real-world medical entrance exam question, please give the true answer based on the question and selection. Topic name: None, Subject name: Gynaecology & Obstetrics","A 61 years old post-menopausal woman with a family history of ovarian cancer presents with pain abdomen. She is on hormone replacement therapy. An abdominal ultrasound revealed a smooth cyst in the right ovary. What should be done next? Select and explain. A. Observe and reassure the patient, B. Laparoscopic surgery to visualize the nature of the cyst, C. Drilling of cysts, D. Check CA-125 levels and advise regular follow-up if normal",D: Check CA-125 levels and advise regular follow-up if normal,"Answer: D: Check CA-125 levels and advise regular follow-up if normal Explanation: Answer-D. Check CA-125 levels and advise regular follow-up if normalThis is a post-menopausal patient with a simple (smooth) qtst in the ovary. Such a cyst doesn't needfuher investigations,but this patient has two risk factors for malignancy: Family history of ovarian tamor and history of Hormonal therapy.Hence, CA-125 levels should be evaluated and the RMI-I score (Risk of Malignancy index) calcalated before proceedingfor regular follow up. If RMI score exceeds 200,fuher imaging laparoscopic surgery will be needed in this patient." "This is a real-world medical entrance exam question, please give the true answer based on the question and selection. Topic name: None, Subject name: Pharmacology","A drug X was given continuous intravenous infusion at 1.6 mg/min. The clearance of the drug is 640 mL/min. With a half-life of 1.8 hours, what would be the steady state plasma concentration of drug? Select and explain. A. 0.002 mg/mL, B. 0.004 trig/mL, C. 2.88 mg/mL, D. 3.55 mg/mL",A: 0.002 mg/mL,"Answer: A: 0.002 mg/mL Explanation: Answer- A. 0.002 mg/mLCpss is directly propoional to the dose rate & inversely propoional to the clearance of drug.Cpss = Dose rate/clearanceDose rate= 1.6 mgiml; Clearance= 640 ml/minCpss = 1.61640 = 0.0025 mg/ml = 0.002 mg/ml" "This is a real-world medical entrance exam question, please give the true answer based on the question and selection. Topic name: None, Subject name: Gynaecology & Obstetrics","Which of these is seen in Asherman syndrome? Select and explain. A. Oligomenorrhea, B. Hypomenorrhea, C. Metromenorrhagia, D. Polymenorrhea",B: Hypomenorrhea,"Answer: B: Hypomenorrhea Explanation: Ans: B. Hypomenorrhea(Ref: Shaw's 16/e p250; Novaks 13/e p351)Hypomenorrhea:Seen in Asherman syndromeAsherman's syndrome:More common with secondary amenorrhea or hypomenorrhea.Causes:In patients with risk factors for endometrial or cervical scarring (history of uterine or cervical surgery), infections related to IUD use & severe pelvic inflammatory disease.Found in 39% patients undergoing hysterosalpingography with previous postpaum curettage.Rare cause: Infections (tuberculosis & schistosomiasis)." "This is a real-world medical entrance exam question, please give the true answer based on the question and selection. Topic name: AIIMS 2017, Subject name: Surgery","Most common cause of recurrent laryngeal nerve palsy is: Select and explain. A. Bronchogenic carcinoma, B. Thyroid surgery, C. Mediastinal tumors, D. Pancoast tumor",B: Thyroid surgery,"Answer: B: Thyroid surgery Explanation: ""Previous surgery, mostly thyroid surgery, is the most common cause of laryngeal nerve paralysis. Revision thyroidectomy bears a paicularly high risk for inferior and superior laryngeal nerve trauma."" - Surgery of Larynx and Trachea by Marc Remacle (2015)/ p271" "This is a real-world medical entrance exam question, please give the true answer based on the question and selection. Topic name: None, Subject name: Gynaecology & Obstetrics","A G2 P1+0+0 diabetic mother present at 32 weeks pregnancy, there is history of full term fetal demise in last pregnancy. Her vitals are stable, sugar is controlled and fetus is stable. Which among the following will be the most appropriate management? Select and explain. A. To induce at 38 weeks, B. To induce at 40 weeks, C. Cesarean section at 38 weeks, D. To wait for spontaneous delivery",A: To induce at 38 weeks,"Answer: A: To induce at 38 weeks Explanation: The most common time of IUD in a diabetic patient is last two weeks of pregnancy, since in this patient there is history of a full term demise as well, so logically speaking we should terminate her pregnancy at 38 weeks.This is what logic says, now let us see what references have to say- High risk gestational diabetes: History of stillbirth History of neonatal death History of fetal macrosomia Concomitant obesity and/or hypertension Development of oligohydramnios, polyhydramnios preeclampsia or fetal macrosomia Inadequate metabolic control with diet alone. “High risk gestational diabetic patients should have their labor induced when they reach 38 weeks with exception of those with a macrosomia fetus (Efw > 4000 g) who should be delivered by cesarean section because of the increased risk of shoulder dystocia”. Fernando Arias 3/e, p 449 Induction of labor: The indications are Diabetic women controlled on insulin (GDM or class B diabetes) are considered for induction of labor after 38 completed weeks Women with vascular complications (pre-eclampsia, IUGR) often require induction after 37 weeks." "This is a real-world medical entrance exam question, please give the true answer based on the question and selection. Topic name: None, Subject name: Surgery","Numbness on shaving after parotidectomy due to which nerve injury? Select and explain. A. Facial nerve, B. Auriculotemporal nerve, C. Greater auricular nerve, D. Occipital nerve",B: Auriculotemporal nerve,"Answer: B: Auriculotemporal nerve Explanation: Answer- B. Auriculotemporal nerve" "This is a real-world medical entrance exam question, please give the true answer based on the question and selection. Topic name: None, Subject name: Dental","According to RAMJFORD, occlusal interferences with nervous tension triggers which of the following habit: Select and explain. A. Bruxism, B. Thumb sucking, C. Tongue thrusting, D. All of the above",A: Bruxism,"Answer: A: Bruxism Explanation: None" "This is a real-world medical entrance exam question, please give the true answer based on the question and selection. Topic name: None, Subject name: Pediatrics","True about cow\'s milk are all except – Select and explain. A. Cow's milk contains 80% whey protein not casein, B. Cow milk has less carbohydrate than mothers milk, C. Has more K+ and Na+ than infant formula feeds, D. All of the above",A: Cow's milk contains 80% whey protein not casein,"Answer: A: Cow's milk contains 80% whey protein not casein Explanation: Whey protein constitutes 80% of the protein in human milk, while the main protein in cow's milk is casein Table : Compariso, of human milk and cow's milk" "This is a real-world medical entrance exam question, please give the true answer based on the question and selection. Topic name: None, Subject name: Dental","While performing biomechanical preparation, apical constriction is enlarged to 50 sized instrument. What would be the next step? Select and explain. A. Use reverse filing technique, B. Reinstrument with larger instrument 1 mm short of previous preparation, C. Fit a 50 mm cone and condense carefully, D. Fill with 40 sized cone",B: Reinstrument with larger instrument 1 mm short of previous preparation,"Answer: B: Reinstrument with larger instrument 1 mm short of previous preparation Explanation: None" "This is a real-world medical entrance exam question, please give the true answer based on the question and selection. Topic name: AIIMS 2017, Subject name: Anatomy","Root value of cremaster reflex is Select and explain. A. L1, L2, B. L2, L3, C. S1, S2, D. S3, S4","A: L1, L2","Answer: A: L1, L2 Explanation: In Men, Cremaster muscle & cremaster fascia form middle/2nd covering of spermatic cord. This muscle & Its associated fascia are supplied by genital branch of GENITOFEMORAL NERVE(L1 L2) Contraction of this muscle & resulting elevation of testis can be stimulated by reflex arc Gently touching the skin at & around the anterior aspect of superior pa of thigh stimulates sensory fibers in femoral branch of genito-femoral N/ Ilio-Inguinal N. These fibers enter spinal cord at L1 At L1, Sensory fibers stimulate motor fibers carried in genital branch of genito-femoral N, which results in contraction of cremaster muscle & elevation of testis. Cremasteric reflex is more active in children, tending to diminish with age. As with many reflexes, it may be absent in ceain neurological disorders. Although, it can be used for testing spinal cord function at level L1 in men, Its clinical use is limited." "This is a real-world medical entrance exam question, please give the true answer based on the question and selection. Topic name: None, Subject name: Anatomy","All of the following are true about venous drainage of esophagus except: Select and explain. A. Thoracic esophagus drains into the azygos vein, B. Esophageal veins drain into a submucosal plexus, C. The cervical esophagus drains directly into the right brachiocephalic vein, D. Lower esophageal veins anastomose with the left gastric vein",C: The cervical esophagus drains directly into the right brachiocephalic vein,"Answer: C: The cervical esophagus drains directly into the right brachiocephalic vein Explanation: Ans: C. The cervical esophagus drains directly into the right brachiocephalic vein(Ref: Gray's 411e p1111, 40Ie p950).Cervical pa of esophagus is drained into inferior thyroid vein, not the right brachiocephalic vein.Venous drainage of esophagus:Cervical pa is drained by inferior thyroid vein.Thoracic pa is drained by azygous and hemiazygous veins.Abdominal pa is drained by two venous channels, hemiazygous vein and left gastric vein." "This is a real-world medical entrance exam question, please give the true answer based on the question and selection. Topic name: AIIMS 2017, Subject name: Anaesthesia","A Patient in medical intensive care unit who is intubated ,suddenly removes the endotracheal tube. What should be done next? Select and explain. A. Sedate and reintubate, B. Make him sit and do physiotherapy, C. Assess the patient and give bag and mask ventilation and look for spontaneous breathing, D. Give bag and mask ventilation and intubate",C: Assess the patient and give bag and mask ventilation and look for spontaneous breathing,"Answer: C: Assess the patient and give bag and mask ventilation and look for spontaneous breathing Explanation: As the ET tube was removed by the patient by himself, it indicates adequate spontaneous breathing and muscle power. Hence assess the patient first, give bag and mask ventilation and look for spontaneous breathing" "This is a real-world medical entrance exam question, please give the true answer based on the question and selection. Topic name: None, Subject name: Radiology","Disadvantages of grids are: Select and explain. A. Skin exposure of patient, B. Increase exposure time, C. Doesn't improve image, D. Overall density of radiograph is increased",B: Increase exposure time,"Answer: B: Increase exposure time Explanation: None" "This is a real-world medical entrance exam question, please give the true answer based on the question and selection. Topic name: None, Subject name: Dental","Most commonly missing tooth is Select and explain. A. Upper 3rd molar, B. Lower 3rd molar, C. Upper lateral incisor, D. Lower 2nd premolar",B: Lower 3rd molar,"Answer: B: Lower 3rd molar Explanation: None" "This is a real-world medical entrance exam question, please give the true answer based on the question and selection. Topic name: None, Subject name: Pathology","True about enameloma Select and explain. A. A globule like structure usually found on root due to misplaced ameloblast, B. True neoplasm of ameloblast, C. Tumor resembling enamel present on gingiva, D. Tumor of ameloblast which fail to differentiate terminally",A: A globule like structure usually found on root due to misplaced ameloblast,"Answer: A: A globule like structure usually found on root due to misplaced ameloblast Explanation: None" "This is a real-world medical entrance exam question, please give the true answer based on the question and selection. Topic name: None, Subject name: Dental","Chewing stroke starts from? Select and explain. A. CO, B. CR, C. Maximum intercuspation, D. CR or CO depends on patient to patient",D: CR or CO depends on patient to patient,"Answer: D: CR or CO depends on patient to patient Explanation: None" "This is a real-world medical entrance exam question, please give the true answer based on the question and selection. Topic name: None, Subject name: Dental","Molecule with permanent dipole Select and explain. A. Oxygen, B. Helium, C. Liquid nitrogen, D. Water",D: Water,"Answer: D: Water Explanation: None" "This is a real-world medical entrance exam question, please give the true answer based on the question and selection. Topic name: AIIMS 2017, Subject name: Pharmacology","A diabetic patient presented with uncontrolled blood sugar level. He has history of pancreatitis and family history of urinary bladder carcinoma. He does not want to take injectable drugs. Which of the following drug can be added to control his blood sugar? Select and explain. A. Liraglutide, B. Sitagliptin, C. Canagliflozin, D. Pioglitazone",C: Canagliflozin,"Answer: C: Canagliflozin Explanation: Linaglutide(-ide) is a peptide, so not given orally. It is a GLP 1(incretin) analogue and it can cause pancreatitis.Sitagliptin is a DPP 4(dipeptidyl peptidase 4) inhibitor; can cause pancreatitis.Pioglitazone(PPAR- gamma agonist) can cause urinary bladder carcinoma; so can't be given.So, canagliflozin (SGLT 2 inhibitor) can be added." "This is a real-world medical entrance exam question, please give the true answer based on the question and selection. Topic name: None, Subject name: Gynaecology & Obstetrics","A pregnant female with known cardiac disease presents to you in the first trimester with history of warfarin embryopathy what should be advised now? Select and explain. A. Continue warfarin throughout the pregnancy, B. Replace warfarin with heparin in First trimester, C. Give acicoumarin, D. Use LMW heparin",B: Replace warfarin with heparin in First trimester,"Answer: B: Replace warfarin with heparin in First trimester Explanation: Ans. B. Replace warfarin with heparin in First trimesterRef: Williams Obstetrics, 24'h ed.Warf'arin has a low molecular weight and readily crosses the placenta.Exposure between the 6th and 9th weeks may result in warfarin embryopathy characterized by stippling of the veebrae and femoral epiphyses and by nasal hypoplasia with depression ofthe nasal bone." "This is a real-world medical entrance exam question, please give the true answer based on the question and selection. Topic name: None, Subject name: Dental","Which muscle is responsible for translation of condyle? Select and explain. A. Medial pterygoid, B. Masseter, C. Superior constrictor of pharynx, D. Lateral pterygoid",D: Lateral pterygoid,"Answer: D: Lateral pterygoid Explanation: None" "This is a real-world medical entrance exam question, please give the true answer based on the question and selection. Topic name: None, Subject name: Biochemistry","Average daily dose of vitamin-C is: Select and explain. A. 30 - 40 mg, B. 50 - 60 mg, C. 60 - 100 mg, D. 100 - 150 mg",B: 50 - 60 mg,"Answer: B: 50 - 60 mg Explanation: None" "This is a real-world medical entrance exam question, please give the true answer based on the question and selection. Topic name: None, Subject name: Dental","It is likely that cell mediated immune reactions (delayed hypersensitivity) occur in periodontitis because subjects with periodontitis have Select and explain. A. High levels of histamine in involved gingival tissue, B. IgG antibodies reactive with plaque bacterial antigens, C. Ttymphocytes sensitized to bacterial plaque antigens, D. High Levels of collagenase in gingival fluids",C: Ttymphocytes sensitized to bacterial plaque antigens,"Answer: C: Ttymphocytes sensitized to bacterial plaque antigens Explanation: None" "This is a real-world medical entrance exam question, please give the true answer based on the question and selection. Topic name: AIIMS 2019, Subject name: Medicine","Which of the following is NOT the criteria to discharge a patient of dengue haemorrhagic fever? Select and explain. A. Afebrile for 24 hours without antipyretics, B. Platelet count greater than 50,000 cells, C. Return of appetite, D. Atleast 24 hours since recovery from shock",D: Atleast 24 hours since recovery from shock,"Answer: D: Atleast 24 hours since recovery from shock Explanation: Normal urine output- 1.5 l/day For discharge patient should become afebrile 24 hrs from shock recovery would not be enough should be kept a little longer to know general well being of patient. Therefore, option a, b, d ruled out. Patients with dengue hemorrhagic fever or dengue shock syndrome may be discharged from the hospital when they meet the following criteria: Afebrile for 24 hours without antipyretics Good appetite, clinically improved condition Adequate urine output Stable hematrocit level Atleast 48 hours since recovery from shock No respiratory distress Platelet count greater than 50,000 cells." "This is a real-world medical entrance exam question, please give the true answer based on the question and selection. Topic name: None, Subject name: Pediatrics","A mother is exposed to Diethyl stilbesterol during pregnancy. All the following features may be seen in the child after birth except : Select and explain. A. Clear cell carcinoma, B. Microglandular hyperplasia, C. Malformation of the vagina and uterus, D. Vaginal adenosis",B: Microglandular hyperplasia,"Answer: B: Microglandular hyperplasia Explanation: ""Women who were exposed to diethylstilbestrol in utero have a high incidence of adenosis of the vagina and cervis. These patient also have, potential reproductive abnormalities, including infertility, habitual abortions and tubal and uterine cavity abnormality. Clear cell adenocarcinoma of the vagina and cervix is a rare sequaele of DES exposure in utero""." "This is a real-world medical entrance exam question, please give the true answer based on the question and selection. Topic name: None, Subject name: Dental","According to 2011 census, life expectancy at birth for male and female is Select and explain. A. 67 and 70, B. 70 and 67, C. 61 and 69, D. 69 and 61",A: 67 and 70,"Answer: A: 67 and 70 Explanation: None" "This is a real-world medical entrance exam question, please give the true answer based on the question and selection. Topic name: None, Subject name: Pathology","which one of the following shows 'Blueberry lips'? Select and explain. A. MEN syndrome, B. Cyanotic heart disease, C. Albright syndrome, D. Gardner syndrome",A: MEN syndrome,"Answer: A: MEN syndrome Explanation: None" "This is a real-world medical entrance exam question, please give the true answer based on the question and selection. Topic name: None, Subject name: Surgery","A surgeon with less experience of laparoscopic cholecystectomy while doing laparoscopic surgery found some stone in common bile duct. What should he ideally do? Select and explain. A. Open cholecystectomy with choledochoduodenostomy, B. Laparoscopic exploration of CBD and removal of stone, C. Laparoscopic CBD exploration through the cystic duct, D. Open CBD exploration",D: Open CBD exploration,"Answer: D: Open CBD exploration Explanation: Ans. d. Open CBD explorationLaparoscopic CBD exploration is an advanced surgery and if a surgeon is not trained for this then he should conve the case and do open cholecystectomy and open CBD exploration.The preparation and indications for cholecystectomy are the same whether it is performed by laparoscopy or by open techniques.For patients in whom a laparoscopic approach is not indicated or in whom conversion from a laparoscopic approach is required an open cholecystectomy is performed." "This is a real-world medical entrance exam question, please give the true answer based on the question and selection. Topic name: None, Subject name: Pediatrics","Macrocephaly is seen in which of the following syndromes ? Select and explain. A. Metachromatic leucodystrophy, B. Adrenoleukodystrophy, C. Canavan's disease, D. Krabbe's disease",C: Canavan's disease,"Answer: C: Canavan's disease Explanation: LEUCO J ITS TROPHY Leucodystrophy refers to progressive degeneration of the white matter of the brain clue to imperfect growth or development of the myelin sheath, the fatty covering that acts as an insulator around nerve fibre. Myelin which lends its color to the white matter of the brain, is a complex substance made up of at least ten different chemicals. The leucodystrophy are a group of disorders that are caused by genetic defects in hoW myelin produces or metabolizes these chemicals. Each of the leucodystrophies in the result of a defect in the gene that controls one (and only one) of the chemicals. Specific leusodystrophies include - Metachroinatic leucodystrophy          ❑ Adrenoleucodystrophy                                 ​❑ Canavan disease Krabbe's disease                                                 ❑ Pelizaeus-Merzhacher disease                                                         ​ ❑ Alexander disease Symptoms vary according to the specific type of leucodystrophy and may be difficult to recognize in the early stages of the disease. Canavan's disease Autosomal recessive disorder Caused due to deficiency of the enzyme N-Aspertoacylase. This leads to accumulation of N-Acetyl aspartic acid in brain and urine. It is characterized by the clinical traid of --> Hypotonia, Head lag, Macrocephaly ​​Adrenoleucodvstrophy X-linked recessive disorder Caused due to deficiency of AcyI-C'oA synthetase. Onset is about 5-10 years Main symptoms are ataxia, spasticity, motor deficits, cortical blindness. Marocephaly is not a key feature Metachromatic leucodystrophy Autosomal recessive disorder Caused due to Arylsulfatase A deficiency Onset is in the 2nd year. Symptoms are into-ordination, especially gait disturbance, then general regression, optic atrophy, combined upper and lower motor neuron signs. Marocephaly usually late. Krabbe leucodvstrophy Autosomal recessive Caused due to deficiency of Beta-galactosidase deficiency Onset is in the first 6 months of life Optic atrophy, spasticity Head often small" "This is a real-world medical entrance exam question, please give the true answer based on the question and selection. Topic name: None, Subject name: Dental","According to wein if there is bone resorption but no tooth resorption then substract ……..from the radiographic length Select and explain. A. 1mm, B. 1.5 mm, C. 2mm, D. 2.5 mm",B: 1.5 mm,"Answer: B: 1.5 mm Explanation: None" "This is a real-world medical entrance exam question, please give the true answer based on the question and selection. Topic name: None, Subject name: Anatomy","The 3 embryonic germ layers: ectoderm, mesoderm and endoderm are derived from: Select and explain. A. Epiblast only, B. Ectoderm from epiblast; mesoderm and endoderm from hypoblast, C. Ectoderm and mesoderm from epiblast; endoderm from hypoblast, D. Hypoblast only",A: Epiblast only,"Answer: A: Epiblast only Explanation: All three embryonic germ layers (ectoderm, mesoderm and endoderm) are derived from the epiblast only. The hypoblast contributes to the development of the extraembryonic mesoderm and yolk sac and plays a key role in signaling to establish axial patterning in the embryo itself" "This is a real-world medical entrance exam question, please give the true answer based on the question and selection. Topic name: None, Subject name: Pharmacology","A patient presented with acute exacerbation of bronchial asthma. Salbutamol inhalation didn't improve the condition of the patient. So, intravenous coicosteroids and aminophylline were added and the condition improved. What is the mechanism of action of coicosteroids in this condition? Select and explain. A. They cause bronchodilatation when given with xanthines, B. They increase bronchial responsiveness to salbutamol, C. They increase the action of aminophylline on adenosine receptors, D. They increase the mucociliary clearance",B: They increase bronchial responsiveness to salbutamol,"Answer: B: They increase bronchial responsiveness to salbutamol Explanation: Ans. b. They increase bronchial responsiveness to salbutamol Coicosteroids has a dual effect in acute asthma with an early facilitator effect on airway beta-2 adrenoreceptor sensitivity and a later effect on airway inflammation, which fuher emphasizes the need fir coicosteroids to he administered as early as possible during an acute asthma attack.""he molecular mechanism of action of coicosteroids involves several effects on the inflammatory process. The major effect of coicosteroids is to switch off the transcription of multiple activated genes that encode inflammatory proteins such as cytokines, chemokines, adhesion molecules, and inflammatory enzymes. This effect involves several mechanisms, including inhibition of the transcription factor NF-KB, but an impoant mechanism is recruitment of HDAC2 to the inflammatory gene complex, which reverses the histone acetylation associated with increased gene transcription. Coicosteroids also activate anti-inflammatory genes, such as mitogen-activated protein (MAP) kinase phosphatase-1, and increase the expression of IQ-receptors. Most of the metabolic and endocrine side effects of coicosteroids are also mediated through transcriptional activation." "This is a real-world medical entrance exam question, please give the true answer based on the question and selection. Topic name: None, Subject name: Dental","A mucostatic impression for a prosthesis is taken for which of the following cases Select and explain. A. Lower RPD with Kennedy I classification, B. Lower RPD with Kennedy II Classification, C. Upper RPD with Kennedy IV classification, D. Full upper and lower denture",C: Upper RPD with Kennedy IV classification,"Answer: C: Upper RPD with Kennedy IV classification Explanation: None" "This is a real-world medical entrance exam question, please give the true answer based on the question and selection. Topic name: None, Subject name: Surgery","In sequencing for panfacial trauma, requires the first reduction and fixation of: Select and explain. A. Mandible, B. ZMC, C. NOE region with frontal bone fracture, D. First can be any of the above but it should end with maxilla",D: First can be any of the above but it should end with maxilla,"Answer: D: First can be any of the above but it should end with maxilla Explanation: None" "This is a real-world medical entrance exam question, please give the true answer based on the question and selection. Topic name: None, Subject name: Medicine","The best preferred biochemical marker for MI Select and explain. A. CPK MM, B. CPK MB, C. CPK BB, D. All of the above",B: CPK MB,"Answer: B: CPK MB Explanation: None" "This is a real-world medical entrance exam question, please give the true answer based on the question and selection. Topic name: None, Subject name: Anatomy","Reichert's cartilage is: Select and explain. A. 1st arch cartilage., B. 2nd arch cartilage., C. 3rd arch cartilage., D. 4th arch cartilage.",B: 2nd arch cartilage.,"Answer: B: 2nd arch cartilage. Explanation: None" "This is a real-world medical entrance exam question, please give the true answer based on the question and selection. Topic name: None, Subject name: Pathology","An epithelial lining is typically found in all of the following except: Select and explain. A. Sinuses, B. Fistulae, C. Aneurismal bone cysts, D. Keratocytes",C: Aneurismal bone cysts,"Answer: C: Aneurismal bone cysts Explanation: None" "This is a real-world medical entrance exam question, please give the true answer based on the question and selection. Topic name: None, Subject name: Social & Preventive Medicine","A girl is has white hair. 40% of her classmates are also suffering from the same problem. None of the other people from the same village have white hair. Her brother who is staying with an aunt in the neighbouring village is also normal and does not have white hair. Which of the following explains the probable cause of her white hair? Select and explain. A. Environmental, B. Genetic, C. Nutritional, D. Infection",A: Environmental,"Answer: A: Environmental Explanation: Ans. a. EnvironmentalA girl and 40% of her classmates are suffering from white hair. Her brother who is staying with an aunt in the neighboring village is normal and does not have white hair. In the given question, as the girl and many of her classmates (from a single geographical area, in this case, the school) have developed grey hair, the most likely cause would be environmental.Similarly, as many other children from the same school have white hair, the cause is also unlikely to be nutritional.As the girl's brother living in a different village does not have white hair, the cause of the white hair is unlikely to be genetic" "This is a real-world medical entrance exam question, please give the true answer based on the question and selection. Topic name: None, Subject name: Dental","Non- working contact is needed in all except: Select and explain. A. Natural dentition., B. Complete denture., C. Distal extension RPDs, D. None.",A: Natural dentition.,"Answer: A: Natural dentition. Explanation: In natural dentition non working contact is most damaging hence should be removed as soon as possible." "This is a real-world medical entrance exam question, please give the true answer based on the question and selection. Topic name: AIIMS 2019, Subject name: Ophthalmology","Which of the following is true? Select and explain. A. Latanoprost is used with caution in patients of bronchial asthma, B. Topiramate can cause bilateral angle closure glaucoma, C. Methazolamide causes decrease in ocular blood flow, D. Central scotoma is seen in open angle glaucoma",B: Topiramate can cause bilateral angle closure glaucoma,"Answer: B: Topiramate can cause bilateral angle closure glaucoma Explanation: 1. Bronchial asthma pt. we have to avoid b - blockers, therefore , latanoprost cannot be used in bronchial asthma. Latanoprost can be used in: Herpes Uveitis Cystoid macular edema 2. Topiramate is a sulfa derivative They increase the vascularity of the cilio choroidal circulation due to which the ciliary body and choroid plexus swells up and it pushes the lense forward and it blocks the angle by pushing the iris forward leading to drug induced angle closure glaucoma. 3. Methazolamide is a carbonic anhydrase inhibitor Increases the ocular blood flow Used in low tension glaucoma 4. Central scotoma - seen in optic neuritis / toxic neuritis / toxic amblyopia. 5. Paracentral scotoma - seen in open angle glaucoma." "This is a real-world medical entrance exam question, please give the true answer based on the question and selection. Topic name: None, Subject name: Medicine","A Patient is having Ph- 7.12, HCO3- 28 and PCO2- S0mmhg. What is the acid base disorder in this patient? Select and explain. A. Metabolic acidosis with respiratory compensation, B. Metabolic alkalosis with respiratory compensation, C. Respiratory acidosis with renal compensation, D. Respiratory alkalosis with renal compensation",C: Respiratory acidosis with renal compensation,"Answer: C: Respiratory acidosis with renal compensation Explanation: Ans: C. Respiratory acidosis with renal compensationpH-7.12 so its acidosis.PCO2 is 5o mmHg (normal range 35-45 mmHg) which is higher than the normal s/o respiratory acidosis.HCO3- is 28 mmol/L (18-24 mmol/L) which is higher than the normal range s/o metabolic compensated alkalosis." "This is a real-world medical entrance exam question, please give the true answer based on the question and selection. Topic name: None, Subject name: Dental","Fibroblasts in cell-rich zone primarily secretes: Select and explain. A. Type I collagen, B. Type VI collagen, C. Type V collagen, D. Type IV collagen",A: Type I collagen,"Answer: A: Type I collagen Explanation: None" "This is a real-world medical entrance exam question, please give the true answer based on the question and selection. Topic name: None, Subject name: Medicine","A young female patient presents with jaundice and elevated liver enzymes. She has a history of two smilar episodes in the past. Her serum ANA was 1:40 and IgG was 2400 IU. Her serum copper levels were normal and viral markers were negative. A liver biopsy was carried out and based on the pathology repo, you treated her with immunosuppressants and the patient's condition improved. What was the most likely finding on pathology repo to suggest the treatment? Select and explain. A. Non-alcoholic steatohepatitis, B. Hemosiderosis, C. Autoimmune hepatitis, D. Primary biliary cirrhosis",C: Autoimmune hepatitis,"Answer: C: Autoimmune hepatitis Explanation: Ans. c. Autoimmune hepatitisMany of the clinicalfeatures of autoimmune hepatitis are similar to those described for chronic viral hepatitiso.. Fatigue, malaise, anorexia, amenorrhea, acne, ahralgias, and jaundice are commono.. Occasionally ahritis, maculopapular eruptions (including cutaneous vasculitis), erythema nodosum' colitis'pleurisy pericarditis, anemia, azotemiao and sicca syndromeType I autoimmune hepatitis- Associated with marked hyperglobulinemia, lupoid features, circulating ANAsa' andHLA-DR3 or HLA-DR4 (especially BB-DRB1-03).Associated autoantibodies: Autoantibodies against actin as well as atypical pANCAType ll autoimmune hepatitis- Linked to HLA-DRB1 and HLA-DQBI haplotypesAssociated with anti-LKMIType lll autoimmune hepatitis- These patients lack ANA and anti-LKMI but have circulating antibodies to solubleliver antigen/liver pancreas antigen" "This is a real-world medical entrance exam question, please give the true answer based on the question and selection. Topic name: None, Subject name: Anatomy","On applying pressure on angle of jaw while maintaining patent airway which nerve is likely to be damaged: Select and explain. A. 6th, B. 7th, C. 4th, D. 9th",B: 7th,"Answer: B: 7th Explanation: None" "This is a real-world medical entrance exam question, please give the true answer based on the question and selection. Topic name: None, Subject name: Dental","Which is not present in porcelain: Select and explain. A. Silica, B. Feldspar, C. Calcium carbonate, D. Amorphous material",D: Amorphous material,"Answer: D: Amorphous material Explanation: None" "This is a real-world medical entrance exam question, please give the true answer based on the question and selection. Topic name: None, Subject name: Dental","For data of ""GLOBOCAN 2012"" screening examination for carcinoma of lip incidence was measured. Which of the following is measured for data collection worldwide: Select and explain. A. Age specific only, B. Age specific with demographics, C. Age specific with gender, D. Age specific with growth rate",C: Age specific with gender,"Answer: C: Age specific with gender Explanation: GLOBOCAN 2012 Main aim was providing world wise estimate separately for sex- and age-specific on incidence, prevalence and mortality from major types of cancer, at national level" "This is a real-world medical entrance exam question, please give the true answer based on the question and selection. Topic name: None, Subject name: Pharmacology","Which of the following is a schedule X drug? Select and explain. A. Thalidomide, B. Colistin, C. Ketamine, D. Halothane",C: Ketamine,"Answer: C: Ketamine Explanation: Ans. C. Ketamine* The following are the Schedule X drugs mentioned in the Drugs and Cosmetics Rules 1945: Amobarbital, Amphetamine, Barbital, Cyclobarbital, Dexamphetamine, Ethchlorvynol, Glutethimide, Meprobamate, Methamphetamine, Methylphenidate, Methylphenobarbital, Phencyclidine, Phenmetrazine, Secobarbital." "This is a real-world medical entrance exam question, please give the true answer based on the question and selection. Topic name: None, Subject name: Ophthalmology","Diffuse mottling of retina with focal areas of increased and decreased pigmentation between posterior pole and equator (Salt and pepper retinopathy) is observed in all of the following except: Select and explain. A. Resolving retinal detachment, B. Phenothiazine toxicity, C. Congenital rubella, D. Fundus flavimaculatus",A: Resolving retinal detachment,"Answer: A: Resolving retinal detachment Explanation: Ans. a. Resolving retinal detachment (Ref: Kanki 7th/657- 666, Yanoffs 3rd/560-567)Salt and Pepper Fundus is a nonspecific term to describe areas of hypopigmentation due to atrophic retinal changes (salt) with pigmentary alterations from pigment migration (pepper) in large poions of retina.Causes of Salt and Pepper Fundus:Rubella (Congenital)Syphillis (Congenital)Leber amaurosis (Congenital)Mayous Batten disease (Maculo-cerebral facial degeneration)Phenothiazine toxicityRetinal dystrophiesFundus flavimaculatus (Stargardt's Disease)AlbinismRetinitis PigmentosaCystinosis" "This is a real-world medical entrance exam question, please give the true answer based on the question and selection. Topic name: None, Subject name: Gynaecology & Obstetrics","Birth weight of a baby can be increased by: Select and explain. A. Cessation of smoking, B. Aspirin, C. Ca++ and vitamin D supplement, D. Bed rest",A: Cessation of smoking,"Answer: A: Cessation of smoking Explanation: “Smoking is the single most preventable cause of IUGR in infants born in the united states—women who quit smoking at 7 months gestation have newborns with higher mean birthweights than do women who smoke throughout the pregnancy. Women who quit smoking before 16 weeks of gestation are not at any increased risk for an IUGR infant.” COGDT 10/e, p 293 The answer is further supported by Williams 24/e, p 882 “In prevention of fetal growth restriction – smoking cessation is critical” Williams 24/e, p 882" "This is a real-world medical entrance exam question, please give the true answer based on the question and selection. Topic name: AIIMS 2019, Subject name: Psychiatry","Z tracking technique is used in? Select and explain. A. Administering long acting antipsychotic, B. Lithium monitoring, C. Carbamazepine monitoring, D. Nicotine patch",A: Administering long acting antipsychotic,"Answer: A: Administering long acting antipsychotic Explanation: Z track technique Used for giving i.m. injections. In this technique, the skin and tissue are pulled and held firmly while injection is given, and after removing the needle, skin and tissue are released. This prevents tracking (leakage) of the medication into the subcutaneous tissue (underneath the skin) as the track that needle forms is zig zag and drug can't come out." "This is a real-world medical entrance exam question, please give the true answer based on the question and selection. Topic name: None, Subject name: Ophthalmology","Ex-Press glaucoma implant is made up of? Select and explain. A. Silicone, B. Titanium, C. Gold, D. Stainless steel",D: Stainless steel,"Answer: D: Stainless steel Explanation: Ans. d. Stainless steelIt is made of made of surgical-grade stainless steel, nonvalved filtration device designed to shunt aqueous humor from the anterior chamber to a subconjunctival filtration bleb.The EX-PRESS Glaucoma Filtration Device is intended to reduce IOP in glaucoma patients when medication and conventional surgical treatments have failed.The device channels aqueous humor through a secure lumen (of either 50 mm or 200 mm) to a half-thickness scleral flap, creating a subconjunctival filtration bleb.It is a minimally invasive glaucoma surgery (MIGS) to increase aqueous drainage.It has lower complications of ocular hypotony.The commonly used devices are as follows: Shunts using episcleral explants:* Ahmed glaucoma valve: Silicone* Molteno implant: Polypropelene plate and silicone plate* Baerveldt implant: Silicone Mini-shunts:* Ex-Press(tm) Mini-Shunt: Stainless steel * iStent(r): Titanium" "This is a real-world medical entrance exam question, please give the true answer based on the question and selection. Topic name: AIIMS 2020, Subject name: ENT","A child presents with high grade fever, inspiratory stridor and develops swallowing difficulty with drooling of saliva since last 4-6 hours. Which of the following treatment is recommended apa from general airway management? Select and explain. A. Nebulized racemic epinephrine, B. Anti-diphtheria toxin, C. Coicosteroids, D. IV ceftriaxone",A: Nebulized racemic epinephrine,"Answer: A: Nebulized racemic epinephrine Explanation: ACUTE EPIGLOTTITIS- The classical presentation of acute epiglottitis is welldescribed. It is of a toxic child with a sho history of sorethroat, inspiratory stridor, muffled voice and droolingdue to odynophagia and dysphagia. Left untreated thereis progressive respiratory distress. The child is febrile. It is commonest between the ages of 2 and 8 Nebulized epinephrine (1mL of 1 in 1000 epinephrinediluted in 3mL of 0.9% saline) has an established role inthe acute paediatric airway in reducing mucosal oedemaby an alpha-agonist effect causing vasoconstriction andbronchodilation; a maximum effect is achieved within30-60 minutes. It may postpone or eliminate the need for an aificial airway, or give symptomaticrelief until effective treatment can be given." "This is a real-world medical entrance exam question, please give the true answer based on the question and selection. Topic name: None, Subject name: Dental","A measure of location which divides the distribution the ratio of 3:1 is: Select and explain. A. Median, B. First quartile, C. Third quartile, D. Mode",C: Third quartile,"Answer: C: Third quartile Explanation: None" "This is a real-world medical entrance exam question, please give the true answer based on the question and selection. Topic name: AIIMS 2019, Subject name: Surgery","Rearrange the sequence of hand washing: a. b. c. d. Select and explain. A. a,c,d,b, B. b,c,a,d, C. d,a,c,b, D. c,a,b,d","A: a,c,d,b","Answer: A: a,c,d,b Explanation: None" "This is a real-world medical entrance exam question, please give the true answer based on the question and selection. Topic name: None, Subject name: ENT","A patient presents to your clinic for evaluation of defective hearing. Rinne's test shows air conduction greater than the bone conduction on both sides with Weber test lateralized to right ear. What is the next logical step? Select and explain. A. Normal test, B. Schwabach's test, C. Repeat Rinne's test on right side, D. Wax removal",B: Schwabach's test,"Answer: B: Schwabach's test Explanation: Ans: B. Schwabach's test(Ref Dhingra 7/e p23, 5/e p25-27)Lateralization of Weber's test to right implies either right-sided conductive deafness or left sided sensorineural deafness.Now in conductive deafness bone conduction is better than air conduction in Rinnie's test, hence the patient probably has sensorineural deafness involving the left ear, as per the findings of Rinnie's and Weber tests.In such a case, Schwabach's test should be performed to see the absolute bone." "This is a real-world medical entrance exam question, please give the true answer based on the question and selection. Topic name: None, Subject name: Ophthalmology","Phenol red thread test is used for dry eye: Select and explain. A. In the test, volume of tears is measured as it changes color on contact with tears, B. If the color changes to blue, it depicts surface mucin deficiency, C. Requires pH meter for reading the result, D. Requires topical anesthetic agent","A: In the test, volume of tears is measured as it changes color on contact with tears","Answer: A: In the test, volume of tears is measured as it changes color on contact with tears Explanation: Ans: A. In the test, volume of tears is measured as it changes color on contact with tears(Ref: Yanoff & Duker 4/e p277)Phenol red dye test measures the production of tears without topical anesthesia, as the dye changes its color to red on contact with tears.It doesn't require pH ureter for reading the result.Phenol red is a pH indicator used in cell biopsy as it changes colour from yellow to red on pH range from 6.8 to 8.2.Used to measure residual tears in inferior conjunctival sac, especially in dry eye patients." "This is a real-world medical entrance exam question, please give the true answer based on the question and selection. Topic name: None, Subject name: Dental","Specific plaque hypothesis state that Select and explain. A. All plaque is pathogenic, B. Only specific micro-organisms are responsible for pathogenicity of plaque, C. Only specific microbes cause caries, D. Plaque is pathogenic only when signs of associated disease are present",D: Plaque is pathogenic only when signs of associated disease are present,"Answer: D: Plaque is pathogenic only when signs of associated disease are present Explanation: None" "This is a real-world medical entrance exam question, please give the true answer based on the question and selection. Topic name: None, Subject name: Medicine","Organism which causes bacterial endocarditis: Select and explain. A. Lacto bacillus, B. Streptococcus sanguis, C. Streptococcus viridans, D. Streptococcus pyogenes",C: Streptococcus viridans,"Answer: C: Streptococcus viridans Explanation: None" "This is a real-world medical entrance exam question, please give the true answer based on the question and selection. Topic name: None, Subject name: Dental","Which of the following alloys has the highest surface roughness? Select and explain. A. Stainless steel, B. Elgiloy, C. NiTi, D. TMA",C: NiTi,"Answer: C: NiTi Explanation: The surface of NiTi is rougher (because of surface defects, not the quality of polishing) than that of beta-Ti, which in turn is rougher than steel. However, there is little or no correlation for orthodontic wires between the coefficients of friction and surface roughness. (i.e., interlocking and plowing are not significant components of the total frictional resistance). Ref: Profitt 4th edition page 378" "This is a real-world medical entrance exam question, please give the true answer based on the question and selection. Topic name: AIIMS 2017, Subject name: Microbiology","An infant presented with the following lesions on his face and limbs. Which of the following is the most likely causative organism? Select and explain. A. HSV, B. HPV, C. Coxsackie, D. CMV",C: Coxsackie,"Answer: C: Coxsackie Explanation: Hand foot and mouth disease can be caused by Coxsackie virus A16, Coxsackie virus B and Enterovirus 71. It is characterized by oral and pharyngeal ulcerations and vesicular rashes of the palms and soles which heal without crusting." "This is a real-world medical entrance exam question, please give the true answer based on the question and selection. Topic name: AIIMS 2018, Subject name: Pathology","Which of the following cannot be diagnosed without positive ANA? Select and explain. A. Drug induced lupus, B. SLE, C. Sjogren syndrome, D. Scleroderma",B: SLE,"Answer: B: SLE Explanation: Repeated negative testing for ANA indicates least chances of having SLE. ANA is positive in 98% of patients with SLE Limited form of scleroderma known as CREST has anti centromere antibody positive and generalized form has anti topoisomerase positive antibody Anti histone antibodies are positive in drug induced lupus Anti SS-A (RO) antibodies and anti SS-B(LA) antibodies are positive in Sjogren's Syndrome" "This is a real-world medical entrance exam question, please give the true answer based on the question and selection. Topic name: None, Subject name: Pathology","Lesions of oral tuberculosis are associated with all the following except: Select and explain. A. Central caseation, B. Hyaline degeneration, C. Giant cells in the center, D. Presence of epithelioid cells",B: Hyaline degeneration,"Answer: B: Hyaline degeneration Explanation: None" "This is a real-world medical entrance exam question, please give the true answer based on the question and selection. Topic name: None, Subject name: Dental","Which of the following statement is true as far as spring back bending of super-elastic NiTi and non-super elastic NiTI wires is concern? Select and explain. A. Spring back in bending is much greater for superelastic NiTi wires than non-superelastic NiTi, B. Spring back in bending is much lesser for superelastic NiTi wires than non-superelastic, C. Spring back in bending is same for superelastic Nill wires as well as non-superelastic NiTi wires, D. No comparison can be made between the spring back in bending of these two wires",A: Spring back in bending is much greater for superelastic NiTi wires than non-superelastic NiTi,"Answer: A: Spring back in bending is much greater for superelastic NiTi wires than non-superelastic NiTi Explanation: Springback—The amount of elastic strain that a metal can recover when loaded to and unloaded from its yield strength; an important property of orthodontic wires. Reference: Phillips 12th ed page no 396" "This is a real-world medical entrance exam question, please give the true answer based on the question and selection. Topic name: None, Subject name: Pathology","Intra-epithelial bulla are found in Select and explain. A. Pemphigus, B. Bullous pemphigoid, C. Bullous Lichen planus, D. Pemphigoid",A: Pemphigus,"Answer: A: Pemphigus Explanation: None" "This is a real-world medical entrance exam question, please give the true answer based on the question and selection. Topic name: None, Subject name: Surgery","Kraissl's lines arc: Select and explain. A. Collagen and elastin lines in stab wounds, B. Point of maximum tension in a fracture, C. Point of tension in hanging, D. Relaxed tension lines in skin",D: Relaxed tension lines in skin,"Answer: D: Relaxed tension lines in skin Explanation: Answer- D. Relaxed tension lines in skinKraissl's lines are essentially exaggerated wrinkle lines obtained by studying the loose skin of elderly faces whilst contracting the muscles of facial expression These lines for the most pa correspond to Relaxed skin tension lines (RSTLs), but slight variation exists on the face, especially on the lateral side of the nose, the lateral aspect of the orbit, and the chin." "This is a real-world medical entrance exam question, please give the true answer based on the question and selection. Topic name: None, Subject name: Surgery","A surgeon decides to operate a patient of carcinoma cecum and perform a right hemicolectomy through a midline laparotomy approach. You have been instructed to prepare the pas of the patient for surgery. What will you do? Select and explain. A. Clean and drape from the level of nipple to mid-thigh, B. Clean and drape from chin to knee, C. Clean and drape from umbilicus to mid-thigh, D. Clean and drape from rib cage to inguinal regions",A: Clean and drape from the level of nipple to mid-thigh,"Answer: A: Clean and drape from the level of nipple to mid-thigh Explanation: Ans: A. Clean and drape from the level of nipple to mid-thighArea of Cleaning & Draping in SurgeriesCranial surgeryDepends upon surgeonThyroid or neck surgeryChin to nipple with shoulder & axilladegEye surgeryCut eyelashes of affected eyeNasal surgeryNo shaving unless with mustacheEar surgeryTwo & half inches around eardegChest surgeryBase of neck to waist, axilla & inner armdegAbdominal & pelvic surgeryBase of neck to waist, axilla & inner armdegKidney-anteriorNipple to perineum, side to side; supra scapular region to buttocksVaginal, scrotal, rectal surgeryWaist to perineum plus anterior & inner aspect of thigh & 6 inches from groin; posterior?entire buttocks & anusdegLower extremitiesDigits 2 inches above knee, entire extremity and groindegUpper extremitiesDistal arm 2 inches above elbow, elbow up to axilladeg" "This is a real-world medical entrance exam question, please give the true answer based on the question and selection. Topic name: None, Subject name: Social & Preventive Medicine","Human Developmental Index is a composite measure, which uses? Select and explain. A. Life expectancy at age one, literacy and infant moality, B. Freedom, spice and right to express oneself, C. Life expectancy at bih, infant moality and quality of life, D. Life expectancy at bih, knowledge and decent standard of living.","D: Life expectancy at bih, knowledge and decent standard of living.","Answer: D: Life expectancy at bih, knowledge and decent standard of living. Explanation: Ans: D. Life expectancy at bih, knowledge and decent standard of living.(Ref Park 24/e p17, 23/e p17)Human Development Index (HDI):Published on 4 November 2010.Updated on 10 June 2011.Composite statistic of life expectancy, education & income indices.Used to rank countries into 4 tiers of human development. 2010 HDI Repo:Combines three dimensions:A long & healthy life: Life expectancy at bih.Education index: Mean years of schooling and Expected years of schooling.A decent standard of living: GNI per capita (PPP US$)." "This is a real-world medical entrance exam question, please give the true answer based on the question and selection. Topic name: None, Subject name: Pediatrics","NESTROFT test is a screening test for – Select and explain. A. β–thalassemia, B. Hereditary spherocytosis, C. Autoimmune hemolytic anemia, D. Megaloblastic anemia",A: β–thalassemia,"Answer: A: β–thalassemia Explanation: NESTROFT ( Naked Eye Single Tube Red Cell Osmotic Fragility Test) is a screening test for detection of beta thalassemia trait. Thalassemia screening Widespead prevlence of thalessemia has led to a pressing demand for community, screening. Various methods for screening of thalassemia trait are available which include peripheral smear examination, RBC indices, 1VIeintzer's fraction, discriminant functions, NESTROFT etc. NESTROFT TEST is used in many centres in India for screening of thalassemia trait. If mother is NESTROFT positive, the confirmatory test for HbA2 is done for mother and the father is subjected to screening by NESTROFT. If father is also NESTROFT positive, confirmatory test for HbA2 is done for father. If both the parents have thalessemia trait, there is 1:4 chance of fetus having thalassemia major. Therefore, prenatal diagnosis is done by chorionic villus sampling (CVS) in first trimester. If CVS confirms the fetus to be having thalasemia major, termination of pregnancy is indicated after counseling the parents. If the fetus has thalassemia minor (trait) pregnancy is continued as such and baby will have normal lifespan. It is worth noting here that a positive NESTROFT test is seen in other conditions beside beta thalassemia trait. These are iron deficiency anemia, alpha thalassemia trait, homozygous and heterozygous HbE, HbS as well as hereditary persistence of fetal hemoglobin. Therefore, a positive NESTROFT test should always be followed by a confirmatory test for HBA2 ( eg. serum electrophoresis)." "This is a real-world medical entrance exam question, please give the true answer based on the question and selection. Topic name: None, Subject name: Dental","Difference between the size of the cranium and the cephalogram is: Select and explain. A. No difference, B. 5% reduction, C. 5% enlargement, D. 10% enlargement",C: 5% enlargement,"Answer: C: 5% enlargement Explanation: None" "This is a real-world medical entrance exam question, please give the true answer based on the question and selection. Topic name: AIIMS 2017, Subject name: Biochemistry","Which amino acid does not include post translational modification? Select and explain. A. Selenocystiene, B. Triiodothyronine, C. Hydroxy-proline, D. Hydroxy-lysine",A: Selenocystiene,"Answer: A: Selenocystiene Explanation: Selenocysteine (21st amino acid) & Pyrrolysine (22nd amino acid) Formed by co-translational modification Not by post-translational modifications. Selenocysteine and Pyrrolysine 21st amino acid-Selenocysteine- UGA 22nd amino acid - Pyrrolysine - UAG Both UGA, UAG are stop codon but co-translational modification can give rise to amino acid selenocysteine and pyrrolysine. co-translational modification. Enzymes that use selenocysteine as their active sites are called Seleno Proteins. Selenocysteine containing proteins: Glutathione peroxidase Thioredoxin reductase selenoprotein P 5' deiodinases" "This is a real-world medical entrance exam question, please give the true answer based on the question and selection. Topic name: None, Subject name: Physiology","A politician is shot in the back during a rally at level of T8 veebral immediately after the shot he loses all the sensation below level of lesion. Chance of regeneration of spinal cord due to the fact that injured nerve is not able to regenerate is due to reason all except: Select and explain. A. Lack of endoneural tubes, B. Lack of growth factors, C. Presence of glial scar, D. Lack of myelin inhibiting substance",D: Lack of myelin inhibiting substance,"Answer: D: Lack of myelin inhibiting substance Explanation: Ans. d. Lack of myelin inhibiting substance(Ref GanonGr 90; Clinical Box 4-)Following CNS injuries several events which provide inappropriate environment for regeneration are: (Ganong 23/e p90) Astrocytic proliferation)Activation of microgliaScar formationInflammationInvasion of immune cellsCNS neurons do not have the growth promoting chemical needed for the regenerationCNS myelin is a potent inhibitor of axonal growthAxon Regeneration in CNSThe proximal stump of a damaged axon in the CNS will form sho sprouts, but distant stump recovery is rare, and the damaged axons are unlikely to form new synapses. This is because:CNS neurons do not have the growth promoting chemical needed for the regenerationCNS myelin is a potent inhibitor of axonal growth.That is why treatment of brain and spinal cord injuries frequently focuses on rehabilitation rather than reversing the nerve damage. Following CNS injuries, several events which provide inappropriate environment for regeneration are:Astrocytic proliferationActivation of microgliaformationInflammationInvasion of immune cellsNew research is aiming to identify ways to initiate and maintain axonal growth, to direct regenerating axons to reconnect with their target neurons and to reconstitute original neuronal circuit." "This is a real-world medical entrance exam question, please give the true answer based on the question and selection. Topic name: None, Subject name: Biochemistry","Enzyme that catalyses the transfer of a molecule group from one molecule to another is: Select and explain. A. Oxidases, B. Oxidases, C. Transferases, D. Lipases",C: Transferases,"Answer: C: Transferases Explanation: None" "This is a real-world medical entrance exam question, please give the true answer based on the question and selection. Topic name: None, Subject name: Dental","Bacteria are populated most commonly on Select and explain. A. Buccal surface, B. Lingual surface, C. Interproximal surface, D. Occlusal surface",C: Interproximal surface,"Answer: C: Interproximal surface Explanation: None" "This is a real-world medical entrance exam question, please give the true answer based on the question and selection. Topic name: None, Subject name: Physiology","Stretch fibers are present in striated muscle, innervation is carried by Select and explain. A. A delta fiber, B. C fiber, C. unmyelinated fiber, D. Gamma fiber",D: Gamma fiber,"Answer: D: Gamma fiber Explanation: None" "This is a real-world medical entrance exam question, please give the true answer based on the question and selection. Topic name: None, Subject name: Radiology","Which film is used for caries detection? (or) Which film is used for caries detection in children? Select and explain. A. D speed, B. E speed, C. B speed, D. F speed",D: F speed,"Answer: D: F speed Explanation: None" "This is a real-world medical entrance exam question, please give the true answer based on the question and selection. Topic name: None, Subject name: Dental","Sending the parents outside the operatory and threatening the child that they won't be back until he stops crying-is an example of which of the following behavior management techniques: Select and explain. A. Omission, B. Negative reinforcement, C. Positive reinforcement, D. Modelling",A: Omission,"Answer: A: Omission Explanation: None" "This is a real-world medical entrance exam question, please give the true answer based on the question and selection. Topic name: None, Subject name: Pharmacology","Black deposits on conjunctiva in a patient with glaucoma are seen with the use of: Select and explain. A. Prostaglandins, B. Carbonic anhydrase inhibitors, C. Epinephrine, D. Beta blocker",C: Epinephrine,"Answer: C: Epinephrine Explanation: Ans: C. Epinephrine(Ref: Goodman Gilman 12/e p286, 326; Katzung 13/e p161, 12/e p160; The Cornea by Smolin and Thoft's/p504)Black deposits on conjunctiva in a patient with glaucoma are seen with use of epinephrine. Latanoprost is associated with iris hyperpigmentation.Epinephrine is unstable in alkaline solution; when exposed to air or light, it turns pink from oxidation to adrenochrome and then brown from formation of polymers." "This is a real-world medical entrance exam question, please give the true answer based on the question and selection. Topic name: None, Subject name: Pathology","Which is responsible for adhesion of platelet on exposed collagen fibril after an injury Select and explain. A. Von willebrand factor, B. Factor 8, C. Factor 9, D. Fibronectin",A: Von willebrand factor,"Answer: A: Von willebrand factor Explanation: None" "This is a real-world medical entrance exam question, please give the true answer based on the question and selection. Topic name: None, Subject name: Pathology","Transformation into osteosarcoma is seen with Select and explain. A. Pagets disease and polyostotic fibrous dysplasia, B. Pagets disease and osteopetrosis, C. Cherubism and polyostotic fibrous dyspLasia, D. Cherubism and polyostotic fibrous dyspLasia",A: Pagets disease and polyostotic fibrous dysplasia,"Answer: A: Pagets disease and polyostotic fibrous dysplasia Explanation: None" "This is a real-world medical entrance exam question, please give the true answer based on the question and selection. Topic name: None, Subject name: Dental","Accessory canals are most numerous in apical third, which one is 2nd numerous in this regard Select and explain. A. Middle Third, B. Coronal Third, C. Both are equal, D. None of the above",B: Coronal Third,"Answer: B: Coronal Third Explanation: Apical third has accessory canals: 74% of cases Coronal 3rd have accessory canals: 15% of cases Middle third has accessory canals: 11% of cases" "This is a real-world medical entrance exam question, please give the true answer based on the question and selection. Topic name: None, Subject name: Surgery","Injury to which of the following vessels is least likely to result in significant damage: Select and explain. A. Renal aery, B. Superior mesenteric aery, C. Inferior mesenteric aery, D. Celiac trunk",C: Inferior mesenteric aery,"Answer: C: Inferior mesenteric aery Explanation: Ans. c. Inferior mesenteric aeryInjury to inferior mesenteric aery is least likely to result in significant damage among the given options.Occlusion of the inferior mesenteric aery does not always result in irreversible ischaemia of the descending and sigmoid colon, because the marginal aery of the colon usually receives an adequate supply from the left branch of the middle colic aery." "This is a real-world medical entrance exam question, please give the true answer based on the question and selection. Topic name: None, Subject name: Dental","A 65 year old edentulous patient is treated for squamous cell carcinoma involving 2/3rd of tongue with 50 Gy radiation dose. Which of the following material is best for making impression in this patient? Select and explain. A. Polysulfide, B. Polyether, C. ZOE, D. Irreversible hydrocolloids",A: Polysulfide,"Answer: A: Polysulfide Explanation: The patient has undergone radiation therapy, which leads to xerostomia. Therefore, in this patient, hydrophilic impression material cannot be used. ZOE impression material is also contraindicated in this patient, since it can adhere to mucosa and leads to ulceration on removal. So, among the options provided, only polysulfide is the best suitable impression material for these kind of patients." "This is a real-world medical entrance exam question, please give the true answer based on the question and selection. Topic name: None, Subject name: Dental","Tooth develops from: (Or) Calcified structures of oral cavity develops from Select and explain. A. Ectoderm, mesoderm, B. Ectoderm, C. Mesoderm, D. Ectoderm and endoderm","A: Ectoderm, mesoderm","Answer: A: Ectoderm, mesoderm Explanation: Most of skeletal and connective tissues with exception of enamel are derived from ectomesenchymal tissue. Enamel is derived from ectoderm only. Posterior part of oral cavity is formed from endoderm." "This is a real-world medical entrance exam question, please give the true answer based on the question and selection. Topic name: None, Subject name: Surgery","In a 5-year-old child with mandibular fracture, on a follow up recall after 2 months, what should be done to check the bone healing: Select and explain. A. OPG at every 2 month till one year, B. NCCT, C. Single contrast CT, D. Clinical examination adequate",D: Clinical examination adequate,"Answer: D: Clinical examination adequate Explanation: Clinically stable and symmetric bone, with a normal occlusion and a pain free chewing is indication enough for a completed bone healing in a child." "This is a real-world medical entrance exam question, please give the true answer based on the question and selection. Topic name: None, Subject name: Medicine","Ciliocytophthoria caused by Select and explain. A. Kaagener syndrome, B. Fungal infection, C. Viral infection, D. Bacterial infection",C: Viral infection,"Answer: C: Viral infection Explanation: Answer- C. Viral infectionIt is well known that acute inflammation of the upper airways are caused mainly by viruses, even though after the viral infection a bacterial overlapped infection follows, paly ored by the cytopathic effect of the virus itself on the mucosa." "This is a real-world medical entrance exam question, please give the true answer based on the question and selection. Topic name: None, Subject name: Pharmacology","The principal central action of caffeine is on the: Select and explain. A. Cerebral cortex, B. Corpus callosum, C. Hypothalamus, D. Spinal cord",A: Cerebral cortex,"Answer: A: Cerebral cortex Explanation: None" "This is a real-world medical entrance exam question, please give the true answer based on the question and selection. Topic name: None, Subject name: Dental","In root fracture of the apical one-thrid of permanent anterior teeth, the teeth usually Select and explain. A. Discolor rapidly, B. Remain in function and are vital, C. Undergo papal necrosis and become ankylosed, D. Are indicated for extraction and prosthetic replacement",B: Remain in function and are vital,"Answer: B: Remain in function and are vital Explanation: None" "This is a real-world medical entrance exam question, please give the true answer based on the question and selection. Topic name: None, Subject name: Dental","Mode is Select and explain. A. Value that most frequently occurs in a set of observation, B. When observations are arranged in ascending or descending order the value that comes in the centre, C. Calculated by adding up the values of a group of items and dividing the sum by the number of items, D. Best measure for central tendency in a highly scented distribution",A: Value that most frequently occurs in a set of observation,"Answer: A: Value that most frequently occurs in a set of observation Explanation: None" "This is a real-world medical entrance exam question, please give the true answer based on the question and selection. Topic name: None, Subject name: Radiology","The base of an X-ray film is composed of: Select and explain. A. Polyester, B. Gelatin, C. Nylon, D. Bromide",A: Polyester,"Answer: A: Polyester Explanation: None" "This is a real-world medical entrance exam question, please give the true answer based on the question and selection. Topic name: None, Subject name: Dental","Location of MB2 in maxillary first molar Select and explain. A. Distopalatal to MB1, B. Mesiobuccal to MB1, C. Distobuccal to MB1, D. Mesiopalatal to MB1",D: Mesiopalatal to MB1,"Answer: D: Mesiopalatal to MB1 Explanation: None" "This is a real-world medical entrance exam question, please give the true answer based on the question and selection. Topic name: None, Subject name: Gynaecology & Obstetrics","Which of the following abnormalities can be diagnosed in the I st trimester of pregnancy? Select and explain. A. Anencephaly, B. Encephalocele, C. Meningocele, D. Microcephaly",A: Anencephaly,"Answer: A: Anencephaly Explanation: Answer- A. Anencephaly""Anencephaly is diagnosed by the absence of cranial vault (calvarium) and telencephalon. Brain tissue is angiomatous.Early diagnosis is possible at aboat 13 weeks" "This is a real-world medical entrance exam question, please give the true answer based on the question and selection. Topic name: None, Subject name: Medicine","A 50 years old female on junk food diet presented with point hemorrhages in scalp, bleeding in joints and erythemataous lesions in the skin. X-ray of knee joint was suggestive of hemahrosis. Problem lies with: Select and explain. A. Hydroxylation of proline and lysine, B. Carboxylation of glutamic acid, C. Carboxylation of coagulation factors, D. Platelet aggregation defect",A: Hydroxylation of proline and lysine,"Answer: A: Hydroxylation of proline and lysine Explanation: Answer- A. Hydroxylation of proline and lysineVitamin C is required for post-translational modification of lysine and proline in collagen synthesisHydroxyproline is formed by post-translational hydroxylation catalyzed by prolyl hydroxylase, whose cofactors are ascorbic acid and Keto- alpha glutarate." "This is a real-world medical entrance exam question, please give the true answer based on the question and selection. Topic name: None, Subject name: Pathology","In a 60 yr old hypertensive male with renal failure, renal biopsy shows onion skin appearance. The most likely diagnosis is Select and explain. A. Hyaline arteriosclerosis, B. Thrombophlebitis obliterans, C. Hyperplastic arteriosclerosis, D. Arteriosclerosis obliterans",C: Hyperplastic arteriosclerosis,"Answer: C: Hyperplastic arteriosclerosis Explanation: None" "This is a real-world medical entrance exam question, please give the true answer based on the question and selection. Topic name: None, Subject name: Dental","The inter-arch distance measured when the occlusal rims are in uniform contact is: Select and explain. A. Freeway space, B. Vertical dimension of occlusion, C. Vertical dimension of rest, D. Christensen's space",B: Vertical dimension of occlusion,"Answer: B: Vertical dimension of occlusion Explanation: None" "This is a real-world medical entrance exam question, please give the true answer based on the question and selection. Topic name: None, Subject name: Biochemistry","Which of the following is used by RBCs in the fasting state? Select and explain. A. Glucose, B. Alanine, C. Ketone body, D. Fatty acid",A: Glucose,"Answer: A: Glucose Explanation: Ans: A. GlucoseGlucose is used by RBCs in fasting state.OrganFedFastingStarvationBrainGlucoseGlucoseKetone bodiesHeaFatty Fatty acidsKetone bodiesLiverGlucoseFatty acidsAmino acidsMusclesGlucoseFatty acidsFatty acids & ketoneAdipose tissueGlucoseFatty acidsFatty acids & ketone RBCsGlucoseGlucose (Ref Harper 30/e p146, 150, 28/e p141)." "This is a real-world medical entrance exam question, please give the true answer based on the question and selection. Topic name: None, Subject name: Dental","A condition where there is loss of interdental bone on facial and lingual plate, without involvement of radicular bone is: Select and explain. A. Ledge, B. Crater, C. Intrabony defect, D. Reverse architecture",D: Reverse architecture,"Answer: D: Reverse architecture Explanation: Reversed Architecture Reverse (or negative) alveolar bone architecture is the result of a loss  of  interdental  bone,  without  a  concomitant  loss  of  radicular (buccal or lingual/palatal) bone, thereby reversing the normal (or positive)  architecture .  Negative  architecture  is  more common in the maxilla of patients with periodontitis. Ledges Ledges are plateau-like bone margins that are caused by the resorption of thickened bony plates . Osseous Craters Osseous craters are a specific type of two-wall defect; they present as concavities in the crest of the interdental bone that is confined within the facial and lingual walls. Craters have been found to make up about one-third (35.2%) of all defects and about two-thirds (62%) of all mandibular defects; they occur twice as often in posterior segments as in anterior segments. Ref: Newman and Carranza’s Clinical Periodontology, thirteenth edition; page no 325" "This is a real-world medical entrance exam question, please give the true answer based on the question and selection. Topic name: None, Subject name: Dental","Till what age tetracycline should not be given to prevent discoloration Select and explain. A. 3 year, B. 8 year, C. 12 year, D. 18 year",B: 8 year,"Answer: B: 8 year Explanation: None" "This is a real-world medical entrance exam question, please give the true answer based on the question and selection. Topic name: AIIMS 2019, Subject name: Medicine","A 50 year old tall thin smoker presents with dyspnoea and palpitations. ECG was done. Diagnosis: Select and explain. A. Atrial fibrillation, B. Multifocal atrial tachycardia, C. AV nodal re-entrant tachycardia, D. Monomorphic VT",B: Multifocal atrial tachycardia,"Answer: B: Multifocal atrial tachycardia Explanation: Question mentions - H/o smoker first thing that comes in mind is COPD & MC arrhythmia atrial fibrillation. We first see lead II on ECG RR approx. 1.5 and more towards left 2.5. HR=300/1.5=200/min Therefore, Patient suffers from tachycardia with variable RR interval. D/D Atrial fibrillation Multifocal atrial tachycardia P wave absent P wave present Variable RR interval Therefore ruled out P wave varying amplitual and answer is MAT. Trick to solve: First check if patient has tachycardia/bradycardia RR interval Narrow QRS suggests supraventricular tachycardia COPD is the reason for multifocal atrial tachycardia due to right ventricular strain causing RV enlargement causing right axis duration. Visible P wave variable amplitude RR interval changing Narrow QRS All point towards diagnosis of multifocal atrial tachycardia. Monomorphic VT is ruled out as it has broad QRS complex. Between MAT & AV nodal reentrant tachycardia the H/o smokers narrows down the diagnosis to MAT." "This is a real-world medical entrance exam question, please give the true answer based on the question and selection. Topic name: None, Subject name: Pediatrics","Which of the following is not a feature of Juvenile Idiopathic Arthritis – Select and explain. A. Rheumatoid nodule, B. Spikes of high fever, C. Uveitis, D. Raynaud's phenomenon",D: Raynaud's phenomenon,"Answer: D: Raynaud's phenomenon Explanation: Juvenile idiopathic arthritis Juvenile idiopathic arthritis is broad term that describes a clinically heterogenous group of arthritides of unknown cause, which begin before 16 years of age. This term encompasses several disease categories : - 1. Juvenile rheumatoid arthritis (see previous explanations) : - i) Pauciarticular (oligoarticular) JRA ii) Poly-articular JRA RA. factor (+) ye RA factor (-) ye iii) Systemic JRA Juvenile psoriatic arthritis Juvenile enthesitis related arthritis (ERA) ​Subgroups of Juvenile Idiopathic Arthritis (JIM" "This is a real-world medical entrance exam question, please give the true answer based on the question and selection. Topic name: None, Subject name: Dental","For an unbounded amalgam restoration, what should be done before copal varnish is applied ? Select and explain. A. Dentine desensitiser is applied, B. Smear layer is left intact, C. Smear layer is modified, D. After removal of enamel smear layer only",A: Dentine desensitiser is applied,"Answer: A: Dentine desensitiser is applied Explanation: The use of a dentin desensitizer over the prepared tooth structure before placing amalgam is generally recommended.  The dentin desensitizer is rubbed onto the prepared tooth surface for 30 seconds and excess moisture is removed without desiccating the dentin. Sturdevant's Art and Science of Operative Dentistry. Edition 6. Page: 417" "This is a real-world medical entrance exam question, please give the true answer based on the question and selection. Topic name: None, Subject name: Dental","In CPITN index zero score means: Select and explain. A. Gingival bleeding, B. Grade 1 mobility of teeth, C. Healthy tooth, D. Spontaneous bleeding",C: Healthy tooth,"Answer: C: Healthy tooth Explanation: None" "This is a real-world medical entrance exam question, please give the true answer based on the question and selection. Topic name: None, Subject name: Pediatrics","A five year old male child presents with complaints of fever and abdominal distension. IIe is having vomiting for the last five days. On examination there are 6–8 Pus Cell/hpf in urine. WBC count shows 78% neutrophils. What is the best line of management – Select and explain. A. Send urine for culture and sensitivity and wait for results, B. Send urine for culture and sensitivity and start I.V. antibiotics immediately, C. Send urine for culture, do an USG and start chloroquine, D. Radio nucleotide studies",B: Send urine for culture and sensitivity and start I.V. antibiotics immediately,"Answer: B: Send urine for culture and sensitivity and start I.V. antibiotics immediately Explanation: None" "This is a real-world medical entrance exam question, please give the true answer based on the question and selection. Topic name: None, Subject name: Forensic Medicine","A doctor who performed the autopsy on a 26-year-old married woman, committing suicide found the cause of death to be aluminium phosphide poisoning. She was summoned in a cou of law where he willingly hides this information. This is punishable under which section? Select and explain. A. IPC 193, B. CrPC 175, C. CrPC 69, D. IPC 189",A: IPC 193,"Answer: A: IPC 193 Explanation: Ans: A. IPC 193(Ref: Reaco 34/e p I 3, 33/e p392; Textbooi, on the Indian Penal Code by Krishna 1) Gaur 4/e p594; the-Indian-penal-code-pdfd74214920)Section 193 - In Indian Penal Code deals with punishment for giving false evidence." "This is a real-world medical entrance exam question, please give the true answer based on the question and selection. Topic name: AIIMS 2019, Subject name: Gynaecology & Obstetrics","Loading dose MgSO4 (IV) to be prepared as Select and explain. A. 4 ml 50% w/v plus 16 ml NS, B. 8 ml 50% w/v plus 12 ml NS, C. 12 ml 50% w/v plus 8 ml NS, D. 16 ml 50% w/v plus 4 ml NS",B: 8 ml 50% w/v plus 12 ml NS,"Answer: B: 8 ml 50% w/v plus 12 ml NS Explanation: Loading dose of Mgso4: Total: 14 gms Ampule commercially available is: Each 2 ml of injection (50%) = 1 gm of MgSO4 4 gm I/V as a 20 % solution that is 4 ampules (8 mL=4 gm) + 12 mL NS making it 20 % solution + 10 gm I/M (5gm IM in each buttock as 50 % solution)" "This is a real-world medical entrance exam question, please give the true answer based on the question and selection. Topic name: None, Subject name: Biochemistry","Apoprotein B-48 is found in Select and explain. A. VLDL, B. LDL, C. HDL, D. Chylomicrons",D: Chylomicrons,"Answer: D: Chylomicrons Explanation: None" "This is a real-world medical entrance exam question, please give the true answer based on the question and selection. Topic name: None, Subject name: Social & Preventive Medicine","If the distribution of intra-ocular pressure (IOP) seen in 100 glaucoma patients has an average 30 mm with a SD of 1.0, what is the lower limit of the average IOP that can be expected 95% of times? Select and explain. A. 26, B. 28, C. 32, D. 259",B: 28,"Answer: B: 28 Explanation: In the given question, Mean (μ) = 30 mm and SD (σ) = 1.0 mm Thus, 95% values are contained in the range of Mean ± 2 SD (µ ± 2σ) or 30 ± 2 (1) So, 95% values are contained in the range 30 – 2 mm and 30 + 2 mm OR between 28 and 30 mm." "This is a real-world medical entrance exam question, please give the true answer based on the question and selection. Topic name: None, Subject name: Surgery","The radiographic view of choice for diagnosing horizontally favorable and unfavorable # is: Select and explain. A. OPG, B. Occlusal, C. Lateral Oblique view, D. Transpharyngeal",C: Lateral Oblique view,"Answer: C: Lateral Oblique view Explanation: None" "This is a real-world medical entrance exam question, please give the true answer based on the question and selection. Topic name: None, Subject name: Gynaecology & Obstetrics","Which type of Hb is not affected by Rh isoimmunisation: Select and explain. A. Anti C, B. Anti E, C. Anti lewis, D. Anti D",C: Anti lewis,"Answer: C: Anti lewis Explanation: “An antigen frequently found in routine antenatal screening is the Lewis group (Lea and Leb). The Lewis antigens do not cause erythroblastosis fetalis and differ from all of the other red cell antigens in that they are not synthesized in the red cell membrane but are absorbed into it.” Fernando Arias 2/e, p 116" "This is a real-world medical entrance exam question, please give the true answer based on the question and selection. Topic name: None, Subject name: Ophthalmology","The ETDRS cha is used for visual evaluation in diabetic patients. What does ETDRS stand for Select and explain. A. Extended treatment for diabetic retinopathy study, B. Early treatment for diabetic retinopathy study, C. Emergency treatment for diabetic retinopathy study, D. Emerging treatment for diabetic retinopathy study",B: Early treatment for diabetic retinopathy study,"Answer: B: Early treatment for diabetic retinopathy study Explanation: Answer: B. Early treatment for diabetic retinopathy studyETDRS stands for Early treatment for diabetic retinopathy study" "This is a real-world medical entrance exam question, please give the true answer based on the question and selection. Topic name: None, Subject name: Dental","Working side contact means: Select and explain. A. Contact on lateral movement., B. Contact on medial movement., C. No contact on lateral movement., D. None of the above.",A: Contact on lateral movement.,"Answer: A: Contact on lateral movement. Explanation: None" "This is a real-world medical entrance exam question, please give the true answer based on the question and selection. Topic name: None, Subject name: Dental","What percentage of accessory canals found in apical third of the root Select and explain. A. 74%, B. 54%, C. 94%, D. 60%",A: 74%,"Answer: A: 74% Explanation: None" "This is a real-world medical entrance exam question, please give the true answer based on the question and selection. Topic name: None, Subject name: Pathology","The resolving power of a microscope depends upon all of the following except: Select and explain. A. Size of the apeure, B. Focal length of the eyepiece, C. Thickness of the film, D. Wavelength of light source used",C: Thickness of the film,"Answer: C: Thickness of the film Explanation: Ans: C. Thickness of the film(Ref Bancroft Theory & Practice of Histochemical Techniques/p45-47).Resolving power of microscope:Microscope specific.Inversely distance between two objects that can be resolved.Doesn't depend on specimen nature or thickness.According to Abbe's Criterion." "This is a real-world medical entrance exam question, please give the true answer based on the question and selection. Topic name: None, Subject name: Dental","Which type of trauma is associated with destruction of protective layer? Select and explain. A. Intrusion, B. Extrusion, C. Avulsion, D. Lateral luxation",A: Intrusion,"Answer: A: Intrusion Explanation: None" "This is a real-world medical entrance exam question, please give the true answer based on the question and selection. Topic name: None, Subject name: Surgery","During diagnostic laparoscopy for undescended testis, there are absent testicular vessels. What should be done next? Select and explain. A. Explore fuher, B. Nothing is to be done, C. Inguinal exploration, D. Scrotal examination",C: Inguinal exploration,"Answer: C: Inguinal exploration Explanation: Answer- C" "This is a real-world medical entrance exam question, please give the true answer based on the question and selection. Topic name: None, Subject name: Dental","Excessive occlusal forces can be superimposed in all cases, except Select and explain. A. Normal periodontium with normal height of bone, B. Normal periodontium with reduced height of bone, C. Marginal periodontitis with reduced height of bone, D. Marginal periodontitis with normal height of bone",D: Marginal periodontitis with normal height of bone,"Answer: D: Marginal periodontitis with normal height of bone Explanation: None" "This is a real-world medical entrance exam question, please give the true answer based on the question and selection. Topic name: None, Subject name: Dental","The reverse bevel incision is made to Select and explain. A. Allow atraumatic reflection of the gingival margin, B. Remove the infected tissue in the sulcus, C. Provide access to the alveolar crest, D. All of the above",B: Remove the infected tissue in the sulcus,"Answer: B: Remove the infected tissue in the sulcus Explanation: None" "This is a real-world medical entrance exam question, please give the true answer based on the question and selection. Topic name: None, Subject name: Dental","Partial veneer crown is contraindicated in Select and explain. A. Long teeth, B. Short teeth, C. Tipped teeth, D. Tapered teeth",B: Short teeth,"Answer: B: Short teeth Explanation: None" "This is a real-world medical entrance exam question, please give the true answer based on the question and selection. Topic name: AIIMS 2017, Subject name: ENT","Target sign is seen in? Select and explain. A. Spontaneous CSF Rhinorrhea, B. Traumatic CSF Rhinorrhea, C. Both, D. None",B: Traumatic CSF Rhinorrhea,"Answer: B: Traumatic CSF Rhinorrhea Explanation: Leakage of CSF into the nose is called CSF rhinorrhea. It may be clear fluid or mixed with blood as in acute head injuries. TRAUMA. Most of the cases follow trauma.it can be accidental or surgical. Inflammation, mucoceles of sinuses, sinunasal polyposis, fungal infection of sinuses & osteomyelitis, can all erode the bone and dura. Neoplasms, idiopathic- where cause is unknown and patient has spontaneous leak. SITE OF Leakage- CSF from anterior cranial fossa reaches the nose (1) cribriform plate, (2) roof of ethmoid air cells or (3) frontal sinus. CSF from middle cranial fossa follows injuries to sphenoid sinus, cribriform plate. In fracture of temporal bone, CSF reaches the middle ear and then escapes through the Eustachian tube into the nose (CSF OTORHINORRHOEA) DIAGNOSIS- there is history of clear watery discharge from the nose on bending the head or straining. CSF rhinorrhea should be differentiated from nasal discharge as the former is sudden, gushes in drops when bending and cannot be sniffed back. CSF rhinorrhea after head trauma is mixed with blood and shows ""target sign"" when collected on filter paper.it shows central red spot and peripheral lighter halo sign. LAB TEST = BETA-2 transferrin is seen in CSF and not in the nasal discharge. its presence is a specific and sensitive test." "This is a real-world medical entrance exam question, please give the true answer based on the question and selection. Topic name: None, Subject name: Surgery","LA deposited in Inferior alveolar nerve block at Select and explain. A. Sigmoid notch, B. Coronoid notch, C. Lingula, D. Groove of mandibular neck",C: Lingula,"Answer: C: Lingula Explanation: None" "This is a real-world medical entrance exam question, please give the true answer based on the question and selection. Topic name: None, Subject name: Surgery","In extraction, best time to administer analgesics is: Select and explain. A. Before anaesthesia wears off, B. Prior to extraction, C. When pain is moderate to severe, D. After anaesthesia wears off",A: Before anaesthesia wears off,"Answer: A: Before anaesthesia wears off Explanation: None" "This is a real-world medical entrance exam question, please give the true answer based on the question and selection. Topic name: None, Subject name: Dental","Mantle dentin is a layer next to: Select and explain. A. DE], B. Pulp, C. Dentin, D. Enamel",A: DE],"Answer: A: DE] Explanation: None" "This is a real-world medical entrance exam question, please give the true answer based on the question and selection. Topic name: None, Subject name: ENT","The wave ' V ' in BERA represents activity in which of the following ? Select and explain. A. Lateral lemniscus, B. Superior olivary complex, C. Cochlear nerve, D. Cochlear nucleus",A: Lateral lemniscus,"Answer: A: Lateral lemniscus Explanation: The wave 'V' in BERA represents activity in lateral lemniscus.Auditory brainstem response (ABR):Is used to elicit brainstem responses to auditory stimulation by clicks or tone bursts.It is a non-invasive technique to find the integrity of central auditory pathways through the VIIIth nerve, pons and midbrain.The first, third and fifth waves are most stable and are used in measurements.Wave V Generation of wave V reflects the activity of multiple anatomic auditory structures. The sharp positive peak of wave V arises mainly from the lateral lemniscus, following slow negative wave represents the dendritic potential in the inferior colliculus." "This is a real-world medical entrance exam question, please give the true answer based on the question and selection. Topic name: None, Subject name: Gynaecology & Obstetrics","Preconceptional intake of which of the following results in decrease in incidence of neural tube defect: Select and explain. A. Vitamin A, B. Folate, C. Vitamin E, D. Vitamin C",B: Folate,"Answer: B: Folate Explanation: “Folic acid has been shown to effectively reduce the risk of neural tube defects (NTDs). A daily 4 mg dose is recommended for patients who have had a previous pregnancy affected by neural tube defects. It should be started atleast 1 month (ideally 3 months) prior to pregnancy and continued through the first 6–12 weeks of pregnancy.” COGDT 10/e, p 197 Remember: Therapeutic dose of folic acid (to be given in females with previous history of baby with NTD) - 4 mg. Prophylactic dose of folic acid = 0.4 mg i.e. 400 mg Duration: It should be started 1 month before conception and continued till 3 months of pregnancy." "This is a real-world medical entrance exam question, please give the true answer based on the question and selection. Topic name: None, Subject name: Dental","Lower 2nd molar has Select and explain. A. ‘+’ shape, B. Distorted ‘+’ shape, C. Y shaped, D. H shaped",A: ‘+’ shape,"Answer: A: ‘+’ shape Explanation: None" "This is a real-world medical entrance exam question, please give the true answer based on the question and selection. Topic name: None, Subject name: Dental","Failure of partial dentures due to poor clasp design can best be avoided by: Select and explain. A. Using stress breakers, B. Using bar type clasps, C. Altering tooth contours, D. Clasping only those teeth with fairly long crowns and normal bone support",C: Altering tooth contours,"Answer: C: Altering tooth contours Explanation: None" "This is a real-world medical entrance exam question, please give the true answer based on the question and selection. Topic name: None, Subject name: Physiology","In excitable cells, repolarization is closely associated with one of the following events: Select and explain. A. Na+ efflux, B. Na+ influx, C. K+ efflux, D. K+ influx",C: K+ efflux,"Answer: C: K+ efflux Explanation: None" "This is a real-world medical entrance exam question, please give the true answer based on the question and selection. Topic name: None, Subject name: Dental","Hapten is? Select and explain. A. Antigenic and proteinaceous substance, B. Protein and non antigenic, C. Nonprotein and antigenic, D. Nonprotein and non antigenic",A: Antigenic and proteinaceous substance,"Answer: A: Antigenic and proteinaceous substance Explanation: None" "This is a real-world medical entrance exam question, please give the true answer based on the question and selection. Topic name: None, Subject name: Ophthalmology","Right trochlear nerve palsy can lead to all except: Select and explain. A. Diplopia on upward gaze and adduction, B. Right head tilt, C. Exotropia, D. Hyperopia",B: Right head tilt,"Answer: B: Right head tilt Explanation: Ans: B. Right head tilt(Ref. Yanoff and Duker 4/e p1228)Right trochlear nerve palsy:Accompanied by compensatory contralateral head tilt (left head tilt), not right head tilt.Signs:Classical sign of unilateral fouh nerve palsy is contralateral head tilt (an 'ocular' toicollis).Exhibited by most patients & usually sole presenting sign in children.Non-ophthalmological causes also considered.Presents with veical diplopia.Torsional diplopia - Due to ocular cyclotorsion accompanies veical diplopia in acquired fouh nerve palsy." "This is a real-world medical entrance exam question, please give the true answer based on the question and selection. Topic name: None, Subject name: Dental","The lingual extension of a Lower denture is Limited by all of the following (except) Select and explain. A. Modiolus, B. Mylohyoid muscle, C. Origin of genioglossus, D. Sublingual salivary glands",A: Modiolus,"Answer: A: Modiolus Explanation: None" "This is a real-world medical entrance exam question, please give the true answer based on the question and selection. Topic name: None, Subject name: Radiology","Patient with history of tachyarrhythmias is on implantable cardioveer defibrillator. He develops shock. Best method to know the position and integrity of ICD is: Select and explain. A. CECT, B. MRI, C. USG, D. Plain radiograph",A: CECT,"Answer: A: CECT Explanation: Ans: A. CECT (Ref Hurst 's The Hea 13/e p1050)Plain radiograph:Best method to know ICD position & integrity.ICD Lead dislodgement:Radiographically visible.Significant increase in pacing threshold /decline in electrogram amplitude." "This is a real-world medical entrance exam question, please give the true answer based on the question and selection. Topic name: None, Subject name: Surgery","Morris retractor is most commonly used to retract Select and explain. A. Left hypochondrium, B. Right hypochondrium, C. Hernia surgery, D. None",A: Left hypochondrium,"Answer: A: Left hypochondrium Explanation: Answer- A. Left hypochondrium Left hypochondrium- usesUsed to retract the abdominal wall in a wide mannerInstrument of choice or retraction in left subcostal region because it avoids the risk of splenic injury." "This is a real-world medical entrance exam question, please give the true answer based on the question and selection. Topic name: None, Subject name: Microbiology","Which of the following is not true about H. influenza? Select and explain. A. Rarely presents as meningitis in children less than 2 months of age, B. Capsular polypeptide protein is responsible for virulence, C. Requires factor V and X for growth, D. Most common invasive disease of H. influenza is meningitis",B: Capsular polypeptide protein is responsible for virulence,"Answer: B: Capsular polypeptide protein is responsible for virulence Explanation: None" "This is a real-world medical entrance exam question, please give the true answer based on the question and selection. Topic name: AIIMS 2019, Subject name: Medicine","Which drug is not given in Wilson disease? Select and explain. A. Trientine, B. Calcium, C. Penicillamine, D. Zinc",B: Calcium,"Answer: B: Calcium Explanation: Wilson disease Trientine- copper chelator Penicillamine- also chelator but less used due to nephrotic side effect. Zinc- is competitive inhibitor of copper in absorption of gut. Zinc acetate is DOC for Wilson disease." "This is a real-world medical entrance exam question, please give the true answer based on the question and selection. Topic name: AIIMS 2018, Subject name: Forensic Medicine","Bullet fingerprinting is: Select and explain. A. Human fingerprints on bullet, B. Primary marking, C. Secondary marking, D. Distoed bullet",C: Secondary marking,"Answer: C: Secondary marking Explanation: Bullet fingerprinting- Primary markings- The rifled firearm leaves its signature on the caridge case and on the bullet. With all rifled firearms, the bullet is slightly larger than the barrel, and as it passes through the barrel, its sides are marked by the rifling of the barrel. Secondary markings- They are produced on the surface of the bullet by imperfection on the inner surface of the barrel. These irregularities are produced by sticking of paicles of the bullet to the bore when shots are fired and is known as metallic fouling. They also result accidentally during the manufacturing process, usually microscopic in nature and have random distribution. They are useful in identifying the specific gun which was fired." "This is a real-world medical entrance exam question, please give the true answer based on the question and selection. Topic name: None, Subject name: Dental","Following intracoronal bleaching immediate composite restoration required, what has to be done: Select and explain. A. Treat with catalase, B. Wait for 7 days is mandatory, C. Treat with H2O2 for 3 minutes, D. Not possible",A: Treat with catalase,"Answer: A: Treat with catalase Explanation: None" "This is a real-world medical entrance exam question, please give the true answer based on the question and selection. Topic name: None, Subject name: Anatomy","Which of the following is developed from the 1st arch: Select and explain. A. Stylohyoid ligament., B. Spenomandibular ligament., C. Stapes., D. Styloid process.",B: Spenomandibular ligament.,"Answer: B: Spenomandibular ligament. Explanation: None" "This is a real-world medical entrance exam question, please give the true answer based on the question and selection. Topic name: None, Subject name: Forensic Medicine","A 70 years old male patient presents with amblyopia, exeional chest pain, episodic tachycardia and extra systoles on FCC. What is the probable cause? Select and explain. A. Cocaine poisoning, B. Chronic nicotine poisoning, C. Arsenophagia, D. Cannabis ingestion",B: Chronic nicotine poisoning,"Answer: B: Chronic nicotine poisoning Explanation: Answer- B. Chronic nicotine poisoningSymptoms of chronic nicotine poisoning are cough, wheezing, dyspnoea, anorexia, vomiting, diarrhea, anemia, faintness,tremors, impaired memory, amblyopia, blindness, irregularity of the hea with extra-systoles and occasionally attacks pain suggesting angina pectoris" "This is a real-world medical entrance exam question, please give the true answer based on the question and selection. Topic name: None, Subject name: Dental","All of the following reflex appear at birth except: Select and explain. A. Landau reflex, B. Babinski reflex, C. Startle reflex, D. Moro's reflex",A: Landau reflex,"Answer: A: Landau reflex Explanation: All the reflexes appears at birth except Landau and parachute reflex." "This is a real-world medical entrance exam question, please give the true answer based on the question and selection. Topic name: None, Subject name: Pediatrics","A child had repeated vomiting and developed metabolic alkalosis. The treatment given is – Select and explain. A. Ringer lactate, B. I.V. normal saline and potassium, C. ORS, D. I.V. Normal saline",B: I.V. normal saline and potassium,"Answer: B: I.V. normal saline and potassium Explanation: Treatment of metabolic alkalosis due to vomiting or nasogastric suction. Most children with metabolic alkalosis have one of the chloride responsive etiology (e.g., vomiting). In these situations, administration of suffieeint sodium chloride and potassium chloride to correct volume deficit is necessary to correct metabolic alkalosis. Adequate replacement of gastric losses of sodium and potassium in a child with vomiting can minimize or prevent the development of metabolic alkalosis. With adequate intravascular volume and a normal serum potassium concentration, the kidney is able to excrete the excess bicarbonate within a couple of days." "This is a real-world medical entrance exam question, please give the true answer based on the question and selection. Topic name: None, Subject name: Pathology","Most used selective medium for streptococcus rnutans is Select and explain. A. Mac conkey medium, B. Mitus salivarius bacitracin agar, C. Nutrient agar, D. Tellurite medium",B: Mitus salivarius bacitracin agar,"Answer: B: Mitus salivarius bacitracin agar Explanation: None" "This is a real-world medical entrance exam question, please give the true answer based on the question and selection. Topic name: None, Subject name: Dental","Which characteristic of orthodontic wire describe energy storage capacity? Select and explain. A. Range, B. Resilience, C. Formability, D. Proportional limit",B: Resilience,"Answer: B: Resilience Explanation: Popularly, the term resilience is associated with springiness, but it means precisely the amount of energy absorbed within a unit volume of a structure when it is stressed to its proportional limit.  The resilience of two or more materials can be compared by observing the areas under the elastic region of their stress-strain plots assuming that they are plotted on the same scale. The material with the larger elastic area has higher resilience. Key Concept: Resilience—The amount of elastic energy per unit volume that is sustained on loading and released upon unloading of a test specimen.   Reference: Phillips science of dental materials, 12th ed page no 48,55" "This is a real-world medical entrance exam question, please give the true answer based on the question and selection. Topic name: None, Subject name: Dental","Thickness of luting cement is: Select and explain. A. 20-40 μ, B. 10-20 μ, C. 60-80 μ, D. 1-2 μ",A: 20-40 μ,"Answer: A: 20-40 μ Explanation: None" "This is a real-world medical entrance exam question, please give the true answer based on the question and selection. Topic name: None, Subject name: Surgery","Midface is defined as the portion between: Select and explain. A. Hairline to subnasale, B. Glabella to subnasale, C. Glabella to menton, D. Subnasale to menton",B: Glabella to subnasale,"Answer: B: Glabella to subnasale Explanation: None" "This is a real-world medical entrance exam question, please give the true answer based on the question and selection. Topic name: None, Subject name: Physiology","Which of the following anterior pituitary hormones is primarily under inhibitory control of hypothalamus. Select and explain. A. TSH, B. GRH, C. Somatostatin, D. Prolactin",D: Prolactin,"Answer: D: Prolactin Explanation: None" "This is a real-world medical entrance exam question, please give the true answer based on the question and selection. Topic name: None, Subject name: Dental","Buccal coil spring used to regain space between 1st premolar and 1st molar causes following common post treatment complication: Select and explain. A. Pain, B. Gingival irritation, C. Tendency of 1st premolar to rotate, D. Tendency of 1st premolar to intrude",C: Tendency of 1st premolar to rotate,"Answer: C: Tendency of 1st premolar to rotate Explanation: None" "This is a real-world medical entrance exam question, please give the true answer based on the question and selection. Topic name: None, Subject name: Surgery","Direct inter dental wiring is also known as: Select and explain. A. Risdon's wiring, B. Gilmer's wiring, C. Eyelet wiring, D. Col. Stouts wiring",B: Gilmer's wiring,"Answer: B: Gilmer's wiring Explanation: None" "This is a real-world medical entrance exam question, please give the true answer based on the question and selection. Topic name: None, Subject name: Pathology","On biopsy report of CGCG on the basis of histologically & morphological similarities differential diagnosis is made between: Select and explain. A. Fibrous dysplasia, B. Hyperparathyroidism, C. Osteitis deformans, D. Hyperthyroidism",B: Hyperparathyroidism,"Answer: B: Hyperparathyroidism Explanation: None" "This is a real-world medical entrance exam question, please give the true answer based on the question and selection. Topic name: None, Subject name: Surgery","Best fluid for factor VIII replacement is: Select and explain. A. fresh whole blood, B. cryoprecipitate, C. FFP, D. Platelet concentrate",B: cryoprecipitate,"Answer: B: cryoprecipitate Explanation: None" "This is a real-world medical entrance exam question, please give the true answer based on the question and selection. Topic name: None, Subject name: Dental","Which of the following feature is not seen in tongue thrusting? Select and explain. A. Spaced dentition, B. Crowding and crossbite, C. Open bite, D. Flush terminal plane",B: Crowding and crossbite,"Answer: B: Crowding and crossbite Explanation: Characteristics of tongue thrusting Spaced dentition Primate spaces Deep bite Flush terminal plane Malocclusion Various malocclusions have been reported to be caused due to tongue thrust. These can further be subdivided as: a. Features pertaining to the maxilla Proclination of maxillary anteriors resulting in an increase in overjet. Generalised spacing between the teeth. Maxillary constriction. b. Features pertaining to the mandible Retroclination or proclination of mandibular teeth depending on the type of tongue thrust present. c. Intermaxillary relationships Anterior or posterior open bite based on the posture of the tongue. Posterior teeth crossbite." "This is a real-world medical entrance exam question, please give the true answer based on the question and selection. Topic name: None, Subject name: Surgery","Best way to monitor perfusion of free flap: Select and explain. A. Pulse oximeter, B. Laser doppler velocimetery, C. Prick test, D. Flourescin & dermo flourometer",C: Prick test,"Answer: C: Prick test Explanation: None" "This is a real-world medical entrance exam question, please give the true answer based on the question and selection. Topic name: None, Subject name: Dental","Which of the following structures is not of ectodermal origin: Select and explain. A. Hunter-schreger bands, B. Enamel spindles, C. Enamel tufts, D. Enamel lamellae",B: Enamel spindles,"Answer: B: Enamel spindles Explanation: Enamel spindles are odontoblastic process, which extend into enamel. Enamel spindles are mesenchymal in origin." "This is a real-world medical entrance exam question, please give the true answer based on the question and selection. Topic name: None, Subject name: Forensic Medicine","Paradoxical undressing seen in - Select and explain. A. Immersion syndrome, B. Heat cramps, C. Hypothermia, D. Heat exhaustion",C: Hypothermia,"Answer: C: Hypothermia Explanation: Ans- c- hypothermiaParadoxical undressing is a term for a phenomenon frequently seen in cases of lethal hypothermia. Sholy before death, the person will remove all their clothes, as if they were burning up, when in fact they are freezing." "This is a real-world medical entrance exam question, please give the true answer based on the question and selection. Topic name: None, Subject name: Radiology","The single most effective method of reducing patient somatic exposure when taking radiograph is to use: Select and explain. A. A lead apron, B. Speed E film, C. Added filtration, D. An open ended, shielded cone",B: Speed E film,"Answer: B: Speed E film Explanation: Intraoral dental X-ray film is available in two speed groups: D and E/F. Clinically, film of speed group E/F is approximately twice as fast (sensitive) as film of group D and thus requires only half the exposure. Fast films are desirable from the standpoint of exposure reduction. Multiple studies have found that E/F-speed film is preferred, because it has the same useful density range, latitude, contrast, and image quality as D-speed films and can be used in routine intraoral radiographic examinations without sacrifice of diagnostic information.  Current digital sensors offer equal or greater dose savings than E/F-speed film and comparable diagnostic utility. Reference: White and Pharoah's Oral Radiology Principles and Interpretation 7th edition page no 33" "This is a real-world medical entrance exam question, please give the true answer based on the question and selection. Topic name: None, Subject name: Surgery","There is a swelling in a patient with respect to maxillary LI region, since last 48 hours, the swelling is hot & palpable & rebounds on pressure, the treatment of choice is Select and explain. A. Incision & drainage, B. Antibiotic coverage, C. Antibiotics of heat only, D. Aspiration",A: Incision & drainage,"Answer: A: Incision & drainage Explanation: None" "This is a real-world medical entrance exam question, please give the true answer based on the question and selection. Topic name: None, Subject name: Anatomy","True about occulomotor nerve are all except Select and explain. A. Carries parasympathetic fibres, B. Causes constriction of pupils, C. Supplies inferior oblique muscle, D. Passes through inferior orbital fissure",D: Passes through inferior orbital fissure,"Answer: D: Passes through inferior orbital fissure Explanation: None" "This is a real-world medical entrance exam question, please give the true answer based on the question and selection. Topic name: None, Subject name: Dental","In a free gingival graft, what happens to epithelium of the graft? It: Select and explain. A. Remains as such, B. Proliferates, C. Degenerates, D. Has to be removed by the surgeon",C: Degenerates,"Answer: C: Degenerates Explanation: None" "This is a real-world medical entrance exam question, please give the true answer based on the question and selection. Topic name: None, Subject name: Medicine","A boy is suffering from acute pyelonephritis. The most specific investigation is Select and explain. A. Histopathological examination, B. Leucocyte esterase test, C. Nitrite test, D. Bacteria in gram stain",A: Histopathological examination,"Answer: A: Histopathological examination Explanation: The answer is A Acute pyelonephritis is a bacterial infection causing inflammation of the kidneys and is one of the most common diseases of the kidney.Pyelonephritis occurs as a complication of an ascending urinary tract infection (UTI) which spreads from the bladder to the kidneys and their collecting systems.Symptoms usually include fever, flank pain, nausea, vomiting, burning on urination, increased frequency, and urgency. The 2 most common symptoms are usually fever and flank pain. E. coli is the most common bacteria causing acute pyelonephritis due to its unique ability to adhere to and colonize the urinary tract and kidneys.Histopathology will usually reveal necrosis or putrid abscess formation within the renal parenchyma. The renal tissues are infiltrated with neutrophils, macrophages and plasma cells. However, architecture is not completely disorganized.The presence of white blood cells within or upon casts strongly suggests pyelonephritis, direct infection of the kidney. They may also be seen in inflammatory states, such as acute allergic interstitial nephritis, nephrotic syndrome, or post-streptococcal acute glomerulonephritis." "This is a real-world medical entrance exam question, please give the true answer based on the question and selection. Topic name: None, Subject name: Dental","Setting expansion of grey MTA when mixed with water is? Select and explain. A. 0.68%, B. 0.12%, C. 1.02%, D. 0.80%",C: 1.02%,"Answer: C: 1.02% Explanation: None" "This is a real-world medical entrance exam question, please give the true answer based on the question and selection. Topic name: None, Subject name: Pediatrics","All of the following syndromes are associated with uniparental disomy except- Select and explain. A. Prader-Willi syndrome, B. Russell-Silver syndrome, C. Bloom syndrome, D. Angelman syndrome",C: Bloom syndrome,"Answer: C: Bloom syndrome Explanation: Answer- C. Bloom syndromeBloom syndrome is not associated wilh uniparental disomy.Uniparental disomy is the term used when both chromosome of a pair of chromosome in a person with normal number ofchromosome, have been inherited from only one parent." "This is a real-world medical entrance exam question, please give the true answer based on the question and selection. Topic name: AIIMS 2017, Subject name: Medicine","Which of the following has a male:female ratio of 1:1? Select and explain. A. Rheumatoid ahritis, B. Gout, C. Seronegative spondyloahritis, D. Polymyalgia rheumatica",C: Seronegative spondyloahritis,"Answer: C: Seronegative spondyloahritis Explanation: Seronegative spondyloahritis has a male : female ratio of 1:1. Rheumatoid ahritis and Polymyalgia rheumatica are seen more commonly in females. Gout is more common in males." "This is a real-world medical entrance exam question, please give the true answer based on the question and selection. Topic name: AIIMS 2017, Subject name: Pharmacology","A diabetic and hypeensive patient taking several drugs presented with septicemia. Serum creatinine levels are 5.7 mg/dL. Which of the following drug should be stopped? Select and explain. A. Insulin, B. Metoprolol, C. Linagliptin, D. Metformin",D: Metformin,"Answer: D: Metformin Explanation: Serum creatinine levels are raised indicating renal failure. In renal failure and hepatic failure --> Metformin is C/I as it |ses the risk of developing lactic acidosis. Contraindicated in patients with an eGFR below 30 mL/minute/1.73 m2. eGFR should be measured annually for patients taking metformin therapy. eGFR is preferred over serum creatinine concentration to assess renal function for a person taking metformin therapy Linagliptin is a DPP-4 inhibitor. Metoprolol is a b blocker." "This is a real-world medical entrance exam question, please give the true answer based on the question and selection. Topic name: None, Subject name: Pediatrics","Drug of choice for Rheumatic fever prophylaxis in penicillin allergic patient – Select and explain. A. Erythromycin, B. Clindamycin, C. Vancomycin, D. Gentamycin",A: Erythromycin,"Answer: A: Erythromycin Explanation: T/T of Rhernatic fever Drug of choice --> Penicillin Drug of choice in penicillin allergic patients -4 Erythromycin Drug of choice for prophylaxis in Rheumatic fever -4 enzathine penicillin Primary prevention            -       Benzathine penicillin administared before the 9th day of streptococcal pharyngitis Secondary prevention       -       Benzathine penicillin is administered of patients who have had an episode of Rheumatic fever this prevents further attacks In penicillin allergic patients erythromycin is administered." "This is a real-world medical entrance exam question, please give the true answer based on the question and selection. Topic name: None, Subject name: Dental","Placement of graft will be failure in which class of recession Select and explain. A. class I, B. Class II, C. Class III, D. Class IV",D: Class IV,"Answer: D: Class IV Explanation: None" "This is a real-world medical entrance exam question, please give the true answer based on the question and selection. Topic name: None, Subject name: Dental","The vascular supply of the periodontal ligament is: Select and explain. A. Greatest in the middle-third of a single rooted tooth, B. Greatest in the middle-third of a multirooted tooth, C. A net-like plexus that runs closer to the cementum than to the bone, D. A net-like plexus that runs closer to the bone than to the cementum",D: A net-like plexus that runs closer to the bone than to the cementum,"Answer: D: A net-like plexus that runs closer to the bone than to the cementum Explanation: None" "This is a real-world medical entrance exam question, please give the true answer based on the question and selection. Topic name: None, Subject name: Surgery","Commonest cause of TMJ ankylosis is Select and explain. A. Trauma, B. Development disturbances, C. Infections, D. Atrophy",A: Trauma,"Answer: A: Trauma Explanation: None" "This is a real-world medical entrance exam question, please give the true answer based on the question and selection. Topic name: None, Subject name: Dental","To prevent sensitivity caused by acid etching and to protect pulp in deep cavities which of the following should be used. Select and explain. A. Ca(OH)2 liner, B. ZOE, C. Light cured GIC liner, D. Varnish",A: Ca(OH)2 liner,"Answer: A: Ca(OH)2 liner Explanation: None" "This is a real-world medical entrance exam question, please give the true answer based on the question and selection. Topic name: None, Subject name: Pediatrics","All are true about rheumatic fever, except – Select and explain. A. Common in poor socioeconomic group, B. Develops after streptococcal pharyngitis, C. Communicable disease, D. Seen in 5–15 years of children",C: Communicable disease,"Answer: C: Communicable disease Explanation: Rheumatic fever Rheumatic fever is an acute immunologically mediated multisystem inflammatory disease that occurs few weeks after an episode of group A streptococcal pharyngitis. The disease is immune mediated, not a communicable disease. Epidemiology ofAcute Rheumatic fever. The epidemiology of acute Rheumatic fever is identical to that of group A streptococcal upper respiratory tract infection. As is the case of streptococcal sore throat, acute rheumatic fever most often occurs in children, the peak age related incidence is between 5-15 years. Epidemiological risk factor for rheumatic fever, include lower standards of living especially crowding, the disease has been more common among socially and economically disadvantaged populations. Pathogenesis of Rheumatic fever It is strongly suspected that acute Rheumatic fever is a hypersensitivity reaction induced by group A streptococci but the exact pathogenesis remains uncertain despite many years of investigation. Although the precise factor or factors that confer this property are unknown, highly rheumatogenic strains, share certain biological characteristic. It is believed that antibodies directed against the M proteins of certain strains cross reacts with glycoprotein antigens in the heart, joints and other tissues. These strains are (1, 3, 5, 6, 18). The M proteins molecules has a particular surface exposed antigenic domain against which Rheumatic fever patients mount a strong antibody response. These antibodies against the M proteins cross reacts with glycoproteins antigens in the heart, joints and other tissues because these glycoproteins share similar features with M antigenic domain." "This is a real-world medical entrance exam question, please give the true answer based on the question and selection. Topic name: AIIMS 2019, Subject name: Anaesthesia","The case of biliary duct stricture with retching and vomiting, given 3mg morphine epiduraly daily, one day 12 mg mistakenly dose of epidural, morphine given. Not present:- Select and explain. A. Itching, B. Urinary retention, C. Increase vomiting, D. Overstimulation of respiratory centre",D: Overstimulation of respiratory centre,"Answer: D: Overstimulation of respiratory centre Explanation: Side effect of high dose epidural opioid: Nausea and vomiting Pruritis Urinary retention Respiratory depression High dose of opioid can spread centrally and stimulate CTZ center and can precipitate nausea and vomiting. Pruritis due to morphine is not due to histamine release, high dose morphine can spread centrally, act on trigeminal nerve and cause pruritis. Urinary retention in morphine use is due to it's action on sacral spinal cord, therefore it can happen even at normal dose and not related to high dose. Respiratory depression in morphine overdose due to migration of morphine centrally." "This is a real-world medical entrance exam question, please give the true answer based on the question and selection. Topic name: None, Subject name: Dental","Bonding agents have hydrophilic and hydrophobic groups. The hydrophobic group binds which structure? Select and explain. A. Calcium in enamel, B. Hydroxyapatite group in hard tissue, C. Resin in restoration, D. Collagen in dentin",C: Resin in restoration,"Answer: C: Resin in restoration Explanation: Ideally, a dentin bonding agent should have both hydrophilic and hydrophobic ends. The hydrophilic end displaces the dentinal fluid to wet the surface. The hydrophobic end bonds to the composite resin. Bonding: The forces or energies between atoms or molecules at an interface that hold two phases together. Mechanism of bonding: Dentin adhesive molecule has a bifunctional structure: M-R-X Where, M is the double bond of methacrylate which copolymerizes with composite resin. R is the spacer which makes the molecule large. X is a functional group for bonding, which bonds to an inorganic or organic portion of dentin. Ref: Textbook of Operative Dentistry, Nisha and Amit Garg, 3rd edition, Pg no:239" "This is a real-world medical entrance exam question, please give the true answer based on the question and selection. Topic name: None, Subject name: Pharmacology","A patient on 300 mg of aspirin will show all the following except: Select and explain. A. Irreversible inhibition of cyclooxygenase path way, B. Prolonged bleeding time, C. Inhibition of thromboxane TXA2, D. Inhibition of prostaglandin PGI2",A: Irreversible inhibition of cyclooxygenase path way,"Answer: A: Irreversible inhibition of cyclooxygenase path way Explanation: None" "This is a real-world medical entrance exam question, please give the true answer based on the question and selection. Topic name: None, Subject name: Pharmacology","Antimalarial drug used for causal prophylaxis act at which stage of developmental cycle Select and explain. A. Gametogony, B. Erythrocytic schizogony, C. Pre-erythrocytic schizogony, D. Exo-erythrocytic schizogony",C: Pre-erythrocytic schizogony,"Answer: C: Pre-erythrocytic schizogony Explanation: Ans. C. Pre-erythrocytic schizogonyPre-erythrocytic schizogony* Occurs inside the parenchymal cells of liver.* Merozoites are liberated into blood.* Drugs acting on this stage will prevent infection of RBC and clinical attack of malaria causal prophylaxis.* Drugs are* Proguanil & Tetracycline - for falciparum." "This is a real-world medical entrance exam question, please give the true answer based on the question and selection. Topic name: None, Subject name: Pathology","A 3-year-old patient reports of painless progressive bilateral facial swellings. The tentative diagnosis is Select and explain. A. Cherubism, B. Monostotic fibrous dysplasia, C. Polyostotic fibrous dysplasia, D. Central giant cell granuloma",A: Cherubism,"Answer: A: Cherubism Explanation: None" "This is a real-world medical entrance exam question, please give the true answer based on the question and selection. Topic name: None, Subject name: Dental","Normal WBC count Select and explain. A. 4,000-11,000, B. 40000 -80000, C. 13000-17000, D. 1000-3000","A: 4,000-11,000","Answer: A: 4,000-11,000 Explanation: None" "This is a real-world medical entrance exam question, please give the true answer based on the question and selection. Topic name: AIIMS 2018, Subject name: Psychiatry","Provision of WHO mental action gap are all, except: Select and explain. A. Human rights, B. Communication regarding care and career, C. Screening family members, D. Social suppo",C: Screening family members,"Answer: C: Screening family members Explanation: WHO developed mental health action gap to improve the care of mental, neurological, substance abuse disorder (low and middle country) These disorder cause large economics costs and are association with violation of human rights The key pa was to develop an evidence-based guideline meant for use by non specialists Mental health action gap involves mobilization of suppo and establish panerships" "This is a real-world medical entrance exam question, please give the true answer based on the question and selection. Topic name: None, Subject name: Anatomy","Retraction of the protruded mandible is done by Select and explain. A. Medial Pterygoid, B. Lateral pterygoid, C. Masseter, D. Temporalis",D: Temporalis,"Answer: D: Temporalis Explanation: Depression of mandible is caused by lateral pterygoid (main), digastric, geniohyoid & mylohyoid. And Elevation of mandible by temporalis, medial pterygoid & masseter." "This is a real-world medical entrance exam question, please give the true answer based on the question and selection. Topic name: None, Subject name: Pharmacology","Methacholine acts at which receptor? Select and explain. A. M1, B. M2, C. M3, D. M4",B: M2,"Answer: B: M2 Explanation: Ans: B. M2Methacholine is an M2 receptor agonist.ParameterM1M2M3Agonists (Relatively selective)Oxotremorine, MCN-343AMethacholineBethanecholAntagonists (Relatively selective)Pirenzepine, TelenzepineMethoctramine, TripitramineDarifenacin, Solifenacin" "This is a real-world medical entrance exam question, please give the true answer based on the question and selection. Topic name: None, Subject name: Dental","Name the curve which passes through the cusps of all the posteriors, reaches to condyle: Select and explain. A. Curve of Wilson., B. Anti monsoon curve., C. Monsoon curve., D. Curve of spee.",D: Curve of spee.,"Answer: D: Curve of spee. Explanation: None" "This is a real-world medical entrance exam question, please give the true answer based on the question and selection. Topic name: None, Subject name: Dental","4 yr old child has 0.2 ppm fluoride in drinking water. The child can be administered: Select and explain. A. 1 mg tab/day, B. Rinses 0.2% NaF, C. Topical application of fluoride, D. Fluoride containing dentifrices",A: 1 mg tab/day,"Answer: A: 1 mg tab/day Explanation: None" "This is a real-world medical entrance exam question, please give the true answer based on the question and selection. Topic name: None, Subject name: Social & Preventive Medicine","Which or tile following is true about antigenic drift? Select and explain. A. It is seen only in influenza virus type A, B. It is a result of framesshift mutation, C. It mostly affects the matrix protein, D. It mostly affects the matrix protein",D: It mostly affects the matrix protein,"Answer: D: It mostly affects the matrix protein Explanation: Ans: D. It mostly affects the matrix protein(Ref: Park 24/e p164, 23/e p154, 22/e p145)Antigenic drift:Mechanism for variation in viruses that involves the accumulation of mutations within genes that code for antibody binding sites.Results from genetic recombination of human with animal or an virus, providing a major antigenic change.This can cause a major epidemic or pandemic involving most or all age groups.Antigenic drift involves point mutation in the genes owing to selection pressure by immunity in the host population.This results in a new strain of virus paicles, which cannot be inhibited as effectively by the antibodies that were originally targeted against previous strains, making it easier for the virus to spread throughout a paially immune population, responsible for seasonal epidemics of influenza." "This is a real-world medical entrance exam question, please give the true answer based on the question and selection. Topic name: None, Subject name: Dental","Most commonly used technique for determination of working length of a 8 year old child patient of teeth maxillary central incisor: Select and explain. A. Tactile sensation, B. Moisture on paperpoint, C. Digital, D. None",B: Moisture on paperpoint,"Answer: B: Moisture on paperpoint Explanation: As 8 year old patient has central incisor with open apex, hence paper points are most commonly used method." "This is a real-world medical entrance exam question, please give the true answer based on the question and selection. Topic name: None, Subject name: Surgery","A Iaday 35 years old lactating mother Presented with a Painful breast lump. Most appropriate initial investigation should be: Select and explain. A. Mammography, B. USG, C. MRI, D. X-ray",B: USG,"Answer: B: USG Explanation: Ans. b. USG(Ref Sutton's Radiology 7/e p1456; Schwaz 9/e p467; Sabiston 19/e p2035-p846) Indications of Breast Ultrasound: ,Itton's Radiology 7/e p1456)Symptomatic breast lump in women < 35 yearsBreast lump during lactation and pregnancyAssessment of mammographic abnormality (+- fuher mammographic views)Assessment of MRI or scintimammography detected lesionsClinical breast mass with negative mammogramBreast inflammationAugmented breast (together with MRI)" "This is a real-world medical entrance exam question, please give the true answer based on the question and selection. Topic name: None, Subject name: Pharmacology","Which of the following is caused by Amphotericin B Select and explain. A. Hypo kalemia, B. Hyperkalemia, C. Hypermagnesemia, D. Hyponatremia",A: Hypo kalemia,"Answer: A: Hypo kalemia Explanation: None" "This is a real-world medical entrance exam question, please give the true answer based on the question and selection. Topic name: None, Subject name: Dental","The most common curvature of the palatal root of maxillary first molar is: Select and explain. A. Facial, B. Lingual, C. Distal, D. Mesial",A: Facial,"Answer: A: Facial Explanation: None" "This is a real-world medical entrance exam question, please give the true answer based on the question and selection. Topic name: None, Subject name: Pathology","A new marker for mantle cell lymphoma especially useful in Cyclin DI negative cases is: Select and explain. A. SOX 11, B. Annexin V, C. MYD88, D. ITRA I",A: SOX 11,"Answer: A: SOX 11 Explanation: Answer- A. SOX 11SOXII expression h a highly specific for mantle cetl lymphoma and identifies the cyclin Dl-negative subtype.Mantle Cell Lymphoma (MCL)Immunophenotype:CD20 CD79a & PAX-5 positive establish B lineageCD5, CD43, FMC-7, cyclin Dl & SOX11 positiveSurface IgM & IgD positiveCDS+ve & CD23 -ve which help to distinguish it from CLL/SLL." "This is a real-world medical entrance exam question, please give the true answer based on the question and selection. Topic name: None, Subject name: Anatomy","Type of skull seen in Beckwith hypoglycemic syndrome and fanconi syndrome: Select and explain. A. Platycephaly., B. Microcephaly., C. Anencephaly., D. Brachcephaly.",B: Microcephaly.,"Answer: B: Microcephaly. Explanation: None" "This is a real-world medical entrance exam question, please give the true answer based on the question and selection. Topic name: None, Subject name: Dental","Centre of rotation during intrusion is at: Select and explain. A. Infinity, B. Middle third of tooth, C. CEJ, D. Outside the tooth",D: Outside the tooth,"Answer: D: Outside the tooth Explanation: None" "This is a real-world medical entrance exam question, please give the true answer based on the question and selection. Topic name: None, Subject name: Pathology","Koebner phenomenon is seen in Select and explain. A. Impetigo, B. Pemphigoid, C. Erythema multiforme, D. Psoriasis",D: Psoriasis,"Answer: D: Psoriasis Explanation: None" "This is a real-world medical entrance exam question, please give the true answer based on the question and selection. Topic name: None, Subject name: Dental","Most abundant cells in Cell rich zone Select and explain. A. Fibroblasts, B. Odontoblasts, C. Ameloblasts, D. Undifferentiated mesencymal cells",A: Fibroblasts,"Answer: A: Fibroblasts Explanation: None" "This is a real-world medical entrance exam question, please give the true answer based on the question and selection. Topic name: None, Subject name: Physiology","Countercurrent mechanism is not seen in: Select and explain. A. Kidney, B. Testes, C. Eye, D. Intestine",C: Eye,"Answer: C: Eye Explanation: Ans: C. Eye(Ref Ganong 25/e p417, 420, 685, 24/e p687, 419, 422) .Countercurrent mechanism:Not seen in eye.Inflow runs parallel to, counter to & in close proximity to outflow for some distance.Seen in kidney, limbs, testis & intestine.In renal system - Both loops of Henle & vasa recta in renal medulla.Intestine - Villi where oxygen directly diffuses from aerioles to vein." "This is a real-world medical entrance exam question, please give the true answer based on the question and selection. Topic name: None, Subject name: Skin","Which component of cement causes allergic contact dermatitis?? Select and explain. A. Cobalt, B. Nickel, C. Iron, D. Chromium",D: Chromium,"Answer: D: Chromium Explanation: Ans : D: ChromiumMost common allergen is hexavalent chromium salts.Allergic contact dermatitis more than irritant contact dermatitis.Level of exposure needed for allergic contact dermatitis 100-400 nglcm2.Taarate, glycine, silicone, dapsone, ferrous sulphate causes conversion from chromium (VI) to chromium (III), which is less allergenic." "This is a real-world medical entrance exam question, please give the true answer based on the question and selection. Topic name: None, Subject name: Pharmacology","A bank employee felt depressed with no interest in activities came to AIIMS OPD. He was staed on Escitalopram. Which of these adverse effects cannot be explained with escitalopram? Select and explain. A. Vivid dreaming, B. Anorgasmia, C. Sialorrhea, D. Nausea",C: Sialorrhea,"Answer: C: Sialorrhea Explanation: Ans. C. SialorrheaEscitalopram belongs to a class of drugs known as selective serotonin reuptake inhibitors (SSRI).Improves your energy levels and feelings of well-being and decrease nervousness.Used to treat depression and anxiety.MOA: Works by helping to restore the serotonin balance in the brain.S/E:Headache, Nausea, Ejaculation disorder, Somnolence, Insomnia, Dry mouth, Constipation, Fatigue, Libido decreased, Inability to achieve orgasm, Gas (flatulence), Toothache, Weight gain, Menstrual disorder, Neck/shoulder pain, Runny nose & Flu-like syndrome." "This is a real-world medical entrance exam question, please give the true answer based on the question and selection. Topic name: None, Subject name: Anatomy","True statements about osteoblasts are all except: Select and explain. A. Derived from osteoprogenitor cells, B. Regulated by BM P, C. Have a plasma membrane showing multiple folds, D. Have neuropeptide receptors",C: Have a plasma membrane showing multiple folds,"Answer: C: Have a plasma membrane showing multiple folds Explanation: Ans: C. Have a plasma membrane showing multiple folds(Ref Gray's 40/e p87, 88, 91)Characteristic of osteoclasts:Plasma membrane showing multiple folds.I.e. ruffled borders are involved in osteocytic resorption.Osteoclasts:Large (40 or more) polymorphic cells.Have upto 20 oval, closely packed nuclei.Howship's lacunae - Lie in close contact with bone surface in resorption bays.Cytoplasm:Contains numerous coated transpo vesicles & microtubule arrays.Involved in vesicles transpo between Golgi stacks & ruffled membrane.Ruffled membrane - Highly infolded cell surface of active osteoclasts at local bone resorption sites.Osteoblasts:Derived from osteoprogenitor (stem) cells of mesenchymal origin.Present in bone marrow & other connective tissues.Proliferate & differentiate after stimulated by bone morphogenetic proteins (BMPs a into osteoblasts prior to bone formation." "This is a real-world medical entrance exam question, please give the true answer based on the question and selection. Topic name: None, Subject name: Physiology","Heart muscle, true are all except: Select and explain. A. Act as syncitium, B. Has multiple nuclei, C. Has gap junctions, D. Has branching",B: Has multiple nuclei,"Answer: B: Has multiple nuclei Explanation: None" "This is a real-world medical entrance exam question, please give the true answer based on the question and selection. Topic name: None, Subject name: Biochemistry","IDL has Select and explain. A. Apo B100 and Apo E, B. Apo C, C. Apo E, D. All of the above",A: Apo B100 and Apo E,"Answer: A: Apo B100 and Apo E Explanation: None" "This is a real-world medical entrance exam question, please give the true answer based on the question and selection. Topic name: None, Subject name: Surgery","Fracture of mandible having mandibular height 8 mm, line of treatment: Select and explain. A. Champy's plate, B. Lag screw, C. Reconstruction plate, D. All of the above",C: Reconstruction plate,"Answer: C: Reconstruction plate Explanation: As reconstruction plate is load bearing plate." "This is a real-world medical entrance exam question, please give the true answer based on the question and selection. Topic name: None, Subject name: Social & Preventive Medicine","Natural disaster causing maximum deaths Select and explain. A. Hydrological, B. Meterological, C. Geological, D. Fires",A: Hydrological,"Answer: A: Hydrological Explanation: Ans is 'a' i.e. Hydrological DisasterA ""disaster"" can be defined as 'any occurrence that causes damage, ecological disruption, loss of human life or deterioration of health and health services on a scale stiffly-lent to warrant an extraordinary response from outside the affected community or area'.* A ""hazard"" can be defined as any phenomenon that has the potential to cause disruption or damage to people and their environment. - There are many types of disasters such as eahquakes, cyclones, floods, tidal waves, landslides, volcanic eruptions, fires, snowstorms, building collapse, toxicologic accidents, and warfare, etc.* Hydrological disasters (i.e. floods, tsunamis) are associated with causing a maximum death toll. Geological disasters: includes Eahquakes, Avalanches & Volcanic eruptionsMeteorological disasters: include Cyclones, Tornadoes, Hailstorms, Drought, Heatwaves, Blizzards (Blizzards are severe winter storms characterized by low temperature, strong winds, and heavy snow)." "This is a real-world medical entrance exam question, please give the true answer based on the question and selection. Topic name: None, Subject name: Surgery","A young healthy male patient presented with abdominal pain and history of altered bowel habits from the last 6 months. On CT examination, there was dilated distal pa of ileum, thickened ileocecal junction with thickened cecum with presence of sacculations on the antimesenteric border. The vascularity of adjoining mesentery is also increased and there is surrounding mesentery fat. Which of the following is not a differential diagnosis? Select and explain. A. Ulcerative colitis, B. Crohn's disease, C. Tuberculosis, D. Ischemic bowel disease",D: Ischemic bowel disease,"Answer: D: Ischemic bowel disease Explanation: Answer- D. Ischemic bowel diseaseCT scans associated with bowel ischemia include dilation of the bowel lumen, bowel wall thickening abnormal bowel wall enhancement, aerial occlusion, venous thrombosis, and intramural or poal venous gasDilation of an ischemic bowel segment suggests interruption of normal peristaltic activity.Symmetrical bowel wall thickening greater than 3 mm in a distended segment." "This is a real-world medical entrance exam question, please give the true answer based on the question and selection. Topic name: None, Subject name: Dental","Kroll designed Select and explain. A. RPI clasp, B. Bar clasp, C. Circumferential clasp, D. Altered clasp",A: RPI clasp,"Answer: A: RPI clasp Explanation: None" "This is a real-world medical entrance exam question, please give the true answer based on the question and selection. Topic name: None, Subject name: Radiology","If multiple myeloma is suspected in a patient's history and intraoral radiograph, which of the following radiographs should be taken to confirm the diagnosis? Select and explain. A. Lateral skull, B. Anterior posterior view, C. Lateral oblique, D. Posterior anterior view",A: Lateral skull,"Answer: A: Lateral skull Explanation: None" "This is a real-world medical entrance exam question, please give the true answer based on the question and selection. Topic name: None, Subject name: Surgery","All of the following are signs of respiratory insufficiency except: Select and explain. A. Hypoxia, B. Inability to speak, C. Strider during inspiration, D. All of the above",D: All of the above,"Answer: D: All of the above Explanation: Ans: D. All of the above(Ref Bailey 27/e p929-925, 26/e p303: Sobiston 20/e p557: Harrison 19/e p1661, 1731, 1732).Respiratory Insufficiency:Inability of lungs to function properly & maintain normal processes of oxygen uptake & carbon dioxide removal.Sign & symptoms of respiratory insufficiency:Fatigue, shoness of breath, heavy breathing, rapid breathing, exercise intolerance, hypoxia, inability to speak & stridor during inspiration." "This is a real-world medical entrance exam question, please give the true answer based on the question and selection. Topic name: None, Subject name: Surgery","Eutectic mixture of 2.5% lignocaine and 2.5% prilocaine is used for Select and explain. A. Gow gates technique for mandibular nerve block, B. Gasserion ganglion block, C. Intraputpal anesthesia, D. For anesthetizing intact mucosa",D: For anesthetizing intact mucosa,"Answer: D: For anesthetizing intact mucosa Explanation: None" "This is a real-world medical entrance exam question, please give the true answer based on the question and selection. Topic name: None, Subject name: Pathology","Primordial cyst develops Select and explain. A. In place of missing teeth, B. In teeth in which crown development is completed, C. In periapical region, D. In mandibular body",A: In place of missing teeth,"Answer: A: In place of missing teeth Explanation: None" "This is a real-world medical entrance exam question, please give the true answer based on the question and selection. Topic name: None, Subject name: Biochemistry","Precipitation of proteins is done by all of these except: Select and explain. A. Adding trichloroacetic acid., B. Adding acetyl alcohol and acetone., C. Adjusting pH to other than the isoelectric point., D. Salts of heavy metals.",C: Adjusting pH to other than the isoelectric point.,"Answer: C: Adjusting pH to other than the isoelectric point. Explanation: Ans: C. Adjusting pH to other than the isoelectric pointProtein precipitation is widely used to concentrate proteins and purify them from various contaminants.Protein precipitation occurs at the isoelectric point of the protein and not at any other pH.Selective precipitation exploits differences in relative solubility of individual proteins as a function of pH (isoelectric precipitation), polarity (precipitation with ethanol or acetone), or salt concentration (salting out with ammonium sulfate)." "This is a real-world medical entrance exam question, please give the true answer based on the question and selection. Topic name: None, Subject name: Pharmacology","Which of the following is not true for rifabutin as compared to rifampicin? Select and explain. A. Rifabutin has the longer half-life than rifampicin, B. Rifabutin has lesser incidence of drug-interactions, C. Rifabutin is more efficacious against MAC as compared to rifampicin, D. Rifabutin is more efficacious for pulmonary TB as compared to rifampicin",D: Rifabutin is more efficacious for pulmonary TB as compared to rifampicin,"Answer: D: Rifabutin is more efficacious for pulmonary TB as compared to rifampicin Explanation: Ans: D. Rifabutin is more efficacious for pulmonary TB as compared to rifampicin(Ref Goodman Gilman 12/e p1550, 1552, 1554, 1568; Katzung 12/e p845)Rifabutin:Stronger activity against Mycobacterium avium complex (compared to rifampicin).Ineffective against Mycobacterium tuberculosis. RifabutinRifampicin (Rifampin)RifapentineProtein binding 718597Oral 2068deg-tmax (hours)2.5-4.01.5-2.05.0-6.0degHalf-life (hours)32-67deg2-5deg14-18Intracellular/extracellular9524-60degAutoinduction40%deg38%20%CYP3A inductionWeakPronouncedModerateCYP3A substrateYesdegNoNo" "This is a real-world medical entrance exam question, please give the true answer based on the question and selection. Topic name: None, Subject name: Dental","Which of the following is most stable restoration for a G.V. Black class V unstable erosive lesion? Select and explain. A. Acid etch composite, B. GIC, C. Resin modified GIC, D. Compomer",C: Resin modified GIC,"Answer: C: Resin modified GIC Explanation: None" "This is a real-world medical entrance exam question, please give the true answer based on the question and selection. Topic name: None, Subject name: Pathology","What is seen in syndrome X? Select and explain. A. Prediabetics, B. Prehypertensive, C. X chromosome mutation, D. More chances of syncope during dental extractions",A: Prediabetics,"Answer: A: Prediabetics Explanation: Syndrome X is a term used as a placeholder for a newly discovered group of symptoms. It may refer to: Cardiac syndrome X Metabolic syndrome Metabolic syndrome (Formerly known as syndrome x) can dramatically increase your risk of serious health problems, such as diabetes, heart attacks, and strokes yet often people don't even know what it is. Metabolic syndrome is generally defined as a cluster of risk factors, including high blood sugar, extra abdominal fat, high blood pressure, and unhealthy cholesterol levels. Insulin resistance. Insulin is used to metabolize glucose. In people with insulin resistance, there is resistance against insulin action, due to which the body keeps making more and more of it to cope with the rising level of glucose. Eventually, this can lead to diabetes." "This is a real-world medical entrance exam question, please give the true answer based on the question and selection. Topic name: None, Subject name: Dental","The stiffness of the bristles of a nylon toothbrush is dependent on the: Select and explain. A. Diameter and length of filament, B. Amount of polish of filament, C. Colour of filament, D. Consistency of shape of the filament",A: Diameter and length of filament,"Answer: A: Diameter and length of filament Explanation: None" "This is a real-world medical entrance exam question, please give the true answer based on the question and selection. Topic name: None, Subject name: Radiology","Campbell's lines are seen in: Select and explain. A. Transorbital, B. Transpharyngeal, C. Occipitomental, D. Transcranial",C: Occipitomental,"Answer: C: Occipitomental Explanation: None" "This is a real-world medical entrance exam question, please give the true answer based on the question and selection. Topic name: None, Subject name: Dental","Free gingival groove represents Select and explain. A. Histologic depth of gingival sulcus, B. False pocket depth, C. Depth of gingival sulcus, D. None of the above",A: Histologic depth of gingival sulcus,"Answer: A: Histologic depth of gingival sulcus Explanation: None" "This is a real-world medical entrance exam question, please give the true answer based on the question and selection. Topic name: None, Subject name: Biochemistry","All of the following are true regarding oxygenases except : Select and explain. A. Incorporate 2 atoms of oxygen, B. Incorporate 1 atom of oxygen, C. Required for hydroxylation of steroids, D. Required for carboxylation of drugs",D: Required for carboxylation of drugs,"Answer: D: Required for carboxylation of drugs Explanation: Ans. d. Required for carboxylation reaction (Ref Harper 2R/e p9R-102: Lehninger 5/e p8l 6).Oxygenase (which belongs to oxido-reductase class) cause incorporation of 2 atoms of 0, (dioxygenase) or 1 atom of 02 (monoxygenase or hydroxylase). Carboxylase enzyme, which belongs to ligase class is required for carboxylation.Cytochromes P450 are (heme containing) monoxygenasesdeg which take pa in hydroxylase cycledeg (NADH, NADPH and Fe2 S2 required), detoxify drugs in liver microsomesdeg (with cytochrome b5, Fe2 S2 not required)." "This is a real-world medical entrance exam question, please give the true answer based on the question and selection. Topic name: AIIMS 2019, Subject name: Physiology","In females adrenal glands are the exclusive source for secretion of? Select and explain. A. Estrogen, B. Testosterone, C. DHEAS, D. Progesterone",C: DHEAS,"Answer: C: DHEAS Explanation: Adrenal Androgens:- Adrenal gland exclusively secretes DHEA (DEHYDROEPIANDROSTERONE) from adrenal coex, especially during fetal life. It also secretes progesterone & oestrogens in minute quantity, which are female sex hormones, but they primarily come from ovaries. Testosterone in females comes from both ovaries and adrenal glands, but majority from ovaries." "This is a real-world medical entrance exam question, please give the true answer based on the question and selection. Topic name: None, Subject name: Pharmacology","What is the dose of adrenaline in anaphylactic shock? Select and explain. A. 0.5 ml in 1:1000, B. 0.5 ml in 1:10000, C. 1 ml in 1:1000, D. 1.5 ml in 1:1000",A: 0.5 ml in 1:1000,"Answer: A: 0.5 ml in 1:1000 Explanation: Ans. A. 0.5 ml in 1:1000Severe hypersensitivity reactions, anaphylactic shockIM Injection:* Adults: The usual dose is 500 micrograms (0.5ml of adrenaline 1/1000). If necessary, this dose may be repeated several times at 5-minute intervals according to blood pressure, pulse and respiratory function.* Half doses of adrenaline may be safer for patients who are taking amitriptyline, imipramine or a beta blocker.Paediatric population:* The following doses of adrenaline 1/1,000 are recommended:AgeDoseOver 12 years0.5 mg IM (0.5ml 1:1000 solution)6 - 12 years0.3 mg IM (0.3ml 1:1000 solution)6 months - 6 years0.15 mg IM (0.15ml 1:1000 solution)Under 6 months0.01mg/kg IM (0.01ml/kg 1:1000 solution)* If necessary, these doses may be repeated at 5-15 -minute intervals according to blood pressure, pulse and respiratory function." "This is a real-world medical entrance exam question, please give the true answer based on the question and selection. Topic name: None, Subject name: Surgery","Which combination forms day care anesthesia Select and explain. A. Fentanyl, propofol, Isoflurane, B. Pethidine, Propofol, Isoflurane, C. Thiopental pethidime, Halothane, D. Thiopentane, isoflurane fantanyt","A: Fentanyl, propofol, Isoflurane","Answer: A: Fentanyl, propofol, Isoflurane Explanation: None" "This is a real-world medical entrance exam question, please give the true answer based on the question and selection. Topic name: None, Subject name: Dental","CEREC AC which is true Select and explain. A. Creates the image by video recording, B. Laser beam is used, C. LED is used, D. Composite metal resin alloy etc",C: LED is used,"Answer: C: LED is used Explanation: None" "This is a real-world medical entrance exam question, please give the true answer based on the question and selection. Topic name: None, Subject name: Pharmacology","All are used for postpaum hemorrhage except - Select and explain. A. Misoprostol, B. Dinoprostone, C. Prostaglandin F2 alpha, D. Oxytocin",B: Dinoprostone,"Answer: B: Dinoprostone Explanation: Ans. B. DinoprostonePostpaum hemorrhage, the loss of more than 500 mL of blood after delivery, occurs in up to 18 percent of bihs and is the most common maternal morbidity in developed countries. Dinoprostone:Naturally occurring prostaglandin E2 (PGE2). It has impoant effects in labour.It also stimulates osteoblasts to release factors which stimulates bone resorption by osteoclasts.As a prescription drug it is used as a vaginal suppository, to prepare the cervix for labour and to induce labour. Indication:For the termination of pregnancy during the second trimester (from the 12th through the 20th gestational week as calculated from the first day of the last normal menstrual period), as well as for evacuation of the uterine contents in the management of missed aboion or intrauterine fetal death up to 28 weeks of gestational age as calculated from the first day of the last normal menstrual period.In the management of nonmetastatic gestational trophoblastic disease (benign hydatidiform mole).Other indications include improving the cervical inducibility (cervical ""ripening"") in pregnant women at or near term with a medical or obstetrical need for labor induction, and the management of postpaum hemorrhage." "This is a real-world medical entrance exam question, please give the true answer based on the question and selection. Topic name: None, Subject name: Dental","Ratio of polymer to monomer in cold cure resin Select and explain. A. 3:1 by volume and 2:1 by weight, B. 3:1 by weight and 2:1 by volume, C. 2:1 by volume and 2:1 by weight, D. 3:1 by volume and 3:1 by weight",A: 3:1 by volume and 2:1 by weight,"Answer: A: 3:1 by volume and 2:1 by weight Explanation: When the powder and liquid components are mixed in the proper proportions, a doughlike mass results. Commercial products generally use a polymer-to-monomer ratio of approximately 3:1 by volume." "This is a real-world medical entrance exam question, please give the true answer based on the question and selection. Topic name: None, Subject name: Dental","According to WHO classification of periapical pathology, 4.81 means: Select and explain. A. Periapical cyst, B. Lateral cyst, C. Residual cyst, D. Inflammatory periodontal cyst",C: Residual cyst,"Answer: C: Residual cyst Explanation: WHO Classification: Ko 4.4 Acute Apical Periodontitis Ko 4.5 Chronic Apical Periodontitis Ko 4.6 Periapical Abscess with sinus formation 4.60 Sinus to maxillary antrum 4.61 Sinus to nasal floor 4.62 Sinus to oral cavity 4.63 Sinus to outer skin of face Ko 4.7 Periapical Abscess without sinus Ko 4.8 Periapical cyst 4.80 Apical / lateral cyst 4.81 Residual cyst 4.82 Inflammatory paradental cyst." "This is a real-world medical entrance exam question, please give the true answer based on the question and selection. Topic name: None, Subject name: Dental","Termination of diseases by extermination of micro organisms is? Select and explain. A. Eradication, B. Control, C. Elimination, D. Regional elimination",A: Eradication,"Answer: A: Eradication Explanation: None" "This is a real-world medical entrance exam question, please give the true answer based on the question and selection. Topic name: AIIMS 2019, Subject name: Biochemistry","Which TCA intermediate is used in haem synthesis: Select and explain. A. Fumarate, B. Alpha keto gluterate, C. Malate, D. Succinyl CoA",D: Succinyl CoA,"Answer: D: Succinyl CoA Explanation: Solutions: - The first step of haem synthesis uses succinyl CoA which is intermediate to TCA. Fumarate, Alpha keto gluterate and malate (4C) is also intermediate to TCA. Malonate (3C) or malonyl CoA is inhibitor of 3 pathways in body - Succinate DH of TCA - Complex 2 of ETC - Beta oxidation of Fatty acid Acetyl CoA is not an intermediate of TCA. Haem synthesis It occurs in Mitochondria + cytoplasm (So, doesn't occurs in RBC)" "This is a real-world medical entrance exam question, please give the true answer based on the question and selection. Topic name: None, Subject name: Medicine","Presence of glucose in urine indicates: Select and explain. A. Patient suffers from diabetes insipidus, B. Patient has consumed excess sugar over long years, C. Renal threshold for glucose is exceeded, D. None of the above",C: Renal threshold for glucose is exceeded,"Answer: C: Renal threshold for glucose is exceeded Explanation: None" "This is a real-world medical entrance exam question, please give the true answer based on the question and selection. Topic name: AIIMS 2019, Subject name: Pathology","What among the following is the use of this? Select and explain. A. Prevent viral infections, B. Prevent transfusion related reactions, C. Prevent bacterial contamination, D. Prevent blood mismatch",B: Prevent transfusion related reactions,"Answer: B: Prevent transfusion related reactions Explanation: The picture given above shows the WBC filter. | WBC's entering the body of the recipient | are the chances of development of transfusion related reactions. Purpose of having this kind of filter is that: - lesser chances of entry of WBC inside the body of the host. - Prevent the entry of (Cell) aggregates inside the body of the recipient. The size of the WBC filters can range from 170- 200m. The size of the transfusion needle is 18-19 gauge." "This is a real-world medical entrance exam question, please give the true answer based on the question and selection. Topic name: None, Subject name: Physiology","A cell membrane is damaged by inseion of a micro needle, repair shall occur by which of the following process? Select and explain. A. Lateral movement of proteins, B. Enzyme catalysed process, C. Hydrophobic interactions, D. An active process involving hydrolysis of ATP",A: Lateral movement of proteins,"Answer: A: Lateral movement of proteins Explanation: Ans. a. Lateral movement of proteins (Ref Harper 29/e p464, Ganong 24/e p36)Harper says 'Another indicator of the dynamic nature of membranes is that a variety of studies have shown that lipids and ceain proteins exhibit lateral diffusion in the plane of their membranes" "This is a real-world medical entrance exam question, please give the true answer based on the question and selection. Topic name: None, Subject name: Surgery","The common complication of the local anesthetic prilocaine is: Select and explain. A. Agranulocytosis, B. Hepatic dysfunction, C. Methemoglobinemia, D. Loss of taste",C: Methemoglobinemia,"Answer: C: Methemoglobinemia Explanation: None" "This is a real-world medical entrance exam question, please give the true answer based on the question and selection. Topic name: None, Subject name: Dental","Cross sectional from and dimension of lingual bar major connector is: Select and explain. A. 4 gauge full pear shape, B. 6 gauge half pear shape, C. 6 gauge round shape, D. gauge flat",B: 6 gauge half pear shape,"Answer: B: 6 gauge half pear shape Explanation: None" "This is a real-world medical entrance exam question, please give the true answer based on the question and selection. Topic name: AIIMS 2019, Subject name: Microbiology","A 22 year old college boy with history of sex with commercial workers came to Derma OPD with lesion in genital region, tissue from this lesion was examined, which is the most likely finding? Select and explain. A. Intracytoplasmic vacuolations, B. Multinucleated giant cells, C. Belongs to Herpes family, D. It's a RNA virus",A: Intracytoplasmic vacuolations,"Answer: A: Intracytoplasmic vacuolations Explanation: MOLLUSCUM CONTAGIOSUM VIRUS It is an obligate human pox Virus that produces characteristic skin lesions. Clinical Manifestations Pink pearly wa-like lesions (2-5 mm size),umbilicated with a characteristic dimple at the centre. Lesions are characterized by: - Lack of associated inflammation and necrosis - Found singly or in clusters Distribution: lesions are found anywhere on the body except on palms and soles. Genital lesions are seen in adults Molluscum bodies are the Intracytoplasmic Eosinophilic inclusions seen in skin scrapings stained with histopathological stains." "This is a real-world medical entrance exam question, please give the true answer based on the question and selection. Topic name: None, Subject name: Dental","Which of the following is advantage of sub-marginal flop? Select and explain. A. Cause less gingival shrinking, B. Cause Less scars, C. Less intra-operative bleeding, D. None of the above",A: Cause less gingival shrinking,"Answer: A: Cause less gingival shrinking Explanation: None" "This is a real-world medical entrance exam question, please give the true answer based on the question and selection. Topic name: None, Subject name: Dental","Glossy smooth margins of a casting are due to: Select and explain. A. Shrinkage of alloy on cooling, B. Investment breakdown, C. Incomplete wax elimination, D. Incomplete gas elimination",C: Incomplete wax elimination,"Answer: C: Incomplete wax elimination Explanation: None" "This is a real-world medical entrance exam question, please give the true answer based on the question and selection. Topic name: None, Subject name: Dental","What is the working time of zinc polycarboxylate cement? Select and explain. A. 1 minute, B. 2 minutes, C. 3 minutes, D. 4 minutes",C: 3 minutes,"Answer: C: 3 minutes Explanation: Control of Working Time The working time for polycarboxylate cement is much shorter than that for zinc phosphate cement, approximately 2.5 versus 5 minutes.   The graph shows the relative viscosities and working times of zinc phosphate, polycarboxylate, and GICs as they set.  A cool slab lengthens the working time for zinc carboxylate cement, although it causes the polyacrylic acid to thicken, which hinders mixing.  Refrigerating the powder is useful, because it retards the reaction without raising the viscosity of the liquid. Polycarboxylate cements are more viscous than a comparable mix of GIC; however, the polycarboxylate mix undergoes thinning at an increased shear rate.  Clinically, this means that rapid spatulation and fast seating will reduce the viscosity of the polycarboxylate cement to ensure complete seating." "This is a real-world medical entrance exam question, please give the true answer based on the question and selection. Topic name: None, Subject name: Medicine","Risk factors for Coronary artery disease are all except Select and explain. A. Smoking, B. Increased fibrinogen level, C. Hypertension, D. Alcohol consumption",D: Alcohol consumption,"Answer: D: Alcohol consumption Explanation: None" "This is a real-world medical entrance exam question, please give the true answer based on the question and selection. Topic name: None, Subject name: Surgery","In esophageal varices bleeding, which of the following is not used? Select and explain. A. Endoscopy banding, B. Octreotide, C. Platelet transfusion, D. TIPS",C: Platelet transfusion,"Answer: C: Platelet transfusion Explanation: Answer- C. Platelet transfusionBeta-blockersEndoscopic sclerotherapyEndoscopic variceal ligation (banding)A medication called octreotide - will lower the pressure in the swollen veins by tightening the blood vessels and reducing blood flow.Transjugular intrahepatic poosystemic shunt (TIPS) procedure is another potential treatment option for recurrent bleeding esophageal varices" "This is a real-world medical entrance exam question, please give the true answer based on the question and selection. Topic name: None, Subject name: Physiology","Posterior pituitary insufficiency leads to: Select and explain. A. Diabetes mellitus, B. Diabetes insipidus, C. Dwarfism, D. Cretinism",B: Diabetes insipidus,"Answer: B: Diabetes insipidus Explanation: ADH SYNTHESIS IN SUPRAOPTIC AND PARAVENTRICULAR NUCLEI OF THE HYPOTHALAMUS AND ADH RELEASE FROM THE POSTERIOR PITUITARY Figure shows the neuroanatomy of the hypothalamus and the pituitary gland, where ADH is synthesized and released. The hypothalamus contains two types of magnocellular (large) neurons that synthesize ADH in the supraoptic and paraventricular nuclei of the hypothalamus, about five sixths in the supraoptic nuclei and about one sixth in the paraventricular nuclei. Both of these nuclei have axonal extensions to the posterior pituitary. The most important renal action of ADH is to increase the water permeability  of  the  distal  tubule,  collecting  tubule, and collecting duct epithelia. This effect helps the body to conserve water in circumstances such as dehydration. In the absence of ADH, the permeability of the distal tubules and collecting ducts to water is low, causing the kidneys to excrete large amounts of dilute urine, a condition called diabetes insipidus. Thus, the actions of ADH play a key role in controlling the degree of dilution or concentration of the urine." "This is a real-world medical entrance exam question, please give the true answer based on the question and selection. Topic name: None, Subject name: Gynaecology & Obstetrics","All are used in preterm labour to decrease uterine contractility except: Select and explain. A. Methyl alcohol, B. Ritodrine, C. Magnesium sulphate, D. Dexamethasone",D: Dexamethasone,"Answer: D: Dexamethasone Explanation: None" "This is a real-world medical entrance exam question, please give the true answer based on the question and selection. Topic name: None, Subject name: Pathology","Site specificity is seen in: Select and explain. A. Syphilis, B. Recurrent herpes Labialis, C. Carcinoma, D. Traumatic aphthous ulcer",B: Recurrent herpes Labialis,"Answer: B: Recurrent herpes Labialis Explanation: None" "This is a real-world medical entrance exam question, please give the true answer based on the question and selection. Topic name: None, Subject name: Dental","Remodelling theory of craniofacial growth was given by Select and explain. A. Brash, B. John hunter, C. Vander Klauuw, D. Sicher & Weinmann",A: Brash,"Answer: A: Brash Explanation: None" "This is a real-world medical entrance exam question, please give the true answer based on the question and selection. Topic name: None, Subject name: Dental","10-year-old boy visits dentist with complicated crown fracture with abscess formation in 12. X-ray reveals radiolucency in relation to 11, 12. 12 is having immature blunderbuss canal while 11 has complete root formation, vitality of 11, 21 is negative. What would be the management of this patient? Select and explain. A. Apexification of 12 and RCT of 11, B. Revascularization in 12 and RCT of 11, C. RCT of 12 and 11, D. Extraction of 12 and RCT in 11",B: Revascularization in 12 and RCT of 11,"Answer: B: Revascularization in 12 and RCT of 11 Explanation: Since 11 is mature and has complete root formation, RCT will be preferred for this tooth. Due to immature status of 12 as evident from blunder- buss canal revascularization should be attempted for this tooth." "This is a real-world medical entrance exam question, please give the true answer based on the question and selection. Topic name: None, Subject name: Dental","Lymphocytes found in the pulp proper are: Select and explain. A. T lymphocytes, B. B lymphocytes, C. Both, D. None",A: T lymphocytes,"Answer: A: T lymphocytes Explanation: Lymphocytes, both T and B, are found in cell-rich zone and function in an immune capacity of the pulp. But in pulp-proper zone, only T-lymphocytes are present." "This is a real-world medical entrance exam question, please give the true answer based on the question and selection. Topic name: None, Subject name: Pediatrics","A 2 months old child was brought to the subcenter by his mother with complaints of fever for two days. Weight of the child is 2 kg. On examination, the child is restless and irritable, skin pinch went back in 2 seconds, oral mucosa is dry and eyes were sunken. There were ten pustules on his forehead. What should be done at the subcenter? Select and explain. A. Refer to higher center with mother giving frequent sips of ORS., B. Immediately admit the child, give IV fluids and then refer to higher center., C. Give first dose of antibiotic and refer to higher center in an ambulance with sips of ORS along the way., D. Send child home with few packets of ORS and call after 3 days.",C: Give first dose of antibiotic and refer to higher center in an ambulance with sips of ORS along the way.,"Answer: C: Give first dose of antibiotic and refer to higher center in an ambulance with sips of ORS along the way. Explanation: Ans: C. Give first dose of antibiotic and refer to higher center in an ambulance with sips of ORS along the way.(Ref .Ghai 8th/e p752-753: IMNCI Guidelines).Multiple signs of possible serious bacterial infection (10 skin pustules, diarrhea with severe dehydration & weight = 2 kg which is <-3SD).Hence, child should be immediately given first dose of injectable antibiotic, kept warm & referred to hospital with sips of ORS along the way" "This is a real-world medical entrance exam question, please give the true answer based on the question and selection. Topic name: None, Subject name: Dental","In a left skewed curve, true statement is: Select and explain. A. Mean = Median, B. Mean < Mode, C. Mean >Mode, D. Mean = Mode",B: Mean < Mode,"Answer: B: Mean < Mode Explanation: None" "This is a real-world medical entrance exam question, please give the true answer based on the question and selection. Topic name: None, Subject name: Pathology","Which of the following are seen in ectodermal dysplasia? Select and explain. A. Hyperpyrexia, B. Protuberant lips and frontal bossing, C. Defective or absence of sweat glands, D. Any of the above",D: Any of the above,"Answer: D: Any of the above Explanation: Hyper pyrexia due to hypohidrosis, hypotrichosis, hypodontia, protuberant lips and depressed nasal and supraorbital bridges are important features of ectodermal dysphasia" "This is a real-world medical entrance exam question, please give the true answer based on the question and selection. Topic name: None, Subject name: Pathology","A 60-year-old has got severe bulla and target lesion which erythema around halo and genital lesions: Select and explain. A. Stevens Jhonson syndrome, B. Herpes zoster, C. Herpes simplex, D. Herpangina",A: Stevens Jhonson syndrome,"Answer: A: Stevens Jhonson syndrome Explanation: Stevens — Johnson syndrome is very severe bullous form of erythema multiformae and is characterized by oral, ocular and genital lesions." "This is a real-world medical entrance exam question, please give the true answer based on the question and selection. Topic name: AIIMS 2017, Subject name: Medicine","A patient comes to ER with headache describing it as worst headache in his life. What is the next step? Select and explain. A. CT brain, B. Lumbar puncture, C. MRI brain, D. Observation and analgesics",A: CT brain,"Answer: A: CT brain Explanation: Worst headache of my life s/o SAH Any intracranial bleeding, we need to do NCCT Shows blood in syln fissure from MCA (Middle cerebral aery) Lumbar puncture- C/I as patient may be having | ICT" "This is a real-world medical entrance exam question, please give the true answer based on the question and selection. Topic name: None, Subject name: Dental","According to Vertucci’s classification of root canal, the fig shows: Select and explain. A. Type III, B. Type IV, C. Type V, D. Type VI",C: Type V,"Answer: C: Type V Explanation: None" "This is a real-world medical entrance exam question, please give the true answer based on the question and selection. Topic name: None, Subject name: Pediatrics","In SCHWARTZ formula for calculation of creatinine clearance in a child, the constant depends on the following except – Select and explain. A. Age, B. Method of estimation of creatinine, C. Mass, D. Severity of renal failure",D: Severity of renal failure,"Answer: D: Severity of renal failure Explanation: Schwartz formula (for creatine clearance in child). Creatinine clearance = K x height./ creatinine. K = constant K depends upon --> Age, Muscle mass, Method of creatinine estimation" "This is a real-world medical entrance exam question, please give the true answer based on the question and selection. Topic name: None, Subject name: Dental","Pulpal medication and thermal protection is given by Select and explain. A. Solution liners (2-5 μm), B. Suspension Liners (25-30 μm), C. Traditional Liner (0.2-0.3 mm), D. Cement bases",C: Traditional Liner (0.2-0.3 mm),"Answer: C: Traditional Liner (0.2-0.3 mm) Explanation: None" "This is a real-world medical entrance exam question, please give the true answer based on the question and selection. Topic name: None, Subject name: Pathology","Epstein pearls are: Select and explain. A. Gingival cyst of newborn, B. Gingival cyst of adult, C. Enamel pearls, D. Epithelial rests",A: Gingival cyst of newborn,"Answer: A: Gingival cyst of newborn Explanation: None" "This is a real-world medical entrance exam question, please give the true answer based on the question and selection. Topic name: None, Subject name: Dental","White flecks yellow or brown spots on surface of teeth covering, giving papery white appearance Select and explain. A. Mottled enamel, B. Incipient caries, C. Dental stains, D. Smooth caries",A: Mottled enamel,"Answer: A: Mottled enamel Explanation: None" "This is a real-world medical entrance exam question, please give the true answer based on the question and selection. Topic name: None, Subject name: Microbiology","Adult stage of filarial worms responsible for diseases in all of the following except: Select and explain. A. Brugia malayi, B. Onchocerca volvulus, C. Mansonella ozzardi, D. Wuchereria bancrofti",B: Onchocerca volvulus,"Answer: B: Onchocerca volvulus Explanation: Ans: B. Onchocerca volvulus Adult stage of Onchocerca volvulus filarial worms does not cause any illness or disease.Onchocerciasis:Primarily affects skin, eyes & lymph nodes.In contrast to pathology in lymphatic filariasis, onchocerciasis damage elicited by microfilariae & not by adult parasites.Onchocerca microfilariae:Released from female worms causing most severe damage.Migrating microfilariae (exclusively found in interstitial fluids of skin & subdermal tissues not bloodstream).Cause changes in skin pigment & loss of elastic fibers a ""hanging groin, severe intractable & intolerable pruritus.Blindness (primarily men) develops over many years - Due to microfilariae accumulation in vitreous humor.Are not blood borne & hence, concentrate & remain in eye.Visual clouding, photophobia & ultimately retinal damage a Incurable blindness.(Ref: Harrison 19/e p1420)." "This is a real-world medical entrance exam question, please give the true answer based on the question and selection. Topic name: None, Subject name: Dental","Tooth in the mandibular arch which is most likely to be displaced due to arch size discrepancy is Select and explain. A. First molar, B. Second molar, C. First premolar, D. Second premolar",D: Second premolar,"Answer: D: Second premolar Explanation: Mandibular 2nd premolar is the most likely tooth to be displaced due to arch size discrepancy." "This is a real-world medical entrance exam question, please give the true answer based on the question and selection. Topic name: None, Subject name: Anatomy","Inferiohyroid aery is a branch ol? Select and explain. A. Thyrocervical trunk, B. ICA, C. Costocervical trunk, D. ECA",A: Thyrocervical trunk,"Answer: A: Thyrocervical trunk Explanation: Ans. A. Thyrocervical trunkRef: Grays Anatomy,4Ia ed., pg. 455458Inferior thyroid aery is branch of thyrocervical trunk of subclan aery.The thyrocervical trunk arises fiom the front of the first pa of the subclan aery near the medial border of scalenus anterior, and divides into the inferior thyroid, suprascapular and superficial cervical aeries." "This is a real-world medical entrance exam question, please give the true answer based on the question and selection. Topic name: None, Subject name: Dental","Secondary flare provides a marginal metal of: Select and explain. A. 10-20 degrees, B. 30-40 degrees, C. 140-150 degrees, D. None of the above",B: 30-40 degrees,"Answer: B: 30-40 degrees Explanation: Secondary flare: It is a flat plane superimposed peripherally to the primary flare. Advantages are: Secondary flare ensures cavosurface margins to extend into embrasures (self-cleansing area). Permits easy burnishing and finishing of the restoration. Produces more obtuse-angled and stronger cavosurface margin. In class II cast gold inlay, proximal box should have flares so as to have metal of 30-40° which is burnishable." "This is a real-world medical entrance exam question, please give the true answer based on the question and selection. Topic name: None, Subject name: Dental","A patient comes to a dentist with decayed mandibular 1st molar which involves the mesiobuccal and distobuccal cusp. The restoration of choice is: Select and explain. A. Intermediate restoration, B. Composite, C. Cast metal, D. GIC",C: Cast metal,"Answer: C: Cast metal Explanation: The cast-metal inlay is an alternative to amalgam or composite when the higher strength of a casting alloy is needed or when the superior control of contours and contacts that the indirect procedure provides is desired. The cast-metal onlay is often an excellent alternative to a crown for teeth that have been greatly weakened by caries or by large, failing restorations but where the facial and lingual tooth surfaces are relatively unaffected by disease or injury. For such weakened teeth, the superior physical properties of a casting alloy are desirable to withstand the occlusal loads placed on the restoration; also the onlay can be designed to distribute occlusal loads over the tooth in a manner that decreases the chance of tooth fracture in the future. Sturdevant operative dentistry 7th Edition, Page No:e94" "This is a real-world medical entrance exam question, please give the true answer based on the question and selection. Topic name: None, Subject name: Surgery","Marathon runner had pain in anteromedial tibia on regular walking/jogging for long hours. X ray is normal. Doctor orders a bone scan. What may be the probable diagnosis Select and explain. A. Jones fracture, B. Shin splint, C. Lisfranc fracture, D. Nutcracker",B: Shin splint,"Answer: B: Shin splint Explanation: Answer: B. Shin splintShin splints (medial tibial stress syndrome) is an inflammation of the muscles, tendons, and bone tissue around your tibia.Pain typically occurs along the inner border of the tibia, where muscles attach to the bone.Shin splint pain most often occurs on the inside edge of your tibia" "This is a real-world medical entrance exam question, please give the true answer based on the question and selection. Topic name: None, Subject name: Pathology","Radiographic finding in Pindborg tumour is Select and explain. A. burst appearance, B. Onion-peel appearance, C. Driven-snow appearance, D. Cherry-blossom appearance",C: Driven-snow appearance,"Answer: C: Driven-snow appearance Explanation: None" "This is a real-world medical entrance exam question, please give the true answer based on the question and selection. Topic name: None, Subject name: Medicine","Risk of endocarditis is more in all in all of the following lesions except: Select and explain. A. VSD., B. MR., C. AR., D. ASD.",D: ASD.,"Answer: D: ASD. Explanation: Risk of endocarditis is less in low pressure lesions like Atrial septal defect (ASD) while it is more in high pressure lesions like ventricular septal defect (VSD), mitral regurgitation and aortic regurgitation." "This is a real-world medical entrance exam question, please give the true answer based on the question and selection. Topic name: None, Subject name: Dental","The green stains frequently seen on children's teeth are caused by Select and explain. A. Materia alba, B. Enamel defects, C. Dentinal defects, D. Chromogenic bacteria",D: Chromogenic bacteria,"Answer: D: Chromogenic bacteria Explanation: None" "This is a real-world medical entrance exam question, please give the true answer based on the question and selection. Topic name: None, Subject name: Pharmacology","At low infusion rates of 3-5 mcg/kg/min, what action is produced by dopamine? Select and explain. A. Vasoconstriction, B. Increased renal blood flow, C. Increased cardiac contractility, D. Decreased blood pressure",B: Increased renal blood flow,"Answer: B: Increased renal blood flow Explanation: Ans: B. Increased renal blood flow(Ref: Goodman Gilman 12/e p355; KDT 7/e p134. 6/e p507)Action of dopamine:At low concentrations (2 to 5 mcg/kg per minute):Primarily acts on vascular D1 receptors.Mainly renal, mesenteric & coronary beds.By activating adenylyl cyclase a raising intracellular cyclic AMP concentrations.D1 receptor stimulation a Vasodilation.Increases in glomerular filtration rate, renal blood flow & Nat+ K+ - ATPase pump." "This is a real-world medical entrance exam question, please give the true answer based on the question and selection. Topic name: None, Subject name: Dental","Posselt's diagram indicates? Select and explain. A. Motion of mandible, B. Protrusive relation of central incisor, C. Antero-superior relation of condyle in articular fossa in CR, D. None of the above",A: Motion of mandible,"Answer: A: Motion of mandible Explanation: None" "This is a real-world medical entrance exam question, please give the true answer based on the question and selection. Topic name: None, Subject name: Surgery","Which is the most common site for fracture in zygomatic arch fracture? Select and explain. A. At the suture, B. Anterior to zygomaticotemporal suture, C. Posterior to zygomaticotemporal suture, D. None of the above",C: Posterior to zygomaticotemporal suture,"Answer: C: Posterior to zygomaticotemporal suture Explanation: None" "This is a real-world medical entrance exam question, please give the true answer based on the question and selection. Topic name: None, Subject name: Dental","While taking final impression tray should be seated first: Select and explain. A. Anteriorly, B. Posteriorly, C. Anteriorly or posteriorly depending upon operators choice, D. Anteriorly and posteriorly simultaneously",A: Anteriorly,"Answer: A: Anteriorly Explanation: None" "This is a real-world medical entrance exam question, please give the true answer based on the question and selection. Topic name: None, Subject name: Surgery","Following bilateral mandibular fracture in the canine region, the following muscles will tend to pull the mandible back: Select and explain. A. Genioglossus and anterior belly of digastric, B. Genioglossus and mylohyoid, C. Genioglossus and thyrohyoid, D. Genioglossus and masseter",A: Genioglossus and anterior belly of digastric,"Answer: A: Genioglossus and anterior belly of digastric Explanation: None" "This is a real-world medical entrance exam question, please give the true answer based on the question and selection. Topic name: None, Subject name: Medicine","The anemia associated with leukemia Select and explain. A. Iron deficiency, B. Megaloblastic type, C. Myelophthisic type, D. All of the above",C: Myelophthisic type,"Answer: C: Myelophthisic type Explanation: None" "This is a real-world medical entrance exam question, please give the true answer based on the question and selection. Topic name: None, Subject name: Dental","To regain arch length in anterior segment by moving incisors labially Select and explain. A. Cephalometric analysis have to be made before taking up treatment, B. Use tongue blade therapy 20 times 1 day, C. Use of 7 spring with Hawley's retainer, D. Use on anterior bite plane",A: Cephalometric analysis have to be made before taking up treatment,"Answer: A: Cephalometric analysis have to be made before taking up treatment Explanation: None" "This is a real-world medical entrance exam question, please give the true answer based on the question and selection. Topic name: None, Subject name: Dental","According to scientists, recent studies says about the pattern of enamel prism demineralization, true is: Select and explain. A. Head and tail both are resistant to dissolution, B. Head and tail both are not resistant to dissolution, C. Head is more resistant to dissolution, D. Tail is more resistant to dissolution",D: Tail is more resistant to dissolution,"Answer: D: Tail is more resistant to dissolution Explanation: Enamel is mainly composed of millions of enamel rods or prisms, sheaths and a cementing inter rod substance. Each rod has a head and tail.  The head is directed occlusally and the tail is directed cervically.  Basically, acid etching creates a 5 to 50 micron deep microporous layer into which adhesive resin flows.  This results in a long-lasting enamel bond achieved via micromechanical interlocking between the resin and enamel. Etching produces irregular and pitted surface with numerous microscopic undercuts with uneven dissolution of heads and tails, the latter being more resistant. Ref: Textbook of Operative Dentistry, Nisha and Amit Garg, 3rd edition, Pg no:28,237" "This is a real-world medical entrance exam question, please give the true answer based on the question and selection. Topic name: None, Subject name: Social & Preventive Medicine","Central value of a set of 180 values can be obtained by Select and explain. A. 2nd tertile, B. 90th percentile, C. 2nd quartile, D. 9th decile",C: 2nd quartile,"Answer: C: 2nd quartile Explanation: Quartile: Divides a distribution into 4 equal parts, so the number of intercepts required will be 3, i.e. Q1, Q2, Q3 – So, Zero – Q1 covers 25% values – Similarly, Q2 – Q1, Q3 – Q2 and 100 – Q3 all cover 25% values each – Thus, Q2 – Zero, 100 – Q2 and Q3 – Q1 all cover 50% values each – Q1 divides a distribution in a ratio of 25 : 75 OR 1 : 3 – Q2 divides a distribution in a ratio of 50 : 50 OR 1 : 1, Thus second quartile is equivalent to median – Q3 divides a distribution in a ratio of 75 : 25 OR 3 : 1. In the given question, n = 180 Thus Q2 which is equivalent to median, divides a distribution in a ratio of 50 : 50 OR 1 : 1." "This is a real-world medical entrance exam question, please give the true answer based on the question and selection. Topic name: None, Subject name: Dental","In order to reduce non-working side interference, reduce Select and explain. A. Maxillary supporting cusp, B. Mandibular supporting cusp, C. Both maxillary and mandibular supporting cusps, D. Either of the jaw’s supporting cusps",D: Either of the jaw’s supporting cusps,"Answer: D: Either of the jaw’s supporting cusps Explanation: None" "This is a real-world medical entrance exam question, please give the true answer based on the question and selection. Topic name: None, Subject name: Surgery","What will be treatment plan for a parasymphysial fracture in a 55 year old man. Select and explain. A. IMF for 3 weeks, B. IMF for 6 weeks, C. Compression plate is must, D. Non union will occur if reconstruction plate is not given",B: IMF for 6 weeks,"Answer: B: IMF for 6 weeks Explanation: None" "This is a real-world medical entrance exam question, please give the true answer based on the question and selection. Topic name: None, Subject name: Dental","ADA sp. Number fir dental excavating burs: Select and explain. A. 20, B. 21, C. 22, D. 23",D: 23,"Answer: D: 23 Explanation: None" "This is a real-world medical entrance exam question, please give the true answer based on the question and selection. Topic name: None, Subject name: Dental","False about enamel rods is Select and explain. A. They are perpendicular to tooth surface, B. They are parallel to each other, C. Diameter of enamel rods is 30micrometer at DEJ, D. Diameter of enamel rod is 5 micrometer at DEJ",C: Diameter of enamel rods is 30micrometer at DEJ,"Answer: C: Diameter of enamel rods is 30micrometer at DEJ Explanation: Option C is false as diameter of enamel rod (avg) is 5 micrometer at DEJ and 8 micrometer at surface." "This is a real-world medical entrance exam question, please give the true answer based on the question and selection. Topic name: None, Subject name: Radiology","Radiographs are of no help in the diagnosis of Select and explain. A. Cementoma, B. Subluxation teeth, C. Acute alveolar abscess, D. Impacted canines",C: Acute alveolar abscess,"Answer: C: Acute alveolar abscess Explanation: None" "This is a real-world medical entrance exam question, please give the true answer based on the question and selection. Topic name: None, Subject name: Medicine","CPR should have a ratio of chest compression to mouth breathing of: Select and explain. A. 1:04, B. 4:01, C. 2:03, D. 3:02",B: 4:01,"Answer: B: 4:01 Explanation: None" "This is a real-world medical entrance exam question, please give the true answer based on the question and selection. Topic name: None, Subject name: Dental","The use of stress breaker in modern dentistry is avoided because Select and explain. A. It affects abutment more, B. It affects alveolar ridge, C. It affects both abutment and alveolar ridge, D. None of the above",B: It affects alveolar ridge,"Answer: B: It affects alveolar ridge Explanation: None" "This is a real-world medical entrance exam question, please give the true answer based on the question and selection. Topic name: None, Subject name: Pediatrics","Which of the following does not complicate into CHF – Select and explain. A. Co–relation of aorta, B. Transposition of great vessels, C. Tetralogy of fallot's, D. Patent ductus arteriorus",C: Tetralogy of fallot's,"Answer: C: Tetralogy of fallot's Explanation: The ventricular septal defect of TOF is always large enough to allow free exit to the right to left shunt. Since the right ventricle is effectively decompressed by VSD, Congestive cardiac failure never occurs in TOE Exceptions to this rule are : - Anemia                                                                                                      Infective endocarditis iii) Systemic hypertension iv) Myocarditis complicating TOF V) Aortic or pulmonic regurgitation. Remember: Following do not occur in TOF -->       o Cardiomegaly          o CHF              o Recurrent chest infections" "This is a real-world medical entrance exam question, please give the true answer based on the question and selection. Topic name: None, Subject name: Dental","Leeway space is due to ? Select and explain. A. Space differential between deciduous canine and molar and their succedaneous permanent teeth, B. Space differential between deciduous incisors and their succedaneous permanent teeth, C. Difference between deciduous and permanent maxillary and mandibular canines only, D. Difference between deciduous and permanent maxillary and mandibular molars only",A: Space differential between deciduous canine and molar and their succedaneous permanent teeth,"Answer: A: Space differential between deciduous canine and molar and their succedaneous permanent teeth Explanation: When the primary molars and canine are exfoliated the combined width of primary molars and canine are significantly larger than the premolars and permanent canine that replace them, and the “leeway space” provided by this difference offers an excellent opportunity of natural or orthodontic adjustment of occlusal relationships at the end of the dental transition" "This is a real-world medical entrance exam question, please give the true answer based on the question and selection. Topic name: None, Subject name: Anatomy","All are sensory to the palate except: Select and explain. A. Maxillary division of the trigeminal nerve, B. Facial nerve, C. Glossopharyngeal nerve, D. Hypoglossal nerve",D: Hypoglossal nerve,"Answer: D: Hypoglossal nerve Explanation: None" "This is a real-world medical entrance exam question, please give the true answer based on the question and selection. Topic name: None, Subject name: Physiology","According to Weber-Fechner's law, strength of stimulus perceived is directly propoional to: Select and explain. A. Intensity of stimulus, B. Amplitude of action potential, C. Number of neurons stimulated, D. Number of receptors stimulated",A: Intensity of stimulus,"Answer: A: Intensity of stimulus Explanation: Ans: A. Intensity of stimulus(Ref: Guyton 13/e p615)According to Weber-Fechner's Law, strength of stimulus perceived is directly propoional to intensity of stimulus.""Weber-Fechner Principle--Detection of ""Ratio"" of Stimulus Strength: In the mid 1800s, Weber first and Fechner later proposed the principle that gradations of stimulus strength are discriminated approximately in propoion to the logarithm of stimulus strength." "This is a real-world medical entrance exam question, please give the true answer based on the question and selection. Topic name: None, Subject name: Dental","In a functional implant, bone loss seen annually after 1 year is: Select and explain. A. 1.5 to 2 mm, B. Less than 0.1 mm, C. 1 to 2 mm, D. 1 to 1.5 mm",B: Less than 0.1 mm,"Answer: B: Less than 0.1 mm Explanation: None" "This is a real-world medical entrance exam question, please give the true answer based on the question and selection. Topic name: None, Subject name: Pediatrics","A baby presents with tetany. First thing to be done is administration of : Select and explain. A. Diazepam, B. Vitamin D, C. Calcium gluconate, D. Calcitonin",C: Calcium gluconate,"Answer: C: Calcium gluconate Explanation: The baby presenting with tetany is hypocalcemic, so must be treated immediately with calcium gluconate I.V. Treatment of hypocalcemia. Acute or Severe Tetany The objective of the treatment is to correct hypocalcemia immediately with calcium gluconate I.V Maintenance management of hypoparathyroidism and chronic hypocakenzia. The objective is to maintain serum calcium and serum phosphase at approximately normal levels. It usually requires. 1. Diet  ---> Calcium supplements and diet rich in calcium. 2. Vitamin D supplementation  ---> Ergocakiferol or dihydrotachysterol." "This is a real-world medical entrance exam question, please give the true answer based on the question and selection. Topic name: None, Subject name: Gynaecology & Obstetrics","Maximum level of alpha fetoprotein is seen in: Select and explain. A. Fetal serum, B. Placenta, C. Amniotic fluid, D. Maternal serum",A: Fetal serum,"Answer: A: Fetal serum Explanation: Alpha fetoprotein is the most abundant protein in the fetal serum throughout fetal development. It is transferred from fetus to amniotic fluid when fetus passes urine. The concentration of AFP in amniotic fluid is approximately 100 folds less than in fetal serum, peaks at 13-14 weeks and then decreases in the second trimester (by 10% per week). AFP reaches the maternal serum by diffusion across the amniotic membranes and via the placenta. The level of AFP in maternal serum is less than fetal serum as is suggested by: “Fetal serum contains AFP in a concentration 150 times that of maternal serum”. COGDT 10/e, p 185 “Ordinarily high level of fetoprotein are found in developing fetus and low levels exist in maternal serum and amniotic fluid”. Mannual of Laboratory and Diagnostic Test Fishback 7/e, p 995" "This is a real-world medical entrance exam question, please give the true answer based on the question and selection. Topic name: None, Subject name: Medicine","Anthracis causes which pulmonary manifestations- Select and explain. A. Atypical pneumonia, B. Hemorrhagic mediastinitis, C. Lung abscess, D. Bronchopulmonary pneumonia",B: Hemorrhagic mediastinitis,"Answer: B: Hemorrhagic mediastinitis Explanation: Answer- B. Hemorrhagic mediastinitisA low-grade fever and a nonproductive cough. Substernal discomfo early in the illness. Inhalational anthrax progresses rapidly, causing hemorrhagic mediastinitisHigh feverSevere shoness of breathTachypneaCyanosisProfuse diaphoresisHematemesis" "This is a real-world medical entrance exam question, please give the true answer based on the question and selection. Topic name: None, Subject name: Dental","Which of the following vaccine is not included in EPI schedule Select and explain. A. DPT, B. MMR, C. BCG, D. OPV",B: MMR,"Answer: B: MMR Explanation: None" "This is a real-world medical entrance exam question, please give the true answer based on the question and selection. Topic name: None, Subject name: Pharmacology","Vasopressin antagonist acts on which pa of the nephron? Select and explain. A. Proximal convoluted tubule, B. Distal convoluted tubule, C. Coical collecting tubule, D. Medullary collecting duct",D: Medullary collecting duct,"Answer: D: Medullary collecting duct Explanation: Answer- D (Medullary collecting duct)'The mechanism by which vasopressin exes its anti-diuretic effect is activated by V2 receptors and involves the inseion of aquaporin-2 into the apical (luminal) membranes at the principle cells of the collecting duct.'" "This is a real-world medical entrance exam question, please give the true answer based on the question and selection. Topic name: AIIMS 2018, Subject name: Pharmacology","Which of the following anti-tubercular drug can cause ophthalmological toxicity? Select and explain. A. Isoniazid, B. Rifampicin, C. Ethambutol, D. Pyrazinamide",C: Ethambutol,"Answer: C: Ethambutol Explanation: Ethambutol side effect: causes Red Green color blindness (Ophthalmological toxicity) Isoniazid side effects: Neurotoxicity, Hepatotoxicity. Pyrazinamide side effects: Hyperuricemia (gouty ahritis). Rifampicin side effects: Hepatotoxicity, orange red urine and tears etc." "This is a real-world medical entrance exam question, please give the true answer based on the question and selection. Topic name: None, Subject name: Pharmacology","Compared to unfractionated heparin, Low molecular weight heparin (LMWH) has reliable anticoagulant action because: Select and explain. A. It interferes with thrombin and antithrombin III simultaneously, B. It is less protein bound, C. It is given subcutaneously, D. It is cleared by macrophages",B: It is less protein bound,"Answer: B: It is less protein bound Explanation: Answer- B. It is less protein boundCompared to unfractionated heparin, Low molecular weight heparin (LMWH) has reliable anticoagulant action because it is less protein bound.Indications of LMW heparins:Prophylaxis of DVT and pulmonary embolism in high-risk patients undergoing surgery stroke or other immobilized patientsTreatment of established DVTUnstable anginaTo maintain patency of cannulae and shunts in dialysis patients and in extracorporeal circulation." "This is a real-world medical entrance exam question, please give the true answer based on the question and selection. Topic name: None, Subject name: Dental","Rake angle for tungsten carbide burs: Select and explain. A. Positive., B. Neutral., C. Slight negative., D. Both BC",D: Both BC,"Answer: D: Both BC Explanation: None" "This is a real-world medical entrance exam question, please give the true answer based on the question and selection. Topic name: None, Subject name: Social & Preventive Medicine","You went to a sub center as pa of an audit. How many infants should be registered with a health worker working there? Select and explain. A. 50, B. 100, C. 150, D. 200",B: 100,"Answer: B: 100 Explanation: Ans: B. 100Ref: K Park, 23d ed., pg. 524, 567Estimated no. of infants in a health sub centre covering 5000 population can be calculated as follows:No. of live bih = Bih rate x Population/ 1000= 20.4 x 5000/1000 = 102Infant moality for sub centre = No. of live bih x IMR/1000= 102 x 34/1000= 3.5 (for calculation purpose we assume it 4)Therefore, total number of infants = 102 -- 4= 98Hence correct answer is 100 (nearest value)." "This is a real-world medical entrance exam question, please give the true answer based on the question and selection. Topic name: None, Subject name: Dental","Dr Bernhard Giesenhagen gave which of the following Select and explain. A. Bone ring technique, B. Bone flap technique, C. Overlap technique, D. Bone pouch technique",A: Bone ring technique,"Answer: A: Bone ring technique Explanation: None" "This is a real-world medical entrance exam question, please give the true answer based on the question and selection. Topic name: None, Subject name: Dental","Modification spaces are: Select and explain. A. Spaces other than the original class, B. Space posterior to all teeth, C. Space anterior to all teeth, D. Additional space in class IV",A: Spaces other than the original class,"Answer: A: Spaces other than the original class Explanation: None" "This is a real-world medical entrance exam question, please give the true answer based on the question and selection. Topic name: None, Subject name: Anatomy","What is the nerve supply of the angle of the jaw? Select and explain. A. Mandibular nerve, B. Maxillary nerve, C. Lesser occipital nerve, D. Greater auricular nerve",D: Greater auricular nerve,"Answer: D: Greater auricular nerve Explanation: Ans: D. Greater auricular nerve(Ref: Gray's 41Ie p407, 413, 40Ie p435)The skin over the angle of mandible is supplied by the greater auricular nerve, which carries branches from anterior ramus of C2 and C3. Branches of Upper Cervical Spinal NervesGreat auricular nerveBranches from anterior rami of 2"" & 3rd cervical spinal nervesdegInnervates skin over the angle of mandible, parotid gland & earlobedegLesser occipital nerveBranches from anterior rami of 2"" & 3rd cervical spinal nervesdegInnervates scalp behind the eardeg.Greater occipital nerveBranching off the posterior ramus of the 2"" cervical spinal nervesdegInnervates scalp of occipital areadeg3rd occipital nerve:Branches off the posterior ramus of 3rd cervical nervedegInnervates scalp in occipital & sub occipital areasdeg." "This is a real-world medical entrance exam question, please give the true answer based on the question and selection. Topic name: AIIMS 2019, Subject name: Social & Preventive Medicine","A child has received full Rabies vaccination in December 2018 and now presented with oozing wound on Great toe and the pet had vaccination also. Next line of management is Select and explain. A. No vaccine required, B. RIG + 5 doses of vaccine, C. 5 doses of vaccines only, D. 2 doses of Rabies vaccine",D: 2 doses of Rabies vaccine,"Answer: D: 2 doses of Rabies vaccine Explanation: According to NRCP 2015,National Rabies prophylaxis Guidelines (NRPG) says even the pet is vaccinated, no chance is taken, Rabies is 100% Fatal disease. So, Pet's vaccination status is not so relevant in India guidelines Post exposure prophylaxis. 2 IM doses of Rabies vaccine on Day 0,3. Day 0 3 7 14 28 ESSEN Regimen IM 1 1 1 1 1 THAI updated ID Red close R. 2 2 2 0 2 All these vaccinations come under primary level prevention & specific protection. New Recommended Regimens/ Schedules { NEW GUIDELINES} Type of prophylaxis Regimen POST EXPOSURE INTRAMUSCULAR Essen Regimen (1-1-1-1-1) Day 0,3,7,14,28 POST EXPOSURE INTRADERMAL Updated Thai Red Cross Regimen (2-2-2-0-2) Day 0,3,7,28 POST-EXPOSURE IN VACCINATED INDIVIDUALS Day 0,3 PRE-EXPOSURE PROPHYLAXIS Day 0,7,21/28" "This is a real-world medical entrance exam question, please give the true answer based on the question and selection. Topic name: None, Subject name: Pathology","All of the following are typically associated with the loss of 40% of the circulating blood volume except Select and explain. A. A decrease in the blood pressure, B. A decrease in the central venous pressure, C. A decrease in the heart rate, D. A decrease in the urine output",C: A decrease in the heart rate,"Answer: C: A decrease in the heart rate Explanation: None" "This is a real-world medical entrance exam question, please give the true answer based on the question and selection. Topic name: None, Subject name: Pathology","Disappearance of nuclear chromatin is called as Select and explain. A. Pyknosis, B. Karyolysis, C. Karyorhexis, D. None",B: Karyolysis,"Answer: B: Karyolysis Explanation: None" "This is a real-world medical entrance exam question, please give the true answer based on the question and selection. Topic name: None, Subject name: Physiology","Biomineralization is a process by which inorganic `metals' are incorporated in the body and made a pa of biological structures. Example of Biomineralization is seen in: Select and explain. A. Pinna (anatomy), B. Hooves of cattle, C. Honeycomb, D. Mollusc shell",D: Mollusc shell,"Answer: D: Mollusc shell Explanation: Ans. d. Mollusc shell (Ref en.wilkipedia.org/wiki/Biomineralization)The mollusc shell is a biogenic composite material that has been the subject of much interest in materials science because of its unusual propeies and its model character for biomineralization.Biomineralization is the process by which, living organisms produce minerals, often to harden or stiffen existing tissues. Such tissues are called mineralized tissues." "This is a real-world medical entrance exam question, please give the true answer based on the question and selection. Topic name: None, Subject name: Pharmacology","There was an outbreak of MRSA in the hospital and it was found that a nurse of NICU had MRSA colonisation of anterior nares. What is the best treatment? Select and explain. A. Topical bacitracin, B. Oral Vancomycin, C. Inhaled colistin, D. IV cefazolin",A: Topical bacitracin,"Answer: A: Topical bacitracin Explanation: Ans. A. Topical bacitracinMRSA (methicillin-resistant Staphylococcus aureus) infection is a life-threatening bacterial infection caused by Staphylococcus aureus that is resistant to the antibiotic methicillin. Severe cases usually require hospitalization for treatment.MRSA infections are usually treated with systemic antibiotics, either orally or intravenously.Topical bacitracin in combination with neomycin and polymyxin B might be added to the therapy." "This is a real-world medical entrance exam question, please give the true answer based on the question and selection. Topic name: None, Subject name: Anatomy","What is the location of Meissner's corpuscles? Select and explain. A. Lucidum, B. Basale, C. Reticular dermis, D. Papillary dermis",D: Papillary dermis,"Answer: D: Papillary dermis Explanation: Ans. d. Papillary dermisMeissner corpuscles are encapsulated nerve endings present in dermal papillae.Merkel cells location: Stratum basalePacinian corpuscle location: Reticular layer of dermis" "This is a real-world medical entrance exam question, please give the true answer based on the question and selection. Topic name: None, Subject name: Pediatrics","All are signs of impending Eisenmenger except – Select and explain. A. Increased flow murmur across tricuspid & pulmonary valve, B. Single S2, C. Loud P2, D. Graham steel murmur",A: Increased flow murmur across tricuspid & pulmonary valve,"Answer: A: Increased flow murmur across tricuspid & pulmonary valve Explanation: Eisenmenger syndrome refers to patients with a VSD in which blood is shunted from right to left as a result of development of pulmonary vascular resistance. Initially shunt is from left to right as the systemic vascular pressure is greater than pulmonary vascular pressure. With time pulmonary vascular resistance increases due to change in pulmonary vessel wall as a result of increased flow in pulmonary vessels. When pulmonary vascular pressure exceeds the systemic vascular resistance, reversal of shunt into right to left shunt occurs. This development of right to left shunt due to reversal of left to right shunt as a result of development of pulmonary vascular resistance and pulmonary hypertension is called Eisenmenger syndrome." "This is a real-world medical entrance exam question, please give the true answer based on the question and selection. Topic name: None, Subject name: Dental","Spiral angle can be defined as: Select and explain. A. Angle between long axis and blade., B. Angle between long axis and cutting edge., C. Angle formed by the blades., D. All are same.",A: Angle between long axis and blade.,"Answer: A: Angle between long axis and blade. Explanation: None" "This is a real-world medical entrance exam question, please give the true answer based on the question and selection. Topic name: AIIMS 2019, Subject name: Pharmacology","G-protein coupled receptor that does not act through opening of potassium channels is Select and explain. A. Muscarinic M2 receptor, B. Dopamine D2 receptor, C. Serotonin 5 HT 1 receptor, D. Angiotensin 1 receptor",D: Angiotensin 1 receptor,"Answer: D: Angiotensin 1 receptor Explanation: * DGPCR which act through opening of K+ channels lead to hyper polarisation, thus leading to inhibition.So, receptor which causes stimulation will not act through opening of K+ channels. o Muscarinic M2 receptor, Dopamine D2 receptor and serotonin 5HT1 receptor act through opening of K+ channels.o Whereas Angiotensin 1 receptor act by increasing Ca+2." "This is a real-world medical entrance exam question, please give the true answer based on the question and selection. Topic name: None, Subject name: Gynaecology & Obstetrics","Which of these is not a non-contraceptive use of levonorgestrel? Select and explain. A. Endometriosis, B. Premenstrual tension, C. Complex endometrial hJperplasia, D. Emergencycontraception",B: Premenstrual tension,"Answer: B: Premenstrual tension Explanation: Ans: B. Premenstrual tension(Ref Williams 24/e p701: Goodman Gilman 12/e p1184, 1190)Pre-menstrual tension is not a non-contraceptive use of levonorgestrel.Therapeutic Uses - Levonorgestrel:Emergency contraception:Useful within 120 hours as emergency bih control.Idiopathic menorrhagia:Excessively heavy, regular menses in the absence of intracavitary pathology or coagulopathy.Menometrorrhagia:Excessive bleeding in amount prolonged in duration (regular or irregular intervals).An alternative delivery system - Protect against endometrial hyperplasia in women taking SERM.Management of recurrent pelvic pain secondary to multi- treated endometriosis.LNG-IUS is licensed for use in menorrhagia & to provide endometrial protection to perimenopausal & postmenopausal women on estrogen replacement therapy.LNG-IUS beneficial in endometriosis, adenomyosis, fibroids, endometrial hyperplasia & early stage endometrial cancere (where the patient is deemed unfit for primary surgical therapy)." "This is a real-world medical entrance exam question, please give the true answer based on the question and selection. Topic name: None, Subject name: Dental","Vestibuloplasty all are true except: Select and explain. A. Mucosal advancement, B. Use of hydroxyapatite, C. Secondary epithelialization, D. Usage of epithelial grafts",B: Use of hydroxyapatite,"Answer: B: Use of hydroxyapatite Explanation: Vestibuloplasty The reduction of alveolar ridge size is frequently accompanied by an apparent encroachment of muscle attachments on the crest of the ridge. These so-called high (mandibular) or low (maxillary) attachments serve to reduce the available denture-bearing area and to undermine denture stability.  Key concept: Vestibuloplasty is a soft tissue procedure, and does not necessitate bone grafts. TYPES OF VESTIBULOPLASTY:- Mucosal advancement (Submucous) V’plasty: The mucous membrane of the vestibule is undermined and advanced to line both sides of the extended vestibule. Secondary epithelization vestibuloplasty: The mucosa of the vestibule is used to line one side of the extended vestibule, and the other side heals by growing a new epithelial surface. Grafting vestibuloplasty: Skin, mucous membrane and dermis can be used as a free graft to line one or both sides of the extended vestibule." "This is a real-world medical entrance exam question, please give the true answer based on the question and selection. Topic name: None, Subject name: Dental","Following metal alloy held in reducing hardness of orthodontic wires? Select and explain. A. Chromium, B. Cobalt, C. Silicon, D. Carbon",B: Cobalt,"Answer: B: Cobalt Explanation: None" "This is a real-world medical entrance exam question, please give the true answer based on the question and selection. Topic name: None, Subject name: Anatomy","All of the following are structures in the lateral wall of cavernous sinus except: Select and explain. A. Occulomotor nerve, B. Ophthalmic nerve, C. Optic chiasma, D. Maxillary nerve",C: Optic chiasma,"Answer: C: Optic chiasma Explanation: Structures in the Lateral Wall of the Sinus, from above Downwards (a) Oculomotor nerve: In the anterior part of the sinus. It divides into superior and inferior divisions which leave the sinus by passing through the superior orbital fissure: (b) Trochlear nerve: In the anterior part of the sinus, it crosses superficial to the oculomotor nerve, and enters the orbit throught the superior orbital fissure; (c) Ophthalmic nerve: In the anterior part of the sinus. It divides into the lacrimal, frontal and nasocillary nerves; (d) Maxillary nerve: It leaves the sinus by passing through the foramen rotundum on its way to the pterygopalatine fossa. (e) Trigeminal ganglion: The ganglion and its dural cave project into the posterior part of the lateral wall of the sinus." "This is a real-world medical entrance exam question, please give the true answer based on the question and selection. Topic name: None, Subject name: Dental","Fixation period of intrusive luxation injury is? Select and explain. A. 2 weeks, B. 4 weeks, C. 4 weeks, if semi-rigid & 2 weeks, if rigid, D. 6-8 weeks",B: 4 weeks,"Answer: B: 4 weeks Explanation: None" "This is a real-world medical entrance exam question, please give the true answer based on the question and selection. Topic name: None, Subject name: Surgery","Judet view of X-ray is for: Select and explain. A. Calcaneum, B. Scaphoid, C. Shoulder, D. Pelvis",D: Pelvis,"Answer: D: Pelvis Explanation: Ans: D. Pelvis(Ref Maheshwari 5/e p38, 369)Judet's view of X-ray - For acetabular (pelvic) fracture.Judet's views:Standard radiographic projections employed in acetabulum fractures patients.Basically 45"" oblique of affected hip.45"" angle best achieved by rolling patient.Uses:Generally only performed as supplementary view.Useful in demonstrating or confirming acetabular fractures in acute injury cases." "This is a real-world medical entrance exam question, please give the true answer based on the question and selection. Topic name: None, Subject name: Pathology","Prostaglandins are synthesized from: Select and explain. A. RNA template, B. Rough endoplasmic reticulum, C. Polyunsaturated fatty acids, D. None of the above",C: Polyunsaturated fatty acids,"Answer: C: Polyunsaturated fatty acids Explanation: None" "This is a real-world medical entrance exam question, please give the true answer based on the question and selection. Topic name: None, Subject name: Dental","EDTA does not cause which of the following action Select and explain. A. Chelating action, B. Irrigation, C. Remove smear layer, D. Pulp fixation",D: Pulp fixation,"Answer: D: Pulp fixation Explanation: None" "This is a real-world medical entrance exam question, please give the true answer based on the question and selection. Topic name: None, Subject name: Pathology","Dentigerous cyst is likely to cause which neoplasia Select and explain. A. Ameloblastoma, B. Adeno carcinoma, C. Fibrosarcoma, D. All of the above",A: Ameloblastoma,"Answer: A: Ameloblastoma Explanation: None" "This is a real-world medical entrance exam question, please give the true answer based on the question and selection. Topic name: None, Subject name: Social & Preventive Medicine","Key indicator for AFP surveillance ?. Select and explain. A. At least one case of non-polio AFP per year per 1000 population of under 5 years, B. At least one case of non-polio AFP per year per 100000 population of under 5 year, C. At least one case of non-polio AFP per year per 1000 population of under 15 years, D. At least one case of non-polio AFP per year per 100000 population of under 15 years",D: At least one case of non-polio AFP per year per 100000 population of under 15 years,"Answer: D: At least one case of non-polio AFP per year per 100000 population of under 15 years Explanation: Ans.d) At least one case of non-polio AFP per year per 100000 population of under 15 years The number of AFP cases repoed each year is used as an indicator of a country's ability to detect polio, even in countries where the disease no longer occurs. Polio surveillance It is the most impoant pa of whole polio eradication intiative. It has two components:?Acute flaccid paralysis (AFP) surveillanceAcute flaccid paralysis is defined as acute onset (< 4 weeks) of flaccid paralysis (reduced tone) without other obvious cause in children WHO recommends the immediate repoing and investigation of every case of AFP in children less than 15 years." "This is a real-world medical entrance exam question, please give the true answer based on the question and selection. Topic name: None, Subject name: Dental","Premaxilla is derived from Select and explain. A. Maxillary protuberance, B. Palatine bones, C. Frontonasal process, D. Median process",C: Frontonasal process,"Answer: C: Frontonasal process Explanation: None" "This is a real-world medical entrance exam question, please give the true answer based on the question and selection. Topic name: None, Subject name: Dental","If periodontal surgery is necessary in case of horizontal bone toss in upper anterior region. Which of the following procedure is the most appropriate? Select and explain. A. Papilla preservation flap, B. Widman flap, C. Coronally displaced flap, D. Gingivectomy",A: Papilla preservation flap,"Answer: A: Papilla preservation flap Explanation: None" "This is a real-world medical entrance exam question, please give the true answer based on the question and selection. Topic name: None, Subject name: Pharmacology","Prolonged administration of streptomycin may result in damage to the: Select and explain. A. Optic nerve, B. Facial nerve, C. Auditory nerve, D. Trigeminal nerve",C: Auditory nerve,"Answer: C: Auditory nerve Explanation: Streptomycin (antitubercular) causes vestibular damage on prolonged use. Adverse effects This class of drugs is notorious for their adverse effects like ototoxicity and nephrotoxicity. About 1/5th patients given streptomycin 1 gm BD i.m. experience vestibular disturbances although auditory disturbances are less common.  Streptomycin has the lowest nephrotoxicity among aminoglycosides; probably because it is not concentrated in the renal cortex. Reference: Essentials of Medical Pharmacology Eighth Edition KD TRIPATHI page no 94,798" "This is a real-world medical entrance exam question, please give the true answer based on the question and selection. Topic name: None, Subject name: Physiology","Considering the latent period of muscle twitch to be 10 ms, contraction time 40 ms and relaxation time 50 ms, what will be the tetanizing frequency for this muscle? Select and explain. A. 25 Hz, B. 50 Hz, C. 100 Hz, D. 75 Hz",A: 25 Hz,"Answer: A: 25 Hz Explanation: Ans: A. 25 HzStimulation frequency at which summation of contractions occurs -Determined by twitch duration of the paicular muscle under study.Example:If twitch duration = 10 ms.Frequencies less than 1/10 ms (100/s) - Cause discrete responses interrupted by complete relaxation.Frequencies greater than 100/s cause summation.Tetany:Continuous contraction of muscle fibers, without latent period & relaxation time.Hence tetanizing frequency depends only on contraction time, i.e. twitch duration.Twitch duration = 40 milliseconds = 0.04 secTetanizing frequency = 1/0.04 = 25 Hz" "This is a real-world medical entrance exam question, please give the true answer based on the question and selection. Topic name: None, Subject name: Anatomy","Compament of leg without neurovascular bundle? Select and explain. A. Anterior, B. Lateral, C. Deep posterior, D. Superficial posterior",D: Superficial posterior,"Answer: D: Superficial posterior Explanation: Ans. d. Superficial posteriorRef: Grays Anatomy, 4lst ed, pg- 1406-1412 and Last anatomy 12th ed, pg. 142Contents of flexor (posterior) compament:Superficial pa:Gastrocnemius, plantaris and soleus musclesDeep pa:Popliteus, Flexor digitorum longus, flexor hallucis longus and Tibialis posterior musclePosterior tibial and peroneal vesselsTibial nerve" "This is a real-world medical entrance exam question, please give the true answer based on the question and selection. Topic name: None, Subject name: Dental","Which of the following condition must be met for a valid student-t test between means of two groups: Select and explain. A. Number of observation must be same - no, B. Standard deviation must be approximately same - no, C. Mean should be approximately same - no, D. Sample must be small",D: Sample must be small,"Answer: D: Sample must be small Explanation: None" "This is a real-world medical entrance exam question, please give the true answer based on the question and selection. Topic name: None, Subject name: Pathology","V) hich of the following can change the gene expression by methylation and acetylation without affecting the content of the gene? Select and explain. A. Epigenetics, B. Translocation, C. Inversion, D. Transduction",A: Epigenetics,"Answer: A: Epigenetics Explanation: Ans.: A. Epigenetics""Epigenetics is defined as changes that alter the pattern of gene expression that persist across at least one cell division but are not caused by changes in the DNA code. Epigenetic changes include alterations of chromatin structure mediated bymethylation of cytoosine residues in CpG dinucleotides, modification of histones by acetylation or methylation, or changesin higher-order chromosome structure- "" Harrison""" "This is a real-world medical entrance exam question, please give the true answer based on the question and selection. Topic name: None, Subject name: Dental","What type of fibers are principal fibers of the PDL? Select and explain. A. Elastic, B. Reticular, C. Collagenous, D. Collagenous and elastic",C: Collagenous,"Answer: C: Collagenous Explanation: None" "This is a real-world medical entrance exam question, please give the true answer based on the question and selection. Topic name: None, Subject name: Physiology","Find faulty statement regarding muscle spindle: Select and explain. A. Central zone has no actin and myosin, B. Type 1 nuclear bag fibres has low myosin ATPase activity, C. Peripheral zone has no actin and myosin, D. Type 2 nuclear bag has high level of myosin ATPase activity",C: Peripheral zone has no actin and myosin,"Answer: C: Peripheral zone has no actin and myosin Explanation: Each muscle spindle has three essential elements:  (1) A group of specialized intrafusal muscle fibers  with contractile polar ends and a noncontractile center.  (2) Large diameter myelinated afferent nerves (types Ia and II) originating in the central portion of the intrafusal fibers.  (3) Small diameter myelinated efferent nerves supplying the polar contractile regions of the intrafusal fibers." "This is a real-world medical entrance exam question, please give the true answer based on the question and selection. Topic name: None, Subject name: Dental","Injecting dye for diagnosis of salivary gland Select and explain. A. Angiogram, B. Sialography, C. Angiogram, D. Tomography",B: Sialography,"Answer: B: Sialography Explanation: None" "This is a real-world medical entrance exam question, please give the true answer based on the question and selection. Topic name: None, Subject name: Psychiatry","All are causes of subcortical dementia except - Select and explain. A. Alzheimer's disease, B. Parkinson's disease, C. Supranuclear palsy, D. HIV associated dementia",A: Alzheimer's disease,"Answer: A: Alzheimer's disease Explanation: Alzheimer’s disease is a cortical dementia." "This is a real-world medical entrance exam question, please give the true answer based on the question and selection. Topic name: None, Subject name: Dental","Which growth factor Is not present in PRP? Select and explain. A. PDGF aa, B. PDGF bb, C. PDGF cc, D. PDGF Fab",C: PDGF cc,"Answer: C: PDGF cc Explanation: None" "This is a real-world medical entrance exam question, please give the true answer based on the question and selection. Topic name: None, Subject name: Forensic Medicine","A 18-year-old girl was brought to OPD, labia majora separated, labia minora flabby, fourchette tear present and vagina is roomy but Hymen is intact. What could be possible? Select and explain. A. Virgin, B. False virgin, C. Premenstrual stage, D. Molestation",B: False virgin,"Answer: B: False virgin Explanation: Ans: b. False virginRef: KSN Reddy's The Essentials of Forensic Medicine and Toxicology 33' edn; Page no. 392False VirginHymen is intact but thick and the woman has had sexual intercourse. Labia majora: Flabby, may gap on abduction of thighLabia minora: Loose, blackish-brown, cutaneousVagina deep, roomy, dilated and capacious with less rugosed wall Vagina easily admits two fingers.True VirginLabia majora: Firm, rounded and completeLabia minora: Soft, sensitive and pink in colorFourchette and posterior commissure: Intact * Vaginal wall: Closely approximatedVagina mucosa: Rugose, reddish, sensitive to touchHymen: Intact" "This is a real-world medical entrance exam question, please give the true answer based on the question and selection. Topic name: None, Subject name: Pathology","A diffuse spreading inflammatory lesion is due to bacterial enzyme Select and explain. A. Coagulase, B. Hyaluronidase, C. Peroxidase, D. Bradykinin",B: Hyaluronidase,"Answer: B: Hyaluronidase Explanation: None" "This is a real-world medical entrance exam question, please give the true answer based on the question and selection. Topic name: None, Subject name: Pharmacology","Which of the following drugs can be given in renal failure safely? Select and explain. A. Saxagliptin, B. Linagliptin, C. Vildagliptin, D. Sitagliptin",B: Linagliptin,"Answer: B: Linagliptin Explanation: Ans: B. Linagliptin(Ref Goodman Gilman 12/e p1264; Katzung 13/e p740, 12/e p761; FDA website: http://wwwfda.gov/ Safety/ MedWatch/Safetylnformation/ircm3 I 9215.htm)Linagliptin:Given safely in renal failure.Chronic kidney disease:Major complication in type 2 diabetes.Metformin usage is challenging.More severe renal disease - Less likely for metformin use.Due to safety concerns." "This is a real-world medical entrance exam question, please give the true answer based on the question and selection. Topic name: None, Subject name: Dental","Zones of periapical pathology were described by: Select and explain. A. Palmer, B. Fish, C. Black, D. Hugues",B: Fish,"Answer: B: Fish Explanation: None" "This is a real-world medical entrance exam question, please give the true answer based on the question and selection. Topic name: None, Subject name: Anatomy","Brachcephaly with narrow foramen magnum is seen in: Select and explain. A. Down syndrome., B. Fanconi syndrome., C. Achondroplasia., D. Cleidocranial dysplasia.",C: Achondroplasia.,"Answer: C: Achondroplasia. Explanation: None" "This is a real-world medical entrance exam question, please give the true answer based on the question and selection. Topic name: None, Subject name: Pharmacology","Gingival hyperplasia is caused by all except Select and explain. A. Sodium valproate, B. Carbamazepine, C. Verapamil, D. Bleomycin",B: Carbamazepine,"Answer: B: Carbamazepine Explanation: None" "This is a real-world medical entrance exam question, please give the true answer based on the question and selection. Topic name: None, Subject name: Pediatrics","A triad of seizure, mental retardation and sebaceous adenoma is seen in – Select and explain. A. Congenital syphilis, B. Tuberous sclerosis, C. Toxoplasmosis, D. Hypothyroidism",B: Tuberous sclerosis,"Answer: B: Tuberous sclerosis Explanation: Tuberous sclerosis : It is an autosomal dominant neurocutaneous syndrome. Pathology Characteristics brain, lesion, consists of tubers in the brain. They are typically present in subependyneal region where they undergo calcification producing candle dripping appearance." "This is a real-world medical entrance exam question, please give the true answer based on the question and selection. Topic name: None, Subject name: Medicine","Which is not seen in neuroimaging of patient of tuberous sclerosis? Select and explain. A. White matter lesion, B. Ependymoma, C. Subependymal nodules, D. Giant cell astrocytoma",B: Ependymoma,"Answer: B: Ependymoma Explanation: None" "This is a real-world medical entrance exam question, please give the true answer based on the question and selection. Topic name: None, Subject name: Dental","Cholesterol crystals in apical periodontitis lesions are believed to be formed from cholesterol released by: Select and explain. A. Disintegrating erythrocytes of stagnant blood vessels within the lesion, B. Lymphocyte, plasma cells, macrophages that die and disintegrate in chronic apical lesion, C. Circulating plasma lipids, D. All of the above",D: All of the above,"Answer: D: All of the above Explanation: Apical periodontitis can be caused by both exogenous and endogenous factors.  Exogenous factors include microbes and their toxins and noxious metabolic by-products, chemical agents, mechanical irritation, foreign bodies, and trauma.  Endogenous factors include the host’s metabolic products, such as urate and cholesterol crystals, as well as cytokines or other inflammatory mediators that activate osteoclasts. Occasionally, foamy macrophages and giant cells are seen, especially associated with cholesterol crystal deposits, which are products of disintegrated cell membranes, disintegrating erythrocytes of stagnant blood vessels within the lesion, lymphocytes, plasma cells, macrophages that die and disintegrate in chronic apical lesion." "This is a real-world medical entrance exam question, please give the true answer based on the question and selection. Topic name: None, Subject name: Dental","The analytical study where population is the unit of study is Select and explain. A. Cross-sectional, B. Ecological, C. Case-control, D. Cohort study",B: Ecological,"Answer: B: Ecological Explanation: None" "This is a real-world medical entrance exam question, please give the true answer based on the question and selection. Topic name: None, Subject name: Dental","What % is the incidence of vertucci type 8 Select and explain. A. 6%, B. 10%, C. 2%, D. 12%",A: 6%,"Answer: A: 6% Explanation: None" "This is a real-world medical entrance exam question, please give the true answer based on the question and selection. Topic name: None, Subject name: Surgery","Which of the following is expected when a tooth is lost? Select and explain. A. Decrease in trabecular pattern, B. Loss of bone width, C. Loss of bone height, D. All of the above",D: All of the above,"Answer: D: All of the above Explanation: None" "This is a real-world medical entrance exam question, please give the true answer based on the question and selection. Topic name: None, Subject name: Dental","In Mandibular 1st molar smallest cusp is Select and explain. A. Distobuccal, B. Distal, C. Mesiobuccal, D. Mesial",B: Distal,"Answer: B: Distal Explanation: None" "This is a real-world medical entrance exam question, please give the true answer based on the question and selection. Topic name: None, Subject name: Dental","In a village, despite of health education for oral cancer, people don't follow instructions even after referral with persuasive reminders, people are still reluctant. It best fits under which model: Select and explain. A. Trans-theoretical model, B. Health belief model, C. Social compliance, D. Public health model",A: Trans-theoretical model,"Answer: A: Trans-theoretical model Explanation: Trans-theoretical Model Developed by Prochaska and DiClemente. Based on assumption that behavior change is evolutional in nature and occurs in different stages of change through which individuals can progress. It holds habitual behaviors mainly like smoking, nutritional diet, screening for cancer, alcohol consumption, etc., in a series of steps through a cyclical process." "This is a real-world medical entrance exam question, please give the true answer based on the question and selection. Topic name: AIIMS 2019, Subject name: Anatomy","Which of the following finger has two dorsal interossei (AIIMS May 2019) Select and explain. A. Little, B. Ring, C. Middle, D. Index",C: Middle,"Answer: C: Middle Explanation: Dorsal interossei: Dorsal interossei are major abductors of index, middle & ring fingers. Most dorsally situated of all intrinsic muscles. Palpated through skin on dorsal aspect of hand. Tendons of dorsal interossei pass dorsal to deep transverse metacarpal ligament. - 1st dorsal interosseous muscle - largest & inses into lateral side of index finger. - 2nd& 3rd interosseous muscle - inse into lateral & medial sides respectively of middle finger. - 4th dorsal interosseous muscle - inses into medial side of ring finger. Thumb & little finger do not have dorsal interossei as they have their own abductors in thenar & hypothenar muscle groups respectively. Dorsal interossei + palmar interossei + lumbricals flex-metacarpophalangeal joint & extend inter - phalangeal joint." "This is a real-world medical entrance exam question, please give the true answer based on the question and selection. Topic name: None, Subject name: Dental","Dimensional stability of elastomeric impression material can be given in descending order as : Select and explain. A. Polysulphide > Polyether > Condensation silicone > Addition silicone, B. Polyether > Condensation silicone > Polysulphide > Addition silicone, C. Addition silicone > Polyether > Polysulphide > Condensation silicone, D. Addition silicone > Condensation silicone Polysulphide > Polyether",C: Addition silicone > Polyether > Polysulphide > Condensation silicone,"Answer: C: Addition silicone > Polyether > Polysulphide > Condensation silicone Explanation: None" "This is a real-world medical entrance exam question, please give the true answer based on the question and selection. Topic name: None, Subject name: Gynaecology & Obstetrics","In a female, intraocular metastasis most commonly occurs from which of the following gynaecological primary? Select and explain. A. Breast, B. Ovary, C. Cervix, D. Endometrium",A: Breast,"Answer: A: Breast Explanation: Answer- A (Breast)Breast cancer is the most common tumor to metastasize to the eye followed by lung cancer.Intraocular metastases are the most common malignancy of eye, and the primary cause is breast cancer." "This is a real-world medical entrance exam question, please give the true answer based on the question and selection. Topic name: None, Subject name: Forensic Medicine","Calculate the Age of fetus having CRL 23 cm, foot length 7 cm. Talus is present and testes are present at superficial inguinal ring? Select and explain. A. 5 months, B. 6 months, C. 7 months, D. 8 months",C: 7 months,"Answer: C: 7 months Explanation: Ans: C. 7 monthsGestational age can be determined fiom maturation of chorionic villi; foot length and ossification centres.Seventh month:Length 35 cm; crown-rump length 23 cm; foot length 8 cm; weight 900 to 1200 g.Nails are thick.Eyelids open and pupillary membrane disappears.Skin is dusky-red, thick and fibrous.Meconium is found in the entire large intestine.Testes are found at external inguinal ring.Gallbladder contains bile and caecum is seen in the right iliac fossa.Ossification centre is present in the talus." "This is a real-world medical entrance exam question, please give the true answer based on the question and selection. Topic name: None, Subject name: Physiology","Serum is: Select and explain. A. Plasma-clotting factors, B. Plasma-fibrin, C. Blood-RBC, D. Blood-all cellular elements",B: Plasma-fibrin,"Answer: B: Plasma-fibrin Explanation: None" "This is a real-world medical entrance exam question, please give the true answer based on the question and selection. Topic name: None, Subject name: Radiology","Which of the following has a radiolucency closest to the dentin: Select and explain. A. ZOE, B. Calcium hydroxide, C. Silicophosphate, D. Gutta percha",C: Silicophosphate,"Answer: C: Silicophosphate Explanation: None" "This is a real-world medical entrance exam question, please give the true answer based on the question and selection. Topic name: None, Subject name: Dental","Indirect retainers are situated: Select and explain. A. As far anteriorly from the fulcrum line, B. As close to the fulcrum line, C. At the canine region, D. At the mesial marginal ridge of the premolars in the class I case",A: As far anteriorly from the fulcrum line,"Answer: A: As far anteriorly from the fulcrum line Explanation: None" "This is a real-world medical entrance exam question, please give the true answer based on the question and selection. Topic name: None, Subject name: Dental","Which of the following is a 3-D curve: Select and explain. A. Curve of spee., B. Anti monsoon curve., C. Monsoon curve.., D. None.",C: Monsoon curve..,"Answer: C: Monsoon curve.. Explanation: Monsoon’s curve 3-D curve Spherical in shape Center – Glabella Radius – 8” Surface of phase is thought or said to pass through the occlusal plane including the condyle." "This is a real-world medical entrance exam question, please give the true answer based on the question and selection. Topic name: None, Subject name: Radiology","Cementosis are commonly seen in: Select and explain. A. Females in maxilla, B. Females in mandible, C. Males in maxilla, D. Males in mandible",B: Females in mandible,"Answer: B: Females in mandible Explanation: None" "This is a real-world medical entrance exam question, please give the true answer based on the question and selection. Topic name: None, Subject name: Anatomy","Facial nerve innervates all of the following except: Select and explain. A. Platysma, B. Anterior belly of digastric, C. Stapedius, D. Auricular muscles",B: Anterior belly of digastric,"Answer: B: Anterior belly of digastric Explanation: All of the following muscles are innervated by facial nerve. Muscles of facial expression Occipito frontalis Platysma Posterior belly of digastric Stapedius Stylohyoid Auricular muscles." "This is a real-world medical entrance exam question, please give the true answer based on the question and selection. Topic name: None, Subject name: Dental","Amalgam restorations give the best clinical service when the residual mercury content is: Select and explain. A. 38-42%, B. 48-52%, C. 58-62%, D. 68-72%",B: 48-52%,"Answer: B: 48-52% Explanation: None" "This is a real-world medical entrance exam question, please give the true answer based on the question and selection. Topic name: None, Subject name: Pathology","About intraoperative histopathological analysis, all are true except: Select and explain. A. Gives an immediate definitive diagnosis of tumor, B. Used for detecting positive margins after resection, C. Used to confirm suspected metastasis, D. Sentinel lymph node biopsy in breast carcinoma is an example",A: Gives an immediate definitive diagnosis of tumor,"Answer: A: Gives an immediate definitive diagnosis of tumor Explanation: Ans: A. Gives an immediate definitive diagnosis of tumor (Ref http:/,www.nchi.iilm.nih.gov/pmc/arii,PMC3347896).lntraoperative Histopathological Analysis:Frozen section procedure/cryosection:Rapid microscopic specimen analysis.Used most often in oncological surgery.Slide quality produced by frozen section lower than formalin-fixed paraffin-embedded tissue processing.Used for presumptive diagnosis.More accurate diagnosis - Fixed tissue processing." "This is a real-world medical entrance exam question, please give the true answer based on the question and selection. Topic name: None, Subject name: Medicine","A 23 years old boy, a badminton player, sustained injury of left ankle. He was immobilized for 3 months, the cast was removed and patient was able to walk normally. Later he complained of pain and swelling in the left calf, left ankle and foot. His mother massaged him for 30 minutes. After a while he developed acute onset of breathlessness and was brought to emergency and died. Most likely cause of death is: Select and explain. A. Pulmonary thromboembolism, B. Congestive cardiac failure, C. Massive stroke, D. Hypovolemic shock",A: Pulmonary thromboembolism,"Answer: A: Pulmonary thromboembolism Explanation: Answer- A. Pulmonary thromboembolismHistory of immobilization for long duration suggests a probahly subclinical deep venous thrombosis that had developed in the patient. On massaging, the thrombus of DVT got dislodged in the pulmonary circulation, causing pulmonary thromboembolism and death." "This is a real-world medical entrance exam question, please give the true answer based on the question and selection. Topic name: None, Subject name: Pathology","Petechial hemorrhage is seen in Select and explain. A. cyclic neutropenia, B. agranulocytosis, C. pernicious anemia, D. thrombocytopenic purpura",D: thrombocytopenic purpura,"Answer: D: thrombocytopenic purpura Explanation: None" "This is a real-world medical entrance exam question, please give the true answer based on the question and selection. Topic name: None, Subject name: Pharmacology","Digoxin is obtained from a plant product and has a half-life of 36 hours. How does this information help us in formulating treatment? Select and explain. A. To adjust maintenance dose of digoxin required to keep the blood levels within therapeutic range, B. Intravenous administration in emergency and urgent dosing, C. Long half-life permits altemate day dosing, D. It requires a high loading dose to be administered",A: To adjust maintenance dose of digoxin required to keep the blood levels within therapeutic range,"Answer: A: To adjust maintenance dose of digoxin required to keep the blood levels within therapeutic range Explanation: Ans: A. To adjust maintenance dose of digoxin required to keep the blood levels within therapeutic range(Ref Goodman Gilman 12/e p33. 37: KDT 7/e p31, 515)Half- life of drug:Determines the maintenance dose of digoxin.Helps maintain therapeutic plasma levels.Maintenance dose also determined by creatinine clearance.Half-life of dioxin:36 to 48 hrs in patients with normal renal function.3.5 to 5 days in anuric patients.In patients with normal renal function, an oral daily maintenance dose without loading dose - steady-state blood concentration in approximately 7 days.Hence daily maintenance dose decided based on half-life ofthe drug.Usual digoxin therapeutic serum concentrations range - 0.8 to 2 ng/mL.(Note: Loading dose governed by volume of distribution)." "This is a real-world medical entrance exam question, please give the true answer based on the question and selection. Topic name: None, Subject name: Social & Preventive Medicine","You have been asked to design a study for a disease whose prevalence in the community is 10%. The alpha error has to be kept at 5% with a relative precision of 20% and a power of 20%. What will be the accurate sample size for this study? Select and explain. A. 400, B. 900, C. 1800, D. 3600",B: 900,"Answer: B: 900 Explanation: Ans: B. 900" "This is a real-world medical entrance exam question, please give the true answer based on the question and selection. Topic name: None, Subject name: Dental","The anteroposterior curve is Select and explain. A. Curve of Spee, B. Curve of Wilson, C. Curve of Monson, D. Bonwill's curve",A: Curve of Spee,"Answer: A: Curve of Spee Explanation: None" "This is a real-world medical entrance exam question, please give the true answer based on the question and selection. Topic name: None, Subject name: Dental","Which periodontal fibres are consistent and are reconstructed even after the destruction of the alveolar bone? Select and explain. A. Apical, B. Alveolar crest group, C. Oblique, D. Transseptal",D: Transseptal,"Answer: D: Transseptal Explanation: None" "This is a real-world medical entrance exam question, please give the true answer based on the question and selection. Topic name: None, Subject name: Dental","Condensation reaction occurs in Select and explain. A. Agar, B. Alginate, C. Polysulfide, D. ZOE",C: Polysulfide,"Answer: C: Polysulfide Explanation: None" "This is a real-world medical entrance exam question, please give the true answer based on the question and selection. Topic name: None, Subject name: Pediatrics","The following cardiac defects are characterized by ductus dependent blood flow except – Select and explain. A. Transposition of great arteries with intact septum, B. Interrupted aortic arch, C. Truncus arteriosus, D. Hypoplastic left heart syndrome",C: Truncus arteriosus,"Answer: C: Truncus arteriosus Explanation: Congenital heart disease in the newborn can be broadly categorized by the relationship between the patients cardiac defect and the patent ductus arteriosus; this categorization yields four distinct groups: I. Pulmonary flow ductal dependence. Newborns with congenital heart disease who are dependent on the potency of their ductus for pulmonary blood flow present with varying degrees of cyanosis. Critical Pulmonary Valve Stenosis with Intact Ventricular Septum. Li Tricuspid Atresia: Tetralogy of Fallot Ni Systemic flow ductal dependence. These newborns are dependent on their ductus arteriosus for systemic blood flow and unlike the babies who are pulmonary flow dependent, these newborns present with severely decreased cardiac output. This decreased systemic flow is characterized by pallor, diminished peripheral pulses, low urine output, cool extremities and varying degrees of metabolic acidosis. This includes newborns with left ventricular outflow tract obstruction at various levels. Congenital Valvular Aortic Stenosis Coarctation of the Aorta Interrupted Aortic Arch Hypoplastic Left Heart Syndrome (HLHS) IlL     xygenation ductal dependence. These are babies who are born with congenital heart disease that requires a patent ductus for adequate mixi g of saturated and desaturated blood. Transposition of the Great Vessels (TGV) IV J uctal independence. These are neonates who are not dependent on a patent ductus but still require urgent operation. Total anomalous pulmonary venous return (TAPVR) Truncus Arteriosus Anomalous origin of the left main coronary alloy from the pulmonaly artery." "This is a real-world medical entrance exam question, please give the true answer based on the question and selection. Topic name: None, Subject name: Dental","The function of 2% potassium sulphate in a gypsum product is: Select and explain. A. To regulate the setting expansion, B. Regulate setting time, C. Acts as retarder, D. None",B: Regulate setting time,"Answer: B: Regulate setting time Explanation: None" "This is a real-world medical entrance exam question, please give the true answer based on the question and selection. Topic name: AIIMS 2018, Subject name: Radiology","What is the most poable diagnosis based on the findings in the CT scan given below? Select and explain. A. Extradural hemorrhage, B. Subdural hamorrhage, C. Intracerebral hamorrhage, D. Subarachnoid hamorrhage",A: Extradural hemorrhage,"Answer: A: Extradural hemorrhage Explanation: Epidural Hematoma- Biconvex/lentiform hyperdensity seen Subdural Hematoma - Concavo- convex or crescentic shaped. Subarachnoid Hemorrhage- Shows Hyperdensity in:* Syln fissure* Basal cisterns* Sulcal Spaces" "This is a real-world medical entrance exam question, please give the true answer based on the question and selection. Topic name: None, Subject name: Surgery","Mandibular fracture managed by adapting mini plates is a type of Select and explain. A. Load bearing osteosynthesis, B. Load sharing, C. Non rigid osteosynthesis, D. Rigid osteosynthesis",B: Load sharing,"Answer: B: Load sharing Explanation: None" "This is a real-world medical entrance exam question, please give the true answer based on the question and selection. Topic name: None, Subject name: Microbiology","The incubation period of LGV is? Select and explain. A. 3-7 days, B. 7-10 days, C. 10-30 days, D. 30-90 days",C: 10-30 days,"Answer: C: 10-30 days Explanation: Ans. C. 10-30 daysThe incubation period for LGV (the time interval between sexual contact and the appearance of symptoms) varies on average from 10 to 14 days. At times the incubation period may be as long as up to 6 weeks after sexual contact with an infected paner. Feature SyphilisHerpes ChancroidLGV Donovanosis Incubation period 9-90 days 2-7 days 1-14 days3 days-6weeks 1-4 weeks(up to 6 months)" "This is a real-world medical entrance exam question, please give the true answer based on the question and selection. Topic name: None, Subject name: Dental","All of the following are true for ZOE based sealer except? Select and explain. A. Expands on setting & hence provide impervious seal, B. Absorb if extruded periapically, C. Antibacterial, so prevent entombing of Bacterial, D. Slow setting, so results in increased inflammation initially",A: Expands on setting & hence provide impervious seal,"Answer: A: Expands on setting & hence provide impervious seal Explanation: None" "This is a real-world medical entrance exam question, please give the true answer based on the question and selection. Topic name: None, Subject name: Dental","Maximum amount of phosphate is seen in Select and explain. A. Bone, B. Enamel, C. Gingiva, D. Dentine",B: Enamel,"Answer: B: Enamel Explanation: None" "This is a real-world medical entrance exam question, please give the true answer based on the question and selection. Topic name: None, Subject name: Pediatrics","What should be the ideal temperature in delivery room for the neonates to be kept in warmer? Select and explain. A. 22-26degC, B. 28-30degC, C. 30-35degC, D. 37degC",A: 22-26degC,"Answer: A: 22-26degC Explanation: Ans: A. 22-26degC(Ref Ghai 8/e p133).Nursery temperature:Ideal temperature in delivery room for neonates in warmer state - 22-26degC (72-78""F).Should be free from drill of air.Ventilation system for each delivery & resuscitation room designed to control ambient temperature between 72-78 degrees Fahrenheit (22-26 degrees Centigrade) during delivery, resuscitation & stabilization of newborn." "This is a real-world medical entrance exam question, please give the true answer based on the question and selection. Topic name: AIIMS 2018, Subject name: Medicine","What is the best management of the case shown? Select and explain. A. Dapsone plus steroids, B. Stop smoking and screen for cancer, C. Vitamin supplements, D. Antifungals for oral candidiasis",D: Antifungals for oral candidiasis,"Answer: D: Antifungals for oral candidiasis Explanation: Candidiasis Rx - Topical clotrimazole - Nystatin lozenges Median rhomboid glossitis Appearance - seen in Candidiasis infection" "This is a real-world medical entrance exam question, please give the true answer based on the question and selection. Topic name: None, Subject name: Anatomy","Spinal cord ends at what level in adults? Select and explain. A. T12, B. L1, C. L2, D. L3",B: L1,"Answer: B: L1 Explanation: Ans. b. L1 Ref: Grays Anatomy, 4l'' ed., pg. 762-763In the adult, the spinal cord terminates on average at the level of the middle third of the body of the first lumbar veebra which corresponds approximately to the transpyloric plane.Spinal cord extends from C I to L I (lower border) in adults.Filum terminale extends from lower end of spinal cord to the tip of coccyx.Dural sheath extends up to 52 veebrae.Subdural space extends up to 52.Subarachnoid space extends up to 52.Piamater extends up to tip of coccyx." "This is a real-world medical entrance exam question, please give the true answer based on the question and selection. Topic name: None, Subject name: Gynaecology & Obstetrics","Amniotic fluid contains acetyl cholinesterase enzyme. What is the diagnosis ? Select and explain. A. Open spina bifida, B. Gastroschisis, C. Omphalocele, D. Osteogenesis imperfecta",A: Open spina bifida,"Answer: A: Open spina bifida Explanation: Amniotic fluid Acetylcholineesterase level is elevated in open neural tube defect: It has a better diagnostic value than AFP. In case of suspected neural tube defect, on Amniocentesis, if amniotic fluid AFP levels are raised but Acetylcholinesterase levels are normal, patient should be reassured that elevated AFP levels are probably caused by fetal blood contamination but, if acetyl cholinesterase is also elevated along with AFP it is indicative of NTD. It also helps to distinguish between neural tube defect and abdominal wall defects (both of which cause elevated MSAFP): – Acetyl cholinesterase is raised in open NTD, but is low in abdominal wall defects. – In patients with NTD, the ratio of acetylcholinesterase to butyrlcholinesterase levels is 0.14 or more. In case of abdominal wall defects this ratio is less than 0.14." "This is a real-world medical entrance exam question, please give the true answer based on the question and selection. Topic name: None, Subject name: Pharmacology","Which of the following cause an increase in ACh release from the presynaptic neuron? Select and explain. A. Blocking calcium channels on presynaptic, B. Blocking chloride channel on presynaptic, C. Blocking Na channel, D. Blocking K channels",D: Blocking K channels,"Answer: D: Blocking K channels Explanation: Ans. D. Blocking K channels* Potassium channel blocker will cause an increase in ACh release from the presynaptic neuron.* Blockade of presynaptic potassium channels promotes activation of voltage-gated calcium channels leading to the increased release of acetylcholine in the neuromuscular junction. Aminopyridines used in the treatment of Lambe-Eaton myasthenic syndrome acts by this mechanism and improves the muscle strength in the patients" "This is a real-world medical entrance exam question, please give the true answer based on the question and selection. Topic name: None, Subject name: Pathology","Bifid tongue, a congenital anomaly occurs due to non-fusion of: Select and explain. A. Tuberculum impar and lateral lingual swellings, B. Hypobranchial eminence and tuberculum impar, C. The two lateral lingual swellings, D. Some of the above",C: The two lateral lingual swellings,"Answer: C: The two lateral lingual swellings Explanation: None" "This is a real-world medical entrance exam question, please give the true answer based on the question and selection. Topic name: None, Subject name: Medicine","Hypereosinophilic syndrome is a disease characterized by elevated eosinophil count in the blood for at least 6 months, without any recognizable cause, with involve-ment of either hea, nervous system or bone marrow. What should be the eosinophil count for diagnosis? Select and explain. A. >500/mm3, B. >1000/ mm3, C. >1500/ mm3, D. >2000/ mm3",C: >1500/ mm3,"Answer: C: >1500/ mm3 Explanation: Ans: C. >1500/ mm3(Ref Harrison 19/e p1686, 135e-8, 18/e p481 e-21.5)Eosinophil count for hypereosinophilia syndrome diagnosis = >1.5 x 109/L (>1500/mm).Blood Eosinophilia:Neither secondary or clonal.Cause: Idiopathic.Hypereosinophilic Syndrome (HES):Subcategory of idiopathic eosinophilia with persistent increase of AEC to >1.5 x 109/L (>1500/ mm3).Features:Presence of eosinophil-mediated organ damage -Cardiomyopathy, gastroenteritis, cutaneous lesions, sinusitis, pneumonitis, neuritis & vasculitis.Also thromboembolic complications, hepatosplenomegaly & either cytopenia or cytosis." "This is a real-world medical entrance exam question, please give the true answer based on the question and selection. Topic name: None, Subject name: Pediatrics","Treatment of kawasaki disease in children is – Select and explain. A. Oral steroids, B. IV steroids, C. IV Immuno globulins, D. Mycophenolate mefentil",C: IV Immuno globulins,"Answer: C: IV Immuno globulins Explanation: None" "This is a real-world medical entrance exam question, please give the true answer based on the question and selection. Topic name: None, Subject name: Dental","The effectiveness of condensing porcelain powder to reduce shrinkage is determined by Select and explain. A. Shape and size of the particle, B. Thickness of the platinum matrix, C. Speed of increase of furnace heat, D. Using powder of same particle size",A: Shape and size of the particle,"Answer: A: Shape and size of the particle Explanation: None" "This is a real-world medical entrance exam question, please give the true answer based on the question and selection. Topic name: None, Subject name: Pathology","Russell's bodies are found in: Select and explain. A. Activated macrophages, B. Histiocytes, C. Plasma cells, D. Erythrocytes",C: Plasma cells,"Answer: C: Plasma cells Explanation: None" "This is a real-world medical entrance exam question, please give the true answer based on the question and selection. Topic name: None, Subject name: Microbiology","When horse serum is injected intravenously in to a rabbit and again in to the skin two or three weeks later, what is the necrotising reaction that occurs at the site of the second injection? Select and explain. A. Atopy, B. Anaphylaxis, C. Arthus phenomenon, D. Serum sickness",C: Arthus phenomenon,"Answer: C: Arthus phenomenon Explanation: None" "This is a real-world medical entrance exam question, please give the true answer based on the question and selection. Topic name: None, Subject name: Ophthalmology","on examination 18 month old child has inward detion of both eyes, which of the following test should be done? Select and explain. A. Forced duction test, B. Cover Uncover Test, C. Fundus Examination, D. All of the above",B: Cover Uncover Test,"Answer: B: Cover Uncover Test Explanation: Answer- B- Cover Uncover TestSince the diagnosis is latent strabismus the correct answer is B > Cover-uncover test. It tells about the presence and type of heterophoria. To perform it, one eye is covered with an occluder and the other is made to fix an object. In the presence of heterophoria, the eye under cover will dete. After a few seconds the cover is quickly removed and the movement of the eye (which was under cover) is observed. Direction of movement of the eyeball tells the type of heterophoria >The forced duction test is performed in order to determine whether the absence of movement of the eye is due to a neurological disorder or a mechanical restriction. The anesthetized conjunctiva is grasped with forceps and an attempt is made to move the eyeball in the direction where the movement is restricted. > FUNDUS EXAMINATION This is essential to diagnose the diseases of the vitreous, optic nerve head, retina and choroid." "This is a real-world medical entrance exam question, please give the true answer based on the question and selection. Topic name: None, Subject name: Pathology","Serum sickness syndrome is: Select and explain. A. A transplant immunity, B. An anaphylactic shock, C. Systemic arthus reaction, D. None of the above",C: Systemic arthus reaction,"Answer: C: Systemic arthus reaction Explanation: None" "This is a real-world medical entrance exam question, please give the true answer based on the question and selection. Topic name: None, Subject name: Social & Preventive Medicine","Which of the following is used to denote a continuous variable? Select and explain. A. Simple bar, B. Histogram, C. Pie diagram, D. Multiple bar",B: Histogram,"Answer: B: Histogram Explanation: None" "This is a real-world medical entrance exam question, please give the true answer based on the question and selection. Topic name: None, Subject name: Gynaecology & Obstetrics","What dose of misoprostol is used orally to control bleeding in post paum hemorrhage? Select and explain. A. 400 micrograms, B. 600 micrograms, C. 800 micrograms, D. 1000 micrograms",B: 600 micrograms,"Answer: B: 600 micrograms Explanation: Ans: B. 600 micrograms(Ref Williams 24/e 1785)Approved dose of misoprostol in emergent management of postpaum hemorrhage = 600 pg.Misoprostol (Cytotec):Synthetic prostaglandin E1 analogue.Effective in prevention & treatment of atony & postpaum hemorrhage.Derman (2006) compared a 600 ug oral dose given at delivery against placebo and found that the drug decreased hemorrhage incidence from 12 to 6 percent and that of severe hemorrhage from 1.2 to 0.2 percent." "This is a real-world medical entrance exam question, please give the true answer based on the question and selection. Topic name: None, Subject name: Social & Preventive Medicine","Which of the following is the best for determining the threshold for diagnosis of a positive test? Select and explain. A. Analysis of variance, B. Pearson coefficient, C. Receiver-operating characteristic curve, D. Pre-test probability",C: Receiver-operating characteristic curve,"Answer: C: Receiver-operating characteristic curve Explanation: Ans: C. Receiver-operating characteristic curve(Ref http:ilvvwwlexjansen.com/nesteg/ne.sug10/hl/h107.pdf)Receiver-operating characteristic (ROC) curve:Defined as a plot of test sensitivity as they coordinate versus its I-specificity or false positive rate (FPR) as the x coordinate.Graphic presentation of relationship between both sensitivity & specificity.Uses:Effective method for evaluating diagnostic test's performances.Excellent way to compare diagnostic tests.Best for determining threshold for diagnosis of positive test.Helps decide optimal model through determining best threshold for diagnostic test." "This is a real-world medical entrance exam question, please give the true answer based on the question and selection. Topic name: AIIMS 2017, Subject name: Dental","Highest concentration of desmosomes are seen in which location? Select and explain. A. A, B. B, C. C, D. D",B: B,"Answer: B: B Explanation: Desmosomes help in attaching keratinocytes and are present in Epidermis which rules out Option C & D A is St. basale and B is St. Spinosum. Highest concentration of desmosomes are prominently seen in St. spinosum, So answer is B" "This is a real-world medical entrance exam question, please give the true answer based on the question and selection. Topic name: None, Subject name: Pediatrics","An 8 year old female child following URTI developed maculopapular rash on the jaw spreading onto the trunk which cleared on the 3T day without desquamation and tender post auricular and suboccipital lymphadenopathy. The diagnosis is – Select and explain. A. Kawasaki disease, B. Erythema infectiosiun, C. Rubella, D. Measles",C: Rubella,"Answer: C: Rubella Explanation: None" "This is a real-world medical entrance exam question, please give the true answer based on the question and selection. Topic name: None, Subject name: Pediatrics","A 6 months old child having severe dehydration comes to the casualty with weak pulse and unrecordable B.P. Repeated attempt in gaining I.V. access has failed – The next best step is – Select and explain. A. Try again, B. Jugular vein catheterization, C. Intra osseus I.V. fluids, D. Venesection",C: Intra osseus I.V. fluids,"Answer: C: Intra osseus I.V. fluids Explanation: ""If I.V. route is unsuccessful, introasseus access in the proximal tibia of an uninjured leg is the preferred alternative for children younger than 6 yrs. In children older than 6 yrs a percutaneous femoral venous line should be attempted." "This is a real-world medical entrance exam question, please give the true answer based on the question and selection. Topic name: None, Subject name: Social & Preventive Medicine","All the statements are true about standardization, except Select and explain. A. Standardization allows comparison to be made between two different populations, B. The national population is always taken as the standard population, C. For direct standardization age specific rates of the study population are applied to that of the standard population, D. For indirect standardization age specific rates of the standard population are applied to the study population",B: The national population is always taken as the standard population,"Answer: B: The national population is always taken as the standard population Explanation: Standard population: Is a population where numbers in each age and sex group are known Two frequently used standard populations are: Segi world population European standard population Choice of standard population is arbitrary: Available standard populations may be used Standard population may also be created using 2 populations The national population need not always be taken as the standard population Is commonly used in occupational studies: Comparison of mortality in an industry and general population Can be used for occurrence of disease (rather than death)" "This is a real-world medical entrance exam question, please give the true answer based on the question and selection. Topic name: None, Subject name: Surgery","Which of the following is not removed in radical neck dissection Select and explain. A. Sternocleidomastoid muscle, B. Digastric muscle, C. Spinal accessory nerve, D. Internal jugular vein",B: Digastric muscle,"Answer: B: Digastric muscle Explanation: Answer- B. Digastric muscle* The radical neck dissection consists of removal of all five lymph node groups of one side of the neck (levels I-V).* This includes removal of the sternocleidomastoid muscle, the internal jugular vein, and the spinal accessory nerve." "This is a real-world medical entrance exam question, please give the true answer based on the question and selection. Topic name: None, Subject name: Pharmacology","Loading dose depends on Select and explain. A. Volume of distribution, B. Clearance, C. Rate of administration, D. Half life",A: Volume of distribution,"Answer: A: Volume of distribution Explanation: None" "This is a real-world medical entrance exam question, please give the true answer based on the question and selection. Topic name: None, Subject name: Pathology","A 68 years old man had severe chest pain. The patient died on the way to the hospital. In the hospital, at autopsy tetrazolium chloride staining of the hea was done. What will be the color of ble myocardium? Select and explain. A. Red, B. Blue, C. Dark brown, D. Pink",A: Red,"Answer: A: Red Explanation: Answer- A. RedTriphenyltetrazolium chloride (TTC) stain impas a brick-red color to intact, non-infarcted myocardium where the dehydrogenase enzymes are preserved.If the infarct preceded death by 2 to 3 hours, it is possible to highlight the area of necrosis by immersion of tissue slices in a solution of triphenyltetrazolium chloride. This gross histochemical stain impas a brick-red color to intact, non-infarcted myocardium where lactate dehydrogenase activily is preserved." "This is a real-world medical entrance exam question, please give the true answer based on the question and selection. Topic name: None, Subject name: Pediatrics","A neonate has central cyanosis and short systolic murmur on the 2nd day of birth. The diagnosis is – Select and explain. A. Tetralogy of Fallot's, B. Transposition of great vessels, C. Atrial septal defect, D. Ventricular Septal defect",B: Transposition of great vessels,"Answer: B: Transposition of great vessels Explanation: Out of the four options, only 1st two are cyanotic heart disease i.e. TOF & TGV. TOF can be ruled out as it does not cause cyanosis at birth. Nelson says - ""Often cyanosis is not present of birth, but with increasing hypertrophy of the right ventricular infiindibulum and patient growth, Cyanosis occurs later in the 1st year of life."" Short systolic murmur is not of much help in reaching the diagnosis as it is present in both the conditions. Transposition of great vessels Transposition of great vessels is defined as : - i) Aorta arising from right ventricle. ii)Pulmonary artery arising from the left ventricle. Anatomically TGA is divided into :- A) Complete variety Right atrium is connected to right ventricle and right ventricle is connected to aorta. Left atrium is connected to left ventricle and left ventricle is connected to pulmonary artery. This results in complete separation of systemic and pulmonary circulation : -" "This is a real-world medical entrance exam question, please give the true answer based on the question and selection. Topic name: None, Subject name: Pathology","Christmas disease is due to deficiency of Select and explain. A. Hagernan Factor, B. Platelets, C. Plasma thromboplastin antecedent, D. Plasma thromboplastin component",D: Plasma thromboplastin component,"Answer: D: Plasma thromboplastin component Explanation: None" "This is a real-world medical entrance exam question, please give the true answer based on the question and selection. Topic name: None, Subject name: Pharmacology","Latest drug reccntly approved for Amyotrophic Lateral sclerosis is? Select and explain. A. Piracetam, B. Ceftriaxone, C. Edaravone, D. Doxycycline",C: Edaravone,"Answer: C: Edaravone Explanation: Ans: C. EdaravoneRef href=""https://www/"">https://www.br ai ntrop ic. c om/noo t rop ics/p i rac etam/Edaravone is a novel neuroprotective agent for the treatment of amyotrophic lateral sclerosis (ALS)" "This is a real-world medical entrance exam question, please give the true answer based on the question and selection. Topic name: None, Subject name: Dental","Water content in enamel by volume Select and explain. A. 2-3%, B. 6-7%, C. 20-25%, D. 10-12%",B: 6-7%,"Answer: B: 6-7% Explanation: None" "This is a real-world medical entrance exam question, please give the true answer based on the question and selection. Topic name: AIIMS 2018, Subject name: Surgery","The given below device is/used for: Select and explain. A. Pneumatic compression stocking, B. Varicose vein, C. Hypothermia, D. Cellulitis",A: Pneumatic compression stocking,"Answer: A: Pneumatic compression stocking Explanation: Pneumatic compression stocking is used to prevent DVT in patients with immobilization for longer duration of time. These are helpful in patients at moderate risk for DVT as a prophylactic measure but these are not effective against established thrombosis. It is used especially in bariatric population and can be used in their room, operation room or post-operative area. - DVT should be initially managed by heparin or low-molecular-weight heparin. - Long-term management is done by anti-coagulation with warfarin. - Inferior vena cava filter is indicated for patients not benefiting from medical treatment." "This is a real-world medical entrance exam question, please give the true answer based on the question and selection. Topic name: None, Subject name: Physiology","Autogenic inhibition is afunction of Select and explain. A. Intrafusal fibre, B. Extrafusal fibre, C. Muscle spindle, D. Golgi tendon organ",D: Golgi tendon organ,"Answer: D: Golgi tendon organ Explanation: None" "This is a real-world medical entrance exam question, please give the true answer based on the question and selection. Topic name: None, Subject name: Biochemistry","Which is not a monosaccharide? Select and explain. A. Glucose, B. Galactose, C. Maltose, D. Fructose",C: Maltose,"Answer: C: Maltose Explanation: None" "This is a real-world medical entrance exam question, please give the true answer based on the question and selection. Topic name: None, Subject name: Social & Preventive Medicine","A patient with cough was sputum AFB negative but chest X-ray was suggestive of TB. What should be the next step according to RNTCP? Select and explain. A. Line probe assay, B. Culture, C. Nucleic acid amplification test, D. Tuberculin test",C: Nucleic acid amplification test,"Answer: C: Nucleic acid amplification test Explanation: Answer- C. Nucleic acid amplification testCaridge Based Nucleic Acid Amplification Test (CB-NAAT): The CB'NAAT is known as the GeneXpeft in mostcountries other than India This is prefered first diagnostic test in children and people with TB and HIC co-infection" "This is a real-world medical entrance exam question, please give the true answer based on the question and selection. Topic name: None, Subject name: Social & Preventive Medicine","A 42 years old woman from a dry state who ingested rye for long time presented with complaints of weakness in both lower limbs, nausea and fatigue. Over due course of time, she is completely unable to walk. What is the most likely cause? Select and explain. A. Argemone mexicana, B. Amanita, C. Ergot alkaloids, D. Lathyrus sativus",D: Lathyrus sativus,"Answer: D: Lathyrus sativus Explanation: Ans: D. Lathyrus sativus(Ref Park 24/e p682, 23/e p644, 657).This case is suggestive of food adulteration & neurolathyrism.Caused by Lathyrus sativus.Neurolathyrism:Crippling disease of CNS.Features:Characterized by gradually developing spastic paralysis of lower limbs.Mostly in adults consuming pulse, Lathyrus sativus." "This is a real-world medical entrance exam question, please give the true answer based on the question and selection. Topic name: None, Subject name: Surgery","In Ludwig's angina, the 1st step when patient report to emergency room Select and explain. A. Incision & drainage under local anaesthesia, B. Tracheostomy under general anaesthesia, C. Tracheostomy under local anaesthesia, D. I/V antibiotics and fluids",C: Tracheostomy under local anaesthesia,"Answer: C: Tracheostomy under local anaesthesia Explanation: None" "This is a real-world medical entrance exam question, please give the true answer based on the question and selection. Topic name: None, Subject name: Anatomy","Arterial supply of Submandibular gland is through which branch of ECA: Select and explain. A. Ascending pharyngeal, B. Superior pharyngeal, C. Maxillary, D. Facial",D: Facial,"Answer: D: Facial Explanation: None" "This is a real-world medical entrance exam question, please give the true answer based on the question and selection. Topic name: None, Subject name: Dental","The community periodontal index of treatment needs is recorded for: Select and explain. A. Quadrants, B. Sextants, C. One arch, D. Whole dentition",B: Sextants,"Answer: B: Sextants Explanation: None" "This is a real-world medical entrance exam question, please give the true answer based on the question and selection. Topic name: None, Subject name: Pathology","Syndrome which is characterized by 2X chromosomes and 1Y chromosome is: Select and explain. A. Kleinfelter syndrome, B. Down syndrome, C. Turner syndrome, D. Marfan syndrome",A: Kleinfelter syndrome,"Answer: A: Kleinfelter syndrome Explanation: Klinefelter’s syndrome is associated with an extra X chromosome (47, XXY). Mohan H. Textbook of pathology. Jaypee Brothers Medical Publishers; 2015. Edition 7. Page 198" "This is a real-world medical entrance exam question, please give the true answer based on the question and selection. Topic name: None, Subject name: Radiology","A patient who has been given tumoricidal dose of radiotherapy recently reports to dental clinic with painful carious exposed mandibular first molar with periapical abscess. The treatment of choice should be: Select and explain. A. Extraction of the offending tooth, B. Root canal treatment if possible, C. No treatment should be given, D. Incision and drainage of abscess",B: Root canal treatment if possible,"Answer: B: Root canal treatment if possible Explanation: None" "This is a real-world medical entrance exam question, please give the true answer based on the question and selection. Topic name: None, Subject name: Biochemistry","A material that speeds up a chemical reaction without chemically becoming involved is called: Select and explain. A. A solvent, B. A base, C. A catalyst, D. Thermoplastic",C: A catalyst,"Answer: C: A catalyst Explanation: None" "This is a real-world medical entrance exam question, please give the true answer based on the question and selection. Topic name: None, Subject name: Pathology","Histopathologically rosettes are seen in Select and explain. A. Retinoblastoma, B. Neurocysticercosis, C. PNET, D. Medulloblastoma",A: Retinoblastoma,"Answer: A: Retinoblastoma Explanation: Ans. A retinoblastomaRosettes are little round groupings of cells found in tumors.Usually consist of cells in a spoke-wheel or halo arrangement surrounding a central, acellular region seen commonly in Retinoblastoma.In well-differentiated retinoblastomas, the tumor cells are characteristically arranged in rosettes. The rosettes may be of 2 types-- Flexner-Wintersteiner rosettes characterized by small tumor cells arranged around a lumen with their nuclei away from the lumen, and Homer-Wright rosettes having the radial arrangement of tumor cells around the central neurofibrillar structurePNET (Primitive Neuro-Ectodermal Tumour) :A PNET is composed of sheets of uniform small, round cells that are slightly larger than lymphocytes,Medulloblastoma:Individual tumor cells of medulloblastoma are small, with little cytoplasm and hyperchromatic nuclei that are frequently elongated or crescent-shaped" "This is a real-world medical entrance exam question, please give the true answer based on the question and selection. Topic name: None, Subject name: Medicine","A 20 years old female came with complaints of headache, vomiting and decrease in movement of right leg. In the past, she had episodes of violent and aggressive behaviour and abdominal pain. Which of the following is the most probable diagnosis? Select and explain. A. Conversion disorder, B. Mitochondria disorder, C. Acute inflammatory demyelinating, paralysis, D. Acute intermittent porphyria",D: Acute intermittent porphyria,"Answer: D: Acute intermittent porphyria Explanation: Answer- D. Acute intermittent porphyriaPeriodic abdominal pain (GI symptoms), peripheral neuropathy, headache (neurological symptoms) and psychiatric disorders (aggressive behavior) is highly suggestive of acute intermittent porphyria.Asymptomatic in majority of patients prior to pubeyMC symptom: Abdominal painIleus, abdominal distention & decreased bowel sounds are common.Nausea; vomiting; constipation; tachycardia; hypeension; mental symptoms; pain in limbs, head, neck, or chest; muscle weakness; sensory loss; dysuria; and urinary retention are characteristic.Tachycardia, hypeension, restlessness, tremors & excess sweating are due to sympathetic overactivity.Peripheral neuropathy is due to axonal degeneration and primarily affects motor neurons.Motor neuropathy affects the proximal muscles initially, more often in the shoulders & arms.Mental symptoms: Anxiety, insomnia, depression, disorientation, hallucinations, paranoia & seizures" "This is a real-world medical entrance exam question, please give the true answer based on the question and selection. Topic name: None, Subject name: Gynaecology & Obstetrics","The commonest cause of occipitoposterior position of fetal head during labor is: Select and explain. A. Maternal obesity, B. Multiparity, C. Deflexion of fetal head, D. Android pelvis",D: Android pelvis,"Answer: D: Android pelvis Explanation: In vertex presentation when the occiput is placed posteriorly over the sacroiliac joint or directly over the sacrum. It is called as occiptoposterior position. Causes of Occipitoposterior Position: Most common cause of occipitoposterior position is anthropoid and android pelvis:" "This is a real-world medical entrance exam question, please give the true answer based on the question and selection. Topic name: None, Subject name: Pathology","Which of the following is not associated with gingival Lesions? Select and explain. A. Herpes, B. Recurrent apthae, C. Pemphigus, D. Pyugenic granuloma",B: Recurrent apthae,"Answer: B: Recurrent apthae Explanation: None" "This is a real-world medical entrance exam question, please give the true answer based on the question and selection. Topic name: None, Subject name: Pediatrics","Exclusive breast feeding may be associated with all of the following except – Select and explain. A. Hemolysis due to Vit–K deficiency, B. Evening colic, C. Golden colour stool, D. Prolongation of physiological jaundice",B: Evening colic,"Answer: B: Evening colic Explanation: Breast feeding protects against evening colic. Evening colic may be seen as a manifestation of allergy to cow's milk, but not with breast milk. Haemorrhage due to vit K deficiency may be seen. Breast milk contains very little Vit K - Dutta 4th/e p. 515 Hypoprothrombinemia, may therefore occur along with defeciency of other vit K dependent coagulation factors. (VII, IX, X). This predisposes to haemorrhagic disease in new horns. There is strong association between exclusive breast feeding and neonatal jaundice. It is presumed to be due to inhibitory substance in the breast milk, that intefere with bilirubin conjugation e.g. pregananediol and free fatty acids. Golden colour stool may be seen." "This is a real-world medical entrance exam question, please give the true answer based on the question and selection. Topic name: None, Subject name: Surgery","A 60 years old man presents to the emergency depament with history of A, in unconscious state and gains consciousness in between but again becomes unconscious. The time when he becomes conscious is called as: Select and explain. A. Lucid Interval, B. Extradural hematoma, C. Subdural hematoma, D. None",A: Lucid Interval,"Answer: A: Lucid Interval Explanation: Answer-A. Lucid intervalThis is a period occurring in insanity, during which all the symptoms of insanity disappear completelyThe individual is able to judge his acts soundly, and he becomes legally liable for his actsLucid interval is seen in mania, melanocholia and Extradural hemorrhage." "This is a real-world medical entrance exam question, please give the true answer based on the question and selection. Topic name: None, Subject name: Pharmacology","Which of the following anticholinergic agent does not cross the blood brain barrier? Select and explain. A. Glycopyrrolate, B. Atropine, C. Hyoscine butylbromide, D. Hyoscine hydrobromide",D: Hyoscine hydrobromide,"Answer: D: Hyoscine hydrobromide Explanation: Answer- D. Hyoscine hydrobromideGlycopyrrolate is a synthetic quaternary anticholinergic, which doesn't cross the blood brain barrier and completely lacks central effects." "This is a real-world medical entrance exam question, please give the true answer based on the question and selection. Topic name: None, Subject name: Dental","According to psychosexual theory development of personality and skills occurs: Select and explain. A. Anal stage., B. Phallic stage., C. Latency stage., D. Genital stage.",C: Latency stage.,"Answer: C: Latency stage. Explanation: None" "This is a real-world medical entrance exam question, please give the true answer based on the question and selection. Topic name: None, Subject name: Surgery","On radiograph of upper lateral incisor a periapical lesion is found after 2 years following RCT curretage was done and histopathology showed presence of apical cyst. Lesion continued to enlarge even after surgery, What is the reason for this. Select and explain. A. Leaking from main canal, B. Unobturated accessory canal, C. Apex was not resected, D. Actinomycoses infection",A: Leaking from main canal,"Answer: A: Leaking from main canal Explanation: None" "This is a real-world medical entrance exam question, please give the true answer based on the question and selection. Topic name: None, Subject name: Physiology","Which of the following nerve root is the control center for the stapedial reflex? Select and explain. A. Superior olivary complex, B. Lateral lemniscus, C. Inferior colliculus, D. Medial geniculate body",A: Superior olivary complex,"Answer: A: Superior olivary complex Explanation: Ans: A. Superior olivary complex(Ref Gray's 40/e p628, Ganong 25/e 207, 24/e p207)Superior olivary complex - Control center for stapedial reflex." "This is a real-world medical entrance exam question, please give the true answer based on the question and selection. Topic name: None, Subject name: Pathology","KLK4 gene mutation is associated with which type of amelogenesis imperfecta? Select and explain. A. Hypoplastic, B. Hypomaturation, C. Hypocalcification, D. Hypoplastic hypomaturation",B: Hypomaturation,"Answer: B: Hypomaturation Explanation: None" "This is a real-world medical entrance exam question, please give the true answer based on the question and selection. Topic name: AIIMS 2017, Subject name: Physiology","Absolute refractoriness of a neuron is due to? Select and explain. A. Closure of activated Na channels, B. Closure of inactivated Na channels, C. Hyperpolarization of CI channels, D. Opening of rectifier K + channels",B: Closure of inactivated Na channels,"Answer: B: Closure of inactivated Na channels Explanation: Absolute refractory period: The period during which 2nd action potential cannot be elicited, even with strong stimulus is called ""ABSOLUTE REFRACTORY PERIOD"". Reason for this restriction is that sholy after action potential is initiated, the sodium channels (or calcium channels / both) become inactivated & no amount of excitatory signal applied to these channels at this point will open inactivated gates. Only condition that will allow them to reopen is when membrane potential returns to or gets close to original resting membrane potential level. Then, within another small fraction of second, inactivated gates of channels open & new action potential can be initiated This period for large myelinated nerve fibers is about 1/2500 second thus, such fibers can transmit maximum of about 2500 impulses per second - Na have 2 gates, activated gate & an inactivated gate. At resting state - active gate - closed - inactive gate - open At stimulus - both gates are open At absolute refractory period - active gate - open; inactive gate - closed Next action potential is generated only after inactivated gate is open & both gates go back to resting state." "This is a real-world medical entrance exam question, please give the true answer based on the question and selection. Topic name: None, Subject name: Pathology","Dinesh, a 24-year-old male, complains of loose teeth in a single quadrant. His radiograph shows irregular bone loss and histopathology reveals eosinophils and histiocytes. The most probable diagnosis is: Select and explain. A. Hand-Schuller-Christian disease, B. Paget's disease, C. Osteoclastoma, D. Albright's syndrome",A: Hand-Schuller-Christian disease,"Answer: A: Hand-Schuller-Christian disease Explanation: None" "This is a real-world medical entrance exam question, please give the true answer based on the question and selection. Topic name: None, Subject name: Pathology","Perception of taste even in absence of stimuli is known as Select and explain. A. Ageusia, B. Dysguesia, C. Cocoguesia, D. Phantoguesia",D: Phantoguesia,"Answer: D: Phantoguesia Explanation: None" "This is a real-world medical entrance exam question, please give the true answer based on the question and selection. Topic name: None, Subject name: Surgery","The anesthetic drug injected for paravertebral block is least likely to diffuse to Select and explain. A. Epidural space, B. Subarachnoid space, C. Intercostal space, D. Superior and inferior paravertebral space",B: Subarachnoid space,"Answer: B: Subarachnoid space Explanation: None" "This is a real-world medical entrance exam question, please give the true answer based on the question and selection. Topic name: None, Subject name: Dental","Which of the local anesthetic agent can be given in liver failure cases? Select and explain. A. Articaine, B. Lignocaine, C. Bupivacaine, D. Benzocaine",D: Benzocaine,"Answer: D: Benzocaine Explanation: Benzocaine can be given in liver failure patients since it is ester type, and can be metabolized by cholinesterase in plasma." "This is a real-world medical entrance exam question, please give the true answer based on the question and selection. Topic name: None, Subject name: Medicine","To assure surgical hemostasis, the best blood product administered preoperatively to patients with classic hemophilia is: Select and explain. A. Whole blood, B. Factor 8 concentrate, C. Fresh frozen plasma, D. Factor 9 concentrate",B: Factor 8 concentrate,"Answer: B: Factor 8 concentrate Explanation: None" "This is a real-world medical entrance exam question, please give the true answer based on the question and selection. Topic name: None, Subject name: Dental","Which of the following clasp is preferable on a partially erupted tooth for adequate retention of removable orthodontic appliance? Select and explain. A. Circumferential clasp, B. Adams clasp, C. Jacksons clasp, D. South end clasp",B: Adams clasp,"Answer: B: Adams clasp Explanation: None" "This is a real-world medical entrance exam question, please give the true answer based on the question and selection. Topic name: AIIMS 2019, Subject name: Pediatrics","About Juvenile Myoclonic epilepsy, all are true EXCEPT: Select and explain. A. Valproate is contraindicated, B. Lamotrigine can be given, C. Phenytoin is not the preferred drug, D. Polygenic inheritance",A: Valproate is contraindicated,"Answer: A: Valproate is contraindicated Explanation: Juvenile Myoclonic epilepsy: also known as Janz syndrome Seen in adoloscent age group Typically presents with myoclonic jerks causing patient to drop objects GTCS, clonic seizures and juvenile absences can also be seen Attacks are more prominent during early morning Drug of choice: Valproate; Lamotrigine can also be used" "This is a real-world medical entrance exam question, please give the true answer based on the question and selection. Topic name: None, Subject name: Biochemistry","Enzymes are classified according to: Select and explain. A. Action on substrate, B. Action of coenzymes, C. Nature of the enzymes, D. All of the above",A: Action on substrate,"Answer: A: Action on substrate Explanation: None" "This is a real-world medical entrance exam question, please give the true answer based on the question and selection. Topic name: AIIMS 2019, Subject name: Ophthalmology","Enlargement of the blind spot occurs in which of the following Select and explain. A. Primary open angle glaucoma, B. Diabetic macular edema, C. Optic nerve hypoplasia, D. Papilledema",D: Papilledema,"Answer: D: Papilledema Explanation: Primary open angle glaucoma - baring of the blind spot Diabetic macular edema - doesn't lead to any visual field defect Optic nerve hypoplasia - it is not developed enough to develop a proper visual field defect Papilledema (it is raised intracranial pressure causing disc edema ) - leads to enlargement of blind spot" "This is a real-world medical entrance exam question, please give the true answer based on the question and selection. Topic name: AIIMS 2019, Subject name: Surgery","Percentage of surgical site infection in patients with clean contaminated wound after prophylactic antibiotic? Select and explain. A. 1-2%, B. <10%, C. 10-20%, D. 20-30%",B: <10%,"Answer: B: <10% Explanation: Wounds classification: Clean - 5% Clean contaminated - 10% Contaminated - 20-30% Diy - 30-40% Wounds Class Definition I: Clean Uninfected operative wound without inflammation Respiratory, alimentary, genital or infected urinary tract is not entered Wounds are closed primarily, if necessary drained with closed drain Examples of Clean Wound Inguinal hernia Joint replacement Thyroidectomy Mastectomy Abdominal aoic aneurysm (AAA) repair II: Clean contaminated Operative wound in which respiratory tract GIT or genitourinary tract is entered under controlled condition without unusual contamination Examples of Clean Contaminated Wound Cholecystectomy CBD exploration Elective GI surgeries (elective colonic resection, gastrectomy) III: Contaminated Open, fresh accidental wounds Operations with major break in sterile techniques Gross spillage from GIT Incision in which acute non-purulent inflammation is encountered Examples of Contaminated Wound Spill during elective GI surgery Perforated gastric ulcer Appendicular perforation Penetrating abdominal trauma Enterotomy during bowel obstruction Human bite Open fracture TYPE OF SURGERY INFECTION RATE WITH PROPHYLAXIS IN % INFECTION RATE WITHOUT PROPHYLAXIS IN % CLEAN(no viscus opened) 1-2 1-2 CLEAN CONTAMINATED (viscus opened, minimal spillage) 3 6-9 CONTAMINATED (open viscus with spillage or inflammatory disease) 6 13-20 DIY (pus/perforation/incision through abscess) 7 40 Table no. 5.4 from Bailey and love's Sho practice of surgery, 27 Ed , Pg no-53" "This is a real-world medical entrance exam question, please give the true answer based on the question and selection. Topic name: None, Subject name: Surgery","Direct impact on the bone will produce a: Select and explain. A. Transverse fracture, B. Oblique fracture, C. Spiral fracture, D. Comminuted fracture",A: Transverse fracture,"Answer: A: Transverse fracture Explanation: None" "This is a real-world medical entrance exam question, please give the true answer based on the question and selection. Topic name: None, Subject name: Social & Preventive Medicine","A psychiatrist is not posted at: Select and explain. A. PHC, B. Military hospitals, C. District hospitals, D. Hospitals with medical colleges",A: PHC,"Answer: A: PHC Explanation: Ans: A. PHC(Ref Park 24/e p944, 23/e p9Ob, 221e p847).A psychiatrist is not posted at PHC.For sub-centre area of PHCApa from the essential staff, the desirable staff for both type A and type B PHC are:One of the two medical officers (MBBS) should be lady doctor, if the delivery case load is 30 or more per month.One AYUSH medical officer to provide choice to the people, where as AYUSH facility is not available in the vicinity.One staff nurse/nurse midwife.One health educator at the PHC.That makes total staff at type A PHC 18 and at type B PHC 21." "This is a real-world medical entrance exam question, please give the true answer based on the question and selection. Topic name: None, Subject name: Dental","For posterior abutment with acceptable crown root ratios, average crown length, normal esthetic requirement, retainer of choice is: Select and explain. A. Full crown, B. Partial veneer crown, C. Resin veneer, D. PFM",B: Partial veneer crown,"Answer: B: Partial veneer crown Explanation: None" "This is a real-world medical entrance exam question, please give the true answer based on the question and selection. Topic name: None, Subject name: Gynaecology & Obstetrics","A 45 years old patient presented with complaints of pain in abdomen and menorrhagia. Endometrial biopsy was normal and sonogram of uterus showed diffusely enlarged uterus with no adnexal mass. What is the diagnosis? Select and explain. A. Fibroid uterus, B. Endometritis, C. Endometriosis, D. Adenomyosis",D: Adenomyosis,"Answer: D: Adenomyosis Explanation: Ans: D: Adenomyosis(Ref: Shaw's 16Ie p413-413, 13/c p4T 3; Novak's 13Ie p184; Robbins 9Ie p1012)Clinical features like abdominal pain and menorrhagia with normal endometrial biopsy and on ultrasound diffuse, symmetrical enlargement of uterus, in a perimenopausal women without any focal lesion is highly suggestive of Adenomyosis.Clinical symptoms include menometrorrhagia (irregular and heavy menses), colicky dysmenorrhea, dyspareunia, and pelvic pain, paicularly during the premenstrual period.Coexist with endometriosis.Often asymptomatic.Uterus is diffusely enlarged, although usually less than 14 cm in size, and is often soft and tender, paicularly at the time of menses." "This is a real-world medical entrance exam question, please give the true answer based on the question and selection. Topic name: None, Subject name: Dental","A patient is involved in chronic mouth breathing, the clinical examination of the patient reveals Select and explain. A. Convex profile, long face, narrow arches, B. Concave profile, Long face, broad arches, C. Convex profile, short face, broad arches, D. Concave profile, short face, narrow arches","A: Convex profile, long face, narrow arches","Answer: A: Convex profile, long face, narrow arches Explanation: None" "This is a real-world medical entrance exam question, please give the true answer based on the question and selection. Topic name: None, Subject name: Anatomy","All of the following bones are involves in wrist joint formation; except: Select and explain. A. Scaphoid, B. Ulna, C. Radius, D. Triquetral",B: Ulna,"Answer: B: Ulna Explanation: Ans. b. UlnaWrist Joint (Radiocarpal Joint)It is a synol ellipsoid joint.It is formed by aiculation of the distal end of the radius and the aicular disc above and the scaphoid, lunate, and triquetral bones below.Ulna excluded from the wrist joint by the aicular disc." "This is a real-world medical entrance exam question, please give the true answer based on the question and selection. Topic name: None, Subject name: Pathology","Hodgkin's disease is considered to be: Select and explain. A. Follicular reticulosis, B. Inflammatory disease, C. Chronic granulomatous disease, D. A malignant neoplasm",D: A malignant neoplasm,"Answer: D: A malignant neoplasm Explanation: None" "This is a real-world medical entrance exam question, please give the true answer based on the question and selection. Topic name: None, Subject name: Dental","Majority of oral microorganisms are: Select and explain. A. Strict anaerobes, B. Gram-positive bacilli, C. Spirochetes, D. Facultative anaerobes",D: Facultative anaerobes,"Answer: D: Facultative anaerobes Explanation: None" "This is a real-world medical entrance exam question, please give the true answer based on the question and selection. Topic name: None, Subject name: Pediatrics","Which of the following is considered as high risk infant? Select and explain. A. Working mother, B. Antenatal preeclampsia, C. Third child, D. Twins",A: Working mother,"Answer: A: Working mother Explanation: Ans. A. Working mother Identification of 'At Risk' Infants * Bih weight < 2.5 Kg* Bih order 5 or more* Aificial feeding* Weight below 70% of expected weight (i.e. grade II & III malnutrition)* Failure to gain weight during 3 successive months* Children with PEM, Diarrhea.* Working mother, one parent.* Spacing of less than 1 year." "This is a real-world medical entrance exam question, please give the true answer based on the question and selection. Topic name: None, Subject name: Ophthalmology","Cells affected in glaucomatous optic neuropathy are: Select and explain. A. Amacrine cells, B. Bipolar cells, C. Ganglion cells, D. Rods and cones",C: Ganglion cells,"Answer: C: Ganglion cells Explanation: Answer- C. Ganglion cellsGlaucoma, the second leading cause ofblindness, is characterized by changes in the optic disc and visual field defects.The elevated intraocular pressure was considered the prime factor responsible for the glaucomatous optic neuropathy involving death of retinal ganglion cellsaand their axons.Glaucoma, a leading cause of irreversible visual loss, is characterized by loss of retinal ganglion cells (RGC) and their axons over a period of many years.Mainly the ganglions cellsa are affected in glaucoma patients, which may lead to glaucomatus optic atrophy.Glaucomatous optic neuropathy is characterized by changes in the optic disc and visual field defects.The morphologic changes in the optic disc are in the form of thinning of neuro retinal rim, pallor and progressive cupping of the optic disc.The hemorrhage-associated retinal nerve fiber layer defects precede measurable changes ofthe optic disc configuration.The visual field defects in glaucoma are often detected only after 40% of the axons are loss." "This is a real-world medical entrance exam question, please give the true answer based on the question and selection. Topic name: None, Subject name: Pediatrics",". The most common fungal infection in the neonates transmitted by caregiver's hand is: Select and explain. A. Candida albicans, B. Candida glabrata, C. Candida tropicalis, D. Candida parapsilosis",D: Candida parapsilosis,"Answer: D: Candida parapsilosis Explanation: Ans. d. Candida parapsilosis""C. parapsilosis infections are esptecially associated with hyperalimentation solutions, prosthetic devices, and indwellingcatheters, as well as the nosoeomial sprcad of disease throagh the hands of health care workers" "This is a real-world medical entrance exam question, please give the true answer based on the question and selection. Topic name: AIIMS 2018, Subject name: Surgery","A Young Male complained of intermittent pain , swelling and discharge at the base of spine. He also had episodes of fever and repeated abscesses that had burst spontaneously , in recent past. By occupation , he is a jeep driver. On physical examination, diagnosis of pilonidal sinus was made. Which flap-based procedure is used for pilonidal sinus surgery? Select and explain. A. Rhomboid flap, B. Circular flap, C. Free flap, D. Rotational flap",A: Rhomboid flap,"Answer: A: Rhomboid flap Explanation: Pilonidal sinus surgery - excision of pilonidal sinus and cyst and repair of defect by flap. Flaps used: 1. Limber's flap 2.Karydakis flap 3. Rhomboid flap 4. Bascom procedure" "This is a real-world medical entrance exam question, please give the true answer based on the question and selection. Topic name: None, Subject name: Dental","Insufficient closure of the flasks during packing of denture base resin can result in: Select and explain. A. Poor colour stability, B. Less polymerization shrinkage, C. Increase in vertical dimension, D. Decrease in vertical dimension",C: Increase in vertical dimension,"Answer: C: Increase in vertical dimension Explanation: None" "This is a real-world medical entrance exam question, please give the true answer based on the question and selection. Topic name: AIIMS 2019, Subject name: Dental","Which is a true association: Select and explain. A. Lichen planus and malignant melanoma, B. Melasma and malignant melanoma, C. Psoriasis and metabolic syndrome, D. Vitiligo and metabolic syndrome",C: Psoriasis and metabolic syndrome,"Answer: C: Psoriasis and metabolic syndrome Explanation: Chronic plaque psoriasis is an immune mediated inflammatory skin disease that is strongly associated with the clinical features of the metabolic syndrome (Mets), including abdominal obesity, hypeension, atherogenic dyslipidemia, type 2 diabetes, insulin resistance and nonalcoholic atty liver disease. There are complex interrelationships between the metabolic syndrome and chronic inflammation. Vitiligo is associated with autoimmune diseases like Thyroid diseasea (hypehyroidism and hypothyroidism ), Pernicious anaemia, Addison disease, Diabetes, Myasthenia gravis, Alopecia areata" "This is a real-world medical entrance exam question, please give the true answer based on the question and selection. Topic name: None, Subject name: Dental","Dental plaque adheres to the tooth because: Select and explain. A. Levans are gummy, B. Dextrans are insoluble and sticky, C. Plaque grows into the irregularities, D. Microorganisms produce sticky lipoproteins",B: Dextrans are insoluble and sticky,"Answer: B: Dextrans are insoluble and sticky Explanation: None" "This is a real-world medical entrance exam question, please give the true answer based on the question and selection. Topic name: None, Subject name: Dental","Arestin is: Select and explain. A. 2% minocycline, B. 1% minocycline, C. 0.5% minocycline, D. 0.1% minocycline",A: 2% minocycline,"Answer: A: 2% minocycline Explanation: None" "This is a real-world medical entrance exam question, please give the true answer based on the question and selection. Topic name: None, Subject name: Physiology","Histamine is present on surface of: Select and explain. A. Lymphocytes, B. Mast cells, C. Neutrophils, D. Erythrocytes",B: Mast cells,"Answer: B: Mast cells Explanation: None" "This is a real-world medical entrance exam question, please give the true answer based on the question and selection. Topic name: None, Subject name: Medicine","Physical sign of severe aortic stenosis is: Select and explain. A. A late peaking holosystolic murmur, B. A delayed carotid upstroke, C. A diastolic rumble, D. An opening snap",B: A delayed carotid upstroke,"Answer: B: A delayed carotid upstroke Explanation: None" "This is a real-world medical entrance exam question, please give the true answer based on the question and selection. Topic name: None, Subject name: Surgery","Nasal antrostomy usually done from through Select and explain. A. Middle concha, B. Inferior concha, C. Middle meatus, D. Inferior meatus",D: Inferior meatus,"Answer: D: Inferior meatus Explanation: None" "This is a real-world medical entrance exam question, please give the true answer based on the question and selection. Topic name: None, Subject name: Dental","Increased VDO results in Select and explain. A. Strained facial appearance, B. Trauma to underlying tissues, C. Clicking of teeth, D. All of the above",D: All of the above,"Answer: D: All of the above Explanation: None" "This is a real-world medical entrance exam question, please give the true answer based on the question and selection. Topic name: None, Subject name: Anatomy","Mesodermal basis of lateral part of lip is formed by: Select and explain. A. Frontonasal process., B. Maxillary process., C. Globular process., D. All of the above",B: Maxillary process.,"Answer: B: Maxillary process. Explanation: None" "This is a real-world medical entrance exam question, please give the true answer based on the question and selection. Topic name: None, Subject name: Anatomy","Sensory supply of cornea is by? Select and explain. A. Infraorbital nerve, B. Supraorbital nerve, C. Infratrochlear nerve, D. Nasolacrimal nerve (probably printing mistake - probably Nasociliary nerve).",D: Nasolacrimal nerve (probably printing mistake - probably Nasociliary nerve).,"Answer: D: Nasolacrimal nerve (probably printing mistake - probably Nasociliary nerve). Explanation: Ans: D. Nasolacrimal nerve (probably printing mistake - probably Nasociliary nerve).Cornea:Richly supplied by nerves (without myelin sheaths & Schwann cell sheath).Extremely sensitive structure - Due to its dense nerve supply.Ooriginate from small ophthalmic division of trigeminal nerve, mainly by long ciliary nerve.Long ciliary nerve - Branch of Nasociliary nerve." "This is a real-world medical entrance exam question, please give the true answer based on the question and selection. Topic name: None, Subject name: Dental","Average Diameter of Apical foramen in an 18-25 years old adult is Select and explain. A. 400 micrometer, B. 502 micrometer, C. 681 micrometer, D. 602 micrometer",B: 502 micrometer,"Answer: B: 502 micrometer Explanation: None" "This is a real-world medical entrance exam question, please give the true answer based on the question and selection. Topic name: None, Subject name: Skin","A patient came with history of joint pain and fever for one week. For which the patient took NSAIDS foilowing which he developed rash and hyperpigmentation on nose. What is the most likelv diagnosis? Select and explain. A. Dengue, B. fixed Drug eruption, C. Chikungunya, D. melasma",C: Chikungunya,"Answer: C: Chikungunya Explanation: Ans : C ChikungunyaRef: Hypapigmqtdion and &ilangurwfetEn &hasnw A"" An has Demaor 20 I 6;9 I :g60-I r.Chikungunya fever (cF) is an acute febrile illness presenting with symptoms like intense asthenia, ahralgia, myalgia and headache and is caused by chikungunya virus (CV).Among the skin lesions, maculopapular rash is common, but seen in several viral illnesses, therefore, not useful in suspecting CF.Hyperpigmentation is a unique feature noted in CF.Hyperpigmentation associated with CF is macular and most commonly affects nose and cheeks." "This is a real-world medical entrance exam question, please give the true answer based on the question and selection. Topic name: None, Subject name: ENT","Which focal length in the objective piece of microscope is commonly used for ear surgery? Select and explain. A. 100mm, B. 250 mm, C. 450mm, D. 950 mm",B: 250 mm,"Answer: B: 250 mm Explanation: Focal length of objective lens: For ear surgery - 200 mm/ 250 mm For Nose / Paranasal sinus surgery - 300 mn For Laryngeal surgery-400 mn" "This is a real-world medical entrance exam question, please give the true answer based on the question and selection. Topic name: None, Subject name: Gynaecology & Obstetrics","What is the level of proteinuria to diagnose severe preeclampsia? Select and explain. A. 20 mg, B. 200 mg, C. 300 mg, D. 3000 mg",C: 300 mg,"Answer: C: 300 mg Explanation: Ans: C. 300 mg(Ref: Hypeension in pregnancy (ACOG laskforce on hypeension in pregnancy)-Obstetrics and gynaecology, Vol-122, No.5, November 2013; William's 24/e p181; Danfoh 10/e p264)Diagnostic criteria for pre-eclampsia:Proteinuria - 2300 mg per 24 hour urine collection.Removed from essential criterion for pre-eclampsia diagnosis by American College of Obstetricians and Gynecologists in 2013.In absence of diagnostic requirement of 5 gm as massive proteinaria for severe eclampsia - General definition of proteinuria (>300 mg) sufficient." "This is a real-world medical entrance exam question, please give the true answer based on the question and selection. Topic name: None, Subject name: Pharmacology","Not true about piperacilline is: Select and explain. A. Active against pseudomonas aeruginosa, B. Inhibits Beta-lactamase, C. It is a ureido penicillin, D. Given parenterally",B: Inhibits Beta-lactamase,"Answer: B: Inhibits Beta-lactamase Explanation: None" "This is a real-world medical entrance exam question, please give the true answer based on the question and selection. Topic name: None, Subject name: Pediatrics","Which of the following disorder of mother leads to microcephaly in baby –a) SLEb) Hepatitis A c) Phenylketonuriad) Rubella Select and explain. A. ab, B. cd, C. bd, D. ac",B: cd,"Answer: B: cd Explanation: Microcephaly is defined as a head circumference that measures more than three standard deviation below the mean for age. Causes of microcephaly" "This is a real-world medical entrance exam question, please give the true answer based on the question and selection. Topic name: AIIMS 2018, Subject name: Anatomy","Left superior intercostal vein drains into ? Select and explain. A. Azygous vein, B. Hemiazygous vein, C. Brachiocephalic vein, D. Internal thoracic vein",C: Brachiocephalic vein,"Answer: C: Brachiocephalic vein Explanation: 2,3,4-posterior intercostal vein on left side drains into left superior intercostal which fuher drains into left brachiochephalic vein . Right and left brachiocephalic veins joins to form superior vena cava that opens into right atrium Posterior Intercostal Veins and their drainage: Right side drains into 1stposterior intercostal vein Right brachiocephalic vein 2,3,4 posterior intercostal vein Arch of azygos vein 5 to 11 posterior intercostal vein Azygos vein Left side drains into 1stposterior intercostal vein Left brachiocephalic vein 2,3,4-posterior intercostal vein Left brachiocephalic vein 5,6,7 posterior intercostal vein Superior hemiazygous vein 8,9,10 posterior intercostal vein Inferior hemiazygous vein" "This is a real-world medical entrance exam question, please give the true answer based on the question and selection. Topic name: None, Subject name: Dental","Bakers anchorage is a type of: Select and explain. A. Intramaxillary anchorage, B. Intermaxillary anchorage, C. Extraoral anchorage, D. Muscular anchorage",B: Intermaxillary anchorage,"Answer: B: Intermaxillary anchorage Explanation: Baker Anchorage In most of the malocclusion cases, the upper teeth protruded excessively and the lower teeth and jaw seemed to be underdeveloped. Obviously, the mere expansion of the upper and lower arches would not achieve the desired correction. Some means had to be found to correct the protrusion of the upper teeth and the underdevelopment of the lower teeth and jaws. This was achieved by so-called Baker anchorage, a method of using intermaxillary rubber bands." "This is a real-world medical entrance exam question, please give the true answer based on the question and selection. Topic name: None, Subject name: Anatomy","Early fusion of the coronal suture results in Select and explain. A. Brachycephaly, B. Oxycephaly, C. Trigonocephaly, D. Scaphocephaly",A: Brachycephaly,"Answer: A: Brachycephaly Explanation: None" "This is a real-world medical entrance exam question, please give the true answer based on the question and selection. Topic name: None, Subject name: Dental","In 1921, school dental nurse was started in Select and explain. A. Adelaide, B. Wellington, C. New cross, D. Connecticut",B: Wellington,"Answer: B: Wellington Explanation: None" "This is a real-world medical entrance exam question, please give the true answer based on the question and selection. Topic name: None, Subject name: Dental","The proximal walls of a class II cavity on a deciduous molar: Select and explain. A. Follow directions of enamel rods, B. Converge toward occlusal surface, C. Diverge toward occlusal surface, D. Undermine enamel which is very thin",B: Converge toward occlusal surface,"Answer: B: Converge toward occlusal surface Explanation: None" "This is a real-world medical entrance exam question, please give the true answer based on the question and selection. Topic name: None, Subject name: Social & Preventive Medicine","Which of the following is used for selecting patients with respect to potential factors that will affect the results? Select and explain. A. Systematic random sampling, B. Simple random sampling, C. Stratified random sampling, D. Cluster sampling",C: Stratified random sampling,"Answer: C: Stratified random sampling Explanation: Ans: C. Stratified random samplingRef. Park 24/e p886, 23/e p850, 22/e p792; BK Malunan 6th/100-101).Stratified random sampling is used for selecting patients with respect to potential factors that will affect the results.Stratified random sample:The sample is deliberately drawn in a systematic way so that each poion of the sample represents corresponding strata of universe.This method is paicularly useful where one is interested in analyzing the data by a ceain characteristic of population, viz. Hindus, Christians, Muslims, age-groups etc." "This is a real-world medical entrance exam question, please give the true answer based on the question and selection. Topic name: None, Subject name: Physiology","Which of the following is not a physiological method of heat loss from the body? Select and explain. A. Sweating, B. Vasodilation, C. Posture, D. Peripheral vasoconstriction",D: Peripheral vasoconstriction,"Answer: D: Peripheral vasoconstriction Explanation: None" "This is a real-world medical entrance exam question, please give the true answer based on the question and selection. Topic name: None, Subject name: Dental","Single central pit is formed on the occlusal surface of Select and explain. A. Lower 1st premolar, B. Upper 1st premolar, C. Lower 2nd premolar, D. Upper 2nd premolar",C: Lower 2nd premolar,"Answer: C: Lower 2nd premolar Explanation: None" "This is a real-world medical entrance exam question, please give the true answer based on the question and selection. Topic name: None, Subject name: Surgery","After a surgery, the surgeon asked the intern to remove the Foley's catheter but he could not do it. The surgeon himself tried to remove the Foley's catheter but he was unsuccessful. What should be done next? Select and explain. A. CT-guided rupture of bulb of Foley's, B. Inject ether to dissolve the balloon and pull it out, C. Inject water to overdistend the balloon until it bursts and Foley's can be removed, D. Use ultrasound guidance to locate and prick the balloon and then remove the catheter.",D: Use ultrasound guidance to locate and prick the balloon and then remove the catheter.,"Answer: D: Use ultrasound guidance to locate and prick the balloon and then remove the catheter. Explanation: Ans: D. Use ultrasound guidance to locate and prick the balloon and then remove the catheter.http://www.aafp. orglafp12000109151p1397).Best technique in this situation is ultrasound-guided rupture of balloon." "This is a real-world medical entrance exam question, please give the true answer based on the question and selection. Topic name: None, Subject name: Pathology","True about ythT cells: Select and explain. A. Are helper t cells, B. Oligoclonal proliferation, C. Antigen presentation, D. Opsonization",B: Oligoclonal proliferation,"Answer: B: Oligoclonal proliferation Explanation: Ans: B. Oligoclonal proliferationCell TypeSubsetMain mediatorsEstablished roleFindings in allergic rhinitis (AR)gd T cell* IFNg * IL-17A * IL-17F* IL-22* proinflammatory and immune-modulating functions at epithelial surfaces* innate and adaptive immunity paicipation* ydT cytokines induce B cell IgE synthesis* suppo Th2 inflammatory response* oligoclonal proliferation in nasal mucosa (?)* | circulating ydT cell percentages correlating with Th17 numbers in AR* negative correlation with Tregs" "This is a real-world medical entrance exam question, please give the true answer based on the question and selection. Topic name: None, Subject name: Dental","Lateral shift (Bennett's movement) influences: Select and explain. A. Faciolingual position of teeth, B. Mesiodistal position of the teeth, C. Both of the above, D. None of the above",A: Faciolingual position of teeth,"Answer: A: Faciolingual position of teeth Explanation: None" "This is a real-world medical entrance exam question, please give the true answer based on the question and selection. Topic name: None, Subject name: Anatomy","Abnormal development of a joint is known as: Select and explain. A. Synostosis., B. Dysostosis., C. Both, D. None",A: Synostosis.,"Answer: A: Synostosis. Explanation: None" "This is a real-world medical entrance exam question, please give the true answer based on the question and selection. Topic name: None, Subject name: Dental","For a loose complete denture relining and rebasing can be done in which of the following condition Select and explain. A. Excessive alveolar bone loss, B. When centric relation and centric occlusion do not coincide, C. The patient is poor and cannot afford new denture, D. More than 2mm loss of alveolar bone height",C: The patient is poor and cannot afford new denture,"Answer: C: The patient is poor and cannot afford new denture Explanation: None" "This is a real-world medical entrance exam question, please give the true answer based on the question and selection. Topic name: None, Subject name: Dental","Odontoblastic layer’s cell size: Select and explain. A. Directly depends on no. of dentinal tubules, B. Tall columnar in crown, C. Squamous in apical portion, D. All of the above",D: All of the above,"Answer: D: All of the above Explanation: None" "This is a real-world medical entrance exam question, please give the true answer based on the question and selection. Topic name: None, Subject name: Dental","Percentage of Phosphoric acid where Dicalcium phosphate monohydrate is formed that cannot be rinsed off. Select and explain. A. 25%, B. 37%, C. 50%, D. 85%",A: 25%,"Answer: A: 25% Explanation: None" "This is a real-world medical entrance exam question, please give the true answer based on the question and selection. Topic name: None, Subject name: Dental","Preferred haemostatic agents for perforation repair include all except? Select and explain. A. Freeze dried bone, B. Calcium Hydroxide, C. Ferric Sulphate, D. Collagen",C: Ferric Sulphate,"Answer: C: Ferric Sulphate Explanation: None" "This is a real-world medical entrance exam question, please give the true answer based on the question and selection. Topic name: None, Subject name: Medicine","A patient with Hb 7 was to be transfused with 2 packs of blood. The first pack was transfused in 2 hours after which vitals were stable and then the transfusion of the next pack was staed . But suddenly the patient develops breathlessness and hypeension. What can be the cause of this sudden reaction? Select and explain. A. transfusion related circulatory overload (TACO), B. allergic reaction to transfused blood, C. transfusion related acute lung injury (TRALI), D. transfusion related acute renal failure",C: transfusion related acute lung injury (TRALI),"Answer: C: transfusion related acute lung injury (TRALI) Explanation: Answer- C. transfusion related acute lung injury (TRALI)Transfusion-related acute lung injury (TRALI) is a serious blood transfusion complication characterized by the acute onset of non-cardiogenic pulmonary edema following transfusion of blood products.Hypotension, and fever that develop within 6 hours after transfusion and usually resolve with suppoive care within 48 to 96 hours. Although hypotension is considered one of the impoant signs in diagnosing TRALI, hypeension can occur in some cases." "This is a real-world medical entrance exam question, please give the true answer based on the question and selection. Topic name: None, Subject name: Medicine","A 50 years old male presented with frontal bossing, enlarged nasal bone, enlarged jaw and spade like fingers. Which of the following test will you do for diagnosis? Select and explain. A. IGF1, B. ACTH, C. TSH, D. Serum coisol",A: IGF1,"Answer: A: IGF1 Explanation: Answer- A. IGF1An IGF-I level provides a useful laboratory screening measure when clinical features raise the possibility of acromegaly.Acromegaly-Diagnosis:Screening investigation of choice in acromegaly: IGF-lConfirmatory & Gold standard: Oral Glucose Tolerance tests" "This is a real-world medical entrance exam question, please give the true answer based on the question and selection. Topic name: None, Subject name: Surgery","Which of the following inducing agent has analgesic property? Select and explain. A. Nitrous oxide, B. Hatothane, C. Enflurane, D. Sevoflurane",A: Nitrous oxide,"Answer: A: Nitrous oxide Explanation: None" "This is a real-world medical entrance exam question, please give the true answer based on the question and selection. Topic name: None, Subject name: Anatomy","Which of the following is not present in posterior triangle of neck? Select and explain. A. Phrenic nerve, B. Subclavian nerve, C. External jugular vein, D. Hypoglossal nerve",D: Hypoglossal nerve,"Answer: D: Hypoglossal nerve Explanation: None" "This is a real-world medical entrance exam question, please give the true answer based on the question and selection. Topic name: AIIMS 2018, Subject name: Medicine","A patient of hypeension on Metoprolol, Verapamil was given. This is will result in? Select and explain. A. Atrial fibrillation, B. Bradycardia with AV Block, C. Torsades De pointes, D. Tachycardia",B: Bradycardia with AV Block,"Answer: B: Bradycardia with AV Block Explanation: Metoprolol - |HR Verapamil - A-V nodal delay, |HR So, Bradycardia with AV Block - Torsade's De points: Class IA / IC / III: anti antiarrhythmic" "This is a real-world medical entrance exam question, please give the true answer based on the question and selection. Topic name: None, Subject name: Medicine","A patient with history of frequent abortions and pain in calf muscles from past 4 years is due to deficiency of Select and explain. A. Factory VII, B. Protein c, C. Plasmin, D. Thrombin",B: Protein c,"Answer: B: Protein c Explanation: None" "This is a real-world medical entrance exam question, please give the true answer based on the question and selection. Topic name: None, Subject name: Surgery","Which of the following is true regarding Galeazzi's fracture dislocation? Select and explain. A. Interosseous membrane tear with ulnar shaft fracture, B. Radial collateral ligament tear with interosseous membrane tear with radial shaft fracture, C. Interosseous membrane tear with triangular fibro-cailage complex (TFCC) tear and ulnar shaft fracture, D. Interosseous membrane tear with triangular fibro-cailage complex (TFCC) tear and radial shaft fracture",D: Interosseous membrane tear with triangular fibro-cailage complex (TFCC) tear and radial shaft fracture,"Answer: D: Interosseous membrane tear with triangular fibro-cailage complex (TFCC) tear and radial shaft fracture Explanation: Answer- D. Interosseous membrane tear with triangular fibro-cailage complex (TFCC) tear and radial shaft fractureGuleazzi'sfracture is afracture of lower one-third of radius wilh dislocation of distal radioulnar joint. It is associated with tearing of interosseous membrane and triangular fibrocailage complex (TFCC).""Components of Distal Radioulnar Joint (DR(LJ): The distal radius and ulna are linked to each other by the interosseus membrane, the capsule of DRUJ and the triangular fibrocailage complex (TFCC)." "This is a real-world medical entrance exam question, please give the true answer based on the question and selection. Topic name: None, Subject name: Dental","In the intercuspal position, the lingual cusp of maxillary 2nd premolar contacts the: Select and explain. A. Distal fossa of mandibular 2nd premolar, B. Mesial fossa of mandibular 1st molar, C. Distal marginal ridge of mandibular 2nd molar, D. Distal marginal ridge of mandibular 1st premolar",A: Distal fossa of mandibular 2nd premolar,"Answer: A: Distal fossa of mandibular 2nd premolar Explanation: None" "This is a real-world medical entrance exam question, please give the true answer based on the question and selection. Topic name: None, Subject name: Medicine","A Female patient was brought to the ER with altered sensorium. On examination BP was 88/67mm with a pulse of 60/min. Rectal temperature was 34 Celsius. There was associated history of constipation, dry skin and menorrhagia. What is the diagnosis? Select and explain. A. Myxedema Coma, B. Septic Shock, C. Hypothermia, D. Cardiogenic Shock",A: Myxedema Coma,"Answer: A: Myxedema Coma Explanation: Ans: A. Myxedema ComaRef: Harrison, I8't' ed., pg. 2922.Myxedema coma is a state of decompensated hypothyroidism.A person may have lab values identical to a ""normal"" hypothyroid state, but a stressful event (such as an infection, myocardial infarction or stroke) precipitates the myxedema coma state, usually in the elderly.Primary symptolns of myxedema coma are altered rnental status and low body ternperature.Low blood sugar, low blood pressure, hyponatrereria, hypercapnia, hypoxia, slowed hea rate, and hypoventilation may also occur." "This is a real-world medical entrance exam question, please give the true answer based on the question and selection. Topic name: AIIMS 2019, Subject name: Anaesthesia","Duration of action of proparacaine: Select and explain. A. 2min, B. 5min, C. 10min, D. 20min",D: 20min,"Answer: D: 20min Explanation: Proparacaine (It is an ester containing LA) has fast onset (within 30 sec) and sho duration of action (10-20 minutes) Used in ophthalmic surgeries. Metabolized by pseudocholinesterase" "This is a real-world medical entrance exam question, please give the true answer based on the question and selection. Topic name: None, Subject name: Dental","When it is made up of same steel, size 55, which will be more prone to fracture: Select and explain. A. Triangular reamer, B. Rhomboid cross section file, C. Square cross section file, D. Headstrom file",D: Headstrom file,"Answer: D: Headstrom file Explanation: None" "This is a real-world medical entrance exam question, please give the true answer based on the question and selection. Topic name: None, Subject name: Dental","Which of the following is true about master apical file? Select and explain. A. The last file which is selected for the obturation, B. The initial file which is selected for initial radiographic working length, C. The smaller file used in between the other file during biomechanical preparation, D. The last file which is used for removal of soft debris from root canal",A: The last file which is selected for the obturation,"Answer: A: The last file which is selected for the obturation Explanation: None" "This is a real-world medical entrance exam question, please give the true answer based on the question and selection. Topic name: None, Subject name: Medicine","A patient presents to the emergency with altered sensorium. All of these tests should be done except: Select and explain. A. Complete blood counts, B. Random blood sugar, C. Lumbar puncture, D. Non-contrast CT head",C: Lumbar puncture,"Answer: C: Lumbar puncture Explanation: Ans: C. Lumbar puncture(Ref Harrison 19/e p1773, 18/e p2251)Recommends imaging study prior to lumbar puncture.Excludes raised 1CP causes (large intracranial mass lesion) a Preventing herniation." "This is a real-world medical entrance exam question, please give the true answer based on the question and selection. Topic name: None, Subject name: Dental","Figure depicts: Select and explain. A. Balanced force technique, B. Anticurvature filing, C. Standarized technique, D. None",B: Anticurvature filing,"Answer: B: Anticurvature filing Explanation: Anticurvature Filing This technique is designed to keep files away from the furcational zone of curved root canals thus preventing excessive removal of dentin and strip perforations at this critical zone. This goal should be achieved by use of precurved instruments (files) at the outer side of the curvature. When the file can be easily negotiated around the ledge, anticurvature filing will enable the clinician to blend the ledge into the canal preparation." "This is a real-world medical entrance exam question, please give the true answer based on the question and selection. Topic name: None, Subject name: Pathology","Under polarised light, the congo red stained amyloid shows- Select and explain. A. Pink or red color, B. White color, C. Apple green birefringence, D. None",C: Apple green birefringence,"Answer: C: Apple green birefringence Explanation: Answer- C. Apple green birefringenceAmyloid has an affinity for Congo red stain; therefore this method is used for confirmation of amyloid of all types. If the stained section is viewed in polarised light, the amyloid characteristically shows apple-green birefringence due to cross- b-pleated sheet configuration of amyloid fibrils.The stain can also be used to distinguish between AL and AA amyloid (primary and secondary amyloid respectively)." "This is a real-world medical entrance exam question, please give the true answer based on the question and selection. Topic name: None, Subject name: Dental","In sleep apnea syndrome, AHI Index for mild sleep apnea is Select and explain. A. >5, B. >15, C. >10, D. >30",A: >5,"Answer: A: >5 Explanation: None" "This is a real-world medical entrance exam question, please give the true answer based on the question and selection. Topic name: None, Subject name: Pharmacology","All are true of midazolam over diazepam except Select and explain. A. 2 to 3 times more potent than diazepam, B. It has low water solubility and shows minimal risk for thrombophlebitis, C. Has more affinity for benzodiazepine receptors, D. Better anxiolysis and amnesia",B: It has low water solubility and shows minimal risk for thrombophlebitis,"Answer: B: It has low water solubility and shows minimal risk for thrombophlebitis Explanation: None" "This is a real-world medical entrance exam question, please give the true answer based on the question and selection. Topic name: None, Subject name: Dental","Which of the following is non parametric test Select and explain. A. Pearson correlation, B. ANOVA, C. Mann whitney test, D. Student t test",C: Mann whitney test,"Answer: C: Mann whitney test Explanation: None" "This is a real-world medical entrance exam question, please give the true answer based on the question and selection. Topic name: None, Subject name: Surgery","Haglund's deformity is seen in which joint? Select and explain. A. Elbow, B. Wrist, C. Knee, D. Ankle",D: Ankle,"Answer: D: Ankle Explanation: Answer- D. AnklePrbminence of posterosuperior poion of calcaneus leading to heel pain is called Haglund deformity.A Haglund deformity, or pump bump, is caused by chronic inflammation of the adventitious superficial pretendinousAchilles bursa that separutes the Achilles tendon from the overlying skin.""-Campbell's operative" "This is a real-world medical entrance exam question, please give the true answer based on the question and selection. Topic name: AIIMS 2018, Subject name: Pathology","Naive T cell get activated by: Select and explain. A. NK cell, B. Dendritic cell, C. Macrophage, D. B-lymphocyte",B: Dendritic cell,"Answer: B: Dendritic cell Explanation: Naive T cell get activated by antigen presenting cells such as Dendritic cell Macrophage B-lymphocyte Most impoant APC in activating naive T cell is dendritic cell Dendritic cell on skin is called as Langerhans's cell. They have granules called as Bier beck's granules These granules have rod like appearance with a dilated end hence known to have tennis racquet appearance Markers of Langerhans's cell CD 1 A S-100 HLA-DR Dendritic cell in lymph node is known as follicular dendritic cell. Follicular dendritic cell acts as a reservoir for HIV" "This is a real-world medical entrance exam question, please give the true answer based on the question and selection. Topic name: None, Subject name: Pharmacology","Lidocaine produces its antiarrhythmic effects by Select and explain. A. Increasing A-V conduction, B. Decreasing cardiac excitability, C. Increasing cardiac conduction velocity, D. Increasing spontaneous pacemaker activity",B: Decreasing cardiac excitability,"Answer: B: Decreasing cardiac excitability Explanation: None" "This is a real-world medical entrance exam question, please give the true answer based on the question and selection. Topic name: None, Subject name: Dental","Marginal bone loss around mandibular implant depends on all except Select and explain. A. Abutment, B. Boning technique, C. Connector, D. Implant fixture",B: Boning technique,"Answer: B: Boning technique Explanation: Bone loss around Implant- One of the primary complications Abnormal if more than 0.2mm/year For peri-implantitis 1 mm to 1.5 mm detectable bone loss recommended Important factors on which bone loss depends around an implant- Implant size and shape (improper) Inadequate in number compare to desired Wrong positioning Bone quality already poor or amount is inadequate Initial instability achieved Compromised healing phase Improper prosthesis fit like Implant fixture Defective design (like excessive cantilever, poor hygiene access) Non maintained or excessive occlusal forces Improper fitting of abutment components Poor oral hygiene Some systemic conditions like uncontrolled diabetes Poor oral habits like tobacco chewing, smoking" "This is a real-world medical entrance exam question, please give the true answer based on the question and selection. Topic name: None, Subject name: Dental","Size of apical foramen Select and explain. A. Remain constant, B. Increases with age, C. Decreases with age, D. May increase or decrease with age",B: Increases with age,"Answer: B: Increases with age Explanation: None" "This is a real-world medical entrance exam question, please give the true answer based on the question and selection. Topic name: None, Subject name: Dental","In order to have balance in hand instrument: Select and explain. A. Blade edge should be within 2 mm of the axis of handle, B. Blade edge should be within 2 mm of the axis of shank, C. Blade edge should be within 2.5 mm of the axis of handle, D. Blade edge should be within 2.5 mm of the axis of shank",A: Blade edge should be within 2 mm of the axis of handle,"Answer: A: Blade edge should be within 2 mm of the axis of handle Explanation: Balance is accomplished by designing the angles of the shank so that the cutting edge of the blade lies within the projected diameter of the handle and nearly coincides with the projected long axis of the handle. For optimal antirotational design, the blade edge must not be positioned away from the axis by more than 1 to 2 mm. Sturdevants operative dentistry 7th edition page e1" "This is a real-world medical entrance exam question, please give the true answer based on the question and selection. Topic name: None, Subject name: Pharmacology","A patient of septic shock was given intravenous norepinephrine. The response to this drug is best checked by: Select and explain. A. Increase in hea rate, B. Decrease in hea rate, C. Increase in mean aerial pressure, D. Decreased renal perfusion and reduced urine output",C: Increase in mean aerial pressure,"Answer: C: Increase in mean aerial pressure Explanation: Ans. c. Increase in mean aerial pressureNoradrenaline is a powerful peripheral vasoconstrictor and inotrope and used in patients of septic shock and cardiogenic shock. Noradrenaline causes peripheral vasoconstriction thereby increasing diastolic blood pressure as well as venous return. The increase in diastolic blood pressure and systolic blood pressure can increase mean aerial pressure (MAP) which is the therapeutic outcome expected in any septic shock patient as a response to vasopressor." "This is a real-world medical entrance exam question, please give the true answer based on the question and selection. Topic name: None, Subject name: Dental","In facial porcelain inlay what should be kept in mind? Select and explain. A. Facial surface should be perpendicular to cavity margins, B. Facial surface should converge to cavity margins, C. Facial surface should be parallel to cavity margins, D. Axial wall is 2.5 mm deeper than normal",A: Facial surface should be perpendicular to cavity margins,"Answer: A: Facial surface should be perpendicular to cavity margins Explanation: None" "This is a real-world medical entrance exam question, please give the true answer based on the question and selection. Topic name: None, Subject name: Dental","Rapid tooth separation works on following principle: Select and explain. A. Wedge principle., B. Traction principle., C. Both of the above., D. None.",C: Both of the above.,"Answer: C: Both of the above. Explanation: None" "This is a real-world medical entrance exam question, please give the true answer based on the question and selection. Topic name: None, Subject name: Pediatrics","All are true regarding administration of betamethasone to a mother with premature delivery except – Select and explain. A. Neonatal morbidity better, B. Helps lung maturity, C. Prevents hyperbilirubinemia, D. Decreases intraventricular hemorrhage",C: Prevents hyperbilirubinemia,"Answer: C: Prevents hyperbilirubinemia Explanation: Prevention of Prenatal steroids are effective in preventing HMD. Steroids acts by enhancing lung maturity." "This is a real-world medical entrance exam question, please give the true answer based on the question and selection. Topic name: None, Subject name: Dental","Method of increasing attached gingiva Select and explain. A. Apical flap, B. Widman flap, C. Undisplaced flap, D. Modified widman flap",A: Apical flap,"Answer: A: Apical flap Explanation: None" "This is a real-world medical entrance exam question, please give the true answer based on the question and selection. Topic name: AIIMS 2017, Subject name: Pathology","A patient presented with headache and fever. His investigations revealed hemoglobin of 16g/dl, TLC of 21,000/ uL, platelet count of 3,75,000. His DLC showed Neutrophils (25%), Lymphocytes (20%), Metamyelocytes and myelocytes 40%and eosinophils5%. Which of the following is the next best investigation in this patient? Select and explain. A. JAK 2 mutation, B. EPO level, C. Philadelphia chromosome, D. Bone marrow biopsy",C: Philadelphia chromosome,"Answer: C: Philadelphia chromosome Explanation: In the above question, lab investigation of the patient reveals normal hemoglobin, Elevated TLC, normal platelet count, decreased Neutrophils, Elevated metamyelocytes and myelocytes. This lab picture clearly indicates that immature WBC's are much more in number than the mature WBC'S. Such a condition is given the name ""shift to the left"". This is seen in association with two types of conditions: 1.) Leukemoid reaction 2.) CML To distinguish between the two condition, we can do: LAP score (not commonly performed now). - CML- decreased, Leukemoid reaction- increased. Philadelphia chromosome presence (confirmatory test)- CML- Philadelphia chromosome (+ve), Leukemoid reaction - Philadelphia chromosome(-ve) ." "This is a real-world medical entrance exam question, please give the true answer based on the question and selection. Topic name: None, Subject name: Gynaecology & Obstetrics","A pregnant female, 38 years old, had a child with Down's syndrome. How do you assess the risk of Down's syndrome in the present pregnancy: Select and explain. A. Material alpha-feto protein, B. Material HCG, C. USG, D. Chorionic villous biopsy",D: Chorionic villous biopsy,"Answer: D: Chorionic villous biopsy Explanation: In patients with previous history of Down syndrome “The risk of recurrence is greater than the risk of genetic diagnosis and these patients should be advised to seek genetic counselling and to have a genetic diagnosis.” Fernando Arias 3/e, p 27 Therefore amniocentesis /Chorionic villous biopsy should be done." "This is a real-world medical entrance exam question, please give the true answer based on the question and selection. Topic name: None, Subject name: Medicine","With reference to Bacteroides fragilis the following statements are true except: Select and explain. A. B. fragilis is the most frequent anaerobe isolated from clinical samples, B. B. fragilis is sensitive to metronidazole, C. The lipopolysaccharide formed by B. fragilis is structurally & functionally different from conventional endotoxin., D. Shock & DIC are common in Bacteroides bacteremia",D: Shock & DIC are common in Bacteroides bacteremia,"Answer: D: Shock & DIC are common in Bacteroides bacteremia Explanation: None" "This is a real-world medical entrance exam question, please give the true answer based on the question and selection. Topic name: None, Subject name: Forensic Medicine","A lady was brought from village, unconscious, about 12 hours after ingesting some kind of unknown poison. Her hea rate was 103/min, blood pressure in 90/50 mm Hg and respiratory rate is 19/min. Her breath smelled like kerosene. All of the following should be done in her management, except: Select and explain. A. Gastric lavage should be done, B. Atropine should be administered till signs of recovery, C. Vasopressors should be administered intravenously, D. Immediate airway management",C: Vasopressors should be administered intravenously,"Answer: C: Vasopressors should be administered intravenously Explanation: Ans: C. Vasopressors should be administered intravenously(Ref Reddy 34/e p487, 33/e p; Parikh 6/e p10.41-10.45).Signs & symptoms suggestive of organophosphorus poisoning.Antidote of choice - Atropine.Reves bradycardia & hypotension.Vasopressors contraindicated - Due to cholinergic effect combated by atropine." "This is a real-world medical entrance exam question, please give the true answer based on the question and selection. Topic name: None, Subject name: Anatomy","Located medial to the third molar at the junction of the maxilla and the horizontal plate of the palatine bone: Select and explain. A. Posterior nasal spine, B. Mylohyoid line, C. Pterygoid hamulus, D. Greater palatine foramen",D: Greater palatine foramen,"Answer: D: Greater palatine foramen Explanation: Greater palatine foramen located medial to the third molar at the junction of the maxilla and the horizontal palate of the palatine bone. Greater palatine foramen transmits descending palatine vessels and palatine nerve." "This is a real-world medical entrance exam question, please give the true answer based on the question and selection. Topic name: None, Subject name: Surgery","After Le-Fort 1 osteotomy the blood supply of maxilla is maintained by Select and explain. A. Posterior superior alveolar artery, B. Anterior superior alveolar artery, C. Greater palatine artery, D. Ascending palatine branch of facial artery",D: Ascending palatine branch of facial artery,"Answer: D: Ascending palatine branch of facial artery Explanation: None" "This is a real-world medical entrance exam question, please give the true answer based on the question and selection. Topic name: AIIMS 2019, Subject name: Social & Preventive Medicine","Which of the following does not include Specific protection under primary prevention? Select and explain. A. Tab Rifampicin to those in contact with meningitis, B. Health education, C. Pentavalent vaccination, D. Wheat flour foified with added iron",B: Health education,"Answer: B: Health education Explanation: Tab. Rifampicin to the meningitis patients is chemoprophylaxis which is given when a risk factor is present. Hence this is primary level prevention specific protection. Pentavalent vaccines; vaccines are always given when risk factors are present. Primary level prevention with Specific protection. Foification done when there is need or deficiency in a population (risk factors): primary level prevention. Protection from occupational hazards, Road traffic accidents, carcinogens will come under primary level Specific protection. Health education is primordial prevention." "This is a real-world medical entrance exam question, please give the true answer based on the question and selection. Topic name: None, Subject name: Dental","Which of the following Universal pro-taper re-treatment file has end cutting tip? Select and explain. A. D1, B. D2, C. D3, D. All of the above",A: D1,"Answer: A: D1 Explanation: None" "This is a real-world medical entrance exam question, please give the true answer based on the question and selection. Topic name: None, Subject name: Surgery","The proximal segment of mandibular angle fracture usually displaced in which direction Select and explain. A. Anterior and Superior, B. Posterior and interior, C. Interior only, D. Posterior and superior",A: Anterior and Superior,"Answer: A: Anterior and Superior Explanation: None" "This is a real-world medical entrance exam question, please give the true answer based on the question and selection. Topic name: None, Subject name: Forensic Medicine","Under section 304A of IPC, for a proved case of medical negligence, The maximum punishment of imprisonment is up to: Select and explain. A. 1 year, B. 2 years, C. 3 years, D. 5 years",B: 2 years,"Answer: B: 2 years Explanation: Answer: b. 2 years (Ref Reddy 33/e p35, 29/e p260,Under section 304A of IPC, for a proved case of medical negligence, the maximum punishment of imprisonment is up to 2 years." "This is a real-world medical entrance exam question, please give the true answer based on the question and selection. Topic name: None, Subject name: Anaesthesia","The action of proparacaine stas within? Select and explain. A. 2 minutes, B. 5 minutes, C. 15 minutes, D. 20 minutes",A: 2 minutes,"Answer: A: 2 minutes Explanation: Ans. A. 2 minutesProparacaine and tetracaine are indicated to produce local anesthesia of sho duration for ophthalmic procedures including measurement of intraocular pressure (tonometry), removal of foreign bodies and sutures, and conjunctival and corneal scraping in diagnosis and gonioscopy.Onset of action:* Proparacaine--Within 20 seconds.* Tetracaine--Approximately 15 seconds.Duration of action:* Proparacaine--15 minutes or longer.* Tetracaine--10 to 20 minutes; average 15 minutes." "This is a real-world medical entrance exam question, please give the true answer based on the question and selection. Topic name: None, Subject name: Dental","Type of cry, which is not a cry at all, there are no tears, no sobs, just a constant whining noise: Select and explain. A. Obstinate cry., B. Frighten cry., C. Hurt cry., D. Compensatory cry.",D: Compensatory cry.,"Answer: D: Compensatory cry. Explanation: The four types of crying classified by Elsbach are:" "This is a real-world medical entrance exam question, please give the true answer based on the question and selection. Topic name: None, Subject name: Pathology","The cyst which is found within the bone at the junction of the globular process, the Lateral nasal process & maxillary process is: Select and explain. A. Naso-alveolar cyst, B. Globulomaxillary cyst, C. Naso palatine cyst, D. Mid palatine cyst",B: Globulomaxillary cyst,"Answer: B: Globulomaxillary cyst Explanation: None" "This is a real-world medical entrance exam question, please give the true answer based on the question and selection. Topic name: AIIMS 2019, Subject name: Gynaecology & Obstetrics","In which of surgeries this retractor is commonly used. Select and explain. A. LSCS, B. Vaginal hysterectomy, C. Fothergill repair, D. Manchester operation",A: LSCS,"Answer: A: LSCS Explanation: Helps to retract the bladder so, you can enter in uterus Used in LSCS" "This is a real-world medical entrance exam question, please give the true answer based on the question and selection. Topic name: None, Subject name: Pathology","Not seen in classical triad of megaloblastic anemia Select and explain. A. Howell-Jolly bodies, B. Reticulocytosis, C. Hypersegmented neutrophil, D. Macro-ovalocytes",B: Reticulocytosis,"Answer: B: Reticulocytosis Explanation: None" "This is a real-world medical entrance exam question, please give the true answer based on the question and selection. Topic name: None, Subject name: Anaesthesia","According to `AHA 2010 Guidelines' which of the following drug is not used in CPCR? Select and explain. A. Adrenalin, B. Vasopressin, C. Atropine, D. Amiodarone",C: Atropine,"Answer: C: Atropine Explanation: Answer- C. Atropine'According to AHA 2020 Guidelines, Atropine is no longer recommended for routine use in the management of pulseless electrical activity (PEA)asystole.'ACLS- medications for pulses arrestAtropine: deleted from pulseless arrest algorithmEpinephrine: dose, interval unchangedVasopressin: dose, use unchangedAmiodarone: dose, indications unchangedLidocaine: dose, indications unchangedSodium Bicarbonate: routine use not recommendedCalcium: for treatment of cardiac arrest not recommended" "This is a real-world medical entrance exam question, please give the true answer based on the question and selection. Topic name: None, Subject name: Surgery","Cleft lip is treated by all of the following methods except: Select and explain. A. Von-Langenback, B. Millard's method, C. Tannison rindal method, D. Le-mesurier",A: Von-Langenback,"Answer: A: Von-Langenback Explanation: None" "This is a real-world medical entrance exam question, please give the true answer based on the question and selection. Topic name: None, Subject name: Surgery","Which of the following does not cause head and neck squamous cell carcinoma Select and explain. A. Alcohol, B. Betel nut, C. HPV, D. EBV",D: EBV,"Answer: D: EBV Explanation: Answer- D .EBVThe most impoant risk factors for developing HNSCC are tobacco smoking and alcohol consumption.Consumed in betel quids containing areca nut increases the risk of developing HNSCC.HNSCCs of the oral cavity and oropharynx, are becoming more prevalent, which may be related to an increase in oral and oropharyngeal HPV infection." "This is a real-world medical entrance exam question, please give the true answer based on the question and selection. Topic name: None, Subject name: Biochemistry","In lead poisoning, there is an inhibition of some of the enzymes of the heme biosynthetic pathway. This is reflected by the accumulation of what substance in blood? Select and explain. A. Uroporphyrinogen III, B. Ferrochelatase, C. Porphob I inogen, D. Delta amino levulinic acid",D: Delta amino levulinic acid,"Answer: D: Delta amino levulinic acid Explanation: Ans: D. Delta amino levulinic acid(Ref: Harper 30/e n329)Lead poisoning:Affect heme metabolism by combining with SH groups in ferrochelatase & ALA (delta-amino levulinic acid) dehydratase enzymes.Elevated protoporphyrin levels - Found in RBC's.Elevated ALA & coproporphyrin levels - Found in urine." "This is a real-world medical entrance exam question, please give the true answer based on the question and selection. Topic name: None, Subject name: Pediatrics","A 30–year old lady delivered a healthy baby at 37 week of gestation. She was a known case of chronic hepatitis B infection She was positive for HBsAG o but negative for HBeAG. Which of the following is the most appropriate treatment for the baby – Select and explain. A. Both active and passive immunization soon after birth, B. Passive immunization soon afte birth and active immunization at 1 year of age, C. Only passive immunization soon after birth, D. Only active immunization soon after birth",A: Both active and passive immunization soon after birth,"Answer: A: Both active and passive immunization soon after birth Explanation: Immunization of infants born to i sAg positive women. These infants should receive both active and passive immunization soon after birth. The first dose of active immunization should be accompanied by administration of .5 ml of HBIG as soon after delivery is possible, because the effectiveness decreases rapidly with increased time after birth. Subsequent doses of active immunization should be given at 1-2mo, and 6 mo of age." "This is a real-world medical entrance exam question, please give the true answer based on the question and selection. Topic name: None, Subject name: Dental","CHX mouthwash daily use till 2 week for streptococcus mutanus is effective for: Select and explain. A. 14-18 weeks, B. 12-14 weeks, C. 12-16 weeks, D. 12-26 weeks",D: 12-26 weeks,"Answer: D: 12-26 weeks Explanation: None" "This is a real-world medical entrance exam question, please give the true answer based on the question and selection. Topic name: None, Subject name: Dental","Which of the following can lead to uncontrolled bleeding during oral prophylaxis Select and explain. A. Patient on Heparin, B. Patient on Nifedipine, C. Patient on phenytoin, D. None of the above",A: Patient on Heparin,"Answer: A: Patient on Heparin Explanation: None" "This is a real-world medical entrance exam question, please give the true answer based on the question and selection. Topic name: None, Subject name: Dental","Obturation with gutta-percha cones is indicated in primary teeth only when: Select and explain. A. Canals are too wide, B. Successor is missing, C. Canals are very long but straight, D. When stainless steel crown recommended",A: Canals are too wide,"Answer: A: Canals are too wide Explanation: None" "This is a real-world medical entrance exam question, please give the true answer based on the question and selection. Topic name: None, Subject name: Pediatrics","Short child with low T4 and raised TSH and swelling of pituitary, what is the diangosis ? Select and explain. A. Primary hypothyrodism, B. Pituitary tumor, C. TSH Secreting pituitary adenoma, D. TSH resistance",A: Primary hypothyrodism,"Answer: A: Primary hypothyrodism Explanation: Normally, T4 and T3 exert negative feedback on TSH secretion by pituitary in two ways: Block the secretion of TSH by pituitary directly --> major action Block the secretion of TRH by hypothalamus (TRH stimulates the secretion of TSH by pituitary) In primary hypothyrodism, T4 and T3 are not produced or produced in low concentration by thyroid. This results in abolition of negative feedback on TSH secretion. Thus, there is elevated TSH and pituitary can become hyperplastic to produce more TSH. In pituitary cause of hypothyroidism (secondary hypothyroidism) both TSH and Thyroid hormone (T4, T3) are low as pituitary does not secrete TSH and TSH is the major stimulation for production of T4 and T3. About option d TSH resistance can also produce same picture i.e. raised TSH and low T4 with pituitary swelling. There is thyroid insensitivity to TSH which results in hypothyroidism (1T4 and T3). Because of reduced thyroid hormone feedback, TSH is markedly elevated. However, Amongst the given options best option is primary hypothyrodism because TSH resistance is a very rare condition and further T4 levels are normal in Mild and moderate (Partial) TSH resistance." "This is a real-world medical entrance exam question, please give the true answer based on the question and selection. Topic name: None, Subject name: Dental","D1 bone found mainly in which site? Select and explain. A. Anterior maxilla, B. Anterior mandible, C. Posterior mandible, D. Posterior maxilla",B: Anterior mandible,"Answer: B: Anterior mandible Explanation: None" "This is a real-world medical entrance exam question, please give the true answer based on the question and selection. Topic name: None, Subject name: Social & Preventive Medicine","A researcher said he has discovers a new drug which is effective in chronic hypeensives with a p value of < 0.10. Which of the following is true regarding the same? Select and explain. A. The test is 90% reproducible, B. 90% of test results could have occurred by chance, C. Not more than 10% of the people benefitted by the drug could he due to chance, D. 90% of patients will be benefitted by giving the drug",C: Not more than 10% of the people benefitted by the drug could he due to chance,"Answer: C: Not more than 10% of the people benefitted by the drug could he due to chance Explanation: Answer- C. Not more than 10% of the people benefitted by the drug could he due to chanceThe p (probability) value is used when we wish to see how likely is that a hypothesis is true, The hypothesis is usually that there is no difference between two treatments, known as ""null hypothesis'.The p value gives the probability of any observed difference having happened by chance.p:0.5 means that the probability of any observed difference having happened by chance is 0.5 in I or 50:50.Similarly, a p valae of <0.10 means that the probubility of it happening by chance is 10%.In the question, the probability that the drug affected the chronic hypeensives by chance is less than 1096. Hence, 90% ofthe subjects were benefitted by the drug." "This is a real-world medical entrance exam question, please give the true answer based on the question and selection. Topic name: None, Subject name: Surgery","Which metabolic condition has phosphaturia and osteomalacia? Select and explain. A. Fibrosarcoma, B. Osteosarcoma, C. Undifferentiated sarcoma, D. Malignant peripheral nerve sheath tumor",A: Fibrosarcoma,"Answer: A: Fibrosarcoma Explanation: Ans: A. FibrosarcomaPhosphatonin (FGF-23) oncogenic osteomalacia (paraneoplastic syndrome) seen in fibrosarcoma." "This is a real-world medical entrance exam question, please give the true answer based on the question and selection. Topic name: AIIMS 2017, Subject name: Surgery","How will you check the functioning of an ICD tube? Select and explain. A. By observing for continuous air bubbles coming out of the underwater drain, B. By observing the movement of air water column in the tube during respiration, C. By taking X ray chest repeatedly, D. By auscultation",B: By observing the movement of air water column in the tube during respiration,"Answer: B: By observing the movement of air water column in the tube during respiration Explanation: Chest drainage system: Follow-up: Patency of chest tube is assessed by observing the oscillations in water seal chamber with respiratory movements. Position of chest tube & resolution of intrapleural air or liquid is checked by x-ray (AP & cross table lateral). Tube should be pulled back if it crosses the mediastinum Removal: Chest tubes are generally removed when there has been air or fluid drainage of <200 ml in 24 hours for >24 hours ." "This is a real-world medical entrance exam question, please give the true answer based on the question and selection. Topic name: AIIMS 2018, Subject name: Medicine","A smoker presents with recent onset breathlessness and ECG was done. The diagnosis is? Select and explain. A. Atrial fibrillation, B. Paroxysmal supraventricular tachycardia, C. Acute myocardial infarction, D. Multi-focal atrial tachycardia",D: Multi-focal atrial tachycardia,"Answer: D: Multi-focal atrial tachycardia Explanation: Multi - focal Atrial tachycardia: - Irregular R-R internal - Right axis detion - Variable amplitude P-wave - Normal PR interval - Non progression of R-wave (suggestive of Right ventricular etiology Atrial fibrillation Irregular R-R intervals and absence of P waves. AF Ruled out because these is P wave present in ECG. Acute myocardial infarction No STEMI No T wave inversion PSVT ruled out - Rhythm normal" "This is a real-world medical entrance exam question, please give the true answer based on the question and selection. Topic name: None, Subject name: Surgery","During mandibular nerve block which nerve also gets damaged? Select and explain. A. Abducence, B. Facial, C. Vagus, D. Trochlear",B: Facial,"Answer: B: Facial Explanation: None" "This is a real-world medical entrance exam question, please give the true answer based on the question and selection. Topic name: AIIMS 2019, Subject name: Medicine","Which of the following is most associated with respiratory alkalosis:- Select and explain. A. Assisted control mode ventilation, B. Non invasive ventilation, C. Pressure controlled, D. SIMV",A: Assisted control mode ventilation,"Answer: A: Assisted control mode ventilation Explanation: Option A- means that every few second breath will be delivered to the patient by the computer so this will be computer aided breathing by patient. Also, if patient is breathing himself computer will help the patient. Option B- it is an uncomfoable procedure for patient as its a tight-fitting mask so d/t discomfo patient tries to remove it. | Causing compromised ventilation | CO2 || in body | Respiration acidosis Option C-here number of breath/min remain same but the pressure exeed by the ventilation is extra.which might cause barotrauma But will not cause any derangement so option c is ruled out. It may contribute hypoventilation but never Hyperventilation. Hypoventilation if pressure Delivered is not proper then expansion of chest will not occur contributing to accum. of CO2 in body. Option D- Synchronized intermittent mandatory ventilation this ventilator modality mode is used when we try to vain of the patient on ventilator. Ventilatory Mode Variables set by user (independent) Variable Monitored by User (Dependent) Trigger cycle Limit Advantages Disadvantages ACMV (assist control veilation) Tidal volume ventilator rate FO2 PEEP level pressure limit Peak, mean, and plateau airway pressure VE ABG I/E ratio Patient effo Timer Pressure limit Patient control Guaranteed ventilation Potential hyperventilation Barotraumia and volume trauma Every effective breath generates & ventilator volume IMV (intermittent mandatory ventilation) Tidal volume Mandatory ventilator rate Fio2 PEEP Level Pressure limit Spontaneous breaths between assisted breaths Peak, mean, and plateau airway pressure VE ABG I/E ratio Patient effo Timer Pressure limit Patient control Comfo from spontaneous breaths Guaranteed ventilation Potential dysynchrony Potential hypo ventilation PSV (presssure-suppo ventilaton) Inspiratory pressure level Fio2 PEEP Pressure limit Tidal volume Respiratory rate VE ABG Pressure limit Inspiratory flow Patient control Comfo Assures synchromy No timer backup Potential hypo-ventilation NV (non invasive ventilation) Inspiratory and expiratory level Tidal volume Respiratory rate Pressure limit Inspiratory flow Patient control Mask interface may cause discomfo and facial bruising" "This is a real-world medical entrance exam question, please give the true answer based on the question and selection. Topic name: AIIMS 2019, Subject name: Physiology","Which of the following hormone crosses the plasma membrane for its action? Select and explain. A. Epinephrine, B. Glucagon, C. Insulin, D. Thyroxine",D: Thyroxine,"Answer: D: Thyroxine Explanation: Intracellular hormone receptors: - - Several hormones, including adrenal & gonadal steroid hormones, thyroid hormones, retinoid hormones and vitamin D, bind with protein receptors inside the cell. - Because these hormones are lipid soluble. Cell surface receptors: - - All the peptide hormones uses it - They are water soluble. - They act secondary messengers (CGMP, CAMP etc.) Examples: Catecholamines (epinephrine, dopamine) Insulin Glucagon Prolactin Growth hormone etc." "This is a real-world medical entrance exam question, please give the true answer based on the question and selection. Topic name: None, Subject name: Dental","Cross-sectional studies are carried on Select and explain. A. Same individuals but of same age, B. Different individuals but of same age groups, C. Same individuals at different ages, D. Different individuals of different age groups",D: Different individuals of different age groups,"Answer: D: Different individuals of different age groups Explanation: None" "This is a real-world medical entrance exam question, please give the true answer based on the question and selection. Topic name: None, Subject name: Anatomy","In gene studies, the specific site to which the enzyme CRE recombinase binds is: Select and explain. A. RE site, B. LoxP site, C. NT site, D. F site",B: LoxP site,"Answer: B: LoxP site Explanation: Ans: B. LoxP siteCre-Lox recombination - Known as ""Site-specific recombinase technology"".* Widely used to carry out deletions, inseions, translocations & inversions at specific sites in cellular DNA.* Consists of Cre-recombinase enzyme. - Recombines a pair of sho target sequences called ""Lox sequences"".* Cre enzyme & original Lox site - Referred as ""LoxP sequence"" - Derived from bacteriophage P1.* Lox P (locus of X-over P1) - Site on bacteriophage P1 consisting of 34 bp.Ref https://en.wikipedia.oig/wiki/Cre-Lox_recombination)" "This is a real-world medical entrance exam question, please give the true answer based on the question and selection. Topic name: None, Subject name: Gynaecology & Obstetrics","Which of the following is not used in DIC? Select and explain. A. Heparin, B. Epsilon amino caproic acid, C. Blood transfusion, D. Intravenous fluids.",B: Epsilon amino caproic acid,"Answer: B: Epsilon amino caproic acid Explanation: Well friends, we have discussed the causes and investigations of DIC. Now here let’s take a look at its management.\ Management of DIC The most important step is to terminate the pregnancy- vaginal delivery without episiotomy is preferred to cesarean section Volume replacement by crystalloids or colloids will reduce the amount of whole blood needed to restore the blood volume. 500 ml of fresh blood raises the fibrinogen level approximately by 12.5 mg/100 ml and platelets by 10,000–15,000 cu mm. Fresh blood- helps in flushing out fibrin degradation product and improving the micro circulation. To replace fibrinogen- Fresh frozen plasma should be given: Fresh frozen plasma (FFP) is extracted from whole blood. It contains fibrinogen, anti-thrombin III, clotting factors V, XI, XII. FFP transfusion provides both volume replacement and coagulation factors. One unit of FFP (250 mL) raises the fibrinogen by 5-10 mg/dL. FFP does not need to be ABO or Rh compatible. Cryoprecipitate is obtained from thawed FFP. It is rich in fibrinogen, factor VIII, Von Willebrand’s factor, and XIII. Cryoprecipitate provides less volume (40 mL) compared to FFP (250 mL). So it should not be used for volume replacement. One unit of cryoprecipitate increases the fibrinogen level by 5-10 mg/dL. In case of active bleeding with platelet counts < 50,000/ml or prophylactically with platelet count 20–30,000/ml – platelet replacement should be done. Platelet should ABO and Rh specific. 1 units (50 ml) raises the platelet count by 7500/ ml Recombinant activated factors VIIA: (60-100 μg/kg IV) can reverse DIC within 10 minute as it is a precusor for extrinsic clotting cascade which is replaced. Role of Heparin According to Williams Obs. “Heparin is not used in DIC.” According to COGDT 10/e, p 999 “Heparin acts as an anticoagulant by activating antithrombin III but has little effect on activated coagulation factors. Anticoagulation is contraindicated in patients with fulminant DIC and central nervous system insults, fulminant liver failure, or obstetric accidents. The one instance, however, in which heparin has been demonstrated to benefit pregnancy-related DIC is in the case of the retained dead fetus with intact vascular system, where heparin may be administered to interrupt the coagulation process and thrombocytopenia for several days until delivery may be implemented.” As far as EACA is concerned- Williams Obs. 22/e, p 844 says – “EACA is not recommended in case of DIC.” According to Williams Obs 23/e, p 787 “It use in most types of obstetric coagulopathy has not been efficacious & not recommended”" "This is a real-world medical entrance exam question, please give the true answer based on the question and selection. Topic name: None, Subject name: Gynaecology & Obstetrics","Kalindi 25 years female admitted as a case of septic abortion with tricuspid valve endocarditis. Vegetation from the valve likely to affect is: Select and explain. A. Liver, B. Spleen, C. Brain, D. Lung",D: Lung,"Answer: D: Lung Explanation: “Right sided endocarditis which usually involves the tricuspid valve causes septic pulmonary emboli occasionally with infarction and lung abscesses.”" "This is a real-world medical entrance exam question, please give the true answer based on the question and selection. Topic name: None, Subject name: Surgery","High velocity gunshot injury with periosteal denudation & comminuted fracture is best treated as Select and explain. A. 'Bag of bones' & IMF, B. Reconstruction plates & closure of fracture, C. Immediate reconstruction & grafting, D. Initially debridement is done, than load bearing reconstruction plates, and grafting in secondary procedure","D: Initially debridement is done, than load bearing reconstruction plates, and grafting in secondary procedure","Answer: D: Initially debridement is done, than load bearing reconstruction plates, and grafting in secondary procedure Explanation: None" "This is a real-world medical entrance exam question, please give the true answer based on the question and selection. Topic name: None, Subject name: Dental","Palatal secretions affect the setting of all the impression materials except: Select and explain. A. Silicone, B. Agar-agar, C. Impression paste, D. Impression plaster",B: Agar-agar,"Answer: B: Agar-agar Explanation: None" "This is a real-world medical entrance exam question, please give the true answer based on the question and selection. Topic name: None, Subject name: Skin","A 28-year-old patient of neurocysticercosis develops generalized peeling of skin all over except palms and soles staing one month after taking anti-epileptics. What is the most probable diagnosis? Select and explain. A. Fixed drug eruption, B. Pemphigus, C. Steven Johnson syndrome, D. TEN",D: TEN,"Answer: D: TEN Explanation: Ans: D. TEN (Ref Neena Khanna 4/e p83, 174)Probable diagnosis is toxic epidermal necrolysis.Toxic epidermal necrolysis (TEN/ Lyell's syndrome:Rare, life-threatening skin condition that is usually caused by a reaction to drugs.Etiology of Epidermal Necrolysis Drugs MiscellaneousIdiopathic Anticonvulsants: Carbamazepine, phenytoin barbiturates, lamotrigineChemotherapeutic agents: sulfonamides, penicillinNSAIDs: Butazones, oxicams Others: Allopurinol, nevirapine SSE, GVHD, Lymphoreticular malignanciesInfections (Mycoplasma pneumoniae, herpes virus infection) 5% of patients" "This is a real-world medical entrance exam question, please give the true answer based on the question and selection. Topic name: AIIMS 2019, Subject name: Pathology","Which of the following finding are there in iron deficiency anemia? Select and explain. A. | TIBC, | Ferritin, | Transferrin saturation, B. | TIBC, | Ferritin, | Transferrin saturation, C. | TIBC, | Ferritin, | Transferrin saturation, D. | TIBC, | Ferritin, | Transferrin saturation","A: | TIBC, | Ferritin, | Transferrin saturation","Answer: A: | TIBC, | Ferritin, | Transferrin saturation Explanation: The diagnosis of iron deficiency anemia ultimately rests on laboratory studies. The serum iron and ferritin are low, and the total plasma iron-binding capacity (reflecting elevated transferrin levels) is high. Low serum iron with increased ironbinding capacity results in a reduction of transferrin saturation to below 15%. Reduced iron stores inhibit hepcidin synthesis, and its serum levels fall." "This is a real-world medical entrance exam question, please give the true answer based on the question and selection. Topic name: None, Subject name: Pediatrics","Characteristic features of growth hormone deficiency include all of the following except – Select and explain. A. Short stature since birth, B. Symptomatic hypoglycemia, C. Delayed tooth eruption, D. Sexual infantilism",A: Short stature since birth,"Answer: A: Short stature since birth Explanation: The child with hypopituitarism is usually of normal size and weight at birth. Here is a differential diagnosis of Short stature See the following table:" "This is a real-world medical entrance exam question, please give the true answer based on the question and selection. Topic name: None, Subject name: Pathology","The host tissue response in acute inflammation is all except: Select and explain. A. Exudative, B. Necrotizing, C. Granulomatous, D. Cytopathic",C: Granulomatous,"Answer: C: Granulomatous Explanation: None" "This is a real-world medical entrance exam question, please give the true answer based on the question and selection. Topic name: None, Subject name: Surgery","An empty cavity in the mandible with no lining is mostly likely to be: Select and explain. A. Aneurysmal bone cyst, B. Idiopathic bone cavity, C. Dentigerous cyst, D. Keratocyst",B: Idiopathic bone cavity,"Answer: B: Idiopathic bone cavity Explanation: None" "This is a real-world medical entrance exam question, please give the true answer based on the question and selection. Topic name: None, Subject name: Surgery","Which of the following is true about supracondvlar fracture of humerus? Select and explain. A. Distal segment is dislocated anteriorly more than posterior, B. Cubitus valgus more common than cubitus varus during malunion, C. Nerve injury related manifestations are transitory, D. Injury causes weakness of elbow flexion",C: Nerve injury related manifestations are transitory,"Answer: C: Nerve injury related manifestations are transitory Explanation: Answer- C. Nerve injury related manifestations are transitorySupracondylar Fracture Humerus:Distal segment is dislocated posteriorly more than anteriorcubitus varus more common than cubitus varus during malunioncubitus valgus more common than cubitus varus during nonunionNerve injury related manifestations are transitory due to neuropraxiaWeakness of elbow flexion is not seen in supracondylar fracture" "This is a real-world medical entrance exam question, please give the true answer based on the question and selection. Topic name: None, Subject name: Dental","Advantage of the submarginal falp for periradicular surgery is? Select and explain. A. Crestal bone less, B. Bleeding lees, C. Gingival shrinking less, D. Flap closure easy",A: Crestal bone less,"Answer: A: Crestal bone less Explanation: None" "This is a real-world medical entrance exam question, please give the true answer based on the question and selection. Topic name: AIIMS 2018, Subject name: Pharmacology","A patient is of warfarin therapy. All of the following drugs increase the risk of bleeding with warfarin except? Select and explain. A. Isoniazid, B. Amiodarone, C. Carbamazepine, D. Cimetidine",C: Carbamazepine,"Answer: C: Carbamazepine Explanation: Warfarin (anti-coagulant) is metabolized by microsomal enzymes. So, enzyme inducer decreases its effects whereas enzyme inhibitors lead to its toxicity. Enzyme inhibitors include valproate, ketoconazole, cimetidine, ciprofloxacin, erythromycin, isoniazid, amiodarone etc. These can increase the risk of bleeding with warfarin. Carbamazepine is an enzyme inducer, it decrease the effect of warfarin." "This is a real-world medical entrance exam question, please give the true answer based on the question and selection. Topic name: AIIMS 2019, Subject name: Biochemistry","Which of the following binds mRNA with ribosome in prokaryotes? Select and explain. A. tRNA, B. Shine Dalgarno sequence, C. 7 methyl guanosine capping, D. Poly A tail",B: Shine Dalgarno sequence,"Answer: B: Shine Dalgarno sequence Explanation: Shine Dalgarno sequence Only in prokaryotes present at -10 position on mRNA, Rich in purines. SD sequence is complementary to 16 s rRNA, Their binding helps in binding of mRNA with ribosome during initiation of translation * In eukaryotic cells, ribosomes recognize most eukaryotic mRNAs by binding to the 7-methylguanosine cap at their 5' terminus." "This is a real-world medical entrance exam question, please give the true answer based on the question and selection. Topic name: None, Subject name: Medicine","Which heart sound indicates stiffness of ventricular wall Select and explain. A. S1, B. S2, C. S3, D. S4",D: S4,"Answer: D: S4 Explanation: If S4 is heard as a result of stiffness of ventricular walls, as relaxation is impaired due to decreased ATP (as blood flow to heart is decreased)." "This is a real-world medical entrance exam question, please give the true answer based on the question and selection. Topic name: None, Subject name: Medicine","If the cycle stas with ' a ' wave of jvp . Arrange the following in order- Select and explain. A. R wave, B. T wave, C. 1st hea sound and Rapid ejection phase, D. All",D: All,"Answer: D: All Explanation: Answer- D. AllThe ""a"" wave is approximately synchronous with the first hea sound (S1) and just precedes the carotid upstroke.Sholy after the a-wave there is a second peak , the c-wave. The c-wave immediately follows the r wave of the ECG waveform.The v-wave corresponds to the end of the t wave in the ECG waveform." "This is a real-world medical entrance exam question, please give the true answer based on the question and selection. Topic name: None, Subject name: Medicine","Window period for thrombolysis in a stroke patient is: Select and explain. A. 1.5 hours, B. 2.5 hours, C. 3.5 hours, D. 4.5 hours",D: 4.5 hours,"Answer: D: 4.5 hours Explanation: Answer- D. 4.5 hours""Intravenous thrombolysis is usually practiced within the window period of 4.5 hours." "This is a real-world medical entrance exam question, please give the true answer based on the question and selection. Topic name: None, Subject name: Dental","99.73 variation involves how many SD? Select and explain. A. 1, B. 2, C. 3, D. 4",C: 3,"Answer: C: 3 Explanation: None" "This is a real-world medical entrance exam question, please give the true answer based on the question and selection. Topic name: None, Subject name: Dental","The best way to remove a hydrocolloid impression from the patient's mouth is: Select and explain. A. Slight rocking of the impression to disengage it from the undercut, B. Wetting the periphery of the impression with moist cotton to break the peripheral seal, C. Sudden jerking of the impression to prevent tearing, D. Supporting the impression along with the tray to prevent disengaging of the tray alone",C: Sudden jerking of the impression to prevent tearing,"Answer: C: Sudden jerking of the impression to prevent tearing Explanation: Since alginate is a viscoelastic material, its tear strength is increased when the impression is removed along a vertical path with a snap. The speed of removal must be a compromise between a  rapid movement and a slower rate that is more comfortable for the patient. Usually an alginate impression does not adhere to the oral tissues as well as some of the elastomers do, so it is easier to remove the alginate impression rapidly. However, it is always best to avoid torquing or twisting the impression in an effort to remove it quickly. Specifically, the handle should be used minimally during breaking of the air seal (“suction”) or removal of the tray from the teeth. Phillips dental materials 12th edition page no 174" "This is a real-world medical entrance exam question, please give the true answer based on the question and selection. Topic name: None, Subject name: Anatomy","Buccinator is pierced by all of the following except: Select and explain. A. Labial branch of facial nerve, B. Buccal branch of mandibular nerve, C. Parotid duct, D. Molar mucous glands",B: Buccal branch of mandibular nerve,"Answer: B: Buccal branch of mandibular nerve Explanation: Ans: B. Buccal branch of mandibular nerve(Ref Grays 41/e 4(Y 10/e p487; Snells 9/e p582)Buccal branch of facial run crosses the buccinator muscle and innervates it, without piercing it.Buccinator - Structures passing:Anteriorly (Superficial buccinator surface):Muscles - zygomaticus major, risorius, levator & depressor anguli oris muscles.Duct - Parotid duct.Pierces buccinator opposite third upper molar tooth & lies on deep muscle surface before opening into mouth opposite maxillary second molar tooth.Blood vessel - Crossed by facial aery, facial vein & branches of facial & buccal nerves." "This is a real-world medical entrance exam question, please give the true answer based on the question and selection. Topic name: None, Subject name: Surgery","Main disadvantage of dry heat used for sterilization is that: Select and explain. A. It is time consuming, B. Rusts the instruments, C. Dulls the sharp instruments, D. Ineffective sterilization",A: It is time consuming,"Answer: A: It is time consuming Explanation: None" "This is a real-world medical entrance exam question, please give the true answer based on the question and selection. Topic name: None, Subject name: Pathology","Reed Sternberg cells in Hodgkin's lymphoma are derived from Select and explain. A. Macrophages, B. N K cell, C. T cell, D. B cell",D: B cell,"Answer: D: B cell Explanation: None" "This is a real-world medical entrance exam question, please give the true answer based on the question and selection. Topic name: None, Subject name: Anatomy","Broca's area is related to Select and explain. A. Reading, B. Sentence formation, C. Word formation, D. Comprehension",C: Word formation,"Answer: C: Word formation Explanation: None" "This is a real-world medical entrance exam question, please give the true answer based on the question and selection. Topic name: None, Subject name: Anaesthesia","Which of the following statements is true or false regarding the CPR technique?1. Czan be given irrespective of rib fracture.2. An adult chest compression : breath is 30 : 2 to 15 : 2 even if 2nd rescuer present.3. In infants ratio change from 30 : 2 to 15 : 2 when 2nd rescuer arrive.4. Chest compression at rate of 100 - 120 / min on adults and 90 per minute in infants. Select and explain. A. a is false and b, c, d are true, B. a, b are true & c, d are false, C. a, c, d are true & b is false, D. b, c are true & a, d are false","A: a is false and b, c, d are true","Answer: A: a is false and b, c, d are true Explanation: Ans: A is false & b, c, d are trueCPR technique cannot be given to patients with rib fracture.In CPR technique, an adult chest compression : breath is 30 : 2 to 15 : 2 even if 2nd rescuer present.In infants ratio change from 30 : 2 to 15 : 2 when 2nd rescuer arrive.Chest compression at rate of 100 - 120 / min on adults and 90 per minute in infants." "This is a real-world medical entrance exam question, please give the true answer based on the question and selection. Topic name: None, Subject name: Pediatrics","CASE -2 SR visit again but the condition is not improved but this time IV cannula was set. What drug should be given now? Select and explain. A. Midazolam, B. I/V Phenobarbital, C. Oral valproate, D. IV carbamazepine",B: I/V Phenobarbital,"Answer: B: I/V Phenobarbital Explanation: Ans. In this case the First Line has failed . So for second line therapy I/V Phenobarbital is prefferedReference - 2, B. Positive shake test, C. Increased phosphatidyl glycerol, D. Blue cells in Nile Blue Test",D: Blue cells in Nile Blue Test,"Answer: D: Blue cells in Nile Blue Test Explanation: Ans D. Blue cells in Nile Blue TestRef: DC Dutla's textbook of Obstetrics,9't' ed.All of the above are correct except that it is the presence of more than 50 % orange coloured cells in Nile Blue test that suggests fetal pulmonary maturity." "This is a real-world medical entrance exam question, please give the true answer based on the question and selection. Topic name: AIIMS 2017, Subject name: Medicine","A 30 year old patient has right upper limb BP of 180/95 mm Hg and left upper limb BP of 130/90 mm Hg. He also has early diastolic murmur in right 2 intercostal space. Which of the following would be LEAST likely associated with these findings? Select and explain. A. Supravalvular aoic Stenosis, B. Coarctation of aoa, C. Takayasu aeritis, D. Aoic dissection",B: Coarctation of aoa,"Answer: B: Coarctation of aoa Explanation: Supra valvular AS - pulse disparity (Coanda effect) - Systolic pressure in the right arm> left arm Streaming of the jet toward the innominate, RCCA, and right SCA Option B - Continuous murmur /systolic murmur Option C - MC blood vessel involved in Takayasu aeritis is Subclan aery. - DIASTOLIC murmur is seen. Option D -unequal BP may be present" "This is a real-world medical entrance exam question, please give the true answer based on the question and selection. Topic name: None, Subject name: Dental","Acid neutralizing substance in saliva Select and explain. A. NH3, B. Carbonate, C. Bicarbonate, D. Chlorides",C: Bicarbonate,"Answer: C: Bicarbonate Explanation: None" "This is a real-world medical entrance exam question, please give the true answer based on the question and selection. Topic name: None, Subject name: Gynaecology & Obstetrics","A pregnant lady acquires chickenpox 3 days prior to delivery. She delivers by normal vaginal route which of the following statement is true? Select and explain. A. Both mother and baby are safe, B. Give antiviral treatment to mother before delivery, C. Give antiviral treatment to baby, D. Baby will develop neoatal varicella syndrome",D: Baby will develop neoatal varicella syndrome,"Answer: D: Baby will develop neoatal varicella syndrome Explanation: Varicella infection in pregnancy: If varicella infection occurs in a pregnant female during first half of pregnancy (M/C time of transmission-13 to 20 weeks) it results in congenital varicella syndrome in the fetus. Congenital varicella syndrome in characterized by chorioretinitis, microophthalmia, cerebral cortical atrophy, IUGR, hydronephrosis and skin or bone defects. Congenital varicella syndrome is an indication for doing MTP. Congenital defects rarely occurs if varicella infection occurs after 20 weeks. The terminology varicella embryopathy is not used these days. Neonatal varicella iin characterized by pneumonitis, hepatitis and DIC. The severity of neonatal infection is inversely related to the concentration of maternal antibodies present in the newborn circulation. Mother starts producing and transferring antibodies approximately 5 days after the onset of her disease.Thus, babies born 5 days or more from the beginning of maternal disease will be protected. Fernando Arias 3/e, p 156 Perinatal varicella exposure just before or during delivery poses a serious threat to newborns and so Varicella Ig should be given to all neonates of born to mothers who have clinical evidence of varicella 5 days before and upto 2 days after delivery. The use of VZIG decreases the chances of neonatal varicella and also modify the clinical course but it does not always prevent severe or fatal varicella. Expectant treatment with close observation, followed by prompt initiation of antiviral therapy on suspicion of neonatal varicella is recommended. Antiviral treatment (acyclovir) is given to neonates only if they develop neonatal varicella syndrome. Vaccine is not secreted in breast milk, so postpartum vaccination should not be delayed because of breast feeding." "This is a real-world medical entrance exam question, please give the true answer based on the question and selection. Topic name: None, Subject name: Dental","Method for slow tooth separation: Select and explain. A. Use of 26 gauge spring wire around proximal contacts, B. Use of soft brass wire around contacts, C. Use of oversized resin, D. All of the above",D: All of the above,"Answer: D: All of the above Explanation: Other than these methods, elastics are also used around contact points." "This is a real-world medical entrance exam question, please give the true answer based on the question and selection. Topic name: None, Subject name: Medicine","A patient presents to the ER after a A with multiple rib injuries. He is conscious, speaking single words. RR = 40/min, BP= 90/40 mmHg. What is the next immediate step in management? Select and explain. A. Intubate the patient, B. Urgent fluid infusion, C. Chest X ray, D. Needle inseion in 2nd ICS",D: Needle inseion in 2nd ICS,"Answer: D: Needle inseion in 2nd ICS Explanation: Answer- D. Needle inseion in 2nd ICSThis is a case of tension pneumothorax. Although the new ATLS update is 5th intercostal space in mid maxillary line, but in this question we will go with a time tested method of needle in 2nd intercostal space." "This is a real-world medical entrance exam question, please give the true answer based on the question and selection. Topic name: None, Subject name: Pediatrics","A 3 – month old infant presents with bilateral medullary nephrocalcinosis. All of the following can cause medullary nephrocalcinosis except : Select and explain. A. Hyperoxaluria, B. Bartter's syndrome, C. Prolonged use of furosemide, D. ARPKD",D: ARPKD,"Answer: D: ARPKD Explanation: Nephrocalcinosis : Deposition of calcium in the substance of the kidney. Causes of nephrocalcinosis : Idiopathic hypercalciuria. Hyperoxaluria. Distal renal tubular acidosis. Diuretics for preterm baby. Bartter syndrome. Dent's disease. Primary hyperparathyroidism. Familial hypomagnesemia, hypercalciuria with nephrocalcinosis." "This is a real-world medical entrance exam question, please give the true answer based on the question and selection. Topic name: None, Subject name: ENT","A patient has bilateral conductive deafness, tinnitus with positive family history. The diagnosis is - Select and explain. A. Otospongiosis, B. Tympanosclerosis, C. Meniere's disease, D. B/L otitis media",A: Otospongiosis,"Answer: A: Otospongiosis Explanation: Conductive deafness means the disease process leading to deafness is limited to external ear tympanic membrane, middle ear including the footplate of stapes. Bilateral conductive deafness rules out meniere’s disease (as it presents with SNHL). Amongst the remaining three options, positive family history is seen mainly in case of otosclerosis (Otospongiosis), so it is our answer. Ref. Dhingra 6/e, p 30, 87" "This is a real-world medical entrance exam question, please give the true answer based on the question and selection. Topic name: None, Subject name: Pediatrics","In a child with tetralogy of Fallot with fever and diarrhea, which of the following is the surest sign of a cyanotic spell? Select and explain. A. Hepatomegaly, B. Absence of murmur, C. S3 gallop rhythm, D. Aerial oxygen saturation of less than 75%",B: Absence of murmur,"Answer: B: Absence of murmur Explanation: Ans: B: Absence of murmurExplanation:(Ref: Nelson 20Ie p2212; Ghai 8/e p422)During cyanotic spell, temporary disappearance or a decrease in intensity of the systolic murmur is usual as flow across the right ventricular outflow tract diminishes.Paroxysmal hypercyanotic attacks (hypoxic, ""blue,"" or ""tet"" spells) are a paicular problem during the 1st 2 years of life.The infant becomes hyperpneic and restless, cyanosis increases, gasping respirations ensue, and syncope may follow.Most frequently in morning on initially awakening or after episodes of vigorous crying." "This is a real-world medical entrance exam question, please give the true answer based on the question and selection. Topic name: None, Subject name: Dental","Resolution of IOPA radiograph is? Select and explain. A. 16 lp, B. 22 lp, C. 30 lp, D. 10 lp",A: 16 lp,"Answer: A: 16 lp Explanation: None" "This is a real-world medical entrance exam question, please give the true answer based on the question and selection. Topic name: AIIMS 2018, Subject name: Medicine","Which of the following is used for laboratory diagnosis of alcoholic hepatitis? Select and explain. A. ALP, B. AST, C. LDH, D. GGT",B: AST,"Answer: B: AST Explanation: AST/SGOT (found in the mitochondria of Hepatocyte, and Alcohol get metabolize in mitochondria) - GGT - Fatty liver - Alcoholic | NAFLD/NASH: Syndrome X, Reye syndrome ALT - Obstructive jaundice" "This is a real-world medical entrance exam question, please give the true answer based on the question and selection. Topic name: None, Subject name: Dental","A radiograph is shown below. This type of root morphology has highest incidence in which population? Select and explain. A. African, B. Caucasians, C. Mongoloids, D. Asians",D: Asians,"Answer: D: Asians Explanation: A variation in root morphology is the presence of an extra distolingual root. Usually, this root has a type 1 canal configuration. Two-thirds of the first mandibular molars found in a Chinese population had this variation. Similarly, this distolingual root occurred in 4% of mandibular first molars of a Kuwaiti population. These results confirm the observation that East Asian populations have more three-rooted mandibular first molars than do other racial groups. Mandibular molars, mainly first molars, may also have an additional root located lingually or buccally. Although this is a rare occurrence in Caucasian populations, it is more common in Asian populations. The radix entomolaris (RE) is a supernumerary root located distolingually in mandibular molars, whereas the radix paramolaris (RP) is an extra root located mesiobuccally. Each root usually contains a single root canal. The orifice of the RE is located distolingually to mesiolingually from the main canal or canals of the distal root; the orifice of the RP is located mesiobuccally to distobuccally from the main mesial root canals.  Key Concept: The figure in the question show radix entomolaris." "This is a real-world medical entrance exam question, please give the true answer based on the question and selection. Topic name: None, Subject name: Microbiology","Virus is cultured in: Select and explain. A. Sabouraud's agar, B. Rogosa medium, C. Nonembryonated egg, D. Embryonated egg",D: Embryonated egg,"Answer: D: Embryonated egg Explanation: Embryonated eggs were used initially for the growth of viruses. Embryonated chick egg was used first for cultivation of viruses by Goodpasture in 1931.  Usually, 8–11 days old chick eggs are used for culture of viruses. The viruses are isolated in different sites of the egg, such as yolk sac, amniotic cavity, and allantoic cavity, and chorioallantoic membrane (CAM)." "This is a real-world medical entrance exam question, please give the true answer based on the question and selection. Topic name: AIIMS 2018, Subject name: Biochemistry","Vitamin K in its coenzyme form is regenerated by which enzyme? Select and explain. A. Glutathione reductase, B. Pyruvate carboxylase, C. Dihydrofolate reductase, D. Epoxide reductase",D: Epoxide reductase,"Answer: D: Epoxide reductase Explanation: Vitamin K - Vitamin K hydroquinone is oxidized to the epoxide Epoxide activates a glutamate residue in the protein substrate to a carbanion, which reacts nonenzymically with carbon dioxide to form g-carboxyglutamate. Vitamin K epoxide is reduced to the quinone by an epoxide reductase, and the quinone is reduced to the active hydroquinone by Vitamin K reductase. Dicumarol & warfarin inhibits the enzyme (reductase) that conves epoxide to active Vitamin K (hydroquinone)." "This is a real-world medical entrance exam question, please give the true answer based on the question and selection. Topic name: None, Subject name: Pediatrics","18 months old child, who has received one dose of DPT and OPV at 2 months of age. What will be your next immunization plan : Select and explain. A. Restart immunization schedule, as per age, B. Measles, BCG, booster dose of DPT and OPV, C. Measles, booster dose of DPT and OPV, D. BCG, 2ND dose of DPT and OPV","D: BCG, 2ND dose of DPT and OPV","Answer: D: BCG, 2ND dose of DPT and OPV Explanation: A lapse in the immunization schedule does not require reinitiation of entire vaccine series. Immunization should be given at the next visit in the usual doses assuming that the minimum interval has already elapsed and the imunization should be completed at the next available opportunity"". D.P.T. and oral polio vaccine which the child already had should be considered as the first dose. The child would need further 2 doses of this vaccine to complete the primary vaccination. Booster doses are administered, once the primary immunization is complete. Thus you cannot administer booster doses of D.P.T. and O.P.V. as their primary immunization is not complete." "This is a real-world medical entrance exam question, please give the true answer based on the question and selection. Topic name: None, Subject name: Dental","HTR polymer is Select and explain. A. Polyether, B. Polytmethymethacrylate, C. Silica, D. Hydroxyapatite",B: Polytmethymethacrylate,"Answer: B: Polytmethymethacrylate Explanation: None" "This is a real-world medical entrance exam question, please give the true answer based on the question and selection. Topic name: None, Subject name: ENT","The treatment of choice for stage I cancer larynx is- Select and explain. A. Radical surgery, B. Chemotherapy, C. Radiotherapy, D. Surgery followed by radiotherapy",C: Radiotherapy,"Answer: C: Radiotherapy Explanation: Friends remember 2 very important concepts regarding laryngeal Ca: If the site of larynx caner viz supra glottis, glottis or subglottis is not mentioned, the cancer should be considered glottic (since it is the M/C variety) Generally stage I, II, III, IV means stage T1 , T2 , T3 , T4 respectively. According to Dhingra Radiotherapy is the treatment of choice for all stage I cancers of larynx, which neither impair mobility nor invade cartilage or cervical nodes. The greatest advantage of radiotherapy over surgery in Ca larynx glottic cancer is - preservation of voice." "This is a real-world medical entrance exam question, please give the true answer based on the question and selection. Topic name: None, Subject name: Dental","Gingival Recession is most common due to Select and explain. A. Tooth brush trauma, B. TFO, C. Deflective contacts, D. ANUG",A: Tooth brush trauma,"Answer: A: Tooth brush trauma Explanation: None" "This is a real-world medical entrance exam question, please give the true answer based on the question and selection. Topic name: None, Subject name: Dental","Which of the following prevents fracture of MO amalgam: Select and explain. A. Beveling of axiopulpal line angle, B. Occlusal dovetails, C. Pin retained amalgam, D. Removal of unsupported enamel rods",A: Beveling of axiopulpal line angle,"Answer: A: Beveling of axiopulpal line angle Explanation: None" "This is a real-world medical entrance exam question, please give the true answer based on the question and selection. Topic name: None, Subject name: Pathology","Bacteria associated with caries are able to cause the same due to: Select and explain. A. Production of extxacellular polysaccharides like dextrans and levans, B. Production of acid to hydrolyse diet carbohydrates, C. Production of enzymes which inhibit normal flora, D. Inhibition of secretory IgA",A: Production of extxacellular polysaccharides like dextrans and levans,"Answer: A: Production of extxacellular polysaccharides like dextrans and levans Explanation: None" "This is a real-world medical entrance exam question, please give the true answer based on the question and selection. Topic name: AIIMS 2019, Subject name: Pharmacology","Most commonly used route of administration of heparin for post-operative thromboprophylaxis is? Select and explain. A. Subcutaneous, B. Intravenous, C. Inhalational, D. Intramuscular",A: Subcutaneous,"Answer: A: Subcutaneous Explanation: UFH is injected s.c. every 8- 12 hours, staed before surgery and continued for 7-10 day, or till the patient stas moving about. This regimen has been found to prevent postoperative deep vein thrombosis (post operative thromboprophylaxis) without increasing surgical bleeding. It also does not prolong aPTT or clotting time. However, it should not he used in case of neurosurgery or when spinal anaesthesia is to be given." "This is a real-world medical entrance exam question, please give the true answer based on the question and selection. Topic name: None, Subject name: Gynaecology & Obstetrics","In which part of fallopian tube ectopic pregnancy will have longest survival: Select and explain. A. Isthmus, B. Ampulla, C. Cornua, D. Interstitium",D: Interstitium,"Answer: D: Interstitium Explanation: M.C site of ectopic pregnancy – Fallopian tubes. In Fallopian Tubes – M/C sites in descending order are: Ampulla > mus > infundibulum > interstitium Rarest overall site of ectopic pregnancy is cervix or cesarean section scan Average period of survival of ectopic pregnancy is 8 weeks. Ectopic pregnancy survives for longest time in its annual pregnancy." "This is a real-world medical entrance exam question, please give the true answer based on the question and selection. Topic name: AIIMS 2018, Subject name: Pharmacology","Major mechanism of action of nitrates in acute attack of angina is: Select and explain. A. Coronary vasodilation, B. Decrease in preload, C. Decrease in afterload, D. Decrease in hea rate",B: Decrease in preload,"Answer: B: Decrease in preload Explanation: Aim for the treatment of classical angina is decrease in work of hea. It can be accomplished by decrease in preload, decrease in afterload or decrease in hea rate. Nitrates act by production of nitric oxide (NO). NO stimulates guanylate cyclase in smooth muscles which leads to formation of cGMP. Later acts on smooth muscles to cause vasodilation. Nitrates mainly produce NO in veins leading to its predominant action as venodilation. Dilation of veins results in decrease in pre-load. Therefore major mechanism of action of nitrates in angina is decrease in preload. However, in variant angina these drugs benefit by causing coronary vasodilation as variant angina occurs due to vasospasm of coronary aery." "This is a real-world medical entrance exam question, please give the true answer based on the question and selection. Topic name: None, Subject name: Dental","Which of the following is least sensitive to radiation exposure Select and explain. A. Bone, B. Hematopoietic tissue, C. Nerve, D. Muscle",C: Nerve,"Answer: C: Nerve Explanation: None" "This is a real-world medical entrance exam question, please give the true answer based on the question and selection. Topic name: None, Subject name: Dental","Which of the following least resemble permanent tooth? Select and explain. A. Incisor, B. Canine, C. Mandibular 1st Molar, D. Mandibular 2nd Molar",C: Mandibular 1st Molar,"Answer: C: Mandibular 1st Molar Explanation: None" "This is a real-world medical entrance exam question, please give the true answer based on the question and selection. Topic name: None, Subject name: Anatomy","Umblical cord contains: Select and explain. A. 3 arteries & 1 vein, B. 1 artery& 3 veins, C. 1 artery & 1 vein, D. 2 arteries & 1 vein",D: 2 arteries & 1 vein,"Answer: D: 2 arteries & 1 vein Explanation: A patent opening called the primitive umbilical ring exists on the ventral surface of the developing embryo through which three structures pass: the yolk sac (vitelline duct), connecting stalk, and allantois. The allantois is not functional in humans and degenerates to form the median umbilical ligament in the adult. As the amnion expands, it pushes the vitelline duct, connecting stalk, and allantois together to form the primitive umbilical cord. The definitive umbilical cord at term is pearl-white, 1–2 cm in  diameter,  50–60  cm  long, eccentrically  positioned,  and contains the right and left umbilical arteries, left umbilical vein, and mucus connective tissue (Wharton’s jelly). The right and left umbilical arteries carry deoxygenated blood from the fetus to the placenta. The left umbilical vein carries oxygenated blood from the placenta to the fetus. Key Concept: Umblical cord contains the right and left umbilical arteries, left umbilical vein, and mucus connective tissue (Wharton’s jelly)." "This is a real-world medical entrance exam question, please give the true answer based on the question and selection. Topic name: None, Subject name: Dental","Looseness seen in a recently delivered RPD is due to: Select and explain. A. Defective occlusal balancing, B. Passive clasp on abutment, C. Thin flange, D. Insufficient overlap on posterior teeth",A: Defective occlusal balancing,"Answer: A: Defective occlusal balancing Explanation: None" "This is a real-world medical entrance exam question, please give the true answer based on the question and selection. Topic name: None, Subject name: Pathology","In a patient with Hepatitis B infection. Which one of the markers will be increased? Select and explain. A. HbsAg, B. HbcAg, C. Anti HbsAg IgG, D. Anti HbcAg IgG",D: Anti HbcAg IgG,"Answer: D: Anti HbcAg IgG Explanation: Answer- D. Anti HbcAg IgGBest marker for diagnosing acute hepatitis B is IgM anti-HBc as it is found only in the acute phase of hepatitis B (In chronic hepatitis IgG anti-HBc is found)." "This is a real-world medical entrance exam question, please give the true answer based on the question and selection. Topic name: None, Subject name: Pathology","Sunlight is one of the etiological causes of: Select and explain. A. Squamous cell carcinoma, B. Malignant melanoma, C. Basal cell carcinoma, D. Port-wine stain",C: Basal cell carcinoma,"Answer: C: Basal cell carcinoma Explanation: None" "This is a real-world medical entrance exam question, please give the true answer based on the question and selection. Topic name: None, Subject name: Microbiology","In a patient with typhoid, diagnosis after 15 days of onset of fever is best done by: Select and explain. A. Blood culture, B. Widal test, C. Stool culture, D. Urine culture",C: Stool culture,"Answer: C: Stool culture Explanation: None" "This is a real-world medical entrance exam question, please give the true answer based on the question and selection. Topic name: None, Subject name: Surgery","Which pa of 2nd metatarsal is involved in the March fracture? Select and explain. A. Head, B. Neck, C. Shaft, D. Base",B: Neck,"Answer: B: Neck Explanation: Answer- B. NeckMost common site of stress fracture or March fracture is second metatar'sal neck.likely site for stress fracturces are common in distance runners and ballet dancers. The second metatarsal neck is the most likety sitefor stressfractures, but all metatarsals are susceptible" "This is a real-world medical entrance exam question, please give the true answer based on the question and selection. Topic name: None, Subject name: Anatomy","Branchial fistula is persistent of: Select and explain. A. 1st branchial cleft., B. 1st branchial arch., C. 2nd branchial cleft., D. 2nd branchial arch.",C: 2nd branchial cleft.,"Answer: C: 2nd branchial cleft. Explanation: None" "This is a real-world medical entrance exam question, please give the true answer based on the question and selection. Topic name: None, Subject name: Dental","A patient reported with Disto-oclusal amalgam restoration in 47 & complaints of sticking of food in interproximal area. Most common reason is? Select and explain. A. Broad contact area Buccolingually, B. Wide contact area occlugingivaly, C. Contact area too occlusively, D. Contact area too gingivally",D: Contact area too gingivally,"Answer: D: Contact area too gingivally Explanation: None" "This is a real-world medical entrance exam question, please give the true answer based on the question and selection. Topic name: None, Subject name: Medicine","All of the following clinical features are seen in Zika fever except: Select and explain. A. Guillain-Barre syndrome, B. Petechial rash, C. Fever with ahralgia, D. Petechial rash",B: Petechial rash,"Answer: B: Petechial rash Explanation: Answer- B. Petechial rash ""Zika virus is an emerging pathogen that is transmitted among nonhuman primates and humans by Aedes mosquitoes.Zia virus infection is characterized by influenza-like clinical signs, including fever, headaches, and malaise. A maculopapular rash, conjunctivitis,myalgia,and ahralgia usually accompany or follow those manfestations.""" "This is a real-world medical entrance exam question, please give the true answer based on the question and selection. Topic name: None, Subject name: Ophthalmology","First order neuron of visual pathway - Select and explain. A. Photoreceptor, B. bipolar neuron, C. lateral geniculate body, D. all of the above",A: Photoreceptor,"Answer: A: Photoreceptor Explanation: Answer-:A: photoreceptorRods and cones are the first-order receptor cells that respond directly to light stimulation.Bipolar neurons are the second-order neurons that relay stimuli from the rods and cones to the ganglion cells.Ganglion cells third-order neurons that form the optic nerve (CN II)." "This is a real-world medical entrance exam question, please give the true answer based on the question and selection. Topic name: None, Subject name: Dental","Patient with complete denture complains of pain while swallowing and there is overextension in the distolingual aspect. The muscle involved is: Select and explain. A. Superior constrictor, B. Palatopharyngeus, C. Palatoglossus, D. Styloharyngeus",A: Superior constrictor,"Answer: A: Superior constrictor Explanation: None" "This is a real-world medical entrance exam question, please give the true answer based on the question and selection. Topic name: None, Subject name: Surgery","Urobilinogen levels in obstructed jaundice due to gallstones will be: Select and explain. A. Markedly raised, B. Slightly increased, C. Normal, D. Completely absent",D: Completely absent,"Answer: D: Completely absent Explanation: Ans: D. Completely absent(Ref: Harrison 19Ie p280, 18Ie p325)Suggestive of obstructive jaundice. Function testPrehepatic jaundiceHepatic jaundicePost-hepatic (Obstructive) jaundiceTotal bilirubinNormal/ increasedIncreaseddegIncreaseddegConjugated bilirubinNormalIncreaseddegIncreaseddegUnconjugated bilirubinNormal/ increasedIncreaseddegNormalUrobilinogenNormal/ increasedDecreaseddegDecreased/ negativedegUrine colorNormalDark (urobilinogen +conjugated bilirubin)degDark (conjugated bilirubin)degStool colorNormalNormal/paledegPaledegAlkaline phosphatase levelsNormalIncreaseddegIncreaseddegAlanine transferase & aspaate transferase levelsNormalIncreaseddegIncreaseddegConjugated bilirubin in urineAbsentdegPresentdegPresentdegSplenomegalyPresentdegPresentdegAbsentdeg" "This is a real-world medical entrance exam question, please give the true answer based on the question and selection. Topic name: None, Subject name: Pathology","Which is not true about phenotype? Select and explain. A. It is modified with the passage of time, B. It is the appearance of an individual, C. It is genetic sequence of an individual, D. It is influenced by genotype",C: It is genetic sequence of an individual,"Answer: C: It is genetic sequence of an individual Explanation: None" "This is a real-world medical entrance exam question, please give the true answer based on the question and selection. Topic name: None, Subject name: Surgery","Both bone and disc spaces are destroyed in- Select and explain. A. Tuberculosis, B. Metastasis, C. Lymphoma, D. Multiple myeloma",A: Tuberculosis,"Answer: A: Tuberculosis Explanation: Answer- A. TuberculosisThe spine is the MC site of skeletal tuberculosis, accounting for 5OVo cases followed by hip and knees.MC infective pathology of spine is tuberculosis." "This is a real-world medical entrance exam question, please give the true answer based on the question and selection. Topic name: None, Subject name: Medicine","The following drug is not useful for MRSA? Select and explain. A. Cefaclor, B. Cotrimoxazole, C. Ciprofloxacin, D. Vancomycin",A: Cefaclor,"Answer: A: Cefaclor Explanation: None" "This is a real-world medical entrance exam question, please give the true answer based on the question and selection. Topic name: AIIMS 2018, Subject name: Anatomy","Loss of flexion in the interphalangeal joint of thumb in supracondylar fracture is due to involvement of which nerve Select and explain. A. Anterior interosseous, B. Posterior interosseous, C. Median, D. Ulnar",A: Anterior interosseous,"Answer: A: Anterior interosseous Explanation: Flexion at IP joint of thumbs - by flexor pollicis longus Nerves that may be involved in supracondylar humerus : 1. Median nerve - most common Paial injury occurs usually. i.e. fibers involved in forming AIN branch. 2. Radial nerve - less often 3. Ulnar nerve - least commonly involved. AIN supply: Flexor pollicis longus * Lateral 1/2 of flexor digitorum profundus * Pronator quadratus" "This is a real-world medical entrance exam question, please give the true answer based on the question and selection. Topic name: None, Subject name: Dental","Study model with mounted base and trimmed height is: Select and explain. A. 55 mm, B. 70 mm, C. 60 mm, D. 90 mm",B: 70 mm,"Answer: B: 70 mm Explanation: The cast should be such that the base of the lower model is equal in thickness to that of the maxillary model. The total height of both casts in occlusion should be about 70–75 mm." "This is a real-world medical entrance exam question, please give the true answer based on the question and selection. Topic name: None, Subject name: Biochemistry","In noncompetitive antagonism, the true statement is: Select and explain. A. Km value decrease;.vmaxdecreases, B. no change in vmax;Km value decrease, C. V max decreased; Km value normal, D. Km value increased; V max increased",C: V max decreased; Km value normal,"Answer: C: V max decreased; Km value normal Explanation: Answer is c. vmax decreases, km unchangedRef: Harper's Illustrated Biochemistry 30th edn; Page no. 82Noncompetitive inhibitors bind to the enzyme or the enzyme-substrate complex at a site different from the active site, decreasing the activity of the enzyme. Thus, Vmax is decreased.Competitive inhibitionNoncompetitive inhibitionIt is reversed by increasingIrreversiblesubstrate concentration.Excess substrate does not abolishExcess substrate abolishesthe inhibition.inhibition.Km remains the same, but thevmax remains the same, but theapparent vmax is decreased.apparent Km is increased.Inhibitors have no structuralInhibitor will be structuralanalogue of substrate.resemblance to substrate." "This is a real-world medical entrance exam question, please give the true answer based on the question and selection. Topic name: None, Subject name: Physiology","T-lymphocytes play a primary role in Select and explain. A. Production of Antibodies, B. Production of lymphokines and delayed hypersensitivity, C. Activation of complement system, D. Immediate Hypersensitivity",B: Production of lymphokines and delayed hypersensitivity,"Answer: B: Production of lymphokines and delayed hypersensitivity Explanation: None" "This is a real-world medical entrance exam question, please give the true answer based on the question and selection. Topic name: None, Subject name: Pathology","Apoptosis is suggestive of: Select and explain. A. Liquefaction degeneration, B. Coagulative necrosis, C. Neo angiogenesis, D. Epithelial dysplasia",B: Coagulative necrosis,"Answer: B: Coagulative necrosis Explanation: None" "This is a real-world medical entrance exam question, please give the true answer based on the question and selection. Topic name: None, Subject name: Dental","Electric resistance between oral mucosa and PDL is always constant that is: Select and explain. A. 4.5 K ohm, B. 5.5 K ohm, C. 6.5 K ohm, D. 7.5 K ohm",C: 6.5 K ohm,"Answer: C: 6.5 K ohm Explanation: FIRST-GENERATION APEX LOCATORS (RESISTANCE APEX LOCATORS) They are also known as resistance apex locators that measure opposition to the flow of direct current, that is, resistance. It is based on the principle that resistance offered by periodontal ligament and oral mucous membrane is the same, that is, 6.5 K ohms. Initially, Sono-Explorer was imported from Japan by Amadent, (Port Jefferson, New York) but nowadays first generation apex locators are off the practice. Blood, pus, chelating agents, irrigants, and other materials used within the canal can give false readings." "This is a real-world medical entrance exam question, please give the true answer based on the question and selection. Topic name: None, Subject name: Pathology","Frey's syndrome results from surgery of the Select and explain. A. a)Submandibular salivary gland, B. Parotid gland, C. Sublingual salivary gland, D. TMJ",B: Parotid gland,"Answer: B: Parotid gland Explanation: Auriculo temporal syndrome or Frey syndrome or gustatory sweating is due to damage of auricutotemporal nerve following surgery in parotid and mandibular ramus areas, and subsequent innervation of sweat glands by parasympathetic salivary fibres. The patient typically exhibits flushing and sweating of the involved side of the face, chiefly in the temporal area, during eating." "This is a real-world medical entrance exam question, please give the true answer based on the question and selection. Topic name: AIIMS 2017, Subject name: Pathology","In a platelet poor plasma sample calcium and tissue thromboplastin is added. This is used to assess which of the following pathway? Select and explain. A. Extrinsic, B. Intrinsic, C. Fibrinolytic, D. Common",A: Extrinsic,"Answer: A: Extrinsic Explanation: Whenever we want to assess the Extrinsic or Intrinsic pathway, the sample that is taken is Platelet poor plasma sample. Assessment of EXTRINSIC PATHWAY - tissue thromboplastin is added along with calcium Intrinsic pathway - addition of Ca2+ along with negatively charged substance like Kaolin or Cephalin." "This is a real-world medical entrance exam question, please give the true answer based on the question and selection. Topic name: AIIMS 2019, Subject name: Pharmacology","All of the following can be used to decreased intraocular pressure in glaucoma except? Select and explain. A. Mannitol, B. Methazolamide, C. Clonidine, D. Dexamethasone",D: Dexamethasone,"Answer: D: Dexamethasone Explanation: Explanation- Dexamethasone is a coicosteroid. coicosteroids are contraindicated in Glaucoma , as they result in fuher increase in the intraocular pressure. Mannitol is drug of choice for acute congestive glaucoma. For acute attack of close angle glaucoma mannitol is given IV it result in considerable decrease in Intraocular pressure.Hypeonic mannitol decongest the eye by osmotic action. TOPICAL DRUGS FOR GLAUCOMA b-Adrenergic blockers a-Adrenergic agonists Prostaglandin analogues Carbonic anhydrase inhibitors Miotics Timolol Betaxolol Levobunolol Caeolol Dipivefrine Apraclonidine Brimonidine Latanoprost Travoprost Bimatoprost Dorzolamide Brinzolamide Pilocarpine Physostigmine Echothiophate Methazolamide is carbonic anhydride inhibitor and clonidine is alpha 2 agonist. These can also cause decrease in intra ocular pressure." "This is a real-world medical entrance exam question, please give the true answer based on the question and selection. Topic name: None, Subject name: Dental","The undesirable side effect most commonly associated with the use of finger spring to tip the crown of a tooth is: Select and explain. A. Pain, B. Gingival irritation, C. Tendency for the root apex to move in the direction opposite from the crown, D. Tendency for the tooth to intrude",C: Tendency for the root apex to move in the direction opposite from the crown,"Answer: C: Tendency for the root apex to move in the direction opposite from the crown Explanation: Excessive forces can cause unwanted tooth movement. Unwanted excessive tipping of maxillary central incisor root is caused by over activation of finger spring." "This is a real-world medical entrance exam question, please give the true answer based on the question and selection. Topic name: AIIMS 2019, Subject name: Surgery","A chronic alcoholic patient came to emergency with severe pain in epigastrium and multiple episodes of vomiting. On examination, guarding was present in upper epigastrium. Chest X-ray was normal. What is the next best step? Select and explain. A. Upper GI endoscopy, B. Serum lipase, C. Alcohol breath test, D. CECT",B: Serum lipase,"Answer: B: Serum lipase Explanation: From the given case: ACUTE PANCREATITIS- Chronic Alcoholic- Severe pain in Epigastrium- Episodes of vomiting* Normal CXR- Perforation peritonitis is ruled out - no pneumoperitoneum Acute Pancreatitis: Etiology Clinical Presentation Gall stones Pain in Epigastrium- radiating to left back Alcoholism Ileus- Nausea & Vomiting Trauma to Abdomen Tachypnea Ampullary tumor Abdominal distention Drugs Guarding +nt Hyperparathyroidism Hypotension maybe +nt. Hypercalcemia Pancreatic divisum Viral infections Scorpion bite Malnutrition Idiopathic INVESTIGATIONS:Initial investigation:- Clinical findings- Amylase - Not specific- Lipase - Specific (preferred) Imaging: - CECT - IOC + Staging investigation.- Balthazar criteria- To be performed after 72 hours as pancreatic necrosis is evident 1st 24 hrs.- Upper GIE & Alcohol breath test - Not done as the patient is alcoholic- No history of upper GI bleeding" "This is a real-world medical entrance exam question, please give the true answer based on the question and selection. Topic name: None, Subject name: Dental","Process shown in color plate is: Select and explain. A. Bleeding on probing., B. Bone sounding., C. Pocket measurement, D. Attached ginigiva measurement",B: Bone sounding.,"Answer: B: Bone sounding. Explanation: None" "This is a real-world medical entrance exam question, please give the true answer based on the question and selection. Topic name: None, Subject name: Surgery","In oral and maxillofacial surgery, 'danger space' is known as Select and explain. A. Carotid sheath, B. Posterior to carotid sheath in posterior triangle of neck, C. Posterior to transverse process of vertebrae, D. Space between alar &prevertebral fascia",D: Space between alar &prevertebral fascia,"Answer: D: Space between alar &prevertebral fascia Explanation: None" "This is a real-world medical entrance exam question, please give the true answer based on the question and selection. Topic name: AIIMS 2019, Subject name: Pharmacology","Doc of bacterial vaginosis in pregnancy is Select and explain. A. Clindamycin, B. Metronidazole, C. Erythromycin, D. Rovamycin",B: Metronidazole,"Answer: B: Metronidazole Explanation: - Metronidazole is drug of choice for bacterial vaginosis ( with or without pregnancy )- Alternative to metronidazole is clindamycin.- Rovamycin i.e. spiramycin (which is a macrolide) is drug of choice for toxoplasmosis in pregnancy. Metronidazole -* It is the prototype nitroimidazole having broad-spectrum cidal activity against anaerobic protozoa, including Giardia lamblia , trichomonas and amoeba. * Many anaerobic and microaerophilic bacteria, such as Bact. fragilis, Fusobacterium, Clostridium perfringens, Cl. difficile, Helicobacter pylori, Campylobacter, peptococci, spirochetes and anaerobic Streptococci are sensitive. * Metronidazole does not affect aerobic bacteria. Mechanism of action- After entering the cell by diffusion, its nitro group is reduced by ceain redox proteins operative only in anaerobic microbes to a highly reactive nitro radical which exes cytotoxicity. The nitro radical of metronidazole acts as an electron sink which competes with the biological electron acceptors of the anaerobic organism for the electrons generated by the pyruvate : ferredoxin oxidoreductase (PFOR) enzyme pathway of pyruvate oxidation. The energy metabolism of anaerobes that have no mitochondria is thus, disrupted. Aerobic environment attenuates cytotoxicity of metronidazole by inhibiting its reductive activation. Moreover, O2 competes with the nitro radical of metronidazole for the free electrons generated during energy metabolism of anaerobes. Metronidazole, in addition, has been found to inhibit cell mediated immunity, to induce mutagenesis and to cause radiosensitization. Uses of metronidazole- o Amoebiasiso Giardiasiso Trichomonas vaginitiso Bacterial vaginosiso Anaerobic bacterial infection o Pseudomembranous enterocolitiso Acute necrotising ulcerative gingivitiso Helicobacter pylori gastritis o Guinea worm infestation" "This is a real-world medical entrance exam question, please give the true answer based on the question and selection. Topic name: None, Subject name: Dental","The biofilms found on tooth surfaces are termed as: Select and explain. A. Enamel, B. Dental caries, C. Dental plaque, D. Saliva",C: Dental plaque,"Answer: C: Dental plaque Explanation: None" "This is a real-world medical entrance exam question, please give the true answer based on the question and selection. Topic name: None, Subject name: Pediatrics","A girl child with fever, cough, dyspnoea with x–ray showing right lower lobe patchy consolidation, for which treatment was given. After 8 weeks symptom improved but x-ray showed more dense consolidation involving the whole of the right lower lobe. What is the next best line of investigation ? Select and explain. A. Bronchoscopy, B. Culture from nasopharynx, C. Barium esophagogram, D. Allergic skin test",A: Bronchoscopy,"Answer: A: Bronchoscopy Explanation: A history of recurrent pneumonia in the same lobe or segment or slow resolution (> 3 months) of pneumonia on successive radiographs suggests the possibility of bronchial obstruction and the need for bronchoscopy." "This is a real-world medical entrance exam question, please give the true answer based on the question and selection. Topic name: None, Subject name: Dental","Mouth Breathing is most commonly seen in: Select and explain. A. Athetoid type of cerebral palsy., B. Spastic type of cerebral palsy., C. Ataxic type of cerebral palsy., D. Mixed type.",A: Athetoid type of cerebral palsy.,"Answer: A: Athetoid type of cerebral palsy. Explanation: None" "This is a real-world medical entrance exam question, please give the true answer based on the question and selection. Topic name: None, Subject name: Forensic Medicine","According to 'DELHI ANATOMY ACT 1957', a person died in road traffic accident, the dead body is said to be unclaimed after. Select and explain. A. 24 hours, B. 48 hours, C. 72 hours, D. 96 hours",C: 72 hours,"Answer: C: 72 hours Explanation: Answer- C. 72 hoursAccording to 'DELHI ANATOMY ACT 1957', a person died in road traffic accident the dead body is said to be unclaimed after 72 hours.According to Delhi Anatomy Act 1953, the hospital authority can utilize an unclaimed body (a person who dies in hospital, prison or public places, which has not been claimed by any of his near relatives or personal friends within the prescribed timeline of 48 hours) for the purpose of conducting anatomical examination and dissection or other similar purpose." "This is a real-world medical entrance exam question, please give the true answer based on the question and selection. Topic name: AIIMS 2018, Subject name: Pharmacology","Which of the following hormone is/are under inhibitory control of hypyothalamus? Select and explain. A. Prolactin, B. Only Prolactin, C. Only Growth hormone, D. Both Prolactin and Growth hormone",D: Both Prolactin and Growth hormone,"Answer: D: Both Prolactin and Growth hormone Explanation: Inhibitory hormones secreted by hypothalamus are somatostatin and dopamine. Somatostatin inhibits GH, ACTH, TSH. Dopamine inhibits prolactin. GH is under both stimulatory (GHRH) and inhibitory control (Somatostatin) Prolactin is only under inhibitory control (dopamine). Note: Read the question carefully; if they ask which hormones of anterior pituitary are under inhibitory control (as in this question), answer should be both prolactin and GH. However, if they ask the question, which hormone is ONLY under inhibitory control, then answer should be only prolactin." "This is a real-world medical entrance exam question, please give the true answer based on the question and selection. Topic name: None, Subject name: Social & Preventive Medicine","Annual new case detection rate of leprosy as on 31st March, 2016 is: Select and explain. A. 0.66 /10,000 population, B. 0.66/1,00,000 population, C. 9.7/10,000 population, D. 9.7/1,00,000 population","D: 9.7/1,00,000 population","Answer: D: 9.7/1,00,000 population Explanation: Ans. d. 9.7/1,00,000 populationAnnual new case detection rate of leprosy as on 31"" March, 2016 is 9.7/1,00,000 population.NLEP - Annual Repo for the year 2015-16:Based on the repos received from all the States and UTs for the year of 2015 -16 (Annexure -I), current leprosy situation in the country is as below:A total of 127334 new cases were detected during the year 2015-16, which gives Annual New Case Detection Rate (ANCDR) of 9.71 per 100,000 population, as against 125785 cases in 2014-15.A total of 86028 leprosy cases are on record as on 1st April 2016, giving a Prevalence Rate (PR) of 0.66 per 10,000 population, as against 88833 cases in 1 't April 2015.Detailed information on new leprosy cases detected during 2015-16 indicates the propoion of MB (51.27%), Female (38.33%), Child (8.94%), Grade II Deformity (4.60%), ST cases (18.79%) and SC cases (18.57%).A total of 5851 Gr. II disability detected amongst the New Leprosy Cases during 2015-16, indicating the Gr. II Disability Rate of 4.46 / million population (Annexure-II)A total of 11389 child cases were recorded, indicating the Child Case rate of 8.94% (Annexure-III)." "This is a real-world medical entrance exam question, please give the true answer based on the question and selection. Topic name: None, Subject name: Anaesthesia","What is the most reliable site to measure core temperature during general anesthesia? Select and explain. A. Pulmonary aery, B. Distal esophagus, C. Rectum, D. Tympanic membrane",D: Tympanic membrane,"Answer: D: Tympanic membrane Explanation: Ans: D. Tympanic membrane(Ref Miller's 7/e p1550)Though pulmonary aery is the gold standard site for core temperature measurement, esophagus has similar reliability and is the most commonly used site in the anesthetic practice fior temperature monitoring.Core-Temperature Monitoring:Sites for Core Temperature MeasurementGold standard site for core temperature measurementPulmonary aeryMost accurate for brain temperatureTympanic membraneBest for brain temperatureNasopharynxBest site & most commonly used for core body temperatureLower end of esophagus" "This is a real-world medical entrance exam question, please give the true answer based on the question and selection. Topic name: None, Subject name: Pediatrics","All of the following are characteristic features of Tricuspid Atresia except – Select and explain. A. Left Axis deviation, B. Right ventricular hypoplasia, C. Pulmonary vascularity is diminished, D. Splitting of S2",D: Splitting of S2,"Answer: D: Splitting of S2 Explanation: Atresia of the tricuspid valve results in the absence of a communication between the right atrium and right ventricle therefore the right ventricle is underdeveloped the inflow portion being absent. The only exit for systemic venous blood coming to the right atrium is by way ofAtrial Septal defect. Through this the blood goes to left atrium from where it enters left ventricle. A ventricular septal defect provides communication between the left ventricle and the outflow portion of the right ventricle. The left ventricle therefore maintain both the systemic as well as the pulmonary circulation thus there is hypertrophy of the left ventricle which is reflected by left axis deviation in ECG. The pulmonary blood flow is dependent on the size of the ventricular defect, the smaller the VSD, the lesser the pulmonary blood flow. 90% patients of Triscuspid Atresia have diminished pulmonary blood flow. Auscultatory finding in case of Tricuspid Atresia -' S 1- Normal                o SZ - Single           o Murmur grade III to grade 1111 / VI" "This is a real-world medical entrance exam question, please give the true answer based on the question and selection. Topic name: None, Subject name: Physiology","Fick's law deals with Select and explain. A. Passive diffusion of molecules along the concentration gradient, B. Active diffusion of molecules along the concentration gradient, C. Both passive and active diffusion of molecules along the concentration gradient, D. Not concerned with the diffusion of molecules",A: Passive diffusion of molecules along the concentration gradient,"Answer: A: Passive diffusion of molecules along the concentration gradient Explanation: Ans.A. Passive diffusion of molecules along the concentration gradient.Fick's law of diffusion: The magnitude of the diffusing tendency is directly propoional to the cross-sectional area across which diffusion is taking place and the concentration gradient or chemical gradient, which is the difference in the concentration of the diffusing substance divided by the thickness of the membrane. The time required for equilibrium by diffusion is propoional to the square of diffusion distance.Diffusion is the process by which a gas or substance in solution expands, because of the motion of its paicles, to fill all of the available volumes." "This is a real-world medical entrance exam question, please give the true answer based on the question and selection. Topic name: None, Subject name: Pathology","Which of the following is most common location of intracranial neurocysticercoses: Select and explain. A. Brain parenchyma, B. Spinal cord., C. Basal cisternae., D. Medulla oblongata.",A: Brain parenchyma,"Answer: A: Brain parenchyma Explanation: Cysticercosis is a parasitic infection that results from ingestion of eggs from the adult tapeworm. When cysticercosis involves the central nervous system, it is called neurocysticercosis. The cyst degenerates, leaks fluid from the larval cyst into the brain parenchyma, generating a strong immune response, characterized by enhancement on CT and MRI Downs Syndrome It is first seen either with seizures (70-90%) or headache." "This is a real-world medical entrance exam question, please give the true answer based on the question and selection. Topic name: None, Subject name: Radiology","Which of the following statements about developing a dental radiograph is false? Select and explain. A. It involves chemically precipitating metallic silver, B. If it is too long, it results in an over dark radiograph, C. If high temperatures are used, it results in a fogged, over dark radiograph, D. It typically involves a developer containing sodium thiosulphate",D: It typically involves a developer containing sodium thiosulphate,"Answer: D: It typically involves a developer containing sodium thiosulphate Explanation: None" "This is a real-world medical entrance exam question, please give the true answer based on the question and selection. Topic name: None, Subject name: Gynaecology & Obstetrics","A 16 years old girl walks into your clinics and asks for Ca cervix vaccination.Which of the following should be administered? Select and explain. A. Gardasil 9, B. Biovac, C. Polymer 7, D. Cohen 5",A: Gardasil 9,"Answer: A: Gardasil 9 Explanation: Ans. A.Gardasil 9Gardasil 9 is an HPV vaccine approved by the U.S. Food and Drug Administration and can be used for both girls and boys.This vaccine can prevent most cases of cervical cancer if given before a girl or woman is exposed to the virus.HPV Vaccines* Cervarix - Protects against HPV-16, 18.* Gardasil 4 Protects against HPV - 6, 11, 16, 18.* Gardasil -9 - Protects against HPV - 6, 11, 16, 18, 31, 33, 45, 52, 58." "This is a real-world medical entrance exam question, please give the true answer based on the question and selection. Topic name: None, Subject name: Dental","Dunlop's beta hypothesis is useful for the treatment of: Select and explain. A. Bruxism., B. Thumb sucking., C. Mouth breathing., D. Tongue thrusting.",B: Thumb sucking.,"Answer: B: Thumb sucking. Explanation: None" "This is a real-world medical entrance exam question, please give the true answer based on the question and selection. Topic name: AIIMS 2019, Subject name: Biochemistry","Iron enters enterocyte by : Select and explain. A. Divalent cation transpoer, B. Ferropoin, C. Hephaestin, D. Ceruloplasmin",A: Divalent cation transpoer,"Answer: A: Divalent cation transpoer Explanation: *.Cytochrome b reductase 1 or Duodenal cytochrome b (Dcytb) is a ferric reductase enzyme conves Fe3+ to Fe2+, and aid the entry of Fe2+ into the mucosal cell *The ferrous iron is then transpoed in the cell by a divalent metal transpoer (DMT-1). *Ferropoin and Hephaestin proteins are at the basolateral membrane and helps in transpo of iron from intestine cell to plasma. Divalent Metal/Cation Transpoer 1(DMT1 or DCT1) /Natural resistance-associated macrophage protein 2(NRAMP 2) *Located on the apical membrane of enterocytes * Transpo of ferrous iron * DMT1 expression is regulated by body iron stores to maintain iron homeostasis. Hephaestin, also known as HEPH * Transmembrane protein, *Homology with ceruloplasmin * Involved in the metabolism and homeostasis of iron * Mainly involved in iron efflux at the basolateral membrane, in association with ferropoin. * The highest expression of hephaestin is found in small intestine." "This is a real-world medical entrance exam question, please give the true answer based on the question and selection. Topic name: None, Subject name: Gynaecology & Obstetrics","Lady wth MS + MR with full term gestation, obstetrician planning to conduct normal delivery, what would be anesthesia of choice? Select and explain. A. Parenteral opioids, B. Spinal anesthesia, C. Inhalational analgesia, D. Neuraxial analgesia",D: Neuraxial analgesia,"Answer: D: Neuraxial analgesia Explanation: Pain relief is important for heart disease patients as pain can cause tachycardia, which in turn can cause cardiac failure. Epidural and spinal techniques are the most effective means of providing pain relief for labor. These are also known as regional techniques because pain relief is limited to a specific anatomical region. These modalities are also known as neuraxial techniques, since both the approaches involve administration of drugs that exert their effects in the axial portion of the CNS" "This is a real-world medical entrance exam question, please give the true answer based on the question and selection. Topic name: None, Subject name: Surgery","The most appropriate method to differentiate between a dentigerous cyst and an ameloblastoma is through: Select and explain. A. Radiographic examination, B. Aspiration cytology, C. Microscopic examination, D. Clinical features",C: Microscopic examination,"Answer: C: Microscopic examination Explanation: None" "This is a real-world medical entrance exam question, please give the true answer based on the question and selection. Topic name: AIIMS 2018, Subject name: Pharmacology","A hypeensive patient was on metoprolol treatment. Verapamil was added to the therapy of this patient. This can result in:- Select and explain. A. Atrial fibrillation, B. Bradycardia with AV block, C. Torsades de pointes, D. Tachycardia",B: Bradycardia with AV block,"Answer: B: Bradycardia with AV block Explanation: Metoprolol (Cardioselective beta blocker) decrease hea rate and AV conduction. Verapamil and Diltiazem: CCB (calcium channel blocker): These also inhibits SA Node and AV Node, hence depress the hea. So, when verapamil (or Diltiazem) is combined with metoprolol, they both depress hea and lead to bradycardia with AV block. Other CCBs include dihydropyridines (like amlodipine etc). These dipines can be used with Metoprolol as they cause reflex tachycardia." "This is a real-world medical entrance exam question, please give the true answer based on the question and selection. Topic name: None, Subject name: Microbiology","Rabies vaccine not to be given in case of? Select and explain. A. Rat bite, B. Cat bite, C. Dog bite, D. Monkey bite",A: Rat bite,"Answer: A: Rat bite Explanation: None" "This is a real-world medical entrance exam question, please give the true answer based on the question and selection. Topic name: None, Subject name: Pathology","Asymmetric widening of the periodontal Ligament around two or more teeth is seen in Select and explain. A. metastatic breast carcinoma, B. osteosarcoma, C. Paget's disease, D. Fibrous dysplasia",B: osteosarcoma,"Answer: B: osteosarcoma Explanation: None" "This is a real-world medical entrance exam question, please give the true answer based on the question and selection. Topic name: None, Subject name: Gynaecology & Obstetrics","Most accurate and safe method to diagnose ble pregnancy at 6weeks- Select and explain. A. Doppler assessment of fetal cardiac activity, B. USG for fetal cardiac activity, C. Urinary Beta-hCG determination, D. Per vaginal examination of uterine size corresponding to 6 weeks gestation",B: USG for fetal cardiac activity,"Answer: B: USG for fetal cardiac activity Explanation: Answer- B. USG for fetal cardiac activityUSG for fetal cardiac activity:At 6 weeks it is routine to detect fetal cardiac activity by ultrasound5 1/2 Yz to 6 weeks is usually a very good time to detect either a fetal pole or even a fetal hea beat by vaginalultrasound." "This is a real-world medical entrance exam question, please give the true answer based on the question and selection. Topic name: None, Subject name: Radiology","Reversal lines are characteristic on the radiograph of: Select and explain. A. Osteoma, B. Osteosarcoma, C. Epidermoid cyst, D. Paget's disease of bone",D: Paget's disease of bone,"Answer: D: Paget's disease of bone Explanation: None" "This is a real-world medical entrance exam question, please give the true answer based on the question and selection. Topic name: None, Subject name: Dental","Function of molybdenum in base metal alloys Select and explain. A. Increases strength, B. Decreases strength, C. Increase CTP., D. Decrease corrosion resistance",A: Increases strength,"Answer: A: Increases strength Explanation: None" "This is a real-world medical entrance exam question, please give the true answer based on the question and selection. Topic name: None, Subject name: Dental","Vaccine for caries is based on which immunoglobulin Select and explain. A. IgG, B. IgA, C. IgE, D. IgM",B: IgA,"Answer: B: IgA Explanation: None" "This is a real-world medical entrance exam question, please give the true answer based on the question and selection. Topic name: None, Subject name: Dental","As we move from anterior to posterior gingival embrasure become Select and explain. A. Smaller, B. Larger, C. Remain same, D. Can be any of the above",A: Smaller,"Answer: A: Smaller Explanation: None" "This is a real-world medical entrance exam question, please give the true answer based on the question and selection. Topic name: None, Subject name: Dental","What % collagen fibers and fibroblasts are present In gingival connective tissue Select and explain. A. 60% & 5%, B. 65% & 5%, C. 60%& 10%, D. 65%& 10%",A: 60% & 5%,"Answer: A: 60% & 5% Explanation: None" "This is a real-world medical entrance exam question, please give the true answer based on the question and selection. Topic name: None, Subject name: Physiology","Immunologicaly active cells are: Select and explain. A. Plasma cells, B. MAST cells, C. Eosinophils, D. R.B.C s",A: Plasma cells,"Answer: A: Plasma cells Explanation: None" "This is a real-world medical entrance exam question, please give the true answer based on the question and selection. Topic name: None, Subject name: Dental","All of the following about use of aerotor hand piece are false except: Select and explain. A. The sound frequency greater than 75 decibel (dB) causes damage to the ear, B. The frequency of rotation of bur is 6000-8000rpm, C. It will harm more in elderly than very young, D. Its use in younger teeth is contraindicated.",A: The sound frequency greater than 75 decibel (dB) causes damage to the ear,"Answer: A: The sound frequency greater than 75 decibel (dB) causes damage to the ear Explanation: None" "This is a real-world medical entrance exam question, please give the true answer based on the question and selection. Topic name: None, Subject name: Dental","Which of the following kit has been designed & preferred specially for removal of fibre reinforced composite post? Select and explain. A. Ruddle post removal system, B. Gonion kit, C. Gyro kit, D. JS extractor kit",C: Gyro kit,"Answer: C: Gyro kit Explanation: None" "This is a real-world medical entrance exam question, please give the true answer based on the question and selection. Topic name: None, Subject name: Dental","Distance of the blade from the shank to have antirotational stability of the instrument? Select and explain. A. 2 mm, B. 3 mm, C. 4 mm, D. 5 mm",A: 2 mm,"Answer: A: 2 mm Explanation: None" "This is a real-world medical entrance exam question, please give the true answer based on the question and selection. Topic name: None, Subject name: Dental","The procedure preferred for periodontal surgery in mandibular anterior teeth is Select and explain. A. Gingivectomy, B. Periodontal flap, C. Subgingival curettage, D. Grafting",B: Periodontal flap,"Answer: B: Periodontal flap Explanation: None" "This is a real-world medical entrance exam question, please give the true answer based on the question and selection. Topic name: None, Subject name: Social & Preventive Medicine","A researcher selected all possible samples from a population and plotted their means on a line graph. This distribution is called as: Select and explain. A. Sample distribution, B. Sampling distribution, C. Population distribution, D. Parametric distribution",B: Sampling distribution,"Answer: B: Sampling distribution Explanation: Ans: B. Sampling distribution(Ref Research Methods For Communication Science James H. Watt;Sampling distribution / finite-sample distribution:Probability distribution of a given statistic based on a random sample.They allow analytical considerations to be based on the sampling distribution of a statistic, rather than on the joint probability distribution of all the individual sample values." "This is a real-world medical entrance exam question, please give the true answer based on the question and selection. Topic name: None, Subject name: Anatomy","Motor division of trigeminal division comes out via: Select and explain. A. Foramen spinosum, B. Foramen magnum, C. Foramen ovale, D. Foramen lacerum",C: Foramen ovale,"Answer: C: Foramen ovale Explanation: None" "This is a real-world medical entrance exam question, please give the true answer based on the question and selection. Topic name: None, Subject name: Medicine","Helmet cells are characteristic of anemia of Select and explain. A. Hemolytic uremic syndrome, B. Polysplenia, C. Spherocytosis, D. Acanthocytosis",A: Hemolytic uremic syndrome,"Answer: A: Hemolytic uremic syndrome Explanation: None" "This is a real-world medical entrance exam question, please give the true answer based on the question and selection. Topic name: None, Subject name: Pediatrics","In chronic renal failure : a) Urine output is more than 3 litres per dayb) Urine concentration is decreasedc) Sodium conservation is poord) Polycythemia is present Select and explain. A. a, B. c, C. bc, D. ad",C: bc,"Answer: C: bc Explanation: None" "This is a real-world medical entrance exam question, please give the true answer based on the question and selection. Topic name: None, Subject name: Anaesthesia","All of the following are induction agents except:September 2007 Select and explain. A. Thiormtal, B. Halothane, C. Nitrous oxide, D. Propofol",C: Nitrous oxide,"Answer: C: Nitrous oxide Explanation: Ans. C: Nitrous oxideGeneral anaesthesia can be induced by intravenous (IV) injection, or breathing a volatile anaesthetic through a facemask (inhalational induction).Onset of anaesthesia is faster with IV injection than with inhalation, taking about 10-20 seconds to induce total unconsciousness.This has the advantage of avoiding the excitatory phase of anaesthesia, and thus reduces complications related to induction of anaesthesia.An inhalational induction may be chosen by the anesthesiologist where IV access is difficult to obtain, where difficulty maintaining the airway is anticipated, or due to patient preference (e.g. children).Commonly used IV induction agents include propofol, sodium thiopental, etomidate, and ketamine.The most commonly-used agent for inhalational induction is sevoflurane because it causes less irritation than other inhaled gasesIn order to prolong anaesthesia for the required duration (usually the duration of surgery), anaesthesia must be maintained. Usually this is achieved by allowing the patient to breathe a carefully controlled mixture of oxygen, nitrous oxide, and a volatile anaesthetic agent or by having a carefully controlled infusion of medication, usually propofol, through an IV.Induction Characteristics & Dosage Requirements for Currently AvailableSedative & Hypnotic DrugsDrug NameInduction Dose (mg/kg)Onset (sec)Duration (min)Thiopental3-6<305-10Methohexital1-3<305-10Propofol1.5-2.515-455-10Midazolam0.2-0,430-9010-30Diazepam0.3-0.645-9015-30Lorazepam0.03-0.0660-12060-120Etomidate0.2-0.33-123-12Ketamine1-210-2010-20" "This is a real-world medical entrance exam question, please give the true answer based on the question and selection. Topic name: None, Subject name: Dental","Most common organism responsible for acute apical abscess: Select and explain. A. Trepenoma denticola, B. Dilaster invisus, C. E. faecalis, D. All of the above",A: Trepenoma denticola,"Answer: A: Trepenoma denticola Explanation: The bacteria most often recovered from infected vital pulps are streptococci and staphylococci, but many other microorganisms including anaerobes have also been isolated. With the introduction of improved methods of molecular identification of pathogens, many new organisms have been identified by researchers.  Species that have been found significantly include Porphyromonas gingivalis, Porphyromonas endodontalis, Fusobacterium nucleatum and others.  Polymerase chain reaction (PCR) methods show higher prevalence of Treponema denticola, a recognized periodontal pathogen, in association with Tannerella forsythia and P. gingivalis." "This is a real-world medical entrance exam question, please give the true answer based on the question and selection. Topic name: None, Subject name: Dental","Indirect retention is designed to Select and explain. A. Stabilize tooth borne RPD, B. Engage an undercut area of the abutment tooth, C. Resist tissue ward movement of an extension base partial denture, D. Resist dislodgement of an extension base partial denture in occlusal direction",D: Resist dislodgement of an extension base partial denture in occlusal direction,"Answer: D: Resist dislodgement of an extension base partial denture in occlusal direction Explanation: None" "This is a real-world medical entrance exam question, please give the true answer based on the question and selection. Topic name: None, Subject name: Surgery","One unit of PRBC raises HCT by: (In exam instead of HCT, they wrote HCV, which we assume to be a spelling error): Select and explain. A. 1%, B. 2%, C. 3-5%, D. 5-8%",C: 3-5%,"Answer: C: 3-5% Explanation: None" "This is a real-world medical entrance exam question, please give the true answer based on the question and selection. Topic name: None, Subject name: Forensic Medicine","All are true for the following about methanol poisoning EXCEPT: Select and explain. A. Fomepizole is a competitive inhibitor of aldehyde dehydrogenase, B. Minimum lethal dose of methanol is 1.25 ml/kg body weight, C. Formic acid is mainly responsible for toxicity, D. Methanol causes snow field vision",A: Fomepizole is a competitive inhibitor of aldehyde dehydrogenase,"Answer: A: Fomepizole is a competitive inhibitor of aldehyde dehydrogenase Explanation: Answer- a-Fomepizole is a competitive inhibitor of aldehyde dehydrogenaseFomepizole:Fomepizole (4-methylpyrazole) is a specifc inhibitor of alcohol dehydrogenase- retards methanol metabolismLoading dose is 15 mglkg IV followed by l0 mg/kg every 12 hours till serum methanol falls below 20 g/dl, has been found effective and safe.Minimum lethal dose of methanol is 1.25mVkg body weight: Minimum lethal dose of methanol is 0.3-1.0 gm/kgFormic acid is malnly rcsponsible for toxicity:The methyl alcohol poisoning there is CNS depression, cardiac depression and optic nerve atrophy. These effects are produced due to Formaldehyde and formic acid.The specifc toxicity of formic acid is retinal damage." "This is a real-world medical entrance exam question, please give the true answer based on the question and selection. Topic name: None, Subject name: Dental","All factors governing dentine permeability EXCEPT Select and explain. A. Smear Layer, B. Fluid Convection, C. Length of dentinal tubules, D. Diffusion coefficient",B: Fluid Convection,"Answer: B: Fluid Convection Explanation: None" "This is a real-world medical entrance exam question, please give the true answer based on the question and selection. Topic name: None, Subject name: Pediatrics","Injection Glucagon is effective for management of persistent hypoglycemia in all, except – Select and explain. A. Large for date baby, B. Galactosemia, C. Infant of diabetic mother, D. Nesidioblastosis",B: Galactosemia,"Answer: B: Galactosemia Explanation: Glucagon is used to t/t hypoglycemia caused by hyperinsulinism. Glucagon mainly acts by increasing hepatic Glycogenolysis and Gluconeogenesis Hyperinsulinism is a feature of all the four conditions mentioned in the questions so Glucagon should have been effective in all these conditions, but it is not so. Glucagon does not have any effect on hypoglycemia in Galactosemic patients. In Galactosemia liver is the first organ to be affected so there is no question of increased glycogenolysis and gluconeogenesis from the already damaged liver therefore Glucagon will be ineffective." "This is a real-world medical entrance exam question, please give the true answer based on the question and selection. Topic name: None, Subject name: Dental","Which of the following is not characteristic feature of Binder syndrome? Select and explain. A. Acute naso-labial angle, B. Flat nasal bridge, C. Class III malocclusion, D. Class II malocclusion",D: Class II malocclusion,"Answer: D: Class II malocclusion Explanation: None" "This is a real-world medical entrance exam question, please give the true answer based on the question and selection. Topic name: AIIMS 2018, Subject name: Biochemistry","Which of the following vitamin increases the absorption of iron? Select and explain. A. Vitamin A, B. Vitamin C, C. Thiamin, D. Riboflavin",B: Vitamin C,"Answer: B: Vitamin C Explanation: iron absorption - Increase iron absorption Decrease iron absorption Vitamin C Cysteine In iron deficiency anemia iron absorption increase by 2-10 times Small peptides and amino acid Phytate (found in cereals) oxalate (found in leafy vegetable) diet with high phosphate content malabsorption syndromes such as steatorrhea In patients with paial or total surgical removal of stomach and/or intestine Vitamin C and Cysteine conve ferric iron to ferrous iron. Ferrous iron is soluble and easily absorbed." "This is a real-world medical entrance exam question, please give the true answer based on the question and selection. Topic name: None, Subject name: Pharmacology","Which of the following injection is available for subcutaneous administration? Select and explain. A. Albuterol, B. Terbutaline, C. Metaproteronol, D. Pirbuterol",B: Terbutaline,"Answer: B: Terbutaline Explanation: Ans. b. Terbutaline (Ref KDT 7/e p133, 223, 6/e p127, 323; Katzung 11/e p344, 227)Terbutaline can be given by subcutaneous route.Terbutaline is adrenergic agonist. Its sabcutaneous injection is used in patients with severe exacerbations of asthma." "This is a real-world medical entrance exam question, please give the true answer based on the question and selection. Topic name: None, Subject name: Surgery","Gillis approach is used in Select and explain. A. Open reduction of zygomatic fracture, B. Mandible, C. Closed reduction of zygomatic fracture, D. None of the above",C: Closed reduction of zygomatic fracture,"Answer: C: Closed reduction of zygomatic fracture Explanation: None" "This is a real-world medical entrance exam question, please give the true answer based on the question and selection. Topic name: None, Subject name: Dental","All of the following are resorbable Guided Tissue Regeneration (GTR) except? Select and explain. A. Poly lactic acid, B. Poly galactic acid, C. Polylactic acid, polyglycolic acid, and trimethylene carbonate, D. PTFE",D: PTFE,"Answer: D: PTFE Explanation: None" "This is a real-world medical entrance exam question, please give the true answer based on the question and selection. Topic name: AIIMS 2017, Subject name: ENT","Which of the following sinus grows till early adulthood:- Select and explain. A. Maxillary, B. Ethmoidal, C. Frontal, D. Sphenoid",C: Frontal,"Answer: C: Frontal Explanation: Development of paranasal sinuses Sinus Present at bih 1st X-Ray appearance, Adult size Maxillary Yes 4-5 months after bih 15-18 yrs. Ethmoid Yes 1yr 12yr Sphenoid Yes 6-7yr 15yr Frontal No 4-5yr 18yr" "This is a real-world medical entrance exam question, please give the true answer based on the question and selection. Topic name: None, Subject name: Dental","Mode:is defined as? Select and explain. A. Average of the values, B. Relating to a value or quantity lying at the midpoint of a data after arranging in ascending or descending order, C. Adding up the values and then dividing by the number of values, D. The number which appears most often in a set of numbers",D: The number which appears most often in a set of numbers,"Answer: D: The number which appears most often in a set of numbers Explanation: None" "This is a real-world medical entrance exam question, please give the true answer based on the question and selection. Topic name: AIIMS 2017, Subject name: Microbiology","Given below is the diagrammatic representation of a bacterium showing its flagella. Which of the following most closely resembles the organism depicted in the diagram? Select and explain. A. Leptospria, B. H. Pylori, C. Vibrio cholera, D. Salmonella typhi",A: Leptospria,"Answer: A: Leptospria Explanation: Endoflagella is a feature of spirochetes which arises from periplasmic space. They have rotatory type of motility. Treponema Barrelia Leptospira Size 6-14 um x 0.2 um 10-30 um x 0.2-0.5um 6-20 um x 0.1 um Spirals (In number) 6-12 3-10 Numerous and tightly coiled with hooked ends Wavelength 1 um 3 um 0.5 um Amplitude of spiral 1-1.5 um Up to 2 um 0.1 um Endoflagella at each pole 3-4 7-11 1 Staining Do not take up ordinary stains, can be stained only bu silver impregnation stains, except Bornelia which is poorly Gram-stained." "This is a real-world medical entrance exam question, please give the true answer based on the question and selection. Topic name: AIIMS 2017, Subject name: Anaesthesia","Time required for pre-oxygenation before tracheal intubation Select and explain. A. 3 min, B. 5 min, C. 2 min, D. 1 min",A: 3 min,"Answer: A: 3 min Explanation: Preoxygenation before intubation : Process of replacing nitrogen in the lungs with oxygen. Preoxygenation with tidal volume breathing of 3 mins is required before tracheal intubation. This delays the onset of critical hypoxia during the period of apnea by 5-8 minutes." "This is a real-world medical entrance exam question, please give the true answer based on the question and selection. Topic name: None, Subject name: Dental","With aging changes in periodontal ligament are Select and explain. A. Decrease in number of cells and increase in collagen fibers, B. Increase in number of cells and decrease in collagen fibers, C. Increase in number of elastic fibers, D. Hyalinisation changes",C: Increase in number of elastic fibers,"Answer: C: Increase in number of elastic fibers Explanation: None" "This is a real-world medical entrance exam question, please give the true answer based on the question and selection. Topic name: None, Subject name: Microbiology","""Grape bunch"" shaped colonies are seen in: Select and explain. A. Streptococcus, B. Staphylococcus, C. E.coli, D. Gonococci",B: Staphylococcus,"Answer: B: Staphylococcus Explanation: None" "This is a real-world medical entrance exam question, please give the true answer based on the question and selection. Topic name: None, Subject name: Anatomy","House Brackman grading scale used for evaluation of which nerve? Select and explain. A. Facial nerve, B. Hypoglossal, C. Vagus, D. Trigeminal",A: Facial nerve,"Answer: A: Facial nerve Explanation: None" "This is a real-world medical entrance exam question, please give the true answer based on the question and selection. Topic name: None, Subject name: Dental","To prevent ventilator associated pneumonia, the most effective and evidence based results are seen with which of the following for critically ill patients: Select and explain. A. Betadine mouthwash, B. Oral hygiene procedures plus chlorhexidine, C. Manual brushing, D. Powered brushing",B: Oral hygiene procedures plus chlorhexidine,"Answer: B: Oral hygiene procedures plus chlorhexidine Explanation: None" "This is a real-world medical entrance exam question, please give the true answer based on the question and selection. Topic name: None, Subject name: Radiology","The darkening of radiograph X-ray depends on all except: Select and explain. A. Thickness of object, B. Quality and quantity of X-rays, C. Angulation cone, D. Velocity of electron emitted from cathode",C: Angulation cone,"Answer: C: Angulation cone Explanation: None" "This is a real-world medical entrance exam question, please give the true answer based on the question and selection. Topic name: None, Subject name: Dental","To remove non working side interference, reduce: Select and explain. A. Maxillary supporting cusps., B. Mandibular supporting cusps., C. Supporting cusps of both the arches., D. Either of the arch's supporting cusps should be reduced.",D: Either of the arch's supporting cusps should be reduced.,"Answer: D: Either of the arch's supporting cusps should be reduced. Explanation: None" "This is a real-world medical entrance exam question, please give the true answer based on the question and selection. Topic name: None, Subject name: Pathology","Tumor represented T3N2M0 is classified as : Select and explain. A. Stage I, B. Stage II, C. Stage III, D. Stage IV",D: Stage IV,"Answer: D: Stage IV Explanation: None" "This is a real-world medical entrance exam question, please give the true answer based on the question and selection. Topic name: None, Subject name: Forensic Medicine","A bomb blast took place in Delhi following which 2 persons died. All of the following are true about their injuries except: Select and explain. A. Injuries occurred due to burns air blast, B. Force of explosion decreases rapidly, C. Force of explosion is directional, D. Bruise, laceration, fractures are triad of main explosive injuries seen","D: Bruise, laceration, fractures are triad of main explosive injuries seen","Answer: D: Bruise, laceration, fractures are triad of main explosive injuries seen Explanation: Ans: D. Bruise, laceration, fractures are the triad of main explosive injuries seenMarshall's triad is diagnostic of explosive injury.Marshall's triad -Bruises, abrasions & puncture lacerations.Does not include fracture." "This is a real-world medical entrance exam question, please give the true answer based on the question and selection. Topic name: None, Subject name: Social & Preventive Medicine","Broken ampula is thrown into which coloured bin? Select and explain. A. Red, B. Yellow, C. Blue, D. Black",C: Blue,"Answer: C: Blue Explanation: Ans. C. BlueCOLOR CODING & TYPES OF CONTAINER: Colour codingType of ContainerWaste CategoryTreatment optionsYellowPlastic BagsHuman and animal wastes, Microbial and Biological wastes and soiled wastes(Cat 1,2,3 and 6)Incineration/ Deep BurialRedDisinfected container/ Plastic bagsMicrobiological and Biological wastes, Soiled wastes, Solid wastes(Cat 3,6,7)Autoclave/ Microwave/ Chemical Treatment)Blue/ White/ TransparentPlastic bag, Puncture proof containerWaste sharps and solid waste( Cat 4 &7)Autoclave/ Microwave/ Chemical Treatment Destruction and ShreddingBlackPlastic bagDiscarded medicines, Cytotoxic drugs, Incineration ash and chemical waste(Cat 5,9 & 10)Disposal in secured land fillsGreenPlastic ContainerGeneral waste such as office waste, food waste & garden wasteDisposed in secured landfills" "This is a real-world medical entrance exam question, please give the true answer based on the question and selection. Topic name: None, Subject name: Microbiology","Phage mediated change in C. diphtheria is due to which of the following? Select and explain. A. Conjugation, B. Transformation, C. Transduction, D. None of the above",C: Transduction,"Answer: C: Transduction Explanation: Ans: C. Transduction(Ananthanarayan 10/e p59, 8/e p65-67; Jawetz 27/e p112).Phage mediated change in C. diphtheria is due to Transduction.In transduction, donor DNA is carried by a phage coat and is transferred into the recipient by the mechanism used for phage infection.Methods of transferMechanismNature of DNA transferredConjugationTransfer of DNA form one bacterium toChromosomal or plasmid DNAdegTransductionTransfer of DNA form one bacterium toAny gene in generalized transduction, only selected genes in specialized transductiondegTransformationTransfer of DNA from one bacterium toAny genedeg" "This is a real-world medical entrance exam question, please give the true answer based on the question and selection. Topic name: AIIMS 2018, Subject name: Microbiology","What is not true regarding Zika Virus: Select and explain. A. Belong to family flaviviridae, B. Transmission happens by Mosquitoes, C. Not transmitted from mother to newborn, D. Possibly can cause microcephaly",C: Not transmitted from mother to newborn,"Answer: C: Not transmitted from mother to newborn Explanation: Zika Virus belongs to family flaviviridae( which comes under Arbovirus). It is transmitted through mosquitoes (Aedes aegypticus) or sexual transmission .It also shows veical transmission i.e. from mother to child. If pregnant female gets infected with zika virus it may cause Microcephaly and GBS (Guillian Barre syndrome) in new born." "This is a real-world medical entrance exam question, please give the true answer based on the question and selection. Topic name: None, Subject name: Gynaecology & Obstetrics","A 23-year-old lady taking antiepileptics for a seizure disorder gets married. When should folic acid supplementation advised to the patient? Select and explain. A. Any time as soon as she presents to the clinic irrespective of pregnancy, B. Three months before becoming pregnant, C. 1st trimester, D. As soon as pregnancy is confirmed",A: Any time as soon as she presents to the clinic irrespective of pregnancy,"Answer: A: Any time as soon as she presents to the clinic irrespective of pregnancy Explanation: Ans: A. Any time as soon as she presents to the clinic irrespective of pregnancyIf a pregnancy is planned in high-risk women (previously affected child with neural tube defects), supplementation should be staed with 4 mg (= 4000 microgram) of folic acid daily, beginning 1 month before the time of the planned conception.Recommendations:By U.S. Public Health Service.Folic acid 0.4 mg daily - For all women of childbearing age & ones capable of becoming pregnant.Folic acid 4 mg (= 4000 microgram) daily - For planned pregnancy in high-risk women (previously affected child) - Beginning 1 month before time of planned conception." "This is a real-world medical entrance exam question, please give the true answer based on the question and selection. Topic name: None, Subject name: Medicine","Infective endocarditis is most commonly seen in: Select and explain. A. ASD, B. VSD, C. PDA, D. Pulmonary stenosis",B: VSD,"Answer: B: VSD Explanation: Infective endocarditis typically occurs at sites of pre-existing endocardial damage, but infection with particularly virulent or aggressive organisms such as Staphylococcus aureus can cause endocarditis in a previously normal heart. Staphylococcal endocarditis of the tricuspid valve is a common complication of intravenous drug use. Many acquired and congenital cardiac lesions are vulnerable, particularly areas of endocardial damage caused by a high-pressure jet of blood, such as ventricular septal defect, mitral regurgitation and aortic regurgitation, many of which are haemodynamically insignificant. In contrast, the risk of endocarditis at the site of haemodynamically important low-pressure lesions, such as a large atrial septal defect, is minimal. Ref: Davidson 23rd ed page no 527" "This is a real-world medical entrance exam question, please give the true answer based on the question and selection. Topic name: None, Subject name: Pharmacology","Clavulanic acid added to ampicillin causes: Select and explain. A. Increase effectivity against gram-negative organisms, B. Inhibition of β-lactamases, C. Increases the effectiveness against delta lactamase producing organism, D. Increase effectivity against gram-positive organisms",B: Inhibition of β-lactamases,"Answer: B: Inhibition of β-lactamases Explanation: None" "This is a real-world medical entrance exam question, please give the true answer based on the question and selection. Topic name: None, Subject name: Surgery","Epidural hematoma is caused by Select and explain. A. Middle meningeal artery, B. Vertebral artery, C. Anterior cerebral artery, D. Posterior cerebellar artery",A: Middle meningeal artery,"Answer: A: Middle meningeal artery Explanation: None" "This is a real-world medical entrance exam question, please give the true answer based on the question and selection. Topic name: None, Subject name: Surgery","With respect to peripheral vascular disease, all of the following statements are true except: Select and explain. A. Ankle brachial index < 0.5 indicates critical limb ischemia, B. The ankle-brachial index is different at rest than that during exercise, C. L-arginine is useful for providing endothelium-independent vasorelaxation, D. Smoking has a greater association with peripheral vascular disease than coronary vascular disease",D: Smoking has a greater association with peripheral vascular disease than coronary vascular disease,"Answer: D: Smoking has a greater association with peripheral vascular disease than coronary vascular disease Explanation: Ans. d. Smoking has a greater association with peripheral vascular disease than coronary vascular disease > c. L-arginine is useful for providing endothelium-independent vasorelaxationSainston I9/e p/ 730; Schwaz 9/e p751, Bailey 26th/S75, 25th/900-902)Smoking has greater association with coronary aery disease than peripheral vascular disease. L-Arginine is a precursor of Nitric oxide so helps in vasodilation due to synthesis of NO by endothelium. Hence it is endothelium-dependent." "This is a real-world medical entrance exam question, please give the true answer based on the question and selection. Topic name: None, Subject name: Gynaecology & Obstetrics","Pregnant women with following is called systemic hypeension: Select and explain. A. Hypeension diagnosed at 10 weeks of gestation, B. Diabetic retinopathy, C. Diabetic nephropathy, D. Episode of seizure",A: Hypeension diagnosed at 10 weeks of gestation,"Answer: A: Hypeension diagnosed at 10 weeks of gestation Explanation: Ans. a. Hypeension diagnosed at 10 weeks of gestationChronic underlying hypeension or systemic hypeension is diagnosed in women with documented blood pressures > 140/90 mm Hg before pregnancy or before 20 weeks' gestation, or both." "This is a real-world medical entrance exam question, please give the true answer based on the question and selection. Topic name: None, Subject name: Pharmacology","The highest risk associated with the use of oral contraceptives is in: Select and explain. A. Hepatic necrosis, B. Permanent sterility, C. Thromboembolic disorders, D. Cancer of breast",C: Thromboembolic disorders,"Answer: C: Thromboembolic disorders Explanation: Serious Complications Leg vein thrombosis and pulmonary embolism: The older preparations increased the incidence of venous thromboembolism, but this is found to be less marked (only 2-3 patients per 1000 women years) with the newer reduced steroid content pills. However, even low-dose pills pose significant risk in women > 35 years of age, diabetics, hypertensives and in those who smoke. The excess risk is due to the estrogen component of the OC, and normalizes shortly after stopping the OC. Rise in BP: Occurred in 5-10% women taking the earlier pills. The again is less frequent and smaller in magnitude with the low-dose pills of today. If the BP rises, best is to stop OCs: the BP normalizes in the next 3-6 months. Both the estrogen and progestin components are responsible for this effect, probably by increasing plasma angiotensinogen level and renin activity which have been found to be raised during OC use. Aldosterone secretion is enhanced resulting in salt and water retention. Reference: Essentials of Medical Pharmacology Eighth Edition KD TRIPATHI page no 351" "This is a real-world medical entrance exam question, please give the true answer based on the question and selection. Topic name: None, Subject name: Pathology","A neuralgia with trigger zones in the oropharynx and pain in the ear pharynx, nasopharynx, tonsils and posterior tongue is most likely: Select and explain. A. Trigeminal neuralgia, B. Bell's patsy, C. GlossopharyngeaL neuralgia, D. Sphenopalatine neuralgia",C: GlossopharyngeaL neuralgia,"Answer: C: GlossopharyngeaL neuralgia Explanation: None" "This is a real-world medical entrance exam question, please give the true answer based on the question and selection. Topic name: None, Subject name: Gynaecology & Obstetrics","Ergometrine is contraindicated in: Select and explain. A. Eclampsia, B. Abortion, C. Induction of labour, D. Post partum hemorrhage",A: Eclampsia,"Answer: A: Eclampsia Explanation: Contraindications for the use of Ergometrine are:" "This is a real-world medical entrance exam question, please give the true answer based on the question and selection. Topic name: AIIMS 2019, Subject name: Anaesthesia","Which is not an obvious advantage of high pressure oxygen through cannula: Select and explain. A. Bypassing nasopharyngeal dead space, B. Decreases need for intubation, C. PEEP, D. Hot and Humidification of air",A: Bypassing nasopharyngeal dead space,"Answer: A: Bypassing nasopharyngeal dead space Explanation: High pressure oxygen through cannula: Oxygen supply system capable of delivering up to 100% humidified and heated oxygen at a flowrate of up to 60 liters per minute. This device increases the FiO2 (21-100%) and reduces the need for intubation. It also creates some amount of PEEP: 3-5mmhg" "This is a real-world medical entrance exam question, please give the true answer based on the question and selection. Topic name: None, Subject name: Dental","Silicate cement is indicated in: Select and explain. A. Mouth breathers, B. Patients with high caries index, C. In the restoration of posterior tooth, D. None of the above",B: Patients with high caries index,"Answer: B: Patients with high caries index Explanation: Up to the early 1970s, silicate cement restorations were used for anterior esthetic restorations. This material was used because of its tooth-colored properties in addition to its ability to release significant amounts of fluoride. The anticariogenic potential of silicate cement confirms the ability of F- to inhibit demineralization, that’s why silicate cement has use in patients with high caries index. Reference: PHILLIPS’ SCIENCE OF DENTAL MATERIALS, 12th ed page no 14" "This is a real-world medical entrance exam question, please give the true answer based on the question and selection. Topic name: None, Subject name: Radiology","Effect of radiation is: Select and explain. A. Cumulative, B. Fractionalized, C. Concentrated, D. Dependent on genetic make-up of individuals",A: Cumulative,"Answer: A: Cumulative Explanation: None" "This is a real-world medical entrance exam question, please give the true answer based on the question and selection. Topic name: None, Subject name: Pathology","Ossifying fibroma manifests as: Select and explain. A. Unilocular radiolucency., B. Unilocular radiopoacity., C. Multilocular radiolucency., D. Multilocular radiopoacity.",B: Unilocular radiopoacity.,"Answer: B: Unilocular radiopoacity. Explanation: None" "This is a real-world medical entrance exam question, please give the true answer based on the question and selection. Topic name: None, Subject name: Pediatrics","All of the following groups of newborns are at an increased risk of hypoglycemia except – Select and explain. A. Birth asphyxia, B. Respiratory distress syndrome, C. Maternal diabetes, D. Post term infant",D: Post term infant,"Answer: D: Post term infant Explanation: Neonatal hypoglycemia is, seen in preterm infants (not post term)" "This is a real-world medical entrance exam question, please give the true answer based on the question and selection. Topic name: None, Subject name: Dental","Resorption of cementum as compared to bone occurs: Select and explain. A. Less readily, B. More readily, C. Same, D. Undermining resorption",A: Less readily,"Answer: A: Less readily Explanation: None" "This is a real-world medical entrance exam question, please give the true answer based on the question and selection. Topic name: None, Subject name: Dental","Mandibulectomy cantor Curtis class 2 defect Is Select and explain. A. Midline, B. lateral to canine, C. Alveolus, D. discontinuity defect",B: lateral to canine,"Answer: B: lateral to canine Explanation: None" "This is a real-world medical entrance exam question, please give the true answer based on the question and selection. Topic name: None, Subject name: Surgery","A 40 year woman was brought to the casualty 8 hours after sustaining burns on the abdomen, both the limbs and back. What will be the best formula to calculate amount of fluid to be replenished? Select and explain. A. 2 mL/kg x %TBSA, B. 4 mL/kg x %TBSA, C. 8 mL/kg x %TBSA, D. 4 mL/kg x %TBSA in first 8 hours followed by 2 mL/ kg/hour x %TBSA",A: 2 mL/kg x %TBSA,"Answer: A: 2 mL/kg x %TBSA Explanation: Answer- A. 2 mL/kg x %TBSAFluid resuscitation in burns:According to ATLS 10th edition, 2018, the resuscitation formula used in thermal burns is 2 mL/kg x % TBSA.First 8 hours: 1 mL/kg x % TBSA (lactated Ringer solution)Next 16 hours: 1 mL/kg x % TBSA (lactated Ringer solution)For a patient presenting 8 hours after sustaining thermal burns, the total volume using resuscitation formula should be given in the next 16 hours i.e; 2 mL/kg x % TBSA in the next 16 hours while maintaining target urine output of 0.5 mL/kg/hr for adults.In adults, urine output should be maintained between 30 and 50 cc/hr to minimize potential over-resuscitation.The total percentage of the burnt area is calculated clinically using the rule of nines. Burns - Formulae for Fluid Calculation(Updated according to ATLS 10th edition 2018 guidelines)" "This is a real-world medical entrance exam question, please give the true answer based on the question and selection. Topic name: None, Subject name: Surgery","Most common functional tumors of endocrine pancreas? Select and explain. A. Vipoma, B. Gastrinoma, C. Glucagonoma, D. Somatostatinoma",B: Gastrinoma,"Answer: B: Gastrinoma Explanation: Answer- B. GastrinomaInsulinoma is the most common functional tumour but if that wasnt there in the optios then gastrinoma is the next answer." "This is a real-world medical entrance exam question, please give the true answer based on the question and selection. Topic name: None, Subject name: ENT","The main vessel involved in bleeding from Juvenile nasopharyngeal angiofibroma: Select and explain. A. Facial aery, B. Ascending pharyngeal aery, C. Internal maxillary aery, D. Anterior ethmoidal aery",C: Internal maxillary aery,"Answer: C: Internal maxillary aery Explanation: Ans: C. Internal maxillary aery(Ref Dhingra 7/e p281-282, 5/261-263)Juvenile Nasopharyngeal Angiofibroma:MC benign tumor of nasopharynx (but overall angiofibroma rare).MC site:Posterior pa of nasal cavity close to the margin of sphenopalatine foramen.Seen exclusively in males of 10-20 years.Testosterone dependent tumor - Seen in pre-pubeal to adolescent males.Features:Locally invasive vasoformative tumor consisting of endothelium lined vessels with no muscle coat.Main feeding vessel in most cases = Internal maxillary aery.Internal maxillary aery - External carotid aery branch." "This is a real-world medical entrance exam question, please give the true answer based on the question and selection. Topic name: None, Subject name: Anatomy","Incus and malleus are derived from: Select and explain. A. Dorsal end of meckels cartilage., B. Ventral end of meckels cartilage., C. Inferior end of meckels cartilage., D. Any of the above.",A: Dorsal end of meckels cartilage.,"Answer: A: Dorsal end of meckels cartilage. Explanation: None" "This is a real-world medical entrance exam question, please give the true answer based on the question and selection. Topic name: None, Subject name: Pathology","The most common odontogenic tumour which occurs in relation to an unerupted tooth in the anterior maxilla: Select and explain. A. Odontogenic adenomatoid tumour, B. Odontoma, C. Myxoma, D. Cementifying fibroma",A: Odontogenic adenomatoid tumour,"Answer: A: Odontogenic adenomatoid tumour Explanation: None" "This is a real-world medical entrance exam question, please give the true answer based on the question and selection. Topic name: None, Subject name: Pathology","Brachytherapy is: Select and explain. A. Irradiation of tissues from a distance, B. Irradiation of tissues from a distance of 3 cm, C. Irradiation of tissues by implants within the tissues, D. Irradiation of tissues by radiopharmaceuticals",C: Irradiation of tissues by implants within the tissues,"Answer: C: Irradiation of tissues by implants within the tissues Explanation: None" "This is a real-world medical entrance exam question, please give the true answer based on the question and selection. Topic name: None, Subject name: Anatomy","Golgi apparatus in serous acinar cells is at: Select and explain. A. Supranuclear, B. Apical, C. Adjacent to nucleus, D. Basal",A: Supranuclear,"Answer: A: Supranuclear Explanation: Ans. a. SupranuclearSerous cells have a pyramidal shape.They possess large amounts of rough endoplasmic reticulum (RER) with many ribosome free regions, supranuclear Golgi complex and numerous spherical secretory granules." "This is a real-world medical entrance exam question, please give the true answer based on the question and selection. Topic name: None, Subject name: Dental","An imaginary occlusal curve that contacts the buccal and lingual cusp tips of mandibular buccal teeth is called the Select and explain. A. Catenary curve, B. Curve of spee, C. Monson curve, D. Wilson curve",D: Wilson curve,"Answer: D: Wilson curve Explanation: None" "This is a real-world medical entrance exam question, please give the true answer based on the question and selection. Topic name: None, Subject name: Biochemistry","An infant presented to the OPD with a history of vomiting and malnutrition. On investigation, the Guthrie test was found to be positive.All are true regarding this disease except? Select and explain. A. Due to Phenylalanine hydroxylase enzyme defect, B. White patch of hair due to tryptophan deficiency, C. Phenyl acetate positive in urine, D. Mental retardation is present",B: White patch of hair due to tryptophan deficiency,"Answer: B: White patch of hair due to tryptophan deficiency Explanation: Ans: B. White patch of hair due to tryptophan deficiencyThe deficiency of phenylalanine hydroxylase is the cause of this disease.The genetic mutation may be such that either the enzyme is not synthesized, or a non-functional enzyme is synthesized.The classical Phenylketonuria child is mentally retarded with an IQ of 50.Guthrie test is a rapid screening test.Ferric chloride test: The urine of the patient contains phenyl ketones about 500-3000 mg/day.This could be detected by adding a drop of ferric chloride to the urine.A transient blue-green colour is a positive test." "This is a real-world medical entrance exam question, please give the true answer based on the question and selection. Topic name: None, Subject name: Dental","The porosity formed due to simultaneous nucleation of solid grains and gas bubbles at the first moment that metal freezes at the mold walls is called: Select and explain. A. Pinhole porosity, B. Gas inclusion porosity, C. Micro porosity, D. Subsurface porosity.",D: Subsurface porosity.,"Answer: D: Subsurface porosity. Explanation: None" "This is a real-world medical entrance exam question, please give the true answer based on the question and selection. Topic name: None, Subject name: Pediatrics","A 6 days old neonate weighing 2800 gm (bih weight 3200 gm) was brought with the complaints of fever, poor feeding and poor activity. There was no history of vomiting or diarrhea. Axillary temperature was 39degC with depressed fontenalle, sunken eyes, decreased urine output and decreased skin turgor. Her mother has the history of decreased milk production. What is your diagnosis? Select and explain. A. Neonatal sepsis, B. Galactosemia, C. Fever & dehydration, D. Acute renal failure",A: Neonatal sepsis,"Answer: A: Neonatal sepsis Explanation: Ans: A. Neonatal sepsisInitial Signs & Symptoms of Infection in Newborn InfantsGeneralCardiovascular SystemFever, temperature instabilitydegNot doing well, poor feedingdegEdemadegPallor, mottling, cold clammy skindegHypotension, tachycardiadegBradycardiaGastrointestinal SystemCentral Nervous SystemAbdominal distentiondegVomiting, diarrheadegHepatomegalyIrritability, lethargy, high pitched cryTremors, seizuresdegHyporeflexia, hypotonia, abnormal Moro's reflexdegRespiratory SystemHematological SystemApnea, dyspnea, tachypneadegRetractions, flaring, gruntingdegCyanosisdegPallor, jaundice, splenomegalydegBleedingPetechiae, purpura" "This is a real-world medical entrance exam question, please give the true answer based on the question and selection. Topic name: AIIMS 2019, Subject name: Social & Preventive Medicine","Pneumococcal vaccine PCV 23 polysaccharide has shown the best results in the following people Select and explain. A. Sickle cell anemia, B. Cystic fibrosis, C. Child less than 2 year age, D. Recurrent otitis media and sinusitis",A: Sickle cell anemia,"Answer: A: Sickle cell anemia Explanation: - PCV 23 vaccine is recommended for, - Older people > 65 years age - 2-64 years age if they have, Chronic cardiovascular disease Chronic pulmonary disease Diabetes mellitus Alcoholic Chronic smokers Chronic liver disease / cirrhosis Asplenia like sickle cell disease or splenectomy Immunosuppressed like HIV Organ transplantation Renal disease / transplantation - For children < 2 years age, PCV- 23 is strictly contraindicated." "This is a real-world medical entrance exam question, please give the true answer based on the question and selection. Topic name: None, Subject name: Dental","Light transmitting wedges transmit: Select and explain. A. 70-80% of incident light, B. 80-85% of incident light, C. 85-90% of incident light, D. 90-95% of incident light",D: 90-95% of incident light,"Answer: D: 90-95% of incident light Explanation: Light transmitting wedges As the name indicates, these types of wedges transmit 90 to 95 percent of incident light. They are a type of plastic wedges. Transparent in nature. Designed for use in the cervical area of class II composite resin restoration. Nisha Garg, Amit Garg. Textbook of Operative Dentistry.  Edition 3. Page:209" "This is a real-world medical entrance exam question, please give the true answer based on the question and selection. Topic name: None, Subject name: Pathology","Chronic hyperpLastic pulpitis is Select and explain. A. necrotizing, B. suppurative lesion, C. proliferation of a chronically inflamed pulp, D. also called as phoenix abscess",C: proliferation of a chronically inflamed pulp,"Answer: C: proliferation of a chronically inflamed pulp Explanation: None" "This is a real-world medical entrance exam question, please give the true answer based on the question and selection. Topic name: None, Subject name: Physiology","What is responsible for rhythm generation of inspiration? Select and explain. A. Dorsal nucleus of vagus, B. Pre botzinger complex, C. ventral respiratory neurons, D. Pneumotaxic center",B: Pre botzinger complex,"Answer: B: Pre botzinger complex Explanation: Ans. B. Pre botzinger complexRhythmic respiration is initiated by a small group of synaptically coupled pacemaker cells in the pre-Botzinger complex (pre-BOTC) on either side of the medulla between the nucleus ambiguus and the lateral reticular nucleus.The main components of the respiratory control pattern generator responsible for automatic respiration are located in the medulla.Rhythmic respiration is initiated by a small group of synaptically coupled pacemaker cells in the pre-Botzinger complex (pre-BOTC) on either side of the medulla between the nucleus ambiguus and the lateral reticular nucleus." "This is a real-world medical entrance exam question, please give the true answer based on the question and selection. Topic name: None, Subject name: Radiology","In leukemia patient, post extraction bleeding doesn't stop because of: Select and explain. A. Increased leukocyte count, B. Increased Ca+ level in blood, C. Platelet disorder, D. Deficiency of clotting factors",C: Platelet disorder,"Answer: C: Platelet disorder Explanation: None" "This is a real-world medical entrance exam question, please give the true answer based on the question and selection. Topic name: None, Subject name: Gynaecology & Obstetrics","All of the following constitute the active management of third stage of labour for the prevention of postpaum hemorrhage (PPH) except: Select and explain. A. Direct injection of oxytocin after delivery of anterior shoulder, B. Constant controlled cord traction, C. Early cord clamping and cutting, D. Prophylactic misoprostol",C: Early cord clamping and cutting,"Answer: C: Early cord clamping and cutting Explanation: Answer- C (Early cord clamping and cutting) Current evidence shows that delayed cord clamping is beneficial for the babv.Immediate cord clamping has been shown to increase the incidence of iron deficiency and anemia.For premature and low bih weight babies immediate cord clamping can also increase the risk of intraventricular hemorrhage and late onset sepsis." "This is a real-world medical entrance exam question, please give the true answer based on the question and selection. Topic name: None, Subject name: Dental","Optimum BMP dose for osteogenesis is: Select and explain. A. 50 μg/mL, B. 2.5 μg/mL, C. 100 μg/mL, D. 100 μg/mL",A: 50 μg/mL,"Answer: A: 50 μg/mL Explanation: None" "This is a real-world medical entrance exam question, please give the true answer based on the question and selection. Topic name: None, Subject name: Dental","Anchorage not being applied here: Select and explain. A. Reciprocal, B. Stationary, C. Intramaxillary, D. Intraoral",B: Stationary,"Answer: B: Stationary Explanation: Different types of anchorage Intraoral Anchorage Anchorage units are present within the oral cavity. Anchorage sources can be teeth, palate, etc. Simple Anchorage When application of force tends to change the axial inclination of the tooth or teeth that form the anchorage unit in the plane of space in which the force is being applied. Stationary anchorage Anchor tooth or source does not move against the forces of teeth. In real sense, only the extra-oral source of anchorage like can be from headgears would be anchorage. Anchor tooth being housed in a bioactive environment would show some degree of movement and hence cannot be classified as stationary anchorage in a real sense. Reciprocal Anchorage: Teeth which are pulled against each other to close spaces. Midline diastema in the upper arch with elastic arch. Cross elastics to correct molar cross-bite. Arch expansion using a midline screw. The molar rotator.  Single or Primary Anchorage The tooth to be moved against a tooth which can have greater alveolar support area. Like a molar along with adjacent premolars used to align another molar. Compound Anchorage The use of more teeth with greater anchorage potential to move a tooth or group of teeth with lesser support. Like in the fixed orthodontic appliances retracting incisors using loop mechanics. Reinforced Anchorage Reinforcing of anchorage units by use of more than one type of resistance units. Like headgears along with routine fixed mechano-therapy or (extra-oral anchorage and intra arch compound anchorage) Key Concept:  Since the figure shows the movement of both the units towards each other, this doesn’t fit the definition of stationary anchorage." "This is a real-world medical entrance exam question, please give the true answer based on the question and selection. Topic name: AIIMS 2018, Subject name: Medicine","Which of the following drugs is not used in management of rheumatoid ahritis? Select and explain. A. Etanercept, B. Leftunomide, C. Febuxostat, D. Methotrexate",C: Febuxostat,"Answer: C: Febuxostat Explanation: Febuxostat- hypouricemic agent It can be used in patient of chronic kidney disease where probenecid cannot be used Probenecid used in under excretion in chronic gout. Chronic Gout: under excreter (Probenecid/Febuxostat)" "This is a real-world medical entrance exam question, please give the true answer based on the question and selection. Topic name: None, Subject name: Pathology","Biopsy of a clinically suspicious lesion is negative. The most appropriate treatment is Select and explain. A. Tell patient no malignancy, B. Repeat the biopsy, C. Observe the patient for twelve months, D. Observe the patient for three months",B: Repeat the biopsy,"Answer: B: Repeat the biopsy Explanation: None" "This is a real-world medical entrance exam question, please give the true answer based on the question and selection. Topic name: AIIMS 2019, Subject name: Gynaecology & Obstetrics","A 36 week primigravida was admitted in view of a single seizure episode. On examination her BP is 170/100, PR is 90/min, fetal hea rate is present. Immediate next step in management is? Select and explain. A. Inj. mgso4, B. Inj. Calcium gluconate, C. Inj Phenetoin, D. MRI brain",A: Inj. mgso4,"Answer: A: Inj. mgso4 Explanation: This patient appears to have Eclampsia as her BP is raised, immediate management of such seizure episode is inj MgSO4, which is the drug of choice for prevention and management of eclampsia. Management of eclampsia involves Inj MgSO4, IV labetalol (drug of choice for control of BP in pre eclampsia), definitive management is termination of pregnancy." "This is a real-world medical entrance exam question, please give the true answer based on the question and selection. Topic name: None, Subject name: Pathology","For primary herpes simplex, the diagnosis is made by which gene Select and explain. A. Culture with giernsa stain, B. Culture with wright strain, C. Routine cytology, D. Flourescent stain for cytology",D: Flourescent stain for cytology,"Answer: D: Flourescent stain for cytology Explanation: None" "This is a real-world medical entrance exam question, please give the true answer based on the question and selection. Topic name: None, Subject name: Social & Preventive Medicine","In a village of 20,000 population 456 bihs occurred in april, 56 showed no signs of life at bih, 56 died before 28 days of life, 34 died between 28 days- 1 year, 500 was total number of deaths in that year. Infant moality rate? Select and explain. A. 197.4, B. 320, C. 225, D. 125",C: 225,"Answer: C: 225 Explanation: Ans.C.225IMR = Number of deaths of children less than 1 year of age in a year X 1000/number of live bihs in the same year.IMR=56+34 X1000/400= 225" "This is a real-world medical entrance exam question, please give the true answer based on the question and selection. Topic name: AIIMS 2017, Subject name: Forensic Medicine","A middle aged lady was found in a robbed room lying in a pool of blood on forensic examination there was an entry wound of size around 2 x 2 cm on the left temporal region with tattooing and blackening around the wound. There was also an exit wound with beveling at the right temporal region. On fuher examination two bullet fragments were found inside the brain parenchyma. Which of the following could be used to determine the distance from which the weapon was fired? Select and explain. A. Hair, B. Clothes, C. Bullet fragments, D. Blood",A: Hair,"Answer: A: Hair Explanation: To calculate the distance from which weapon was fired we have to find range.To calculate blood and bullet fragments are not suitable since they are very small and blood being a liquid is not suitable.Out of hair and clothes since hair is more on head so it is most suitable to calculate range." "This is a real-world medical entrance exam question, please give the true answer based on the question and selection. Topic name: None, Subject name: Forensic Medicine","A 16-year-old female with normal pubic hair and breast development, presented with complaints of primary amenorrhea. Investigation shows normal karyotype. Most likely diagnosis is:AFMC 12; JIPMER 12; AIIMS 12, 13 Select and explain. A. Mullerian agenesis, B. Turner syndrome, C. Testicular feminization, D. Kallmann syndrome",A: Mullerian agenesis,"Answer: A: Mullerian agenesis Explanation: Ans. Mullerian agenesis" "This is a real-world medical entrance exam question, please give the true answer based on the question and selection. Topic name: None, Subject name: Surgery","On massive blood transfusion the following complication occurs- Select and explain. A. Respiratory acidosis, B. Metabolic acidosis, C. Metabolic alkalosis, D. Respiratory alkalosis",C: Metabolic alkalosis,"Answer: C: Metabolic alkalosis Explanation: Answer- C. Metabolic alkalosisBanked blood has a pH of 6.8 and is acidotic.After massive transfusion citrate in the stored blood is conveed into bicarbonate resulting in metabolic alkalosis.Other complications are-HypokalemiaHypothermiaHypocalcemiaRef: Transfusion Medicine and Hemostasis: Clinical and Laboratory Aspects By Christopher D. Hillye, Page 291 ; Yao and Ausio's Anesthesiology: Problem-Oriented Patient Management By Fun-Sun F. Yao, 6th Edition, Page 1148" "This is a real-world medical entrance exam question, please give the true answer based on the question and selection. Topic name: None, Subject name: Pediatrics","Ponderal index is: Select and explain. A. Square root of height in feet by weight in grams, B. Weight in kilograms by cube of height in meters, C. Mid-upper arm circumference to head circumference ratio, D. Head circumference to abdominal circumference ratio",B: Weight in kilograms by cube of height in meters,"Answer: B: Weight in kilograms by cube of height in meters Explanation: Ans: B. Weight in kilograms by cube of height in meters(Ref Dafiary Manual of Obstetrics 3/e p199)Ponderal Index:Calculated by multiplying weight in grams by hundred and then dividing by cube of length in cm.Ponderal index = Bih weight (gm)/ Length (cm)3 x 100Interpretation:Usually less than 2 in asymmetric growth retardation.2 or more in babies with normal growth or symmetric growth retardation." "This is a real-world medical entrance exam question, please give the true answer based on the question and selection. Topic name: None, Subject name: Biochemistry","Which of the following is inclusion of cytoplasm? Select and explain. A. Endoplasmic reticulum, B. Ribosome, C. Golgi body, D. Glycogen",D: Glycogen,"Answer: D: Glycogen Explanation: None" "This is a real-world medical entrance exam question, please give the true answer based on the question and selection. Topic name: None, Subject name: Pathology","Which process shows phases of acute inflammation- Select and explain. A. Apoptosis, B. Pyroptosis, C. Necroptosis, D. Necrosis",D: Necrosis,"Answer: D: Necrosis Explanation: Answer- D NecrosisDirect injury to the endothelium causes cell necrosis and appearance of physical gaps at the sites of detached endothelial cells.In necrosis phospholipase activation occurs that leads to cell damage and leakage of enzymes outside the cell is followed by inflammatory cells resulting in inflammation." "This is a real-world medical entrance exam question, please give the true answer based on the question and selection. Topic name: None, Subject name: Surgery","Investigation of choice to rule out biliary atresia in a 2-month-old child is: Select and explain. A. Hepatic scintigraphy, B. ERCP, C. USG, D. CT scan.",A: Hepatic scintigraphy,"Answer: A: Hepatic scintigraphy Explanation: Ans: A. Hepatic scintigraphy(Ref. Nelson 20/e p1934, 19/e p1385; Sabiston 20/e p1880, I9/e p1852-1853; Schwaz 10/e p1628, 9/1438-1440; Bailey 27/e p1196, 26/e p1104-1105; Blumga 5/e p595-603; Shackelford 7/e p1390-1396)* Investigation of choice to rule out biliary atresia in a 2- month-old child = Hepatic scintigraphy.Hepatobiliary scintigraphy with technetium-labeled iminodiacetic acid derivatives (HIDA scan):* Used to differentiate biliary atresia from non-obstructive causes of cholestasis.Normal hepatic uptake -* In biliary atresia.* Excretion into intestine is absent.Impaired uptake -* In neonatal hepatitis.* Excretion into intestine occur.* Follow-up scan after 24 hours - Determines the biliary tree patency.* Phenobarbital administration (5 mg/kg/day) for 5 days before scan recommended.* Enhances biliary isotope excretion.Hepatobiliary scintigraphy:* Very sensitive.* Non-specific test for biliary atresia.* Fails to identify other structural abnormalities of biliary tree or vascular anomalies." "This is a real-world medical entrance exam question, please give the true answer based on the question and selection. Topic name: None, Subject name: Biochemistry","HDL is synthesised in Select and explain. A. Liver and GIT, B. Liver, C. GIT, D. None of the above",A: Liver and GIT,"Answer: A: Liver and GIT Explanation: None" "This is a real-world medical entrance exam question, please give the true answer based on the question and selection. Topic name: None, Subject name: Forensic Medicine","A man throws sulphuric acid on the face of his wife after a fight following which she comes to the emergency for suppoive management. All of these statements are true about chemical burns except: Select and explain. A. Blisters are present, B. Ulcerated patches are present, C. Absence of singeing of hairs, D. Coagulation necrosis occurs at the site of burn",A: Blisters are present,"Answer: A: Blisters are present Explanation: Ans: A. Blisters are present(Ref Parikh 6/e p4.168) Vesicles & blisters are usually absent in chemical burns.Chemical burn:Occurs when living tissue is exposed to a corrosive substance such as a strong acid or base.Follows standard burn classification.Cause extensive tissue damage.Main types of irritant and/or corrosive products are:Acids, bases, oxidizersreducing agents, solvents, and alkylants.Chemical weapons, e.g. vesicants such as mustard gas and lewisite, or uicants such as phosgene oxime." "This is a real-world medical entrance exam question, please give the true answer based on the question and selection. Topic name: None, Subject name: Gynaecology & Obstetrics","Which is best method to assess fetal damage in a diabetes mother in 1st trimester is: Select and explain. A. Blood sugar estimation, B. Urine ketone assay, C. Amniocentesis to see level of sugar in amniotic fluid, D. Glycosylated Hb",D: Glycosylated Hb,"Answer: D: Glycosylated Hb Explanation: In diabetic patients: Most sensitive test/best test to assess the risk of fetal malformation is maternal HbA1 c levels The best test to detect fetal malformations is USG. Now question says - which is the most sensitive screening test to detect congenital malformations. Undoubtedly ultrasound should be the first choice but it is not given in the options." "This is a real-world medical entrance exam question, please give the true answer based on the question and selection. Topic name: None, Subject name: Surgery","Which of the following is not an indicator of difficult mask ventilation? Select and explain. A. Patient having history of snoring, B. Patient having beard, C. Old age patient, D. Full set of teeth present",D: Full set of teeth present,"Answer: D: Full set of teeth present Explanation: None" "This is a real-world medical entrance exam question, please give the true answer based on the question and selection. Topic name: None, Subject name: Physiology","muscle spindle is Select and explain. A. Receptor for a variety of multisynaptic reflexes, B. Receptor for stretch receptor reflex, C. Occurs only in antigravity extensor muscles, D. Excited by both stretch and contraction of the muscles in which it is located",B: Receptor for stretch receptor reflex,"Answer: B: Receptor for stretch receptor reflex Explanation: None" "This is a real-world medical entrance exam question, please give the true answer based on the question and selection. Topic name: AIIMS 2017, Subject name: Pathology","A 6 year old patient with anemia, on electrophoresis shows HbF of 90% and HbA2 of 3%. Which of the following will be seen on peripheral smear Select and explain. A. A,B, B. A,C, C. B,C, D. ABC",D: ABC,"Answer: D: ABC Explanation: The peripheral smears given shows: A - Target cell B - Howell jolly bodies C - Poikilocytosis. - All these features are characteristic to b- Thalassemia major. - Best investigation for the diagnosis of any kind of hemoglobinopathy-HPLC (High Performance Liquid Chromatography)" "This is a real-world medical entrance exam question, please give the true answer based on the question and selection. Topic name: None, Subject name: Dental","Recently two methods, CPAP conventional and bubble CPAP were compared, in the conventional method 90 out of 160 showed extubation failure while 40 out of 160 showed extubation failure in the bubble method. Which test would be the best to compare the statically significance between the rates of extubation in the two tests.? Select and explain. A. Paired test, B. Chi square test, C. Student test, D. ANOVA",B: Chi square test,"Answer: B: Chi square test Explanation: Chi-square Test It is a test used for qualitative data in which there is no motion of magnitude or size of the attitude. The data are classified by counting the individuals having the same characteristic of attribute. Student t-test It is used for quantitative data. The data has a magnitude and is normally distributed in the population (I.e. Continuous data) eg. Each individual has one measurement from a continuous spectrum or range such as body temperature, height, weight, blood pressure. Student t-test is of three types: Student t-test for single small sample. Student t-test for independent samples (also called as unpaired test). Student t-test for paired sample (paired t test). Paired t-test It compares the means of two paired samples (when two reading, are taken from a same group, before and after an intervention)." "This is a real-world medical entrance exam question, please give the true answer based on the question and selection. Topic name: AIIMS 2019, Subject name: Pharmacology","Which of the following is used for the treatment of paucibacillary leprosy? Select and explain. A. 2 drugs for 6 months, B. 2 drugs for 12 months, C. 3 drugs for 6 months, D. 3 drugs for 12 months",C: 3 drugs for 6 months,"Answer: C: 3 drugs for 6 months Explanation: According to the latest guidelines treatment of multibacillary and paulibacillary leprosy is same expect for duration of treatment. Multibacillary Paulibacillary Rifampin 600 mg once a month supervised 600 mg once a month supervised Clofazimine 300 mg once a month supervised + 50 mg daily self administered 300 mg once a month supervised + 50 mg daily self administered Dapsone 100 mg daily self administered 100 mg daily self administered Duration 12 months 6 months" "This is a real-world medical entrance exam question, please give the true answer based on the question and selection. Topic name: None, Subject name: Dental","The pin used in pin restoration of a root canal treated tooth is: Select and explain. A. Self thread pin, B. Cemented pin, C. Frictional pin, D. Any of the above",B: Cemented pin,"Answer: B: Cemented pin Explanation: None" "This is a real-world medical entrance exam question, please give the true answer based on the question and selection. Topic name: None, Subject name: Dental","Etching of dentin does not include: Select and explain. A. Removal of smear layer, B. Exposure of collagen fibres, C. Opening of dentinal tubules, D. Increases surface energy",D: Increases surface energy,"Answer: D: Increases surface energy Explanation: None" "This is a real-world medical entrance exam question, please give the true answer based on the question and selection. Topic name: AIIMS 2019, Subject name: Ophthalmology","ETDRS grading of vision is done for Select and explain. A. Endothelial count, B. Corneal topography, C. Primary open angle glaucoma, D. Classification of diabetic retinopathy",D: Classification of diabetic retinopathy,"Answer: D: Classification of diabetic retinopathy Explanation: ETDRS - Early treatment diabetic retinopathy study It is the best cha to measure the vision It is used to early treatment diabetic retinopathy" "This is a real-world medical entrance exam question, please give the true answer based on the question and selection. Topic name: None, Subject name: Dental","Bacteriocidal toxin is found in: Select and explain. A. Zone of contamination, B. Zone of infection, C. Zone of stimulation, D. Zone of irritation",B: Zone of infection,"Answer: B: Zone of infection Explanation: The reaction of the periradicular tissues to noxious products of tissue necrosis, bacterial products and antigenic agents from the root canal has been described by Fish. The four zones  Zone of infection Zone of contamination Zone of irritation Zone of stimulation Zone of Infection: This is present in the center of the lesion. It is characterized by PMNL's. The bacterial toxins are found in this zone. Zone of Contamination: It is characterized by round cell infiltration. It demonstrates cellular destruction due to toxins from the central zone or zone of infection. Because of autolysis and death of bone cells, the lacunae were empty. Lymphocytes are prevalent everywhere. Zone of Irritation: This zone is characterized by macrophages and osteoclasts. The collagen framework was digested by phagocytic cells, the macrophages, while osteoclasts attacked the bone tissue. This opens a gap in the bone all around the center of lesion. That space becomes filled with PMNL's. It contains cholesterol crystals. This zone demonstrates much activity preparatory to repair. Zone of Stimulation: (Peripheral zone) This zone is characterized by fibroblasts and osteoblasts. In this zone, the effects of toxins were mild enough to be stimulant which results in laying down of collagen fibers around the zone of irritation that acts both as a wall of defense and as a scaffolding on which the osteoblasts built new bone. This new bone was built in an irregular fashion." "This is a real-world medical entrance exam question, please give the true answer based on the question and selection. Topic name: None, Subject name: Medicine","Which of the following is not a feature of infective endocarditis. Select and explain. A. Roth spot., B. Osler's nodes., C. Aschoff nodules., D. Positive blood culture.",C: Aschoff nodules.,"Answer: C: Aschoff nodules. Explanation: Aschoff nodules are pathognomonic feature of rheumatic heart disease." "This is a real-world medical entrance exam question, please give the true answer based on the question and selection. Topic name: AIIMS 2017, Subject name: Pharmacology","A 27-year-old female presented to OPD of infeility clinic. She was prescribed bromocriptine. What could be the possible reason? Select and explain. A. Hyperprolactinemia, B. Polycystic ovarian disease, C. Hypogonadotropic hypogonadism, D. Pelvic inflammatory disease",A: Hyperprolactinemia,"Answer: A: Hyperprolactinemia Explanation: Bromocriptine is a D2 agonist, so it decreases the prolactin levels.Increased prolactin can lead to galactorrhea and infeility (by decreasing gonadotropins)." "This is a real-world medical entrance exam question, please give the true answer based on the question and selection. Topic name: None, Subject name: Dental","In case of buccolingual wide proximal box, which of the following wedgging method should be used: Select and explain. A. Wedge wedgging., B. Double wedgging., C. Piggy back., D. None.",B: Double wedgging.,"Answer: B: Double wedgging. Explanation: None" "This is a real-world medical entrance exam question, please give the true answer based on the question and selection. Topic name: None, Subject name: Anatomy","Nociceptive signals are transmitted primarily to which of the following? Select and explain. A. Nucleus caudalis, B. Limbic system, C. Reticular system, D. Superior cervical ganglion",A: Nucleus caudalis,"Answer: A: Nucleus caudalis Explanation: Trigeminal Nucleus caudalis is an important site for processing orofacial nociceptive input. After activation of C and A delta fibres, they transmit nociceptive signals primarily via trigeminal nerves to Trigeminal Nucleus caudalis. It is also called as medullary dorsal horn as it has similar anatomical resemblance to spinal dorsal horn." "This is a real-world medical entrance exam question, please give the true answer based on the question and selection. Topic name: None, Subject name: Dental","At what stage of embryonic development does initial tooth development begins: Select and explain. A. 3rd week, B. 6th week, C. 8th week, D. 4th week",B: 6th week,"Answer: B: 6th week Explanation: The odontogenesis of the primary dentition begins in the embryonic period, between the sixth and the seventh week of prenatal development. The first stage of tooth development is initiation, where ectoderm induces the mesenchymal tissue in order to initiate the process. There are 4 main stages of development of the tooth: The first stage begins in the fetus at about 6 weeks of age. This is when the basic substance of the tooth forms. Next, the hard tissue that surrounds the teeth is formed, around 3 to 4 months of gestation." "This is a real-world medical entrance exam question, please give the true answer based on the question and selection. Topic name: None, Subject name: Dental","Unique characteristic of attached gingiva in children: Select and explain. A. Interdental clefts., B. Retrocuspid papilla., C. Both., D. Stillman's cleft.",C: Both.,"Answer: C: Both. Explanation: The two unique characteristics of the attached gingiva in children are interdental clefts and the retrocuspid papilla. The inter dental clefts are normal anatomic features found in the interradicular zones underlying the saddle areas. The retrocuspid papilla is found approximately 1mm below the free gingiva on the attached gingiva lingual to mandibular canine . It occurs in 85°/0 of children and apparently decreases with age." "This is a real-world medical entrance exam question, please give the true answer based on the question and selection. Topic name: None, Subject name: Dental","Effect of thumb sucking: Select and explain. A. Localized anterior open bite., B. Increase overjet., C. Constricted maxilla, D. All of the above.",D: All of the above.,"Answer: D: All of the above. Explanation: Clinical features of thumb sucking: Well localized Anterior open bite Secondary tongue thrusting(May develop). Increased overjet - Proclination of upper teeth, Retroclination of lower teeth Decreased overbite Maxillary constriction (greater maxillary arch depth & decreased maxillary width) -Posterior cross bite Mandible rotates downwards and backwards (vertical growth pattern) Hypotonic upper lip because of proclination of upper anteriors Hypertonic lower lip Hypertonic active mentalis activity causes Long face and convex profile." "This is a real-world medical entrance exam question, please give the true answer based on the question and selection. Topic name: None, Subject name: Medicine","Patient presented with PSVT after carotid management, BP is 70/30 mm/H & vagal maneuver failed and refractory to adenosine given. what will be the next step- Select and explain. A. Synchronised Cardioversion, B. Repeated adenosine, C. Inj. Amiodarone, D. DC shock",A: Synchronised Cardioversion,"Answer: A: Synchronised Cardioversion Explanation: Answer- A. Synchronised CardioversionThe next best step of management in this patient is synchronized cardioversion.The ECG is suggestive of paroxysmal supraventricular tachycardia (PSVT) at a rate of 150/min. In this case, carotid massage and IV adenosine have failed to control PSVT and the patient continues to have hemodynamic instability (B.P. - 60/30 mmHg). In this condition, synchronized direct current cardioversion (and not just DC shock) should be initiated with the staing dose of 50-100J." "This is a real-world medical entrance exam question, please give the true answer based on the question and selection. Topic name: None, Subject name: Pediatrics","A 7–day old infant has a leaky meningomyelocele. The most useful test for diagnosis and management of the condition is – Select and explain. A. Blood–culture and sensitivity, B. Urine–culture and sensitivity, C. Rectal swab–culture and sensitivity, D. Wound swab–culture and sensitivity",A: Blood–culture and sensitivity,"Answer: A: Blood–culture and sensitivity Explanation: Most cases of meningocele are recommended for treatment as soon after birth as possible. In the case of a sac which is leaking fluid, the treatment is most urgent. Generally, treatment includes a surgical procedure to close and remove the soft tissue covering of back. As there are chances of bacterial meningitis and sepsis due to CSF leak, CSF and blood sample should be send for culture and sensitivity and peri-operative antibiotics (broad spectrum) can be changed to the specific antibiotics after the blood and CCF culture and sensitivity report." "This is a real-world medical entrance exam question, please give the true answer based on the question and selection. Topic name: None, Subject name: Radiology","The radiopacity that frequently obliterate the root apices of maxillary molars is Select and explain. A. Maxillary sinus, B. Palatine bone, C. Zygoma and Zygomatic process of Maxilla, D. Orbital process of Zygomatic bone",C: Zygoma and Zygomatic process of Maxilla,"Answer: C: Zygoma and Zygomatic process of Maxilla Explanation: FigureA  shows  the  inferior  border  of  the  zygomatic  process  of  the  maxilla  over the  molars.  This  structure  lies  buccal  to  the  teeth  and  appears to  move  mesially  as  the  x-ray  beam  is  oriented  more  from the  distal.  Similarly,  as  the  angulation  of  the  beam  is  increased vertically,  the  zygomatic  process  is  projected  occlusally  over the  teeth." "This is a real-world medical entrance exam question, please give the true answer based on the question and selection. Topic name: None, Subject name: Radiology","Disadvantages of using a GRID during radiography is: Select and explain. A. Increased patient exposure due to more exposure time, B. Decreased exposure time, C. Increased secondary radiation fog to film, D. Reduces secondary radiation to film",A: Increased patient exposure due to more exposure time,"Answer: A: Increased patient exposure due to more exposure time Explanation: None" "This is a real-world medical entrance exam question, please give the true answer based on the question and selection. Topic name: None, Subject name: Pathology","Brittle bone syndrome is caused by Select and explain. A. Improper synthesis of procollagen, B. Polymerisation of collagen, C. Increased osteoclastic activity, D. Increased fibroblastic activity",A: Improper synthesis of procollagen,"Answer: A: Improper synthesis of procollagen Explanation: None" "This is a real-world medical entrance exam question, please give the true answer based on the question and selection. Topic name: None, Subject name: Dental","Full mouth disinfection is done in: Select and explain. A. 24 hours, B. 48 hours, C. 72 hours, D. 36 hours",A: 24 hours,"Answer: A: 24 hours Explanation: None" "This is a real-world medical entrance exam question, please give the true answer based on the question and selection. Topic name: None, Subject name: Anatomy","The nerve which constitutes the sensory nerve supply of capsule of TMJ is: Select and explain. A. Auriculotemporal nerve, B. Facial nerve, C. Masseteric nerve, D. Auricular nerve",A: Auriculotemporal nerve,"Answer: A: Auriculotemporal nerve Explanation: Branches of auriculotemporal nerve i. Slender filaments to the posterior part of the capsule of TMJ ii. One or two thick branches to the parotid gland which mingle with the branches of the facial nerve in its substance. iii. Cutaneous branches to the auricle and temple." "This is a real-world medical entrance exam question, please give the true answer based on the question and selection. Topic name: None, Subject name: Dental","For a bonding agent to be effective wetting angle should be Select and explain. A. Minimum with dental hard tissue, B. Minimum with dentin and Maximum with enamel, C. Maximum with dentin and Minimum with enamel, D. Maximum with dental hard tissue",A: Minimum with dental hard tissue,"Answer: A: Minimum with dental hard tissue Explanation: None" "This is a real-world medical entrance exam question, please give the true answer based on the question and selection. Topic name: None, Subject name: Dental","The strength of a gypsum investment is dependent on: Select and explain. A. Carbon content, B. Silica content, C. Gypsum, D. Copper",C: Gypsum,"Answer: C: Gypsum Explanation: α-hemihydrate of gypsum, and quartz or cristobalite are forms of silica. Most investments contain the α-hemihydrate of gypsum, because of its greater strength. This gypsum product serves as a binder for the other ingredients and to provide rigidity. The strength of the investment is dependent on the amount of binder used. The investment powder may contain 25% to 45% of calcium sulfate hemihydrate. Gypsum shrinks considerably, when it is heated. If the proper forms of silica are employed in the investment, this contraction during heating can be eliminated and changed to an expansion. Ref: Phillips 12th edition page 201" "This is a real-world medical entrance exam question, please give the true answer based on the question and selection. Topic name: None, Subject name: Gynaecology & Obstetrics","Which of the following is the impoant marker of male infeility in semen analysis? Select and explain. A. Motility, B. Concentration, C. Volume, D. Sperm Count",B: Concentration,"Answer: B: Concentration Explanation: Ans. B. ConcentrationRef: Clinical Gynecologic Endocrinology Infeility, 8' ed.Sperm concentrationand progressive motility is impoant in distinguishing feile from infeile men but strict sperm morphology is one of the most discriminating value." "This is a real-world medical entrance exam question, please give the true answer based on the question and selection. Topic name: None, Subject name: Anaesthesia","A pregnant woman with placenta pre staed to bleed as she went into labor. Her blood pressure was 80/50 mm Hg. A lower segment caesarean section was planned in view of acute shock. What type of anesthesia will you plan for this patient? Select and explain. A. General anesthesia with IV induction by ketamine, B. Spinal anesthesia up to L4 level, C. General anesthesia with IV induction by propofol followed by maintenance with fluranes, D. Sedation and epidural analgesia",A: General anesthesia with IV induction by ketamine,"Answer: A: General anesthesia with IV induction by ketamine Explanation: Ans: A. General anesthesia with IV induction by ketamine(Ref Williams 24/e p516; Morgan 4/e p197-199)This patient of placenta pre is in labor and has bled into shock.She should be delivered by cesarean section under general anesthesia.General anesthesia is preferred as it is a more controllable modality and there is a significant risk of hypotension associated with spinal anesthesia.Ketamine is the preferred agent in cases of acute shock.Stimulates sympathetic system causing tachycardia and hypeension, so it is intravenous anaesthetic of choice for shock." "This is a real-world medical entrance exam question, please give the true answer based on the question and selection. Topic name: None, Subject name: Pharmacology","A patient requires 180 mg ceftriaxone. The l contains 500 mg/5ml of ceftriaxone. You have a 2 ml syringe with l0 divisions per ml. How many divisions in the 2 ml syringe will you fill to give 180 mg ceftriaxone? Select and explain. A. 9, B. 18, C. l0, D. 12",B: 18,"Answer: B: 18 Explanation: Ans: B. 18500 mg/ 5 ml means 100 mg/l ml. Since 100 mg is present in 1 ml, therefore, 180 mg of drug is present in 1.8 ml.Now it is clear that we must administer 1.8 ml of the above solution to administer the desired amount of drug.Last thing is to calculate the divisions to administer 1.8 ml.10 division per ml means each 0. 1 ml is equal to 1 division.Therefore, 1 .8 ml will be equal to l8 divisions." "This is a real-world medical entrance exam question, please give the true answer based on the question and selection. Topic name: None, Subject name: Surgery","An adult patient sustained a subcondylar fracture on the left side. Clinically it is seen that there is : Select and explain. A. Moderate intraoral bleeding, B. Trismus and bilateral crepitus, C. Deviation of the mandible to the right on protrusion, D. Inability to deviate the mandible to the right",D: Inability to deviate the mandible to the right,"Answer: D: Inability to deviate the mandible to the right Explanation: None" "This is a real-world medical entrance exam question, please give the true answer based on the question and selection. Topic name: None, Subject name: Gynaecology & Obstetrics","What is the best time to give anti-D to a pregnant patient? Select and explain. A. 12 weeks, B. 28 weeks, C. 36 weeks, D. After delivery",B: 28 weeks,"Answer: B: 28 weeks Explanation: Ans: B. 28 weeks (Ref Williams 24/e p312: COGT 11/e p353; FERNANDO ARIAS 4/e p374).Best time to give anti-D to a pregnant patient is 28 weeks.According to American College of Obstetricians and Gynecologists, 2010:Anti-D immune globulin is given prophylactically to all Rh D-negative, unsensitized women at approximately 28 weeks.Second dose is given after delivery if the infant is Rh D-positive ." "This is a real-world medical entrance exam question, please give the true answer based on the question and selection. Topic name: None, Subject name: Dental","Gingivitis Select and explain. A. Inevitably progresses to periodontitis, B. Affects approximately 40% of adolescents, C. Is characterized by true pocketing, D. Is a reversible lesion",D: Is a reversible lesion,"Answer: D: Is a reversible lesion Explanation: None" "This is a real-world medical entrance exam question, please give the true answer based on the question and selection. Topic name: None, Subject name: Biochemistry","The protein rich in basic amino acids, which functions in the packaging of DNA in chromosomes, is: Select and explain. A. Histone, B. Collagen, C. Hyaluronic acid binding protein, D. Fibrinogen",A: Histone,"Answer: A: Histone Explanation: None" "This is a real-world medical entrance exam question, please give the true answer based on the question and selection. Topic name: None, Subject name: Dental","Statistics to check out mean of values is done by: Select and explain. A. Chi-square test, B. Student's analysis, C. Probability test, D. Horizontal studies",B: Student's analysis,"Answer: B: Student's analysis Explanation: None" "This is a real-world medical entrance exam question, please give the true answer based on the question and selection. Topic name: None, Subject name: Dental","In class II cavity for inlay, the cavosurface margin of the gingival seat clears the adjacent tooth by: Select and explain. A. 0.20+ 0.05mm, B. 0.50+ 0.20mm, C. 0.80 ± 0.35mm, D. 1.10 ± 0.45mm",B: 0.50+ 0.20mm,"Answer: B: 0.50+ 0.20mm Explanation: None" "This is a real-world medical entrance exam question, please give the true answer based on the question and selection. Topic name: None, Subject name: Pharmacology","Aspirin acts on which part of brain? Select and explain. A. Substantia gelatinosa, B. Limbic system, C. Medulla, D. Cortex",A: Substantia gelatinosa,"Answer: A: Substantia gelatinosa Explanation: None" "This is a real-world medical entrance exam question, please give the true answer based on the question and selection. Topic name: None, Subject name: Social & Preventive Medicine","A latex agglutination test for detection of meningitis was approved. Calculate the sensitivity and specificity of the test based on the data given below: Test PositiveTest NegativeDiseased273Non-diseased595 Select and explain. A. Sensitivity 90% Specificity 95%, B. Specificity 90% Sensitivity 95%, C. Sensitivity 80% Specificity 90%, D. Sensitivity 75% Specificity 95%",A: Sensitivity 90% Specificity 95%,"Answer: A: Sensitivity 90% Specificity 95% Explanation: Ans: A. Sensitivity 90% Specificity 95%Sensitivity:Introduced as a statistical index of diagnostic accuracy.Defined as the ability of a test to identify correctly all those who have the disease, that is 'true positive:""Specificity:Measures the propoion of negatives that are correctly identified as such, e.g., the percentage of healthy people who are correctly identified as not having the condition, that is true negative. Sensitivity = Propoion of persons with the condition who test positive: a /(a + c)`t = 27/30 = 90% .Specificity = Propoion of persons without the condition who test negative: d /(b + d)deg = 95/100 = 95%." "This is a real-world medical entrance exam question, please give the true answer based on the question and selection. Topic name: None, Subject name: Medicine","A 40 years old patient came with complaints of spikes of fever and difficulty in breathing. Transesophageal ECHO found out the vegetations in the hea. The culture was positive for Burkholderia cepacia. Drug of choice for Burkholderia cepacia pneumonia is: Select and explain. A. Aminoglycoside and colistin, B. Carbapenems with 3rd generation cephalosporins, C. Tigecycline and cefipime, D. Cotrimoxazole with 3rd generation cephalosporins",D: Cotrimoxazole with 3rd generation cephalosporins,"Answer: D: Cotrimoxazole with 3rd generation cephalosporins Explanation: Answer- D. Cotrimoxazole with 3rd generation cephalosporinsDrug of choice for B. cepacia is TMP-SMX and alternative agents are ceftazidime, chloramphenicolB. cepacia is the cause of a rapidly fatal syndrome of respiratory distress & septicemia ('cepacia syndrome')in cystic fibrosis Patients.Predisposing factors: cystic fibrosis & chronic granulomatous diseaseaB. cepacia inhabits moist environments and is found in rhizosphere.possesses multiple virulence factors & colonizing factors capable of binding to lung mucus (predilection of B.cepacia for the lungs in cystic fibrosis).Treatment:DOC for B. cepacia: TMP-SMXAlternative agents: Meropenem & doxycycline" "This is a real-world medical entrance exam question, please give the true answer based on the question and selection. Topic name: None, Subject name: Dental","Which of the following bone defects offers the best chance for bone fill? Select and explain. A. 3 Walled defect, B. 2 Walled defect, C. Osseous crater, D. Hemisepta",A: 3 Walled defect,"Answer: A: 3 Walled defect Explanation: None" "This is a real-world medical entrance exam question, please give the true answer based on the question and selection. Topic name: None, Subject name: Medicine","Hand to knee gait in polio is due to involvement of which muscle? Select and explain. A. Gastrocnemius, B. Gluteus medius, C. Quadriceps, D. Hamstring",C: Quadriceps,"Answer: C: Quadriceps Explanation: Answer-C. QuadricepsDeformity at hip: Flexion, abduction & external rotationDeformity at knee: Triple deformity of flexion' posterior subluxation & external rotationFoot: Equinovarus is most deformity followed by equinovalgus, Calcaneovalgus, calcaneovarusUpper limb: Shoulder and elbow muscles" "This is a real-world medical entrance exam question, please give the true answer based on the question and selection. Topic name: AIIMS 2017, Subject name: Anatomy","Which of the following marked muscles is Involved in Opening of jaw? Select and explain. A. A, B. B, C. C, D. D",A: A,"Answer: A: A Explanation: Depression of Mandible done by Lateral Pterygoid Mylohyoid Anterior belly of digastric muscle *In the image Marker A - Lateral Pterygoid Marker B - Lateral to mandible bone - Masseter Marker C - TowardsTemporal region - Temporalis (retraction of Mandible) Marker D - Subcutaneous Muscle - Buccinator (Cheek muscle - Accessory Muscle for mastication) *Angle of mandible has Masseter laterally Medial pterygoid medially *ELEVATORS- Mnemonic - MTMe M - Masseter T - Temporalis M - Medial Pterygoid" "This is a real-world medical entrance exam question, please give the true answer based on the question and selection. Topic name: None, Subject name: Medicine","Urine analysis of a patient with hematuria and hypercalciuria is most likely to reveal; Select and explain. A. Isomorphic RBCs, B. RBC casts, C. Nephrotic range proteinuria, D. Eosinophiluria",A: Isomorphic RBCs,"Answer: A: Isomorphic RBCs Explanation: Ans. a. Isomorphic RBCsUrine analysis of a patient with hematuria and hypercalciuria is most likely to reveal isomorphic RBCs.""Hypercalciuria is a cause of non-glomerular hematuria. RBCs from a non-glomerular source more closely resemble peripheral blood on microscopy, with isomorphic RBCs and absence of casts.""" "This is a real-world medical entrance exam question, please give the true answer based on the question and selection. Topic name: None, Subject name: Surgery","A patient presents to the ER after a A with multiple rib injuries. He is conscious, speaking single words.RR= 40/ minute, BP= 90/40mm Hg. What is the next immediate step in management? Select and explain. A. Intubate the patient, B. Urgent fluid infusion, C. Chest X- ray, D. Needle inseion in 2nd ICS",D: Needle inseion in 2nd ICS,"Answer: D: Needle inseion in 2nd ICS Explanation: Answer- D. Needle inseion in 2nd ICSThis is a case of pneumothorax. Although the new ATLS update is 5th intercostal space in mid axillary line but in this question we will go with a time tested method of needle in the 2nd ICS." "This is a real-world medical entrance exam question, please give the true answer based on the question and selection. Topic name: None, Subject name: Pediatrics","A neonate presented with cicatrizing skin lesions all over the body with hypoplasia of all limbs. An MRI of the brain revealed diffuse cerebral atrophy. An ophthalmologic evaluation reveals chorioretinitis. Which of these tests is most likely to show a positive result in this patient? Select and explain. A. Anti-HCMV antibodies, B. Anti-toxoplasma antibodies, C. Anti-VZV antibody, D. Anti-rubella antibody",C: Anti-VZV antibody,"Answer: C: Anti-VZV antibody Explanation: Ans: C. Anti-VZV antibody(Ref Ghai 8/e p2215).Suggestive of congenital varicella infections detected using Anti-VZV antibodies.Congenital varicella syndrome:Characterized by cicatricial skin scarring in a zoster-like distribution, limb hypoplasia, and neurologic (e.g., microcephaly, coical atrophy, seizures, and mental retardation), eye (e.g., chorioretinitis, microphthalmia, and cataracts), renal (e.g., hydroureter and hydrottephrosis) and autonomic nervous system abnormalities (neurogenic bladder, swallowing dysfunction, and aspiration pneumonia)." "This is a real-world medical entrance exam question, please give the true answer based on the question and selection. Topic name: None, Subject name: Anatomy","Which part of body is underdeveloped at birth: Select and explain. A. Eyes, B. Ears., C. Face., D. Brain",C: Face.,"Answer: C: Face. Explanation: None" "This is a real-world medical entrance exam question, please give the true answer based on the question and selection. Topic name: None, Subject name: Physiology","To preserve blood for transfusion later: Select and explain. A. Dilute with equal volume of 0.9% saline, B. Add solution of sodium citrate, C. Add solution of calcium chloride, D. Add fibrinogen",B: Add solution of sodium citrate,"Answer: B: Add solution of sodium citrate Explanation: None" "This is a real-world medical entrance exam question, please give the true answer based on the question and selection. Topic name: None, Subject name: Pathology","False about Bone marrow biopsy Select and explain. A. Can be done in prone or lateral position, B. To find out infiltrative and granulomatous disorders, C. Breath holding not necessary, D. Contraindicated when platelet count is below 40,000","D: Contraindicated when platelet count is below 40,000","Answer: D: Contraindicated when platelet count is below 40,000 Explanation: Ans: D. Contraindicated when platelet count is below 40,000CT-guided bone marrow biopsy is safe in thrombocytopenic patients, with a hemorrhagic complication rate below 1.6% for patients with a platelet count of 20,000-50,000/mL. Routine preprocedure platelet transfusion may not be necessary for patients with a platelet count of 20,000-50,000/mL." "This is a real-world medical entrance exam question, please give the true answer based on the question and selection. Topic name: None, Subject name: Medicine","Hypeensive hemorrhage is most commonly seen in: Select and explain. A. Basal ganglia, B. Thalamus, C. Brain stem, D. Cerebrum",A: Basal ganglia,"Answer: A: Basal ganglia Explanation: Ans. A. Basal gangliaHypeensive hemorrhage:Causes: Results from spontaneous rupture of small penetrating aery deep in the brain.Most common site:Basal ganglia (especially putamen), thalamus, cerebellum & pons.Small aeries here are most prone to hypeension-induced vascular injury." "This is a real-world medical entrance exam question, please give the true answer based on the question and selection. Topic name: None, Subject name: Surgery","When Isoflurane is used as a general anesthetic agent, what is the maximum dose of adrenaline that can be given: Select and explain. A. 3.4 mug/kg, B. 6.7 mug/kg, C. 2.1 mug/kg, D. 10.9 mug/kg",B: 6.7 mug/kg,"Answer: B: 6.7 mug/kg Explanation: None" "This is a real-world medical entrance exam question, please give the true answer based on the question and selection. Topic name: None, Subject name: Anatomy","Preganglionic parasympathetic fibres to the Otic ganglion are carried in the: Select and explain. A. Greater petrosal nerve, B. Lesser petrosal nerve, C. Chorda tympani, D. Auriculotemporal nerve",B: Lesser petrosal nerve,"Answer: B: Lesser petrosal nerve Explanation: None" "This is a real-world medical entrance exam question, please give the true answer based on the question and selection. Topic name: None, Subject name: Dental","A 10 year old patient reported to department with BCLP (bilateral cleft lip & palate); the maxillary transverse width was normal, lateral incisor was congenitally missing with impacted canine. What will be the treatment approach? Select and explain. A. SABG only, B. SABG followed by expansion, C. Expansion followed by SABG, D. Only expansion",A: SABG only,"Answer: A: SABG only Explanation: Timing of SABG surgery: Secondary alveolar bone grafting (SABG) is done at an age when the growth inhibition effects of the surgery on maxilla are minimised, and it can help the maxillary canine or lateral incisor to erupt normally through the cancellous bone. After the age of 9 years, maxillary growth is minimal. Secondary bone grafting is done during the mixed dentition stage after the eruption of permanent incisors, but before the eruption of permanent canines. The timing of bone graft is based on the root formation and eruption pattern of the maxillary lateral incisor and maxillary canine. In case, insufficient bone is available in the cleft area for the lateral incisors to erupt, bone graft can be done around 7 years of age. Note: In this patient, since the transverse width is normal, there is no need for expansion. Orthodontics: Diagnosis and Management of Malocclusion and Dentofacial Deformities 3rd ed Om P. Kharbanda pdf no 3709" "This is a real-world medical entrance exam question, please give the true answer based on the question and selection. Topic name: None, Subject name: Anatomy","The secretion of salivary gland may be described as? Select and explain. A. Exocrine and merocrine, B. Exocrine and holocrine, C. Endocrine and holocrine, D. Endocrine and merocrine",A: Exocrine and merocrine,"Answer: A: Exocrine and merocrine Explanation: None" "This is a real-world medical entrance exam question, please give the true answer based on the question and selection. Topic name: None, Subject name: Biochemistry","CG islands in our DNA are impoant for: Select and explain. A. Methylation, B. Acetylation, C. t-RNA synthesis, D. DNA replication",A: Methylation,"Answer: A: Methylation Explanation: Ans: A. Methylation(Ref Harper 30/e p438, 439; Harrison 19/e p102e-7)CG islands:Also referred as ""CpG islands"".Plays vital role in gene expression regulation.Cytosine (C) residues in CG rich islands undergo methylation by DNA methyl transferase.The DNA methyltransferase can methylate only the CG sequence paired with methylated CG. The CG sequence not paired with methylated CG will not be methylated.Methylation of deoxycytidine residues (in sequence 5'-in CpG-3') in DNA may effect gross changes in chromatin so as to preclude its active transcription." "This is a real-world medical entrance exam question, please give the true answer based on the question and selection. Topic name: None, Subject name: Medicine","Adrenocorticosteroids in excess may cause: Select and explain. A. Osteoporosis, B. Osteosclerosis, C. Osteochondritis, D. None of the above",A: Osteoporosis,"Answer: A: Osteoporosis Explanation: None" "This is a real-world medical entrance exam question, please give the true answer based on the question and selection. Topic name: None, Subject name: Surgery","Chances of recurrence of multicystic ameloblastoma after surgical enucleation is: Select and explain. A. <50%, B. <10%, C. 50-100%, D. 18-25%",C: 50-100%,"Answer: C: 50-100% Explanation: None" "This is a real-world medical entrance exam question, please give the true answer based on the question and selection. Topic name: None, Subject name: Dental","True about conversion of monomer to polymer is Select and explain. A. Density changes from 1.19g/cm3 to 0.9 and volume shrinkage is 21%, B. Density changes from 1.19g/cm3 to 0.9 and volume shrinkage is 7%, C. Density changes from 0.9g/cm3 to 1.19 and volume shrinkage is 21%, D. Density changes from 0.9g/cm3 to 1.19 and volume shrinkage is 7%",C: Density changes from 0.9g/cm3 to 1.19 and volume shrinkage is 21%,"Answer: C: Density changes from 0.9g/cm3 to 1.19 and volume shrinkage is 21% Explanation: None" "This is a real-world medical entrance exam question, please give the true answer based on the question and selection. Topic name: None, Subject name: Gynaecology & Obstetrics","Before ovulation development of granules in a cell is dependent on: Select and explain. A. Estrogen, B. Progesterone, C. FSH, D. LH",A: Estrogen,"Answer: A: Estrogen Explanation: Ans. a. EstrogenEstrogens are a steroid hormone which exists in three forms each of unique significance during a woman's life.Estradiol is the most common moiety during the non-pregnant reproductive years. It is conveed from androgens (produced from cholesterol in the follicular theca cells), which diffuse into the follicular granulosa cells containing the aromatase enzyme that completes the transformation into estradiol." "This is a real-world medical entrance exam question, please give the true answer based on the question and selection. Topic name: None, Subject name: Surgery","In long surgical procedure intubation method is Select and explain. A. Nasotracheal tube with cuff, B. LIMA, C. Nasotracheal tube without cuff, D. Endotracheal tube",A: Nasotracheal tube with cuff,"Answer: A: Nasotracheal tube with cuff Explanation: None" "This is a real-world medical entrance exam question, please give the true answer based on the question and selection. Topic name: None, Subject name: Dental","Working cusps are also known as: Select and explain. A. Guiding cusps., B. Supporting cusps., C. Holding cusps., D. Balancing cusps.",B: Supporting cusps.,"Answer: B: Supporting cusps. Explanation: None" "This is a real-world medical entrance exam question, please give the true answer based on the question and selection. Topic name: None, Subject name: Medicine","All of the following are criteria for admission in upper GI bleed except: Select and explain. A. Shock index >1.5, B. Hematocrit > 40%, C. Frank blood in nasogastric aspirate, D. BP <100 mm Hg",B: Hematocrit > 40%,"Answer: B: Hematocrit > 40% Explanation: Answer- B. Hematocrit > 40%Hematocrit > 40% is not a criteria for admission in upper GI bleed.Shock index >1.5, Frank blood in nasogastric aspirate and BP <100 mm Hg are the indications of hospital admission and resuscitation in upper GI bleed." "This is a real-world medical entrance exam question, please give the true answer based on the question and selection. Topic name: None, Subject name: Dental","Caries in a 4 year old child is best assessed by: Select and explain. A. Previous caries experience, B. Maternal caries, C. School fluoridation, D. Daily fluoride mouthwash",A: Previous caries experience,"Answer: A: Previous caries experience Explanation: None" "This is a real-world medical entrance exam question, please give the true answer based on the question and selection. Topic name: None, Subject name: Pathology","Strength of collagen is due to: Select and explain. A. Hydroxy glycine, B. Glycine, C. Praline, D. Hydroxyproline",D: Hydroxyproline,"Answer: D: Hydroxyproline Explanation: None" "This is a real-world medical entrance exam question, please give the true answer based on the question and selection. Topic name: None, Subject name: Biochemistry","At physiological pH, which of these amino acids has a positive charge? Select and explain. A. Valine, B. Aspaic acid, C. Arginine, D. Isoleucine",C: Arginine,"Answer: C: Arginine Explanation: Ans: C. ArginineAt physiological pH, arginine has a positive charge.Negative Charged (Acidic Side Chains) - Aspaic acid & glutamic acid.Positive Charged (Basic Side Chains) - Histidine, arginine & lysine." "This is a real-world medical entrance exam question, please give the true answer based on the question and selection. Topic name: None, Subject name: Dental","Gycosylated Hb is done for the assessment of: Select and explain. A. Diabetic patient, B. Cushing disease, C. Hyperparathyroidism, D. Addison disease",A: Diabetic patient,"Answer: A: Diabetic patient Explanation: None" "This is a real-world medical entrance exam question, please give the true answer based on the question and selection. Topic name: None, Subject name: Pathology","GNAS-1 gene mutation is associated with Select and explain. A. Fibrous dysplasia, B. Osteogenesis imerfecta, C. Amelogenesis imperfect, D. Cleidocranial dysplasia",A: Fibrous dysplasia,"Answer: A: Fibrous dysplasia Explanation: Fibrous dysplasia is caused by a mutation in GNAS1 gene." "This is a real-world medical entrance exam question, please give the true answer based on the question and selection. Topic name: None, Subject name: Dental","Fill the missing data in the Ottawa charter for health promotion Select and explain. A. Promotion of health services, B. Reorienting health services, C. Prevention of disease, D. Effective health services",B: Reorienting health services,"Answer: B: Reorienting health services Explanation: None" "This is a real-world medical entrance exam question, please give the true answer based on the question and selection. Topic name: None, Subject name: Microbiology","Helminth implicated in causing pernicious anemia is: Select and explain. A. Diphyllobothrium latum, B. Ascaris, C. Taenia solium, D. Hymenolepis nana",A: Diphyllobothrium latum,"Answer: A: Diphyllobothrium latum Explanation: Ans: A. Diphyllobothrium latum (Ref: Paniker's 7/e p118, b/e p142; lawetz 27/e p731; Harrison 19/e p1434.Helminth implicated in causing pernicious anemia is Diphyllobothrium latum.Diphyllobothrium latum:Disease caused by tapeworms is chiefly vague abdominal discomfo and loss of appetite, leading to weight loss.D latum has an unusual capacity to absorb vitamin B12 a causing vitamin B12 deficiency a Pernicious anemia may rarely develop." "This is a real-world medical entrance exam question, please give the true answer based on the question and selection. Topic name: None, Subject name: Gynaecology & Obstetrics","All are true about polycystic ovarian disease except: Select and explain. A. Persistently elevated LH, B. Increased LH/FSH ratio, C. Increased Dheas, D. Increased prolactin",D: Increased prolactin,"Answer: D: Increased prolactin Explanation: In patients of PCOS: Alterations in gonadotropin releasing hromone pulsatality leads to preferential production of LH, as compared to FSH. Also estrogen has a positive feedback on LH and negative feedback on FSH which leads to increase in LH and decrease in FSH such that LH/FSH is > 2.1. Now the question arises whether this increase in LH is persistent. Most of the books do not state anything clearly except that there is an increase in the LH pulse amplitude and frequency. So I had to look up in Leon Speroff (which is the BAAP of all problems related to endocrinology and infertility in Gynae)." "This is a real-world medical entrance exam question, please give the true answer based on the question and selection. Topic name: None, Subject name: Pharmacology","Which is gametocidal for all species: Select and explain. A. Quinine, B. Chloroquine, C. Primaquine, D. None",C: Primaquine,"Answer: C: Primaquine Explanation: Ans: C. PrimaquineRef: Goodman Gilman's The Pharmacological Basis of Therapeutics 13"" edn; Page no. 980, NVBD GuidelinesDrugsActionChloroquineSchizontocidal for all speciesGametocidal for PV, PO and PMNo action on hypnozoitesQuininePrimary blood schizontocidalLittle effect on sporozoiteGametocidal to PV and PMAemisinin groupBlood schizontocidal Gametocidal action recently describedMefloquineStrong schizontocidal action against all speciesGametocidal against PV, PM and POSporonticidal actHalofantrineSchizontocidal to all speciesNo action on latent tissue form of PV and gametocytesAtovaquoneBlood schizontocidal (Used primarily for MDR PF)PyronaridineSchizontocidal for PF, PV and MDR PFSulfadoxine -PyrimethamineActive against blood schizonts of PF. Less active against other speciesPrimaquineDestroys late hepatic stage and latent forms of PV and POGametocidal to all species, mainly PF.No action on erythrocyte stage of PF, though activeProguanilWeak schizontocidal action against all species." "This is a real-world medical entrance exam question, please give the true answer based on the question and selection. Topic name: None, Subject name: Radiology","Radiation effect blood supply because of Select and explain. A. Anemia, B. Endarteritis of small blood vessels, C. Infection, D. All of the above",B: Endarteritis of small blood vessels,"Answer: B: Endarteritis of small blood vessels Explanation: None" "This is a real-world medical entrance exam question, please give the true answer based on the question and selection. Topic name: None, Subject name: Microbiology","Lepromin test is useful for Select and explain. A. Diagnosis, B. Prognosis, C. Treatment plan, D. Epidemiology",B: Prognosis,"Answer: B: Prognosis Explanation: None" "This is a real-world medical entrance exam question, please give the true answer based on the question and selection. Topic name: None, Subject name: Dental","A statistical test which indicates the chance or probability of an observed difference between two means occurring by chance is called, Select and explain. A. Tests of significance, B. Means, C. Ratio, D. Normalcy",A: Tests of significance,"Answer: A: Tests of significance Explanation: None" "This is a real-world medical entrance exam question, please give the true answer based on the question and selection. Topic name: AIIMS 2019, Subject name: Medicine","Which of these is the Renal feed for CKD patients? Select and explain. A. Low calorie low volume, B. Low calorie high volume, C. High calorie low volume, D. High calorie high volume",C: High calorie low volume,"Answer: C: High calorie low volume Explanation: OPTION C - CKD patients have catabolic metabolism so they will require higher caloric as lot of muscle wasting. Since the disease CKD has affected kidney, we prefer to give low volume due to decrease capacity of kidney to excrete." "This is a real-world medical entrance exam question, please give the true answer based on the question and selection. Topic name: None, Subject name: Pathology","Hemophilics show: Select and explain. A. Increased bleeding time and clotting time, B. Decreased bleeding time and clotting time, C. Decreased bleeding time and increased clotting time, D. Normal bleeding time and increased clotting time",D: Normal bleeding time and increased clotting time,"Answer: D: Normal bleeding time and increased clotting time Explanation: None" "This is a real-world medical entrance exam question, please give the true answer based on the question and selection. Topic name: None, Subject name: Dental","Process shown in color plate is called: Select and explain. A. Mandibular mesial root resection, B. Mandibular distal root resection, C. Hemisection., D. All of the above",C: Hemisection.,"Answer: C: Hemisection. Explanation: None" "This is a real-world medical entrance exam question, please give the true answer based on the question and selection. Topic name: None, Subject name: Medicine","Tourniquet test is used in daily follow-up of patients with: Select and explain. A. Zika virus, B. Dengue virus, C. Chikungunya, D. Swine flu",B: Dengue virus,"Answer: B: Dengue virus Explanation: Ans: B. Dengue virus(Ref: Harrison 19/e p1322)Tourniquet test is used in daily follow up of patients with dengue virusThe tourniquet test (capillary-fragility test):Pa of the new WHO case definition for dengue.The test is a marker of capillary fragility and it can be used as a triage tool to differentiate patients with acute gastroenteritis, for example, from those with dengue.It is a clinical diagnostic method to determine a patient's hemorrhagic tendency, fragility of capillary walls and thrombocytopenia." "This is a real-world medical entrance exam question, please give the true answer based on the question and selection. Topic name: None, Subject name: Biochemistry","What high energy phosphate compound is formed in the citric acid cycle through substrate level phosphorylation: Select and explain. A. ATP, B. TTP, C. ITP, D. GTP",D: GTP,"Answer: D: GTP Explanation: None" "This is a real-world medical entrance exam question, please give the true answer based on the question and selection. Topic name: None, Subject name: Dental","When instrument with primary cutting edge is perpendicular to blade then: Select and explain. A. 1st digit of 4 digit formula omitted., B. 2nd digit of 4 digit formula omitted., C. 3rd digit of 4 digit formula omitted., D. 4th digit of 4 digit formula omitted.",B: 2nd digit of 4 digit formula omitted.,"Answer: B: 2nd digit of 4 digit formula omitted. Explanation: None" "This is a real-world medical entrance exam question, please give the true answer based on the question and selection. Topic name: None, Subject name: Dental","A rest seat is prepared as: Select and explain. A. Spoon shaped depression with the apex towards the center of the tooth, B. Concavity with hemisphere shape, C. Elliptical shape, D. Rhomboid depression limited to enamel",A: Spoon shaped depression with the apex towards the center of the tooth,"Answer: A: Spoon shaped depression with the apex towards the center of the tooth Explanation: None" "This is a real-world medical entrance exam question, please give the true answer based on the question and selection. Topic name: None, Subject name: Pathology","A patient showing inability to close the right corner of the mouth is most probably suffering from: Select and explain. A. Myasthenia gravis, B. Bell's palsy, C. TMJ dysfunction syndrome, D. Multiple sclerosis",B: Bell's palsy,"Answer: B: Bell's palsy Explanation: Bell's palsy is manifested by drooping of corner of mouth, drooling of saliva, watering of eye, inability to blink the eye. The patient has a typical mask-like or expressionless appearance. The patient will have speech difficulty and occasionally the taste sensation to anterior portion of tongue is lost or altered. In supranuclear lesions of facial nerve, only the lower part of the face is paralysed. The upper part (frontalis and part of orbicularis oculi) escapes due to bilateral representation in the cerebral cortex. In infranuclear lesions of facial nerve (Bell's palsy) half of the whole of face is paralysed. The face becomes asymmetrical, and any attempt to smile draws the mouth to the NORMAL SIDE." "This is a real-world medical entrance exam question, please give the true answer based on the question and selection. Topic name: None, Subject name: Anatomy","Tensor veli palatini is supplied by: Select and explain. A. Facial nerve, B. Trigeminal nerve, C. Glossopharyngeal nerve, D. Pharyngeal plexus",B: Trigeminal nerve,"Answer: B: Trigeminal nerve Explanation: None" "This is a real-world medical entrance exam question, please give the true answer based on the question and selection. Topic name: None, Subject name: Dental","According to ISO 6876-2001 ADA specifications 57 Endodontics sealer penetration is upto? Select and explain. A. 150 μm, B. 50 μm, C. 300 μm, D. 100 μm",B: 50 μm,"Answer: B: 50 μm Explanation: None" "This is a real-world medical entrance exam question, please give the true answer based on the question and selection. Topic name: None, Subject name: Medicine","Which of the following disease cause damage of vertebral body & intervertebral space? Select and explain. A. Tuberculosis, B. Multiple myeloma, C. Lymphoma, D. Metastasis",A: Tuberculosis,"Answer: A: Tuberculosis Explanation: None" "This is a real-world medical entrance exam question, please give the true answer based on the question and selection. Topic name: None, Subject name: Dental","Mechanical objectives of cleaning ans shaping are given by: Select and explain. A. Schilder, B. Wein, C. Brook, D. Grossman",A: Schilder,"Answer: A: Schilder Explanation: None" "This is a real-world medical entrance exam question, please give the true answer based on the question and selection. Topic name: None, Subject name: Medicine","All of the following are features of glucocoicoid deficiency except: Select and explain. A. Fever, B. Hyperkalemia, C. Postural hypotension, D. Weight loss",B: Hyperkalemia,"Answer: B: Hyperkalemia Explanation: Ans: B. Hyperkalemia(Ref: Harrison 19/c p2325, 18/c p2957)Hyperkalemia is seen in mineralocoicoid deficiency but not in isolated glucocoicoid deficiency.Fever is due to release of inflammatory mediators, which is suppressed by steroids, is seen in glucocoicoid deficiency." "This is a real-world medical entrance exam question, please give the true answer based on the question and selection. Topic name: None, Subject name: Pediatrics","Transient tachypnea of new born (TTN) is commonly seen in which of the following situations – Select and explain. A. Term delivery requiring forceps, B. Term requiring ventouse, C. Elective caesarean section, D. Normal vaginal delivery",C: Elective caesarean section,"Answer: C: Elective caesarean section Explanation: In text books, both elective caesarian section and normal preterm or term vaginal delivery have been mentioned as risk factors for transient tachypnea of newborn. But the best answer is caesarian section - ""Delivery by caesarian section and gestational age are the risk factors for TTN"". —Articles Obs & Gynae Transient tachvpnea of Newborn (TTN) Transient tachypnea of the newborn is a benign self-limiting disease occuring usually in term neonates and is due to delayed clearance of lung fluid. It is also called respiratory distress syndrome type H TTN follows - Uneventful normal preterm or term vaginal delivery Cesarean delivery TTN is believed to be secondary to slow absorption of fetal lung fluid resulting in decreased pulmonary compliance and tidal volume and increased dead space therefore also known as wet lung. Clinical manifestations Early onset of tachypnea El Sometimes refraction or expiratory grunting u Occasionally cyanosis Patients usually recover rapidly within 3 days. Hypoxemia, hypercapnia and acidosis are uncommon." "This is a real-world medical entrance exam question, please give the true answer based on the question and selection. Topic name: None, Subject name: Dental","KRI paste composition is: Select and explain. A. Iodoform+ZOE, B. Iodoform+chlorophenol+menthol, C. Iodoform+CaOH, D. Iodoform only",B: Iodoform+chlorophenol+menthol,"Answer: B: Iodoform+chlorophenol+menthol Explanation: None" "This is a real-world medical entrance exam question, please give the true answer based on the question and selection. Topic name: None, Subject name: Dental","Steepest cusp is seen in: Select and explain. A. Maxillary 1st premolar, B. Maxillary 1st molar, C. Mandibular IInd molar, D. Mandibular 1st molar",A: Maxillary 1st premolar,"Answer: A: Maxillary 1st premolar Explanation: None" "This is a real-world medical entrance exam question, please give the true answer based on the question and selection. Topic name: None, Subject name: Dental","Which of the following is true about Chi-square test? Select and explain. A. Measures qualitative data, B. Measures both qualitative and quantitative data, C. Measures the qualitative data between two proportion, D. Measure the quantitative data between two proportion",C: Measures the qualitative data between two proportion,"Answer: C: Measures the qualitative data between two proportion Explanation: None" "This is a real-world medical entrance exam question, please give the true answer based on the question and selection. Topic name: None, Subject name: Dental","After 24 hrs. of setting tensile stength of GIC is: Select and explain. A. Equal to Znpo4, B. Greater to Znpo4, C. Lesser to Znpo4, D. None of the above",B: Greater to Znpo4,"Answer: B: Greater to Znpo4 Explanation: None" "This is a real-world medical entrance exam question, please give the true answer based on the question and selection. Topic name: None, Subject name: Pathology","""Strawberry Tongue"" is associated with Select and explain. A. syphilis, B. measles, C. scarlet fever, D. typhoid",C: scarlet fever,"Answer: C: scarlet fever Explanation: None" "This is a real-world medical entrance exam question, please give the true answer based on the question and selection. Topic name: None, Subject name: Biochemistry","Thiamine is a cofactor for all of the following enzymes except: Select and explain. A. Alpha ketoglutarate dehydrogenase, B. Branched-chain keto-acid dehydrogenase, C. Succinate dehydrogenase, D. Pyruvate dehydrogenase.",C: Succinate dehydrogenase,"Answer: C: Succinate dehydrogenase Explanation: Ans: C. Succinate dehydrogenaseThiamin as coenzyme:Catalyzes oxidative decarboxylation reactions.3 multi-enzyme complexes catalyzing oxidative decarboxylation reactions:Branched-chain ketoacid dehydrogenase - Involved in metabolism of leucine, isoleucine & valineAlpha-ketoglutarate dehydrogenase - In citric acid cyclePyruvate dehydrogenase - In carbohydrate metabolismTransketolase reaction - In pentose phosphate pathway.Succinate dehydrogenase:Involved in redox reaction catalyzed by FMN & FAD." "This is a real-world medical entrance exam question, please give the true answer based on the question and selection. Topic name: None, Subject name: Dental","Name the type of cry which is a sort of coping mechanism to an unpleasant stimuli: Select and explain. A. Obstinate cry, B. Hurt cry, C. Compensatory cry, D. Frightened cry",C: Compensatory cry,"Answer: C: Compensatory cry Explanation: CRY ELSBACH IN 1963 There are 4 Types A. Obstinate Cry:  Loud, high pitched. Siren like weil. External response to anxiety. B. Hurt Cry:  Loud & more frequent. Tears rolling down on the cheeks. Without the child making any resistance to treatment procedure. C. Compensatory Cry: No cry at all. Sort of coping mechanism to an unpleasant stimuli. Not high or low – It’s MONOTONOUS. No tears. D. Frightened Cry:  Sharp shrilled, extremely high pitched cry. Torrent of tears with convulsive breath catching sobs." "This is a real-world medical entrance exam question, please give the true answer based on the question and selection. Topic name: None, Subject name: Pediatrics","A five year old child presents with left ventricular hypertrophy and central cyanosis what is the most probable diagnosis – Select and explain. A. Tricuspid atresia, B. Eisenmenger syndrome, C. Tetrology of Fallot, D. Total anomalous pulmonary venous drainage",A: Tricuspid atresia,"Answer: A: Tricuspid atresia Explanation: ""Left axis deviation and left ventricular hypertrophy are usually present on electrocardiogram distinguishing tricuspid atresia from most other cyanotic heart lesions. The combination of cyanosis and left axis deviation is highly suggestive of tricuspid atresia."" - Nelson irk p. 1913 Tricuspid atresia Congenital absence of tricuspid valve is called tricuspid atresia TA has following features : - Absence of tricuspid valve Hypoplastic right ventricle with absence of inflow portion of RV So, blood can not flow directly from right atrium to right ventricle and following associated defect are neccessary for survival : - For flow of blood from RA to LA Patent foramen ovale or ASD. For flow of blood from LV to RV --> VSD (VSD is always muscular in tricuspid atresia) Therefore blood flow in following direction." "This is a real-world medical entrance exam question, please give the true answer based on the question and selection. Topic name: None, Subject name: Dental","In dental materials setting time is measured by which test Select and explain. A. Rockwell test, B. Cold blend test, C. Vickers test, D. Vicat needle method",D: Vicat needle method,"Answer: D: Vicat needle method Explanation: None" "This is a real-world medical entrance exam question, please give the true answer based on the question and selection. Topic name: AIIMS 2018, Subject name: Forensic Medicine","In case of professional misconduct Patients records on demand should be provided within? Select and explain. A. 36 hours, B. 36 hours, C. 72 hours, D. 7 days",C: 72 hours,"Answer: C: 72 hours Explanation: According to MCI Regulations- Patient's records should be maintained upto 3 years for inpatients Routine case records maintained - 6 Yrs after completion of treatment & 3 Yrs after death. Records should be provided within 72 hours after request by patient or relatives" "This is a real-world medical entrance exam question, please give the true answer based on the question and selection. Topic name: None, Subject name: Dental","The number of sites examined to assess the stages of bone maturation in Fishman's skeletal maturation index are: Select and explain. A. 4, B. 5, C. 6, D. 7",C: 6,"Answer: C: 6 Explanation: None" "This is a real-world medical entrance exam question, please give the true answer based on the question and selection. Topic name: None, Subject name: Anatomy","All of the following muscles are derived from pharyngeal arches except: Select and explain. A. Tensor tympani, B. Levator palpebrae superioris, C. Palatine tensor, D. Orbicularis oculi",B: Levator palpebrae superioris,"Answer: B: Levator palpebrae superioris Explanation: Levator palpebrae superioris is a somatic muscle that develops from the mesoderm. Tensor tympani & palatine tensor (tensor veli palatine) develops from first pharyngeal arch while orbicularis oculi (muscles of facial expression) develops from the second pharyngeal arch (facial nerve)" "This is a real-world medical entrance exam question, please give the true answer based on the question and selection. Topic name: None, Subject name: Surgery","In a polytrauma patient, chest tube insertion caused sudden drainage of 1,500 mL blood from left side. Oxygen saturation is 92%. What should be the immediate next step: Select and explain. A. Transfer to trauma center, B. Prepare for immediate open thoracotomy, C. Auscultate for left chest breath sounds and wait for saturation to rise, D. Clamp the chest tube",D: Clamp the chest tube,"Answer: D: Clamp the chest tube Explanation: None" "This is a real-world medical entrance exam question, please give the true answer based on the question and selection. Topic name: None, Subject name: Dental","As compared to relining, in rebasing of a denture a change is effected in: Select and explain. A. Centric occlusion, B. Centric relation, C. Entire denture base, D. Tissue surface",C: Entire denture base,"Answer: C: Entire denture base Explanation: None" "This is a real-world medical entrance exam question, please give the true answer based on the question and selection. Topic name: None, Subject name: Dental","Dentin island are frequently found in the root canals of which of the following permanent teeth Select and explain. A. Upper 2nd Molar, B. Upper 2nd Premolar, C. Lower 2nd Molar, D. Lower 2nd Premolar",B: Upper 2nd Premolar,"Answer: B: Upper 2nd Premolar Explanation: None" "This is a real-world medical entrance exam question, please give the true answer based on the question and selection. Topic name: None, Subject name: Dental","Which of the following muscle helps in depressing the mandible: Select and explain. A. Temporalis, B. Massetor, C. Lateral pterygoid, D. Medial pterygoid",C: Lateral pterygoid,"Answer: C: Lateral pterygoid Explanation: None" "This is a real-world medical entrance exam question, please give the true answer based on the question and selection. Topic name: None, Subject name: Biochemistry","A 48-year old lady presented with bony pain and hepatosplenomegaly. On examination of biopsy speci!men from spleen, crumpled tissue paper appearance is seen. Which of the following product is likely to have accumulated? Select and explain. A. Ganglioside, B. Sulfatide, C. Sphingomyelin, D. Glucocerebroside",D: Glucocerebroside,"Answer: D: Glucocerebroside Explanation: Ans: D. GlucocerebrosideGaucher's disease:* MC lysosomal storage disorder, autosomal recessive in inheritance.* Caused by deficiency of tissue enzyme glucocerebrocidase.Glucocerebrocidase:* Splits glucose from glucosyl ceramide. Hence, accumulates glucocerebroside.* Glucosyl ceramide - Cerebrocide accumulates in cell of reticuloendothelial system.Clinical history:* Hepatosplenomegaly with crumpled tissue paper appearance on biopsy - Highly suggestive of Gaucher's disease.* Pancytopenia & thrombocytopenia secondary to hypersplenism.* Widened marrow cavity - Due to Gaucher's cells deposition. - Expansion of bone is prominent - Especially at lower end of femur & humerus. - Causes pathologic fracture & bone pain.Treatment:* Enzyme replacement therapy done with natural or recombinant glucocerebrocidase.(Ref Harrison 19/e p432-e5; Robbins 9/e p151-154, 8/e p153; Nelson 20/e p708, 19/e p487, 488, 500)" "This is a real-world medical entrance exam question, please give the true answer based on the question and selection. Topic name: None, Subject name: Medicine","Drug of choice in acute bacterial meningitis Select and explain. A. Erythromycin, B. Ceftriaxone, C. Sulfisoxazole, D. Cefoperazone",B: Ceftriaxone,"Answer: B: Ceftriaxone Explanation: None" "This is a real-world medical entrance exam question, please give the true answer based on the question and selection. Topic name: None, Subject name: Pathology","IgA deposits in dermal papilla are characteristically seen in? Select and explain. A. A. Dermatitits herpetiformis, B. B. Pemphigus vulgaris, C. C. Bullous pemphigoid, D. D. IgA dermatosis of childhood",A: A. Dermatitits herpetiformis,"Answer: A: A. Dermatitits herpetiformis Explanation: A. Dermatitits herpetiformisDermatitits herpetiformis is diagnosed by a blood test for IgA antibodies, and by a skin biopsy in which the pattern of IgA deposits in the dermal papillae, revealed by direct immunofluorescence, distinguishes it from linear IgA bullous dermatosis and other forms of dermatitis." "This is a real-world medical entrance exam question, please give the true answer based on the question and selection. Topic name: None, Subject name: Gynaecology & Obstetrics","A drop in fetal heart rate that typically last less than 2 minutes and usually associated with umbilical cord compression is called: Select and explain. A. Early deceleration, B. Late deceleration, C. Variable deceleration, D. Prolonged deceleration",C: Variable deceleration,"Answer: C: Variable deceleration Explanation: None" "This is a real-world medical entrance exam question, please give the true answer based on the question and selection. Topic name: None, Subject name: Pathology","Microscopic examination of chromosomes shape, size and arrangement is known as Select and explain. A. Chromosomal mapping, B. Karyotyping, C. Inheritance testing, D. Genotyping",B: Karyotyping,"Answer: B: Karyotyping Explanation: None" "This is a real-world medical entrance exam question, please give the true answer based on the question and selection. Topic name: None, Subject name: Pathology","Chimerism phenomenon is associated with which of the following- Select and explain. A. Paternity test, B. Maternity test, C. Person identification test, D. organ transplantation case",D: organ transplantation case,"Answer: D: organ transplantation case Explanation: Answer- D. organ transplantation caseChimerism can occur in animals is by organ transplantation, giving one individual tissues that developed from a different genome. For example, transplantation of bone marrow often determines the recipient's ensuing blood type." "This is a real-world medical entrance exam question, please give the true answer based on the question and selection. Topic name: None, Subject name: Surgery","In a middle aged female swelling that presents in the midline of neck is most likely originating from: Select and explain. A. Larynx, B. Trachea, C. Thyroid, D. Hyoid",C: Thyroid,"Answer: C: Thyroid Explanation: None" "This is a real-world medical entrance exam question, please give the true answer based on the question and selection. Topic name: AIIMS 2018, Subject name: Pharmacology","Which of the following drug has gametocidal action all species of Plasmodium? Select and explain. A. Primaquine, B. Chloroquine, C. Quinine, D. None of these",A: Primaquine,"Answer: A: Primaquine Explanation: Primaquine: It is gameticidal against all species of Plasmodium i.e. vivax, ovale, falciparum, malariae. Hence, it is DOC to prevent transmission of malaria. It is also used for Radical cure of vivax and ovale malaria as it can kill hypnozoites as well. Chloroquine and Quinine: Gameticidal against P. vivax species only" "This is a real-world medical entrance exam question, please give the true answer based on the question and selection. Topic name: None, Subject name: Dental","Treatment of diastema because of a thick labial frenum is done: Select and explain. A. After frenectomy, B. Before eruption of canines, C. After eruption of canines, D. Before frenectomy",C: After eruption of canines,"Answer: C: After eruption of canines Explanation: None" "This is a real-world medical entrance exam question, please give the true answer based on the question and selection. Topic name: None, Subject name: Social & Preventive Medicine","Which of the following is not true about screw feed technology Select and explain. A. Reduces weight by 30%, B. Reduces volume by 80%, C. Ideal for pathological waste, D. Non-burn heat sterilization technique",C: Ideal for pathological waste,"Answer: C: Ideal for pathological waste Explanation: Answer- C. Ideal for pathological wasteScrew-Feed Technology:A non-burn, dry thermal disinfection processWaste is reduced by 80% in volumeWaste is reduced by 20-35 % in weightSuitable for treating infectious waste and sharpsShould not be used to process pathological, cytotoxic, or radioactive waste." "This is a real-world medical entrance exam question, please give the true answer based on the question and selection. Topic name: None, Subject name: Physiology","Bacteria are most commonly ENGULFED by: Select and explain. A. Neutrophilic leukocytes, B. Large granular lymphocytes, C. Small lymphocytes, D. Killer cells",A: Neutrophilic leukocytes,"Answer: A: Neutrophilic leukocytes Explanation: None" "This is a real-world medical entrance exam question, please give the true answer based on the question and selection. Topic name: None, Subject name: Pathology","Which of the following is true about intracellular iron homeostasis in iron deficiency anemia? Select and explain. A. Transferrin receptor-1 iron responsive elements increase transferrin receptor mRNA concentration and synthesis, B. Transferrin receptor-1 iron responsive elements decrease transferrin receptor mRNA concentration and synthesis, C. Apoferritin mRNA iron response element decreases and ferritin synthesis decreases, D. Apoferritin mRNA iron response element decreases and ferritin synthesis increases",A: Transferrin receptor-1 iron responsive elements increase transferrin receptor mRNA concentration and synthesis,"Answer: A: Transferrin receptor-1 iron responsive elements increase transferrin receptor mRNA concentration and synthesis Explanation: Answer- A. Transferrin receptor-1 iron responsive elements increase transferrin receptor mRNA concentration and synthesisIn iron deficiency anemia, transferrin receptor-I iron responsive elements increase transfetin receptor mRNA concentration and synthesis.""The regulation of iron metabolism at the cytoplasmic mRNA level by interaction of iron regulalory protein (IRP-I) and the iron-responsive elements (IREs) to apoferritin mRNA and transferrin receptor mRNA. When the cytoplasmic iron concentration is low, IRP-I binds to the IREs of both mRNAs." "This is a real-world medical entrance exam question, please give the true answer based on the question and selection. Topic name: None, Subject name: Pathology","Which of the following is anaplastic lymphoma kinase (ALK) positive neoplasm? Select and explain. A. Synol sarcoma, B. Fibromatosis, C. Ewing sarcoma, D. Inflammatory myofibroblastic tumor",D: Inflammatory myofibroblastic tumor,"Answer: D: Inflammatory myofibroblastic tumor Explanation: Answer- D. Inflammatory myofibroblastic tumorInflammatory myofibroblastic tumor is anaplastic lymphoma kinase (ALK) positive neoplasm.Inflammatory myofibroblastic tumor, though rare, is more common in children, with an equal male-to-female ratio.Presenting symptoms include fever, cough, chest pain, and hemopytsis. It may also be asymptomatic.Anaplastic Lymphoma Kinase (ALK)- Anaplastic lymphoma kinase also known as ALK tyrosine kinase receptor or CD246 (cluster of diff-erentiation 246) is an enzyme that in humans is encoded by ALK gene.Anaplastic Lymphoma Kinase (ALK) Positive NeoplasmsAnaplastic large-cell lymphomasAdenocarcinoma of lungFamilial neuroblastomaInflammatory myofibroblastic tumor" "This is a real-world medical entrance exam question, please give the true answer based on the question and selection. Topic name: None, Subject name: Ophthalmology","%lost radio-resistant cells in retina Select and explain. A. Retinal pigment epithelium, B. Ganglion cell layer, C. Rods and cones, D. Bipolar cells",B: Ganglion cell layer,"Answer: B: Ganglion cell layer Explanation: Ans. b. Ganglion cell layer (Ref Radiation Retinopatliv. ,(1991) 5. 239-251)Ganglion cells are an example of highly specialized cells that have undergone extensive maturation, are the most radioresistant cells in the retina." "This is a real-world medical entrance exam question, please give the true answer based on the question and selection. Topic name: None, Subject name: Pathology","Port wine stains are seen in: Select and explain. A. Nevus, B. Haemangionna, C. Melanoma, D. All the above",B: Haemangionna,"Answer: B: Haemangionna Explanation: None" "This is a real-world medical entrance exam question, please give the true answer based on the question and selection. Topic name: None, Subject name: Ophthalmology","As compared to blood, vitreous humor has high concentration of: Select and explain. A. Sodium, B. Potassium, C. Glucose, D. Ascorbate",D: Ascorbate,"Answer: D: Ascorbate Explanation: Ans: D. Ascorbate(Ref Yanoff and Duker 4/e p353)As compared to blood, vitreous humor has high concentration of ascorbate." "This is a real-world medical entrance exam question, please give the true answer based on the question and selection. Topic name: None, Subject name: Skin","An 8-year-old boy presents with well defined annular lesion over the buttock with central scarring that is gradually progressive over the last 8 months. The diagnosis is Select and explain. A. Annular psoriasis, B. Lupus Vulgaris, C. Tinea Corporis, D. Chronic granulomatous disease",B: Lupus Vulgaris,"Answer: B: Lupus Vulgaris Explanation: B i.e. Lupus vulgarisAnnular plaque with central scarring - Typically seen in Lupus vulgaris.Most common sites in India = Buttocks and face.Cause:The underlying focus of TB (bone, joint or lymph node).Contiguous extension of disease from underlying affected tissue.By hematogenous or lymphatic spread.Arise after exogenous inoculation.A complication of BCG vaccination.Features & manifestations:Cutaneous TB more commonly affects buttocks & extremities rather than the face.Such a pattern is usually due to reinoculation and may relate to playing without clothing or shoes.Lesions:Becomes flat plaques with a serpiginous or polycyclic outline & a smooth surface or psoriasiform scaling: there may be erosions, ulceration, and scarring.Hyperophic forms appear as a soft mass with a nodular, hyperkeratotic surface." "This is a real-world medical entrance exam question, please give the true answer based on the question and selection. Topic name: None, Subject name: Pathology","Which of the following is a reactive lesion of the gingiva that may demonstrate bone radiographically and often even microscopically? Select and explain. A. Osteoma, B. Peripheral ossifying fibroma, C. Traumatic neuroma, D. Irritation fibroma",B: Peripheral ossifying fibroma,"Answer: B: Peripheral ossifying fibroma Explanation: None" "This is a real-world medical entrance exam question, please give the true answer based on the question and selection. Topic name: None, Subject name: Dental","The percentage of fluoride used in iontophoresis is: Select and explain. A. 1%, B. 2%, C. 4%, D. 8%",B: 2%,"Answer: B: 2% Explanation: None" "This is a real-world medical entrance exam question, please give the true answer based on the question and selection. Topic name: None, Subject name: Dental","Glazing of the layer crack is seen because Of Select and explain. A. Thermal shock, B. Contamination, C. Incompatible, D. None",A: Thermal shock,"Answer: A: Thermal shock Explanation: None" "This is a real-world medical entrance exam question, please give the true answer based on the question and selection. Topic name: None, Subject name: Dental","MTA barrier in open apex is made up to? Select and explain. A. 3mm, B. 5mm, C. Depend on length of canal, D. 1mm",B: 5mm,"Answer: B: 5mm Explanation: None" "This is a real-world medical entrance exam question, please give the true answer based on the question and selection. Topic name: None, Subject name: Microbiology","Which of the following is microaerophilic - Select and explain. A. Campylobacter, B. Vibrio, C. Bacteroides, D. Pseudomonas",A: Campylobacter,"Answer: A: Campylobacter Explanation: Microaerophilic: Require small amount of oxygen (5%) e.g. Campylobacter, Helicobacter and Mycobacterium bovis." "This is a real-world medical entrance exam question, please give the true answer based on the question and selection. Topic name: None, Subject name: Surgery","Oncotype Dx test is done to for the following in breast cancer: Select and explain. A. Chemotherapy in hormone receptor positive patients, B. Hormone therapy in hormone positive, C. Chemotherapy in hormone receptor negative patients, D. Herceptin in Her-2-neu +ve.",A: Chemotherapy in hormone receptor positive patients,"Answer: A: Chemotherapy in hormone receptor positive patients Explanation: Ans: A. Chemotherapy in hormone receptor positive patients(Ref Harrison I9/e p528; DX:Multigene tests.Uses:To guide chemotherapy decisions for,Node-negative cases.Hormone receptor-positive cases.HER2-negative breast cancer.Used clinically for early-stage breast cancer predicting recurrence risk & guide adjuvant chemotherapy decisions." "This is a real-world medical entrance exam question, please give the true answer based on the question and selection. Topic name: None, Subject name: Pharmacology","A 25 yr old person with history of repeated episodes of rheumatic fever is hypersensitive to penicillin. Which of following drug can be prescribed to him? Select and explain. A. Penicillin G, B. Sulfisoxazole, C. Sulfasalazine, D. Streptomycin",B: Sulfisoxazole,"Answer: B: Sulfisoxazole Explanation: None" "This is a real-world medical entrance exam question, please give the true answer based on the question and selection. Topic name: AIIMS 2018, Subject name: Pharmacology","Major determinant of loading dose of a drug is:- Select and explain. A. Half life, B. Clearance, C. Volume of distribution, D. Bioavailability",C: Volume of distribution,"Answer: C: Volume of distribution Explanation: LOADING DOSE = Vd * target plasma concentration The loading dose is primarily dependent on the volume of distribution of drugs. The loading dose is administered to achieve rapid action of drugs having high the volume of distribution. MAINTENANCE DOSE = Clearance * target plasma concentration" "This is a real-world medical entrance exam question, please give the true answer based on the question and selection. Topic name: None, Subject name: Anatomy","Auriculotemporal nerve encircles which artery Select and explain. A. Superficial temporal artery, B. Superficial temporal vein, C. Middle meningeal artery, D. Deep temporal",C: Middle meningeal artery,"Answer: C: Middle meningeal artery Explanation: None" "This is a real-world medical entrance exam question, please give the true answer based on the question and selection. Topic name: None, Subject name: Surgery","A young patient has been admitted with A and had massive hemorrhage. He needs to be transfused with large amounts of fluids. Which IV cannula is preferred? Select and explain. A. Grey, B. Green, C. Blue, D. Pink",B: Green,"Answer: B: Green Explanation: Ans: B. Greenused for routine blood transfusions, transfusing large volumes of fluid, intravenous feeding of patients and the harvesting and separation of stem cells.Refi ATLS 18th edn; 2018, Chapter 3 ShockColorSizeExternal diameter(mm)Length(mm)Water flow rate (mL/min)Recommended usesOrange14G2.1 mm45 mm-240 mL/minTrauma, rapid blood transfusion, surgeryGray16G1.8 mm45 mm--180 mL/minRapid fluid replacement, trauma, rapid blood transfusionGreen18G1.3 mm32 mm--90 mL/minLarge volumes of fluid, Rapid fluid replacement, trauma, rapid blood transfusionPink20G1.1 mm32 mm--60 mL/minMost infusions, rapid fluid replacement, trauma, routine blood transfusionBlue22G0.9 mm25 mm--36 mL/minMost infusionsNeonate, pediatric, older adults routine blood transfusion Yellow 24G 0.7 mm 19 mm --20 mL/minMost infusions neonate, pediatric, older adults, routine blood transfusion, neonate or pediatric blood transfusion Purple 26G 0.6 mm 19 mm --13 mL/minPediatrics, Neonate" "This is a real-world medical entrance exam question, please give the true answer based on the question and selection. Topic name: AIIMS 2018, Subject name: ENT","Cranial nerve that is not involved in olfaction:- Select and explain. A. Glossopharyngeal, B. Vagus, C. Hypoglossal, D. Trigeminal",C: Hypoglossal,"Answer: C: Hypoglossal Explanation: Olfaction - 1. Ohonasal (odor in inspired air) 2. Retro nasal (odor in expired air) Food in mouth - swallowing and deglutition 1. Chorda tympani (branch of facial nerve): taste from anterior 2/3rd tongue 2. Lingual nerve: pain, tactile and temperature from anterior tongue 3. Greater superficial petrosal nerve: taste from palate 4. 9th and 10th CN: taste from posterior tongue and throat CN 5,7,9 & 10 help to regulate olfaction. Add smell to taste. Hypoglossal nerve that is pure motor nerve supply muscle of tongue." "This is a real-world medical entrance exam question, please give the true answer based on the question and selection. Topic name: None, Subject name: Dental","Difference between mandibular and maxillary premolars is that in the mandibular premolars: Select and explain. A. The buccal and lingual cusps are more prominent, B. The lingual cusp is sharper and longer, C. The lingual surface rounded and less developed, D. The buccal surface is rounded and Less developed",C: The lingual surface rounded and less developed,"Answer: C: The lingual surface rounded and less developed Explanation: None" "This is a real-world medical entrance exam question, please give the true answer based on the question and selection. Topic name: None, Subject name: Dental","Simple random sampling is ideal for: Select and explain. A. Vaccinated people, B. Heterogenous population, C. Homogenous population, D. All of the above",C: Homogenous population,"Answer: C: Homogenous population Explanation: None" "This is a real-world medical entrance exam question, please give the true answer based on the question and selection. Topic name: None, Subject name: Radiology","Dead bone appears on a radiograph as: Select and explain. A. Radiolucent, B. Radio-opaque, C. Cotton wool appearance, D. Salt and pepper appearance",B: Radio-opaque,"Answer: B: Radio-opaque Explanation: None" "This is a real-world medical entrance exam question, please give the true answer based on the question and selection. Topic name: None, Subject name: Radiology","Which of the following resembles caries in radiographs? Select and explain. A. Zinc oxide eugenol, B. Zinc phosphate, C. GIC, D. Polymethyl methacnilate",D: Polymethyl methacnilate,"Answer: D: Polymethyl methacnilate Explanation: None" "This is a real-world medical entrance exam question, please give the true answer based on the question and selection. Topic name: None, Subject name: Dental","Beveling of the pulpoaxial line angle of a class II cavity is done to: Select and explain. A. Increase the strength of the restoration, B. Improved marginal adaptation, C. To prevent the fracture of enamel, D. To prevent the fracture of amalgam",D: To prevent the fracture of amalgam,"Answer: D: To prevent the fracture of amalgam Explanation: None" "This is a real-world medical entrance exam question, please give the true answer based on the question and selection. Topic name: None, Subject name: Surgery","Most potent topical anesthestic is Select and explain. A. Mepivacaine, B. Prilocaine, C. Benzocaine, D. Tetracaine",D: Tetracaine,"Answer: D: Tetracaine Explanation: None" "This is a real-world medical entrance exam question, please give the true answer based on the question and selection. Topic name: None, Subject name: Biochemistry","A 7-year old boy presented with severe abdominal pain. On examination, he had xanthoma. Blood sample was taken for work-up blood sample had milky appearance of plasma. Which of the following lipoprotein is increased? Select and explain. A. LDL, B. HDL, C. Chylomicron, D. Chylomicron remnants",C: Chylomicron,"Answer: C: Chylomicron Explanation: Ans: C. ChylomicronType I or V hyperlipoproteinemia:Features:Severe abdominal pain, xanthomas & milky appearance of plasma.Elevated levels of chylomicrons.(Ref. Harrison 19/e p2438-2447, 18/e p3148)." "This is a real-world medical entrance exam question, please give the true answer based on the question and selection. Topic name: AIIMS 2019, Subject name: Pharmacology","Which of the following is not used in hea failure? Select and explain. A. Metoprolol, B. Trimetazidine, C. Sacubitril, D. Nesiritide",B: Trimetazidine,"Answer: B: Trimetazidine Explanation: Beta blockers in hea failure -Beta blockers are contraindicated in acute hea failure but they can be used in chronic hea failure. At first beta blockers should be staed at low dose, dose should be increased gradually so that predominant action is on kidneys not on hea. Beta blockers used are - Carvedilol, metoprolol and bisoprolol. Sacubitril - It is NEP(neutral endopeptidase) inhibitor, which is required for metabolism of BNP(brain natriuretic peptide) as a result BNP levels are increased resulting in natriuresis and vasodilation. Thus can be used in CHF. Nesiritide- It is recombinant BNP. It is given through subcutaneous route. Trimetazidine -it is a metabolic modulator. it paially inhibits beta oxidation of fatty acids which results in shifting of metabolism of hea muscles from fatty acids to glucose Which require less amount of oxygen so beneficial for the patient of angina pectoris but not used in hea failure." "This is a real-world medical entrance exam question, please give the true answer based on the question and selection. Topic name: None, Subject name: Pediatrics","The most common gene defect in idiopathic steroid resistant nephrotic syndrome – Select and explain. A. ACE, B. NPHS 2, C. HOX 11, D. PAX",B: NPHS 2,"Answer: B: NPHS 2 Explanation: None" "This is a real-world medical entrance exam question, please give the true answer based on the question and selection. Topic name: None, Subject name: Anatomy","Ligamentum flavum consists of: Select and explain. A. Type-II collagen, B. Type-III collagen, C. Reticulin, D. Elastin",D: Elastin,"Answer: D: Elastin Explanation: Ans. D. ElastinLigamentum flavum connects the laminae of adjacent veebrae.The ligamentum flavum has a high content of elastin. So under tension it can be stretched by 80% without damage.Functions of ligamentum flavum to provide a constant smooth lining to the veebral canal, which is never overstretched in flexion and which never goes slack in extension." "This is a real-world medical entrance exam question, please give the true answer based on the question and selection. Topic name: None, Subject name: Pediatrics","All of following are recognized manifestation of acute Rheumatic fever except –a) Abdominal painb) Epistaxisc) Choread) Subcutaneous nodules Select and explain. A. ac, B. a, C. ad, D. ab",D: ab,"Answer: D: ab Explanation: Subcutaneous nodules and chorea are the major criteria. Epistaxis and abdominal pain are nonspecific and usually do not occur." "This is a real-world medical entrance exam question, please give the true answer based on the question and selection. Topic name: None, Subject name: Pathology","Following are functions of prostaglandins except: Select and explain. A. Increase capillary permeability, B. Uterine contraction, C. Lowers blood pressure, D. Elevates blood pressure",D: Elevates blood pressure,"Answer: D: Elevates blood pressure Explanation: None" "This is a real-world medical entrance exam question, please give the true answer based on the question and selection. Topic name: None, Subject name: Dental","Complete denture teeth are in edge to edge contact in centric occlusion. What should be done? Select and explain. A. Grind the centric holding cusp, B. Grind the lingual fossa of upper and lower incisors, C. Grind the cusp in such a way that the incline of the upper cusp slides buccally and lower cusp slides lingually, D. Grind the cusp in such a way that the incline of the upper cusp slides lingually and lower cusp slides buccally",C: Grind the cusp in such a way that the incline of the upper cusp slides buccally and lower cusp slides lingually,"Answer: C: Grind the cusp in such a way that the incline of the upper cusp slides buccally and lower cusp slides lingually Explanation: None" "This is a real-world medical entrance exam question, please give the true answer based on the question and selection. Topic name: None, Subject name: Anatomy","Position of adenoid is in the ? Select and explain. A. Hypopharynx, B. Oropharynx, C. Trachea, D. Nasopharynx",D: Nasopharynx,"Answer: D: Nasopharynx Explanation: None" "This is a real-world medical entrance exam question, please give the true answer based on the question and selection. Topic name: None, Subject name: Anatomy","Anterior two third of the tongue is demarcated from the posterior one third by: plexus, 2-long thoracic plexus, 2-long thoracic Select and explain. A. Passavant's ridge, B. Sulcus terminalis, C. Circurnvallatepapillae, D. Filifonn papilla",B: Sulcus terminalis,"Answer: B: Sulcus terminalis Explanation: Ans: B. Sulcus terminalis(Ref Gray's 41/e p5 I I, 40/e p503)Anterior two third of the tongue is demarcated from the posterior one-third by sulcus terminalis.Tongue:Divided by a V-shaped sulcus terminalis.Fuher divides tongue into,Anterior, oral (presulcal) pa facing upwards.Posterior, pharyngeal (postsulcal) pa facing posteriorly.Anterior pa forms about two-thirds of tongue length." "This is a real-world medical entrance exam question, please give the true answer based on the question and selection. Topic name: AIIMS 2020, Subject name: Orthopaedics","Fish tail deformity in a child is seen after injury to? Select and explain. A. Distal Tibia, B. Distal Femur, C. Distal humerus, D. Distal Radius",C: Distal humerus,"Answer: C: Distal humerus Explanation: Fishtail deformity of the elbowis characterized by a contour abnormality of the distal humerus, which develops when the lateral trochlear ossification centers fails to develop or resorbs. It is an uncommon complication usually following a distal humeral fracture in childhood.Whilst initially presumed to be a benign condition, long-term follow-up suggests that patients withfishtail deformityare prone to: functional impairment ongoing pain development of early osteoahrosis" "This is a real-world medical entrance exam question, please give the true answer based on the question and selection. Topic name: None, Subject name: Pathology","A radiograph of the mandibular posteriors in a patient reveals radiopacity above the apices of right 1st molar. No restoration or cavity present. There is no pain or swelling and the pulp is vital. The diagnosis is: Select and explain. A. Periapical granuloma, B. Cementoblastoma, C. Radicular cyst, D. Chronic abscess",B: Cementoblastoma,"Answer: B: Cementoblastoma Explanation: Radiographic Features: The tumor mass is attached to the tooth root and appears as a well-circumscribed dense radiopaque mass often surrounded by a thin, uniform radiolucent line. The outline of the affected root is generally obliterated, because of resorption of the root and fusion of the mass to the tooth. The associated tooth is vital, unless coincidentally involved. The lesion is slow-growing and may cause expansion of cortical plates of bone, but is usually otherwise asymptomatic. Ref: Shafer's textbook of oral pathology 7th edition page 736" "This is a real-world medical entrance exam question, please give the true answer based on the question and selection. Topic name: None, Subject name: Dental","Major fimbrial structural component of P. gingivalis is Select and explain. A. Kgr, B. KgP, C. Fim A, D. Fim C",C: Fim A,"Answer: C: Fim A Explanation: None" "This is a real-world medical entrance exam question, please give the true answer based on the question and selection. Topic name: None, Subject name: Gynaecology & Obstetrics","A female of 36 weeks gestation presents with hypertension, blurring of vision and headache. Her blood pressure reading was 180/120 mm Hg and 174/110 mm Hg after 20 minutes. How will you manage the patient? Select and explain. A. Admit the patient and observe, B. Admit the patient, start antihypertensives and continue pregnancy till term., C. Admit the patient, start antihypertensives, MgSO4 and terminate the pregnancy, D. Admit oral antihypertensives and follow up in out-patient department","C: Admit the patient, start antihypertensives, MgSO4 and terminate the pregnancy","Answer: C: Admit the patient, start antihypertensives, MgSO4 and terminate the pregnancy Explanation: In the question, patient is pres enting with Headache Blurring of vision B/P = 180/120 mm of Hg (later 174/110 mm of Hg) i.e. she is a case of severe pregnancy induced hypertension. First step in the management of this case would be to prevent seizures i.e. give MgSO4. Her B/P should be controlled with antihypertensive and since pregnancy is >34 weeks, therefore terminate pregnancy (which is the definitive management)." "This is a real-world medical entrance exam question, please give the true answer based on the question and selection. Topic name: None, Subject name: Biochemistry","Which of the following is not glucogenic? Select and explain. A. Pyruvate, B. Oxaloacetate, C. Acetyl-CoA, D. Lactate",C: Acetyl-CoA,"Answer: C: Acetyl-CoA Explanation: Ans: C. Acetyl-CoA(Ref. Harper 30/e p185, 29/e p187)Acetyl CoA is not a substrate for gluconeogenesis (not glucogenic) and cannot be conveed back to glucose.""Acetyl CoA is not a substrate for gluconeogenesis and cannot be conveed back to glucosee. This is because acetyl CoA cannot be conveed back to pyruvateQ since its carbon backbone is lost in citric acid cycle as CO2.""" "This is a real-world medical entrance exam question, please give the true answer based on the question and selection. Topic name: None, Subject name: Dental","Reduction in the fusion temperature of dental gold casting alloys is caused by presence of: Select and explain. A. Platinum, B. Copper, C. Silver, D. Gold",B: Copper,"Answer: B: Copper Explanation: None" "This is a real-world medical entrance exam question, please give the true answer based on the question and selection. Topic name: None, Subject name: Surgery","Highly vascular tumor of brain and spinal cord in adults: Select and explain. A. Metastasis, B. Pilocytic astrocytoma, C. Hemangioblastoma, D. Cavernous malformation",C: Hemangioblastoma,"Answer: C: Hemangioblastoma Explanation: Answer- C (Hemangioblastoma)Hemangioblastoma represents the MC primary intra-axial tumor in the adult posterior fossa.Occur almost exclusively in the posterior fossa (cerebelluma)Solid or cystic with a mural nodule." "This is a real-world medical entrance exam question, please give the true answer based on the question and selection. Topic name: None, Subject name: Biochemistry","The patient suffered from hypogonadism, failure to thrive, loss of taste and unable to maintain stability. This shows the deficiency of: Select and explain. A. Zinc, B. Chromium, C. Copper, D. Potassium",A: Zinc,"Answer: A: Zinc Explanation: None" "This is a real-world medical entrance exam question, please give the true answer based on the question and selection. Topic name: None, Subject name: Dental","At which stage of Nolla's, the tooth starts erupting? Select and explain. A. Stage 5, B. Stage 6, C. Stage 7, D. Stage 8",B: Stage 6,"Answer: B: Stage 6 Explanation: None" "This is a real-world medical entrance exam question, please give the true answer based on the question and selection. Topic name: None, Subject name: Anatomy","The primitives palate is formed from: Select and explain. A. Frontonasal process., B. 2 palatal process., C. Maxillary process., D. All of the above.",A: Frontonasal process.,"Answer: A: Frontonasal process. Explanation: Palate is formed by fusion of 3 components.  Two palatal processes, derived from maxillary process.  The primitive palate, formed from the fronto nasal process" "This is a real-world medical entrance exam question, please give the true answer based on the question and selection. Topic name: None, Subject name: Dental","Palatogingival groove is seen in Select and explain. A. Mandibular lateral incisor, B. Maxillary lateral incisor, C. Mandibular 1st premolar, D. Maxillary first premolar",B: Maxillary lateral incisor,"Answer: B: Maxillary lateral incisor Explanation: PALATOGINGIVAL GROOVE: In Maxillary central and lateral incisors May contribute to  Periodontal (AND/OR)  Pulpal pathology To detect the effect:  Vitality testing  Probing Radiograph Treatment:  Burning out the groove  Surgical management" "This is a real-world medical entrance exam question, please give the true answer based on the question and selection. Topic name: None, Subject name: Anatomy","Which is not a branch of the external carotid aery supplying nasal septum? Select and explain. A. Sphenopalatine, B. greater palatine, C. superior labial, D. anterior ethmoidal",D: anterior ethmoidal,"Answer: D: anterior ethmoidal Explanation: Ans. D. anterior ethmoidalThe nasal septum also derives its blood supply from :Branches from the external carotid aery are the sphenopalatine aery, the greater palatine aery, the superior labial aery, and the angular aery.The main branches from the interior carotid are the anterior ethmoidal aery, and the posterior ethmoidal aery that supplies the septum, and these derive from the ophthalmic aery." "This is a real-world medical entrance exam question, please give the true answer based on the question and selection. Topic name: AIIMS 2019, Subject name: Pharmacology","Prucalopride is a Select and explain. A. 5HT4 agonist, B. 5HT2b agonist, C. 5HT2b antagonist, D. 5HT1a paial agonist",A: 5HT4 agonist,"Answer: A: 5HT4 agonist Explanation: Prucalopride-* It is a selective 5-HT4 receptor agonist marketed recently in Europe, UK and Canada for the treatment of chronic constipation in women, when other laxatives fail to provide adequate relief.* It activates prejunctional 5-HT4 receptors on intrinsic enteric neurones to enhance release of the excitatory transmitter ACh, thereby promoting propulsive contractions in ileum and more prominently in colon.* Prucalopride is shown to have low affinity for 5-HT1B/1D receptor, as well as for cardiac K+ channels. It is therefore, believed to be free of cardiovascular risk. No Q-T prolongation has been noted during clinical trial.* Side effects are headache, dizziness, fatigue, abdominal pain and diarrhoea; but generally subside during use." "This is a real-world medical entrance exam question, please give the true answer based on the question and selection. Topic name: None, Subject name: Dental","Dicor restoration is: Select and explain. A. Two coloured restorations, B. Heat pressed ceramics, C. Castable ceramic, D. None of the above",C: Castable ceramic,"Answer: C: Castable ceramic Explanation: None" "This is a real-world medical entrance exam question, please give the true answer based on the question and selection. Topic name: None, Subject name: Pathology","Cysts associated with vital teeth are: Select and explain. A. Dentigerous cyst, Globulomaxillary cyst, lateral periodontal cyst, OKC, B. Dentigerous cyst, Globulomaxillary cyst, radicular cyst, C. Dentigerous cyst, OKC, radicular cyst, D. 'B' & 'C'","A: Dentigerous cyst, Globulomaxillary cyst, lateral periodontal cyst, OKC","Answer: A: Dentigerous cyst, Globulomaxillary cyst, lateral periodontal cyst, OKC Explanation: None" "This is a real-world medical entrance exam question, please give the true answer based on the question and selection. Topic name: None, Subject name: Dental","Metal coping seen in upper anteriors is suggestive of: Select and explain. A. Cast post & core, B. PFM crowns, C. Metal coping with high stiffness, D. Metal coping with low stiffness",A: Cast post & core,"Answer: A: Cast post & core Explanation: Radiopaque post and core seen in anterior teeth suggestive of a cast post with core." "This is a real-world medical entrance exam question, please give the true answer based on the question and selection. Topic name: None, Subject name: Anatomy","Common carotid artery divides to ICA & ECA at: Select and explain. A. Cricoid cartilage, B. Thyroid cartilage, C. 1st tracheal ring, D. Cricothyroid membrane",B: Thyroid cartilage,"Answer: B: Thyroid cartilage Explanation: None" "This is a real-world medical entrance exam question, please give the true answer based on the question and selection. Topic name: None, Subject name: Dental","Algophobia is: Select and explain. A. Phobia of closed space., B. Phobia of large space., C. Phobia of pain., D. Phobia of animals.",C: Phobia of pain.,"Answer: C: Phobia of pain. Explanation: None" "This is a real-world medical entrance exam question, please give the true answer based on the question and selection. Topic name: None, Subject name: Anatomy","Which of the following is unpaired bone of facial skeleton: Select and explain. A. Nasal, B. Lacrimal, C. Inferior nasal concha, D. Vomer",D: Vomer,"Answer: D: Vomer Explanation: Mandible and vomer are the two unpaired bones of facial skeleton." "This is a real-world medical entrance exam question, please give the true answer based on the question and selection. Topic name: None, Subject name: ENT","Which of the following technique is used to move tongue away from post pharyngeal wall Select and explain. A. Jaw thrust maneuver, B. Triple maneuver, C. Head maneuver, D. Neck maneuver",A: Jaw thrust maneuver,"Answer: A: Jaw thrust maneuver Explanation: Ans. A. Jaw thrust maneuver* The jaw-thrust maneuver is a first aid and medical procedure used to prevent the tongue from obstructing the upper airways" "This is a real-world medical entrance exam question, please give the true answer based on the question and selection. Topic name: None, Subject name: Dental","Which of the following cry is characterized by loud, high pitched and siren-like wail: Select and explain. A. Obstinate cry., B. Frighten cry., C. Hurt cry., D. Compensatory cry.",A: Obstinate cry.,"Answer: A: Obstinate cry. Explanation: None" "This is a real-world medical entrance exam question, please give the true answer based on the question and selection. Topic name: None, Subject name: Social & Preventive Medicine","A recently delivered woman with a 15 days old child suffering from cough, sneezing and fever needs help. She has no money for transpoation to nearby hospital. Which of the national programme can help this woman? Select and explain. A. JSSK, B. Indira Gandhi YojanaF-IMNCI, C. F-IMNCI, D. Home-based Care",A: JSSK,"Answer: A: JSSK Explanation: Ans: A. JSSK(Ref Park 24/e p476, 23/e p456, 22/e p420; Shishu Suraksha Karyakaram (JSSK), the national programme can help this woman. Janani-Shishu Suraksha Karyakram (JSSK)The initiative entitles all pregnant women delivering in public health institutions to absolutely free and no expense to delivery, including cesarean section.Includes free drugs and consumables, free diet up to 3 days during normal delivery and up to 7 days for cesarean section, free diagnostics, and free blood wherever required.Provides for free transpo from home to institution, between facilities in case of referral and drop back home.Similar entitlements for all sick newborns & infants accessing public health institutions for treatment till 30 days after bih.Aims to eliminate out of pocket expenses incurred by the pregnant women and sick new borns while accessing services at Government health facilities." "This is a real-world medical entrance exam question, please give the true answer based on the question and selection. Topic name: None, Subject name: Dental","2nd Mesiobuccal canal can be seen from naked eyes in what percentage of cases Select and explain. A. 62-70%, B. 90-95%, C. 30%, D. 40-55%",A: 62-70%,"Answer: A: 62-70% Explanation: None" "This is a real-world medical entrance exam question, please give the true answer based on the question and selection. Topic name: None, Subject name: Dental","Direct pulp capping is not done in which of these situations? Select and explain. A. Mechanical exposure, B. No bleeding, C. Pinpoint exposure, D. Carious exposure with more than 1 mm in size",D: Carious exposure with more than 1 mm in size,"Answer: D: Carious exposure with more than 1 mm in size Explanation: “Direct pulp capping is defined as 1 mm2 or less than 1 mm2 exposure due to mechanical exposure during cavity preparation or caries or trauma left behind with a sound surrounding dentin and dressed with a biocompatible radiopaque base in contact with the exposed pulp and should heal the tissue and deposit the reparative dentin prior to placing the restoration.” Reference: Pediatric Dentistry Principles and Practice SECOND EDITION, MS Muthu; pdf no 780" "This is a real-world medical entrance exam question, please give the true answer based on the question and selection. Topic name: None, Subject name: Dental","Tip of ultrasonic scaler after usage, cleaned by Select and explain. A. Under running H2O for 2 min, B. chemical cleaning, C. Autoclave, D. Not Recalled",C: Autoclave,"Answer: C: Autoclave Explanation: None" "This is a real-world medical entrance exam question, please give the true answer based on the question and selection. Topic name: None, Subject name: Social & Preventive Medicine","Which of the following is the best study design to assess in quick time the strength of association between smoking and lung cancer? Select and explain. A. Cross-sectional study, B. Case controlstudy, C. Randomized controlled trial, D. Coho study",B: Case controlstudy,"Answer: B: Case controlstudy Explanation: Ans: B. Case controlstudy(Ref: Park 24/e p76, 23Ie p71; 22/c p76).Best study design to assess in quick time the strength of association between smoking and lung cancer is case control study.Permanent method of epidemiological investigation.Case control studies, often called retrospective studies are a common first approach to test causal hypothesis.Distinct features:Both exposure and outcome (disease) have occurred before the sta of study.Study proceeds backwards from effect to cause.Uses a control or comparison group to suppo or refute an inference." "This is a real-world medical entrance exam question, please give the true answer based on the question and selection. Topic name: None, Subject name: Surgery","Tumor marker for seminoma: Select and explain. A. PLAP, B. LDH, C. AFP, D. HCG",A: PLAP,"Answer: A: PLAP Explanation: Answer- A. PLAPPLAP (Placental alkaline phosphatase):Elevated levels present in as many as 40% patients with advanced disease.Most useful as a marker for bulk disease.Elevated in seminomaa" "This is a real-world medical entrance exam question, please give the true answer based on the question and selection. Topic name: None, Subject name: Anatomy","Nerve supply of the mucosa of larynx, from epiglottis to the level of the cords is : Select and explain. A. Superior laryngeal, B. External laryngeal, C. External laryngeal and recurrent laryngeal, D. Internal laryngeal and recurrent laryngeal",A: Superior laryngeal,"Answer: A: Superior laryngeal Explanation: The larynx is innervated bilaterally by the superior laryngeal nerve (supplies mucosa from the epiglottis to the level of the cords) and the recurrent laryngeal nerve (supplies mucosa below the cords), both branches of the vagus nerve (CN X).." "This is a real-world medical entrance exam question, please give the true answer based on the question and selection. Topic name: None, Subject name: Anatomy","Which of the following muscle is not inseed to the greater tubercle of humerus? Select and explain. A. Supraspinatus, B. lnfraspinatus, C. Teres minor, D. Subscapularis",D: Subscapularis,"Answer: D: Subscapularis Explanation: Ans: D SubscapularisSubscapularis muscle - Largest component of the posterior wall of the axilla.Origin: From subscapularis fossa & also fills it.Inseion: To lesser tubercle of humerus.Inseions of greater tubercle of humerusSupraspinatusInfraspinatusTeres minor" "This is a real-world medical entrance exam question, please give the true answer based on the question and selection. Topic name: None, Subject name: Pharmacology","Adverse effect of Foscarnet includes all except? Select and explain. A. Hypercalcemia, B. Hyperkalemia, C. Hypocalcemia, D. Hypokalemia",B: Hyperkalemia,"Answer: B: Hyperkalemia Explanation: Ans. B. HyperkalemiaPotential adverse effects of foscarnet include :-* Renal impairment, hypo- or hypercalcemia, hypo- or hyperphosphatemia, hypokalemia, and hypomagnesemia.* Penile ulceration associated with Foscarnet therapy may be due to high level of ionized drug in the urine.* Nausea, vomiting, anemia, elevation of liver enzymes, and fatigue have been repoed.* The risk of anemia may be additive in patients receiving concurrent zidovudine.* CNS toxicities include headache, hallucinations, and seizures." "This is a real-world medical entrance exam question, please give the true answer based on the question and selection. Topic name: None, Subject name: Pathology","All of the following statements about acute leukemia in children are true except Select and explain. A. It characteristically causes gross gingival swelling, B. It may he manifested by mucosal pallor, C. It can cause obvious purpura, D. It is usually of the lymphoblastic variety",A: It characteristically causes gross gingival swelling,"Answer: A: It characteristically causes gross gingival swelling Explanation: None" "This is a real-world medical entrance exam question, please give the true answer based on the question and selection. Topic name: None, Subject name: Pediatrics","A 10 days old neonate is posted for pyloric stenosis in surgery. The investigation report shows a serum calcium level of 6 mg/dL. What information would you like to know before you supplement calcium to this neonate – Select and explain. A. Blood glucose, B. Serum protein, C. Serum bilirubin, D. Oxygen saturation",B: Serum protein,"Answer: B: Serum protein Explanation: Normal calcium level is 8.9-10.1 mg/dl (total calcium). Slightly less than half of the total serum calcium exists in free or ionized form. Remainder is bound to protein (mostly albumin). ionized calcium is relevant for cell fuction. ""There are few clinical situations in which the total calcium is not an adequate surrogate for the ionized Ca++ concentration. The most common and severe problem is the presence of hypoalbuminemia"". Each 1 gin/di of albumin in the serum binds about 0.8 mg/dl of calcium. A low total calcium concentration may be normal in a patient with significant hypoalbuminemia. Now, it is clear from above that a low level of serum protein lowers the total plasma calcium but not the ionized calcium, So, before treating hypocalcemia, measure the serum protein level." "This is a real-world medical entrance exam question, please give the true answer based on the question and selection. Topic name: None, Subject name: Dental","In heavy calculus formers, early plaque consists of Ca, PO4 and k in what proportions: Select and explain. A. Calcium > phosphorous > potassium, B. Phosphorous > potassium > calcium, C. Phosphorous > calcium > potassium, D. Potassium > phosphorous > calcium",C: Phosphorous > calcium > potassium,"Answer: C: Phosphorous > calcium > potassium Explanation: None" "This is a real-world medical entrance exam question, please give the true answer based on the question and selection. Topic name: None, Subject name: Pediatrics","Most common cause of Blood stained diaper in a Neonate is : Select and explain. A. Bilharziasis, B. Sickle cell trait, C. Meatal stenosis, D. Urethral hemangioma",D: Urethral hemangioma,"Answer: D: Urethral hemangioma Explanation: It is the answer from exclusion of other options­. Bilharziasis does not occur in neonatal period. Sickle cell trait is usually asymptomatic. Meatal stenosis presents between 3-8 years. Now we are left with option `d' only." "This is a real-world medical entrance exam question, please give the true answer based on the question and selection. Topic name: None, Subject name: Dental","The purpose of pulpotomy in young permanent tooth Select and explain. A. Prevent tooth fracture, B. To induce root formation, C. Formation of hard base to support restoration, D. To minimize infection",B: To induce root formation,"Answer: B: To induce root formation Explanation: None" "This is a real-world medical entrance exam question, please give the true answer based on the question and selection. Topic name: AIIMS 2019, Subject name: Medicine","Deep y descent in JVP is seen in all except:- Select and explain. A. Cardiac tamponade, B. RCM, C. Constrictive pericarditis, D. Tricuspid regurgitation",A: Cardiac tamponade,"Answer: A: Cardiac tamponade Explanation: Option A- Cardiac tamponade- y descent absent as hea can't relax Bag of fluid squeezing hea from outside passive filling of blood in the ventricle. Option D- Tricuspid regurgitation CV wave -feature of severe TR as CV wave is positive. Step y descent Steep x & Step Y causes= Constrictive pericarditis Restrictive cardiomyopathy Rapid ventricular filling || d/t Calcification outside hea Fibrosis of hea( Diastasis impaired) Step x absent Y = in cardiac tamponade | As no relaxation Option B- shows steep y Option C- shows steep y" "This is a real-world medical entrance exam question, please give the true answer based on the question and selection. Topic name: None, Subject name: Forensic Medicine","All of these stains are used in tattooing except: Select and explain. A. Osmium blue, B. Prussian blue, C. Vermillion, D. India ink",A: Osmium blue,"Answer: A: Osmium blue Explanation: Ans: A. Osmium blue(Ref Reddy 34/e 1,89, 33/e p92).Osmium blue:Not used for tattooing.Tattoo marks:Design made in skin by multiple puncture wounds with needles or an electric vibrator dipped in colouring matter.Commonly used dyes:Indian ink, carbon (black), cinnabar or vermilion (mercuric sulphide) red, chromic acid (green), indigo, cobalt, Prussian blue (ferric ferrocyanide), ultramarine (blue).Osmium tetroxide:Used in optical microscopy.Stains lipids.Dissolves in fats & reduced by organic materials to elemental osmium.Osmium - An easily visible black substance." "This is a real-world medical entrance exam question, please give the true answer based on the question and selection. Topic name: None, Subject name: ENT","A patient of carcinoma larynx with stridor presents in casualty, immediate management is - Select and explain. A. Planned tracheostomy, B. Immediate tracheostomy, C. High dose steroid, D. Intubate, give bronchodilator and wait for 12 hours, if no response, proceed to tracheostomy",B: Immediate tracheostomy,"Answer: B: Immediate tracheostomy Explanation: Carcinoma larynx presenting with stridor means it is subglottic laryngeal carcinoma .Ideally in such cases emergency laryngectomy should be performed. “In the case of a large subglottic tumour presenting with respiratory obstruction a case could be made for doing an emergency laryngectomy.” But it is not given in the options: Intubation can not be done as growth is seen in subglottic area therefore tube can not be put. Planned tracheostomy can not be done as patient is suffering from stridor, which is an emergency. Therefore we will have to do emergency tracheostomy. With the precaution that the area of cancer should be removed within 72 hours." "This is a real-world medical entrance exam question, please give the true answer based on the question and selection. Topic name: None, Subject name: Dental","The penetration of the fissure sealant into the fissure: Select and explain. A. Is not influenced by the wettability of the enamel, B. Is independent of the chemical composition of the enamel, C. Is the result of a chemical bond between the sealant and enamel, D. Must occur before the polymerization of the material",D: Must occur before the polymerization of the material,"Answer: D: Must occur before the polymerization of the material Explanation: None" "This is a real-world medical entrance exam question, please give the true answer based on the question and selection. Topic name: AIIMS 2018, Subject name: Gynaecology & Obstetrics","Which of the following markers is not used in quadruple test for antenatal detection of Down syndrome? Select and explain. A. AFP, B. ss-hCG, C. Estradiol, D. Inhibin",C: Estradiol,"Answer: C: Estradiol Explanation: Quadruple test is the second trimester measurement of maternal serum 1 Alpha fetoprotein ( MSAFP) : decreased2 unconjugated estriol: decreased3 HCG: increased4 Inhibin A : increased - It is done between 15 - 20 weeks" "This is a real-world medical entrance exam question, please give the true answer based on the question and selection. Topic name: None, Subject name: Dental","High Cu alloys have all of the following except: Select and explain. A. High tensile strength, B. Low creep, C. High corrosion, D. None of the above",C: High corrosion,"Answer: C: High corrosion Explanation: None" "This is a real-world medical entrance exam question, please give the true answer based on the question and selection. Topic name: None, Subject name: Surgery","When soft palate is paralysed, which is not seen? Select and explain. A. Clefting of the palate, B. Nasal regurgitation, C. Nasal twang, D. Flat palate",A: Clefting of the palate,"Answer: A: Clefting of the palate Explanation: None" "This is a real-world medical entrance exam question, please give the true answer based on the question and selection. Topic name: AIIMS 2018, Subject name: Surgery","Sentinel lymph node biopsy is most useful for: Select and explain. A. Carcinoma cervix, B. Carcinoma endometrium, C. Carcinoma vulva, D. Carcinoma vagina",C: Carcinoma vulva,"Answer: C: Carcinoma vulva Explanation: SLN biopsy is usually done in: CA breast, CA penis & Malignant melanoma SLN biopsy is also applied successfully in cancers of head & neck & vulva" "This is a real-world medical entrance exam question, please give the true answer based on the question and selection. Topic name: AIIMS 2017, Subject name: Forensic Medicine","Atropine is not an antidote in: Select and explain. A. Tik 20, B. Endrin, C. Baygon, D. Parathion",B: Endrin,"Answer: B: Endrin Explanation: Atropine is used for organophosphate and carbamate poisoning but not for organochloride poisoining. Since endrin is an organochloride so atropine is not an antidote for endrin." "This is a real-world medical entrance exam question, please give the true answer based on the question and selection. Topic name: AIIMS 2018, Subject name: Forensic Medicine","As per mental health care act, an individual with a known psychotic disorder on treatment and is not a minor, can choose to decide the caretaker and the course of treatment. This is called as:- Select and explain. A. Advance directive, B. Treatment directive, C. Mental will, D. Future directive",A: Advance directive,"Answer: A: Advance directive Explanation: Advance directives:- Every person except minor has right to take advance directive by writing. It empowers the patient to choose his/her treatment and appoint a representative to take decision on behalf of patient. If patient is minor, his/her parent or caretaker will act as representative." "This is a real-world medical entrance exam question, please give the true answer based on the question and selection. Topic name: None, Subject name: Social & Preventive Medicine","In a population of 10000 people, the prevalence of a disease is 20%. The sensitivity of a screening test is 95% and specificity is 80%. The positive predictive value of the test will be - Select and explain. A. 45.70%, B. 54.30%, C. 15.30%, D. 98.50%",B: 54.30%,"Answer: B: 54.30% Explanation: In the given question, Sensitivity = 95% Specificity = 80% Prevalence = 20% Thus," "This is a real-world medical entrance exam question, please give the true answer based on the question and selection. Topic name: AIIMS 2018, Subject name: Medicine","Which of the following is the most commonly used drug for management of community acquired pneumonia? Select and explain. A. Vancomycin, B. Ceftriaxone, C. Azithromycin, D. Streptomycin",C: Azithromycin,"Answer: C: Azithromycin Explanation: Azithromycin for management of pneumococcal pneumonia Modality Regimen Previously healthy and no antibiotics in past 3 months: A macrolide or Doxycycline (100 mg PO BD) Comorbidities or antibiotics in past 3 months: select an alternative from a different class Fluoroquinolone or A b- lactam plus a macrolide In regions with a high rate of ""high-level"" pneumococcal macrolide resistance,consider alternatives listed above for patients with comorbidities" "This is a real-world medical entrance exam question, please give the true answer based on the question and selection. Topic name: None, Subject name: Dental","Water powder ratio for class IV stone is: Select and explain. A. 0.22-0.24, B. 0.24-0.28, C. 0.65-0.70, D. 0.45-0.50",A: 0.22-0.24,"Answer: A: 0.22-0.24 Explanation: None" "This is a real-world medical entrance exam question, please give the true answer based on the question and selection. Topic name: AIIMS 2019, Subject name: Orthopaedics","Most specific for diagnosing Ankylosing spondyliti? Select and explain. A. HLA B27, B. B/l sactoilitis, C. Lumbar movement, D. ESR",B: B/l sactoilitis,"Answer: B: B/l sactoilitis Explanation: According to Modified New York criteria, the diagnosis of definite AS requires the following: established sacroiliitis on radiographs (unilateral or bilateral) and at least one of the following clinical criteria: (1) low back pain and stiffness for more than 3 months improving with activity, (2) reduced chest expansion (3) limited lumbar movements. HLA B27 is present in > 90% cases of ankylosing spondylitis but it is not specific for AS." "This is a real-world medical entrance exam question, please give the true answer based on the question and selection. Topic name: None, Subject name: Pathology","Abnormality in elastin protein can lead to all except ? Select and explain. A. Fractures, B. Joint laxity, C. Aoic aneurysm, D. Subluxation of lens",A: Fractures,"Answer: A: Fractures Explanation: Ans. A. FracturesRef Clinical Outcomes of Elastin Fibre DeJbcts, J Cytol Histol 201 3, 4: IAbnormality in elastin protein can lead to abnormality in nrany systern.It can lead to aoic aneurysm, joint laxity and subluxation of lens." "This is a real-world medical entrance exam question, please give the true answer based on the question and selection. Topic name: None, Subject name: Medicine","A 40 years old female is diagnosed to have epilepsy and is staed phenytoin and valproate. Four weeks later, she developed a diffuse rash all over her body, which gradually disappeared on stopping both the drugs. In how many weeks, will the rash reappear when she is re-challenged with phenytoin and valproate? Select and explain. A. 1 day, B. 1 week, C. 2 weeks, D. 4 weeks",A: 1 day,"Answer: A: 1 day Explanation: Ans: A. 1 day(Ref: Harrison 19/c p382, 18/c p3263; Goodman Gillman 12/c p592; Andrews' Diseases of the Skin: Clinical Dermatology By William D. Tames Timothy Berger, Dirk Elstoulp 113)Phenytoin Hypersensitivity Syndrome (PHS):Hypersensitivity due to anti-epileptic drugs.Type 4 Hypersensitivity reaction (Steven-Johnson Syndrome).Usually at 3-4 weeks.On re-challenging drug - pre-formed mediators released much quicker - Causes rashes - On same day of staing treatment." "This is a real-world medical entrance exam question, please give the true answer based on the question and selection. Topic name: AIIMS 2018, Subject name: Pharmacology","A patient of HIV is on treatment with indinavir, zidovudine, lamivudine and ketoconazole. He developed nephrolithiasis, hyperlipidemia, central obesity, hyperglycemia and insulin resistance. Which of the following drug is likely to produce above adverse effects? Select and explain. A. Lamivudine, B. Indinavir, C. Zidovudine, D. Ketoconazole",B: Indinavir,"Answer: B: Indinavir Explanation: Protease inhibitors (-navirs) can cause lipodystrophy syndrome characterized by weight gain, increased glucose, increased lipids and insulin resistance. Lipodystrophy syndrome can also be caused by anti-psychotic drugs. Special side effects of various protease inhibitors: Indinavir: Renal stones Tipranavir: Intracranial hemorrhage Saquinavir: QT prolongation. Atazanavir: Hyperbilirubinemia." "This is a real-world medical entrance exam question, please give the true answer based on the question and selection. Topic name: None, Subject name: Medicine","What is the most common tumor of mediastinum? Select and explain. A. Neurogenic, B. Thymoma, C. Lymphoma, D. Metastatic tumor",A: Neurogenic,"Answer: A: Neurogenic Explanation: Answer- A. Neurogenicthe most common mediastinal masses are neurogenic tumors (23%), thymomas (21%), lymphomas (13%) and germ cell tumors (12%). Mediastinal masses are most frequently located in the anterosuperior mediastinum (54%), with the posterior (26%) and middle mediastinum (20%) being less frequently involved" "This is a real-world medical entrance exam question, please give the true answer based on the question and selection. Topic name: None, Subject name: Social & Preventive Medicine","A new method of measuring Haemoglobin levels has been developed. Ten successive readings of a single sample are as follows: 9.4, 10.4, 9.6, 9.1, 10.8, 12.1, 10.1, 9.8, 9.2, 9.5. But the Haemoglobin measured by standard calorimetry was 10.2. Therefore the given method has Select and explain. A. Low validity, low reliability, B. High validity, low reliability, C. High validity, high reliability, D. Low validity, high reliability","B: High validity, low reliability","Answer: B: High validity, low reliability Explanation: Reliability is precision (repeatability) and Validity is accuracy (close to true/actual value) In the given question, 10 successive readings are all different and they have a mean value of 9.4+10.4+9.6+9.1+10.8+12.1+10.1+9.8+9.2+9.5/10 = 10.0 Thus it has low reliability (non-consistent) and high validity (close to true/actual value of 10.2)" "This is a real-world medical entrance exam question, please give the true answer based on the question and selection. Topic name: None, Subject name: Dental","Using stopko tip irrigation which of these is used cleaning and drying of pulp chamber: Select and explain. A. 5.25% NaOCl + 17% EDTA, B. 95% alcohol+17% EDTA, C. 5.25 NaOCl+17% CHX, D. 95% ethanol + 17% EDTA",D: 95% ethanol + 17% EDTA,"Answer: D: 95% ethanol + 17% EDTA Explanation: Direct lines from book ""warming of NaOCI to 122° F (50' C) leads to increase in collagen dissolution as potential to disinfecting, but it can leads to detrimental corrosion effects on NiTi instruments immersion for 1 hr""." "This is a real-world medical entrance exam question, please give the true answer based on the question and selection. Topic name: None, Subject name: Pediatrics","Ideal route of drug delivery in neonatal resuscitation is: Select and explain. A. Intraosseous, B. Through umbilical vein, C. Through peripheral vein, D. Through umbilical aery",B: Through umbilical vein,"Answer: B: Through umbilical vein Explanation: Ans: B. Through umbilical vein(Ref: Ghai 8/e p132).Umbilical vein - Preferred route for drug delivery during resuscitation.Due to ease of approach.Veins in scalp or extremities are difficult to access during resuscitation.For umbilical vein catheterization, 3.5 Fr or 5 Fr umbilical catheter inseed into umbilical vein such that its tip is just inside the skin surface and there is free flow of blood.Direct injection into umbilical cord is not desirable.No intracardiac injection recommended." "This is a real-world medical entrance exam question, please give the true answer based on the question and selection. Topic name: AIIMS 2019, Subject name: Gynaecology & Obstetrics","Which of the following drugs used for management of preterm labor for also has Neuro- protective role in fetus:- Select and explain. A. MgSO4, B. Nifedipine, C. Ritodrine, D. Isoxsuprine",A: MgSO4,"Answer: A: MgSO4 Explanation: Preterm labour is labour staing before 37 weeks MANAGEMENT FOR PRETERM LABOUR :- 1) For lung maturity - steroids 2) Tocolytics E.g. - Nifedipine - First line & safest drug MgSo4 - Neuroprotective Very low bih wt. neonates whose mothers were treated with MgSo4 for preterm labor or preeclampsia were found to have a reduced incidence of cerebral palsy at 3 years. Rate of both neonatal death & cerebral palsy were lower in the Mg treated group." "This is a real-world medical entrance exam question, please give the true answer based on the question and selection. Topic name: None, Subject name: Dental","Most modern precapsulated dental amalgam formulated with Select and explain. A. 50-55% hg by wt., B. 40-45% hg by wt., C. 25-35% hg by wt., D. 15-25% hg by wt.",B: 40-45% hg by wt.,"Answer: B: 40-45% hg by wt. Explanation: None" "This is a real-world medical entrance exam question, please give the true answer based on the question and selection. Topic name: None, Subject name: Physiology","Which of the following hormones will be affected most after the change in sex hormone binding globulin? Select and explain. A. Testosterone, B. Estrogen, C. Progesterone, D. DHEA",A: Testosterone,"Answer: A: Testosterone Explanation: Ans: A. Testosterone(Ref Ganong 25/e p 422, 24/e p424; Harper 30/e p 516. 517)Testosterone binds to SHBG with higher affinity than does estradiol.Hence SHBG level change a Greater change in free testosterone level than in free estradiol.Sex hormone binding globulin:Glycoprotein binding sex steroids with high affinity as.Order:DHT>Testosterone>>Esterone/ estradiol. Due to higher SHBG affinity for DHT & Testosterone, SHBG variation has profound effects on androgens levels." "This is a real-world medical entrance exam question, please give the true answer based on the question and selection. Topic name: None, Subject name: Dental","Chronic carrier state is seen in all except Select and explain. A. Measles, B. Diptheria, C. Typhoid, D. Gonorrhea",A: Measles,"Answer: A: Measles Explanation: None" "This is a real-world medical entrance exam question, please give the true answer based on the question and selection. Topic name: None, Subject name: Dental","Sealants are effective in Select and explain. A. Initial caries, B. Secondary caries, C. Recurrent caries, D. Rampant caries",A: Initial caries,"Answer: A: Initial caries Explanation: None" "This is a real-world medical entrance exam question, please give the true answer based on the question and selection. Topic name: AIIMS 2019, Subject name: Biochemistry","Which of the following change can be done in insulin structure so that there is least change in the function of insulin: Select and explain. A. Interchange of A1 & A4, B. Interchange of B29 and B30, C. Interchange of A5 & A6, D. Breaking disulphide linkages",B: Interchange of B29 and B30,"Answer: B: Interchange of B29 and B30 Explanation: In insulin structure.:- A chain has total 21 Amino Acid. 6th, 7th, 11th, and 20th Amino Acid Are Cysteine and these residuals are responsible for the formation of disulphide bond. If we break disulphide bond in a protein this will lead to denaturation of the protein. B Chain has total 30 amino acid.7th and 19th are cysteine So they also cannot be changed, or if changed will lead to denaturation of the protein Thus options C and D can be ruled out. Interchange of A1 & A4, i.e. glycine and glutamate, which can also be ruled out because glycine is an amino acid which is conserved during evolution in most of the proteins. Interchange of B29 and B30: - The insulin which is used in the treatment - Insulin lispro actually has a change in the carboxy terminal of B chain. So, changes can be made here for changing little bit of propeies and not going to damage the structure much. EXTRA EDGE: Gene coding for insulinis localized on11p15 site of chromosome." "This is a real-world medical entrance exam question, please give the true answer based on the question and selection. Topic name: None, Subject name: Ophthalmology","A 15-year-oldboy presented with headache and blurring of vision. On examination there was diptopia on looking towards left in the right eye.What is your diagnosis? Select and explain. A. Tb meningitis, B. Internuclear ophthalmoplegia, C. Cranial neuritis, D. Demyelination",B: Internuclear ophthalmoplegia,"Answer: B: Internuclear ophthalmoplegia Explanation: Ans: B. Internuclear ophthalmoplegiaDemyelinationRef Bilateral Internuclear ophthalmoplegio in Multiple sclerosis,iejm journal,20l6Typically, symptoms & examination findings i.e. diplopia on looking towards left in the right eye is characteristic of internuclear ophthalmoplegia." "This is a real-world medical entrance exam question, please give the true answer based on the question and selection. Topic name: None, Subject name: Anatomy","Duct of parotid gland crosses over masseter muscle and pierces the: Select and explain. A. Buccinator muscle, B. Superior constrictor, C. Masseter, D. Zygomaticus major",A: Buccinator muscle,"Answer: A: Buccinator muscle Explanation: None" "This is a real-world medical entrance exam question, please give the true answer based on the question and selection. Topic name: None, Subject name: Gynaecology & Obstetrics","A gravida 2 patient with previous LSCS comes at 37 weeks, has BP= 150/100 mm of hg. And on pervaginal examination, cervix is 50% effaced station-3, os is closed and pelvis is adequate. Protein uria is +1, Most appropriate step at the moment would be: Select and explain. A. Antihypertensive regime and wait for spontaneous labor, B. Wait and watch, C. Induce labour, D. caesarean section",C: Induce labour,"Answer: C: Induce labour Explanation: This patient has BP: 150/100 mm hg Proteinuria: +1 Therefore it is classified as mild preeclampsia In mild preeclampsia – if gestational age is > than labour induced should be 37 wks (here in the question = gestational age is >37 weeks). Here BP is 150/100, (Therefore, it is not necessary to start antihypertensive). The NICE clinical guidelines suggest treating moderate hypertension (BP-150/100-159/109 mm Hg) with antihypertensives to keep B I P <150/80-100 range. The benefits or disadvantages of this intervention have not been elucidated by adequate clinical trials. Fernando Arias 4/e, p 209. “There is a consensus that if BP is below 150/100 mm Hg, there is no need for antihypertensive therapy. An exception may be if mild hypertension is associated with markers of potential severe disease or sign of organ dysfunction, (heavy proteinuria, liver dysfunction, hematological dysfuntion)”. Fernando Arias 4/e, p 209. Thus, in this patient, the role of antihypertensive is not confirmatory as BP is 150/100 mm Hg. But role of induction of labor is confirmed, as patient is 37 weeks pregnant with mild hypertension. PIH is not a contraindication for VBAC (Vaginal birth after cesarean) and further more that the pelvis of this patient is adequate – so there is no harm in inducing labour, rather it is advantageous, because it will help in developing lower uterine segment. At any point of time; if there is scar tenderness or if patients BP rises immediately perform cesarean section’ therefore the best answer here is – Antihypertensive regime and then induce labour" "This is a real-world medical entrance exam question, please give the true answer based on the question and selection. Topic name: None, Subject name: Social & Preventive Medicine","Extremely axle insecticide according to WHO classification, are coded as: Select and explain. A. Red, B. Green, C. Yellow, D. Blue",A: Red,"Answer: A: Red Explanation: Answer- A. RedExtremely toxic insecticide, according to WHO classification, is coded as Red." "This is a real-world medical entrance exam question, please give the true answer based on the question and selection. Topic name: None, Subject name: Gynaecology & Obstetrics","A 26 years old female presented with mild pain in lower abdomen. She has had 2 full-term normal delivery earlier. Her last menstrual period was 3 weeks back. On pelvic examination, you find a palpable mass in the adnexa. On USG pelvis, you find a 5 cm ovarian cyst. What should be your next step? Select and explain. A. Observation and follow-up for cyst after 2-3 months, B. CA-125 levels, C. Diagnostic exploratory laparotomy, D. CECT of pelvis",A: Observation and follow-up for cyst after 2-3 months,"Answer: A: Observation and follow-up for cyst after 2-3 months Explanation: Ans: A. Observation and follow-up for cyst after 2-3 months(Ref: Shaw's 15/e p369)The patient is premenopausal, 3 weeks after LMP (likely to have ovulated) with a 5 cm cyst.The scenario presented here likely represents the patient having a corpus luteum cyst.These cysts need not be approached aggressively unless causing significant symptoms." "This is a real-world medical entrance exam question, please give the true answer based on the question and selection. Topic name: None, Subject name: Surgery","Removal of veebral disc can be done by all these approaches except: Select and explain. A. Laminotomy, B. Laminectomy, C. Laminoplasty, D. Hemilaminectomy",C: Laminoplasty,"Answer: C: Laminoplasty Explanation: Ans: C. Laminoplasty(Ref Maheshwari 5/e p257)Removal of veebral disc can be done by all these approaches except laminoplasty.Laminoplasty:Describes process of increasing available space for spinal cord by laminar arch reconstruction by posterior approach. Methods of Removal of DiscFenestrationThe ligamentum flavum bridging the two adjacent laminae is excised and the spinal canal at the level exposed.LaminotomyIn addition to fenestration, a hole is made in the lamina for wider exposure.Hemi-laminectomyThe whole of the lamina on one side is removedLaminectomyThe laminae on both sides, with the spinous process are removed. Such a wide exposure is required for a big, central disc producing cauda equina syndrome." "This is a real-world medical entrance exam question, please give the true answer based on the question and selection. Topic name: None, Subject name: Medicine","Drug not used in H. pylori? Select and explain. A. Metronidazole, B. Omeprazole, C. Mosapride, D. Amoxicillin",C: Mosapride,"Answer: C: Mosapride Explanation: None" "This is a real-world medical entrance exam question, please give the true answer based on the question and selection. Topic name: None, Subject name: Pharmacology","All are true about nitric oxide, EXCEPT: Select and explain. A. Regulates vasomotor tone, B. Acts via cAMP, C. Present in cigarette smoke, D. It is important in penile erection",B: Acts via cAMP,"Answer: B: Acts via cAMP Explanation: Nitric oxide is a potent vasodilator and serves to regulate vascular tone. Produced in the endothelium it freely diffuses to the nearby smooth muscle cell and activates a soluble guanylyl cyclase (sGC) raising cytosolic cGMP concentration. This in turn activates a cGMP-dependent protein kinase (PKG)." "This is a real-world medical entrance exam question, please give the true answer based on the question and selection. Topic name: AIIMS 2018, Subject name: Orthopaedics","A patient of supracondylar humerus fracture is unable to flex interphalangeal joint of the thumb. Which nerve is most likely injured? Select and explain. A. Median nerve, B. Superficial branch of ulnar nerve, C. AIN, D. Pin",C: AIN,"Answer: C: AIN Explanation: Flexion of interphalangeal joint of thumb is carried out by flexor pollicis longus. It is supplied by AIN, a branch of median nerve. Muscles which are supplied by AIN are flexor digitorum profundus (Lateral 1/2 ), flexor pollicis longus and pronator quadratus. PIN a branch of radial nerve causes finger drop Supra condylar fracture of Humerus - Order of nerve involvement in children:AIN > Median Nerve > Radial Nerve > Ulnar Nerve" "This is a real-world medical entrance exam question, please give the true answer based on the question and selection. Topic name: None, Subject name: Surgery","Which of the following is not a cause of clubfoot in newborns? Select and explain. A. CTEV, B. Ahogryposis multiplex cngenita, C. Polio, D. Spina bifida",C: Polio,"Answer: C: Polio Explanation: Answer- C. PolioEtiology of club footIdiopathic (MC) or CTEVSecondary club foot:Neurological disorders and neural tube defects (myelomeningocele, spinal dysraphism)Paralytic disorders as spina bifida, myelodysplasia and Freidreich's ataxiaAhrogryposis multiplex congenitaLarsen syndromeFreeman-Sheldon (Mobius) syndromeDiastrophic dwarfismSacral agenesis, tibial deficiency, constriction rings and amniotic bandsFetal alcohol syndromeDown's syndromeLarsen syndrome" "This is a real-world medical entrance exam question, please give the true answer based on the question and selection. Topic name: None, Subject name: Medicine","Which of the following clinical features of demyelinating myelopathy least likely suggests a progression to multiple sclerosis? Select and explain. A. Complete cord transection, B. Bilateral visual loss, C. Absence of oligoclonal bands, D. Poor prognosis",C: Absence of oligoclonal bands,"Answer: C: Absence of oligoclonal bands Explanation: Answer: c. Absence of oligoclonal bands (Ref Harrison I9/e p2656. 18/e p3372)Oligoclonal bands are variable; but when bands are present, a diagnosis of MS is more likely, thus absence of oligoclonal bands suggests a less likely progression to MS." "This is a real-world medical entrance exam question, please give the true answer based on the question and selection. Topic name: None, Subject name: Medicine","According to 2010 ACLS guidelines, all of the following are true except Select and explain. A. Defibrillation whenever done, it should be done with maximum available energy, B. Atropine is for asystole, C. Immediately call for help in witnessed or unwitnessed cases, D. Cardiac massage resuscitation immediately after defibrillation without waiting for assessing rhythm.",B: Atropine is for asystole,"Answer: B: Atropine is for asystole Explanation: None" "This is a real-world medical entrance exam question, please give the true answer based on the question and selection. Topic name: None, Subject name: Dental","To assess outcome from labour, cost, time Select and explain. A. Cost benefit, B. Cost Effectiveness, C. Cost utility, D. Risk Benefit",A: Cost benefit,"Answer: A: Cost benefit Explanation: None" "This is a real-world medical entrance exam question, please give the true answer based on the question and selection. Topic name: None, Subject name: Surgery","Jersey finger is caused by rupture of: Select and explain. A. Flexor digitorum superficialis, B. Flexor digitorum profundus, C. Extensor digiti minirni, D. Extensor indicis proprius",B: Flexor digitorum profundus,"Answer: B: Flexor digitorum profundus Explanation: Ans: B. Flexor digitorum profundus(Ref Bailey 26/e p465)Rugger jersey finger:Caused by rupture of flexor digitorum profundus.Injury to flexor profundus tendon at its attachment point to distal phalanx.Occurs mostly in American football when a player grabs another player's jersey with the tips of one or more fingers while that player is pulling or running away.""" "This is a real-world medical entrance exam question, please give the true answer based on the question and selection. Topic name: None, Subject name: Pediatrics","Which of the following is least likely in PDA? Select and explain. A. CO, wash out, B. Necrotizing enterocolitis, C. Bounding pulse, D. Pulmonary hemorrhage","A: CO, wash out","Answer: A: CO, wash out Explanation: Ans. a. CO, wash outInfants and children with a small PDA are generally asymptomatic; infants with a large PDA present with signs of hea failure.'Premature newborns can't tolerate PDA, so it results in hea failure, respiratory distress or necrotizing enterocolitis.Premature infants may present with respiratory distress, apnea, worsening mechanical ventilation requirement or other serious complications (e.g. necrotizing enterocolitis)Signs of hea failure occur earlier in premature o infants than in full-term infants and may be more severe.A large ductal shunt in a premature infant often is a major contributor to the severity of the lung disease of prematurity" "This is a real-world medical entrance exam question, please give the true answer based on the question and selection. Topic name: None, Subject name: Biochemistry","Glycoprotein which is Lubricant & Protective Select and explain. A. Mucin, B. Immunoglobulin, C. Ovalbumin, D. None of these",A: Mucin,"Answer: A: Mucin Explanation: Answer- A. Mucin* glycoproteins found in the body is mucins, which are secreted in the mucus of the respiratory and digestive tracts. The sugars when attached to mucins give them considerable water-holding capacity and also make them resistant to proteolysis by digestive enzymes.* O-Linked Glycans: * Mucins, which are found extensively in salivary secretions, contain many sho O-linked glycans. * Increase the viscosity of the fluids" "This is a real-world medical entrance exam question, please give the true answer based on the question and selection. Topic name: None, Subject name: Pathology","Most common tumour of parotid gland is: Select and explain. A. Pleomorphic adenoma, B. Adenoid cystic carcinoma, C. Cylindroma, D. Epidermoid carcinoma",A: Pleomorphic adenoma,"Answer: A: Pleomorphic adenoma Explanation: None" "This is a real-world medical entrance exam question, please give the true answer based on the question and selection. Topic name: None, Subject name: Dental","Which of the following is not a function of guiding plane? Select and explain. A. Provide one path of insertion and removal of the restoration, B. Provide stabilizing characteristics against horizontal rotation of denture, C. Eliminate detrimental strain to abutment teeth during placing or removing restoration, D. Engage the abutment tooth in such a manner as to resist displacement of restoration away from basal seat",D: Engage the abutment tooth in such a manner as to resist displacement of restoration away from basal seat,"Answer: D: Engage the abutment tooth in such a manner as to resist displacement of restoration away from basal seat Explanation: None" "This is a real-world medical entrance exam question, please give the true answer based on the question and selection. Topic name: None, Subject name: Dental","The means by which one part of R.P.D opposes the action of the retainer in the function is called Select and explain. A. Tripoding, B. Reciprocation, C. Retention, D. Stress breaking",B: Reciprocation,"Answer: B: Reciprocation Explanation: None" "This is a real-world medical entrance exam question, please give the true answer based on the question and selection. Topic name: None, Subject name: Pharmacology","All are true about metformin except: Select and explain. A. Inhibits cellular respiration in mitochondria, B. Excreted unchanged in urine, C. Reduce hepatic glucose production, D. Used in decompensated heart failure",D: Used in decompensated heart failure,"Answer: D: Used in decompensated heart failure Explanation: None" "This is a real-world medical entrance exam question, please give the true answer based on the question and selection. Topic name: None, Subject name: Biochemistry","Coenzyme A contains which of the following vitamins: Select and explain. A. Biotin, B. Pyridoxine, C. Pantothenic acid, D. Niacin",C: Pantothenic acid,"Answer: C: Pantothenic acid Explanation: None" "This is a real-world medical entrance exam question, please give the true answer based on the question and selection. Topic name: None, Subject name: Dental","""Multifactorial Causation"" theory was given by Select and explain. A. McMohan Pugh, B. Pettenkofer of Munich, C. Winslow, D. Robert Koch",B: Pettenkofer of Munich,"Answer: B: Pettenkofer of Munich Explanation: None" "This is a real-world medical entrance exam question, please give the true answer based on the question and selection. Topic name: None, Subject name: Anatomy","Lateral movement of condyle takes place by: Select and explain. A. Contralateral lateral pterygoid, B. Ispilateral lateral pterygoid, C. Same side medial pterygoid, D. None of the above",A: Contralateral lateral pterygoid,"Answer: A: Contralateral lateral pterygoid Explanation: Lateral movement of condyle is due to the contraction of contralateral pterygoid: Contraction of right lateral pterygoid- Left lateral working movement. Contraction of left lateral pterygoid- Right lateral working movement" "This is a real-world medical entrance exam question, please give the true answer based on the question and selection. Topic name: None, Subject name: Dental","Causative organism for ANIIG is: Select and explain. A. Fusospirochetal bacilli, B. Treponema pallidum and spirochetes, C. Streptococcus sanguis, D. Sptreptococcus epidermidis",A: Fusospirochetal bacilli,"Answer: A: Fusospirochetal bacilli Explanation: None" "This is a real-world medical entrance exam question, please give the true answer based on the question and selection. Topic name: None, Subject name: Medicine","causes of thyroid storm Select and explain. A. ineffective pre-operative hormonal control, B. manhandling of the thyroid gland during surgery, C. parathyroid tail left after surgery, D. Poor Preoperative preparation of the patient",D: Poor Preoperative preparation of the patient,"Answer: D: Poor Preoperative preparation of the patient Explanation: Answer- D. Poor Preoperative preparation of the patientThis is a rare but life-threatening complication of thyrotoxicosis.The most prominent signs are fever, agitation, delirium, tachycardia or atrial fibrillation and, in the older patient, cardiac failure. Thyrotoxic crisis is a medical emergency and has a moality of 10% despite early recognition and treatment.It is most commonly precipitated by an infection in a patient with previously unrecognized or inadequately treated thyrotoxicosis.It may also develop in known thyrotoxicosis sholy after thyroidectomy in an ill-prepared patient or within a few days of 131I therapy, when acute radiation damage may lead to a transient rise in serum thyroid hormone levels.Patients should be rehydrated and given propranolol, either orally (80 mg 4 times daily) or intravenously (1-5 mg 4 times daily). Sodium iodate (500 mg per day orally) will restore serum T3 levels to normal in 48-72 hours.This is a radiographic contrast medium that not only inhibits the release of thyroid hormones but also reduces the conversion of T4 to T3 and is, therefore, more effective than potassium iodide or Lugol's solution." "This is a real-world medical entrance exam question, please give the true answer based on the question and selection. Topic name: None, Subject name: Dental","This curve is known as Select and explain. A. Cephalocaudal curve, B. Scammon’s curve, C. S shaped curve, D. None of the above",B: Scammon’s curve,"Answer: B: Scammon’s curve Explanation: None" "This is a real-world medical entrance exam question, please give the true answer based on the question and selection. Topic name: None, Subject name: Surgery","Lidocaine is used more commonly in dentistry because lidocaine : Select and explain. A. Causes Less depression of CNS, B. Causes less cardiovascular collapse, C. Causes lesser incidence of allergic reactions, D. Is 50 times more potent than procaine",C: Causes lesser incidence of allergic reactions,"Answer: C: Causes lesser incidence of allergic reactions Explanation: None" "This is a real-world medical entrance exam question, please give the true answer based on the question and selection. Topic name: None, Subject name: Dental","Which material is carried in a custom tray? Select and explain. A. High fusing compound, B. Reversible hydrocolloid, C. Metallic oxide paste, D. Irreversible hydrocolloid",C: Metallic oxide paste,"Answer: C: Metallic oxide paste Explanation: None" "This is a real-world medical entrance exam question, please give the true answer based on the question and selection. Topic name: None, Subject name: Dental","Surface roughness of an orthodontic mini-screw implant can be measured by: Select and explain. A. Gyrometer, B. Androidometer, C. Profilometer, D. Resonance analyzer",C: Profilometer,"Answer: C: Profilometer Explanation: The quality of the surface finish and polish can be characterized by the measurement of the surface roughness using a profilometer, an optical microscope or an SEM. Reference: Phillips 12th ed page no 236" "This is a real-world medical entrance exam question, please give the true answer based on the question and selection. Topic name: None, Subject name: Dental","Which of the following is main cause of pain during pulpal injury progression? Select and explain. A. Increased vascular permeability, B. Decreased threshold of nerve fibres to pain, C. Arteriolar dilatation, D. Decrease pressure",A: Increased vascular permeability,"Answer: A: Increased vascular permeability Explanation: None" "This is a real-world medical entrance exam question, please give the true answer based on the question and selection. Topic name: None, Subject name: Biochemistry","A child was brought to the hospital was found to have hypoglycemia, hepatomegaly and accumulation of highly branched glycogen called limit dextrins. He is likely to be suffering from: Select and explain. A. McArdle's disease, B. Anderson's disease, C. von Gierke's disease, D. Cori's disease",D: Cori's disease,"Answer: D: Cori's disease Explanation: Ans. 'D' Cori's diseaseCori's disease is also called Limit dextrinosis. It is a Type IIa glycogen storage disorder.It occurs due to the deficiency of the enzyme liver and muscle debranching enzyme.The clinical features are fasting hypoglycemia, hepatomegaly in infancy, accumulation of characteristic branched polysaccharide (limit dextrin) and muscle weakness." "This is a real-world medical entrance exam question, please give the true answer based on the question and selection. Topic name: None, Subject name: Radiology","Decay of enamel following radiotherapy is initiated due to: Select and explain. A. Decrease in salivary flow, B. Direct radiations contacting the enamel, C. Dessication of tooth structure, D. Hypocalcification and pitting of enamel",A: Decrease in salivary flow,"Answer: A: Decrease in salivary flow Explanation: Radiation caries is a rampant form of dental decay that may occur in individuals who receive a course of radiotherapy that includes exposure of the salivary glands. Patients receiving radiation therapy to oral structures have increase in Streptococcus mutans, Lactobacillus, and Candida. Caries results from changes in the salivary glands and saliva, including reduced flow, decreased pH, reduced buffering capacity, increased viscosity, and altered flora.  White and Pharoah, pg-22" "This is a real-world medical entrance exam question, please give the true answer based on the question and selection. Topic name: None, Subject name: Dental","Maximum infective stage of hepatitis B is Select and explain. A. HBsAg, B. HBe Ag, C. HBc Ag, D. Anti-HBs",B: HBe Ag,"Answer: B: HBe Ag Explanation: None" "This is a real-world medical entrance exam question, please give the true answer based on the question and selection. Topic name: None, Subject name: Pharmacology","Lente insulin is composed of: Select and explain. A. 30% Amorphous + 70% Crystalline insulin, B. 30% Crystalline + 70% Amorphous insulin, C. Same as NPH insulin, D. Only 70% amorphous insulin",A: 30% Amorphous + 70% Crystalline insulin,"Answer: A: 30% Amorphous + 70% Crystalline insulin Explanation: Answer- A. 30% Amorphous + 70% Crystalline insulinLente insulin is a 7:3 mixture of long acting ultralente (crystalline) and sho-acting semilente (amorphous) insulin zincsuspension.Long Actinglnsulin glargineInsulin detemirInsulin degludecProtamine zinc insulin" "This is a real-world medical entrance exam question, please give the true answer based on the question and selection. Topic name: None, Subject name: Dental","Materials which has high compressive strength and low tensile strength is classified with property of Select and explain. A. Ductility, B. Brittleness, C. Malleability, D. Resilient",B: Brittleness,"Answer: B: Brittleness Explanation: None" "This is a real-world medical entrance exam question, please give the true answer based on the question and selection. Topic name: None, Subject name: Dental","Apical foramen is offcenter the root apex Select and explain. A. 0.5-3 mm, B. 0.5- 1 mm, C. 1.5-4 mm, D. 1.5-3 mm",A: 0.5-3 mm,"Answer: A: 0.5-3 mm Explanation: None" "This is a real-world medical entrance exam question, please give the true answer based on the question and selection. Topic name: None, Subject name: Dental","When there is a prematurity in centric occlusion but not in eccentric or other movements then reduce; Select and explain. A. Cusps of opposing teeth, B. Mesiodistal position of teeth, C. Opposing fassa or marginal ridge, D. None of the above",C: Opposing fassa or marginal ridge,"Answer: C: Opposing fassa or marginal ridge Explanation: None" "This is a real-world medical entrance exam question, please give the true answer based on the question and selection. Topic name: None, Subject name: Surgery","A condition of the mouth which increases the caries activity in the oral cavity is Select and explain. A. Xerostomia, B. Fissured tongue, C. Hairy tongue, D. Watery saliva",A: Xerostomia,"Answer: A: Xerostomia Explanation: None" "This is a real-world medical entrance exam question, please give the true answer based on the question and selection. Topic name: None, Subject name: Pediatrics","Hyaline membrane disease of lungs is characterized by – Select and explain. A. FRC is smaller than closing volume, B. FRC is greater than closing volume, C. FRC is equal to closing volume, D. FRC is independent of closing volume",A: FRC is smaller than closing volume,"Answer: A: FRC is smaller than closing volume Explanation: ""Hyaline membrane disease reduces compliance and causes a fall in FRC (to levels below the normal closing volume of the lung)"". _______________________________________________ Handbook of pediatric intensive care" "This is a real-world medical entrance exam question, please give the true answer based on the question and selection. Topic name: AIIMS 2019, Subject name: Pharmacology","Which of the following is clinical use of tafenoquine? Select and explain. A. Radical cure of Plasmodium vivax, B. Prophylaxis of malaria in pregnancy, C. Treatment of severe falciparum malaria, D. Treatment of endemic malaria in children < 2 years",A: Radical cure of Plasmodium vivax,"Answer: A: Radical cure of Plasmodium vivax Explanation: Radical cure - About 8-30% P.v. cases relapse due to persistance of exoerythrocytic stage. Drugs which attack this stage (hypnozoites) given together with a clinical curative achieve total eradication of the parasite from the patient's body. Drug of choice for radical cure of vivax and ovale malaria is: * Primaquine 15 mg daily for 14 days * Tafenoquine is a new long-acting exoerythrocytic schizontocide, has been developed as a single dose anti-relapse drug for vivax malaria." "This is a real-world medical entrance exam question, please give the true answer based on the question and selection. Topic name: None, Subject name: Pharmacology","Anaesthestic agent causing adrenal suppression Select and explain. A. Midazolom, B. Thiopentone, C. Etomidate, D. Propofol",C: Etomidate,"Answer: C: Etomidate Explanation: None" "This is a real-world medical entrance exam question, please give the true answer based on the question and selection. Topic name: None, Subject name: Physiology","Which of the following is not a monomeric intermediate filament? Select and explain. A. Vimentin, B. Keratin, C. Desmin, D. Tubulin",D: Tubulin,"Answer: D: Tubulin Explanation: Ans. D. TubulinThe building block of a microtubule is the tubulin subunit, a heterodimer of a- and b-tubulin." "This is a real-world medical entrance exam question, please give the true answer based on the question and selection. Topic name: None, Subject name: Forensic Medicine","A patient comes with pinpoint pupil, salivation, tremors, and red tears. Cholinesterase activity was 30% normal. Probable diagnosis is: Select and explain. A. Opium, B. Organophosphate poisoning, C. Dhatura, D. Organochloride pesticide poisoning",B: Organophosphate poisoning,"Answer: B: Organophosphate poisoning Explanation: Ans.B i.e. Organophosphorus* These are characteristic features of anti-cholinesterase (organophosphate and carbamate) poisoning.* Features of Organophosphate poisoning: - Muscarinic symptoms: Pinpoint pupil, salivation, lacrimation, urination, defecation, gastrointestinal distress, vomiting, bronchospasm, bradycardia. - Nicotinic symptoms: Fasciculations and fibrillations of muscle, tachycardia, tachypnea. - CNS symptoms: Tremors, giddiness, ataxia, coma. - Red tears: Due to accumulation of porphyrin in the lacrimal glands" "This is a real-world medical entrance exam question, please give the true answer based on the question and selection. Topic name: None, Subject name: Medicine","45-year-old presents with persistent productive cough and clubbing. On auscultation, wheeze and crackles are present. Which is the most specific investigation to confirm the diagnosis? Select and explain. A. HRCT, B. Bronchoscopy with bronchoalveolar lavage, C. Chest X-ray, D. MRI",A: HRCT,"Answer: A: HRCT Explanation: Ans. a. HRCTPersistence of a productive cough, clubbing, wheeze, and crackles is suggestive of bronchiectasis. HRCT is the diagnostic procedure of choice (most specific).HRCT findings in bronchiectasis:Airway dilation (detected as parallel ""tram tracks"" or as the ""signet-ring sign"").Lack of bronchial taperingBronchial wall thickening in dilated airwaysInspissated secretions (e.g., the ""tree-in-bud"" pattern)Cysts emanating from the bronchial wall (especially pronounced in cystic bronchiectasis)" "This is a real-world medical entrance exam question, please give the true answer based on the question and selection. Topic name: None, Subject name: Dental","The ""Pulse oximetry"" used in the determination of: Select and explain. A. Rate of flow, B. Oxygen saturation, C. Blood volume, D. Blood coefficient",B: Oxygen saturation,"Answer: B: Oxygen saturation Explanation: None" "This is a real-world medical entrance exam question, please give the true answer based on the question and selection. Topic name: None, Subject name: Dental","The iron carbide in orthodontic wire is in the form of: Select and explain. A. Martensite, B. Ferrite, C. Carbide, D. Austenite",D: Austenite,"Answer: D: Austenite Explanation: None" "This is a real-world medical entrance exam question, please give the true answer based on the question and selection. Topic name: None, Subject name: Forensic Medicine","Ewing's postulates concerns with which of the following: Select and explain. A. Growth at the site following trauma, B. Growth after a neurological injury, C. Age related changes in the teeth, D. Old seminal stains",A: Growth at the site following trauma,"Answer: A: Growth at the site following trauma Explanation: Ans: a. Growth at the site following trauma(Ref Parikh 6/e p4.89).Ewing's Postulates:Ceain criteria, known as `Ewing's postulates', must be satisfied before a relationship between trauma and new growth is accepted:The tumor must arise exactly at the site injuredDefinite and substantial trauma must be provedThe tumor must be confirmed pathologicallyThe tissue at the site must have been healthy before the traumaA reasonable interval-neither too long or too sho--must elapse between the time of the trauma and the appearance of the tumorThough not one of the Ewing's original postulates, there should be some good scientific reason for ascribing the tumor formation to the injury and this is rarely possible." "This is a real-world medical entrance exam question, please give the true answer based on the question and selection. Topic name: None, Subject name: Surgery","A hemophilia B patient needs a major surgery. Factor replacement should be done at what rate: Select and explain. A. 80-100 units/ kg every 12 hours, B. 80-100 units/kg every 24 hours, C. 60 units/kg every 12 hours, D. 80-100 units/kg every 6 hours",B: 80-100 units/kg every 24 hours,"Answer: B: 80-100 units/kg every 24 hours Explanation: None" "This is a real-world medical entrance exam question, please give the true answer based on the question and selection. Topic name: None, Subject name: Medicine","All of the following are true about severe malaria except: Select and explain. A. Hypoglycemia, blood sugar level less than 40 mg, B. Creatinine more than 3 mg/dL, C. LDH > 750 U/L, D. Hematocrit more than 15",D: Hematocrit more than 15,"Answer: D: Hematocrit more than 15 Explanation: Answer- D. Hematocrit more than 15Hematocrit <15 (not more than 15) is suggestive of severe malaria.Manifestations of Severe Falciparum Malaria:Hematocrit of <15% or hemoglobin level of <50 g/L (<5 g/dl) with parasitemia level of > 100,000/microLSerum creatinine level of >265 micromol,{L (>3 mg/dl)Plasma glucose level of <2.2 mmol/L (<40 mg/dL)" "This is a real-world medical entrance exam question, please give the true answer based on the question and selection. Topic name: None, Subject name: Forensic Medicine","In post moem body traumatic injury of face with one eye missing and some injury on mouth and nose ( right eye ball was missing , blood in socket area ) Cause of injury Select and explain. A. Blunt rupture to eyeball, B. Evisceration by sharp weapon, C. Post moem aefact, D. None of the Above",B: Evisceration by sharp weapon,"Answer: B: Evisceration by sharp weapon Explanation: Answer B. Evisceration by sharp weaponIn blunt rupture of eye the complete eye will not be missing. Also since the blood is present in the socket so post moem aifact is ruled out.Evisceration by Sharp object the eye ball will be completely missing. Some injury to mouth or nose can be due to trauma in those areas." "This is a real-world medical entrance exam question, please give the true answer based on the question and selection. Topic name: None, Subject name: Dental","The Chamfer finish line is used in Select and explain. A. Lingual surface of metal ceramic crown, B. Facial surface of metal ceramic crown, C. Facial surface of porcelain jacket crown, D. Lingual surface of porcelain jacket crown",A: Lingual surface of metal ceramic crown,"Answer: A: Lingual surface of metal ceramic crown Explanation: None" "This is a real-world medical entrance exam question, please give the true answer based on the question and selection. Topic name: None, Subject name: Dental","The minimum incidence of cleft palate is seen in which of the following: Select and explain. A. Mongoloid, B. Afghans, C. Negroes, D. South Americans",C: Negroes,"Answer: C: Negroes Explanation: None" "This is a real-world medical entrance exam question, please give the true answer based on the question and selection. Topic name: None, Subject name: Pharmacology","Angiotensin receptor blocker (ARB) with PPARgamma function as well is: Select and explain. A. Olmesaan, B. Candesaan, C. Telmisaan, D. Eprosaan",C: Telmisaan,"Answer: C: Telmisaan Explanation: Ans: C. Telmisaan(Ref: Yamagishi S, Takeuchi M. Telmisaan is a promising cardiornetabolic saan due to its unique PPAR-gamma-inducing propey. Med Hypotheses. 2005;64(3):476-8)Angiotensin receptor blocker (ARB) with PPAR-gamma function as well is telmisaan.Telmisaan:Act as a paial agonist of peroxisome prolilerator-activated receptor-gamma (PPAR-gamma).Due to its unique PPAR-gamma-modulating activity, telmisaan is a promising `cardiontetabolic saan', that targets both diabetes and CVD in hypeensive patients.The binding affinity to PPAR-gamma is highest for telmisaan followed by lisinopril and valsaan." "This is a real-world medical entrance exam question, please give the true answer based on the question and selection. Topic name: None, Subject name: Dental","An isolated area in which root is denuded of bone is intact and covered only by periosteum and overlying gingiva is called Select and explain. A. Fenestration, B. Dehiscence, C. Infracrestal pocket, D. Hemiseptum defect",A: Fenestration,"Answer: A: Fenestration Explanation: None" "This is a real-world medical entrance exam question, please give the true answer based on the question and selection. Topic name: None, Subject name: Dental","Which of the following metals shows the property of twinning: Select and explain. A. Stainless steel, B. Nickel-titanium alloy, C. Cobalt chromium, D. All of the above",B: Nickel-titanium alloy,"Answer: B: Nickel-titanium alloy Explanation: None" "This is a real-world medical entrance exam question, please give the true answer based on the question and selection. Topic name: None, Subject name: Dental","Which of the following impression material requires a hardener? Select and explain. A. Agar, B. Alginate, C. Elastomers, D. Impression compound",A: Agar,"Answer: A: Agar Explanation: None" "This is a real-world medical entrance exam question, please give the true answer based on the question and selection. Topic name: None, Subject name: Surgery","A driver wearing seat belt applied brake suddenly to avoid a collision. Which of the following body pas is most likely to be injured? Select and explain. A. Liver, B. Spleen, C. Mesentery, D. Abdominal aoa",C: Mesentery,"Answer: C: Mesentery Explanation: Answer- C (Mesentry)Most common organ injured in seat belt injury is Mesentery." "This is a real-world medical entrance exam question, please give the true answer based on the question and selection. Topic name: None, Subject name: Gynaecology & Obstetrics","Drug not given in PCOD in a 30-year-old lady with infeility? Select and explain. A. Clomiphene, B. Tamoxifen, C. OCPs, D. Metformin",B: Tamoxifen,"Answer: B: Tamoxifen Explanation: Ans: B. Tamoxifen(Ref: Jeffcott 6/e p205; Shaws 16/e p431-434, 15/e p371, 14/331-332, 13/353-354; Novak 's 15/e pl 076. Duna Gvnae 6/e p470)Drug not given in 30-year-old PCOD lady with infeility - Tamoxifen.Treatment of PCOD:Dexamethasone 0.5 mg at bedtime - Reduces androgen production.In Clomiphene failed group - Ovulation induced with FSH or GnRH analogues.DOC - Metformin - Treats root cause of PCOS, rectifies endocrine & metabolic functions and improves feility.Surgery (laparoscopic multiple puncture of cyst) - Reserved for failed medical therapy, hyperstimulation cases & GnRH analogue usage." "This is a real-world medical entrance exam question, please give the true answer based on the question and selection. Topic name: None, Subject name: Dental","According to Kim and colleagues which of the following describe type IV isthmus? Select and explain. A. Two canals with a definite connection between them, B. Complete or incomplete isthmus between three or more canals, C. Very short, complete Isthmus between two canals, D. Two or three canal openings without visible connections",B: Complete or incomplete isthmus between three or more canals,"Answer: B: Complete or incomplete isthmus between three or more canals Explanation: None" "This is a real-world medical entrance exam question, please give the true answer based on the question and selection. Topic name: None, Subject name: Dental","Heavy and light body impression materials are used with: Select and explain. A. Mercaptan, B. Silicone, C. Reversible hydrocolloid, D. 1 and 2",D: 1 and 2,"Answer: D: 1 and 2 Explanation: Elastomeric impression materials: They are supplied in two components, a base paste and a catalyst paste (or liquid) that are mixed before making impressions. They are often formulated in several consistencies, including extra low, low, medium, heavy, and putty, in increasing order of filler content. Extra-low and putty forms are available only for condensation and addition silicones. Polysulfide is provided only in light-body and heavy-body consistencies. There is no heavy-body product for condensation silicone. Pigments are added to give each material a distinct color. The base paste, is a polysulfide polymer that contains a multifunctional mercaptan (-SH) called a polysulfide polymer, a suitable  filler  (such as lithopone or titanium dioxide) to provide the required strength, a plasticizer (such as dibutyl phthalate) to confer the appropriate viscosity to the paste, and a small quantity of sulfur, approximately 0.5%, as an accelerator. Reference: PHILLIPS’ SCIENCE OF DENTAL MATERIALS, 12th ed page no 153" "This is a real-world medical entrance exam question, please give the true answer based on the question and selection. Topic name: None, Subject name: Dental","In comparison to maxillary central incisor, maxillary canine has a height of contour that is: Select and explain. A. More, B. Less, C. Same, D. Different on different teeth",A: More,"Answer: A: More Explanation: None" "This is a real-world medical entrance exam question, please give the true answer based on the question and selection. Topic name: None, Subject name: Medicine","Bite cells are seen in Select and explain. A. G6PD deficiency, B. SCA, C. Hereditory spherocytosis, D. Trauma",A: G6PD deficiency,"Answer: A: G6PD deficiency Explanation: None" "This is a real-world medical entrance exam question, please give the true answer based on the question and selection. Topic name: None, Subject name: Physiology","The decreased phosphate levels seen in hyper parathyroidism is due to: Select and explain. A. Decreased intestinal phosphate absorption, B. Increased calcium excretion, C. Decreased renal phosphate absorption, D. Increased loss of phosphate in urine",C: Decreased renal phosphate absorption,"Answer: C: Decreased renal phosphate absorption Explanation: None" "This is a real-world medical entrance exam question, please give the true answer based on the question and selection. Topic name: AIIMS 2017, Subject name: Medicine","A patient on Anti - tubercular therapy develops tingling sensation in the limbs. Which of the following when substituted can result in improvement of symptoms? Select and explain. A. Thiamine, B. Pyridoxine, C. Folic acid, D. Methylcobalamine",B: Pyridoxine,"Answer: B: Pyridoxine Explanation: Tingling sensation in limb on ATT: ISONIAZID toxicity (Peripheral neuritis) which occurs due to deficiency of vitamin B6 (Pyridoxine) So pyridoxine supplementation is given in this case." "This is a real-world medical entrance exam question, please give the true answer based on the question and selection. Topic name: None, Subject name: Dental","A permanent tooth with fracture involving apical one third of root after treatment, would most likely: Select and explain. A. Exhibit internal root resorption, B. Remain vital and functional, C. Be ankylosed, D. Exhibit external root resorption and exfoliate",B: Remain vital and functional,"Answer: B: Remain vital and functional Explanation: None" "This is a real-world medical entrance exam question, please give the true answer based on the question and selection. Topic name: None, Subject name: Dental","The effect of enlarged Adenoids on the maxillary growth is by Select and explain. A. Narrowing of maxilla, B. Widening of maxilla, C. Palatal plane tipped upwards at PNS, D. Palate descends down",A: Narrowing of maxilla,"Answer: A: Narrowing of maxilla Explanation: None" "This is a real-world medical entrance exam question, please give the true answer based on the question and selection. Topic name: None, Subject name: Microbiology","Which of the following is not true of boiling water Select and explain. A. It can kill hepatitis B virus, B. Used for sterilizing surgical instruments, C. Can kill mycobacterium tuberculosis, D. Can kill vegetative bacterial cells",B: Used for sterilizing surgical instruments,"Answer: B: Used for sterilizing surgical instruments Explanation: None" "This is a real-world medical entrance exam question, please give the true answer based on the question and selection. Topic name: None, Subject name: Medicine","Marker appearing last in MI Select and explain. A. LDH, B. CPK-MB, C. Troponin T, D. Troponin I",A: LDH,"Answer: A: LDH Explanation: None" "This is a real-world medical entrance exam question, please give the true answer based on the question and selection. Topic name: None, Subject name: Dental","Patient reports with severe throbbing pain in relation to mandibular second molar. The tooth is sensitive to both hot and cold food and elicits a painful response on percussion. What would be your treatment of choice? Select and explain. A. Incision and drainage of Periapical area, B. Endodontic therapy, C. Indirect pulp capping, D. Partial pulpectomy",B: Endodontic therapy,"Answer: B: Endodontic therapy Explanation: None" "This is a real-world medical entrance exam question, please give the true answer based on the question and selection. Topic name: None, Subject name: Anatomy","Which of the following is not true about the larynx? Select and explain. A. All intrinsic muscles are supplied by the recurrent laryngeal nerve, B. Cricothyroid is supplied by the external laryngeal nerve, C. Posterior cricoarytenoid abduct the vocal cords, D. Lymphatic drainage of the larynx is to the deep cervical nodes",A: All intrinsic muscles are supplied by the recurrent laryngeal nerve,"Answer: A: All intrinsic muscles are supplied by the recurrent laryngeal nerve Explanation: All intrinsic muscles are supplied by the recurrent laryngeal nerve except cricothyroid is supplied by external laryngeal nerve." "This is a real-world medical entrance exam question, please give the true answer based on the question and selection. Topic name: None, Subject name: Pharmacology","Etanercept is a disease-modifying drug used in management of rheumatoid ahritis. What is its mechanism of action? Select and explain. A. Inhibition of TNF alpha, B. COX-2 inhibition, C. IL-6 inhibition, D. Stabilization of mast cells",A: Inhibition of TNF alpha,"Answer: A: Inhibition of TNF alpha Explanation: Ans: A. Inhibition of TNF alpha Gilman 12/e p182; KDT7/e p883, 6/ep205).Etanercept:Fusion protein produced by recombinant DNA.It fuses the TNF receptor to the constant end of the IgG1 antibody.It reduces the effect of naturally present TNF.Hence a TNF inhibitor, functioning as a decoy receptor that binds to TNF." "This is a real-world medical entrance exam question, please give the true answer based on the question and selection. Topic name: None, Subject name: Pediatrics","A6 month old child with Tetralogy of Fallot develops cyanotic spell initiated by crying. Which one of the following drugs you would like to avoid – Select and explain. A. Sodium bicarbonate, B. Propranolol, C. Phenylephrine, D. Isoprenaline",D: Isoprenaline,"Answer: D: Isoprenaline Explanation: In Tetralogy of Fallot there is right outflow obstruction due to pulmonary stenosis with supravalvular pulmonary artery obstruction. In case of severe obstruction, the right ventricular pressure becomes greater than the left ventricular presence and the deoxygenated blood starts moving to the left ventricle resulting in severe cyanosis and erythrocytosis. In this situation the aim is to increase blood flow towards the right heart. The methods adopted are compression of Aorta (knee chest position) and use of vasopressor drugs like methoxamine (Phenylepinephrine is also a vasopressor). Any condition or drug which causes peripheral pooling of blood should be avoided because this will reduce return of blood to heart. Therefore isoprenaline which has marked 13, agonistic action should be avoided as it will cause vasodilatation which leads to peripheral pooling of blood. This will be harmful in a case of Tetralogy of Fallot." "This is a real-world medical entrance exam question, please give the true answer based on the question and selection. Topic name: None, Subject name: Dental","For root fracture it is imperative to take X-ray at? Select and explain. A. 45°, 90°, B. 90°, C. 90°, 110°, D. 45°, 90°, 110°","D: 45°, 90°, 110°","Answer: D: 45°, 90°, 110° Explanation: None" "This is a real-world medical entrance exam question, please give the true answer based on the question and selection. Topic name: AIIMS 2019, Subject name: Pediatrics","A 5 year child is brought with brown coloured urine and oliguria since 3 days with mild facial puffiness and pedal edema with 3+ proteinuria, BP 126/90. Urine examination shows RBCs 100/hpf and granular casts. Which of the following doesn't present with this finding? Select and explain. A. Minimal change disease, B. Membranous glomerulonephritis, C. FSGS, D. IgA nephropathy",A: Minimal change disease,"Answer: A: Minimal change disease Explanation: Glomerular Syndromes Nephrotic Syndrome Nephritic Syndrome Urinary albumin >3.0 - 3.5 gm/24 hours Hematuria/RBC casts Hypoalbuminemia Oliguria Edema Hypeension Hyperlipidemia Decreased GFR Lipiduria Proteinuria +/- Causes SGN (HIV), MGN (SLE, Hep B, Cancer - solid tumors), Mi minimal (children), MPGN (HepC)F Others : DM, Malignancy, vasculitis, amyloidosis Focal glomerulonephritis -IgA nephropathy - Focal SLE (Type III) Diffuse glomerulonephritis - Post infectious - Diffuse SLE (Type IV) IgA nephropathy: Most common presentation asymptomatic microhematuria with mild proteinuria" "This is a real-world medical entrance exam question, please give the true answer based on the question and selection. Topic name: None, Subject name: Pediatrics","An alert 6 months old child is brought with vomiting & diarrhea. RR–45/min, HR–130/min, S P–85 mm of Hg. Capillary refilling time is 4 secs. Diagnosis is – Select and explain. A. Early compensated hypovolemic shock, B. Early decompensated hypovolemic shock, C. Late compensated hypovolemic shock, D. Late decompensated shock due to SVT",A: Early compensated hypovolemic shock,"Answer: A: Early compensated hypovolemic shock Explanation: Hypovolemic shock in children may have following stages : - Early compensated : - Immediately after hypovolemia, body tries to maintain the BP to maintain adequate perfusion to vital organs through a compensatory mechanisms. Late uncompensated : - If shock state continues or the compensatory mechanisms are not enough to maintain the metabolic needs of the tissue, the shock, goes into uncompensated phase." "This is a real-world medical entrance exam question, please give the true answer based on the question and selection. Topic name: None, Subject name: Skin","Postherpetic neuralgia is defined as pain lasting beyond how many weeks? Select and explain. A. 1 week, B. 2 weeks, C. 3 weeks, D. 4 weeks",D: 4 weeks,"Answer: D: 4 weeks Explanation: Ans: D. 4 weeks(Ref Fitzpatrick 6/e p2302, 7/e p490-493, 1873-1898; Rooks 8/e p33.14-33.22; Roxburgh 18/e p52-54).Post-herpetic neuralgia (PHN):Most common & impoant neurologic complications of herpes zoster.Defined as pain after the rash has healed or pain 1 month or 3 months after rash onset.Best answer would be 4 weeks." "This is a real-world medical entrance exam question, please give the true answer based on the question and selection. Topic name: None, Subject name: Pathology","Swelling and redness of the orifices of minor salivary glands of the palate occurs in: Select and explain. A. Nicotinic stornatitis, B. Leukoplakia, C. Fovea palatine, D. Erythroplakia",A: Nicotinic stornatitis,"Answer: A: Nicotinic stornatitis Explanation: None" "This is a real-world medical entrance exam question, please give the true answer based on the question and selection. Topic name: None, Subject name: Pediatrics","A child is diagnosed to have acute gastroenteritis. The consulting pediatrician wants to send a stool sample to a lab which is 16-18 hours away. Which of the following mediums should he use to send this sample – Select and explain. A. Charcol cotton bud, B. Carry Blair medium, C. Sterilized jar, D. A medium with high CO2 content",B: Carry Blair medium,"Answer: B: Carry Blair medium Explanation: In acute gastroenteritis stool culture should be obtained as early as possible. ""Fecal specimens that cannot immediately be plated for culture can be transported to the laboratory in a non-nutrient holding medium such as Carry-Blair to prevent drying or overgrowth of specific organisms""." "This is a real-world medical entrance exam question, please give the true answer based on the question and selection. Topic name: None, Subject name: Dental","GIC was developed by: Select and explain. A. Barnum., B. Wilson and Kent., C. Bowen., D. Herman.",B: Wilson and Kent.,"Answer: B: Wilson and Kent. Explanation: None" "This is a real-world medical entrance exam question, please give the true answer based on the question and selection. Topic name: None, Subject name: Surgery","The first sensation to be lost following administration of local anesthetic is Select and explain. A. Proprioception, B. Pain, C. Touch, D. Temperature",B: Pain,"Answer: B: Pain Explanation: None" "This is a real-world medical entrance exam question, please give the true answer based on the question and selection. Topic name: None, Subject name: Pharmacology","Drug causing toxic optic neuropathy is:March 2009 Select and explain. A. Pyrazinamide, B. Chloroquine, C. Ethambutol, D. Tetracycile",C: Ethambutol,"Answer: C: Ethambutol Explanation: Ans. C: EthambutolCauses of nutritional optic neuropathy include tobacco, ethanol, thiamine, and vitamin B-12.Causes of toxic optic neuropathy include chemicals and drugs, such as methanol, ethylene glycol, ethambutol, isoniazid, digitalis, cimetidine, vincristine, cyclosporine, toluene, and amiodarone.Chloroquine causes loss of vision due to retinal damage. Corneal deposits may also occur and affect vision, but are reversible on discontinuation." "This is a real-world medical entrance exam question, please give the true answer based on the question and selection. Topic name: AIIMS 2019, Subject name: Pediatrics","A 6 years old child with development delay, can ride a tricycle, can climb upstairs with alternate feet, but downstairs with 2 feet per step, can tell his name, known his own sex, but cannot narrate a story. What is his development age? Select and explain. A. 3 years, B. 4 years, C. 5 years, D. 2 years",A: 3 years,"Answer: A: 3 years Explanation: Milestones at 2-4 yrs of age: Age Gross motor Fine motor Language 24 months Walks up and downstairs (2 feet/step) Jumps Tower of 6 blocks Draws veical and circular stroke Turn pages of a book, one at a time Puts 3 words together (subject, verb, object) 36 months Rides a tricycle, can steer and pedal Alternate feet going upstairs Can build a tower with 9 blocks Can copy circle Can dress and undress fully Knows full name,age and sex 48 months Hopes on one foot Alternate feet going Downstairs Throws ball overhand Copies cross Tells story So the child in the question though has chronological age of 6 years, but his developmental milestones are that of a 3 year old child. Hence, his Developmental age is 3 years Extra Edge: Developmental Quotient (DQ) = Developmental age/ Chronological age * 100 = 3 yr. / 6 yr. * 100 = 50" "This is a real-world medical entrance exam question, please give the true answer based on the question and selection. Topic name: None, Subject name: Gynaecology & Obstetrics","A 20 years old woman has been brought to casualty with BP 70/40 mm Hg, pulse rate 120/min. and a positive urine pregnancy test. She should be managed by: Select and explain. A. Immediate laparotomy, B. Laparoscopy, C. Culdocentesis, D. Resuscitation and Medical management",A: Immediate laparotomy,"Answer: A: Immediate laparotomy Explanation: Patient is being brought to the casualty with - BP = 70/40mm, P/R = 120/min (i.e. she is in shock). Her urine pregnancy test is positive i.e. she is a case of ruptured ectopic." "This is a real-world medical entrance exam question, please give the true answer based on the question and selection. Topic name: AIIMS 2018, Subject name: Pharmacology","Major mechanism of action of nitrates in acute attack of angina is:- Select and explain. A. Coronary vasodilation, B. Decreases in preload, C. Decreases in afterload, D. Decreases in hea rate",B: Decreases in preload,"Answer: B: Decreases in preload Explanation: Aim for the treatment of classical angina is decrease in work of hea. It can be accomplished by decrease in preload, decrease in afterload or decrease in hea rate. Nitrates act by production of nitric oxide (NO). NO stimulates guanylate cyclase in smooth muscles which leads to formation of cGMP. Later acts on smooth muscles to cause vasodilation. Nitrates mainly produce NO in veins leading to its predominant action as venodilation. Dilation of veins results in decrease in pre-load. Therefore major mechanism of action of nitrates in angina is decrease in preload. However, in variant angina these drugs benefit by causing coronary vasodilation as variant angina occurs due to vasospasm of coronary aery." "This is a real-world medical entrance exam question, please give the true answer based on the question and selection. Topic name: None, Subject name: Dental","Black Extension for prevention to MID (Minimally invasive dentistry), all are true except Select and explain. A. Site & size of lesion, B. Technique & Material used, C. Increase number of cariogenic bacteria, D. Minimal preparation of cavity",C: Increase number of cariogenic bacteria,"Answer: C: Increase number of cariogenic bacteria Explanation: None" "This is a real-world medical entrance exam question, please give the true answer based on the question and selection. Topic name: None, Subject name: Dental","In dental composite 75 % by weight filler is approximately what % volume of filler? Select and explain. A. 85, B. 90, C. 60, D. 50",D: 50,"Answer: D: 50 Explanation: None" "This is a real-world medical entrance exam question, please give the true answer based on the question and selection. Topic name: None, Subject name: Microbiology","In penicillin allergy, penicillin acts as a: Select and explain. A. Hapten, B. Carrier, C. Super antigen, D. Toxin",A: Hapten,"Answer: A: Hapten Explanation: None" "This is a real-world medical entrance exam question, please give the true answer based on the question and selection. Topic name: None, Subject name: Pediatrics","A 12 years old boy develops sore throat of 4 day s duration. On examination, yellow grayish patch seen over both the tonsils and foul smell coming from his mouth. Which of the following non-suppurative complication is of concern? Select and explain. A. Acute rheumatic fever, B. Acute glomerulonephritis, C. Both acute rheumatic fever and acute glomerulonephritis, D. Scarlet fever",C: Both acute rheumatic fever and acute glomerulonephritis,"Answer: C: Both acute rheumatic fever and acute glomerulonephritis Explanation: Answer- C (Both acute rheumatic fever and acute glomerulonephritis)In streptococcal infections, both acute rheumatic fever and acute glomerulonephritis are non-suppurative complication and is of concern." "This is a real-world medical entrance exam question, please give the true answer based on the question and selection. Topic name: None, Subject name: Medicine","In a patient with implanted cardiac pacemaker (defibrillator),which of the following investigation is useful to know the position of the misplaced implant ?. Select and explain. A. X-ray, B. CT scan, C. MR1, D. USG",A: X-ray,"Answer: A: X-ray Explanation: Ans. a. X-ray (Ref Hurst s The Hea 13/e p1050)Hurst's says ""Lead dislodgement maybe radiographically visible or it maybe a microdislodgement, where there is no radiographic change in position, but there is significant increase in pacing threshold and/or decline in electrogram amplitude.Lead dislodgement may result in an increasing pacing threshold, failure to capture, or failure to sense. Lead dislodgement maybe radiographically visible or it maybe a microdislodgement, where there is no radiographic change in position, but there is significant increase in pacing threshold and/or decline in electrogram amplitude." "This is a real-world medical entrance exam question, please give the true answer based on the question and selection. Topic name: None, Subject name: Medicine","What is the effective management of a dengue patient with warning signs without shock and haemorrhage- Select and explain. A. Steroids, B. Platelet transfusion, C. IV fluids, D. Antiviral",C: IV fluids,"Answer: C: IV fluids Explanation: Answer- C. IV fluidsSuppoive care with analgesics, fluid replacement, and bed rest is usually sufficient." "This is a real-world medical entrance exam question, please give the true answer based on the question and selection. Topic name: None, Subject name: Pathology","Most impoant but nonspecific regulator of iron metabolism is: Select and explain. A. Hepcidin, B. DMT I, C. Ferropoin, D. Ferritin",A: Hepcidin,"Answer: A: Hepcidin Explanation: Ans: A. Hepcidin(Ref Robbins 91e p650 848 8/e p660)Hepcidin:Main regulator protein for iron absorption.Encoded by HAMP gene.Small circulating peptide synthesized & released from liver in response to increased intrahepatic iron levels.Proteins Involved in Iron MetabolismCeruloplasmindeg (ferrioxidase activity)DMT1degFerrireductasedeg (cytochrome b reductase I)FerritindegFerropinHeme transpoerHemojuvelinHepcidindegHephaestindegHFEdegIron-responsive element-bindingproteindegTransferrindegTransferrin receptors 1 & 2" "This is a real-world medical entrance exam question, please give the true answer based on the question and selection. Topic name: None, Subject name: Surgery","Treatment of solitary langerhans histiocytoma of mandible is by Select and explain. A. Curettage, B. Radiotherapy, C. No treatment required, D. Chemotherapy",A: Curettage,"Answer: A: Curettage Explanation: None" "This is a real-world medical entrance exam question, please give the true answer based on the question and selection. Topic name: None, Subject name: Physiology","In PHC, which anticoagulant is used to send the blood sample for blood glucose: Select and explain. A. EDTA, B. Heparin, C. Potassium oxalate + sodium fluoride, D. Potassium oxalate",C: Potassium oxalate + sodium fluoride,"Answer: C: Potassium oxalate + sodium fluoride Explanation: None" "This is a real-world medical entrance exam question, please give the true answer based on the question and selection. Topic name: None, Subject name: Social & Preventive Medicine","HIV sentinel surveillance provides data: Select and explain. A. To monitor disease trends, B. To identify high risk population, C. To estimate disease incidence, D. To assess the quality of related services",A: To monitor disease trends,"Answer: A: To monitor disease trends Explanation: Ans. a. To monitor disease trendsAlthough the HIV sentinel surveillance data has been primarily used for monitoring the trends, i.e. to assess how rapidly HIV infection increases or decreases over the time in different groups and areas, it can also provide an estimate of the total burden of HIV infection in the counttyHIV Sentinel SurveillanceAfter the establishment of the fact that the HIV infection is present in wide geographic areas, the objective of the sentinel surveillance was redefined to monitor the trends of HIV infectionQThe objective of the surveillance is best achieved by annual cross-sectional survey of the risk group in the same place over a few years by unlinked anonymous serological testing procedures by two ERS (i.e. when HIV testing is carried out without identification of name of samples collected for other purposes such as VDRL in STD clinics.) The number of samples to be screened must represent the risk group under study and the sample size is determined accordingly.Although the HIV sentinel surveillance data has been primarily used for monitoring the trends, i.e. to assess how rapidly HIV infection increases or decreases over the time in different groups and areas, it can also provide an estimate of the total burden of HIV infection in the countryQ" "This is a real-world medical entrance exam question, please give the true answer based on the question and selection. Topic name: None, Subject name: Dental","1,1,1,1 Tetra-floroethylene is applied during cold test testing with? Select and explain. A. # 4 Cotton pellet, B. # 2 Cotton pellet, C. # 6 Cotton pellet, D. # 8 Cotton pellet",B: # 2 Cotton pellet,"Answer: B: # 2 Cotton pellet Explanation: None" "This is a real-world medical entrance exam question, please give the true answer based on the question and selection. Topic name: None, Subject name: Social & Preventive Medicine","A radiotherapist prescribes a new drug combination of chemotherapy and immunotherapy for metastatic melanoma. It prolongs the survival. Which of the following is true in this situation? Select and explain. A. Incidence reduces and prevalence increases, B. Incidence remains the same and prevalence increases, C. Incidence reduces and prevalence remains the same, D. Incidence increases and prevalence reduces",B: Incidence remains the same and prevalence increases,"Answer: B: Incidence remains the same and prevalence increases Explanation: Answer- B. Incidence remains the same and prevalence increasesIncidence: Number of new caseq occurring in a defined population during a specified period of time.It is expressed as per 1000 per year.Incidence is a rateIncidence is not affected by duration of diseaseUse of in incidence is generally restricted to acute conditions.This is a much more accurate measure of risk than prevalence." "This is a real-world medical entrance exam question, please give the true answer based on the question and selection. Topic name: None, Subject name: Physiology","Receptors for olfactory sensation are present in: Select and explain. A. Lateral walls, B. Nasal septum, C. Lateral wall and nasal septum above superior concha, D. None of the above",C: Lateral wall and nasal septum above superior concha,"Answer: C: Lateral wall and nasal septum above superior concha Explanation: None" "This is a real-world medical entrance exam question, please give the true answer based on the question and selection. Topic name: AIIMS 2019, Subject name: Pathology","What processing should be done of the blood before transfusion to reduce chances of febrile non-hemolytic transfusion reaction (FNHTR)? Select and explain. A. Irradiation, B. Washing, C. Leucocyte reduction, D. Glycolisation",C: Leucocyte reduction,"Answer: C: Leucocyte reduction Explanation: 1. The reason for the development of allergy is the presence of plasma and the process of washing helps in the removal of residual plasma. if the Patient is having a history of allergy - Antihistaminic drugs -reduce the chances of development of allergic reactions during blood transfusion. 2. Irradiation - reduces the chance of development of Graft Versus Host Disease. Irradiation - causes - reduction in the number of the immunocompetent cells (lymphocytes mainly) Irradiation of cellular blood components (red blood cells, platelets, and granulocytes) is indicated to prevent the development of transfusion-associated graft-versus-host disease (TA-GVHD). Patients at risk of TA-GVHD include immunocompromised patients who are receiving a bone marrow or stem cell transplant and fetuses undergoing an intrauterine transfusion. Irradiation is also indicated for recipients of components collected from a blood relative or HLA-matched donors. 3. Leukocyte reduction - Leukoreduction of whole blood and blood components has been shown to reduce recurrent febrile non-hemolytic transfusion reactions, reduce alloimmunization to leukocyte antigens that may complicate care of patients whoundergo transplantation or chronic transfusion therapy, and protect against transmission of cytomegalovirus (CMV) to patients at increased risk of CMV disease. antibodies in the recipient - react with donor leukocytes - release of cytokines - cause fever- known as Febrile Non-Hemolytic Transfusion Reaction. 4. Glycolisation - addition of glycol. useful in autologous blood transfusion - increases the life span of RBC. If the patient is having a history of development of anaphylaxis after blood transfusion, the patient is most likely to be suffering from IgA deficiency." "This is a real-world medical entrance exam question, please give the true answer based on the question and selection. Topic name: None, Subject name: Anatomy","All the following nerves are related to the mandible except: Select and explain. A. Lingual nerve, B. Auriculotemporal nerve, C. Chorda tympani, D. Mental nerve",C: Chorda tympani,"Answer: C: Chorda tympani Explanation: The sensory information form the mucous membranes of the nasopharynx and palate is carried along the greater petrosal nerve, while the chorda tympani nerve (and lingual nerve) carries taste input from the anterior two thirds of the tongue, floor of mouth, and palate." "This is a real-world medical entrance exam question, please give the true answer based on the question and selection. Topic name: None, Subject name: Dental","Which of the following statements about high copper silver alloy compared to conventional alloy in not true? Select and explain. A. It has increased tensile and compressive strength, B. It has poor tarnish and corrosion resistance, C. Its edge strength is greater, D. Mercury content in the final restoration is less",B: It has poor tarnish and corrosion resistance,"Answer: B: It has poor tarnish and corrosion resistance Explanation: None" "This is a real-world medical entrance exam question, please give the true answer based on the question and selection. Topic name: None, Subject name: Dental","After amalgam polishing, the outermost surface has crystals arranged flattened to tooth surface. This layer is known as Select and explain. A. Beilby layer, B. Beillyl layer, C. Beibyl Layer, D. none of the above",A: Beilby layer,"Answer: A: Beilby layer Explanation: None" "This is a real-world medical entrance exam question, please give the true answer based on the question and selection. Topic name: None, Subject name: Dental","Nutra sweet is: Select and explain. A. Saccharin., B. Aspartame., C. Xylitol., D. Sorbitol.",B: Aspartame.,"Answer: B: Aspartame. Explanation: Saccharin and aspartame are two artificial sweeteners commonly available. Saccharin is 100 times sweeter than sucrose and contains no calories. Aspartame, commonly known as nutra sweet, is a combination of the aminoacids, aspartic acid and phenylalanine. It is 200 times more sweeter than sucrose." "This is a real-world medical entrance exam question, please give the true answer based on the question and selection. Topic name: None, Subject name: Pharmacology","Which of the following drugs is not used in typhoid fever? Select and explain. A. Amikacin, B. Ciprofloxacin, C. Cefixime, D. Azithromycin",A: Amikacin,"Answer: A: Amikacin Explanation: Ans: A. AmikacinRef: Sharma & Sharma's Principles of Pharmacologt, 3d ed., pg. 74t-749 and htrys://www.uplodate.com/contents/treatment-and-prevention-of-enteric-typhoid-und-paratyphoid-feverAmikacin is a commonly used aminoglycoside.Antimicrobial spectrum of aminoglycosides does not include salmonella.It is used in gentamycin and tobramycin resistant infections.It is used for organisms like Pseudomonas, Proteus, and Serratia. It is also effective in MDR-TB." "This is a real-world medical entrance exam question, please give the true answer based on the question and selection. Topic name: None, Subject name: Forensic Medicine","Opium poisoning is treated with:MAHE 06; Bihar 12; UPSC 13 Select and explain. A. Naloxone, B. Atropine, C. Neostigmine, D. Physostigmine",A: Naloxone,"Answer: A: Naloxone Explanation: Ans. A. Naloxone" "This is a real-world medical entrance exam question, please give the true answer based on the question and selection. Topic name: None, Subject name: ENT","Onodi cells and Haller cells are seen in relation to following respectively - Select and explain. A. Optic nerve and floor of the orbit, B. Optic nerve and internal carotid artery, C. Optic nerve and nasolacrimal duct, D. Orbital floor and nasolacrimal duct",A: Optic nerve and floor of the orbit,"Answer: A: Optic nerve and floor of the orbit Explanation: The Onodi and Haller cells are posterior ethmoidal air cells." "This is a real-world medical entrance exam question, please give the true answer based on the question and selection. Topic name: None, Subject name: Dental","Junctional epithelium Select and explain. A. Lines the gingival sulcus, B. Is permeable, C. Is derived initially from the outer enamel epithelium, D. Is attached to enamel by desmosomes",B: Is permeable,"Answer: B: Is permeable Explanation: None" "This is a real-world medical entrance exam question, please give the true answer based on the question and selection. Topic name: None, Subject name: Pediatrics","A child present with recurrent sinusitis and recurrent chest infections. Chest X–ray reveals dextrocardia and situs invertus. The diagnosis is – Select and explain. A. Kartagener's syndrome, B. Good–pasture's syndrome, C. Ehlers–Danlos syndrome, D. William Campbell syndrome",A: Kartagener's syndrome,"Answer: A: Kartagener's syndrome Explanation: Kartagener's syndrome is a subgroup of primary ciliary dyskinesia. Primary ciliary dyskinesia: It is an autosomal recessive syndrome. It is characterized by poorly functioning cilia. There is absence or shortening of Dynein arms that are responsible for the coordinated bending of cilia. Approximately half of the patients with primary ciliary dyskinesia have kartagener's syndrome." "This is a real-world medical entrance exam question, please give the true answer based on the question and selection. Topic name: None, Subject name: Dental","In expansion screws an expansion of 90° causes an expansion of: Select and explain. A. 0.20 mm, B. 0.10 mm, C. 0.36 mm, D. 0.09 mm",A: 0.20 mm,"Answer: A: 0.20 mm Explanation: None" "This is a real-world medical entrance exam question, please give the true answer based on the question and selection. Topic name: None, Subject name: Dental","The function of a sprue is to: Select and explain. A. Form an opening for molten metal to enter the mold, B. Help polish cast restorations, C. Eliminate air bubbles on the wax pattern, D. Reproduce fine detail",A: Form an opening for molten metal to enter the mold,"Answer: A: Form an opening for molten metal to enter the mold Explanation: None" "This is a real-world medical entrance exam question, please give the true answer based on the question and selection. Topic name: None, Subject name: Pharmacology","A nurse got accidental prick from the HIV infected needle. Which of the following is T/F regarding the management of this nurse?a. Lamivudine used as monotherapy in post-exposure prophylaxis (t/f)b. Washing hands advised(t/f)c. Viral markers at the time of prick(t/f)d. Repeat serology at 6 weeks (t/f) Select and explain. A. b,c,d true a false, B. a, b true c,d false, C. b,c true a,d false, D. All true","A: b,c,d true a false","Answer: A: b,c,d true a false Explanation: Ans. A. b,c,d true a false* Lamivudinemonotherapy is the incorrect option. Instead, dual drug therapy of Tenofovir 300mg + Lamivudine 300mg (FDC) is recommended." "This is a real-world medical entrance exam question, please give the true answer based on the question and selection. Topic name: None, Subject name: Medicine","Peripheral eosinophila with renal failure is least likely to be possible in: Select and explain. A. Drug-induced interstitial nephritis, B. Drug-induced interstitial nephritis, C. Atheroembolic renal failure, D. Polyangitis nodosa",D: Polyangitis nodosa,"Answer: D: Polyangitis nodosa Explanation: Answer. d. Polyangitis nodosa (ref- Wallach interpretation of diagnostic test 8/e pg 95-96)Peripheral eosinophilia with renal failure is least likely to be possible in polyangitis nodosa." "This is a real-world medical entrance exam question, please give the true answer based on the question and selection. Topic name: None, Subject name: Dental","Largest permanent tooth in mouth is Select and explain. A. Maxillary 1st molar, B. Mandibular 1st molar, C. Maxillary canine, D. Mandibular canine",A: Maxillary 1st molar,"Answer: A: Maxillary 1st molar Explanation: None" "This is a real-world medical entrance exam question, please give the true answer based on the question and selection. Topic name: None, Subject name: Physiology","Which of the following is TRUE about propagated nerve action potential? Select and explain. A. Decremental, B. Not affected by hypoxia, C. Fastest in C fibers, D. Not affected by membrane capacitance",B: Not affected by hypoxia,"Answer: B: Not affected by hypoxia Explanation: Acute and chronic hypoxias are associated with reduced nerve conduction velocity, neuronal action potential generation and axonal transport. In the central neurons, the reduction of nerve cell excitability in hypoxia is primarily because of increased K+ conductance. So, hypoxia also affects action potential. But compared to other statements, its can be the answer." "This is a real-world medical entrance exam question, please give the true answer based on the question and selection. Topic name: None, Subject name: Pathology","Translocation seen in Ewing's sarcoma is: Select and explain. A. t(8,11), B. t(11,22), C. t(11,18), D. t(14,18)","B: t(11,22)","Answer: B: t(11,22) Explanation: Ans: B. t(11,22)Genetic exchange between chromosomes can cause cells to become cancerous. Most cases of Ewing's sarcoma (85%) are the result of a translocation between chromosomes 11 and 22, which fuses the EWS gene of chromosome 22 to the FLI1 gene of chromosome 11. Other translocations are at t(21;22) and t(7;22)." "This is a real-world medical entrance exam question, please give the true answer based on the question and selection. Topic name: None, Subject name: Medicine","A 60 years old male with alcoholic liver disease presented to emergency with a history of hematemesis. Which of the following is false regarding his management' Select and explain. A. Somatostatin infusion is indicated, B. Negative nasogastric aspirate does not exclude variceal bleed, C. Nasogastric tube inseion can provoke variceal bleeding, D. Upper GI endoscopy is done initially",C: Nasogastric tube inseion can provoke variceal bleeding,"Answer: C: Nasogastric tube inseion can provoke variceal bleeding Explanation: Answer- C. Nasogastric tube inseion can provoke variceal bleedingOctreotide is the preferred pharmacologic agent for initial management of acute variceal bleeding.Management of Acute Variceal Bleeding:Patients should be admitted to an ICU for resuscitation and managementa.Blood resuscitation should be performed to a hemoglobin level of 8 g/dL.Over-replacement of packed red blood cells and the overzealous administration of saline can lead to both rebleeding and increased moality.Administration of FFP and platelets in patients with severe coagulopathy.Shunt therapy (surgical shunts or TIPS) has been shown to control refractory variceal bleeding.Combination of pharmacologic and EVL therapy improve initial control of bleeding and increase the 5-day hemostasis rate." "This is a real-world medical entrance exam question, please give the true answer based on the question and selection. Topic name: None, Subject name: Microbiology","Which of the following is not used as a disinfectant? Select and explain. A. 1-2% Cetrimide, B. 100% Alcohol, C. 2% Lysol, D. 5% Chloroxylene",B: 100% Alcohol,"Answer: B: 100% Alcohol Explanation: A 70% solution of alcohol takes more time in evaporation from the surface, increasing the contact time. Therefore, 70% isopropyl alcohol is suitable for disinfection.  100% isopropyl alcohol coagulates the protein instantly creating a protein layer that protects the remaining protein from further coagulation." "This is a real-world medical entrance exam question, please give the true answer based on the question and selection. Topic name: None, Subject name: Dental","Young’s formula for calculating pediatric dose of a drug is: Select and explain. A. Age / Age+12 x Adult dose, B. Age / 20 x Adult dose, C. Weight (pounds) / 150 x Adult dose, D. None of the above",A: Age / Age+12 x Adult dose,"Answer: A: Age / Age+12 x Adult dose Explanation: Remember, the age denoted here is chronological age. Option 1 is Young’s formula to calculate pediatric dose. Option 2 is Dilling's rule to calculate pediatric dose. Option 3 is Clark's rule to calculate pediatric dose." "This is a real-world medical entrance exam question, please give the true answer based on the question and selection. Topic name: None, Subject name: Pathology","Most common site for melanotic neuroectodermal tumour of infancy is (MNTI): Select and explain. A. Maxilla, B. Mandible, C. Ethmoid bone, D. Cervical spine",A: Maxilla,"Answer: A: Maxilla Explanation: None" "This is a real-world medical entrance exam question, please give the true answer based on the question and selection. Topic name: AIIMS 2019, Subject name: Anatomy","Liver is divided into eight segments according to Couinaud's classification based upon Select and explain. A. Bile Duct, B. Poal vein, C. Hepatic Vein, D. Hepatic aery",B: Poal vein,"Answer: B: Poal vein Explanation: Segmental anatomy of liver. Liver is divided into 8 segments by DAV structures (bile duct, hepatic aery, poal vein) Poal vein is most impoant among them Left hepatic vein separates medial & lateral segments Right hepatic vein separates anterior & posterior segments Poal vein separates upper & lower segments From surgical perspective, right hepatectomy - involve division of liver in principle plane in which segment V, VI, VII, VIII would be removed leaving segments I, II, III, IV Hepatic vein is now removed from couinaud's classification" "This is a real-world medical entrance exam question, please give the true answer based on the question and selection. Topic name: None, Subject name: Medicine","Mantle cell Carcinoma shows Select and explain. A. CD 5+, CD 25-, B. CD 5+, CD 10+, C. CD 5+, CD 23+, D. CD 5+, CD 23-","D: CD 5+, CD 23-","Answer: D: CD 5+, CD 23- Explanation: None" "This is a real-world medical entrance exam question, please give the true answer based on the question and selection. Topic name: None, Subject name: Pharmacology","The drug which gives orange colour to the urine is: Select and explain. A. Rifampicin, B. Ethambutol, C. INH, D. Streptomycin",A: Rifampicin,"Answer: A: Rifampicin Explanation: None" "This is a real-world medical entrance exam question, please give the true answer based on the question and selection. Topic name: None, Subject name: Medicine","A 27 years old pregnant lady comes with severe jaundice and altered sensorium. On examination, the patient is deeply icteric, not responding to commands and pelvic sonogram reveals intrauterine fetal death. Serum bilirubin levels are 28.8 mg/dL (direct = 18.6 mg/dL), AST levels are 1063 and ALT levels are 1191. The viral markers are as follows. What is the likely diagnosis?Anti-HAV IgGReactiveAnti-HAV IgMNonreactiveHbSAgNon reactiveAnti-HbSAgNonreactiveAnti-HBc IgMNonreactiveAnti-HBc IgGReactiveAnti-HCV IgGNonreactiveAnti-HEV IgMReactiveAnti-HEV IgGNon reactive Select and explain. A. Acute hepatitis E superimposed on chronic liver failure due to hepatitis B, B. Fulminant hepatitis due to hepatitis B infection, C. Acute hepatitis E with chronic hepatitis A, D. Fulminant hepatitis due to hepatitis E infection",D: Fulminant hepatitis due to hepatitis E infection,"Answer: D: Fulminant hepatitis due to hepatitis E infection Explanation: Ans: D. Fulminant hepatitis due to hepatitis E infection(Ref: Harrison 191e p2018, 18/e p2546)In the question, anti-HBc IgG is reactive but HbsAg is negative, the case is a recovered case from hepatitis B.Hepatitis A is rarely chronic, can he ruled out from the given options.Anti-HAV IgG suggests recovered case from HAV infections.Anti-HEV 1gM is an indicator of acute hepatitis E infection.History of pregnancy and high serum bilirubin with raised AST and ALT is suggestive of fulminant hepatitis E.Commonly Encountered Serologic Patterns of Hepatitis B InfectionHBsAgAnti-HBsAnti-HBcHBeAgAnti-HBeInterpretation+-IgM+-* Acute hepatitis B, high infectivity+-IgG+-* Chronic hepatitis B, high infectivitydeg+ IgG-+* Late acute or chronic hepatitis B, low infectivitydeg* HBeAg-negative (`precore-mutant) hepatitis B (chronic or rarely acute)" "This is a real-world medical entrance exam question, please give the true answer based on the question and selection. Topic name: None, Subject name: Surgery","In a patient of liver disease, the possible complication during extraction is: Select and explain. A. Dry socket, B. Fascial space infection, C. Bleeding, D. Loss of clot",C: Bleeding,"Answer: C: Bleeding Explanation: None" "This is a real-world medical entrance exam question, please give the true answer based on the question and selection. Topic name: None, Subject name: Pediatrics","Dengue shock syndrome is characterized by the following except : Select and explain. A. Hepatomegaly, B. Pleural effusion, C. Thrombocytopenia, D. Decreased haemoglobin",D: Decreased haemoglobin,"Answer: D: Decreased haemoglobin Explanation: Dengue hemorrhagic fever : Fever. Minor or Major hemorrhgic manifestations. Hepatomegaly. Thrombocytopenia 100,000/mm3. Hypoalbuminemia. Objective evidence of increased capillaty permeability (hematocrit 20%). Pleural effusion (by chest radiograph). ​​Criteria for Dengue shock syndrome : It includes those for dengue hemorrhagic fever plus, Hypotension or narrow pulse." "This is a real-world medical entrance exam question, please give the true answer based on the question and selection. Topic name: None, Subject name: Pathology","Vascular event of inflammation in order-a) delayed prolonged- leukocytes mediated injuryb) immediate transient - mast cell activation lead to release of histamine bradykininc) immediate prolonged- direct endothelial injury by bacterial toxind) Somewhat delayed prolonged- sun exposure /heat stroke lead to mild endothelial injury Select and explain. A. ABCD, B. DCBA, C. ACDB, D. BDCA",D: BDCA,"Answer: D: BDCA Explanation: Answer- DB, D, C, A" "This is a real-world medical entrance exam question, please give the true answer based on the question and selection. Topic name: None, Subject name: Microbiology","Endotoxin of gram -ve bacteria that do not cause pathogenesis of natural disease Select and explain. A. Mycobacterium, B. Klebsiella, C. Vibrio, D. E.coli",C: Vibrio,"Answer: C: Vibrio Explanation: None" "This is a real-world medical entrance exam question, please give the true answer based on the question and selection. Topic name: None, Subject name: Skin","A 24 years old female presented with patchy hair loss in the right temporal and occipital region. Examination revealed non- scarring alopecia with multiple small broken hairs. Scrapings from scalp showed mild inflammation, peri-follicular hemorrhage and surrounding mild lymphocytic infiltration. What is the most likely diagnosis? Select and explain. A. Alopecia areata, B. Androgenic alopecia, C. Loose anagen hair, D. Trichotillomania",D: Trichotillomania,"Answer: D: Trichotillomania Explanation: Ans: D. Trichotillomania (Ref: Rooks 8/e p55, Fitzputrick 6/e p/55).Patchy hair loss in the right temporal and occipital region in a 24-years-old female non- scarring alopecia with multiple small broken hairs and mild (sparsity) of a perifollicular inflammatory infiltrate is highly suggestive of trichotillomania." "This is a real-world medical entrance exam question, please give the true answer based on the question and selection. Topic name: None, Subject name: Dental","Decalcification and discoloration of teeth occurs during orthodontic treatment due to: Select and explain. A. Acid etching, B. Poor oral hygiene, C. Leakage of cement around bands, D. All of the above",D: All of the above,"Answer: D: All of the above Explanation: None" "This is a real-world medical entrance exam question, please give the true answer based on the question and selection. Topic name: None, Subject name: Pathology","A 4 yr old girl child suffering from vomiting, cerebral edema and is being treated for viral infection from past 5 days. Liver biopsy in this patient may show Select and explain. A. Central haemorrhagic necrosis, B. Non-alcoholic steatohepatitis, C. Autoimmune hepatitis, D. Microvesicular steatohepatitis",D: Microvesicular steatohepatitis,"Answer: D: Microvesicular steatohepatitis Explanation: None" "This is a real-world medical entrance exam question, please give the true answer based on the question and selection. Topic name: None, Subject name: Dental","Currently used radiographic method for working length determination is: Select and explain. A. Grossman technique, B. Ingels technique., C. Bregman technique., D. None of the above.",B: Ingels technique.,"Answer: B: Ingels technique. Explanation: None" "This is a real-world medical entrance exam question, please give the true answer based on the question and selection. Topic name: None, Subject name: Medicine","A patient who is a known case of CKD has complaints of vomiting. His ABG repos are as follows: pH - 7.40, pCO2 - 40, HCO3 - 25. Na -145, chloride-100. Select and explain. A. Normal anion gap met acidosis, B. High anion gap met acidosis, C. No acid base abnormality, D. High anion gap metabolic acidosis with metabolic alkalosis",D: High anion gap metabolic acidosis with metabolic alkalosis,"Answer: D: High anion gap metabolic acidosis with metabolic alkalosis Explanation: Ans. D. High anion gap metabolic acidosis with metabolic alkalosis Even though ABG looks completely normal - clinical history is the key here.CKD patients generally have high AG metabolic acidosis. On the background of that he has developed vomiting (which is an alkalotic state). Both opposing disorders have normalized the ABG. But the patient is actually having a double disorder." "This is a real-world medical entrance exam question, please give the true answer based on the question and selection. Topic name: None, Subject name: Dental","Impression compound base: Select and explain. A. Low thermal conductivity, B. Crystalline structure, C. Formation of cross linkage with heating, D. High fusion temperature",A: Low thermal conductivity,"Answer: A: Low thermal conductivity Explanation: Impression compound  The thermal conductivity of these materials is very low, indicating the need to allow an extended time to achieve thorough cooling and heating of the compound. The material should be uniformly soft at the time it is placed in the tray and thoroughly cooled in the tray before the impression is withdrawn from the mouth. Cold water can be sprayed on the tray while it is in the mouth until the compound is thoroughly hardened prior to removal of the impression tray. Failure to attain a complete hardening of the material before withdrawing the impression can result in a significant distortion of the impression by relaxation. Reference: PHILLIPS’ SCIENCE OF DENTAL MATERIALS, 12th ed page no 178" "This is a real-world medical entrance exam question, please give the true answer based on the question and selection. Topic name: AIIMS 2018, Subject name: Surgery","Patient had a A and put on mechanical ventilation, he is opening his eyes on verbal command, Moves all his 4 limbs spontaneously. What will be his GCS score? Select and explain. A. 9, B. 10, C. 11, D. 12",A: 9,"Answer: A: 9 Explanation: GLASGOW COMA SCALE Revised GCS (2014) Eye Opening (E) Verbal Response (V) Best Motor Response (M) Spontaneous 4 Oriented 5 Obeying commands 6 To Speech 3 Confused 4 Localizing 5 To pressure 2 Words 3 Normal flexion (withdrawal) 4 None 1 Sounds 2 Abnormal flexion 3 None 1 Extension 2 None 1 Pupils Unreactive to Light Pupil Reactivity Score Both Pupils 2 One Pupil 1 Neither Pupil 0 GCS specifically recommends avoiding sternal rubs as it causes bruising & responses can be difficult to interpret. They also do not recommend routine use of retromandibular pressure. Revised GCS (2014) changes are underlined & highlighted in the above table. Maximum score -15, minimum score- 3 Best predictor of outcome: Motor response Repoing of Non-testable Score Aspects: In cases of a non-testable aspect, the new GCS should only be noted in its components. Any element that cannot be tested should be marked as NT. for ""not testable"". For intubated patients or patients with tracheostomy, VNT is used. It is no longer recommend to assign 1 point to non-testable elements, therefore a combined score should not be used. E- Opening of eyes on verbal command - 3 M-Movement of all 4 limbs spontaneously - 6 V-Patient on mechanical ventilation - Non-testable-0 The GCS score is (E3VNTM6) 9 for this patient." "This is a real-world medical entrance exam question, please give the true answer based on the question and selection. Topic name: None, Subject name: Biochemistry","Which of the following enzyme is common between glycogenesis and glycogenolysis? Select and explain. A. Glycogen synthase, B. Glycogen phosphorylase, C. Glucan transferase, D. Phosphoglucomutase",D: Phosphoglucomutase,"Answer: D: Phosphoglucomutase Explanation: Ans: D.PhosphoglucomutasePhosphoglucomutase enzyme is common between glycogenesis & glycogenolysis.Reaction catalyzed by phosphoglucomutase is reversible.Hence, glucose 6-phosphate can be formed from glucose I-phosphate.In liver & kidney, (not in muscle) glucose 6-phosphatase hydrolyzes glucose 6-phosphate a glucose a Increases blood glucose concentration.(Ref Harper's 29/e p180; Harper 30/e p176, 177, 28/e p158, 333)." "This is a real-world medical entrance exam question, please give the true answer based on the question and selection. Topic name: None, Subject name: Dental","Common sign of occlusal trauma (TFO) is Select and explain. A. Tooth mobility, B. Fractures of cusps, C. Resorption of alveolar ridge, D. Widening of P.D ligament",A: Tooth mobility,"Answer: A: Tooth mobility Explanation: None" "This is a real-world medical entrance exam question, please give the true answer based on the question and selection. Topic name: AIIMS 2019, Subject name: Orthopaedics","What to be done next? Select and explain. A. Extended curettage with allograft, B. Bone biopsy, C. Curettage, D. Extended curettage with autograft",B: Bone biopsy,"Answer: B: Bone biopsy Explanation: The given image shows a lytic area in the lower end of femur which extends up to the joint. The findings show Giant cell tumor , the only tumour which goes till the joint. The treatment of choice - extended curettage with autograft. To confirm GCT, bone biopsy is done next as histology is diagnostic." "This is a real-world medical entrance exam question, please give the true answer based on the question and selection. Topic name: None, Subject name: Pharmacology","Mechanism of action of Oseltamivir (Tamiflu) as an antiviral agent is: Select and explain. A. Inhibition of M2 receptor, B. Neuraminidase inhibition, C. Inhibition of RNA dependent DNA polymerase, D. Apoptosis of infected cells",B: Neuraminidase inhibition,"Answer: B: Neuraminidase inhibition Explanation: Answer- B. Neuraminidase inhibitionThe neuraminidase inhibitors oseltamivir and zanamivir, analogs of sialic ocid, interfere with release of progenyinfluenza vitus from infected host cells, thas halting the spread of infection within the respiratory tract. These agentscompetitively and reversibly interact with the active enzyme site to inhibit viral neuraminidase activity at low nanomolar concentrations." "This is a real-world medical entrance exam question, please give the true answer based on the question and selection. Topic name: None, Subject name: Anatomy","Which of the following cranial nerve not associated with olfaction? Select and explain. A. XII, B. IX, C. V, D. X",A: XII,"Answer: A: XII Explanation: Ans. A. XIIXII nerve is a motor nerve; does not play role in olfaction and gustation.Cranial Nerves -- Components and FunctionsNumberNameFunctionIOlfactory nerve Smell sensationIIOptic nerve VisionIIIOculomotor Turns eyeball upward, downward,and medially; Raises upper eyelid, alsoconstricts pupil; accommodates eyeIVTrochlear nerve It helps in turning eyeball downwardand laterallyVTrigeminalMaxillarydivisionSupplies dura mater of anterior paof middle cranial fossa, conjunctiva ofinferior eyelid, skin of face over maxilla;teeth of upper jaw; mucous membrane NumberNameFunction OphthalmicdivisionSupplies cornea, superior conjunctiva,skin of dorsum of external nose,forehead, scalp, superior eyelids, andalso mucous nasal cavity, ethmoid,frontal and sphenoid sinuses MandibulardivisionSkin of lower lip, buccal, parotid andtemporal regions of face, external ear(Auricle, tympanic membrane andacoustic meatus), mucous membraneof mouth and anterior two-third paof tongue. Supplies 4 muscles of mastication,mylohyoid, anteriorbelly of digastric, tensor tympani andtensor veli palatini VI Abducent Lateral rectus muscle turns eyeballlaterally NumberName FunctionVIIFacial Taste from anterior two-thirdsof tongue, from floor of mouth andpalate Muscles of face and scalp,stapedius muscle, posterior belly ofdigastric and stylohyoid muscles Submandibular and sublingualsalivary glands,the lacrimal gland, andglands of nose and palate VIII VestibulocochlearCochlear Organ of Coi--hearing VestibularFrom utricle and saccule andsemicircular canals--positionand movement of headIXGlossopharyngeal General sensation and taste fromposterior one-third of tongue andpharynx; carotid sinus (baroreceptor);and carotid body (chemoreceptor) NumberName Function Stylopharyngeus muscle--assistsswallowingParasympathetic parotid salivary gland X Vagus Hea and great thoracic blood vessels;larynx, trachea, bronchi, and lungs;alimentary tract from pharynx tosplenic flexure ofcolon; liver, kidneys, and pancreas XI Spinal accessory Cranial rootSternocleidomastoid and trapeziusmuscles Cranial root Muscles of soft palate (excepttensor veli palatini), pharynx (exceptstylopharyngeal), and larynx (exceptcricothyroid) in branches of vagus XII Hypoglossal Muscles of tongue (exceptpalatoglossus) controlling its shape andmovement" "This is a real-world medical entrance exam question, please give the true answer based on the question and selection. Topic name: None, Subject name: Gynaecology & Obstetrics","Regimen followed in expectant management of placenta previa: Select and explain. A. Liley's method, B. Crede's method, C. Macafee and Johnson regime, D. Brandt-Andrews Method",C: Macafee and Johnson regime,"Answer: C: Macafee and Johnson regime Explanation: None" "This is a real-world medical entrance exam question, please give the true answer based on the question and selection. Topic name: None, Subject name: Social & Preventive Medicine","Recommnended daily intake of iodine in pregnancy- Select and explain. A. 90 microgram, B. 120 microgram, C. 150 microgram, D. 250 microgram",D: 250 microgram,"Answer: D: 250 microgram Explanation: Answer- D. 250 microgramRecommended daily intake of iodine in pregnancy is 250 microgram." "This is a real-world medical entrance exam question, please give the true answer based on the question and selection. Topic name: None, Subject name: Anatomy","True about Levator Ani except - Select and explain. A. Converge downwards & medially, B. Suppos viscera, C. Made up of iliococcygeus & pubococcygeus, D. Attached to pelvic brim",D: Attached to pelvic brim,"Answer: D: Attached to pelvic brim Explanation: Ans. D. Attached to pelvic brimThe levator ani is made up of three pas:Iliococcygeus musclePubococcygeus musclePuborectalis muscleOrigin & Inseion:The levator ani arises, in front, from the posterior surface of the superior pubic ramus lateral to the symphysis behind, from the inner surface of the spine of the ischium and between these two points, from the obturator fascia.The fibers pass downward and backward to the middle line of the floor of the pelvis The most posterior are inseed into the side of the last two segments of the coccyx; those placed more anteriorly unite with the muscle of the opposite side, in a median fibrous ridge called the anococcygeal body or raphe, which extends between the coccyx and the margin of the anus.Action: Suppos the viscera in pelvic cavity" "This is a real-world medical entrance exam question, please give the true answer based on the question and selection. Topic name: None, Subject name: Dental","Glossy smooth margins of a casting are due to Select and explain. A. Shrinkage of alloy on cooling, B. Investment breakdown, C. incomplete wax elimination, D. incomplete gas elimination",C: incomplete wax elimination,"Answer: C: incomplete wax elimination Explanation: None" "This is a real-world medical entrance exam question, please give the true answer based on the question and selection. Topic name: None, Subject name: Pediatrics","A 5–years old male child presents with episodic anaemia and jaundice since birth. He is least likely to have which of the following Select and explain. A. Hereditary spherocytosis, B. Siclde cell anemia, C. PNH, D. G–6–PD deficiency",C: PNH,"Answer: C: PNH Explanation: Causes of Jaundice since birth are: (i) Rh incompatibility (erythroblastosis fetalis) ABO incompatibility Congenital infections (TORCH) Sepsis Concealed hemorrhage Red cell membrane defect (hereditary spherocytosis) Red cell enzyme defect (G6PD deficiency) So, option a & d can cause jaundice since birth. In sickle cell anemia, affected infants do not develop symptoms in the first few months of life because the hemoglobin produced by the developing fetus (fetal hemoglobin) protects the red blood cells from sickling. This fetal hemoglobin disappears after 5 month of age so that by 5 months of age, the sickling of the red blood cells is prominent and symptoms begin. PNH is manfested in adults. So, both PNH and sickle cell anemia does not cause jaundice since birth. But among these two I would prefer PNH as the answer because it is manifested in adulthood while the patient in question is a 5-years old child. Sickle cell anemia symptoms develop at the age of 5 months and it is one of the cause of jaundice" "This is a real-world medical entrance exam question, please give the true answer based on the question and selection. Topic name: None, Subject name: Dental","Best medium to transport/store an avulsed tooth, which is not to be reimplanted immediately? (OR) the medium which maintains the vitality of PDl and thus indicated for long duration transport of an avulse tooth Select and explain. A. Viaspan, B. Hanks balanced salt solution, C. Coconut water, D. Milk",A: Viaspan,"Answer: A: Viaspan Explanation: None" "This is a real-world medical entrance exam question, please give the true answer based on the question and selection. Topic name: None, Subject name: Social & Preventive Medicine","True statements regarding the calculation of HDI are all of the following except Select and explain. A. In gross enrolment, only secondary education is considered, not primary education, B. 1/3rd weightage is given to adult literacy, C. 2/3`dweightage is given to gross enrolmen, D. Country's achievement is computed on the basis of adult literacy",D: Country's achievement is computed on the basis of adult literacy,"Answer: D: Country's achievement is computed on the basis of adult literacy Explanation: Ans. d. Country's achievement is computed on the basis of adult literacyThe knowledge and education index of a country is computed on the basis of adult literacy rate and combined gross enrollment ratio.""" "This is a real-world medical entrance exam question, please give the true answer based on the question and selection. Topic name: None, Subject name: ENT","In complete bilateral palsy of recurrent laryngeal nerves, there is - Select and explain. A. Complete loss of speech with stridor and dyspnea, B. Complete loss of speech but not difficulty in breathing, C. Preservation of speech with severe stridor and dyspnea, D. Preservation of speech and not difficulty in breathing",C: Preservation of speech with severe stridor and dyspnea,"Answer: C: Preservation of speech with severe stridor and dyspnea Explanation: None" "This is a real-world medical entrance exam question, please give the true answer based on the question and selection. Topic name: None, Subject name: Pediatrics","Delayed neuronal migration and organization leads to ceain disorders. Which of the following is the least likely possibility? Select and explain. A. Lissencephaly, B. Schizencephaly, C. Polymicrogyria, D. Focal coical dyplasia without balloon cells",D: Focal coical dyplasia without balloon cells,"Answer: D: Focal coical dyplasia without balloon cells Explanation: Ans. d. Focal coical dyplasia without balloon cells (Ref http://enm kipedia.org/wiki/Neuronal migration_ disorder)Delayed neuronal migration and organization leads to ceain disorders. Least likely possibility is focal coical dyplasia without balloon cells" "This is a real-world medical entrance exam question, please give the true answer based on the question and selection. Topic name: None, Subject name: Biochemistry","Enzyme involved in gluconeogenesis are all except: Select and explain. A. Pyruvate carboxylase, B. PEP carboxykinase, C. Pyruvate kinase, D. Glucose-6-phosphatase",C: Pyruvate kinase,"Answer: C: Pyruvate kinase Explanation: Answer-c- Pyruvate kinaseMitochondrial pyruvate carboxylase catalyzes the carboxylation of Pyruvate to Oxaloacetate, It is an ATP-requiring reaction, Biotin is the coenzyme.Phosphoenolpyruvate Carboxykinase: Catalyzes the decarboxylation and phosphorylation of oxaloacetate to phosphoenolpyruvate(PEPCK) (Cytosol) using GTP as the phosphate donor.The conversion of glucose-6-phosphate to glucose is catalyzed by glucose 6-phosphatase" "This is a real-world medical entrance exam question, please give the true answer based on the question and selection. Topic name: None, Subject name: Dental","Concentration of adrenaline in local anaesthetic solution for achieving haemostasis & vasoconstriction at surgical site? Select and explain. A. 1:20000, B. 1:50000, C. 1:80000, D. 1:100000",B: 1:50000,"Answer: B: 1:50000 Explanation: None" "This is a real-world medical entrance exam question, please give the true answer based on the question and selection. Topic name: None, Subject name: Anatomy","While performing drainage of fluid from the pleural cavity, the needle is introduced through all of the following structures except- Select and explain. A. Skin, B. Thoracic fascia, C. Pulmonary pleura, D. Intercostal muscles",C: Pulmonary pleura,"Answer: C: Pulmonary pleura Explanation: Answer- C (Pulmonary pleura)(Ref. BDC 5/e p Vol-l/p2l2)Pleural space lies between the parietal pleura and the visceral (Pulmonary) pleura.Parietal pleural lines the inner surface of the chest wall.Visceral pleura is invested on the surface of the lung.Thus to reach the pleural space for pleural fluid tapping, parietal pleura is pierced and not the visceral pleura." "This is a real-world medical entrance exam question, please give the true answer based on the question and selection. Topic name: None, Subject name: Pharmacology","Inter dose interval depends on? Select and explain. A. Half life of drug, B. Dose of drug, C. Age of patient, D. Bioavailability of drug",A: Half life of drug,"Answer: A: Half life of drug Explanation: None" "This is a real-world medical entrance exam question, please give the true answer based on the question and selection. Topic name: None, Subject name: ENT","Which is the most common type of congenital ossicular dysfunction? Select and explain. A. Isolated stapes defect, B. Stapes defect with fixation of footplate and lenticular process involvement., C. Defective lenticular process of incus, D. None of the above.",B: Stapes defect with fixation of footplate and lenticular process involvement.,"Answer: B: Stapes defect with fixation of footplate and lenticular process involvement. Explanation: Ans: B. Stapes defect with fixation of footplate and lenticular process involvement.(Ref Nelson 20/e p3071)Most common type of congenital ossicular dysfunction is stapes defect with fixation of footplate and lenticular process involvement." "This is a real-world medical entrance exam question, please give the true answer based on the question and selection. Topic name: None, Subject name: Pathology","Earliest transient change following tissue injury Select and explain. A. Neutrophilia, B. Neutropenia, C. Monocytoses, D. Lymphocytoses",A: Neutrophilia,"Answer: A: Neutrophilia Explanation: None" "This is a real-world medical entrance exam question, please give the true answer based on the question and selection. Topic name: None, Subject name: Dental","Canine can be classified as Select and explain. A. Haplodont, B. Triconodont, C. Tritubercular teeth, D. None of the above",A: Haplodont,"Answer: A: Haplodont Explanation: None" "This is a real-world medical entrance exam question, please give the true answer based on the question and selection. Topic name: None, Subject name: Ophthalmology","Microaneurysms are the earliest manifestation of diabetic retinopathy. Which of the following layer is involved in diabetic etinopathy? Select and explain. A. Outer plexiform layer, B. Inner nuclear layer, C. Layer of rods and cones, D. Retinal pigment epithelium",B: Inner nuclear layer,"Answer: B: Inner nuclear layer Explanation: Ans. b. Inner nuclear layerMlcroaneurysms are the earliest clinical sign of diabetic retinopathy and occur secondary tocapillary wall outpouching due to pericyte loss.. Thev appear as small red dots' MicroaneurysStructural changes in the retinal microcirculation have been associated with a physiologic breakdown in the bloodretinalbarrier. ihus, the retinal microcirculation in diabetics may be exceptionally leaky, giving rise to macularedema, a common cause of visual loss in these patients.The vascular changes may also produce exudates that accumulate in the outer plexiform layerq" "This is a real-world medical entrance exam question, please give the true answer based on the question and selection. Topic name: None, Subject name: Dental","The type of bone present in the inter-radicular area is Select and explain. A. Cortical, B. Cancellous, C. Osteophytic, D. Exophytic",B: Cancellous,"Answer: B: Cancellous Explanation: None" "This is a real-world medical entrance exam question, please give the true answer based on the question and selection. Topic name: AIIMS 2017, Subject name: Medicine","Asymptomatic hep B is common in 2-3% normal population, but there is increased risk of transmission into hepatocellular carcinoma. Why? Select and explain. A. Inability to induce inflammation to remove the organism, B. High level of transaminases, C. High rate of proliferation of virus, D. Integration of viral DNA with host DNA",D: Integration of viral DNA with host DNA,"Answer: D: Integration of viral DNA with host DNA Explanation: Option A- Explanation for carrier stage Option C- Refers to high infectivity of the virus so basically higher chances of infection but not cancer. Option D- This leads to inhibition of apoptosis so might cause cancer of liver (HCC)" "This is a real-world medical entrance exam question, please give the true answer based on the question and selection. Topic name: None, Subject name: Dental","Which of the following alloys has the highest coefficient of friction? Select and explain. A. SS, B. TMA, C. Ni-Ti, D. Elgiloy",B: TMA,"Answer: B: TMA Explanation: The surface of NiTi is rougher (because of surface defects, not the quality of polishing) than that of beta-Ti, which in turn is rougher than steel. However, there is little or no correlation for orthodontic wires between the coefficients of friction and surface roughness. (i.e., interlocking and plowing are not significant components of the total frictional resistance).  Although NiTi has greater surface roughness, beta-Ti has greater frictional resistance, it turns out that as the titanium content of an alloy increases, its surface reactivity increases, and the surface chemistry is a major influence on frictional behavior. Thus, beta-Ti, at 80% titanium, has a higher coefficient of friction than NiTi at 50% titanium, and there is greater frictional resistance to sliding with either than with steel. Key Concept Although NiTi has greater surface roughness, beta-Ti has greater frictional resistance. Ref: Proffit 4th edition page 378" "This is a real-world medical entrance exam question, please give the true answer based on the question and selection. Topic name: None, Subject name: Dental","Which of the following has strong familial inheritance: Select and explain. A. Functional class III, B. Class I type I, C. Class II div I, D. Class II div II",D: Class II div II,"Answer: D: Class II div II Explanation: None" "This is a real-world medical entrance exam question, please give the true answer based on the question and selection. Topic name: None, Subject name: Pharmacology","Which anti-cholinergic drug doesn't cross the blood brain barrier and placenta? Select and explain. A. Glycopyrrolate bromide, B. Hyoscine bromide, C. Hyoscine butylbromide, D. Atropine",A: Glycopyrrolate bromide,"Answer: A: Glycopyrrolate bromide Explanation: None" "This is a real-world medical entrance exam question, please give the true answer based on the question and selection. Topic name: AIIMS 2018, Subject name: Biochemistry","Which of the following is a primarily RNA based technique? Select and explain. A. Next generation sequencing, B. PCR, C. Sanger's technique, D. Western blotting",B: PCR,"Answer: B: PCR Explanation: PCR is Reverse Transcriptase PCR, where staing material is RNA. It is used to make cDNA, which is amplified. Thus RNA can be amplified by this method and quantification of mRNA can also be done (see fig). Here we use Tth polymerase enzyme which has both reverse transcriptase and polymerase activity WITH REGARD TO OTHER OPTIONS: Western blot is to detect protein. Next generation sequencing & Sanger's technique are for DNA sequencing. ADDITIONAL EDGE: Used to detect RNA expression Sample here is RNA. This RNA is conveed to cDNA by enzyme reverse transcriptase. Tth polymerase (Thermus thermophilus HB-8) has Polymerase as well as reverse transcriptase activity when mixed with manganese ions and can thus be used for the amplification of RNA to cDNA. This polymerase is thermostable but does not has proofreading activity. So it is usually combined with a proofreading enzyme." "This is a real-world medical entrance exam question, please give the true answer based on the question and selection. Topic name: None, Subject name: Dental","A child accidentally swallows 10 cc of a 10% fluoride solution. The most effective immediate action is to: Select and explain. A. Have the child drink copious quantities of water, B. Have the child drink a 100/0 sodium bicarbonate solution, C. Have the child drink milk or some other calcium containing liquid, D. Send the child to the family physician",C: Have the child drink milk or some other calcium containing liquid,"Answer: C: Have the child drink milk or some other calcium containing liquid Explanation: None" "This is a real-world medical entrance exam question, please give the true answer based on the question and selection. Topic name: None, Subject name: Pharmacology","I.V. anesthesia is: Select and explain. A. Propofol, B. Sevoflurane, C. Flumazanil, D. Naloxane",A: Propofol,"Answer: A: Propofol Explanation: None" "This is a real-world medical entrance exam question, please give the true answer based on the question and selection. Topic name: None, Subject name: Pathology","Which of the following is carcinoma of the skin, spreads by local invasion and has no tendency to metastasise? Select and explain. A. Malignant melanoma, B. Basal cell carcinoma, C. Fibrosarcoma, D. Leukoplakia",B: Basal cell carcinoma,"Answer: B: Basal cell carcinoma Explanation: None" "This is a real-world medical entrance exam question, please give the true answer based on the question and selection. Topic name: None, Subject name: Surgery","Chronic alcoholic comes with pain epigastrium and recurrent vomiting. On examination guarding in upper epigastrium. Chest X-ray normal. What to do next? Select and explain. A. UGI endoscopy, B. Serum lipase, C. CECT, D. Alcohol breath test",B: Serum lipase,"Answer: B: Serum lipase Explanation: Answer- B. Serum lipaseSerum amylase and lipases are the initial investigations done in patients with acute pancreatitis.CECT is the investigation of choice but the initial investigation in such patients." "This is a real-world medical entrance exam question, please give the true answer based on the question and selection. Topic name: None, Subject name: Dental","Lateral movement of condyle is caused by: Select and explain. A. Contralateral lateral pterygoid muscle, B. Both lateral and medial pterygoid muscle, C. Epsilateral lateal pterygoid muscle, D. Bilateral contraction of lateral pterygoid muscle",B: Both lateral and medial pterygoid muscle,"Answer: B: Both lateral and medial pterygoid muscle Explanation: None" "This is a real-world medical entrance exam question, please give the true answer based on the question and selection. Topic name: None, Subject name: Surgery","Submandibular duct is exposed via intraoral approach by incising the: Select and explain. A. Buccinator, B. Mucous membrane, C. Masseter, D. All of the above",B: Mucous membrane,"Answer: B: Mucous membrane Explanation: None" "This is a real-world medical entrance exam question, please give the true answer based on the question and selection. Topic name: AIIMS 2018, Subject name: Anatomy","Purkinje fibers project to which of the following Select and explain. A. Caudate nucleus, B. Red nucleus, C. Dentate nucleus, D. Ventero lateral nucleus of thalamus",C: Dentate nucleus,"Answer: C: Dentate nucleus Explanation: Dentate nucleus Information processed in the Purkinje cell layer of cerebellum sent to deep cerebellar nuclei (4 in total), which form the cerebellar output. Largest collection of fibers originates from the dentate nucleus, which then project to contralateral ventral nucleus of thalamus after decussating in superior cerebellar peduncle." "This is a real-world medical entrance exam question, please give the true answer based on the question and selection. Topic name: None, Subject name: Radiology","The bisecting angle technique is used to take Select and explain. A. Bitewing radiographs, B. Periapical radiographs, C. True occlusal radiographs of the mandible, D. None of the above",B: Periapical radiographs,"Answer: B: Periapical radiographs Explanation: None" "This is a real-world medical entrance exam question, please give the true answer based on the question and selection. Topic name: None, Subject name: Biochemistry","Physiological active form of cholesterol is: Select and explain. A. Lipoprotein, B. Lipid, C. Free cholesterol, D. None of the above",C: Free cholesterol,"Answer: C: Free cholesterol Explanation: Free Cholesterol is considered as physiological active form of the cholesterol." "This is a real-world medical entrance exam question, please give the true answer based on the question and selection. Topic name: None, Subject name: Dental","Most difficult access cavity preparation is in Select and explain. A. Mandibular incisors, B. Mandibular Canine, C. Mandibular premolar, D. Mandibular molar",A: Mandibular incisors,"Answer: A: Mandibular incisors Explanation: None" "This is a real-world medical entrance exam question, please give the true answer based on the question and selection. Topic name: AIIMS 2018, Subject name: Pediatrics","A 4 - year - old child presented with painless genu valgum. X - ray bilateral knee was done and is shown below. Which of the following is most likely diagnosis? Select and explain. A. Rickets, B. Scurvy, C. Congenital anomaly, D. Trauma to epiphysis",B: Scurvy,"Answer: B: Scurvy Explanation: Scurvy X-ray - pencil thin outline of coex Subperiosteal hemorrhage Wimberger sign Pelkan spur Trummerfeld zone" "This is a real-world medical entrance exam question, please give the true answer based on the question and selection. Topic name: None, Subject name: Forensic Medicine","A Patient is admitted with Insomnia, Agitation, Diarrhoea, Dilated pupils and Sweating, what is the type of poisoning? Select and explain. A. Heroin, B. Cocaine, C. Cannabis, D. Ecstasy",A: Heroin,"Answer: A: Heroin Explanation: Ans. A. Heroin* Insomnia, Agitation,Diarrhea and Altered sensorium* There may be white froth from mouth, in PM examination * Methadone is the drug of Choice" "This is a real-world medical entrance exam question, please give the true answer based on the question and selection. Topic name: None, Subject name: Dental","A conventional gingivectomy will Select and explain. A. Eliminate infra-bony pockets, B. Eliminate false pockets, C. Preserve width of attached gingiva, D. Facilitate healing by primary intention",B: Eliminate false pockets,"Answer: B: Eliminate false pockets Explanation: None" "This is a real-world medical entrance exam question, please give the true answer based on the question and selection. Topic name: None, Subject name: Dental","Increased depth in posterior palatal seal area of denture causes: Select and explain. A. Tingling sensation, B. Gagging, C. Increased retention, D. Unseating of denture base",D: Unseating of denture base,"Answer: D: Unseating of denture base Explanation: None" "This is a real-world medical entrance exam question, please give the true answer based on the question and selection. Topic name: None, Subject name: Pharmacology","Pegloticase used in which of the following conditions? Select and explain. A. Chronic Gout, B. Psoriatic ahritis, C. RA, D. Paralytic Ileus",A: Chronic Gout,"Answer: A: Chronic Gout Explanation: Ans. A. Chronic Gout* Pegloticase is a medication for the treatment of Severe, Treatment - Refractory, Chronic Gout.* It is a third line treatment in those in whom other treatments are not tolerated.* Drug is administered by infusion intravenously." "This is a real-world medical entrance exam question, please give the true answer based on the question and selection. Topic name: None, Subject name: Biochemistry","True about cardiolipin is all except Select and explain. A. Found in inner mitochondrial membrane, B. Play role in process of apoptosis, C. Is Diphosphatidyl glycerol, D. Has choline base which has labile methyl group",D: Has choline base which has labile methyl group,"Answer: D: Has choline base which has labile methyl group Explanation: Cardiolipin (diphosphatidylglycerol) is a phospholipid present in mitochondria.   It is formed from phosphatidylglycerol, which in turn is synthesized from CDP-diacylglycerol and glycerol 3-phosphate. Cardiolipin, found in the inner membrane of mitochondria, has a key role in mitochondrial structure and function and is also thought to be involved in programmed cell death (apoptosis). Cephalin: Has choline base which has labile methyl group.   Harper's Illustrated Biochemistry 30th edition page no 248" "This is a real-world medical entrance exam question, please give the true answer based on the question and selection. Topic name: None, Subject name: Surgery","During cryosurgery Select and explain. A. Cells get evaporated, B. Cells will not die, only freeze, C. Cell death occurs when the temperature falls below -20°C, D. Patients need general anesthesia",C: Cell death occurs when the temperature falls below -20°C,"Answer: C: Cell death occurs when the temperature falls below -20°C Explanation: None" "This is a real-world medical entrance exam question, please give the true answer based on the question and selection. Topic name: AIIMS 2019, Subject name: Physiology","Golgi tendon organ function is? Select and explain. A. Detects the dynamic change in muscle length, B. Detects the muscle tension, C. Detects the muscle stretch, D. Detects the muscle strength",B: Detects the muscle tension,"Answer: B: Detects the muscle tension Explanation: There are two proprioceptors in a muscle: - Muscle spindle Golgi tendon organ - Situated in belly or muscle - Formed by intra-fusal fibres - Detects the length of muscle when muscle is stationary - Detect rate of change of length - Present in tendon - Made up of tendon fibres - Detects the muscle tension. - Detects rate of change of tension." "This is a real-world medical entrance exam question, please give the true answer based on the question and selection. Topic name: None, Subject name: Dental","A cephalogram of a 10-year-old child shows ANB = -4° and facial angle of 98°. The case is of: Select and explain. A. Maxillary hypoplasia, B. Pseudoclass III malocclusion, C. True Class III malocclusion, D. Maxillary prognathism",C: True Class III malocclusion,"Answer: C: True Class III malocclusion Explanation: None" "This is a real-world medical entrance exam question, please give the true answer based on the question and selection. Topic name: None, Subject name: Dental","Tobacco Chewing/ smoking is the most common predisposing cause of which of the following: Select and explain. A. Cyclic neutropenia, B. Juvenile periodontitis, C. Necrotising periodontitis, D. Necrotising ulcerative gingivitis (ANUG)",D: Necrotising ulcerative gingivitis (ANUG),"Answer: D: Necrotising ulcerative gingivitis (ANUG) Explanation: None" "This is a real-world medical entrance exam question, please give the true answer based on the question and selection. Topic name: None, Subject name: Anatomy","The main arterial trunk supplying the infra temporal fossa is: Select and explain. A. Infratemporal artery, B. Deep temporal artery, C. Maxillary artery, D. Posterior superior alveolar artery",C: Maxillary artery,"Answer: C: Maxillary artery Explanation: None" "This is a real-world medical entrance exam question, please give the true answer based on the question and selection. Topic name: None, Subject name: Medicine","Reticulocytosis is seen in: Select and explain. A. Hemolytic anemia, B. Megaloblastic anemia, C. Aplastic anemia, D. Iron deficiency anemia",A: Hemolytic anemia,"Answer: A: Hemolytic anemia Explanation: Red cell precursors formed in the bone marrow from the erythroid (CFU–E) progenitor cells are called erythroblasts or normoblasts. These divide and acquire haemoglobin, which turns the cytoplasm pink; the nucleus condenses and is extruded from the cell. The first non-nucleated red cell is a reticulocyte, which still contains ribosomal material in the cytoplasm, giving these large cells a faint blue tinge (‘polychromasia’). Reticulocytes lose their ribosomal material and mature over 3 days, during which time they are released into the circulation. Increased numbers of circulating reticulocytes (reticulocytosis) reflect increased erythropoiesis. Proliferation and differentiation of red cell precursors is stimulated by erythropoietin, a polypeptide hormone produced by renal interstitial peritubular cells in response to hypoxia. Failure of erythropoietin production in patients with renal failure causes anaemia, which can be treated with exogenous recombinant erythropoietin or similar pharmacological agents called erythropoiesis-stimulating agents, e.g. darbepoetin. Reference: : Davidson P R I N C I P L E S   and Practice O F M E D I C I N E 23rd edition page no  915" "This is a real-world medical entrance exam question, please give the true answer based on the question and selection. Topic name: None, Subject name: Dental","The material used in its pure form in dentistry is: Select and explain. A. Composite, B. Silver, C. Gold, D. Amalgam",C: Gold,"Answer: C: Gold Explanation: Pure gold has been in use in dentistry in the United States for more than 100 years. Various techniques have been advanced for its use in the restoration of teeth. Ref: Sturdevant operative dentistry 7 ed page no e69" "This is a real-world medical entrance exam question, please give the true answer based on the question and selection. Topic name: None, Subject name: Dental","MB2 (In maxillary first molar)can be seen through Dental operating microscope in what percentage of cases Select and explain. A. 90-95%, B. 70-85%, C. 62-70%, D. 30-40%",A: 90-95%,"Answer: A: 90-95% Explanation: None" "This is a real-world medical entrance exam question, please give the true answer based on the question and selection. Topic name: None, Subject name: Surgery","Post-auricular ecchymosis in cases of fracture of the base of the skull is called: Select and explain. A. Battle's sign, B. Tinel's sign, C. Trousseau's sign, D. Nikolsky's sign",A: Battle's sign,"Answer: A: Battle's sign Explanation: None" "This is a real-world medical entrance exam question, please give the true answer based on the question and selection. Topic name: None, Subject name: Surgery","Transverse symphyseal fracture of mandible can be managed by all of the following, except Select and explain. A. Lag screws, B. 2 mm compression plate, C. 2.5 mm monocortical plate, D. 1.5 mm single miniplates",D: 1.5 mm single miniplates,"Answer: D: 1.5 mm single miniplates Explanation: None" "This is a real-world medical entrance exam question, please give the true answer based on the question and selection. Topic name: None, Subject name: Dental","Modified access cavity in maxillary molar to accommodate accessory orifice is known as Select and explain. A. Clove leaf, B. Shamrock preparation, C. Both, D. None",C: Both,"Answer: C: Both Explanation: None" "This is a real-world medical entrance exam question, please give the true answer based on the question and selection. Topic name: None, Subject name: Surgery","Most common cause of interochanteric fracture in old is - Select and explain. A. Osteoporosis, B. Paget's dis, C. Osteopetrosis, D. Osteomalacia",A: Osteoporosis,"Answer: A: Osteoporosis Explanation: Answer- A. OsteoporosisHaving osteoporosisHaving a history of other bone problems or fracturesHaving low bone density and low muscle mass" "This is a real-world medical entrance exam question, please give the true answer based on the question and selection. Topic name: None, Subject name: Dental","Split finger technique is used for Select and explain. A. Gingival augmentation, B. Sinus augmentation procedures, C. 1^st stage implant surgery, D. Creation of inter-implant papillae",D: Creation of inter-implant papillae,"Answer: D: Creation of inter-implant papillae Explanation: None" "This is a real-world medical entrance exam question, please give the true answer based on the question and selection. Topic name: None, Subject name: Biochemistry","Second messenger for smooth muscle relaxation mediated by NO is: Select and explain. A. Ca', B. cAMP, C. cGMP, D. Magnesium",C: cGMP,"Answer: C: cGMP Explanation: Ans: C. cGMPRef.: Harper 30/e p290, 437)Second messenger for smooth muscle relaxation mediated by NO is cGMP.""GTP serves as an allosteric regulator and as an energy source for protein synthesis, and cGMP serves as a second messenger in response to nitric oxide (NO) during relaxation of smooth muscle." "This is a real-world medical entrance exam question, please give the true answer based on the question and selection. Topic name: None, Subject name: Pharmacology","Which factor of the following contributes to short duration of anesthetic action of single dose thiopental Select and explain. A. Rapid biotransformation, B. Rapid accumulation in body fat, C. High lipid solubility, D. Ability to enter and leave brain tissue every rapidly",C: High lipid solubility,"Answer: C: High lipid solubility Explanation: None" "This is a real-world medical entrance exam question, please give the true answer based on the question and selection. Topic name: None, Subject name: Dental","Fibroblasts in pulp produces collagen Select and explain. A. Type I only, B. Type II and III, C. Type I and III, D. Type II and I",C: Type I and III,"Answer: C: Type I and III Explanation: None" "This is a real-world medical entrance exam question, please give the true answer based on the question and selection. Topic name: None, Subject name: Pharmacology","If a bacteria were susceptible to both penicillin and Erythromycin, then it would not be appropriate to treat the patients with both antibiotics at the same time because Select and explain. A. Penicillin is inactivated by erythromycin, B. Erythromycin exerts its effects extracellularly, C. Erythromycin enhances the β-lactamases activity, D. Penicillin is only effective against growing cells",D: Penicillin is only effective against growing cells,"Answer: D: Penicillin is only effective against growing cells Explanation: None" "This is a real-world medical entrance exam question, please give the true answer based on the question and selection. Topic name: None, Subject name: Ophthalmology","Which of the following is false about indirect opthalmoscopy? Select and explain. A. Convex lens is used, B. Image is viual and erect, C. Magnification is 4-5 times, D. It is so bright that regular haziness is penetrated",B: Image is viual and erect,"Answer: B: Image is viual and erect Explanation: Answer- B. Image is viual and erectIndirect Ophthalmoscopy:Convex lens is used of +16 to +18 DiopterImage is real and inveedMagnification is 4-5 timesIt is so bright that regular haziness is penetrated, useful for hazy media." "This is a real-world medical entrance exam question, please give the true answer based on the question and selection. Topic name: None, Subject name: Pediatrics","A child with 22-25 stool/day, 3 day old pneumonitis, no passage of urine from 36 hours. low B.P. Blood pH – 7.21. Urine Na+–18 meq/L, S. Urea 120, serum Creatinine 1.2 indicate – Select and explain. A. Acute cortical necrosis, B. Acute tubular necrosis, C. Pre–renal Azotemia, D. Acute medullary necrosis",C: Pre–renal Azotemia,"Answer: C: Pre–renal Azotemia Explanation: History and clinical features of the child suggests that he is suffering from acute renal failure. From the given values we have to get the cause of this renal failure. H/o diarrhea with low BP suggest dehydration —> a cause of prerenal ARE" "This is a real-world medical entrance exam question, please give the true answer based on the question and selection. Topic name: None, Subject name: Dental","Number of fossa in upper 1st molar Select and explain. A. 2 major 2 minor, B. 3 major 2 minor, C. 2 major 1 minor, D. 1 major 2 minor",A: 2 major 2 minor,"Answer: A: 2 major 2 minor Explanation: 2 major fossa = Distal and Central 2 minor fossa also known as triangular fossa= Distal and Mesial" "This is a real-world medical entrance exam question, please give the true answer based on the question and selection. Topic name: None, Subject name: Pathology","For wound healing which mineral is helpful? Select and explain. A. Calcium, B. Selenium, C. Magnesium, D. Copper",C: Magnesium,"Answer: C: Magnesium Explanation: None" "This is a real-world medical entrance exam question, please give the true answer based on the question and selection. Topic name: AIIMS 2018, Subject name: Medicine","Which of the following drugs is used for treatment of cancer associated thromboembolism? Select and explain. A. LMW heparin, B. Anti-thrombin III inhibitors, C. Direct factor Xa inhibitors, D. Warfarin",A: LMW heparin,"Answer: A: LMW heparin Explanation: Cancer is a hypercoagulable state, so there is clot formation in the leg of patient and that goes to brain and cause stroke like presentation. Rx= LMW heparin (enoxaparin 40 mg OD 4 weeks)" "This is a real-world medical entrance exam question, please give the true answer based on the question and selection. Topic name: None, Subject name: Dental","Nuk Sauger Nipple is : Select and explain. A. Non-physiologic nipple., B. Conventional nipple., C. Physiologic nipple., D. None.",C: Physiologic nipple.,"Answer: C: Physiologic nipple. Explanation: None" "This is a real-world medical entrance exam question, please give the true answer based on the question and selection. Topic name: None, Subject name: Pediatrics","A child who was normal at bih develops chronic liver failure and muscle weakness at 3 months of age. On investigations, serum glucose is low, along with ketoacidosis and decreased pH. ALT and AST are raised. Blood lactate and uric acid levels are normal. Intravenous glucagon given after meals raises the blood glucose levels, but does not raise glucose when given after an overnight fast. Liver biopsy shows increased glycogen in liver. Which is the enzyme likely to be defective in this child? Select and explain. A. Glucose-6-phosphatase, B. Muscle phosphorylase, C. Branching enzyme, D. Debranching enzyme",D: Debranching enzyme,"Answer: D: Debranching enzyme Explanation: Ans: D: Debranching enzyme(Ref: Nelson 20/e p717-720; Harrison 19/e p433 e-2, 18Ie p3200, 3201)In this child, a combination of liver and muscle involvement with ketoacidosis and raised liver enzymes points towards Type III glycogen storage disease, i.e. Cori's disease caused by deficiency of debranching enzyme.Type IIIa Glycogen Storage Disease or Cori's Disease or Forbes Disease:Due to deficiency ofglycogen debranching enzyme activity.Debranching enzyme, together with phosphorylase, is responsible for complete degradation of glycogen. When debranching enzyme is defective, glycogen breakdown is incomplete and an abnormal glycogen with sho outer branch chains and resembling limit dextrin accumulates.Deficiency of glycogen debranching enzyme causes hepatomegaly, hypoglycemia, sho stature, variable skeletal myopathy & variable eardiomyopathy." "This is a real-world medical entrance exam question, please give the true answer based on the question and selection. Topic name: None, Subject name: Surgery","Ameloblastoma histologically resembles: Select and explain. A. BCC, B. SCC, C. Osteosarcoma, D. Fibrosarcoma",A: BCC,"Answer: A: BCC Explanation: None" "This is a real-world medical entrance exam question, please give the true answer based on the question and selection. Topic name: None, Subject name: Dental","The depth of clinical gingival sulcus is the distance between the gingival margin to the Select and explain. A. Cementoenamel Junction, B. Alveolar crest, C. Apical extension of junctional epithelium, D. Apical penetration of the probe",D: Apical penetration of the probe,"Answer: D: Apical penetration of the probe Explanation: None" "This is a real-world medical entrance exam question, please give the true answer based on the question and selection. Topic name: AIIMS 2018, Subject name: Medicine","ECG shows a mean axis of 90 degrees. In which of the following would be present the maximum voltage of R wave? Select and explain. A. III, B. I, C. aVF, D. aVL",C: aVF,"Answer: C: aVF Explanation: Lead I = R wave taller than S = +ve Lead avF = R wave taller than S = +ve" "This is a real-world medical entrance exam question, please give the true answer based on the question and selection. Topic name: None, Subject name: Dental","Which of the following is a tertiary monoblock system of obturation of the root canal. Select and explain. A. Resilon monoblock obturation, B. Active Gutta percha, C. Orthograde obturation with MTA, D. Gutta Percha and AH plus sealer",B: Active Gutta percha,"Answer: B: Active Gutta percha Explanation: None" "This is a real-world medical entrance exam question, please give the true answer based on the question and selection. Topic name: AIIMS 2018, Subject name: Pathology","Which is false about apoptosis? Select and explain. A. No inflammation, B. Plasma membrane intact, C. Organelle swelling, D. Affected by dedicated genes",C: Organelle swelling,"Answer: C: Organelle swelling Explanation: Apoptosis is genetically regulated, hence apoptosis is sometimes referred to as programmed cell death. Plasma membrane remains intact in apoptosis hence there is no leakage of enzymes and inflammation Apoptosis cause activation of caspase and protease which cause cells to shrink . Hence organelle swelling is not a feature of apoptosis. It is a feature of reversible cell injury or necrosis Characteristic feature of apoptosis microscopically is chromatin condensation" "This is a real-world medical entrance exam question, please give the true answer based on the question and selection. Topic name: None, Subject name: Dental","During polymerisation of acrylic resin, above what temperature benzoyl peroxide form free radicals? Select and explain. A. 25°C, B. 37°C, C. 45°C, D. 60°C",D: 60°C,"Answer: D: 60°C Explanation: The most commonly employed initiator is benzoyl peroxide, which is activated rapidly between 50°C and 100°C to release two free radicals per benzoyl peroxide molecule. Reference: PHILLIPS’ SCIENCE OF DENTAL MATERIALS, 12th ed page no 101" "This is a real-world medical entrance exam question, please give the true answer based on the question and selection. Topic name: None, Subject name: Radiology","Radiolucencies of the jaw may be seen in: Select and explain. A. Multiple myeloma, B. Giant cell tumours, C. Hyperparathyroidism, D. All of the above",D: All of the above,"Answer: D: All of the above Explanation: None" "This is a real-world medical entrance exam question, please give the true answer based on the question and selection. Topic name: None, Subject name: Dental","Test to exclude a disease Select and explain. A. Positive predictivity, B. Negative predictivity, C. Specificity, D. Sensitivity",C: Specificity,"Answer: C: Specificity Explanation: None" "This is a real-world medical entrance exam question, please give the true answer based on the question and selection. Topic name: None, Subject name: ENT","Pharyngeal Pseudosulcus is seen secondary to- Select and explain. A. Vocal Abuse, B. Laryngopharyngeal Reflux, C. Tuberculosis, D. Corticosteroid usage",B: Laryngopharyngeal Reflux,"Answer: B: Laryngopharyngeal Reflux Explanation: Vocal Sulcus/Laryngeal Sulcus It is a groove along the mucosa and can be classified into three types: Laryngeal sulcus" "This is a real-world medical entrance exam question, please give the true answer based on the question and selection. Topic name: None, Subject name: Dental","During cobalt chromium crown reconstruction, temperature of soldering torch suddenly increases. In such case, which of the following property will change? Select and explain. A. Yield strength decreases and percent elongation increases, B. Yield strength and percent elongation both increases, C. Yield strength increases and percent elongation decreases, D. Yield strength and percent elongation both decreases",D: Yield strength and percent elongation both decreases,"Answer: D: Yield strength and percent elongation both decreases Explanation: Heat treatments of cobalt-based alloys reduce both the yield strength and elongation. If for any reason, some soldering or welding must be performed on these removable dental prostheses, the lowest possible temperature should be used with the shortest possible heating time. Ref: Craig’s 14th edition page 194." "This is a real-world medical entrance exam question, please give the true answer based on the question and selection. Topic name: None, Subject name: Dental","4th generation apex locators are based on? Select and explain. A. Frequency based, B. Impedence based, C. Multiple frequency ratio based, D. Resistance based",C: Multiple frequency ratio based,"Answer: C: Multiple frequency ratio based Explanation: None" "This is a real-world medical entrance exam question, please give the true answer based on the question and selection. Topic name: None, Subject name: Dental","Law of pulse oxymetry is based on Select and explain. A. Doppler's law, B. Pascal's law, C. Poille's law, D. Beer Lambert's law",D: Beer Lambert's law,"Answer: D: Beer Lambert's law Explanation: None" "This is a real-world medical entrance exam question, please give the true answer based on the question and selection. Topic name: None, Subject name: Pathology","The most common intraoral location for a pigmented nevi is the Select and explain. A. Hard palate, B. Soft palate, C. Buccal mucosa, D. Floor of mouth",A: Hard palate,"Answer: A: Hard palate Explanation: None" "This is a real-world medical entrance exam question, please give the true answer based on the question and selection. Topic name: None, Subject name: Dental","AIDS, secondary infection will be all except Select and explain. A. Rubella, B. HSV, C. Candida, D. kaposis sarcoma",A: Rubella,"Answer: A: Rubella Explanation: None" "This is a real-world medical entrance exam question, please give the true answer based on the question and selection. Topic name: None, Subject name: Surgery","NN hick of the following tractions is not used in lower limb? Select and explain. A. Gallows, B. Bryant, C. Dunlop, D. Perkin",C: Dunlop,"Answer: C: Dunlop Explanation: Ans: C. Dunlop(Ref Maheshwari 5/e p27)Dunlop traction:Used for supracondylar fracture of humerus, not for lower limb. Moleskin covered with elastic bandage:Uses of TractionsNameUseBryant's Traction('Fracture shaft of femur in children <2 yearsGallow's TractiondegFracture shaft of femur in children <2 yearsRussel's TractiondegFracture shaft of femur in older childrenPerkin's TractiondegFracture shaft of femur in adults90deg-90deg TractionFracture shaft of femur in childrenAgnes-Hunt TractionCorrection of Hip deformityWell-Leg TractionCorrection of adduction or abduction deformity of hipDunlop TractionSupracondylar fracture of humerusSmith's TractionSupracondylar fracture of humerusCalcaneal TractionOpen fractures of ankle or legMetacarpal TractionOpen forearm fracturesHead-Halter TractionCervical spine injuriesCrutchfield TractiondegCervical spine injuriesHalo-Pelvic TractionScoliosis" "This is a real-world medical entrance exam question, please give the true answer based on the question and selection. Topic name: None, Subject name: Pediatrics","Earliest valvular lesion in a case of acute rheumatic fever is – Select and explain. A. Mitral regurgitation (MR), B. Aortic Regurgitation(AR), C. Mitral stenosis (MS), D. Aortic Stenosis (AS)",A: Mitral regurgitation (MR),"Answer: A: Mitral regurgitation (MR) Explanation: In rheumatic fever valvular abnormalities usually are associated with chronic disease but mitral regurgitation is seen in acute rheumatic fever. Mitral regurgitation is the commonest manifestation of acute as well as previous rheumatic carditis." "This is a real-world medical entrance exam question, please give the true answer based on the question and selection. Topic name: None, Subject name: Medicine","Serum glutamic oxaloacetic transaminase is increased in: Select and explain. A. Renal failure, B. Prostatic carcinoma, C. Myocardial infarction, D. Pulmonary edema",C: Myocardial infarction,"Answer: C: Myocardial infarction Explanation: None" "This is a real-world medical entrance exam question, please give the true answer based on the question and selection. Topic name: None, Subject name: Social & Preventive Medicine","Which is the best index for burden of disease? Select and explain. A. Case fatality rate, B. Morbidity data, C. Dependency ratio, D. Disability adjusted life years",D: Disability adjusted life years,"Answer: D: Disability adjusted life years Explanation: Disability adjusted life years : Is a measure of the burden of disease in a defined population and the effectiveness of interventions; It expresses years lost to premature death and years lived with disability adjusted for its’ severity" "This is a real-world medical entrance exam question, please give the true answer based on the question and selection. Topic name: None, Subject name: Surgery","Peau d'Orange of breast is due to:September 2005, 2010, March 2007 Select and explain. A. Obstruction of Vein, B. Obstruction of lymphatic ducts., C. Obstruction of glandular ducts, D. Obstruction of aeries",B: Obstruction of lymphatic ducts.,"Answer: B: Obstruction of lymphatic ducts. Explanation: Ans. B: Obstruction of lymphatic ductsInflammatory breast cancer (IBC) is a very aggressive type of breast cancer in which the cancer cells block the lymph vessels in the skin of the breast.This type of breast cancer is called ""inflammatory"" because the breast often looks swollen and red, or ""inflamed"", sometimes overnight, and can be misdiagnosed as mastitis.Invasion of the local lymphatic ducts impairs drainage and causes edematous swelling of the breast. Because the skin of the breast is tethered by the suspensory ligament of Cooper, the accumulation of fluid may cause the skin of the breast to assume a dimpled appearance similar to an orange peel (peau d'orange).IBC is sometimes misdiagnosed as an insect bite or breast infection. In the case of IBC, a lump is usually not present as in other forms of breast cancer." "This is a real-world medical entrance exam question, please give the true answer based on the question and selection. Topic name: None, Subject name: Dental","Most accurate method of determination of working length in a tooth with open apex: Select and explain. A. Moisture on paperpoint, B. Electronic apex locator, C. Other digital method, D. Both 1 and 2",A: Moisture on paperpoint,"Answer: A: Moisture on paperpoint Explanation: Advantages of apex locators Provide objective information with high degree of accuracy. Accurate in reading (90-98% accuracy). Some apex locators are also available in combination with pulp tester, so can be used to test pulp vitality. Disadvantages of apex locators Can provide inaccurate readings in following cases: Presence of pulp tissue in canal Too wet or too dry canal Use of narrow file Blockage of canal Incomplete circuit Low battery Chances of overestimation. May pose problem in teeth with immature apex. Incorrect readings in teeth with periapical radiolucencies, and necrotic pulp associated with root resorption, etc., because of lack of viable periodontal ligament." "This is a real-world medical entrance exam question, please give the true answer based on the question and selection. Topic name: None, Subject name: Surgery","Infection of masticatory space is usually associated with Select and explain. A. Tonsillar abscess, B. Mandibular molar, C. Lateral pharyngeal space, D. Parotid space infection",B: Mandibular molar,"Answer: B: Mandibular molar Explanation: None" "This is a real-world medical entrance exam question, please give the true answer based on the question and selection. Topic name: None, Subject name: Dental","The patients with class II division 1 malocclusion have Select and explain. A. Hypertonic Lower Lip, B. Hypotonic Lower lip, C. Hypertonic upper Lip, D. Hypotonic upper lip",D: Hypotonic upper lip,"Answer: D: Hypotonic upper lip Explanation: None" "This is a real-world medical entrance exam question, please give the true answer based on the question and selection. Topic name: None, Subject name: Surgery","Following road traffic accident, a patient with vague abdominal pain was immediately taken to the operation theatre for emergency laparotomy. On examination, a large, contained, stable, non-pulsatile retroperitoneal hematoma was found on the right side. One-shot IVU shows a barely discernible nephrogram on the right side and prompt uptake and excretion on the left side. What should be the next step to be done? Select and explain. A. Nephrectomy, B. Isolate the proximal renal vessels, open the Gerota's fascia and explore the kidney, C. Perform on table retrograde pyelography, D. Perform on table angiography","B: Isolate the proximal renal vessels, open the Gerota's fascia and explore the kidney","Answer: B: Isolate the proximal renal vessels, open the Gerota's fascia and explore the kidney Explanation: Ans. b. Isolate the proximal renal vessels, open the Gerota's fascia and explore the kidney-'With I7/e p281-286; Campbell 10th/1169-1178; Bailey 26th/1286-1288, 25th/1290-1291; JT Sturm, .IF Perry. Jr and A S Cass. Renal aery and vein injury following blunt trauma.Annals of surgery.Vol. 182 (6: 696-698.)" "This is a real-world medical entrance exam question, please give the true answer based on the question and selection. Topic name: None, Subject name: Surgery","In mandibular angle fracture, which is most appropriate treatment? Select and explain. A. 1 mini plate on oblique ridge, B. 2 mini plates on lateral surface of mandible, C. Compression plates, D. Reconstruction plates",A: 1 mini plate on oblique ridge,"Answer: A: 1 mini plate on oblique ridge Explanation: None" "This is a real-world medical entrance exam question, please give the true answer based on the question and selection. Topic name: None, Subject name: Dental","Which is not true about Juvenile periodontitis Select and explain. A. Rate of bone loss is faster than normal periodontitis, B. There is predilection for female patients, C. The inflammatory process starts about simultaneously as the bone loss, D. Frequently bilateral symmetrical patterns of bone loss Occur",C: The inflammatory process starts about simultaneously as the bone loss,"Answer: C: The inflammatory process starts about simultaneously as the bone loss Explanation: None" "This is a real-world medical entrance exam question, please give the true answer based on the question and selection. Topic name: None, Subject name: Dental","In normal dentition, in centric occlusion, opposing contact may be expected at? Select and explain. A. Buccal slopes of lingual cusps of maxillary posterior teeth., B. Buccal slopes of buccal cusps of maxillary posterior teeth., C. Lingual slopes of lingual cusps of mandibular posterior teeth., D. All of the above.",A: Buccal slopes of lingual cusps of maxillary posterior teeth.,"Answer: A: Buccal slopes of lingual cusps of maxillary posterior teeth. Explanation: None" "This is a real-world medical entrance exam question, please give the true answer based on the question and selection. Topic name: None, Subject name: Medicine","An elderly, hypeensive patient presented with sudden onset headache, vomiting, neck rigidity without focal neurological deficit. Diagnosis: Select and explain. A. Subarachnoid hemorrhage, B. Ischemic stroke, C. Subdural hemorrhage, D. Meningitis",A: Subarachnoid hemorrhage,"Answer: A: Subarachnoid hemorrhage Explanation: Ans. a. Subarachnoid hemorrhage (Ref Harrison l9/e p1784, I8/e p2262-2263; Sobiston I9/e p1880-1882; Schwaz 9/e p1534-1536; Bailey 25/e p304)An elderly, hypeensive patient presented with sudden onset headache, vomiting, neck rigidity without focal neurological deficit, diagnosis in this case is subarachnoid hemorrhage." "This is a real-world medical entrance exam question, please give the true answer based on the question and selection. Topic name: None, Subject name: Dental","The setting time of irreversible hydrocolloids can be decreased by: Select and explain. A. Raising the temperature of water used for mixing, B. Using excess water for mixing, C. Lowering the temperature of water used for mixing, D. None of the above",A: Raising the temperature of water used for mixing,"Answer: A: Raising the temperature of water used for mixing Explanation: None" "This is a real-world medical entrance exam question, please give the true answer based on the question and selection. Topic name: None, Subject name: Pharmacology","Drug binds to activate the receptor in same fashion, which directly or indirectly brings effect, is: Select and explain. A. Inverse agonist, B. Antagonist, C. Agonist, D. Partial agonist",C: Agonist,"Answer: C: Agonist Explanation: None" "This is a real-world medical entrance exam question, please give the true answer based on the question and selection. Topic name: None, Subject name: Pediatrics","Infective endocarditis is not seen in – Select and explain. A. ASD, B. TOF, C. VSD, D. MR",A: ASD,"Answer: A: ASD Explanation: Endocarditis tends to occur in : High pressure areas (left side of heart) Downstream from sites where blood flows at a high velocity through a narrow orifice from a high to low-pressure chamber (distal to constriction in Coarctation of aorta) Endocarditis is unusual in sites with a small pressure gradient as ASD Infective endocarditis is very rare in patients of ASD Endocarditis occurs more frequently in patients with valvular incompetence than in those with pure stenosis. Risk of endocarditis in various lesions are :" "This is a real-world medical entrance exam question, please give the true answer based on the question and selection. Topic name: None, Subject name: Dental","The systolic blood pressure in normal healthy males of age group of 40 — 60 has a normal distribution with a mean of 130 in a sample of 100. People with systolic blood pressure more than 130 will be: Select and explain. A. 100, B. 75, C. 50, D. 25",C: 50,"Answer: C: 50 Explanation: None" "This is a real-world medical entrance exam question, please give the true answer based on the question and selection. Topic name: None, Subject name: Pharmacology","Which of the following drugs does not affect DNA synthesis? Select and explain. A. Rifampicin, B. Linezolid, C. Nitrofurantoin, D. Metronidazole",B: Linezolid,"Answer: B: Linezolid Explanation: Ans: B. Linezolid(Ref Goodman Gilman I3/e p796, 12/e p1537; Katzung 12/e p817; KDT 7/e p758, 817, 6/e p669)Linezolid inhibits protein synthesis by binding to the P site of the 50S ribosomal subunit and preventing formation of the larger ribosomal-fMet-tRNA complex that initiates protein synthesis.Nitrofurantoin - Works by damaging bacterial DNA.Metronidazole - Forms toxic free radical metabolites in the bacterial cell that damage DNA.Rifampicin - Inhibit DNA-dependent RNA polymerase." "This is a real-world medical entrance exam question, please give the true answer based on the question and selection. Topic name: None, Subject name: Dental","Periodontosis manifests itself mainly in: Select and explain. A. Molars, B. Upper incisors, C. Molars and incisors, D. Premolars",C: Molars and incisors,"Answer: C: Molars and incisors Explanation: None" "This is a real-world medical entrance exam question, please give the true answer based on the question and selection. Topic name: None, Subject name: Dental","Nerve fibres in pulp are Select and explain. A. Sympathetic efferent post ganglionic, B. Parasympathetic post ganglionic, C. Motor fibres, D. Both autonomic parasympathetic and sympathetic fibres",A: Sympathetic efferent post ganglionic,"Answer: A: Sympathetic efferent post ganglionic Explanation: None" "This is a real-world medical entrance exam question, please give the true answer based on the question and selection. Topic name: AIIMS 2019, Subject name: Pharmacology","A patient was schedule for surgery. Before giving anaesthesia, he was administered glycopyrrolate. What is rationale of giving glycopyrrolate before anaesthesia? Select and explain. A. To allay anxiety, B. To decrease secretions, C. As inducing agent, D. For muscle relaxation",B: To decrease secretions,"Answer: B: To decrease secretions Explanation: Preanaesthetic medication refers to the use of drugs before anaesthesia to make it safe and less unpleasant. When inhalation anaesthetic are given they inhibit the ciliary function therefore secretions in the respiratory pathway are not removed so we have to stop the secretions by administration of anti cholinergic drugs like atropine or glycopyrrolate. Glycopyrrolate is twice as potent and longer acting quaternary antimuscarinic which does not produce central effects. Antisecretory action is more marked than atropine and tachycardia is less marked. It acts rapidly when given i.v. and is the preferred antimuscarinic in anaesthetic practice." "This is a real-world medical entrance exam question, please give the true answer based on the question and selection. Topic name: None, Subject name: Pharmacology","All of the following are true about Ifosfamide except: Select and explain. A. It is nitrogen mustard, B. Metabolised by CYP3A4 to form active metabolite, C. Chloracetaldehyde is active form, D. Less neurotoxic than cyclophosphamide",D: Less neurotoxic than cyclophosphamide,"Answer: D: Less neurotoxic than cyclophosphamide Explanation: Ans. d. Less neurotoxic than cyclophosphamideAt equivalent doses, the rate of chloroacetaldehyde generation with ifosfamide is 40 times greater than with cyclophosphamide. At equivalent doses, the rate of chloroacetaldehyde generation with ifosfamide is 40 times greater than with cyclophosphamide therefore it produces more platelet suppression, neurotoxicity and urothelial damage.IfosfamideIfosfamide is a synthetic analogue of cyclophosphamideIt is a nitrogen mustard alkylating agentMetabolism:Ifosfamide is a prodrug that requires metabolic activation by microsomal liver enzymes to produce biologically active compounds.Activation is mediated by cytochrome p450 --> CYP3A4 and deactivated by CYP3A-4 and CYP2B6The metabolic activation of Ifosfamide is an autoinducible process and besides producing active cytotoxic metabolites it also results in generation of some toxic metabolic byproducts that are responsible Side-Effects/Toxicity:Ifosfamide is a cyclophosphamide analogue, it has all the potential adverse effects Ifosfamide metabolite, rather than the parent drug is responsible for toxicity.Well known toxic metabolites of Ifosfamide: Acrolein and chloroacetaldehyde.Acrolein is responsible for Ifosfamide induced hemorrhagic cystitis.Chloroacetaldehyde is responsible for renal tubular damage and neurotoxicity.At equivalent doses, the rate of chloroacetaldehyde generation with ifosfamide is 40 timescyclophosphamide therefore it produces more platelet suppression, neurotoxicity and urothelial" "This is a real-world medical entrance exam question, please give the true answer based on the question and selection. Topic name: None, Subject name: Medicine","The organism which is involved in anaerobic infections of head and neck region is Select and explain. A. Staphylococcus aureus, B. S. albus, C. Streptococci, D. Bacteroids",D: Bacteroids,"Answer: D: Bacteroids Explanation: None" "This is a real-world medical entrance exam question, please give the true answer based on the question and selection. Topic name: None, Subject name: Gynaecology & Obstetrics","Earliest diagnosis of pregnancy can be established safely by: Select and explain. A. USG for fetal cardiac activity, B. Fetal cardiac Doppler study, C. hCG levels, D. MRI pelvis",A: USG for fetal cardiac activity,"Answer: A: USG for fetal cardiac activity Explanation: Ans: A. USG for fetal cardiac activity (Ref Williams 24/e p196; Ultrasound Obstet Gynecol 2011; 37:625-628; Dutta 8/e p77-78, 7/e p68)Earliest diagnosis of pregnancy:Most accurate & safest method diagnose ble pregnancy at 6 weeks = USG for fetal cardiac activity.Transvaginal sonography:By 5 weeks:Reliably visualizes intrauterine gestational sac.Embryo visible transvaginally once mean sac diameter is 20 mm.Otherwise is anembryonic gestation.By 6 weeks:Embryo with cardiac activity.Cardiac motion visible when embryo length is 5 mm.If embryo <7 mm is unidentified with cardiac activity - Subsequent examination recommended in 1 week (American Institute of Ultrasound in Medicine, 2013a).Doppler:Most sensitive but unsafe in early pregnancy.Doppler examination of fetal vessels in early pregnancy should not be performed without a clinical indication." "This is a real-world medical entrance exam question, please give the true answer based on the question and selection. Topic name: None, Subject name: Anatomy","Infarction of anterior inferior cerebellar artery may cause damage to Select and explain. A. Pyramidal tract, B. Lateral spinothalamic tract, C. Vestibular nuclei, D. Spinal nucleus of trigeminal nerve",C: Vestibular nuclei,"Answer: C: Vestibular nuclei Explanation: None" "This is a real-world medical entrance exam question, please give the true answer based on the question and selection. Topic name: None, Subject name: Dental","Oral mucosa is? Select and explain. A. keratinized squamous, B. stratified squamous, C. Squamous, D. Columnar",B: stratified squamous,"Answer: B: stratified squamous Explanation: None" "This is a real-world medical entrance exam question, please give the true answer based on the question and selection. Topic name: None, Subject name: Dental","All of the following statements about cast cobalt alloys are true EXCEPT: Select and explain. A. They have a modulus of elasticity twice that of gold alloys., B. They are more rigid than gold alloys., C. They have higher ductility than gold alloys., D. They have a lower proportional limit than gold alloys.",C: They have higher ductility than gold alloys.,"Answer: C: They have higher ductility than gold alloys. Explanation: Gold is the most ductile and malleable metal. This property enables manufacturers to beat it into thin foils. Silver is second. Among other metals platinum ranks third in ductility and copper ranks third in malleability. Mannapalli 3 ed page no 16" "This is a real-world medical entrance exam question, please give the true answer based on the question and selection. Topic name: None, Subject name: Surgery","Initial clicking of TMJ while opening is due to Select and explain. A. Protruded condyle in respect to articular disc, B. Retruded condyle in respect to articular disc, C. Perforated disc, D. Lateral displacement of the condyle",B: Retruded condyle in respect to articular disc,"Answer: B: Retruded condyle in respect to articular disc Explanation: None" "This is a real-world medical entrance exam question, please give the true answer based on the question and selection. Topic name: None, Subject name: Medicine","A 5-year old boy passed 18 loose stools in last 24 hours and vomited twice in last 4 hours. He is irritable but drinking fluids. The optional therapy for this child is: Select and explain. A. Intravenous fluids, B. Oral rehydration therapy, C. Intravenous fluid initially for 4 hours followed by oral fluids, D. Plain water ad libitum",B: Oral rehydration therapy,"Answer: B: Oral rehydration therapy Explanation: None" "This is a real-world medical entrance exam question, please give the true answer based on the question and selection. Topic name: None, Subject name: Dental","Nickel is added to wrought gold alloys as Select and explain. A. Strengthener, B. Hardener, C. Whitens the alloy, D. Scavenger",A: Strengthener,"Answer: A: Strengthener Explanation: None" "This is a real-world medical entrance exam question, please give the true answer based on the question and selection. Topic name: None, Subject name: Dental","All of these are dental auxiliary dental materials except: Select and explain. A. Dental impression wax, B. Ceramic, C. Casting investment, D. Impression compound",B: Ceramic,"Answer: B: Ceramic Explanation: Auxiliary dental material—Substance that is used in the construction of a dental prosthesis but that does not become a part of the structure. Auxiliary dental materials are substances used in the process of fabricating dental prostheses and appliances but that do not become part of these devices. These include acid-etching solutions, impression materials, casting investments, gypsum cast and model materials, dental waxes, acrylic resins for impression and bleaching trays, acrylic resins for mouth guards and occlusion aids, and finishing and polishing abrasives. Reference: PHILLIPS’ SCIENCE OF DENTAL MATERIALS, 12th ed page no 3, 5" "This is a real-world medical entrance exam question, please give the true answer based on the question and selection. Topic name: None, Subject name: Dental","Treatment resins are given for which type of edentulous ridges? Select and explain. A. Abused ridges, B. Flabby ridges, C. Ridges with large tori on them, D. Ridges with bony resorption",A: Abused ridges,"Answer: A: Abused ridges Explanation: None" "This is a real-world medical entrance exam question, please give the true answer based on the question and selection. Topic name: None, Subject name: Dental","Resilient silicone denture lining materials are superior to plasticized acrylic resin as: Select and explain. A. Bond strength with a denture is increased, B. They remain resilient for a short time, C. They do not support fungal growth, D. Tissue adaptation is better",C: They do not support fungal growth,"Answer: C: They do not support fungal growth Explanation: None" "This is a real-world medical entrance exam question, please give the true answer based on the question and selection. Topic name: None, Subject name: Skin","A patient presents with focal alopecia areata. All of the following are associations of alopecia areata except: Select and explain. A. Nail pitting, B. Atopy, C. Geographic tongue, D. Exclamatory mark",C: Geographic tongue,"Answer: C: Geographic tongue Explanation: Answer- C. Geographic tongueGeographic tongue (also known as benign migratory glossitis, erythema migrans lingualis, glossitis areata migrans, and transitory benign plaques ofthe tongue). An inflammatory condition of the mucous membrane of the tongue, usually on the dorsal surface." "This is a real-world medical entrance exam question, please give the true answer based on the question and selection. Topic name: None, Subject name: Pathology","Delayed eruption of at least part of the dentition is a recognized feature of all of the following except: Select and explain. A. Rickets, B. Congenital hyperthyroidism, C. Cleidocranial dysplasia, D. Cherubism",B: Congenital hyperthyroidism,"Answer: B: Congenital hyperthyroidism Explanation: None" "This is a real-world medical entrance exam question, please give the true answer based on the question and selection. Topic name: None, Subject name: Social & Preventive Medicine","After the appearance of rash, prophylactic isolation of measles case is necessary for a minimum of:September 2007 Select and explain. A. 2 days, B. 5 days, C. 7 days, D. 9 days",B: 5 days,"Answer: B: 5 days Explanation: Ans. B: 5 days" "This is a real-world medical entrance exam question, please give the true answer based on the question and selection. Topic name: None, Subject name: Dental","The usefulness of a screening test depends upon it Select and explain. A. Sensitivity, B. Specificity, C. Reliability, D. Predictive value",A: Sensitivity,"Answer: A: Sensitivity Explanation: None" "This is a real-world medical entrance exam question, please give the true answer based on the question and selection. Topic name: None, Subject name: Dental","Brushing technique for gingival recession Select and explain. A. Bass, B. Stillman, C. Charter, D. Modified stillman",D: Modified stillman,"Answer: D: Modified stillman Explanation: None" "This is a real-world medical entrance exam question, please give the true answer based on the question and selection. Topic name: None, Subject name: Pharmacology","The intramuscular administration of 0.6 mg of atropine sulphate to a 50 kg adult may produce all of the following effects except: Select and explain. A. Bradycardia, B. Decreased salivation, C. Decreased sweating, D. Mydriasis",A: Bradycardia,"Answer: A: Bradycardia Explanation: None" "This is a real-world medical entrance exam question, please give the true answer based on the question and selection. Topic name: None, Subject name: Surgery","In a 19 years old patient, with a swelling over the left angle of the mandible, temperature of 38°C and negative history of trauma, one should suspect: Select and explain. A. Spontaneous fracture of the mandible, B. Pericoronal infection, C. Mumps, D. Sjogren's syndrome",B: Pericoronal infection,"Answer: B: Pericoronal infection Explanation: None" "This is a real-world medical entrance exam question, please give the true answer based on the question and selection. Topic name: None, Subject name: Dental","2nd optimal landmark for termination of cleaning, shaping and obturation is Select and explain. A. Apical constriction, B. Cementodentinal junction, C. Major diameter, D. None of the above",A: Apical constriction,"Answer: A: Apical constriction Explanation: None" "This is a real-world medical entrance exam question, please give the true answer based on the question and selection. Topic name: None, Subject name: Dental","Borders of major connectors adjacent to gingival crevices, should be no closer to the crevice than? Select and explain. A. 1-2 mm, B. 2-3 mm, C. 3-6 mm, D. 7-10 mm",C: 3-6 mm,"Answer: C: 3-6 mm Explanation: In maxillary arch, border of major connector must be no closer than 6 mm to gingival crevices of teeth. In mandibular arch, border of major connector must be no closer than 3 mm to gingival crevices of teeth." "This is a real-world medical entrance exam question, please give the true answer based on the question and selection. Topic name: None, Subject name: Biochemistry","Golgi bodies function as: Select and explain. A. Protein lipid synthesis, B. Protein synthesis, C. Sorting of glycoproteins, D. None of the above",C: Sorting of glycoproteins,"Answer: C: Sorting of glycoproteins Explanation: None" "This is a real-world medical entrance exam question, please give the true answer based on the question and selection. Topic name: None, Subject name: Pathology","In Prothrombin time (PT) estimation, on addition of calcium and thromboplastin to platelet poor plasma. which of the following pathway is activated? Select and explain. A. Extrinsic, B. Intrinsic, C. Fibrinolysis, D. Common",A: Extrinsic,"Answer: A: Extrinsic Explanation: Ans. a. ExtrinsicIn Prothrombin time (PT) estimation, addition of calcium and thromboplastin to platelet poor plasma activates extrinsic pathway.The prothrombin time (PT) assay assesses the function of the proteins in the extrinsic pathway (factors VII, X,and .fibrinogen). In brief tissue factor, phospholipids, and calcium are added to plasma and the time for a fibrin clot to form is recorded.Prothrombin time (PT): This test assesses the extrinsic and common coagulation pathways. The clotting of plasma after addition of an exogenous source of tissue thromboplastin (e.g., brain extract) and Ca"" ions is measured in seconds. A prolonged PT can result from deficiency or dysfunction of factor V, factor VII, factor X, prothrombin. or fibrinogen" "This is a real-world medical entrance exam question, please give the true answer based on the question and selection. Topic name: None, Subject name: Physiology","true statement about golgi tendon organ is Select and explain. A. High threshold, B. Dynamic response only, C. Detects length change, D. 3-25 muscle fibres",D: 3-25 muscle fibres,"Answer: D: 3-25 muscle fibres Explanation: None" "This is a real-world medical entrance exam question, please give the true answer based on the question and selection. Topic name: None, Subject name: Surgery","Treatment for KOT is: Select and explain. A. Mandible resection with continuity defect and immediate reconstruction, B. Mandible resection without continuity defect and immediate reconstruction, C. Enucleation and peripheral osteotomy, chemical cauterization, D. Cauterization and delayed reconstruction","C: Enucleation and peripheral osteotomy, chemical cauterization","Answer: C: Enucleation and peripheral osteotomy, chemical cauterization Explanation: None" "This is a real-world medical entrance exam question, please give the true answer based on the question and selection. Topic name: None, Subject name: Dental","Lingual bar is used in conditions where: Select and explain. A. Lingual sulcus is deep, B. Lingual sulcus is shallow, C. When splinting of adjacent teeth is require, D. When abutment teeth have poor bony support",A: Lingual sulcus is deep,"Answer: A: Lingual sulcus is deep Explanation: None" "This is a real-world medical entrance exam question, please give the true answer based on the question and selection. Topic name: None, Subject name: Gynaecology & Obstetrics","All of the following are used for screening cancers in females except Select and explain. A. CA-125: Ovarian cancer, B. Office endometrial aspirate: Endometrial carcinoma, C. Pap smear: Cervical cancer, D. Mammography: Breast cancer",B: Office endometrial aspirate: Endometrial carcinoma,"Answer: B: Office endometrial aspirate: Endometrial carcinoma Explanation: Ans. b. Office endometrial aspirate: Endometrial carcinomaOffice endometrial aspirate is not used for screening of Endometrial carcinoma.Transvaginal ultrasound and endometrial sampling have been advocated as screening tests for endometrial cancer but benefit from routine screening have not been shown.`Screening for endometrial cancer should currently not be undeaken because of a lack of an appropriate, cost-effective and acceptable test that reduces moality. Routine PAP testing is an inadequate test that reduces moality. Routine PAP testing is inadequate and endometrial cytology assessment is too insensitive and non-specific to be useful in screening for endometrial cancer even in a high-risk population.' Screening TestDisease ScreenedPapanicolaou (Pap's) smearCervical cancerMammographyBreast cancerCA-125Ovarian cancerScreening TestThe screening test is used to search for an unrecognized disease or defect, in apparently healthy individuals, using rapidly applied tests, examination or other proceduresScreening TestDisease ScreenedPapanicolaou (Pap's) smearCervical cancerQMammographyBreast cancerQBimanual oral examinationOral cancerQELISAHIVQUrine for sugar, Random blood sugarDiabetes mellitusQAFPDevelopmental anomalies in fetusQDRE + PSAProstate cancerQFecal occult blood testColorectal cancerQCA-125Ovarian cancerQ" "This is a real-world medical entrance exam question, please give the true answer based on the question and selection. Topic name: AIIMS 2018, Subject name: Physiology","Maximum area in homunculus signifies? Select and explain. A. Low control and skilled movements, B. More muscle fibers are present, C. Increased precisiveness of movements, D. None of the above",C: Increased precisiveness of movements,"Answer: C: Increased precisiveness of movements Explanation: 1. Solutions: - Motor homunculus - Area controlling muscles for speech - Muscles of thumb | Increased precision of movements - fine motor activities Ref: CH 56, page no. 707. 2. Sensory homunculus - Lips (greatest area) - Face & thumb | Size of area of represented directly propoional to the number of specialized sensory receptors in the respective peripheral area of the body. Ref: Guyton and Hall Ed 13,CH 48, page no. 612." "This is a real-world medical entrance exam question, please give the true answer based on the question and selection. Topic name: None, Subject name: Dental","All of the following hemostatic agents are used as perforation repair, except? Select and explain. A. Collagen, B. CaOH2, C. Ferric sulphate, D. Fridge dried bone",C: Ferric sulphate,"Answer: C: Ferric sulphate Explanation: None" "This is a real-world medical entrance exam question, please give the true answer based on the question and selection. Topic name: None, Subject name: Dental","Unattached gingiva: Select and explain. A. Is interdental gingiva, B. Is below mucogingival fold, C. c) Cannot be separated by probe, D. d) Is marginal gingiva",D: d) Is marginal gingiva,"Answer: D: d) Is marginal gingiva Explanation: None" "This is a real-world medical entrance exam question, please give the true answer based on the question and selection. Topic name: None, Subject name: Surgery","In case of nodal metastasis, which is not seen on CT scan Select and explain. A. Spiculated node, B. Size more than 6 mm, C. Necrotic node, D. Rounded node",A: Spiculated node,"Answer: A: Spiculated node Explanation: None" "This is a real-world medical entrance exam question, please give the true answer based on the question and selection. Topic name: None, Subject name: Pharmacology","Fixed drug eruptions are frequently seen with? Select and explain. A. Penicillin, B. Sulfonamide, C. Cetirizine, D. Roxithromycin",B: Sulfonamide,"Answer: B: Sulfonamide Explanation: Ans. B. SulfonamideDrugs causing fixed drug eruptionParacetamol (Phenacetin)SulfonamidesNSAIDsAspirinBarbituratesDapsoneTetracyclinesPhenylbutazone" "This is a real-world medical entrance exam question, please give the true answer based on the question and selection. Topic name: None, Subject name: Forensic Medicine","An autopsy was performed on a case of accidental death. It showed two linear fractures on petrous pa of temporal bone. Which of the following rules gives the sequence of fractures? Select and explain. A. McNaughton's rule, B. Puppe's rule, C. Young's rule, D. Dunlop's rule",B: Puppe's rule,"Answer: B: Puppe's rule Explanation: Ans: B. Puppe's rule (Ref. Reddy 34/e p213. 33/e p225, Forensic Pathology 3/e p241, 110)Puppe's rule:Used to determine that which fracture line has occurred before the second one." "This is a real-world medical entrance exam question, please give the true answer based on the question and selection. Topic name: None, Subject name: Anatomy","Pharyngeal arches are rod like thickning of: Select and explain. A. Endoderm., B. Ectoderm., C. Mesoderm., D. All of the above.",C: Mesoderm.,"Answer: C: Mesoderm. Explanation: None" "This is a real-world medical entrance exam question, please give the true answer based on the question and selection. Topic name: None, Subject name: Physiology","The average no of muscle fibres attached to one golgi tendon oragn are Select and explain. A. 3-Jan, B. 10-May, C. 15-Oct, D. 15-75",C: 15-Oct,"Answer: C: 15-Oct Explanation: None" "This is a real-world medical entrance exam question, please give the true answer based on the question and selection. Topic name: None, Subject name: Dental","Gene for Dentin mineralization Select and explain. A. MAP1B, B. PHEX, C. DEN, D. PHIX",B: PHEX,"Answer: B: PHEX Explanation: None" "This is a real-world medical entrance exam question, please give the true answer based on the question and selection. Topic name: None, Subject name: Biochemistry","Which of these is an example of anaplerotic reaction? Select and explain. A. Pyruvate to oxaloacetate, B. Pyruvate to Acetyl CoA, C. Pyruvate to lactic acid, D. Pyruvate to acetaldehyde",A: Pyruvate to oxaloacetate,"Answer: A: Pyruvate to oxaloacetate Explanation: Answer- A. Pyruvate to oxaloacetateCarboxylation of pyruvate to oxuloacetate is an example of anaplerotic reactions.""Anaplerotic reactions are chemical reactions that form intermediates of a metabolic pathway. Examples of such are foundin the citric acid cycle (TCA cycle).Examples of Anaplerotic ReactionsCarboxylation of pyruvate to oxaloacetateTransamination of aspaate to oxaloacetate by aspaate amino transferaseaHydration of glutamate to alpha-ketoglutarate by glutamate-dehydrogenaseBeta-oxidation of fatty acids to succinyl-CoAIn purine synthesis & purine nucleotide cycle: Adenylosuccinate to fumarate catalyzed by adenylosuccinate lyase." "This is a real-world medical entrance exam question, please give the true answer based on the question and selection. Topic name: None, Subject name: Dental","The most effective caustic that may be used locally in the treatment of hypersensitive dentin in the molars is: Select and explain. A. Alcohol, B. Chlorhexidine, C. Trichloro acetic acid, D. Silver nitrate",D: Silver nitrate,"Answer: D: Silver nitrate Explanation: None" "This is a real-world medical entrance exam question, please give the true answer based on the question and selection. Topic name: None, Subject name: Dental","Orthodontically treated midline diastema is best retained by: Select and explain. A. Hawley's appliance for 6 months, B. Hawley's appliance for 12 months, C. Bonded lingual retainer, D. No need for retainer as it is self-retained",C: Bonded lingual retainer,"Answer: C: Bonded lingual retainer Explanation: Advantages of lingual-bonded retainers Lingual bonded retainers are invisible, hence, aesthetically acceptable. Patient comfort and compliance. Recommended in patients with compromised periodontal support. Effectiveness and reliability in the long term and permanent retention and help in retaining the orthodontically treated midline diastema. Orthodontics: Diagnosis and Management of Malocclusion and Dentofacial Deformities 3rd ed O P Kharbanda pdf no 1880" "This is a real-world medical entrance exam question, please give the true answer based on the question and selection. Topic name: None, Subject name: Forensic Medicine","Poisoning by which of these elements causes blind staggers alkali disease in livestock? Select and explain. A. Magnesium, B. Manganese, C. Selenium, D. Molybdenum",C: Selenium,"Answer: C: Selenium Explanation: Ans: C. SeleniumBlind staggers are no longer believed to be caused by selenium but by sulfate toxicity due to the consumption of high-sulfate alkali water and/or high sulfur-containing forages.Since Sulfate is not mentioned in Question we will go for selenium.Excess sulfate (>1% of diet) leads to polioencephalomalacia and the classical signs of blind staggers.Blind StaggersThis syndrome may occur within a brief period (a few days to a few weeks) after the animal has begun to take in very high dietary levels of selenium. The symptoms of this condition are: impaired vision, a depressed appetite, and wandering in circlesProlonged oral exposure of cattle to elevated dietary selenium (Se) is associated historically with 2 clinical syndromes:Alkali disease.""Blind staggers"".These two syndromes are not necessarily independent of one another. An animal may have only symptoms of one or the other or a combination of the symptoms of both. Both are associated with cardiac and skeletal muscle damage as well as hepatic damage." "This is a real-world medical entrance exam question, please give the true answer based on the question and selection. Topic name: None, Subject name: Dental","In a patient with allergy to costume jewelry, which of the following is contraindicated: Select and explain. A. High gold, B. Low gold, C. Palladium — Ag, D. Ni — Cr",D: Ni — Cr,"Answer: D: Ni — Cr Explanation: None" "This is a real-world medical entrance exam question, please give the true answer based on the question and selection. Topic name: AIIMS 2019, Subject name: Pathology","Which of the following is the complication of massive blood transfusion? Select and explain. A. Metabolic acidosis, B. Metabolic alkalosis, C. Respiratory alkalosis, D. Respiratory acidosis",B: Metabolic alkalosis,"Answer: B: Metabolic alkalosis Explanation: Transfused blood contains chemical known as citrate and the liver conves this citrate into bicarbonate . High amount of bicarbonate in body leads to metabolic alkalosis" "This is a real-world medical entrance exam question, please give the true answer based on the question and selection. Topic name: None, Subject name: Dental","Turku’s sugar study was related to Select and explain. A. Fluorosis, B. Dental Caries, C. Both, D. None",B: Dental Caries,"Answer: B: Dental Caries Explanation: None" "This is a real-world medical entrance exam question, please give the true answer based on the question and selection. Topic name: None, Subject name: Physiology","Maximum alveolar aerial oxygen difference is seen in: Select and explain. A. Pulmonary embolism, B. Severe asthma, C. Interstitial lung disease, D. Foreign body in upper airway",A: Pulmonary embolism,"Answer: A: Pulmonary embolism Explanation: Ans: A. Pulmonary embolism (Ref: Ganong 25/e p634, 24/e p634)In pulmonary embolism gradient increases to very high level (I0 to 83 mm Hg from normal level of 5 mm Hg).But normal Pa02 level on aerial blood gas analysis does not exclude the diagnosis of acute pulmonary embolism.It is also increased in ILD." "This is a real-world medical entrance exam question, please give the true answer based on the question and selection. Topic name: None, Subject name: Pharmacology","Isosorbide 5 — mononitrate is preferred because Select and explain. A. Less tolerance develops, B. Sublingual absorption is better, C. Longer duration of action, D. Direct coronary vasodilatory action is more",C: Longer duration of action,"Answer: C: Longer duration of action Explanation: None" "This is a real-world medical entrance exam question, please give the true answer based on the question and selection. Topic name: None, Subject name: Dental","Dicor is Select and explain. A. Castable ceramic, B. Metavite, C. Vitallium, D. Vita ceramic",A: Castable ceramic,"Answer: A: Castable ceramic Explanation: None" "This is a real-world medical entrance exam question, please give the true answer based on the question and selection. Topic name: None, Subject name: Dental","Which of the following premolars has a mesial marginal ridge more cervical than the distal marginal ridge: Select and explain. A. Maxillary 1st premolar, B. Maxillary 2nd premolar, C. Mandibular 1st premolar, D. Mandibular 2nd premolar",C: Mandibular 1st premolar,"Answer: C: Mandibular 1st premolar Explanation: None" "This is a real-world medical entrance exam question, please give the true answer based on the question and selection. Topic name: None, Subject name: Dental","Correlation between the height and weight of children is best represented by: Select and explain. A. Histogram, B. Line diagram, C. Scatter diagram, D. Bar diagram",C: Scatter diagram,"Answer: C: Scatter diagram Explanation: None" "This is a real-world medical entrance exam question, please give the true answer based on the question and selection. Topic name: None, Subject name: Pathology","Which among the following shows pseudo-epitheliomatous hyperplasia? Select and explain. A. Sq. cell carcinoma, B. Basal cell carcinoma, C. Verrucous carcinoma, D. Granular cell myoblastorna",D: Granular cell myoblastorna,"Answer: D: Granular cell myoblastorna Explanation: None" "This is a real-world medical entrance exam question, please give the true answer based on the question and selection. Topic name: None, Subject name: Forensic Medicine","A person was advised by his ohopedic surgeon to get regular dressing of his wound done. But the patient did not give much care. During follow-up, patient was repeatedly told to get the dressing done timely but patient didn't do the dressing himself, saying that he was busy. Finally the wound enlarged and the underlying bone developed osteomyelitis. Which of the following statement is true regarding above-mentioned situation? Select and explain. A. Doctor is guilty under ""Last clear chance"" doctrine, B. Doctor is not guilty under ""Contributory negligence"", C. Doctor is punishable under avoidable negligence, D. Doctor is guilty as he prescribed wrong medicines","A: Doctor is guilty under ""Last clear chance"" doctrine","Answer: A: Doctor is guilty under ""Last clear chance"" doctrine Explanation: Ans: A. Doctor is guilty under ""Last clear chance"" doctrine(Ref Reddy 34/e p38, 33/e p40)Last Clear Chance Doctrine:Possibility of doctor saving the patient (plaintiff) by dressing wound properly the 2nd time, hence curing without osteomyelitis development.Hence, doctor is paly guilty for the outcome (osteomyelitis).Criteria included:Failure of doctor to prevent damage resulting from negligent act of patient, even after getting clear time, he cannot plead contributory negligence in civil cases.Elements of proof:Defendant- Doctor.Defendant recognized the danger & acquired a duty to avoid it.Defendant failed to avoid danger.Plaintiff - Patient.Plaintiff placed themselves in a situation of risk or danger through their own negligence.Plaintiff could not avoid the danger.Plaintiff was injured as a result of the defendant's failure." "This is a real-world medical entrance exam question, please give the true answer based on the question and selection. Topic name: AIIMS 2019, Subject name: Social & Preventive Medicine","Dengue discharge protocol includes Select and explain. A. 24 hours after Recovery from shock, B. Urine volume > 200 ml, C. 24 hours after absence of fever with use of Paracetamol, D. Return of normal appetite",D: Return of normal appetite,"Answer: D: Return of normal appetite Explanation: National Vector Borne Disease Control Programme (NVBDCP) of India given the dengue discharge criteria as follows: 1. Absence of fever > 24 hours (without paracetamol) 2. Return of appetite 3. Good urine output 4. Platelet > 50,000 5. No Respiratory distress 6. > 2-3 days after recovery form shape 7. Visible clinical improvement" "This is a real-world medical entrance exam question, please give the true answer based on the question and selection. Topic name: None, Subject name: Gynaecology & Obstetrics","Which of the following statements is true about Swyer svndrome? Select and explain. A. Can be feile with surrogacy, B. Can be feile with ovum donation, C. Presents with primary feility, D. Gonadectomy is indicated for all patients",A: Can be feile with surrogacy,"Answer: A: Can be feile with surrogacy Explanation: Ans: A. Can be feile with surrogacy(Ref Shaw's 16/e p145)Swyer syndrome:Patients can be feile with surrogacy, with ovum donation.Gonadectomy indicated for all patients due to malignancy risk." "This is a real-world medical entrance exam question, please give the true answer based on the question and selection. Topic name: None, Subject name: Anatomy","Arterial supply to floor of mouth & suprahyoid muscle is Select and explain. A. Lingual artery, B. Maxillary artery, C. Inferior alveolar artery, D. Facial artery",A: Lingual artery,"Answer: A: Lingual artery Explanation: Lingual artery is a branch of external carotid artery. The root of tongue is also supplied by tonsillar and ascending pharyngeal arteries. Venous drainage is mainly by the deep lingual vein." "This is a real-world medical entrance exam question, please give the true answer based on the question and selection. Topic name: None, Subject name: Radiology","In intraoral radiograph, patient radiation is decreased by all except: Select and explain. A. High speed film, B. Intensifying screens, C. Lead apron, D. Decreasing kVp",D: Decreasing kVp,"Answer: D: Decreasing kVp Explanation: Kilovoltage: The optimal operating potential of dental X-ray units is between 60 and 70 kVp (ADA 2012). Although image diagnosis may be improved slightly with increased image contrast (low kVp) images, the patient dose is reduced with higher kVp exposures. Most intraoral machines use 60 to 70 kVp. The availability of constant-potential (fully rectified), high-frequency or direct current (DC) dental X-ray units have made possible the production of radiographs with lower kilovoltage and at reduced levels of radiation. The surface exposure required to produce a comparable radiographic density using a constant-potential unit is approximately 25% less than that of a conventional self-rectified unit operating at the same kilovoltage. At the present time, several manufacturers produce DC units. Thus, in intraoral radiograph, patient radiation is not decreased by decreasing kVp.  Reference: White and Pharoah's Oral Radiology Principles and Interpretation 7th edition page no 36" "This is a real-world medical entrance exam question, please give the true answer based on the question and selection. Topic name: None, Subject name: Medicine","Recently approved drug by FDA for treatment of Lennox-Gestaut syndrome: Select and explain. A. Lacosamide, B. Vigabatrin, C. Zonisamide, D. Rufinamide",D: Rufinamide,"Answer: D: Rufinamide Explanation: Answer- D. RufinamideRecently approved drug by FDAfor treatment of Lennox-Gestaul syndrome is rufinamide.Rufinamide:Rufinamide was approved by the US Food and Drug Administration on November 14, 2008 as adjunctive treatment of seizures associated with Lennox-Gastaut syndrome in children 4 years and older and adults." "This is a real-world medical entrance exam question, please give the true answer based on the question and selection. Topic name: None, Subject name: Medicine","Not true about fresh frozen plasma Select and explain. A. Supplies major coagulation factors, B. ABO match not required, C. Should be used in replacement of factors in DIC/Trauma, D. To be used within 30 minutes of having trauma",D: To be used within 30 minutes of having trauma,"Answer: D: To be used within 30 minutes of having trauma Explanation: None" "This is a real-world medical entrance exam question, please give the true answer based on the question and selection. Topic name: None, Subject name: Pediatrics","A newborn presents with congestive heart failure, on examination has bulging anterior fontanelle with a bruit on auscultation. Transfontanellar USG shows a hypoechoic midline mass with dilated lateral ventricles. Most likely diagnosis is – Select and explain. A. Medulloblastoma, B. Encephalocele, C. Vein of Galen malformation, D. Arachnoid cyst",C: Vein of Galen malformation,"Answer: C: Vein of Galen malformation Explanation: Midline intracranial mass with bruit on ausculation and features of hydrocephalus and congestive heart failure ­All point towards Vein of Galen Malformation. Vein of Galen Malformations(VGM) The vein of Galen is a large deep vein at the base of the brain. It is located under the cerebral hemispheres and drains the anterior and central regions of the brain into the sinuses of the posterior cerebral fossa . VGM results from an aneurysmal malformation with an arteriovenous shunting of blood.VGM usually causes high-output heart failure in the newborn resulting from the decreased resistance and high blood flow in the lesion. Associated findings include cerebral ischemic changes such as strokes or steal phenomena that result in progressive hemiparesis. The malformation may result in mass effects, causing progressive neurological impairment. Alternatively, the malformation may cause obstruction of the cerebrospinal fluid (CSF) outflow and result in hydrocephalus. Loud intracranial bruit may be heard because of the blood turbulence in VGM. The most effective treatment for the malformation has been embolization rather than surgery The vein of Galen abnormality is the most frequent arteriovenous malformation in neonates." "This is a real-world medical entrance exam question, please give the true answer based on the question and selection. Topic name: None, Subject name: Gynaecology & Obstetrics","A pregnant, diabetic female on oral hypoglycemics is shifted to insulin. All of the following are true regarding this, except: Select and explain. A. Insulin does not cross placenta, B. During pregnancy insulin requirement increases and cannot be provided with sulphonylureas, C. Tolbutamide crosses placenta, D. Tolbutamide causes PIH",D: Tolbutamide causes PIH,"Answer: D: Tolbutamide causes PIH Explanation: None" "This is a real-world medical entrance exam question, please give the true answer based on the question and selection. Topic name: None, Subject name: Anatomy","Fenestri vestibuli is: Select and explain. A. Rounded bulge produced by first turn of cochlea, B. Oval opening posterosuperior to the promontory, C. Prominence produced due to the lateral semicircular canal, D. It is grooved by tympanic plexus",B: Oval opening posterosuperior to the promontory,"Answer: B: Oval opening posterosuperior to the promontory Explanation: The Medial or Labyrinthine Wall The medial wall separates the middle ear from the internal ear. It presents the following features. (a) The promontory is a rounded bulging produced by the first turn of the cochlea. It is grooved by the tympanic plexus. (b) The fenestra vestibule is an oval opening posterosuperior to the promontory. It leads into the vestibule of the internal ear and is closed by the foot-plate of the stapes. (c) The prominence of the facial canal runs backwards just above the fenestra vestibule, to reach the lower margin of the aditus. The canal then descends behind the posterior wall to end at the stylomastoid foramen. (d) Above the prominence of facial canal lies the prominence produced due to the lateral semicircular canal." "This is a real-world medical entrance exam question, please give the true answer based on the question and selection. Topic name: None, Subject name: Surgery","Lefort 1 osteotomy essentially involves osteotomy of how many bones? Select and explain. A. 2, B. 3, C. 4, D. 5",D: 5,"Answer: D: 5 Explanation: None" "This is a real-world medical entrance exam question, please give the true answer based on the question and selection. Topic name: None, Subject name: Dental","Wilson curve seen in: Select and explain. A. Molars, B. Premolar, C. Anteriors, D. All of the above.",A: Molars,"Answer: A: Molars Explanation: None" "This is a real-world medical entrance exam question, please give the true answer based on the question and selection. Topic name: None, Subject name: Dental","In primary trauma from occlusion: Select and explain. A. The blood vessels present in periodontal ligament rupture and release contents in surrounding space, B. Adaptive capacity of tissues to withstand occlusal forces is impaired resulting in injury from normal occlusal forces, C. The alveolar bone is not damaged, D. Bifurcations and trifurcations are less susceptible to injury related to this condition",A: The blood vessels present in periodontal ligament rupture and release contents in surrounding space,"Answer: A: The blood vessels present in periodontal ligament rupture and release contents in surrounding space Explanation: None" "This is a real-world medical entrance exam question, please give the true answer based on the question and selection. Topic name: None, Subject name: Microbiology","About psedomonas. All are true except Select and explain. A. Strict aerobes, B. Can grow in disinfectants in hospital, C. Infection is mostly due to endogenous source, D. Most common organism in burn patient",C: Infection is mostly due to endogenous source,"Answer: C: Infection is mostly due to endogenous source Explanation: None" "This is a real-world medical entrance exam question, please give the true answer based on the question and selection. Topic name: None, Subject name: Medicine","Which of the heart valve is most likely to be involved by Infective endocarditis following a septic abortion: Select and explain. A. Aortic valve., B. Tricuspid valve., C. Pulmonary valve., D. Mitral valve.",B: Tricuspid valve.,"Answer: B: Tricuspid valve. Explanation: None" "This is a real-world medical entrance exam question, please give the true answer based on the question and selection. Topic name: None, Subject name: Surgery","On loading the fracture mandible posterior to the site of fracture, which of the following is present: Select and explain. A. Tension on upper border, B. Compression on upper border, C. Compression on medial surface, D. Compression on inferior surface",B: Compression on upper border,"Answer: B: Compression on upper border Explanation: None" "This is a real-world medical entrance exam question, please give the true answer based on the question and selection. Topic name: None, Subject name: Gynaecology & Obstetrics","DOC for bacterial vaginosis in pregnancy Select and explain. A. Clindamycin, B. Erythromycin, C. Rovamycin, D. Metronidazole",D: Metronidazole,"Answer: D: Metronidazole Explanation: Ans. D. MetronidazoleMedication--Treatment should include both paners.Oral Metronidazole--500 mg orally twice daily after meals for 7 days. Or 2 g stat.Advisable to defer treatment during first trimester of pregnancy.Side effects: nausea, metallic taste, antabuse - like reaction to alcohol." "This is a real-world medical entrance exam question, please give the true answer based on the question and selection. Topic name: None, Subject name: Pharmacology","Which one of the following antibiotics chelates with calcium ? Select and explain. A. Penicillin, B. Tetracycline, C. Cephalosporins, D. Erythromycin",B: Tetracycline,"Answer: B: Tetracycline Explanation: None" "This is a real-world medical entrance exam question, please give the true answer based on the question and selection. Topic name: None, Subject name: Dental","Gene for odontoblast differentiation Select and explain. A. MAP1B, B. PHEX, C. DEN, D. PHIX",A: MAP1B,"Answer: A: MAP1B Explanation: None" "This is a real-world medical entrance exam question, please give the true answer based on the question and selection. Topic name: AIIMS 2019, Subject name: Gynaecology & Obstetrics","Drugs preventing PPH, all except: Select and explain. A. Misoprostol, B. Dinoprostone, C. PGF-2 alpha, D. Oxytocin",B: Dinoprostone,"Answer: B: Dinoprostone Explanation: *PPH - Any bleed in genital tract after delivery Treatment - Misoprostol - PGE1 - 1000 ug per rectum - Carboprost -PGF2 alpha -IM only - Oxytocin is DOC. By WHO - 5/10 IU IM/IV Prophylactic - 10 -20 IU lV infusion For Rx of PPH. - IV methylergometrine 0.2 mg (peak action of 90 sec)" "This is a real-world medical entrance exam question, please give the true answer based on the question and selection. Topic name: None, Subject name: Dental","Porcelain denture teeth Select and explain. A. Have a higher coefficient of thermal expansion than acrylic teeth, B. Have a lower abrasion resistance than enamel, C. Should be used where the inter alveolar clearance is small, D. Have a higher abrasion resistance than gold",D: Have a higher abrasion resistance than gold,"Answer: D: Have a higher abrasion resistance than gold Explanation: None" "This is a real-world medical entrance exam question, please give the true answer based on the question and selection. Topic name: None, Subject name: Dental","Green Vermillion index measures Select and explain. A. Periodontal disease, B. Oral hygiene, C. Bone level, D. Gingival health",B: Oral hygiene,"Answer: B: Oral hygiene Explanation: None" "This is a real-world medical entrance exam question, please give the true answer based on the question and selection. Topic name: None, Subject name: Medicine","After Dengue hemorrhagic shock, what is the criteria for the patient to be discharged from the hospital? Select and explain. A. After return of appetite, B. After urine output more than 200ml, C. 24hrs after recovery from shock, D. Fever controlled by paracetamol for >24hrs",A: After return of appetite,"Answer: A: After return of appetite Explanation: Answer- A. After return of appetitePatients who are resuscitated from shock rapidly recover. Patients with dengue hemorrhagic fever or dengue shock syndrome may be discharged from the hospital when they meet the following criteria:* Afebrile for 24 hours without antipyretics* Good appetite, clinically improved condition* Adequate urine output* Stable hematocrit level* At least 48 hours since recovery from shock* No respiratory distress* Platelet count greater than 50,000 cells/mL" "This is a real-world medical entrance exam question, please give the true answer based on the question and selection. Topic name: None, Subject name: Pathology","Coxsackie virus causes Select and explain. A. infectious mononucleosis, B. Lymphoma, C. herpangina, D. herpes",C: herpangina,"Answer: C: herpangina Explanation: None" "This is a real-world medical entrance exam question, please give the true answer based on the question and selection. Topic name: None, Subject name: Anatomy","Which of the following is true about nerve supply of adrenal gland? Select and explain. A. Release of catecholamines is not affected by nerve supply to adrenals, B. Preganglionic fibres from lower thoracic and lumbar veebra come sympathetic chain to supply adrenals, C. Adrenal coex doesn't have nerve supply as it has endocrine functions, D. Adrenal medulla doesn't have nerve supply",B: Preganglionic fibres from lower thoracic and lumbar veebra come sympathetic chain to supply adrenals,"Answer: B: Preganglionic fibres from lower thoracic and lumbar veebra come sympathetic chain to supply adrenals Explanation: Ans. B. Preganglionic fibres from lower thoracic and lumbar veebra come sympathetic chain to supply adrenalsRef: Grays Anatomy, 41"" ed., pS. 439Suprarenal gland - Greater autonomic supply than any other organ.The nerves are distributed throughout the gland around blood vessels (regulating blood flow), in the medulla (stimulating the release of catecholamines from chromaffin cells), and in the coex (where they may influence steroid hormone production).A suprarenal plexus lies between the medial aspect ofeach gland and the coeliac and aoicorenal ganglia.It contains predominantly preganglionic syrnpathetic fibres that originate in the lower thoracic spinal segments, reach the plexus branches of the greater splanchnic newes, and synapse on clusters of large medullary chromaffin cells." "This is a real-world medical entrance exam question, please give the true answer based on the question and selection. Topic name: None, Subject name: Dental","According to the instrument formula given by G.V. Black, the primary cutting edge angle should be: Select and explain. A. >50 degree, B. >90 degree, C. <50 degree, D. <90 degree",A: >50 degree,"Answer: A: >50 degree Explanation: Blade angle should be more than 50 degree." "This is a real-world medical entrance exam question, please give the true answer based on the question and selection. Topic name: None, Subject name: Dental","Optimal anti-rotational design in a blade is incorporated by offset not greater than? Select and explain. A. 2 mm, B. 5 mm, C. 3 mm, D. 6 mm",A: 2 mm,"Answer: A: 2 mm Explanation: None" "This is a real-world medical entrance exam question, please give the true answer based on the question and selection. Topic name: None, Subject name: Dental","Which of the following is not true for S/S spreader compared to Ni-Ti Spreader? Select and explain. A. Less depth penetration is achieved with Ni-Ti spreader is achieved, B. More Flexibility with Ni-Ti spreader is achieved, C. Less stress Generated in Ni-Ti spreaded condensation, D. Less incidence of root Fracture with Ni-Ti spreader",A: Less depth penetration is achieved with Ni-Ti spreader is achieved,"Answer: A: Less depth penetration is achieved with Ni-Ti spreader is achieved Explanation: None" "This is a real-world medical entrance exam question, please give the true answer based on the question and selection. Topic name: None, Subject name: Microbiology","All are true about legionella pneumophilia except Select and explain. A. a) quatic bodies are main habitat, B. It can replicate in amoeba, C. Urine test is a reliable method, D. Person to person transmission",D: Person to person transmission,"Answer: D: Person to person transmission Explanation: None" "This is a real-world medical entrance exam question, please give the true answer based on the question and selection. Topic name: None, Subject name: Social & Preventive Medicine","In “Tip of Iceberg Phenomenon”, submerged portion does not consist of Select and explain. A. Healthy population, B. Carriers, C. Undiagnosed cases, D. Latent period",A: Healthy population,"Answer: A: Healthy population Explanation: None" "This is a real-world medical entrance exam question, please give the true answer based on the question and selection. Topic name: None, Subject name: Dental","Curve passing through the buccal and lingual cusp tips of the mandibular buccal teeth is: Select and explain. A. Wilson curve, B. Mansoon curve, C. Curve of Spee, D. Catenary curve",A: Wilson curve,"Answer: A: Wilson curve Explanation: None" "This is a real-world medical entrance exam question, please give the true answer based on the question and selection. Topic name: None, Subject name: Microbiology","Which stage of plasmodium vivax is infective to mosquito? Select and explain. A. Sporozoite, B. Gametocyte, C. Merozoite, D. Zygote",B: Gametocyte,"Answer: B: Gametocyte Explanation: When certain forms of blood stage parasites (gametocytes, which occur in male and female forms) are ingested during blood feeding by a female Anopheles mosquito, they mate in the gut of the mosquito and begin a cycle of growth and multiplication in the mosquito.  After 10-18 days, a form of the parasite called a sporozoite migrates to the mosquito’s salivary glands.  When the Anopheles mosquito takes a blood meal on another human, anticoagulant saliva is injected together with the sporozoites, which migrate to the liver, thereby beginning a new cycle. Ref :https://www.cdc.gov/malaria/about/biology/index.html" "This is a real-world medical entrance exam question, please give the true answer based on the question and selection. Topic name: None, Subject name: Medicine","A lady presents with complaints of abdominla pain. CECT shows bilateral papillary necrosis. Which of the following test shall not be done to investigate the cause of her papillary necrosis? Select and explain. A. Urine acidification test, B. Sickling test, C. Urine -PCR for TB, D. Bacterial culture of urine",A: Urine acidification test,"Answer: A: Urine acidification test Explanation: Ans. a. Urine acidification test(Ref: Harrison 19/e p1860, 18/e p2372)Urine acidification test is mainly done to diagnose A (Renal tubular acidosis), not the causes of papillary necrosis." "This is a real-world medical entrance exam question, please give the true answer based on the question and selection. Topic name: None, Subject name: Pharmacology","Phase 1 clinical trial is done for: Select and explain. A. Drug safety, B. Pharmacodynamics, C. Efficacy, D. Dosing",A: Drug safety,"Answer: A: Drug safety Explanation: Ans: A. Drug safety(Ref Goodman Gilman 12/e p79; .Katzung 13/e p12, 12/e p75; KDT 7/e p63-64, 6/e p77)Safety, pharmacodynamics & dosing - Tested in Phase 1 clinical trial.Drug safety - Most impoant primary end point." "This is a real-world medical entrance exam question, please give the true answer based on the question and selection. Topic name: None, Subject name: Gynaecology & Obstetrics","All are features of infant born to diabetic mother except: Select and explain. A. Obesity, B. Learning disability, C. Ketotic hypogylycemia, D. Future diabetes mellitus",B: Learning disability,"Answer: B: Learning disability Explanation: Late effects of maternal diabetes on children: Increased risk of diabetes in children if: If mother is diabetic – Risk 1–3% If father is diabetic – Risk 6% If both are diabetic – Risk 20% Increased risk of Cardiovascular disease (Cardiomyopathy). Increased risk of obesity Williams Obs. 23/e, p 1109 As far as learning disability is concerned, Williams 22/e, p 1178 says “It is seen that maternal diabetes has a negligible impact on cognitive development of child.” Williams 23/e, p 1116 says “RIZZO and colleagues (1995) used multiple tests of intelligence and psychomotor development to assess 196 children of diabetic mother upto age 9 years. They concluded that maternal diabetes had a negligible impact on cognitive development”." "This is a real-world medical entrance exam question, please give the true answer based on the question and selection. Topic name: None, Subject name: Dental","A permanent tooth bud, which has been accidentally removed during extraction of a deciduous tooth, should be: Select and explain. A. Discarded, B. The socket curetted and the tooth bud replanted, C. Pulpectomy of the tooth bud and placed into the socket, D. Place deep into the socket from where it was removed",D: Place deep into the socket from where it was removed,"Answer: D: Place deep into the socket from where it was removed Explanation: None" "This is a real-world medical entrance exam question, please give the true answer based on the question and selection. Topic name: None, Subject name: Ophthalmology","Which of the following stain is used for diagnosis of Granular dystrophy of cornea? Select and explain. A. Colloidal iron, B. Congo red, C. PAS, D. Masson trichrome",D: Masson trichrome,"Answer: D: Masson trichrome Explanation: Ans: D. Masson trichrome(Ref: Kanski 7/e p212-224; Parson :s 22/e p212-214, 21/e p207-209; Yanoff and Duker 4/e p261).Masson trichrome stain - Used for granular corneal dystrophy diagnosis.Granular dystrophy:AD inheritance with gene locus on 5q31Onset: First decade with recurrent erosionsSigns (in chronological order):Small, white, sharply demarcated deposits resembling crumbs or snowflakes in central anterior stroma.Increasing number of lesions with deeper & outward spread but not reaching limbus.Gradual confluence causing visual acuity impairment.Histology:Shows amorphous hyaline deposits staining bright red with Masson trichrome." "This is a real-world medical entrance exam question, please give the true answer based on the question and selection. Topic name: None, Subject name: Surgery","Which of the following is not a component of quick SOFA (qSOFA) scoring? Select and explain. A. Bilateral undilated pupils, B. Altered Mentation, C. Glasgow Coma Score, D. SBP <= 100 mm Hg",A: Bilateral undilated pupils,"Answer: A: Bilateral undilated pupils Explanation: Answer- A. Bilateral undilated pupilsAssessmentqSOFA scoreLow blood pressure (SBP <= 100 mmHg)1High respiratory rate (>= 22 breaths/min)1Altered mentation (GCS <= 14)1" "This is a real-world medical entrance exam question, please give the true answer based on the question and selection. Topic name: None, Subject name: Dental","Which one of the following is not used as an abrasive in the dentifrice Select and explain. A. Calcium carbonate, B. Silica, C. Sodium Chloride, D. Amylase",D: Amylase,"Answer: D: Amylase Explanation: None" "This is a real-world medical entrance exam question, please give the true answer based on the question and selection. Topic name: None, Subject name: Social & Preventive Medicine","Which of these indicators is used at anganwadi centers for growth monitoring in children? Select and explain. A. Height for age, B. Weight for age, C. Weight for height, D. Mid-arm circumference",B: Weight for age,"Answer: B: Weight for age Explanation: Ans: B. Weight for ageRef: Park 241e p582, 23/e p547, 22/e p506)India has adopted the new WHO Child Growth Standards (2006) in February 2009 for monitoring the young child growth and development within the National Rural Health Mission and the ICDS.These are based on weight for age.Weight for age is used at anganwadi centers for growth monitoring in children." "This is a real-world medical entrance exam question, please give the true answer based on the question and selection. Topic name: None, Subject name: Anatomy","Palatine process begins to fuse at: Select and explain. A. 6th week of fertilization, B. 8th week of fertilization, C. 10th week of fertilization, D. 12th week of fertilization",B: 8th week of fertilization,"Answer: B: 8th week of fertilization Explanation: 7 weeks - palate consists of the primary palate, formed by the fusion of the medial nasal processes. Behind the primary palate down growth of the nasal septum produces two communications between  the oral and nasal cavities the primitive nasal choanae. 8 weeks - Formation of the secondary palate begins with downward growth of the medial parts of the maxillary processes (palatine processes) to a location on either side of the tongue. As the mandible develops and the tongue drops down these palatine processes grow horizontally fusing with the nasal septum in the midline and with the primary palate along their anterior aspect. By 81/2 weeks - the palatal shelves appear above the tongue and in near contact with each other. At 8th & 9th week they come in contact and fusion begins Key Concept - Formation of the secondary palate begins at 8th week with downward growth of the medial parts of  the maxillary processes (palatine processes) to a location on either side of the tongue." "This is a real-world medical entrance exam question, please give the true answer based on the question and selection. Topic name: None, Subject name: Medicine","Shy Drager syndrome is: Select and explain. A. Intestinal polyp, B. Acute adernergic shock, C. Vasovagal hypotension, D. Orthostatic hypotension",D: Orthostatic hypotension,"Answer: D: Orthostatic hypotension Explanation: None" "This is a real-world medical entrance exam question, please give the true answer based on the question and selection. Topic name: None, Subject name: Pathology","Medlar bodies are found in - Select and explain. A. Sporotrichosis, B. Chromoblastomycosis, C. Mycetoma, D. Histoplasmosis",B: Chromoblastomycosis,"Answer: B: Chromoblastomycosis Explanation: Answer- B. ChromoblastomycosisMedlar bodies, also known as a sclerotic or muriform cell, When present in the skin or subcutaneous tissue, the cells are indicative of chromoblastomycosis.Chromoblastomycosis (CBM) is defined as a chronic cutaneous and subcutaneous fungal infection resulting from traumatic implantation of ceain dematiaceous fungi through the skin. In the infected tissue, characteristic dark-colored, thick-walled, muriform cells i.e. sclerotic cells (Medlar bodies) are seen." "This is a real-world medical entrance exam question, please give the true answer based on the question and selection. Topic name: None, Subject name: Anatomy","The muscles of the tongue are supplied by: Select and explain. A. Glossopharyngeal nerve, B. Lingual nerve, C. Chorda tympani, D. Hypoglossal",D: Hypoglossal,"Answer: D: Hypoglossal Explanation: None" "This is a real-world medical entrance exam question, please give the true answer based on the question and selection. Topic name: None, Subject name: Pathology","Transudate is characterized by: Select and explain. A. Associated inflammatory conditions, B. Low protein content, C. Tendency to clot, D. Specific gravity of above 1.018",B: Low protein content,"Answer: B: Low protein content Explanation: None" "This is a real-world medical entrance exam question, please give the true answer based on the question and selection. Topic name: None, Subject name: Dental","Function of major connector is that it: Select and explain. A. Connects parts of prosthesis on one side of arch with those on opposite side, B. Distributes forces aver entire supporting structure, C. Resists flexing and torque, D. All of the above",D: All of the above,"Answer: D: All of the above Explanation: None" "This is a real-world medical entrance exam question, please give the true answer based on the question and selection. Topic name: None, Subject name: Dental","Which of the following can result in Paradoxical excitement in children. Select and explain. A. Morphine, B. Phenobarbitone, C. Nitrous Oxide, D. Amphetamine",B: Phenobarbitone,"Answer: B: Phenobarbitone Explanation: None" "This is a real-world medical entrance exam question, please give the true answer based on the question and selection. Topic name: None, Subject name: Pharmacology","Which one of the following is a gender-specific side-effect of valproate? Select and explain. A. Polycystic ovarian syndrome, B. Alopecia, C. Weight loss, D. Tremor",A: Polycystic ovarian syndrome,"Answer: A: Polycystic ovarian syndrome Explanation: Ans: A. Polycystic ovarian syndrome Side-effects of Valproic AcidMC side effects are transient GI symptoms (anorexia, nausea & vomiting)Effects on the CNS: Sedation, ataxia & Rash, alopecia Stimulation of appetite & weight gain.Increase the chance of polycystic ovary syndrome (PCOS) in women with epilepsy or bipolar disorders Elevation of hepatic transaminases, microvesicular steatosisAcute pancreatitisHyperammonemiaNeural tube defects" "This is a real-world medical entrance exam question, please give the true answer based on the question and selection. Topic name: AIIMS 2019, Subject name: Orthopaedics","History of fall on outstretched hand with the given xray: Select and explain. A. Galeazzi fracture, B. Montaggia fracture, C. Colles fracture, D. Smith fracture",A: Galeazzi fracture,"Answer: A: Galeazzi fracture Explanation: The xray shows fracture of radius + disruption of distal radio-ulnar joint - Galeazzi fracture ""FOOSH"" (Fracture due to fall on outstretched hand)* Fracture clavicle* Surgical neck of humerus fracture* Supracondylar fracture humerus and lateral condyle fracture humerus* Head and neck fracture of radius* Galeazzi fracture dislocation* Colle's fracture (Most common)* Radial styloid fracture* Fracture scaphoid" "This is a real-world medical entrance exam question, please give the true answer based on the question and selection. Topic name: None, Subject name: Surgery","Nitrous oxide alone is not used as a G.A agent because of Select and explain. A. Difficulty in maintaining an adequate oxygen concentration, B. Expense of the agent and its exposure hazards, C. Adverse effect on liver, D. Poor analgesic property",A: Difficulty in maintaining an adequate oxygen concentration,"Answer: A: Difficulty in maintaining an adequate oxygen concentration Explanation: None" "This is a real-world medical entrance exam question, please give the true answer based on the question and selection. Topic name: None, Subject name: Physiology","Maximum pCO2 is seen in which of the following stage in arterial gas analysis? Select and explain. A. A, B. B, C. C, D. D",A: A,"Answer: A: A Explanation: None" "This is a real-world medical entrance exam question, please give the true answer based on the question and selection. Topic name: None, Subject name: Dental","Cholesterol crystals are found in which zone? Select and explain. A. Zone of infection, B. Zone of contamination, C. Zone of irritation, D. Zone of stimulation",C: Zone of irritation,"Answer: C: Zone of irritation Explanation: The reaction of the periradicular tissues to noxious products of tissue necrosis, bacterial products and antigenic agents from the root canal has been described by Fish. The four zones  Zone of infection Zone of contamination Zone of irritation Zone of stimulation Zone of Infection: This is present in the center of the lesion. It is characterized by PMNL's. Zone of Contamination: It is characterized by round cell infiltration. It demonstrates cellular destruction due to toxins from the central zone or zone of infection. Because of autolysis and death of bone cells, the lacunae were empty. Lymphocytes are prevalent everywhere. Zone of Irritation: This zone is characterized by macrophages and osteoclasts. The collagen framework was digested by phagocytic cells, the macrophages, while osteoclasts attacked the bone tissue. This opens a gap in the bone all around the center of lesion. That space becomes filled with PMNL's. It contains cholesterol crystals. This zone demonstrates much activity preparatory to repair. Zone of Stimulation: (Peripheral zone) This zone is characterized by fibroblasts and osteoblasts. In this zone, the effects of toxins were mild enough to be stimulant which results in laying down of collagen fibers around the zone of irritation that acts both as a wall of defense and as a scaffolding on which the osteoblasts built new bone. This new bone was built in an irregular fashion." "This is a real-world medical entrance exam question, please give the true answer based on the question and selection. Topic name: None, Subject name: Biochemistry","Following are the essential amino acids: Select and explain. A. Phenylalanine, Tryptophan, Lysine, B. Phenylalanine, Asparagine, Methionine, C. Phenylalanine, Valine, Glycine, D. Histidine, Glutamine, Valine","A: Phenylalanine, Tryptophan, Lysine","Answer: A: Phenylalanine, Tryptophan, Lysine Explanation: None" "This is a real-world medical entrance exam question, please give the true answer based on the question and selection. Topic name: None, Subject name: Medicine","A 25 years old lady with a history of fever for 1 month presents with headache and ataxia.Brain imaging shows dilated ventricles and significant basal exudates. Which of the following will be the most likely CSF finding? Select and explain. A. Lymphocytosis, Low Glucose, High protein, B. Lymphocytosis, Normal Glucose, High protein, C. Lymphocytosis, Low Glucose, Normal protein, D. Neutrophilia, Low glucose, Low Protein","A: Lymphocytosis, Low Glucose, High protein","Answer: A: Lymphocytosis, Low Glucose, High protein Explanation: Ans. A (Lymphocytosis, Low Glucose, High protein)Presence of significant basal exudates, together with dilated ventricles (hydrocephalus) in a young female with a prolonged history of fever and headache suggests a diagnosis of Tubercular Meningitis.Tubercular Meningitis is characterized by Lymphocytic Pleocytosis, Low Glucose and High Protein within the CSF.The pathological hallmark of Tubercular Meningitis is the predominant involvement of basal cisterns that are observed by the presence of basal inflammatory tissue exudate." "This is a real-world medical entrance exam question, please give the true answer based on the question and selection. Topic name: AIIMS 2018, Subject name: Pediatrics","A sick intubated neonate is having bilateral jerk of both right and left upper limbs with some occasional twitching of neck as well. Likely type of seizures:- Select and explain. A. Focal clonic, B. Multifocal clonic, C. Multifocal tonic clonic, D. Focal tonic",B: Multifocal clonic,"Answer: B: Multifocal clonic Explanation: Though most common type of neonatal seizure is subtle seizure, subtle seizures are very difficult to recognise, and might manifest as just some lip smacking movements, or cyclical movement of limbs. However, in the given scenario, the newborn has bilateral jerk of both right and left upper limbs with some occasional twitching of neck. This is suggestive of multifocal clonic seizures." "This is a real-world medical entrance exam question, please give the true answer based on the question and selection. Topic name: None, Subject name: Medicine","Sickle cell mutation is Select and explain. A. Point mutation, B. Selective mutation, C. Frame shift mutation, D. Nonsense mutation",A: Point mutation,"Answer: A: Point mutation Explanation: None" "This is a real-world medical entrance exam question, please give the true answer based on the question and selection. Topic name: None, Subject name: Microbiology","Resistance of Staphylococcus aureus is due to: Select and explain. A. Pleomorphism, B. Elaboration of an enzyme that destroy penicillin, C. Penicillin analogue production, D. Lack of nucleic acid in the cell wall",B: Elaboration of an enzyme that destroy penicillin,"Answer: B: Elaboration of an enzyme that destroy penicillin Explanation: None" "This is a real-world medical entrance exam question, please give the true answer based on the question and selection. Topic name: None, Subject name: Dental","All information must be explained in comprehensible, non medical terms preferable in local language about the diagnosis, nature of treatment, risks involved, prospects of success and alternative methods of treatment known Select and explain. A. Implied consent, B. Express consent, C. Informed consent, D. Substitute consent",C: Informed consent,"Answer: C: Informed consent Explanation: None" "This is a real-world medical entrance exam question, please give the true answer based on the question and selection. Topic name: None, Subject name: Medicine","Proteins elevated in the plasma in myocardial infarction Select and explain. A. Troponin T, B. Troponin H, C. Troponin K, D. Troponin M",A: Troponin T,"Answer: A: Troponin T Explanation: None" "This is a real-world medical entrance exam question, please give the true answer based on the question and selection. Topic name: None, Subject name: Dental","Dystrophic calcification is seen most commonly in which of the following oral tissues Select and explain. A. Enamel, B. Pulp, C. Dentin, D. Periodontal ligament",B: Pulp,"Answer: B: Pulp Explanation: None" "This is a real-world medical entrance exam question, please give the true answer based on the question and selection. Topic name: None, Subject name: Gynaecology & Obstetrics","A pregnant woman with G3P2L0 presented to you with a pregnancy at period of gestation of 9 weeks. She has a history of conization one year back currently on follow up with no recurrence on PAP smean She also has the history of preterm bihs at 30 and 32 weeks during her last 2 pregnancy. What is your next step in the management of this patient? Select and explain. A. USG to see cervical length, B. Cervical cerclage, C. Complete bed rest, D. Abdominal cerclage",A: USG to see cervical length,"Answer: A: USG to see cervical length Explanation: Ans: A. USG to see cervical lengthRef NICE guidelines; ACOG guidelines; SOGC guidelines.The current review fromACOG recommends cervical cerclage for women with a current singleton pregnancy, prior spontaneous preterm bih at less than 34 weeks of gestation, and cervical length less than 25 mm." "This is a real-world medical entrance exam question, please give the true answer based on the question and selection. Topic name: None, Subject name: Dental","Angles between Adhesive and Adherent is Zero degree, it indicates: Select and explain. A. Complete wetting of surfaces, B. Rough surfaces between Adhesive & Adherent, C. Adherent and adhesive molecules are tangent to each other, D. Irregularities present between Adherent surfaces",A: Complete wetting of surfaces,"Answer: A: Complete wetting of surfaces Explanation: None" "This is a real-world medical entrance exam question, please give the true answer based on the question and selection. Topic name: None, Subject name: Dental","Which of the following require delayed separation? Select and explain. A. Tilted tooth, B. Drifted tooth, C. Rotated tooth, D. All of the above",D: All of the above,"Answer: D: All of the above Explanation: None" "This is a real-world medical entrance exam question, please give the true answer based on the question and selection. Topic name: None, Subject name: Pathology","Acetone free methyl alcohol is present in Leishmann's stain for: Select and explain. A. It fixes cells to the slide, B. It colors the red cells, C. It prevents the cells from sticking to the slide surface, D. It stops metabolic and enzymatic activity of the cell",D: It stops metabolic and enzymatic activity of the cell,"Answer: D: It stops metabolic and enzymatic activity of the cell Explanation: Answer- D. It stops metabolic and enzymatic activity of the cellTt is a type ofAcidic dye stains the basic components of cell & basic dye stains the acidic components of cell.Leishman's stain contains eosin & methylene blue in acetone free methyl alcohol.Methyl alcohol acts as a fixative.Acetone if present, will destroy the cell membraneMethylene blue (""polychromed""), the basic dye and eosin, the acidic dye exists as thiazine eosinate, which dissociates into the component dyes, when diluted with distilled water.Methyl blue stains the nucleus & basophilic granules of WBC, whereas eosin stains the eosinophilic granules.It is generally used to differentiate & identily leucocytes, malaria parasites & trypanosomas" "This is a real-world medical entrance exam question, please give the true answer based on the question and selection. Topic name: None, Subject name: Skin","Treatment of choice for erythrodermic psoriasis: Select and explain. A. Methotrexate, B. Coicosteroids, C. Coal tar topical, D. Topical steroids",A: Methotrexate,"Answer: A: Methotrexate Explanation: Ans: A. Methotrexate (Ref Fitzpatrick 6/e p2676)Methotrexate or oral retinoids (Acitretin) - DOC for erythrodermic psoriasis management.Current use of methotrexate to treat psoriasis is most common in patients with severe or refractory plaque-type disease that requires systemic treatment.It is also very useful for erythrodermic and pustular psoriasis, as well as psoriatic ahritis." "This is a real-world medical entrance exam question, please give the true answer based on the question and selection. Topic name: None, Subject name: Dental","An 8 year old child got trauma and got his central incisor avulsed 20 minutes back which got contaminated with debris. The child rushes to dental clinic with avulsed tooth, the treatment is: Select and explain. A. Clean root surface with saline and reimplant, B. Clean and curette root surface and reimplant, C. Clean root surface, do RCT and reimplant, D. Scrubbing of root surface and reimplant",A: Clean root surface with saline and reimplant,"Answer: A: Clean root surface with saline and reimplant Explanation: If the tooth has been out for 15 minutes to 60 minutes: Tooth with closed apex Clean the root surface with saline. Do not touch a viable root with hands, forceps, gauze or anything, or try to scrub or clean it to avoid injury to the periodontal ligament which makes it difficult to revascularize the reimplanted tooth. Examine alveolar socket after cleaning it with saline. Do not overlook fracture of tooth and alveolar ridge. Reimplant the tooth gently with firm finger pressure. Ask the patient to bite down firmly on a piece of gauze to help stabilize the tooth. After evaluating the occlusion, stabilize the tooth if required. Ideal splint required for an avulsed tooth is a flexible splint. The commonly used flexible splints are made of Gortex, metallic mesh stripes, synthetic clothes or orthodontic wire. A thick strip of composite bonded to the avulsed and the adjacent teeth also act as a splint. The splint should engage several teeth around the avulsed tooth and it should be kept in place for not more than 7 to 10 days. Continue with the same treatment as above. Tooth with open apex: Clean the root surface with saline. Examine the alveolar socket after cleaning it with saline. Cover the root surface with minocycline hydrochloride microspheres before reimplanting to kill bacteria which could enter the immature apex and form an abscess. Continue with the same treatment as above. Ref: Textbook of endodontics, Nisha Garg and Amit Garg, 3rd edition, pg no:477" "This is a real-world medical entrance exam question, please give the true answer based on the question and selection. Topic name: None, Subject name: Dental","In 3 mix paste; discoloration occurs due to presence of: Select and explain. A. Ciprofloxacin, B. Minocycline, C. Metronidazole, D. Tetracycline",B: Minocycline,"Answer: B: Minocycline Explanation: None" "This is a real-world medical entrance exam question, please give the true answer based on the question and selection. Topic name: None, Subject name: Dental","Ear prosthesis and ocular prosthesis is made from Select and explain. A. Methyl methacrylate, B. Metallic implants, C. Silicone, D. Epoxy resin",C: Silicone,"Answer: C: Silicone Explanation: None" "This is a real-world medical entrance exam question, please give the true answer based on the question and selection. Topic name: None, Subject name: Microbiology","Following hepatitis B infection through blood transfusion, disease manifests in: Select and explain. A. 1 week, B. 6 weeks, C. 3 months, D. 1 year",C: 3 months,"Answer: C: 3 months Explanation: None" "This is a real-world medical entrance exam question, please give the true answer based on the question and selection. Topic name: None, Subject name: Social & Preventive Medicine","Which of these statements is true about Rashtriya Swasthya Bima Yojana? Select and explain. A. Cashless benefit on presenting smacard and fingerprints, B. Valid for up to 4 family members, C. Can be used only in 1 district, D. Treatment only in government hospitals",A: Cashless benefit on presenting smacard and fingerprints,"Answer: A: Cashless benefit on presenting smacard and fingerprints Explanation: Ans: A. Cashless benefit on presenting smacard and fingerprintsRashtriya Swasthya Bima Yojana (RSBY):Government- run health insurance scheme for the Indian poor.It provides fir cashless insurance for hospitalization in public as well as private hospitals.Provide health insurance coverage for Below Povey Line (BPL) families.Objective is to provide protection to BPL households from financial liabilities arising out of health shocks that involve hospitalization." "This is a real-world medical entrance exam question, please give the true answer based on the question and selection. Topic name: None, Subject name: Biochemistry","The activity of the following enzyme is affected by biotin deficiency: Select and explain. A. Transketolase, B. Dehydrogenase, C. Oxidase, D. Carboxylase",D: Carboxylase,"Answer: D: Carboxylase Explanation: None" "This is a real-world medical entrance exam question, please give the true answer based on the question and selection. Topic name: None, Subject name: Radiology","For maintaining parallelism between object the and film, film is placed at a distance to avoid distortion what should be done: Select and explain. A. Increase target to object distance, B. Decrease target to object distance, C. Both of the above, D. None of the above",A: Increase target to object distance,"Answer: A: Increase target to object distance Explanation: None" "This is a real-world medical entrance exam question, please give the true answer based on the question and selection. Topic name: None, Subject name: Pediatrics","Which type of cerebral palsy is commonly associated with scoliosis and other orthopedic problems – Select and explain. A. Spastic quadriplegia, B. Anterior cerebral palsy, C. Spastic deplegia, D. Atonic cerebral palsy",A: Spastic quadriplegia,"Answer: A: Spastic quadriplegia Explanation: ""Scollosis is common, occuring in 15 to 25 percent of the total body CP (spastic quadriplegia cp)"".  Cerebral palsy (CP) Cerebral palsy is defined as a non progressive neuromotor disorder of cerebral origin. It has following features - Static (nonprogressive) Disorder of movement/motor disorder (but sometimes sensory involvement may occur) Cerebral in origin Etiopathogenesis CP is due to an insult of developing brain. The insult results in maldevelopment and disorderly anatomic organization of the brain. The insult may be prenatal (most of the cases), during delivery or in the postnatal period. Classification CP may be classified into following types -" "This is a real-world medical entrance exam question, please give the true answer based on the question and selection. Topic name: None, Subject name: Biochemistry","Vitamin K antagonizes Select and explain. A. Corticosteroids, B. Thrombin formation, C. Bishydroxy coumarin, D. Production of clotting factors by liver",C: Bishydroxy coumarin,"Answer: C: Bishydroxy coumarin Explanation: Bishydroxy coumarin (dicoumarol), a vitamin K antagonist may be considered as an oral anticoagulant.  This acts by reducing  the synthesis of certain clotting factors (ll, Vll, lX and X). Ref : Lippincott’s illustrated reviews for biochemistry , 7th edition." "This is a real-world medical entrance exam question, please give the true answer based on the question and selection. Topic name: None, Subject name: Physiology","Which of the following does golgi tendon organ detect? Select and explain. A. Muscle Tension, B. Dynamic muscle length, C. Static muscle length, D. Muscle action",A: Muscle Tension,"Answer: A: Muscle Tension Explanation: Ans. A. Muscle Tension(Ref.: Ganong 25/e p232; Guyton 13/e p697, 701).Golgi tendon organ senses muscle tension.The Golgi organ (also called Golgi tendon organ, GTO, tendon organ, neurotendinous organ or neurotendinous spindle) senses changes in muscle tension.It is a proprioceptive sensory receptor organ that is at the origin and inseion of skeletal muscle fibers into the tendons of skeletal muscle.It provides the sensory component of the Golgi tendon reflex." "This is a real-world medical entrance exam question, please give the true answer based on the question and selection. Topic name: None, Subject name: Gynaecology & Obstetrics","Maximum strain of parturient heart occurs during: Select and explain. A. At term, B. Immediate postpartum, C. Ist trimester, D. IInd trimester",B: Immediate postpartum,"Answer: B: Immediate postpartum Explanation: “Significant hemodynamic alterations are apparent early in pregnancy, women with severe cardiac dysfunction may experience worsening of heart failure before mid pregnancy. In others, heart failure develops after 28 weeks, when pregnancy induced hypervolemia is maximal (32 weeks). In the majority, however heart failure develops peripartum when the physiological capability for rapid changes in cardiac out put may be overwhelmed in presence of structural cardiac disease.” Williams 22/e, p 1018, 23/e, p 958, 959 Reading the above text, from Williams Obs., it is clear that maximum chances of heart failure are in the peripartum period. But it is not clear whether maximum chances are during labour or immediate postpartum. Dutta Obs. 7/e, p53 provides answer to this: “The cardiac output starts to increase from 5th week of pregnancy, reaches its peak 40-50% at about 30-34 weeks. Thereafter the cardiac output remains static till term”. “Cardiac output increases further during labour (+50%) and immediately following delivery (+70%) over the pre labour values.” So, maximum chances of heart failure are in immediate postpartum period when cardiac output is maximum." "This is a real-world medical entrance exam question, please give the true answer based on the question and selection. Topic name: None, Subject name: Microbiology","Schizonts and late trophozoite stages of plasmodium falciparum not seen in peripheral blood smear because: Select and explain. A. They are sequestered in the spleen, B. Due to adherence to the capillary endothelium, they are not seen in peripheral blood, C. Due to antigen-antibody reaction and removal, D. They are seen in mosquito blood","B: Due to adherence to the capillary endothelium, they are not seen in peripheral blood","Answer: B: Due to adherence to the capillary endothelium, they are not seen in peripheral blood Explanation: Ans: B. Due to adherence to the capillary endothelium, they are not seen in peripheral blood(Ref: Harrison 1y/e p1371, Jawetz 27/e p719)Plasmodium falciparum:Only ring stages or gametocytes are seen in infected peripheral blood.Schizonts & late trophozoite stages of Plasmodium falciparum are not seen in infected peripheral blood.Parasites make red cells sticky a Tend to be retained in deep capillary beds (except in overwhelming fatal infections)." "This is a real-world medical entrance exam question, please give the true answer based on the question and selection. Topic name: None, Subject name: Surgery","In depressed zygomatic arch fracture, difficulty in opening the mouth is caused by impingement of: Select and explain. A. Condyles, B. Ramus, C. Petrous temporal, D. Coronoid process",D: Coronoid process,"Answer: D: Coronoid process Explanation: None" "This is a real-world medical entrance exam question, please give the true answer based on the question and selection. Topic name: None, Subject name: Gynaecology & Obstetrics","A pregnant female delivered a baby with normal expulsion of an intact placenta. After half hour she staed bleeding per vaginaly. On examination she was hypotensive and boggy mass is palpated per abdomen. USG showed retained placental tissues. what is the likely diagnosis? Select and explain. A. Placenta succenturiata, B. Adenomyosis, C. Placenta accreta, D. Membranous placenta",A: Placenta succenturiata,"Answer: A: Placenta succenturiata Explanation: Ans: A. Placenta succenturiataRef: DC Dutta's tubook of Obstetrics, gh ed.Placenta succenturiata has one (usual) or more small lobes of placenta placed at a varying margin from the main placental margin.A leash of vessels connects the small lobe with the main lobe.Many times, succenturiate is retained and it presents as postpaum hemorrhage which may be primary or secondary." "This is a real-world medical entrance exam question, please give the true answer based on the question and selection. Topic name: None, Subject name: Medicine","Crescendo angina is Select and explain. A. Stable angina, B. Heart failure, C. Unstable angina, D. MI",C: Unstable angina,"Answer: C: Unstable angina Explanation: None" "This is a real-world medical entrance exam question, please give the true answer based on the question and selection. Topic name: None, Subject name: Pediatrics","After the delivery of an infant of diabetic mother, blood glucose of the infant was 60 mg/dt. Which other investigation docs the sister expects that the physician would ask her to do? Select and explain. A. Serum potassium, B. CBC, C. Serum calcium, D. Serum chloride",C: Serum calcium,"Answer: C: Serum calcium Explanation: Ans: C. Serum calciumRef Ghai Essential Pedistrics 8,h ed, pg. 181 und Nelson Textbook of Pediatrics 20h ed"" pg. 897 Infants of a diabetic mother are ata higher risk of metabolic complications as compared to normal infants.These complications include:* Hypoglycemia* Hypocalcemia* Hypomagnesemia* Hence the infant needs to be checked for these as soon as possible." "This is a real-world medical entrance exam question, please give the true answer based on the question and selection. Topic name: None, Subject name: Surgery","About giant cell tumor, all are true except: Select and explain. A. Commonly presents in the 20-40 year age group, B. Matrix consists of proliferating mononuclear cells, C. Osteoclast giant cells constitute the proliferative component of the tumor, D. It is a benign tumor which may have lung metastasis",C: Osteoclast giant cells constitute the proliferative component of the tumor,"Answer: C: Osteoclast giant cells constitute the proliferative component of the tumor Explanation: Ans: C. Osteoclast giant cells constitute the proliferative component of the tumor(Ref: Robbins 9Ie p1204; Apley's 9Ie p202).In giant cell tumors, osteoclast giant cells are the malignant cells, which induce the proliferation of mononuclear macrophage lineage cells, hence the dividing cell population is the mononuclear cells which form the matrix.Osteoclast giant cells do not constitute the proliferative component of the tumor." "This is a real-world medical entrance exam question, please give the true answer based on the question and selection. Topic name: None, Subject name: Dental","During recording of PPS, face is titled down 30 degrees to touch the sternum: Select and explain. A. To activate the muscles of palate & pharynx, B. To activate the muscles of soft palate only, C. To prevent flow of material to throat, D. Help in sucking & swallowing",B: To activate the muscles of soft palate only,"Answer: B: To activate the muscles of soft palate only Explanation: None" "This is a real-world medical entrance exam question, please give the true answer based on the question and selection. Topic name: None, Subject name: Pathology","A 6 year old patient with extra cusp on maxillary central incisor is associated with all, except Select and explain. A. Mohr's syndrome, B. Sturge-Weber Syndrome, C. Rubinstein Taybi Syndrome, D. Proteus Syndrome",D: Proteus Syndrome,"Answer: D: Proteus Syndrome Explanation: None" "This is a real-world medical entrance exam question, please give the true answer based on the question and selection. Topic name: None, Subject name: Dental","High copper dental amalgams are superior to other amalgams because high copper dental amalgams: Select and explain. A. Have less marginal breakdown, B. Are workable at lower Hg-alloy ratio, C. Have a higher ratio of tensile to compressive strength, D. Have less resistance to tarnish and corrosion",A: Have less marginal breakdown,"Answer: A: Have less marginal breakdown Explanation: None" "This is a real-world medical entrance exam question, please give the true answer based on the question and selection. Topic name: None, Subject name: Dental","Study models are used: Select and explain. A. As references in orthodontic cases, B. To show shape, size and position of teeth, C. As an aid in treatment planning, D. All of the above",D: All of the above,"Answer: D: All of the above Explanation: None" "This is a real-world medical entrance exam question, please give the true answer based on the question and selection. Topic name: None, Subject name: Dental","A patient is giving; history of avulsed tooth 20 minutes back, comes to dentist what should be done? Select and explain. A. Scrub the tooth and reimplant, B. Rinse with saline and reimplant, C. Sterilize tooth and reimplant, D. Scrub the tooth do RCT and reimplant",B: Rinse with saline and reimplant,"Answer: B: Rinse with saline and reimplant Explanation: If a patient provided history that a tooth has been avulsed and it can be determined that the injury is without other oral. neurologic, or higher-priority physical complications. it is best to replace it in the socket immediately and to hold it in place with light finger pressure. If the avulsion occurred in a clean environment, nothing should be done to the tooth before replants it. If the tooth is dirty, an attempt should be made to clean the root surface, but it is very important to preserve any remnants of the periodontal ligament that are still attached to the root. 'therefore the parent would then be instructed to keep the tooth immersed in a suitable storage medium and bring the child and the tooth for immediate care. The patient should receive immediate attention after arriving at the dental office; and if it seems any contamination than it should be rinse thoroughly with saline only do not attempt to scrub tooth by any means to prevent vitality of periodontal fibers and re-implant it in socket. If patient or parent cannot or will not replant it than procedure to maintain vitality of tooth Allowing the avulsed tooth to dehydrate before replantation is damaging to a favourable prognosis. Hanks buffered saline, isotonic saline, and pasteurized bovine milk may be the most favourable known storage media. If none of these solutions is readily available, human saliva is acceptable short-term substitute storage liquid. Presumably, the patient's saliva (and perhaps blood) would be readily available. Although tap water has been a commonly recommended storage solution (and its use would be preferable to allowing dehydration of the tooth), saliva is a better storage medium. Neither water nor saliva is as good as milk or saline, if the tooth must be stored for a long period (more than 30 minutes before replantation). Because water is hypotonic, its use leads to rapid cell lysis and increased inflammation on replantation." "This is a real-world medical entrance exam question, please give the true answer based on the question and selection. Topic name: None, Subject name: Ophthalmology","14 year old child with blindness, sensorineural hearing loss, progressive hematuria, hypeension with similar family history in father Select and explain. A. Alpo syndrome, B. Goldenhar syndrome, C. Goodpasture syndrome, D. Nager syndrome",A: Alpo syndrome,"Answer: A: Alpo syndrome Explanation: Answer: A. Alpo syndrome* Alpo syndrome is a genetic condition characterized by kidney disease, hearing loss, and eye abnormalities.* People with Alpo syndrome experience progressive loss of kidney function.* Almost all affected individuals have blood in their urine (hematuria), which indicates abnormal functioning of the kidneys." "This is a real-world medical entrance exam question, please give the true answer based on the question and selection. Topic name: None, Subject name: Medicine","Coarse tremors of tongue is seen in all except: Select and explain. A. Parkinsonism, B. Alcohol, C. Thyrotoxicosis, D. General paresis",C: Thyrotoxicosis,"Answer: C: Thyrotoxicosis Explanation: Answer- C. ThyrotoxicosisTypically the tremor of Parkinson's disease is asymmetrical, at least initially, and affects an upper limb involving the ipsilateral leg after a latency of approximately two years. The disease may also produce tremor of the lips, tongue or jaw but it rarely causes significant head or vocal tremor.Fine tremors- Thyrotoxicosis" "This is a real-world medical entrance exam question, please give the true answer based on the question and selection. Topic name: None, Subject name: Dental","Most probable chance of developing caries in the following curve is at: Select and explain. A. Point A, B. Point B, C. Point C, D. Point D",B: Point B,"Answer: B: Point B Explanation: It is Stephen's curve, which is used for illustration of rapid decrease in pH after sucrose consumption. Exposure of dental plaque to a fermentable carbohydrate leads to decrease in pH rapidly,  reaching minimum in approx. 5-10 minutes. It recovers to its initial value in 20-60 minutes gradually." "This is a real-world medical entrance exam question, please give the true answer based on the question and selection. Topic name: None, Subject name: Dental","Dustless alginate is produced by Select and explain. A. Reducing the diatomaceous earth, B. Adding heavy metal salts, C. Coating with dihydric alcohol, D. Altering the matrix",C: Coating with dihydric alcohol,"Answer: C: Coating with dihydric alcohol Explanation: None" "This is a real-world medical entrance exam question, please give the true answer based on the question and selection. Topic name: None, Subject name: Dental","A-alpha fibers have a speed of conduction of ------ m/s. Select and explain. A. 20-Oct, B. 40-50, C. 50-70, D. 70-120",D: 70-120,"Answer: D: 70-120 Explanation: None" "This is a real-world medical entrance exam question, please give the true answer based on the question and selection. Topic name: None, Subject name: Social & Preventive Medicine","Which of these is not true about randomization in a clinical trial? Select and explain. A. Reduces confounding, B. Decreases selection bias, C. Ensures comparability of two groups, D. Increases external validity of the trial",D: Increases external validity of the trial,"Answer: D: Increases external validity of the trial Explanation: Ans: D. Increases external validity of the trial(Ref Park 24/e p87, 23/e p82, 22/e p79)External validity is the validity of generalized (causal) inferences in scientific research, usually based on experiments as experimental validity.In other words, it is the extent to which the results of a study can be generalized to other situations and to other people.This cannot be ensured by randomization." "This is a real-world medical entrance exam question, please give the true answer based on the question and selection. Topic name: None, Subject name: Dental","Which of the following is a dye used for dentinal caries detection: Select and explain. A. Calcein, B. Zygo ZL-22., C. Fuschin., D. Acid red system.",D: Acid red system.,"Answer: D: Acid red system. Explanation: Dyes used in caries detection are: Enamel caries Calcein  Zygo ZL-22 Fuschin Dentinal caries    Acid red system α - Aminoacridine" "This is a real-world medical entrance exam question, please give the true answer based on the question and selection. Topic name: None, Subject name: Pharmacology","Ritonavir inhibits metabolism of all of the following drugs except: Select and explain. A. Amiodarone, B. Phenytoin, C. Cisapride, D. Midazolam",B: Phenytoin,"Answer: B: Phenytoin Explanation: Ans. b. Phenytoin (Ref Goodman Gillman Ilth/1302; Katzung II/e plI44)Ritonavir inhibits metabolism of Amiodarone, Cisaprlde, Midazolam and but not the phenytoin'The majority (95%o) of phenytoin is metabolized principalty in the hepatic endoplasmic reticulum by CYP2C9/10 and to a lesser extent CYP2C9.'" "This is a real-world medical entrance exam question, please give the true answer based on the question and selection. Topic name: None, Subject name: Dental","About gutta-percha all are true, except: Select and explain. A. It contains 60-70% gutta-percha and 20% ZnO, B. Can be sterilized by heating, C. With time they become brittle, D. It has two forms α and β",B: Can be sterilized by heating,"Answer: B: Can be sterilized by heating Explanation: None" "This is a real-world medical entrance exam question, please give the true answer based on the question and selection. Topic name: None, Subject name: Dental","A patient shows one or more of the following: advanced bone loss, grade II and III furcation involvements, tooth mobility, inaccessible areas, systemic/environmental factors represents: Select and explain. A. Questionable prognosis, B. Poor prognosis, C. Fair prognosis, D. Hopeless prognosis",A: Questionable prognosis,"Answer: A: Questionable prognosis Explanation: Good prognosis: Control of etiologic factors and adequate periodontal support ensure the tooth will be easy to maintain by the patient and clinician. Fair prognosis: Approximately 25% attachment loss or grade I furcation invasion (location and depth allow proper maintenance with good patient compliance). Poor prognosis: 50% attachment loss, grade II furcation invasion (location and depth make maintenance possible but difficult). Questionable prognosis: >50% attachment loss, poor crown-to-root ratio, poor root form, grade II furcation invasion (location and depth  make  access  difficult)  or  grade  III  furcation  invasion; mobility no. 2 or no. 3; root proximity. Hopeless  prognosis:  Inadequate  attachment  to  maintain  health, comfort, and function. Ref: Newman and Carranza’s Clinical Periodontology, thirteenth edition; page no 413" "This is a real-world medical entrance exam question, please give the true answer based on the question and selection. Topic name: None, Subject name: Physiology","Carotid sinus/baroreceptor is located at the origin of Select and explain. A. Common carotid artery, B. Internal carotid artery, C. Aorta, D. External carotid artery",B: Internal carotid artery,"Answer: B: Internal carotid artery Explanation: None" "This is a real-world medical entrance exam question, please give the true answer based on the question and selection. Topic name: None, Subject name: Surgery","Maximum permissible dose of Articaine in a healthy patient is Select and explain. A. 1.3 mg/kg, B. 5 mg/kg, C. 2 mg/kg, D. 7 mg/kg",D: 7 mg/kg,"Answer: D: 7 mg/kg Explanation: None" "This is a real-world medical entrance exam question, please give the true answer based on the question and selection. Topic name: None, Subject name: Dental","ADA specification of orthodontic wire: Select and explain. A. 29, B. 25, C. 32, D. 30",C: 32,"Answer: C: 32 Explanation: None" "This is a real-world medical entrance exam question, please give the true answer based on the question and selection. Topic name: None, Subject name: Dental","Which is true about mucogingival flap designs Select and explain. A. Flap should be wider at the base, B. Flap should be narrower at the base, C. Flap margins should not rest on bone, D. Mucogingival flaps should be avoided",A: Flap should be wider at the base,"Answer: A: Flap should be wider at the base Explanation: None" "This is a real-world medical entrance exam question, please give the true answer based on the question and selection. Topic name: None, Subject name: Dental","An anterior bite plate should be trimmed so that Select and explain. A. It is included at 10° to occlusal plane, B. It is inclined at 30° to occlusal plane, C. Posterior teeth are 5 mm apart, D. Posterior teeth are 2 mm apart",D: Posterior teeth are 2 mm apart,"Answer: D: Posterior teeth are 2 mm apart Explanation: None" "This is a real-world medical entrance exam question, please give the true answer based on the question and selection. Topic name: None, Subject name: Dental","In resorption of roots of the primary teeth, the dental pulp Select and explain. A. Plays a passive role, B. Becomes a fibrotic non—vital mass, C. Initiates resorption from the inner surface of roots, D. Aids in formation of secondary dentin slowing down resorption",A: Plays a passive role,"Answer: A: Plays a passive role Explanation: None" "This is a real-world medical entrance exam question, please give the true answer based on the question and selection. Topic name: None, Subject name: Surgery","A 2 years old child suffers flame burns involving face, bilateral upper limbs and front of chest and abdomen. What is the body surface area involved? Select and explain. A. 40%, B. 45%, C. 54%, D. 60%",A: 40%,"Answer: A: 40% Explanation: Ans: A. 40%(Ref Sabiston 201e p507, 19/e p523; Schwaz 91e p199-200; Bailey 261e p389)The best way to measure the area burned accurately is the Lund and Browder cha.Total body surface area affected in burns of a 2 years old child involving face (8.5%), bilateral upper limb (20%), front of chest & abdomen (13%) is 41.5% (approximately 40%)." "This is a real-world medical entrance exam question, please give the true answer based on the question and selection. Topic name: None, Subject name: Pediatrics","About carey coombs murmur which is false – Select and explain. A. Delayed diastole murmur, B. Seen in rheumatic fever, C. Can be associated with AR, D. Low pitched murmur",C: Can be associated with AR,"Answer: C: Can be associated with AR Explanation: Carey-Coombs Murmur Heard in patients with acute rheumatic fever. Occurs due to inflammation of mitral valve cusps or excessive left atrial blood flow as a consequence of MR. Soft middiastolic murmur usually follows" "This is a real-world medical entrance exam question, please give the true answer based on the question and selection. Topic name: None, Subject name: Pathology","The probable reasons for a high incidence of dental caries in the teenage population relates most directly to: Select and explain. A. Rapid growth, B. Frequency of sucrose intake, C. Negligence in visiting the dentist, D. Carelessness in oral hygiene habits",B: Frequency of sucrose intake,"Answer: B: Frequency of sucrose intake Explanation: None" "This is a real-world medical entrance exam question, please give the true answer based on the question and selection. Topic name: AIIMS 2018, Subject name: Psychiatry","Which of the following is not an adverse effect of Escitalopram? Select and explain. A. Nausea, B. Vivid dreams, C. Anorgasmia, D. Sialorrhoea",D: Sialorrhoea,"Answer: D: Sialorrhoea Explanation: Adverse effects of SSRI's MC: GI side effects- Nausea, Vomiting Dyspepsia, Diarrhea Sexual side effects- Decreased libido, Anorgasmia, Delayed ejaculation Increased sweating Decreased platelet aggregation Vivid dreams Sialorrhea is a very common side effect of Clozapine." "This is a real-world medical entrance exam question, please give the true answer based on the question and selection. Topic name: None, Subject name: Pathology","In a 30 years old female patient with polyahritis, testing reveals nucleolar pattern of ANA staining. What is the likely course of this patient? Select and explain. A. Malar rash, alopecia and renal failure, B. Sclerodactyly, esophageal dysmotility and Raynaud's phenomenon, C. Sjogren's syndrome, D. Painful genital and oral blisters and ulcers","B: Sclerodactyly, esophageal dysmotility and Raynaud's phenomenon","Answer: B: Sclerodactyly, esophageal dysmotility and Raynaud's phenomenon Explanation: Answer- B. Sclerodactyly, esophageal dysmotility and Raynaud's phenomenonNucleolsr ANA positivity is most likely suggestive of systemic sclerosis or scleroderma, which comprises of diffuse or limited disease-CREST syndrome (Calcinosis, Raynaud syndrome, Esophageal dysmotility, Sclerodactyly, Telangiectasia)Antinuclear antibodies are directed against nuclear antigens, grouped into: ( I ) antibodies to DNA (2) antibodies to histones (3) antibodies to non-histone proteins bound to RNA (4) antibodies to nucleolar antigens.Most widely used method for detecting ANAs: Indirect immunoffuorescence (identify antibodies that bind to a variety of nuclear antigens, including DNA, RNA & proteins).Pattern of nuclear fluorescence suggests the type of antibody present in the patient's serum." "This is a real-world medical entrance exam question, please give the true answer based on the question and selection. Topic name: AIIMS 2017, Subject name: Pharmacology","Which of the following drug can decrease the size of prostate? Select and explain. A. Tamsulosin, B. Sildenafil, C. Finasteride, D. Prazosin",C: Finasteride,"Answer: C: Finasteride Explanation: In BPH, urinary obstruction is present because of increased size of prostate(static component). This is increased by spasm of urethra(dynamic component).To dilate urethra; alpha 1A receptor blockers like tamsulosin and silodosin are used. These provide the immediate symptomatic relief by treating the dynamic component.To decrease the size of prostate, 5 alpha reductase inhibitors are used which inhibit the conversion of testosterone to DHT(dihydrotestosterone). Drugs are dutasteride and finasteride. These drugs are given for long duration." "This is a real-world medical entrance exam question, please give the true answer based on the question and selection. Topic name: None, Subject name: Dental","Problem associated with wide divergent roots of primary teeth Select and explain. A. During resorption, apical 1/3rd may remain unresorbed, B. During extraction, fracture of apical 1/3rd may be there, C. Both, D. None",C: Both,"Answer: C: Both Explanation: None" "This is a real-world medical entrance exam question, please give the true answer based on the question and selection. Topic name: None, Subject name: Medicine","Which of the following is not an autoimmune disorder? Select and explain. A. Ulcerative colitis, B. Grave's disease, C. Rheumatoid ahritis, D. SLE",A: Ulcerative colitis,"Answer: A: Ulcerative colitis Explanation: Ans: A. Ulcerative colitis(Ref Harrison 19/e p377, 18/e p2723)Autoimmune Diseases Organ SpecificOrgan Nonspecific (Systemic)Grave's diseaseVitiligoSystemic lupus erythematosusHashimoto's thyroiditisAutoimmune hemolytic anemiaRheumatoid ahritisAutoimmune polyglandular syndromeAutoimmune thrombocytopenic purpuraSystemic necrotizing vasculitisType 1 diabetes mellitusPernicious anemiaGranulomatosis with polyangitisInsulin-resistant diabetes mellitusMyasthenia gravis(Wegener's)Immune-mediated infeilityMultiple sclerosisAntiphospholipid syndromeAutoimmune Addison's diseaseGuillain-Barre syndromeSjogren's syndromPemphigus vulgarisStiff-man syndrome Pemphigus foliaceusAcute rheumatic fever Dermatitis herpetiformisSympathetic ophthalmia Autoimmune alopeciaGoodpasture's syndrome" "This is a real-world medical entrance exam question, please give the true answer based on the question and selection. Topic name: None, Subject name: Dental","During relining procedures fear lies in the alteration of Select and explain. A. Balanced occlusion, B. Centric occlusion, C. Vertical dimension, D. Condylar guidance",C: Vertical dimension,"Answer: C: Vertical dimension Explanation: None" "This is a real-world medical entrance exam question, please give the true answer based on the question and selection. Topic name: None, Subject name: Biochemistry","Klenow fragment lacks the activity of? Select and explain. A. 3'-5' exonuclease, B. 5'-3' exonuclease, C. 5'-3' DNA polymerase, D. 3'-5' DNA polymerase",B: 5'-3' exonuclease,"Answer: B: 5'-3' exonuclease Explanation: Ans: B. 5'-3' exonucleaseDNA polymerase I is not the primary enzyme of replication; instead, it performs a host of clean-up functions during replication, recombination, and repair.When the 5'-3' exonuclease domain is removed, the remaining fragment (Mr 68,000), the large fragment or Klenow fragment, retains the polymerization and proofreading activities.Klenow fragment is a large protein fragment produced when DNA polymerase I from E. coli is enzymatically cleaved by the protease subtilisin." "This is a real-world medical entrance exam question, please give the true answer based on the question and selection. Topic name: AIIMS 2019, Subject name: Anatomy","Lesion of the marked structure affects all EXCEPT (AIIMS May 2019) Select and explain. A. Superior rectus, B. Superior oblique, C. Inferior oblique, D. Medial rectus",B: Superior oblique,"Answer: B: Superior oblique Explanation: Nerve pointed in diagram is oculomotor nerve (cranial nerve III) Extrinsic muscles of the eyeball are innervated by 3 cranial nerves. 3 | lateral rectus innervated by 6th cranial nerve Superior oblique by 4th cranial nerve Remaining muscles by 3rd cranial nerve. Among these 3 cranial nerves, oculomotor nerve is most medial, trochlear is most lateral & abducens pierces dura mater b/w these two at base of cranial cavity. MUSCLE OF EYE ACTION SUPERIOR RECTUS ELEVATION, ADDUCTION, MEDIAL ROTATION OF EYE BALL. LEVATOR PALPEBRAE SUPERIORIS ELEVATION OF UPPER EYELID MEDIAL RECTUS ADDUCTION OF EYEBALL. INFERIOR RECTUS DEPRESSION, ADDUCTION, LATERAL ROTATION OF EYEBALL. INFERIOR OBLIQUE ELEVATION, ABDUCTION, LATERAL ROTATION OF EYEBALL SUPERIOR OBLIQUE DEPRESSION, ABDUCTION, MEDIAL ROTATION OF EYEBALL CILIARIS AND SPHINCTER PUPILLAE CONSTRICTS CILIARY BODY & PUPIL" "This is a real-world medical entrance exam question, please give the true answer based on the question and selection. Topic name: None, Subject name: Surgery","Patient with severe acidosis is treated with Select and explain. A. i.v. NaHCo3, B. Ringers lactate, C. Dextrose, D. None of the above",A: i.v. NaHCo3,"Answer: A: i.v. NaHCo3 Explanation: None" "This is a real-world medical entrance exam question, please give the true answer based on the question and selection. Topic name: None, Subject name: Dental","Diagnodent uses what for detecting caries Select and explain. A. Sound wave, B. Visible light, C. LASER, D. Nanotechnology",C: LASER,"Answer: C: LASER Explanation: None" "This is a real-world medical entrance exam question, please give the true answer based on the question and selection. Topic name: None, Subject name: Social & Preventive Medicine","Under National Health Mission which committee makes plan for village health? Select and explain. A. Panchayat health committee (PHC), B. Village health planning and management committee (VHPMC), C. Village health sanitation and nutrition committee (VHSNC), D. RogiKalyan Samiti",C: Village health sanitation and nutrition committee (VHSNC),"Answer: C: Village health sanitation and nutrition committee (VHSNC) Explanation: Ans: C. Village health sanitation and nutrition committee (VHSNC)Ref: K Park, 23'd ed,, pg. 450-451 and Health Policies and Programmes in India, DK Taneja, I2'h ed., pg. 82VHSNC:Sub-committee or a standing committee of the Gram Panchayat.The VHSNC acts as a platform for convergence between different depaments & committees at village level.The VHSNCs shall be suppoed to develop village health plans to ensure convergent action on social determinants of health, ensure access to health services, especially of the more marginalised sections in the village, and suppo the organization of the Village Health and Nutrition Day." "This is a real-world medical entrance exam question, please give the true answer based on the question and selection. Topic name: None, Subject name: Pathology","Anodontia affects the growth of: Select and explain. A. Maxilla, B. Mandible, C. Alveolar bone, D. Cranium",C: Alveolar bone,"Answer: C: Alveolar bone Explanation: None" "This is a real-world medical entrance exam question, please give the true answer based on the question and selection. Topic name: None, Subject name: Dental","The most mucostatic impression material is Select and explain. A. Thin mix of plaster of paris, B. Zinc oxide eugenol impression paste, C. Free flowing wax, D. Reversible hydrocolloids",A: Thin mix of plaster of paris,"Answer: A: Thin mix of plaster of paris Explanation: Impression plaster is a β-calcium sulfate hemihydrate used at a water/powder ratio of approximately 0.5 to 0.6. Its fluidity makes it suitable for making impressions of soft tissues in the uncompressed state, a characteristic of mucostatic impression material. Rest all options are mucocompressive impression materials. Ref: Phillips 12edition page no 177" "This is a real-world medical entrance exam question, please give the true answer based on the question and selection. Topic name: None, Subject name: Dental","Wrong among following about Irritated mandible Select and explain. A. Anterior mandible is most common site, B. HBO THERAPY success rate is 94 to 100%, C. Mandible is most commonly affected bone in the entire craniofacial skeleton, D. Radiation less than 55 is low risk for implant success",A: Anterior mandible is most common site,"Answer: A: Anterior mandible is most common site Explanation: None" "This is a real-world medical entrance exam question, please give the true answer based on the question and selection. Topic name: None, Subject name: Medicine","Facial nerve injury during forceps delivery in labour is due to: Select and explain. A. Mastoid process is absent at birth, B. Parotid gland in developing stage, C. Beak of forceps engages main trunk of facial Nerve, D. Sublingual hematoma during delivery causes neuropraxia",A: Mastoid process is absent at birth,"Answer: A: Mastoid process is absent at birth Explanation: None" "This is a real-world medical entrance exam question, please give the true answer based on the question and selection. Topic name: None, Subject name: Anaesthesia","A sevoflurane vaporizer can accurately deliver the dose of an anesthetic agent. It resembles it in which of the following propeies? Select and explain. A. Molecular weight, B. Oil gas paition coefficient, C. Blood gas paition coefficient, D. Vapor pressure",D: Vapor pressure,"Answer: D: Vapor pressure Explanation: Ans: D. Vapor pressureVapor pressure is directly propoional to temperature.Increasing temperature will increase the ratio of gas:liquid molecules, thereby increasing vapor pressure.Vapor pressure is independent of atmospheric pressure and is contingent only on the temperature and physical characteristics of the liquid." "This is a real-world medical entrance exam question, please give the true answer based on the question and selection. Topic name: AIIMS 2018, Subject name: Anatomy","Which of the following projects efferent fibers through the marked structure:- Select and explain. A. Hippocampus, B. Mammillary body, C. Caudate nucleus, D. Amygdala",A: Hippocampus,"Answer: A: Hippocampus Explanation: Hippocampus The marked structure is the fornix which is the major output tract for the hippocampus, which is a functional pa of the papez circuit Papez circuit: pa of limbic system associated with memory & emotions." "This is a real-world medical entrance exam question, please give the true answer based on the question and selection. Topic name: AIIMS 2019, Subject name: Physiology","Sequential arrangement question on JVP. Staing with ""a"" wave. A-First hea sound B-T wave C-R wave D-Rapid ejection phase Select and explain. A. A,B,C,D, B. A,C,D,B, C. C,A,D,B, D. D,C,B,A","C: C,A,D,B","Answer: C: C,A,D,B Explanation: Sequential arrangement - JVP waves: - A wave - atrial systole C wave - Iso volumetric contraction V wave - Iso volumetric relaxation Arrangement of ventricular events - Atrial systole - 'A' wave in JVP, | Ventricular systole - 'QRS' complex (ventricular depolarization) | AV valves closes - 1st hea sound | SL valve open - rapid ejection | Ventricular repolarization - T - wave" "This is a real-world medical entrance exam question, please give the true answer based on the question and selection. Topic name: None, Subject name: Dental","Maximum masticatory forces taken up by which fibers: Select and explain. A. Alveolar crest, B. Oblique, C. Horizontal, D. Apical",B: Oblique,"Answer: B: Oblique Explanation: None" "This is a real-world medical entrance exam question, please give the true answer based on the question and selection. Topic name: None, Subject name: Pathology","Most characteristic feature of mandibulofacial dysostosis is Select and explain. A. Normal hearing, B. Normal vision with coloboma, C. Mandibular prognathism, D. Vertical maxillary excess",B: Normal vision with coloboma,"Answer: B: Normal vision with coloboma Explanation: None" "This is a real-world medical entrance exam question, please give the true answer based on the question and selection. Topic name: None, Subject name: Radiology","What is the function of filtration process while taking radiographs? Select and explain. A. Remove the short wavelength photons, B. Remove the portion of long wavelength photons, C. To increase the radiation dose to patient, D. To increase scatter of secondary radiation",B: Remove the portion of long wavelength photons,"Answer: B: Remove the portion of long wavelength photons Explanation: None" "This is a real-world medical entrance exam question, please give the true answer based on the question and selection. Topic name: None, Subject name: Pediatrics","An 8 year old boy complaints of increasing muscle weakness. On examination, his calves are bulky and show muscle tightening. His serum creatine kinase levels are increasing with age. Which of the following is the most likely diagnosis? Select and explain. A. Hereditary sensorimotor neuropathy, B. Myelin deficiency, C. Dystrophin deficiency, D. Congenital myopathy",C: Dystrophin deficiency,"Answer: C: Dystrophin deficiency Explanation: Answer- C. Dystrophin deficiencyMost likely diagnosis is duchenne Muscular Dystrophy due to Dystrophin deficiency." "This is a real-world medical entrance exam question, please give the true answer based on the question and selection. Topic name: None, Subject name: Dental","Transfer of hinge opening axis is compulsory in: Select and explain. A. Partially edentulous, B. Complete edentulous, C. Orthodontics, D. Planning to change vertical dimensions",D: Planning to change vertical dimensions,"Answer: D: Planning to change vertical dimensions Explanation: None" "This is a real-world medical entrance exam question, please give the true answer based on the question and selection. Topic name: None, Subject name: Forensic Medicine","Which of these findings is not specific of blast injury? Select and explain. A. Abrasion, B. Bruise, C. Puncture laceration, D. Fracture",D: Fracture,"Answer: D: Fracture Explanation: Ans: D. Fracture(Ref: Reddy 34/e 1)225-227, 33/e p239-240; Parikh 6/e p4.183; Sabiston 19/e p6I2-613; Bailey 26/e p430 25/e p422-423)Fracture is not specific of blast injury.Marshall's triad:Diagnostic of explosive injury.Includes bruises, abrasions & puncture lacerations." "This is a real-world medical entrance exam question, please give the true answer based on the question and selection. Topic name: None, Subject name: Dental","An investigator want to know the similarity of the mean peak flow of expiratory rates and non-smokers, light smokers, moderate smokers, & heavy smokers. Which is statistical test of significance: Select and explain. A. One way ANOVA, B. Two way ANOVA, C. Student-t test, D. Chi square test",A: One way ANOVA,"Answer: A: One way ANOVA Explanation: None" "This is a real-world medical entrance exam question, please give the true answer based on the question and selection. Topic name: None, Subject name: Dental","Crab claw separator is Select and explain. A. Farrier separator, B. Elliot separator, C. Both, D. None",B: Elliot separator,"Answer: B: Elliot separator Explanation: None" "This is a real-world medical entrance exam question, please give the true answer based on the question and selection. Topic name: None, Subject name: Dental","All are secondary colonizers except Select and explain. A. S. sanguis, B. P. intermedia, C. Fusobacteria, D. P. gingivalis",A: S. sanguis,"Answer: A: S. sanguis Explanation: None" "This is a real-world medical entrance exam question, please give the true answer based on the question and selection. Topic name: None, Subject name: Anatomy","Passavant’s ridge is located on which of the following: Select and explain. A. Posterior wall of oropharynx, B. Anterior wall of oropharynx, C. Posterior wall of nasopharynx, D. Anterior wall of nasopharynx",C: Posterior wall of nasopharynx,"Answer: C: Posterior wall of nasopharynx Explanation: The upper fibres of palatopharyngeus form a spincter internal to superior constrictor. These fibres constitute passavant’s muscle, which on contraction raises passavent’s ridge on the posterior wall of nasopharynx." "This is a real-world medical entrance exam question, please give the true answer based on the question and selection. Topic name: None, Subject name: Dental","Apical foramen is found at root apex in Select and explain. A. 17-46% of cases, B. 5-10 % of cases, C. 10-30% of cases, D. 100% of cases",A: 17-46% of cases,"Answer: A: 17-46% of cases Explanation: None" "This is a real-world medical entrance exam question, please give the true answer based on the question and selection. Topic name: AIIMS 2017, Subject name: Microbiology","1-3 beta - d - glucan assay is done for which infection? Select and explain. A. Invasive candidiasis, B. Cryptococcus, C. Penicillium, D. Rhinicerebral mucormycosis",A: Invasive candidiasis,"Answer: A: Invasive candidiasis Explanation: Beta-d- glucan is an attractive antigen in that it is found in a broad range of fungal agents, including the commonly encountered agents Candida spp., Aspergillus spp., Pneumocystis jirovecii. Cryptococcus- capsular antigen detection from CSF or serum by latex agglutination test is a rapid and sensitive test. Pneumocystis carinii- no serological test available Mucormycosis- Can be diagnosed by KOH mount showing , non-septated hyphae branching at obtuse angle" "This is a real-world medical entrance exam question, please give the true answer based on the question and selection. Topic name: None, Subject name: Physiology","Which of the following is rapid source of energy by resynthesizing ATP for exercising muscles is? Select and explain. A. Glycolysis, B. Glycogenolysis, C. TCA cycle, D. Phosphocreatine",A: Glycolysis,"Answer: A: Glycolysis Explanation: Ans: A. Glycolysis(Ref Guyton 13/e p1087. Ganong 25/e p108, 24/e p106)Phosphocreatine:Rapid source of energy by resynthesizing ATP for exercising muscles.During periods of high activity, cycling of phosphorylcreatine allows for quick release of ATP to sustain muscle activity." "This is a real-world medical entrance exam question, please give the true answer based on the question and selection. Topic name: None, Subject name: Dental","Midazolam cannot be given by which of the following routes: Select and explain. A. Oral, B. Inhalation, C. Intra muscular, D. Intra venous",B: Inhalation,"Answer: B: Inhalation Explanation: None" "This is a real-world medical entrance exam question, please give the true answer based on the question and selection. Topic name: None, Subject name: Pediatrics","All are signs of hydrocephalus in a neonate except –a) Enlarged headb) Sunset signc) Crack post signd) Depressed fontanelle Select and explain. A. ab, B. cd, C. bd, D. ac",B: cd,"Answer: B: cd Explanation: ""Cracked pot or inacewan sign indicates raised intracranial pressure after sutures and fontanels have closed"" Fontanelle is wide and bulging (not depressed) Signs of Increased intracranial tension in infants Separation of cranial sutures —> Earliest sign Wide bulging anterior fontanelle Increased head circumference Papilledema does not occur in infants because separation of sutures and open fontanelle compensate for increase in the intracranial pressure. However, if ICT rises very rapidly papilledema may occur. Macewan or cracked pot sign occurs after sutures and fontanelle have closed (not in neonate) Projectile vomiting Headache -3 Particularly in early morning Diplopia & sixth nerve palsy Sun set sign --> Eyes deviate downward because of impingement of the dilated suprapineal recess." "This is a real-world medical entrance exam question, please give the true answer based on the question and selection. Topic name: AIIMS 2019, Subject name: Psychiatry","About depersonalization, which of the following is false: Select and explain. A. More common in females than male, B. Common in patients with seizure and migraine, C. Common with post life threatening accidents, D. Reality testing is lost",D: Reality testing is lost,"Answer: D: Reality testing is lost Explanation: Depersonalization: Feeling of unreality of self or detachment from self, also called 'as if' phenomenon. Derealization: Feeling of unreality of the world. The reality testing is intact in both the phenomenon (according to DSM-V)" "This is a real-world medical entrance exam question, please give the true answer based on the question and selection. Topic name: None, Subject name: Pharmacology","Antiemetic action is due to which propey of metoclopramide? Select and explain. A. 5-HT3 antagonist, B. D, antagonist, C. 5-HT4 agonist, D. M3 antagonist","B: D, antagonist","Answer: B: D, antagonist Explanation: Ans: B. D, antagonist(Ref: Goodman Gilman 12Ie p1325; Katzung 13/e p1062, 12Ie 0092; KDT 7Ie p665).Metoclopramide:Dopamine D, receptor antagonist.Mixed 5-HT 3 eceptor antagonist & 5-HT, receptor agonist.Antiemetic action due to its antagonist activity at D2 receptors in the chemoreceptor trigger zone in the central nervous system-this action prevents nausea & vomiting triggered by most stimuli.At higher doses, 5-HT, antagonists activity may also contribute to the antiemetic effect." "This is a real-world medical entrance exam question, please give the true answer based on the question and selection. Topic name: None, Subject name: Dental","Epsilon in high copper amalgam is? Select and explain. A. Cu6Sn5, B. Ag3Sn, C. Cu3Sn, D. Ag2Hg3",C: Cu3Sn,"Answer: C: Cu3Sn Explanation: None" "This is a real-world medical entrance exam question, please give the true answer based on the question and selection. Topic name: AIIMS 2017, Subject name: Microbiology","Blood on OT floor is cleaned by? Select and explain. A. Phenol, B. Alcohol based compounds, C. Chlorine based compounds, D. Quaternary ammonium compounds",C: Chlorine based compounds,"Answer: C: Chlorine based compounds Explanation: Sodium hypochlorite (1%) is used as laboratory disinfectant for disinfectingblood and other specimens and is also used for disinfecting the blood spillage area." "This is a real-world medical entrance exam question, please give the true answer based on the question and selection. Topic name: AIIMS 2019, Subject name: Social & Preventive Medicine","Regarding Influenza, not true statement is Select and explain. A. Secondary attack rate 5-15%, B. Virus shedding present before the patient presents with symptoms, C. 1-5 years age is not a high risk age group, D. Aquatic birds are reservoir",C: 1-5 years age is not a high risk age group,"Answer: C: 1-5 years age is not a high risk age group Explanation: - High risk age group of influenza include, - those > 65 years & - those < 5 years" "This is a real-world medical entrance exam question, please give the true answer based on the question and selection. Topic name: None, Subject name: Dental","Which one of the following is the role of barrier membrane in GTR? Select and explain. A. To help overall healing, B. Prevention of epithelial migration, C. To stop bleeding, D. To prevent the underlying tissues from the infection",B: Prevention of epithelial migration,"Answer: B: Prevention of epithelial migration Explanation: None" "This is a real-world medical entrance exam question, please give the true answer based on the question and selection. Topic name: None, Subject name: Dental","Suture technique is called as: Select and explain. A. Simple loop suture., B. Sling suture., C. Figure eight suture., D. Simple sling suture.",C: Figure eight suture.,"Answer: C: Figure eight suture. Explanation: None" "This is a real-world medical entrance exam question, please give the true answer based on the question and selection. Topic name: None, Subject name: Dental","Functional appliances in skeletal class II are indicated when: Select and explain. A. Maxilla is normal, mandible is retrognathic, B. Maxilla is prognathic, C. Mandible is retrognathic in a mature individual, D. When severe crowding is present","A: Maxilla is normal, mandible is retrognathic","Answer: A: Maxilla is normal, mandible is retrognathic Explanation: Moderate to severe skeletal class-II malocclusion division 1 cases due to a short or retrognathic mandible are ideally suited for functional appliance treatment. Class II division 2 type of malocclusions may be treated with functional appliances after correcting the axial inclinations of maxillary anteriors." "This is a real-world medical entrance exam question, please give the true answer based on the question and selection. Topic name: None, Subject name: Social & Preventive Medicine","A healthy center repos 40 to 50 cases in a week in the community. This week there are 48 cases normally. This is called: Select and explain. A. Epidemic, B. Sporadic, C. Endemic, D. Outbreak",C: Endemic,"Answer: C: Endemic Explanation: Ans. C. EndemicRef: Park's Text book Preventive & Social Medicine 24th edn; Page no. 98 EndemicConstant presence of a disease or infectious agent in a defined geographical area.It is the usual or expected frequency of a disease in a population." "This is a real-world medical entrance exam question, please give the true answer based on the question and selection. Topic name: None, Subject name: Dental","Which of the following is a phosphoryn indicating dentin presence Select and explain. A. Nestin, B. Arestin, C. Collagen, D. All of the above",A: Nestin,"Answer: A: Nestin Explanation: None" "This is a real-world medical entrance exam question, please give the true answer based on the question and selection. Topic name: None, Subject name: Anatomy","Cartilage of larynx are developed from: Select and explain. A. 3rd arch cartilage., B. 4th arch cartilage., C. 4&5 arch cartilage., D. 4&6 arch cartilage.",D: 4&6 arch cartilage.,"Answer: D: 4&6 arch cartilage. Explanation: None" "This is a real-world medical entrance exam question, please give the true answer based on the question and selection. Topic name: None, Subject name: Surgery","A 13 year old girl with soft painful swelling on posterior aspect of thigh. Color Doppler showed multiple venous channels without any major arterial feeder. All of the following are liquid embolizing agents except Select and explain. A. Cyanoarylate, B. Sodiumtetradecyl sulphate, C. Polyvinyl alcohol, D. Absolute alcohol",C: Polyvinyl alcohol,"Answer: C: Polyvinyl alcohol Explanation: Polyvinyl alcohol is not a liquid embolizing agent." "This is a real-world medical entrance exam question, please give the true answer based on the question and selection. Topic name: None, Subject name: Social & Preventive Medicine","The mean systolic blood pressure was measured in a sample population of elderly females and came out to be 125 mm Hg with a standard detion of 10. 95 percent of people would have blood pressure above: Select and explain. A. 105 mm Hg, B. 110 mm Hg, C. 115 mm Hg, D. 140 mm Hg",B: 110 mm Hg,"Answer: B: 110 mm Hg Explanation: Ans: B. 110 mm Hg(Ref: Park 24/e p 886)" "This is a real-world medical entrance exam question, please give the true answer based on the question and selection. Topic name: None, Subject name: Dental","Sequence of treatment in patient with caries in relation to 51, 52, 61 & 62: Select and explain. A. History taking-oral care assessment-caries risk assessment-diet counseling-restoration-fluoride application, B. History taking-oral care assessment-caries risk assessment-diet counseling-fluoride application-restoration, C. History taking-caries risk assessment- oral care assessment–diet counseling -restoration-fluoride application, D. History taking-oral care assessment caries risk assessment-restoration-fluoride application diet counseling",B: History taking-oral care assessment-caries risk assessment-diet counseling-fluoride application-restoration,"Answer: B: History taking-oral care assessment-caries risk assessment-diet counseling-fluoride application-restoration Explanation: Sequence of treatment in patient with caries in relation to 51, 52, 61 & 62 is history taking-oral care assessment-caries risk assessment-diet counseling-fluoride application-restoration." "This is a real-world medical entrance exam question, please give the true answer based on the question and selection. Topic name: None, Subject name: Biochemistry","Pernicious anaemia occurs in: Select and explain. A. Vit B1 deficiency, B. Vit B12 deficiency, C. Vit C deficiency, D. Vit D deficiency",B: Vit B12 deficiency,"Answer: B: Vit B12 deficiency Explanation: The most important disease associated with vitamin B12 deficiency is pernicious anemia. lt is characterized  by  low  hemoglobin  levels, decreased  number  of  erythrocytes  and neurological manifestations. One or more of the following causes are attributed to the occurrence of pernicious anemia. Autoimmune destruction of gastric parietal cells that secrete intrinsic factor. In the absence of lF, vitamin B12 cannot be absorbed. Hereditary malabsorption of vitamin B12 Partial or total gastrectomy-these individuals become intrinsic factor deficient. Insufficient production of lF and/or gastric HCl, occasionally seen in older people. Dietary deficiency of B12,  is seen among the  strict  vegetarians of  low  socioeconomic group in the developing countries (lndia, Srilanka etc.). Key Concept:  The most important disease associated with vitamin B12 deficiency is pernicious anemia. Reference: Satyanarayana Biochemistry 3rd  ed page no 155" "This is a real-world medical entrance exam question, please give the true answer based on the question and selection. Topic name: None, Subject name: Medicine","Which of the following is not a characteristic feature of multiple myeloma? Select and explain. A. Increased Ig levels in serum, B. Positive ANA, C. Plasmacytosis, D. M spike on electrophoresis",B: Positive ANA,"Answer: B: Positive ANA Explanation: Ans. B. Positive ANASerum electrophoresis: Abnormal spike in the region of gamma globulin (myeloma spike) is present in 90% of cases.* Bone marrow plasmacytosis in multiple myeloma is characteristically more than 30%.* The immunoglobulin least commonly involved in Multiple Myeloma is IgD.* The immunoglobulin most commonly involved in Multiple Myeloma is IgG. - Low haemoglobin, - high ESR (usually very high), - increased total protein, - A/G ratio reversed, - increased serum calcium, - normal alkaline phosphatase. - Urine: Bence Jones proteins are found in 30% of cases." "This is a real-world medical entrance exam question, please give the true answer based on the question and selection. Topic name: None, Subject name: Dental","Major pattern of accessory canals in mandibular molar is? Select and explain. A. A lateral canal extends from coronal 3rd to furcal region in distal canal., B. Presence of both lateral & furcal canal, C. Single furcation canal extend from pulp chamber to intra-radicular region, D. Variable pattern & depend on age of patient",A: A lateral canal extends from coronal 3rd to furcal region in distal canal.,"Answer: A: A lateral canal extends from coronal 3rd to furcal region in distal canal. Explanation: None" "This is a real-world medical entrance exam question, please give the true answer based on the question and selection. Topic name: None, Subject name: Dental","All are true about enamel rods except Select and explain. A. Length of enamel rods is same as the thickness of enamel, B. Enamel rods are narrow at DEJ and wide at tooth surface., C. Under light microscope seen as Fish scale, D. Tail is more resistant to caries than the core",A: Length of enamel rods is same as the thickness of enamel,"Answer: A: Length of enamel rods is same as the thickness of enamel Explanation: Enamel rods have wavy path, hence length of enamel rod is more than the thickness of the enamel." "This is a real-world medical entrance exam question, please give the true answer based on the question and selection. Topic name: None, Subject name: Anaesthesia","A patient with history of coronary aery disease presents with pulse rate of 48/min and low BP. Patient has decreased myocardial contractility on Echo. Which of these anesthetic agents is contraindicated? Select and explain. A. Fentanyl, B. Etomidate, C. Ketamine, D. Dexmedetomidine",D: Dexmedetomidine,"Answer: D: Dexmedetomidine Explanation: Ans: D. Dexmedetomidine(Ref KDT 7/e p384; Katzung 13/e p I 45, 12/e p445)This case patient has bradycardia & hypotension.Hence, Dexmedetomidine is contraindicated in hypovolemia, hypotension, hea block and congestive hea failure.Dexmedetomidine - Side effects:Similar to those with clonidine (hypotension, bradycardia & dry mouth ).Contraindications:Hypovolemia.HypotensionHea block.CHF prior to administration." "This is a real-world medical entrance exam question, please give the true answer based on the question and selection. Topic name: None, Subject name: Anatomy","Which of the following nerves supplies the ear lobule? Select and explain. A. Greater auricular nerve, B. Lesser occipital nerve, C. Facial nerve d, D. Auriculotemporal nerve",A: Greater auricular nerve,"Answer: A: Greater auricular nerve Explanation: Ans:. A. Greater auricular nerve (Ref Gray's 41/e p627, 40/e p620)Greater auricular nerve supplies ear lobule.Sensory nerves of ear:Great auricular nerve - Supplies most of cranial surface & posterior pa of lateral surface (helix, antihelix, lobule).Lesser occipital nerve - Supplies upper pa of cranial surface.Auricular branch of vagus - Supplies concavity of concha & posterior pa of eminentia.Auriculotemporal nerve - Supplies tragus, crus of helix & adjacent pa of helix.Facial nerve + auricular branch of vagus - Supplies small areas on both aspects of auricle, concha depression & its eminence." "This is a real-world medical entrance exam question, please give the true answer based on the question and selection. Topic name: None, Subject name: Pharmacology","All are true of paracetamol poisoning expect? Select and explain. A. Acetylcysteine in antidote, B. Asymptomatic for 24 to 30 hours, C. 10 mg is safe dose, D. Acetylcysteine block glutathione in paracetamol poisoning",B: Asymptomatic for 24 to 30 hours,"Answer: B: Asymptomatic for 24 to 30 hours Explanation: None" "This is a real-world medical entrance exam question, please give the true answer based on the question and selection. Topic name: None, Subject name: Dental","Theories which define the etiology of thumb sucking: Select and explain. A. Sigmond Freud theory., B. Oral drive theory., C. Benjamin theory of rooting reflex., D. All of the above.",D: All of the above.,"Answer: D: All of the above. Explanation: None" "This is a real-world medical entrance exam question, please give the true answer based on the question and selection. Topic name: None, Subject name: Social & Preventive Medicine","Which of the following is true for ESI act? Select and explain. A. Funeral benefit is up to Rs 50,000, B. The State Government's share of expenditure on medical care is 1/8; the ESI Corporation's share of expenditure on medical care is 7/8 of total cost, C. Person with daily wages of Rs 70 has to contribute Rs 300 towards ESI, D. Employee has to contribute 4.75% and employer contributes 8.75%",B: The State Government's share of expenditure on medical care is 1/8; the ESI Corporation's share of expenditure on medical care is 7/8 of total cost,"Answer: B: The State Government's share of expenditure on medical care is 1/8; the ESI Corporation's share of expenditure on medical care is 7/8 of total cost Explanation: Answer- B. The State Government's share of expenditure on medical care is 1/8; the ESI Corporation's share of expenditure on medical care is 7/8 of total costESI Act: The State Government's share of expenditure on medical care is 1/8; the ESI Corporation's share of expenditure on medical care is 7/8 of total cost.Employees State lnsurance Act (1948):Funeral benefit is up to Rs 5000The State Government's share of expenditure on medical care is 1/8 of total cost of medical care; the ESI Corporation's share of expenditure on medical care is 7/8 of total cost of medical care.Person with daily wages of less Rs.70 per day are exempted from making any paymentsThe employer contributes 4.75% of total wage bill; the employee contributes 1.75% of wages." "This is a real-world medical entrance exam question, please give the true answer based on the question and selection. Topic name: None, Subject name: Pharmacology","Nausea and vomiting postoperatively can be because of all except Select and explain. A. Blood ingested, B. N2O, C. Opioid, D. Acetaminophen",D: Acetaminophen,"Answer: D: Acetaminophen Explanation: None" "This is a real-world medical entrance exam question, please give the true answer based on the question and selection. Topic name: None, Subject name: Microbiology","Disease caused by Staphylococcus aureus which is not mediated through a toxin is Select and explain. A. Food poisoning, B. Septicemic shock, C. Toxic shock syndrome, D. Staphylococcal scalded skin syndrome",B: Septicemic shock,"Answer: B: Septicemic shock Explanation: Answer- B. Septicemic shockSepticemia is mainly due to endotoxin like activity as seen in Gram-negative bacilli and not due to exotoxin. Clinically, Staphylococcus aureus sepsis presents like that documented for sepsis due to other bacteria. Septicemic shock is not directly caused due to toxins and other virulence factors play a role like protein A, fibrinolysin and coagulase, which are anti-phagocytic, suppress host immunity and helps in the spread of infection. Septicemia is mainly due to endotoxin tike activity as seen in Gram-negative bacilli and not due to exotoxin. S.aureus produces free types of toxin: cytotoxins, pyrogenic toxin superantigens, and exfoliative toxins. Both epidemiologic data and studies in animals suggest that antitoxin antibodies are protective against illness in TSS' staphylococcal food poisoning, and staphylococcal scalded skin syndrome (SSSS). Illness develops after toxin synthesis and absorption and the subsequent toxin-initiated host response" "This is a real-world medical entrance exam question, please give the true answer based on the question and selection. Topic name: None, Subject name: Medicine","A patient had a femur fracture for which internal fixation was done. Two days later, the patient developed sudden onset shoness of breath with low-grade fever. What is the likely cause? Select and explain. A. Pneumothorax, B. Fat embolism, C. Pleural effusion, D. Congestive hea failure",B: Fat embolism,"Answer: B: Fat embolism Explanation: Answer- B. Fat embolism""Fat embolism is a common phenomenon following limb fractures. Circulating fat globules larger than 10 pm in diameter occur in most edults after closed fractures of long bones and histological traces of fat can be found in the lungs and other internal organs." "This is a real-world medical entrance exam question, please give the true answer based on the question and selection. Topic name: None, Subject name: Physiology","Localization of the receptor of a hormone Xis found to be in the nucleus. What is likely to be X? Select and explain. A. Adrenaline, B. Insulin, C. Thyroxine, D. FSH",C: Thyroxine,"Answer: C: Thyroxine Explanation: Ans: C. ThyroxineRef Ganong 25/e p300, 24Ie p300; Harper 30Ie p501).Thyroxine is a lipophilic hormone that acts on nuclear receptor.Steroids and thyroid hormones are distinguished by their predominantly intracellular sites of action, since they can diffuse freely through the cell membrane.They bind to a family of largely cytoplasmic proteins known as nuclear receptors.Upon ligand binding, the receptor-ligand complex translocates to the nucleus where it either homodimerizes, or associates with a distinct liganded nuclear receptor to form a heterodimer.In either case, the dieter binds to DNA to either increase or decrease gene transcription in the target tissue." "This is a real-world medical entrance exam question, please give the true answer based on the question and selection. Topic name: None, Subject name: Physiology","Endo-cochlear potential is: Select and explain. A. +45 mV, B. --45mV, C. -60mV, D. +85 mV",D: +85 mV,"Answer: D: +85 mV Explanation: Ans: D.+85 mV(Ref Ganong 25/e p204, 24/e p203; Guyton I 3/e p677)Endocochlear potential:Cells in stria vascularis have a high concentration of Na, K ATPase.Exists all time between endolymph & perilymph.Positivity inside scala media & negativity outside.This is called endocochlear potential.Generated by continual secretion of positive potassium ions into scala media by stria vascularis.Normal: +85 mV" "This is a real-world medical entrance exam question, please give the true answer based on the question and selection. Topic name: None, Subject name: Social & Preventive Medicine","Statistical `Q' test is used for: Select and explain. A. Comparing the propoion of means of 2 groups, B. To determine outliars, C. To determine normality distribution, D. Comparing the propoion of means of more than 2 groups",B: To determine outliars,"Answer: B: To determine outliars Explanation: Answer- B. To determine outliarsDixon's Q test, or simply the Q test, is used for identification and rejection of outliers.In statistics, an outlier is an observation point that is distant from other observations.An outlier may be due to variability in the measurement or it may indicate experimental errorOutliers can occur by chance in any distribution, but they are often indicative either of measurement error or that the population has a heavy-tailed distribution.This test should be applied sparingly and never more than once to a single data set." "This is a real-world medical entrance exam question, please give the true answer based on the question and selection. Topic name: None, Subject name: Pharmacology","Drug acting on cell wall of gram positive bacteria:March 2009 Select and explain. A. Gentamycin, B. Ciprofloxacin, C. Tetracycline, D. Vancomycin",D: Vancomycin,"Answer: D: Vancomycin Explanation: Ans. D: VancomycinVancomycins bind to the peptides of the peptidoglycan monomers and block both the formation of gycosidic bonds between the sugars by the transgycosidase enzymes and the formation of the peptide cross-links by the transpeptidase enzymes. This results in a weak cell wall and osmotic lysis of the bacterium.The fluoroquinolones (norfloxacin, lomefloxacin, fleroxacin, ciprofloxacin, enoxacin, trovafloxacin, gatifloxacin, etc.) work by inhibiting one or more of a group of enzymes called topoisomerase, enzymes needed for supercoiling, replication and separation of circular bacterial DNA.For example, DNA gyrase is a topoisomerase that catalyzes the negative supercoiling of the circular DNA found in bacteria. Topoisomerase IV, on the other hand, is involved in the relaxation of the supercoiled circular DNA, enabling the separation of the interlinked daughter chromosomes at the end of bacterial DNA replication.In gram-positive bacteria, the main target for fluoroquinolones is DNA gyrase (topoisomerase II), an enzyme responsible for supercoiling of bacterial DNA during DNA replication; in gram-negative bacteria, the primary target is topoisomerase IV, an enzyme responsible for relaxation of supercoiled circular DNA and separation of the inter-linked daughter chromosomes.The tetracyclines (tetracycline, doxycycline, demeclocycline, minocycline, etc.) block bacterial translation by binding reversibly to the 30S subunit and distoing it in such a way that the anticodons of the charged tRNAs cannot align properly with the codons of the mRNA.The aminoglycosides (streptomycin, neomycin, netilmicin, tobramycin, gentamicin, amikacin, etc.) bind irreversibly to the 30S subunit of bacterial ribosomes." "This is a real-world medical entrance exam question, please give the true answer based on the question and selection. Topic name: None, Subject name: Biochemistry","Typical lipoprotein contains Select and explain. A. Free cholesterol + Phospholipid at periphery, B. Cholesteryl ester + FA at center, C. Proteins as apoproteins, D. All of the above",D: All of the above,"Answer: D: All of the above Explanation: None" "This is a real-world medical entrance exam question, please give the true answer based on the question and selection. Topic name: None, Subject name: Surgery","Treatment of choice in a patient with a staghorn calculus with mild hydronephrosis? Select and explain. A. ESWL, B. PCNL, C. RIRS, D. Open surgery",B: PCNL,"Answer: B: PCNL Explanation: Answer- B. PCNLPCNL is the best answer as the patient has hydronephrosis and in an obstructed system ESWL is not useful as stone fragments don't get cleared." "This is a real-world medical entrance exam question, please give the true answer based on the question and selection. Topic name: None, Subject name: Dental","Pericytes are found: Select and explain. A. In thymus, B. Around capillaries of pulp, C. In gallbladder, D. Along with heparin on surface of mast cells",B: Around capillaries of pulp,"Answer: B: Around capillaries of pulp Explanation: None" "This is a real-world medical entrance exam question, please give the true answer based on the question and selection. Topic name: None, Subject name: Medicine","Which of the following Slatements is not true? Select and explain. A. Parathyroid hormone-related protein is responsible for causing hypercalcemia in cancer patients, B. The unionized fraction of calcium in the plasma is an impoant determinant of PTH secretion, C. Mg2+ influences PTH secretion in the same direction as Ca2+ but is a less potent secretagogue, D. Ca2+influences PTH secretion by acting on a calcium sensor G-protein coupled receptor located in the parathyroid gland",B: The unionized fraction of calcium in the plasma is an impoant determinant of PTH secretion,"Answer: B: The unionized fraction of calcium in the plasma is an impoant determinant of PTH secretion Explanation: Ans. b. The unionized fraction of calcium in the plasma is an impoant determinant of PTH secretion""Circulating ionized calcium acts directly on the parathyroid glands in a negative feedback fashion to regulate the secretion of PTH.""Magnesium is required to maintain normal parathyroid secretory responses. Impaired PTH release along with diminished target responses to PTH account fir the hypocalcaemia that occasionally occurs in magnesium deficiency" "This is a real-world medical entrance exam question, please give the true answer based on the question and selection. Topic name: None, Subject name: Pharmacology","A 35-year-old male is put on thiazides for the treatment of primary hypeension. What would be the status of his urinary sodium, potassium and calcium in the first 24-hours of drug administration? Select and explain. A. Sodium and potassium increases, calcium increases, B. Sodium and potassium decreases, calcium decreases, C. Sodium and calcium increases, potassium decreases, D. Potassium and calcium increases sodium decreases","A: Sodium and potassium increases, calcium increases","Answer: A: Sodium and potassium increases, calcium increases Explanation: Answer: a. Sodium and potassium increases, calcium increases (1(e j..Ciuodnian unman iLe pod,* Urinary sodium, potassium and calcium increase in the first 24 hours of thiazide administration.* The acute effects of thiazide on Ca"" excretion is variable:* It has been seen in various trials that urinary excretion increases in the initial stages of thiazide diuretic therapy. When administered chronically, thiazide diuretics decrease Ca"" excretion." "This is a real-world medical entrance exam question, please give the true answer based on the question and selection. Topic name: None, Subject name: Dental","Cytokines associated with periapical lesions Select and explain. A. IL-1, B. IL-6, C. IL-8, D. TNF-α",A: IL-1,"Answer: A: IL-1 Explanation: None" "This is a real-world medical entrance exam question, please give the true answer based on the question and selection. Topic name: None, Subject name: Physiology","Rapid axonal flow in the neurons is mediated by all except: Select and explain. A. Dynein, B. Kinesin, C. Neurofilaments, D. Microtubules",C: Neurofilaments,"Answer: C: Neurofilaments Explanation: None" "This is a real-world medical entrance exam question, please give the true answer based on the question and selection. Topic name: None, Subject name: Surgery","A 49 years old male with 35 pack years presented with painless mass in left scrotal sac and microscopic hematuria. On laboratory investigation, Alphafetoprotein and lactate dehydrogenase was negative. What is the most probable diagnosis? Select and explain. A. Epididymitis, B. Seminoma, C. Renal cell carcinoma, D. Carcinoma lung",C: Renal cell carcinoma,"Answer: C: Renal cell carcinoma Explanation: Answer- C (Renal cell carcinoma)MC type of RCC, mainly sporadica.Spread:Characteristic feature of RCC is tendency to invade renal vein. Fuher extension produces a continuous cord of tumor in IVC and even in right side of hea.MC route is hematogenousMC sites of distant metastasis are lungs (cannon ball deposits and pulsating secondaries) Q> bone> liver> brain.Lymphatic spread occurs when tumor extends beyond renal capsule.Response to biological response modifiers (IL-2 and lFN-alpha)MC and consistent presentation is hematuriaClassical triad of gross hematuria, abdominal mass and pain is seen in 10% cases" "This is a real-world medical entrance exam question, please give the true answer based on the question and selection. Topic name: None, Subject name: Dental","Severe alveolar bone loss, as observed in juvenile periodontitis is associated with: Select and explain. A. Cyclic neutropenia, B. Lysis of neutrophils, C. Increased phagocytosis, D. Neutrophil chemotactic defects or Impaired neutrophil chemotaxis",D: Neutrophil chemotactic defects or Impaired neutrophil chemotaxis,"Answer: D: Neutrophil chemotactic defects or Impaired neutrophil chemotaxis Explanation: None" "This is a real-world medical entrance exam question, please give the true answer based on the question and selection. Topic name: None, Subject name: Forensic Medicine","A 14 years old rape victim with 22 weeks of gestation coming to hospital. All of the following can be done except: Select and explain. A. Male doctor can examine her with female attendant, B. Gynecologist can abo the fetus upon the patient request, C. No need to collect vaginal swab, D. UPT not required",B: Gynecologist can abo the fetus upon the patient request,"Answer: B: Gynecologist can abo the fetus upon the patient request Explanation: Ans. b. Gynecologist can abo the fetus upon the patient requestConsent of woman is required before conducting aboion. Written consent of guardian is required if the woman is a minor (<18 years) or mentally ill person. In the question, age of patient is 14 years only, so the consent of the guardian is required for termination. Termination is permitted upto 20 weeks of pregnancy only. So, gynecologist cannot abo the fetus upon the patient request." "This is a real-world medical entrance exam question, please give the true answer based on the question and selection. Topic name: None, Subject name: Physiology","Which of the following site doesn't contain brown adipose tissues? Select and explain. A. Scapula, B. Subcutaneous tissue, C. Around blood vessel, D. Around adrenal coex",A: Scapula,"Answer: A: Scapula Explanation: Ans: A. ScapulaBrown fat:More abundant in infants but also in adults.Location:Between scapulas, at nape of neck, along great vessels in thorax and abdomen & in other scattered locations in body." "This is a real-world medical entrance exam question, please give the true answer based on the question and selection. Topic name: None, Subject name: Dental","All are true about RVG, except: Select and explain. A. 80% reduction of patient exposure, B. Instant imaging, C. Easy to storage and retrieval, D. Image is sharper than cause by halogen halide",D: Image is sharper than cause by halogen halide,"Answer: D: Image is sharper than cause by halogen halide Explanation: None" "This is a real-world medical entrance exam question, please give the true answer based on the question and selection. Topic name: None, Subject name: Dental","Modulus of elasticity of which cement is best to support under complex amalgam restoration? Select and explain. A. GIC, B. ZnPO4, C. ZOE, D. Polycarboxylate",B: ZnPO4,"Answer: B: ZnPO4 Explanation: None" "This is a real-world medical entrance exam question, please give the true answer based on the question and selection. Topic name: None, Subject name: Dental","Hyalinization in orthodontic movement means: Select and explain. A. A cell free zone histologically similar to hyaline appearance, B. Change to hyaline cartilage, C. Lamina dura converts into hyaline cartilage, D. Periodontal Ligament changes to hyaline cartilage",A: A cell free zone histologically similar to hyaline appearance,"Answer: A: A cell free zone histologically similar to hyaline appearance Explanation: None" "This is a real-world medical entrance exam question, please give the true answer based on the question and selection. Topic name: None, Subject name: Pharmacology","Which of the following drugs is least efficacious in the treatment of temporal lobe epilepsy? Select and explain. A. Phenobarbitone, B. Phenytoin sodium, C. Primidone, D. Carbamazepine",C: Primidone,"Answer: C: Primidone Explanation: Complex partial seizures (CPS, temporal lobe epilepsy, psychomotor): attacks of bizarre and confused behaviour dream-like state and purposeless movements, or even walking, unaware emotional changes lasting 1-2 min along with impairment of consciousness. The patient has no recollection of the attack. An aura often precedes. The seizure focus is located in the temporal lobe.​ Phenobarbitone was the first efficacious anti-epileptic introduced in 1912.​ Primidone A deoxybarbiturate, which is converted by liver to phenobarbitone and phenylethyl malonamide (PEMA). Its antiepileptic activity is mainly due to these active metabolites because t1/2 of primidone (6-14 hr) is less than that of its active metabolites. About 1/3 primidone is excreted unchanged by kidney. Dose to dose primidone is less potent, but antiepileptic efficacy is similar to phenobarbitone. It is seldom used now in GTGC and partial epilepsy, mainly as an adjuvant to phenytoin or carbamazepine. Adverse effects are similar to phenobarbitone. In addition, anemia, leukopenia, psychotic reaction and lymph node enlargement occur rarely. Dose: Start with 250 mg OD, then 250-500 mg BD, children 10-20 mg/kg/day. MYSOLINE 250 mg tab. Reference: Essentials of Medical Pharmacology Eighth Edition KD TRIPATHI page no 438,443​" "This is a real-world medical entrance exam question, please give the true answer based on the question and selection. Topic name: None, Subject name: Medicine","Serology profile done for a patient is mentioned below. What is the likely cause of the abnormal findings:HbsAg-Non-reactiveHBV DNA-UndetectableHbeAg-Non-reactiveIgG Anti-HbC-Reactive Select and explain. A. Chronic hepatitis inactive state, B. Chronic hepatitis recovery state, C. Pre-core mutant infection, D. Window period",B: Chronic hepatitis recovery state,"Answer: B: Chronic hepatitis recovery state Explanation: Ans: B. Chronic hepatitis recovery stateFrom the given serology profile and table given below it is clear that the patient has had hepatitis B infection in the remote past or is a low-level Hepatitis B carrier.Commonly Encountered Serologic Patterns of Hepatitis B InfectionHBsAgAnti-HBsAnti-HBcHBeAgAnti-HBeInterpretation+-IgM+-Acute hepatitis B, high infectivity+-IgG+-Chronic hepatitis B, high infectivitydeg+-IgG-+Late acute or chronic hepatitis B, low infectivitydegHBeAg-negative ('precoremutant') hepatitis B (chronic or rarely acute)++++1-+1-HBsAg of one subtype and heterotypic anti-HBs (common)Process of seroconversion from HBsAg to anti-HBs (rare)--IgM+/-+/-Acute hepatitis BdegAnti-HBc `window'0--IgG-+/-Low-level hepatitis B carrierdegHepatitis B in remote pastdeg-+IgG-+/-Recovery from hepatitis Bdeg-+---Immunization with HBsAg (after vaccination)Hepatitis B in the remote past (?)False-positive" "This is a real-world medical entrance exam question, please give the true answer based on the question and selection. Topic name: None, Subject name: Microbiology","Beta-1,3-D Glucan testing is done for all except- Select and explain. A. Aspergillosis, B. Mucormycosis, C. Candidiasis, D. Pneumocystis",B: Mucormycosis,"Answer: B: Mucormycosis Explanation: Ans- B. Mucormycosis* Detection of fungal antigens in body fluids, including cryptococcus capsular polysaccharide, histoplasma antigen, galactomannan, and b-d-glucan, is viewed as being clinically useful for at least the presumptive diagnosis of invasive fungal infections.b-d-Glucan is an attractive antigen in that it is found in a broad range of fungal agents, including the commonly encountered agents Candida spp., Aspergillus spp., and Pneumocystis jirovecii." "This is a real-world medical entrance exam question, please give the true answer based on the question and selection. Topic name: None, Subject name: Biochemistry","Maximum thermic effect of food is seen with Select and explain. A. Carbohydrates, B. Protein, C. Fat, D. Not dependent on macronutrients",B: Protein,"Answer: B: Protein Explanation: Answer- B. ProteinMaximum thermic effect off is seen with proteins (20-30%) >carbohydrates (5-6%) > fat (2.5-4%)." "This is a real-world medical entrance exam question, please give the true answer based on the question and selection. Topic name: None, Subject name: Dental","Latest technique by which bone graft is obtained without damaging vital structure, vessels/veins is based on? Select and explain. A. Piezoelectric vibration, B. Osteotome, C. Ultrasonic/soft lasers, D. Electrocautery",A: Piezoelectric vibration,"Answer: A: Piezoelectric vibration Explanation: None" "This is a real-world medical entrance exam question, please give the true answer based on the question and selection. Topic name: None, Subject name: Dental","Enlarged tonsils are only seen in: Select and explain. A. Infantile swallow., B. Mature swallow., C. Simple tongue thrust., D. Complex tongue thrust.",D: Complex tongue thrust.,"Answer: D: Complex tongue thrust. Explanation: None" "This is a real-world medical entrance exam question, please give the true answer based on the question and selection. Topic name: AIIMS 2019, Subject name: ENT","Which of the following is not a cause of Squamous cell Carcinoma for Head & Neck: Select and explain. A. EBV, B. HPV, C. Betel Nut, D. Vitamin A",D: Vitamin A,"Answer: D: Vitamin A Explanation: Risk factors for head and neck cancer Tobacco Alcohol Prolonged sun exposure Viruses (HPV) & (EBV) Gender M>F, age > 40yrs Poor oral / dental hygiene; betel nut consumption Environmental or occupational factors Poor nutrition - vitamin A & B deficiency, iron deficiency In question they have mentioned vitamin A NOT vitamin A deficiency" "This is a real-world medical entrance exam question, please give the true answer based on the question and selection. Topic name: AIIMS 2018, Subject name: Microbiology","Episodes of repeated thin stools with mucus, subjective feeling of fever and lower abdominal pain, with leukocytes in stool. Which of the following is likely? Select and explain. A. Giardia, B. Entamoeba, C. Staph, D. Clostridium perfringens",B: Entamoeba,"Answer: B: Entamoeba Explanation: As leucocytes are present in stools so it is inflammatory type of diarrhea. So the likely organism is Entamoeba As Staph and C. perfringens are responsible for water diarrhea (non-inflammatory diarrhea) Giardia cause malabsorption syndrome." "This is a real-world medical entrance exam question, please give the true answer based on the question and selection. Topic name: None, Subject name: Dental","Permanent bending of an orthodontic wire during its clinical application will leads to work hardening of the alloy, this might cause: Select and explain. A. Increase in strength, B. Decrease in ductility, C. Development of internal stresses, D. All of the above",D: All of the above,"Answer: D: All of the above Explanation: None" "This is a real-world medical entrance exam question, please give the true answer based on the question and selection. Topic name: None, Subject name: Ophthalmology","Which of the following is true about degenerative myopia? Select and explain. A. More common in males as compared to females, B. Myopic degeneration can lead to retinal detachment, C. It is seen in less than 6 dioptres of myopia., D. The condition has no racial prediliction.",B: Myopic degeneration can lead to retinal detachment,"Answer: B: Myopic degeneration can lead to retinal detachment Explanation: Ans. b. Myopic degeneration can lead to retinal detachmentDegenerative myopia: ? It is seen in more than 6 dioptres of myopia.? More common in females as compared to males.? The condition has a racial predilection; it is more common in Jews and Japenese people, and most cases are of genetic origin.Patients with degenerative myopia typically complain of decreased vision, headaches, and sensitivity to light. If retinal degeneration or detachment is present, patients may also repo light flashes and floaters, which are associated with retina changes. Those with degenerative myopia have an increased incidence of cataract formation (nuclear cataracts are most typical).Some of the most typical features of degenerative myopia are:Vitreous liquefaction and posterior vitreous detachmentPeripapillary atrophy appearing as temporal choroidal or scleral crescents or rings around the optic discLattice degeneration in the peripheral retinaTilting or malinseion of the optic disc, usually associated with myopic conusThinning of the retinal pigment epithelium with resulting atrophic appearance of the fundusEctasia of the sclera posteriorly (posterior staphyloma)Breaks in Bruch's membrane and choriocapillaris, resulting in lines across the fundus called ""lacquer cracks""Fuchs spot in the macular area." "This is a real-world medical entrance exam question, please give the true answer based on the question and selection. Topic name: None, Subject name: Pharmacology","Metronidazole: Select and explain. A. Has no side-effects, B. Is used in management of ANUG, C. Is mainly concentrated in saliva, D. Is active against gram-positive aerobes",B: Is used in management of ANUG,"Answer: B: Is used in management of ANUG Explanation: None" "This is a real-world medical entrance exam question, please give the true answer based on the question and selection. Topic name: None, Subject name: Dental","Growth of nasal bone is completed by: Select and explain. A. 10 years, B. 11 years, C. 13 years, D. 7 years",A: 10 years,"Answer: A: 10 years Explanation: Growth of the nasal bone is complete at about age 10. Growth thereafter is only of the nasal cartilage and soft tissues, both of which undergo a considerable adolescent spurt. The result is that the nose becomes much more prominent at adolescence, specially in boys. Ref: Proffit 6th ed page no 37" "This is a real-world medical entrance exam question, please give the true answer based on the question and selection. Topic name: None, Subject name: Anatomy","The typical cervical differs from thoracic vertebra in that it Select and explain. A. Has a triangular body, B. Has a foramen transversarium, C. Superior articular facet directed backwards and upwards, D. Has a large vertebral body",B: Has a foramen transversarium,"Answer: B: Has a foramen transversarium Explanation: The cervical vertebrae are identified by the presence of foramen transversarium in the transverse process. This is an unique identification feature of cervical vertebrae, and is not present in thoracic and lumbar vertebra." "This is a real-world medical entrance exam question, please give the true answer based on the question and selection. Topic name: None, Subject name: Dental","Phosphate bonded investment should be completely carbon free, otherwise: Select and explain. A. Investment chips off when molten metal enters the mold space, B. Black castings, C. Carbon makes the casting brittle, D. Smooth casting",C: Carbon makes the casting brittle,"Answer: C: Carbon makes the casting brittle Explanation: Carbon is often added to the powder to produce clean castings and facilitate the divesting of the casting from the investment mold. This addition is appropriate when the casting alloy is gold, but there is disagreement regarding the effects of carbon in phosphate investments used for casting silver-palladium alloys, palladium-silver alloys, or base metal alloys. It is believed that carbon can embrittle the alloys, even though the investment is heated to temperatures that burn out the carbon. Some evidence indicates that palladium reacts with carbon at temperatures above 1504 °C. Thus, if the casting temperature of an alloy with high palladium content exceeds this critical point, a phosphate investment without carbon should be used. Ref: Phillip’s 12th edition page 208" "This is a real-world medical entrance exam question, please give the true answer based on the question and selection. Topic name: None, Subject name: Radiology","In cephalometric radiography, the distance between the subject and the source of X-ray is Select and explain. A. 2 feet, B. 48 inches, C. 4.8 metres, D. 5 feet",D: 5 feet,"Answer: D: 5 feet Explanation: By convention, the distance from the X-ray source to the subject's midsagittal plane is 5 feet. Contemporary Orthodontics, William Proffit, 6th ed, pg-174" "This is a real-world medical entrance exam question, please give the true answer based on the question and selection. Topic name: None, Subject name: Dental","Most atypical (unique) tooth Select and explain. A. Primary lower 1st molar, B. Primary lower 2nd molar, C. Primary upper 1st molar, D. Primary upper 2nd molar",A: Primary lower 1st molar,"Answer: A: Primary lower 1st molar Explanation: None" "This is a real-world medical entrance exam question, please give the true answer based on the question and selection. Topic name: None, Subject name: Ophthalmology","Capillary microaneurysms is an earliest sign of:March 2013 Select and explain. A. Vitreous hemorrhage, B. Non-proliferative diabetic retinopathy, C. Trauma, D. Hypeensive retinopathy",B: Non-proliferative diabetic retinopathy,"Answer: B: Non-proliferative diabetic retinopathy Explanation: Ans. B i.e. Non-proliferative diabetic retinopathyDiabetic retinopathyFundus examination in DM:- NIDDM: As early as possible- IDDM: 5 years after diagnosis of DMIncidence of diabetic retinopathy increases with disease duration" "This is a real-world medical entrance exam question, please give the true answer based on the question and selection. Topic name: AIIMS 2019, Subject name: Microbiology","Which of the following marker persists in chronic hepatitis and recurrent hepatitis? Select and explain. A. IgG Anti HbcAg, B. HBsAg, C. IgG Anti HBsAG, D. Anti Hbs",A: IgG Anti HbcAg,"Answer: A: IgG Anti HbcAg Explanation: Interpretation of common serological patterns in HBV infection Virus / Antibody markers Interpretation HBsAg HBeAg Anti - HBc Anti - HBs Anti - HBe + + IgM - - Acute HBV infection, highly infectious + + IgG - - Late/chronic HBV infection or carrier state: highly infectious + - IgG - +/- Late /chronic HBV infection or carrier state; low infectivity - +/- IgM - +/- Seen rarely in early acute HBV infection; infectious - - IgG +/- +/- Remote HBV infection; infectivity nil or very low - - - + - Immunity following HBV vaccine" "This is a real-world medical entrance exam question, please give the true answer based on the question and selection. Topic name: None, Subject name: Medicine","A 22-year-old man presents with diarrhea and intolerance to dairy products, on investigation he was found to have lactase deficiency. Agent least likely to cause lactose intolerance among these is? Select and explain. A. Condensed MILK, B. Skimmed Milk, C. Yoghu, D. Ice cream",D: Ice cream,"Answer: D: Ice cream Explanation: Ans: D. Ice cream" "This is a real-world medical entrance exam question, please give the true answer based on the question and selection. Topic name: None, Subject name: Dental","Object permanence seen in which stage of Jean Piaget’s theory Select and explain. A. Sensorimotor, B. Pre-operational stage, C. Concrete operations stage, D. Formal operations stage",A: Sensorimotor,"Answer: A: Sensorimotor Explanation: None" "This is a real-world medical entrance exam question, please give the true answer based on the question and selection. Topic name: AIIMS 2019, Subject name: Ophthalmology","A patient with hypeension and diabetes presents with blurred vision. Fluorescein angiography shows Select and explain. A. Macular edema, B. Sub macular edema, C. Papilledema, D. Pre macular hemorrhage",A: Macular edema,"Answer: A: Macular edema Explanation: M/C cause of loss of vision in a diabetic = CYSTOID MACULAR EDEMA. PETALLOID APPERENCE" "This is a real-world medical entrance exam question, please give the true answer based on the question and selection. Topic name: None, Subject name: Pathology","Which of the following malignancy is least commonly associated with lymphatic spread ? Select and explain. A. Basal Cell Carcinoma, B. Squamous cell Carcinoma, C. Malignant melanoma, D. Merkel cell Carcinoma",A: Basal Cell Carcinoma,"Answer: A: Basal Cell Carcinoma Explanation: Ans. A. Basal Cell CarcinomaRef: Robbins & Cotran, 9'' ed., pg. I155-l I 57Basal cell carcinoma is a locally aggressive cutaneous tumor.The rest of three tumors in the options have more tendency for lymphatic spread compared to BCC." "This is a real-world medical entrance exam question, please give the true answer based on the question and selection. Topic name: None, Subject name: Surgery","While extracting a mandibular third molar, it is noted that the distal root is missing. The root tip is most likely in the: Select and explain. A. Submental space, B. Submandibular space, C. Para pharyngeal space, D. Pterygomandibular space",B: Submandibular space,"Answer: B: Submandibular space Explanation: None" "This is a real-world medical entrance exam question, please give the true answer based on the question and selection. Topic name: None, Subject name: Biochemistry","Enzyme regulating the conversion of ethanol to acetaldehyde: Select and explain. A. Alcohol dehydrogenase, B. Acetaldehyde dehydrogenase, C. Catalase, D. Enolase",A: Alcohol dehydrogenase,"Answer: A: Alcohol dehydrogenase Explanation: None" "This is a real-world medical entrance exam question, please give the true answer based on the question and selection. Topic name: None, Subject name: Dental","Caries status in child is most likely suggested by Select and explain. A. DMFT and past caries experience, B. Salivary flow and composition, C. Frequency of sugar intake, D. Fluoride concentration in the area",A: DMFT and past caries experience,"Answer: A: DMFT and past caries experience Explanation: None" "This is a real-world medical entrance exam question, please give the true answer based on the question and selection. Topic name: None, Subject name: Dental","Bar-clasp assembly mostly used is: Select and explain. A. Mesio-occlusal rest, B. Disto-occlusal rest, C. Buccal approach, D. None of the above",A: Mesio-occlusal rest,"Answer: A: Mesio-occlusal rest Explanation: None" "This is a real-world medical entrance exam question, please give the true answer based on the question and selection. Topic name: None, Subject name: Pathology","Large anterior fontanelles, open sutures, slanting eyes, decreased sexual development, macroglossia and enamel hypoplasia are seen in: Select and explain. A. Craniofacial dysostosis, B. Down's syndrome, C. Treacher Collins syndrome, D. Marfan's syndrome",B: Down's syndrome,"Answer: B: Down's syndrome Explanation: None" "This is a real-world medical entrance exam question, please give the true answer based on the question and selection. Topic name: None, Subject name: Dental","The following appliance is used in: Select and explain. A. Space regaining, B. Maxillary arch expansion, C. Distalization of molar, D. None of the above",B: Maxillary arch expansion,"Answer: B: Maxillary arch expansion Explanation: This is a HYRAX appliance used for maxillary arch expansion. Several methods have been developed for RME, such as bonded Hyrax developed by McNamara and Brudon (a Hyrax with an acrylic splint bounded to the occlusal surface of posterior teeth) to minimize the vertical effects of conventional expansion appliances." "This is a real-world medical entrance exam question, please give the true answer based on the question and selection. Topic name: None, Subject name: Dental","Folic acid deficiency is precipitated by: Select and explain. A. Aspirin, B. Ranitidine, C. Cyclosporin, D. Phenytoin",D: Phenytoin,"Answer: D: Phenytoin Explanation: None" "This is a real-world medical entrance exam question, please give the true answer based on the question and selection. Topic name: None, Subject name: Physiology","Group 2 sensory fibres are attached to: Select and explain. A. Annulospiral ending, B. Golgi tendon, C. Flower spray ending, D. Pacinian corpuscle",C: Flower spray ending,"Answer: C: Flower spray ending Explanation: Numerical classification of sensory nerve fibers:" "This is a real-world medical entrance exam question, please give the true answer based on the question and selection. Topic name: None, Subject name: Anatomy","Lamina cribrosa is a modification of : Select and explain. A. Sclera, B. Choroid, C. Optic nerve sheath, D. Retina",A: Sclera,"Answer: A: Sclera Explanation: The sclera (skleros=hard) is opaque and forms the posterior five-sixths of the eyeball. It is composed of dense fibrous tissue which is firm and maintains the shape of the eyeball. It is thickest behind, near the entrance of the optic nerve, and thinnest about 6 mm behind the sclerocorneal junction where the recti muscles are inserted. However, it is weakest at the entrance of the optic nerve. Here the sclera shows numerous perforations for passage of fibres of the optic nerve. Because of its sieve-like appearance this region is called the lamina cribrosa (crib=sieve)." "This is a real-world medical entrance exam question, please give the true answer based on the question and selection. Topic name: None, Subject name: Dental","Mgo added to alginate power to modify which property: Select and explain. A. Increases surface character., B. Increases shelf life., C. Increases shear and tear strength., D. Improve strength.",A: Increases surface character.,"Answer: A: Increases surface character. Explanation: MgO added in alginate powder leads to improve its surface character mainly as well as it increased tensile strength, hardness of model, elastic modulus & Improved thermal stability." "This is a real-world medical entrance exam question, please give the true answer based on the question and selection. Topic name: None, Subject name: Dental","2 root canals are found in what percentage of Mandibular incisors Select and explain. A. 22-40%, B. 11-22%, C. 20-50 %, D. 5-15%",A: 22-40%,"Answer: A: 22-40% Explanation: None" "This is a real-world medical entrance exam question, please give the true answer based on the question and selection. Topic name: None, Subject name: Surgery","Most difficult maxillary tooth to anaesthetize by infiltration is : Select and explain. A. 1st molar, B. 1st premolar, C. Canine, D. 3rd molar",A: 1st molar,"Answer: A: 1st molar Explanation: None" "This is a real-world medical entrance exam question, please give the true answer based on the question and selection. Topic name: None, Subject name: Microbiology","(gd) T cell is associated with? Select and explain. A. CD4, B. CD5, C. Cd8, D. First line defense against bacterial peptides",D: First line defense against bacterial peptides,"Answer: D: First line defense against bacterial peptides Explanation: Ans. D-First line defense against bacterial peptides(gd) T cell: Gamma delta (gd) T cells have a TCR that is made up of one g (gamma) chain and one d (delta) chain.This group of T cells is usually much less common than ab T cells, but are at their highest abundance in the gut mucosa, within a population of lymphocytes known as intraepithelial lymphocytes (IELs).Constitute 5% of total cells, express y/6 chains of TCR chains; instead of ?/b chains.They lack both CD4 and CD8 molecules.They do not require antigen processing and MHC presentation of peptides.They lack both CD4 and CD8 molecules.They do not require antigen processing and MHC presentation of peptides.They are pa of innate immunity as the(gd) receptors exhibit limited diversity for the antigen.They are usually found in the gut mucosa, as intraepithelial lymphocytes (IELs).The function of (gd) T-cells is not known, they may encounter the lipid antigens that enter through the intestinal mucosa." "This is a real-world medical entrance exam question, please give the true answer based on the question and selection. Topic name: None, Subject name: Dental","Health education includes all except Select and explain. A. Development of reflexive behavior, B. Appeals to reason, C. Positive instilled behaviour, D. People to think for themselves",A: Development of reflexive behavior,"Answer: A: Development of reflexive behavior Explanation: None" "This is a real-world medical entrance exam question, please give the true answer based on the question and selection. Topic name: None, Subject name: Dental","Main crystalline component of calculus is Select and explain. A. Hydroxyapatite, B. Octacalcium phosphate, C. Magnisium whitlockite, D. Brushite",A: Hydroxyapatite,"Answer: A: Hydroxyapatite Explanation: Hydroxyapatite and octacalcium phosphate(constitute the bulk of the specimen) are detected most frequently in all supragingival calculus. While in Subgingival calculus ratio of calcium to phosphate is higher subgingivally and sodium content also increases with the depth of periodontal pocket." "This is a real-world medical entrance exam question, please give the true answer based on the question and selection. Topic name: None, Subject name: Dental","PPS anatomically is: Select and explain. A. Pterygomaxillary notches & fovea palatine, B. Pterygomaxillary notches & PN5, C. Maxillary tuberosity & pterygornaxillary raphae, D. Posterior Limit of palatine bone",A: Pterygomaxillary notches & fovea palatine,"Answer: A: Pterygomaxillary notches & fovea palatine Explanation: None" "This is a real-world medical entrance exam question, please give the true answer based on the question and selection. Topic name: None, Subject name: Radiology","There is <2 cm painful swelling responsive to salicylates, this is: Select and explain. A. Osteoma, B. Osteochondroma, C. Osteoid osteoma, D. Osteoblastoma",C: Osteoid osteoma,"Answer: C: Osteoid osteoma Explanation: None" "This is a real-world medical entrance exam question, please give the true answer based on the question and selection. Topic name: None, Subject name: Anatomy","Card test is done for which of the following muscle? Select and explain. A. Palmar interossei, B. Dorsal interossei, C. Lumbricals, D. Adductor pollicis",A: Palmar interossei,"Answer: A: Palmar interossei Explanation: Ans: A. Palmar interosseiNerveTestTestMuscleUlnar nerveBook testAdductor pollicis Card testPalmar interossei Froment's signFlexor pollicis substitutes for adduc?tor pollicis" "This is a real-world medical entrance exam question, please give the true answer based on the question and selection. Topic name: None, Subject name: Pediatrics","The chances of having an unaffected baby, when both parents have achondroplasia, are – Select and explain. A. 0%, B. 25%, C. 50%, D. 100%",B: 25%,"Answer: B: 25% Explanation: First see general features of autosomal dominant disorders - Autosomal dominant disorders are manifested in heterozygous state. So, at least one of the parent should be affected to transmit the disease to child. Homozygous state does not exist as it is not compitable with life, i.e. homozygous fetus usually dies in utero. So, all the affected individuals will be heterozygous. If an affected person marries an unaffected one, i.e. if one of the parents is affected than every child has 50% of chance of having the disease and 50% of chance not having the disease." "This is a real-world medical entrance exam question, please give the true answer based on the question and selection. Topic name: None, Subject name: Dental","Ionic exchange between enamel surface and environment: Select and explain. A. Does not take place once enamel matures, B. Stops after 2 years of eruption, C. Continues till adult life, D. Continues through out life",D: Continues through out life,"Answer: D: Continues through out life Explanation: None" "This is a real-world medical entrance exam question, please give the true answer based on the question and selection. Topic name: None, Subject name: Pathology","The common site for necrotizing sialometaplasia Select and explain. A. cheeks, B. dorsum of tongue, C. palate, D. gingival",C: palate,"Answer: C: palate Explanation: None" "This is a real-world medical entrance exam question, please give the true answer based on the question and selection. Topic name: None, Subject name: Dental","Which of the following is a major pattern of accessory canals in mandibular first molar is? Select and explain. A. Single furcation canal which is extend from pulp chamber to intra-redicular region, B. A lateral canal extends from coronal third to furcal region particular in distal canal, C. Presence of both lateral and furcal canal, D. Pattern is variable and age dependent",B: A lateral canal extends from coronal third to furcal region particular in distal canal,"Answer: B: A lateral canal extends from coronal third to furcal region particular in distal canal Explanation: None" "This is a real-world medical entrance exam question, please give the true answer based on the question and selection. Topic name: None, Subject name: Pathology","45 chromosomes are seen in Select and explain. A. Turner's syndrome, B. Down's syndrome, C. Dentinogenesis imperfecta, D. Treacher's Collins syndrome",A: Turner's syndrome,"Answer: A: Turner's syndrome Explanation: None" "This is a real-world medical entrance exam question, please give the true answer based on the question and selection. Topic name: None, Subject name: Dental","Which of following will happen with a flexible major connector: Select and explain. A. It will make other components more effective, B. Will cause PDL destruction of abutment teeth, C. Cause ill-fitting of RPD, D. Will cause difficulty in occlusion",B: Will cause PDL destruction of abutment teeth,"Answer: B: Will cause PDL destruction of abutment teeth Explanation: None" "This is a real-world medical entrance exam question, please give the true answer based on the question and selection. Topic name: AIIMS 2018, Subject name: Medicine","A 50 year male presented with high BP of 160/100 mm Hg and hea rate of 120/min. CECT is shown below. Which is best management of this condition? Select and explain. A. Surgical repair, B. LMW heparin, C. Beta blocker, D. Vitamin K inhibitors",A: Surgical repair,"Answer: A: Surgical repair Explanation: Tennis ball appearance of ascending aoa is feature of Aoic dissection. - Type A Rx- Surgical Repair Extra - mile Type B Aoic dissection Type B aoic dissections involve transverse and/or descending aoa. For uncomplicated/ stable distal lesions and intramural type B hematomas medical therapy is preferred. For complicated type B surgical correction is indicated." "This is a real-world medical entrance exam question, please give the true answer based on the question and selection. Topic name: None, Subject name: Surgery","Most common site of zygomatic arch fracture is: Select and explain. A. Anterior to Zygomaticotemporal suture, B. Posterior to Zygomaticotemporal suture, C. Zygomaticotemporal suture, D. No specific location",B: Posterior to Zygomaticotemporal suture,"Answer: B: Posterior to Zygomaticotemporal suture Explanation: The zygomatic bone usually fractures in the region of the  zygomaticofrontal  suture, the  zygomaticotemporal suture and the zygomaticomaxillary suture. It is unusual for the zygomatic bone itself to be fractured, but in extreme violence, the bone may be comminuted or split across. Most common site of zygomatic arch fracture is posterior to Zygomaticotemporal suture. The isolated zygomatic arch fracture may occur without displacement of the zygomatic bone. Ref: Oral and maxillofacial surgery , Neelima Anil Malik 4th ed page no 534" "This is a real-world medical entrance exam question, please give the true answer based on the question and selection. Topic name: None, Subject name: Forensic Medicine","Which of the following principles governs biomedical research in human subjects? Select and explain. A. Geneva declaration, B. Helsinki declaration, C. Hippocratic oath, D. International code of medical ethics",B: Helsinki declaration,"Answer: B: Helsinki declaration Explanation: Ans: B. Helsinki declaration(Ref: Reddy 34/e p600, 33/e p26, 400, 647; Parikh 6/e p1.26)Helsinki declaration governs biomedical research in human subjects.The declaration of Helsinki is a set of ethical principles regarding human experimentation developed for the medical community by the World Medical Association.It is widely regarded as the cornerstone document on human research ethics." "This is a real-world medical entrance exam question, please give the true answer based on the question and selection. Topic name: None, Subject name: Pathology","Median rhomboid glossitis is due to: Select and explain. A. Inflammation of the tongue, B. Persistence of tuberculum impar, C. Hypertrophy of filiform papillae, D. Atrophy of filiform papillae",B: Persistence of tuberculum impar,"Answer: B: Persistence of tuberculum impar Explanation: None" "This is a real-world medical entrance exam question, please give the true answer based on the question and selection. Topic name: None, Subject name: Microbiology","A 2 weeks old infant has conjunctivitis, which later developed into respiratory distress and pneumonia. Chest X-ray showed bilateral lung infiltrates. WBC count was 14,300/dL. Which of the following is the most likely organism? Select and explain. A. Chlamydia trachomatis, B. Streptococcus agalactiae, C. Gonococcus, D. Haemophilus influenzae",A: Chlamydia trachomatis,"Answer: A: Chlamydia trachomatis Explanation: Answer- A. Chlamydia trachomatisChlamydia trachomatis: Of newborns infected by the mother, 10-20% may develop respiratory tract involvement 2-12 weeks after bih, culminating in pneumonia.Affected newborns have nasal obstruction or discharge, striking tachypnea, a characteristic paroxysmal staccato cough, an absence of fever, and eosinophilia. Interstitial infiltrates and hyperinflation can be seen on radiographs. The diagnosis should be suspected if pneumonitis develops in a newborn who has inclusion conjunctivitis and can be established by isolation of C. trachomatis from respiratory secretions." "This is a real-world medical entrance exam question, please give the true answer based on the question and selection. Topic name: None, Subject name: Anatomy","Independent assoment of maternal and paternal chromosome occurs at which stage of spermatocyte maturation: Select and explain. A. Spermatogonia to primary spermatocyte, B. Primary spermatocyte to secondary spermatocyte, C. Secondary spermatocyte to spermatids, D. Spermatid to spermatozoa",B: Primary spermatocyte to secondary spermatocyte,"Answer: B: Primary spermatocyte to secondary spermatocyte Explanation: Ans: B. Primary spermatocyte to secondary spermatocyteDuring spermatogenesis - Independent assoment of paternal & maternal chromosomes occurs during meiosis I.Also, primary spermatocyte (2n) is conveed into two secondary spermatocytes." "This is a real-world medical entrance exam question, please give the true answer based on the question and selection. Topic name: None, Subject name: Dental","During post preparation in distal canal of mandibular first molar, maximum chance of perforation is seen in: Select and explain. A. Lingual surface, B. Distal surface, C. Buccal surface, D. Mesial surface",D: Mesial surface,"Answer: D: Mesial surface Explanation: None" "This is a real-world medical entrance exam question, please give the true answer based on the question and selection. Topic name: None, Subject name: Dental","Ratio of mesiodistal width to height of crown of maxillary canine Select and explain. A. 8:10, B. 10:08, C. 6:08, D. 8:06",A: 8:10,"Answer: A: 8:10 Explanation: None" "This is a real-world medical entrance exam question, please give the true answer based on the question and selection. Topic name: AIIMS 2018, Subject name: Pharmacology","Which of the following hormone is/are under inhibitory of hypothalamus? Select and explain. A. Prolactin, B. Only prolactin, C. Only growth hormone, D. Both prolactin and growth hormone",D: Both prolactin and growth hormone,"Answer: D: Both prolactin and growth hormone Explanation: Inhibitory hormones secreted by hypothalamus are somatostatin and dopamine. Somatostatin inhibits GH, ACTH, TSH. Dopamine inhibits prolactin. GH is under both stimulatory (GHRH) and inhibitory control (Somatostatin) Prolactin is only under inhibitory control (dopamine). Note: Read the question carefully; if they ask which hormones of anterior pituitary are under inhibitory control (as in this question), answer should be both prolactin and GH. However, if they ask the question, which hormone is ONLY under inhibitory control, then answer should be only prolactin." "This is a real-world medical entrance exam question, please give the true answer based on the question and selection. Topic name: None, Subject name: Biochemistry","All of the following are the reasons responsible for ketosis in a patient of Von Gierke's disease? Select and explain. A. They have hypoglycemia, B. Have low blood sugar levels, C. Oxaloacetate is required for gluconeogenesis, D. Low fat mobilization",D: Low fat mobilization,"Answer: D: Low fat mobilization Explanation: Answer- D. Low fat mobilizationExcessive fatty acid mobilization (not the Low fat mobilization) from adipose tissues leads to ketosis in Von-Gierke's disease.Type I glycogen storage disorder, inherited as autosomal recessive, due to the deficiency of glucose-6 phosphatase.Excessive fatty acid mobilization from adipose tissues leads to ketosis.Deficiency of glucose-6 phosphatase leads to low blood sugar and hypoglycemic attacks." "This is a real-world medical entrance exam question, please give the true answer based on the question and selection. Topic name: None, Subject name: Social & Preventive Medicine","How much population falls between median and median plus one standard deviation in a normal distribution ? Select and explain. A. 0.34, B. 0.68, C. 0.17, D. 0.47",A: 0.34,"Answer: A: 0.34 Explanation: NORMAL DISTRIBUTION Shape is bilaterally symmetrical Mean = Median = Mode (coincide) 50% of all values lie above Mean (or Median or Mode) Mean + 1SD cover 68% values (Mean + 1SD cover 34% values) Mean + 2SD cover 95% values (Mean + 2SD cover 47.5% values) Mean + 3SD cover 99% values (Mean + 3SD cover 49.5% values)" "This is a real-world medical entrance exam question, please give the true answer based on the question and selection. Topic name: None, Subject name: Dental","Which crown would be suitable for a patient with high caries index, minimum horizontal overlap of maxillary central incisors? Select and explain. A. Metal ceramic crown, B. Three fourth crown, C. Porcelain jacket crown, D. Aluminium crown",A: Metal ceramic crown,"Answer: A: Metal ceramic crown Explanation: None" "This is a real-world medical entrance exam question, please give the true answer based on the question and selection. Topic name: None, Subject name: Pathology","NK cells are effective against viral infected cells only if the cell with infection: Select and explain. A. Express MHC class I proteins, B. Unable to express MHC class I proteins, C. Express MHC class II proteins, D. Unable to express MHC class II proteins",B: Unable to express MHC class I proteins,"Answer: B: Unable to express MHC class I proteins Explanation: None" "This is a real-world medical entrance exam question, please give the true answer based on the question and selection. Topic name: None, Subject name: Dental","Main concern during prosthodontics surgery and better prosthetic prognosis? Select and explain. A. Continuity defects, B. Minimal invasion, C. Occlusion, D. Cost",B: Minimal invasion,"Answer: B: Minimal invasion Explanation: None" "This is a real-world medical entrance exam question, please give the true answer based on the question and selection. Topic name: None, Subject name: Dental","Serial technique Or telescoping technique is another name of: Select and explain. A. Step back technique., B. Crown down technique., C. Standarize technique., D. Balance force technique.",A: Step back technique.,"Answer: A: Step back technique. Explanation: Schilder called the step back technique as serial Or telescoping technique." "This is a real-world medical entrance exam question, please give the true answer based on the question and selection. Topic name: None, Subject name: Pathology","Clinical diagnosis of candidiasis is confirmed by Select and explain. A. Characteristic odour, B. Demonstration of mycelia and spores in scrapping, C. Response to injection of vitamin B12, D. Demonstration of ray fungus in granules",B: Demonstration of mycelia and spores in scrapping,"Answer: B: Demonstration of mycelia and spores in scrapping Explanation: None" "This is a real-world medical entrance exam question, please give the true answer based on the question and selection. Topic name: None, Subject name: Radiology","The best radiographic view of temporomandibular joint is given by: Select and explain. A. Transorbital view, B. Reverse Towne's view, C. Transpharyngeal, D. Panorex",C: Transpharyngeal,"Answer: C: Transpharyngeal Explanation: None" "This is a real-world medical entrance exam question, please give the true answer based on the question and selection. Topic name: None, Subject name: Surgery","Submandibular calculus can be removed by: Select and explain. A. Dilatation of the duct, B. Excision of the opening of duct, C. Removal of the gland, D. Incision of the duct and removal of calculus",D: Incision of the duct and removal of calculus,"Answer: D: Incision of the duct and removal of calculus Explanation: None" "This is a real-world medical entrance exam question, please give the true answer based on the question and selection. Topic name: None, Subject name: Dental","The best way to fill a bony defect will be with which of the following? Select and explain. A. Hydroxy apatite, B. Donor graft, C. Xenograft, D. Autograft",D: Autograft,"Answer: D: Autograft Explanation: None" "This is a real-world medical entrance exam question, please give the true answer based on the question and selection. Topic name: AIIMS 2019, Subject name: Physiology","A 36 year old female is found to have a large pituitary mass on MRI imaging. She underwent transsphenoidal hypophysectomy. Postop replacement of which of the following hormone is not needed? Select and explain. A. L-Thyroxine, B. Estradiol, C. Glucocoicoids, D. Mineralocoicoids",D: Mineralocoicoids,"Answer: D: Mineralocoicoids Explanation: Aldosterone is majorly regulated by K+ & plasma volume The following factors play essential roles in aldosterone regulation: 1. | K+ concentration in ECF - | Aldosterone 2. | Angiotensin concentration in ECF - | Aldosterone3. | Na++ in ECF - decreases aldosterone very slightly4. ACTH from anterior pituitary - necessary for aldosterone sec but has little effect on controlling the rate of secretion in most physiological conditions. ACTH appears to play a permissive role in regulation of aldosterone of these, K+ ion conc. & RAAS are the most potent in regulating aldosterone secretion. Regulated by Hormone TSH Thyroid FSH Estrogen ACTH Glucocoicoids" "This is a real-world medical entrance exam question, please give the true answer based on the question and selection. Topic name: None, Subject name: Pharmacology","Barbiturates in pediatrics is: Select and explain. A. Contraindicated, B. Low safety, C. Can be used safely, D. Not much use",B: Low safety,"Answer: B: Low safety Explanation: None" "This is a real-world medical entrance exam question, please give the true answer based on the question and selection. Topic name: None, Subject name: Dental","Which of the following type of behavior is seen in stubborn child: Select and explain. A. Obstinate., B. Timid., C. Whinning., D. Hysterical.",A: Obstinate.,"Answer: A: Obstinate. Explanation: None" "This is a real-world medical entrance exam question, please give the true answer based on the question and selection. Topic name: None, Subject name: Dental","Recently two methods of cPAP; conventional and bubble cPAP were compared. In the conventional method, 90 out of 160 showed extubation failure while 40 out of 160 showed extubation failure in bubble method, which test would be the best to compare the statistical significance between the rates of extubation failure in the two tests? Select and explain. A. Chi square test, B. Student's test, C. Paired t test, D. Analysis of variance",A: Chi square test,"Answer: A: Chi square test Explanation: None" "This is a real-world medical entrance exam question, please give the true answer based on the question and selection. Topic name: None, Subject name: Dental","Angle between rake face and radial line is called: Select and explain. A. Edge angle., B. Rake angle., C. Clearance angle., D. Spiral angle.",B: Rake angle.,"Answer: B: Rake angle. Explanation: None" "This is a real-world medical entrance exam question, please give the true answer based on the question and selection. Topic name: AIIMS 2019, Subject name: Anatomy","Which of the following are true regarding levator ani EXCEPT? Select and explain. A. Levator ani muscle is attached at pelvic brim, B. Pubococcygeus and iliococcygeus are components, C. Fibres are directed posterior and medial, D. Suppos pelvic viscera",A: Levator ani muscle is attached at pelvic brim,"Answer: A: Levator ani muscle is attached at pelvic brim Explanation: Levator ani: Two levator ani muscles originate from each side of pelvic wall, course medially, inferiorly & join together in midline Levator ani muscle include 3 collections of muscle fibers- Pubococcygeus + Puborectalis + Iliococcygeus muscle Levator ani muscles help suppo pelvic viscera & maintain closure of rectum & vagina, maintain angle b/w rectum & anal canal. They are innervated by branches from anterior ramus of S4 and by branches of pudendal nerve (S2 to S4) It is attached to pubis & obturator fascia. Levator ani + ischio-coccygeus form - pelvis" "This is a real-world medical entrance exam question, please give the true answer based on the question and selection. Topic name: None, Subject name: Dental","True congenital anodontia is: Select and explain. A. X linked dominant, B. X linked recessive, C. Autosomal dominant, D. Polygenic",C: Autosomal dominant,"Answer: C: Autosomal dominant Explanation: Congenital absence of teeth is a hereditary phenomenon passed through generations by an autosomal dominant pattern. Such a patient often presents with poor esthetics, mastication and disturbed social behavior." "This is a real-world medical entrance exam question, please give the true answer based on the question and selection. Topic name: None, Subject name: Dental","Most advantageous indication of acid etching is Select and explain. A. Decrease micro leakage, B. Decrease polymerization shrinkage, C. Decrease coefficient of thermal expansion, D. Decrease porosity in restorative material",A: Decrease micro leakage,"Answer: A: Decrease micro leakage Explanation: None" "This is a real-world medical entrance exam question, please give the true answer based on the question and selection. Topic name: None, Subject name: Physiology","Which hormone act by crossing cell membrane - Select and explain. A. Thyroxine, B. Insulin, C. Glucagon, D. Calcitonin",A: Thyroxine,"Answer: A: Thyroxine Explanation: Ans. A. ThyroxineIntracellular receptors are transcription factors that have binding sites for the hormone (ligand) and for DNA and function as ligand (hormone)-regulated transcription factors.Steroid hormones and the steroid derivative vitamin D3 fulfill this requirement.Thyroid hormones must be actively transpoed into the cell." "This is a real-world medical entrance exam question, please give the true answer based on the question and selection. Topic name: None, Subject name: Surgery","I.V. diazepam causes following in a patient on dental chair: Select and explain. A. Tinel's sign, B. Virrel's sign, C. Batters sign, D. Bell's sign",B: Virrel's sign,"Answer: B: Virrel's sign Explanation: None" "This is a real-world medical entrance exam question, please give the true answer based on the question and selection. Topic name: None, Subject name: Pharmacology","Preferred drug for the treatment of uncomplicated grade 2 hypeension in a 48 year old man is Select and explain. A. Chlohalidone, B. Triamterene, C. Spironolactone, D. Furosemide",A: Chlohalidone,"Answer: A: Chlohalidone Explanation: Ans. A. Chlohalidone Thiazide diuretics are inexpensive, are generally well tolerated, and are recommended as a first-line therapy in the treatment of hypeension in the elderly population.Thiazide diuretics are as effective as any drug for first-line treatment of hypeension in the elderly population is the Antihypeensive and Lipid-Lowering Treatment to Prevent Hea Attack Trial (ALLHAT) study.Patients who received the diuretic had a lower incidence of CV events (secondary outcomes) compared with the other groups. The diuretic treatment group had lower HF rates compared with the CCB group" "This is a real-world medical entrance exam question, please give the true answer based on the question and selection. Topic name: None, Subject name: Dental","Which is not true Select and explain. A. Actinomycin D is a mono clonal antibody, B. Actinomycin D is not a cell specific but It's a cell cycle specific, C. Vinblastine is not a cell specific but its a cycle/phase specific, D. None of above",B: Actinomycin D is not a cell specific but It's a cell cycle specific,"Answer: B: Actinomycin D is not a cell specific but It's a cell cycle specific Explanation: None" "This is a real-world medical entrance exam question, please give the true answer based on the question and selection. Topic name: None, Subject name: Anatomy","Bone better described as ""bat with extended wings"" is: Select and explain. A. Ethmoid, B. Sphenoid, C. Nasal, D. Mandible",B: Sphenoid,"Answer: B: Sphenoid Explanation: None" "This is a real-world medical entrance exam question, please give the true answer based on the question and selection. Topic name: None, Subject name: Dental","""Cut back"" incision made on laterally displaced flap is: Select and explain. A. Mesial cut toward the donor site, B. Distal cut toward the donor site, C. Mesial cut toward the recipient site, D. Distal cut toward the recipient site",D: Distal cut toward the recipient site,"Answer: D: Distal cut toward the recipient site Explanation: None" "This is a real-world medical entrance exam question, please give the true answer based on the question and selection. Topic name: None, Subject name: Surgery","Which is the best incision preferred for diaphragmatic surgery?(AIIMS May 2015, May 2014) Select and explain. A. Circumferential, B. Radial, C. Veical, D. Transverse",A: Circumferential,"Answer: A: Circumferential Explanation: Ans: A. Circumferential(Ref Sahiston and Spencer's Surge, of Chest 8/chapter 7)Circumferential incision: generally taken for diaphragmatic surgery.Circumferential incisions:In periphery regions - Result in little loss of function.Must be at least 5 cm lateral to edge of central tendon.To avoid posterolateral & anterolateral branches of phrenic nerve.Difficult to correctly realign after long operation.Placement of surgical clips on each side of muscular incision greatly facilitates correct spatial orientation on closing." "This is a real-world medical entrance exam question, please give the true answer based on the question and selection. Topic name: AIIMS 2019, Subject name: Microbiology","CLED media better than Macconkey media Select and explain. A. It stimulates growth of Staph and Candida as it is non selective, B. Inhibits swarming of proteus, C. Differentiates between Lactose fermenter and non-lactose fermenters, D. Sodium taurocholate is used as selective agent.",A: It stimulates growth of Staph and Candida as it is non selective,"Answer: A: It stimulates growth of Staph and Candida as it is non selective Explanation: Cysteine electrolyte deficient agar (CLED) is a non-selective media and it stimulates the growth of Staphylococcus and Candida whereas Mac Conkey agar is a selective media. Both CLED and MAC Conkey Agar inhibits the swarming of proteus and differentiate between lactose fermenter and non-lactose fermenter. Both use sodium Taurocholate as a selective agent and so first option is a better option." "This is a real-world medical entrance exam question, please give the true answer based on the question and selection. Topic name: None, Subject name: Pediatrics","A 2-day-old neonate in the neonatal ICU develops seizures. Which of the following would be the next best investigation for the child? Select and explain. A. Transcranial ultrasound, B. CT Head, C. MRI brain, D. X-ray",A: Transcranial ultrasound,"Answer: A: Transcranial ultrasound Explanation: Answer- A (Transcranial ultrasound)`Cranial ultrasonouraphy is the main imagine modality of premature neonates and well suited for the study of neonates in general." "This is a real-world medical entrance exam question, please give the true answer based on the question and selection. Topic name: None, Subject name: Pathology","Which of the following is the most common lesion of the mandible? Select and explain. A. Adamantinoma, B. Osteogenic sarcoma, C. Squamous cell carcinoma, D. Osteoclastoma",A: Adamantinoma,"Answer: A: Adamantinoma Explanation: None" "This is a real-world medical entrance exam question, please give the true answer based on the question and selection. Topic name: AIIMS 2018, Subject name: Medicine","A medical student presented to the ED with protracted vomiting. For this he was given and anti-emetic drug following which he developed abnormal posturing. Which of the following is the most likely drug to be given to the patient? Select and explain. A. Metoclopramdie, B. Ondansetron, C. Domperidone, D. Dexamethasone",A: Metoclopramdie,"Answer: A: Metoclopramdie Explanation: Abnormal posturing - Metoclopramide (repeated or large dose) | Decrease dopamine level in brain | Extrapyramidal manifestation- acute dystonia" "This is a real-world medical entrance exam question, please give the true answer based on the question and selection. Topic name: None, Subject name: Pathology","A patient complains of loss of visual acuity, deafness and enlargement of maxilla: Select and explain. A. Paget's disease, B. Osteomalacia, C. Fibrous dysplasia, D. Osteogenesis imperfecta",A: Paget's disease,"Answer: A: Paget's disease Explanation: None" "This is a real-world medical entrance exam question, please give the true answer based on the question and selection. Topic name: None, Subject name: Medicine","The genetic mutation seen in the most common type of maturity onset diabetes of young (MOM') is: Select and explain. A. Hepatocyte nuclear factor-4, B. Hepatocyte nuclear factor-1, C. Glucokinase, D. Insulin promoter factor-1",B: Hepatocyte nuclear factor-1,"Answer: B: Hepatocyte nuclear factor-1 Explanation: Answer-b. Hepatocyte nuclear factor-1 Maturity onset Diabetes in Young (MODY) TypesMODY-1HNF-4 alphaMODY-2GlucokinaseMODY-3HNF-1 alpha (most common)MODY-4IPF-1MODY-5HNF-1 betaMODY-6NeuroD1/beta2" "This is a real-world medical entrance exam question, please give the true answer based on the question and selection. Topic name: None, Subject name: Pediatrics","Which of the following is the most common inherited malignancy : Select and explain. A. Infant leukemia, B. Retinoblastoma, C. Wilm's tumour, D. Neuroblastoma",B: Retinoblastoma,"Answer: B: Retinoblastoma Explanation: ""Retinoblastoma is the most strking example of inhirited cancer syndrome. Approximately 40% of retinoblastomas are familial. Carrier of this gene have a 10000 fold increased risk of developing retinoblastoma usualy bilateral"". Each child of parent with familial bilateral retinoblastoam has a 50% risk of inheriting the retinoblastoma gene, of these 90% will develop retinoblastoma." "This is a real-world medical entrance exam question, please give the true answer based on the question and selection. Topic name: None, Subject name: Anatomy","All of the following are true about grey communicans except: Select and explain. A. U nmyel inated, B. Connects to spinal nerves, C. Pregang I ion ic, D. Present medial to the white ramus cornmunicans",C: Pregang I ion ic,"Answer: C: Pregang I ion ic Explanation: Ans. c. PreganglionicEach spinal nerve receives a branch called a gray ramus communicans from the adjacent paraveebral ganglion of the sympathetic trunk. The gray rami communicans contain postganglionic nerve fibers of the sympathetic nervous system and are composed of largely unmyelinated neurons. This is in contrast to the white rami communicans, in which heavily myelinated neurons give the rami their white appearance. ""The gray ramus communicans connects the sympathetic trunk or a ganglion to the anterior ramus and contains the postganglionic sympathetic fibers. It appears gray because postganglionic fibers are nonmyelinated. The gray ramus communicans is positioned medial to the white ramus communicans.""" "This is a real-world medical entrance exam question, please give the true answer based on the question and selection. Topic name: None, Subject name: Surgery","Which of the following structure is not removed in radical neck dissection- Select and explain. A. Spinal accessory nerve, B. Submandibular, C. Tail of parotid, D. Level 2 b lymph nodes",C: Tail of parotid,"Answer: C: Tail of parotid Explanation: Answer- CStructures removed during radical neck dissection-The classic operation involves resection of the cervical lymphatics and lymph nodes and those structures closely associated:the internal jugular vein,the accessory nerve,the submandibular gland, andthe sternocleidomastoid muscle." "This is a real-world medical entrance exam question, please give the true answer based on the question and selection. Topic name: None, Subject name: Surgery","In fracture of atrophic mandible with bone loss, what is the best treatment modality? Select and explain. A. Bone grafting and load bearing, B. Bone grafting and load sharing, C. Semi-rigid fixation, D. IMF with open reduction",A: Bone grafting and load bearing,"Answer: A: Bone grafting and load bearing Explanation: None" "This is a real-world medical entrance exam question, please give the true answer based on the question and selection. Topic name: None, Subject name: Physiology","Which of the following hormones has a permissive action at the onset of pubey? Select and explain. A. Insulin, B. Leptin, C. GnRH, D. Growth hormone",B: Leptin,"Answer: B: Leptin Explanation: Ans: B. LeptinRef: Ganong 25/e p397, 24/e p399; Knobil and Neill's Physiology of Reproduction 2014/p1607; Novaks 13/e p410).Leptin has a permissive action at the onset of pubey." "This is a real-world medical entrance exam question, please give the true answer based on the question and selection. Topic name: None, Subject name: Dental","Contraindication of band and Loop space maintainer are all except Select and explain. A. High caries susceptibility, B. Single tooth missing in posterior region, C. Moderate to severe space loss, D. Lower anterior crowding",B: Single tooth missing in posterior region,"Answer: B: Single tooth missing in posterior region Explanation: None" "This is a real-world medical entrance exam question, please give the true answer based on the question and selection. Topic name: None, Subject name: Dental","A tooth can be made to appear shorter by positioning? Select and explain. A. Gingival Height of contour more incisally, B. Gingival Height of contour more gingivally, C. Developmental depression more far., D. Mesial and distal ling angle closure",A: Gingival Height of contour more incisally,"Answer: A: Gingival Height of contour more incisally Explanation: None" "This is a real-world medical entrance exam question, please give the true answer based on the question and selection. Topic name: None, Subject name: Dental","In 7-year-old patient; 74 and 85 are indicated for extraction; which of the following will be appropriate space maintainer for the child? Select and explain. A. Bilateral band and loop, B. Lingual arch, C. Nance space maintainer, D. None of the above",A: Bilateral band and loop,"Answer: A: Bilateral band and loop Explanation: Band and loop space maintainer  Advantages Effective space maintainer for unilateral loss of single tooth in buccal segments. Economical to make and construct. Little chairside time, if preformed bands are used. Adjusts easily to accommodate the changing dentition. Reference: Pediatric Dentistry Principles and Practice SECOND EDITION, MS Muthu; pdf no 1012" "This is a real-world medical entrance exam question, please give the true answer based on the question and selection. Topic name: None, Subject name: Pediatrics","Following are true about bronchial cyst except – Select and explain. A. Mostly mediastinal, B. 50–70% occur in lungs, C. Usually multiloculated, D. Are infected quite often",B: 50–70% occur in lungs,"Answer: B: 50–70% occur in lungs Explanation: Only 15% of bronchogenic cysts occur in lungs (pulmonary parenchyma). Bronchogenic cysts Bronchogenic cyst is a congenital lung defect, which develops from abnormal lung budding of primitive foregut. Most of the bronchogenic cyst arise in the mediastinum, where they are usually located in the mid mediastinum  near carina. About 15% of Bronchogenic cyst also occurs in pulmonary parenchyma. When located in the lung they are more common in right lung and lower lobe. They are lined by cuboidal or ciliated columnar epithelial cells and are usually filled with mucoid material. Complications of Bronchogenic cyst :- Infection of the cyst leading to lung abscess (illediastinal bronchogenic cysts do not communicate with the bronchi but those situated within the lung may communicate with the airways leading to abscess formation). Compression can produce either hyperinflation of the lung or atelactasis due to obstruction. Tension pneumothorax (d/t rupture of cyst that communicates with bronchus)." "This is a real-world medical entrance exam question, please give the true answer based on the question and selection. Topic name: None, Subject name: Radiology","To localize a supernumerary or an impacted tooth and determine its exact relationship to the other teeth, which of the following radiographs would be most effective? Select and explain. A. A periapical and an occlusal view, B. An occlusal view using a high angle, C. A panoramic radiograph, D. Two or more periapical views at different angles and an occlusal view",D: Two or more periapical views at different angles and an occlusal view,"Answer: D: Two or more periapical views at different angles and an occlusal view Explanation: None" "This is a real-world medical entrance exam question, please give the true answer based on the question and selection. Topic name: None, Subject name: Ophthalmology","Cells most commonly affected in glaucomatous optic atrophy? Select and explain. A. Amacrine cells, B. Bipolar cells, C. Ganglion cells, D. Rods and cones",C: Ganglion cells,"Answer: C: Ganglion cells Explanation: Answer-C. Ganglion cellsGlaucoma, the second leading cause of blindness, is characterized by changes in the optic disc and visual field defects'.The elevated intraocular pressure was considered the prime factor responsible for the glaucomatous optic neuropathy involving death of retinal ganglion cells and their axons.Glaucoma, a leading cause of irreversible visual loss, is characterized by loss of retinal ganglion cells (RGC) and their axons over a period of many years.Mainly the ganglions cells are affected in glaucoma patients, which may lead to glaucomatus optic atrophy'" "This is a real-world medical entrance exam question, please give the true answer based on the question and selection. Topic name: None, Subject name: Anatomy","All of the following muscles have parallel oriented fibers except: Select and explain. A. Saorius, B. Rectus abdominis, C. Sternohyoid, D. Tibialis anterior",D: Tibialis anterior,"Answer: D: Tibialis anterior Explanation: Ans: D. Tibialis anterior(Ref Gray's. 41/e p112, 40/e p104-105).Individual fibers of muscle are arranged either parallel or oblique to long axis of the muscle.Saorius, rectus abdominis & sternohyoid - Parallel oriented fibers.Tibialis anterior muscle - Multipennate muscle with oblique fibers. Muscles with Parallel FasciculiMuscles with Oblique FasciculiMuscles in which fasciculi are parallel to the line of pull & have greater degree of movement.Types:Quadrilateral: ThyrohyoidStrap-like: Sternohyoid & saorius Strap-like with tendinous intersections:Rectus AbdominisFusiform: Biceps brachii, digastricMuscles in which fasciculi are oblique to the line ofpull, muscle may be triangular, or pennate (feather? like) in the constructionArrangement makes muscle more powerful.Reduces range of movement.Types:Triangular: Temporalis, adductor longusdegUnipennate: Flexor pollicis longus, extensor digitorum longusdegBipennate: Rectus femoris, flexor hallucis longusdegMultipennate: Tibialis anterior, submscapularis, deltoid (acromial fibers)." "This is a real-world medical entrance exam question, please give the true answer based on the question and selection. Topic name: None, Subject name: Dental","Amount of energy actually producing cutting is: Select and explain. A. Cutting efficiency., B. Cutting effectiveness., C. Cutting fraction., D. None.",A: Cutting efficiency.,"Answer: A: Cutting efficiency. Explanation: None" "This is a real-world medical entrance exam question, please give the true answer based on the question and selection. Topic name: None, Subject name: Dental","Supragingival plaque undergoes which of the following changes with time? Select and explain. A. Plaque mass decreases, B. Plaque microflora becomes more gram positive, C. Plaque microflora becomes gram negative, D. Plaque microflora becomes predominantly spirochetal",C: Plaque microflora becomes gram negative,"Answer: C: Plaque microflora becomes gram negative Explanation: None" "This is a real-world medical entrance exam question, please give the true answer based on the question and selection. Topic name: None, Subject name: Dental","James classified habits as: Select and explain. A. Useful and Harmful., B. Intentional and Unintentional., C. Compulsive and Non-compulsive., D. None of the above.",A: Useful and Harmful.,"Answer: A: Useful and Harmful. Explanation: None" "This is a real-world medical entrance exam question, please give the true answer based on the question and selection. Topic name: None, Subject name: Gynaecology & Obstetrics","Transmission of herpes is maximum in- Select and explain. A. IInd trimester, B. IIIrd trimester, C. During parturition, D. 1st trimester",C: During parturition,"Answer: C: During parturition Explanation: In case of herpes infection transplacental infection is not common, instead the fetus becomes affected by virus shed from the cervix or lower genital tract during vaginal delivery. Baby may sometimes be affected in utero from contaminated liquor following rupture of membranes." "This is a real-world medical entrance exam question, please give the true answer based on the question and selection. Topic name: None, Subject name: Dental","An investigator wants to study the association between maternal intake of iron supplements (Yes or No) and birth weights (in gm) of new born babies. He collects relevant data from 100 pregnant women and their newborns. What statistical test of hypothesis would you advise for the investigator in this situation? Select and explain. A. Chi-square test, B. Unpaired or independent t - test, C. Analysis of variance, D. Student t — test",B: Unpaired or independent t - test,"Answer: B: Unpaired or independent t - test Explanation: None" "This is a real-world medical entrance exam question, please give the true answer based on the question and selection. Topic name: None, Subject name: Dental","Which is the most numerous component of plaque? Select and explain. A. Minerals, B. Food debris, C. Microorganisms, D. Leucocytes",C: Microorganisms,"Answer: C: Microorganisms Explanation: None" "This is a real-world medical entrance exam question, please give the true answer based on the question and selection. Topic name: AIIMS 2018, Subject name: Pharmacology","Which of the following drug must be sold by retail only on production of a prescription by a registered medical practitioner? Select and explain. A. Schedule H, B. Schedule G, C. Schedule X, D. Schedule M",A: Schedule H,"Answer: A: Schedule H Explanation: Schedule H: Drugs which can be sold only with prescription. On the strip of drug a long red line is present. Rx is written on the top right corner. NRx- is written on Narcotic (habit forming) drugs. XRx- not available for general market, only given under special conditions where health care provider can give the drug. OTC- Over The Counter drugs can be given without prescription." "This is a real-world medical entrance exam question, please give the true answer based on the question and selection. Topic name: AIIMS 2019, Subject name: ENT","Arrange the sequence of auditory pathway from peripheral to central: Inferior colliculus Cochlear nucleus Auditory coex Medial geniculate body Select and explain. A. B>A>D>C, B. B>D>C>A, C. A>D>B>C, D. C>A>B>D",A: B>A>D>C,"Answer: A: B>A>D>C Explanation: Auditory neural pathway. From below upwards are ( Mnemonic - E. COLI-MA) E - Eighth nerve (vestibulocochlear nerve) C - Cochlear nucleus (spiral ganglion) O - Superior Olivary complex (opposite) L - Lateral lemniscus (largest) I - Inferior colliculus M - Medial geniculate body A - Auditory coex E - 8th nerve COLI - Brainstem MA - Cerebrum Crossing over phenomenon takes places at Superior Olivary complex" "This is a real-world medical entrance exam question, please give the true answer based on the question and selection. Topic name: None, Subject name: Dental","According to WHO 1986, a molar filled with temporary restoration is classified as: Select and explain. A. Sound tooth, B. Filled but decay, C. Filled with no decay, D. Decayed tooth",D: Decayed tooth,"Answer: D: Decayed tooth Explanation: None" "This is a real-world medical entrance exam question, please give the true answer based on the question and selection. Topic name: None, Subject name: Biochemistry","A Middle aged woman presents with fissures in mouth, tingling sensation and peripheral neuropathy. Investigations showed reduced glutathione reductase activity. which vitamin deficiency is the likely cause of this? Select and explain. A. Vitamin B1, B. Vitamin 82, C. Vitamin 86, D. Vitamin B l2",B: Vitamin 82,"Answer: B: Vitamin 82 Explanation: Ans: B. Vitamin 82Ref: Harper's illustrated biochemistry, 3Oh ed., pg. 556Deficiency of riboflavin (vitamin B2) is characterized by cheilosis, desquamation and inflammation of the tongue, and a seborrheic dermatitis." "This is a real-world medical entrance exam question, please give the true answer based on the question and selection. Topic name: None, Subject name: Dental","Creation of an apical stop to prevent extrusion of matrix beyond apex is called: Select and explain. A. Resistance form, B. Retention form, C. Conenience form, D. None of the above.",A: Resistance form,"Answer: A: Resistance form Explanation: None" "This is a real-world medical entrance exam question, please give the true answer based on the question and selection. Topic name: None, Subject name: Surgery","A Jawan posted in Siachen was brought to the emergency depament with hypothermia. Which is the ideal site to measure his temperature? Select and explain. A. Oral, B. Rectal, C. Tympanic membrane, D. Armpit",B: Rectal,"Answer: B: Rectal Explanation: Answer- B. Rectal* Normal body core temperature ranges from about 98 F to 100 F (36.6 C to 37.7 C); core temperature is best measured by a rectal thermometer." "This is a real-world medical entrance exam question, please give the true answer based on the question and selection. Topic name: None, Subject name: Pediatrics","A 3.5 kg male infant born at term after an uncomplicated pregnancy and delivery develops respiratory distress sholy after bih and requires mechanical ventilation. The chest radiograph reveals a normal cardiothymic silhouette but a diffuse ground glass appearance to the lung fields. Surfactant replacement fails to improve gas exchange. Over the first week life, the hypoxemia worsens. Results of routing culture and echocardiographic findings are negative. A term female sibling died at 1 month of age with respiratory distress. Which of the following is the most likely diagnosis? Select and explain. A. Neonatal pulmonary alveolar proteinosis, B. Meconium aspiration, C. Total anomalous pulmonary venous return, D. Disseminated herpes simplex infection",A: Neonatal pulmonary alveolar proteinosis,"Answer: A: Neonatal pulmonary alveolar proteinosis Explanation: Ans: A. Neonatal pulmonary alveolar proteinosis(Ref. Nelson 20/e p852, 2119)Suggestive of neonatal pulmonary alveolar proteinosis.Pulmonary alveolar proteinosis:Disorder characterized by intra-alveolar accumulation of pulmonary surfactant.Two clinically distinct forms of pulmonary alveolar proteinosis are seen:Fatal form: Presenting sholy after bih (congenital PAP)Gradually progressive form: Presenting in older infants & children.Clinical manifestation:Immediately apparent in the newborn period & rapidly leads to respiratory failure.Clinically and radiographically indistinguishable from more common disorders of the newborn that lead to respiratory failure including pneumonia, generalized bacterial infection, respiratory distress syndrome and total anomalous pulmonary venous return with obstruction." "This is a real-world medical entrance exam question, please give the true answer based on the question and selection. Topic name: None, Subject name: Gynaecology & Obstetrics","A 26-year-old primigravida with juvenile myoclonic epilepsy comes to you at 4 months with concern regarding continuing sodium-valproate treatment. Your advice is: Select and explain. A. Add lamotrigine to sodium valproate, B. Taper sodium valproate and add lamotrigine, C. Switch on to carbamazepine, D. Continue sodium valproate with regular monitoring of serum levels",D: Continue sodium valproate with regular monitoring of serum levels,"Answer: D: Continue sodium valproate with regular monitoring of serum levels Explanation: As per ACOG and RCOG guidelines, there is no particular drug of choice for epilepsy in pregnancy Valproate increases chances of birth defects much more than phenytoin, carbamazepine or phenobarbitone and hence if valproate is being used, it should be substituted by a lesser teratogenic drug. Now in this question: A 26-year-old primigravida with juvenile myoclonic epilepsy who has been using valproic acid comes to you at fourth month of pregnancy for advice. Logically speaking if patient has myoclonic epilepsy in non pregnant states-DOC is valproic acid —Harrsion 18/e, p 3266 or lamotrigine, so her physician must have prescribed valproic acid to her. Ideally valproic acid should not be used during pregnancy as it is associated with a high risk of congenital malformations in the fetus. So if this patient would have come in the first trimester, I would have substituted it with some other antiepileptic drug like lamotrigine." "This is a real-world medical entrance exam question, please give the true answer based on the question and selection. Topic name: None, Subject name: Medicine","On laboratory investigations in a patient. pH = 7.3, pCO2 = 35 mm Hg. What is the likely acid base imbalance? Select and explain. A. Respiratory acidosis, B. Metabolic acidosis, C. Metabolic alkalosis, D. Respiratory alkalosis",B: Metabolic acidosis,"Answer: B: Metabolic acidosis Explanation: Ans: B. Metabolic acidosis(Ref: Harrison 19/e p317, 18Ie p365)In the question, pH is decreased (acidosis) & pCO, is decreased (Normal= 40-45 mm Hg).A decreased pCO2 will try to increase pH, hence it must be secondary compensating mechanism.So, the primary mechanism causing the acid base imbalance must be a decrease in serum bicarbonate concentration i.e. metabolic acidosis." "This is a real-world medical entrance exam question, please give the true answer based on the question and selection. Topic name: None, Subject name: Dental","Lingually placed mandibular posteriors will cause all, except Select and explain. A. Gagging, B. Speech difficulties, C. Excessive pressure on ridge, D. Tongue interference",C: Excessive pressure on ridge,"Answer: C: Excessive pressure on ridge Explanation: None" "This is a real-world medical entrance exam question, please give the true answer based on the question and selection. Topic name: None, Subject name: Dental","__________ is helpful in making radicular access after entry: Select and explain. A. Gates glidden drill, B. K-File, C. K-reamer, D. Round bur",A: Gates glidden drill,"Answer: A: Gates glidden drill Explanation: None" "This is a real-world medical entrance exam question, please give the true answer based on the question and selection. Topic name: None, Subject name: Biochemistry","Functions of apoproteins are/is Select and explain. A. Structural component of lipoprotein, B. Enzyme cofactors, C. Enzyme inhibitors, D. All of the above",D: All of the above,"Answer: D: All of the above Explanation: Apolipoproteins carry out several roles: (1) they can form part of the structure of the lipoprotein, for example, apo B; (2) they are enzyme cofactors, for example, C-II for lipoprotein lipase,  A-I  for  lecithin:cholesterol  acyltransferase,  or  enzyme inhibitors, for example, apo A-II and apo C-III for lipoprotein lipase, apo C-I for cholesteryl ester transfer protein; and (3) they Apolipoproteins carry out several roles: (1) they can form part of the structure of the lipoprotein, for example, apo B; (2) they are enzyme cofactors, for example, C-II for lipoprotein lipase,  A-I  for  lecithin:cholesterol" "This is a real-world medical entrance exam question, please give the true answer based on the question and selection. Topic name: None, Subject name: Dental","Cleaning of base metal alloy is done by: Select and explain. A. Warm HCl, B. Cold HF, C. Cold H2SO4, D. Sandblasting with aluminium oxide",D: Sandblasting with aluminium oxide,"Answer: D: Sandblasting with aluminium oxide Explanation: SANDBLASTING Sandblasting is the process by which particles of an abrasive (usually aluminum oxide) is projected at high velocity using compressed air in a continuous stream. The casting is held in a sandblasting machine to clean the remaining investment from the surface of alloy." "This is a real-world medical entrance exam question, please give the true answer based on the question and selection. Topic name: AIIMS 2017, Subject name: Anatomy","Shape of trapezium muscle is Select and explain. A. Triangular, B. Quadrilateral, C. Trapezium, D. Quadrangular",A: Triangular,"Answer: A: Triangular Explanation: *Trapezius is triangular muscle present in upper back region. *Attachments & Origin: Origin: Superior nuchal line, external occipital protuberance, ligamentum nuchae, spinous processes of C7-T12 Inseion: Lateral 1/3rd of clavicle, acromion, spine of scapula. Innervation: Motor - Accessory nerve (CN-11) Proprioception - C3, C4. FUNCTION: Assists in laterally rotating scapula during abduction of humerus above horizontal *Upper fibers - Elevation of scapula *Middle fibers - Adduct scapula *Lower fibers - Depress scapula All fibers together rotates scapula laterally so glenoid cavity projects upwards taking humerus upwards. (Overhead Abduction)" "This is a real-world medical entrance exam question, please give the true answer based on the question and selection. Topic name: None, Subject name: Medicine","In medically intractable seizures, which of the following modalities of treatment has shown the best seizure-free period? Select and explain. A. Epileptic surgery, B. Deep brain stimulation, C. Vagal stimulation, D. Ketogenic diet",A: Epileptic surgery,"Answer: A: Epileptic surgery Explanation: Ans. a. Epileptic surgery (Ref Harrison l9/e p2556. I8/e p3267; Ketogenic diet for epilepsy by R Levy, P Cooper - Cochrune Database Syst Rev, 2003 - Wiley Online Library)Epileptic surgery:Surgery can be extremely effective in substantially reducing seizure frequency and even providing complete seizure control." "This is a real-world medical entrance exam question, please give the true answer based on the question and selection. Topic name: AIIMS 2019, Subject name: Pediatrics","An affected male does not have affected children but an affected female always has affected children. Type of inheritance? Select and explain. A. X linked recessive, B. Autosomal recessive, C. X linked dominant, D. Mitochondrial",D: Mitochondrial,"Answer: D: Mitochondrial Explanation: Mitochondrial inheritance- an affected female passes disease to all her offspring but the affected male can not. Because mitochondrial dna is present in cytoplasm, the child receives cytoplasm and mitochondria of the mother's ovum. Diseases of mitochondrial inheritance- Kearn sayre syndrome Leber hereditary optic neuropathy MELAS (Mitochondrial encephalopathy, lactic acidosis, stroke like syndrome) MERRF (myoclonic epilepsy with ragged red fibres) NARP ( neuropathy ataxia retinitis pigmentosa) CPEO (Chronic progressive external ophthalmoplegia) Pearson syndrome." "This is a real-world medical entrance exam question, please give the true answer based on the question and selection. Topic name: None, Subject name: Psychiatry","Most common complication of ECT is - Select and explain. A. Antegrade amnesia, B. Retrograde amnesia, C. Psychosis, D. Depression",B: Retrograde amnesia,"Answer: B: Retrograde amnesia Explanation: Amnesia is the most common side effect of ECT. Both retrograde and anterograde amnesia are seen, however retrograde amnesia is much more common." "This is a real-world medical entrance exam question, please give the true answer based on the question and selection. Topic name: None, Subject name: Pediatrics","What is seen in severe AS – Select and explain. A. Pulsus bisferiens, B. Late ejection systolic click, C. Heaving shifted apex, D. Loud S2",B: Late ejection systolic click,"Answer: B: Late ejection systolic click Explanation: Signs of severe aortic stenosis Narrow pulse pressure —> Narrower the pulse pressure, more severe the AS. Systolic thrill in the second left intercostal space. Later the peak of ejection systolic murmur, severe the AS. Parodoxically split S. Presence of S3 or S4. ST and T changes on ECG. Cardiomegaly on x-ray." "This is a real-world medical entrance exam question, please give the true answer based on the question and selection. Topic name: AIIMS 2019, Subject name: Medicine","Beta 1,3 Glucan test is positive in all except? Select and explain. A. Aspergillus, B. Candida, C. Mucormycosis, D. Pneumocystis Jirovecii",C: Mucormycosis,"Answer: C: Mucormycosis Explanation: Beta glucan test is PAN investigation for invasive fungal infection with febrile neutropenia. It helps pick up aspergillus, candida, jirovecii infection but poor in mucormycosis identification. Beta 1,3 glucan test used in :MNEMONIC - CAP C- Candida A- Aspergillus P- P.jirovecii" "This is a real-world medical entrance exam question, please give the true answer based on the question and selection. Topic name: None, Subject name: Pathology","If a patient with Raynaud's disease puts his hand in cold water, the hand appears: Select and explain. A. Red, B. Yellow, C. White, D. Blue",C: White,"Answer: C: White Explanation: Raynaud's phenomenon is due to intense vasospasm of peripheral arteries that results in change in colour of fingertips as a response to cold or emotion. It is seen in scleroderma and L.E. One variant of systemic sclerosis is the CREST syndrome, an acronym of the five major findings: calcinosis cutis, Raynaud's phenomenon, oesophageal dysfunction, sclerodactyly, and telangiectasia. This form of the disease is sometimes not as severe as the usual systemic type.  Raynaud syndrome. A vascular spasm in response to cooling, seen in the digits in connective tissue disorders." "This is a real-world medical entrance exam question, please give the true answer based on the question and selection. Topic name: None, Subject name: Pediatrics","A 2-year-old child with fever and barking cough for last 2 days presented to the pediatric emergency at 2.30 am. On examination, respiratory rate is 36/ min, temperature of 39 degC and stridor heard only on crying. No other abnormality is found. What is the next best step in management? Select and explain. A. High-dose dexamethasone, B. Racemic epinephrine nebulization, C. Reassurance, D. Intravenous antibiotics",A: High-dose dexamethasone,"Answer: A: High-dose dexamethasone Explanation: Ans: A. High-dose dexamethasone(Ref Ghai 8/e p376, 398: Nelson 20/e p2032-2034)Diagnostic of laryngotracheobronchitis or croup of mild severity.Hence, high-dose dexamethasone will be the treatment of choice.Treatment:Cornerstone of Treatment: Glucocoicoids & nebulized epinephrinedegGlucocoicoids:Useful in mild, moderate & severe croup.Dexamethasone is most effective coicosteroid. Nebulized epinephrine:Useful in moderate to severe distress.By adrenergic stimulation causes,Constriction of precapillary aerioles.Decreases capillary hydrostatic pressure leading to fluid resorption from interstitium.Improvement in laryngeal mucosal edema.Antibiotics are not indicatedHeliox (mixture of oxygen & helium) has low viscosity & low specific gravity.Allows for greater laminar airflow through respiratory tract.Considered in treatment of children with severe croup." "This is a real-world medical entrance exam question, please give the true answer based on the question and selection. Topic name: None, Subject name: Biochemistry","A patient presents with Von-Gierk's disease and ketosis was detected on investigation. All of the following would he associated findings except: Select and explain. A. There is hypoglycemia, B. Have low blood sugar levels, C. Oxaloacetate is required for gluconeogenesis, D. Low fat mobilization",D: Low fat mobilization,"Answer: D: Low fat mobilization Explanation: Ans. d. Low fat mobilization (Ref Harper 's 27/e p/66)Excessive fatty acid mobilization (not the Low fat mobilization) from adipose tissues leads to ketosis in Von-Gierke's disease. Von Gierke's Disease:Type I glycogen storage disorder, inherited as autosomal recessiveQ, due to the deficiency of glucose-6 phosphataseDeficiency of glucose-6 phosphatase leads to low blood sugar and hypoglycemic attacks.Gluconeogenesis is enhanced, that's why Oxaloacetate is required for gluconeogenesis Excessive fatty acid mobilization from adipose tissues leads to ketosis" "This is a real-world medical entrance exam question, please give the true answer based on the question and selection. Topic name: None, Subject name: Biochemistry","Chylomicron are formed in Select and explain. A. GIT, B. Liver, C. Liver and GIT, D. None of the above",A: GIT,"Answer: A: GIT Explanation: formed CM – in GIT VLDL – in liver HDL – liver & GIT LDL – not formed It is converted from VLDL" "This is a real-world medical entrance exam question, please give the true answer based on the question and selection. Topic name: None, Subject name: Physiology","Potential generated due to movement of freely diffusible ions across a semi-permeable membrane is calculated using: Select and explain. A. Nemst equation, B. Gibbs equation, C. Goldman-Hodgkin-Katz equation, D. Fick principle",A: Nemst equation,"Answer: A: Nemst equation Explanation: Ans: A. Nemst equation(Ref Ganong 25/e9, p24/e Guyton 13ze p61).Nernst equation:Calculates potential generated due to movement of freely diffusible ions across a semi-permeable membrane.Describes ""relation of diffusion potential to ion concentration difference across a membrane""." "This is a real-world medical entrance exam question, please give the true answer based on the question and selection. Topic name: None, Subject name: Social & Preventive Medicine","Mean and standard deviation can be worked out only if data is on Select and explain. A. Nominal scale, B. Dichotomous scale, C. Interval/Ratio scale, D. Ordinal scale",C: Interval/Ratio scale,"Answer: C: Interval/Ratio scale Explanation: Most satisfying scale for measurement of quantities: Metric scale – Mean and SD can only be worked out on: Interval/Ratio scale Measurements are east to handle in: Ratio scale." "This is a real-world medical entrance exam question, please give the true answer based on the question and selection. Topic name: None, Subject name: Surgery","Infection from maxillary first molar drains into Select and explain. A. Submandibular space, B. Infratemporal space, C. Buccal space, D. Infraorbital space",C: Buccal space,"Answer: C: Buccal space Explanation: None" "This is a real-world medical entrance exam question, please give the true answer based on the question and selection. Topic name: None, Subject name: Social & Preventive Medicine","Mean and standard deviation can be worked out only if data is on - Select and explain. A. Interval/Ratio scale, B. Dichotomous scale, C. Nominal scale, D. Ordinal scale",A: Interval/Ratio scale,"Answer: A: Interval/Ratio scale Explanation: Most satisfying scale for measurement of quantities: Metric scale – Mean and SD can only be worked out on: Interval/Ratio scale Measurements are east to handle in: Ratio scale." "This is a real-world medical entrance exam question, please give the true answer based on the question and selection. Topic name: None, Subject name: Pediatrics","Coarctation of aorta is associated with all, except – Select and explain. A. Turner's syndrome, B. Bicuspid aortic valve, C. Pulmonary stenosis, D. Atresia of aortic arch",C: Pulmonary stenosis,"Answer: C: Pulmonary stenosis Explanation: Coarctation of Aorta is a quite common structural abnoimality. Males are affected twice often as females although females with Turner syndrome frequently have a coarctation. Two classic forms of coarctation ofAorta have been described. 1) An infantile form with tubular hypoplasia of the Aortic arch proximal to a P A that is often asymptomatic in childhod. (This is known as coarctation syndrome). 2)An ""adult"" form in which there is discrete ridge like infolding of the Aorta just opposite to closed ductus arteriosus. Although coarctation of Aorta may occur as a solitary defect it is accompanied by - Bicuspid aortic valve in 50% cases and may be associated with -: Congenital Aortic stenosis, ASD, VSD, mitral regurgitation and Berry aneurysm of the circle of willis." "This is a real-world medical entrance exam question, please give the true answer based on the question and selection. Topic name: None, Subject name: Pediatrics","The cause of death in congenital diaphragmatic hernia is – Select and explain. A. Septicaemia, B. Hemorrhage, C. Pulmonary hypoplasia, D. Intestinal obstruction",C: Pulmonary hypoplasia,"Answer: C: Pulmonary hypoplasia Explanation: Most common cause of death in congenital diaphragmatic hernia is                       Pulmonary complications 2""d most common cause of death in congenital diephragmatic hernia in  ---> Intestinal obstruction Congenital diaphrogmatic hernia Also known as ochalek hernia. There is herniation of abdominal contents, i.e. stomach (most of the time), intestine, liver; into thorax through a defect in the diaphragm. Females are affected more than males More common on left side and is posterolateral. Components of CDH are Herniation of abdominal contents into thorax Pulmonary hypoplasia Malrotation of intestine Associated anomalies may be seen 30% of cases -> CNS lesions, esophageal atresia, omphalocele and CVS lesions. Most cases are sporadic. CDH is a reconized part of several chromosomal syndromes ---> Trisomy 21, 18, 13, turner syndrome, Pallister-Killian, Fryn. Clinical features CDH may present as Soon (within 6 hrs) after birth (most of the cases) or After neonatal period (small group) 1)   Soon after birth Respiratory distress is a cardinal sign --> presents as tachypnea, grunting, chest retraction, cyanosis. Scaphoid abdomen Increased chest wall diameter Bowel sounds may be heard in the chest with decreased breath sound. Cardiac impulse is displaced away from the side of hernia. 2)   After neonatal period Vomiting as a result of intestinal obstruction Mild respiratory symptoms Occasionally, incarceration of the intestine will proceed to ischemia with sepsis and shock. Diagnosis Prenatal ultrasound can diagnose CDH between 16 and 14 wk. After delivery chest X-ray and nasal gastric tube is all that is usually required to confirm the diagnosis." "This is a real-world medical entrance exam question, please give the true answer based on the question and selection. Topic name: None, Subject name: Gynaecology & Obstetrics","Sudden hyperflexion of thigh over abdomen (Mc Roberts manoeuvre) with of the following nerve is commonly involved? Select and explain. A. Common peroneal nerve, B. Obturator nerve, C. Lumbosacral trunk, D. Lateral cutaneous Nerve of thigh",D: Lateral cutaneous Nerve of thigh,"Answer: D: Lateral cutaneous Nerve of thigh Explanation: McRoberts’ manoeuvre consists of forcible abduction of patients legs by sharply flexing them on the abdomen. It is the single most effective manoeuvre and should be the first manoeuvre to be performed in case of shoulder dystocia. McRobert’s manoeuvre results in straightening of the sacrum relative to the lumbar vertebra along with rotation of symphysis pubis towards the maternal head and it decreases the angle of pelvic inclination. Sometimes, over zealous use of McRobert’s manoeuvre may result in separation of the maternal pubic symphysis and injury to lateral cutaneous nerve of thigh." "This is a real-world medical entrance exam question, please give the true answer based on the question and selection. Topic name: None, Subject name: Gynaecology & Obstetrics","In which of the following conditions would maternal serum alpha-fetoprotein values be the highest: Select and explain. A. Down's syndrome, B. Omphalocele, C. Gastroschisis, D. Spina bifida occulta",C: Gastroschisis,"Answer: C: Gastroschisis Explanation: Friends, many questions are asked on serum alpha fetoprotein, therefore basic knowledge of this protein is quite vital. Alpha Fetoprotein: It is a glycoprotein synthesized by the fetal yolk sac in the early weeks of gestation and by the gastrointestinal tract and liver later. It is the most abundant protein in the fetal serum. It circulates in fetal serum and passes into fetal urine and amniotic fluid. Concentration of AFP increases steadily in fetal serum till 13 weeks, (3 mg/ml) after which the level rapidly decreases throughout the rest of pregnancy. AFP level in fetal serum declines following birth and by one year of age, its concentration is 1 ng/ml which persists throughout life. AFP passes from the fetus to amniotic fluid when fetus passes urine. It passes into the maternal serum by diffusion across the placental membranes and via placental circulation and is found in steadily increasing quantities in maternal serum after 12 weeks. These are the usual ways of entry of serum alfa fetoprotein in maternal serum but serum alfa fetoprotein can find its way in maternal serum in other ways too. Open fetal body wall defects uncoverd by integument permit allows additional AFP to leak into the amniotic fluid and thus maternal serum AFP are increased. This is the reason for increase in serum alpha fetoprotein in neural tube defects and ventral wall defects. Maternal screening is done between 15-20 weeks (according to Williams 23/e, p 289, Fernando Arias 3/e, p 58). It is measured in nanograms per ml and reported as a multiple of the median (MOM). MSAFP of 2.5 MOM is considered as the upper limit of normal (for twin pregnancy it is 3.5 MOM). Now after having this basic knowledge lets have a look at the question. In the question alpha fetoprotein will be increased in the following conditions. Gastroschisis – Ventral wall defect Omphalocele – Ventral wall defect Spina bifida occulta – Neural tube defect In Down’s syndrome - AFP levels are decreased. Spina bifida occulta: Spina bifida occulta is usually a small, clinically asymptomatic defect, covered by skin, so there are less chances of mixing of fetal serum and maternal serum. Therefore the maternal serum alphafetoprotein level usually does not increase in spina bifida occulta. “In the fetus with a defect such as anencephaly or spina bifida, AFP enters the amniotic fluid in increased amounts, leading to higher levels in the maternal serum as well. Levels of AFP are elevated in amniotic fluid and maternal serum only when such lesions are “open,” i.e., when the neural tissue is exposed or covered by only a thin membrane. When NTDs are skin-covered, AFP does not escape from the fetal circulation, and such defects are generally not detected by maternal serum AFP (MSAFP) screening”. USG in Obs. and Gyane by Callen 4/e, p 25 Omphalocele:  Fernando Arias 3/e, p 84 It is a midline defect of the anterior abdominal wall characterized by herniation of the abdominal viscera into the base of the umbilical cord. The protruding organs are typically covered by a thin aminoperitoneal membrane. Omphalocele has a strong association with high levels of maternal serum alpha-fetoprotein because the ventral wall defect allows mixing of fetal and maternal circulation. Gastroschisis: Fernando Arias 2/e, p 83 Gastroschisis is a paraumbilical defect of the anterior abdominal wall, through which abdominal viscera herniates. The defect is usually located on the right side of the cord insertion and compromises the full thickness of the abdominal wall. There is no sac or membrane covering the herniated organs. This defect is associated with high alpha feto protein titre. Both gastroschisis and omphalocele are ventral wall defects containing abdominal organs and both are associated with high alphafetoprotein level in maternal serum. But it is likely that alphafetoprotein level will be higher in patients with gastroschisis as there is no sac or membrane which covers the herniated organs in this defect. So there is more possibility of fetoprotein leak into the maternal serum or amniotic fluid in Gastroschisis. The answer is further supported by the following graph from USG in Obs. and Gynae by Callen 4/e, p 28" "This is a real-world medical entrance exam question, please give the true answer based on the question and selection. Topic name: None, Subject name: Dental","Increased size of filler particle in composite: Select and explain. A. Increases strength, B. Decreases surface roughness, C. Increases exposure time, D. decreases viscosity",A: Increases strength,"Answer: A: Increases strength Explanation: None" "This is a real-world medical entrance exam question, please give the true answer based on the question and selection. Topic name: None, Subject name: Pharmacology","In iron poisoning drug Desferoxamine is given.what is its mechanism of action - Select and explain. A. By binding with trivalent Fe3+, B. -by inhibiting Hepcidin, C. -by inhibiting DMT1, D. -by inhibiting ferropoin",A: By binding with trivalent Fe3+,"Answer: A: By binding with trivalent Fe3+ Explanation: Ans. A. By binding with trivalent Fe3+Deferoxamine works in treating iron toxicity by binding trivalent (ferric) iron (for which it has a strong affinity), forming ferrioxamine, a stable complex which is eliminated the kidneys.100 mg of deferoxamine is capable of binding approximately 8.5 mg of trivalent (ferric) iron.Deferoxamine works in treating aluminum toxicity by binding to tissue-bound aluminum to form aluminoxamine, a stable, water-soluble complex.The formation of aluminoxamine increases blood concentrations of aluminum, resulting in an increased concentration gradient between the blood and dialysate, boosting the removal of aluminum during dialysis.100 mg of deferoxamine is capable of binding approximately 4.1 mg of aluminum." "This is a real-world medical entrance exam question, please give the true answer based on the question and selection. Topic name: None, Subject name: Surgery","First local anaesthesia to be used clinically was Select and explain. A. Cocaine, B. Bupivacaine, C. Procane, D. Lignocaine",A: Cocaine,"Answer: A: Cocaine Explanation: None" "This is a real-world medical entrance exam question, please give the true answer based on the question and selection. Topic name: None, Subject name: Dental","In universal pro-taper retreatment file end cutting tip is seen in? Select and explain. A. D-1, B. D-2, C. D-3, D. None of the above",A: D-1,"Answer: A: D-1 Explanation: None" "This is a real-world medical entrance exam question, please give the true answer based on the question and selection. Topic name: None, Subject name: Pharmacology","The drug which is used to treat laryngospasm is Select and explain. A. Atropine, B. Diazepam, C. Neostigmine, D. Succinylcholine",D: Succinylcholine,"Answer: D: Succinylcholine Explanation: None" "This is a real-world medical entrance exam question, please give the true answer based on the question and selection. Topic name: None, Subject name: Physiology","The best stimulator of respiratory centre is: Select and explain. A. Decrease in arterial carbon dioxide level, B. Increase in arterial carbon dioxide level, C. Increase in arterial oxygen level, D. None of the above",B: Increase in arterial carbon dioxide level,"Answer: B: Increase in arterial carbon dioxide level Explanation: None" "This is a real-world medical entrance exam question, please give the true answer based on the question and selection. Topic name: None, Subject name: Pediatrics","A 10 year old girl presented with fever convulsions, neck rigidity. CSF findings are protein 150 mg, sugar 40 mg, chloride 2500 mg with lymphadenopathy – Select and explain. A. Viral meningitis, B. Pyogenic meningitis, C. Cryptococcal meningitis, D. Tuberculous meningitis",D: Tuberculous meningitis,"Answer: D: Tuberculous meningitis Explanation: Typical signs of meningitis (Fever, convulsion and neck rigidity) with increased CSF protein and decreased sugar level and decreased chloride level associated with lymphadenopathy suggest the diagnosis of tubercular meningitis." "This is a real-world medical entrance exam question, please give the true answer based on the question and selection. Topic name: None, Subject name: Pathology","A 7-year-old child having yellowish discoloured spot of maxillary central incisor. His mother presents a history of injury to deciduous tooth 3 years back with recurrent infections & swelling. The diagnosis is Select and explain. A. Turners hypoplasia, B. Generalised dental fluorosis, C. Syphilitic hypoplasia, D. Rickets hypomineralisation",A: Turners hypoplasia,"Answer: A: Turners hypoplasia Explanation: None" "This is a real-world medical entrance exam question, please give the true answer based on the question and selection. Topic name: None, Subject name: Dental","Which of the following structures is not found in a living pulp? Select and explain. A. Collagen fibres, B. Haversian fibres, C. Non-myelinated nerve fibres, D. Reticulum",B: Haversian fibres,"Answer: B: Haversian fibres Explanation: None" "This is a real-world medical entrance exam question, please give the true answer based on the question and selection. Topic name: None, Subject name: Physiology","When a person lies down : Select and explain. A. There is immediate increase in venous return, B. Heart rate becomes stable at slightly more than normal, C. Cerebral blood flow increases & becomes stable at more than normal, D. Blood flow to the Apex of lung decreases",A: There is immediate increase in venous return,"Answer: A: There is immediate increase in venous return Explanation: None" "This is a real-world medical entrance exam question, please give the true answer based on the question and selection. Topic name: None, Subject name: Pediatrics","Long term complication of infants born to I.D.D.M mother AJE – Select and explain. A. DM, B. Ketotic hypoglycemia, C. Obesity, D. Blindness",B: Ketotic hypoglycemia,"Answer: B: Ketotic hypoglycemia Explanation: There is nonketotic hypoglycemia." "This is a real-world medical entrance exam question, please give the true answer based on the question and selection. Topic name: None, Subject name: Dental","Dark points on teeth along with tooth destruction of 4 years old patient Select and explain. A. ECC, B. Staining, C. Fluorosis, D. Deep caries",A: ECC,"Answer: A: ECC Explanation: None" "This is a real-world medical entrance exam question, please give the true answer based on the question and selection. Topic name: None, Subject name: Surgery","The incision used for treatment of traumatic telecanthus is Select and explain. A. Bicoronal incision, B. Upper blepharoplasty, C. Infraorbital incision, D. Preauricular incision",A: Bicoronal incision,"Answer: A: Bicoronal incision Explanation: None" "This is a real-world medical entrance exam question, please give the true answer based on the question and selection. Topic name: None, Subject name: Pharmacology","Which of the following drugs is not used in detoxifica-tion of chronic alcoholics? Select and explain. A. Flumazenil, B. Disulfiram, C. Acamprosate, D. Naltrexone",A: Flumazenil,"Answer: A: Flumazenil Explanation: Ans: A. Flumazenil(Ref: Goodman Gilman 12/e p468; Kaaung 13/e p3 7 7, 394, 12/e p381, 399; KDT 6/e p385; Harrison 19/e p2727, 8/c 1)2727)Flumazenil:GABAa receptor antagonist.Used intravenously to treat benzodiazepine overdoses.Help reverse anesthesia.Not used in alcohol detoxification.Rest given drugs have a role in alcohol detoxification." "This is a real-world medical entrance exam question, please give the true answer based on the question and selection. Topic name: None, Subject name: Ophthalmology","What is the angle subtended by the largest letter in the Snellen cha on a person's eye who is reading it from a distance of 6 meters? Select and explain. A. 1 minute, B. 10 minutes, C. 50 minutes, D. 60 minutes",C: 50 minutes,"Answer: C: 50 minutes Explanation: Ans: C. 50 minutes(Ref: Yanoff & Duker 4/e p42; Parson 22/e p98, 21/e p98-100)The angular spacing between the bars of the C or E in Snellen's cha is 1 minute for the 6/6 letter (smallest letter).The largest letter on the Snellen's cha is the 6/60 letter.When viewed from a distance of 6 meter, this letter subtends an angle of 50 minutes in the eye and the bars of the letter subtend an angle of 10 minutes." "This is a real-world medical entrance exam question, please give the true answer based on the question and selection. Topic name: None, Subject name: Microbiology","Which of the following enzyme is required to induce the formation of a plasma clot pathogenic staphylococci Select and explain. A. Lysozyme, B. Fibrinolysin, C. Coagulase, D. S protein",C: Coagulase,"Answer: C: Coagulase Explanation: None" "This is a real-world medical entrance exam question, please give the true answer based on the question and selection. Topic name: None, Subject name: Pathology","Patient giving history of thrombocytopenic purpura reports for extraction. What could be the most common postoperative complication? Select and explain. A. Oedema, B. Haemorrhage, C. Infection spreading through tissue spaces, D. Dry socket",B: Haemorrhage,"Answer: B: Haemorrhage Explanation: None" "This is a real-world medical entrance exam question, please give the true answer based on the question and selection. Topic name: None, Subject name: Gynaecology & Obstetrics","All are seen with scar dehiscence, except: Select and explain. A. Maternal bradycardia, B. Fetal bradycardia, C. Vaginal bleeding, D. Hematuria",A: Maternal bradycardia,"Answer: A: Maternal bradycardia Explanation: Uterine rupture typically is classified as either complete (all layers of the uterine wall separated) or incomplete (uterine muscle separated but visceral peritoneum is intact). Incomplete rupture is commonly referred to as scar dehiscence. Scar dehiscence is an intraoperative finding always. The greatest risk factor for either complete or incomplete uterine rupture is prior cesarean delivery. Following uterine rupture the most common electronic fetal monitoring finding is sudden, severe heart rate decelerations that may evolve into late decelerations, bradycardia, and undetectable fetal heart action. In some cases in which the fetal presenting part has entered the pelvis with labor, loss of station may be detected by pelvic examination. If the fetus is partly or totally extruded from the site of uterine rupture, abdominal palpitation or vaginal examination may be helpful to identify the presenting part, which will have moved away from the pelvic inlet. A firm contracted uterus may at times be felt alongside the fetus. With rupture and expulsion of the fetus into the peritoneal cavity, the chances for intact fetal survival are dismal, and reported morality rates range from 50% to 75%. Clinical features of Ruptured Uterus:" "This is a real-world medical entrance exam question, please give the true answer based on the question and selection. Topic name: None, Subject name: Dental","An average diameter of apical foramen in patients of age above 55 years is Select and explain. A. 400 micrometer, B. 502 micrometer, C. 681 micrometer, D. 602 micrometer",C: 681 micrometer,"Answer: C: 681 micrometer Explanation: None" "This is a real-world medical entrance exam question, please give the true answer based on the question and selection. Topic name: None, Subject name: Dental","Mesial contact area of the permanent canine is at the: Select and explain. A. Middle third, B. Junction of the incisal and middle thirds, C. Junction of the middle and cervical thirds, D. None of the above",B: Junction of the incisal and middle thirds,"Answer: B: Junction of the incisal and middle thirds Explanation: None" "This is a real-world medical entrance exam question, please give the true answer based on the question and selection. Topic name: None, Subject name: Physiology","The pathway of pain from teeth and temperature is carried by: Select and explain. A. Corlicospinal tract, B. Corticocerebral tract, C. Lateral spinothalamic tract, D. Ventral spinothalamic tract",C: Lateral spinothalamic tract,"Answer: C: Lateral spinothalamic tract Explanation: None" "This is a real-world medical entrance exam question, please give the true answer based on the question and selection. Topic name: None, Subject name: Dental","Which of these is a qualitative test for bacteria: Select and explain. A. Snyder test, B. Allen's test, C. Dentocult test, D. Salivary reductase test",D: Salivary reductase test,"Answer: D: Salivary reductase test Explanation: None" "This is a real-world medical entrance exam question, please give the true answer based on the question and selection. Topic name: None, Subject name: Surgery","The radiographic feature of sinusitis includes Select and explain. A. Fluid Levels, B. Erosion of bone, C. Clouding of antra, D. Clouding and fluid level",D: Clouding and fluid level,"Answer: D: Clouding and fluid level Explanation: None" "This is a real-world medical entrance exam question, please give the true answer based on the question and selection. Topic name: None, Subject name: Medicine","Acquired cause of pure red cell aplasia are all except: Select and explain. A. ABO incompatibility in bone marrow transplantation, B. Lymphoma, C. Drug induced-NSAIDs, D. Chromosome 5q deletion syndrome (5q monosomy)",C: Drug induced-NSAIDs,"Answer: C: Drug induced-NSAIDs Explanation: Answer- C. Drug induced-NSAIDsNSAIDs are not the cause of Acquired pure red cell aplasia.Acquired pure red cell aplasia:Cancer: Thymoma, Lymphoid malignancies, Paraneoplastic to solid tumors.Drugs: Phenytoin, azathioprine, Chloramphenicol, Procainamide, isoniazid" "This is a real-world medical entrance exam question, please give the true answer based on the question and selection. Topic name: None, Subject name: Anatomy","All of the following cells are of endodermal origin except: Select and explain. A. Epithelial cells of intestine, B. Odontoblasts, C. Goblet cells, D. Hepatocytes",B: Odontoblasts,"Answer: B: Odontoblasts Explanation: Endodermal derivatives: Epithelial lining of respiratory tract, lungs, gut, bladder,part of urethra; parenchymal cells of tonsils,thymus, thyroid, parathyroid, liver, pancreas; epithelial lining of tympanic cavity and auditory tube. Odontoblasts are derived from neural crest derived ectomesenchyme." "This is a real-world medical entrance exam question, please give the true answer based on the question and selection. Topic name: None, Subject name: Surgery","A middle-aged female presented with recurrent bloody diarrhea. Colonoscopy reveals multiple geographic ulcers and histopathological examination is shown below. What is the likely diagnosis? Select and explain. A. Crohn's disease, B. Adenocarcinoma colon, C. Pseudomembranous colitis, D. Ulcerative colitis",A: Crohn's disease,"Answer: A: Crohn's disease Explanation: Answer- A. Crohn's diseaseThe microscopic features of active Crohn disease include abundant neutrophils that infiltrate and damage crypt epithelium. Clusters of neutraphils within a crypt are referred to as crypt abscesses and are often associated with crypt destruction Ulceration is common in Crohn disease." "This is a real-world medical entrance exam question, please give the true answer based on the question and selection. Topic name: None, Subject name: ENT","CA maxillary sinus stage III (T3 No Mo), treatment of choice is / Ca maxillary sinus is treated by - Select and explain. A. Radiotherapy, B. Surgery + Radiotherapy, C. Chemotherapy, D. Chemotherapy + Surgery",B: Surgery + Radiotherapy,"Answer: B: Surgery + Radiotherapy Explanation: None" "This is a real-world medical entrance exam question, please give the true answer based on the question and selection. Topic name: None, Subject name: Dental","About polymerization shrinkage of composite all are true, except: Select and explain. A. Polymerization shrinkage is greater if bonded surface area is lesser than unbounded surface area, B. Polymerization shrinkage is high if within the enamel margins, C. Acid etching and priming will decrease polymerization shrinkage, D. Microleakage can occur because of polymerization shrinkage",A: Polymerization shrinkage is greater if bonded surface area is lesser than unbounded surface area,"Answer: A: Polymerization shrinkage is greater if bonded surface area is lesser than unbounded surface area Explanation: None" "This is a real-world medical entrance exam question, please give the true answer based on the question and selection. Topic name: None, Subject name: Physiology","Within which parts of a gastric gland are chief cells located Select and explain. A. Fundus, B. Isthmus, C. Neck, D. Gastric pit",A: Fundus,"Answer: A: Fundus Explanation: Fundic glands (chief cells, parietal & mucous neck cells) are located in body and fundus of stomach." "This is a real-world medical entrance exam question, please give the true answer based on the question and selection. Topic name: AIIMS 2019, Subject name: Physiology","Feed forward mechanism.. Select and explain. A. Feeling thirsty while walking in hot temperature, B. Shivering on exposure to cold temperature, C. Salivation on smelling food, D. Increase blood pressure during supine posture",C: Salivation on smelling food,"Answer: C: Salivation on smelling food Explanation: Feed forward mechanism Feedback mechanism Controller anticipates changes & takes a desired action. No time lag present Examples: - Cephalic phase of gastric acid secretion Increase ventilatory drive in exercise. Change occur in controlled variable & that change is feedback to controller & then the controller takes action Time lag is present. Type: Negative feed back - Kidney body fluid mechanism - Temperature regulation - Baroreceptor mechanism Positive feed back - Circulatory shock - Oxytocin in paurition - Platelet plug / clot formation - LH surge leading to ovulation - Bladder filling to micturition" "This is a real-world medical entrance exam question, please give the true answer based on the question and selection. Topic name: None, Subject name: Microbiology","A 15 years old boy presents with loose motions and intermittent abdominal pain over the past 1 year. Wet mount stool specimen showed the presence of multiple ova, which are more than 100 p in diameter. The causative organism for the disease shall not include: Select and explain. A. Fasciola gigantica, B. Gastrodiscoides hominis, C. Echinostoma ilocanum, D. Opistorchis viverrini",D: Opistorchis viverrini,"Answer: D: Opistorchis viverrini Explanation: Answer- D (Opistorchis viverrini)Inhabit the bile duct of fish eating animalsCaused by ingestion of raw or under cooked freshwater fishAbdominal discomfo, gasFatigue is typicalAssociated with bile duct cancer" "This is a real-world medical entrance exam question, please give the true answer based on the question and selection. Topic name: None, Subject name: Biochemistry","Best method to neasure HbA1c? Select and explain. A. lsoelectric focusing, B. Affinity chromatography, C. Ion exchange chromatography, D. Electrophoresis",C: Ion exchange chromatography,"Answer: C: Ion exchange chromatography Explanation: Ans: C. Ion exchange chromatographyRef: Tietz T*tbook of clinical chemistry and molecular biologt, lh ed^, pg. 1443-1444Techniques based on charge differences (ion-exchange chromatography, HPLC, electrophoresis, and isoelectric focusing), structural differences (affinity chromatography and immunoassay), or chemical analysis (photometry and spechophotometry)." "This is a real-world medical entrance exam question, please give the true answer based on the question and selection. Topic name: None, Subject name: Gynaecology & Obstetrics","All of the following are true regarding Duncan placental separation except: Select and explain. A. Most common method of placental separation, B. Maternal side of the placenta presents at the vulva, C. Separation stas from the periphery, D. Blood collects between the placenta and fetal membranes and escapes through vagina",A: Most common method of placental separation,"Answer: A: Most common method of placental separation Explanation: Answer- A (Most common method of placental separation)Less common than Schultze methodMaternal side of the placenta presents at the vulvaSeparation stas from the peripheryBlood escapes through vagina" "This is a real-world medical entrance exam question, please give the true answer based on the question and selection. Topic name: None, Subject name: Dental","8 year old child had fractured his maxillary central incisor 10 months ago. The pulp shows no response. There is no periapical lesion in the radiograph. The treatment of choice is Select and explain. A. Ca(OH)2 Pulp capping, B. Formocresol pulpotomy, C. Conventional root canal treatment, D. Complete debridement and apexification",D: Complete debridement and apexification,"Answer: D: Complete debridement and apexification Explanation: None" "This is a real-world medical entrance exam question, please give the true answer based on the question and selection. Topic name: None, Subject name: Surgery","Sublingual space is divided from submandibular space by: Select and explain. A. Fibres of mylohyoid, B. Submandibular gland, C. Body of mandible, D. Geniohyoid",A: Fibres of mylohyoid,"Answer: A: Fibres of mylohyoid Explanation: None" "This is a real-world medical entrance exam question, please give the true answer based on the question and selection. Topic name: None, Subject name: Surgery","The most characteristic feature of basal cell carcinoma is: Select and explain. A. Keratin pearls, B. Foam cells, C. Nuclear palisading, D. Psammoma bodies",C: Nuclear palisading,"Answer: C: Nuclear palisading Explanation: Ans. c. Nuclear palisadingThe most characteristic feature of basal cell carcinoma is nuclear palisading.The characteristic finding is of ovoid cells in nests with a single 'palisading' layer. lt is only the outer layer of cells that actively divide, explaining why tumor growth rates are slower than their cell cycle speed would suggest, and why incompletely excised lesions are more aggressive.Morphoeic BCCs synthesis. size type 4 collagenase and so spread rapidly." "This is a real-world medical entrance exam question, please give the true answer based on the question and selection. Topic name: None, Subject name: Surgery","All the following are seen in thyrotoxicosis except Select and explain. A. Tremor, B. Increased temperature, C. Slow pulse rate, D. Exophthalmos",C: Slow pulse rate,"Answer: C: Slow pulse rate Explanation: None" "This is a real-world medical entrance exam question, please give the true answer based on the question and selection. Topic name: AIIMS 2019, Subject name: Gynaecology & Obstetrics","Staging of GTN with lung metastasis. Select and explain. A. Stage 1, B. Stage 2, C. Stage 3, D. Stage 4",C: Stage 3,"Answer: C: Stage 3 Explanation: Stage 1 Disease confined to uterus Stage 2 GTN extending outside uterus but limited to genital structure (adnexa, vagina, broad ligament) Stage 3 GTN extending to lungs with or without known genital tract involvement Stage 4 All other metastatic sites 0 1 2 4 Age (years) <= 39 > 39 Antecedent pregnancy Hydatidiform mole Aboion Term Interval between end of antecedent pregnancy sta of chemotherapy (month) <4 4-6 7-12 >12 Human chronic gonadotropin (IU/L) at the time of GTN diagnosis <103 103 - 104 104 - 105 >105 ABO groups 0 or A B or AB Largest tumor, including uterine (cm) <3 3 - 5 > 5 Site of metaslases Spleen, kidney GI tract Brain, liver Number of metastases 1 drug >= 2 drug" "This is a real-world medical entrance exam question, please give the true answer based on the question and selection. Topic name: AIIMS 2017, Subject name: Biochemistry","Glucosamines used in following condition: Select and explain. A. Ahritis, B. Niemann pick disease, C. Alzheimer's disease, D. Cancer",A: Ahritis,"Answer: A: Ahritis Explanation: option a - Glucosamine is used in ahritis GAGs - it's a tandem repeat of amino sugar + Uronic acid GAGs + Protein - Proteoglycan GAGs + Protein - proteoglycan Major Proteoglycan present in cailage is called Aggrecan That's why Glucosamine is used along with chondroitin sulphate for ahritis. GAGs SIGNIFICANT FEATURES Hyaluronic acid Longest GAG No sulfate present EXCEPTION: GAG which is non-covalently attached to other proteoglycans in ECM. (Whereas mostly GAGs are attached to proteins) Chondroitin sulfate Most abundant (a very large molecule) Major component of cailage Also present in cornea (mainly keratan sulfate) Keratan sulfate Most heterogenous GAGs - Keratin Sulfate I and II (because they contain additional monosaccharides such as L-Fucose, Mannose and NANA) No Uronic Acid present Responsible for corneal transparency Mainly present in cornea Dermatan sulphate Most widely distributed GAG relatively smaller GAG Binds LDL and has role in Atherosclerosis GAG synthesized mainly by aerial Smooth Muscle Cells Heparin Highest Negative Charge Intracellular GAG (mostly GAGs are extracellular) Heparan sulphate Role in retinal cell cell attachment Cell cell adhesion Mucin Clot Test/ Rope Test --> If few drops of Synol fluid is added to Acetic Acid, clot is formed due to polymerization of Hyaluronic Acid. Poor clot formation occurs in Rheumatoid Ahritis, Septic Ahritis, Gouty Ahritis (inflammatory conditions)." "This is a real-world medical entrance exam question, please give the true answer based on the question and selection. Topic name: None, Subject name: Dental","After removal of periodontal dressing what should be done to keep sensitivity to the minimum? Select and explain. A. Use of mouthwash, B. Use of desensitizing paste, C. Keep roots, plaque free, D. None of the above","C: Keep roots, plaque free","Answer: C: Keep roots, plaque free Explanation: None" "This is a real-world medical entrance exam question, please give the true answer based on the question and selection. Topic name: None, Subject name: Gynaecology & Obstetrics","All of the following increase at full term in pregnancy except: Select and explain. A. Minute volume, B. GFR, C. Blood volume, D. Cardiac output",D: Cardiac output,"Answer: D: Cardiac output Explanation: Ans: D. Cardiac output(Ref Dutta 8/e p60-61, 7/e p51-53; Williams 24/e p59)Cardiac output increases from the fifth week of pregnancy and reaches its maximum levels at approximately 32 weeks, after which there is only a slight increase until labor, delivery, and the postpaum period" "This is a real-world medical entrance exam question, please give the true answer based on the question and selection. Topic name: None, Subject name: Pathology","Multiple punched out lesions are seen in: Select and explain. A. Paget's disease, B. Osteosarcoma, C. Ewing sarcoma, D. Multiple myeloma",D: Multiple myeloma,"Answer: D: Multiple myeloma Explanation: None" "This is a real-world medical entrance exam question, please give the true answer based on the question and selection. Topic name: None, Subject name: Dental","Tooth structure cutting is preceded by crack propogation due to tensile forces known as: Select and explain. A. Brittle fracture, B. Ductile fracture, C. Shear fracture, D. All of the above",A: Brittle fracture,"Answer: A: Brittle fracture Explanation: Tooth structure is cut by two types of fracture Brittle fracture Tensile fracture Tensile fracture proceeds by plastic deformation due to shear forces." "This is a real-world medical entrance exam question, please give the true answer based on the question and selection. Topic name: AIIMS 2018, Subject name: Forensic Medicine","According to POCSO, all of the following are aggravated sexual offence except: Select and explain. A. Gang Rape, B. Rape by threatening, C. Rape by police officer, D. Rape during communal violence",B: Rape by threatening,"Answer: B: Rape by threatening Explanation: POCSO- Protection Of Children from Sexual Offences, 2012 Objective- To prevent the children from various froms of Child Sexual Abuse According to POCSO- Child- Any person below 18 Sexual harassment- Sexual remarks , Sexual our requests, involvement of child in pornography. Sexual assault- If a person touches the body pa of the child or any act done with sexual intent. (There is no penetration) Penetrative Sexual Assault- Penetration (Penovaginal, Peno oral Peno anal), Fingering, foreign body inseion Sexual assault and penetrative sexual assault is aggravated if- it is done by police officer, army officer, public servent, a person of trust, authority management, staff of institution. It is done on child of less than 12 years of age. It is commited by a gang It is commited during communal riots or communal violence" "This is a real-world medical entrance exam question, please give the true answer based on the question and selection. Topic name: None, Subject name: Surgery","A 45 year old woman has a nodule in the neck which moves during swallowing. The nodule most probably is attached to: Select and explain. A. Thyroid, B. Larynx, C. Oesophagus, D. Pharynx",A: Thyroid,"Answer: A: Thyroid Explanation: None" "This is a real-world medical entrance exam question, please give the true answer based on the question and selection. Topic name: None, Subject name: Pharmacology","Benzodiazepines are true in: Select and explain. A. Produce distortion in sleep more than other sedatives, B. More safe than other sedatives when take in larger amounts, C. After metabolism of other drugs in liver, D. All have metabolically active substrates",B: More safe than other sedatives when take in larger amounts,"Answer: B: More safe than other sedatives when take in larger amounts Explanation: None" "This is a real-world medical entrance exam question, please give the true answer based on the question and selection. Topic name: None, Subject name: Dental","An example of acute inflammatory gingival enlargement is most likely to be seen in patient exhibiting: Select and explain. A. Periodontal cyst, B. Pregnancy gingivitis, C. Dilantin hyperplasia, D. Hereditary fibromatosis",B: Pregnancy gingivitis,"Answer: B: Pregnancy gingivitis Explanation: None" "This is a real-world medical entrance exam question, please give the true answer based on the question and selection. Topic name: None, Subject name: Medicine","Intake of unsaturated fatty acid is associated with: Select and explain. A. Increased risk of cardiovascular disease, B. Decreased risk of cardiovascular disease, C. Increased risk of GI disturbances, D. Decreased risk of GI disturbances",B: Decreased risk of cardiovascular disease,"Answer: B: Decreased risk of cardiovascular disease Explanation: Diets deficient in fresh fruit, vegetables and polyunsaturated fatty acids are associated with an increased risk of cardiovascular disease. The introduction of a Mediterranean-style diet reduces cardiovascular events. However, dietary supplements, such as vitamins C and E, beta-carotene, folate and fish oils, do not reduce cardiovascular events and, in some cases, have been associated with harm. Reference: : Davidson P R I N C I P L E S   and Practice O F M E D I C I N E 23rd edition page no  486" "This is a real-world medical entrance exam question, please give the true answer based on the question and selection. Topic name: None, Subject name: Surgery","A 14 year old boy has delayed eruption of the second molar. Radiography shows a dentigerous cyst surrounding the crown of the tooth. The treatment of choice is: Select and explain. A. Extraction of the molar, B. Aspiration of the cyst, C. Observe, D. Expose the crown and keep it exposed",D: Expose the crown and keep it exposed,"Answer: D: Expose the crown and keep it exposed Explanation: None" "This is a real-world medical entrance exam question, please give the true answer based on the question and selection. Topic name: None, Subject name: Pediatrics","6 yr old child H/O fever 5 days back which is now afebrile with normal muscle mass, tone and reflex, no neurological deficit but pain on palpating muscles and CPK levels 2000, diagnosis is Select and explain. A. GBS, B. Dermatomyositis, C. DMD, D. Acute viral myositis",D: Acute viral myositis,"Answer: D: Acute viral myositis Explanation: Ans.D. Acute viral myositisThe clinical picture suggests the most probable diagnosis is Viral myositis. The specific history of URI suggests influenza myositis. Infectious myositis has a male predominance and are typically seen in young adults. The typical presentation of the childhood form includes fever, malaise, and rhinorrhea which is usually followed in 1-7 days by severe pain, especially in the calves. The muscle pain is usually worse with movement and the symptoms last for about a week. Muscle weakness, tenderness, and swelling are usually seen with more severity in adults and the Proximal muscles are affected predominantly.In children, toe walking and wide-based gait may be seen because of the involvement of the gastrocnemius-soleus muscles. Lab features of influenza myositis includeElevated CK may be as high as 500 times normalUrine myoglobin is usually positive" "This is a real-world medical entrance exam question, please give the true answer based on the question and selection. Topic name: None, Subject name: Pathology","MHC antigens are absent on? Select and explain. A. Platelet, B. Erythrocyte, C. Neutrophil, D. Monocyte",B: Erythrocyte,"Answer: B: Erythrocyte Explanation: Ans: B. Erythrocyte(Ref: Ananthanarayan 10/e p141-143, 8/e p 132-135)MHC antigens:Absent on erythrocytes.HLA antigens:Class 1 antigens (A, B and C) - Found on surface of viually all nucleated cells.Principal antigens.Involved in graft rejection & cell-mediated cytolysis.Function as components of hormone receptors.HLA class II antigens:More restricted in distribution.Found only on cells of immune system-macrophages, dendritic cell activated T cells & mainly on B cells." "This is a real-world medical entrance exam question, please give the true answer based on the question and selection. Topic name: None, Subject name: Dental","The cohesion of direct gold filling at room temperature is an example of: Select and explain. A. Wedging, B. Annealing, C. Adaptability, D. Atomic attraction",D: Atomic attraction,"Answer: D: Atomic attraction Explanation: None" "This is a real-world medical entrance exam question, please give the true answer based on the question and selection. Topic name: None, Subject name: Social & Preventive Medicine","92. A clinical trail was conducted with 15225 hypeensive patients alloted in the intervention group (New drug) and control group (Old drug) respectively. Results of the research study are given in the following table. Calculate the absolute risk reduction (ARR) and relative risk (RR): Control group (old drug)Intervention group (new drug)Developed HT complications18001620Did not develop HT complications1342513605Total subjects1522515225 Select and explain. A. ARR = 10% and RR = 0.9, B. ARR = 1% and IZR = 9, C. ARR = 1% and RR = 0.9, D. ARIZ = 10% and RR = 9",C: ARR = 1% and RR = 0.9,"Answer: C: ARR = 1% and RR = 0.9 Explanation: Ans. C. ARR = 1% and RR = 0.9(Ref. Park 24/e p83)In the given situation, absolute risk reduction (ARR) is 1% and relative risk (RR) is 0.9.Relative Risk (RR)Attributable Risk (AR)Population Attributable Risk (PAR)RR=IExp/INon-Exp AR=(IExp-INon-Exp)/INon-ExpX100 AR=(Total-INon-exp)/ITotalX100RR is the ratio of the risk in the exposed divided by the risk in the unexposed.AR indicates the number of cases of a disease among exposed individuals that can be attributed to that exposureIt is excess risk or risk differenceUseful measure of extent of public health problem caused by an exposure PAR indicates the number (or propoion) of cases that would not occur in a population if the factor were eliminated. Control group (old drug)Intervention group (New drug)Developed HT complications18001620Did not develop HT complications1342513605Total subjects1522515225IC = Incidence of events (hypeensive complications) in the control group = 1800/15225 = 0.118 = 11.8%IT = Incidence of events (hypeensive complications) in the treatment group = 1620/15225 = 0.106 = 10.6%Absolute risk reduction (ARR) is also known as risk difference.ARR = IC-IT = 11.8 -10.6 = 1.2%Relative risk (RR) = IT/IC = 0.106/0.118 = 0.9" "This is a real-world medical entrance exam question, please give the true answer based on the question and selection. Topic name: None, Subject name: Anatomy","Lacrimation is affected due to damage to which of the following nerves? Select and explain. A. Nasociliary nerve, B. Greater petrosal nerve, C. Trigeminal nerve, D. Anterior ethmoid nerve",B: Greater petrosal nerve,"Answer: B: Greater petrosal nerve Explanation: Answer- B (Greater petrosal nerve)The greater petrosal nerve is a branch of the facial nerve. It gives secretomotor fibers to the pterygopalatine ganglion along with lesser petrosal nerve. The postganglionic fibers innervate the lacrimal gland and are responsible for the formation of tears.Greater petrosal nerve damage will deprive the lacrimal gland of its secretomotor fibers and this would lead to the absence of tears.Greater petrosal nerve damage can happen in case of intrapetrosal mass lesions, a trauma in case of temporal bone injuries, etc." "This is a real-world medical entrance exam question, please give the true answer based on the question and selection. Topic name: None, Subject name: Pathology","Insulin resistance is seen in liver disease because of- Select and explain. A. Decreased secretion of insulin, B. Hepatic steatosis, C. Hepatocytedysfunction, D. Low 'C'peptide level",B: Hepatic steatosis,"Answer: B: Hepatic steatosis Explanation: Ans. b. Hepatic steatosis (Ref Robbins 9/e p842, 845, 8/e p 1136, http://wwwnews-medical.net/health/What-is-InsulinResistance.aspx )'Insulin resistance may also be caused by the damage of liver cells having undergone a defect of insulin receptors in hepatocytes.''Nonalcoholic fatty liver disease (NAFLD) refers to the presence of hepatic steetosis when no other causes for secondary hepatic fat accumulation (eg. heavy alcohol consumption) are present." "This is a real-world medical entrance exam question, please give the true answer based on the question and selection. Topic name: None, Subject name: Dental","Access cavity shape in mandibular 1st molar with 4 canals Select and explain. A. Trapezoidal, B. Round, C. Oval, D. Triangular",A: Trapezoidal,"Answer: A: Trapezoidal Explanation: None" "This is a real-world medical entrance exam question, please give the true answer based on the question and selection. Topic name: None, Subject name: Gynaecology & Obstetrics","A 25-year-old lady with submucosal fibroid was undergoing myomectomy. The surgeon was using 1.5 yo glycine as irrigating fluid for the cavity. During the surgery the nurse informs the surgeon that there is a 500m1 fluid deficit. What is the next step to be done? Select and explain. A. Stop the'surgery, B. Change the fluid to normal saline, C. Continue the surgery with careful monitoring of fluid status, D. Give furosemide to the patient and continue surgery",C: Continue the surgery with careful monitoring of fluid status,"Answer: C: Continue the surgery with careful monitoring of fluid status Explanation: Ans. C. Continue the surgery with careful monitoring of fluid statusRef. BSGE/ESGE guideline on manage,nent of fluid disrension metlia in operative hysteroscopy.Asymptomatic hypervolemia can be managed by fluid restriction with or without diuretics.Patient should be observed for symptoms of hyponatremia and continued electrolyte monitoring should be done." "This is a real-world medical entrance exam question, please give the true answer based on the question and selection. Topic name: None, Subject name: Social & Preventive Medicine","If the distribution of intraocular pressure (IOP) seen in 100 glaucoma patients has an average 30 mm with an SD of 1, what is the lower limit of the average that can be expected 95% of times - Select and explain. A. 28, B. 26, C. 32, D. 25",A: 28,"Answer: A: 28 Explanation: In the given question, Mean (μ) = 30 mm and SD (σ) = 1.0 mm Thus, 95% values are contained in the range of Mean ± 2 SD (µ ± 2s) or 30 ± 2 (1) So, 95% values are contained in the range 30 – 2 mm and 30 + 2 mm OR between 28 and 30 mm." "This is a real-world medical entrance exam question, please give the true answer based on the question and selection. Topic name: None, Subject name: Dental","Metal oxides are added in porcelain provide all except Select and explain. A. Strength, B. Improve bonding, C. Cosmetic, D. Impart colour",A: Strength,"Answer: A: Strength Explanation: None" "This is a real-world medical entrance exam question, please give the true answer based on the question and selection. Topic name: None, Subject name: Physiology","Anatomical dead space measured by - Select and explain. A. Bohler's method, B. Xenon dilution technique, C. Spirometry, D. Single breath nitrogen test",D: Single breath nitrogen test,"Answer: D: Single breath nitrogen test Explanation: Ans. D. Single breath nitrogen test(Ref: Ganong 25/e p633, 634, 24/e p633, 634)Anatomical dead space - Calculation:By Bohr's equation - Uses single breath nitrogen inhalation technique.Xenon/Helium dilution technique:Used to measure functional residual capacity of lung.Spirometry:Cannot measure residual or dead space volumes." "This is a real-world medical entrance exam question, please give the true answer based on the question and selection. Topic name: None, Subject name: Pediatrics","Suprasellar cystic mass in children is – Select and explain. A. Medulloblastoma, B. Craniopharyngioma, C. Meningioma, D. Secondaries",B: Craniopharyngioma,"Answer: B: Craniopharyngioma Explanation: Craniopharyngjoma Craniophyngioma is the most common supratentorial tumor in children. It is a suprasellar tumor which presents as cystic mass. It arises from squamous epithelial cells crests of embryonic rathke's pouch. Clinical presentation of craniopharyngioma In children they present as --> visual loss, growth failure, Bitemporal hemianopsia In adults they present as —> visual loss & endocrine dysfunction (D'1, delayed puberty).." "This is a real-world medical entrance exam question, please give the true answer based on the question and selection. Topic name: None, Subject name: Dental","Incidence of extra canal in mandibular 1st molar is seen in Select and explain. A. 10%, B. 20%, C. 30%, D. 40%",C: 30%,"Answer: C: 30% Explanation: None" "This is a real-world medical entrance exam question, please give the true answer based on the question and selection. Topic name: None, Subject name: ENT","A 55 year old male presents with a lesion of 2.5*3.5 cm on lateral border of tongue with induration. There is no associated lymphadenopathy. What is the staging according to the 8th AJCC guidelines? Select and explain. A. T2N0M0, B. T3N2M0, C. T4N3M0, D. T3N2M0",A: T2N0M0,"Answer: A: T2N0M0 Explanation: Ans: A. T2N0M0Ref: AJCC Cancer staging munual I'h ed., ch-7, pg. 79,90Cancer of the lip and oral cavity TNM staging AJCC UICC 2017" "This is a real-world medical entrance exam question, please give the true answer based on the question and selection. Topic name: None, Subject name: Dental","First permanent molars begin to calcify at: Select and explain. A. 6 months of intra uterine life, B. At birth, C. Before deciduous incisors, D. About 1 year after birth",B: At birth,"Answer: B: At birth Explanation: None" "This is a real-world medical entrance exam question, please give the true answer based on the question and selection. Topic name: None, Subject name: Dental","Lactobacillus count AFTER CARIES ACTIVITY test is 10,500CFU. What is the rate of caries progression ? Select and explain. A. Slight, B. Mild, C. Moderate, D. Severe",D: Severe,"Answer: D: Severe Explanation: None" "This is a real-world medical entrance exam question, please give the true answer based on the question and selection. Topic name: None, Subject name: Dental","5 years and 4 month old child having small chin, he clinically presents with distal step 2nd molar relation and narrow width of maxilla, having normal SNA angle, decreased SNB angle, with low FMA angle, then. What would be the appropriate treatment protocol for this patient? Select and explain. A. Wait and watch for 6 years, B. Kloehn head gear to treat maxillary protrusion, C. FR II appliance, D. Twin block appliance",A: Wait and watch for 6 years,"Answer: A: Wait and watch for 6 years Explanation: When the facial growth pattern is viewed against the perspective of the cephalocaudal gradient, it is not surprising that the mandible, being farther away from the brain, tends to grow more and later than the maxilla, which is closer. Growth of the mandible continues at a relatively steady rate before puberty. Ramus height increases 1 to 2 mm per year and body length increases 2 to 3 mm per year. These cross-sectional data tend to smooth out the juvenile and pubertal growth spurts, which do occur in growth of the mandible. Growth in length and height of both jaws continues through the period of puberty. Since the age of the child is less than 6 years and growth spurts are yet to occur. So at this point of time, in this patient, the best treatment protocol will be wait and watch. Ref: Proffit 4th Edition" "This is a real-world medical entrance exam question, please give the true answer based on the question and selection. Topic name: None, Subject name: Gynaecology & Obstetrics","A 32-year-old woman is 9 weeks pregnant and has a 10 year old Down's syndrome child. What test would you recommend for the mother so that she can know about her chances of getting a Down's syndrome baby is this present pregnancy. How will you assure the mother about the chances of Down's syndrome in the present pregnancy? Select and explain. A. Blood test, B. USG, C. Chorionic villus sampling, D. Assure her there is no chance since she is less than 35 years of age",C: Chorionic villus sampling,"Answer: C: Chorionic villus sampling Explanation: Friends – In Patients with previous H/O Down’s syndrome the only confirmatory test, which tells us with 100% reliability of the chances of Down’s syndrome in present pregnancy is “Karyotyping”. The sample for karyotyping can be obtained in first-trimester by – chorionic villi sampling and in 2nd trimester by amniocentesis. So obviously we will think of marking option ‘c’ i.e. chorionic villi sampling as the correct answer but, The Question specifically mentions that patient is 9 weeks pregnant and we all know that if CVS is done before 10 weeks– it can lead to limb reduction defects and oromandibular defects in the fetus \Some people argue CVS is not the correct thing to do at this stage. Read for yourself what Williams has to say on this issue. “Early reports of an association between CVS and limb. Reduction defects and oromandibular limb hypogenesis caused a great deal of concern (Burton, 1992; Firth, 1991, 1994; Hsieh, 1995, and all their colleagues). Subsequently, it was shown that limb-reduction defects were associated with CVS performed earlier in gestation—typically around 7 weeks” Williams Obs 23/e, p 300" "This is a real-world medical entrance exam question, please give the true answer based on the question and selection. Topic name: None, Subject name: Dental","Which types of wooden wedges are commonly available Select and explain. A. Triangular, B. Rectangular, C. Round, D. All are common",A: Triangular,"Answer: A: Triangular Explanation: None" "This is a real-world medical entrance exam question, please give the true answer based on the question and selection. Topic name: None, Subject name: Pharmacology","Short-acting non-depolarising muscle relaxant is: Select and explain. A. Succinylcholine, B. Pancuronium, C. Mivacurum, D. Decamethonium",C: Mivacurum,"Answer: C: Mivacurum Explanation: None" "This is a real-world medical entrance exam question, please give the true answer based on the question and selection. Topic name: None, Subject name: Medicine","A patient with Tubercular meningitis was taking ATT regularly. At end of 1 month of regular intake of drugs deterioration in sensorium is noted in condition of the patient despite good compliance for drugs. Which of the following investigations is not required as emergency condition? Select and explain. A. MRI, B. NCCT, C. CSF examination, D. Liver function tests",C: CSF examination,"Answer: C: CSF examination Explanation: Answer- C. CSF examinationAltered sensorium might be due to raised intracranial tension and lumbar punctare can precipitate the ominous herniation of the brain. So, lumbar puncture (for CSF examination) is not required on emergency evaluation in the above mentioned patient.Altered sensorium in a patient of Turbercular meningitis, who was taking ATT from the last one month maybe due toHepatic encephalopathy secondary to hepatotoxicity of ATT. Hence, LFT should be performed.Diagnosis:Lumbar puncture is cornerstone of diagnosis (should not be done in cases or raised ICT).CSF examination reveals high leukocyte counto (usually predominance of lymphocytes but often with a predominance of neutrophils in the early stage), a protein content of I to 8 g/L (100 to 800 mg/dl) and a low glucose concentration.Culture of CSF is diagnostic in 80% cases.CT or MRI may show hydrocephalus and abnormal enhancement of basal cisterns or ependyma." "This is a real-world medical entrance exam question, please give the true answer based on the question and selection. Topic name: None, Subject name: Dental","Ability of a bur blade to describe a complete circle during rotation is called as: Select and explain. A. Concentricity., B. Staticity., C. Run out., D. Symmetry",C: Run out.,"Answer: C: Run out. Explanation: None" "This is a real-world medical entrance exam question, please give the true answer based on the question and selection. Topic name: AIIMS 2019, Subject name: Surgery","Which of the following is true about prostate cancer? Select and explain. A. Histopathology is determined by Gleason score, B. Most common region involved is central zone, C. Most common type is squamous cell Carcinoma, D. It is not a hormone dependent cancer",A: Histopathology is determined by Gleason score,"Answer: A: Histopathology is determined by Gleason score Explanation: Prostate cancer: Adenocarcinoma Malignancy arising from gland Hormone dependent malignancy PROSTATE Ca. BREAST Ca.| |TESTOSTERONE PROGESTERONE - M/C malignant tumor in ? > 65 years- M/C site for malignancy - Peripheral zone Grading (Pathologic): GLEASON GRADE PRIMARY (1-5) SECONDARY (1-5) Gleason Score: Primary + secondary grading= 2-10 Screening: | Levels of PSA (prostate specific Antigen)- not specific" "This is a real-world medical entrance exam question, please give the true answer based on the question and selection. Topic name: AIIMS 2017, Subject name: Pathology","Which of the following anticoagulant preservative can be used to store blood, so that it can be kept for 35 days? Select and explain. A. Acid citrate dextrose (ACD), B. CPD Citrate phosphate dextrose, C. Citrate phosphate dextrose-adenine (CPD-A), D. CP2D - citrate phosphate double dextrose",C: Citrate phosphate dextrose-adenine (CPD-A),"Answer: C: Citrate phosphate dextrose-adenine (CPD-A) Explanation: Shelf life of the blood after adding above chemicals:- Acid citrate dextrose -> 21 days Citrate phosphate Dextrose -> 21 days Citrate phosphate Dextrose Adenine -> 35 days Saline Adenine Glucose Mannitol -> 42 days." "This is a real-world medical entrance exam question, please give the true answer based on the question and selection. Topic name: None, Subject name: Dental","All ceramic crown is placed on mandibular canine, what can we do to prolong the life of crown: Select and explain. A. Glazing, B. Polishing., C. Out of occlusion (Infraocclusion)., D. Group function.",D: Group function.,"Answer: D: Group function. Explanation: None" "This is a real-world medical entrance exam question, please give the true answer based on the question and selection. Topic name: None, Subject name: Orthopaedics","All of the following are complications of supracondylar fracture of humerus in children,except - Select and explain. A. Compartment syndrome, B. Myositis ossificans, C. Malunion, D. Non-Union",D: Non-Union,"Answer: D: Non-Union Explanation: None" "This is a real-world medical entrance exam question, please give the true answer based on the question and selection. Topic name: None, Subject name: Dental","Diagnosis of mouth breathing can be done by all of the following except: Select and explain. A. Cephalometric, B. Mouth mirror test, C. Rhinomanometry, D. Kinesiography",D: Kinesiography,"Answer: D: Kinesiography Explanation: Kinesiography is used to record the rest position of the mandible three dimensionally." "This is a real-world medical entrance exam question, please give the true answer based on the question and selection. Topic name: None, Subject name: Dental","In reversible hydrocolloid the property by which the transformation from sol to gel and gel to sol occurs, is a function of the: Select and explain. A. Concentration of fillers and plasticizer, B. Percentage composition by weight of water, C. Concentration of potassium sulphate, D. Temperature",D: Temperature,"Answer: D: Temperature Explanation: The physical change of agar from sol to gel is induced by lowering the temperature. The gel liquefies to a sol when it is heated to a temperature known as the liquefaction temperature (70°C to 100°C). When the sol is cooled, it becomes a gel at a point known as the gelation temperature (between 37°C and 50°C). Thus, it is called a reversible hydrocolloid. The gelation temperature is critical for impression making. Phillips dental materials 12th edition page no 168,170" "This is a real-world medical entrance exam question, please give the true answer based on the question and selection. Topic name: None, Subject name: Medicine","A patient presents with ascending muscle weakness for 2 days. On examination, the limb is flaccid. What investigation should be done first? Select and explain. A. Serum potassium, B. Serum creatinine, C. Serum magnesium, D. Serum calcium",A: Serum potassium,"Answer: A: Serum potassium Explanation: Answer- A. Serum potassiumHypokalemia can cause flaccid ascending paralysis, which is a differential diagnosis for GBS. It can also cause periodic paralysis. Hypocalcemia and hypomagnesemia usually cause tetany and hypercalcemia won't cause ascending paralysis.A low serum potassium level during an attack, excluding secondary causes, establishes the diagnosis of Hypokalemic Periodic Paralysis." "This is a real-world medical entrance exam question, please give the true answer based on the question and selection. Topic name: None, Subject name: Medicine","Acetone breath is a feature of: Select and explain. A. Liver disease, B. Sinusitis, C. Renal disease, D. Diabetes mellitus",D: Diabetes mellitus,"Answer: D: Diabetes mellitus Explanation: None" "This is a real-world medical entrance exam question, please give the true answer based on the question and selection. Topic name: None, Subject name: Dental","Which of the following bone(s) are used for the determination of growth in an individual? Select and explain. A. Clavicle, B. Sternum, C. Cervical vertebrae, D. Mandible",C: Cervical vertebrae,"Answer: C: Cervical vertebrae Explanation: Cervical vertebrae maturation index (CVMI) The size and shape changes in the bodies of five cervical vertebrae (second through sixth) are an accurate indicator of skeletal maturity. These can be assessed on a lateral cephalogram. Ref: Orthodontics: Diagnosis and Management of Malocclusion and Dentofacial Deformities, 3e, Om P. Kharbanda pdf no 873" "This is a real-world medical entrance exam question, please give the true answer based on the question and selection. Topic name: None, Subject name: Dental","Shape of the central groove in Mandibular 2nd premolar is Select and explain. A. Y shaped, B. H shaped, C. U shaped, D. All of the above",D: All of the above,"Answer: D: All of the above Explanation: None" "This is a real-world medical entrance exam question, please give the true answer based on the question and selection. Topic name: None, Subject name: Gynaecology & Obstetrics","Amniotic fluid is mainly produced by: Select and explain. A. Placenta, B. Fetus, C. Chorion, D. Amnion",B: Fetus,"Answer: B: Fetus Explanation: “The precise origin of the amniotic fluid remains is still not well understood. It is probably of mixed maternal and fetal origin.” Dutta Obs. 8/e, p 43 But this cannot help us to solve this question. Let’s see what Williams Obs. has to say on Origin of Amniotic fluid. “In early pregnancy, amniotic fluid is an ultrafiltrate of maternal plasma. By the beginning of second trimester, it consists largely of extracellular fluid which diffuses through the fetal skin, and thus reflects the composition of fetal plasma”. After 20 weeks, however, the cornification of fetal skin prevents this diffusion and amniotic fluid is composed largely of fetal urine.” Williams Obs. 23/e, p 88, 89 Reading the above text, it can be concluded that in early pregnancy - Mother is the main contributor whereas during rest of the pregnancy - Fetus is the main contributor." "This is a real-world medical entrance exam question, please give the true answer based on the question and selection. Topic name: None, Subject name: Medicine","A 40 years old female currently on a drug for psychiatric illness and hypeension presents with NYHA class III hea failure with dyspnea, pedal edema and K+ levels of 5.5 mEq/L and creatinine 2.5 mg%. Which of the following drug is best avoided? Select and explain. A. Carvedilol, B. Enalapril, C. Spironolactone, D. Digoxin",C: Spironolactone,"Answer: C: Spironolactone Explanation: Answer- C. Spironolactone In this case patient is having heafailure with hyperkalemia and renal dysfunction with psychiatric illness. Spironolactone and enalapril, both causes hyperkalemia hut spironolactone csuses CNS depression also. Digoxin causes hypokalemia In this patient, spironolactone should be avoided.Mechanism of ActionPharmacologic antagonist of aldosterone in collecting tubulesWeak antagonism of androgen receptorsWeak inhibitor of testosterone synthesis" "This is a real-world medical entrance exam question, please give the true answer based on the question and selection. Topic name: None, Subject name: Dental","Among the following solvent which of the following is safest and effective gutta percha solvent? Select and explain. A. Terpentine, B. Choloroform, C. Methylchioroform, D. Halothane",B: Choloroform,"Answer: B: Choloroform Explanation: None" "This is a real-world medical entrance exam question, please give the true answer based on the question and selection. Topic name: None, Subject name: Dental","Type-IV isthmii is? Select and explain. A. complete or incomplete isthmus between three or more canals, B. Complete communication between two canal, C. No communication between any canal, D. two or three canal openings without visible connections",A: complete or incomplete isthmus between three or more canals,"Answer: A: complete or incomplete isthmus between three or more canals Explanation: None" "This is a real-world medical entrance exam question, please give the true answer based on the question and selection. Topic name: None, Subject name: Pathology","Green discoloration of teeth is seen in: Select and explain. A. Tetracycline therapy, B. Fluorosis, C. Erythroblastosis fetalis, D. None of the above",C: Erythroblastosis fetalis,"Answer: C: Erythroblastosis fetalis Explanation: None" "This is a real-world medical entrance exam question, please give the true answer based on the question and selection. Topic name: None, Subject name: Dental","Improved marginal adaptation in a composite restoration can be achieved by: Select and explain. A. Moisture free environment, B. Application of pressure, C. Acid etching, D. Continued packing",C: Acid etching,"Answer: C: Acid etching Explanation: None" "This is a real-world medical entrance exam question, please give the true answer based on the question and selection. Topic name: None, Subject name: Pharmacology","In an animal model, the phenomenon of vasomotor reversal of dale can be demonstrated by: Select and explain. A. Stimulation of alpha-1 followed by stimulation of beta-2, B. Block of alpha-1 followed by stimulation of beta-2, C. Stimulation of alpha-1 followed by block of beta-2, D. Stimulation of beta-1 receptor followed by block of beta-2 receptor",B: Block of alpha-1 followed by stimulation of beta-2,"Answer: B: Block of alpha-1 followed by stimulation of beta-2 Explanation: Answer- B. Block of alpha-1 followed by stimulation of beta-2In an animal model, the phenomenon of vasomotor reversal of dale can be demonstrated hy block of alpha-l followed by stimulation of beta-2." "This is a real-world medical entrance exam question, please give the true answer based on the question and selection. Topic name: None, Subject name: Physiology","Which hormone, together with the catecholamines, enhances the tone of vascular smooth muscle and assists in elevating blood pressure? Select and explain. A. Parathyroid hormone, B. Glucagon, C. Thyroxine, D. Cortisol",D: Cortisol,"Answer: D: Cortisol Explanation: None" "This is a real-world medical entrance exam question, please give the true answer based on the question and selection. Topic name: None, Subject name: Dental","The purpose of plane of orientation is: Select and explain. A. To serve as a guide in establishing occlusal plane, B. To aid in measuring height and Length of posterior teeth, C. To serve as an arbitrary plane when established on the articulator, D. To divide the distance between upper and Lower cast equally",A: To serve as a guide in establishing occlusal plane,"Answer: A: To serve as a guide in establishing occlusal plane Explanation: None" "This is a real-world medical entrance exam question, please give the true answer based on the question and selection. Topic name: None, Subject name: Dental","The two cements which hold with the tooth are: Select and explain. A. GIC and polycarboxylate, B. GIC and ZnPO4, C. ZnPO4 and silicate, D. GIC and silicate",A: GIC and polycarboxylate,"Answer: A: GIC and polycarboxylate Explanation: None" "This is a real-world medical entrance exam question, please give the true answer based on the question and selection. Topic name: None, Subject name: Dental","Inflammation of the periapical tissue is sustained by: Select and explain. A. Stagnant tissue fluid, B. Necrotic tissue, C. Microorganisms, D. Pus cells",C: Microorganisms,"Answer: C: Microorganisms Explanation: Apical periodontitis is a chronic inflammatory disorder of periradicular tissues caused by aetiological agents of endodontic origin.  Persistent apical periodontitis occurs when root canal treatment of apical periodontitis has not adequately eliminated the intraradicular infection (microbes)." "This is a real-world medical entrance exam question, please give the true answer based on the question and selection. Topic name: None, Subject name: Surgery","A young lady with symptoms of hypehyroidism with elevated T4 and TSH levels were 8.5. Fuher examination reveals bitemporal hemianopia. Next step of management: Select and explain. A. Sta antithyroid drugs, and do urgent MRI brain, B. Sta beta-blockers, C. Conservative management, D. Sta antithyroid drugs and wait for symptoms to resolve.","A: Sta antithyroid drugs, and do urgent MRI brain","Answer: A: Sta antithyroid drugs, and do urgent MRI brain Explanation: Ans: A. Sta antithyroid drugs, and do urgent MRI brain(Ref Harrison 19/e p2274. 18/e p2880; Sabiston 20/e p982, 19/e p1890; Schwaz 10/e p1533, 9/e p1541: Bailey 27/e p8 1 1, 26/e p614-616)Hypehyroidism with elevated T4 and TSH levels and bitemporal hemianopia is highly suggestive of TSH-secreting adenoma.MRI - Confirm TSH-secreting pituitary adenoma diagnosis.TSH producing macroadenomas:Rare.Often large & locally invasive.Along with thyroid goiter & hypehyroidism, reflecting TSH overproduction.Diagnosis:Elevated serum free T4 levels.Inappropriately normal or high TSH secretion.MRI - Evidence of pituitary adenoma." "This is a real-world medical entrance exam question, please give the true answer based on the question and selection. Topic name: None, Subject name: Pathology","Which of the following is a soft tissue cyst which do not produce any radiographic changes Select and explain. A. Nasolabial cyst, B. Nasopalatine cyst, C. Mid alveolar cyst, D. Palatine cyst",A: Nasolabial cyst,"Answer: A: Nasolabial cyst Explanation: None" "This is a real-world medical entrance exam question, please give the true answer based on the question and selection. Topic name: None, Subject name: Dental","Pure histological landmark for termination of cleaning, shaping and obturation is Select and explain. A. CDJ, B. Apical foramen, C. Apical constriction, D. All of the above",A: CDJ,"Answer: A: CDJ Explanation: None" "This is a real-world medical entrance exam question, please give the true answer based on the question and selection. Topic name: None, Subject name: Dental","Mesial and distal wall of class V cavity depends on: Select and explain. A. Direction of enamel rods, B. Presence of carious lesion, C. Contour of gingiva, D. Location of contact area",A: Direction of enamel rods,"Answer: A: Direction of enamel rods Explanation: None" "This is a real-world medical entrance exam question, please give the true answer based on the question and selection. Topic name: None, Subject name: Gynaecology & Obstetrics","A 20 year young female presented for antenatal checkup. She was in 1st trimester and was diagnosed to have ovarian cyst. Treatment of choice: Select and explain. A. Surgical removal in IInd trimester, B. Removal after delivery, C. Termination of pregnancy and cyst removal, D. Observation",A: Surgical removal in IInd trimester,"Answer: A: Surgical removal in IInd trimester Explanation: Patient is presenting in the first trimester with ovarian cyst. The principle of treatment in case of ovarian tumour is to remove the tumour as soon as the diagnosis is made. But this principle should not be followed in the first trimester. Surgery in the first trimester is best avoided, as during surgery a corpus luteal cyst or ovary might be removed which will be detrimental to the pregnancy, which may end up in a miscarriage. Therefore, all such cases should be operated (ovariotomy/cystectomy) in the second trimester. Therefore, the best time of elective operation for an ovarian tumor in pregnancy is between 14 to 18 weeks, as the chances of abortion are less and access to the pedicle is easy." "This is a real-world medical entrance exam question, please give the true answer based on the question and selection. Topic name: None, Subject name: Pathology","Addison's disease is related to: Select and explain. A. Adrenal medulla, B. Adrenal cortex, C. Post pituitary, D. Parathyroid gland",B: Adrenal cortex,"Answer: B: Adrenal cortex Explanation: None" "This is a real-world medical entrance exam question, please give the true answer based on the question and selection. Topic name: AIIMS 2017, Subject name: Pediatrics","A child presented with Microcephaly, Hepatomegaly and periventricular calcification. Best site from where specimen should be obtained for diagnosis of CMV by PCR? Select and explain. A. Urine, B. Blood, C. Liver biopsy, D. CSF",A: Urine,"Answer: A: Urine Explanation: Congenital CMV infection C/F - Microcephaly Intracranial periventricular calcifications Chorioretinitis SNHL The diagnosis of congenital CMV infection is best confirmed by isolation of the virus from urine." "This is a real-world medical entrance exam question, please give the true answer based on the question and selection. Topic name: None, Subject name: Dental","The force applied to cause root movement should be Select and explain. A. Positive intermittent force, B. Torque or moment, C. Extra - oral, D. Gentle action",B: Torque or moment,"Answer: B: Torque or moment Explanation: None" "This is a real-world medical entrance exam question, please give the true answer based on the question and selection. Topic name: None, Subject name: Physiology","Which of the following hormones is controlled by feedforward control? Select and explain. A. ADH, B. Insulin, C. Coicosteroids, D. Prolactin",A: ADH,"Answer: A: ADH Explanation: Ans. A. ADHOsmolarity and volume status are the two greatest factors that affect ADH secretion.However, a variety of other factors promote ADH secretion as well.These include angiotensin II, pain, nausea, hypoglycemia, nicotine, opiates, and ceain medications.ADH secretion is also negatively affected by ethanol, alpha-adrenergic agonists, and atrial natriuretic peptide.Ethanol's inhibitory effect helps to explain the increased diuresis experienced during intoxicated states as well as increased free water loss; without appropriate ADH secretion, more water is excreted by the kidneys." "This is a real-world medical entrance exam question, please give the true answer based on the question and selection. Topic name: None, Subject name: Pharmacology","Which one of the following is penicillinase resistant penicillin: Select and explain. A. Amoxycillin, B. Cloxacillin, C. Ampicillin, D. Penicillin G",B: Cloxacillin,"Answer: B: Cloxacillin Explanation: None" "This is a real-world medical entrance exam question, please give the true answer based on the question and selection. Topic name: None, Subject name: Forensic Medicine","IPC section dealing with illegal aboion with woman consent? Select and explain. A. 310, B. 312, C. 313, D. 314",B: 312,"Answer: B: 312 Explanation: Answer- B. 312'Indian Penal Code (IPC) Sections 312 (causing miscarriage), 315 (act done with intent to prevent child being born alive or to cause to die after bih), 316 (causing death of quick unborn child by oct amount to culpable homicide) and Sections 3 and 4 of the Medical Termination of Pregnancy (MTP) Act I971 for performing MTP without license and without taking medical advice.'" "This is a real-world medical entrance exam question, please give the true answer based on the question and selection. Topic name: None, Subject name: Microbiology","In a suspected patient of dengue, all of these are acceptable investigations at day 3 of presentation except: Select and explain. A. Viral culture and isolation in C6/36 cell line, B. ELISA for antibody against dengue virus, C. NS I antigen detection, D. #NAME?",B: ELISA for antibody against dengue virus,"Answer: B: ELISA for antibody against dengue virus Explanation: Ans: B. ELISA for antibody against dengue virus(Ref Ananthanarayan 10/e p529, 8/e p519: Harrison 19/c 1)1319).ELISA in antibody detection against Dengue virus:Yields diagnostic results after 5 days (acute infection phase) for antibody detection against dengue virus.Other diagnosis methods:For mosquito inoculation/mosquito cell culture - Virus readily isolated from blood in acute phase.Laboratory tests for Dengue:Virus isolation within six days: Serum, plasma, autopsy tissueViral nucleic acid detection (-PCR assay).Diagnostic MethodTime to ResultsSpecimenTime of collection after onset of symptomsViral isolation & serotype identification1-2 weeksWhole blood, serum, tissues1-5 daysNucleic acid detection1 or 2 daysTissues, whole blood, serum, plasma1-5 daysdegAntigen detection1 daySerum1-6 daysdeg >1 dayTissue for immunochemistryNAIgM ELISA1-2 daysSerum, plasma, whole blood After 5 daysa" "This is a real-world medical entrance exam question, please give the true answer based on the question and selection. Topic name: None, Subject name: Social & Preventive Medicine","Government initiative to improve the facilities in labour room in all govt hospitals is under Select and explain. A. Ayushman Bharat Scheme, B. Laqshya, C. Newborn delivery room program, D. Janani suraksha yojana",B: Laqshya,"Answer: B: Laqshya Explanation: Ans.B. Laqshya'LaQshya' programme of the Ministry of Health and Family Welfare aims at improving quality of care in labour room and maternity Operation Theatre (OT). Objective:To reduce maternal and newborn moality & morbidity due to APH, PPH, retained placenta, preterm, preeclampsia & eclampsia, obstructed labour, puerperal sepsis, newborn asphyxia, and sepsis, etc.To improve Quality of care during the delivery and immediate post-paum care, stabilization of complications and ensure timely referrals, and enable an effective two-way follow-up system.To enhance satisfaction of beneficiaries visiting the health facilities and provide Respectful Maternity Care (RMC) to all pregnant women attending the public health facility." "This is a real-world medical entrance exam question, please give the true answer based on the question and selection. Topic name: None, Subject name: Dental","When length of a spring is doubled, the effect is: Select and explain. A. Doubled, B. Four times, C. Six times, D. Eight times",D: Eight times,"Answer: D: Eight times Explanation: None" "This is a real-world medical entrance exam question, please give the true answer based on the question and selection. Topic name: None, Subject name: Anaesthesia","Effective strategies to decrease the risk of post puncture dural headache are all except: Select and explain. A. Use of small bore needle, B. Use of atraumatic needle, C. Supplementation of fluids, D. Replacement of stylet prior to removal of needle",D: Replacement of stylet prior to removal of needle,"Answer: D: Replacement of stylet prior to removal of needle Explanation: Answer- D. Replacement of stylet prior to removal of needlePatient on antihypeensive medication is not a contraindication for neuraxial block.Neuraxial Block:Patients with platelet <80,000/ml: Relative contraindicatlonMarked coagulopathy, blood dyscrasias or full anticoagulant therapy: Absolute contraindicationMarked skin sepsis and marked spinal deformity: Absolute contraindication" "This is a real-world medical entrance exam question, please give the true answer based on the question and selection. Topic name: None, Subject name: Medicine","Severe acidosis occurs during cardiopulmonary resuscitation. The best method to counteract acidosis is to administer: Select and explain. A. Glucose intravenously, B. Epinephrine intravenously, C. Epinephrine intramuscularly, D. Sodium bicarbonate intravenously",D: Sodium bicarbonate intravenously,"Answer: D: Sodium bicarbonate intravenously Explanation: None" "This is a real-world medical entrance exam question, please give the true answer based on the question and selection. Topic name: AIIMS 2019, Subject name: Microbiology","Giemsa stained smear cannot detect Select and explain. A. Coxiella burnetti, B. Baonella, C. Toxoplasmosis, D. E. Chaffnessis",A: Coxiella burnetti,"Answer: A: Coxiella burnetti Explanation: Giemsa stained smear can detect Baonella, tachyzoites of Toxoplasma Gondii, E.chaffnessis but not for CoxiellaBurnetti which causes Q fever but can be detected by serology." "This is a real-world medical entrance exam question, please give the true answer based on the question and selection. Topic name: None, Subject name: Dental","Separation through rubber ring or band takes Select and explain. A. 2-3 days to 1 week, B. 1-2 days, C. 1-3 weeks, D. None of the above",A: 2-3 days to 1 week,"Answer: A: 2-3 days to 1 week Explanation: None" "This is a real-world medical entrance exam question, please give the true answer based on the question and selection. Topic name: None, Subject name: Pathology","Gardner syndrome does not include: Select and explain. A. Osteomas, B. Epidermoid cysts, C. Osteosarcoma, D. Impacted permanent teeth",C: Osteosarcoma,"Answer: C: Osteosarcoma Explanation: None" "This is a real-world medical entrance exam question, please give the true answer based on the question and selection. Topic name: None, Subject name: Pediatrics","Pediatric patient presented with 45 mins h/o continuous convulsions.CASE 1: SR told to give iv lorazepam but J.R cant secured iv line. Then what he has given? Select and explain. A. Rectal diazepam, B. Inhalation Phenobarbital, C. IV carbamazepine, D. Subcutaneous midazolam",A: Rectal diazepam,"Answer: A: Rectal diazepam Explanation: Answer A. Rectal diazepamBenzodiazepines are Drug of Choice and in pediatric patients rectal route should be preffered.Reference - Dose > Pharmacokinetics > Safety" "This is a real-world medical entrance exam question, please give the true answer based on the question and selection. Topic name: None, Subject name: Dental","Bennett angle is: Select and explain. A. The angle between sagittal condylar path and Frank fort plane, B. The angle which the path of balancing side condyle makes with the sagittal plane during Lateral excursion, C. Measured by using a face bow, D. None of the above",B: The angle which the path of balancing side condyle makes with the sagittal plane during Lateral excursion,"Answer: B: The angle which the path of balancing side condyle makes with the sagittal plane during Lateral excursion Explanation: None" "This is a real-world medical entrance exam question, please give the true answer based on the question and selection. Topic name: None, Subject name: Anatomy","Couinaud classified liver into 8 segments,which structure differentiates the segments anatomically Select and explain. A. Poal vein, B. Hepatic vein, C. Bile duct, D. Hepatic aery",B: Hepatic vein,"Answer: B: Hepatic vein Explanation: Ans. B. Hepatic veinThe Couinaud classification divides the liver into 8 functional segments.The hepatic veins are found at the periphery of each segment, whereas the center has branches of the poal veins, hepatic aeries, and bile ducts.The middle hepatic vein divides the liver into left and right lobes. The left hepatic vein divides the left lobe into lateral (2, 3) and medial (4a, 4b) segments. The right hepatic vein divides the right lobe into anterior (5, 8) and posterior (6, 7) segments. The caudate lobe (1) has hepatic veins that often drain directly into the IVC." "This is a real-world medical entrance exam question, please give the true answer based on the question and selection. Topic name: None, Subject name: Dental","'Care index' with respect to epidemiology of oral disease is Select and explain. A. Ratio of filled teeth to total teeth with caries experience, B. Ratio of filled teeth to teeth present, C. No. of filled teeth per person, D. Ratio of denture users to non-denture users",A: Ratio of filled teeth to total teeth with caries experience,"Answer: A: Ratio of filled teeth to total teeth with caries experience Explanation: None" "This is a real-world medical entrance exam question, please give the true answer based on the question and selection. Topic name: None, Subject name: Pharmacology","A morbidly obese diabetic woman was on failed metformin therapy. She has the history of pancreatitis and family history of bladder cancer. Patient does not want to take injections. Which of the following would be suitable to reduce her glucose levels? Select and explain. A. Liraglutide, B. Sitagliptin, C. Canagliflozin, D. Pioglitazone",C: Canagliflozin,"Answer: C: Canagliflozin Explanation: Ans. C. Canagliflozin Canagliflozin is a Sodium-Glucose Co-Transpoer 2 Inhibitor, given orally and reduces body weight. For the given clinical scenario, Canagliflozin is the preferred drug, as Sitagliptin increases the risk of acute pancreatitis, use of pioglitazone is associated with a small increased risk of bladder cancer and Liraglutide is given subcutaneously." "This is a real-world medical entrance exam question, please give the true answer based on the question and selection. Topic name: None, Subject name: Dental","Cavity varnish is used for: Select and explain. A. To reduce thermal conductivity, B. To reduce the permeability of acid into dentinal tubules, C. Action as an obtundant, D. All of the above",B: To reduce the permeability of acid into dentinal tubules,"Answer: B: To reduce the permeability of acid into dentinal tubules Explanation: None" "This is a real-world medical entrance exam question, please give the true answer based on the question and selection. Topic name: None, Subject name: Dental","A patient shows gingival swelling in lower premolar region. Biopsy was taken and histopathological examination showed endothelium-lined vascular spaces and the extreme proliferation of fibroblasts and budding endothelial cells. What could this lesion be: Select and explain. A. Pyogenic granuloma, B. Fibroma, C. Aneurysmal bone cyst, D. Traumatic bone cyst",A: Pyogenic granuloma,"Answer: A: Pyogenic granuloma Explanation: Clinically, pyogenic granuloma is a raised lesion on either a sessile or a pedunculated base. Its surface may have a smooth, lobulated, or, occasionally, warty appearance that is erythematous and often ulcerated. Depending on the age of the lesion, the texture varies from soft to firm and is suggestive of an ulcerated fibroma.  Because of the pronounced vascularity of these lesions, they often bleed easily when probed. Histologically, the pyogenic granuloma presents as a remarkable proliferation of plump fibroblasts and endothelial cells with the formation of prominent numbers of  thin-walled, endothelium-lined vascular channels. A polymorphous inflammatory cell infiltrate is present, and the overlying surface epithelium is often ulcerated. Treatment consists of surgical excision, with care being taken to completely remove any local irritant that may still be present that would predispose to recurrence of the lesion. Reference: McDONALD AND AVERY’S DENTISTRY for the CHILD and ADOLESCENT, 10th ed page no 604,605" "This is a real-world medical entrance exam question, please give the true answer based on the question and selection. Topic name: None, Subject name: Physiology","Most common sign of airway obstruction Select and explain. A. Slow pounding pulse, B. Strenuous breathing, C. Increase pulse rate, D. Decrease pulse rate",B: Strenuous breathing,"Answer: B: Strenuous breathing Explanation: None" "This is a real-world medical entrance exam question, please give the true answer based on the question and selection. Topic name: None, Subject name: Pharmacology","GPCR that does not act through opening of potassium channels is? Select and explain. A. Muscarinic M2 receptor, B. Dopamine D2 receptor, C. Serotonin 5 HT1 receptor, D. Angiotensin 1 receptor",D: Angiotensin 1 receptor,"Answer: D: Angiotensin 1 receptor Explanation: Ans. D. Angiotensin 1 receptor* Four G protein gated inwardly-rectifying potassium (GIRK) channel subunits have been identified in mammals: GIRK1, GIRK2, GIRK3, and GIRK4. The GIRK subunits come together to form GIRK ion channels.* Activation of the IKACh channels begins with release of acetylcholine (ACh) from the vagus nerve onto pacemaker cells in the hea.* ACh binds to the M2 muscarinic acetylcholine receptors, which interact with G proteins." "This is a real-world medical entrance exam question, please give the true answer based on the question and selection. Topic name: None, Subject name: Forensic Medicine","After a building collapse, among remnants, a person's length of humerus is 24.5 cm. What is the predicted height of this person? Select and explain. A. 90 cm, B. 110 cm, C. 130 cm, D. 146 cm",C: 130 cm,"Answer: C: 130 cm Explanation: Ans: C. 130 cm(Ref Parikh 6/e p2.75, 12.2).In general, humerus represents 20%, Tibia 22%, Femur 27% and Spine 34% of the stature.Multiplication factor for humerus is 5.31 for calculation of stature.In the question, length of femur is 24.5 cm.Hence, predicted height = 5.31x24.5 = 130 cm." "This is a real-world medical entrance exam question, please give the true answer based on the question and selection. Topic name: AIIMS 2019, Subject name: Medicine","A patient of CKD has presented with protracted vomiting. ABG shows pH = 7.40, pCO2 = 40 mm Hg, HCO3 = 25 mEq, Na=145 mEq/L, Chloride = 100 mEq/L, HCO3 = 24 mEq/L. What is the observation? Select and explain. A. Normal anion gap metabolic acidosis, B. High anion gap metabolic acidosis, C. No acid base abnormality, D. High anion gap metabolic acidosis and metabolic alkalosis",D: High anion gap metabolic acidosis and metabolic alkalosis,"Answer: D: High anion gap metabolic acidosis and metabolic alkalosis Explanation: AG(Anion Gap)= Na+ - (Cl- + HCO3-) = 145-124 = 21 meq Normal 8-10meq Rules out option a. - CKD is condition where patient experiences impaired ability to excrete H+ - Vomiting contributes to metabolic alkalosis - Two processes simultaneously occurring So answer is High anion gap metabolic acidosis and metabolic alkalosis." "This is a real-world medical entrance exam question, please give the true answer based on the question and selection. Topic name: AIIMS 2019, Subject name: Anaesthesia","Purpose of giving Glycopyrrolate before GA Select and explain. A. Decrease laryngeal secretions, B. Muscle relaxation, C. Prevents aspiration, D. Provides analgesia",A: Decrease laryngeal secretions,"Answer: A: Decrease laryngeal secretions Explanation: Glycopyrolate in preanesthetic medication: Glycopyrolate unlike atropine and scopolamine, it is a quaternary ammonium compound, it doesn't crosses the blood brain barrier. It's role: To decrease salivary, tracheobronchial secretions. It can be use IV/IM It's anti-sialogue activity is best when given at 0.004mg/kg BW IM 30-60 minutes prior peak effects. Tachycardiac effects of glycopyrrolate are minimize when given IM. Decrease the gastric secretion and increases PH of gastric content. To counteract parasympathetic overdrive, dose: 0.1 IV, repeat every 3-5 minute. To counteract parasympathetic effect, atropine is preferred due to much faster onset." "This is a real-world medical entrance exam question, please give the true answer based on the question and selection. Topic name: None, Subject name: Biochemistry","what does not occur in 5'- 3'direction? Select and explain. A. DNA repair, B. DNA replication, C. RNA editing, D. Transcription",C: RNA editing,"Answer: C: RNA editing Explanation: Ans -C- RNA editingIn rna editing editosome can edit only in 3- 5 direction, along with primary rna transcript.DNA replication goes in the 5' to 3' direction because DNA polymerase acts on the 3'-OH of the existing strand for adding free nucleotides.In transcription the complementary RNA is created in the opposite direction, in the 5' - 3' direction, matching the sequence of the sense strand with the exception of switching uracil for thymine.When the strand containing the mis-match is identified, an endonuclease nicks the strand and the mis-matched nucleotide(s) is/are removed by an exonuclease." "This is a real-world medical entrance exam question, please give the true answer based on the question and selection. Topic name: None, Subject name: Dental","Highest level of evidence is seen in: Select and explain. A. Case control studies, B. Meta-analysis, C. Cohort studies, D. Systematic review",B: Meta-analysis,"Answer: B: Meta-analysis Explanation: None" "This is a real-world medical entrance exam question, please give the true answer based on the question and selection. Topic name: AIIMS 2019, Subject name: Medicine","Which of the following cannula is used in patient with severe dehydration and diarrhea? Select and explain. A. Blue, B. Green, C. Grey, D. Pink",C: Grey,"Answer: C: Grey Explanation: Severe shock - 16G /Grey Pediatrics/ Geriatrin - 22g/blue Routine maintenance IVF/ Adult male Antibiotic / blood transfusion - 18g/ green Adult female antibiotic - 20g/pink" "This is a real-world medical entrance exam question, please give the true answer based on the question and selection. Topic name: AIIMS 2019, Subject name: Social & Preventive Medicine","Which of the following diseases is NOT transmitted by it? Select and explain. A. Kala Azar, B. Chandipura encephalitis, C. Babesiosis, D. Carrion's Disease",C: Babesiosis,"Answer: C: Babesiosis Explanation: The image is of SANDFLY, identifiable by the long, slender legs, lanceolate wings, branching of the second longitudinal vein, densely hairy antennae, legs, body and wings. Diseases transmitted: Kala Azar Sandfly fever Oriental sore Oroya fever Carrion's disease Chandipura virus Babesiosis is transmitted by the Hard Tick." "This is a real-world medical entrance exam question, please give the true answer based on the question and selection. Topic name: None, Subject name: Surgery","All of the following statements of nasal fractures are true except: Select and explain. A. Even if minor, they may be followed by bilateral ecchymosis and facial oedema, B. They may need to be reduced for a few weeks, C. They need not be complicated by traumatic telecanthus, D. They may lead to the telescoping of the nasal complex into the frontal sinus",B: They may need to be reduced for a few weeks,"Answer: B: They may need to be reduced for a few weeks Explanation: None" "This is a real-world medical entrance exam question, please give the true answer based on the question and selection. Topic name: None, Subject name: Dental","Etchant preferred in gel form than in liquid form: Select and explain. A. Better control over placement, B. Enhance and concentrate the action of acid, C. Helps in visualization while placement, D. None of the above",A: Better control over placement,"Answer: A: Better control over placement Explanation: Generally, the etchant is supplied as an aqueous gel to allow precise placement over a specific area. These gels are often made by adding colloidal silica (the same fine particles used in microfilled composites) or polymer beads to the acid. Brushes are used to place the acidic gel, or the acid may be supplied in a disposable syringe from which it can be expressed onto enamel and dentin.  During placement, it is important to be aware of the risk for air bubbles that may be introduced at the interface. Regions with air pockets will not be etched. Reference: PHILLIPS’ SCIENCE OF DENTAL MATERIALS, 12th ed page no 262" "This is a real-world medical entrance exam question, please give the true answer based on the question and selection. Topic name: AIIMS 2018, Subject name: Microbiology","A known HIV patient on anti - retroviral therapy presented with diarrhea of six months duration. Stoll microscopy was done in which 10-30 micrometer cysts were seen, Kinyoun stain was positive. What is the most likely diagnosis? Select and explain. A. Cystoisospora, B. Cryptosporidium, C. Balantidium coli, D. Strongyloides",A: Cystoisospora,"Answer: A: Cystoisospora Explanation: The identification of the parasite is based on the size of oocyst since the size of oocyst is 20 to 30 Micron the most likely parasite is Isospora. If the size was 4-8 micron the parasite would have been Cryptosporidium parvum." "This is a real-world medical entrance exam question, please give the true answer based on the question and selection. Topic name: AIIMS 2018, Subject name: Pediatrics","How long should a child be isolated after being diagnosed with bacterial meningitis to prevent fuher transmission? Select and explain. A. Till 24 hours after staing antibiotics, B. Till cultures become negative, C. Till antibiotics course is complete, D. Till 12 hrs after admission",A: Till 24 hours after staing antibiotics,"Answer: A: Till 24 hours after staing antibiotics Explanation: - CSF becomes sterile within 24 of initiation of appropriate antibiotic therapy. - Hence, child should be isolated for 24 hrs after staing antibiotics to prevent fuher transmission." "This is a real-world medical entrance exam question, please give the true answer based on the question and selection. Topic name: None, Subject name: Pathology","On stretching the cheek the lesion disappears in: Select and explain. A. Leukoplakia, B. Focal hyperkeratosis, C. Leukoedema, D. Typhoid",C: Leukoedema,"Answer: C: Leukoedema Explanation: None" "This is a real-world medical entrance exam question, please give the true answer based on the question and selection. Topic name: None, Subject name: Dental","Extra palatal root when present on mesial side its known as Select and explain. A. Radix mesiopalatalis, B. Radix mesialis, C. Radix mesiolingualis, D. None of the above",C: Radix mesiolingualis,"Answer: C: Radix mesiolingualis Explanation: None" "This is a real-world medical entrance exam question, please give the true answer based on the question and selection. Topic name: None, Subject name: Surgery","A 56-year-old female presents with nocturnal pain in the right thumb, index and middle finger for the past 3 months. All of the following provocative tests can be performed for the diagnosis of the condition except : Select and explain. A. Phalen's test, B. Finkelstein test, C. Tinel's sign, D. Tourniquet test",B: Finkelstein test,"Answer: B: Finkelstein test Explanation: Ans. b. Finkelstein test (Ref. Maheshwuri 3/eNocturnal pain in the right thumb, index and middle finger for the past 3 months in a 56-year old female is highly suggestive of Carpal tunnel syndrome. Finkelstein test is used to diagnose DeQuervain's tenosynovitis, not the carpal tunnel syndrome." "This is a real-world medical entrance exam question, please give the true answer based on the question and selection. Topic name: None, Subject name: Pharmacology","Which of the following drug is contraindicated in pregnancy? Select and explain. A. Enalapril, B. Amlodipine, C. b-blockers, D. Propylthiouracil",A: Enalapril,"Answer: A: Enalapril Explanation: None" "This is a real-world medical entrance exam question, please give the true answer based on the question and selection. Topic name: None, Subject name: Dental","Interglobular dentin Select and explain. A. Has less mineral content, B. Shows PAS reaction, C. Both, D. None",C: Both,"Answer: C: Both Explanation: None" "This is a real-world medical entrance exam question, please give the true answer based on the question and selection. Topic name: None, Subject name: Gynaecology & Obstetrics","All of the following indicate superimposed pre-eclampsia in a pregnant female of chronic hypeension except: (Asked twice) Select and explain. A. New onset proteinuria, B. Platelet count < 75,000, C. Increase in systolic BP by 30 mm Hg and diastolic by 15 mm Hg, D. Fresh retinal hypeensive changes",C: Increase in systolic BP by 30 mm Hg and diastolic by 15 mm Hg,"Answer: C: Increase in systolic BP by 30 mm Hg and diastolic by 15 mm Hg Explanation: Answer- C. Increase in systolic BP by 30 mm Hg and diastolic by 15 mm HgSevere range BP despite escalation of antihypeensive therapy, Thrombocytopenia (Platelet count<1,00,000/mL)Elevated liver transaminases (two times the upper limit of normal concentration for paicular laboratory)New onset and worsening renal insufficiencyPulmonary edema.Persistent cerebral or visual disturbances" "This is a real-world medical entrance exam question, please give the true answer based on the question and selection. Topic name: None, Subject name: Dental","Basic fear of a 2-year-old child during his first visit to the dentist is related to Select and explain. A. Fear of an injection, B. Fear of separation from parent, C. Fear of dental instruments, D. Not understanding the reason for dental treatment",B: Fear of separation from parent,"Answer: B: Fear of separation from parent Explanation: None" "This is a real-world medical entrance exam question, please give the true answer based on the question and selection. Topic name: None, Subject name: Dental","Which of the following show chemical bond with enamel: Select and explain. A. Composites, B. Direct filling resins, C. Polycarboxylate cements, D. BIS-GMA resins in pit and fissure sealants",C: Polycarboxylate cements,"Answer: C: Polycarboxylate cements Explanation: None" "This is a real-world medical entrance exam question, please give the true answer based on the question and selection. Topic name: None, Subject name: Social & Preventive Medicine","A Study conducted in a population. Diastolic Blood Pressure mean 110 mm Hg with SD of 11 mm Hg, Vitamin D mean 18 ng/ml with SD of 3 ng/ml. what is the relation between the two variance Select and explain. A. Variance of Vitamin D > DBP, B. Variance of DBP > 4 times Vitamin D, C. Data insufficient to comment upon, D. None",B: Variance of DBP > 4 times Vitamin D,"Answer: B: Variance of DBP > 4 times Vitamin D Explanation: Ans.b). Variance of DBP > 4 times Vitamin DCoefficient of Variance (CV) is a tool to compare variability of two different characteristics (eg. BP, serum creatinine, height weight etc.) in the same group of subjects or compare variability of the same character in two or more different groups.Thus it is a measure used to compare relative variability.For example, coefficient of variance measureswhether weight varies more than height in a group of studentwhether weight varies more in girls of boys. Calculation:CV= standard detion x 100/ meanFor Blood pressure- 110x100/11=1000For Vitamin D- 3X100/18= 16.7" "This is a real-world medical entrance exam question, please give the true answer based on the question and selection. Topic name: None, Subject name: Surgery","Main mechanism of action of heparin Is to prevent: Select and explain. A. Conversion of fibrinogen to fibrin, B. Conversion of prothrombin to thrombin, C. PTC to PTA, D. Factor VIIa inhibition",B: Conversion of prothrombin to thrombin,"Answer: B: Conversion of prothrombin to thrombin Explanation: None" "This is a real-world medical entrance exam question, please give the true answer based on the question and selection. Topic name: None, Subject name: Dental","For class II die 1 malocclusion of 14 years old boy, which analysis is indicated to detect the tooth extraction Select and explain. A. Bolton's analysis, B. Pont's analysis, C. Peck & peck analysis, D. Ashley & Howe's analysis",D: Ashley & Howe's analysis,"Answer: D: Ashley & Howe's analysis Explanation: None" "This is a real-world medical entrance exam question, please give the true answer based on the question and selection. Topic name: None, Subject name: Physiology","Diffusion capacity of carbon monoxide is decreased in all except: Select and explain. A. Polycythemia, B. Interstitial lung disease, C. Emphysema, D. Pulmonary vascular disease",A: Polycythemia,"Answer: A: Polycythemia Explanation: Ans: A. Polycythemia(Ref Ganong 25/e p635. 24/e p635)Diffusion capacity of carbon monoxide is increased in polycythemia.Diffusing capacity of lung:Directly propoional to surface area of alveolo-capillary membrane.Inversely propoional to alveolo-capillary membrane thickness.Diffusing capacity for- CO (DICO):Measured as an index of diffusing capacity.Due to diffusion-limited uptake. Factors Affecting DLCOIncreased DLCODecreased DLCOExerciseSupine positionMuller maneuver (inspiration against closed mouth & nose after forced expiration)Pulmonary hemorrhagePolycythemiaLeft-to-right shunt (e.g. atrial septal defectdeg)ObesityAsthmaChronic bronchitis without major'PregnancyPost-exerciseStandingValsalva maneuveLung resectionPulmonary emphysemaPulmonary emphysema hypeension & chronic venous thromboembolism)Interstitial lung diseasesAnemiaDrugs: Amiodarone, bleomycin, Pulmonary lymphangitic carcinomatosis" "This is a real-world medical entrance exam question, please give the true answer based on the question and selection. Topic name: None, Subject name: Dental","An avulsed tooth which is replanted shows loss of lamina dura radiographically, high sound on percussion. The type of resorption is Select and explain. A. Internal resorpton, B. External resorption, C. Replacement resorption, D. Cervical resorption",C: Replacement resorption,"Answer: C: Replacement resorption Explanation: None" "This is a real-world medical entrance exam question, please give the true answer based on the question and selection. Topic name: None, Subject name: Dental","Extra-radicular source of persistent endodontic infection is? Select and explain. A. Propionobacterium, B. E.fecalis, C. V.parvula), D. T.denticola",A: Propionobacterium,"Answer: A: Propionobacterium Explanation: None" "This is a real-world medical entrance exam question, please give the true answer based on the question and selection. Topic name: None, Subject name: Anatomy","Which of the following is not a content of mesorectal fascia? Select and explain. A. Inferior rectal vein, B. Superior rectal vein, C. Pararectal lymph node, D. Inferior mesenteric plexus",A: Inferior rectal vein,"Answer: A: Inferior rectal vein Explanation: Ans: A. Inferior rectal veinMesorectum is enclosed by mesorectal fascia, which is derived from the visceral peritoneum.Known as visceral fascia of mesorectum, fascia propria of rectum or presacral wing of hypogastric sheath.Upper rectum is derived from the embryological hind gut, it is surrounded by mesorectumdeg. Contents of Mesorectal fasciaSuperior rectal aery and its branchesSuperior rectal vein and tributariesLymphatic vessels and nodes along superior rectal aeries.Branches from inferior mesenteric plexus to innervate rectum.Loose adipose connective tissue down to the level of levator ani (pelvic floor)." "This is a real-world medical entrance exam question, please give the true answer based on the question and selection. Topic name: None, Subject name: Physiology","A motorcyclist after road traffic accident is having raised aerial pCO2 on ABG. Probable cause can be: Select and explain. A. Damage to respiratory apparatus, B. Damage to respiratory centre, C. Damage to both respiratory apparatus and respiratory centre, D. Massive lung contusion alone",C: Damage to both respiratory apparatus and respiratory centre,"Answer: C: Damage to both respiratory apparatus and respiratory centre Explanation: Answer- C. Damage to both respiratory apparatus and respiratory centre'Depression of the respiratory center by a variety of drugs, injury, or disease can produce respiratory acidosis.This may occur acutely with general anesthetics, sedatives, and head trauma or chronically with sedatives, alcohol, intracranial tumors, and the syndromes of sleep-disordered breathing including the primary alveolar and obesity-hypoventilation syndromes.'- Harrison I8/e p371" "This is a real-world medical entrance exam question, please give the true answer based on the question and selection. Topic name: None, Subject name: Dental","Pulp mummification is indicated in: Select and explain. A. Non-vital tooth, B. Deep caries on a symptomatic vital tooth, C. Traumatic exposure of a vital tooth, D. Traumatic exposure of a vital tooth with open wide apex",C: Traumatic exposure of a vital tooth,"Answer: C: Traumatic exposure of a vital tooth Explanation: None" "This is a real-world medical entrance exam question, please give the true answer based on the question and selection. Topic name: None, Subject name: Pathology","""Hair-on-end"" appearance in a skull roentgenogram is seen in : Select and explain. A. Fibrous dysplasia, B. Thalassemia, C. Garre's Osteomyelitis, D. Pagets disease",B: Thalassemia,"Answer: B: Thalassemia Explanation: None" "This is a real-world medical entrance exam question, please give the true answer based on the question and selection. Topic name: None, Subject name: Pediatrics","Poor prognostic indicator of ALL is – Select and explain. A. Female sex, B. Leukocyte count < 50,000, C. Age greater than 1 year, D. Hypodiploidy",D: Hypodiploidy,"Answer: D: Hypodiploidy Explanation: Prognostic factors in ALL" "This is a real-world medical entrance exam question, please give the true answer based on the question and selection. Topic name: None, Subject name: Pharmacology","L-asparaginase is used in the treatment of: Select and explain. A. AML, B. ALL, C. CML, D. CLL",B: ALL,"Answer: B: ALL Explanation: Ans: B. ALL(Ref: Goodman Cilrmm 12/c p1720; Katzung 13Ie p938-939, 12/e p968; KDT 7Ie p868).* Asparaginase (L-asparagine amidohydrolase) is an enzyme used to treat childhood ALL* Drug is isolated and purified from Escherichia coli or Erwinia chrysanthemi for clinical use. - It hydrolyzes circulating L-asparagine to aspaic acid and ammonia. - Because tumor cells in ALL lack asparagine synthetase, they require an exogenous source of L!asparagine. - Thus, depletion of L-asparagine results in effective inhibition of protein synthesis.* In contrast, normal cells can synthesize L-asparagine and thus are less susceptible to the cytotoxic action of asparaginase." "This is a real-world medical entrance exam question, please give the true answer based on the question and selection. Topic name: None, Subject name: Dental","According to Glickman, maximum accumulation of plaque takes place in approximately Select and explain. A. 7 days, B. 15 days, C. 30 days, D. 60 days",C: 30 days,"Answer: C: 30 days Explanation: None" "This is a real-world medical entrance exam question, please give the true answer based on the question and selection. Topic name: AIIMS 2017, Subject name: Medicine","Which of the following is a false statement? Select and explain. A. Acetaminophen does not have anti-inflammatory action, B. NSAIDs with least cardiovascular risk in Naproxen, C. Gastric irritation is more severe with NSAIDs compared to aspirin, D. Non selective COX-2 inhibitors are contraindicated in postoperative patients",C: Gastric irritation is more severe with NSAIDs compared to aspirin,"Answer: C: Gastric irritation is more severe with NSAIDs compared to aspirin Explanation: Gastric irritation is most severe with aspirin. It can cause gastric mucosal erosion and ulceration leading to bleeding and perforation." "This is a real-world medical entrance exam question, please give the true answer based on the question and selection. Topic name: None, Subject name: Pediatrics","Most common cause of death in case of acute poliomyelitis is – Select and explain. A. Intercostal muscles paralysis, B. Convulsion, C. Cardiac arrest, D. Respiratory failure",D: Respiratory failure,"Answer: D: Respiratory failure Explanation: Death is usually due to complications arising from respiratory dysfunction. Paralytic polio In less than 1% of infections. Paralysis is characterized as : Descending  Asymmetrical  Proximal muscles > distal muscles Non progressive No sensory involvement  No autonomic disturbance Lower motor neuron type Most common muscle affected -4 Quadriceps Most common muscle undergoes complete paralysis -4 Tibialis anterior Most common muscle affected in hand —> Opponens pollicis. M.C. cause of death ---> Respiratory paralysis Following signs can be elicited : Tripod sign   —>      Child is asked to sit up unassisted. He assumes tripod posture. Kiss the knee Test           —>         The child cannot kiss his knees due to spine stiffness. Head drop sign    —>        Hand is placed under the patients shoulder and the trunk is raised. The head lags behind simply." "This is a real-world medical entrance exam question, please give the true answer based on the question and selection. Topic name: None, Subject name: Dental","Which of the following statements is true for rapid prototyping? Select and explain. A. Photopolymer resin cures with laser, B. Powder by fused deposition modeling, C. Selective laser sintering, D. All of the above",D: All of the above,"Answer: D: All of the above Explanation: Rapid prototyping is the fast fabrication of a physical part, model or assembly using 3D computer aided design (CAD). The creation of the part, model or assembly is usually completed using additive manufacturing, or more commonly known as 3D printing. A photopolymer or autopolymerising resin is deposited layer by layer and sets according to its chemical nature. Laser can also be used in the polymerization process. It is applicable in dentistry for printing prostheses, stents and models." "This is a real-world medical entrance exam question, please give the true answer based on the question and selection. Topic name: None, Subject name: Surgery","In choriocarcinoma, lung metastasis occurs in which stage? Select and explain. A. Stage 1, B. Stage 2, C. Stage 3, D. Stage 4",C: Stage 3,"Answer: C: Stage 3 Explanation: Answer- C. Stage 3FIGO Classification* Stage I Disease confined to uterus.* Stage II Extends outside of the uterus but is limited to the genital structure ( adnexa, vagina, broad ligament.)* Stage III Extends of the lungs , with or without known genital tract involvement.* Stage IV meatastases to other organs ( Brain, liver, Kidney, Ovaries, Bowel)." "This is a real-world medical entrance exam question, please give the true answer based on the question and selection. Topic name: None, Subject name: Dental","Role of plasticizer in synthetic resins in dentistry is Select and explain. A. Increase smoothness, B. To increase bulk, C. To prevent polymerisation shrinkage, D. To reduce softening and fusion temperature",D: To reduce softening and fusion temperature,"Answer: D: To reduce softening and fusion temperature Explanation: None" "This is a real-world medical entrance exam question, please give the true answer based on the question and selection. Topic name: None, Subject name: Pathology","In which of the following disorders a circulating antibody directed to intercellular cementing substance of stratified squamous epithelium is observed: Select and explain. A. Lichen planus, B. Verrucous vulgaris, C. Bullous pemphigoid, D. Pemphigus vulgaris",D: Pemphigus vulgaris,"Answer: D: Pemphigus vulgaris Explanation: None" "This is a real-world medical entrance exam question, please give the true answer based on the question and selection. Topic name: None, Subject name: Pediatrics","A 10 year old boy is having polyuria, polydipsia,laboratory data showed (in mEq/lit) – Na– 154 K– 4.5 HCO3– 22 Serum osmolality – 295 Blood urea – 50 Urine specific gravity – 1.005 The likely diagnosis is – Select and explain. A. Diabetes insipidus, B. Renal tubular acidosis, C. Barter's syndrome, D. Recurrent UTI",A: Diabetes insipidus,"Answer: A: Diabetes insipidus Explanation: Lets see each option one by one : artter's syndrome : is characterized by hypokalemia, metabolic alkalosis and normal to low blood pressure. It can be easily ruled out as the patient is having normal potassium level.  Renal tubular acidosis : is characterized by hyperchloremic metabolic acidosis with a normal serum anion gap. It is ruled out as this patient is having normal bicarbonate level (Bicarbonate level is decreased in RTA). Recurrent UTI Symptoms seen are : Fever, dysuria, frequency, urgency, suprapubic pain. It is ruled out as the above given symptoms are not mentioned in this patient. Hypernatremia (Na is 154) is not seen in UTI. iabetes insipidus : Clinical presentation of hypernatremia, polyuria, polydypsia, thin urine of low specific gravity (1.005) is suggestive of diabetes insipidus." "This is a real-world medical entrance exam question, please give the true answer based on the question and selection. Topic name: None, Subject name: Surgery","FNAC can not diagnose Select and explain. A. Papillary carcinoma of thyroid, B. Follicular carcinoma of thyroid, C. Anaplastic carcinoma of thyroid, D. Medullary carcinoma of thyroid",B: Follicular carcinoma of thyroid,"Answer: B: Follicular carcinoma of thyroid Explanation: FNAC helps in diagnosis of- Papillary carcinoma Medullary carcinoma Anaplastic carcinoma Colloid nodules Thyroiditis FNAC cannot diagnose follicular carcinoma and Hurthle carcinoma" "This is a real-world medical entrance exam question, please give the true answer based on the question and selection. Topic name: None, Subject name: Medicine","Lacunar cells are seen in which type of Hodgkin’s lymphoma Select and explain. A. Lymphocyte predominance, B. Lymphocyte depletion, C. Nodular sclerosing, D. Mixed cellularity",C: Nodular sclerosing,"Answer: C: Nodular sclerosing Explanation: None" "This is a real-world medical entrance exam question, please give the true answer based on the question and selection. Topic name: None, Subject name: Dental","Maximum fluoride content is found in: Select and explain. A. Sea fish, B. Tea leaves, C. Human milk, D. Coconut water",B: Tea leaves,"Answer: B: Tea leaves Explanation: None" "This is a real-world medical entrance exam question, please give the true answer based on the question and selection. Topic name: AIIMS 2018, Subject name: Dental","A patient with acute history of blistering and denudation involving >30% BSA along with erosions of the lips with hemorrhagic crusting and other mucosa for few days. What is the most common triggering factor? Select and explain. A. Bacterial infection, B. Viral infection, C. Drug induced, D. Idiopathic",C: Drug induced,"Answer: C: Drug induced Explanation: Lesions >30% BSA , erosion over lips and mucosa : Dx TEN TEN SJS: Haemorrhagic Crusting Sloughing of mucosa Targetoid lesion (2 rings) Nikolsky sign (Pseudo) due to epidermal necrosis. Epidermal necrosis (SJS/TEN) Etiology: Drugs: -Antiepileptic( MC) - CBZ, Lamotrigine * NSAIDS * Penicillin * Nevirapine CBZ induced SJS/TEN is a/w HLA B1502" "This is a real-world medical entrance exam question, please give the true answer based on the question and selection. Topic name: None, Subject name: Surgery","Organic component of bone comprises of: Select and explain. A. 35% collagen, B. 35% noncollagenous protein, C. 65% noncollagenous protein, D. 90% collagen protein",D: 90% collagen protein,"Answer: D: 90% collagen protein Explanation: None" "This is a real-world medical entrance exam question, please give the true answer based on the question and selection. Topic name: None, Subject name: Medicine","What is the difference between RIFLE & KDIGO criteria in differentiating a tubular injury? Select and explain. A. Uosm, B. Urinary Na, C. Urinary NGAL, D. FeNa (UN, x Scr / SNa x Ucr)",C: Urinary NGAL,"Answer: C: Urinary NGAL Explanation: Ans. C. Urinary NGALRIFLE originally used urinary sodium, urine osmolarity, FeNa to define tubular dysfunction/injury.Currently urinary NGAL/Kim-1/Cystatin C/L-FABP all are being evaluated as more sensitive and specific markers of tubular injury (KDIGO 2017/18)." "This is a real-world medical entrance exam question, please give the true answer based on the question and selection. Topic name: None, Subject name: Biochemistry","Lipids are ………………group of compounds: Select and explain. A. Heterogenous, B. Homogenous, C. None of the above, D. Any of the above",A: Heterogenous,"Answer: A: Heterogenous Explanation: Lipids are heterogenous group of compounds. They are soluble in non-polar solvents( Ether, Benzene, Chloroform,Formalin, Acetone), but insoluble in polar solvent( water, plasma, alcohol." "This is a real-world medical entrance exam question, please give the true answer based on the question and selection. Topic name: None, Subject name: Physiology","A couple comes for evaluation of infeility. The HSG was normal but semen analysis revealed azoospermia. What is the diagnostic test to differentiate between testicular failure and vas deferens obstruction? Select and explain. A. Serum FSH, B. Karyotyping, C. Testosterone levels, D. Testicular FNAC",A: Serum FSH,"Answer: A: Serum FSH Explanation: Ans: A. Serum FSH(Ref. Ganong 25Ie p419, 425, 24Ie p427)Prior to initiating treatment for a couple, in whom the man has azoospermia, it is impoant to distinguish whether the lack of sperm in the ejaculate is from an obstructive or non-obstructive process.The presence of normal volume testes with bilaterally indurated epididymis and/or absent vas deferens will point to an obstructive etiology for azoospermia.A history of cryptorchidism in the presence of small or soft testes suggests non-obstructive azoospermia, especially if associated with an elevated serum (FSH) level.A decreased spermatogenesis leads to decrease in production of inhibin, which causes an elevation in FSH." "This is a real-world medical entrance exam question, please give the true answer based on the question and selection. Topic name: AIIMS 2019, Subject name: Microbiology","Which of the following toxin is causing this: Select and explain. A. Botulism, B. Tetanus, C. Diphtheria, D. Cholera",A: Botulism,"Answer: A: Botulism Explanation: Botulinum toxin contains 2 chains- heavy and light. The heavy Chain binds presynaptically to nerve terminals and is responsible for mediating translocation of light chain into the cell. The light chain is active pa which cleaves host protein snap 25 The cleaved snap 25 prevent the release of neurotransmitter Acetylcholine and is responsible flaccid paralysis. The tetanus toxin causes cleavage of synaptobrevin and prevents the release of neurotransmitters GABA and Glycine which causes spastic paralysis. Diphtheria toxin causes ADP ribosylation of elongation factor 2 which inhibits protein synthesis. Cholera toxin causes ADP ribosylation of G stimulatory protein which causes increase in cyclin AMP and causes watery diarrhoea So according to the question the answer is botulinum toxin." "This is a real-world medical entrance exam question, please give the true answer based on the question and selection. Topic name: None, Subject name: Radiology","Radio density can be increased by Select and explain. A. Decreasing MA, B. Decreasing kVp, C. Decreasing target film distance, D. Increasing target film distance",C: Decreasing target film distance,"Answer: C: Decreasing target film distance Explanation: None" "This is a real-world medical entrance exam question, please give the true answer based on the question and selection. Topic name: None, Subject name: Biochemistry","In uncontrolled diabetes mellitus, elevated triglyceride and VLDL levels are seen due to: Select and explain. A. Increased activity of lipoprotein lipase and decreased activity of hormone sensitive lipase, B. Increased activity of hormone sensitive lipase and decreased activity of lipoprotein lipase, C. Increase in peripheral LDL receptors, D. Increased activity of hepatic lipase",B: Increased activity of hormone sensitive lipase and decreased activity of lipoprotein lipase,"Answer: B: Increased activity of hormone sensitive lipase and decreased activity of lipoprotein lipase Explanation: Answer- B. Increased activity of hormone sensitive lipase and decreased activity of lipoprotein lipaseIn uncontrolled diabetes mellitus, elevated triglyceride and VLDL levels sre seen due to increased activity of hormone sensitive lipase (which insulin inhibits) and decreased activity of lipoprotein lipase (which insulin stimalates)." "This is a real-world medical entrance exam question, please give the true answer based on the question and selection. Topic name: None, Subject name: Dental","A 9-year-old child comes with Ellis class-III fracture in 11 after 24 hours, what will be the first step of management? Select and explain. A. Take radiograph and Cvek's pulpotomy, B. Take radiograph and pulpectomy, C. Indirect pulp capping, D. Observation and restore with composite resin",A: Take radiograph and Cvek's pulpotomy,"Answer: A: Take radiograph and Cvek's pulpotomy Explanation: Indication and contraindication for pulpotomy Indications: Vital tooth with healthy periodontium. Pain, if present, is neither spontaneous nor persistent. Tooth which is restorable. The tooth that possesses at least 2/3rd of its root length. Hemorrhage from the amputation site is pale red and easy control. In mixed dentition stage, primary tooth is preferable to a space maintainer. Contraindications Evidence of internal resorption. Presence of interradicular bone loss. Existence of abscesses or fistula in relation to the teeth. Radiographic signs of calcific globules seen in the pulp chamber. Caries penetrating the floor of the pulp chamber. Tooth close to natural exfoliation." "This is a real-world medical entrance exam question, please give the true answer based on the question and selection. Topic name: None, Subject name: Surgery","In a 40 years old patient with head injury, which of the following is the best strategy to decrease the intracerebral pressure? Select and explain. A. Limiting pCO, of the patient, B. Administer sedatives, C. Oxygen supplementation by mechanical ventilation, D. Administer nimodipine","A: Limiting pCO, of the patient","Answer: A: Limiting pCO, of the patient Explanation: Ans:. a. Limiting pCO, of the patient(Ref Harrison 19/e p1780, 18/e p2257)Hyperventilation and decreasing pCO2 is the initial and one of the most impoant treatment strategies to lower the ICP in a patient of head injury.For ICP >20-25 mmHg for >5 minPressor therapy - Phenylephrine, dopamine, or norepinephrine to maintain adequate MAP to ensure CPP 60 mmHg (maintain euvolemia to minimize deleterious systemic effects of pressors)Consider second-tier therapies for refractory elevated 1CP:High-dose barbiturate therapy (""pentobarb coma"")Aggressive hyperventilation to PaCO2 <30 mmHgHypothermiaHemicraniectomy" "This is a real-world medical entrance exam question, please give the true answer based on the question and selection. Topic name: None, Subject name: Medicine","All the following are seen in myasthenia gravis except: Select and explain. A. Ptosis, B. Muscle fatigability, C. Absent DTRs, D. Normal pupillary reflex",C: Absent DTRs,"Answer: C: Absent DTRs Explanation: Answer- C. Absent DTRsDeep tendon reflexes (DTRs) are present in patients of Myasthenia gravis.""The limb weakness in myasthenia gravis (MG) is often proximal and may be asymmetric. Despite the muscle weakness, deep tendon reflexes are preserved." "This is a real-world medical entrance exam question, please give the true answer based on the question and selection. Topic name: None, Subject name: Pathology","A patient notices a well demarcated area of depapillation on his tongue which has been there for as long as he can remember. The most probable diagnosis: Select and explain. A. Median rhomboid glossitis, B. Geographic tongue, C. Black hairy tongue, D. Moeller's glossitis",A: Median rhomboid glossitis,"Answer: A: Median rhomboid glossitis Explanation: None" "This is a real-world medical entrance exam question, please give the true answer based on the question and selection. Topic name: None, Subject name: Dental","Root end resection is done? Select and explain. A. 1mm, B. 4mm, C. 6mm, D. 3mm",D: 3mm,"Answer: D: 3mm Explanation: None" "This is a real-world medical entrance exam question, please give the true answer based on the question and selection. Topic name: AIIMS 2018, Subject name: Biochemistry","All of the following should be avoided by a patient with lactose intolerance, EXCEPT: Select and explain. A. Condensed milk, B. Ice-cream, C. Skimmed milk, D. Yoghu",D: Yoghu,"Answer: D: Yoghu Explanation: Lactose intolerance due to a defect in the enzyme lactose (beta galactosidase) is very common. The treatment advocated is severe restriction of lactose (any kind of milk and milk products). Yoghu - In yogu some of the bacteria produce lactase enzyme that can digest lactose." "This is a real-world medical entrance exam question, please give the true answer based on the question and selection. Topic name: None, Subject name: Dental","For testing the statistical significance of the difference in heights of school children among three socio economic groups the most appropriate statistical test is: Select and explain. A. Student's 't' test, B. Chi-squared test, C. Paired 't' test, D. One way analysis of variance (one way ANOVA)",D: One way analysis of variance (one way ANOVA),"Answer: D: One way analysis of variance (one way ANOVA) Explanation: None" "This is a real-world medical entrance exam question, please give the true answer based on the question and selection. Topic name: None, Subject name: Surgery","Inheritance of cleft lip and palate is Select and explain. A. Monogenic, B. Polygenic, C. Multifactorial, D. Sex linked",C: Multifactorial,"Answer: C: Multifactorial Explanation: None" "This is a real-world medical entrance exam question, please give the true answer based on the question and selection. Topic name: None, Subject name: Dental","Name the parts of the implant assembly Select and explain. A. 1 - fixture, 2 – abutment, 3 - abutment screw, 4 - crown, B. 1 - crown, 2 – abutment screw, 3 - abutment. 4 - fixture, C. 1 - crown, 2 - healing cap, 3 - abutment. 4 - root form implant, D. 1 - crown, 2 - healing cap, 3 - cover screw, 4 - fixture","B: 1 - crown, 2 – abutment screw, 3 - abutment. 4 - fixture","Answer: B: 1 - crown, 2 – abutment screw, 3 - abutment. 4 - fixture Explanation: None" "This is a real-world medical entrance exam question, please give the true answer based on the question and selection. Topic name: None, Subject name: Social & Preventive Medicine","In a survey of sleep apnea scores among 10 people, the highest sample of 58 was entered by mistake as 85. This will affect the result as: Select and explain. A. Increased mean, decreased median, B. Increased mean, increased median, C. Increased mean. no change in median, D. No change in mean, increased median.",C: Increased mean. no change in median,"Answer: C: Increased mean. no change in median Explanation: Ans: C. Increased mean. no change in median(Ref. Park 24/e p885, 23/e p847, 22/e p786)Value of median is not affected by abnormal very high or very low value.Mean is unduly influenced by abnormal values (either very high or very low) in distribution." "This is a real-world medical entrance exam question, please give the true answer based on the question and selection. Topic name: None, Subject name: Medicine","Localised suppuration of lung is called: Select and explain. A. An abscess, B. Empyema, C. Emphysema, D. Anasarca",A: An abscess,"Answer: A: An abscess Explanation: None" "This is a real-world medical entrance exam question, please give the true answer based on the question and selection. Topic name: None, Subject name: Anatomy","Artery of 4th pharyngeal arch: Select and explain. A. Right and left CCA., B. Right and left subclavian artery and arch of aorta., C. Right and left pulmonary arteries., D. None of the above.",B: Right and left subclavian artery and arch of aorta.,"Answer: B: Right and left subclavian artery and arch of aorta. Explanation: None" "This is a real-world medical entrance exam question, please give the true answer based on the question and selection. Topic name: None, Subject name: Social & Preventive Medicine","Among a 100 women with average Hb of 10 gm%, the standard deviation was 1, what is the standard error? Select and explain. A. 0.01, B. 0.1, C. 1, D. 10",B: 0.1,"Answer: B: 0.1 Explanation: In the given question, n = 100 women, mean Hemoglobin (m) = 10 gm%, standard deviation (s) = 1, Thus standard error (SE) =" "This is a real-world medical entrance exam question, please give the true answer based on the question and selection. Topic name: None, Subject name: Dental","The age of the child used in the determination of child mortality rate is: Select and explain. A. 0-5 years, B. 1-4 years, C. 5-8 years, D. 0-1 years",B: 1-4 years,"Answer: B: 1-4 years Explanation: None" "This is a real-world medical entrance exam question, please give the true answer based on the question and selection. Topic name: None, Subject name: Dental","The formation of the dental lamina is initiated by Select and explain. A. Forebrain, B. Rathke's pouch, C. Neural crest cells, D. Odontoblasts",C: Neural crest cells,"Answer: C: Neural crest cells Explanation: None" "This is a real-world medical entrance exam question, please give the true answer based on the question and selection. Topic name: None, Subject name: Physiology","The most outer covering of nerve fibres is called as: Select and explain. A. Neurolemma, B. Perineurium, C. Axolemma, D. Myelin sheath",B: Perineurium,"Answer: B: Perineurium Explanation: None" "This is a real-world medical entrance exam question, please give the true answer based on the question and selection. Topic name: None, Subject name: Anaesthesia","A 46 years old male patient was given subarachnoid block with bupivacaine (heavy) by the anesthetist. After 10 minutes he was found to have a BP of 72/44 mm Hg and hea rate of 52/min. On checking the level of block it was found to be T6. What is the likely explanation for the bradvcardia? Select and explain. A. Bezold-Jarisch reflex, B. Bainbridge reflex, C. Block of Cardio-accelerator fibers of synthetic origin, D. Reverse Bainbridge reflex",A: Bezold-Jarisch reflex,"Answer: A: Bezold-Jarisch reflex Explanation: Ans: A. Bezold-Jarisch reflex (Ref 8/e p1970, Miller 7/e p409)The Bezold-Jarisch reflex involves a variety of cardiovascular and neurological processes which cause hypopnea (excessively shallow breathing or an abnormally low respiratory rate) & bradycardia (abnormally low resting hea rate).Possible cause of profound bradycardia and circulatory collapse after spinal anesthesia.Cardioprotective reflex.Implicated in physiologic response to a range of cardiovascular conditions such as myocardial ischemia or infarction, thrombolysis, or revascularization and syncope.Natriuretic peptide receptors stimulated by endogenous ANP or BNP may modulate the Bezold-Jarisch reflex.Less pronounced in patients with cardiac hyperophy or atrial fibrillation" "This is a real-world medical entrance exam question, please give the true answer based on the question and selection. Topic name: AIIMS 2018, Subject name: Forensic Medicine","This specific sign is seen in: Select and explain. A. Lead poisoning, B. Organophosphorus poisoning, C. Arsenic poisoning, D. Zinc poisoning",C: Arsenic poisoning,"Answer: C: Arsenic poisoning Explanation: Rain drop Pigmentation- Brownish pigmentation present on palm and trunk. It occurs in chronic Arsenic Poisoning. Chronic arsenic Poisoning- Skin- Rain drop pigmentation, Hyperkeratosis of palms and soles. Nails- Aldrich mees lines. Sensory neuropathy Black foot disease (Gangrene of peripheries) Tests done for diagnosis of Arsenic Poisoning- Marsh test Reinsch test NAA AAS" "This is a real-world medical entrance exam question, please give the true answer based on the question and selection. Topic name: None, Subject name: Medicine","Patient who is a known case of thalassemia major already on repeated blood transfusions with history of iron overload previously treated with chelating agents. She also has a history of cardiac arrhythmia. She came for BT now. During BT patient complained of backache and looks extremely anxious. What is next management? Select and explain. A. Observe for a change in colour of the urine, B. Continue BT, do ECG, C. Stop BT and wait for patient to get normal and sta, D. Stop BT and Do clerical check",D: Stop BT and Do clerical check,"Answer: D: Stop BT and Do clerical check Explanation: Ans. d. Stop BT and do clerical checkThe patient has ominous signs of a major Acute Hemolytic transfusion reaction ( Backache/anxiety). STOP THE TRANSFUSION WITHOUT ANY DELAY.Also fuher work up is mandatory(especially the basic clerical work like checking the blood product details and cross matching repos) and do not resta transfusion until it is complete" "This is a real-world medical entrance exam question, please give the true answer based on the question and selection. Topic name: None, Subject name: Dental","Which of the following has a bifid root? Select and explain. A. 32, 42, B. 31, 41, C. 34, 44, D. 13, 23","C: 34, 44","Answer: C: 34, 44 Explanation: None" "This is a real-world medical entrance exam question, please give the true answer based on the question and selection. Topic name: AIIMS 2018, Subject name: Orthopaedics","Which Bone does not form the wrist joint Select and explain. A. Radius, B. Triquetrum, C. Scaphoid, D. Ulna",D: Ulna,"Answer: D: Ulna Explanation: * Bones forming the wrist joint are: radius, scaphoid, lunate, triquetrum and other carpal bones.* The wrist joint is a synol joint between the distal end of radius and the aicular disc overlying the distal end of ulna and the scaphoid, lunate and triquetrum.* Ulna is not pa of wrist joint, only the overlying aicular disc is pa of the wrist joint." "This is a real-world medical entrance exam question, please give the true answer based on the question and selection. Topic name: None, Subject name: Ophthalmology","Stenopic slit'is used for all except- Select and explain. A. Fincham's test, B. Determine the axis of cylinder, C. Corneal tattooing, D. lridectomy",C: Corneal tattooing,"Answer: C: Corneal tattooing Explanation: Ans: C. Corneal tattooingRef: A, K. Khurana Comprehensive Ophthalmologt,4,h ed.Stenopic slit is useful in finding out the axis of the cylinder.(option b) stenopic slit is used in cases of corneal opacities to find out the optimal site for optical iridectomy.After dilating the pupil with a mydriatic, the slit is rotated in front of the eye and the axis which gives maximum clarity is chosen for optical iridectomy" "This is a real-world medical entrance exam question, please give the true answer based on the question and selection. Topic name: None, Subject name: Surgery","In a patient with obstructive jaundice, what is the possible explanation for a bilirubin level of 40 mg/dL? Select and explain. A. Malignant obstruction, B. Complete obstruction of common bile duct, C. Renal failure, D. Liver failure",C: Renal failure,"Answer: C: Renal failure Explanation: Ans: C. Renal failure(Ref Zakim and Boyer s Hepatologv (2016)/p109; Bailey 25/e p1128, Textbook of hepatologv 6/e p20t).High bilirubin content:Causes of cholestatic jaundice (malignant obstruction, complete CBD obstruction).Presence of concomitant renal failure a Very high bilirubin beyond 30 mg/dL.Conjugated hyperbilirubinemia:Results from impaired intrahepatic bilirubin excretion or extrahepatic obstruction.Due to continued urinary excretion, maximum serum bilirubin levels plateau at approximately 500 mmol/L (30 Ing/dL) even with complete bile duct obstruction.Extreme hyperbilirubinemia:Levels higher than 500 mmol/L (30 mg/dL).Commonly indicates severe parenchymal liver disease in association with hemolysis (as in sickle cell anemia) or renal failure." "This is a real-world medical entrance exam question, please give the true answer based on the question and selection. Topic name: None, Subject name: Pharmacology","Drugs used in urea cycle disorders to reduce ammonia? Select and explain. A. Phenyl butarone, B. Phenyl butyrate, C. L - Carnitine, D. Isoleucin",B: Phenyl butyrate,"Answer: B: Phenyl butyrate Explanation: Ans: B. Phenyl butyrate Urea cycle disorders are inherited disorders where ammonia is not turned into uric acid properly and so it builds up in the blood. Regular intake of sodium phenylbutyrate helps reduce the levels of ammonia." "This is a real-world medical entrance exam question, please give the true answer based on the question and selection. Topic name: None, Subject name: Dental","Calcium In blood increased by Select and explain. A. Calcitonin, B. Parathormone, C. TNF alpha, D. Insulin",B: Parathormone,"Answer: B: Parathormone Explanation: None" "This is a real-world medical entrance exam question, please give the true answer based on the question and selection. Topic name: AIIMS 2019, Subject name: Social & Preventive Medicine","Mass Drug Administration is not helpful for Select and explain. A. Lymphatic Filariasis, B. Vitamin A Deficiency, C. Worm infestation, D. Scabies",D: Scabies,"Answer: D: Scabies Explanation: - Lymphatic Filariasis is treated with Diethyl carbamazine (DEC) (Or) a combination of DEC + Albendazole/ Ivermectin - Vitamin A deficiency - Prophylaxis- 2 lakhs IU to 1-6 years of age - Worm infestation - Ascariasis once every 2-3 months; it does not interrupt transmission but useful in reduction of worm load in community - Scabies - No mass drug administration done along with patient his/her family members should be treated not the whole community." "This is a real-world medical entrance exam question, please give the true answer based on the question and selection. Topic name: None, Subject name: Dental","If continuous hypersensitivity develops in recently restored tooth: Select and explain. A. No treatment necessary, B. Remove the restoration and place ZOE, C. Perform pulpectomy, D. Perform direct pulp capping",C: Perform pulpectomy,"Answer: C: Perform pulpectomy Explanation: Since the patient gives a history of continuous hypersensitivity, it is indicative of pulpitis and hence, pulpectomy is recommended as the best option." "This is a real-world medical entrance exam question, please give the true answer based on the question and selection. Topic name: None, Subject name: Social & Preventive Medicine","Immune thrombocytopenic purpura (ITP) is a complication observed with which of the following vaccines? (Asked twice) Select and explain. A. MMR, B. HIV conjugate polysaccharide vaccine, C. Typhoid Vi polysaccharide vaccine, D. Influenza vaccine",A: MMR,"Answer: A: MMR Explanation: Answer- A. MMRITP develop in one in every 24,000 doses of the MMR vaccine givenBruise-like spotsIn rare cases, a child may get a small rash of bruise like spots a but two weeks after having the MMR vaccine.This side effect is linked to the rubella vaccine and is known as idiopathic thrombocytopenic purpura (ITP)." "This is a real-world medical entrance exam question, please give the true answer based on the question and selection. Topic name: None, Subject name: Social & Preventive Medicine",". The prospectively evaluated, double-blinded, randomized clinical trail represents the 'gold-standard' for providing evidence for therapeutic decision making. This was first proposed by the father of evidence-based medicine: Select and explain. A. Tolstoy, B. Sackett, C. Hippocrates, D. da Vinci",B: Sackett,"Answer: B: Sackett Explanation: Ans. b. Sackett""David Lawrence Sackett is a Canadian medical doctor and a pioneer in evidence-based medicine. He founded the first depament of clinical epidemiology in Canada at McMaster University, and the Oxford Centre for Evidence-Based Medicine. He is well known for his textbooks Clinical Epidemiology and Evidence-Based Medicine."" Father of modern medicineHippocratesQFather of Indian medicineCharakaQFather of modern surgeryAmbroise PareQFather of Indian surgeryShushrutaQFather of epidemiologyJohn SnowQFather of bacteriologyLouis PasteuQFather of modern anatomyVesaliusaQFather of physiologyClaude BernardQFather of antisepsisJoseph ListerQ" "This is a real-world medical entrance exam question, please give the true answer based on the question and selection. Topic name: None, Subject name: Pharmacology","Hepatic First pass metabolism will be bypassed by the foltowing routes of drug adminisration excePt? Select and explain. A. Oral, B. Intravenous, C. Sublingual, D. Intradermal",A: Oral,"Answer: A: Oral Explanation: Ans: A. OralRef: Goodman & Gilman, I3'h ed., pg. 16-17 and sharma & sharms's Principles of Pharmacologt, 3'd ed., pg. I8-20Oral route has ceain disadvantages notably the 1st pass metabolism in liver before reaching the systemic circulation." "This is a real-world medical entrance exam question, please give the true answer based on the question and selection. Topic name: None, Subject name: Ophthalmology","Cherry red spot after trauma is seen in children due to: Select and explain. A. CRAO, B. CRVO, C. Berlin's edema, D. Niemann-Pick's disease",C: Berlin's edema,"Answer: C: Berlin's edema Explanation: Ans: C. Berlin's edema(Ref Kanski 7/e p882: Parson's 22/e p392, 21/e p382, 20/e p367; Yanoff and Duker 4/e p671)Cherry red spot after trauma in children due to Berlin's edema.Commotio retinae (Berlin's Edema):Common occurrence following a eye blow.Manifests as milky white cloudiness involving posterior pole with a 'cherry-red spot' in foveal region.Appear after some days or may be followed by pigmentary changes.Most frequently affect temporal fundus." "This is a real-world medical entrance exam question, please give the true answer based on the question and selection. Topic name: None, Subject name: Dental","Single accessory canal arises from coronal 1/3rd in what percentage of mandibular molars Select and explain. A. 23%, B. 13%, C. 10%, D. None of the above",A: 23%,"Answer: A: 23% Explanation: None" "This is a real-world medical entrance exam question, please give the true answer based on the question and selection. Topic name: None, Subject name: Pharmacology","What does low volume of distribution of a drug mean? Select and explain. A. Low bioavailability, B. Does not accumulates in tissues, C. Low absorption, D. Not metabolized in the body",B: Does not accumulates in tissues,"Answer: B: Does not accumulates in tissues Explanation: Ans: B. Does not accumulates in tissues(Ref: KDT 7/e p17, 18, 6/e p18)Low volume of distribution:Implies that the drug remains confined to the plasma compament without getting distributed in the body tissues." "This is a real-world medical entrance exam question, please give the true answer based on the question and selection. Topic name: None, Subject name: Biochemistry","The conversion of glucose-6-P to fructose-6-P is an example of which of the following reactions: Select and explain. A. Phosphate transfer, B. Isomerisation, C. Dehydration, D. Aldol cleavage",B: Isomerisation,"Answer: B: Isomerisation Explanation: None" "This is a real-world medical entrance exam question, please give the true answer based on the question and selection. Topic name: None, Subject name: Psychiatry","A person with histrionic, shy, anxious avoidant per!sonality comes under which cluster? Select and explain. A. A, B. B, C. C, D. D",C: C,"Answer: C: C Explanation: Ans: C. C(Ref Niraj Ahuja 7/e p113)ClusterDisordersCluster A (Odd and eccentric)Paranoid PDSchizoid PDSchizotypal PDCluster B (Dramatic, emotional and Antisocial PDHistrionic PDNarcissistic PDBorderline PDCluster C (Anxious and fearful)Anxious (avoidant) PDDependent PDAnankastic (Obsessive and compulsive) PD" "This is a real-world medical entrance exam question, please give the true answer based on the question and selection. Topic name: None, Subject name: Pharmacology","Which of the following is third generation cephalosporin? Select and explain. A. Cefoperazone, B. Cefuroxime, C. Cefoxitin, D. Cefadroxil",A: Cefoperazone,"Answer: A: Cefoperazone Explanation: None" "This is a real-world medical entrance exam question, please give the true answer based on the question and selection. Topic name: None, Subject name: Dental","Ideal denture base material should not have Select and explain. A. Sufficient strength, B. Dense and non-irritating surface, C. High specific gravity, D. Esthetically acceptable",C: High specific gravity,"Answer: C: High specific gravity Explanation: None" "This is a real-world medical entrance exam question, please give the true answer based on the question and selection. Topic name: None, Subject name: Anatomy","Buccopharyngeal membrane is composed of: Select and explain. A. Ectoderm., B. Endoderm., C. Mesoderm, D. Both AB",D: Both AB,"Answer: D: Both AB Explanation: None" "This is a real-world medical entrance exam question, please give the true answer based on the question and selection. Topic name: None, Subject name: Surgery","A 11 year old boy with grossly carious right maxillary lateral incisor, LA is given, complains of burning sensation in right eye, swelling in right eyelid, best statement to explain this condition. Select and explain. A. Middle meningeal artery has tortuous course, B. LA reaches the eyelid via infraorbital foramen, C. Reaction is due to preservative in the L.A., it will subside in some time, D. Pain will subside in few minutes","C: Reaction is due to preservative in the L.A., it will subside in some time","Answer: C: Reaction is due to preservative in the L.A., it will subside in some time Explanation: None" "This is a real-world medical entrance exam question, please give the true answer based on the question and selection. Topic name: None, Subject name: Dental","Sibilant sounds are produced at: Select and explain. A. Rest position, B. Occluding position, C. Open from resting position, D. Between rest and occluding position",D: Between rest and occluding position,"Answer: D: Between rest and occluding position Explanation: None" "This is a real-world medical entrance exam question, please give the true answer based on the question and selection. Topic name: None, Subject name: Dental","Temporo-mandibular ligament is attached to: Select and explain. A. Lateral aspect of TMJ, B. Posterior aspect of TMJ, C. Mandibular condyle, D. Coronoid process",A: Lateral aspect of TMJ,"Answer: A: Lateral aspect of TMJ Explanation: None" "This is a real-world medical entrance exam question, please give the true answer based on the question and selection. Topic name: None, Subject name: Dental","What should be the best treatment option for the extraction socket shown in Fig Select and explain. A. Ovate Pontic, B. Modified ridge lap, C. Sanitary pontic, D. All of the above",A: Ovate Pontic,"Answer: A: Ovate Pontic Explanation: OVATE PONTICS The extraction of a tooth entails removal of the contact point and half the interproximal embrasure; as a consequence, the papilla is not compressed but flattens out, and esthetics  are  compromised.   The papilla can be maintained if at the time of extraction an ovate  pontic  is created  that  will provide the contact point and lateral embrasure support  that the papilla needs.  An ovat  pontic is inserted 2.5 mm into the extraction site. The size and shape of the ovate pontic should be the same as the tooth that was extracted.   A site preservation bone grafting procedure should be performed at the time of the extraction. If the bone levels remain stable, the papilla will also be stable.  Key Concept Ovate pontics are used in cases where the residual ridge is defective or incompletely healed. Rosensteil 5th ED Page no 131,132" "This is a real-world medical entrance exam question, please give the true answer based on the question and selection. Topic name: None, Subject name: Radiology","The maximum penetration among the following is seen with which ray: Select and explain. A. Alpha, B. Beta, C. Gamma, D. Electron Beam",C: Gamma,"Answer: C: Gamma Explanation: None" "This is a real-world medical entrance exam question, please give the true answer based on the question and selection. Topic name: None, Subject name: Medicine","The red blood cells in beta thalassemia are typically: Select and explain. A. Macrocytic and normochromic, B. Microcytic and hypochromic, C. Normocytic and hypochromic, D. Normocytic and normochromic",B: Microcytic and hypochromic,"Answer: B: Microcytic and hypochromic Explanation: None" "This is a real-world medical entrance exam question, please give the true answer based on the question and selection. Topic name: None, Subject name: Medicine","A 60 years old patient who had myocardial infarction 2 weeks back, the lipid profile is done for a patient and reveals HDL 32 mgldL, LDL 126 mg/dl, TG 276 mgl/dL. Which of the following is preferred for this patient ? Select and explain. A. Rosuvastatin + Fenofibrate, B. Fenofibrate alone, C. Rosuvastatin 10 mg, D. Atorvastatin 80 mg",D: Atorvastatin 80 mg,"Answer: D: Atorvastatin 80 mg Explanation: Answer- D. Atorvastatin 80 mgWhen lipid-lowering agents are indicated, the treatment is staed with IIMG Co-A reductase inhibitors (statins). High intensity statin therapy should be staed in the patients with presence of clinical atherosclerotic cardiovascular events (MI in this patient). In high intensity statin therapy, either atorvastatin (40-80 mg/day) or rosuvastatin (20-40 mg/day) should be given.High lntensity Statin TherapyLowers LDL cholesterol by approximately 50% .Drugs used:Atorvastatin (40-80 mg)Rosuvastatin (20-40 mg)" "This is a real-world medical entrance exam question, please give the true answer based on the question and selection. Topic name: None, Subject name: Biochemistry","RNAi causes the following in a gene: Select and explain. A. Knock in, B. Knock out, C. Knock down, D. Knock up",C: Knock down,"Answer: C: Knock down Explanation: Ans. C. Knock down (Ref Robbins 9/e p5) RNAi causes Knock down in a gene.""Small interfering RNAs (siRNAs): Another species of gene-silencing RNA, called small interfering RNAs (siRNAs), works in a manner quite similar to that of miRNA. siRNAs are becoming powerful tools for studying gene function and may in the future be used therapeutically to silence specific genes, such as oncogenes, whose products are involved in neoplastic. transformation.""-Robbins 9/c, p5.Knock outTargeted gene is completely removedfrom the DNA sequence by replacing itwith an aificial piece of DNAdegKnock downGene is not completely removed but itsexpression is suppressed by using RNAinterference technologydegKnock inSegment of a gene is inseed into aDNA sequencedeg" "This is a real-world medical entrance exam question, please give the true answer based on the question and selection. Topic name: None, Subject name: Gynaecology & Obstetrics","In a patient with heart disease, which of the following should not be used to control PPH: Select and explain. A. Methylergometrine, B. Oxytocin, C. Misoprostol, D. Carboprost",A: Methylergometrine,"Answer: A: Methylergometrine Explanation: None" "This is a real-world medical entrance exam question, please give the true answer based on the question and selection. Topic name: None, Subject name: Dental","Pulpectomy and pulp capping are more successful in primary teeth because of: Select and explain. A. Increased odontoblastic activity in the young, B. Increased blood supply through the wide apex, C. Increased volume of pulp in primary teeth, D. Lesser amount of collagen fibres in young",B: Increased blood supply through the wide apex,"Answer: B: Increased blood supply through the wide apex Explanation: None" "This is a real-world medical entrance exam question, please give the true answer based on the question and selection. Topic name: None, Subject name: Physiology","Which are all the factors determining the GFR? Select and explain. A. Afferent aeriolar resistance, B. Efferent aeriolar resistance, C. Aerial pressure, D. All of the above",D: All of the above,"Answer: D: All of the above Explanation: Ans. D. All of the aboveDeterminants of GFR:Increase glomerular capillary hydrostatic pressure increases GFR. Glomerular hydrostatic pressure is determined by the following variables : i) Aerial pressure, ii)Afferent aeriolar resistance; iii) Efferent aeriolar resistance.Contraction of mesangial cells (therefore decrease in GFR) is produced by : - Endothelin, angiotensin II, vasopressin, norepinephrine, PAF, PDGF, thromboxane A2,PGF2, Leukotrienes C4 & D4, Histamine." "This is a real-world medical entrance exam question, please give the true answer based on the question and selection. Topic name: None, Subject name: Dental","Sclerotic dentin has following features: Select and explain. A. Caries susceptible, B. Insensitive, C. Hypersensitive, D. Resistant to caries",D: Resistant to caries,"Answer: D: Resistant to caries Explanation: None" "This is a real-world medical entrance exam question, please give the true answer based on the question and selection. Topic name: None, Subject name: Pathology","Complete obliteration of pulp is seen in all except: Select and explain. A. Type I dentinogenesis imperfecta, B. Type II dentinogenesis imperfecta, C. Type III dentinogenesis imperfecta, D. Dentin dysplasia",B: Type II dentinogenesis imperfecta,"Answer: B: Type II dentinogenesis imperfecta Explanation: None" "This is a real-world medical entrance exam question, please give the true answer based on the question and selection. Topic name: None, Subject name: Physiology","A spirometry curve of a patient has been provided below. Calculate the FEV1/FVC ratio from the curve: Select and explain. A. 60-69%, B. 70-79%, C. 80-89%, D. 90-99%",C: 80-89%,"Answer: C: 80-89% Explanation: Answer- C. 80-89%" "This is a real-world medical entrance exam question, please give the true answer based on the question and selection. Topic name: None, Subject name: Physiology","Minimum blood pressure is in: Select and explain. A. Aorta, B. Arteries, C. Capillaries, D. Venules",D: Venules,"Answer: D: Venules Explanation: None" "This is a real-world medical entrance exam question, please give the true answer based on the question and selection. Topic name: None, Subject name: Pathology","Streptococcus mutans is considered to be a principal etiologic agent of caries because it produces organic acids and it: Select and explain. A. Forms a gelatinous matrix, B. bMetabolizes substrate from saliva, C. Derives energy from enamel constituents, D. Lives symbiotically with lactobacillus",A: Forms a gelatinous matrix,"Answer: A: Forms a gelatinous matrix Explanation: None" "This is a real-world medical entrance exam question, please give the true answer based on the question and selection. Topic name: None, Subject name: Pediatrics","In Rh lso Immunisation, exchange transfusion is indicated if – Select and explain. A. Cord blood hemoglobin is less than 10 g %, B. Cord bilirubin is more than 5 mg, C. History of previous sibling affected, D. All of these",D: All of these,"Answer: D: All of these Explanation: Indications of Exchange transfusion Cord hemoglobin      _.10g/dL           o Bilirubin protein ratio >3.5        o Prematurity Cord bilirubin          >5mg/d             o Reticulocyte count > 15% Previous kernicterus or severe erythroblastosis in a sibling" "This is a real-world medical entrance exam question, please give the true answer based on the question and selection. Topic name: None, Subject name: Unknown","The following angle is associated with fracture of Select and explain. A. Calcaneum, B. Talus, C. Navicular, D. Cuboid",A: Calcaneum,"Answer: A: Calcaneum Explanation: It is flattening of angle subtended by the posterior articular surface and the upper surface of body posterior to the joint (Bohler angle) occurs in fracture of calcaneum. It is 25—40° normally. It is also known as tuber joint angle." "This is a real-world medical entrance exam question, please give the true answer based on the question and selection. Topic name: None, Subject name: Microbiology","Which of the following is not a feature of anaphylaxis? Select and explain. A. Delayed reaction, B. Immediate reaction, C. Passive transfer by serum, D. None of the above",A: Delayed reaction,"Answer: A: Delayed reaction Explanation: None" "This is a real-world medical entrance exam question, please give the true answer based on the question and selection. Topic name: None, Subject name: Pediatrics","An 8 year old boy during a routine checkup is found to have E. coli 1,00,000 cc/ml on a urine culture. The urine specimen was obtained by mid–stream clean–catch void. The child is asymptomatic. Which is the most appropriate next step in the management – Select and explain. A. Treat as an acute episode of urinary tract infection, B. No therapy, C. Prophylactic antibiotics for 6 months, D. Administer long term urine alkalinizer",A: Treat as an acute episode of urinary tract infection,"Answer: A: Treat as an acute episode of urinary tract infection Explanation: If the urine culture shows 3100000 colony counts/ml, the definitive diagnosis of U.T.I. is confirmed. It does not matter whether the patient is symptomatic or asymptomatic. If the diagnosis of U.T.I. is confirmed on urine culture, the patient should be treated accordingly. Criteria for diagnosis of U.T.L on midstream urine sample - ci If 3100,000 colonies/ml --> UT! confirmed (Does riot matter whether it is symptomatic or asymptomatic). If 10000 colonies/ml and the patient is asymptomatic --> UTI unlikely If 10000 colonies/m1 with sympt6ms of U.T.I. --> U.T.I is likely" "This is a real-world medical entrance exam question, please give the true answer based on the question and selection. Topic name: None, Subject name: Radiology","Which type of laser is used in waterlase system? Select and explain. A. Argon laser, B. Diode Laser, C. Nd:YAG laser, D. YSGG Laser",D: YSGG Laser,"Answer: D: YSGG Laser Explanation: It is an erbium laser (Er,Cr:YSGG) that has the ability to remove both hard and soft tissue safely and effectively. The WaterLase iPlus™ laser from BIOLASE, he says, is exceptionally versatile. “This is an all-tissue laser. Not only can it manipulate soft tissue by removing it for procedures such as biopsies and gingivectomies, it can also safely and effectively ablate tooth and bone structure." "This is a real-world medical entrance exam question, please give the true answer based on the question and selection. Topic name: None, Subject name: Pediatrics","Which of the following statements regarding Kawasaki disease is true – Select and explain. A. Associated with coronary artery aneurysm in up to 25% of untreated cases, B. It is the most common cause of vasculitis in children, C. IV immunoglobulins are recommended only if coronary artery is involved, D. Lymph node biopsy is used for diagnosis",A: Associated with coronary artery aneurysm in up to 25% of untreated cases,"Answer: A: Associated with coronary artery aneurysm in up to 25% of untreated cases Explanation: ""Coronary artery aneurysm develop in up to 25% of untreated patients"" - Nelson Kawasaki disease Kawasaki disease, also known as lymph node syndrome, mucocutaneous-lymph node syndrome, and infant polyarteritis, is a poorly understood self-limited vasculitis that affects many organs, including the skin mucous membrane, lymph nodes, heart and blood vessel walls. It is usually seen in children younger than 5 years. Kawasaki disease predominantly affect medium sized vessels, but may also affect small or large vessels. Presentation Presenting features may be divided into : - 1. Mucocutaneous, lymphnode involved Often begins with high grade fever. Erythma of mouth, red cracked lips, a strawberry tongue. Bilateral conjuctival injection with iritis and keratic precipitate. Red palms and soles Rash, which may take many forms (but not vesicular), on trunk. Rash with peeling of skin in genital area perineal desquamation. Cervical lymphadenopathy Beau's lines (transverse grooves on nails). Arthralgia 2. Heart The cardiac complications are the most important aspect of the disease. These are due to coronary vasculitis. Vasculitis causes coronary ectasia and coronary artery aneurism. Aneurism may lead to MI and sudden death. Myocarditis, pericarditis with small pericardial effusion. Valvular regurgitation Laborator findings Lymph node biopsy does not aid the diagnosis of Kawasaki disease (not used) The diagnosis of Kawasaki disease is based on presence of specific clinical signs (Clinical criteria). These is no specific laboratory test" "This is a real-world medical entrance exam question, please give the true answer based on the question and selection. Topic name: AIIMS 2018, Subject name: Medicine","A patient has had recurrent optic neuritis bilaterally with transverse myelitis. Visual acuity in right eye is 6/60 and left eye is 6/18. Patient showed a 50% response to steroids. Diagnosis is? Select and explain. A. Neuromyelitis Optica, B. Subacute combined degeneration of spinal cord(SACD), C. Post cerebral aery stroke, D. Neuro-syphilis",A: Neuromyelitis Optica,"Answer: A: Neuromyelitis Optica Explanation: Combination of transverse myelitis and optic neuritis is diagnosis of Neuromyelitis optica. SACD:- has only spinal cord involvement. Babinski sign (+), Areflexia, Romberg sign Post cerebral aery stroke - Do not cause myelitis & response to steroids. Mid brain stroke: P1 segment involvement Coical blindness d/t inv. of occipital lobe (P2 segment) Neuro-syphilis - Responsive to Antibiotics rather than steroid Argyll Robeson pupil Lhermitte sign Sensory ataxia/tabes dorsalis Neuromyelitis Optica (Devic's Disease) Optic neuritis Transverse myelitis Area postrema syndrome Diencephalic syndrome Acute brainstem syndrome Cerebral syndrome (coical syndrome)" "This is a real-world medical entrance exam question, please give the true answer based on the question and selection. Topic name: None, Subject name: Gynaecology & Obstetrics","All of the following are reversible long acting contraceptives except:? Select and explain. A. Copper T, B. Laparoscopic tubal ligation, C. LNG- IUS, D. Implanon",B: Laparoscopic tubal ligation,"Answer: B: Laparoscopic tubal ligation Explanation: Ans. b. Laparoscopic tubal ligationIntrauterine device (IUD), also known as long-acting reversible contraception (LARC).Laparoscopic tubal ligation is the most common modality of pregnancy prevention. It is irreversible in nature.Spring-loaded clip (Filshie clip, Hulka clip) or Silastic rubber band (Falope ring) are most commonly used.Destruction or removal of a segment of the oviduct is performed in an operating room through a transabdominal approach usually using a laparoscopy or minilaparotomy.An ectopic pregnancy should be ruled out; if the procedure fails.Failure rate is 1 in 200." "This is a real-world medical entrance exam question, please give the true answer based on the question and selection. Topic name: None, Subject name: Surgery","The maxillary sinus drains into the Select and explain. A. Middle meatus, B. Inferior meatus, C. Superior meatus, D. Sphenoethmoidal recess",A: Middle meatus,"Answer: A: Middle meatus Explanation: None" "This is a real-world medical entrance exam question, please give the true answer based on the question and selection. Topic name: None, Subject name: Pharmacology","Drug not requiring dose adjustment in renal disease is Select and explain. A. Aminoglycoside, B. Vancomycin, C. Calcium channel blocker, D. Metronidazole",C: Calcium channel blocker,"Answer: C: Calcium channel blocker Explanation: None" "This is a real-world medical entrance exam question, please give the true answer based on the question and selection. Topic name: None, Subject name: Dental","Occlusal adjustment is affected by all except: Select and explain. A. Compensating curve, B. Cusp height, C. Incisal guidance, D. Postural position",D: Postural position,"Answer: D: Postural position Explanation: None" "This is a real-world medical entrance exam question, please give the true answer based on the question and selection. Topic name: AIIMS 2017, Subject name: Pediatrics","Dried blood spotes test in neonates is used in testing for:- Select and explain. A. Inborn error of metabolism, B. Blood group, C. Total cell count, D. Creatinine and bilirubin",A: Inborn error of metabolism,"Answer: A: Inborn error of metabolism Explanation: Dried blood spots are used in Tandem Mass Spectrometry as screening test for inborn errors of metabolism. Another screening test used- GCMS gas chromatography mass spectroscopy." "This is a real-world medical entrance exam question, please give the true answer based on the question and selection. Topic name: None, Subject name: Social & Preventive Medicine","A research was undeaken by a group of psychiatrists and obstetricians to assess postpaum depression in mothers giving bih to male versus female children according to the Edinburgh depression scale (EPDS). What test should be used to compare the outcomes? Select and explain. A. Student's t-test, B. Paired t-test, C. Chi-square test, D. Pearson's correlation coefficient",C: Chi-square test,"Answer: C: Chi-square test Explanation: Answer- C. Chi-square test Chi-square test- Comparison of two independent qualitative variablesA 'non-parametric test' of significanceUsed to 'test significance of association between 2 or more qualitative characteristicsUsed to compare propoions in 2 or more groupsUsed for non-Normal (non-Gaussian) distributions" "This is a real-world medical entrance exam question, please give the true answer based on the question and selection. Topic name: None, Subject name: Dental","Splinting of cervical root fracture is done for Select and explain. A. 40 days, B. 4 months, C. 4 weeks, D. 14 days",B: 4 months,"Answer: B: 4 months Explanation: None" "This is a real-world medical entrance exam question, please give the true answer based on the question and selection. Topic name: None, Subject name: Physiology","true is all except regarding intrafusal fibres Select and explain. A. Nuclear bag fibres are lesser, B. Nuclear chain fibres are shorter and thinner, C. Secondary endings excited by chain fibres only, D. Primary endings excited by bag fibres only",D: Primary endings excited by bag fibres only,"Answer: D: Primary endings excited by bag fibres only Explanation: None" "This is a real-world medical entrance exam question, please give the true answer based on the question and selection. Topic name: None, Subject name: Biochemistry","Which acid is formed in the citric acid cycle? Select and explain. A. Oxaloacetic acid, B. Glutamic acid, C. Nitric acid, D. None of the above",A: Oxaloacetic acid,"Answer: A: Oxaloacetic acid Explanation: None" "This is a real-world medical entrance exam question, please give the true answer based on the question and selection. Topic name: None, Subject name: Anatomy","Foramen caecum is seen in: Select and explain. A. Ethmoid bone, B. Tongue, C. Sphenoid, D. Option 1 and 2",D: Option 1 and 2,"Answer: D: Option 1 and 2 Explanation: None" "This is a real-world medical entrance exam question, please give the true answer based on the question and selection. Topic name: None, Subject name: Surgery","A 48-years-old male reported to you with fractured central incisor with fracture line extending 4 mm gingival below CEJ with thin bone buccally and thick apicopalatally, treatment of choice is: Select and explain. A. Socket shield technique, B. Post and core, C. Extraction and implant, D. Socket preservation",A: Socket shield technique,"Answer: A: Socket shield technique Explanation: None" "This is a real-world medical entrance exam question, please give the true answer based on the question and selection. Topic name: None, Subject name: Surgery","Open reduction & fixation of fracture of mandibular condyle is indicated when Select and explain. A. Shortening of ramal height is more than 5 mm, B. Superior 5 mm condyle is fractured, C. Green stick fracture in children, D. Condylar fracture which is not dislocated",A: Shortening of ramal height is more than 5 mm,"Answer: A: Shortening of ramal height is more than 5 mm Explanation: None" "This is a real-world medical entrance exam question, please give the true answer based on the question and selection. Topic name: None, Subject name: Pediatrics","A child presented with respiratory distress was brought to emergency with bag and mask ventilation. Now child is intubated. Chest X–ray shows right–sided deviation of mediastinum with scaphoid abdomen. His Pulse rate is increased. What is the next step ? Select and explain. A. Endotracheal intubation, B. Put a nasogastric tube, C. Surgery, D. End tidal CO2 to confirm intubation",B: Put a nasogastric tube,"Answer: B: Put a nasogastric tube Explanation: Mediastnal deviation, scaphoid abdomen with respiratory distress suggest the diagnosis of congenital diaphragmatic hernia (CDH). The resuscitation of CDH patient consist of Stabilization by mechanical ventilation with 100% 02. Nasogastric suction This child has already been intubated. Now nasogastric suction should be done to aspirate swallowed air and to prevent distension of the herniated bowel. which would further compress the lung." "This is a real-world medical entrance exam question, please give the true answer based on the question and selection. Topic name: AIIMS 2017, Subject name: Anatomy","In the following diagram, the structure marked with arrow has all of the following cell junctions except? Select and explain. A. Zona occludens, B. Zona adherens, C. Macula adherens, D. Gap junction",A: Zona occludens,"Answer: A: Zona occludens Explanation: The image given is cardiac muscle, as it is showing Intercalated discs which is typical for cardiac muscle & Perinuclear halo is seen. It is striated muscle (not as striated as skeletal muscle) Usually in a cell, the following cell junctions are seen in a sequence Surface of cell Tight junctions (most superficial) - Absent in cardiac muscle Zona adherens (20nm) Desmosomes/Macula adherens (25mm) - holds 2 cells together Gap Junctions - imp in cardiac muscle Gap Junctions are narrow gaps usually working as electrical synapses, they let pass ca+2 ions & maintain quick transfer of cardiac impulses & maintain rhythmic contractions If gap junctions are compromised, Problem occurs in electrical synapses which leads to arrhythmia. Tight junctions (Zona occludens) are absents in cardiac muscle. Hemi-desmosomes attach cell to basement membrane. Given structure is not skeletal muscle because skeletal muscle is more striated & there is no peri-nuclear halo in it. It is not smooth muscle because smooth muscle is spindle shaped and is not striated." "This is a real-world medical entrance exam question, please give the true answer based on the question and selection. Topic name: None, Subject name: Surgery","Best graft for alveolar cleft surgery (Or) Patient with cleft palate graft taken from Select and explain. A. Posterior iliac crest, B. Anterior iliac crest, C. Clavicle, D. Rib",A: Posterior iliac crest,"Answer: A: Posterior iliac crest Explanation: None" "This is a real-world medical entrance exam question, please give the true answer based on the question and selection. Topic name: None, Subject name: Medicine","Calculate the base deficit in a patient of weight 75 kg with a pH = 6.96, pCO2 = 30 mm Hg and HCO3- = 6 mEq/L: Select and explain. A. 300 mEq, B. 400 mEq, C. 500 mEq, D. 800 mEq",C: 500 mEq,"Answer: C: 500 mEq Explanation: Ans: C. 500 mEq(Ref Harrison 19/e p317, 12/e p368).Base deficit in question = 400 mEq.Base excess & base deficit:Refer to an excess or deficit respectively in amount of base present in blood.Total bicarbonate deficit = 0.3 x weight (kg) x base deficitBase deficit in mEq = 0.4 x weight (kg) x Total Base Deficit = (24-6) x 0.4 x 75 = 540 mEq = 500 mEq (approx.)To obtain deficit in grams, divide mEq by 12." "This is a real-world medical entrance exam question, please give the true answer based on the question and selection. Topic name: None, Subject name: Dental","About zinc oxide eugenol all are true except Select and explain. A. ZnO is converted to Zn(OH)2, B. Reaction is autocatalytic, C. Water is the byproduct of reaction, D. Dehydrated ZnO reacts with dehydrated eugenol",D: Dehydrated ZnO reacts with dehydrated eugenol,"Answer: D: Dehydrated ZnO reacts with dehydrated eugenol Explanation: None" "This is a real-world medical entrance exam question, please give the true answer based on the question and selection. Topic name: None, Subject name: Dental","In the triangle formed by projection of the orifices of the canals of maxillary molars Select and explain. A. Line connecting lingual with mesial is longest, B. Line connecting mesial to distal is longest, C. Line connecting distal to lingual is longest, D. All lines are equal",A: Line connecting lingual with mesial is longest,"Answer: A: Line connecting lingual with mesial is longest Explanation: None" "This is a real-world medical entrance exam question, please give the true answer based on the question and selection. Topic name: AIIMS 2018, Subject name: Pathology","Storage temperature of RBC, Platelet, and Fresh Frozen Plasma (FFP) are: Select and explain. A. RBC 2-6oC, Platelet 20-22oC, FFP-30oC, B. RBC - 30oC, FFP 2-6oC, Platelet 20-22oC, C. RBC 20-22oC, Platelet 2-6oC, FFP-30oC, D. RBC 20-22oC, FFP-30o C, Platelet 2-6oC","A: RBC 2-6oC, Platelet 20-22oC, FFP-30oC","Answer: A: RBC 2-6oC, Platelet 20-22oC, FFP-30oC Explanation: Platelets are stored at 20-24?C with continuous agitation. Since they are present at room temperature transfusion related infections are high with platelet transfusion Packed RBC's are stored at a temperature of 2-6?C FFP and cryoprecipitate are stored at -18 to -30? C" "This is a real-world medical entrance exam question, please give the true answer based on the question and selection. Topic name: None, Subject name: Dental","C shaped root canals are preferably found in (Maximum incidence) Select and explain. A. Mand 1st molar, B. Maxillary 2nd molar, C. Maxillary 1stmolar, D. Mand 2nd molar",D: Mand 2nd molar,"Answer: D: Mand 2nd molar Explanation: None" "This is a real-world medical entrance exam question, please give the true answer based on the question and selection. Topic name: None, Subject name: Surgery","A patient underwent thyroidectomy for hypehyroidism. Two days later he was presented with features of thyroid storm. What is the most likely cause? Select and explain. A. Poor antibiotic coverage, B. Rough handling during surgery, C. Removal of parathyroid, D. Inadequate preoperative preparation",D: Inadequate preoperative preparation,"Answer: D: Inadequate preoperative preparation Explanation: Answer DThyrotoxic crisis (storm)- This is an acute exacerbation of hypehyroidism.It occurs if a thyrotoxic patient has been inadequately prepared for thyroidectomy and is now extremely rare.Features: They present in 12-24 hours after surgery; with severe dehydration, circulatory collapse, hypotension, hyperpyrexia, tachypnoea, hyperventilation, palpitation, restlessness, tremor, delirium, diarrhea, vomiting, and cardiac failure; later coma.Treatment: Injection hydrocoisone, oral antithyroid drugs, tepid sponging of the whole body, beta-blocker injection, oral iodides, a large amount of IV fluids for rehydration, digitoxin, cardiac monitor, often ventilator suppo, with close observation." "This is a real-world medical entrance exam question, please give the true answer based on the question and selection. Topic name: None, Subject name: Surgery","A 27 years old lady with 20 weeks pregnancy presented with a thyroid nodule on right side. FNAC from the nodule was suggestive of papillary carcinoma. Which of the following is contraindicated in her management? Select and explain. A. Total thyroidectomy plus neck node dissection, B. Right lobectomy, C. Radioactive iodine ablation, D. Total thyroidectomy",C: Radioactive iodine ablation,"Answer: C: Radioactive iodine ablation Explanation: Answer- BRadioactive iodine ablation is contraindicated in pregnancy because of severe teratogenic effects.Contraindications-ChildhoodPregnancyLactationThe standard primary treatment for patients with thyroid cancer remains total or near total thyroidectomy. In a pregnant woman, both maternal and fetal outcomes shall be considered before taking the decision of surgery." "This is a real-world medical entrance exam question, please give the true answer based on the question and selection. Topic name: None, Subject name: Anatomy","All the following features of skull of a newborn are true except: Select and explain. A. Diploe not formed, B. Styloid process has not fused with rest of temporal bone, C. Anterior fontanelle open, D. Mastoid process is of adult size",D: Mastoid process is of adult size,"Answer: D: Mastoid process is of adult size Explanation: Features of foetal skull: Large skull / calvaria.  No diploe, which appears by 4th year of life. Prominent frontal and parietal protuberance. Presence of metopic suture between 2 halves of frontal bone. Presence of unossified 6 fontanelles at angles of parietal bones. Non fusion of styloid process with rest of temporal bone. Mastoid process is absent, which appears by 2nd year of life. Paranasal air sinuses are rudimentary/absent. Internal ear, tympanic cavity, tympanic antrum and ear ossicles are of adult size." "This is a real-world medical entrance exam question, please give the true answer based on the question and selection. Topic name: None, Subject name: Medicine","Diminution in normal size of the body, well proportioned body, fine sparse hair on the head and other hairy regions wrinkled atrophic skin and often hypogonadism could be diagnosed as suffering from: Select and explain. A. Hypothyroidism, B. Hypopituitarism, C. Hyperthyroidism, D. None of the above",B: Hypopituitarism,"Answer: B: Hypopituitarism Explanation: None" "This is a real-world medical entrance exam question, please give the true answer based on the question and selection. Topic name: None, Subject name: ENT","In Dysphonia plica ventricularis, sound is produced by - Select and explain. A. False vocal cords, B. True vocal cords, C. Ventricle of larynx, D. Tongue",A: False vocal cords,"Answer: A: False vocal cords Explanation: In dysphonia plica ventricularis voice is produced by false vocal cords (ventricular folds)." "This is a real-world medical entrance exam question, please give the true answer based on the question and selection. Topic name: None, Subject name: Skin","Maculopapular rash is not seen in ? Select and explain. A. Dengue, B. Rubella, C. measles, D. Zoster",D: Zoster,"Answer: D: Zoster Explanation: Ans. D. ZosterMaculopapular rashes are seen in:* Rubella* Dengue(febrile phase)* Measles* Chikungunya (acute infectious phase)* Erythema infectiosum* Zika virus* West nile virus* Infectious mononucleosis" "This is a real-world medical entrance exam question, please give the true answer based on the question and selection. Topic name: None, Subject name: Anaesthesia","Which anesthetic agent can cause pain on IV adminis-tration? Select and explain. A. Ketamine, B. Propofol, C. Thiopentone, D. Midazolam",B: Propofol,"Answer: B: Propofol Explanation: Ans: B. Propofol(Ref: Miller 6/e p318-320: Morgan 4/e p200-202: Lee 13/e p158-160).Propofol:Oil-based preparation containing soyabean oil, egg lecithin & glycerol.As it contains oil - Hence painful injection IV administration.Should be preceded or mixed with lignocaine." "This is a real-world medical entrance exam question, please give the true answer based on the question and selection. Topic name: None, Subject name: Medicine","Bacteria responsible for post-operative endocarditis is: Select and explain. A. Streptococcus viridans., B. Staphylococcus epidermidis., C. Streptococcus aureus., D. Streptococcus pneumoniae.",B: Staphylococcus epidermidis.,"Answer: B: Staphylococcus epidermidis. Explanation: None" "This is a real-world medical entrance exam question, please give the true answer based on the question and selection. Topic name: None, Subject name: Dental","Which tooth has a non functional cusp Select and explain. A. Lower 1st premolar, B. Upper 1st premolar, C. Lower 2nd premolar, D. Upper 1st premolar",A: Lower 1st premolar,"Answer: A: Lower 1st premolar Explanation: None" "This is a real-world medical entrance exam question, please give the true answer based on the question and selection. Topic name: None, Subject name: Dental","In periodontal flap most important factor to be considered is: Select and explain. A. Depth of vestibule, B. Frenum attachment, C. Amount of attached gingiva, D. Free gingiva",C: Amount of attached gingiva,"Answer: C: Amount of attached gingiva Explanation: None" "This is a real-world medical entrance exam question, please give the true answer based on the question and selection. Topic name: None, Subject name: Medicine","A young donor came to the blood bank for the first time for platelet apheresis with platelet count of 1.9L. During the course he developed paresthesias, circumoral numbness during donation. His vitals remain stable though. ECG showed tachycardia with ST-T changes. What is the reason for his symptoms? Select and explain. A. Hypovolemic shock, B. Hypocalcemia, C. Seizures, D. Allergic reaction",B: Hypocalcemia,"Answer: B: Hypocalcemia Explanation: Ans. b. HypocalcemiaClinical findings of paresthesias, circumoral numbness and ECG showing tachycardia with ST-T changes are highly suggestive of hypocalcemia, which has been caused by extensive transfusion of citrated blood.""Citrate, commonly used to anticoagulate blood components, chelates calcium and thereby inhibits the coagulation cascade. Hypocalcemia, manifested by circumoral numbness and/or tingling sensation of the fingers and toes, may result from multiple rapid transfusions.""-H,""Transient hypocalcemia is seen with severe sepsis, burns, acute kidney injury, and extensive transfusions with citrated blood." "This is a real-world medical entrance exam question, please give the true answer based on the question and selection. Topic name: None, Subject name: Dental","Mesiolingual developmental groove is seen in Select and explain. A. Maxillary 1st premolar, B. Mandibular lateral incisor, C. Maxillary lateral incisor, D. Mandibular 1st premolar",D: Mandibular 1st premolar,"Answer: D: Mandibular 1st premolar Explanation: Mesial Aspect  Lingual cusp is around two third the size of the buccal cusp  Extreme lingual slope is seen at the mesial marginal ridge  Between the mesial marginal ridge and the mesiolingual cusp ridge is the mesiolingual developmental groove  Mesial contact area is in the middle third" "This is a real-world medical entrance exam question, please give the true answer based on the question and selection. Topic name: None, Subject name: Gynaecology & Obstetrics","Most common cause of post neonatal mortality is: Select and explain. A. Genetic cause, B. Maternal health during pregnancy, C. Environmental causes, D. Conditions effecting in early neonatal period",C: Environmental causes,"Answer: C: Environmental causes Explanation: Deaths occurring from 28 days of life to under one year are called postneonatal death." "This is a real-world medical entrance exam question, please give the true answer based on the question and selection. Topic name: None, Subject name: Biochemistry","A patient has normal blood glucose level as estimated by glucose-oxidase peroxidase method, shows positive Benedicts test in urine. Which of the following is the most likely cause? Select and explain. A. Fructosemia, B. Denaturation of glucose, C. Galactosemia, D. False positive",C: Galactosemia,"Answer: C: Galactosemia Explanation: Ans: C. Galactosemia(Ref Harper 30, e p205,- Actsan 20,c p726,- Harrison I9/e p433e-5)Galactosemia:Apa from glucose, both fructose & galactose gives positive Benedict's test.Classic galactosemia:Caused by galactose 1-phosphate uridyl transferase (GALT) deficiency.Diagnosis:Urine of patient shows reducing sugar (galactose) - Detected by Benedict's reagent.Negative glucose oxidase test - Specific for glucose.Presence of reducing sugar (galactose) in urine." "This is a real-world medical entrance exam question, please give the true answer based on the question and selection. Topic name: None, Subject name: Pediatrics","Which of the following inborn errors of metabolism is associated with mental retardation –a) Alkaptonuriab) Homocystinuriac) Pentosuriad) Galactosemia Select and explain. A. c, B. d, C. bd, D. ac",C: bd,"Answer: C: bd Explanation: Metabolic disorders associated with mental retardation. Aminoaciduria —› Phenylketonuria, homocystinemia, histidinemia Galactosemia Mucopolysaccharidosis" "This is a real-world medical entrance exam question, please give the true answer based on the question and selection. Topic name: None, Subject name: Pediatrics","A 6 weeks old baby presents with cough and cold for the last three days. Respiratory rate is 48/min. Patient is febrile, there are no chest retractions but wheezing is present. Which of the following statement is not true? Select and explain. A. Antibiotics are not required, B. Child is suffering from pneumonia, C. Treat only wheezing, D. Treat only fever",B: Child is suffering from pneumonia,"Answer: B: Child is suffering from pneumonia Explanation: Answer- B. Child is suffering from pneumoniaPneumonia:The World Health Organization has defined pneumonia in children clinically based on either a cough or difficulty breathing and a rapid respiratory rate, chest indrawing, or a decreased level of consciousness.A rapid respiratory rate is defined as >60 breaths/minute in children <2 months old, 50 breaths/minute in children 2 months to 1 year old, or >40 breaths/minute in children 1 to 5 years old." "This is a real-world medical entrance exam question, please give the true answer based on the question and selection. Topic name: None, Subject name: Surgery","A patient with Ludwig's angina comes to you. What will be your first treatment step: Select and explain. A. Incision and drainage, B. Start IV antibiotics and ICU admission, C. Tracheostomy under GA, D. Fiberoptic intubation/tracheostomy under LA",D: Fiberoptic intubation/tracheostomy under LA,"Answer: D: Fiberoptic intubation/tracheostomy under LA Explanation: None" "This is a real-world medical entrance exam question, please give the true answer based on the question and selection. Topic name: None, Subject name: Dental","Single accessory canal arising from coronal third of the mandibular molars arises majorly in Select and explain. A. Distal root, B. Mesial root, C. Both equally, D. Does not arise at all",A: Distal root,"Answer: A: Distal root Explanation: None" "This is a real-world medical entrance exam question, please give the true answer based on the question and selection. Topic name: None, Subject name: Dental","Indirect Retainer is placed: Select and explain. A. Near direct retainer, B. As far as possible from fulcrum line, C. Near fulcrum line, D. Near edentulous area",B: As far as possible from fulcrum line,"Answer: B: As far as possible from fulcrum line Explanation: None" "This is a real-world medical entrance exam question, please give the true answer based on the question and selection. Topic name: None, Subject name: Pharmacology","Post operative thromboprophylaxis by low molecular weight heparin is best done by which route of administration? Select and explain. A. Subcutaneous, B. Intravenous, C. Inhalational, D. Intramuscular",A: Subcutaneous,"Answer: A: Subcutaneous Explanation: Ans. A. Subcutaneous* Both heparin 5,000 units subcutaneously every 8 hours and enoxaparin 40 mg subcutaneously once daily provide highly effective and safe prophylaxis for patients undergoing colorectal surgery." "This is a real-world medical entrance exam question, please give the true answer based on the question and selection. Topic name: None, Subject name: Dental","Most important initiative factor for periodontitis is: Select and explain. A. Dental plaque, B. Calculus, C. Trauma from occlusion, D. Food debris layer",A: Dental plaque,"Answer: A: Dental plaque Explanation: None" "This is a real-world medical entrance exam question, please give the true answer based on the question and selection. Topic name: None, Subject name: Medicine","According to WHO guidelines, latent TB should be ruled out in all the following situations except: Select and explain. A. Before treatment with TN F-alpha inhibitors, B. Chronic alcoholics, C. Silicosis, D. Hemodialysis",B: Chronic alcoholics,"Answer: B: Chronic alcoholics Explanation: Ans: B. Chronic alcoholicsAccording to WHO guidelines, latent TB should be ruled out before treatment with TNF-alpha inhibitors, in patients of silicosis and patients undergoing hemodialysis among the provided options.Specific High-risk PopulationsHIV infectiondegRecent contact with an infectious patientInitiation of an anti-tumor necrosis factor (TNF) treatmentdegReceiving dialysisdegReceiving an organ or hematologic transplantation SilicosisdegBeing in prisonBeing an immigrant from highTB burden countriesBeing a homeless personBeing an illicit drug user" "This is a real-world medical entrance exam question, please give the true answer based on the question and selection. Topic name: None, Subject name: Radiology","Maximum recommended dose for dental X-ray operator in a week is Select and explain. A. 100 milliroentgen, B. 10 roentgen, C. 100 roentgen, D. 300 roentgen",A: 100 milliroentgen,"Answer: A: 100 milliroentgen Explanation: None" "This is a real-world medical entrance exam question, please give the true answer based on the question and selection. Topic name: None, Subject name: Surgery","When adequate bone buttressing is present, the AO system of fixation suggests Select and explain. A. Compression Osteogenesis, B. Load bearing fixation, C. Load sharing fixation, D. Non-rigid fixation",C: Load sharing fixation,"Answer: C: Load sharing fixation Explanation: None" "This is a real-world medical entrance exam question, please give the true answer based on the question and selection. Topic name: None, Subject name: Surgery","A 36 years old patient underwent breast conservation therapy and chemotherapy for a 1.5 x 1.2 cm ER positive breast cancer with one positive axillary lymph node. She is now on tamoxifen. How will you follow-up the patient? Select and explain. A. Annual bone scan, B. Assessment of tumor markers 6 monthly, C. Routine clinical examination 3 monthly in 1st year with annual mammogram, D. Routine clinical examination 3 monthly and 6 monthly liver function tests",C: Routine clinical examination 3 monthly in 1st year with annual mammogram,"Answer: C: Routine clinical examination 3 monthly in 1st year with annual mammogram Explanation: Answer- C. Routine clinical examination 3 monthly in 1st year with annual mammogramBreast cancer follow-up should be done with history & physical examination Every 3 to 6 months for the first 3 years, every 6 to 12 months,4 and 5 years, annually thereafter; Mammography annually, beginning no earlier than 6 months after radiation therapy. There is currently no routine role for repeated measurements of tumour markers or imaging other than mammography." "This is a real-world medical entrance exam question, please give the true answer based on the question and selection. Topic name: None, Subject name: Biochemistry","On laboratory investigations in a patient, LDL was highly elevated but the level of LDL receptors was normal. Which of the following is most probable cause? Select and explain. A. Phosphorylation of LDL receptors, B. Lipoprotein lipase deficiency, C. Apo B-100 mutation, D. Cholesterol Acyl Co-A transferase deficiency",C: Apo B-100 mutation,"Answer: C: Apo B-100 mutation Explanation: Ans: C. Apo B-100 mutationIn Type Ila Familial hypercholesterolemia:Highly elevated levels of LDL with normal level of LDL receptors.Characterized mutation in ligand region of apoB-100.Differential diagnosis:""Familial Defective apoB-100 (FDB)""/""Autosomal dominant hypercholesterolemia (ADH) type 2"":Dominantly inherited disorder.Clinically resembles heterozygous familial hypercholesterolemia (FH) with elevated LDL-C levels and normal TGs.FDB - Caused by mutations in gene encoding apoB-100, specifically in LDL receptor-binding domain of apoB-100.Ref (Harrison 19/e p2438-2447, 18/e p3148)" "This is a real-world medical entrance exam question, please give the true answer based on the question and selection. Topic name: None, Subject name: Dental","Which is true regarding lathe cut silver alloy: Select and explain. A. Requires least amount of mercury, B. Achieves lowest compressive strength at 1 hr., C. Has tensile strength both at 15 min & 7 days comparable to high copper unicompositional alloy, D. Has Low creep",C: Has tensile strength both at 15 min & 7 days comparable to high copper unicompositional alloy,"Answer: C: Has tensile strength both at 15 min & 7 days comparable to high copper unicompositional alloy Explanation: None" "This is a real-world medical entrance exam question, please give the true answer based on the question and selection. Topic name: None, Subject name: Medicine","All of the following are indications for thrombolysis in a patient with stroke except: Select and explain. A. Age <18 years, B. Sustained BP >185/110 mm Fig despite treatment, C. On CT scan, edema less than 1/3rd of middle cerebral aery territory, D. Acute ischemic stroke within 3 hours of onset",B: Sustained BP >185/110 mm Fig despite treatment,"Answer: B: Sustained BP >185/110 mm Fig despite treatment Explanation: Ans: B. Sustained BP >185/110 mm Fig despite treatment(Ref: Harrison 19/e p759, 1605, 18/e p3273)A sustained BP >185/110 mull Hg despite treatment is a contraindication for thrombolytic therapy as it may lead to hemorrhage. Administration of IV Recombinant Tissue Plasminogen Activator (PA) for Acute Ischemic Stroke:IndicationContraindicationClinical diagnosis of strokeOnset of symptoms to time of drug administration <3 hoursCT scan showing no hemorrhage or edema of >1/3 of the MCA territoryAge > 18 years Sustained BP >185/110 mm Hg despite treatmentPlatelets <100,000; HCT <25%; glucose <50 or>400 mg/dLUse of heparin within 48 hours & prolonged PTT, or elevated INR" "This is a real-world medical entrance exam question, please give the true answer based on the question and selection. Topic name: AIIMS 2018, Subject name: Pharmacology","Morphine should not be used in the treatment of: Select and explain. A. Ischemic pain, B. Biliary colic, C. Cancer pain, D. Post operative pain",B: Biliary colic,"Answer: B: Biliary colic Explanation: Morphine (an opioid) is a strong analgesic. It is used in treatment of all types of pain like Crush injury, Fracture, MI, cancer pain, post-operative pain etc. However in Biliary colic (e.g. due to any stone blocking bile duct), morphine constricts sphincter of Oddi and increase the intrabiliary pressure. This increases the chances of rupture of bile duct. Therefore opioids like morphine are contra-indicated in biliary colic." "This is a real-world medical entrance exam question, please give the true answer based on the question and selection. Topic name: None, Subject name: Surgery","Most common site of esophageal carcinoma? Select and explain. A. Middle 1/3rd of esophagus, B. Upper 2/3rd of esophagus, C. Lower 2/3rd of esophagus, D. Middle 2/3rd of esophagus",A: Middle 1/3rd of esophagus,"Answer: A: Middle 1/3rd of esophagus Explanation: Answer- A. Middle 1/3rd of esophagus* Esophageal Carcinoma Most common site of ca esophagus is middle 1/3 of esophagus." "This is a real-world medical entrance exam question, please give the true answer based on the question and selection. Topic name: None, Subject name: Medicine","Serum chemistry analysis in a patient reveals pCo2=30mm Hg, pO2=105mm Hg, pH=7.46. This profile indicates Select and explain. A. Metabolic alkalosis, B. Metabolic acidosis, C. Respiratory alkalosis, D. Respiratory acidosis",C: Respiratory alkalosis,"Answer: C: Respiratory alkalosis Explanation: None" "This is a real-world medical entrance exam question, please give the true answer based on the question and selection. Topic name: None, Subject name: Surgery","If impacted 3rd molar is to be extracted in patient planned for bilateral sagittal split osteotomy, then extraction should be done Select and explain. A. 8-12 weeks after surgery, B. At the time of surgery, C. 1 month after surgery, D. 6 month before surgery",D: 6 month before surgery,"Answer: D: 6 month before surgery Explanation: None" "This is a real-world medical entrance exam question, please give the true answer based on the question and selection. Topic name: None, Subject name: Dental","To remove any occlusal pre-maturity, what should be the first step to be done: Select and explain. A. Check working side interference., B. Check non working interference., C. Check the occlusion in centric., D. Any of the above.",C: Check the occlusion in centric.,"Answer: C: Check the occlusion in centric. Explanation: To remove any occlusal pre maturity, we should always check the occlusion in centric first." "This is a real-world medical entrance exam question, please give the true answer based on the question and selection. Topic name: None, Subject name: Pathology","Swollen joint, anemic, loose teeth & dentin dysplasia are because of deficiency of: Select and explain. A. Vitamin C, B. Vitamin D, C. Vitamin B1, D. Vitamin E & D",A: Vitamin C,"Answer: A: Vitamin C Explanation: None" "This is a real-world medical entrance exam question, please give the true answer based on the question and selection. Topic name: None, Subject name: Microbiology","Which of the following structure is disrupted by Vibrio cholerae? Select and explain. A. Hemi desmosome, B. Gap junctions, C. Zona occludens, D. Zona adherens",C: Zona occludens,"Answer: C: Zona occludens Explanation: Ans: C. Zona occludens(Ref Sleisenger and Fordtran's textbook of Gastrointestinal and Liver Disease 10/e p1903).vibrio cholerae also produces additional toxins that may contribute to disease, including the zonula occludens toxin (ZOT) that alters intestinal permeability by acting on intestinal epithelial cells tight junctions, and the accessory choleraendotoxin (ACE)."" Cholera toxin:Vibrio cholerae secretes enterotoxin protein.Encoded by bacteriophage (CT Xfi resident in V. cholera).Enterotoxin:Made up of 1A subunit (composed of 1A1 & 1 A2 peptide joined by disulfide link) & 5B subunits.Other toxins:Zonula occludens toxin (ZOT):Alters intestinal permeability by acting on intestinal epithelial cells tight junctions.Accessory cholera endotoxin (ACE)." "This is a real-world medical entrance exam question, please give the true answer based on the question and selection. Topic name: None, Subject name: Medicine","In which of the following glucocoicoid is used? Select and explain. A. E.coli septicemia, B. Severe typhoid, C. Cerebral Malaria, D. Leishmaniasis",B: Severe typhoid,"Answer: B: Severe typhoid Explanation: Answer- B. Severe typhoidIn critically ill patients with enteric fever, the administration of dexamethasone with chloramphenicol was asssociated with a sabstantially lower moality rate than was treatment with chloramphenicol alone." "This is a real-world medical entrance exam question, please give the true answer based on the question and selection. Topic name: None, Subject name: Biochemistry","Which of the following is not required for protein synthesis of eukaryotes: Select and explain. A. RNA polymerase, B. Ribosomes, C. Peptidyl transferase, D. Amino acyl tRNA synthetase",A: RNA polymerase,"Answer: A: RNA polymerase Explanation: Ans. a. RNA polymerase (Ref Harper 28/e p362) RNA polymerase enzyme is involved in transcription process, not in translation.The a-amino group of the new aminoacyl-tRNA in the A site carries out a nucleophilic attach on the esterified carboxyl group of the peptidyl-tRNA occupying the P site (peptidyl or polypeptide site).Peptidyl transferase: Catalyses two reactions, peptide bond formation between amino acids and together with release factor, peptide release." "This is a real-world medical entrance exam question, please give the true answer based on the question and selection. Topic name: None, Subject name: Dental","3 weeks after insertion of distal extension partial denture, the patient reports to the dental clinic with complaint of sensitivity to chewing pressure. The most probable reason would be: Select and explain. A. Improper occlusion, B. Abrasion of tooth by clasp, C. Wearing off the cement base, D. Passive retentive arm",A: Improper occlusion,"Answer: A: Improper occlusion Explanation: None" "This is a real-world medical entrance exam question, please give the true answer based on the question and selection. Topic name: None, Subject name: Physiology","In the formula for urea clearance, C = U x V/P, U denotes: Select and explain. A. Urinary concentration in gm/24 hours, B. Urine osmolarity, C. Urinary concentration in mg/ml, D. Urine volume per minute",C: Urinary concentration in mg/ml,"Answer: C: Urinary concentration in mg/ml Explanation: Ans: C. Urinary concentration in mg/ml In the Given Formula:C = Clearance of the substanceU = Urinary concentration of the substance in mg/mlP = Plasma concentration of the substance in mg/mlV = Volume of urine" "This is a real-world medical entrance exam question, please give the true answer based on the question and selection. Topic name: None, Subject name: Dental","Prevention of any incipient lesion at initial stage comes under Select and explain. A. Primary prevention, B. Secondary prevention, C. Tertiary prevention, D. Primordial prevention",B: Secondary prevention,"Answer: B: Secondary prevention Explanation: None" "This is a real-world medical entrance exam question, please give the true answer based on the question and selection. Topic name: None, Subject name: Medicine","Relative bradycardia is uncommon in: Select and explain. A. Brucellosis, B. Typhoid, C. Malaria, D. Factitious fever",C: Malaria,"Answer: C: Malaria Explanation: Ans. C. MalariaRelative bradycardia at the peak of high fever is the early findings in <50% cases of typhoid.The presentation of Brucellosis often fits three patterns, among one febrile illness that resembles to thyroid, but it is less severe. Relative bradycardia seen in this case.In malaria relative bradycardia is uncommon." "This is a real-world medical entrance exam question, please give the true answer based on the question and selection. Topic name: None, Subject name: Surgery","The treatment of choice for stage-I cancer larynx is: Select and explain. A. Radical surgery, B. Chemotherapy, C. Radiotherapy, D. Surgery followed by radiotherapy",C: Radiotherapy,"Answer: C: Radiotherapy Explanation: None" "This is a real-world medical entrance exam question, please give the true answer based on the question and selection. Topic name: None, Subject name: Dental","Least pain sensitivity in oral cavity is in which mucosa Select and explain. A. Gingiva, B. Buccal mucosa, C. Labial mucosa, D. Dorsal mucosa of tongue",B: Buccal mucosa,"Answer: B: Buccal mucosa Explanation: None" "This is a real-world medical entrance exam question, please give the true answer based on the question and selection. Topic name: None, Subject name: Gynaecology & Obstetrics","A 16 years old girl came for evaluation of primary amenorrhea. She was having hirsutism, irregular bleeding and infeility, diagnosed as PCOS. Which of the following drugs should not be given? Select and explain. A. Spironolactone, B. Tamoxifen, C. OCPs, D. Clomiphene citrate",B: Tamoxifen,"Answer: B: Tamoxifen Explanation: Answer- B. TamoxifenMedical Treatment of PCOSEstrogen best given with progesterone (combined OCPs) with no androgenic propeiesHirsutism is treated with cyproterone acetate or spironolactone.Infeility is treated with Clomiphene, 80% ovulate and 40% conceive.In Clomiphene failed group, ovulation can be induced with FSH or GnRH analogues.Metformin treats the root cause of PCOS, rectifies endocrine and metabolic functions and improves feility and isdrug of choice." "This is a real-world medical entrance exam question, please give the true answer based on the question and selection. Topic name: None, Subject name: Dental","Which of the following are characteristic feature of high copper amalgam alloy? Select and explain. A. Low compressive strength, B. High marginal breakdown, C. Less marginal #, D. High creep",C: Less marginal #,"Answer: C: Less marginal # Explanation: None" "This is a real-world medical entrance exam question, please give the true answer based on the question and selection. Topic name: AIIMS 2018, Subject name: Pharmacology","Low apparent volume of distribution of a drug indicates That Select and explain. A. Drug has low half life, B. Drug has low bioavailability, C. Drug has low efficacy, D. Drug is not extensively distribution to tissue",D: Drug is not extensively distribution to tissue,"Answer: D: Drug is not extensively distribution to tissue Explanation: Vd means ""the plasma volume that would accommodate all the drug in the body, if the concentration throughout was same as in plasma"". Drugs with high volume of distribution are more distributed in body and vice-versa. Low volume of distribution tells that most of the drug is retained in plasma." "This is a real-world medical entrance exam question, please give the true answer based on the question and selection. Topic name: None, Subject name: Dental","The tissue response to oral hygiene instruction is best assessed by Select and explain. A. Probing the base of the socket, B. Changes in plaque scores, C. Reduced tendency to bleed on probing the gingival margin, D. Reduced tooth mobility",C: Reduced tendency to bleed on probing the gingival margin,"Answer: C: Reduced tendency to bleed on probing the gingival margin Explanation: None" "This is a real-world medical entrance exam question, please give the true answer based on the question and selection. Topic name: None, Subject name: Pediatrics","A pregnant lady had no complaints but mild cervical lymphadenopathy in first trimester. She was prescribed spiramycin but she was noncompliant. Baby was born with hydrocephalus and intracerebral calcification. Which of these is likely cause ? Select and explain. A. Rubella, B. Toxoplasmosis, C. CMV, D. Herpes",B: Toxoplasmosis,"Answer: B: Toxoplasmosis Explanation: Hydrocephalus and intracerebral calcification can occur in all the four options either commonly (Toxoplasma, CIVIV) or uncommonly (HSV, rubella). See previous explanation. However, spiramycin in pregnancy is given for toxoplasmosis. Acute maternal toxoplasmosis is usually asymptomatic, some patients may present with posterior cervical lymphadenopathy, fatigue, lassitude and maculopapular rash. Spiramycin reduces the risk of congenital infection in acute toxoplasmosis during pregnancy. As the patient was not compliant with spiramycin, baby developed congenital toxoplasmosis." "This is a real-world medical entrance exam question, please give the true answer based on the question and selection. Topic name: None, Subject name: Dental","Rampant caries should be applied to a caries rate of: Select and explain. A. 5 or more new lesions per year., B. 7 or more new lesions per year., C. 10 or more new lesions per year., D. 15 or more new lesions per year.",C: 10 or more new lesions per year.,"Answer: C: 10 or more new lesions per year. Explanation: None" "This is a real-world medical entrance exam question, please give the true answer based on the question and selection. Topic name: None, Subject name: Anatomy","Unpaired structures in the brain: Select and explain. A. Basilar artery, B. Vertebral artery, C. Middle cerebral artery, D. Anterior cerebral artery",A: Basilar artery,"Answer: A: Basilar artery Explanation: Unpaired structure in the brain is basilar artery." "This is a real-world medical entrance exam question, please give the true answer based on the question and selection. Topic name: AIIMS 2018, Subject name: Pharmacology","A paediatrician was called for attending a new born baby in the labour ward. The serum unconjugated bilirubin of this baby was 33 mg/dL. Which of the following drug taken by mother in late 3rd trimester may have lead to this problem? Select and explain. A. Cotrimoxazole, B. Azithromycin, C. Ampicillin, D. Chloroquine",A: Cotrimoxazole,"Answer: A: Cotrimoxazole Explanation: Bilirubin given in question is very high; Bilirubin binds to Albumin in blood, so when any drug displaces bilirubin from albumin, it can lead to increased free bilirubin. Cotrimoxazole is a combination of trimethoprim and sulfamethoxazole (a sulfonamide). Sulfonamides (acidic drugs) can displace bilirubin from albumin resulting in excessive free bilirubin. This can cross blood brain barrier in newborn babies which can cause Kernicterus; hence avoided in pregnancy." "This is a real-world medical entrance exam question, please give the true answer based on the question and selection. Topic name: None, Subject name: Pathology","Which of the following is not a small round cell tumor? Select and explain. A. Neuroblastoma, B. Retinoblastoma, C. Ewing's sarcoma, D. Hemangiosarcoma",D: Hemangiosarcoma,"Answer: D: Hemangiosarcoma Explanation: Ans. is 'd' i.e., hemangiosarcomaSmall round blue cell tumors:* Histologically many of the malignant paediatric neoplasms are unique.* They tend to have a more primitive (embryonal) rather than pleomorphic anaplastic microscopic appearance.* Often characterized by sheets of cells with small round nuclei.* Frequently exhibit features of organogenesis specific to the site of tumour origin (therefore these tumours are frequently designated by suffix - blastoma).* Owing to their primitive histological appearance many childhood tumours have been collectively referred to as small round blue cell tumourThe differential diagnosis of small round blue cell tumours includesNeuroblastomaRhabdomyosarcomaRetinoblastomaWilms tumoursEwings sarcomaMedulloblastomaLymphomaPrimitive neuroectodermal tumoursBurkitt lymphoma" "This is a real-world medical entrance exam question, please give the true answer based on the question and selection. Topic name: None, Subject name: Dental","Increased collum angle is seen in Select and explain. A. Class I bimaxillary protrusion, B. Class II div 1, C. Class II div 2, D. Class III",C: Class II div 2,"Answer: C: Class II div 2 Explanation: Collum angle The angulation of the root to the crown, particularly of the single rooted anterior teeth is  known as Collum angle. It is the angle formed by the intersection of the long axis of the crown and root and it is measured using the lateral cephalogram. Dental features of class II div 2 include a class II molar and canine relationship, deep traumatic bite, retroclined upper four incisors or retroclined central incisors with a labial inclination of the laterals and supra-erupted, upright lower incisors. The tooth size may be  small, and upper incisors may have increased collum angle between the crown and the root. The maxillary incisors in class II division 2 malocclusions are about 12° more vertical than those in normal occlusion. The mandibular incisors are upright in a class II division 2 malocclusion, but to a lesser extent than the upper incisors." "This is a real-world medical entrance exam question, please give the true answer based on the question and selection. Topic name: None, Subject name: Physiology","Which of the following is not a vitamin K dependent procoagulant? Select and explain. A. Factor II, B. Factor VII, C. Factor IX, D. Factor XI",D: Factor XI,"Answer: D: Factor XI Explanation: None" "This is a real-world medical entrance exam question, please give the true answer based on the question and selection. Topic name: None, Subject name: Anatomy","A patient got trauma and a metal rod inserted in superior orbital fissure causes damage of? Select and explain. A. Occulomotor nerve, B. Occulomotor and ophthalmic nerve, C. Only ophthalmic nerve, D. Trochlear and vagus nerve",B: Occulomotor and ophthalmic nerve,"Answer: B: Occulomotor and ophthalmic nerve Explanation: Superior orbital fissure- Present between lesser and greater wings of the sphenoid within the orbital cavity." "This is a real-world medical entrance exam question, please give the true answer based on the question and selection. Topic name: None, Subject name: Physiology","Which of the following defines the pressure in the vascular system in the absence of blood flow? Select and explain. A. Pulse pressure, B. Critical closing pressure, C. Mean circulatory filling pressure, D. Perfusion pressure",C: Mean circulatory filling pressure,"Answer: C: Mean circulatory filling pressure Explanation: Answer- C. Mean circulatory filling pressureMean Circulatory Filling pressure (MCFP) is equilibrium pressure that is reached throughout the cardiovascular system when cardiac output is stopped completely. MCFP can be measured by stopping blood flow (e.g., by stopping hea pumping by giving shock to hea with electricity) and allowing the pressure throughout the circulatory system to reach equilibrium. MCFP is equilibrium pressure everywhere in circulation." "This is a real-world medical entrance exam question, please give the true answer based on the question and selection. Topic name: None, Subject name: Pediatrics","Pure left sided failure may be seen with –a) ASDb) Aortic stenosisc) Patent ductus arteriosusd) Pulmonary valvular obstruction Select and explain. A. a, B. c, C. bc, D. ad",C: bc,"Answer: C: bc Explanation: ASD usually does not cause HF. AS & PDA increase the strain on left ventricle -> LVF. PS increases the strain on right ventricular failure ---> RVF." "This is a real-world medical entrance exam question, please give the true answer based on the question and selection. Topic name: None, Subject name: Surgery","What is the advantage of chromic gut over plain gut suture in deep wounds? Select and explain. A. Greater strength, B. Greater ease of use, C. Delayed resorption, D. Less tissue irritation",C: Delayed resorption,"Answer: C: Delayed resorption Explanation: None" "This is a real-world medical entrance exam question, please give the true answer based on the question and selection. Topic name: None, Subject name: Dental","Pellicle formation involves Select and explain. A. Adsorption of acidic glycoproteins from saliva, B. Focal areas of mineralisation, C. Focal areas of necrosis, D. Bacterial colonization",A: Adsorption of acidic glycoproteins from saliva,"Answer: A: Adsorption of acidic glycoproteins from saliva Explanation: None" "This is a real-world medical entrance exam question, please give the true answer based on the question and selection. Topic name: None, Subject name: Dental","Increase in the residual mercury in silver amalgam filling can: Select and explain. A. Cause fracture of the filling, B. Tarnish and corrosion, C. Increase the strength, D. Decrease condensation pressure",A: Cause fracture of the filling,"Answer: A: Cause fracture of the filling Explanation: None" "This is a real-world medical entrance exam question, please give the true answer based on the question and selection. Topic name: None, Subject name: Social & Preventive Medicine","Not included in the human poverty index is: Select and explain. A. % of population not surviving up to 40 yrs age, B. Occupation, C. Underweight for age, D. % population not using safe water supply",B: Occupation,"Answer: B: Occupation Explanation: None" "This is a real-world medical entrance exam question, please give the true answer based on the question and selection. Topic name: None, Subject name: Ophthalmology","A 33 years old male came with pain and watering in the right eye for 36 hours. On examination, a 3 x 2 cm corneal ulcer is seen with elevated margins, feathery hyphae, finger like projections and minimal hypopyon in cornea. What is the likely causative organism? Select and explain. A. Aspergillosis, B. Pseudomonas, C. Acanthamoeba, D. HSV-1",A: Aspergillosis,"Answer: A: Aspergillosis Explanation: Ans: A. AspergillosisRef: Yanoff & Duker 4/e p225; Parson 22/e p203, 21/e p199)Finding suggestive of fungal corneal ulcer. In the question, Aspergillus is the only fungus.Mycotic or fungal keratitis:Commonly due to Aspergillus, Fusarium or Candida albicans.Fungal ulcers are typically seen after injury with vegetable matter such as a thorn or wooden stick and are characterized by a relatively indolent course.Symptoms are much milder than the clinical signs would suggest.Dry slough with feathery borders, surrounded by a yellow line of demarcation, which gradually deepens into a gutter.An immune ring (Wessely) may be visible due to deposition of immune complexes and inflammatory cells around the ulcer.Marked ciliary and conjunctival congestion.Hypopyon is thick and immobile - Due to direct invasion into the anterior chamber of fungal hyphae enmeshed in thick exudates." "This is a real-world medical entrance exam question, please give the true answer based on the question and selection. Topic name: None, Subject name: Surgery","In a patient of oral & maxillofacial trauma, cervical injury is not ruled out, intubation of preference will be: Select and explain. A. LMA, B. Combitube, C. Normal laryngoscopy & endotracheal intubation, D. Fiberoptic intubation",D: Fiberoptic intubation,"Answer: D: Fiberoptic intubation Explanation: None" "This is a real-world medical entrance exam question, please give the true answer based on the question and selection. Topic name: None, Subject name: Surgery","Fine needle aspiration cytology (FNAC) is not enough to diagnose: Select and explain. A. Papillary carcinoma of thyroid, B. Follicular carcinoma of thyroid, C. Carcinoma breast, D. Adenocarcinoma lung",B: Follicular carcinoma of thyroid,"Answer: B: Follicular carcinoma of thyroid Explanation: Ans: B. Follicular carcinoma of the thyroid(Ref: Robbin's 9/e p1094, 8/e p1123).FNAC:Not enough to diagnose follicular carcinoma.Since follicular carcinoma is differentiated from follicular adenoma based on capsular invasion & vascular invasion (not diagnosed by FNAC).Limitations of FNAC in Thyroid Diseases:Not able to distinguish follicular adenoma from follicular carcinoma.Not able to distinguish Huhle cell adenoma from Huhle cell carcinomaUseless in Reidel's thyroiditis (Biopsy is preferred).FNAC is less reliable in patients who have a history of head & neck irradiation or family history of thyroid cancer due to the higher likelihood of multifocal lesions & occult cancer." "This is a real-world medical entrance exam question, please give the true answer based on the question and selection. Topic name: None, Subject name: Dental","Animal test to check the biocompatibility of dental material are all except Select and explain. A. Buehler test, B. Implantation test, C. Ame's test, D. Sensitization test",C: Ame's test,"Answer: C: Ame's test Explanation: None" "This is a real-world medical entrance exam question, please give the true answer based on the question and selection. Topic name: AIIMS 2017, Subject name: Medicine","Normal anion gap metabolic acidosis seen in all except? Select and explain. A. Proximal A, B. Severe pancreatitis, C. Diarrhea, D. Salicylates poisoning",D: Salicylates poisoning,"Answer: D: Salicylates poisoning Explanation: Classification of Anion GAP: High anion gap M.A Normal Anion gap M.A High Mud FLES Normal USed CARP Methanol Uremia DKA Paraldehyde IEM, Iron, INH Lactic acidosis Ethanol, athylene glycol Salicylate Ureterostomy Small bowel fistula Extra chloride Diarrhea Carb anhydrase inhibitor Adrenal insufficiency RTA Pancreatic Fistula" "This is a real-world medical entrance exam question, please give the true answer based on the question and selection. Topic name: None, Subject name: Psychiatry","A 40 years old male with history of fall and one episode of vomiting near an alcohol shop. He was brought to casualty and he was awake with open eyes and had retrograde amnesia. Which of the following is most likely cause? Select and explain. A. Diffuse axonal injury, B. Concussion, C. Drunkenness, D. Cerebral venous thrombosis",B: Concussion,"Answer: B: Concussion Explanation: Ans.BConcussion: It is the most common type of traumatic brain injury.During head trauma, the head usually moves back and foh violently and there is a mismatch between the rapid deceleration and acceleration of the skull and the brain so that the brain hits repeatedly against the skull leading to concussion injury.Anterograde amnesia/Posttraumatic amnesia is a hallmark of concussion.In this type of amnesia, the events following the injury cannot be recalled.Headache, dizziness, nausea, vomiting like symptoms may present" "This is a real-world medical entrance exam question, please give the true answer based on the question and selection. Topic name: None, Subject name: Physiology","DOPA and 5 - Hydroxytryptophan are clinically important because Select and explain. A. They cross Blood Brain Barriers, B. They are acidic precursors of Brain amines, C. They act as neuromodulators, D. They are metabolites of various neurogenic amines",A: They cross Blood Brain Barriers,"Answer: A: They cross Blood Brain Barriers Explanation: None" "This is a real-world medical entrance exam question, please give the true answer based on the question and selection. Topic name: None, Subject name: Surgery","Which among the following should be the last resort for giving L.A technique to the patient Select and explain. A. Intrapulpal, B. Intraligamentary, C. Intraosseous, D. Infiltration",A: Intrapulpal,"Answer: A: Intrapulpal Explanation: None" "This is a real-world medical entrance exam question, please give the true answer based on the question and selection. Topic name: None, Subject name: Pharmacology","Which of the following drug is used to counter act the gastric irritation produced by administration of NSAID: Select and explain. A. Roxatidine, B. Pirenzipine, C. Betaxolol, D. Misoprostol",D: Misoprostol,"Answer: D: Misoprostol Explanation: None" "This is a real-world medical entrance exam question, please give the true answer based on the question and selection. Topic name: None, Subject name: Pathology","Which of the following is associated with defective apoptosis and increased cell survival Select and explain. A. Neuro degenerative diseases, B. Auto immune disorders, C. Myocardial infarction, D. Stroke",B: Auto immune disorders,"Answer: B: Auto immune disorders Explanation: None" "This is a real-world medical entrance exam question, please give the true answer based on the question and selection. Topic name: None, Subject name: Forensic Medicine","A 16-year-old girl comes to a doctor with fractured forearm. She told she tripped and fell but cigarette burns were observed on her forearm. What will be your Next step? Select and explain. A. To inform higher authorities, B. To do a complete physical examination, C. To tell or discuss with colleagues that she is a case of abuse, D. To call local social worker for help",B: To do a complete physical examination,"Answer: B: To do a complete physical examination Explanation: Ans: B. To do a complete physical examinationThe first step after suspecting s case of abuse is to do a complete detailed history taking & physical examination of the patient to see any other injuries which can confirm the findings.Concluding just by seeing a burn that looks like a cigarette burn on the forearm can be misleading at times so a thorough examination of the whole body is mandatory to confirm the above finding to child abuse.after confirmation According to the POSCO Act, every crime of child sexual abuse should be repoed. If a person who has information about any abuse fails to repo, they may face imprisonment up to six months or maybe fined or both" "This is a real-world medical entrance exam question, please give the true answer based on the question and selection. Topic name: None, Subject name: Dental","Which of the following Is not an etiological agent for gingival recession Select and explain. A. Gingival abrasion, B. Gingival ablation, C. Gingival inflammation, D. TFO",D: TFO,"Answer: D: TFO Explanation: None" "This is a real-world medical entrance exam question, please give the true answer based on the question and selection. Topic name: None, Subject name: Gynaecology & Obstetrics","Maximum risk of ureter injury is seen after: Select and explain. A. Vaginal hysterectomy, B. Weheim's hysterectomy, C. Laparoscopic abdominal hysterectomy, D. Anterior colporrhaphy",B: Weheim's hysterectomy,"Answer: B: Weheim's hysterectomy Explanation: Answer- B. Weheim's hysterectomyRadical or Weheim's hysterectomy is associated with the highest risk of ureteric injury. The risk of ureteral injury at vaginal hysterectomy is higher (0.6%) than with an open abdominal approach (0.07%)." "This is a real-world medical entrance exam question, please give the true answer based on the question and selection. Topic name: AIIMS 2018, Subject name: Medicine","A 45 year patient working in a factory for past 20 years presents with breathlessness. HRCT chest shows pleural thickening and fibrosis. Histopathology of the lesion shows? Select and explain. A. Asbestosis, B. Cotton fiber, C. Coal worker pneumoconiosis, D. Silicosis",A: Asbestosis,"Answer: A: Asbestosis Explanation: Option A: In interalveolar septa There is presence of Irregular lobulated fusiform beaded bodies Ferruginous bodies- Diagnostic of asbestosis Pleural thickening Option B: Cotton Fiber-Byssinosis Option C: Coal worker pneumoconiosis Pleural thickening manifestation not seen. Option D: Silicosis is associated with Bilateral HilarLymphederopathy & development of infiltrative lung disease." "This is a real-world medical entrance exam question, please give the true answer based on the question and selection. Topic name: None, Subject name: Ophthalmology","All of the following can be used to decrease IOP in glaucoma except Select and explain. A. Mannitol, B. Carbonic anhydrase inhibitor, C. Apraclonidine, D. Dexamethasone",D: Dexamethasone,"Answer: D: Dexamethasone Explanation: Answer: D. DexamethasoneSteroids cannot be given in glaucoma as they can themselves lead to increase in intraocular pressure" "This is a real-world medical entrance exam question, please give the true answer based on the question and selection. Topic name: None, Subject name: Dental","In which of the following conditions is the role of microbial plaque most obscure: Select and explain. A. Periodontitis, B. Juvenile periodontitis, C. Desquamative gingivitis, D. Necrotising ulcerative gingivitis",C: Desquamative gingivitis,"Answer: C: Desquamative gingivitis Explanation: None" "This is a real-world medical entrance exam question, please give the true answer based on the question and selection. Topic name: None, Subject name: Gynaecology & Obstetrics","What is the first sign of pubey in a girl? Select and explain. A. Thelarche, B. Menarche, C. Adrenarche, D. Pubarche",A: Thelarche,"Answer: A: Thelarche Explanation: Ans: A. ThelarcheThe first physical sign of pubey in girls is usually a firm, tender lump under the center of the areola of one or both breasts; occurring on average at about 10.5 years of age. This is referred to as thelarche.Order of Signs of PubeyMales (TPAM)Females (TPM)Testicular enlargement (First sign)degPubarchedegAdrenarchedegMoustache & BearddegThelarche (First sign)degPubarchedegMenarchedeg Onset of PubeyMalesFemalesGrowth of testes ( 24 mL in volume or 2.5 cm in longest diameter) & thinning of scrotum are first signs of pubey (11- 12 year)deg.These are followed by pigmentation of scrotum & growth of penis & by pubarchedeg.Appearance of axillary hair usually occurs in mid-pubeydeg.In males, unlike in females, acceleration of growth is maximal at genital stages IV-V (typically between 13 & 14 years of age)deg.In males, growth spu occurs approximately 2 year later than in females & growth may continue beyond 18 years of agedeg.Breast development (thelarche) is usually first sign of pubey (10-11 years of age)Followed by the appearance of pubic hair (pubarche) 6-12 months laterdeg.Interval to the onset of menstrual activity (menarche) is usually 2-2.5 years, but may be as long as 6 yearsdeg.Peak height velocity occurs early (at breast stages 11-81 typically between 11-12 years of age) in girls and always precedes menarchedeg.Mean age of menarche is approximately 12.75 yearsdeg." "This is a real-world medical entrance exam question, please give the true answer based on the question and selection. Topic name: None, Subject name: Pediatrics","Hypoxic Ischemic encephalopathy true is – Select and explain. A. Lower limbs affected more than upper limbs, B. Prox. Muscles > distal muscles, C. Seizure, D. Trunk involved",C: Seizure,"Answer: C: Seizure Explanation: Clinical features of hypoxic ischemic encephalopathy Encephalopathy progress over time - Birth to 12 hours --> Decreased level of conciousness, poor tone, decreased spontaneous movement, periodic breathing or apnea, seizures. 12-24 hours --> More seizuers, Apneic spells, jitteriness, weakness. After 24 hours —> Hypotonia, conciousness, poor feeding, brainstem signs (oculomotor) and pupillary disturbances." "This is a real-world medical entrance exam question, please give the true answer based on the question and selection. Topic name: AIIMS 2017, Subject name: Pathology","Which is the best anticoagulant to send sample for serum electrolyte measurement? Select and explain. A. EDTA, B. Lithium heparin, C. Sodium fluoride, D. Citrate",B: Lithium heparin,"Answer: B: Lithium heparin Explanation: EDTA- it is an anticoagulant that is preferred when we want to look at the morphology or we want to do any of the blood counts. Sodium fluoride - it inhibits enolase enzyme thus inhibits glycolysis process , so it is used for glucose estimation . Citrate - 3.2%Trisodium citrate - it is used for the determining the value of ESR and is also used for doing any kind of coagulation study. All the three (sodium EDTA, sodium fluoride, trisodium citrate) above use sodium. therefore, if we use any of these chemicals for the purpose of Serum Electrolyte measurement, the value of sodium and other electrolytes are going to be altered. So WHO recommends that for the purpose of Serum Electrolyte measurement we use an anticoagulant Lithium Heparin. It doesn't cause any alterations in the serum electrolytes." "This is a real-world medical entrance exam question, please give the true answer based on the question and selection. Topic name: None, Subject name: Dental","Known Hypertensive patient with BP of 170/120, what will be done for oral prophylaxis Select and explain. A. Sent him back to physician, B. Wait, monitor & do the treatment, C. Give him intravenous drug and do the oral prophylaxis, D. Not recalled",A: Sent him back to physician,"Answer: A: Sent him back to physician Explanation: None" "This is a real-world medical entrance exam question, please give the true answer based on the question and selection. Topic name: None, Subject name: Anatomy","The two roots of auriculotemporal nerve encircles which structure Select and explain. A. Superficial temporal vein, B. Superficial temporal artery, C. Anterior tympanic artery, D. Middle meningeal artery",D: Middle meningeal artery,"Answer: D: Middle meningeal artery Explanation: None" "This is a real-world medical entrance exam question, please give the true answer based on the question and selection. Topic name: None, Subject name: Microbiology","The most effective way of preventing tetanus is : Select and explain. A. Surgical debridement and toilet, B. Hyperbaric oxygen, C. Antibiotics, D. Tetanus toxoid",D: Tetanus toxoid,"Answer: D: Tetanus toxoid Explanation: Remember : Best way of prevention = Active immunization = TT Best passive immunization = Antitoxin = Human tetanus immunoglobulin." "This is a real-world medical entrance exam question, please give the true answer based on the question and selection. Topic name: None, Subject name: Pathology","Erosive lichen planus resembles which of the following Select and explain. A. Monilial gingivitis, B. Desquamative gingivitis, C. Herpetic gingivitis, D. Acute ulcerative gingivitis",B: Desquamative gingivitis,"Answer: B: Desquamative gingivitis Explanation: Desquamative gingivitis represents oral manifestations of various diseases like Lichen pLanus (Erosive or atrophic form), Cicatrialpemphigold or Benign mucous membrane pemphigoid pemphigus." "This is a real-world medical entrance exam question, please give the true answer based on the question and selection. Topic name: None, Subject name: Pharmacology","A patient taking which of the following drug should not eat cheese, etc: Select and explain. A. Tetracycline, B. Amoxycillin, C. Doxycycline, D. Triamterene",A: Tetracycline,"Answer: A: Tetracycline Explanation: None" "This is a real-world medical entrance exam question, please give the true answer based on the question and selection. Topic name: None, Subject name: Dental","Need for tooth separation is Select and explain. A. Diagnosis, B. Cavity preparation, C. Matrix placement, D. All of the above",D: All of the above,"Answer: D: All of the above Explanation: None" "This is a real-world medical entrance exam question, please give the true answer based on the question and selection. Topic name: None, Subject name: Dental","Stress breakers are not used in partial denture construction because: Select and explain. A. They are expensive, B. Increase the vertical dimension, C. They are difficult to design and fabricate, D. Harmful effect on the abutment",C: They are difficult to design and fabricate,"Answer: C: They are difficult to design and fabricate Explanation: None" "This is a real-world medical entrance exam question, please give the true answer based on the question and selection. Topic name: None, Subject name: Physiology","What is the sequence of clotting factors in coagulation pathway? Select and explain. A. XII, XIII, X, XI, B. XIII, XII, XI, X, C. XII, XI, IX, X, D. X, XIII, XII, XI","C: XII, XI, IX, X","Answer: C: XII, XI, IX, X Explanation: Ans. C. XII, XI, IX, X" "This is a real-world medical entrance exam question, please give the true answer based on the question and selection. Topic name: None, Subject name: Pediatrics","A 6 years male child comes with complaints of bedwetting. The child is continent during the day and problem is only at night. Growth and development of the child were normal. Urine microscopy is normal and urine specific gravity was 1.020. How will you manage? Select and explain. A. Reassure the parents and follow up after 6 months, B. Refer to psychiatrist, C. Complete blood counts, D. Ultrasound-KUB",A: Reassure the parents and follow up after 6 months,"Answer: A: Reassure the parents and follow up after 6 months Explanation: Ans: A: Reassure the parents and follow up after 6 monthsExplanation:(Ref: Nelson 20Ie p2585; Ghai 8/e p504)Bed-wetting is normal till 5 years of age.In a child with only night-time bed wetting, when urinalysis (to rule out infections) and urine osmolality (to rule out diabetes) are normal, only regular follow up is required.Treatment:To reassure the child and parents that the condition is self-limited and to avoid punitive measures that can affect the child's psychologic development adversely.Fluid intake should be restricted to 2 oz after 6 or 7 pm.Parents should be ceain that the child voids at bedtime.Avoiding extraneous sugar and caffeine after 4 pm also is beneficial.If the child snores and the adenoids are enlarged, referral to an otolarvngologist should be considered, because adenoidectomv can cure the enuresis." "This is a real-world medical entrance exam question, please give the true answer based on the question and selection. Topic name: None, Subject name: Gynaecology & Obstetrics","Which is not a feature of HELLP syndrome: Select and explain. A. Thrombocytopenia, B. Eosinophilia, C. Raised liver enzyme, D. Hemolytic anemia",B: Eosinophilia,"Answer: B: Eosinophilia Explanation: None" "This is a real-world medical entrance exam question, please give the true answer based on the question and selection. Topic name: None, Subject name: Medicine","Which laboratory test is most useful for patient under dicumarol therapy? Select and explain. A. PT, B. Platelet count, C. BT, D. CT",A: PT,"Answer: A: PT Explanation: None" "This is a real-world medical entrance exam question, please give the true answer based on the question and selection. Topic name: None, Subject name: Pathology","Infectious mononucleosis is caused by? Select and explain. A. EBV, B. CMV, C. HIV, D. HSV",A: EBV,"Answer: A: EBV Explanation: Epstein-Barr virus (EBV) infections (or HHV-4) is causative for following infections: Infectious mononucleosis Nasopharyngeal carcinoma Burkiu's lymphoma Oral hairy leukoplakia Post-transplant lymphoproliferative diseases" "This is a real-world medical entrance exam question, please give the true answer based on the question and selection. Topic name: None, Subject name: Pharmacology","Drugs causing hyperglycemia: Select and explain. A. β-blocker, B. Glucocorticoids, C. Acetylcholine, D. α-blockers",B: Glucocorticoids,"Answer: B: Glucocorticoids Explanation: None" "This is a real-world medical entrance exam question, please give the true answer based on the question and selection. Topic name: None, Subject name: Pediatrics","In CPR for Infants-1. The sequence followed is Compression - Breathing- Airway2. 30 chest compression is given3. 1 breathe every 15 compressions4. Chest Compression should be 1 1/2 inches in infants5. In infants, the brachial pulse should be assessed. Select and explain. A. True False False True False, B. True False False True True, C. False True False True False, D. False True False True True",D: False True False True True,"Answer: D: False True False True True Explanation: Ans. D. False True False True TrueEmergency care providers should check the victim's pulse for at least 5 seconds but no longer than 10 seconds. For children aged one to adolescence, the pulse should be checked at the carotid aery. In infants, the brachial pulse should be assessed.If there is a palpable pulse within 10 seconds, then a rescue breath should be given every 3 seconds. Breaths should last one second and the chest should be observed for visible rise. If the victim has an advanced airway, then the provider should administer breaths 10-12 times per minute.If the pulse is less than 60/minute, or if the victim has signs of poor perfusion after adequate ventilation and oxygenation, the provider should begin chest compressions. In the absence of a pulse, a lone rescuer should begin CPR with 30 high quality compressions followed by two breaths. If two healthcare providers are available, the cycle of compressions to breaths should be 15:2 (pediatrics).High quality compressions in CPR should be a minimum of 1/3 the AP diameter of the chest, or approximately 1 1/2 inches in infants (4 cm) and 2"" in children from age one to adolescence. The rate of compressions should be 100-120 per minute. Chest recoil should be complete between compressions.Pediatric compression is performed with the head of one hand over the lower 1/2 of the sternum, between the nipples. In infants, use two fingers, or use the thumb encircling technique if multiple providers are available" "This is a real-world medical entrance exam question, please give the true answer based on the question and selection. Topic name: None, Subject name: Radiology","Density of radiograph is affected by all, except: Select and explain. A. mA, B. kVp, C. Cone angulation, D. Thickness of object",C: Cone angulation,"Answer: C: Cone angulation Explanation: Images should have optimal density and contrast to facilitate interpretation. Although milliamperage (mA), peak kilovoltage (kVp), and exposure time are crucial parameters influencing density and contrast, faulty processing can adversely affect the quality of a properly exposed radiograph. White and Pharoah, 7th ed, pg-91" "This is a real-world medical entrance exam question, please give the true answer based on the question and selection. Topic name: None, Subject name: Dental","Child is Evasive and dawdling, what could be the child's mother behavior: Select and explain. A. Overprotective mother., B. Authoritarian mother., C. Rejecting mother., D. Underaffectionate mother.",B: Authoritarian mother.,"Answer: B: Authoritarian mother. Explanation: None" "This is a real-world medical entrance exam question, please give the true answer based on the question and selection. Topic name: None, Subject name: Medicine","Maximum spherocytes is seen in Select and explain. A. Autoimmune hemolytic anemia, B. Vit B12 deficiency, C. Aplastic anemia, D. None",A: Autoimmune hemolytic anemia,"Answer: A: Autoimmune hemolytic anemia Explanation: None" "This is a real-world medical entrance exam question, please give the true answer based on the question and selection. Topic name: None, Subject name: Anatomy","Medial squint is caused by paralysis of Select and explain. A. Occulomotor nerve, B. Abducens nerve, C. Optic nerve, D. Trochlear nerve",B: Abducens nerve,"Answer: B: Abducens nerve Explanation: None" "This is a real-world medical entrance exam question, please give the true answer based on the question and selection. Topic name: None, Subject name: Pathology","Difference in transudate & exudate is that the former has a: Select and explain. A. Low protein, B. Cloudy appearance, C. Increased specific gravity, D. High protein",A: Low protein,"Answer: A: Low protein Explanation: None" "This is a real-world medical entrance exam question, please give the true answer based on the question and selection. Topic name: AIIMS 2018, Subject name: Ophthalmology","The spectacles are used in: Select and explain. A. Progressive glasses for presbyopia, B. Bifocal glasses for presbyopia, C. Bifocals for pediatric pseudophakia, D. Bifocals for adult aphakia",C: Bifocals for pediatric pseudophakia,"Answer: C: Bifocals for pediatric pseudophakia Explanation: The image shows bifocal glasses of executive type which is especially used in case of paediatric pseudophakia and not in adult aphakia or presbyopia. Other types of bifocals are K or D bifocals which are used for adults with presbyopia Progressive glasses avoid the sudden jump in power in the lenses and provide a smooth transition from the power for distance vision to the power for the near vision thereby providing good distance, intermediate and near vision" "This is a real-world medical entrance exam question, please give the true answer based on the question and selection. Topic name: None, Subject name: Medicine","Which of the following complications of stroke need not to be treated? Select and explain. A. Fever, B. Spasticity, C. Dysphagia, D. Numbness",B: Spasticity,"Answer: B: Spasticity Explanation: Answer- B. SpasticitySpasticity is seen in 20% of hemiplegic paitents. Spasticity per se does not produce weakness and other aspects of motor control Overzealous treatment affects the normal ambulation of patients. Hence, treating spasticity, as a rule is not always preferred, treating it is a double-edged sword. Spasticity as a consequence of a stroke may have a ceain beneficiary compensatory aspect." "This is a real-world medical entrance exam question, please give the true answer based on the question and selection. Topic name: None, Subject name: Pathology","In a 32 year female patient, left supraclavicular lymph node biopsy histologically shows acidophilic owl eye nucleoli lying freely in empty spaces showing immunohistocompatibility with CD 15 & CD 30, it is suggestive of Select and explain. A. Nodular sclerosis Hodgkin's lymphoma, B. Lymphocytic predominant Hodgkin's Lymphoma, C. Reactive nodular hyperplasia, D. Large cell lymphoma",A: Nodular sclerosis Hodgkin's lymphoma,"Answer: A: Nodular sclerosis Hodgkin's lymphoma Explanation: None" "This is a real-world medical entrance exam question, please give the true answer based on the question and selection. Topic name: None, Subject name: Forensic Medicine","St. Anthony's fire refers to poisoning by: Select and explain. A. Ergot alkaloids, B. Spanish fly, C. Crotalaria juncea, D. Aflatoxin",A: Ergot alkaloids,"Answer: A: Ergot alkaloids Explanation: Ans: A. Ergot alkaloids(Ref Reddy 34/e p517. 33/e p556: Parikh 6/e p9.32)St. Anthony's Fire:Causes:Due to overdose of ,Ergot medications.After eating flour milled from ergot-infected rye.Humans & livestock a Develop ergotism.Ergotism:Long term effect of ergot poisoning.Due to alkaloids ingestion produced by Claviceps purpurea fungus infected rye & other cereals.Also due to a action of number of ergoline-based drugs.Also known as ""ergotoxicosis, ergot poisoning & Saint Anthony's Fire"".In middle age -Gangrenous poisoning was known as ""holy fire"" or ""Saint Anthony's fire"".Named after monks of Order of Saint Anthony who succeeded in treatment." "This is a real-world medical entrance exam question, please give the true answer based on the question and selection. Topic name: None, Subject name: Pathology","Odontogenic keratocyst has the following feature Select and explain. A. Occurs due to infection periapically, B. Is developmental in origin, C. Can be treated by aspiration, D. Has low recurrence rate",B: Is developmental in origin,"Answer: B: Is developmental in origin Explanation: None" "This is a real-world medical entrance exam question, please give the true answer based on the question and selection. Topic name: None, Subject name: Pathology","Streptococci have the ability to adhere to the tooth surfaces and contribute to caries development as they? Select and explain. A. Are penicillinase resistant, B. Produce acid during metabolism, C. Produce extracellular polysachharides, D. Ferment carbohydrates",C: Produce extracellular polysachharides,"Answer: C: Produce extracellular polysachharides Explanation: Dental plaque is the prerequisite for the development of smooth surface caries. The presence of extracellular polysaccharides (like glucans and levans) have been clearly demonstrated in dental plaque. Glucans, especially their water-insoluble fraction, can serve as a structural component of plaque matrix and help in adhering certain bacteria to the teeth. Even though the ability to produce acid may be a prerequisite for caries induction, but not all acid producing organism are cariogenic." "This is a real-world medical entrance exam question, please give the true answer based on the question and selection. Topic name: None, Subject name: Dental","In gypsum products thinner mix leads to Select and explain. A. Increased setting expansion, B. Decreased setting expansion, C. Increase compressive strength, D. Decrease consistency",B: Decreased setting expansion,"Answer: B: Decreased setting expansion Explanation: None" "This is a real-world medical entrance exam question, please give the true answer based on the question and selection. Topic name: None, Subject name: Dental","TNM Staging is what type of scale Select and explain. A. Ordinal, B. Ratio, C. Nominal, D. Interval",A: Ordinal,"Answer: A: Ordinal Explanation: None" "This is a real-world medical entrance exam question, please give the true answer based on the question and selection. Topic name: AIIMS 2017, Subject name: Biochemistry","Child presents with hypotonia and seizures. It was confirmed to be zellweger syndrome. Which of the following accumulates in brain? Select and explain. A. Glucose, B. Lactic acid, C. Long chain fatty acid, D. Triglycerides",C: Long chain fatty acid,"Answer: C: Long chain fatty acid Explanation: Zellweger syndrome It is also called as Cerebro-Hepato-Renal Syndrome. It is a rare, autosomal recessive disorder. Absence of Peroxisomes in almost all tissues, peroxisomes are responsible for oxidation of very long chain fatty acids which contain more than 22 carbons. Clinical features: Severe neurological symptoms such as Impaired neuronal migration, hypomyelination, hepatomegaly, renal cysts. Accumulation of very long chain fatty acids in brain, with carbon number > 22 Most patients die within 1st year of their life." "This is a real-world medical entrance exam question, please give the true answer based on the question and selection. Topic name: None, Subject name: Dental","Type III gold alloy does not have following features except: Select and explain. A. Harder than type IV, B. Used for small inlays, C. Easily burnishable than type IV, D. Less gold than type IV",C: Easily burnishable than type IV,"Answer: C: Easily burnishable than type IV Explanation: None" "This is a real-world medical entrance exam question, please give the true answer based on the question and selection. Topic name: None, Subject name: Pathology","Which of the following is inherited as an autosomal dominant trait? Select and explain. A. Lichen planus, B. Bullous pemphigoid, C. Pemphigus yulgaris, D. White sponge nevus",D: White sponge nevus,"Answer: D: White sponge nevus Explanation: Lichen planus, pemphigoid, and pemphigus are autoimmune diseases whereas white sponge nevus is inherited as autosomal dominant trait." "This is a real-world medical entrance exam question, please give the true answer based on the question and selection. Topic name: None, Subject name: Microbiology","Anaphylaxis is mediated by Select and explain. A. 5-hydroxytryptamine, B. Heparin, C. ProstaglandinImmunology, D. All of these",D: All of these,"Answer: D: All of these Explanation: Answer- D. All of these* Primary mediators (These are present in mast cell granules) Includes:* Biogenic Amines :- Histamine, 5-Hydroxytryptamine* Proteoglycans : Heparin* Secondary mediators:* Leukotrienes :- LTC4 D4,V* Cytokines:- ILl,lLg,lL4,IL-5, IL4 TNF and GMCSF" "This is a real-world medical entrance exam question, please give the true answer based on the question and selection. Topic name: None, Subject name: Anatomy","Mental foramen is located: Select and explain. A. Between roots of premolars, B. Between roots of molars, C. Near canine, D. Between Incisors",A: Between roots of premolars,"Answer: A: Between roots of premolars Explanation: None" "This is a real-world medical entrance exam question, please give the true answer based on the question and selection. Topic name: None, Subject name: Pathology","Most common cyst associated with adjoining vital teeth? Select and explain. A. Dentigerous cyst, B. Globulomaxillary cyst, C. Periapical cyst, D. Lateral periodontal cyst",A: Dentigerous cyst,"Answer: A: Dentigerous cyst Explanation: None" "This is a real-world medical entrance exam question, please give the true answer based on the question and selection. Topic name: None, Subject name: Surgery","Sorting a patient of trauma of head injury on basis of need of care and availability of resources Select and explain. A. Triage, B. Emergency care, C. Definitive care, D. Hospital care",A: Triage,"Answer: A: Triage Explanation: None" "This is a real-world medical entrance exam question, please give the true answer based on the question and selection. Topic name: None, Subject name: Dental","In a skeletal Class III, the value of ANB will be: Select and explain. A. -10, B. 2, C. 4, D. 6",A: -10,"Answer: A: -10 Explanation: None" "This is a real-world medical entrance exam question, please give the true answer based on the question and selection. Topic name: AIIMS 2018, Subject name: Anatomy","Patient presented with imbalance on walking with eyes closed and has to lift up foot and stomp on ground to walk. The tract involved is:- Select and explain. A. Spinocerebellar tract, B. Dorsal column tract, C. Lateral spinothalamic tract, D. Vestibulospinal tract",B: Dorsal column tract,"Answer: B: Dorsal column tract Explanation: Case described is of SENSORY ATAXIA - d/o damage to dorsal column of spinal cord. Dorsal column tract: - N. gracilis & N. cuneatus - Carry pressure, vibration, stereognosis & conscious proprioception tactile discrimination Damage leads to: - - Sensory ataxia - Stomping gait - d/o loss of conscious proprioception - Romberg test positive (Pt loses balance with closed eyes). Spinocerebellar tract: - - Carry unconscious proprioception - Damage leads to cerebellar ataxia - wide based gait. Spinothalamic tract: - - Carries pain, temperature (lateral spinothalamic) - Pressure & touch (ventral spinothalamic) Note: Other causes of sensory ataxia 1. Neurosyphilis (tabes dorsalis) 2. DM - II (affects dorsal column) 3. Vit B12 deficiency" "This is a real-world medical entrance exam question, please give the true answer based on the question and selection. Topic name: AIIMS 2018, Subject name: Pathology","Which of the following is not a provisional entity as per WHO 2016 classification of Acute leukemia? Select and explain. A. AML with hyperploidy, B. B - ALL with BCR - ABL like mutatio, C. AML with BCR - ABL, D. Early T - cell precursor leukemia / Lymphoma",A: AML with hyperploidy,"Answer: A: AML with hyperploidy Explanation: In Myeloid/lymphoid neoplasms with eosinophilia nad rearrangement of PDGFRA, PDGFRB, OR FGFR1, or with PCM1-JAK2 Provisional entity: Myeloid/ lymphoid neoplasms with PCM1-JAK2 In MDS Provisional entity: Refractory cytopenia of childhood In AML and related neoplasms Provisional entity: AML with BCR-ABL1 Provisional entity: AML with mutated RUNX1 In B cell lymphoblastic leukemia Provision entity: B-lymphoblastic leukemia/ lymphoma, BCR-ABL1-like Provision entity: B-lymphoblastic leukemia/ lymphoma with iAMP21 In T cell lymphoblastic leukemia Provisional entity: Early T-cell precursor lymphoblatic leukemia Provisional entity: Natural killer (NK) cell lymphoblastic leukemia/ lymphoma" "This is a real-world medical entrance exam question, please give the true answer based on the question and selection. Topic name: None, Subject name: Dental","Difference between physical characteristic of reamers and files is: Select and explain. A. The cross-section of reamers in square and files are triangular in cross-section, B. The number of flutes on the blade are more in files than in reamers, C. The reamers have more flutes in the blade, D. Files have two superficial grooves to produce flutes in a double helix design",B: The number of flutes on the blade are more in files than in reamers,"Answer: B: The number of flutes on the blade are more in files than in reamers Explanation: None" "This is a real-world medical entrance exam question, please give the true answer based on the question and selection. Topic name: None, Subject name: Dental","Anterior - posterior spread of implant is: Select and explain. A. Distance from center of most anterior implant to the center of most posterior implant, B. Distance from center of most anterior implant to the distal surface of most posterior implant, C. Distance from distal surface of most anterior implant to the mesial surface of most posterior implant, D. Distance from mesial surface of most anterior implant to the distal surface of most posterior implant",B: Distance from center of most anterior implant to the distal surface of most posterior implant,"Answer: B: Distance from center of most anterior implant to the distal surface of most posterior implant Explanation: None" "This is a real-world medical entrance exam question, please give the true answer based on the question and selection. Topic name: None, Subject name: Dental","For determining periodontal disease an epidemiological survey is done by using: Select and explain. A. Gingival index (G.I.), B. Plaque index (P.I.), C. Periodontal index (P.I), D. None of the above",C: Periodontal index (P.I),"Answer: C: Periodontal index (P.I) Explanation: None" "This is a real-world medical entrance exam question, please give the true answer based on the question and selection. Topic name: None, Subject name: Dental","Arch length analysis of a dentition shows a discrepancy of more than 10 mm. This indicates: Select and explain. A. No extraction required if treated at an early age, B. No extraction required, C. Extraction of posterior teeth, D. Proximal stripping",C: Extraction of posterior teeth,"Answer: C: Extraction of posterior teeth Explanation: None" "This is a real-world medical entrance exam question, please give the true answer based on the question and selection. Topic name: None, Subject name: Social & Preventive Medicine","True about Total Goitre Rate (TGR) is: Select and explain. A. It is an indicator of iron deficiency in the community, B. Community survey of TGR does not require doctors in the team, C. Goitres are classified as not visible, palpable and visible, D. Goitres are classified as not visible, palpable and visible","D: Goitres are classified as not visible, palpable and visible","Answer: D: Goitres are classified as not visible, palpable and visible Explanation: Ans: D. Goitres are classified as not visible, palpable and visible(Ref: Ghai 8Ie p519)The term 'endemic goitre' refers to a total goitre rate of >5 percent in a given community.Total goitre rate is an indicator of iodine deficiency, which causes brain damage and mental retardation.Endemic goiter is present when the prevalence of goiter in a defined population exceeds 5%.Endemic goiter is graded by the method of WHO.Screening estimates of iodine intake are usually derived from 24-hr urinary excretion values or urinary iodine concentration expressed in relation to creatinine concentration." "This is a real-world medical entrance exam question, please give the true answer based on the question and selection. Topic name: None, Subject name: Dental","For which of the following, you will not perform pulpectomy? Select and explain. A. Primary teeth with sinus tract, B. Primary teeth without successor, C. Primary teeth with furcation pathology, D. Pulpless primary teeth next to the line of a palatal cleft",C: Primary teeth with furcation pathology,"Answer: C: Primary teeth with furcation pathology Explanation: Primary teeth with furcation pathology, pulpectomy is not indicated, rather it requires extraction." "This is a real-world medical entrance exam question, please give the true answer based on the question and selection. Topic name: None, Subject name: Surgery","Inorganic component of bone is: Select and explain. A. 35%, B. 45%, C. 85%, D. 65%",D: 65%,"Answer: D: 65% Explanation: None" "This is a real-world medical entrance exam question, please give the true answer based on the question and selection. Topic name: None, Subject name: Medicine","An elderly female presented with blockade, nasal discharge, diplopia and facial swelling. On examination, there is blackish discharge from the nasal cavity with necrosis of nasal mucosa, facial skin and palate. There is fixation of the right globe. There is elevated blood sugar and urinary ketones are positive.Which of the following would be the medication to be used in this patient? Select and explain. A. Amphotericin B, B. Itraconazole, C. Ketoconazole, D. Broad spectrum antibiotics",A: Amphotericin B,"Answer: A: Amphotericin B Explanation: Ans. a. Amphotericin B (Rcf : Harrison l9/e p1350,18/ep l661-l664)This diabetic patient is most likely suffering from mucormycosis, in which there is involvement of peri-orbital region and visual impairment. I.V. Amphotericin BQ is the treatment most commonly used in all forms of Mucormycosis." "This is a real-world medical entrance exam question, please give the true answer based on the question and selection. Topic name: None, Subject name: Pediatrics","Video based question - AIIMS NICU - neonate shown on O2 with nasal prongs, not intubated, sister shown inseing a tube through the mouth, camera zooms in, 18 (calibration) mark seen, the other (green) end as of now not connected to anything, video ends, what is the procedure that is being done?VIDEO LINK: Select and explain. A. Oral suction, B. Oropharyngeal suction, C. Nasogastric tube inse, D. Orogastric tube inseion",D: Orogastric tube inseion,"Answer: D: Orogastric tube inseion Explanation: Ans. D. Orogastric tube inseion" "This is a real-world medical entrance exam question, please give the true answer based on the question and selection. Topic name: None, Subject name: Pathology","Which of following is an oral precancer? Select and explain. A. Oral hairy Leukoplakia, B. White spongy naevus, C. Leukemia, D. Speckled Leukoplakia",D: Speckled Leukoplakia,"Answer: D: Speckled Leukoplakia Explanation: None" "This is a real-world medical entrance exam question, please give the true answer based on the question and selection. Topic name: None, Subject name: Radiology","Patient with carcinoma endometrium treated with pelvic external beam irradiation to whole pelvis. Which of the following organs is most radiosensitive in the pelvic region? Select and explain. A. Ovary, B. Vagina, C. Bladder, D. Rectum",A: Ovary,"Answer: A: Ovary Explanation: Ans: A. Ovary(Ref Perez and Brady's Principles of Radiation Oncology 6/e p65)Among the given options, most radiosensitive organ is ovary >rectum >bladder >vagina.(Radiation tolerance dose: Ovary = 2-3 Gy; Rectum = 60 Gy; Bladder = 65 Gy; Vagina = 90 Gy)." "This is a real-world medical entrance exam question, please give the true answer based on the question and selection. Topic name: None, Subject name: Dental","Repeated fracture of a porcelain fused to metal restoration is primarily due to Select and explain. A. Occlusal trauma, B. Inadequately designed framework, C. Improper firing schedule, D. Failure to use metal conditioner",B: Inadequately designed framework,"Answer: B: Inadequately designed framework Explanation: None" "This is a real-world medical entrance exam question, please give the true answer based on the question and selection. Topic name: None, Subject name: Pathology","White patch is seen on the buccal mucosa consisting of pseudomycelium and chalmydospores with desquamated epithelium adjacent to it, the patient is suffering from Select and explain. A. histoplasmosis, B. cryptococcosis, C. candidiasis, D. coccidionnycosis",C: candidiasis,"Answer: C: candidiasis Explanation: None" "This is a real-world medical entrance exam question, please give the true answer based on the question and selection. Topic name: None, Subject name: Dental","The micro organism previously present in the Periapical tissues following obturation Select and explain. A. Persist and stimulate the formation of a granuloma, B. Are eliminated by the natural defenses, C. Re-enter and re-infect the sterile canal and are removed by surgery, D. Are eliminated by the medicaments used in endodontic treatment",B: Are eliminated by the natural defenses,"Answer: B: Are eliminated by the natural defenses Explanation: None" "This is a real-world medical entrance exam question, please give the true answer based on the question and selection. Topic name: None, Subject name: Anatomy","Cranioveebral joint does not include: Select and explain. A. Occipital condyle, B. Axis, C. Atlas, D. Wings of sphenoid",D: Wings of sphenoid,"Answer: D: Wings of sphenoid Explanation: Ans: D. Wings of sphenoid(Ref Gray's 40/e p733)Cranioveebral Joint:Consists of occipital condyles, atlas & axis.Aiculation between cranium & veebral column specialized - To provide a wider range of movement.Functions as a universal joint.Permits horizontal & veical scanning movements of head.Adapted for eye-head co-ordination." "This is a real-world medical entrance exam question, please give the true answer based on the question and selection. Topic name: None, Subject name: Dental","What type of suturing technique is used here Select and explain. A. Figure of 8, B. Direct loop, C. Horizontal mattress, D. Vertical mattress",A: Figure of 8,"Answer: A: Figure of 8 Explanation: None" "This is a real-world medical entrance exam question, please give the true answer based on the question and selection. Topic name: None, Subject name: Dental","First step in major connector construction is Select and explain. A. Design of the stress bearing areas, B. Design of the non stress bearing areas, C. Marking the outline of the strapline, D. Selection of the strap type",A: Design of the stress bearing areas,"Answer: A: Design of the stress bearing areas Explanation: None" "This is a real-world medical entrance exam question, please give the true answer based on the question and selection. Topic name: None, Subject name: Dental","SI unit of X-ray Select and explain. A. Rad, B. Sv, C. Gray, D. Kerma",C: Gray,"Answer: C: Gray Explanation: None" "This is a real-world medical entrance exam question, please give the true answer based on the question and selection. Topic name: None, Subject name: Dental","Father of statistics Select and explain. A. John Snow, B. John Graunt, C. MacMahon, D. John M. Last",B: John Graunt,"Answer: B: John Graunt Explanation: None" "This is a real-world medical entrance exam question, please give the true answer based on the question and selection. Topic name: None, Subject name: Social & Preventive Medicine","For testing the statistical significance of the difference in heights of school children - Select and explain. A. Student's t test, B. chi-square test, C. Paired 't' test, D. ANOVA",D: ANOVA,"Answer: D: ANOVA Explanation: Student’s t-test: – Paired Student’s t-test: Comparing means (± SD) in paired data (in same group of individuals before and after an intervention) Comparison of mean levels can be done by Paired Student’s t - test – Unpaired Student’s t-test: Comparing means (± SD) in two different group of individuals ANOVA test (F-test/F-ratio): Comparing means (+ SD) in more than two different group of individuals Chi-square test (c2-test): Comparing percentage, proportions & fractions in two or more different group of individuals In the given question, we have to test the statistical significance of the difference in heights of school children; thus mean heights of schools children in different classes/standards/schools will be done Thus, ANOVA test (F - test/ F -ratio) is most suitable." "This is a real-world medical entrance exam question, please give the true answer based on the question and selection. Topic name: None, Subject name: Dental","Odontoblasts are derived from ____________ Select and explain. A. Undifferentiated mesenchymal cells, B. Histocytes, C. Macrophages, D. Lymphocytes",A: Undifferentiated mesenchymal cells,"Answer: A: Undifferentiated mesenchymal cells Explanation: None" "This is a real-world medical entrance exam question, please give the true answer based on the question and selection. Topic name: None, Subject name: Anatomy","The deepest layer of cervical fascia is: Select and explain. A. Vertebral, B. Carotid sheath, C. Deep fascia, D. Prevertebral fascia",D: Prevertebral fascia,"Answer: D: Prevertebral fascia Explanation: The arrangement of layers from outside to inside is: a. Pretracheal fascia b. Investing fascia c. Prevertebral fascia The prevertebral fascia extends medially behind the carotid vessels, where it assists in forming their sheath and passes in front of prevertebral muscles." "This is a real-world medical entrance exam question, please give the true answer based on the question and selection. Topic name: None, Subject name: Pediatrics","A 10–year old male child was presented to the pediatrician for evaluation of a seizure disorder. On examination a vascular plaque was found along the ophthalmic and maxillary divisions of the trigeminal nerve. The mother informed the pediatrician that the lesion was present since birth and there was no change in morphology. The most likely possibility is – Select and explain. A. Sturge Weber syndrome, B. Infantile hemangioma, C. Congenital hemangioma, D. Proteus syndrome",A: Sturge Weber syndrome,"Answer: A: Sturge Weber syndrome Explanation: This child has : Seizure disorder Vascular plaque along the ophthalmic and maxillary divisions of trigeminal nerve Lesion presenting since birth and not chonging in morphology The diagnosis is sturge weber syndrome. Sturge weber syndrome is a rare congenital vascular disorder characterized by facial capillary malformation (port wine stain) and associated capillary venous malformation affecting the brain and eye. It consists of angiomas involving the leptomeninges and skin of face typically in the ophthalmic and maxillary distributions of trigeminal nerve." "This is a real-world medical entrance exam question, please give the true answer based on the question and selection. Topic name: None, Subject name: Microbiology","Dry heat destroys microorganisms by Select and explain. A. Lysis, B. Oxidation, C. Hydrolysis of DNA, D. Coagulation of proteins",B: Oxidation,"Answer: B: Oxidation Explanation: None" "This is a real-world medical entrance exam question, please give the true answer based on the question and selection. Topic name: None, Subject name: Biochemistry","Hyponatremia is defined as a serum Na+ level Select and explain. A. < 135 mmol/L, B. > 135 mmol/L, C. < 160 mmo/L, D. < 100 mmo/L",A: < 135 mmol/L,"Answer: A: < 135 mmol/L Explanation: None" "This is a real-world medical entrance exam question, please give the true answer based on the question and selection. Topic name: None, Subject name: Anatomy","The cribriform plate connects Select and explain. A. Nasal cavity and anterior cranial fossa, B. Nasal cavity and orbit, C. Nasal cavity and oral cavity, D. Anterior cranial fossa and orbit",A: Nasal cavity and anterior cranial fossa,"Answer: A: Nasal cavity and anterior cranial fossa Explanation: Cribriform plate of ethmoid bone separates the anterior cranial fossa from nasal cavity. Anteriorly articulates with frontal bone, at frontoethmoidal suture, marked in the median plane as foramen caecum, which is usually blind. Anteriorly, cribriform plate has midline projection, called crista galli. On each side of crista galli, there are foramina through which anterior ethmoidal nerve and vessels pass to nasal cavity. The plate is also perforated by numerous foramina, for the passage of olfactory nerve rootlets." "This is a real-world medical entrance exam question, please give the true answer based on the question and selection. Topic name: None, Subject name: Pathology","Which of the following is not a tumor suppressor gene? Select and explain. A. pRb, B. p16, C. PTCH, D. c-erbB1",D: c-erbB1,"Answer: D: c-erbB1 Explanation: None" "This is a real-world medical entrance exam question, please give the true answer based on the question and selection. Topic name: AIIMS 2018, Subject name: Physiology","Which hormone has permissive role in pubey? Select and explain. A. Leptin, B. GnRH, C. Insulin, D. GH",B: GnRH,"Answer: B: GnRH Explanation: Pubey means the onset of adult sexual life. Pubey is caused by a gradual increase in GnRH secretion by the pituitary, beginning around 8th year of life. In male and females, the infantile pituitary gland & ovaries are capable of full function if they are appropriately stimulated. However, for reasons not understood, the hypothalamus does not secrete significant quantities of GnRH during childhood. At pubey, the hypothalamus stas secreting pulsatile GnRH, which initiates the other cascading events leading to maturity of the sexual organs and pubey." "This is a real-world medical entrance exam question, please give the true answer based on the question and selection. Topic name: None, Subject name: Dental","In 4-years-old patient S-ECC is present with recurrent gingival swelling and pain in 61 and 62; what will be your line of treatment: Select and explain. A. Endodontic treatment, B. Restoration, C. Extraction, D. Fluoride application",A: Endodontic treatment,"Answer: A: Endodontic treatment Explanation: In 4-years-old patient S-ECC is present with recurrent gingival swelling and pain in 61 and 62, line of treatment is endodontic treatment." "This is a real-world medical entrance exam question, please give the true answer based on the question and selection. Topic name: None, Subject name: Gynaecology & Obstetrics","A middle-aged woman came to OPD with a twin pregnancy. She already had 2 first trimester aboion and she has a 3 years old female child who was born at the end of ninth month of gestation. Which of the following is her accurate representation? C = gravid, P = para? Select and explain. A. G4P1 1+2+1, B. G4P1 0+1+2, C. G5P1 2+0+1, D. G5P0 1+0+2",B: G4P1 0+1+2,"Answer: B: G4P1 0+1+2 Explanation: Answer- B. G4P1 0+1+2The nomenclature for this question is bused on a system called GTPAL systemGravida and parity: Gravida denotes o pregnant state both present and past, irrespective of the period of gestation.Parity denotes a state of previous pregnancy beyond the period of bility" "This is a real-world medical entrance exam question, please give the true answer based on the question and selection. Topic name: None, Subject name: Surgery","The aspirate from a keratocyst will have: Select and explain. A. A low soluble protein content, B. A high soluble protein content, C. Cholesterol crystal, D. inflammatory cells",A: A low soluble protein content,"Answer: A: A low soluble protein content Explanation: None" "This is a real-world medical entrance exam question, please give the true answer based on the question and selection. Topic name: None, Subject name: Surgery","Initially compression osteosynthesis was used for: Select and explain. A. Malunion, B. Non union, C. Reduce length of bone, D. Increase length of bone",B: Non union,"Answer: B: Non union Explanation: None" "This is a real-world medical entrance exam question, please give the true answer based on the question and selection. Topic name: None, Subject name: Physiology","Property affected due to diameter of nerve fiber/axon? Select and explain. A. Conduction velocity, B. Refractory period, C. Latency period, D. Amplitude",A: Conduction velocity,"Answer: A: Conduction velocity Explanation: With increasing diameter of nerve fiber leads to increase in its conduction velocity." "This is a real-world medical entrance exam question, please give the true answer based on the question and selection. Topic name: None, Subject name: Medicine","Routine examination of a 17 years old asymptomatic boy reveals sho PR interval, and delta wave on ECG. The least effective measure for the boy would be: Select and explain. A. Hotter monitoring, B. Treadmill stress test, C. Beta blocker, D. Reassurance",B: Treadmill stress test,"Answer: B: Treadmill stress test Explanation: Ans.b. Tredmill stress testThis is a case of WPW syndrome. There is no use all treadmill test in these cases as results are often misleading.Holter monitoring would be indicated to detect the development of any arrythmias. Beta-blockers have also been used in asymptomatic cases only as a preventive therapy against the development of PSVT." "This is a real-world medical entrance exam question, please give the true answer based on the question and selection. Topic name: None, Subject name: Anaesthesia","A surgeon decides to operate a patient under epidural anesthesia. 3% Xylocaine with adrenaline is used for administering epidural anesthesia. The patient suddenly develops hypotension after 3 minutes of administration. What is the most likely cause for this? Select and explain. A. Systemic absorption of the drug, B. Vasovagal effect, C. Allergy to the drug preparation, D. Penetration into the subarachnoid space",D: Penetration into the subarachnoid space,"Answer: D: Penetration into the subarachnoid space Explanation: Ans: D. Penetration into the subarachnoid space(Ref: Miller 7/e p1895: Barash 5/e p1478, 2405)Penetration of drug into the subarachnoid space sholy causes cardiovascular symptoms like hypotension.Systemic absorption of drugs causes neurological symptoms before hypotension.Vasovagal shock occurs immediately, i.e. even before the needle for epidural anesthesia is injected." "This is a real-world medical entrance exam question, please give the true answer based on the question and selection. Topic name: AIIMS 2019, Subject name: Biochemistry","A person after consuming raw eggs presents with weakness, fatigue & hypoglycemia. Doctor gave him vitamin tablets. Which enzyme deficiency is causing hypoglycemia in this patient: Select and explain. A. Phosphoenol pyruvate carboxykinase, B. Pyruvate carboxylase, C. Glycogen phosphorylase, D. Glucose 6 phosphatase",B: Pyruvate carboxylase,"Answer: B: Pyruvate carboxylase Explanation: Raw eggs contain protein Avidin - tightly binds to Biotin (vit B7) and is excreted along with biotin. Biotin is required for all carboxylases. All carboxylases require ATP also. So, enzyme affected is carboxylase which requires Biotin as coenzyme Pyruvate Carboxylase is the first enzyme of gluconeogenesis. Regarding other options: Glycogen Phosphorylase ROLE of Glycogenolysis Requires vitamin B6 (Pyridoxal phosphate0 No relation with glycogen phosphorylase and egg avidin Glucose 6 phosphatase A common enzyme in gluconeogenesis and glycogenolysis Deficiency leads to hypoglycemia and that is called Von Gierke's disease (Type 1 glycogen storage disease). - Von Gierke's disease is the most common glycogen storage disease, especially in children." "This is a real-world medical entrance exam question, please give the true answer based on the question and selection. Topic name: None, Subject name: Anatomy","Mandibular fossa is a part of: Select and explain. A. Mandible bone, B. Maxilla bone, C. Sphenoid bone, D. Temporal bone",D: Temporal bone,"Answer: D: Temporal bone Explanation: Mandibular fossa is a part of temporal bone. It along with articular tubercle forms the upper articular surface of TMJ. Note:Lower articular surface is formed by head of mandible" "This is a real-world medical entrance exam question, please give the true answer based on the question and selection. Topic name: AIIMS 2019, Subject name: Surgery","Which of the following scoring system is used for wound infection? Select and explain. A. ASA score, B. SIRS score, C. Southampton score, D. Glasgow score",C: Southampton score,"Answer: C: Southampton score Explanation: Southampton Wound Grading System Grade/Appearance Subtype/Appearance 0: Normal healing I: Normal healing with mild bruising or erythema la: some bruising lb: Considerable bruising Ic: Mild erythema II: Erythema plus other signs of inflammation IIa: At one point IIb: Around sutures IIc: Along wound IId: Around wound III: Clear or hemoserous discharge IIIa: At one point only (<2 cm) IIIb: Along wound (>2 cm IIIc: Large volume IIId: Prolonged (>3 days) IV: Pus IVa: At one point only (<2 cm) IVb: Along wound (>2 cm)" "This is a real-world medical entrance exam question, please give the true answer based on the question and selection. Topic name: None, Subject name: Dental","Ellis and deway classification of traumatic fractures are how many Select and explain. A. 9, B. 5, C. 3, D. 4",A: 9,"Answer: A: 9 Explanation: None" "This is a real-world medical entrance exam question, please give the true answer based on the question and selection. Topic name: None, Subject name: Biochemistry","β-oxidation of odd-chain fatty acids produce: Select and explain. A. Succinyl CoA, B. Propionyl CoA, C. Acetyl CoA, D. Malonyl CoA",B: Propionyl CoA,"Answer: B: Propionyl CoA Explanation: None" "This is a real-world medical entrance exam question, please give the true answer based on the question and selection. Topic name: None, Subject name: Social & Preventive Medicine","All of the following can cause epidemic in post-disaster period except Select and explain. A. Leishmaniasis, B. Leptospirosis, C. Rickettsiosis, D. Acute respiratory infections",A: Leishmaniasis,"Answer: A: Leishmaniasis Explanation: Ans. a. LeishmaniasisPost-disaster PeriodAn outbreak of gastroenteritis is the most commonly repoed disease in the post-disaster period. Increased incidence of acute respiratory infections is also common in the displaced population.Vector-borne diseases will not appear immediately but may take several weeks to reach epidemic levels. Displacement of domestic and wild animals increases the risk of transmission of zoonoses. Some of these animals may be reservoirs of infections such as leptospirosis, rickettsiosis, etc.Rickettsial diseases are also seen post-disaster especially in refugee camps.Wild animals are reservoirs of infections, which can be fatal such as equine encephalitis, rabies, and infections still unknown to humans." "This is a real-world medical entrance exam question, please give the true answer based on the question and selection. Topic name: None, Subject name: Dental","Most common morphological variation is seen in: Select and explain. A. Maxillary lateral incisor, B. Mandibular lateral incisor, C. Maxillary canine, D. Mandibular canine",A: Maxillary lateral incisor,"Answer: A: Maxillary lateral incisor Explanation: None" "This is a real-world medical entrance exam question, please give the true answer based on the question and selection. Topic name: AIIMS 2019, Subject name: Surgery","A 10 years old boy came to emergency with complains of severe pain in right scrotum that is not relieved for the past 1 hour. On examination, testicular torsion of the right side is suspected. The surgical intervention should be: Select and explain. A. Immediate scrotal exploration of the affected side and operate opposite side if symptomatic, B. Delay exploration and operate opposite side if symptomatic, C. Immediate exploration of the affected side and operate opposite side even if asymptomatic, D. Delayed exploration on the affected side along with the opposite side even if asymptomatic",C: Immediate exploration of the affected side and operate opposite side even if asymptomatic,"Answer: C: Immediate exploration of the affected side and operate opposite side even if asymptomatic Explanation: Testicular torsion - It is a surgical emergency - Confirm the diagnosis by decreased vascularity in Doppler (IOC) - Emergency orchidopexy- Within 4 hours - In patients of ischemia - Orchidectomy - During orchidopexy- Contralateral hemiscrotum should be explored and fixed because bell clappers deformity is usually bilateral. - Bilateral orchidopexy is preferred for Testicular torsion" "This is a real-world medical entrance exam question, please give the true answer based on the question and selection. Topic name: None, Subject name: Dental","In which of the following condition, mandibular stabilization is not governed by the facial nerve contraction? Select and explain. A. Infantile swallow, B. Mature swallow, C. Complex tongue thrusting, D. Retained infantile swallow",B: Mature swallow,"Answer: B: Mature swallow Explanation: None" "This is a real-world medical entrance exam question, please give the true answer based on the question and selection. Topic name: None, Subject name: Biochemistry","Enzymes help by: Select and explain. A. Lowering the activation energy, B. Increasing the substrate concentration, C. Decreasing the surface tension, D. Increasing the activation energy",A: Lowering the activation energy,"Answer: A: Lowering the activation energy Explanation: None" "This is a real-world medical entrance exam question, please give the true answer based on the question and selection. Topic name: None, Subject name: Biochemistry","Best biochemical description of glucose is? Select and explain. A. Exists as ketose sugar, B. Present in L form in all biosystems, C. Its C-4 epimer is galactose, D. It's a disaccharide",C: Its C-4 epimer is galactose,"Answer: C: Its C-4 epimer is galactose Explanation: Glucose: Simple sugar (monosaccharides) Has six carbon atoms (hexose) Its C-4 epimer is galactose Has 2 isomers - D-isomer (D-glucose/dextrose, occurs widely in nature) and L-isomer (L-glucose - does not occur much in nature) It is stored as a polymer (in plants as starch and in animals as glycogen)" "This is a real-world medical entrance exam question, please give the true answer based on the question and selection. Topic name: None, Subject name: Anatomy","Preganglionic parasympathetic fibres travel to otic ganglion from: Select and explain. A. Facial nerve, B. Glossopharyngeal nerve, C. Trigeminal nerve, D. None of the above",B: Glossopharyngeal nerve,"Answer: B: Glossopharyngeal nerve Explanation: None" "This is a real-world medical entrance exam question, please give the true answer based on the question and selection. Topic name: None, Subject name: Gynaecology & Obstetrics","In a non-diabetic high risk pregnancy the ideal time for non stress test monitoring is: Select and explain. A. 48 hrs, B. 72 hrs, C. 96 hrs, D. 24 hrs",B: 72 hrs,"Answer: B: 72 hrs Explanation: Intervals between NST testing “The interval between tests is arbitrarily set at 7 days. According to ACOG, more frequent testing is advocated for women with posterm pregnancy, type I diabetes mellitus, IUGR or gestational hypertension. In these circumstances some investigators recommend twice weekly (i.e. after 72 hours) with additional testing performed for maternal or fetal deterioration regardless of the time elapsed since the last test. Others recommend NST daily. Generally daily NST is recommended with severe preeclampsia remote from term.”" "This is a real-world medical entrance exam question, please give the true answer based on the question and selection. Topic name: None, Subject name: Dental","Age of preoperational stage is Select and explain. A. 1-2 years, B. 2-7 years, C. 8-11 years, D. 0-2 years",B: 2-7 years,"Answer: B: 2-7 years Explanation: None" "This is a real-world medical entrance exam question, please give the true answer based on the question and selection. Topic name: AIIMS 2019, Subject name: Surgery","A Patient presented to emergence after A with multiple rib fractures. He is conscious speaking single words. On examination, respiratory rate was 40/minute and BP was 90/40 mmHg. What is immediate next step? Select and explain. A. Urgent IV fluid administration, B. Intubate the patient, C. Chest X-ray, D. Needle inseion in 2nd intercostal space",D: Needle inseion in 2nd intercostal space,"Answer: D: Needle inseion in 2nd intercostal space Explanation: A + Multiple Rib Hemothorax Pneumothorax Tension Pneumothorax Flail chest with pulmonary contusion Conscious + Speaking Single words Dyspnea Tachypnea - RR - 40 min - Tachypnea - BP - 90/40 mmHg - Low BP TENSION Pneumothorax Multiple Rib Dyspnea Tachypnea Low BP Tension Pneumothorax: d/t ""One-way valve"" air leak Complete collapse of punctured lung Mediastinum - opposite side displacement | venous return M/C Penetrating chest trauma. Blunt chest trauma with air leak Latrogenic Clinical Picture: Restlessness Tachypnea Dyspnea Distended neck veins. Management: Clinical dx with dire emergency T/t should be never delayed for radiological confirmation. Hence, CXR is not performed. Treatment: Immediate decompression + wide bore needle into 2nd ICS - mid-clavicular line T/f of choice: ICT inseion - 5th ICS (Triangle of Safety) in anterior axillary lines Triangle of Safety:Boundaries:Anterior- Lat. Border of Pec. MajorPosterior- Lat. border of Latissimus dorsiSuperior/Apex- Base of AxillaInferior/Base- Line passing through 5th ICS" "This is a real-world medical entrance exam question, please give the true answer based on the question and selection. Topic name: None, Subject name: Dental","In class 2 inlay preparation, How should the pulpal floor be placed in comparison with pulpal floor in amalgam class 2 preparation? Select and explain. A. Deeper, B. Pulpal floor for amalgam should be placed deeper in dentin, C. Same as Amalgam, D. None of the above",B: Pulpal floor for amalgam should be placed deeper in dentin,"Answer: B: Pulpal floor for amalgam should be placed deeper in dentin Explanation: None" "This is a real-world medical entrance exam question, please give the true answer based on the question and selection. Topic name: None, Subject name: Dental","The fourth root canal if present in a maxillary 1st molar is usually present in: Select and explain. A. Mesiolingual root, B. Mesiobuccal, C. Palatal root, D. Distal root",B: Mesiobuccal,"Answer: B: Mesiobuccal Explanation: None" "This is a real-world medical entrance exam question, please give the true answer based on the question and selection. Topic name: None, Subject name: Dental","Which of the following component acts as an accelerator in ZOE impression paste? Select and explain. A. Zinc Sulphate & Zinc chloride, B. Zinc chloride & Eugenol, C. Zinc Sulphate & Eugenol, D. Glycerin",A: Zinc Sulphate & Zinc chloride,"Answer: A: Zinc Sulphate & Zinc chloride Explanation: None" "This is a real-world medical entrance exam question, please give the true answer based on the question and selection. Topic name: None, Subject name: Pharmacology","Which drug causes flagellate pigmentation of skin? Select and explain. A. Bleomycin, B. Minocycline, C. Vincristine, D. Daunorubicin",A: Bleomycin,"Answer: A: Bleomycin Explanation: Ans: A. BleomycinRef: Goodman and Gilman, I3,h ed., pg. 1193Few case repos of bleomycin induced flagellate dermatitis and pigmentations are available in literature." "This is a real-world medical entrance exam question, please give the true answer based on the question and selection. Topic name: None, Subject name: Dental","If during an application of an orthodontic force, the level declines to zero between activations then the force duration is classified as: Select and explain. A. Continuous force, B. Interrupted force, C. Intermittent force, D. Differential force",B: Interrupted force,"Answer: B: Interrupted force Explanation: None" "This is a real-world medical entrance exam question, please give the true answer based on the question and selection. Topic name: None, Subject name: Dental","Which material undergoes hysteresis? Select and explain. A. Irreversible hydrocolloid, B. Reversible hydrocolloid, C. Impression plaster, D. Metallic oxide paste",B: Reversible hydrocolloid,"Answer: B: Reversible hydrocolloid Explanation: None" "This is a real-world medical entrance exam question, please give the true answer based on the question and selection. Topic name: None, Subject name: Ophthalmology","In 3rd nerve palsy all seen except? Select and explain. A. Pupil dilation, B. Ptosis, C. Outward upward rolling of pupil, D. Impaired pupillary reflex",A: Pupil dilation,"Answer: A: Pupil dilation Explanation: Ans: A. Pupil dilationRef: American Academy of Ophthalmolog 2018, third nerve palsyThe pathway for pupillary constriction for each eye has an afferent limb taking sensory information to the midbrain, and two efferent limbs (one to each eye)." "This is a real-world medical entrance exam question, please give the true answer based on the question and selection. Topic name: None, Subject name: Dental","Mesial marginal developmental groove is seen in Select and explain. A. Maxillary 1st premolar, B. Mandibular lateral incisor, C. Maxillary lateral incisor, D. Mandibular 1st premolar",A: Maxillary 1st premolar,"Answer: A: Maxillary 1st premolar Explanation: Mesial Aspect From the mesial and distal aspect both the buccal and lingual cusps are visible A well developed mesial marginal ridge and a mesial marginal developmental groove is present In the middle of the mesial surface is the mesial developmental depression which continues beyond the cervical line" "This is a real-world medical entrance exam question, please give the true answer based on the question and selection. Topic name: None, Subject name: Pathology","Focal sclerosing osteomyelitis is: Select and explain. A. Due to excessive periosteal bone formation, B. An extremely painful condition, C. Due to low grade chronic infection, D. A common sequel following sequestrectomy",C: Due to low grade chronic infection,"Answer: C: Due to low grade chronic infection Explanation: None" "This is a real-world medical entrance exam question, please give the true answer based on the question and selection. Topic name: None, Subject name: Dental","Counter bevel in tooth should not be given in Select and explain. A. Facial surface of premolar, B. Occlusal surface of premolar, C. Lingual surface of premolar, D. Any surface of premolar",A: Facial surface of premolar,"Answer: A: Facial surface of premolar Explanation: Direct lines: “A counter bevel is not placed on the facial cusps of maxillary premolars and first molars due to esthetic considerations”." "This is a real-world medical entrance exam question, please give the true answer based on the question and selection. Topic name: AIIMS 2018, Subject name: Pharmacology","Which of the following drug has gameticidal action against all species of plasmodium? Select and explain. A. Primaquine, B. Chloroquine, C. Quinine, D. None of these",A: Primaquine,"Answer: A: Primaquine Explanation: Primaquine: It is gameticidal against all species of Plasmodium i.e. vivax, ovale, falciparum, malariae. Hence, it is DOC to prevent transmission of malaria. It is also used for Radical cure of vivax and ovale malaria as it can kill hypnozoites as well. Chloroquine and Quinine: Gameticidal against P. vivax species only" "This is a real-world medical entrance exam question, please give the true answer based on the question and selection. Topic name: None, Subject name: Dental","Arch perimeter can be measured with Select and explain. A. Cephalogram, B. Brass wire, C. Vernier calipers, D. Occlusal radiograph",B: Brass wire,"Answer: B: Brass wire Explanation: None" "This is a real-world medical entrance exam question, please give the true answer based on the question and selection. Topic name: None, Subject name: Dental","Ideal depth for root end filling with MTA is? Select and explain. A. 3mm, B. 5mm, C. 7mm, D. 1mm",A: 3mm,"Answer: A: 3mm Explanation: None" "This is a real-world medical entrance exam question, please give the true answer based on the question and selection. Topic name: None, Subject name: Dental","Primary herpetic gingivostomatitis usually occurs: Select and explain. A. Prior to age 10, B. Between ages 13 to 30, C. At the onset of the menstrual period, D. During menopause",A: Prior to age 10,"Answer: A: Prior to age 10 Explanation: None" "This is a real-world medical entrance exam question, please give the true answer based on the question and selection. Topic name: None, Subject name: Dental","In spherical alloys as compared to lathe cut: Select and explain. A. Less condensing force is required, B. More condensing force is required, C. Both require same condensing force, D. Manipulation is easy",A: Less condensing force is required,"Answer: A: Less condensing force is required Explanation: None" "This is a real-world medical entrance exam question, please give the true answer based on the question and selection. Topic name: AIIMS 2018, Subject name: Gynaecology & Obstetrics","A young female presents to OPD with a spontaneous aboion and secondary amenorrhea since then. FSH was found to be 6 IU/mL. What is the most probable cause of amenorrhea? Select and explain. A. Ovarian failure, B. Pituitary failure, C. Ongoing pregnancy, D. Uterine synechiae",D: Uterine synechiae,"Answer: D: Uterine synechiae Explanation: * Since the lady is having secondary amenorrhea following an aboion, uterine synechiae is the most likely. Although uterine synechiae mostly develops with an overzealous curretage, it is also seen in spontaneous aboions. Also, here FSH levels are normal so it ours an end organ pathology. (Normal serum FSH value in adult woman is 5 - 20 lU/ml).* In case of ovarian failure - FSH will be high* In case of pituitary failure - Level of FSH is low." "This is a real-world medical entrance exam question, please give the true answer based on the question and selection. Topic name: None, Subject name: Pathology","Most common route for metastasis of oral cancer is by: Select and explain. A. Direct extension, B. Lymphatics, C. Blood vessels, D. Aspiration of tumor cells",B: Lymphatics,"Answer: B: Lymphatics Explanation: None" "This is a real-world medical entrance exam question, please give the true answer based on the question and selection. Topic name: None, Subject name: Medicine","All the following are features of Addison's disease, EXCEPT: Select and explain. A. Hypoglycemia, B. Hypocalcaemia, C. Hypotension, D. Hyponatremia",B: Hypocalcaemia,"Answer: B: Hypocalcaemia Explanation: None" "This is a real-world medical entrance exam question, please give the true answer based on the question and selection. Topic name: None, Subject name: Dental","Electric nerve tester detects pain by stimulating which nerve fibres (OR) Sharp, shooting pain in reversible pulpitis indicates stimulation of (OR) In pulp hyperemia which nerve fibers are stimulated Select and explain. A. A alpha, B. A beta, C. A delta, D. C fibres",C: A delta,"Answer: C: A delta Explanation: None" "This is a real-world medical entrance exam question, please give the true answer based on the question and selection. Topic name: None, Subject name: Biochemistry","Which of the following is ω9 family? Select and explain. A. Oleic Acid, B. Linoleic acid, C. Linolenic acid, D. Arachidonic acid",A: Oleic Acid,"Answer: A: Oleic Acid Explanation: None" "This is a real-world medical entrance exam question, please give the true answer based on the question and selection. Topic name: None, Subject name: Physiology","Who discovered the structure and amino acid sequence of insulin as well as the interspecies differences in molecular structure and won the Nobel prize for? Select and explain. A. Prof. JJ Abel, B. Frederick Sanger, C. Rutter, Goodman and Uhlich, D. Frederick Banting and Charles Best",B: Frederick Sanger,"Answer: B: Frederick Sanger Explanation: Ans: b. Frederick Sanger /Ref Lehninger 5/e p903-904, 292)Frederick Sanger discovered the structure and amino acid sequence of insulin as well as the interspecies differences in molecular structure and got the Nobel Prize for that.Fredrick Sanger again with Walter Gil,be got Nobel prize in 1980Jim sequencing of large nucleic acid (DNA) molecule.""" "This is a real-world medical entrance exam question, please give the true answer based on the question and selection. Topic name: None, Subject name: Dental","If rate of loading is decreased, the mechanical properties of material: Select and explain. A. Remains unchanged, B. Increases, C. Decreases, D. Becomes unpredictable",C: Decreases,"Answer: C: Decreases Explanation: Mechanical events are both temperature and time dependant. As the temperature increases, the mechanical property values decrease. The stress strain curve appears to move to the right and downward. The opposite occurs  during cooling. As the rate of loading decreases, the mechanical properties decrease. This is called 'Strain rate sensitivity' and has important clinical implications. To momentarily make a material's behaviour stiffer or elastic, strain it quickly. Eg.: Remove elastic impressions rapidly so that it will be more elastic and more accurately records the absolute dimensions." "This is a real-world medical entrance exam question, please give the true answer based on the question and selection. Topic name: None, Subject name: Physiology","In relaxed state, chest wall and lung recoil are balanced at: Select and explain. A. Minute volume, B. TLC, C. Residual volume, D. FRC",D: FRC,"Answer: D: FRC Explanation: Ans: D. FRC(Ref: Ganong 25/e p629, 24/e p629; Guyton 13/e p502)In relaxed state, chest wall and lung recoil are balanced at functional residual capacity (FRC).Compliance is developed due to the tendency for tissue to resume its original position after an applied force has been removed.After an expiration during quiet breathing (e.g., at the FRC), the lungs have a tendency to collapse and the chest wall has a tendency to expand." "This is a real-world medical entrance exam question, please give the true answer based on the question and selection. Topic name: None, Subject name: Pediatrics","A 35 years old lady has chromosomal translocation 21/21. The risk of down syndrome in the child is – Select and explain. A. 100%, B. 0%, C. 10%, D. 50%",A: 100%,"Answer: A: 100% Explanation: Cvtogenetic aspects of own's syndrome - The most common finding is trisomy 21. Molecular studies have revealed that the additional number 21 chromosome is derived from the mother in 95% of Cases. (The recurrence risk in these families is approximately 1%.) If a female with Down syndrome due to trisomy 21 conceives then there is a risk of 50% that the baby will also have trisomy 21. Males with Down syndrome have rarely if ever reproduced. Chromosome findings in Down syndrome Trisomy 21, e.g. 47, XY, + 21                  -                  95% Mosaicism, e.g. 46, XX, / 47, XX, + 21           -         2% Robertsonian translocation, e.g. 46 XX, - 15,+ t (15q21q)                           -               3% Inheritence in Mosaic if own's syndrome Children with mosaic Down syndrome are usually less severely affected than in the full-blown syndrome, and if only a small proportion of cells are trisomic then these individuals may lead normal lives. In the event of reproduction, of these individuals there is a relatively high risk that the baby will have full trisomy 21 with the precise risk equalling the proportion of gametes which carry an additional number 21 chromosome. Inheritance in Down's syndrome with tranclocation When a child has Down syndrome as a result of an unbalanced Robertsonian translocation, there is a probability of around 25% that one of the parents will carry this in a balanced form. The remaining 75% of cases arise as de novo events and convey a low recurrence risk of approximately 1% If, however, a parent is shown to be a carrier then there will be a significant risk that a future child will be affected, usually of the order of 2-5% for a carrier male and 10-15% for a carrier female. In the very rare event that a parent carries a balanced 21q21e Robertsonian translocation, the risk of Down's   drome in liveborn offs rin• will be 100%. ​Risk of abnormal offspring for a carrier of a balanced translocation" "This is a real-world medical entrance exam question, please give the true answer based on the question and selection. Topic name: None, Subject name: Gynaecology & Obstetrics","Suganti Devi is 30 weeks pregnant with idiopathic cholestasis, is likely to present with following features except: Select and explain. A. Serum bilirubin of 2 mg/dl, B. Serum alkaline phosphatase slightly elevated, C. SGPT of 200 units, D. Prolongation of prothrombin time",D: Prolongation of prothrombin time,"Answer: D: Prolongation of prothrombin time Explanation: Let’s see each option separately. Option “a” Serum bilirubin of 2 mg/dL. Bilirubin level rarely exceed 5 mg%. Bilirubin levels are usually between 2-5 mg/dL i.e. Option “a” is correct. Option “b” Serum alkaline phosphatase of 30 KAU. “Alkaline phosphatase may be mildly elevated.” Robbins 7/e, p 921 i.e. alkaline phosphatase levels may be normal also. Hence, option b is correct. Option “c” SGPT of 200 units. “Serum transaminases levels are normal to moderately elevated but seldom exceed 250 IU/L.” Williams Obs. 23/e, p 1064 i.e. Serum transaminases (SGPT) may be 200 units. Option “d” Prolongation of prothrombin time “Prothrombin time is usually normal unless there is malabsorption.” Mgt of High Risk pregnancy – SS Trivedi, Manju Puri, p 356 Prothrombin is coagulant factor II. Its formation in liver is dependant on fat soluble vitamin K. Absorption of vitamin K occurs with bile acid. In cholestasis absorption of bile acid (vitamin K) is not affected, rather there is accumulation of bile acids, so levels of vitamin K and clotting factors dependant on vitamin K are also normal. The prothrombin time therefore, remains normal in cholestasis. Only when ursodeoxycholic acid or cholestyramine are being given to patients of cholestasis. Prothrombin time needs to be monitored because these drugs decrease the absorption of bile acids and can therefore cause prolongation of prothrombin time." "This is a real-world medical entrance exam question, please give the true answer based on the question and selection. Topic name: None, Subject name: Dental","TMJ develops at the age of Select and explain. A. 18 weeks, B. 10 weeks, C. 6 weeks, D. 29 weeks",B: 10 weeks,"Answer: B: 10 weeks Explanation: TMJ develops at 8-12 weeks of gestation. It is the last joint to develop in the body. The work in the above quoted article establishes three phases in TMJ development: (1) the blastematic stage (weeks 7-8 of development); (2) the cavitation stage (weeks 9-11 of development); and (3) the maturation stage (after week 12 of development). This study identifies the critical period of TMJ morphogenesis as occurring between weeks 7 and 11 of development." "This is a real-world medical entrance exam question, please give the true answer based on the question and selection. Topic name: None, Subject name: Dental","Light cure system, side effect is: Select and explain. A. Iritis, B. Cataract, C. Conjunctivitis, D. Retinal damage",D: Retinal damage,"Answer: D: Retinal damage Explanation: None" "This is a real-world medical entrance exam question, please give the true answer based on the question and selection. Topic name: None, Subject name: Gynaecology & Obstetrics","A Rh-negative mother, who has Indirect Coombs Test (ICT), negative was given Anti-D during 28 weeks of pregnancy. Which of the following is the ideal one? Select and explain. A. Give another dose of Anti-D 72 hours postpaum depending on the baby blood group, B. Give another dose of Anti-D 72 hours postpaum irrespective of baby blood group, C. No need of additional dose since she is ICT negative, D. All of the above",A: Give another dose of Anti-D 72 hours postpaum depending on the baby blood group,"Answer: A: Give another dose of Anti-D 72 hours postpaum depending on the baby blood group Explanation: Answer- A. Give another dose of Anti-D 72 hours postpaum depending on the baby blood groupACOG (2010) recommends anti-D immune globulin to be given prophylactically to all Rh D-negative, unsensitized women at approximately 28 weeks, and a second dose given after delivery if the infant is Rh D-positive. Before the 28-week dose of anti-D immune globulin, repeat antibocly screening is recommended to identify individuals who have become alloimmunized. Following delivery, anti-D immune globulin should be given within 72 hours." "This is a real-world medical entrance exam question, please give the true answer based on the question and selection. Topic name: None, Subject name: Pathology","Which syndrome consists of flushing, warmness and perspiration over the cheek and pinna of the ear on the side following the ingestion of highly seasoned food? Select and explain. A. Fanconi's syndrome, B. Auriculotemporal syndrome, C. Horner's syndrome, D. Cushing's syndrome",B: Auriculotemporal syndrome,"Answer: B: Auriculotemporal syndrome Explanation: Auriculotemporal Syndrome (Frey's Syndrome, Gustatory Sweating) The auriculotemporal syndrome is an unusual phenomenon, which arises as a result of damage to the auriculotemporal nerve and subsequent reinnervation of sweat glands by parasympathetic salivary fibres. Clinical Features  The patient typically exhibits flushing and sweating of the involved side of the face, chiefly in the temporal area, during eating. The severity of this sweating may often be increased by tart foods. Of further interest is the fact that profuse sweating may be evoked by the parenteral administration of pilocarpine or eliminated by the administration of atropine or by a procaine block of the auriculotemporal nerve. There is a form of gustatory sweating which occurs in otherwise normal individuals when they are eating certain foods, particularly spicy or sour ones. This consists of diffuse facial sweating, not simply a perioral sweating and may even be on a hereditary basis. Reference: Shafer’s Textbook of ORAL PATHOLOGY Eighth Edition page no 575" "This is a real-world medical entrance exam question, please give the true answer based on the question and selection. Topic name: None, Subject name: Dental","According to Tweed inclination of lower incisors by 1° labially increases the arch length by Select and explain. A. 0.5 mm, B. 0.8 mm, C. 1.0 mm, D. 1.2 mm",B: 0.8 mm,"Answer: B: 0.8 mm Explanation: None" "This is a real-world medical entrance exam question, please give the true answer based on the question and selection. Topic name: None, Subject name: Gynaecology & Obstetrics","Foltowing are the features of the color of normal amniotic fluid during delivery? Select and explain. A. Milky to yellowish green with mucus flakes, B. Amber colored, C. Clear colorless to Pale Yellow, D. Golden color",C: Clear colorless to Pale Yellow,"Answer: C: Clear colorless to Pale Yellow Explanation: Ans: C. Clear colorless to Pale YellowRef: DC Dutto's textbook of Obstetrics, 9't' ed.Green yellow with flakes (meconium stained)- Fetal distressGolden color- Rh incompatibilityGreenish Ye11ow (saffron)- postmaturityDark colored - concealed accidental hemorrhageDark brown (tobacco juice)- Intruterine demise" "This is a real-world medical entrance exam question, please give the true answer based on the question and selection. Topic name: None, Subject name: Dental","The shape of the occlusal surface of the permanent maxillary first molar is: Select and explain. A. Oval, B. Trapezoidal, C. Triangular, D. Rhomboidal",D: Rhomboidal,"Answer: D: Rhomboidal Explanation: None" "This is a real-world medical entrance exam question, please give the true answer based on the question and selection. Topic name: None, Subject name: Medicine","A patient presented with a steering wheel injury to the right side of chest with breathlessness and shock. How will you differentiate tension pneumothorax and cardiac tamponade? Select and explain. A. Pulse pressure, B. JVP, C. Breath sound, D. Hea sounds",C: Breath sound,"Answer: C: Breath sound Explanation: Ans: C. Breath sound(Ref Harrison 19/e p1573, 1719, 18/e p1972)Raised JVP, pulse pressure and muffled hea sounds along with pulsus paradoxus are seen in both cardiac tamponade and tension pneumothorax.These are differentiated by auscultation for breath sounds, which are normal in cardiac tamponade but absent in tension pneumothorax patients.Pericardial Tamponade:The three principal features of tamponade (Beck's triad) are hypotension, soft or absent hea sounds, and jugular venous distention with a prominent x-descent but an absent y-descent." "This is a real-world medical entrance exam question, please give the true answer based on the question and selection. Topic name: AIIMS 2019, Subject name: Pathology","Which of the following causes vasodilation? Select and explain. A. Thromboxane A2, B. Prostaglandin E2, C. Histamine, D. Serotonin",C: Histamine,"Answer: C: Histamine Explanation: Major vasoactive amine responsible for vasodilation is Histamine. It causes dilation of aerioles and increases the permeability of venules. Prostaglandins can also cause vasodilation but major chemical responsible is histamine. Serotonin (5hydroxytryptamine) is a preformed vasoactive mediator present in platelets and ceain neuroendocrine cells, such as in the gastrointestinal tract, and in mast cells in rodents but not humans." "This is a real-world medical entrance exam question, please give the true answer based on the question and selection. Topic name: None, Subject name: Medicine","A I 6-y ear-old girl presented with abdominal pain after administration of a sulfa drug. She was incoherent and suffered a seizure. Which of the following is the most likely possibility? Select and explain. A. Acute intermittent porphyria, B. Congenital erythropoetic porphyria, C. Adenosine deaminase deficiency, D. HGPase deficiency",A: Acute intermittent porphyria,"Answer: A: Acute intermittent porphyria Explanation: Ans.a Acuteintermittentporphyria (Re/: Harrison 19/ep2521, 18/ep3168-3169)Acute intermittent porphyria is precipitated by barbiturates, alcohol, exposure to heavy metals, sulfa drugs.Acute Intermittent Porphyria:Precipitated by barbiturates, alcohol, exposure to heary metals, sulfa drugsPresent with nonspecific neurovisceral pain: Abdominal pain, psychiatric disorders, seizures, coma and bulbar Paralysis" "This is a real-world medical entrance exam question, please give the true answer based on the question and selection. Topic name: None, Subject name: Pathology","Microdontia is most commonly seen affecting: Select and explain. A. Max. Lateral incisor, B. Mand. second premolar, C. Mand. central incisor, D. Mand. first premolar",A: Max. Lateral incisor,"Answer: A: Max. Lateral incisor Explanation: None" "This is a real-world medical entrance exam question, please give the true answer based on the question and selection. Topic name: None, Subject name: Surgery","Condylar fracture which leads to disruption of TMJ anteriorly is an example of: Select and explain. A. Simple fracture, B. Compound fracture, C. Complex fracture, D. Comminuted fracture",C: Complex fracture,"Answer: C: Complex fracture Explanation: None" "This is a real-world medical entrance exam question, please give the true answer based on the question and selection. Topic name: None, Subject name: Gynaecology & Obstetrics","Which of the following tests is most sensitive for the detection of iron depletion in pregnancy ? Select and explain. A. Serum iron, B. Serum ferritin, C. Serum transferrin, D. Serum iron binding capacity",B: Serum ferritin,"Answer: B: Serum ferritin Explanation: Serum ferritin is the most sensitive test as it correlates best with iron stores and is the first test to become abnormal in case of iron deficiency. Remember: Storage form of iron : – Ferritin Transport form of iron – Transferrin As per CDC serum ferritin less than 15 mg/l confirms iron deficiency anemia." "This is a real-world medical entrance exam question, please give the true answer based on the question and selection. Topic name: None, Subject name: Pathology","Epstein-Barr virus-associated lymphomas are all of these except: Select and explain. A. NK T-cell lymphoma, B. Nodular lymphocyte-predominant Hodgkin's lymphoma, C. Plasmablastic lymphoma, D. Lymphomatoid granulomatosis",B: Nodular lymphocyte-predominant Hodgkin's lymphoma,"Answer: B: Nodular lymphocyte-predominant Hodgkin's lymphoma Explanation: Ans: B. Nodular lymphocyte-predominant Hodgkin's lymphoma(Ref: Robbins 9/e p327, 609)Among Hodgkin's lymphomas, EBV is not associated with nodular sclerosis & lymphocyte predominant Hodgkin's lymphoma.Epstein-Barr Virus (EBV):Member of the herpes virus family.Causative agent for African form of Burkitt lymphoma.B-cell lymphomas in immunosuppressed individuals (paicularly in those with HIV infection or undergoing immunosuppressive therapy after organ or bone marrow transplantation).Subset of Hodgkin lymphoma.Nasopharyngeal.Some gastric carcinomas.Rrare forms of T-cell lymphoma.Natural killer (NK) cell lymphoma." "This is a real-world medical entrance exam question, please give the true answer based on the question and selection. Topic name: None, Subject name: Anatomy","Inferior thyroid artery is a branch of : Select and explain. A. Internal thoracic artery, B. Thyrocervical trunk, C. Dorsal scapular artery, D. Costocervical trunk",B: Thyrocervical trunk,"Answer: B: Thyrocervical trunk Explanation: Thyrocervical trunk is a short, wide vessel which arises from the front of the first part of the subclavian artery, close to the medial border of the scalenus anterior, and between the phrenic and vagus nerves. It almost immediately divides into the inferior thyroid, suprascapular and transverse cervical arteries." "This is a real-world medical entrance exam question, please give the true answer based on the question and selection. Topic name: AIIMS 2018, Subject name: ENT","A person presented to OPD with complaints of rotatory veigo and nausea in the morning on change in position of the head. WHat is your diagnosis? Select and explain. A. Labyrinthitis, B. BPPV, C. Vestibular neuronitis, D. Meniere's disease",B: BPPV,"Answer: B: BPPV Explanation: Positional veigo/ BPPV: It is characterized by veigo when the head is placed in a ceain critical position. There is no hearing loss or other neurologic symptoms. Positional testing establishes the diagnosis and helps to differentiate it from positional veigo of central origin. Disease caused by a disorder of posterior semicircular canal though many patients have history of head trauma and ear infection. It has been demonstrated that otoconial debris, consisting of crystals and calcium carbonate, is released from the degeneration macula of the utricle and floats freely in the endolymph. When it settles on the cupula of posterior semicircular canal in a critical head position, it causes displacement of the cupula and veigo. The veigo is fatigable on assuming the same position repeatedly due to dispersal of the otoconia but can be induced again after a period of rest. Thus, typical history and Hallpike maneuver establishes the diagnosis. The condition can be treated by performing Epley's maneuver. The principle of this maneuver is to reposition the otoconial debris from the posterior semicircular canal back into the utricle. The doctor stands behind the patient and the assistant on the side. The patient is made to sit on the table so that when he is made to lie down, his head is beyond the edge of the table as is done in Dix-Hallpike maneuver. His face is turned 450 to the affected side. The maneuver consists of five positions * Position 1. With the head turned 450, the patient is made to lie down in head-hanging position (Dix-Hallpike). It will cause veigo and nystagmus. Wait till veigo and nystagmus subside. * Position 2. Head is now turned so that affected ear is facing up at a 900 rotation. * Position 3. The whole body and head are now rotated away from the affected ear to a lateral recumbent position in a 900 -rotation face-down position. * Position 4. Patient is now brought to a sitting position with head still turned to the unaffected side by 450. * Position 5. The head is now turned forward and chin brought down 200. There should be a pause at each position till there is no nystagmus or there is slowing of nystagmus, before changing to the next position. After maneuver is complete, patient should maintain an upright posture for 48 h. Eighty percent of the patients will be cured by a single maneuver. If the patients remain symptomatic, the maneuver can be repeated. A bone vibrator placed on the mastoid bone helps to loosen the debris. - Meniere's disease: Has an episodic veigo and it lasts for >20 minutes. - Vestibular neuronitis: Veigo lasts for many days. - Labyrinthitis: There is a history of fever and trauma, and the veigo lasts for days." "This is a real-world medical entrance exam question, please give the true answer based on the question and selection. Topic name: None, Subject name: Microbiology","Which of the following is an obligate intracellular parasite? Select and explain. A. Tropheryma whippelii, B. Baonella henselae, C. Ehrlichia chaffeensis, D. Coxiella burnetii",C: Ehrlichia chaffeensis,"Answer: C: Ehrlichia chaffeensis Explanation: Ans: C. Ehrlichia chaffeensis (Ref: Ananthanarayan 10/e p416, 8/e p409; Jawetz 27/e p346, 347; Harrison 19/e p1159, 1162)Though both Coxiella and Ehrlichia are obligate intracellular pathogens, in 2009 scientists repoed a technique allowing the Q-fever pathogen Coxiella burnetii to grow in an axenic culture and suggested the technique may be useful for study of other pathogens.Hence, Ehrlichia is a better answer in this case." "This is a real-world medical entrance exam question, please give the true answer based on the question and selection. Topic name: None, Subject name: Medicine","Anti-epileptic which is not associated with congenital malformation when used in pregnant woman is: Select and explain. A. Phenytoin, B. Phenobarbitone, C. Carbamazepine, D. Valproate",B: Phenobarbitone,"Answer: B: Phenobarbitone Explanation: Phenobarbitone is considered as safest antiepileptic drug in pregnancy." "This is a real-world medical entrance exam question, please give the true answer based on the question and selection. Topic name: None, Subject name: Dental","Portion of tooth covered by calculus or altered cementum is: Select and explain. A. Toxic zone, B. Deposit zone, C. Instrumentation zone, D. Take away zone",C: Instrumentation zone,"Answer: C: Instrumentation zone Explanation: Calculus is removed by a series of controlled, overlapping, short, powerful strokes primarily using wrist-arm motion . As calculus is removed, resistance to  the  passage  of  the  cutting  edge  diminishes  until  only  a  slight roughness  remains. Longer, lighter  root-planing  strokes  are  then activated with less lateral pressure until the root surface is completely smooth and hard. The instrument handle must be rolled carefully between the thumb and fingers to keep the blade adapted closely to the tooth surface as line angles, developmental depressions, and other changes in tooth contour are followed. Scaling and root-planing strokes should be confined to the portion of the tooth where calculus or altered cementum is found; this area is known as the instrumentation zone. Sweeping the instrument over the crown where it is not needed wastes operating time, dulls the instrument, and causes loss  of control. Key Concept: Scaling and root-planing strokes should be confined to the portion of the tooth where calculus or altered cementum is found; this area is known as the instrumentation zone. Sweeping the instrument over the crown where it is not needed wastes operating time, dulls the instrument, and causes loss  of control. Ref: Newman and Carranza’s Clinical Periodontology, thirteenth edition; page no 545.e15" "This is a real-world medical entrance exam question, please give the true answer based on the question and selection. Topic name: None, Subject name: Dental","Annealing is also known as: Select and explain. A. Soft hardening treatment, B. Hard hardening treatment, C. Precipitating heat treatment, D. Stress relieving heat treatment",A: Soft hardening treatment,"Answer: A: Soft hardening treatment Explanation: None" "This is a real-world medical entrance exam question, please give the true answer based on the question and selection. Topic name: None, Subject name: Gynaecology & Obstetrics","Fetal ECHO shows congenital hea block, what should be the mother screened for? Select and explain. A. SLE, B. Myxoma, C. APLA, D. None",A: SLE,"Answer: A: SLE Explanation: Ans. a). SLECongenital hea block is a rare disorder that occurs in about one out of 22,000 live bihs.In most cases, the cause is not known, but babies of mothers with lupus or other autoimmune diseases, or babies with congenital hea disorders, are at higher risk.Pregnant women who have autoimmune diseases, such as lupus or Sjogren's syndrome, are at an increased risk of having a baby with congenital hea block.Also, a tumor on the baby's hea can cause hea block." "This is a real-world medical entrance exam question, please give the true answer based on the question and selection. Topic name: AIIMS 2018, Subject name: Physiology","During exercise in physiological limits what is the effect on end systolic volume? Select and explain. A. ESV decreases, B. ESV increase, C. ESV remain unchanged, D. ESV first decrease and then increases",A: ESV decreases,"Answer: A: ESV decreases Explanation: During exercise, there is increased sympathetic discharge and venous return to hea. This causes increased stroke volume mainly due to increased myocardial contractility. Due to increased stroke volume, the ESV decreases. In the image given the stoke volume is indicated by the width of the loop and LV pressure by the height of the loop. Three major effects occurring during exercise are : Sympathetic nervous system activation in many tissues with stimulatory effects on circulation. Increase in aerial pressure Increase in cardiac output." "This is a real-world medical entrance exam question, please give the true answer based on the question and selection. Topic name: None, Subject name: Pharmacology","Which of the following drug is commonly used in treatment for cancer associated thromboembotismt Select and explain. A. Low molecular weight heparin, B. anti-thrombin III inhibitors, C. Direct Xainhibitors, D. Warfarin",A: Low molecular weight heparin,"Answer: A: Low molecular weight heparin Explanation: Ans: A. Low molecular weight heparinCurrent guideline-endorsed therapy options for cancer-associated thrombosis include low molecularweight heparin (LMWH), unfractionated heparin (UFH), warfarin & fondaparinux.All current guidelines recommend LMWH for at least 3-6 months in cancer-associated VTE." "This is a real-world medical entrance exam question, please give the true answer based on the question and selection. Topic name: None, Subject name: Dental","Which of the following is not responsible for endogenous staining of teeth during development? Select and explain. A. Tetracycline, B. Rh incompatibility, C. Neonatal liver disease, D. Vitamin-C deficiency",D: Vitamin-C deficiency,"Answer: D: Vitamin-C deficiency Explanation: None" "This is a real-world medical entrance exam question, please give the true answer based on the question and selection. Topic name: None, Subject name: Psychiatry","A young girl hospitalised with anorexia nervosa is on treatment, Even after taking adequate food according to the recommended diet plan for last 1 week, there is no gain in weight, what is the next step in management: Select and explain. A. Increase fluid intake, B. Observe patient for 2 hours after meal, C. Increase the do se of anxiolytics, D. Increase the caloric intake from 1500 kcal to 2000 kcal per day",B: Observe patient for 2 hours after meal,"Answer: B: Observe patient for 2 hours after meal Explanation: Ans. B. Observe patient for 2 hours after mealIn this condition because the food is often regurgitated after meals, the staff may be able to control vomiting by making the bathroom inaccessible for at least 2 hours after meals or by having an attendant in the bathroom to prevent the oppounity for vomiting.The hospital staff should give the patients about 500 calories over the amount required to maintain their present weight (usually 1500 to 2000 calories a day)." "This is a real-world medical entrance exam question, please give the true answer based on the question and selection. Topic name: None, Subject name: Biochemistry","Which of these amino acids does not enter the Krebs cycle by forming Acetyl-CoA pyruvate? Select and explain. A. Glycine, B. Tyrosine, C. Hydroxyprolinc, D. Alanine",B: Tyrosine,"Answer: B: Tyrosine Explanation: Ans: B. Tyrosine(Ref: Harper 30Ie p165)Tyrosine enters Krebs cycle fumarate, while all others form pyruvate to enter the Krebs cycle." "This is a real-world medical entrance exam question, please give the true answer based on the question and selection. Topic name: None, Subject name: Dental","Which of the following type of flow occurs, when on applying critical shear stress, a material turns to liquid? Select and explain. A. Bingham body behaviour, B. Newtonian body, C. Dilatant, D. Pseudoplastic",A: Bingham body behaviour,"Answer: A: Bingham body behaviour Explanation: The viscosity of many dental materials decreases with increasing strain rate until it reaches a nearly constant value. That is, the faster they are stirred, forced through a syringe, or squeezed, the less viscous and more fluid they become. This is pseudoplastic viscosity and is illustrated by the change in slope of the plot in the figure. The viscosity of a pseudoplastic fluid decreases with increasing shear rate.  The tomato-based food condiment ketchup is also pseudoplastic, which makes it difficult to remove from a bottle. Shaking the bottle or rapping the side of the bottle increases its shear rate, decreases its viscosity, and improves its pourability.    A Bingham plastic is a viscoplastic material that behaves as a rigid body at low stresses but flows as a viscous fluid at high stress. It is named after Eugene C. Bingham who proposed its mathematical form." "This is a real-world medical entrance exam question, please give the true answer based on the question and selection. Topic name: None, Subject name: Dental","Facial nerve is located? Select and explain. A. Above SMAS and below parotidomassetric fascia, B. Below SMAS and below parotidomassetric fascia, C. Above SMAS and above parotidomassetric fascia, D. Below SMAS and above parotidomassetric fascia",B: Below SMAS and below parotidomassetric fascia,"Answer: B: Below SMAS and below parotidomassetric fascia Explanation: None" "This is a real-world medical entrance exam question, please give the true answer based on the question and selection. Topic name: None, Subject name: Pathology","RBCs are stored at what temperature? Select and explain. A. -2 to -4 degC, B. 2-6 degC, C. 20-25 degC, D. 37 degC",B: 2-6 degC,"Answer: B: 2-6 degC Explanation: Ans: B. 2-6 degC(Ref UK-NHS Guidelines; Harrison 19/e p138e-2).RBCs should be stored at a temperature of 2-6 ""Celsius.ComponentStorage & durationWhole Blood2-6degC for 42 daysdegPacked RBCs2-6degC for 42 daysdegPlatelets22-24degC for 5 daysdegFFP-18degC for 1 yeardeg -18degC for 1 yeardeg -18degC for 1 year" "This is a real-world medical entrance exam question, please give the true answer based on the question and selection. Topic name: None, Subject name: Dental","Most common complication during ridge split or bone spreading is Select and explain. A. Dehiscence, B. Facial plate splitting, C. Implant micro movement, D. Poor position of implant",B: Facial plate splitting,"Answer: B: Facial plate splitting Explanation: None" "This is a real-world medical entrance exam question, please give the true answer based on the question and selection. Topic name: None, Subject name: Dental","Etching time in teeth with fluorosis Select and explain. A. 10-15 sec, B. 15-30 sec, C. 60-90 sec, D. Can’t be etched",C: 60-90 sec,"Answer: C: 60-90 sec Explanation: Generally, a 20 second time is recommended. Enamel rich with Flourohydroxy apatite may be resistant to etching and may need to be exposed for longer periods. Etching time for enamel varies depending on the type and quality of enamel. Generally, a 15 sec etching with 30 to 40% phosphoric acid is sufficient to reach the characteristic clinical endpoint of a frosty enamel appearance. Deciduous enamel generally contains same prismless enamel that has not yet worn away and requires longer etching time (20-30sec) to create a etched pattern. 5-40% phosphoric acid recommends longer etching times for primary teeth, approximately 30sec." "This is a real-world medical entrance exam question, please give the true answer based on the question and selection. Topic name: None, Subject name: Surgery","Compression of bone is resisted by: Select and explain. A. Protein component, B. lipid component, C. Collagen component, D. Mineral component",D: Mineral component,"Answer: D: Mineral component Explanation: None" "This is a real-world medical entrance exam question, please give the true answer based on the question and selection. Topic name: None, Subject name: Physiology","Golgi tendon organs detect: Select and explain. A. Static muscle length, B. Dynamic muscle length, C. Muscle tension, D. Muscle action",C: Muscle tension,"Answer: C: Muscle tension Explanation: STRUCTURE OF GOLGI TENDON ORGAN Golgi tendon organ  is  situated  in  the  tendon  of skeletal  muscle near the attachment of extrafusal fibers. It is placed in series between the muscle fibers and the  tendon. Golgi tendon organ is formed by a group of nerve endings covered by a connective tissue capsule. NERVE SUPPLY TO GOLGI TENDON ORGAN Sensory nerve fiber supplying the Golgi tendon organ belongs to Ib type. The nerve fiber supplying Golgi tendon organ ramifies into many branches. Each branch ends in the form of a knob.  FUNCTIONS OF GOLGI TENDON ORGAN Golgi tendon organ gives response to the change in the force or tension developed in the skeletal muscle during contraction. It is also the receptor for inverse stretch reflex and lengthening reaction and thereby prevents damage of muscle due to overstretching." "This is a real-world medical entrance exam question, please give the true answer based on the question and selection. Topic name: None, Subject name: Dental","A properly designed rest on lingual surface of a canine is preferred to a rest on incisal surface because: Select and explain. A. Less Leverage is exerted against the tooth by Lingual rest, B. Enamel is thicker on lingual surface, C. Visibility and access is better on lingual surface, D. The cingulum of canine produces a naturalsurface for recess",A: Less Leverage is exerted against the tooth by Lingual rest,"Answer: A: Less Leverage is exerted against the tooth by Lingual rest Explanation: None" "This is a real-world medical entrance exam question, please give the true answer based on the question and selection. Topic name: None, Subject name: Dental","For supporting complex amalgam restoration, which of the following cement has best modulus of elasticity? Select and explain. A. ZOE, B. GIC, C. Polycarboxylate, D. Zinc phosphate",D: Zinc phosphate,"Answer: D: Zinc phosphate Explanation: Zinc phosphate cement has compressive strength of 104 MPa and modulus of elasticity of 13 GPa. In very deep cavities with microscopic exposure, it is recommended that calcium hydroxide should be applied followed by zinc oxide eugenol or zinc phosphate under the restoration.  For amalgam restorations, zinc phosphate is recommended as cavity base." "This is a real-world medical entrance exam question, please give the true answer based on the question and selection. Topic name: None, Subject name: Dental","Perfect example of transition of anterior to posterior is Select and explain. A. Mandibular canine, B. Maxillary canine, C. Mandibular 1st premolar, D. Maxillary 1st premolar",A: Mandibular canine,"Answer: A: Mandibular canine Explanation: None" "This is a real-world medical entrance exam question, please give the true answer based on the question and selection. Topic name: AIIMS 2019, Subject name: Gynaecology & Obstetrics","Sequential order of sperm formation: Spermatogonia Spermatocyte Spermatids Spermatozoa Select and explain. A. 2314, B. 3214, C. 3124, D. 1234",D: 1234,"Answer: D: 1234 Explanation: None" "This is a real-world medical entrance exam question, please give the true answer based on the question and selection. Topic name: None, Subject name: Dental","Inlay wax is used to: Select and explain. A. Temporarily cement inlays, B. Make inlay wax patterns, C. Invest inlay patterns, D. Box models",B: Make inlay wax patterns,"Answer: B: Make inlay wax patterns Explanation: None" "This is a real-world medical entrance exam question, please give the true answer based on the question and selection. Topic name: None, Subject name: Physiology","find false statement regarding sensory endings Select and explain. A. Annulospiral wrap the ends, B. Primary ending is annulospiral, C. Primary ending conduct 1a fibres, D. Flower spray is secondary",A: Annulospiral wrap the ends,"Answer: A: Annulospiral wrap the ends Explanation: None" "This is a real-world medical entrance exam question, please give the true answer based on the question and selection. Topic name: None, Subject name: Dental","Dentition of humans is: Select and explain. A. Herbivorous, B. Omnivorous, C. Carnivorous, D. None of the above",B: Omnivorous,"Answer: B: Omnivorous Explanation: None" "This is a real-world medical entrance exam question, please give the true answer based on the question and selection. Topic name: None, Subject name: Biochemistry","Glycogen synthesis and breakdown takes place in the same cell, having enzymes necessary for both the pathways. Why the glucose-6-phosphate, freshly synthesized during glycogenesis in cytoplasm of hepatocytes, is not immediately degraded by the enzyme glucose!6-phosphatase? Select and explain. A. The thermodynamics does not or such a reaction to occur, B. Glucose-6-phosphatase is present in the endoplasmic reticulum and cannot act on glycogen formed in the cytoplasm, C. Glycogenesis and glycogenolysis are tightly regulated such that enzymes of only one of those is present at a time., D. Steric hindrance due to albumin",B: Glucose-6-phosphatase is present in the endoplasmic reticulum and cannot act on glycogen formed in the cytoplasm,"Answer: B: Glucose-6-phosphatase is present in the endoplasmic reticulum and cannot act on glycogen formed in the cytoplasm Explanation: Ans: B. Glucose-6-phosphatase is present in the endoplasmic reticulum and cannot act on glycogen formed in the cytoplasm(Ref Harper 30/e p178)Glucose-6-phosphate:Formed in cytoplasm of hepatocytes.Glucose-6-phosphatase:Present in lumen of smooth endoplasmic reticulum of cell.Reaction does not take place until gluconeogenesis is ored.Glucose-6-phosphatase catalyzes hydrolysis of glucose-6-phosphate in liver - Expos glucose a increases blood glucose concentration" "This is a real-world medical entrance exam question, please give the true answer based on the question and selection. Topic name: None, Subject name: Social & Preventive Medicine","Number of vision centers under vision 2020, a national program for control of blindness are - Select and explain. A. 20, B. 200, C. 2000, D. 20000",D: 20000,"Answer: D: 20000 Explanation: Ans. is 'd' i.e., 20000 The proposed structure for vision 2020, NPCB : Vision centres 20,000 (Primary level)Service centres 2,000 (Secondary level)Tranning centres 200 (Teiary level)Centres of Excellence 20 (COE)" "This is a real-world medical entrance exam question, please give the true answer based on the question and selection. Topic name: None, Subject name: Surgery","Tachycardia in a patient in shock is response to: Select and explain. A. Hypotension, B. Vasoconstriction, C. CNS anoxia, D. Anxiety",C: CNS anoxia,"Answer: C: CNS anoxia Explanation: None" "This is a real-world medical entrance exam question, please give the true answer based on the question and selection. Topic name: None, Subject name: Anatomy","Which of the following is not a branch of external carotid artery Select and explain. A. Posterior auricular artery, B. Occipital artery, C. Maxillary artery, D. Ophthalmic artery",D: Ophthalmic artery,"Answer: D: Ophthalmic artery Explanation: Branches of ECA:- Anterior - superior thyroid, lingual a., Facial a. Posterior - Occipital a., Posterior auricular a. Medial - Ascending pharyngeal a. Terminal - Maxillary a., Superficial temporal a." "This is a real-world medical entrance exam question, please give the true answer based on the question and selection. Topic name: AIIMS 2019, Subject name: ENT","A 35 years old female patient present with hearing loss with improvement in hearing in noisy environment. On examination Rinne's is negative and weber's is centralized. Following is the audiometry repo. What is the most likely diagnosis: Select and explain. A. Meniere's disease, B. Perilymph fistula, C. Stapedial Otosclerosis, D. Vestibular Schwannoma",C: Stapedial Otosclerosis,"Answer: C: Stapedial Otosclerosis Explanation: POINTS OURING STAPEDIAL OTOSCLEROSIS: - Conductive hearing loss. - MC seen in females 20-30 yrs. - hears better in noisy surrounding (Paraacusis willisii) - Audiometry repo a dip at 2000Hz (Carha's notch) Meniere's disease causes SNHL. Perilymph fistula will cause SNHL. Vestibular schwannoma also will cause SNHL." "This is a real-world medical entrance exam question, please give the true answer based on the question and selection. Topic name: None, Subject name: Medicine","Myocardial infarction; signs are all except Select and explain. A. ‘a’ wave elevation, B. Levine’s sign, C. Kussmaul’s sign, D. Bradycardia in case of infarction of anterior portion of heart",D: Bradycardia in case of infarction of anterior portion of heart,"Answer: D: Bradycardia in case of infarction of anterior portion of heart Explanation: Kussmaul sign is a paradoxical rise in jugular venous pressure (JVP) on inspiration, or a failure in the appropriate fall of the JVP with inspiration Levine's sign is a clenched fist held over the chest to describe ischemic chest pain Bradycardia, in case of ischaemia of inferior part of the heart. Tachycardia, in case of ischaemia of anterior portion of heart." "This is a real-world medical entrance exam question, please give the true answer based on the question and selection. Topic name: None, Subject name: Social & Preventive Medicine","Which one of the following is NOT a socio-economic indicator? Select and explain. A. Literacy rate, B. Life expectancy at birth, C. Housing, D. Family size",B: Life expectancy at birth,"Answer: B: Life expectancy at birth Explanation: Socio-economic indicators: Housing Family size Literacy rate Availability per capitacalorie Per capita GNP Growth rate Level of unEmployment Dependency ratio" "This is a real-world medical entrance exam question, please give the true answer based on the question and selection. Topic name: None, Subject name: Surgery","LA acts on nerve membrane by: Select and explain. A. Blocking conductance of Na+ from interior to exterior, B. Blocking conductance of Na+ from exterior to interior, C. Blocking conductance of K+ from interior to exterior, D. Blocking conductance of K+ from exterior to interior",B: Blocking conductance of Na+ from exterior to interior,"Answer: B: Blocking conductance of Na+ from exterior to interior Explanation: None" "This is a real-world medical entrance exam question, please give the true answer based on the question and selection. Topic name: None, Subject name: Medicine","Sudden death in a patient with an acute myocardial infarction is most commonly due to Select and explain. A. Aortic dissection, B. Atrial fibrillation, C. Cardiac tamponade, D. Ventricular fibrillation",D: Ventricular fibrillation,"Answer: D: Ventricular fibrillation Explanation: None" "This is a real-world medical entrance exam question, please give the true answer based on the question and selection. Topic name: None, Subject name: Anatomy","Inferior thyroid artery ligation during thyroidectomy is done Select and explain. A. For recurrent Laryngeal nerve, B. Superior laryngeal nerve, C. Vagus nerve, D. Glossopharyngeal nerve",A: For recurrent Laryngeal nerve,"Answer: A: For recurrent Laryngeal nerve Explanation: Inferior thyroid artery: It is a branch of thyrocervical trunk from the first part of the subclavian artery. It first runs upwards along the medial border of  scalenus anterior, and then passes medially behind the carotid sheath to reach the back of the thyroid lobe, where it is intimately related to the recurrent laryngeal nerve. The recurrent laryngeal nerve presents a variable relationship with the artery. It may pass behind or in front of the loop of the artery or between the branches of the artery. The artery gives 4 or 5 branches. KEY CONCEPT: During thyroidectomy, the superior thyroid artery is ligated near the gland to save the external laryngeal nerve; and the inferior thyroid artery is ligated away from the gland to save the recurrent laryngeal nerve." "This is a real-world medical entrance exam question, please give the true answer based on the question and selection. Topic name: AIIMS 2019, Subject name: Microbiology","Pulmonary manifestation for inhalational anthrax Select and explain. A. Hemorrhagic mediastinitis, B. Lobar consolidation, C. Bronchopneumonia with type two respiratory failure, D. Can cause pneumonia",A: Hemorrhagic mediastinitis,"Answer: A: Hemorrhagic mediastinitis Explanation: Wool soer's disease or pulmonary anthrax manifests by hemorrhagic mediastinitis causing bloody fluid to accumulate in the chest cavity. It causes initially flu like symptoms fever, shoness of Breath, cough, fatigue and second stage in form of pneumonia occurs." "This is a real-world medical entrance exam question, please give the true answer based on the question and selection. Topic name: None, Subject name: Dental","The inter radicular bone is: Select and explain. A. Compact, B. Cancellous, C. Exophytic, D. Osteophytic",B: Cancellous,"Answer: B: Cancellous Explanation: None" "This is a real-world medical entrance exam question, please give the true answer based on the question and selection. Topic name: None, Subject name: Surgery","Investigation of choice for diagnosis of Zenker's diveiculum is: Select and explain. A. CECT, B. Endoscopy, C. Esophageal manometry, D. Barium swallow",D: Barium swallow,"Answer: D: Barium swallow Explanation: Ans. d. Barium swallowInvestigation of choice for diagnosis of Zenker's diveiculum is a barium swallow.It is not a true esophageal diveicula, as it rises above the upper esophageal sphincter (the cricopharyngeal sphincter)Usually seen in patients over 50 years aMC symptom is dysphagia.Undigested food is regurgitated into the mouth, especially when the patient is in the recumbent position.Swelling of the neck, gurgling noise after eating, halitosis, and a sour metallic taste in the mouth are common symptomsCervical webs are seen associated with 50% of patients with Zenker's diveiculum, which can cause dysphagia postoperatively if not treated." "This is a real-world medical entrance exam question, please give the true answer based on the question and selection. Topic name: None, Subject name: Ophthalmology","Which of the following drug should not be given in a patient of narrow angle glaucoma? Select and explain. A. Phenylephrine, B. Timolol, C. Acetazolamide, D. Homatropine",D: Homatropine,"Answer: D: Homatropine Explanation: Answer- D. HomatropineHomatropine is a mydriatic, which can precipitate angle closure glaucoma and contraindicated in angle closure glaucoma.Phenylephrine, Timolol and Acetazolamide are used for the treatment of angle closure glaucoma.'Mydriatic drugs such as atropine, Homatropine, cyclopentolate, tropicamide and phenylephrine are precipitating factorsfor angle closure glaucoma, so contraindicated in angle closure glaucoma." "This is a real-world medical entrance exam question, please give the true answer based on the question and selection. Topic name: None, Subject name: Dental","The bacteria of oral flora which plays least role in periodontitis is: Select and explain. A. Actinomyces, B. Actinomycetam comitans, C. Spirocheates, D. Bacteriodes",A: Actinomyces,"Answer: A: Actinomyces Explanation: None" "This is a real-world medical entrance exam question, please give the true answer based on the question and selection. Topic name: None, Subject name: Physiology","crossed extensor reflex is a Select and explain. A. Withdrawal reflex, B. Postural reflex, C. Sympathetic, D. Monosynaptic reflex",A: Withdrawal reflex,"Answer: A: Withdrawal reflex Explanation: None" "This is a real-world medical entrance exam question, please give the true answer based on the question and selection. Topic name: None, Subject name: Biochemistry","Cell membrane consists of: Select and explain. A. Lipids and proteins, B. Lipids only, C. Protein only, D. None of the above",A: Lipids and proteins,"Answer: A: Lipids and proteins Explanation: None" "This is a real-world medical entrance exam question, please give the true answer based on the question and selection. Topic name: None, Subject name: Pathology","A patient complains of numbness of lower lip. There is no history of tooth extraction . It could be: Select and explain. A. Infection, B. Metastatic neoplasia., C. Dental menipulation., D. CNS disease.",B: Metastatic neoplasia.,"Answer: B: Metastatic neoplasia. Explanation: None" "This is a real-world medical entrance exam question, please give the true answer based on the question and selection. Topic name: None, Subject name: Anatomy","Lymph from lower lip-middle part drains directly into: Select and explain. A. Submandibular nodes, B. Submental nodes, C. Sublingual nodes, D. Preauricular nodes",B: Submental nodes,"Answer: B: Submental nodes Explanation: None" "This is a real-world medical entrance exam question, please give the true answer based on the question and selection. Topic name: None, Subject name: Pathology","Saliva is increased by: Select and explain. A. Cholinergic drugs, B. Anticholinergic drugs, C. Adrenergic drugs, D. None of the above",A: Cholinergic drugs,"Answer: A: Cholinergic drugs Explanation: None" "This is a real-world medical entrance exam question, please give the true answer based on the question and selection. Topic name: None, Subject name: Dental","False regarding protective co-contraction muscle splinting: Select and explain. A. Reduced mouth opening due to infection, B. Protective co-contraction is not a pathologic condition, C. There is no pain reported when the muscle is at rest, but use of the muscle will typically result in increased pain., D. Altered sensory input or pain, antagonistic muscle groups fire during movement in an attempt to protect the injured part.",A: Reduced mouth opening due to infection,"Answer: A: Reduced mouth opening due to infection Explanation: Protective Co-contraction (Muscle Splinting) Protective co-contraction is the initial response of a muscle to altered sensory or proprioceptive input or injury (or threat of injury). This response has also been called protective muscle splinting or coactivation. It is not associated with reduced mouth opening due to infection." "This is a real-world medical entrance exam question, please give the true answer based on the question and selection. Topic name: None, Subject name: Anatomy","Inferior oblique muscle of the eye is responsible for all of the following, except: Select and explain. A. Downward rotation, B. Lateral rotation, C. Extortion, D. Upward rotation",A: Downward rotation,"Answer: A: Downward rotation Explanation: None" "This is a real-world medical entrance exam question, please give the true answer based on the question and selection. Topic name: None, Subject name: Pediatrics","Craniopagus is defined as fusion of: Select and explain. A. Head and spine, B. Head only, C. Thorax and spine, D. Thorax only",B: Head only,"Answer: B: Head only Explanation: Ans: B. Head onlyRef: Coran Pediatric Surgery 7u ed., pg. 1728Craniopagus:Extent of union:Cranial neuroporeSkull venous sinus &meninges l00%Cerebral coex 37%" "This is a real-world medical entrance exam question, please give the true answer based on the question and selection. Topic name: AIIMS 2019, Subject name: Forensic Medicine","11 yr female in the school brought to principle by teacher that she is always crying unattentive not taking interest in any activity. On fuher investigation the girl told that she was inappropriately touched by her uncle at private pas at her home. Principle inform to whom ? Select and explain. A. Child welfare, B. Parents, C. Police, D. Magistrate",C: Police,"Answer: C: Police Explanation: THE PROTECTION OF CHILDREN FROM SEXUAL OFFENCES (POCSO) ACT, 2012 * Repoing the offence to the police is mandatory. Failure to repo, attracts punishment with imprisonment of up to 6 months or fine or both. Section 19 of the POCSO Act, 2012 provides mandatory repo of child sexual offences to the Special Juvenile Police Unit or the local police" "This is a real-world medical entrance exam question, please give the true answer based on the question and selection. Topic name: None, Subject name: Microbiology","Mw vaccine is made from which bacteria? Select and explain. A. M. Welchii, B. M. Bovis, C. M. Indicus pranii, D. none of these",C: M. Indicus pranii,"Answer: C: M. Indicus pranii Explanation: Answer-C- M. Indicus praniiKilled Mycobacterium indicus pranii (previously known as Mycobacterium w, popularly known as Mw) vaccine has earlier been investigated in genital was with encouraging resultsMycobacterium w vaccine, a useful adjuvant to multidrug therapy in multibacillary leprosy." "This is a real-world medical entrance exam question, please give the true answer based on the question and selection. Topic name: None, Subject name: Surgery","Wound that does not involve dermis is Select and explain. A. Abrasion, B. contusion, C. Laceration, D. Incision",A: Abrasion,"Answer: A: Abrasion Explanation: None" "This is a real-world medical entrance exam question, please give the true answer based on the question and selection. Topic name: AIIMS 2017, Subject name: Microbiology","Which is not a dimorphic fungi? Select and explain. A. P. marneffi, B. Histoplasma capsulatum, C. Blastomyces dermatitidis, D. Pneumocystis jirovecii",D: Pneumocystis jirovecii,"Answer: D: Pneumocystis jirovecii Explanation: Dimorphic fungi :- They exist as molds in the environment at ambient temperature(250C) and as yeasts in human tissues at body temperature(370C). several medically impoant fungi are thermally dimorphic such as:- Histoplasma capsulatum Blastomyces dermatitidis Coccidiodes immitis Paracoccidiodes brasiliensis Penicillium marneffei Sporothrix schenckii All of the above mentioned dimorphic fungi causes systemic mycoses except sporothrix which is responsible for sub cutaneous infection." "This is a real-world medical entrance exam question, please give the true answer based on the question and selection. Topic name: None, Subject name: Dental","True about bisphosphonate mechanism of action Select and explain. A. Inhibit osteoclast mediated resorption, B. Increases rate of osteoid formation, C. Increases mineralization of osteoid, D. All of above",A: Inhibit osteoclast mediated resorption,"Answer: A: Inhibit osteoclast mediated resorption Explanation: None" "This is a real-world medical entrance exam question, please give the true answer based on the question and selection. Topic name: None, Subject name: Medicine","All of the following are true about iron deficiency anemia except: Select and explain. A. Transferrin saturation <16%, B. Detected by serum ferritin levels even in earlier states, C. Mostly presents without any symptoms with abnormal laboratory findings, D. Latent anemia is most prevalent in India",C: Mostly presents without any symptoms with abnormal laboratory findings,"Answer: C: Mostly presents without any symptoms with abnormal laboratory findings Explanation: Answer- C. Mostly presents without any symptoms with abnormal laboratory findingsIron deficiency anemia is widely prevalent in India.IDA is preceded by a stage of latent iron deficiency, where serum ferritin is reduced but hemoglobin is normal.Iron Deficiency Anemia:Decreased MCV and MCHElevated red cell distribution widthIncreased TIBCDecreased serum ironDecreased serum ferritinTransferrin saturation <10." "This is a real-world medical entrance exam question, please give the true answer based on the question and selection. Topic name: None, Subject name: Physiology","If the interstitial hydrostatic pressure is 2 mm Hg, interstitial oncotic pressure is 7 mm Hg and capillary hydrostatic pressure is 25 mm Hg. What should be the capillary oncotic pressure to allow a net filtration pressure of 3 mm Hg? Select and explain. A. 20, B. 21, C. 23, D. 27",D: 27,"Answer: D: 27 Explanation: Ans: D. 27Formula - GFR = Kf l(PGC- PT) - (7r GC-)I* Kf- Glomerular ultrafiltration coefficient. - Product of glomerular capillary wall hydraulic conductivity (i.e. its permeability) & effective filtration surface area.* PGC - Mean hydrostatic pressure in glomerular capillaries = 25* PT - Mean hydrostatic pressure in tubule (Bowman's space) = 2* GC - Oncotic pressure of plasma in glomerular capillaries* T - Interstitial colloidal oncotic pressure = 7* In this question, - GFR, 3 = Kf Kf Kf . - GC = 27 mm/Hg." "This is a real-world medical entrance exam question, please give the true answer based on the question and selection. Topic name: None, Subject name: Surgery","In BSSO setback, fixation in neutral posterior zone is best achieved with: Select and explain. A. Lag screw, B. Position screw, C. Miniplate, D. No fixation is required in neutral position",A: Lag screw,"Answer: A: Lag screw Explanation: None" "This is a real-world medical entrance exam question, please give the true answer based on the question and selection. Topic name: AIIMS 2019, Subject name: Pharmacology","A patient was given ampicillin 2 g intravenously. After that, the person developed rash on skin, hypotension and difficulty in breathing. The patient should be managed by Select and explain. A. 0.5 ml of 1:1000 adrenaline by intramuscular route, B. 0.5 ml of 1:1000 adrenaline by intravenous route, C. 0.5 ml of 1:10000 adrenaline by intramuscular route, D. 0.5 ml of 1:10000 adrenaline by intravenous route",A: 0.5 ml of 1:1000 adrenaline by intramuscular route,"Answer: A: 0.5 ml of 1:1000 adrenaline by intramuscular route Explanation: Penicillins are prone to cause allergic reactions features like rash on skin, hypotension ,difficulty in breathing are suggestive of anaphylactic shock. Drugs Frequently causing allergic reactions Penicillins Aspirin Cephalosporins Indomethacin Sulfonamides Carbamazepine Tetracyclines Allopurinol Quinolones ACE inhibitors Metronidazole Methyldopa Abacavir Hydralazine Antitubercular drugs Local anaesthetics Phenothiazines In case of anaphylactic shock the resuscitation council of UK has recommended the following measure : * Put the patient in reclining position, administer oxygen at high flow rate and perform cardiopulmonary resuscitation if required.* Inject adrenaline 0.5 mg (0.5 ml of I in I 000 solution for adult, 0.3 ml for child 6-12 years and 0.15 ml for child upto 6 years) i.m.; repeat every 5-10 min in case patient does not improve or improvement is transient. This is the only life saving measure. Adrenaline should not be injected i.v. (can itself be fatal) unless shock is immediately life threatening. If adrenaline is to be injected i. v., it should be diluted to 1:10,000 or 1:100,000 and infused slowly with constant monitoring. * Administer a H1 antihistaminic (pheniramine 20-40 mg or chlorpheniramine I 0-20 mg) i.m./slow i.v. It may have adjuvant value.* Intravenous glucocoicoid (hydrocoisone sod. succinate 200 mg) should be added in severe/recurrent cases. It acts slowly, but is specially valuable for prolonged reactions and in asthmatics. It may be followed by oral prednisolone for 3 days." "This is a real-world medical entrance exam question, please give the true answer based on the question and selection. Topic name: None, Subject name: Physiology","Interstitial fluid volume can be determined by: Select and explain. A. Radioactive iodine and radiolabelled water, B. Radioactive water and radiolabelled albumin, C. Radioactive sodium and radioactive water, D. Radioactive sodium and radioactive labelled albumin",D: Radioactive sodium and radioactive labelled albumin,"Answer: D: Radioactive sodium and radioactive labelled albumin Explanation: Ans: D. Radioactive sodium and radioactive labelled albumin(Ref Ganong 25/e p3, 24/e p2; Guvton I p309, 310)Fluid volumeIndicator usedTotal body water volumeDeuterium oxide: D20 (MC used)degTritium oxidedeg, Aminopyrinedeg, AntipyrinedegExtracellular fluid volumeInulin (most accurate)degs22Na, 1251-iothalamate, thiosulfatedegPlasma volumeEvans bluedegSerum albumin labeled with radioactive iodinedeg (1-125)Intracellular fluid(Calculated as total body water - extracellular fluid volume)Blood volume""Cr-labeled RBCsdeg, or calculated as blood volume = plasma volume/ (1 - hematocrit)Interstitial fluid(Calculated as extracellular fluid volume - plasma volume)" "This is a real-world medical entrance exam question, please give the true answer based on the question and selection. Topic name: None, Subject name: Gynaecology & Obstetrics","A 28 year old eclamptic woman develop convulsions.The first measure to be done is: Select and explain. A. Give MgSO4, B. Sedation of patient, C. Immediate delivery, D. Care of airway",D: Care of airway,"Answer: D: Care of airway Explanation: Preeclampsia when complicated with convulsion and / or coma is called eclampsia. Fits occurring in eclampsia are Generalised tonic clonic seizure. In most cases seizures are self limited, lasting for 1 to 2 minutes. Management: “The first priorities are to ensure that the airway is clear and to prevent injury and aspiration of gastric content COGDT 10/e, p 326 Initial management during eclamptic fit: Patients should be kept in an isolated room to protect from noxious stimulus which might provoke further fits. Mouth gag is placed between teeth to prevent tongue bite. Air passage is cleared off the mucus. Oxygen is given. Catheterization is done to monitor urine output. Specific management: A. Medical management i. Seizure treatment The drug of choice for the control and prevention of convulsions is magnesium sulphate (Pritchard’s regimen) Previously used anticonvulsant regimen for eclampsia was ‘Lytic cocktail regimen’ given by Menon using pethidine, chlorpromazine and phenargen. but now it is not used. ii. Treatment of hypertension DOC in eclampsia is labetalol. 2nd DOC in eclampsia is hydralazine. B. Obstetric management in antepartum cases: Immediate termination of pregnancy should be done. Vaginal delivery is preferred but “In current obstetrical practice the large majority of eclamptic women are delivered by cesarean section. The most common exception to cesarean delivery are women with a fetal demise and the rare ones with a very ripe cervix.” Fernando Arias 3/e, p 427" "This is a real-world medical entrance exam question, please give the true answer based on the question and selection. Topic name: None, Subject name: Ophthalmology","IOL placed in the young male., After 10 year what should be done..? Select and explain. A. IOL should be removed in case of Posterior Capsule ossification, B. Never be removed, C. Remove when presbyopia sets in, D. Should be changed after 10 year",A: IOL should be removed in case of Posterior Capsule ossification,"Answer: A: IOL should be removed in case of Posterior Capsule ossification Explanation: Answer A. IOL should be removed in case of Posterior Capsule ossification 1. The indications for removing an intraocular lens (IOL) are:Chronic uveitisEndothelial corneal dystrophyUncontrollalbe glaucomaMetal loop cutting pupillary sphincterGross decentration of IOL (fibrous bands)Extraocular dislocation of IOL G. Recurrent severe hyphemaDevelopment of rubeosis iridisRemoval of iris (1) Iris tumor (2) Epithelial downgrowthEndophthalmitisUnilateral IOL in pending bilateral aphakia 2. The following may be indications for removing and/or replacing and/or replacing an IOL:Wrong dioptric powerForeign body attached to IOLIOL covered with pigmentRepair retinal detachment after extracapsular cataract extractionChoyce lens too shoDannheim IOL with absorbed supramid loop tipsDislocated Ridley IOLSclero-conjunctival erosion of Strampelli's ""external-fixation"" IOL loop Reference - 1 - Indicates positive association between exposure & disease under study.RR = 2 - Indicates incidence rate of disease is 2 times higher in exposed group (compared with unexposed).Impoant in etiological enquiries." "This is a real-world medical entrance exam question, please give the true answer based on the question and selection. Topic name: AIIMS 2019, Subject name: Orthopaedics","Minimum age of screening of osteoporosis: Select and explain. A. 55 years, B. 60 years, C. 65 years, D. 50 years",C: 65 years,"Answer: C: 65 years Explanation: Osteoporosis - screened using DEXA scan staing from the age group of 65 years It measures - bone mineral density and uses a T score to identify osteoporosis. T score: 0 to -1 - normal 1 to -2.5 - osteopenia < -2.5 - osteoporosis" "This is a real-world medical entrance exam question, please give the true answer based on the question and selection. Topic name: None, Subject name: Dental","The total number of vertebrae present in the vertebral column are? Select and explain. A. 22, B. 34, C. 56, D. 52",B: 34,"Answer: B: 34 Explanation: None" "This is a real-world medical entrance exam question, please give the true answer based on the question and selection. Topic name: None, Subject name: Pathology","In a caries-free individual the saliva has: Select and explain. A. Low buffering capacity for acids, B. Medium buffering capacity for acids, C. High buffering capacity for acids, D. Independent of buffering capacity for acids",C: High buffering capacity for acids,"Answer: C: High buffering capacity for acids Explanation: None" "This is a real-world medical entrance exam question, please give the true answer based on the question and selection. Topic name: None, Subject name: Dental","Phallic stage of Freud corresponds to which of the following stages of Piaget Select and explain. A. Sensorimotor, B. Pre operational, C. Concrete operational, D. Formal operational",B: Pre operational,"Answer: B: Pre operational Explanation: None" "This is a real-world medical entrance exam question, please give the true answer based on the question and selection. Topic name: None, Subject name: Gynaecology & Obstetrics","Most common site of primary for intraocular metastasis is from- Select and explain. A. Breast, B. Ovary, C. Cervix, D. Endometrium",A: Breast,"Answer: A: Breast Explanation: Answer- A. BreastMost common site of primary for intraocular metastasis is from breast.'Breast cancer is the most common tumor to metastasize to the eye followed by lung cancer." "This is a real-world medical entrance exam question, please give the true answer based on the question and selection. Topic name: None, Subject name: Anatomy","The cortex of lymph node contains: Select and explain. A. Cords of billroth, B. Hassall's corpuscles, C. Lymphatic nodules, D. White pulp",C: Lymphatic nodules,"Answer: C: Lymphatic nodules Explanation: The cortex of lymph node contains lymphatic nodules. Lymph node consists of 3 parts, i.e.: Capsule Cortex Medulla" "This is a real-world medical entrance exam question, please give the true answer based on the question and selection. Topic name: None, Subject name: Radiology","Occluded submandibular salivary duct can be best observed by Select and explain. A. Palpation, B. Sialography, C. Occlusal X-ray, D. Thermography",B: Sialography,"Answer: B: Sialography Explanation: None" "This is a real-world medical entrance exam question, please give the true answer based on the question and selection. Topic name: None, Subject name: Surgery","In case of facial injuries with voluntary control lost over tongue, the best emergency treatment to prevent tongue from falling back is Select and explain. A. Towel clipping of Tongue, B. Deep traction silk suture of tongue, C. Oropharyngeal airway, D. Definitive treatment",A: Towel clipping of Tongue,"Answer: A: Towel clipping of Tongue Explanation: None" "This is a real-world medical entrance exam question, please give the true answer based on the question and selection. Topic name: None, Subject name: Dental","Chances of ankyloses of mandibular molar which is autotransplanted depends on? Select and explain. A. Splinting of mandibular molar, B. Surgical extraction of molar, C. Socket preparation of molar for autotransplantation, D. Root kept moist in cotton",B: Surgical extraction of molar,"Answer: B: Surgical extraction of molar Explanation: None" "This is a real-world medical entrance exam question, please give the true answer based on the question and selection. Topic name: None, Subject name: Dental","Greater crown bulk distal to the faciolingual dissecting plane of the tooth is most typical of mandibular Select and explain. A. Central incisor, B. Canine, C. Lateral incisor, D. Second premolar",B: Canine,"Answer: B: Canine Explanation: None" "This is a real-world medical entrance exam question, please give the true answer based on the question and selection. Topic name: None, Subject name: Social & Preventive Medicine","Some medicine comes with a label of 'store at a cool place only'. At what temperature should these medicines be kept? Select and explain. A. 8-15 degC, B. 2-8 degC, C. 0 degC, D. 25-28 degC",A: 8-15 degC,"Answer: A: 8-15 degC Explanation: Ans: A. 8-15 degC(Ref: Park 22/e p100)Some medicines come with label of 'store at a cool place only'.These medicines should be kept at 8-15 degC.Definitions of Storage Conditions of Drugs as per Ip 6 (Indian Pharmacopoeia 1996)ColdAny temperature not exceeding 8degC and usually between 2-8degC. A refrigerator is a cold place in which the temperature is maintained thermostatically between 28degC.CoolAny temperature between 8-25degC.An aicle, for which storage in a cool place is directed may alternately,be stored in a refrigerator unless otherwise specified in the individual monograph.Room TemperatureThe temperature prevailing in a working area.WarmAny temperature between 30-40degC.Excessive heatAny temperature above 40degC.Light resistant containersA light-resistant container protect the content from the effect of actinic light by viue of the specific propeies of the material of which it is made.Well closed containerA well-closed container protects the contents from contamination by extraneous liquid & from loss of the aicle under normal condition of handling, shipment, storage & distribution.Well closed containerA tightly closed container protects the contents from contamination by extraneous liquid & solids or vapor, from loss or deterioration of the aicle from effervescence, deliquescent or evaporation under normal condition of handling, shipment, storage & distribution." "This is a real-world medical entrance exam question, please give the true answer based on the question and selection. Topic name: None, Subject name: Pathology","All of the following statements about idiopathic thrombocytopenic purpura are true EXCEPT Select and explain. A. It is associated with platelet-specific auto-antibodies, B. It causes a prolonged bleeding time, C. It is often controllable by immunosuppressive treatment, D. It causes more prolonged hemorrhage than hemophilia",D: It causes more prolonged hemorrhage than hemophilia,"Answer: D: It causes more prolonged hemorrhage than hemophilia Explanation: None" "This is a real-world medical entrance exam question, please give the true answer based on the question and selection. Topic name: None, Subject name: Dental","In a MO cavity prepared for inlay, mechanical advantage to prevent proximal displacement of the restoration is by establishing: Select and explain. A. Adequate pulpal depth, B. Parallel walls, C. Occlusal dovetail, D. Acute lingual inclination in axiogingival area",C: Occlusal dovetail,"Answer: C: Occlusal dovetail Explanation: None" "This is a real-world medical entrance exam question, please give the true answer based on the question and selection. Topic name: None, Subject name: Dental","Major drawback of pro-root MTA is its increased setting time which is decreased by addition of ? Select and explain. A. 3% of NaOH, B. 3% NaCl, C. 5% CaCl, D. Distilled Water",C: 5% CaCl,"Answer: C: 5% CaCl Explanation: None" "This is a real-world medical entrance exam question, please give the true answer based on the question and selection. Topic name: None, Subject name: Dental","Apical constriction is ……………mm coronal to Apical foramen Select and explain. A. 0-0.5, B. 0.5-1.5, C. 1.5-2.5, D. 2-Jan",B: 0.5-1.5,"Answer: B: 0.5-1.5 Explanation: None" "This is a real-world medical entrance exam question, please give the true answer based on the question and selection. Topic name: None, Subject name: Surgery","An elderly male presents 2 months after renal trans-planatation with nephropathy. Which of the following can be a viral etiological agent? Select and explain. A. Polymoa virus BK, B. Human herpes virus type 6, C. Hepatitis C, D. Human papillorna virus, high risk types",A: Polymoa virus BK,"Answer: A: Polymoa virus BK Explanation: Answer- A. Polymoa virus BK Polyoma BK Virus:BK virus is Polyoma virus associated with nephropathy, typically after l-4 moths after transplant.High levels of BK virus replication detected by PCR in urine and blood are predictive of pathology. especially in setting of renal transplantation.Urinary excretion of BK virus and BK virusemia are associated with the development of ureteric strictures, Polyoma virus associated nephropathy." "This is a real-world medical entrance exam question, please give the true answer based on the question and selection. Topic name: AIIMS 2018, Subject name: Physiology","Erythropoietin acts on: Select and explain. A. CFU, B. Late erythtoblast, C. Normoblast, D. Burst forming unit (BFU)",D: Burst forming unit (BFU),"Answer: D: Burst forming unit (BFU) Explanation: Normal duration of erythropoiesis = 7- 9 days Under influence of erythropoietin = 5- 6 days EPO acts on earlier precursors i.e. Formation of burst forming units formation of proerythroblast It has been determined that the impoant effect of erythropoietin is to stimulate the production of proerythroblast from hematopoietic stem cells in the bone marrow. In addition, once the proerythroblasts are formed, the erythropoietin causes these cells to pass more rapidly through the different erythroblastic stages than they normally do, fuher speeding up the process Page 488, CH 33. :" "This is a real-world medical entrance exam question, please give the true answer based on the question and selection. Topic name: None, Subject name: Radiology","Expansion of the contrast filled space in myelography is seen in: Select and explain. A. Intramedullary tumor, B. Intradural extramedullary tumor, C. Spinal dysraphism, D. Extradural tumor",A: Intramedullary tumor,"Answer: A: Intramedullary tumor Explanation: Ans: A. Intramedullary tumor(Ref Sutton 6/e p254: Neurology in Clinical Practice 4/e Vol I p579)Myelography:Expansion of contrast filled space (i.e. subarachnoid space) - Any intramedullaty extradural lesion (meningioma).Sometimes filling defect causing ""meniscus sign"" demonstrable.Levels of Block in MyelographySiteTypical AppearanceExtradural blockFeathered appearanceIntradural Extramedullary blockMeniscus signWidening of ipsilateral subarachnoid spaceIntramedullary blockWidening of the cordTrouser leg appearance" "This is a real-world medical entrance exam question, please give the true answer based on the question and selection. Topic name: None, Subject name: Anatomy","Tensor veli palatini origin and insertion are Select and explain. A. Origin Greater wing of sphenoid and scaphoid fossa; Insertion as palatine aponeurosis into posterior hard palate., B. Origin Lesser wing of sphenoid and scaphoid fossa;Insertion Hamular notch and opposite muscle of same name, C. Origin Lesser wing of sphenoid and scaphoid fossa; Insertion- Posterior hard palate, D. Origin Lesser wing of sphenoid and scaphoid fossa; Insertion: as palatine aponeurosis into posterior hard palate.",A: Origin Greater wing of sphenoid and scaphoid fossa; Insertion as palatine aponeurosis into posterior hard palate.,"Answer: A: Origin Greater wing of sphenoid and scaphoid fossa; Insertion as palatine aponeurosis into posterior hard palate. Explanation: None" "This is a real-world medical entrance exam question, please give the true answer based on the question and selection. Topic name: None, Subject name: Dental","C-factor is associated with? Select and explain. A. Condensing force for amalgam, B. Cavity preparation factor for amalgam, C. Root curvature, D. Polymerisation shrinkage in composite",D: Polymerisation shrinkage in composite,"Answer: D: Polymerisation shrinkage in composite Explanation: None" "This is a real-world medical entrance exam question, please give the true answer based on the question and selection. Topic name: None, Subject name: Medicine","In a patient of rheumatic heart disease with fever and mitral valve vegetations due to infective endocarditis, the vegetation usually do no embolise to: Select and explain. A. Brain, B. Lungs, C. Liver, D. Spleen.",B: Lungs,"Answer: B: Lungs Explanation: None" "This is a real-world medical entrance exam question, please give the true answer based on the question and selection. Topic name: None, Subject name: Dental","Stainless steel orthodontic wire can be hardened by: (OR) Bending orthodontic wire at room temperature is an example of Select and explain. A. Tempering, B. Work/ Strain hardening, C. Age hardening, D. Precipitation hardening",B: Work/ Strain hardening,"Answer: B: Work/ Strain hardening Explanation: None" "This is a real-world medical entrance exam question, please give the true answer based on the question and selection. Topic name: None, Subject name: Gynaecology & Obstetrics","A lady delivered a normal vaginal delivery and was discharged. On third day she came back with fever, tachycardia and seizures. Fundus showed papilledema with no focal deficits. What is the most likely diagnosis? Select and explain. A. Coical vein thrombosis, B. Meningitis, C. Subarachnoid hemorrhage, D. Acute migraine",A: Coical vein thrombosis,"Answer: A: Coical vein thrombosis Explanation: Answer- A. Coical vein thrombosisCoical Vein Thrombosis (CVT)Incidence of CVT is increased during pregnancy & in puerperiumIncidence appears higher in developing countries.Predisposing Factors:Prothrombotic conditions, OCPs, pregnancy, puerperiumMalignancy, infection & head injuryClinical Features:MC presenting symptom: Headaches (of gradual, acute, or thunderclap onset)Associated features may include focal neurological sings, seizures and coma.Diagnosis:Diagnosis is done with MR venography.Treatment:Anticonvulsants for seizures; heparinization is recommended by most, its efflcacy is controversial.Antimicrobials for septic thrombophlebitisFibrinolytic therapy is reserved for those women failing systemic anticoagulation" "This is a real-world medical entrance exam question, please give the true answer based on the question and selection. Topic name: AIIMS 2018, Subject name: Pathology","Infeility in kaagener syndrome is due to which of the following? Select and explain. A. Oligospermia, B. Blockage of epididymis, C. Asthenozoospermia, D. Undescended testis",C: Asthenozoospermia,"Answer: C: Asthenozoospermia Explanation: Kaagener's syndrome is a rare, autosomal recessive genetic ciliary disorder in which defective dynein arm results in defective ciliary motility. Presents with a triad of : Situs inversus Bronchiectasis Sinusitis Reduced ciliary motility leads to asthenozoospermia( slow moving sperms)" "This is a real-world medical entrance exam question, please give the true answer based on the question and selection. Topic name: None, Subject name: Dental","Regeneration leads to formation of which layer of cementum: Select and explain. A. Cellular mixed cementum, B. Cellular intrinsic fibre cementum, C. Acellular extrinsic fiber cementum, D. Acellular afibrillar cementum",C: Acellular extrinsic fiber cementum,"Answer: C: Acellular extrinsic fiber cementum Explanation: None" "This is a real-world medical entrance exam question, please give the true answer based on the question and selection. Topic name: None, Subject name: Pharmacology","Mention the true/false statements about digoxin toxicity? Select and explain. A. The earliest manifestation of digoxin toxicity are gastrointestinal symptoms, B. Non-specific vision changes may be noted in digoxin toxicity, C. Early toxicity may not correlate with serum levels and Neurological symptoms may occur without corresponding cardiovascular changes, D. All",D: All,"Answer: D: All Explanation: Ans. All are true about digoxin toxicity.DIGOXIN TOXICITY:Features:Generally unwell & lethargy.Nausea & vomiting.Confusion.Yellow-green vision.Arrhythmias (e.g. AV block, bradycardia)Dizziness.Precipitating factors:Renal disease HypokalaemiaHypomagnesemia HypoalbuminemiaHypothermiaHypothyroidismHypercalcemia.HypernatremiaAcidosis.Myocardial ischemia.Paial AV block.Drugs:Amiodarone.Quinidine.Verapamil.Spironolactone.Furosemide.Hydrochlorothiazide - Compete with DCT secretion, hence reducing excretion.ManagementDigibind.Correct ventricular arrhythmia by lignocaine.Bradyarrhythmias by propanolol.Atrial tachyarrhythmias by atropine.Phenytoin.Monitor K+" "This is a real-world medical entrance exam question, please give the true answer based on the question and selection. Topic name: None, Subject name: Anatomy","All of the following bones are the pas of inferior wall of orbit except: Select and explain. A. Ethmoid, B. Maxillary, C. Palatine, D. ZygomaticWhile",A: Ethmoid,"Answer: A: Ethmoid Explanation: Answer- A (Ethmoid)Kanski 5/e p558'The inferior wall of orbit consists of three bones: zygomatic, maxillary, and palatinea'.Boundaries of the OrbitRoofLesser wing of the sphenoidOrbital plate of the plateLateral wallGreater wing of the sphenoidZygomaticInferior wallZygomaticMaxillaryPalatineMedial wallFrontal process of the maxillaLacrimalEthmoidBoby of sphenoid" "This is a real-world medical entrance exam question, please give the true answer based on the question and selection. Topic name: None, Subject name: Dental","Not responsible for chronic gingival inflammation? Select and explain. A. Neutrophil, B. Macrophages, C. T-lymphocytes, D. Plasma cells",A: Neutrophil,"Answer: A: Neutrophil Explanation: None" "This is a real-world medical entrance exam question, please give the true answer based on the question and selection. Topic name: None, Subject name: Dental","Which cells are decreased in chronic periodontitis? Select and explain. A. B-lymphocytes, B. T-lymphocytes, C. Mast cells, D. Plasma cells",B: T-lymphocytes,"Answer: B: T-lymphocytes Explanation: None" "This is a real-world medical entrance exam question, please give the true answer based on the question and selection. Topic name: None, Subject name: Dental","For calculation of sample size for a prevalence study all of the following are necessary except Select and explain. A. Prevalence of disease in population, B. Power of the study, C. Significance level, D. Desired precision",D: Desired precision,"Answer: D: Desired precision Explanation: Sample size depends upon: The effect size (usually the difference between 2 groups). The population standard deviation (for continuous data). The desired power of the experiment to detect the postulated effect (Power = 1 – b) The significance level." "This is a real-world medical entrance exam question, please give the true answer based on the question and selection. Topic name: None, Subject name: Medicine","All are features of benign intracranial hypeension except: Select and explain. A. Proptosis, B. Normal size ventricles, C. Headache, D. Papilledema",A: Proptosis,"Answer: A: Proptosis Explanation: Ans: A. ProptosisProptosis is not seen in benign intracranial hypeension.Characteristic features of Pseudotumor Cerebri:Elevated intracranial pressure (intracranial hypeension) with normal or small sized ventricular system.No focal neurological sign.Papilledema (enlarged blind spot in visual fluid).Normal CSF findingNormal CT scanMRI & isotope brain scansExcessive slow-wave activity on ECO2." "This is a real-world medical entrance exam question, please give the true answer based on the question and selection. Topic name: None, Subject name: Ophthalmology","The most common mode of spread of retinoblastoma: Select and explain. A. Optic nerve invasion, B. Lymphatics, C. Vascular, D. Direct invasion",A: Optic nerve invasion,"Answer: A: Optic nerve invasion Explanation: Ans: A. Optic nerve invasion(Ref Kanski 7/e p510-517; Yanoff and Duker 4/e p793)Most common mode of retinoblastoma spread-By optic nerve invasion.Direct extension by continuity to optic nerve & brain seen.Retinoblastoma:Primary malignant intraocular neoplasm.Arising from immature retinoblasts within developing retina.Most common primary intraocular malignancy of childhood in all-racial groups.Strong tendencies invading brain optic nerve & metastasize widely." "This is a real-world medical entrance exam question, please give the true answer based on the question and selection. Topic name: None, Subject name: Pharmacology","The organ most resistant to GA is Select and explain. A. Spinal Cord, B. Medulla oblonagata, C. Medullary cortex, D. Cerebrum",B: Medulla oblonagata,"Answer: B: Medulla oblonagata Explanation: None" "This is a real-world medical entrance exam question, please give the true answer based on the question and selection. Topic name: None, Subject name: Medicine","Continuous murmur can be heard in all except: Select and explain. A. VSD with aoic regurgitation, B. Coronary AV fistula, C. Pulmonary AV fistula, D. Patent ductus aeriosus",A: VSD with aoic regurgitation,"Answer: A: VSD with aoic regurgitation Explanation: Ans: A. VSD with aoic regurgitation(Ref Harrison 19/e p51e-2, 1449, 18/e p 1829)VSD with aoic regurgitation:Produces pansystolic & early diastolic murmur, not continuous.Continuous murmur:Classic example - PDA.Usually heard in second or third interspace.At a slight distance from sternal border.Other causes:Causes of Continuous MurmursContinuous murmur caused by blood flowContinuous murmurs caused by high-to-low pressure shuntsContinuous murmurs secondary to localized aerial obstructionVenous humMammary souffleHemangiomaHypehyroidismAcute alcoholichepatitisHyperemia of neoplasmHepatomaRCCPaget's diseaseSystemic aery to pulmonary aeryPatient ductus aeriosusdegAoopulmonary windowTruncus aeriosusdegPulmonary atresiadegAnomalous left coronary aerySequestration of the lungSystemic aery to right heaRuptured sinus of ValsalvadegAV fistula (systemic or pulmonic)degCoronary AV fistulaLeft-to-right atrial shuntingLutembacher syndromeMitral atresia plus atrialseptal defectVenovenous shuntsAnomalous pulmonaryveinsPoosystemic shuntsBronchiectasisCoarctation of aoaBranch pulmonary aerystenosisCarotid aery occlusionCeliac mesentericocclusionRenal aery occlusionFemoral aery occlusionCoronary aeryocclusion" "This is a real-world medical entrance exam question, please give the true answer based on the question and selection. Topic name: None, Subject name: Dental","Impedence based apex locator is: Select and explain. A. Generation 1, B. Generation 2, C. Generation 3, D. Generation 4",B: Generation 2,"Answer: B: Generation 2 Explanation: Second Generation Apex Locator (Impedance-based Apex Locator) / Low Frequency Apex Locator Inoue introduced the concept of impedance-based apex locator which measures opposition to flow of alternating current or impedance. This apex locator indicates the apex when two impedance values approach each other." "This is a real-world medical entrance exam question, please give the true answer based on the question and selection. Topic name: None, Subject name: Dental","Mechanism of adhesion of GIC restoration with tooth surface is by means of? Select and explain. A. Carboxyl group, B. Chelates with Metal Ions, C. C=C double bond, D. Polymer Chains",A: Carboxyl group,"Answer: A: Carboxyl group Explanation: None" "This is a real-world medical entrance exam question, please give the true answer based on the question and selection. Topic name: AIIMS 2017, Subject name: Surgery","A 10 year old child came to the OPD with pain and mass in right lumbar region with no fever, with right hip flexed. The pain increased on extension and X ray showed spine changes. Most probable diagnosis is: Select and explain. A. Psoas abscess, B. Pyonephrosis, C. Appendicular lump in retrocecal position, D. Torsion of Right undescended testis",A: Psoas abscess,"Answer: A: Psoas abscess Explanation: From the given history: No fever Mass in right lumbar region Spine changes - Pyonephrosis, Appendicular lump in retrocecal position, Torsion of Right undescended testis can be ruled out as these conditions wont present with these clinical features Psoas abscess: Psoas Abscesses: The psoas muscle is another location in which abscesses are encountered. Psoas abscesses may arise from a hematogenous source, by contiguous spread from an intra- abdominal or pelvic process, or by contiguous spread from nearby bony structures (e.g., veebral bodies). Associated osteomyelitis due to spread from bone to muscle or from muscle to bone is common in psoas abscesses. When pott's disease was common, Mycobacterium tuberculosis was a frequent cause of psoas abscess. Currently, either S. aureus or a mixture of enteric organisms including aerobic and anaerobic gram-negative bacilli is usually isolated from psoas abscesses in the United States. S. aureus is most likely to be isolated when a psoas abscess arises from hematogenous spread or a contiguous focus of osteomyelitis; a mixed enteric flora is the most likely etiology when the abscess has an intra- abdominal or pelvic source. Patients with psoas abscesses frequently present with fever, lower abdominal or back pain, or pain referred to the hip or knee. CT is the most useful diagnostic technique.""- Harrison 19/e p852 ""Spinal TB (Pott's disease or tuberculous spondylitis) often involves two or more adjacent veebral bodies. Whereas the upper thoracic spine is the most common site of spinal TB in children, the lower thoracic and upper lumbar veebrae are usually affected in adults. From the anterior superior or inferior angle of the veebral body, the lesion slowly reaches the adjacent body, later affecting the interveebral disk. With advanced disease, collapse of veebral bodies results in kyphosis (gibbus). A paraveebral ""cold"" abscess may also form. In the upper spine, this abscess may track to and penetrate the chest wall, presenting as a soft tissue mass; in the lower spine, it may reach the inguinal ligaments or present as a psoas abscess. ""Harrison 19/e p1110" "This is a real-world medical entrance exam question, please give the true answer based on the question and selection. Topic name: None, Subject name: Pharmacology","The process by which the amount of a drug in the body decreases after administration, but before entering the systemic circulation is called: Select and explain. A. Excretion, B. First pass effect, C. First order elimination, D. Metabolism",B: First pass effect,"Answer: B: First pass effect Explanation: Reduction in the amount of drug before it enters the systemic circulation is called first pass metabolism (also known as first pass effect) whereas, if the amount of drug decreases after entry into the systemic circulation, it is called elimination. Latter includes excretion and metabolism." "This is a real-world medical entrance exam question, please give the true answer based on the question and selection. Topic name: None, Subject name: Dental","Eugenol may be replaced in the zinc oxide eugenol cement by: Select and explain. A. Acetic acid, B. Alginic acid, C. Phosphoric acid, D. Ortho-ethoxy acid",D: Ortho-ethoxy acid,"Answer: D: Ortho-ethoxy acid Explanation: An important improvement of ZOE cements was the development of materials in which the liquid is a mixture of 2-ethoxybenzoic acid (EBA) and eugenol, roughly in a 2:1 proportion. Rather than forming a stronger matrix, the addition of EBA allows for the use of very high powder-to-liquid ratios (6:1) which, per se, increases the strength of the set cement. In these materials, alumina (30%) was added to the powder as a reinforcing agent. Ref: Craig's restorative dentistry ed 13 pg 284" "This is a real-world medical entrance exam question, please give the true answer based on the question and selection. Topic name: None, Subject name: Dental","Cross-linking in resins helps increasing: Select and explain. A. Strength, B. Hardness, C. Crazing resistance, D. All of the above",D: All of the above,"Answer: D: All of the above Explanation: The resin matrix in most dental composites is based on a blend of aromatic and/or aliphatic dimethacrylate monomers such as bis-GMA and urethane dimethacrylate (UDMA), to form highly cross-linked, strong, rigid, and durable polymer structures. Anusavice, Shen, Rawls. Phillip’s Science of Dental Materials. Edition 12. Page:​295" "This is a real-world medical entrance exam question, please give the true answer based on the question and selection. Topic name: None, Subject name: Biochemistry","Which of the following leads to an increase in enzyme activity - Select and explain. A. Increase in temperature, B. Decrease in activation energy, C. Extremes of pH value, D. Low substrate concentration",B: Decrease in activation energy,"Answer: B: Decrease in activation energy Explanation: Answer-B. -Decrease in activation energyThe enzymes speed up chemical reactions by lowering the magnitude of the activation energy banier, i.e.,free energy of Activation" "This is a real-world medical entrance exam question, please give the true answer based on the question and selection. Topic name: None, Subject name: Physiology","Nutrients are mainly absorbed in: Select and explain. A. Small intestine, B. Large intestine, C. Liver, D. Stomach",A: Small intestine,"Answer: A: Small intestine Explanation: Proteins, carbohydrates, fats and other nutritive substances like vitamins, and minerals are absorbed mostly in small intestine. In the small intestine, secondary active transport of Na+ is important in bringing about absorption of glucose, some amino acids, and other substances such as bile acids. Reference: GUYTON AND HALL TEXTBOOK OF MEDICAL PHYSIOLOGY, THIRTEENTH EDITION(INTERNATIONAL EDITION ) page no 464" "This is a real-world medical entrance exam question, please give the true answer based on the question and selection. Topic name: None, Subject name: Pathology","Calcification of falx cerebrl is seen in: Select and explain. A. Sturge Weber syndrome, B. Gorlin goltz syndrome, C. Mobius syndrome, D. Caffey silverman syndrome",B: Gorlin goltz syndrome,"Answer: B: Gorlin goltz syndrome Explanation: None" "This is a real-world medical entrance exam question, please give the true answer based on the question and selection. Topic name: None, Subject name: Dental","70-74.9% cephalic index indicates: Select and explain. A. Hyperdolichocephalic, B. Dolichocephalic, C. Mesocephalic, D. Brachycephalic",B: Dolichocephalic,"Answer: B: Dolichocephalic Explanation: None" "This is a real-world medical entrance exam question, please give the true answer based on the question and selection. Topic name: None, Subject name: Dental","Most commonly used tooth numbering system in books is Select and explain. A. Universal system, B. Zigmondy and Palmar, C. FDI, D. Grid System",A: Universal system,"Answer: A: Universal system Explanation: In clinical practice MC is Zigmondy and palmar system(Grid system)." "This is a real-world medical entrance exam question, please give the true answer based on the question and selection. Topic name: None, Subject name: Dental","A child who has an avulsed tooth 1/2 hour back should carry the tooth to the dentist in: Select and explain. A. Cold milk, B. Normal saline, C. Wrapped in a wet handkerchief, D. Buccal vestibule",A: Cold milk,"Answer: A: Cold milk Explanation: None" "This is a real-world medical entrance exam question, please give the true answer based on the question and selection. Topic name: None, Subject name: Pathology","The term poikilokaryosis refers to Select and explain. A. Alteration of nuclear cytoplasmic ratio, B. Division of nuclei without division of cytoplasm, C. Large, prominent nuclei, D. Loss of polarity and disorientation of cells",B: Division of nuclei without division of cytoplasm,"Answer: B: Division of nuclei without division of cytoplasm Explanation: None" "This is a real-world medical entrance exam question, please give the true answer based on the question and selection. Topic name: None, Subject name: Pathology","Example of Dystrophic calcification- Select and explain. A. Hyperparathyroidism, B. Sarcoidosis, C. Hypervitaminosis D, D. Myositis ossificans",D: Myositis ossificans,"Answer: D: Myositis ossificans Explanation: Answer- D. Myositis ossificansMyositis Ossificans (MO) is an unusual pathological entity still largely unknown, characterized by dystrophic calcification leading to heterotopic ossification of intramuscular connective tissue.DYSTROPHIC CALCIFICATION may occur due to 2 types of causes:Calcification in dead tissueCalcification of degenerated tissue.Calcification in dead tissue:Caseous necrosis in tuberculosisLiquefaction necrosis in chronic abscessesFat necrosis following acute pancreatitis or traumatic fatnecrosis in the breast results in the deposition of calcium soaps.Gamma-Gandy bodies in chronic venous congestion (CVC) of the spleen is characterized by calcific deposits admixed with haemosiderin on fibrous tissue.Infarcts may sometimes undergo dystrophic calcification.Thrombi, especially in the veins, may produce phleboliths.Haematomas in the vicinity of bones may undergo dystrophic calcification.Dead parasites like in hydatid cyst, Schistosoma eggs, and cysticercosis are some of the examples showing dystrophic calcification.Calcification in breast cancer detected by mammography.Congenital toxoplasmosis involving the central nervous system visualized by calcification in the infant's brain.Calcification in degenerated tissues1. Dense old scars may undergo hyaline degeneration and subsequent calcification.2. Atheromas in the aoa and coronaries frequently undergo calcification.3. Mo?nckeberg's sclerosis shows calcification in the tunica media of muscular aeries in elderly people.4. The stroma of tumors such as uterine fibroids, breast cancer, thyroid adenoma, goiter, etc shows calcification.5. Some tumors show characteristic spherules of calcification called psammoma bodies or calcospherites such as in meningioma, papillary serous cystadenocarcinoma of the ovary and papillary carcinoma of the thyroid.6. Cysts that have been present for a long time may show calcification of their walls e.g. epidermal and pilar cysts.7. Calcinosis cutis is a condition of unknown cause in which there are irregular nodular deposits of calcium salts in the skin and subcutaneous tissue.8. Senile degenerative changes may be accompanied by dystrophic calcification such as in coastal cailages, tracheal or bronchial cailages, and pineal glands in the brain, etc." "This is a real-world medical entrance exam question, please give the true answer based on the question and selection. Topic name: None, Subject name: Dental","In National Water Supply and Sanitation programme a problem village is defined as all except? Select and explain. A. Distance of safe water is greater than 1.6 Km, B. Water is exposed to the risk of cholera, C. Water source has excess iron and heavy metals, D. Water infested with Guniea worm",D: Water infested with Guniea worm,"Answer: D: Water infested with Guniea worm Explanation: None" "This is a real-world medical entrance exam question, please give the true answer based on the question and selection. Topic name: None, Subject name: Pathology","Autoinnoculation is seen with? Select and explain. A. Ameloblastoma, B. Papilloma, C. Benign pemphigoid, D. Leukoplakia",B: Papilloma,"Answer: B: Papilloma Explanation: None" "This is a real-world medical entrance exam question, please give the true answer based on the question and selection. Topic name: None, Subject name: Dental","The following condition do not create gingival defects necessitating gingivoplasty except Select and explain. A. Acute necrotizing ulcerative gingivitis, B. Desquamative gingivitis, C. Erosive lichen planus, D. Acute herpetic gingivostomatitis",A: Acute necrotizing ulcerative gingivitis,"Answer: A: Acute necrotizing ulcerative gingivitis Explanation: None" "This is a real-world medical entrance exam question, please give the true answer based on the question and selection. Topic name: None, Subject name: ENT","female patient came with right side hearing loss, better heard in a noisy environment,Audiogram shown with about 30-40 dB gap between AC-BC of right & left ear. Rinne's test negative, Weber's test centralised. Which of the following condition shown? Select and explain. A. Meniere's disease, B. stapedial otosclerosis, C. Presbycusis, D. Vestibular schwannoma",B: stapedial otosclerosis,"Answer: B: stapedial otosclerosis Explanation: Answer-B- stapedial otosclerosisIn otosclerosis the normal dense endochondral layer of the bony otic capsule gets replaced by irregularly laid spongy bone. Most common site is fissula ante fenestram (anterior tothe oval window)Age group affected is 20-45 years (maximum between 20-30 years). Male:female ratio is 1:2.meniere's disease i.e Endolymphatic hydrops leads to SNHL and not conductive hearing loss.Tuning ForkTests in Otosclerosis: Rinnes test-negative ,Weber's test-lateralized to ear with greater conductive loss." "This is a real-world medical entrance exam question, please give the true answer based on the question and selection. Topic name: None, Subject name: Medicine","A middle-aged man comes after a road traffic accident with bleeding from the scalp. He is unconscious. A card in his pocket reveals that he is a known diabetic on Tab Glimepiride + Metformin 2 tablets twice daily. What should the next step in management? Select and explain. A. Send blood for tests, sta IV glucose and send to CT, B. Sta normal saline and send to CT, C. Dextrose solution, CT scan, D. Airway, CT scan. Blood sugar if <70 sta dextrose","D: Airway, CT scan. Blood sugar if <70 sta dextrose","Answer: D: Airway, CT scan. Blood sugar if <70 sta dextrose Explanation: Ans: D.Airway, CT scan. Blood sugar if <70 sta dextrose (Ref Harrison 19/e p2435, 18/e p3008; ATLS 9/e 2013; 72(5): p1363-1366)First testing of blood sugar necessary before glucose administration for dehydration.To rule out diabetic ketoacidotic coma rather than hypoglycemia.Non-contrast head CT (NCCT):Advisable in emergency situations.To avoid cerebral edema due to glucose administration." "This is a real-world medical entrance exam question, please give the true answer based on the question and selection. Topic name: None, Subject name: Dental","Which of the following is a method of achieving slow separation? Select and explain. A. Seperating rubber bands, B. Rubber Dam Sheet, C. Ligature wire, D. All of the following",D: All of the following,"Answer: D: All of the following Explanation: Methods of achieving slow separation 1. Separating rubber rings or bands 2. Rubber dam sheet 3. Ligature wire or copper wire 4. Gutta percha sticks 5. Oversized temporary crowns 6. Fixed orthodontic appliance" "This is a real-world medical entrance exam question, please give the true answer based on the question and selection. Topic name: None, Subject name: Dental","In Caries Assessment Test (CAT), which condition doesn't come in highest risk category? Select and explain. A. Isolated with spot lesion on tooth, B. Enamel hypoplasia, C. Child with special care need, D. Visible plaque on anterior teeth",A: Isolated with spot lesion on tooth,"Answer: A: Isolated with spot lesion on tooth Explanation: None" "This is a real-world medical entrance exam question, please give the true answer based on the question and selection. Topic name: None, Subject name: Dental","High angle case is: Select and explain. A. FMIA > 65, B. IMP > 100, C. FMP > 35°, D. SNA >4°",C: FMP > 35°,"Answer: C: FMP > 35° Explanation: None" "This is a real-world medical entrance exam question, please give the true answer based on the question and selection. Topic name: None, Subject name: Biochemistry","Mother to children's transmission is a key feature of which pattern of inheritance? Select and explain. A. Codominance, B. Autosomal dominant inheritance, C. Recessive inheritance, D. Mitochondrial inheritance",D: Mitochondrial inheritance,"Answer: D: Mitochondrial inheritance Explanation: Ans-d-Mitocondrial inheritance , it is the only non-chromosomal DNA in human cells.Mitochondria! DNA, is always maternally inherited.Mitochondrial and nuclear DNA are located in different places in the cell. During feilization, the sperm and egg cell nuclei fuse to form an embryo.The egg cell is very large compared to the sperm, so although the cells nuclei fuse, the rest of the cell mass in the embryo comes from the egg only.Nuclear DNA is therefore co-inherited but the mitochondrial DNA, which is located outside of the nucleus, is always maternally inherited because all mitochondria in a foetus and later adult are derived from the mitochondria in the mother 's egg.So, in diseases showing mitochondria! inheritance all children from affected mother will inherit the disease but it will not be transmitted from an affected father to his children." "This is a real-world medical entrance exam question, please give the true answer based on the question and selection. Topic name: None, Subject name: Medicine","Sublingual tablet or sublingual GTN metered dose aerosol usually relieve angina in Select and explain. A. 15 sec, B. 2-3 min, C. 10-12 min, D. 30 min",B: 2-3 min,"Answer: B: 2-3 min Explanation: None" "This is a real-world medical entrance exam question, please give the true answer based on the question and selection. Topic name: AIIMS 2019, Subject name: Microbiology","Gene not involved in SCID Select and explain. A. BTK, B. ZAP70, C. IL2RG, D. JAK3",A: BTK,"Answer: A: BTK Explanation: BTK gene is involved in Buon's disease or X linked agammaglobulinemia. Most cases of X linked Severe combined immunodeficiency (SCID) are due to mutations in IL2RG gene. Other mutations are defective enzyme production in Adenosine deaminase and mutations in ZAP -70 which causes defective T cell signalling which causes combined immunodeficiency and mutation occurs in JAK-3." "This is a real-world medical entrance exam question, please give the true answer based on the question and selection. Topic name: None, Subject name: Pediatrics","A 6–month–old infant presents to the 'diarrhoea clinic' unit with some dehydration. The most likely organism causing diarrhea is – Select and explain. A. Entamoeba histolytica, B. Rotavirus, C. Giardia lamblia, D. Shigella",B: Rotavirus,"Answer: B: Rotavirus Explanation: Ghai states - ""In India Rotavirus and enterotoxogenic E.coli account for nearly half of the total diarrhoea) episodes among children""." "This is a real-world medical entrance exam question, please give the true answer based on the question and selection. Topic name: None, Subject name: Dental","Lip and mentalis muscle contraction are not seen in: Select and explain. A. Mature swallow, B. Simple tongue thrust, C. Complex tongue thrust, D. All of the above",A: Mature swallow,"Answer: A: Mature swallow Explanation: None" "This is a real-world medical entrance exam question, please give the true answer based on the question and selection. Topic name: None, Subject name: Social & Preventive Medicine","ABC and VED analysis at PHC are done for: Select and explain. A. Drug inventory, B. Staff management, C. Vaccination coverage, D. National Programs implementation.",A: Drug inventory,"Answer: A: Drug inventory Explanation: Ans: A. Drug inventoryABC and VED analysis at PHC are done for drug inventory.Inventory control is a scientific system which indicates as to what to order, when to order, and how much to order, and how much to stock so that purchasing costs and storing costs are kept as low as possible.It helps to protect against the fluctuation in supply and demand, unceainty and minimise waiting time.There are various methods involved for inventory control but two are commonly used: Always, better and control (ABC) and vital, essential and desirable (VED)." "This is a real-world medical entrance exam question, please give the true answer based on the question and selection. Topic name: None, Subject name: Forensic Medicine","Which of the following is not true regarding teeth features and ethnicity? Select and explain. A. Upper third molar is most commonly absent in Mongolians, B. Carabelli cusps are seen in Caucasians, C. Negros have wide molar cusps and deep, shovel shaped cusps in incisors, D. Prominent lingual ridge and labial ridge is seen in Mongolians","C: Negros have wide molar cusps and deep, shovel shaped cusps in incisors","Answer: C: Negros have wide molar cusps and deep, shovel shaped cusps in incisors Explanation: Answer- C. Negros have wide molar cusps and deep, shovel-shaped cusps in incisors The presence of shovel-shaped incision is a feature of Mongolians, and it is uncommon in Negros.A congenital lack of the third upper molar is most common in Mongolians but can occur in any race. Small nodules on the lingual surface of maxillary molars, called 'Carabelli's cusp', are most common in Caucasian races and rare in the other major racial groups. Negroid races tend to have large teeth and often have more cusps on their molars, even up to eight, with two lingual cusps on the mandibular first premolars as an additional common finding." "This is a real-world medical entrance exam question, please give the true answer based on the question and selection. Topic name: None, Subject name: Dental","Continues use of vibrator can leads to Select and explain. A. Random nodule formation, B. Surface roughness, C. Surface irregularities, D. Discoloration",B: Surface roughness,"Answer: B: Surface roughness Explanation: None" "This is a real-world medical entrance exam question, please give the true answer based on the question and selection. Topic name: None, Subject name: Dental","Most reliable method of recording centric relation Select and explain. A. Graphic tracing with excursive movements, B. Wax cone method, C. Trial and error method, D. None of the above",A: Graphic tracing with excursive movements,"Answer: A: Graphic tracing with excursive movements Explanation: None" "This is a real-world medical entrance exam question, please give the true answer based on the question and selection. Topic name: None, Subject name: Dental","Most common chances of pulpal exposure will be there if pulpal floor is made perpendicular to the long axis of tooth: Select and explain. A. Maxillary 1st premolar, B. Maxillary 1st molar, C. Mandibular 1st premolar, D. Mandibular 2nd premolar",C: Mandibular 1st premolar,"Answer: C: Mandibular 1st premolar Explanation: None" "This is a real-world medical entrance exam question, please give the true answer based on the question and selection. Topic name: AIIMS 2017, Subject name: Microbiology","Diagnosis of CI. Difficile infection is made by which of the following methods? Select and explain. A. Toxin gene detection by polymerase chain reaction (PCR), B. Culture, C. Enzyme - linked immunosorbent assay (ELISA), D. Nagler's reaction",A: Toxin gene detection by polymerase chain reaction (PCR),"Answer: A: Toxin gene detection by polymerase chain reaction (PCR) Explanation: Clostridium difficile is difficult to culture. The best method for its diagnosis is PCR for tox gene. Nagler's reaction is done for Cl. Perfringens. It is a neutralizaton reaction." "This is a real-world medical entrance exam question, please give the true answer based on the question and selection. Topic name: None, Subject name: Anatomy","A 37-year-old patient presented to you with hyper-extension of 4th and 5th metacarpophalangeal joint with flexion at proximal interphalangeal joint. This deformity is due to injury to: Select and explain. A. Deep branch of ulnar nerve, B. Median nerve, C. Radial nerve, D. Superficial branch of median nerve",A: Deep branch of ulnar nerve,"Answer: A: Deep branch of ulnar nerve Explanation: Ans: A. Deep branch of ulnar nerve(Ref: Gray's 41/C p784, 866, 40/e p888)Deformity = Claw hand.Due to injury of deep branch of ulnar nerve." "This is a real-world medical entrance exam question, please give the true answer based on the question and selection. Topic name: None, Subject name: Dental","Myofunctional appliance is given at which stage: Select and explain. A. Primary dentition, B. Mixed dentition, C. Permanent dentition, D. Adulthood",B: Mixed dentition,"Answer: B: Mixed dentition Explanation: The term myofunctional therapy was proposed by lischer. Best period for myofunctional therapy is the late mixed dentition period." "This is a real-world medical entrance exam question, please give the true answer based on the question and selection. Topic name: None, Subject name: Dental","Not true about NRHM Select and explain. A. Appoint ASHA, B. Strengthen institutional delivery under JSY, C. Health and family welfare societies, D. State and district health mission",C: Health and family welfare societies,"Answer: C: Health and family welfare societies Explanation: None" "This is a real-world medical entrance exam question, please give the true answer based on the question and selection. Topic name: None, Subject name: Dental","Model plaster (white) used to cast study models before mixing with water, is largely composed of: Select and explain. A. CaO, B. CaCO3, C. CaSO4  • ½H2O, D. CaSO4 • 2H2O",C: CaSO4  • ½H2O,"Answer: C: CaSO4  • ½H2O Explanation: When plaster is mixed with water, it takes up one and a half molecules of water, i.e, it regains its water of crystallization and becomes calcium sulphate dihydrate. (CaSO4)2.H2O + 3H2O → 2CaSO4.2H2O+ unreacted (CaSO4)2.½H2O + Heat Hemihydrate + Water → Dihydrate + Unreacted hemihydrate + Heat The reaction is exothermic and is the same for all gypsum products. The amount of water required to produce a workable mix varies between the products. As evident from the above reaction, not all of the hemihydrate converts to dihydrate. The amount of conversion is dependent on the type of stone. The highest conversion rate is seen in plaster (90%). In Type 4 and 5 stone, the dihydrate content is about 50%.  Key Concept This corresponds to the first step in the reaction. The principal constituent of gypsum-based products is calcium sulfate hemihydrate . Phillips Science of Dental Materials ed 12 pg183" "This is a real-world medical entrance exam question, please give the true answer based on the question and selection. Topic name: None, Subject name: Dental","Which of the following nonaqueous elastomeric impression materials has the least shelf life Select and explain. A. Poly ether, B. Addition silicone, C. Condensation silicone, D. Polysulphide",A: Poly ether,"Answer: A: Poly ether Explanation: None" "This is a real-world medical entrance exam question, please give the true answer based on the question and selection. Topic name: AIIMS 2017, Subject name: Pharmacology","A female in labor ward was administered opioid analgesic. Which of the following drugs should be kept ready for emergency? Select and explain. A. Lignocaine, B. Naloxone, C. Diphenhydramine, D. Fentanyl",B: Naloxone,"Answer: B: Naloxone Explanation: Naloxone is an opioid anatagonist given intravenously. It blocks mu, kappa and delta receptors. It is the DOC for opioid toxicity. Lignocaine is a local anaesthetic. Diphenhydramine is a first generation antihistaminic. Fentanyl is a synthetic opioid." "This is a real-world medical entrance exam question, please give the true answer based on the question and selection. Topic name: None, Subject name: Dental","When we try to find out significance of difference in height of children between 2 classes in a school, which test is used? Select and explain. A. Unpaired test, B. ANOVA, C. Chisquare, D. Coefficient of variance",A: Unpaired test,"Answer: A: Unpaired test Explanation: None" "This is a real-world medical entrance exam question, please give the true answer based on the question and selection. Topic name: None, Subject name: Dental","Composition of KRI paste is: Select and explain. A. Calcium hydroxide + Iodoform., B. Iodoform+ calcium hydroxide + menthol., C. Iodoform + camphor + parachlorophenol + Menthol., D. Parachlorophenol + Camphor + menthol.",C: Iodoform + camphor + parachlorophenol + Menthol.,"Answer: C: Iodoform + camphor + parachlorophenol + Menthol. Explanation: None" "This is a real-world medical entrance exam question, please give the true answer based on the question and selection. Topic name: None, Subject name: Dental","Which of the following tooth shows C shaped canal Select and explain. A. Mandibular 1st premolar, B. Maxillary 1st premolar, C. Mandibular 2nd premolar, D. Maxillary 2nd premolar",A: Mandibular 1st premolar,"Answer: A: Mandibular 1st premolar Explanation: None" "This is a real-world medical entrance exam question, please give the true answer based on the question and selection. Topic name: AIIMS 2018, Subject name: Psychiatry","SPIKES protocol is used for: Select and explain. A. Triage, B. Communication with patients/attendants regarding bad news, C. Writing death ceificate, D. RCT",B: Communication with patients/attendants regarding bad news,"Answer: B: Communication with patients/attendants regarding bad news Explanation: It's a protocol for breaking the bad news to patients about their illness Involved six steps S-setting up to the interview P- perception (what patient knows about his condition) I- invitation (finding out how much patient want to know) K- knowledge and information (diagnosis and treatment) E -addressing patients' emotion (how patient feel and respond) S- strategy and summary" "This is a real-world medical entrance exam question, please give the true answer based on the question and selection. Topic name: None, Subject name: Pharmacology","Which of the following drugs is a P-glycoprotein inducer? Select and explain. A. Azithromycin, B. Ketoconazole, C. Itraconazole, D. Rifampicin",D: Rifampicin,"Answer: D: Rifampicin Explanation: Answer- D. RifampicinRifampicin induces the isoenqymes CYP3A4, 2C8, 2C9, 2CI9, 286, and the transpoer P-glycoprotein. When coadministeredwith drugs that are substrates of the same enzymes, their metabolism may be accelerated resulting in lowerconcentration and less efficacy.Inducer-AmprenavirClotrimazoleDexamethasoneIndinavirMorphineNelfinavirPhenothiazineRetinoic acidRifampinRitonavirSaquinavirSt John's wo" "This is a real-world medical entrance exam question, please give the true answer based on the question and selection. Topic name: AIIMS 2018, Subject name: Biochemistry","A male child presented with coarse facies, protuberant abdomen , frontal head enlargement, thickening of cardiac valve, hepatosplenomegaly, hearing impairement. What is the most probable diagnosis? Select and explain. A. Hurler's disease, B. Hunter's disease, C. Fragile X syndrome, D. Tay Sach's disease",B: Hunter's disease,"Answer: B: Hunter's disease Explanation: Coarse facial features is a typical feature of Mucopolysaccharidosis (MPS). Hunter's disease is Type II Mucopolysaccharidosis, in which enzyme deficient is Iduronate Sulfatase. This is X-linked recessive, so occurs exclusively in Males. There is no corneal clouding in Hunter's disease. Option 3- Fragile X-Syndrome is a trinucleotide repeat expansion disorder in which patient has large face, large mandible, large testis, large eveed ears and tall stature. It is the second most common cause of mental retardation. Option 4- Tay Sach's disease is because of increased GM2 Gangliosides due to deficiency of enzyme Hexosaminidase A. Clinical features are mental retardation, cherry red spots on macula and progressive neurodegeneration but no hepatosplenomegaly. ADDITIONAL EDGE: Mucopolysaccharidosis: Type Disease name Enzyme defect Special features GAG accumulated MPS I - H (AR) Hurler disease Alpha - L - Iduronidase Inguinal hernias often present DS + HS MPS I - S (AR) Scheie disease Alpha - L - Iduronidase No mental retardation DS MPS II (XR) Hunter disease (Mild Hurler + Aggressive behaviour) Iduronate Sulfatase No corneal clouding, Exclusively males affected DS + HS MPS VI (AR) Maroteaux Lamy Syndrome Aryl Sulfatase B No mental retardation DS AR- Autosomal Recessive; XR- X-linked Recessive Additional Edge: C/F of Hunter disease Copious nasal discharge. Mental retardation. Coarse facies. Protuberant abdomen because of hepatomegaly and also inguinal hernia in these patients. frontal head enlargement. Thickening of cardiac valve. Hearing impairment. These patients have skeletal abnormalities because these mucopolysaccharides will be accumulated in bones also. And, all mucopolysaccharidosis have corneal clouding except hunter's disease." "This is a real-world medical entrance exam question, please give the true answer based on the question and selection. Topic name: None, Subject name: Pediatrics","A term neonate with unconjugated hyperbilirubinemia of 18 mg/dl on 20th day. All are common causes except – Select and explain. A. Breast rnilk jaundice, B. Congenital cholangiopathy, C. G 6PD deficiency, D. Hypothyroidism",B: Congenital cholangiopathy,"Answer: B: Congenital cholangiopathy Explanation: This child has unconjugated hyperbilirubinemia, while congenital cholangiopathy causes conjugated hyperbilirubinemia. reastfeeding jaundice and Breastmilk jaundice There is strong association between exclusive breastfeeding and neonatal jaundice. A few babies who remain on exclusive breast feed develop jaundice in the second week of life and continue well into the third month. This is called breastmilk jaundice. A bilirubin level of over 20 mg/dl may be attained. (It is presumed to be due to inhibitory substances in the breastmilk that interfere with bilirubin conjugation e.g. pregananediol and free fatty acids). Temporary interruption of breastmilk feeds will dramatically reduce the serum levels of bilirubin and there may be slight increase in bilirubin when breast feeding is resumed, but it never reaches the previous levels. Hypothyroidism   Persistent elevation of indirect bilirubin is the first sign of congenital hypothyroidism in neonates. This is due to decreased activity of UDPGT for weeks or months after birth. At least 10% of all infants with congenital hypothyroidism will have jaundice as the presenting symptom. Treatment with thyroxine promptly alleviates jaundice. G-6PD deficiency There is hemolytic anemia that results in increased production of unconjugated bilirubin." "This is a real-world medical entrance exam question, please give the true answer based on the question and selection. Topic name: None, Subject name: Pathology","Which is the most preferred route for drug administration in the management of chronic pain Select and explain. A. Intrathecal, B. Oral, C. Subdermal, D. Intravenous",B: Oral,"Answer: B: Oral Explanation: None" "This is a real-world medical entrance exam question, please give the true answer based on the question and selection. Topic name: AIIMS 2018, Subject name: Physiology","Weber-Fechner law related to: Select and explain. A. Number of nerve Fibers, B. Number of muscle Fibers, C. Number of stimulus, D. Intensity of stimulus",D: Intensity of stimulus,"Answer: D: Intensity of stimulus Explanation: Weber Fechner law: S = K x Log (I) S = sensory perception at coex K = constant (I) = intensity of stimulus at the periphery Perception felt about the intensity of stimulus changes with logarithmic scale of the actual intensity applied at the periphery. Related: Steven's power law S = K x (I)N Sensation felt is propoional to the nth power of intensity of stimulus applied i.e. perception changes as a power function of intensity." "This is a real-world medical entrance exam question, please give the true answer based on the question and selection. Topic name: None, Subject name: Pathology","White, spongy, folded thick mucosa is seen in Select and explain. A. Oral hairy leukoplakia, B. Aspirin burn, C. White spongy nevus, D. Pseudomembranous Candidiasis",C: White spongy nevus,"Answer: C: White spongy nevus Explanation: None" "This is a real-world medical entrance exam question, please give the true answer based on the question and selection. Topic name: AIIMS 2018, Subject name: Physiology","Calculate net filtration pressure with the following data: PGC = 42 mm Hg pGC = 12 mm Hg PBC= 16 mm Hg Assume that no proteins were filtered. Select and explain. A. 34 mm Hg, B. Data not sufficient, C. 14 mm Hg, D. 28 mm Hg",C: 14 mm Hg,"Answer: C: 14 mm Hg Explanation: Force oring filtration = Glomerular capillary hydrostatic pressure(PGC = 42mm Hg) Forces opposing filtration = Plasma colloid oncotic pressure (pGC = 12 mm Hg) Bowman's capsule hydrostatic pressure (PSC = 16mm Hg) : Net = PsC + pGC = 12 + 16= 28mm Hg : NET FILTRATION PRESSURE = 42 - 28 mm Hg = 14mm Hg." "This is a real-world medical entrance exam question, please give the true answer based on the question and selection. Topic name: None, Subject name: Dental","Schielder principle is Select and explain. A. Minimal enlarged apical foramen, B. Shape of the canal can be changed, C. Use of 3-5 times larger than first binding file, D. Original canal curvature should be altered to make it compatible with obturating material",A: Minimal enlarged apical foramen,"Answer: A: Minimal enlarged apical foramen Explanation: None" "This is a real-world medical entrance exam question, please give the true answer based on the question and selection. Topic name: None, Subject name: Dental","While drilling an implant, bone necrosis occurs at what temperatures: Select and explain. A. 65, B. 35, C. 45, D. 55",C: 45,"Answer: C: 45 Explanation: None" "This is a real-world medical entrance exam question, please give the true answer based on the question and selection. Topic name: None, Subject name: Dental","Subantimicrobial dose of doxycycline is: Select and explain. A. 20 mg doxycycline, B. 60 mg doxycycline, C. 80 mg doxycycline, D. 150 mg doxycycline",B: 60 mg doxycycline,"Answer: B: 60 mg doxycycline Explanation: None" "This is a real-world medical entrance exam question, please give the true answer based on the question and selection. Topic name: None, Subject name: Dental","Non radiographic method of working length determination: Select and explain. A. Digital tactile sense, B. Apical periodontal sensitivity, C. Electronic apex locator, D. All of the above",D: All of the above,"Answer: D: All of the above Explanation: None" "This is a real-world medical entrance exam question, please give the true answer based on the question and selection. Topic name: None, Subject name: Pharmacology","Problems associated with nitrous oxide anaesthesia is Select and explain. A. Behavioral problem, B. Sensitizes the heart to adrenaline, C. Prolonged difficult induction, D. Tachycardia and or arrhythmias",A: Behavioral problem,"Answer: A: Behavioral problem Explanation: None" "This is a real-world medical entrance exam question, please give the true answer based on the question and selection. Topic name: None, Subject name: Dental","Roots of primary and secondary teeth are different in that the primary roots are more divergent and flaring. The other difference: Select and explain. A. Primary roots show less accessory and lateral canals, B. Primary roots resorb more easily, C. Primary roots are shorter, D. None of the above",A: Primary roots show less accessory and lateral canals,"Answer: A: Primary roots show less accessory and lateral canals Explanation: None" "This is a real-world medical entrance exam question, please give the true answer based on the question and selection. Topic name: None, Subject name: Dental","Most important factor in rapid progression of Bacteria resulting in pulpal necrosis is? Select and explain. A. Host resistance, B. Virulence of bacteria, C. Dentinal Fluid flow, D. Pulp vitality",D: Pulp vitality,"Answer: D: Pulp vitality Explanation: None" "This is a real-world medical entrance exam question, please give the true answer based on the question and selection. Topic name: None, Subject name: Pathology","Gene for ectodermal dysplasia with cleft lip and palate Select and explain. A. ED-1, B. GJB6, C. PVRL1, D. R P73",C: PVRL1,"Answer: C: PVRL1 Explanation: None" "This is a real-world medical entrance exam question, please give the true answer based on the question and selection. Topic name: None, Subject name: Social & Preventive Medicine","Which of these is the best study to evaluate effect and outcome? Select and explain. A. Clinical trial, B. Coho, C. Case control study, D. Cross sectional study",A: Clinical trial,"Answer: A: Clinical trial Explanation: Ans: A. Clinical trialAmong the given options, clinical trial is the best study to evaluate effect and outcome.Randomized control trial (Experimental study or Intervention study):Considered as the ideal design to evaluate the effectiveness and the side-effects of new forms of intervention.Coho Study:A study design where one or more samples (called cohos) are followed prospectively and subsequent status evaluations with respect to a disease or outcome are conducted to determine which initial paicipants exposure characteristics (risk factors) are associated with it." "This is a real-world medical entrance exam question, please give the true answer based on the question and selection. Topic name: None, Subject name: Biochemistry","All of the following processes take place in mitochondria except: Select and explain. A. Beta-oxidation of fatty acids, B. DNA synthesis, C. Fatty acid synthesis, D. Protein synthesis",C: Fatty acid synthesis,"Answer: C: Fatty acid synthesis Explanation: Ans: C. Fatty acid synthesisMetabolic Pathways (Cycle or Reactions)SiteBeta oxidation.Ketone body utilization.Pyruvate dehydrogenase.Electron transpo chain.TCA cycle (BK PET),Mitochondria" "This is a real-world medical entrance exam question, please give the true answer based on the question and selection. Topic name: None, Subject name: Dental","In moyer's classification of class II type D malocclusion, there is: Select and explain. A. Orthognatic maxilla and orthognathic mandible, B. Prognathic maxilla and orthognathic mandible, C. Orthognathic maxilla and retrognathic mandible, D. Retrognathic maxilla and retrognathic mandible",C: Orthognathic maxilla and retrognathic mandible,"Answer: C: Orthognathic maxilla and retrognathic mandible Explanation: None" "This is a real-world medical entrance exam question, please give the true answer based on the question and selection. Topic name: None, Subject name: Dental","Cephalometric analysis are used to evaluate growth changes by superimposing on: Select and explain. A. Sella - Nasion plane, B. Mandibular plane, C. FHP, D. Occlusal plane",A: Sella - Nasion plane,"Answer: A: Sella - Nasion plane Explanation: None" "This is a real-world medical entrance exam question, please give the true answer based on the question and selection. Topic name: None, Subject name: Microbiology","All of the following are associated with HHV8 except? Select and explain. A. Kaposi sarcoma, B. Primary effusion lymphoma, C. Castleman disease, D. T-cell leukemia",D: T-cell leukemia,"Answer: D: T-cell leukemia Explanation: Ans. D. T-cell leukemiaHuman Herpes Virus-8 Aka Kaposi's sarcoma-associated herpesvirus (KSHV).Human herpesvirus-8 (HHV-8) or Kaposi's sarcoma-associated herpesvirus (KSHV infects B lymphocytes, macrophages, and both endothelial and epithelial cells) appears to be causally related to Kaposi's sarcoma and a subgroup of AIDS-related B-cell body cavity-based lymphomas (primary effusion lymphomas) and multicentric Castleman's disease. HHV-8 infection is more common in pas of Africa than in the United States. Primary HHV-8 infection in immunocompetent children may manifest as fever and maculopapular rash. Among individuals with intact immunity, chronic asymptomatic infection is the rule, and neoplastic disorders generally develop only after subsequent immunocompromise. In patients with AIDS, effective antiretroviral therapy has caused improvement in HHV-8-related disease. The virus is sensitive to ganciclovir, foscarnet, and cidofovir, but the clinical benefit has not been demonstrated in trials. Invasive cervical carcinoma has been causally implicated with human papillomavirus infection." "This is a real-world medical entrance exam question, please give the true answer based on the question and selection. Topic name: None, Subject name: Pediatrics","Presented edema, oliguria and frothy urine. He has no past history of similar complaints. On examination, his urine was positive for 3+ proteinuria, no RBCs/WBCs and no casts. His serum albumin was 2.5 gm/L and serum creatinine was 0.5 mg/dL. The most likely diagnosis is: Select and explain. A. Minimal change disease, B. IgA nephropathy, C. Interstitial nephritis, D. Membranous nephropathy",A: Minimal change disease,"Answer: A: Minimal change disease Explanation: Ans: A.Minimal change disease(Ref Harrison 19/e p184, 18/2345)Most likely diagnosis - Minimal change disease.Minimal Change Disease:Peak age of onset = Between 6-8 years of age (usually <10 years)Type of onset = InsidiousTypical presentation = Nephrotic syndromeHallmark of Nephrotic syndrome =Peripheral edema.Occurs when serum albumin levels < 3 gm/dlLaboratory findings = Proteinuria & hypoalbuminemia." "This is a real-world medical entrance exam question, please give the true answer based on the question and selection. Topic name: None, Subject name: Anatomy","Which of the following is not attached laterally to thyroid cartilage, or all the following muscles are attached to the oblique line of thyroid cartilage except: Select and explain. A. Sternothyroid, B. Thyrohyoid, C. Superior constrictor, D. Inferior constrictor",C: Superior constrictor,"Answer: C: Superior constrictor Explanation: Larynx is composed of nine cartilage out of which 3 are paired and 3 unpaired. Paired - Arytenoids, corniculates and cuneiforms. Unpaired - Thyroid, cricoids, epiglottis. Thyroid cartilage is the largest cartilage of larynx. On the lateral surface of thyroid cartilage three extrinsic laryngeal muscle are connects to it as: Sternohyoid Thyrohyoid Inferior pharyngeal constrictor." "This is a real-world medical entrance exam question, please give the true answer based on the question and selection. Topic name: None, Subject name: Pediatrics","A parent is homozygous and a parent heterozygous for an autosomal recessive gene. What will be the outcome – Select and explain. A. 75% children affected, B. No child affected, but all are carriers, C. 50% children affected, rest are carriers, D. 25% children affected, rest are carriers","C: 50% children affected, rest are carriers","Answer: C: 50% children affected, rest are carriers Explanation: Autosomal recessive disorders Autosomal recessive disorders are manifested in homozygous state, i.e. when both alleles at a given gene locus are mutant. There may be following five situations -" "This is a real-world medical entrance exam question, please give the true answer based on the question and selection. Topic name: None, Subject name: Pathology","Antischkow cells are present in all of the following conditions except Select and explain. A. Sickle cell anaemia, B. Iron deficiency anaemia, C. Apthous ulcer, D. herpes simplex",D: herpes simplex,"Answer: D: herpes simplex Explanation: None" "This is a real-world medical entrance exam question, please give the true answer based on the question and selection. Topic name: None, Subject name: Surgery","Patient on dental chair during extraction becomes unconscious; patient is breathing but not responsive. Your first step will be: Select and explain. A. Recline chair and shout for help and give oxygen, B. Check BP and pulse and intubate, C. Start CPR immediately, D. ECG",A: Recline chair and shout for help and give oxygen,"Answer: A: Recline chair and shout for help and give oxygen Explanation: None" "This is a real-world medical entrance exam question, please give the true answer based on the question and selection. Topic name: None, Subject name: Anatomy","Which of the following structures is not present on the internal surface of mandible? Select and explain. A. Genial tubercle, B. Mylohyoid ridge, C. Lingula, D. Mental foramen",D: Mental foramen,"Answer: D: Mental foramen Explanation: Mental foramen is present on the outer surface of body of mandible between the roots of two premolars" "This is a real-world medical entrance exam question, please give the true answer based on the question and selection. Topic name: None, Subject name: Dental","A mother and her 4 year old son are seated alone in a reception area with the child staring off into space, rocking and constantly twisting a strand of hair about his fingers. Upon entry of another person, the child begins to beat his fist against the side of his face and behaves as though he does not hear his mother speaking to him. This behavior is most characteristic of: Select and explain. A. An autistic child, B. A mentally retarded child, C. First dental appointment anxieties of a 4 year old child, D. A child with a chronic seizure disorder",A: An autistic child,"Answer: A: An autistic child Explanation: None" "This is a real-world medical entrance exam question, please give the true answer based on the question and selection. Topic name: None, Subject name: Dental","ZnPO4 setting time is 4-7 min. All of the following methods are employed to prolong its setting time except: Select and explain. A. Addition of drop of distilled water, B. Prolong spatulation time, C. Small increment of addition of powder to liquid, D. Mixing on cooled glass slab",A: Addition of drop of distilled water,"Answer: A: Addition of drop of distilled water Explanation: Control of setting time​: ​Manufacturing process​ Sintering temperature : The higher the ​temperature, the more slowly the cement sets.​ Particle size : Finer particles react more quickly as a greater surface area is exposed to ​the liquid.​ Water content of liquid : Presence of excess water accelerates, whereas insufficient water ​retards the reaction.​ Buffering agents : When added, slow down the reaction.​ ​Factors under control of operator​ Temperature : Higher temperature accelerates the reaction. Cooling the mixing slab is an effective way of slowing the reaction and prolonging the working time.​ Powder-liquid ratio : More the liquid, slower the reaction. ​ Rate of addition of powder to liquid : The reaction is slower, if the powder is incorporated slowly.​ Mixing time : The longer the mixing time (within practical limits), the slower is the rate of reaction. Ref: Manappalil P: 90​" "This is a real-world medical entrance exam question, please give the true answer based on the question and selection. Topic name: None, Subject name: Pathology","An abnormal resorption pattern in primary teeth, delayed eruption of permanent teeth and a Large tongue are the features of: Select and explain. A. Addison's disease, B. Hypothyroidism, C. Hyperthyroidism, D. Von-Recklinghausen disease",B: Hypothyroidism,"Answer: B: Hypothyroidism Explanation: None" "This is a real-world medical entrance exam question, please give the true answer based on the question and selection. Topic name: None, Subject name: Dental","Which one of the following is not FDA approved Select and explain. A. GEM 215, B. Elyzol, C. Periochip, D. Atridox",B: Elyzol,"Answer: B: Elyzol Explanation: None" "This is a real-world medical entrance exam question, please give the true answer based on the question and selection. Topic name: None, Subject name: Dental","Dynamic creep is the Select and explain. A. Continuing alloying between silver-tin alloy and mercury during the life of restoration, B. Deformation of set amalgam during function, C. Process whereby alloy is ""wetted"" by mercury, D. Spread of amalgam during packing",B: Deformation of set amalgam during function,"Answer: B: Deformation of set amalgam during function Explanation: None" "This is a real-world medical entrance exam question, please give the true answer based on the question and selection. Topic name: None, Subject name: Pediatrics","All are features of non paralytic polio except – Select and explain. A. Absent deep tendon reflexes, B. Head drop, C. Nuchal rigidity, D. Knee heel test negative",A: Absent deep tendon reflexes,"Answer: A: Absent deep tendon reflexes Explanation: Poliomyelitis The usual age group affected by poliomyelitis is between 6 months to 3 years. Incubation period —> 7-14 days. Clinical manifestations Clinical course of Poliomyelitis has following stages. 1. Inapparent (subclinical) infection Occurs in 95% of infected individuals. Patient is asymptomatic 2. Minor (abortive) illness 1114-8% of infection. Fever, sore throat, headache adn malaise. Associated with phase of viremia. Lasts for 1 to 5 days ​ 3. Aseptic meningitis In 1% of infected cases. Signs of menigitis —> Neck rigidity, vomiting etc. 4. Paralytic polio In less than 1% of infections. Note : First 3 stages are non-paralytic polio." "This is a real-world medical entrance exam question, please give the true answer based on the question and selection. Topic name: None, Subject name: Surgery","Which of the following procedures can be used to treat an oro-antral fistula resulting from posterior surgery: (Or) Oro-antral fistula in maxillary 3rd molar region is best/commonly treated by Select and explain. A. Buccal mucoperiosteal flap, B. Palatal mucoperiosteal flap, C. Bridge flap, D. Palatal island flap",D: Palatal island flap,"Answer: D: Palatal island flap Explanation: None" "This is a real-world medical entrance exam question, please give the true answer based on the question and selection. Topic name: None, Subject name: Dental","Fovea palatine are Select and explain. A. Structures through which blood supply take place, B. Mucosal salivary glands, C. Palatal termination of maxillary denture, D. Found in every individual",B: Mucosal salivary glands,"Answer: B: Mucosal salivary glands Explanation: None" "This is a real-world medical entrance exam question, please give the true answer based on the question and selection. Topic name: None, Subject name: Dental","In case of addition silicones what should be done for better cast Select and explain. A. Apply ketone over tray, B. Apply chloroform over tray, C. Add flavouring agent to prevent bad odour, D. Delay pouring of cast",D: Delay pouring of cast,"Answer: D: Delay pouring of cast Explanation: None" "This is a real-world medical entrance exam question, please give the true answer based on the question and selection. Topic name: None, Subject name: Physiology","contractile part of intrafusal fibres Select and explain. A. Centre, B. Ends, C. None, D. Both",B: Ends,"Answer: B: Ends Explanation: None" "This is a real-world medical entrance exam question, please give the true answer based on the question and selection. Topic name: None, Subject name: Dental","The type of strain develop when force is applied perpendicular to surface Select and explain. A. Compressive strain, B. Tensile strain, C. Shear stress, D. Tensile stress",A: Compressive strain,"Answer: A: Compressive strain Explanation: None" "This is a real-world medical entrance exam question, please give the true answer based on the question and selection. Topic name: None, Subject name: Pharmacology","Km value indicates: Select and explain. A. Purity of Enzyme, B. Physiological role, C. The substrate concentration at half maximal velocity, D. Affinity",C: The substrate concentration at half maximal velocity,"Answer: C: The substrate concentration at half maximal velocity Explanation: The dose rate-Cpss relationship is linear only in case of drugs eliminated by first order kinetics. For drugs (e.g. phenytoin) which follow Michaelis Menten kinetics, elimination changes from first order to zero order kinetics over the therapeutic range. Increase in their dose beyond saturation levels causes an increase in Cpss which is out of proportion to the change in dose rate. In their case:" "This is a real-world medical entrance exam question, please give the true answer based on the question and selection. Topic name: None, Subject name: Biochemistry","Which of the following conversions does not require Biotin as a cofactor? Select and explain. A. Gamma carboxylation of glutamate, B. Acetyl Co-A to Malonyl Co-A, C. Propionyl Co-A to methyl malonyl Co-A, D. Pyruvate to oxaloacetate",A: Gamma carboxylation of glutamate,"Answer: A: Gamma carboxylation of glutamate Explanation: Ans: A. Gamma carboxylation of glutamate(Ref: Harrison 19/e p96e-5; Harper 30/e p550, 561)Gamma carboxylation of glutamate:Carried out by Gamma-glutamyl carboxylase.Gamma-glutamyl carboxylase:Vitamin-K dependent enzyme.Catalyzes post-translational modification of vitamin K-dependent proteins.Biotin:Transfers carbon dioxide in reactions like acetyl-CoA carboxylase, pyruvate carboxylase, propionyl-CoA carboxylase & methylcrotonyl-CoA carboxylase.Biotin functions to transfer carbon dioxide in reactionsPyruvate carboxylasePyruvate (3C) to oxaloacetate (4C) in gluconeogenesisdegAcetyl-CoA carboxylaseAcetyl-CoA (2C) to malonyl-CoA (3C) in lipid synthesis.Propionyl-CoA carboxylasePropionyl-Co A (3C) to methylmalonyl-CoA (4C) inMethylmalonyl CoA carboxyl Propionic acid synthesis in bacteriaMethylcrotonyl-CoA carboxylaseLeucine catabolism" "This is a real-world medical entrance exam question, please give the true answer based on the question and selection. Topic name: None, Subject name: Dental","Majority of the syndromic craniosynosteses are associated with: Select and explain. A. FGFR mutations, B. IRFS mutations, C. VWS mutations, D. 22q 11.2 mutations",A: FGFR mutations,"Answer: A: FGFR mutations Explanation: None" "This is a real-world medical entrance exam question, please give the true answer based on the question and selection. Topic name: None, Subject name: Surgery","The facial proportions between upper, middle and lower thirds of fare Is: Select and explain. A. 25:50:25, B. 34:33:33, C. 35:30:35, D. 30:35:35",D: 30:35:35,"Answer: D: 30:35:35 Explanation: None" "This is a real-world medical entrance exam question, please give the true answer based on the question and selection. Topic name: None, Subject name: Medicine","Cold agglutinin is Select and explain. A. IgG, B. IgM, C. IgA, D. IgD",B: IgM,"Answer: B: IgM Explanation: None" "This is a real-world medical entrance exam question, please give the true answer based on the question and selection. Topic name: AIIMS 2019, Subject name: Biochemistry","Thiamine deficiency is assessed by: Select and explain. A. Erythrocyte transketolase activity, B. RBS Glutathione reductase, C. RBC thiamine levels, D. Serum thiamine level",A: Erythrocyte transketolase activity,"Answer: A: Erythrocyte transketolase activity Explanation: Thiamine (vitamin B1) Deficiency It is assessed by Erythrocyte transketolase activity. Thiamine (Vitamin B1) is the marker for transketolase enzyme -which is involved in HMP pathway (in RBCs) Activity of Transketolase is measured (not the quantity). No impoance of checking serum thiamine Riboflavin (Vitamin B2) Deficiency RBC glutathione reductase is the marker for vitamin B2 deficiency. =Enzyme Transaminase activity is checked for Pyridoxine (Vitamin B6) deficiency =If Relation between thiamine and energy is asked then mark enzyme of Link reaction and TCA Because pyruvate dehydrogenase of Link reaction and a keto glutarate dehydrogenase of TCA cycle | give energy and they are requiring vitamins Both link reaction and TCA requires 4 B complex vitamins B1, B2, B3 and B5" "This is a real-world medical entrance exam question, please give the true answer based on the question and selection. Topic name: None, Subject name: Pathology","Calcification of brain is seen in which of the following: Select and explain. A. Papillon lefevre syndrome, B. Gorham stout syndrome, C. Menke syndrome, D. Leukocyte adhesion deficiency syndrome",A: Papillon lefevre syndrome,"Answer: A: Papillon lefevre syndrome Explanation: None" "This is a real-world medical entrance exam question, please give the true answer based on the question and selection. Topic name: None, Subject name: Surgery","Cardiac dysarrythmia during extraction is caused by: Select and explain. A. Trochlear nerve, B. Facial nerve, C. TrigeminaL nerve, D. Occulomotor nerve",C: TrigeminaL nerve,"Answer: C: TrigeminaL nerve Explanation: None" "This is a real-world medical entrance exam question, please give the true answer based on the question and selection. Topic name: None, Subject name: Microbiology","A 35-year-old male farmer presents with multiple discharging cervical sinuses. Which of these stains will be useful for the diagnosis? Where does this organism normally colonise in the body? Select and explain. A. Grocott Methenamine silver, skin, B. PAS, intestine, C. AFB, mouth, D. Gram-stain, oropharynx","C: AFB, mouth","Answer: C: AFB, mouth Explanation: Ans: C. AFB, mouth(Ref Ananthanarayan 10/e p398, 8/e p391-391' I larrison 19/e p1088)In this farmer, who presented with multiple discharging cervical sinuses, the most likely diagnosis is actinomycosis.Actinomyces - Not acid-fast organism.Actinomycosis:Occurs most commonly at an oral, cervical, or facial site, usually as a soft tissue swelling, abscess, or mass lesion that is often mistaken fir a neoplasm.The angle of the jaw is generally involved, but a diagnosis of actinomycosis should be considered with any mass lesion or relapsing infection in the head and neck." "This is a real-world medical entrance exam question, please give the true answer based on the question and selection. Topic name: None, Subject name: Physiology","The concentration of a negative ion is 100 mMoUL extracellular and 10 mMo1/1 intracellular. What will be the Nernst potential for this ion? Select and explain. A. --10 mV, B. +10 mV, C. --61 mV, D. +61 mV",C: --61 mV,"Answer: C: --61 mV Explanation: Ans: C. --61 mVRef: Ganong 25Ie p9, p24Ie p9-10; Guyton 13Ie p61Ek = 61.5 log Ci/CeIntracellular concentration Ci = 100Extracellular concentration Ce = 10EK = 61 C log100/10 = -61 mV." "This is a real-world medical entrance exam question, please give the true answer based on the question and selection. Topic name: AIIMS 2018, Subject name: Pharmacology","A Pediatrician was called for attending a new born baby in the labour ward. The serum unconjugated bilirubin of this baby was 33 mg/dL. Which of the following drug taken by mother in late 3rd trimester may have lead to this problem? Select and explain. A. Cotrimoxazole, B. Azithromycin, C. Ampicillin, D. Chloroquine",A: Cotrimoxazole,"Answer: A: Cotrimoxazole Explanation: Bilirubin given in question is very high; Bilirubin binds to Albumin in blood, so when any drug displaces bilirubin from albumin, it can lead to increased free bilirubin. Cotrimoxazole is a combination of trimethoprim and sulfamethoxazole (a sulfonamide). Sulfonamides (acidic drugs) can displace bilirubin from albumin resulting in excessive free bilirubin. This can cross blood brain barrier in newborn babies which can cause Kernicterus; hence avoided in pregnancy." "This is a real-world medical entrance exam question, please give the true answer based on the question and selection. Topic name: None, Subject name: Surgery","Syncope is usually caused by: Select and explain. A. Vasoconstriction, B. Cerebral ischemia, C. Cerebral hyperemia, D. Decrease in the vascular bed",B: Cerebral ischemia,"Answer: B: Cerebral ischemia Explanation: None" "This is a real-world medical entrance exam question, please give the true answer based on the question and selection. Topic name: None, Subject name: Dental","The enamel has no capacity of self —repair because Select and explain. A. It has only a small percent of organic content, B. Its formative cells are lost once it is completely formed, C. It is essentially a keratin tissue and has no blood vessels, D. It has no direct connection with the active cells of the dental pulp",B: Its formative cells are lost once it is completely formed,"Answer: B: Its formative cells are lost once it is completely formed Explanation: None" "This is a real-world medical entrance exam question, please give the true answer based on the question and selection. Topic name: None, Subject name: Dental","If resin is packed in sandy stage what will be seen in the processed denture? Select and explain. A. Distortion, B. Porosity, C. Inhomogeneous colour, D. Increased strength",B: Porosity,"Answer: B: Porosity Explanation: None" "This is a real-world medical entrance exam question, please give the true answer based on the question and selection. Topic name: None, Subject name: Physiology","Endocrine disorder is the primary cause of: Select and explain. A. Acromegaly, B. Albright's syndrome, C. Paget's disease, D. Fibrous dysplasia",A: Acromegaly,"Answer: A: Acromegaly Explanation: None" "This is a real-world medical entrance exam question, please give the true answer based on the question and selection. Topic name: None, Subject name: Microbiology","All parasites are in hepatocytes Except ? Select and explain. A. Toxoplasma, B. P. falciparium, C. Leishmania, D. Babesia",D: Babesia,"Answer: D: Babesia Explanation: Answer- D. Babesia* Babesia is an parasite which parasitizes RBCs like Plasmodium* visceral leishmaniasis is an infection in hepatocytes ,* Malaria especially with Plasmodium falciparum hepatomegaly can develop early and subsides with treatment." "This is a real-world medical entrance exam question, please give the true answer based on the question and selection. Topic name: None, Subject name: Radiology","The speed with which the electrons travel from the filament of cathode to the anode depends upon Select and explain. A. Potential difference between the two electrodes, B. Number of milliamperes in the tube circuit, C. Angle between the filament and target, D. Voltage of the filament",A: Potential difference between the two electrodes,"Answer: A: Potential difference between the two electrodes Explanation: The primary functions of the power supply of an X-ray machine are to: Provide a low-voltage current to heat the X-ray tube filament. Generate a high potential difference to accelerate electrons from the cathode to the focal spot on the anode. The X-ray tube and two transformers lie within an electrically grounded metal housing called the head of the X-ray machine. White and Pharoah, pg-6" "This is a real-world medical entrance exam question, please give the true answer based on the question and selection. Topic name: None, Subject name: Pediatrics","Recurrent abdominal pain in children in most often due to – Select and explain. A. Roundworms, B. Emotional/behavioural problems, C. Amoebiasis, D. Giardiasis",B: Emotional/behavioural problems,"Answer: B: Emotional/behavioural problems Explanation: Chronic and recurrent abdominal pain in children Chronic abdominal pain is defined as recurrent or persistent bouts of abdominal pain that occur over a minimum of 3 months. The commonest cause of chronic abdominal pain in older children is functional (non organic) that may be due to emotional or behavioral problems. Causes of chronic and recurrent Abdominal pain" "This is a real-world medical entrance exam question, please give the true answer based on the question and selection. Topic name: None, Subject name: Pediatrics","A child presented in the OPD with multiple permeating lesions involving all the bones of the body which of the following is the most probable diagnosis : Select and explain. A. Neuroblastoma, B. Metastasis from osteosarcoma, C. Histiocytosis X, D. Metastasis from Wilm's tumour",C: Histiocytosis X,"Answer: C: Histiocytosis X Explanation: Osteolytic lesions in a child involving all the bones of the body points towards the diagnosis of Histiocytosis X. The bony lesions in Histiocytosis x can present as a single lesion or as a wide spread disease involving virtually any bone of the body. The lesions are osteolytic and the common sites of involvement are --> Skull, Vertebrae, Ribs & Clavicle. ​About other choices : Metastasis from Wilm's tumour  -   It can be easily be excluded as the metastatic from Wilms' tumour does not involve bones. Neuroblastoma  -  Though it can involve any bone of the body, but simultaneously involvement of all the bones of the body without any common manifestation is very unlikely to be neuroblastoma. Metastasis from Osteosarcoma - Metastasis from osteosarcoma commonly goes to lung not bones." "This is a real-world medical entrance exam question, please give the true answer based on the question and selection. Topic name: None, Subject name: Medicine","A patient with suspected cardiac tamponade presents to the AIIMS emergency. You are asked to monitor BP of this patient. All the following precautions should be taken except: Select and explain. A. Patient should be asked to take deep breaths, B. The cuff pressure should be increased to 20 mm over systolic pressure, C. The cuff should be slowly deflated until the first Korotkoff sound is heard only during expiration, D. Pulses paradoxus may not be present",C: The cuff should be slowly deflated until the first Korotkoff sound is heard only during expiration,"Answer: C: The cuff should be slowly deflated until the first Korotkoff sound is heard only during expiration Explanation: Ans: C. The cuff should be slowly deflated until the first Korotkoff sound is heard only during expiration(Ref: Harrison 19/e p1621, 1446 18/e p1825; Ganong 25/e p517)The cuff is slowly deflated and the paradoxical pulse is recorded from the point when first Korotkoff sound is heard only during expiration to when it is heard in both expiration and inspiration.Pulsus paradoxus:Quantified using a blood pressure cuff and stethoscope (Korotkoff sounds), by measuring the variation of the systolic pressure during expiration and inspiration.Inflate cuff until no sounds (as is normally done when taking a BP) slowly decrease cuff pressure until systolic sounds are first heard during expiration but not during inspiration, (note this reading), slowly continue decreasing the cuff pressure until sounds are heard throughout the respiratory cycle, (inspiration and expiration) (note this second reading).If the pressure difference between the two readings is >10mm Hg, it can be classified as pulsus paradoxus." "This is a real-world medical entrance exam question, please give the true answer based on the question and selection. Topic name: None, Subject name: Biochemistry","The most important career of cholesterol in plasma is: Select and explain. A. VLDL, B. LDL, C. HDL, D. Chylomicrons",B: LDL,"Answer: B: LDL Explanation: Transport of Cholesterol between tissues: Dietary cholesterol is incorporated into Chylomicrons. 95% of Chylomicron cholesterol is delivered to liver in Chylomicron remnant. The cholesterol is secreted from liver in VLDL. Most of the cholesterol secreted in VLDL is retained in IDL and ultimately LDL. LDL cholesterol is taken up by liver and extrahepatic tissues (peripheral tissues)." "This is a real-world medical entrance exam question, please give the true answer based on the question and selection. Topic name: None, Subject name: Pathology","An asymptomatic tooth has deep caries on occlusal surface. Radiograph shows radiopaque mass at apex of the tooth; this mass is most likely to be: Select and explain. A. Cementoma, B. Condensing osteitis, C. Chronic apical periodontitis, D. Acute apical periodontitis",B: Condensing osteitis,"Answer: B: Condensing osteitis Explanation: None" "This is a real-world medical entrance exam question, please give the true answer based on the question and selection. Topic name: None, Subject name: Surgery","Best diagnostic investigation for acute appendicitis in children is: Select and explain. A. MRI, B. CECT, C. USG, D. X-ray.",C: USG,"Answer: C: USG Explanation: Ans: C. USGUltrasonography has a sensitivity of approximately 85% and a specificity of more than 90% for the diagnosis of acute appendicitis.Ultrasonography has the advantages of being a noninvasive modality requiring no patient preparation that also avoids exposure to ionizing radiation.Thus, it is commonly used in children and in pregnant patients with equivocal clinical findings suggestive of acute appendicitis." "This is a real-world medical entrance exam question, please give the true answer based on the question and selection. Topic name: None, Subject name: Medicine","Which of the following is true about WILSON disease Select and explain. A. High copper in urine, high copper in serum, B. High ceruloplasmin, C. Low serum copper, D. Low urinary copper",C: Low serum copper,"Answer: C: Low serum copper Explanation: Ans. c. Low serum copperWilson disease is characterized by low serum copper, low ceruloplasmin and high urinary excretion of copper. Wilson disease is characterized by: ATP7B protein deficiency impairs biliary copper excretion, resulting in positive copper balance, hepatic copper accumulation, and copper toxicity from oxidant damage.Defective copper incorporation into apoceruloplasmin leads to excess catabolism and low blood levels of ceruloplasmin.Serum copper levels are usually lower than normal because of low blood ceruloplasmin, which normally binds >90% of serum copper. As the disease progresses, nonceruloplasmin serum copper (""free"" copper) levels increase, resulting in copper buildup in other pas of the body, such as the brain, leading to neurologic and psychiatric disease." "This is a real-world medical entrance exam question, please give the true answer based on the question and selection. Topic name: None, Subject name: Medicine","Which of the following is not true regarding iron supplementation in iron deficiency anemia? Select and explain. A. Administer a small diluted dose first prior to infusion to look for any allergy to iron preparation, B. Oral iron therapy should be stopped once the patient achieves a hemoglobin of >12 mg/dL, C. If gastric intolerance to oral iron therapy occurs, all patients should be administered parenteral forms, D. Parenteral iron supplementation is required in a patient with Hb of less then 7 mg/dL",B: Oral iron therapy should be stopped once the patient achieves a hemoglobin of >12 mg/dL,"Answer: B: Oral iron therapy should be stopped once the patient achieves a hemoglobin of >12 mg/dL Explanation: Ans: B. Oral iron therapy should be stopped once the patient achieves a hemoglobin of >12 mg/dL(Ref: Harrison 19/e p629. 18/e p849)Iron-deficiency anemia - Goal of therapy:Not only repair anemia.Provide stores of at least 0.5-1 g of iron.Sustained treatment for period of 6-12 months after anemia correction necessary to achieve stores." "This is a real-world medical entrance exam question, please give the true answer based on the question and selection. Topic name: None, Subject name: Dental","According to piagets, the period of pre operational stage is: Select and explain. A. Birth to 18 months, B. 12 years onwards., C. 2-6 years., D. 7-12 years.",C: 2-6 years.,"Answer: C: 2-6 years. Explanation: According to Piaget Cognitive theory, the sequence of development has been categorized into 4 major stages: Sensorimotor stage (0 to 2 yrs) Pre-operational stage (2 to 6 yrs) Concrete operation stage (7 to 12 yrs) Formal operation stage (11 to 15 yrs)" "This is a real-world medical entrance exam question, please give the true answer based on the question and selection. Topic name: None, Subject name: Dental","Which of the following is not the diagnostic feature of Apert syndrome: Select and explain. A. Bicoronal craniosynostosis, B. Midface hypoplasia, C. Mandibular hypoplasia, D. Complex syndactyly",C: Mandibular hypoplasia,"Answer: C: Mandibular hypoplasia Explanation: None" "This is a real-world medical entrance exam question, please give the true answer based on the question and selection. Topic name: None, Subject name: Pediatrics","Increase in muscle mass at adolescence is probably caused by – Select and explain. A. Thyroid hormone, B. Adrenal hormone, C. Growth harmone, D. None of the above",C: Growth harmone,"Answer: C: Growth harmone Explanation: During adolescence, there is increased in muscle mass with loss of body fat. The maximum loss of fat and increase in muscle mass correspond to the time of peak hight velocity. ""Both GH and gonadal steroids must be present for normal pubertal growth"". __________ Laurence D. Steinberg Note - thyroid hormone and cortisol (adrenal steroids) are also required but this requirement is not different from that is required during childhood. Gonadal steroids (most imp) and GH are the major hormone for pubertal growth. So, best answer of this question would be all of the above." "This is a real-world medical entrance exam question, please give the true answer based on the question and selection. Topic name: None, Subject name: Dental","Which of the following is not an obstructive cause of mouth breathing: Select and explain. A. Deviated nasal septum., B. Short lip., C. Narrow maxilla., D. Obstructive adenoids.",B: Short lip.,"Answer: B: Short lip. Explanation: Short lip is an anatomic cause." "This is a real-world medical entrance exam question, please give the true answer based on the question and selection. Topic name: None, Subject name: Dental","Epidemiological surveillance is defined as Select and explain. A. Active surveillance, B. passive surveillance, C. Sentinel surveillance, D. Epidemiological surveillance",B: passive surveillance,"Answer: B: passive surveillance Explanation: None" "This is a real-world medical entrance exam question, please give the true answer based on the question and selection. Topic name: None, Subject name: Gynaecology & Obstetrics","Hydrocephalus is best detected antenately by: Select and explain. A. X-ray abdomen, B. Amniocentesis, C. Clinical examination, D. Ultrasonography",D: Ultrasonography,"Answer: D: Ultrasonography Explanation: Hydrocephalus is a condition in which there is an abnormal increase in cerebrospinal fluid within the ventricular and subarachnoid spaces of brain. “The prenatal diagnosis of hydrocephalus is usually made by demonstration of a dilated ventricular system in an ultrasound examination.” Fernando Arias 2/e, p 331 Earliest and most accurate sonographic sign of hydrocephalus – Enlarged lateral ventricles. The lateral ventricle is measured at the level of atrium. Normal transverse diameter of atrium is 7 mm + 1 mm (It remains constant during the second and third trimester). When diameter of atrium is >10 mm, it is called as Ventriculomegaly/Hydrocephalus. Other signs of hydrocephalus on USG: Dangling choroid plexuses. Thinning out of cerebral cortex." "This is a real-world medical entrance exam question, please give the true answer based on the question and selection. Topic name: None, Subject name: Dental","If the pulp of the single rooted canal is triangular in cross-section with the base of the triangle located facially and apex located lingually with the mesial arm longer than the distal, the tooth is most likely: Select and explain. A. Max. central incisor, B. Max. lateral incisor, C. Mand. second premolar, D. Mand. central incisor",A: Max. central incisor,"Answer: A: Max. central incisor Explanation: SHAPE OF ACCESS CAVITY: Maxillary Incisors - Rounded triangle with the apex towards cingulum. Maxillary Canine - Ovoid labiolingually. Maxillary Premolar - Ovoid in Buccolingual direction (Slot shaped). Maxillary Molars - Triangle in the mesial half base towards buccal and apex towards palatal side. Note if asked in exams:            - Rhomboidal (1st choice)             - Rounded Triangle (2nd choice) Modified Access Cavity in maxillary molars (to accommodate accessory orifice):            - Clove leaf.            - Shamrock preparation. MANDIBULAR TEETH: Mandibular Incisors - Narrow rounded triangle or ovoid. Mandibular Canine - Ovoid / funnel shaped. Mandibular Premolar - Ovoid buccolingually. Mandibular Molars - Trapezoidal (1st choice)                     - Rounded Triangle (2nd choice)" "This is a real-world medical entrance exam question, please give the true answer based on the question and selection. Topic name: None, Subject name: Pharmacology","Which of the following is true for octreotide? Select and explain. A. Stimulates growth hormone secretion, B. Used in secretory diarrhea, C. Orally active, D. Contraindicated in acromegaly",B: Used in secretory diarrhea,"Answer: B: Used in secretory diarrhea Explanation: Ans. b. Used in secretory diarrhoea (Ref Goodman Gilman 12/e p1338-1339. 1346: Katling 11/e p650)Octreotide is used in secretory diarrheaOctreotide is an octapeptide analog of somatostatin, effective in inhibiting the severe secretory diarrhea brought about by hormone-secreting tumours of the pancreas and the GI tract.""" "This is a real-world medical entrance exam question, please give the true answer based on the question and selection. Topic name: AIIMS 2017, Subject name: Physiology","Slow wave potential originates in which pa of intestine: Select and explain. A. Interstitial cells of Cajal, B. Parasympathetic neurons, C. Smooth muscle cells, D. Myentric plexus",A: Interstitial cells of Cajal,"Answer: A: Interstitial cells of Cajal Explanation: INTERSTITIAL CELLS OF CAJAL: - These cells synapse with both smooth muscle and neurons of enteric nervous system - They are Pacemaker cell of G.I tract. - Smooth muscle of G.I.T have 2 basic types of electric waves. 1. Slow waves 2. Spikes Slow waves: - Most gastro intestinal contractions occur rhythmically & this rhythm is determined mainly by frequency of 'slow waves' - These are not action potentials, instead they are slow undulating changes in resting membrane potential - These waves are caused by complex interactions among smooth muscle cells & interstitial cells of cajal. - Slow waves usually do not themselves cause muscle contraction (except in stomach) instead they mainly excite appearance of intermittent spike potential & spike potentials in turn actually excite muscle contraction. Parasympathetic neurons - depolarize smooth muscle cell & makes muscle fibers more excitable. Smooth muscle contraction occurs in response to entry of calcium ions into muscle fiber. Enteric nervous system composed of 2 plexuses 1. Myenteric plexus / Auerbach's plexus - outer 2. Submucosal Meissner's plexus - inner Myenteric plexus control G.I movements Submucosal plexus control G.I secretions & local blood flow." "This is a real-world medical entrance exam question, please give the true answer based on the question and selection. Topic name: None, Subject name: Medicine","Glanzmann disease is Select and explain. A. Congenital defect of platelets, B. Congenital defect of RBC, C. Defect of Neutrophils, D. Clotting factor deficiency",A: Congenital defect of platelets,"Answer: A: Congenital defect of platelets Explanation: None" "This is a real-world medical entrance exam question, please give the true answer based on the question and selection. Topic name: None, Subject name: Surgery","All are true about use of articaine in a child except Select and explain. A. It's an amide which is metabolised in both plasma and liver, B. Plasma half-life is 90 min, C. Infiltration produces adequate anaesthesia in primary molar so need for block anesthesia is eliminated, D. More breakdown cause less toxicity",B: Plasma half-life is 90 min,"Answer: B: Plasma half-life is 90 min Explanation: None" "This is a real-world medical entrance exam question, please give the true answer based on the question and selection. Topic name: None, Subject name: Ophthalmology","In which of the following, long spaced collagen present? Select and explain. A. Diaphragm, B. Cornea, C. Basement membrane, D. Tympanic membrane",B: Cornea,"Answer: B: Cornea Explanation: Answer- B. Cornea'Long-spacing collagen is a common component of normal human corneal stroma and its occurrence seems to correlatewith the age-related changes of the tissue.Corneal transparency is mainly dependent on the arrangement of these collagen fibers in stroma." "This is a real-world medical entrance exam question, please give the true answer based on the question and selection. Topic name: None, Subject name: Gynaecology & Obstetrics","Which of the following is the least likely complication of pregnancy-induced hypeension? Select and explain. A. Renal failure, B. Pre-eclampsia, C. HELLP syndrome, D. Fetal macrosomia",D: Fetal macrosomia,"Answer: D: Fetal macrosomia Explanation: Ans. d. Fetal macrosomia (Ref- Dutta6/e , pg227)Fetal macrosomia is the least likely complication of pregnancy-induced hypeension among the options provided." "This is a real-world medical entrance exam question, please give the true answer based on the question and selection. Topic name: None, Subject name: Dental","Good contacts in primary teeth with lack of spacing predicts? Select and explain. A. Normal occlusion in permanent teeth, B. Crowding in permanent teeth, C. Anterior cross bite, D. Spaced permanent dentition",B: Crowding in permanent teeth,"Answer: B: Crowding in permanent teeth Explanation: None" "This is a real-world medical entrance exam question, please give the true answer based on the question and selection. Topic name: None, Subject name: Dental","Determining level of fluoride in community water fluoridation programme depends on: Select and explain. A. Mean annual temperature of the place, B. Economic factors, C. Average weight of children of the area, D. Altitude of area above sea level",A: Mean annual temperature of the place,"Answer: A: Mean annual temperature of the place Explanation: None" "This is a real-world medical entrance exam question, please give the true answer based on the question and selection. Topic name: None, Subject name: Skin","A 35 years old male comes with complain of baldness. On examination, well-defined bald patches were seen with no scarring. Small broken hairs were seen in the surrounding area. What is the likely diagnosis? Select and explain. A. Androgenetic alopecia, B. Alopecia areata, C. Anagen effluvium, D. Telogen Effluvium",B: Alopecia areata,"Answer: B: Alopecia areata Explanation: Ans: B. Alopecia areata(Ref Rooks 8/e p66.13, Fitzpatrick 6/e p732)Findings are highly suggestive of alopecia areata.The scalp appears normal in alopecia areata.In affected areas, anagen is abruptly terminated prematurely and affected hairs move prematurely into telogen, with resultant often precipitous hair shedding.The near pathognomonic 'exclamation point' hairs may be present, paicularly at the periphery of areas of hair loss.These sho broken hairs, whose distal ends are broader than the proximal ends, illustrate their inherent sequence of events: follicular damage in anagen and then a rapid transformation to telogen. White or graying hairs are, frequently spared and probably account, in cases offulminant alopecia areata, far the mysterious phenomenon of 'going gray overnight.'" "This is a real-world medical entrance exam question, please give the true answer based on the question and selection. Topic name: None, Subject name: Dental","Incomplete casting is the result of Select and explain. A. More porous investment in the mold, B. Narrow sprue diameter, C. Hollow sprue pin, D. Large reservoir",B: Narrow sprue diameter,"Answer: B: Narrow sprue diameter Explanation: None" "This is a real-world medical entrance exam question, please give the true answer based on the question and selection. Topic name: None, Subject name: Dental","The occlusal clearance of an onlay is best detected by: Select and explain. A. Depth cuts, B. Visual inspection, C. Wax chew-in, D. Articulating paper",C: Wax chew-in,"Answer: C: Wax chew-in Explanation: None" "This is a real-world medical entrance exam question, please give the true answer based on the question and selection. Topic name: None, Subject name: Social & Preventive Medicine","The probability of a test detecting a truly positive person from the population of diseased is the Select and explain. A. Specificity of the test, B. Positive predictive value of the test, C. Sensitivity of the test, D. Likelihood ratio",C: Sensitivity of the test,"Answer: C: Sensitivity of the test Explanation: Sensitivity of a screening test detects: true positives among all diseased. Specificity of a screening test detects: true negatives among all healthy. PPV detects: true positives among all those who are positive on a screening test NPV detects: true negatives among all those who are negative on a screening test." "This is a real-world medical entrance exam question, please give the true answer based on the question and selection. Topic name: None, Subject name: Dental","Chronic mercury toxicity results from: Select and explain. A. Inhalation while removing old amalgam, B. Skin contact, C. Mercury vapour, D. All of the above",D: All of the above,"Answer: D: All of the above Explanation: All the options given in the question lead to mercury toxicity. Methyl mercury is the most common form that is transformed by natural processes; it is a more toxic form than ethyl mercury (C2H5Hg+) or elemental mercury. Methyl mercury is a major safety concern because it bioaccumulates through the food chain; its pharmacokinetic half-life is longer (1.5 to 3 months) than that of ethyl mercury (less than 1 week). In air, the concentrations of mercury range from about 0.01 to 0.02 µg/m3. Mercury accumulates in the kidneys. In the brain, metallic mercury can be converted to an inorganic form that is retained in the brain. Elemental mercury and mercury vapor have a half-life of 1 to 3 months. Mercury leaves the body by excretion through urine and feces. Exposure to high levels of mercury can injure the brain, kidneys, and developing fetus. Chronic mercury toxicity may be manifested as tremors; memory loss; and changes in personality, vision, and hearing. Reference: PHILLIPS’ SCIENCE OF DENTAL MATERIALS, 12th ed page no 124" "This is a real-world medical entrance exam question, please give the true answer based on the question and selection. Topic name: None, Subject name: Dental","Suitable technique used for diagnosis of posterior interproximal caries in children Select and explain. A. Bite wing with Bisecting angle technique, B. Bite wing with Paralleling technique, C. Bite wing with RVG, D. Panoramic Radiography",C: Bite wing with RVG,"Answer: C: Bite wing with RVG Explanation: None" "This is a real-world medical entrance exam question, please give the true answer based on the question and selection. Topic name: None, Subject name: Dental","Dentin is composed of Select and explain. A. 70% inorganic, B. 30% inorganic, C. 50% inorganic, D. 96% inorganic",A: 70% inorganic,"Answer: A: 70% inorganic Explanation: None" "This is a real-world medical entrance exam question, please give the true answer based on the question and selection. Topic name: None, Subject name: Dental","The optimal time to employ an orthodontic appliance that takes advantage of growth is during: Select and explain. A. Late mixed dentition, B. Early mixed dentition, C. Late primary dentition, D. Early permanent dentition",A: Late mixed dentition,"Answer: A: Late mixed dentition Explanation: None" "This is a real-world medical entrance exam question, please give the true answer based on the question and selection. Topic name: None, Subject name: Dental","Guiding planes on abutment teeth provide: Select and explain. A. Predictable clasp retention, B. Increased resistance towards horizontal movement, C. Transmit forces along long axis of tooth, D. Guides the key into the key way",A: Predictable clasp retention,"Answer: A: Predictable clasp retention Explanation: None" "This is a real-world medical entrance exam question, please give the true answer based on the question and selection. Topic name: None, Subject name: Biochemistry","One of the following is not an amino acid: Select and explain. A. Glycine, B. Hydroxy proline, C. Glutamic acid, D. Choline",D: Choline,"Answer: D: Choline Explanation: None" "This is a real-world medical entrance exam question, please give the true answer based on the question and selection. Topic name: None, Subject name: Dental","Square knot is composed of: Select and explain. A. Two single loops thrown in opposite direction, B. Two double loops thrown in opposite direction, C. Two single loops thrown in same direction, D. Two double loops thrown in Same direction",A: Two single loops thrown in opposite direction,"Answer: A: Two single loops thrown in opposite direction Explanation: None" "This is a real-world medical entrance exam question, please give the true answer based on the question and selection. Topic name: None, Subject name: Dental","Largest incisal embrasure seen in Select and explain. A. Between upper Lateral incisor and Canine, B. Between lower Central incisor and Lateral incisor, C. Between upper canine and premolar, D. Between lower canine and premolar",A: Between upper Lateral incisor and Canine,"Answer: A: Between upper Lateral incisor and Canine Explanation: Largest embrasure Incisal embrasure : between upper LI and Canine Occlusal embrasure: between upper canine and 1st premolar" "This is a real-world medical entrance exam question, please give the true answer based on the question and selection. Topic name: AIIMS 2019, Subject name: Physiology","Patient's lab value of Na+ 137, K+4, Urea 54, Bicarbonate 15, Phosphate 3, Chloride 110, Calcium 10. Calculate the Anion gap? Select and explain. A. 12, B. 16, C. 10, D. 8",A: 12,"Answer: A: 12 Explanation: Anion gap - All Cations can be measured but some of the anions can't be measured, this creates a viual GAP, called as - Anion gap Measured by: - (Na+) + (unmeasured cations) = (CL-) + (HCO-3) + Unmeasured anions - Since cations do not remain unmeasured, all cations gets measured, therefore :- Anion gap = Na minus - - - 137 - 125 - 12" "This is a real-world medical entrance exam question, please give the true answer based on the question and selection. Topic name: None, Subject name: Anatomy","Which of the following is not true regarding trigeminal nerve? Select and explain. A. Roots of trigeminal nerve are attached to Pons, B. It has three main divisions, C. Trigeminal nerve supplies the dura mater of middle cranial fossa, D. The trigeminal ganglion contributes to the sensory root only",D: The trigeminal ganglion contributes to the sensory root only,"Answer: D: The trigeminal ganglion contributes to the sensory root only Explanation: None" "This is a real-world medical entrance exam question, please give the true answer based on the question and selection. Topic name: None, Subject name: Anaesthesia","Which of the following intravenous anesthetic agents is contraindicated in epileptic patients posted for general anaesthesia Select and explain. A. Ketamine, B. Thiopentone, C. Propofol, D. Midazolam",A: Ketamine,"Answer: A: Ketamine Explanation: Ans. a. KetamineKetamine should be avoided in patients with history of seizures as it fuher increases ICP and also causes delirium and hallucinations. Contraindications of Ketamine:Head injury, intracranial space occupying lesion, eye injuryQ (increases ICT, IOT)Ischemic hea disease, vascular aneurysm and hypeensionQ (increases myocardial oxygen demand and hypeension)Psychiatric diseases and drug addictsdeg (more incidence of hallucination and emergence reaction)" "This is a real-world medical entrance exam question, please give the true answer based on the question and selection. Topic name: None, Subject name: Pathology","Which of the following is most common developmental cyst Select and explain. A. Nasopalatine cyst, B. Naso-alveolar cyst, C. Globulomaxillary cyst, D. Median palatal cyst",A: Nasopalatine cyst,"Answer: A: Nasopalatine cyst Explanation: None" "This is a real-world medical entrance exam question, please give the true answer based on the question and selection. Topic name: None, Subject name: Pathology","What does the red cell distribution width represents? Select and explain. A. Anisocytosis, B. Poikilocytosis, C. Level of hypochromia, D. Anisochromia",A: Anisocytosis,"Answer: A: Anisocytosis Explanation: Answer- A. Anisocytosis Red cell distribution width (RDW) is a parameter that measures variation in red blood cell size or red blood cell volume.RDW is elevated in accordance with variation in red cell size (anisocytosis).The ""width"" in RDW refers to the width of the volume curve (distribution width, here presented as the Coefficient of Variation, or CV), not the width of the cells.Anisocytosis is a medical term meaning that a patient's red blood cells are of unequal size.Poikilocytosis is variation in cell shape: poikilocytes may be oval, teardrop-shaped, sickle-shaped or irregularly contracted.Anisochromia is a marked variability in the color density of erythrocytes (red blood cells), which indicates unequal hemoglobin content among the red blood cells.Hypochromia means that the red blood cells have less color than normal when examined under a microscope." "This is a real-world medical entrance exam question, please give the true answer based on the question and selection. Topic name: None, Subject name: Anatomy","Sublingual salivary gland lies: Select and explain. A. Superior to mylohyoid, B. Inferior to mylohyoid, C. Deep to genioglossus, D. Deep to geniohyoid",A: Superior to mylohyoid,"Answer: A: Superior to mylohyoid Explanation: Sublingual salivary gland: This is smallest of the three pairs of large salivary glands.  It lies in the floor of the mouth between the mucus membrane and superior to mylohyoid muscle.  It is almond shaped and rests in the sublingual fossa of the mandible.  It is separated from the base of the tongue by the submandibular duct.  It is mostly mucus in nature and weighs about 3–4 g.  The gland pours its secretion by a series of ducts, about 15 in number, into the oral cavity on the sublingual fold, but a few of them open into the submandibular duct. Key Concept: Sublingual salivary gland is present superior to mylohyoid muscle." "This is a real-world medical entrance exam question, please give the true answer based on the question and selection. Topic name: None, Subject name: Pediatrics","All of the following are sequelae of fetal alcohol syndrome except: Select and explain. A. Macrocephaly, B. Holoprosencephaly, C. Microcephaly, D. Thinning of corpus callosum",A: Macrocephaly,"Answer: A: Macrocephaly Explanation: Answer--A. MacrocephalyFetal alcohol syntlrome is ussociated with microcephuly rather lhan macntcephaly. Holoprosencephaly can occur in extreme forms of fetal alcohol syndrome along with midline hypoplasia.""Fetal Alcohol Syndrome (FAS) No single malformation or characteristic malformation complex has been described in few cases of FAS in wihch neuropathology has been repoed. The neuropathologic changes are varied and nonspecitic and include microcephaly, hydrocephalus, leptomeningeal, white matter and periventricalar neuroglial heterotopias, agenesis of corpas callosum and the cerebellar vermis, incomplete holoprosencephaly, and neural tube defects" "This is a real-world medical entrance exam question, please give the true answer based on the question and selection. Topic name: None, Subject name: Physiology","Which of the following nucleus has cardio inhibitory function? Select and explain. A. Nucleus Ambiguus, B. Nucleus Tractus Solitarius, C. Rostral Ventrolateral Medulla, D. Dorsal motor nucleus of vagus",A: Nucleus Ambiguus,"Answer: A: Nucleus Ambiguus Explanation: Ans. A. Nucleus AmbiguusThe nucleus ambiguus in its ""external formation"" contains cholinergic preganglionic parasympathetic neurons for the hea.These neurons are cardioinhibitory.This cardioinhibitory effect is one of the means by which quick changes in blood pressure are achieved by the central nervous system (the primary means being changes in sympathetic nervous system activity, which constricts aerioles and makes the hea pump faster and harder)." "This is a real-world medical entrance exam question, please give the true answer based on the question and selection. Topic name: None, Subject name: Physiology","Gut associated lymphoid tissue (GALT) is primarily located in Select and explain. A. Lamina propria, B. Submucosa, C. Muscularis, D. Serosa",A: Lamina propria,"Answer: A: Lamina propria Explanation: Apart from its other functions, gastrointestinal tract is a lymphoid organ. The lymphoid tissue within it is collectively known as Gut Associated Lymphoid Tissue, known as GALT. Gut Associated Lymphoid Tissue (GALT) is primarily located in lamina propria.  It may be present diffusely or as solitary or aggregated nodules, known as Peyer's patches in the small intestine." "This is a real-world medical entrance exam question, please give the true answer based on the question and selection. Topic name: None, Subject name: Dental","Which of the following is rare in children? Select and explain. A. Herpangina, B. Dental cysts, C. Cherubism, D. Small pox",B: Dental cysts,"Answer: B: Dental cysts Explanation: None" "This is a real-world medical entrance exam question, please give the true answer based on the question and selection. Topic name: None, Subject name: Dental","In a patient with acute pulpitis, it is difficult for the patient to locate the pain. This is because? Select and explain. A. No nociceptors are present in the pulp, B. Less proprioceptors are present in the pulp, C. Patient cannot speak, D. Patient is not cooperative",B: Less proprioceptors are present in the pulp,"Answer: B: Less proprioceptors are present in the pulp Explanation: None" "This is a real-world medical entrance exam question, please give the true answer based on the question and selection. Topic name: None, Subject name: Anatomy","In an adult, the spinal cord ends at the level of: Select and explain. A. Lower border of L1, B. L2, C. L3, D. L4",A: Lower border of L1,"Answer: A: Lower border of L1 Explanation: In an adult, the terminal end of the spinal cord, called the conus medullaris, ends at about the L1-L2 level of the spinal column and the lumbar and sacral nerve roots descend as the cauda equina below this level to the lower lumbar and sacral exit neuroforamina. Spinal cord in adult  ends at lower border  of lumbar one vertebra. Spinal dura mater and arachnoid  mater extend till sacral two vertebra. Spinal pia mater comprises  an outer epi-pia  and an inner pia-intima. Reference: B D Chaurasia’s Head and Neck anatomy 6th ed page no 187" "This is a real-world medical entrance exam question, please give the true answer based on the question and selection. Topic name: None, Subject name: Pediatrics","Which of the following is the least important prognostic factor in congenital diaphragmatic hernia ? Select and explain. A. Pulmonary Hypertension, B. Delay in emergent surgery, C. Size of defect, D. Gestational age at diagnosis",B: Delay in emergent surgery,"Answer: B: Delay in emergent surgery Explanation: Timing of surgical repair has gradually shifted from an emergency repair, to a policy of stabilization using a variety of ventilatory strategies prior to operation. • Current recommendation is to adopt a conservative approach and delay surgical repair of the CDH until the infant stablizes from a hemodynamic and respiratory point of view. Prognostic factors in CDI-1 Prognostic factors can be divided into: (A) Primary prognostic factors (Pathophysiological) These are the most important prognostic factor which affect morbidity and mortility. These factors are: Pulmonary hypoplasia (most important) Pulmonary hypertension (2nd most important) (B) Secondary or relative predictors (diagnostic/clinical) These poor prognostic factors are: (a)      tinatal Detection at an early gestational age (<24 wks) Associated extradiaphragmatic congenital anomalies Liver herniation into thorax (Liver above diaphragm) Stomach herniationinto thorax (Stomach above diaphragm) Presence of polyhydraminios Small lung to head circumference ratio (LI-IR ratio) Small fetal abdominal circumference (< than 5th percentile) (b) Postnatal Early age (< 6 hours) of presentation PG, and PCO, unresponsive to ventilation Need for extracorporeal membrane oxygenation (ECM0) (c) Side of defect (i) Right sided defect" "This is a real-world medical entrance exam question, please give the true answer based on the question and selection. Topic name: None, Subject name: Dental","Currently used step back technique is modified, it was given by: Select and explain. A. Clam, B. Wein, C. Schilder, D. Mullaney",D: Mullaney,"Answer: D: Mullaney Explanation: None" "This is a real-world medical entrance exam question, please give the true answer based on the question and selection. Topic name: None, Subject name: Biochemistry","Which of the following does not or permissive euchromatin due to changes occurring at cytosine residues at CpG islands in DNA? Select and explain. A. Methylation, B. Alkylation, C. Phosphorylation, D. Sumoylation",A: Methylation,"Answer: A: Methylation Explanation: Ans: A. Methylationref: Harper's illustrated biochemistry, 30th editon., pg. 560.Methylation of cpG sites in the promoter of a gene may inhibit gene expression.There is also evidence that low folate status results in impaired methylation of cpG islands in DNA, which is a factor in the development of colorectal and other cancers." "This is a real-world medical entrance exam question, please give the true answer based on the question and selection. Topic name: None, Subject name: Dental","Which of the following to reduce for occlusal equilibration: Select and explain. A. Upper buccal, B. Lower buccal, C. Upper lingual, D. Both AB",A: Upper buccal,"Answer: A: Upper buccal Explanation: None" "This is a real-world medical entrance exam question, please give the true answer based on the question and selection. Topic name: None, Subject name: Dental","Upper 1st premolar resembles upper 2nd premolar from which aspect Select and explain. A. Buccal, B. Occlusal, C. Lingual, D. Proximal",C: Lingual,"Answer: C: Lingual Explanation: None" "This is a real-world medical entrance exam question, please give the true answer based on the question and selection. Topic name: None, Subject name: Dental","Embrasure characterized by a slight to moderate recession of interdental papilla are: Select and explain. A. Type I, B. Type II, C. Type III, D. Type IV",B: Type II,"Answer: B: Type II Explanation: None" "This is a real-world medical entrance exam question, please give the true answer based on the question and selection. Topic name: None, Subject name: Dental","In Ricketts esthetic plane, lower lip rests: Select and explain. A. 1 mm anterior to plane, B. On the plane, C. 2 mm posterior to plane, D. 1 mm posterior to plane",C: 2 mm posterior to plane,"Answer: C: 2 mm posterior to plane Explanation: Ricketts’ E line: R. M. Ricketts suggested using an esthetic (E) plane to evaluate the relationship between the nose, lips and chin. Ricketts’ E line is drawn from the tip of the nose to the soft tissue chin. A quick method to look at one profile is to imagine a line tangent from the lower chin to the nose tip. Cephalometrically, Ricketts’ E line is drawn from the tip of the nose to the soft tissue chin. Normal values suggest that the upper lip is 4 mm behind the E line, while the lower lip lies 2 mm behind this reference line. It is important to mention that, this reference line is influenced a great deal by the growth of the nose and also varies with age and sex. Ricketts recommended that lip position should be analyzed using the nose–chin line as a reference point. These values are for Caucasians and clearly cannot be applied to all races. Ref: Orthodontics: Diagnosis and Management of Malocclusion and Dentofacial Deformities, 3e, Om P. Kharbanda pdf no 1097" "This is a real-world medical entrance exam question, please give the true answer based on the question and selection. Topic name: None, Subject name: Dental","Which of the following may result if an excessive amount of monomer is incorporated into an acrylic resin mixture? Select and explain. A. Excessive expansion, B. Low impact strength, C. Excessive shrinkage, D. Excessive brittleness",C: Excessive shrinkage,"Answer: C: Excessive shrinkage Explanation: Curing shrinkage arises as the monomer is converted to polymer and the free space it occupies reduces (approximately 20% less than that among unreacted monomers). Reference: PHILLIPS’ SCIENCE OF DENTAL MATERIALS, 12th ed page no 293" "This is a real-world medical entrance exam question, please give the true answer based on the question and selection. Topic name: None, Subject name: Dental","To prevent porosity in dental porcelain it should be baked: Select and explain. A. In presence of air, B. In vacuum, C. For long period, D. Under pressure",B: In vacuum,"Answer: B: In vacuum Explanation: To prevent porosity in dental porcelain, it should be baked in vacuum." "This is a real-world medical entrance exam question, please give the true answer based on the question and selection. Topic name: AIIMS 2018, Subject name: Gynaecology & Obstetrics","Treatment of simple endometrial hyperplasia with atypia in 45 yr female: Select and explain. A. Hysterectomy, B. Progestin, C. Mirena, D. Postmenopausal women",A: Hysterectomy,"Answer: A: Hysterectomy Explanation: Risk of malignancies Management Simple Hyperplasia without Atypia- 1% Complex Hyperplasia without Atypia- 3% Progestin Therapy Simple Hyperplasia with Atypia- 8% Complex Hyperplasia without Atypia - 29% Hysterectomy Hysterectomy is the best treatment at any age with atypical endometrial hyperplasia because of the risk of invasive cancer." "This is a real-world medical entrance exam question, please give the true answer based on the question and selection. Topic name: None, Subject name: Pediatrics","Which of the following drugs is used for the treatment of refractory histiocytosis : Select and explain. A. High dose methotrexate, B. High dose cytarabine, C. Cladribine, D. Fludrabine",C: Cladribine,"Answer: C: Cladribine Explanation: Treatment of LCH : 1. Single system disease  : The clinical course of single system disease is usually benign with a high chance of spontaneous remission. Therefore, treatment should be minimal and should be directed at arresting the progression of a bone lesion that could result in permanent damage before it resolve spontaneously. Curettage or less often low dose local radiotherapy may accomplish this. 2. Multisystem disease : It should be treated with systemic multiagent chemotherapy. Any one, of following two should be included in regimen :  i) Etoposide   ​ii) Vinblastin For unresponsive disease following therapies are available : i) Cyclosporine / antithymocyte globulin. ii) Imatinib. iii) 2-chlorodeoxyadenosine (Cladribine). iv) Stem cell transplantation." "This is a real-world medical entrance exam question, please give the true answer based on the question and selection. Topic name: None, Subject name: Dental","The highest (longest) and sharpest cusp on the deciduous mandibular first molar is: Select and explain. A. Mesiolingual, B. Distolingual, C. Mesiobuccal, D. Distobuccal",A: Mesiolingual,"Answer: A: Mesiolingual Explanation: None" "This is a real-world medical entrance exam question, please give the true answer based on the question and selection. Topic name: None, Subject name: Dental","Calcification of roots of deciduous teeth is completed by: Select and explain. A. 2 years, B. 4 years, C. 6 years, D. 8 years",B: 4 years,"Answer: B: 4 years Explanation: None" "This is a real-world medical entrance exam question, please give the true answer based on the question and selection. Topic name: None, Subject name: Social & Preventive Medicine","To evaluate post-operative vision effects after cataract extraction surgery under PCB, which of the following is used? Select and explain. A. Active surveillance, B. Sentinel surveillance, C. Passive surveillance, D. Routine checkup of all operated cases",D: Routine checkup of all operated cases,"Answer: D: Routine checkup of all operated cases Explanation: Answer- D. Routine checkup of all operated casesTb evaluates postoperative vision effects after cataract extraction surgery under NPCB, routine checkup of all operated cases is used.Strategies under National Programme for Control of Blindness (NPCB)Improving follow-up services of cataract operated patients and treating other causes of epidemics like glaucomaShift eye camp approval to a fixed surgical approach and from conventional surgery to IOL installation for better quality of postoperative vision to operated patients.Expand World Bank Project activities like construction of dedicated eye operation theatres, eye wards at district level, training of other ophthalmic equipment's to the work community.To strengthen paicipants of voluntary organizations in the programme.To enhance the coverage of eye care services in the tribal and underserved areas.Screening of refractive errors in school going children." "This is a real-world medical entrance exam question, please give the true answer based on the question and selection. Topic name: None, Subject name: Forensic Medicine","An adult came to casualty with complaints of rapid hea rate. On examination everything else was normal except for episodic tachycardia and occasional extra-systole and amblyopia. Which of the following is the cause of it? Select and explain. A. Nicotine, B. Cannabis, C. Atropine, D. Cocaine",A: Nicotine,"Answer: A: Nicotine Explanation: Answer- A. NicotineNicotine chronic poisoning can lead to cough, wheezing, dyspnoea, anorexia, vomiting, diarrhea, anaemia, faintness,tremors, impaired memory, amblyopia, blindness, irregularities, of the hea with extra-systoles and occasionally attacks pain suggesting angina pectoris." "This is a real-world medical entrance exam question, please give the true answer based on the question and selection. Topic name: None, Subject name: Dental","In which one of the following mixed dentition analysis of deciduous dentition, there is no use of radiographs? Select and explain. A. Carey's analysis, B. Moyer's analysis, C. Nance Carey's analysis, D. Pont's index",B: Moyer's analysis,"Answer: B: Moyer's analysis Explanation: Moyer’s analysis  This mixed dentition analysis utilises Moyer’s prediction tables. Prediction is based on the premise that there is a reasonably good correlation between the size of erupted permanent incisors and the unerupted canines and premolars. It is a non-radiographic method. The main advantage of non-radiographic prediction methods is that they can be performed by measuring the erupted mandibular incisor(s) without the need of additional measurements from radiographs. Reference: Orthodontics: Diagnosis and Management of Malocclusion and Dentofacial Deformities 3rd O P Kharbanda, pdf no 841" "This is a real-world medical entrance exam question, please give the true answer based on the question and selection. Topic name: None, Subject name: Physiology","During exercise: Select and explain. A. Cerebral bood flow increases if there is increase in systolic blood pressure, B. Body temperature increases, C. Blood flow to muscle increases after 1½ minute, D. Lymphatic flow from muscle decreases",B: Body temperature increases,"Answer: B: Body temperature increases Explanation: None" "This is a real-world medical entrance exam question, please give the true answer based on the question and selection. Topic name: AIIMS 2018, Subject name: Surgery","An intubated patient with eye opening to pain with abnormal flexion. What is the GCS score? Select and explain. A. E2VNTM3, B. E2VTM3, C. E2V1M3, D. E2VTM4",A: E2VNTM3,"Answer: A: E2VNTM3 Explanation: Revised Glasgow Coma Scale 2014 Revised GCS (2014) Eye Opening Verbal Response Best Motor Response(M) Spontaneous 4 Oriented 5 Obeying commands 6 To speech 3 Confused 4 Localizing 5 To pressure 2 Words 3 Normal flexion(withdraw) 4 None 1 Sounds 2 Abnormal flexion 3 None 1 Extension 2 None 1 GCS specifically recommends avoiding sternal rubs as it causes bruising & responses can be difficult to interpret. They also do not recommend routine use of retromandibular pressure. Revised GCS (2014) changes are highlighted in the above table Best predictor of outcome: Motor response Repoing of non-testable score Aspects: In case of a non-testable aspect, the new GCS should be marked as NT, for ""Not teste stable"" For intubated patients or patients with tracheostomy, VNT is used it is no longer recommend to assign 1 point to non-testable elements, therefore a combined score should not be used." "This is a real-world medical entrance exam question, please give the true answer based on the question and selection. Topic name: None, Subject name: Surgery","In a male after laparoscopic cholecystectomy, the specimen is sent for histopathology which shows carcinoma gallbladder stage T1a. What is the most appropriate management in this patient? Select and explain. A. Conservative and follow up., B. Extended cholecystectomy, C. Simple cholecystectomy, D. Radiotherapy",C: Simple cholecystectomy,"Answer: C: Simple cholecystectomy Explanation: With the finding of carcinoma following cholecystectomy, subsequent treatment depends on the depth of penetration of the gallbladder wall and surgical margins.With T1a lesions, in which the carcinoma penetrates the lamina propria but does not invade the muscle layer, simple cholecystectomy should suffice for therapy." "This is a real-world medical entrance exam question, please give the true answer based on the question and selection. Topic name: None, Subject name: Pharmacology","Mechanism of action of chlortetracycline: Select and explain. A. Interfere with metabolism, B. Inhibition of protein synthesis, C. Inhibition of cell wall formation, D. Interfere with DNA function",B: Inhibition of protein synthesis,"Answer: B: Inhibition of protein synthesis Explanation: None" "This is a real-world medical entrance exam question, please give the true answer based on the question and selection. Topic name: None, Subject name: Medicine","A glass factory worker presented with complaints of numbness in hands and feet, generalized weakness and constipation. Radiograph showed linear lines on metaphyses of knee and wrist joints. How will you diagnose this patient? Select and explain. A. Serum mercury levels, B. Vitamin D levels, C. RBCcholinesterase levels, D. Amino levulinic acid levels in urine",D: Amino levulinic acid levels in urine,"Answer: D: Amino levulinic acid levels in urine Explanation: Ans: D. Amino levulinic acid levels in urine(Ref: Harrison 19/e p2689, 427e-2; Harper 30/e p329)High levels of lead can affect bone metabolism by combining with SH groups in enzymes such as ferrochelatase and ALA (delta-amino levulinic acid) dehrdratase.Elevated levels of protoporphyrin are found in red blood cells, and elevated levels of ALA and of coproporphyrin are found in urine.Lead Poisoning:Abdominal pain, irritability, lethargy, anorexia, anemia, Fanconi's syndrome, pyuria, azotemia in children with blood lead level (BPb) >80 i.g/ dL; may also see epiphyseal plate ""lead lines"" on long bone x-rays." "This is a real-world medical entrance exam question, please give the true answer based on the question and selection. Topic name: None, Subject name: Surgery","In symphyseal fracture with lag screw fixation? Select and explain. A. Load bearing, B. Load sharing, C. Adaptation osteosynthesis, D. Compression osteosynthesis",B: Load sharing,"Answer: B: Load sharing Explanation: None" "This is a real-world medical entrance exam question, please give the true answer based on the question and selection. Topic name: None, Subject name: Dental","The highest critical surface tension is for which of the orthodontic brackets? Select and explain. A. Stainless steel, B. Polycarbonate, C. Ceramic alumina, D. Titanium",A: Stainless steel,"Answer: A: Stainless steel Explanation: None" "This is a real-world medical entrance exam question, please give the true answer based on the question and selection. Topic name: None, Subject name: Surgery","Non-visualization of gallbladder in hepatic scintigraphy is suggestive of: Select and explain. A. Chronic cholecystitis, B. Carcinoma gallbladder, C. Acute cholecystitis due to gallstones, D. Gallstones obstructing CBD",C: Acute cholecystitis due to gallstones,"Answer: C: Acute cholecystitis due to gallstones Explanation: Ans: C. Acute cholecystitis due to gallstones(Ref. Sabiston 20/e p1488, 19/e pi 462, Schwuz 10/e p1315, 1320, Bailey 27/c 1)1192. 26/ep1101. Hlumgari 5/e p254-270: Shackelford 7/e p 1306)Non-visualization of gallbladder in hepatic scintigraphy is suggestive of Acute cholecystitis.HIDA Scan:Allows visualization of biliary tree & gall bladder.GB visualized within 30 min of isotope injection in 90% of normal individuals and within 1 hour in the remainder.Bowel is usually seen within 1 hour in majority of patientse.Non-visualization of GB:Suggestive of acute cholecystitis.(Sensitivity & specificity-95%).Delayed/reduced visualization of GB:Suggestive of chronic cholecystitis.Due to contracted gall bladderBiliary scintigraphy:Helpful in diagnosing bile leaks & iatrogenic biliary obstruction.Confirms presence & quantifies leakage." "This is a real-world medical entrance exam question, please give the true answer based on the question and selection. Topic name: None, Subject name: Dental","Bismuth Oxide incorporated in MTA results in grey discolouration of crown. In newer material this Bismuth Oxide is replaced with? Select and explain. A. Terbium-Dysisopropium, B. Tantalam Oxide, C. Bismuth Carbonate, D. Titanium Oxide",B: Tantalam Oxide,"Answer: B: Tantalam Oxide Explanation: None" "This is a real-world medical entrance exam question, please give the true answer based on the question and selection. Topic name: None, Subject name: Biochemistry","All are true about CRISPR cas 9 Except Select and explain. A. gRNA ensures that the Cas9 enzyme cuts at the right point in the genome, B. NHEJ repair & HDR pathway, C. C9 enzyme is used in CRISPR gene editing, D. All of these",D: All of these,"Answer: D: All of these Explanation: Answer-D. All of theseCRISPR-Cas9 was adapted from a naturally occurring genome editing system that can generate double standard Breaks (DSBs).Cas9 is the enzyme that is used most often, other enzymes (for example Cpf1) can also be used- these enzymes can can repair by homologus repair system or by non-homologous end joining in the absence of DNA template.Two general repair pathways:The efficient but error-prone non-homologous end joining (NHEJ) pathwayThe less efficient but high-fidelity homology directed repair (HDR) pathway" "This is a real-world medical entrance exam question, please give the true answer based on the question and selection. Topic name: None, Subject name: Pharmacology","Centrineuraxial (spinaland epidural) anaesthesia is not contraindicated in Select and explain. A. Platelets <80,000, B. Patient on aspirin, C. Patient on oral anticoagulants, D. Raised intracranial pressure",B: Patient on aspirin,"Answer: B: Patient on aspirin Explanation: None" "This is a real-world medical entrance exam question, please give the true answer based on the question and selection. Topic name: None, Subject name: Medicine","In Prinzmetal’s angina Select and explain. A. Generally Right Coronary Artery presents as inferior wall MI, B. Coronary artery spasm most commonly occurs distal to atherosclerotic area., C. No changes in ECG, Q wave become normal after few hours., D. All of the above",D: All of the above,"Answer: D: All of the above Explanation: Pain at rest. Variant may hence ST↓. An inferior wall myocardial infarction — also known as IWMI, or inferior MI, or inferior ST segment elevation MI, or inferior STEMI — occurs when inferior myocardial tissue supplied by the right coronary artery, or RCA, is injured due to thrombosis of that vessel. LAD (Left Anterior Descending Artery) may be involved. ECG no change. Wave become normal after few hours. Coronary artery may be normal. Coronary artery Spasm most commonly occurs distal to the atherosclerotic area." "This is a real-world medical entrance exam question, please give the true answer based on the question and selection. Topic name: None, Subject name: Anatomy","Foramen magnum transmits all except: Select and explain. A. Vertebral artery, B. Spinal branch of 10th nerve, C. Spinal cord, D. Both option 2 and 3",D: Both option 2 and 3,"Answer: D: Both option 2 and 3 Explanation: Structures passing through foramen magnum are: Through anterior part Apical ligament of dens Membrana tectoria Through subarachnoid space Spinal accessory nerve Vertebral arteries Anterior and posterior spinal arteries Through posterior part Lower part of medulla Tonsils of cerebellum Meninges" "This is a real-world medical entrance exam question, please give the true answer based on the question and selection. Topic name: None, Subject name: Dental","Most appropriate method of isolation during tooth preparation in vital tooth: Select and explain. A. Cotton rolls, B. Salivary ejector, C. Isolation with rubber dam, D. Svedopter",C: Isolation with rubber dam,"Answer: C: Isolation with rubber dam Explanation: None" "This is a real-world medical entrance exam question, please give the true answer based on the question and selection. Topic name: None, Subject name: Dental","Of all the pulpal sensibility test, which is best in a patient who is restored with Porcelain fused to metal crown? Select and explain. A. Cold test, B. Heat test, C. EPT, D. Pulse oximetery",A: Cold test,"Answer: A: Cold test Explanation: None" "This is a real-world medical entrance exam question, please give the true answer based on the question and selection. Topic name: None, Subject name: Dental","Teeth selected for scoring of PHP-M (Personal Hygiene Performance Modified) are: Select and explain. A. Six, B. Twelve, C. Eighteen, D. Twenty four",A: Six,"Answer: A: Six Explanation: None" "This is a real-world medical entrance exam question, please give the true answer based on the question and selection. Topic name: None, Subject name: Gynaecology & Obstetrics","Treatment of postpartum hemorrhage is all except: Select and explain. A. Oxytocin, B. Syntometrine, C. Oestrogen, D. Prostaglandins",C: Oestrogen,"Answer: C: Oestrogen Explanation: None" "This is a real-world medical entrance exam question, please give the true answer based on the question and selection. Topic name: AIIMS 2018, Subject name: Physiology","Fibers reaching directly to Purkinje cell of cerebellum arise from which of the following: Select and explain. A. Vestibular nucleus, B. Inferior olivary nucleus, C. Raphe nucleus, D. Locus ceruleus",B: Inferior olivary nucleus,"Answer: B: Inferior olivary nucleus Explanation: Functional unit of cerebellum = Purkinje cell neuronal circuit of the Purkinje cell: - Afferent inputs of cerebellum Climbing fiber type Mossy fiber type Originate from inferior olives of the medulla. Enter cerebellum from multiple sources - cerebrum, brainstem &spinal cord. Send collaterals to excite deep nuclear cells & then proceed to synapse with granule cells. Output from - deep nuclear cell Excitatory influence from direct connection with cerebellum from the brain or periphery Inhibitory influence from Purkinje cell" "This is a real-world medical entrance exam question, please give the true answer based on the question and selection. Topic name: None, Subject name: Pediatrics","Fetal alcohol syndrome is characterized by all except – Select and explain. A. Microcephaly, B. Low intelligence, C. Large proportionate body, D. Septal defects of heart",C: Large proportionate body,"Answer: C: Large proportionate body Explanation: Fetal alcohol syndrome High level of alcohol ingestion in pregnancy can cause damage to fetus, known as fetal alcohol syndrome. The harmful effects may be due to alcohol itself or due to one of its breakdown products. Some evidence suggests that alcohol may impair placental transfer of essential amino acids and zinc, both necessary for protein synthesis, which may account for IUGR. Characterististics of fetal alcohol syndrome include : - HIG1? (not large proportionate body) Microcephaly Congenital heart defects (ASD, VS ) Mental retardation Facial abnormalities -3 Short palpebral fissures, epicanthal folds, maxillary hypoplasia, micrognathia, low set ears, smooth philthrum, thin smooth upper lip. Minor joint anomalies Hyperkinetic movements" "This is a real-world medical entrance exam question, please give the true answer based on the question and selection. Topic name: None, Subject name: Dental","A layman before introduction of fluoride in prevention of dental caries knew about it as a constituent used in Select and explain. A. Pesticide, B. Industrial use, C. Fertilizers, D. Irrigant",A: Pesticide,"Answer: A: Pesticide Explanation: None" "This is a real-world medical entrance exam question, please give the true answer based on the question and selection. Topic name: None, Subject name: Pediatrics","A child presents with abdominal colic and hematuria on ultrasonography a stone 2.5 cm in diameter is seen in the renal pelvis. The next step in management of this case is : Select and explain. A. Pyelolithotomy, B. Nephroureterostomy, C. Conservative, D. ESWL",D: ESWL,"Answer: D: ESWL Explanation: Although there are many methods of renal stone removal, the best is ESWL (extracorporeal shock-wave lithrotripsy]. A conservative approach is chosen only when the stone is upto 0.5 or 0.6 cm." "This is a real-world medical entrance exam question, please give the true answer based on the question and selection. Topic name: None, Subject name: Biochemistry","Which of the following is not used in polymerase chain reaction? Select and explain. A. DNA Polymerase, B. Taq polymerase, C. Dideoxyribonucleotides, D. DNA template",C: Dideoxyribonucleotides,"Answer: C: Dideoxyribonucleotides Explanation: PCR is an in vitro DNA amplification procedure in which millions of copies of a paicular sequence of DNA can be produced within a few hours.The reaction cycle has the following steps:Step 1: Separation (Denaturation): DNA strands are separated (melted) by heating at 95degC for 15 seconds to 2 minutes.Step 2: Priming (Annealing): The primers are annealed by cooling to 50degC for 0.5 to 2 minutes. The primers hybridize with their complementary single-stranded DNA produced in the first step.Step 3: Polymerization: New DNA strands are synthesized by Taq polymerase. This enzyme is derived from bacteria Thermus aquaticus that are found in hot springs.The steps of 1,2 and 3 are repeated. In each cycle, the DNA strands are doubled. Thus, 20 cycles provide for 1 million times amplifications. These cycles are generally repeated by automated instrument, called Tempcycler.5. After the amplification procedure, DNA hybridization technique or Southern blot analysis with a suitable probe shows the presence of the DNA in the sample tissue.Dideoxyribonucleotides are not used in the polymerase chain reaction." "This is a real-world medical entrance exam question, please give the true answer based on the question and selection. Topic name: None, Subject name: Dental","Imaginary line joining MB1, Distal and Palatal orifices in maxillary 1st molar forms Select and explain. A. Maxillary triangle, B. Molar Triangle, C. Molar quadrilateral, D. None of the above",B: Molar Triangle,"Answer: B: Molar Triangle Explanation: None" "This is a real-world medical entrance exam question, please give the true answer based on the question and selection. Topic name: None, Subject name: Gynaecology & Obstetrics","What is the approved dose of misoprostol in emergent management of postpaum hemorrhage? Select and explain. A. 200 mcg, B. 400 mcg, C. 600 mcg, D. 1000 mcg",C: 600 mcg,"Answer: C: 600 mcg Explanation: Ans: C. 600 mcg(Ref Williams 24/e p785)Approved dose of misoprostol in emergent management of postpaum hemorrhage = 600 pg.Misoprostol:Derman (2006) compared a 600 pg oral dose given at delivery against placebo.Drug decreased hemorrhage incidence from 12 to 6 percent & severe hemorrhage from 1.2 to 0.2 percent." "This is a real-world medical entrance exam question, please give the true answer based on the question and selection. Topic name: None, Subject name: Social & Preventive Medicine","Transmission assessment survey (TAS) is done in the following? Select and explain. A. To determine when infections have been reduced below these target thresholds, B. For assessing primary immunization coverage, C. To provide reliable estimates of bih rate, death rate and infant moality rate, D. All",A: To determine when infections have been reduced below these target thresholds,"Answer: A: To determine when infections have been reduced below these target thresholds Explanation: Ans.A. To determine when infections have been reduced below these target thresholdsWHO recommends the transmission assessment survey (TAS) to determine when infections have been reduced below these target thresholds and MDA can stop.Once MDA has stopped, TAS is used as a surveillance tool to determine that infection levels are sustained below target thresholds.Mass drug administration (MDA) is needed to reduce infection in the community to levels below a threshold at which mosquitoes are unable to continue spreading the parasites from person to person and new infections are prevented." "This is a real-world medical entrance exam question, please give the true answer based on the question and selection. Topic name: None, Subject name: Biochemistry","Autosomal dominant hypercholesterolemia type II is due to deficiency of: Select and explain. A. LDL receptor, B. Apoprotein B-100, C. Apoprotein C, D. Lipoprotein lipase",B: Apoprotein B-100,"Answer: B: Apoprotein B-100 Explanation: None" "This is a real-world medical entrance exam question, please give the true answer based on the question and selection. Topic name: None, Subject name: Biochemistry","Binding of Eukaryotic mRNA to Ribosomes is facilitated by Select and explain. A. Capping, B. Poly-A tail, C. tRNA, D. Shine-Dalgarno sequence",A: Capping,"Answer: A: Capping Explanation: Answer- A. Capping* In E. coli mRNA, a leader sequence upstream of the first AUG codon, called the Shine-Dalgarno sequence (also known as the ribosomal binding site AGGAGG), interacts through complementary base pairing with the rRNA molecules that compose the ribosome.* In eukaryotic initiation complex recognizes the 5' cap of the eukaryotic mRNA, then tracks along the mRNA in the 5' to 3' direction until the AUG sta codon is recognized. At this point, the 60S subunit binds to the complex of Met-tRNAi, mRNA, and the 40S subunit." "This is a real-world medical entrance exam question, please give the true answer based on the question and selection. Topic name: None, Subject name: Microbiology","A 5-year-old child from a rural area presented to the OPD with pustular lesions on the lower legs. The cuture from the lesion showed hemolytic colonies on the blood agar which were Gram-positive cocci. Which of the following reactions would help to provisionally confirm the diagnosis of group A streptococcal pyoderma? Select and explain. A. Optochin sensitivity, B. Bacitracin sensitivity, C. Catalase positivity, D. Bile solubility",B: Bacitracin sensitivity,"Answer: B: Bacitracin sensitivity Explanation: Answer: b. Bacitracin sensitivity (Ref Ananthanaravan 8/e pe p205-206)Gram-positive cocci with alpha hemolytic colonies on sheep agar are Streptococcus viridians and Streptococcus pneumoniae.They can be fuher differentiated on basis of optochin sensitivity, Streptococcus viridians-optochin resistantor Streptococcus pneumonia-optochin sensitive." "This is a real-world medical entrance exam question, please give the true answer based on the question and selection. Topic name: None, Subject name: Biochemistry","Which of the following amino acids does not include post-translational modification? Select and explain. A. Selenocysteine, B. Triiodothyronine, C. Hydroxyproline, D. Hydroxylysine",A: Selenocysteine,"Answer: A: Selenocysteine Explanation: Ans. A. SelenocysteinePeptidyl selenocysteine is not the product of a posttranslational modification, but is inseed directly into a growing polypeptide during translation. Selenocysteine is commonly termed as the ""21st amino acid."" However, incorporation of selenocysteine is specified by a large and complex genetic element for the unusual tRNA called tRNASec which utilizes the UGA anticodon that normally signals STOP." "This is a real-world medical entrance exam question, please give the true answer based on the question and selection. Topic name: None, Subject name: Physiology","Centre of activity of autonomic nervous system is: Select and explain. A. Midbrain, B. Cerebrum, C. Hypothalamus, D. Pons",C: Hypothalamus,"Answer: C: Hypothalamus Explanation: None" "This is a real-world medical entrance exam question, please give the true answer based on the question and selection. Topic name: None, Subject name: Dental","Undifferentiated mesenchymal cells found in pulp are: Select and explain. A. Multipotent, B. Totipotent, C. Polypotent, D. Monopotent",A: Multipotent,"Answer: A: Multipotent Explanation: Reserve Cells/Undifferentiated Mesenchymal Cells Undifferentiated mesenchymal cells are descendants of undifferentiated cells of dental papilla, which can dedifferentiate and then redifferentiate into many cell types. Depending on the stimulus, these cells may give rise to odontoblasts and fibroblasts. These cells are found throughout the cell-rich area and the pulp core and often are related to blood vessels. Key concept Multipotent cells are cells that have the capacity to self-renew by dividing and to develop into multiple specialized cell types present in a specific tissue or organ." "This is a real-world medical entrance exam question, please give the true answer based on the question and selection. Topic name: None, Subject name: Pediatrics","In postductal coarctation of the aorta, blood flow to the lower limb is maintained through which of the following arteries – Select and explain. A. Umblical artery and subcostal arteries, B. Thoracic and pericardiophrenic arteries, C. Intercostal arteries and superior epigastric artery, D. Ant and post circumflex arteries",C: Intercostal arteries and superior epigastric artery,"Answer: C: Intercostal arteries and superior epigastric artery Explanation: The coarctaion may be : Preductal - where is narrowing is proximal to the ductus arteriosus or ligamentum arteriosum. or Postductal - where the narrowing is distal to the ductus arteriosus or ligamentum arterosuin. Collateral circulation connects the proximal and the distal aspects of the vessels over time. This collateral circulation will develop mainly from the : - Subclavian,                   u Internal thoracic                                                             u Intercostal arteries u Axillary,                         u Superior and inferior epigastric and Following collateral connections may be seen : In the anterior thoracic wall, the internal mammary arteries (arising from subclavian arteries) and the epigastric arteries join to form collaterals which supply the abdominal wall and the lower extremities. Internal mammary artery gives rise to anterior intercostal arteries which forms anastomoses with the posterior intercostal (post intercostals arise from descending aorta). Superior epigastric artery is terminal branch of internal mammary artery and forms anastomoses with inferior epigastric artery which arises from external iliac artery. Musculophrenic artery is terminal branch of internal mammary artery and forms anastomoses with inferior plirenic and post intercostals. The Para scapular arteries (arising from subclavian artery and axillary arteries) connect with the posterior intercostal arteries to form collaterals which supply the distal aortic compartment. The suprascapular artery from the subclavian; and the thoracoacromial, lateral thoracic and subscapular arteries from the axillary; and the first and second posterior intercostal arteries from the costocervical trunk anastomose with other posterior intercostal arteries. One route of collateral formation can be subclavian artery - vertebral artery-spinal arteries - post intercostals and lumbar artery - aorta." "This is a real-world medical entrance exam question, please give the true answer based on the question and selection. Topic name: None, Subject name: Pediatrics","Absence of which of the following milestone in 3 yr old chitd is called delayed development? Select and explain. A. Hopping on one leg, B. Drawing a square, C. Feeding by spoon, D. Passing a ball to someone",C: Feeding by spoon,"Answer: C: Feeding by spoon Explanation: Ans: C. Feeding by spoonRef: Ghai Essentiul Pediatrics, 8""' ed., pg. 49Hopping on one leg - Should be attained by 4 years ofageDrawing a square - Should be attained by 4Il2 years of ageFeeding by spoon Should be attained by l8 months of agePassing a ball to someone- Should be attained by 3 years" "This is a real-world medical entrance exam question, please give the true answer based on the question and selection. Topic name: None, Subject name: Social & Preventive Medicine","A patient of diabetes and hypeension comes to your clinic. As a doctor, you explain to him the risks of various complications. Which of these is the best tool to demonstrate the complications? Select and explain. A. Pie cha, B. Histogram, C. Scatter plot, D. Venn diagram",D: Venn diagram,"Answer: D: Venn diagram Explanation: Ans: D. Venn diagram(Ref https://en.itikipedia.org/wiki/Venn_diagram)A Venn diagram is an easy tool to make people understand overlapping risk factors, complications and comorbidities to a patient in a simple language." "This is a real-world medical entrance exam question, please give the true answer based on the question and selection. Topic name: None, Subject name: Dental","Distal palatal termination of the maxillary complete denture base is dictated by the: Select and explain. A. Vibrating line, B. Fovea palatine, C. Tuberosity, D. Maxillary torus",A: Vibrating line,"Answer: A: Vibrating line Explanation: None" "This is a real-world medical entrance exam question, please give the true answer based on the question and selection. Topic name: None, Subject name: Physiology","Which of the following hormones increases the sensitivity of heart to epinephrine: Select and explain. A. Parathyroid, B. Insulin, C. Thyroid, D. Glucagon",C: Thyroid,"Answer: C: Thyroid Explanation: None" "This is a real-world medical entrance exam question, please give the true answer based on the question and selection. Topic name: None, Subject name: Pharmacology","Heroin, an addict drug contains Select and explain. A. ASA, B. Acetyl morphine, C. Pentazocine, D. Propoxyphene",B: Acetyl morphine,"Answer: B: Acetyl morphine Explanation: None" "This is a real-world medical entrance exam question, please give the true answer based on the question and selection. Topic name: AIIMS 2019, Subject name: Pathology","Hepcidin inhibits which of the following? Select and explain. A. Hepheastin, B. DMT-1, C. Cerruloplasmin, D. Ferropoin",D: Ferropoin,"Answer: D: Ferropoin Explanation: Iron absorption is regulated by hepcidin, a small circulating peptide that is synthesized and released from the liver in response to increases in intrahepatic iron levels. Hepcidin inhibits iron transfer from the enterocyte to plasma by binding to ferropoin and causing it to be endocytosed and degraded." "This is a real-world medical entrance exam question, please give the true answer based on the question and selection. Topic name: None, Subject name: Anatomy","The weakest part of the pharynx is Select and explain. A. Sinus of morgagni, B. Between thyropharyngeal and cricopharyngeal sphincter, C. Piriform fossa, D. Pharyngeal recess",B: Between thyropharyngeal and cricopharyngeal sphincter,"Answer: B: Between thyropharyngeal and cricopharyngeal sphincter Explanation: In the posterior wall of the pharynx, the lower part of the thyropharyngeus is a single sheet of muscle, not overlapped internally by the upper and middle constrictors. This weak part lies below the level of the vocal folds or upper border of the cricoid lamina and is limited inferiorly by the thick cricopharyngeal sphincter. This area is knows as Killian’s dehiscence.  Pharyngeal diverticula are formed by outpouching of the dehiscence. Such diverticula are normal in the pig. Pharyngeal diverticula are often attributed to neuromuscular incoordination in this region which may be due to the fact that different nerves supply the two parts of the inferior constrictor.  The propulsive thyropharyngeus is supplied by the pharyngeal plexus, and sphincteric cricopharyngeus, by the recurrent laryngeal nerve. If the cricopharyngeus fails to relax when the thyropharyngeus contracts, the bolus of food is pushed backwards, and tends to produce a diverticulum." "This is a real-world medical entrance exam question, please give the true answer based on the question and selection. Topic name: None, Subject name: Dental","The KRI paste is composed of: Select and explain. A. Iodoform, camphor, parachlorophenol and menthol, B. Iodoform and ZOE, C. Parachlorophenol, camphor and menthol, D. Calcium hydroxide and iodoform","A: Iodoform, camphor, parachlorophenol and menthol","Answer: A: Iodoform, camphor, parachlorophenol and menthol Explanation: None" "This is a real-world medical entrance exam question, please give the true answer based on the question and selection. Topic name: None, Subject name: Dental","Curve of occlusion touching the buccal and lingual surfaces of mandibular buccal cuspal teeth is called as: Select and explain. A. Curve of Wilson, B. Curve of Spee, C. Curve of Monsoon, D. Catenary curve",A: Curve of Wilson,"Answer: A: Curve of Wilson Explanation: CURVE OF WILSON It is a curve that contacts the buccal and lingual cusp tips of the mandibular posterior teeth. The curve of Wilson is mediolateral on each side of arch. It results from the inward inclination of the lower posterior teeth. The curve helps in two ways: Teeth aligned parallel to the direction of medial pterygoid for optimum resistance to masticatory forces. The elevated buccal cusps prevent food from going past the occlusal table." "This is a real-world medical entrance exam question, please give the true answer based on the question and selection. Topic name: None, Subject name: Pediatrics","All of the following are good progonostic factor for ALL except – Select and explain. A. Age of onset between 2-8 years, B. Initial WBC count less than 50000, C. Hyperdiploidy, D. t (9 : 22), t (8 : 14), t (4 : 11)","D: t (9 : 22), t (8 : 14), t (4 : 11)","Answer: D: t (9 : 22), t (8 : 14), t (4 : 11) Explanation: Acute Myelogenous leukemia" "This is a real-world medical entrance exam question, please give the true answer based on the question and selection. Topic name: None, Subject name: Pharmacology","After I.V. administration of which of the following anesthetic agent, there is rapid recovery and less headedness? Select and explain. A. Propofol, B. Diazepam, C. Droperidol, D. Midazolam",A: Propofol,"Answer: A: Propofol Explanation: None" "This is a real-world medical entrance exam question, please give the true answer based on the question and selection. Topic name: None, Subject name: Pathology","Which of the following is associated with HIV infection Select and explain. A. Hairy leukoplakia, B. Erythroplakia, C. Oral Lichen planus, D. Bullous pemphigoid",A: Hairy leukoplakia,"Answer: A: Hairy leukoplakia Explanation: None" "This is a real-world medical entrance exam question, please give the true answer based on the question and selection. Topic name: None, Subject name: Medicine","In aerial blood gas analysis of a patient with pCO2- 30, p02-115 and pH-7.45, patient has compensated: Select and explain. A. Respiratory alkalosis, B. Metabolic alkalosis, C. Respiratory acidosis, D. Metabolic acidosis",A: Respiratory alkalosis,"Answer: A: Respiratory alkalosis Explanation: Ans. a. Respiratory alkalosis (Ref Oxford Handbook of Medicine 5/e p684)In aerial blood gas analysis of a patient with pCO2-30, p02-115 and pH-7.45, patient has compensated respiratory alkalosis." "This is a real-world medical entrance exam question, please give the true answer based on the question and selection. Topic name: None, Subject name: Biochemistry","Acetyl Co-A acts as a substrate for all the enzymes except: Select and explain. A. HMG-CoA synthase, B. Malic enzyme, C. Malonyl CoA synthetase, D. Fatty acid synthetase",B: Malic enzyme,"Answer: B: Malic enzyme Explanation: None" "This is a real-world medical entrance exam question, please give the true answer based on the question and selection. Topic name: None, Subject name: Dental","Flow of a material refers to: Select and explain. A. Continued change of the material under a given load, B. The consistency of a material when mixing, C. The homogeneity of gypsum products, D. Dimensional change of the material during settings",A: Continued change of the material under a given load,"Answer: A: Continued change of the material under a given load Explanation: The term flow, rather than creep, has generally been used in dentistry to describe the rheology of amorphous materials. Creep is defined as the time-dependent plastic strain of a material under a static load or constant stress. The creep or flow is measured as the percentage decrease in length that occurs under these testing conditions. Key Concept: Flow of a material refers to continued change of the material under a given load. Reference: PHILLIPS’ SCIENCE OF DENTAL MATERIALS, 12th ed page no 33, 34" "This is a real-world medical entrance exam question, please give the true answer based on the question and selection. Topic name: None, Subject name: Dental","During which stage of tooth formation the size of the tooth is determined? Select and explain. A. Apposition, B. Histodifferentiation, C. Morpho differentiation, D. Calcification",C: Morpho differentiation,"Answer: C: Morpho differentiation Explanation: Morphodifferentiation establishes the morphologic pattern, or basic form and relative size of future tooth." "This is a real-world medical entrance exam question, please give the true answer based on the question and selection. Topic name: AIIMS 2018, Subject name: Microbiology","In which of the following, viral load done by Real Time PCR is of no role in investigative procedures? Select and explain. A. Person with hepatitis B on Tenofovir therapy, B. HSV causing temporal encephalitis, C. BK virus in patient of allograft renal transplant, D. CMV PCR in blood of patient of liver transplant",B: HSV causing temporal encephalitis,"Answer: B: HSV causing temporal encephalitis Explanation: In person with Hepatitis B on Tenofovir therapy along with patients infected with BK virus in patient of allograft renal transplant and in cytomegalovirus PCR in blood of patient of liver transplant viral load is necessary so viral load is done by real time PCR but it in hsv causing temporal aeritis there is a search no role of viral load so there is no such need of real time PCR This is replaced by HSV DNA in CSF by PCR" "This is a real-world medical entrance exam question, please give the true answer based on the question and selection. Topic name: None, Subject name: Surgery","Adeno ameloblastoma treatment Select and explain. A. Enbloc resection of maxilla, B. Marsupialization, C. Enucleation, D. No treatment",C: Enucleation,"Answer: C: Enucleation Explanation: None" "This is a real-world medical entrance exam question, please give the true answer based on the question and selection. Topic name: None, Subject name: Microbiology","A 30 year old female is on antibiotics with prolonged IV cannulation, has spike of fever, the likely cause is : Select and explain. A. Pseudomonas aerugenosa, B. Coagulase negative staphylococcus, C. Streptococcus agalactiae, D. E. coli",B: Coagulase negative staphylococcus,"Answer: B: Coagulase negative staphylococcus Explanation: Staph. epidermidis is MC cause of infection on implanted foreign bodies." "This is a real-world medical entrance exam question, please give the true answer based on the question and selection. Topic name: None, Subject name: Dental","Which is a gypsum product? Select and explain. A. Stone, B. Plaster, C. Investment, D. All of the above",D: All of the above,"Answer: D: All of the above Explanation: None" "This is a real-world medical entrance exam question, please give the true answer based on the question and selection. Topic name: None, Subject name: Dental","Which of the following cements are not be used to cement acrylic temporary crown: Select and explain. A. Zinc oxide eugenol, B. Zinc phosphate, C. Glass ionomer, D. None of the above",C: Glass ionomer,"Answer: C: Glass ionomer Explanation: None" "This is a real-world medical entrance exam question, please give the true answer based on the question and selection. Topic name: None, Subject name: Surgery","In hemorrhagic shock, hypotension occurs when blood loss is more than: Select and explain. A. 10% - 15%, B. 15% - 30%, C. 30% - 40%, D. More than 40%",D: More than 40%,"Answer: D: More than 40% Explanation: None" "This is a real-world medical entrance exam question, please give the true answer based on the question and selection. Topic name: None, Subject name: Pathology","Best test that shows the integrity of intrinsic pathway of clotting mechanism Select and explain. A. bleeding time, B. aPTT, C. prothrombin time, D. clotting time",B: aPTT,"Answer: B: aPTT Explanation: Answer- B (aPTT)The aPTT measures the time necessary to generate fibrin from initiation of the intrinsic pathway.PTT measures the integrity of the intrinsic system (Factors XII, XI, VIII, IX) and common clotting pathways." "This is a real-world medical entrance exam question, please give the true answer based on the question and selection. Topic name: None, Subject name: Dental","Alginate impression material is similar to agar-agar impression material in the following respect: Select and explain. A. Gelation increases in both on increase in temperature, B. Mixing time is increased to reduce the setting time, C. Deformation during removal of impression occurs due to distortion of gel. fibers, D. Both can be re-used for fresh impressions",C: Deformation during removal of impression occurs due to distortion of gel. fibers,"Answer: C: Deformation during removal of impression occurs due to distortion of gel. fibers Explanation: None" "This is a real-world medical entrance exam question, please give the true answer based on the question and selection. Topic name: None, Subject name: Dental","Spaces in deciduous dentition Select and explain. A. Physiological, B. pathological, C. Incisal liability, D. none of above",A: Physiological,"Answer: A: Physiological Explanation: None" "This is a real-world medical entrance exam question, please give the true answer based on the question and selection. Topic name: None, Subject name: Pharmacology","Pigmentation of nail is caused by all of these drugs except: Select and explain. A. Cyclophosphamide, B. Chlorpromazine, C. Chloroquine, D. Amiodarone",D: Amiodarone,"Answer: D: Amiodarone Explanation: Ans: D. Amiodarone(Ref: KDT 7//e p534)Amiodarone:Cause corneal deposits.Not causes nail pigmentation.Causes of Melanonychia or Nail PigmentationPhenothiazines like ChloroquineMinocyclineArsenicClofazimineClomipramineCyclophosphamidFluconazoleFluorideGold saltsIbuprofenKetoconazoleLamivudineMercuryPhenytoinPsoralenRoxithromyciSteroidsSulfonamideTetracyclineThalliumTimoloZidovudine" "This is a real-world medical entrance exam question, please give the true answer based on the question and selection. Topic name: None, Subject name: ENT","Immediate treatment of CSF Rhinorrhoea requires- Select and explain. A. Antibiotics and observation, B. Plugging with paraffin guage, C. Blowing of nose, D. Craniotomy",A: Antibiotics and observation,"Answer: A: Antibiotics and observation Explanation: Early cases of post traumatic CSF rhinorrhea are managed conservatively (by placing the patient in propped up position, avoiding blowing of nose, sneezing and straining) and Prophylactic antibiotics (to prevent meningitis). Persistent cases are treated surgically by nasal endoscopy or by intracranial route. According to Scott-Brown’s Endoscopic closure of CSF leak is now the treatment of choice in majority of patients but it should not be done immediately. First patient should be subjected to diagnostic evaluation and after site of leakage is confirmed, it should be closed endoscopically." "This is a real-world medical entrance exam question, please give the true answer based on the question and selection. Topic name: None, Subject name: Dental","Interdental papilla protruding from the rubber dam, most common cause is Select and explain. A. Inflammation of interdental papillae, B. Use of light weight rubber dam, C. Punch are placed too far, D. Punch are placed too close",D: Punch are placed too close,"Answer: D: Punch are placed too close Explanation: None" "This is a real-world medical entrance exam question, please give the true answer based on the question and selection. Topic name: None, Subject name: Gynaecology & Obstetrics","Paograph represents various stages of labor with respect to time. True about paograph is all except: Select and explain. A. Each small square represents one hour, B. Ale and action lines are separated by a difference of 4 hours, C. Paograph recording should be staed at a cervical dilation of 4 cm, D. Send the patient to first referral unit if the labor progression line crosses the ale line",C: Paograph recording should be staed at a cervical dilation of 4 cm,"Answer: C: Paograph recording should be staed at a cervical dilation of 4 cm Explanation: Ans: C. Paograph recording should be staed at a cervical dilation of 4 cm(Ref: Williams 24/e p452)Paograph recording:Staed after a cervical dilation of 3 cm (not the 4 cm).i.e. During active stage of labor.Designed by WHO for use in developing countries.Stages of labor:Labor a Divided into latent phase.Latent phase should last no longer than 8 hours.Active phase - Stas at 3 cm dilatation a progress should be no slower than 1 cm/hr.Recommended wait period:4-hour before intervention for slow active phase.Labor is graphed & analysis includes use of ale action lines." "This is a real-world medical entrance exam question, please give the true answer based on the question and selection. Topic name: None, Subject name: Forensic Medicine","A patient with Suspected Poisoning with meiosis, Increased Bronchial Secretions & salivation from Angle of mouth. What type of Antidote is to be Given. Select and explain. A. Atropine, B. EDTA, C. Neostigmine, D. Belladonna",A: Atropine,"Answer: A: Atropine Explanation: Answer-A. Atropine* Atropine is a medication used to treat ceain types of nerve agent and pesticide poisonings as well as some types of slow hea rate and to decrease saliva production." "This is a real-world medical entrance exam question, please give the true answer based on the question and selection. Topic name: None, Subject name: Surgery","Cells with greatest affinity to bisphosphonates are: Select and explain. A. Osteoprogenitor cells, B. Osteoblasts, C. Osteoclasts, D. Osteocytes",C: Osteoclasts,"Answer: C: Osteoclasts Explanation: None" "This is a real-world medical entrance exam question, please give the true answer based on the question and selection. Topic name: None, Subject name: Dental","Guiding cusps occlude in: Select and explain. A. Buccal groove., B. Lingual groove., C. Embrassures., D. All of the above.",D: All of the above.,"Answer: D: All of the above. Explanation: None" "This is a real-world medical entrance exam question, please give the true answer based on the question and selection. Topic name: None, Subject name: Surgery","The radiographic view of choice for demonstrating a nasal fracture is: Select and explain. A. Reverse towne's, B. PA view, C. True lateral, D. OPG",C: True lateral,"Answer: C: True lateral Explanation: None" "This is a real-world medical entrance exam question, please give the true answer based on the question and selection. Topic name: None, Subject name: Pharmacology","An oral hypoglycemic agent is: Select and explain. A. Warfarin, B. Insulin, C. Glibeneclamide, D. Gluengon",C: Glibeneclamide,"Answer: C: Glibeneclamide Explanation: None" "This is a real-world medical entrance exam question, please give the true answer based on the question and selection. Topic name: None, Subject name: Biochemistry","Genes involved in X-Linked SCID- Select and explain. A. EGFR, B. CD23, C. IL6, D. IL2RG",D: IL2RG,"Answer: D: IL2RG Explanation: Answer-D. IL2RG* X-linked severe combined immunodeficiency (SCID) is an inherited disorder of the immune system that occurs almost exclusively in males.* Mutations in the IL2RG gene cause X-linked SCID. The IL2RG gene provides instructions for making a protein that is critical for normal immune system function.* Mutations in the IL2RG gene prevent these cells from developing and functioning normally. Without functional lymphocytes, the body is unable to fight off infections." "This is a real-world medical entrance exam question, please give the true answer based on the question and selection. Topic name: None, Subject name: Dental","Dentin conditioner has the following function. Select and explain. A. Removes smear layer, B. Increases surface energy of dentin, C. Forms a thin resin layer over exposed collagen fibrils, D. Helps in bonding with composite",A: Removes smear layer,"Answer: A: Removes smear layer Explanation: None" "This is a real-world medical entrance exam question, please give the true answer based on the question and selection. Topic name: None, Subject name: Surgery","A patient underwent extraction of 3rd molar experiences pain in socket on 3rd day; socket is tender with no fever and swelling: what treatment should be done Select and explain. A. Irrigation of socket with sedative placement and analgesic, B. Curettage of socket and induces bleeding, C. Left untreated and observe for few days, D. Start antibiotics followed by curettage of socket",A: Irrigation of socket with sedative placement and analgesic,"Answer: A: Irrigation of socket with sedative placement and analgesic Explanation: None" "This is a real-world medical entrance exam question, please give the true answer based on the question and selection. Topic name: None, Subject name: Dental","In case of wider osteotomy planned happen which should not be done Select and explain. A. Press buccal and lingual plate, B. Wider implant, C. Bone graft plus implant, D. Deep osteotomy",B: Wider implant,"Answer: B: Wider implant Explanation: None" "This is a real-world medical entrance exam question, please give the true answer based on the question and selection. Topic name: None, Subject name: Radiology","Which among the following is not a risk factor for contrast-induced nephropathy? Select and explain. A. Diabetic nephropathy, B. High osmolar agent, C. Obesity, D. Dehydration",C: Obesity,"Answer: C: Obesity Explanation: Ans. CSide effects of iodinated contrast material are:Idiosyncratic - Anaphylactoid reaction. They are complement-mediated reactions and not IgE. The patient develops bronchospasm and hypotension.Dose-dependent side effect - Contrast-induced nephropathy and seen especially with high osmolar agents.Contrast-induced nephropathy:The patient has non-oliguric transient nephropathy and occurs due to tubular damage. It is defined as the impairment of renal function and is measured as either a 25% increase in serum creatinine from baseline or 0.5 mg/dl increase in absolute value, within 48-72hrs of intravenous contrast administration.Risk Factors for Contrast Medium-Induced Nephropathy:Patient-related:eGFR < 60 mL/min/1.73 m2 before intra-aerial administrationeGFR < 45 mL/min/1.73 m2 before intravenous administrationIn paicular in combination with:Diabetic nephropathyDehydrationCongestive hea failure (NYHA grade 3-4) and low LVEFRecent myocardial infarction (Intra-aoic balloon pumpPeri-procedural hypotensionLow hematocrit levelAge over 70Concurrent administration of nephrotoxic drugsKnown or suspected acute renal failureProcedure-related:Intra-aerial administration of contrast mediumHigh-osmolality agentsLarge doses of contrast mediumMultiple contrast medium administrations within a few daysIf the patient is on Metformin and is having de-arranged RFT, then before giving contrast agent metformin should be stopped since it can precipitate lactic acidosis" "This is a real-world medical entrance exam question, please give the true answer based on the question and selection. Topic name: None, Subject name: Social & Preventive Medicine","Which of the following statements is not true about incidence? Select and explain. A. Incidence decreases when a programme is effective, B. Vaccination strategies decrease the incidence of a disease, C. Newer and effective treatment modalities decrease the incidence, D. Incidence implies number of new cases detected over a fixed time",C: Newer and effective treatment modalities decrease the incidence,"Answer: C: Newer and effective treatment modalities decrease the incidence Explanation: Ans: C. Newer and effective treatment modalities decrease the incidenceRef: Park 24Ic p67, 23Ie p62, 22/c p59).Newer and effective treatment modalities do not decrease the incidence.Improvements in treatment may decrease the duration of illness and may decrease prevalence.Relation between Incidence & PrevalenceGiven the assumption that population is stable & incidence and duration are not changing.Prevalence = Incidence x Mean duration of diseasePrevalence describes balance between incidence, moality & recoverye.Incidence reflects causal factorse.Duration reflects prognostic factorse" "This is a real-world medical entrance exam question, please give the true answer based on the question and selection. Topic name: AIIMS 2018, Subject name: Pharmacology","A hypeensive patient was on metoprolol treatment. Verapamil was added to the therapy of this patient. This can result in Select and explain. A. Atrial fibrillation, B. Bradycardia with AV block, C. Torsades de pointes, D. Tachycardia",B: Bradycardia with AV block,"Answer: B: Bradycardia with AV block Explanation: Metoprolol (Cardioselective beta blocker) decrease hea rate and AV conduction. Verapamil and Diltiazem: CCB (calcium channel blocker): These also inhibits SA Node and AV Node, hence depress the hea. So, when verapamil (or Diltiazem) is combined with metoprolol, they both depress hea and lead to bradycardia with AV block. Other CCBs include dihydropyridines (like amlodipine etc). These dipines can be used with Metoprolol as they cause reflex tachycardia." "This is a real-world medical entrance exam question, please give the true answer based on the question and selection. Topic name: None, Subject name: Surgery","The cannula infuse maximum fluids in dehydration and diarrhea is Select and explain. A. Grey, B. Green, C. Pink, D. Blue",A: Grey,"Answer: A: Grey Explanation: Answer- A. Grey Grey cannula- 16 G -236 ml/minUses:Trauma Patients, Major Surgery, Intra Paum/Post Paum, GI bleeds, Multiple blood transfers, High volume of Fluids." "This is a real-world medical entrance exam question, please give the true answer based on the question and selection. Topic name: None, Subject name: Anatomy","Example of synarthroses is / are: Select and explain. A. Suture, B. Syndesmosis., C. Gomphosis., D. All of the above.",D: All of the above.,"Answer: D: All of the above. Explanation: None" "This is a real-world medical entrance exam question, please give the true answer based on the question and selection. Topic name: None, Subject name: Pharmacology","Which of the following antibiotics is most frequently associated with pseudomembranous colitis? Select and explain. A. Ampicillin, B. Penicillin, C. Clindamycin, D. Cephalosporin",C: Clindamycin,"Answer: C: Clindamycin Explanation: None" "This is a real-world medical entrance exam question, please give the true answer based on the question and selection. Topic name: None, Subject name: Dental","According to American association of Endodontics, which of following terminology is not mentioned in the glossary of terms: Select and explain. A. Chronic apical periodontitis, B. Acute apical abscess, C. Chronic apical abscess, D. Symptomatic apical periodontitis",A: Chronic apical periodontitis,"Answer: A: Chronic apical periodontitis Explanation: None" "This is a real-world medical entrance exam question, please give the true answer based on the question and selection. Topic name: None, Subject name: Dental","Finishing and Polishing of Amalgam make the restoration: Select and explain. A. Increase in tarnish and corrosion resistance, B. Increase the marginal strength, C. Decrease the tarnish and corrosion resistance, D. Increased compressive strength",A: Increase in tarnish and corrosion resistance,"Answer: A: Increase in tarnish and corrosion resistance Explanation: None" "This is a real-world medical entrance exam question, please give the true answer based on the question and selection. Topic name: None, Subject name: Pediatrics","A 2 year old boy has vitamin D refractory rickets. Investigations show serum calcium 9 mg/dl. Phosphate 2.4 mg dl, alkaline phosphate 1040 parathyroid hormone and bicarbonate levels are normal. The most probable diagnosis is – Select and explain. A. Distal renal tubular acidosis, B. Hypophosphatemic rickets., C. Vitamin D dependent rickets, D. Proximal renal tubular acidosis",B: Hypophosphatemic rickets.,"Answer: B: Hypophosphatemic rickets. Explanation: None" "This is a real-world medical entrance exam question, please give the true answer based on the question and selection. Topic name: None, Subject name: Gynaecology & Obstetrics","According to WHO, anemia in pregnancy is diagnosed, when hemoglobin is less than: Select and explain. A. 10.0 gm%, B. 11.0 gm%, C. 12.0 gm%, D. 9.0 gm%",B: 11.0 gm%,"Answer: B: 11.0 gm% Explanation: “According to the standards laid by WHO – Anemia in pregnancy is defined as when hemoglobin is 11 gm/100 ml or less or hematocrit is less than 33%”." "This is a real-world medical entrance exam question, please give the true answer based on the question and selection. Topic name: None, Subject name: Gynaecology & Obstetrics","Premature baby of 34 weeks was delivered. Baby had bullous lesion on the body. X ray shows periosistis what is the next investigation: Select and explain. A. VDRL for mother and baby, B. ELISA for HIV, C. PCR for T.B., D. Hepatitis surface antigen for mother",A: VDRL for mother and baby,"Answer: A: VDRL for mother and baby Explanation: None" "This is a real-world medical entrance exam question, please give the true answer based on the question and selection. Topic name: None, Subject name: Social & Preventive Medicine","The trivalent influenza vaccine contains all of the following strains except: Select and explain. A. H IN I, B. H3N2, C. H2N 1, D. Influenza B",C: H2N 1,"Answer: C: H2N 1 Explanation: Ans: C. H2N 1Each seasonal influenza vaccine contains antigens representing three (trivalent vaccine) or four (quadrivalent vaccine) influenza virus strains: one influenza type A subtype H1 N1 virus strain, one influenza type A subtype H3N2 virus strain, and either one or two influenza type B virus strains." "This is a real-world medical entrance exam question, please give the true answer based on the question and selection. Topic name: None, Subject name: Biochemistry","VLDL is synthesised in Select and explain. A. GIT, B. Liver, C. Liver and GIT, D. None of the above",B: Liver,"Answer: B: Liver Explanation: Four major groups of lipoproteins have been identified that are important physiologically and in clinical diagnosis. These are, Chylomicrons, derived from intestinal absorption of triacylglycerol and other lipids. Very low density lipoproteins (VLDL), derived from the liver for the export of triacylglycerol. Low-density lipoproteins (LDL), representing a final stage in the catabolism of VLDL.  High-density lipoproteins (HDL), involved in cholesterol transport and also in VLDL and chylomicron metabolism. Triacylglycerol is the predominant lipid in chylomicrons and VLDL, whereas cholesterol and phospholipid are the predominant lipids in LDL and HDL, respectively." "This is a real-world medical entrance exam question, please give the true answer based on the question and selection. Topic name: None, Subject name: Pharmacology","Cephalosporins have all of the following interactions & uses, except: Select and explain. A. Show cross sensitivity with penicillins, B. Are penicillinase resistant, C. Have a broad spectrum but are inactive against anaerobes, D. Are used in upper respiratory tract infections",B: Are penicillinase resistant,"Answer: B: Are penicillinase resistant Explanation: CEPHALOSPORINS  These are a group of semisynthetic antibiotics derived from 'cephalosporin-C obtained from a fungus Cephalosporium. They are chemically related to penicillins; the nucleus consists of a beta-lactam ring fused to a dihydrothiazine ring, (7-aminocepha losporanic acid). By addition of different side chains at position 7 of beta-lactam ring (altering spectrum of activity) and at position 3 of dihydrothiazine ring (affecting pharmacokinetics), a large number of semisynthetic compounds have been produced.  Some salient features are: These are not penicillinase resistance (just like penicillins). Attributed to the similar chemical structure to penicillins they show cross sensitivity with penicillins. These are broadspectrum antibiotics but are inactive against anaerobes. These can be used effectively in upper respiratory tract infection. Reference: Essentials of Medical Pharmacology Eighth Edition KD TRIPATHI page no 775,776" "This is a real-world medical entrance exam question, please give the true answer based on the question and selection. Topic name: None, Subject name: Dental","Magnesium deficiency has been reported as an etiological cause for: Select and explain. A. Bruxism., B. Nail biting., C. Thumb sucking., D. All of the above.",A: Bruxism.,"Answer: A: Bruxism. Explanation: None" "This is a real-world medical entrance exam question, please give the true answer based on the question and selection. Topic name: None, Subject name: Dental","In determining the posterior Limit of a maxillary denture base, which of the following is on the posterior border? Select and explain. A. Hamular notch, B. Hamular process, C. Fovea palatine, D. Vibrating Line",A: Hamular notch,"Answer: A: Hamular notch Explanation: None" "This is a real-world medical entrance exam question, please give the true answer based on the question and selection. Topic name: None, Subject name: Pediatrics","A 5 year old child is rushed to casualty reportedly electrocuted while playing in a park. The child is apneic and is ventilated with bag and a mask. Which of the following will be the next step in the management – Select and explain. A. Check pulses, B. Start chest compressions, C. Intubate, D. Check oxygen saturation",A: Check pulses,"Answer: A: Check pulses Explanation: After airway has been opened and two rescue breaths provided, determine the need for chest compression. For this check the pulse in carotid (in children) or branchial artery (in infants). If the pulse is not palpable or heart rate is <60/min, begin chest compression." "This is a real-world medical entrance exam question, please give the true answer based on the question and selection. Topic name: None, Subject name: Biochemistry","Which of the following is maximum in HDL as compared to other lipoproteins? Select and explain. A. Cholesterol, B. Apoproteins, C. Triglycerides, D. Fatty acids",B: Apoproteins,"Answer: B: Apoproteins Explanation: Ans: B. Apoproteins(Ref Harper 30/e p2541).HDL:Highest apoproteins propoion.Has highest density & migrates the least during electrophoresis.Also contains maximum phospholipids.Apolipoprotein or apoprotein:Protein moiety of lipoprotein.Constituting nearly 70% of HDL & 1% chylomicrons." "This is a real-world medical entrance exam question, please give the true answer based on the question and selection. Topic name: None, Subject name: Anatomy","Asseion - Distal pole of scaphoid goes to avascular necrosis after scaphoid fracture.Reason- Blood supply of scaphoid is from distal to proximal. Select and explain. A. Asseion is true but reason is false, B. Both asseion reason are true and reason explains asseion, C. Both asseion and reason are false, D. Reason is true but asseion is false",D: Reason is true but asseion is false,"Answer: D: Reason is true but asseion is false Explanation: Ans. D. Reason is true but asseion is falseThe main blood supply to the scaphoid enters through the non-aicular dorsal ridge at the waist of the bone and the volar tubercle at the distal aspect of the bone.A dorsal branch of the radial aery accounts for 80% of the blood supply of the scaphoid.A separate volar aerial branch to the scaphoid enters the tubercle and accounts for 20-30% of the scaphoid's blood supply, mainly to the distal poion.The proximal pole of the scaphoid relies entirely on intramedullary blood flow.The unusual retrograde nature of the scaphoid's blood supply renders it especially prone to non-union and proximal pole avascular necrosis" "This is a real-world medical entrance exam question, please give the true answer based on the question and selection. Topic name: AIIMS 2017, Subject name: Pathology","A 30 year old male presented with severe dyspnea. His investigations showed mitral stenosis with left atrial enlargement. The histopathology repo from his mitral valve is shown below. What is the likely diagnosis of these patients? Select and explain. A. Sarcoidosis, B. Fungal granuloma, C. Tuberculosis, D. Rheumatic hea disease",D: Rheumatic hea disease,"Answer: D: Rheumatic hea disease Explanation: The above picture shows the presence of Aschoff bodies, which are pathognomic for Rheumatic Hea Disease. Rheumatic fever (RF) is an acute, immunologically mediated, multisystem inflammatory disease classically occurring a few weeks after an episode of group A streptococcal pharyngitis; occasionally, RF can follow streptococcal infections at other sites, such as the skin. Acute rheumatic carditis is a common manifestation of active RF and may progress over time to chronic rheumatic hea disease (RHD), mainly manifesting as valvular abnormalities. RHD is characterized principally by deforming fibrotic valvular disease, paicularly involving the mitral valve; indeed, RHD is viually the only cause of mitral stenosis Type of necrosis seen in RHD - Fibrinoid necrosis. Morphology: Distinctive lesions occur in the hea, called Aschoff bodies, consisting of foci of T lymphocytes, occasional plasma cells, and plump activated macrophages called Anitschkow cell (pathognomonic fo RF).These macrophages have abundant cytoplasm and central round-to-ovoid nuclei (occasionally binucleate) in which the chromatin condenses into a central, slender, wavy ribbon (hence the designation ""caterpillar cells"")." "This is a real-world medical entrance exam question, please give the true answer based on the question and selection. Topic name: None, Subject name: Pharmacology","In the management of anaphylaxis, which action of adrenaline is not observed? Select and explain. A. Bronchodilation by beta-receptors, B. Cardiovascular effects of beta-receptors, C. Action on blood vessels by alpha-receptors, D. Action on presynaptic alpha-receptors",D: Action on presynaptic alpha-receptors,"Answer: D: Action on presynaptic alpha-receptors Explanation: Ans: D. Action on presynaptic alpha-receptors(Ref: Goodman Gilman 12/e p209, 302, 308)Activation of presynaptic ?2 receptors inhibits the release of NE and other co-transmitters front peripheral sympathetic nerve endings.Activation of ?2 receptors in the pontomedullary region of the CNS inhibits sympathetic nervous system activity and leads to a full in blood pressure.Action on presynaptic alpha-receptors (a2 receptors) is not helpful in management of anaphylactic shock." "This is a real-world medical entrance exam question, please give the true answer based on the question and selection. Topic name: None, Subject name: Dental","Normally Maxillary first molar has Select and explain. A. 3 roots and 3 canals, B. 3 roots and 4 canals, C. 2 roots and 3 canals, D. 2 roots and 4 canals",A: 3 roots and 3 canals,"Answer: A: 3 roots and 3 canals Explanation: None" "This is a real-world medical entrance exam question, please give the true answer based on the question and selection. Topic name: None, Subject name: Anatomy","Mark true or false among the following:Content 's of Carotid Sheath areA. Internal jugular veini) trueii) falseB. Cervical sympathetic trunki) trueii) falseC. Vagus nervei) trueii) falseD. Internal carotid aeryi) trueii) falseE. Deep cervical lymph nodesi) trueii) false Select and explain. A. A. ii) B. ii) C. i) D. i) E. i), B. A. i) B. ii) C. ii) D. i) E. i), C. A. i) B. ii) C. i) D. i) E. ii), D. A. i) B. ii) C. i) D. i) E. i)",D: A. i) B. ii) C. i) D. i) E. i),"Answer: D: A. i) B. ii) C. i) D. i) E. i) Explanation: Ans. D: A. i) B. ii) C. i) D. i) E. i)The carotid sheath also is a tubular fascial investment that extends superiorly between the cranial base and inferiorly to the root of the neck. The carotid sheath contains the common and internal carotid aeries, internal jugular vein, and vagus nerve . In addition, the carotid sheath contains deep cervical lymph nodes, sympathetic fibers, and the carotid sinus nerve. The cervical sympathetic trunk lies behind the sheath but is not included within it." "This is a real-world medical entrance exam question, please give the true answer based on the question and selection. Topic name: AIIMS 2018, Subject name: Orthopaedics","Normal Curvature seen in Lumbar Select and explain. A. Lordosis, B. Kyphosis, C. Scoliosis, D. Recurvatum",A: Lordosis,"Answer: A: Lordosis Explanation: * Normal curvature of spine:Lordosis(concavity) ? cervical spine, lumbar spineKyphosis (convexity) ? thoracic & sacral spine * Causes of | lumbar lordosis: (excessive posterior concavity)- Spondylolisthesis- Obesity / Osteoporosis- Achondroplasia- Postural* Scoliosis -abnormal lateral curvature of veebral column * Kyphosis - abnormal curvature of the veebral column in the thoracic region causing ""hunchback"" deformity." "This is a real-world medical entrance exam question, please give the true answer based on the question and selection. Topic name: None, Subject name: Dental","In class III composite preparation, retention points should be placed: Select and explain. A. In the axial wall, B. Entirely in dentine, C. At the dentinoenamel junction, D. At the expense of facial and lingual wall",B: Entirely in dentine,"Answer: B: Entirely in dentine Explanation: None" "This is a real-world medical entrance exam question, please give the true answer based on the question and selection. Topic name: None, Subject name: Dental","Rotation speed of profile instruments Select and explain. A. 50-100 RPM, B. 100-3000 RPM, C. 300-500 RPM, D. 150-300 RPM",D: 150-300 RPM,"Answer: D: 150-300 RPM Explanation: Direct lines from Cohen: The recommended rotational speed for Profile instruments is 150 to 300 rpm, and to ensure a constant rpm level, the preferred means is electrical motors with gear reduction rather than air-driven motors. Also know The profile system was first sold as the ""Series 29"" hand instruments in .02 taper, but available in .04 and .06 taper Tips of the Profile Series 29 rotary instruments - constant proportion of diameter increments (29%) Profile instrument specifications Cross sections: U-shape design with radial lands and a parallel central core Lateral view: 20° helix angle, a constant pitch, and bullet shaped noncutting tips Blades have Neutral or slightly negative rake angle Acts like Reaming action on dentin rather than cutting" "This is a real-world medical entrance exam question, please give the true answer based on the question and selection. Topic name: None, Subject name: Pathology","In biopsy true about formalin as fixative is all except Select and explain. A. To prevent autolysis, B. To make tissue rigid, C. To kill micro organisms, D. 2% forrnaline is used",D: 2% forrnaline is used,"Answer: D: 2% forrnaline is used Explanation: None" "This is a real-world medical entrance exam question, please give the true answer based on the question and selection. Topic name: None, Subject name: Medicine","A 50 years old patient presented with progressive jaundice. Liver function test was done in which conju!gated serum bilirubin-4.8% and total bilirubin-6.7%, alkaline phosphatase- 550 IU, SGOT-50, SGPT-65. Most probable diagnosis is; Select and explain. A. Jaundice due to choledocholithiasis, B. Dubin-Johnson syndrome, C. Viral hepatitis, D. Malignant obstructive jaundice",A: Jaundice due to choledocholithiasis,"Answer: A: Jaundice due to choledocholithiasis Explanation: Answer- A. Jaundice due to choledocholithiasisThe maximum bilirubin level is seldom >256.5 micromol/L (15.0 mg/dl) in patients wilh choledocholithiasis unless concomitant hepatic disease or anotherfactor leading to marked hyperbilirubinemia exists.'-Harrison I8/e p2625.'Serum bilirubin levels >342.0 micromol/L (20 mg/dL) should suggest the possibility of neoplastic obstruction.'-Harrisonl8/e p2625CholedocholithiasisCBD stones should be suspected in any patient with cholecystitis whose serum bilirubin level is >85.5 micromol/L (5 mg/dL).The maximum bilirubin level is seldom >256.5 micromol/L (15.0 mg/dl) in patients with choledocholithiasis." "This is a real-world medical entrance exam question, please give the true answer based on the question and selection. Topic name: AIIMS 2019, Subject name: Gynaecology & Obstetrics","Continuous GnRH therapy is used in All EXCEPT. Select and explain. A. Precocious pubey, B. Prostate cancer, C. Male infeility, D. Endometriosis",C: Male infeility,"Answer: C: Male infeility Explanation: GnRH agonists Precocious pubey in boys & girls Prostate cancer, breast cancer Estrogen dependant disorders: endometriosis, menorhhagia, fibroid uterus, adenomyosis Infeility: in women for controlled ovarian hyperstimulation for A" "This is a real-world medical entrance exam question, please give the true answer based on the question and selection. Topic name: AIIMS 2018, Subject name: Ophthalmology","All are manifestation of dengue virus infection in eye except? Select and explain. A. Cataract, B. Maculopathy, C. Vitreous hemorrhage, D. Optic neuritis",A: Cataract,"Answer: A: Cataract Explanation: The ocular manifestations of dengue include:- Maculopathy - most common Optic neuritis - 2nd most common Vitreous hemorrhage - rarely seen" "This is a real-world medical entrance exam question, please give the true answer based on the question and selection. Topic name: None, Subject name: Dental","Supporting cusps occlude in: Select and explain. A. Central fossa, B. Marginal ridges., C. Embrassures., D. Both AB",D: Both AB,"Answer: D: Both AB Explanation: None" "This is a real-world medical entrance exam question, please give the true answer based on the question and selection. Topic name: None, Subject name: ENT","Which is the most common site for congenital cholesteatoma Select and explain. A. Anterior superior quadrant of tympanic membrane, B. Posterior superior quadrant of tympanic membrane, C. Anterior inferior quadrant of tympanic membrane., D. posterior inferior quadrant of tympanic membrane",A: Anterior superior quadrant of tympanic membrane,"Answer: A: Anterior superior quadrant of tympanic membrane Explanation: Ans: A .Anterior superior quadrant of tympanic membrane followed by posterior superior quadrant.Congenital cholesteatoma was defined as a white, pearly lesion behind an intact tympanic membrane, which after removal was identified on pathological examination as a cholesteatoma.It is the more common causes for onset of childhood conductive hearing loss unrelated to middle ear effusion." "This is a real-world medical entrance exam question, please give the true answer based on the question and selection. Topic name: None, Subject name: Physiology","Iodine is primarily important in the biochemical synthesis of: Select and explain. A. ACTH, B. Thyroxine, C. Adrenaline, D. Calcitonin",B: Thyroxine,"Answer: B: Thyroxine Explanation: None" "This is a real-world medical entrance exam question, please give the true answer based on the question and selection. Topic name: None, Subject name: Pharmacology","Steroids: Select and explain. A. Steroids reduce the rate of repair of tissues thus delays the healing, B. Exacerbate the inflammatory response, C. Can be safely given to immunocompromised patients, D. Indicated in oral thrush",A: Steroids reduce the rate of repair of tissues thus delays the healing,"Answer: A: Steroids reduce the rate of repair of tissues thus delays the healing Explanation: None" "This is a real-world medical entrance exam question, please give the true answer based on the question and selection. Topic name: AIIMS 2018, Subject name: Anatomy","Which of the following DOESN'T supply dura mater Select and explain. A. Middle meningeal aery, B. Meningeal branch of internal carotid aery in posterior cranial fossa, C. Anterior and posterior ethmoidal aeries, D. Accessory meningeal aery",B: Meningeal branch of internal carotid aery in posterior cranial fossa,"Answer: B: Meningeal branch of internal carotid aery in posterior cranial fossa Explanation: Meningeal branch of int carotid aery in post cranial fossa. Explain: AERIAL SUPPLY OF DURA MATER Aerial supply travels in the outer periosteal layers of dura. ANTERIOR CRANIAL FOSSA: Ophthalmic division of internal carotid Ethmoidal aery (meningeal branches) Anterior division of middle meningeal aery. MIDDLE CRANIAL FOSSA: Anterior & posterior divisions of MMA Accessory meningeal aery (br. Of maxillary aery of ICA) Meningeal branches from internal carotid aery POSTERIOR CRANIAL FOSSA: Posterior branch of middle meningeal aery Meningeal branches. 2. Ascending pharyngeal aery 3. Occipital aery 4. Veebral aery" "This is a real-world medical entrance exam question, please give the true answer based on the question and selection. Topic name: None, Subject name: Medicine","A young girl with the diagnosis of acute promyelocytic leukemia (APML) was treated medically. On day 3 of treatment, she developed tachypnea and fever. Chest X-ray shows bilateral pulmonary infiltrates. Which of the follow ing drug should be given next? Select and explain. A. Dexamethasone, B. Cytarabine, C. Dacarbazine, D. Doxorubicin",D: Doxorubicin,"Answer: D: Doxorubicin Explanation: Answer- DTretinoin plus concurrent anthracycline-based (i.e., idarubicin or daunorubicin) chemotherapy appears to be among The most effective treatment for APL. leading to CR rates of 90-95%Arsenic trioxide has significant antileukemic activity and is being explored as pa of initial treatment in clinical trials of APL. Arsenic trioxide increases the risk of APL differentiation syndrome & may prolong the QT interval.As of 2013 the standard of treatment for concurrent chemotherapy has become arsenic trioxide, which combined with ATRA is referred to ATRA-ATO.Before 2013 the standard of treatment was anthracycline (e.g.daunorubicin, doxorubicin, idarubicin or mitoxantrone)-based chemotherapy.Side effect of ATRA therapy as dyspnea, fever, weight gain, peripheral edema and is treated with dexamethasone." "This is a real-world medical entrance exam question, please give the true answer based on the question and selection. Topic name: None, Subject name: Dental","Fear of closed space, open space, and crowded space is called as: Select and explain. A. Acrophobia, B. Agoraphobia, C. Claustrophobia, D. None",C: Claustrophobia,"Answer: C: Claustrophobia Explanation: According to recent concept Agoraphobia is the fear of closed space, open space and crowded space" "This is a real-world medical entrance exam question, please give the true answer based on the question and selection. Topic name: None, Subject name: Dental","Maximum chances of injury in a child patient occurs during: Select and explain. A. 8 months, B. 10-12 months, C. 12-15 months, D. 16 months",C: 12-15 months,"Answer: C: 12-15 months Explanation: As during 12-15 months child starts walking without support." "This is a real-world medical entrance exam question, please give the true answer based on the question and selection. Topic name: None, Subject name: Dental","Gjessing canine is made up of Select and explain. A. 0.18 x 0.25 stainless steel rectangular wire, B. 0.16 x 0.22 stainless steel rectangular wire, C. 0.17 x 0.25 TMA wire, D. 0.15 x 0.22 TMA wire",B: 0.16 x 0.22 stainless steel rectangular wire,"Answer: B: 0.16 x 0.22 stainless steel rectangular wire Explanation: The PG spring is made in 0.016″x.022″ stainless steel rectangular wire. It has gingivally directed overlapping double helices, ovoid in shape, 10 mm in height and 5.5 mm at its widest part. It also has a small occlusal helix 2 mm in diameter. The spring induces bodily movement of teeth and anchorage, reinforcement by a Nance button or headgear is needed in moderate to high anchorage cases. A prerequisite before placing this spring is good rotational correction as well as leveling and alignment of the three teeth involved. PG canine retraction spring was introduced to the orthodontic profession way back in 1985 by Poul Gjessing, hence also known as Gjessing Canine." "This is a real-world medical entrance exam question, please give the true answer based on the question and selection. Topic name: None, Subject name: Dental","The maximum shrinkage during firing process in ceramic occurs in: Select and explain. A. High Bisque stage, B. Low Bisque stage, C. Medium Bisque stage, D. Fusion stage",C: Medium Bisque stage,"Answer: C: Medium Bisque stage Explanation: None" "This is a real-world medical entrance exam question, please give the true answer based on the question and selection. Topic name: None, Subject name: Dental","Least amount of mercury is required in Select and explain. A. Admixed alloy, B. Spherical, C. Hybrid, D. Lathe cut alloy",B: Spherical,"Answer: B: Spherical Explanation: None" "This is a real-world medical entrance exam question, please give the true answer based on the question and selection. Topic name: None, Subject name: Physiology","In human the Hb is : Select and explain. A. HbH, B. HbA, C. HbM, D. HbS",B: HbA,"Answer: B: HbA Explanation: None" "This is a real-world medical entrance exam question, please give the true answer based on the question and selection. Topic name: None, Subject name: Forensic Medicine","Segmentation of blood in blood vessel after death is known as: Select and explain. A. Kevorkian sign, B. Rokitansky sign, C. Kennedy phenomenon, D. Tache noir",A: Kevorkian sign,"Answer: A: Kevorkian sign Explanation: Ans: A.Kevorkian signKevorkian sign:Fragmentation or segmentation (trucking or shunting) of blood columns after death.Appears in retinal vessels within minutes after death & persists for about an hour." "This is a real-world medical entrance exam question, please give the true answer based on the question and selection. Topic name: None, Subject name: Dental","If Patient is identified with Hepatitis B positive, after treatment what should be done with instrument Select and explain. A. Clean with water for 2 min, B. Instruments should be burnt in flame, C. Instruments should be washed and sent for autoclave, D. None of the above",C: Instruments should be washed and sent for autoclave,"Answer: C: Instruments should be washed and sent for autoclave Explanation: None" "This is a real-world medical entrance exam question, please give the true answer based on the question and selection. Topic name: None, Subject name: Gynaecology & Obstetrics","Which of the following is not done for antenatal diagnosis of Down's syndrome: Select and explain. A. Amniotic fluid volume estimation, B. Alpha-fetoprotein estimation, C. Cordocentesis, D. Chorionic villous biopsy",A: Amniotic fluid volume estimation,"Answer: A: Amniotic fluid volume estimation Explanation: None" "This is a real-world medical entrance exam question, please give the true answer based on the question and selection. Topic name: None, Subject name: Radiology","Which type of radiation effect results in radiation induced thyroid cancer? Select and explain. A. Somatic, B. Genetic, C. Teratogenic, D. Autosomal",A: Somatic,"Answer: A: Somatic Explanation: Based on the object that shows the effects: Somatic Effects: These are biological effects that occur on the exposed individuals. Somatic effects can be of 2 types. Prompt somatic effect occur after an acute dose. Eg: Temporary hair loss occurs about three weeks after a dose of 400 rad to scalp. Delayed somatic effects are those that occur years after radiation doses are received. Eg: Increased potential for development of cancer and cataracts. Genetic or heritable effects: Appear in the future generations of the exposed person as a result of radiation damage to reproductive cells." "This is a real-world medical entrance exam question, please give the true answer based on the question and selection. Topic name: None, Subject name: Pediatrics","A 5 years old child brought to the hospital with history of loose stools but no history of fever or blood in stools. Mother says he is irritable and drinks water hastily when given. On examination eyes are sunken and in skin pinch test, the skin retracted within two seconds but not immediately. What is the treatment for this child? Select and explain. A. Administer the first dose of IV antibiotic and immediately refer to hither center, B. Give oral fluids and ask the mother to continue the same and visit again next day, C. Consider severe dehydration, sta IV fluids, IV antibiotics and refer to higher center, D. Give Zinc supplementation and oral rehydration solution only and ask mother to come back if some danger signs develop",D: Give Zinc supplementation and oral rehydration solution only and ask mother to come back if some danger signs develop,"Answer: D: Give Zinc supplementation and oral rehydration solution only and ask mother to come back if some danger signs develop Explanation: Answer- D. Give Zinc supplementation and oral rehydration solution only and ask mother to come back if some danger signs developThe child in this given scenario is having some dehydration, as the child is restless and irritable, drink water readilyand skin pinch goes back slowly (< 2 seconds) with sunken eyes. Treatment includes oral rehydration therapy, zincsupplementation and continued breastfeeding according to the WHO IMNCI protocol plan B." "This is a real-world medical entrance exam question, please give the true answer based on the question and selection. Topic name: AIIMS 2019, Subject name: Pathology","Basophils are activated by Select and explain. A. IL - 5, B. Neutrophils, C. Killer inhibitory peptide, D. Cell fixed Ig E",D: Cell fixed Ig E,"Answer: D: Cell fixed Ig E Explanation: Cell fixed IgE are the activators of basophils . MAST CELL BASOPHIL Foreign antigen binds to the IgE bound to cell and leads to the activation of mast cell or basophil which leads to Type 1 hypersensitivity. IL-5 leads to activation of eosinophils." "This is a real-world medical entrance exam question, please give the true answer based on the question and selection. Topic name: None, Subject name: Dental","Custom tray is better than stock tray for impression of crown due to following reasons except: Select and explain. A. Custom tray is easy to adapt, B. Stock tray may not record full flange, C. Custom tray is cheaper, D. Stock tray can distort easily",C: Custom tray is cheaper,"Answer: C: Custom tray is cheaper Explanation: None" "This is a real-world medical entrance exam question, please give the true answer based on the question and selection. Topic name: None, Subject name: Pediatrics","Palpable purpura is seen in all. except – Select and explain. A. H.S. Purpura, B. Mixed cryoglobulinemia, C. Giant cell arteritis, D. Drug induced vasculitis",C: Giant cell arteritis,"Answer: C: Giant cell arteritis Explanation: None" "This is a real-world medical entrance exam question, please give the true answer based on the question and selection. Topic name: None, Subject name: Dental","In class II malocclusion, ANB angle is Select and explain. A. Greater than normal, B. Less than normal, C. Not altered, D. None of the above",A: Greater than normal,"Answer: A: Greater than normal Explanation: None" "This is a real-world medical entrance exam question, please give the true answer based on the question and selection. Topic name: None, Subject name: Physiology","Blood brain barrier is absent in all of the following areas except Select and explain. A. Subfornical region, B. Habenuclear trigone, C. Area posterma, D. Neurohypophysis",B: Habenuclear trigone,"Answer: B: Habenuclear trigone Explanation: None" "This is a real-world medical entrance exam question, please give the true answer based on the question and selection. Topic name: None, Subject name: Dental","Which of the following is not a major maxillary metal partial denture frame work: Select and explain. A. Lingual U plate, B. Palatal strap, C. Palatal plate, D. Single palatal plate",A: Lingual U plate,"Answer: A: Lingual U plate Explanation: None" "This is a real-world medical entrance exam question, please give the true answer based on the question and selection. Topic name: None, Subject name: Surgery","Fracture passing through mental foramen in mandible with less than 10 mm of bone can be best managed by Select and explain. A. Reconstruction plate, B. MMF, C. Lag screws, D. 3-D plate",A: Reconstruction plate,"Answer: A: Reconstruction plate Explanation: None" "This is a real-world medical entrance exam question, please give the true answer based on the question and selection. Topic name: None, Subject name: Pediatrics","To establish the diagnosis of H-type trachea-esophageal fistula, which if the following is required? Select and explain. A. Chest X-ray, B. Tracheo-bronchoscopy, C. CT scan, D. Esophagoscopy",B: Tracheo-bronchoscopy,"Answer: B: Tracheo-bronchoscopy Explanation: Answer- B. Tracheo-bronchoscopyIsolated tracheoesophageal fistula (TEF) (H-type fistula):Congenital isolated TEF (H-type) is a rare disorder posing diagnostic and management problems.H-type TEF is more frequent than H-type, owing to the oblique angle of the fistula from the trachea (carina or main bronchi) to the oesophagus, anatomically at the level of the neck root (c7-T1).Pressure changes between both structures can cause entry of air into the oesophagus, or entry of oesophageal content into the trachea." "This is a real-world medical entrance exam question, please give the true answer based on the question and selection. Topic name: AIIMS 2018, Subject name: Anatomy","What is normally seen in lumbar spine:- Select and explain. A. Scoliosis, B. Lordosis, C. Kyphosis, D. Recurvatum",B: Lordosis,"Answer: B: Lordosis Explanation: Curvatures of spine: concave anteriorly Fetus - Universal flexion - 10 curvature After bih - Cervical lordosis - 20 curvature begins when child stas to gain head stability Lumber lordosis - Convex anteriorly Cervical lordosis - 2o Thoracic kyphosis - 1o Lumbar lordosis - 2o Sacral kyphosis - 1o" "This is a real-world medical entrance exam question, please give the true answer based on the question and selection. Topic name: None, Subject name: ENT","Most common cause for nose bleeding is- Select and explain. A. Trauma to little's area, B. A.V. aneurysm, C. Postero superior part of nasal septum, D. Hiatus - semilunaris",A: Trauma to little's area,"Answer: A: Trauma to little's area Explanation: Little area (also called as Kiesselbach’s plexus) is a highly vascular area in the anteroinferior part of nasal septum just above the vestibule It is the most common site for nasal bleeding as this area is exposed to the drying effect of inspiratory current and to finger nail trauma." "This is a real-world medical entrance exam question, please give the true answer based on the question and selection. Topic name: AIIMS 2017, Subject name: Medicine","Hyperkalemia management includes all except: Select and explain. A. Calcium gluconate, B. Insulin drip, C. Salbutamol nebulisation, D. MgSO4",D: MgSO4,"Answer: D: MgSO4 Explanation: Option A- Given to antagonize the effect of potassium on hea. Option B & C - K+ influx helps in reducing the load of K+ in blood Option D- used in TDP/Eclampsia/ status asthmaticus" "This is a real-world medical entrance exam question, please give the true answer based on the question and selection. Topic name: None, Subject name: Gynaecology & Obstetrics","Which of the following antihypertensives is not safe in pregnancy: Select and explain. A. Clonidine, B. ACE inhibitors / Enalapril, C. α − Methyldopa, D. Amlodipine",B: ACE inhibitors / Enalapril,"Answer: B: ACE inhibitors / Enalapril Explanation: None" "This is a real-world medical entrance exam question, please give the true answer based on the question and selection. Topic name: None, Subject name: Dental","In an inferior alveolar nerve block given to a child, the position of the needle as compared to that in an adult is more Select and explain. A. Superior, B. Inferior, C. Buccal, D. Lingual",B: Inferior,"Answer: B: Inferior Explanation: None" "This is a real-world medical entrance exam question, please give the true answer based on the question and selection. Topic name: None, Subject name: Gynaecology & Obstetrics","In which of the following heart diseases is materanl mortality during pregnancy found to be the highest: Select and explain. A. Coarctation of aorta, B. Eisenmenger syndrome, C. AS, D. MS",B: Eisenmenger syndrome,"Answer: B: Eisenmenger syndrome Explanation: None" "This is a real-world medical entrance exam question, please give the true answer based on the question and selection. Topic name: AIIMS 2018, Subject name: Pediatrics","An adolescent school girl complaints of dropping objects from hands, it gets precipitated during morning and during exams. There is no history of loss of consciousness and her cousin sister has been diagnosed with epilepsy. EEG was done and was suggestive of epileptic spikes. What is the diagnosis? Select and explain. A. Juvenile myoclonic epilepsy, B. Atypical absence, C. Choreo - athetoid epilepsy, D. Centrotemporal spikes",A: Juvenile myoclonic epilepsy,"Answer: A: Juvenile myoclonic epilepsy Explanation: - History given suggests the diagnosis of Juvunile myoclonic epilepsy Juvunile myoclonic epilepsy ( Janz Syndrome ) - Most common generalized epilepsy in young adults. - Stas in early adolescence with 1 or more of: Myoclonic jerks in morning, causing patient to drop things Generalized tonic-clonic seizures upon awakening Juvenile absences. - Sleep deprivation and photic stimulation can act as precipitants. - EEG shows generalized 4-5 Hz polyspike and slow wave discharge. OTHER OPTIONS: Atypical absence seizures - Less abrupt onset and offset of loss of awareness - Associated myoclonic components and tone changes of head and body - Precipitated by drowsiness - Accompanied by 1 to 2 hz spike and slow wave discharge. Benign childhood epilepsy syndrome with centrotemporal spikes - Rolandic epilepsy - Stas during childhood and is outgrown in adolescence. - Child wakes up at night owing to focal seizures causing buccal and throat tingling and tonic clonic contractions of 1 side of face, with drooling and inability to speak but with preserved consciousness and comprehension. - EEG shows typical broad based centrotemporal spikes that are markedly increased in frequency during drowsiness and sleep. - MRI is normal. - Patient respond well to carbamazepine." "This is a real-world medical entrance exam question, please give the true answer based on the question and selection. Topic name: AIIMS 2019, Subject name: Physiology","Not a monomeric intermediate filament: Select and explain. A. Vimentin, B. Keratin, C. Tubulin, D. Desmin",C: Tubulin,"Answer: C: Tubulin Explanation: Filamentous protein - Are of 3 types based on their diameter. Diameter Microfilaments Intermediate filaments Microtubules <8 nm 8 - 25 nm >25 nm Function Involved in contractility Provide structural suppo Involved in motility (beating of cilia, flagella) Examples Actin Vimentin, keratin, Desmin. Tubulin (a & b tubulin constitutes microtubules)" "This is a real-world medical entrance exam question, please give the true answer based on the question and selection. Topic name: None, Subject name: Pathology","46. A 9 year old child has increased Horizontal anterior bone loss, less cementum and on test shows excretion of phosphoethanolamine in the urine. The child is suffering from. Select and explain. A. Hypophosphatasia, B. Vit. D resistant Rickets, C. Juvenile periodontitis, D. Osteomalacia",A: Hypophosphatasia,"Answer: A: Hypophosphatasia Explanation: None" "This is a real-world medical entrance exam question, please give the true answer based on the question and selection. Topic name: AIIMS 2018, Subject name: Pharmacology","Sodium nitroprusside is metabolized to form an active metabolite. This active metabolized to form an active metabolite. This active metabolite of sodium nitroprusside act activation of: Select and explain. A. Phospholipase A, B. Phospholipase C, C. Guanylate cyclase, D. Protein Kinase C",C: Guanylate cyclase,"Answer: C: Guanylate cyclase Explanation: Nitoprusside and nitrates act by production of NO (nitric oxide). NO stimulates guanylate cyclase in smooth muscles which leads to formation of cGMP. cGMP act on smooth muscles to cause vasodilation." "This is a real-world medical entrance exam question, please give the true answer based on the question and selection. Topic name: None, Subject name: Gynaecology & Obstetrics","Which of the following is the most useful parameter according to WHO in assessing adequacy of sperms for feilization? Select and explain. A. Spermatocyte count, B. Spermatocyte motility, C. Semen volume, D. Spermatocyte morphology",D: Spermatocyte morphology,"Answer: D: Spermatocyte morphology Explanation: Answer- D. Spermatocyte morphologyEvaluation and assessment of semen is very impoant for both diagnosis of male infeility and selection of patienti for treatment with IVF or ICSI. It has been shown that sperm morphologt assessed strictly is most strongly related to feilization rafe than other parometers. In the WHO guidelines for Normal semen analysis, Sperm morphology, ie. > 4% normal forms is the only strict criteria for sperm adequacy." "This is a real-world medical entrance exam question, please give the true answer based on the question and selection. Topic name: None, Subject name: Pathology","A 3 year old child has a fever of 102 degrees F; and following upper respiratory tract infection discrete vesicles and ulcers on the soft plate and pharynx are noted. The most probable diagnosis is Select and explain. A. Herpangina, B. Scarlet fever, C. RubeLlla, D. Herpetic gingivostomatitis",A: Herpangina,"Answer: A: Herpangina Explanation: None" "This is a real-world medical entrance exam question, please give the true answer based on the question and selection. Topic name: None, Subject name: Anatomy","Acrocephaly is also known as: Select and explain. A. Oxycephaly., B. Turticephaly., C. Dome shaped skull., D. All of the above.",D: All of the above.,"Answer: D: All of the above. Explanation: None" "This is a real-world medical entrance exam question, please give the true answer based on the question and selection. Topic name: None, Subject name: Dental","The reverse bevel incision is made to: Select and explain. A. Allow atraumatic reflection of the gingival margin., B. Remove the infected tissue in the sulcus., C. Provide access to the alveolar crest., D. All of the above",B: Remove the infected tissue in the sulcus.,"Answer: B: Remove the infected tissue in the sulcus. Explanation: Internal bevel incision is also known as reverse bevel incision. It is the incision from which the flap is reflected to expose the underlying bone and root. Objectives: • It removes pocket lining • Conserves the uninvolved outer surface of the gingiva • Produces a sharp, thin flap margin for adaptation to the bone-tooth junction" "This is a real-world medical entrance exam question, please give the true answer based on the question and selection. Topic name: None, Subject name: Physiology","Enzymes, which play an important role in calcification, are: Select and explain. A. Enolase and Calcitonin, B. Alkaline phosphatase and catalase, C. Alkaline phosphatase and pyrophosphatase, D. Pyrophosphatase and carbonic anhydrase",C: Alkaline phosphatase and pyrophosphatase,"Answer: C: Alkaline phosphatase and pyrophosphatase Explanation: None" "This is a real-world medical entrance exam question, please give the true answer based on the question and selection. Topic name: None, Subject name: Biochemistry","What does forward scatter in flow cytometry used to assess? Select and explain. A. Cell death, B. Cell size, C. Cell granules, D. Cell fluorescence",B: Cell size,"Answer: B: Cell size Explanation: Ans: B. Cell sizeFlow cytometry:Technique for counting, examining & soing microscopic paicles suspended in a stream of fluid.Allows simultaneous multi-parametric analysis of physical and/or chemical characteristics of single cells flowing through an optical and/or electronic detection apparatus.Measures optical & fluorescence characteristics of single cells.Direction of light & interpretation:Direction of light scattered in forward Scatter (FS):For cell size.Liver cells will have more forward scatter (FS) than dead and apoptotic cellsDirection of light scattered in side scatter (SS):For density of cells (granularity, vacuoles & membrane size).Granulocytes/monocytes have more granularity or vacuoles a more side scattering (SS).(Ref Hematology: Clinical Principles and Applications/p456)." "This is a real-world medical entrance exam question, please give the true answer based on the question and selection. Topic name: None, Subject name: Pathology","Which inflammatory mediator involve in intracellular killing of microbes? Select and explain. A. Catalase, B. Oxidase, C. Lysozyme, D. IL6",C: Lysozyme,"Answer: C: Lysozyme Explanation: The killing of microbes and the destruction of ingested materials are accomplished by reactive oxygen species (ROS, also called reactive oxygen intermediates), reactive nitrogen species, mainly derived from nitric oxide (NO), and lysosomal enzymes. Neutrophils and monocytes contain granules packed with enzymes and anti-microbial proteins that degrade microbes and dead tissues and may contribute to tissue damage. These granules are actively secretory and thus distinct from classical lysosomes.  Neutrophils have two main types of granules: The smaller specific (or secondary) granules contain lysozyme, collagenase, gelatinase, lactoferrin, plasminogen activator, histaminase, and alkaline phosphatase.  The larger azurophil (or primary) granules contain MPO, bactericidal factors (such as defensins), acid hydrolases, and a variety of neutral proteases (elastase, cathepsin G, nonspecific collagenases, proteinase 3). Kumar V, Abbas AK, Aster JC. Robbins basic pathology. Elsevier Health Sciences; 2017. Page 68" "This is a real-world medical entrance exam question, please give the true answer based on the question and selection. Topic name: None, Subject name: Dental","The most effective, scientifically proven and cost efficient caries prevention program in children is: Select and explain. A. Enameloplasty, B. Oral health education, C. Pit and fissure sealants, D. ART",C: Pit and fissure sealants,"Answer: C: Pit and fissure sealants Explanation: None" "This is a real-world medical entrance exam question, please give the true answer based on the question and selection. Topic name: None, Subject name: Surgery","A 24 years old college student while playing hockey injured his right knee. This patient presents after 3 months with instability of knee joint in it full extension without instability at 90 degree of flexion. The structure most commonly damaged is: Select and explain. A. Posterolatilal pa of anterior cruciate ligament, B. Anteromedial pa of anterior cruciate ligament, C. Posterior cruciate ligament, D. Anterior hom of medial meniscus",A: Posterolatilal pa of anterior cruciate ligament,"Answer: A: Posterolatilal pa of anterior cruciate ligament Explanation: Ans. a. Posterolateral pa of anterior cruciate ligament" "This is a real-world medical entrance exam question, please give the true answer based on the question and selection. Topic name: AIIMS 2017, Subject name: Pharmacology","Drug of choice for scrub typhus is: Select and explain. A. Azithromycin, B. Ciprofloxacin, C. Doxycycline, D. Chloramphenicol",C: Doxycycline,"Answer: C: Doxycycline Explanation: Scrub typhus is caused by rickettsia. DOC for rickettsia is Doxycycline(tetracycline). It is the DOC for all kind of typhus like endemic typhus, epidemic typhus, and other rickettsial infections like Q fever, rocky mountain spotted fever. Azithromycin is a macrolide and a protein synthesis inhibitor. Ciprofloxacin is a fluoroquinolone. Chloramphenicol is also a protein synthesis inhibitor." "This is a real-world medical entrance exam question, please give the true answer based on the question and selection. Topic name: None, Subject name: Biochemistry","VLDL is concerned with: Select and explain. A. Delivery of endogenous FA to extrahepatic tissue, B. Delivery of exogenous FA to extrahepatic tissue, C. Delivery of cholesterol to extrahepatic tissue, D. All of the above",A: Delivery of endogenous FA to extrahepatic tissue,"Answer: A: Delivery of endogenous FA to extrahepatic tissue Explanation: VLDL metabolism: Delivery of endogenous FA to the extra-hepatic tissue. VLDL converted to LDL and delivers cholesterol to tissue. VLDL is formed in liver." "This is a real-world medical entrance exam question, please give the true answer based on the question and selection. Topic name: None, Subject name: Microbiology","Malignant pustule is referred to: Select and explain. A. Facio-cervical actinomycosis, B. Cutaneous anthrax, C. Infected squamous cell carcinoma, D. None of the above",B: Cutaneous anthrax,"Answer: B: Cutaneous anthrax Explanation: None" "This is a real-world medical entrance exam question, please give the true answer based on the question and selection. Topic name: None, Subject name: Medicine","A 56yr old underwent transsphenoidal hypophysectomy for pituitary tumour. Now he has low ACTH, TSH, FSH, LH. Which of the hormone will not be given to the patient? Select and explain. A. Glucocoicoids, B. Mineralocoicoids, C. Levothyroxine, D. Estradiol",B: Mineralocoicoids,"Answer: B: Mineralocoicoids Explanation: Answer- B. MineralocoicoidsThis patient does not need supplementation with mineralocoicoids.It is due to the fact that post-hypophysectomy, the basal levels of mineralocoicoids (principally aldosterone) remain normal as the renin-angiotensin-aldosterone feedback loop remains intact." "This is a real-world medical entrance exam question, please give the true answer based on the question and selection. Topic name: None, Subject name: Dental","Cobalt-Chromium alloys contains: Select and explain. A. 30% cobalt and 60% chromium, B. 60% cobalt and 30% chromium, C. 1% palladium, D. 20% gold",B: 60% cobalt and 30% chromium,"Answer: B: 60% cobalt and 30% chromium Explanation: None" "This is a real-world medical entrance exam question, please give the true answer based on the question and selection. Topic name: None, Subject name: Dental","Surveyor is used on master cast to: Select and explain. A. Locate guiding planes, B. Determine aesthetics, C. Delineate areas of maximum convexity, D. Locate undercuts to be used for retention or to be blocked out",D: Locate undercuts to be used for retention or to be blocked out,"Answer: D: Locate undercuts to be used for retention or to be blocked out Explanation: None" "This is a real-world medical entrance exam question, please give the true answer based on the question and selection. Topic name: None, Subject name: Dental","COPRA came into action on: Select and explain. A. 15th April 1987, B. 15th April 1989, C. 25th April 1988, D. 15th April 1986",A: 15th April 1987,"Answer: A: 15th April 1987 Explanation: CONSUMER PROTECTION ACT (CPA/COPRA)  The Consumer Protection Act, 1986 that came into force on 15th April, 1987 is a milestone in the history of socio-economic legislation in the country. It is one of the most progressive and comprehensive pieces of legislation enacted. Essentials of preventive and community dentistry Soben Peter 5th edition" "This is a real-world medical entrance exam question, please give the true answer based on the question and selection. Topic name: AIIMS 2017, Subject name: Microbiology","Cyst of parasite seen in stool microscopy. What is the organism? Select and explain. A. Entamoeba dyspar, B. Balantidium coli, C. Giardia lambia, D. Taenia solium",C: Giardia lambia,"Answer: C: Giardia lambia Explanation: The given cyst have axostyle and 4 nuclei are present - features of Giardia lamblia. Trophohozoites of Giardia lamblia have 4 pairs of flagella and have falling leaf like motility. Balantidium coli- cyst are binucleated Entamoeba dyspar- quadrinucleated cyst no axostyle Taenia solium-no cyst eggs are seen in faeces" "This is a real-world medical entrance exam question, please give the true answer based on the question and selection. Topic name: None, Subject name: Biochemistry","Active form of vitamin D in kidney is: Select and explain. A. 1 dihydroxy cholecalciferol, B. 25 hydroxy cholecalciferol, C. 1, 25 dihydroxy cholecalciferol, D. 7 dihydroxy calciferol","C: 1, 25 dihydroxy cholecalciferol","Answer: C: 1, 25 dihydroxy cholecalciferol Explanation: None" "This is a real-world medical entrance exam question, please give the true answer based on the question and selection. Topic name: None, Subject name: Dental","Enamel lamellae Select and explain. A. Elevation on outer surface of enamel, B. Dentinal tubule in enamel, C. Uniform arrangement of enamel rods, D. Enamel projection in dentin",D: Enamel projection in dentin,"Answer: D: Enamel projection in dentin Explanation: None" "This is a real-world medical entrance exam question, please give the true answer based on the question and selection. Topic name: None, Subject name: Pharmacology","The drug for choice for treatment of infection involving non-penicillinase producing staphylococcus is: Select and explain. A. Ampicillin, B. Erythromycin, C. Penicillin G, D. Methicillin sodium",C: Penicillin G,"Answer: C: Penicillin G Explanation: None" "This is a real-world medical entrance exam question, please give the true answer based on the question and selection. Topic name: None, Subject name: Dental","Before pouring an elastic impression, it is washed with slurry of water and stone to: Select and explain. A. Increase gel strength, B. Prevent syneresis, C. Prevent distortion, D. Wash off saliva on impression",D: Wash off saliva on impression,"Answer: D: Wash off saliva on impression Explanation: None" "This is a real-world medical entrance exam question, please give the true answer based on the question and selection. Topic name: None, Subject name: Dental","Cleaning of files in between of endodontic treatment is done by: Select and explain. A. Gauze soaked in hypochlorite solution, B. 15 seconds in glass bead sterilizer, C. 30 seconds in glass bead sterilizer, D. Chlorhexidine",A: Gauze soaked in hypochlorite solution,"Answer: A: Gauze soaked in hypochlorite solution Explanation: None" "This is a real-world medical entrance exam question, please give the true answer based on the question and selection. Topic name: None, Subject name: Pediatrics","A 10 year old child presented with headache, vomiting, gait instability and diplopia. On examination he had papilledema and gait ataxia. The most probable diagnosis is – Select and explain. A. Hydrocephalus, B. Brain stem tumour, C. Suprasellar tumour, D. Midline posterior fossa tumour",D: Midline posterior fossa tumour,"Answer: D: Midline posterior fossa tumour Explanation: ""The classical triad of headache, nausea and vomiting, and papilledenaa is associated with midline or infratentorial tumors"".   Sign and symptoms of Brain tumors in children Children with brain tumors frequently have symptoms and signs that strongly suggest the appropriate diagnosis. The clinical triad of symptoms associated with increased ICT -4 headache, vomiting, blurred vision / diplopia, is the hallmark of brain tumors in childhood. These symptoms occur more frequently with mid-line posterior fossa (infratentorial) tumors owing to the propensity of lesions close to the ventricular CSF flow system. Symptoms of the brain tumors vary depending upon which part of the brain the tumor is found :­ Brain tumor in cerebrum (supratentorial) Seizures                                 o Hemiperes is or Hemiplegia          o Drowsiness / confusion Visual changes                     o T ICT (less common)                        o Personality changes Brain tumor in brain stem Seizures                                 o Diplopia             o Respiratory changes o T ICT (Rare) Endocrine problem (DM) o Headache            o Cranial N. palsy Brain tumor in cerebellum T ECT (most common)     o Headache          o Uncoordinated movement Vomiting                               o Ataxia Symptoms due to raised ICT --> These are mainly caused by midline posterior fossa tumor --> Medulloblastoma. Headache                                                    o Nausea                                                  o Drowsiness Vomiting (usually in the morning)         o Personality changes                         o Papilledema Diplopia / blurred vision                           o Irritability Note - Brain-stem tumors are also posterior fossa tumors, but they do not present with increased ICT." "This is a real-world medical entrance exam question, please give the true answer based on the question and selection. Topic name: None, Subject name: Gynaecology & Obstetrics","What should be the time of termination of pregnancy of a female with insulin dependent diabetes? Select and explain. A. 40 weeks, B. 38 weeks, C. 37 weeks, D. 34 weeks",A: 40 weeks,"Answer: A: 40 weeks Explanation: Ans. a. 40 weeksIn diabetic mothers, the fetal lung maturation delays due to delay in the completion of surfactant formation in fetal lung. Hence 40 weeks is the choice, the expected delivery can be delayed up to 40 weeks." "This is a real-world medical entrance exam question, please give the true answer based on the question and selection. Topic name: None, Subject name: Surgery","Following a A, a young man was brought to ER. Due to massive blood loss, 2 units of PRBC and 4 platelets obtained from blood bank. Only one IV line was accessible. What will you do? Select and explain. A. Sta PRBC 1st mid store platelet in room temperature, B. Sta platelet and store PRBC in room temperature, C. Only transfuse PRBC, D. Transfuse PRBC and store platelet at 2-6 degrees",A: Sta PRBC 1st mid store platelet in room temperature,"Answer: A: Sta PRBC 1st mid store platelet in room temperature Explanation: Ans: A. Sta PRBC 1st and store platelet in room temperatureRefi ATLS 18th edn; 2018, Chapter 3 ShockFirst preference is PRBC followed by plateletsIn mean time, store platelets at room temperature." "This is a real-world medical entrance exam question, please give the true answer based on the question and selection. Topic name: None, Subject name: Dental","Amount of heat that is required to change boiling water into vapor is referred to as Select and explain. A. Latent Heat of vaporization, B. Latent Heat of sublimation, C. Latent Heat of condensation, D. Latent heat of fusion",A: Latent Heat of vaporization,"Answer: A: Latent Heat of vaporization Explanation: None" "This is a real-world medical entrance exam question, please give the true answer based on the question and selection. Topic name: None, Subject name: Dental","A continuum to sedation, anxiolysis is referred to: Select and explain. A. Conscious sedation, B. Deep sedation, C. Minimal sedation, D. General anaesthesia",C: Minimal sedation,"Answer: C: Minimal sedation Explanation: None" "This is a real-world medical entrance exam question, please give the true answer based on the question and selection. Topic name: None, Subject name: Pathology","Peg-shaped incisors which taper towards the indsal edge are typically seen in all of the following conditions EXCEPT Select and explain. A. Congenital syphilis, B. Rickets, C. Anhyirotic ectodermal dysplasia, D. Supernumerary teeth",B: Rickets,"Answer: B: Rickets Explanation: None" "This is a real-world medical entrance exam question, please give the true answer based on the question and selection. Topic name: None, Subject name: Surgery","Pick out odd drugs for anaesthetic emergencies are: Select and explain. A. AminophylLine, B. Epinephrine, C. Atropine sulphate, D. Amoxycillin",D: Amoxycillin,"Answer: D: Amoxycillin Explanation: None" "This is a real-world medical entrance exam question, please give the true answer based on the question and selection. Topic name: None, Subject name: Dental","A study was done where the sale of anti-asthma drug was recorded and the number of deaths because of asthma was measured over a period of 15 years (1975-1990). This is which type of study. Select and explain. A. Ecological, B. Case reference, C. Experimental, D. Psephology",A: Ecological,"Answer: A: Ecological Explanation: None" "This is a real-world medical entrance exam question, please give the true answer based on the question and selection. Topic name: None, Subject name: Dental","The fear of a 6 year old related to dentistry is primarily: Select and explain. A. Subjective, B. Objective, C. Subjective & objective, D. Psychological",A: Subjective,"Answer: A: Subjective Explanation: None" "This is a real-world medical entrance exam question, please give the true answer based on the question and selection. Topic name: None, Subject name: Dental","The sulcular epithelium acts as a semi permeable membrane through which Select and explain. A. Bacterial products pass in to the gingiva, B. Fluids from the gingiva seeps in to the sulcus, C. Both of the above, D. None of the above",C: Both of the above,"Answer: C: Both of the above Explanation: None" "This is a real-world medical entrance exam question, please give the true answer based on the question and selection. Topic name: None, Subject name: Physiology","A patient inhales a tidal volume of 500 mL. The intrapleural pressure was measured as - 4 cm of water before inspiration and - 9 cm of water after inspiration. Calculate the pulmonary compliance in this patient: Select and explain. A. 0.1 L/cm, B. 0.3 L/cm, C. 0.2 L/cm, D. 0.4 L cm",A: 0.1 L/cm,"Answer: A: 0.1 L/cm Explanation: Answer- A. 0.1 L/cmThe pulmonary compliance in this patient is 0.1 L/cm H2O.Measure of the lung's ability to stretch & expand (distensibility of elastic tissue).Pulmonary surfactant increases compliance by decreasing the surface tension of water.Pulmonary compliance = DV/ DB" "This is a real-world medical entrance exam question, please give the true answer based on the question and selection. Topic name: AIIMS 2018, Subject name: Pediatrics","Maximum steroid produced by fetal adrenal:- Select and explain. A. DHEA-S, B. Coisol, C. Coicosterone, D. Progesterone",B: Coisol,"Answer: B: Coisol Explanation: - steroid hormones are synthesized in adrenal coex. - coisol is major steroid predominantly synthesized by fetal adrenal gland." "This is a real-world medical entrance exam question, please give the true answer based on the question and selection. Topic name: None, Subject name: Dental","Semi colon shaped C shaped canal is classified as (according to Melton and Seitzor) Select and explain. A. Type 1, B. Type 2, C. Type 3, D. Type 4",B: Type 2,"Answer: B: Type 2 Explanation: None" "This is a real-world medical entrance exam question, please give the true answer based on the question and selection. Topic name: AIIMS 2018, Subject name: Dental","Which nerve biopsy is taken to diagnose neuritic leprosy? Select and explain. A. Median nerve, B. Radial cutaneous nerve, C. Ulnar nerve, D. Radial nerve",B: Radial cutaneous nerve,"Answer: B: Radial cutaneous nerve Explanation: Neuritic leprosy - only nerve involved no skin involvement. - Slit Skin smear(SSS) - negative - Nerve thickening - confirms diagnosis. A purely sensory thickened peripheral nerve should be sampled as there are chances of nerve damage during biopsy If its a motor nerve--> deformity . Therefore, option median & ulnar are ruled out. So Radial cutaneous branch is used for sampling during nerve biopsy." "This is a real-world medical entrance exam question, please give the true answer based on the question and selection. Topic name: None, Subject name: Pathology","Low grade infection which leads to localized periosteal reaction is: Select and explain. A. Garre's osteomyelitis, B. Acute osteomyelitis, C. Condensing csteitis, D. Local alveolar osteitis",A: Garre's osteomyelitis,"Answer: A: Garre's osteomyelitis Explanation: None" "This is a real-world medical entrance exam question, please give the true answer based on the question and selection. Topic name: None, Subject name: Medicine","72-y ear-olo gentleman with normal renal functions presents with new onset focal seizures. Which of the following is the best drug to manage the patient? Select and explain. A. Sodium valproate, B. Oxcarbazepine, C. Leviteracetam, D. Pregabalin",B: Oxcarbazepine,"Answer: B: Oxcarbazepine Explanation: Answer: b. Oxcarbazepine (Ref: Harrison 194, p2552. 18/e pc 1)3262)Among the options provided, oxcarbazepine is the best drug to manage this old patient with normal renal functions presents with new onset focal seizures" "This is a real-world medical entrance exam question, please give the true answer based on the question and selection. Topic name: None, Subject name: Gynaecology & Obstetrics","Injury to which of the following deep pa of perinea! body causes cystocele, enterocele and urethral descent? Select and explain. A. Pubococcygeus, B. Ischiocavernosus, C. Bulbospongiosus, D. Sphincter of urethra and anus",A: Pubococcygeus,"Answer: A: Pubococcygeus Explanation: Ans. a. Pubococcygeus When the pubococcygeus muscle contracts, it pulls the rectum, vagina, and urethra anteriorly toward the pubic bone and constricts the lumens of these pelvic organs. It is this contractile propey that is so impoant in maintaining urinary and fecal continence and in providing suppo for the genital organs (vagina, cervix, uterus) that lie upon and are suppoed by the levator plate. Injury to pubococcygeus can lead to rectocele, cystocele and urinary incontinence." "This is a real-world medical entrance exam question, please give the true answer based on the question and selection. Topic name: None, Subject name: Biochemistry","Which of the following is not a post transcriptional modification of RNA? Select and explain. A. Splicing, B. 5' capping, C. 3' polyadenylation, D. Glycosylation",D: Glycosylation,"Answer: D: Glycosylation Explanation: None" "This is a real-world medical entrance exam question, please give the true answer based on the question and selection. Topic name: None, Subject name: Dental","Largest Faciolingual dimension is seen in Select and explain. A. Lower 1st molar, B. Upper 1st molar, C. Lower 2nd molar, D. Upper 2nd molar",B: Upper 1st molar,"Answer: B: Upper 1st molar Explanation: None" "This is a real-world medical entrance exam question, please give the true answer based on the question and selection. Topic name: None, Subject name: Pediatrics","Children with germline retinoblastoma are more likely to develop other primary malignancies in their later lifetime course. Which of the following malignancy can occur in such patients? Select and explain. A. Osteosarcoma of lower limbs, B. Thyroid carcinoma, C. Seminoma, D. Renal cell carcinoma",A: Osteosarcoma of lower limbs,"Answer: A: Osteosarcoma of lower limbs Explanation: Answer- A. Osteosarcoma of lower limbsChildren with germline retinoblastoma are more likely to develop other primary malignancies in their later lifetime course.Osteosarcoma of lower limbs can occur in such patients.Retinoblastoma Syndrome (Primary site malignancy)- Familial Retinoblastoma" "This is a real-world medical entrance exam question, please give the true answer based on the question and selection. Topic name: None, Subject name: Pharmacology","Which of the following drug is commonly used for community acquired pneumonia in OPD? Select and explain. A. Vancomycin, B. Ceftriaxone, C. Azithromycin, D. Streptomycin",C: Azithromycin,"Answer: C: Azithromycin Explanation: Ans: C. AzithromycinRef: n e u m o n ia- i n- a d u I t s -in -t h e - o u tp atie n t - s e tt in g H 4Only one drug which is active orally i.e. Azithromycin.We require OPD based treatment; hence Azithromycin is the best answer here." "This is a real-world medical entrance exam question, please give the true answer based on the question and selection. Topic name: AIIMS 2019, Subject name: Psychiatry","Not a side effect of Escitalopram? Select and explain. A. Nausea, B. Vivid dreams, C. Anorgasmia, D. Sialorrhoea",D: Sialorrhoea,"Answer: D: Sialorrhoea Explanation: Side effects of Escitalopram GI side effects: MC Vivid Dreams Sexual dysfunction on long term intake Sialorrhea is a side effect of clozapine" "This is a real-world medical entrance exam question, please give the true answer based on the question and selection. Topic name: None, Subject name: Medicine","Death due to smoke inhalation is due to: Select and explain. A. Hypoxemia, B. Anemic hypoxia, C. Ischemic hypoxia, D. All of the above",B: Anemic hypoxia,"Answer: B: Anemic hypoxia Explanation: Ans. b. Anemic hypoxiaIn smoke/fire patient generally dies of excess CO poisoning (most frequent cause of death).If there is low PaO, - it is called as hypoxic hypoxia.In smoke (CO poisoning) PaO, will be normal and the problem is COHb.An abnormal Hb or Reduced Hb concentration comes under anemic hypoxia." "This is a real-world medical entrance exam question, please give the true answer based on the question and selection. Topic name: None, Subject name: Social & Preventive Medicine","Child has received full rabies vaccination in December 2018 and now presented with oozing wound on great toe and the pet had vaccination also, what would you do now? Select and explain. A. No vaccine, B. 2 doses of vaccine on day 0 and 3, C. Full 5 doses of vaccines, D. Rabies Ig and full vaccination",B: 2 doses of vaccine on day 0 and 3,"Answer: B: 2 doses of vaccine on day 0 and 3 Explanation: Answer B. 2 doses of vaccine on day 0 and 3Two doses of vaccine on Day 0 & 3 should be administered to this boy.This case is classified under WHO category III since the bleeding was present at the site of dog-bite(toe). This kid should receive PEP without any delay as his immunization was done more than 3 months ago." "This is a real-world medical entrance exam question, please give the true answer based on the question and selection. Topic name: None, Subject name: Pharmacology","Oral infection by penicillinase producing organisms should be treated with: Select and explain. A. Ampicillin, B. Dicloxacillin, C. Erythromycin, D. Any of the above",B: Dicloxacillin,"Answer: B: Dicloxacillin Explanation: None" "This is a real-world medical entrance exam question, please give the true answer based on the question and selection. Topic name: None, Subject name: Medicine","Babu, a 49-year-old patient, complains of foul breath and regurgitation of food eaten few days back. He also complains of dysphagia. The probable diagnosis is: Select and explain. A. Gastrophoresis, B. Achalasia, C. Diabetes, D. Pharyngeal pouch",D: Pharyngeal pouch,"Answer: D: Pharyngeal pouch Explanation: None" "This is a real-world medical entrance exam question, please give the true answer based on the question and selection. Topic name: None, Subject name: Surgery","False statement about Thyroglossal Duct cyst is- Select and explain. A. Infected thyroglossal cyst from sinus, B. Lined by pseudostratified columnar epithelium, C. 40% cases have sub hyoid location, D. It is due to congenital",A: Infected thyroglossal cyst from sinus,"Answer: A: Infected thyroglossal cyst from sinus Explanation: Answer- A. Infected thyroglossal cyst from sinusThyroglossal cysts are the most common cause of midline neck masses and are generally located caudal to (below) the hyoid bone.Thyroglossal Duct Cysts are a bih defect.A thyroglossal cyst is lined by pseudostratified, ciliated columnar epithelium while a thyroglossal fistula is lined by columnar epithelium.A thyroglossal duct cyst may rupture unexpectedly, resulting in a draining sinus known as a thyroglossal fistula." "This is a real-world medical entrance exam question, please give the true answer based on the question and selection. Topic name: None, Subject name: Pediatrics","A 2 year old premature neonate develops GTCS. What is the best investigation done to diagnose the pathology? Select and explain. A. Transcranial ultrasound, B. CT Head, C. MRI brain, D. X-ray",A: Transcranial ultrasound,"Answer: A: Transcranial ultrasound Explanation: Answer- A. Transcranial ultrasound Diagnostic Procedures:Polygraphic video-EEG recording ofsuspected events is probably mandatory for an incontroveible seizure diagnosis.It is performed at the bedside and provides effective assessment of ventricular size and other fluid-containing lesions as well as effective viewing of haemorrhagic and ischaemic lesions and their evolution.CT brain scan is often of secondary or adjunctive impoance to ultrasound. Last-generation CT brain scan imagesare of high resolution, can be generated within seconds and can accurately detect haemorrhage, infarction, grossmalformations and ventricular and other pathological conditions.MRI is much superior for abnormalities of coical development used for the detection of structural abnormalitiessuch as malformations of coical development, intracranial haemorrhage, hydrocephalus and cerebral infarction." "This is a real-world medical entrance exam question, please give the true answer based on the question and selection. Topic name: None, Subject name: Dental","Binder in investment for dental castings: Select and explain. A. Brings about reduction in expansion, B. Provides adequate strength to the investment, holds ingredients together and provides rigidity, C. Is usually a form of silica, D. Can be safely heated above 700°C when present in the form of gypsum","B: Provides adequate strength to the investment, holds ingredients together and provides rigidity","Answer: B: Provides adequate strength to the investment, holds ingredients together and provides rigidity Explanation: None" "This is a real-world medical entrance exam question, please give the true answer based on the question and selection. Topic name: None, Subject name: Pharmacology","Vasodilatation is caused by: Select and explain. A. Serotonin, B. Histamine, C. LT C4, D. Thromboxane A2",B: Histamine,"Answer: B: Histamine Explanation: Ans. B. Histamine* Histamine causes marked dilatation of smaller blood vessels that include aerioles, capillaries, and venules." "This is a real-world medical entrance exam question, please give the true answer based on the question and selection. Topic name: None, Subject name: Surgery","A 56 years old patient came to casualty with history of massive hemoptysis. His routine investigations and chest X-ray was normal. Which of the following is not done to prevent hemoptysis? Select and explain. A. Bronchial aery embolization, B. Pulmonary aery embolization, C. Bronchoscopic laser cauterization, D. Lobectomy of the affected segment",D: Lobectomy of the affected segment,"Answer: D: Lobectomy of the affected segment Explanation: Answer- D. Lobectomy of the affected segmentAll of the given options are used for control of hemoptysis in the following order: Bronchoscopic laser cauterization +Bronchial aery embolization + Pulmonary aery embolization + Lobectomy of the affected segment.""Large-volume hemoptysis, referred to as massive hemoptysis, is variobly defined as hemoptysis of >200-600 mL in 24h. Massive hemoptysis should be considered a medical emergency." "This is a real-world medical entrance exam question, please give the true answer based on the question and selection. Topic name: None, Subject name: Surgery","While doing emergency laparotomy for an intestinal obstruction, which organ will you first visualize to say whether it is small bowel or large bowel obstruction? Select and explain. A. Ileum, B. Sigmoid colon, C. Cecum, D. Rectum",C: Cecum,"Answer: C: Cecum Explanation: Ans: C. CecumCecum is always the first structure which should be seen in bowel obstruction. If it is dilated, then it suggests large bowel obstruction, where as if it is collapsed, then it is signifying small bowel obstruction." "This is a real-world medical entrance exam question, please give the true answer based on the question and selection. Topic name: None, Subject name: Dental","For deep gingival sheath preparation, shape of wedge should be: Select and explain. A. Round, B. Triangular, C. Square, D. All of the above",B: Triangular,"Answer: B: Triangular Explanation: Wedges are devices which are usually preferred for rapid tooth separation. These are used for tooth preparation and restoration. Triangular wedge: Most commonly used. It has two positions — apex and the base. Apex of the wedge usually lies in gingival portion of the contact area. Base lies in contact with gingiva. This helps in stabilization and retraction of gingiva. Used in tooth preparations with deep gingival margins. Ref : Textbook of Operative Dentistry, Nisha and Amit Garg, 3rd edition, Pg no:209" "This is a real-world medical entrance exam question, please give the true answer based on the question and selection. Topic name: None, Subject name: Anatomy","The knowledge about the biomechanics of the muscles attached around the shoulder joint, known as 'rotator cuff muscles' has increased exponentially. However, the role of one of the rotator cuff muscles, which is also now known as 'forgotten rotator cuff muscle' has been ignored or less impoance has been attached to its role. Which of the following muscles best describes this description? Select and explain. A. Supraspinatus, B. Infraspinatus, C. Teres minor, D. Subscapularis",D: Subscapularis,"Answer: D: Subscapularis Explanation: Answer- D (Subscapularis)Subscapularis is the major and most powerful muscle of the rotator cuff and has an enormous meaning in the gleno-humeral stability.Lo and Burkha even tagged the subscapularis tendon hypercritically as the 'forgotten tendon'." "This is a real-world medical entrance exam question, please give the true answer based on the question and selection. Topic name: None, Subject name: Physiology","Electromyography is used to: Select and explain. A. To study muscle activity, B. Determine class II malocclusion, C. Determine the centric relation, D. Determine the centric occlusion",A: To study muscle activity,"Answer: A: To study muscle activity Explanation: None" "This is a real-world medical entrance exam question, please give the true answer based on the question and selection. Topic name: None, Subject name: Pharmacology","Which of the following drugs acting on dilator pupillae has an action analogous to that of pilocarpine on sphincter papillae? Select and explain. A. Timolol, B. Epinephrine, C. Neostigmine, D. Tropicamide",B: Epinephrine,"Answer: B: Epinephrine Explanation: Ans: B. Epinephrine(Ref Goodman Gilman 12/e p326,. Katzung, 13/e p162, 12/e p160,. KDT 7th/e p153. 6th/e p123).Epinephrine acts on dilator pupillae, causing the dilatation of pupil analogous to that of pilocarpine on sphincter papillae.Sympathetic stimulation of adrenergic receptors causes the contraction of the radial muscle and subsequent dilation of the pupil.Parasympathetic stimulation causes contraction of the circular muscle and constriction of the pupil." "This is a real-world medical entrance exam question, please give the true answer based on the question and selection. Topic name: None, Subject name: Pediatrics","A neonate presented with jaundice on first day of life. His mother's blood group is `0' positive. How will you manage this patient? Select and explain. A. Observe only as it is mostly physiological jaundice, B. Exchange transfusion, C. Liver function tests and liver biopsy as it is mostly due to cholestasis, D. Phototherapy",D: Phototherapy,"Answer: D: Phototherapy Explanation: Ans: D. Phototherapy(Ref: Nelson 20/e p873)Jaundice on day 1 of life - Due to some hemolytic disease or congenital infections.Treated using phototherapy.Possibility of Rh incompatibility ruled out as mother blood group is 0+.Ideally basic investigations for hemolytic anemia should be sent simultaneously (not given in the option).Best option would be phototherapy." "This is a real-world medical entrance exam question, please give the true answer based on the question and selection. Topic name: None, Subject name: Dental","After brushing dryness of mouth is caused by Select and explain. A. Glycol, B. sodium Lauryl sulphate, C. Friction due to brushing, D. Sweetner",B: sodium Lauryl sulphate,"Answer: B: sodium Lauryl sulphate Explanation: None" "This is a real-world medical entrance exam question, please give the true answer based on the question and selection. Topic name: None, Subject name: Biochemistry","Rhodopsin deficiency is chiefly associated with: Select and explain. A. Vitamin D deficiency, B. Rickets, C. Vitamin A deficiency, D. Scurvy",C: Vitamin A deficiency,"Answer: C: Vitamin A deficiency Explanation: None" "This is a real-world medical entrance exam question, please give the true answer based on the question and selection. Topic name: None, Subject name: Dental","The maxillary teeth if placed too far anteriorly and superiorly in a complete denture results in the faulty pronunciation of: Select and explain. A. F and V sounds, B. S and T sounds, C. Vowels, D. Consonants",A: F and V sounds,"Answer: A: F and V sounds Explanation: None" "This is a real-world medical entrance exam question, please give the true answer based on the question and selection. Topic name: None, Subject name: Dental","Fill the missing data in the common risk factor approach diagram. Select and explain. A. Diet and smoking, B. Smoking and disease, C. Smoking and prevention, D. Prevention and diet",A: Diet and smoking,"Answer: A: Diet and smoking Explanation: None" "This is a real-world medical entrance exam question, please give the true answer based on the question and selection. Topic name: None, Subject name: Dental","The logical explanation for the unique anticariogenic property of most silicate cements is: Select and explain. A. The reduction in enamel solubility due to fluoride uptake by enamel, B. That beryllium flux is used in silicates, C. That silicates show very little leakage at the margins of the restoration, D. Due to the high silica content",A: The reduction in enamel solubility due to fluoride uptake by enamel,"Answer: A: The reduction in enamel solubility due to fluoride uptake by enamel Explanation: The incidence of secondary caries is markedly less around silicate restorations. This is surprising when considering that severe leakage takes place at its margins.  Also, the incidence of contact caries is less when compared to amalgam restorations (contact caries is the term applied to caries occurring on the proximal surface of the tooth adjacent to the restoration).  The anti-cariogenic property is due to presence of 15% fluoride. Fluoride release is slow and occurs throughout the life of the restoration. Silicate cement was classed as a severe irritant to the pulp because of its low pH (acidic).  For many years, silicate served as a standard for comparing the pulpal response to other materials. In deep cavities, the pulp had to be protected with varnish or calcium hydroxide. Manapallil 3rd Ed P: 88" "This is a real-world medical entrance exam question, please give the true answer based on the question and selection. Topic name: None, Subject name: Pharmacology","An unknown drug is being tested in experimental setup. The results obtained are given in the table. From these actions, new drug is likely to be:ParameterPlacebo treatedNew drug treatedHea rate7286Systolic BP110150Diastolic BP8068TremorsAbsentPresent Select and explain. A. Beta-I and beta-2 agonist, B. Alpha-1 antagonist and beta-2 agonist, C. M2 and M3 agonist, D. Alpha-1 and beta-1 agonist",A: Beta-I and beta-2 agonist,"Answer: A: Beta-I and beta-2 agonist Explanation: Ans. A. Beta-1 and beta-2 agonistBeta-1 stimulation increases hea rate and systolic blood pressure. Beta-2 stimulation cause vasodilation and thus decreases diastolic blood pressure and tremors. Hence, the drug appears to be beta-1 and beta-2 agonist." "This is a real-world medical entrance exam question, please give the true answer based on the question and selection. Topic name: None, Subject name: Pediatrics","All are true regarding Pseudotumour cerebri except – Select and explain. A. Ventricular system is of normal size or small, B. CT Scan shows punctate hypodense areas, C. No focal neurological deficit, D. Intracranial pressure is raised",B: CT Scan shows punctate hypodense areas,"Answer: B: CT Scan shows punctate hypodense areas Explanation: Pseudotumor cerebri Pseudotumor cerebri is characterized by increased intracranial pressure in the absence of an identifiable intracranial mass or hydrocephalus. Cardinal features of pseudotumour cerebri Headache, Clinically resembling that of brains tumor Normal cerebrospial fluid Normal or sin all ventricular size CT & 111R1 Normal No focal neurological deficit Transient visual obscurations and papilledema with enlarged blind spots." "This is a real-world medical entrance exam question, please give the true answer based on the question and selection. Topic name: None, Subject name: Ophthalmology","Changes seen in conjunctiva after vitamti. A deficiency: Select and explain. A. Actinic degeneration, B. Hyperplasia of goblet cells, C. Hyperkeratosis of squamous epithelium, D. Stromal infiltration",C: Hyperkeratosis of squamous epithelium,"Answer: C: Hyperkeratosis of squamous epithelium Explanation: Answer- C. Hyperkeratosis of squamous epitheliumChange seen in coniunctiva after vitamin A deficiency is hyperkeratosis of squamous epithelium.'Vitamin A is necessary for normal differentiation of nonsquamous epithelium; keratinization is a direct consequenceof its deficieny.Reduced aqueous tear production, and irregularities of the keratinized surface may all contribute to stromal melting, which can occur in the absence of inflammatory infiltration or bacterial invasion.Squamous metaplasia and keratinization." "This is a real-world medical entrance exam question, please give the true answer based on the question and selection. Topic name: None, Subject name: Dental","Mandibular 1st premolar normally has Select and explain. A. 1 root, B. 2 roots, C. May have any, D. None",A: 1 root,"Answer: A: 1 root Explanation: None" "This is a real-world medical entrance exam question, please give the true answer based on the question and selection. Topic name: None, Subject name: Pathology","Which one of the following is not a criterion for making a diagnosis of chronic myeloid leukemia in accelerated phase: Select and explain. A. Blasts 10-19% of WBC's in peripheral blood, B. Basophils 10-19% of WBC'S in peripheral blood, C. Increasing spleen size unresponsive to therapy, D. Persistent thrombocytosis (>1000 x 109/L) unresponsive to therapy",D: Persistent thrombocytosis (>1000 x 109/L) unresponsive to therapy,"Answer: D: Persistent thrombocytosis (>1000 x 109/L) unresponsive to therapy Explanation: Answer is D (Persistent thrombocytosis > 1000 x 109/L positive to therapy)Accelerated phase is associated with thrombocytopenia (9/L) and not thrombocytosis.Although multivariate analysis derived criteria also place blood or marrow basophils > 20% in disease criteria, other criteria do consider marrow and peripheral basophils > 10% as a criteria and hence option (D) is the single best answer here.Accelerated phase of CML: It is defined by the development of increasing degrees of anemia unaccounted for by bleeding or chemotherapy and is defined by several criteria. Multivariate analysis derived criteria Other commonly used criteriaCytogenetic clonal evolution Blood or marrow blasts between 10 and 20% Increasing drug dosage requirementBlood or marrow basophils > 20% Marrow reticulum or collagen fibrosis Platelet count < 100 x 10deg/L unrelated to therapy Marrow or peripheral blasts > 10%Marrow or peripheral basophils eosinophils - 10% Triad of WBC > 50 x 109/L, haematocrit < 25% and Mai,100 liP/L not controlled ii1111 therapy Unexplained fever or bone pain" "This is a real-world medical entrance exam question, please give the true answer based on the question and selection. Topic name: None, Subject name: Dental","Desensitization was given by: Select and explain. A. Bandura., B. Joseph wolpe., C. Addleston., D. Wright.",B: Joseph wolpe.,"Answer: B: Joseph wolpe. Explanation: None" "This is a real-world medical entrance exam question, please give the true answer based on the question and selection. Topic name: None, Subject name: Physiology","Which of the following hea sound may be a normal finding during pregnancy? Select and explain. A. Fixed splitting of S2, B. S3, C. S4, D. Pericardial knock",B: S3,"Answer: B: S3 Explanation: Ans. b. S3Ref Ganong's Review of Medical Physiology 2511' edn; Page no. 542Hea SoundsCauseCharactersSite of auscultationPhonocar- diogramECG correlation51-Vibrations set up by suddenclosure of AV valvesat the sta of ventricular systole, during phaseof isovolumetric contraction. Long & soft LUBB Duration -0.15 sFrequency- 25 - 45 HzBest heard over mitral and tricuspid areaSingle group of 9-13 waves - rescendoand diminuendo series of wavesCoincides with peak of R wave52 - Vibrations with closure ofsemilunar valves ust at the onset of ventricular diastole.Heard as a single sound during expiration but during inspiration the increased output of the right hea causes a physiological splitting. Sho,loud High Pitched DUBB Duration0.12 s Frequency 50 HzBest heard over aoic and pulmonary area Single group of 4-6 waves having same amplitudeCoincides with end of T wave CauseCharactersSite of auscultationPhonocar- diogramECG correlation53 -- Vibrations set up in cardiac wallby inrush of blood during rapid fillingphase of ventricular diastole. It may be heard duringlate pregnancy. In children and young adults, is a normalfinding.Sho, soft Low pitched Duration --0.1 sNormally cannot be heard byauscultation with stethoscope 1-- 4 waves grouped togetherAppears between T and P waves54 -- Vibrations set up during atrialsystole Coincides with last rapid filling phaseof ventricular diastole. Examples include concentrichyperophy, aoic stenosis, and myocardial infarction. Sho, low pitched Duration -- 0.03 sFrequency --3 HzNormally cannot be heard by auscultationwith stethoscope1-2 waves with very low amplitudeCoincides with interval betweenend of P wave and onset of Q wave" "This is a real-world medical entrance exam question, please give the true answer based on the question and selection. Topic name: None, Subject name: Pediatrics","All of the following about Vitamin D metabolism are true except Select and explain. A. 25–α hydroxylation takes place in liver, B. 1–α hydroxylation takes place in kidney, C. Daily requirement in the absence of sun–light is 450-6001U/day, D. Williams syndrome is associated with obesity, mental retardation, precocious puberty","D: Williams syndrome is associated with obesity, mental retardation, precocious puberty","Answer: D: Williams syndrome is associated with obesity, mental retardation, precocious puberty Explanation: In Williams syndrome there is failure to gain weight (not obesity) and delay in development (not-precocious puberty) Williams syndrome Williams syndrome is a rare neurodevelopmental disorder characterized by: Unusual facial features: Elfin facial appearance, widely spaced teeth, a long philthrum along with a low nasal bridge, an unusually cheerful demeanor and ease with strangers. Heart defects: Supravalvular aortic stenosis. Development delay: Mental retardation, failure to gain weight (failure to thrive), low muscle tone. Transient hypercalcemia Other features include GI problems (Colic, abdominal pain, diverticulitis), nocturnal enuresis, dental irregularities, hypothyroidism, hyperacussis and photophobia. Vitamin D It occurs mainly in two forms Vit D3 —> Cholecalciferol - synthesized in skin under the influence of UV rays from 7-dehydrocholesterol. Vit D2 —) Calciferol - present in irradiatiate food-yeast, fungi, bread. Activation of Vit D takes place in following manner." "This is a real-world medical entrance exam question, please give the true answer based on the question and selection. Topic name: None, Subject name: Pathology","Oral hairy leukoplakia is seen in AIDS patients. The most likely site of appearance is: Select and explain. A. Lateral borders of tongue, B. Sublingual mucosa, C. Soft palate, D. Buccal mucosa",A: Lateral borders of tongue,"Answer: A: Lateral borders of tongue Explanation: None" "This is a real-world medical entrance exam question, please give the true answer based on the question and selection. Topic name: None, Subject name: Dental","Most appropriate pontic design is: Select and explain. A. It should fill the missing teeth area, B. Greater lingual embrasure, C. Should contact mucosa but should not irritate it, D. None of the above",C: Should contact mucosa but should not irritate it,"Answer: C: Should contact mucosa but should not irritate it Explanation: None" "This is a real-world medical entrance exam question, please give the true answer based on the question and selection. Topic name: None, Subject name: Physiology","Which of the following is not a general compartment of body fluid ? Select and explain. A. Peritoneal, B. Intracellular, C. Interstitial, D. Blood plasma",B: Intracellular,"Answer: B: Intracellular Explanation: None" "This is a real-world medical entrance exam question, please give the true answer based on the question and selection. Topic name: None, Subject name: Pharmacology","Which of the following is true about carbamazepine? Select and explain. A. Not associated with agranulocytosis, B. Has been known to cause Steven-Johnson's syndrome, C. Causes serious nephrogenic toxicity, D. Drug monitoring is not required",B: Has been known to cause Steven-Johnson's syndrome,"Answer: B: Has been known to cause Steven-Johnson's syndrome Explanation: Answer- B. Has been known to cause Steven-Johnson's syndromeCarbamazepine has been known to cause Steven-Johnson's syndrome.High risk-NevirapineOxicam (NSAlDs)ThiacetazoneAllopurinolCarbamazepineLamotrigine" "This is a real-world medical entrance exam question, please give the true answer based on the question and selection. Topic name: None, Subject name: Social & Preventive Medicine","Regarding Evidence Based Medicine (EBM), all of the ing arc true, except: Select and explain. A. EBM depends on clinical model and decision analysis to base its recommendations, B. EBM objectively evaluates the quality of clinical research by critical assessing techniques repoed by researchers in publications, C. The strongest argument for the therapeutic intervention is systemic review or meta-analysis of triple blinded, randomized, placebo-controlled clinical trials with allocation concealment and complete follow-up involving a homogenous population of patients or medical disorders, D. The weakest argument is the opinion of expes or medical boards",A: EBM depends on clinical model and decision analysis to base its recommendations,"Answer: A: EBM depends on clinical model and decision analysis to base its recommendations Explanation: Answer: a. EBM depends on clinical model and decision analysis to base its recommendations It aims to apply the best available evidence gained from the scientific method to medical decision-makingand it seeks to assess the quality for evidence of the risks and benefits of treatments.EBM is the conscientious, explicit and judicious use of current best evidence in making decisions about the care of individual patients" "This is a real-world medical entrance exam question, please give the true answer based on the question and selection. Topic name: None, Subject name: Anatomy","Pharyngotympanic tube is supplied by all except: Select and explain. A. Ascending pharyngeal, B. Ascending palatine, C. Middle meningeal artery, D. Artery of pterygoid canal",B: Ascending palatine,"Answer: B: Ascending palatine Explanation: None" "This is a real-world medical entrance exam question, please give the true answer based on the question and selection. Topic name: None, Subject name: Pathology","Which one of the following is a connective tissue tumour: Select and explain. A. Lipoma, B. Melanoma, C. Carcinoma, D. Papilloma",A: Lipoma,"Answer: A: Lipoma Explanation: None" "This is a real-world medical entrance exam question, please give the true answer based on the question and selection. Topic name: None, Subject name: Surgery","In a patient with thrombocytopenia, what is the target platelet count after transfusion to perform an invasive procedure? Select and explain. A. 30,000, B. 40,000, C. 50,000, D. 60,000","C: 50,000","Answer: C: 50,000 Explanation: Ans: C. 50,000(Ref Bailey 27/e p22-23, 26/e p23, Nelson 20/e p2374).Target platelet count after transfusion for performing invasive procedure in thrombocytopenic patient = 50, 000.Blood ProductStandard GuidelinesFFPIf prothrombin time (PT) or paial thromboplastin time (PTT) > 1.5 times normal.CryoprecipitateIf fibrinogen < 0.8 g/LPlateletsIf platelet count <50 x 109/mL" "This is a real-world medical entrance exam question, please give the true answer based on the question and selection. Topic name: None, Subject name: Pathology","Sjogren's syndrome includes all except: Select and explain. A. Xerostomia, B. Keratoconjunctivitis, C. Arthritis, D. Lymphoma",D: Lymphoma,"Answer: D: Lymphoma Explanation: None" "This is a real-world medical entrance exam question, please give the true answer based on the question and selection. Topic name: None, Subject name: Pathology","Which of the following occurs most commonly on tongue? Select and explain. A. Lymphangioma and granular cell myoblastoma, B. Lipoma and fibroma, C. Neuroblastoma and lipoma, D. Lymphangioma and fibroma",A: Lymphangioma and granular cell myoblastoma,"Answer: A: Lymphangioma and granular cell myoblastoma Explanation: None" "This is a real-world medical entrance exam question, please give the true answer based on the question and selection. Topic name: None, Subject name: Dental","Vitamin D deficiency Select and explain. A. Does not affect enamel, B. Does not affect dentin, C. Does not affect tooth development, D. All of the above.",B: Does not affect dentin,"Answer: B: Does not affect dentin Explanation: None" "This is a real-world medical entrance exam question, please give the true answer based on the question and selection. Topic name: None, Subject name: Dental","Pulp proper resembles Select and explain. A. Loose connective tissue, B. Fine nerve fibers and blood vessels, C. Multipotent cells, D. All of the above",A: Loose connective tissue,"Answer: A: Loose connective tissue Explanation: None" "This is a real-world medical entrance exam question, please give the true answer based on the question and selection. Topic name: AIIMS 2018, Subject name: ENT","Best surgery to relieve intractable veigo in a menieres disease patient is? Select and explain. A. Surgical Labyinthectomy, B. Vestibular neurectomy, C. Endolymphatic sac decompression, D. Cochleosacculotomy",A: Surgical Labyinthectomy,"Answer: A: Surgical Labyinthectomy Explanation: Gold standard treatment for intractable veigo in patient of Menier's disease - Surgical Labyinthectomy Rx of Menier's disease Acute episode: Labyrinthine sedatives Maintenance phase Medical K+sparing diuretics B blockers Antihistamines Surgical Conservative :Decompression of endolymphatic sac ,Vestibular neurectomy. Radical :Surgical Labyinthectomy Intratympanic Gentamycin therapy Silverstein microwick microcatheter Meniett's device" "This is a real-world medical entrance exam question, please give the true answer based on the question and selection. Topic name: None, Subject name: Gynaecology & Obstetrics","A G6+0+0 lady with h/o recurrent missed aboions at 14-16 weeks comes to you with a missed aboion at 12 weeks. Which of the following tests is not warranted? Select and explain. A. Lupus anticoagulant, B. VDRL for husband and wife, C. Anticardiolipin antibody, D. Fetal karyotype",B: VDRL for husband and wife,"Answer: B: VDRL for husband and wife Explanation: Ans: B. VDRL for husband and wife(Ref Williams 24/e p358-359; Dutta 8/e p343, 7/e p167)VDRL:Simple test.Performed in initial work-up for all multiple aboion cases.All aboions are by 16th week while in syphilis, usually there is a improvement in the duration of pregnancy (Kassowitz Law).Kassowitz law:For untreated syphilis woman with series of pregnancies - Have lesser likelihood of infection of fetus from later pregnancies." "This is a real-world medical entrance exam question, please give the true answer based on the question and selection. Topic name: None, Subject name: Surgery","The cyst that moves by protruding the tongue is: Select and explain. A. Thyroglossal cyst, B. Median rhomboid cyst, C. Ranula, D. Tracheal cyst",A: Thyroglossal cyst,"Answer: A: Thyroglossal cyst Explanation: None" "This is a real-world medical entrance exam question, please give the true answer based on the question and selection. Topic name: None, Subject name: Gynaecology & Obstetrics","A woman comes with postdated pregnancy at 42 weeks. The initial evaluation would be: Select and explain. A. Induction of labour, B. Review of previous menstrual history, C. Cesarean section, D. USG",B: Review of previous menstrual history,"Answer: B: Review of previous menstrual history Explanation: A pregnancy continuing beyond two weeks of the expected date of delivery (> 42 weeks or >294 days) is called postmaturity or post-term pregnancy. Pregnancy between 41-42 weeks is called prolonged pregnancy. Most common cause of post term pregnancy is wrong dates so, a careful review of menstrual history is important in all such cases – “If the patient is sure about her date with previous history of regular cycles, it is a fairly reliable diagnostic aid in the calculation of the period of gestation. But in cases of mistaken maturity or pregnancy occurring during lactational amenorrhoea or soon following withdrawal of the pill’, confusion arises. In such cases, the previous well documented antenatal records of first visit in first trimester if available, are useful guides.” Dutta Obs. 6/e, p 319 Once the menstrual history is confirmed, investigations like USG and amniocentesis are done: To confirm fetal maturity To detect any evidence of placental insufficiency" "This is a real-world medical entrance exam question, please give the true answer based on the question and selection. Topic name: None, Subject name: Dental","Lingual ridge is most prominent in Select and explain. A. Upper Canine, B. Lower Canine, C. Upper Central Incisor, D. Lower central incisor",A: Upper Canine,"Answer: A: Upper Canine Explanation: None" "This is a real-world medical entrance exam question, please give the true answer based on the question and selection. Topic name: None, Subject name: Dental","Euphemism pudding paste is used for: Select and explain. A. Sealants., B. Varnish., C. GIC., D. Alginate.",D: Alginate.,"Answer: D: Alginate. Explanation: None" "This is a real-world medical entrance exam question, please give the true answer based on the question and selection. Topic name: None, Subject name: Gynaecology & Obstetrics","In heart patient the worst prognosis during pregnancy is seen in: Select and explain. A. Mitral regurgitation, B. Mitral valve prolapse, C. Aortic stenosis, D. Pulmonary stenosis",C: Aortic stenosis,"Answer: C: Aortic stenosis Explanation: Remember 3 ‘FUNDAS: Highest maternal mortality is associated with Class III of clarkes cl;assification, so any of those diseases are given they will have the worst prognosis. Amongst them also Eisenmengers syndrome has the worst prognosis. ↓ 2. Stenotic heart disease have a worse prognosis than regurgitant lesions. Among stenotic disease-(in alphabetical order)-Aortic stenosiswill have the worst> Mitral stenosis>Pulmonary stenosis. ↓ 3. Congenital heart disease and Mitral valve prolapse have the best prognosis So now this question becomes very easy-Mitarl vave prolapse has the best prognosis, so it is ruled out; Regurgitant lesions have a better prognosis than stenotic lesions so mitral regurgitation is also ruled out. Now we are left with 2 options, aortic stenosis and pulmonary stenosis-as I said go alphabetically, aortic stenosis will have a worse prognosis than pulmonary. Let’s consider each of the options one by one and see what Williams has to say about each of them. Option “a” Mitral regurgitation “M R is well-tolerated during pregnancy probably due to decreased systemic vascular resistance which actually results in less regurgitation. Heart failure only rarely develops during pregnany.” Williams Obs. 22/e, p 1026, 23/e, p 966 Option “b” Mitral valve prolapse “Pregnant women with mitral valve prolapse rarely have cardiac complications. In fact pregnancy induced hypervolemia may improve alignment of mitral valve.” Williams Obs. 22/e, p 1030, 23/e, p 971 Option “c” Aortic stenosis “Although mild to moderate degree of aortic stenosis is well tolerated but severe degree is life threatening.” Williams Obs. 22/e, p 1026, 23/e, p 967 Option “d” Pulmonary stenosis “It is well tolerated during pregnancy and rarely causes any complication.” Williams Obs. 22/e, p 1027, 23/e, p 968" "This is a real-world medical entrance exam question, please give the true answer based on the question and selection. Topic name: None, Subject name: Pharmacology","Acute Barbiturate poisoning results in: Select and explain. A. Renal failure, B. Liver failure, C. Respiratory failure, D. Convulsions",C: Respiratory failure,"Answer: C: Respiratory failure Explanation: None" "This is a real-world medical entrance exam question, please give the true answer based on the question and selection. Topic name: None, Subject name: Dental","The direct bonded orthodontic stainless steel brackets derive retention with composite because of: Select and explain. A. The mechanical interlock with mesh at the bracket base, B. The chemical interlock of composite with bracket base, C. Both mechanical and chemical interlock of composite with the bracket base, D. Biological interlock between the tooth surface and bracket",A: The mechanical interlock with mesh at the bracket base,"Answer: A: The mechanical interlock with mesh at the bracket base Explanation: None" "This is a real-world medical entrance exam question, please give the true answer based on the question and selection. Topic name: None, Subject name: Gynaecology & Obstetrics","A 26 years old patient c/o foul smelling greyish white discharge diagnosed to be Gardnerella vaginalis infection. Microscopic finding is suggestive of? Select and explain. A. Group of bacilli arranged in chain forms, B. Bacteria found to be engulfed by macrophages, C. Bacteria adherent to lining vaginal epithelial cells, D. Bacteria arranged in cluster forms",C: Bacteria adherent to lining vaginal epithelial cells,"Answer: C: Bacteria adherent to lining vaginal epithelial cells Explanation: Ans C. Bacteria adherent to lining vaginal epithelial cellsGardnerella vaginalis, facultatively anaerobic gram-variable rod, is one of the organisms responsible for bacterial vaginosis (BV).The vaginal discharge of BV is characteristically described as a thin, gray, homogeneous fluid that is adherent to the vaginal mucosa. A fishy vaginal odour, which is paicularly noticeable following coitusis present.Demonstration of clue cells on a saline smear is the most specific criterion for diagnosing BV.The whiff test may be positive in up to 70% of BV patients.The vaginal discharge of patients with BV is notable for its lack of polymorphonuclear leukocytes (PMNs), typically 1 or less than 1 PMN per vaginal epithelial cell." "This is a real-world medical entrance exam question, please give the true answer based on the question and selection. Topic name: None, Subject name: Ophthalmology","Optic atrophy is not seen in: Select and explain. A. Retinitis pigmentosa, B. Methanol poisoning, C. Central retinal aerial occlusion (CRAO), D. Polypoidal choroidal vasculopathy",D: Polypoidal choroidal vasculopathy,"Answer: D: Polypoidal choroidal vasculopathy Explanation: Answer- D. Polypoidal choroidal vasculopathyRetinitis pigmentosa, Methanol poisoning and Central retinal aerial occlusion (CRAO) can lead to optic atrophy.Occurs secondary to retinal disease (disease of inner retina or its blood supply)Its ascending type of optic atrophy.Causes:Retinitis pigmentosaCRAOExtensive retino choroiditis" "This is a real-world medical entrance exam question, please give the true answer based on the question and selection. Topic name: AIIMS 2019, Subject name: Social & Preventive Medicine","Which vaccine is effective for Mass vaccination post-disaster? Select and explain. A. Cholera, B. Typhoid, C. Measles, D. Scrub typhus",C: Measles,"Answer: C: Measles Explanation: Measles vaccine is effective post-disaster. Incubation period - 10-14 days IP of Vaccine Induced Measles(VIM) - 7 days, hence immunity is provided before the natural infection. The highest risk of transmission post-disaster is of diarrhoeal and respiratory diseases, which have an incubation period of a few days. ABSOLUTELY contraindicated in post-disaster phase for mass vaccination Typhoid Cholera Tetanus ." "This is a real-world medical entrance exam question, please give the true answer based on the question and selection. Topic name: None, Subject name: Gynaecology & Obstetrics","Dose of radiation for early and locally advanced cancer cervix at point A during brachytherapy? Select and explain. A. 70-75 Gray and 75- 80 Gray, B. 75-80 Gray and 80-85 Gray, C. 80-85 Gray and 85-90 Gray, D. 85-90 Gray and 90-95 Gray",C: 80-85 Gray and 85-90 Gray,"Answer: C: 80-85 Gray and 85-90 Gray Explanation: Answer- C. 80-85 Gray and 85-90 GrayEarly stage disease (nonbulky Stage I-II): 80-85 GyAdvanced stage disease (bulky or Stage IIIB): 85-90 Gy" "This is a real-world medical entrance exam question, please give the true answer based on the question and selection. Topic name: AIIMS 2018, Subject name: Psychiatry","An individual with a known psychiatric disorder or on treatment and is not a minor can chose to decide the care taker and the course of treatment according to mental health act. This is called: Select and explain. A. Advance directive, B. Treatment directive, C. Mental will, D. Future directive",A: Advance directive,"Answer: A: Advance directive Explanation: MENTAL HEALTH CARE ACT 2017 (1) Mental health establishment (2) Capacity to make mental health care and treatment decisions Patient Should be able to understand information given to him Should be able to know about consequences about the decision Should be able to express their decision any form (3) Advanced directive - legal document in which a patient write what action should be taken when they are not able to make decision about their health (4) Nominated representative (5) Ban on direct ECT (6) Ban on ECT in minors (7) Ban on psychosurgery (8) Decriminalisation of suicide" "This is a real-world medical entrance exam question, please give the true answer based on the question and selection. Topic name: None, Subject name: Dental","Sealer penetration into dentinal tubule depends mainly on? Select and explain. A. Type of sealer used, B. Method of obturation, C. Not Recalled, D. Not Recalled",B: Method of obturation,"Answer: B: Method of obturation Explanation: None" "This is a real-world medical entrance exam question, please give the true answer based on the question and selection. Topic name: None, Subject name: Dental","Student t test is: Select and explain. A. Parametric test based on average, B. Non parametric test based on average, C. Parametric test based on variance, D. Non parametric test based on variance",A: Parametric test based on average,"Answer: A: Parametric test based on average Explanation: None" "This is a real-world medical entrance exam question, please give the true answer based on the question and selection. Topic name: None, Subject name: Surgery","During maxillary osteotomy, which of the following can be given to prevent bleeding: Select and explain. A. Acetoacetic acid, B. Lactic acid, C. Heparin, D. Tranexamic acid",D: Tranexamic acid,"Answer: D: Tranexamic acid Explanation: None" "This is a real-world medical entrance exam question, please give the true answer based on the question and selection. Topic name: None, Subject name: Pathology","What is the term for the radio opaque area found at the root apex of young permanent teeth involved with a chronic pulpitis? Select and explain. A. Apical cyst, B. Apical condensing osteitis, C. Chronic apical periodontitis, D. Stage one apical osteofibroses",B: Apical condensing osteitis,"Answer: B: Apical condensing osteitis Explanation: None" "This is a real-world medical entrance exam question, please give the true answer based on the question and selection. Topic name: None, Subject name: Surgery","When force are applied on lateral surface at angle region, compression is generated on: Select and explain. A. Superior surface, B. Lateral surface, C. Inferior surface, D. Medial surface",B: Lateral surface,"Answer: B: Lateral surface Explanation: None" "This is a real-world medical entrance exam question, please give the true answer based on the question and selection. Topic name: None, Subject name: Dental","Child of an Overindulgent mother will be: Select and explain. A. Shy, submissive., B. Aggressive, display temper tantrums., C. Evasive and dawdling., D. Disobedient.","B: Aggressive, display temper tantrums.","Answer: B: Aggressive, display temper tantrums. Explanation: Maternal attitude and child behaviour" "This is a real-world medical entrance exam question, please give the true answer based on the question and selection. Topic name: None, Subject name: Dental","Which of the following is not a behavioral theory: Select and explain. A. Cognitive theory., B. Hierarchy of needs., C. Social learning., D. Classical conditioning.",A: Cognitive theory.,"Answer: A: Cognitive theory. Explanation: None" "This is a real-world medical entrance exam question, please give the true answer based on the question and selection. Topic name: None, Subject name: Physiology","myasthenia gravis is a disorder of Select and explain. A. Peripheral nerve, B. Spinal cord, C. Motor neuron, D. Neuromuscular junction",D: Neuromuscular junction,"Answer: D: Neuromuscular junction Explanation: None" "This is a real-world medical entrance exam question, please give the true answer based on the question and selection. Topic name: None, Subject name: Dental","In occlusion, the teeth have Select and explain. A. Cusp-to-cusp contact, B. Edge-to-edge contact, C. Marginal contact, D. Surface-to-surface contact",D: Surface-to-surface contact,"Answer: D: Surface-to-surface contact Explanation: None" "This is a real-world medical entrance exam question, please give the true answer based on the question and selection. Topic name: None, Subject name: Surgery","Submental intubation is an alternative to tracheostomy in which type of fractures? Select and explain. A. Mandibular fracture, B. Nasal fracture, C. Panfacial fracture, D. Styloid process fracture",C: Panfacial fracture,"Answer: C: Panfacial fracture Explanation: None" "This is a real-world medical entrance exam question, please give the true answer based on the question and selection. Topic name: None, Subject name: Anatomy","Which of the following is supplied by the glossopharyngeal nerve? Select and explain. A. Stylopharyngeus, B. Palatopharyngeus, C. Geniohyoid, D. Genioglossus",A: Stylopharyngeus,"Answer: A: Stylopharyngeus Explanation: None" "This is a real-world medical entrance exam question, please give the true answer based on the question and selection. Topic name: None, Subject name: Anatomy","Cardiac pain may be transmitted to the jaw due to the overlapping of: Select and explain. A. 5th cranial nerve, third cervical nerve and first thoracic nerve, B. 7th cranial nerve, third cervical nerve and first thoracic nerve, C. 5th cranial nerve, second cervical nerve and first thoracic nerve, D. 7th cranial nerve, second cervical nerve and first thoracic nerve","A: 5th cranial nerve, third cervical nerve and first thoracic nerve","Answer: A: 5th cranial nerve, third cervical nerve and first thoracic nerve Explanation: Cardiac pain may be transmitted to the jaw due to the overlapping of 5th cranial nerve, third cervical nerve, and first thoracic nerve" "This is a real-world medical entrance exam question, please give the true answer based on the question and selection. Topic name: None, Subject name: Dental","Considering the morphology of root and pulp canals, a root canal instrument should be placed in what direction to gain access to the Mesiofacial root of permanent maxillary first molar: Select and explain. A. From the mesiobuccal, B. From the distobuccal, C. From the mesiolingual, D. From the distolingual",D: From the distolingual,"Answer: D: From the distolingual Explanation: None" "This is a real-world medical entrance exam question, please give the true answer based on the question and selection. Topic name: None, Subject name: Dental","Optimal landmark for termination of cleaning, shaping and obturation is Select and explain. A. Cementodentinal junction, B. Apical constriction, C. Minor diameter, D. All of the above",A: Cementodentinal junction,"Answer: A: Cementodentinal junction Explanation: None" "This is a real-world medical entrance exam question, please give the true answer based on the question and selection. Topic name: None, Subject name: Gynaecology & Obstetrics","True about congenital diseases in diabetes mellitus is all except: Select and explain. A. Results due to free radical injury, B. 6-10% cases are associated with major congenital abnormality, C. 1-2% of newborns are associated with single umbilical artery, D. Insulin can be given",B: 6-10% cases are associated with major congenital abnormality,"Answer: B: 6-10% cases are associated with major congenital abnormality Explanation: Lets see each option separately - Congenital disease in diabetes mellitus Option a i.e. Results due to free radical injury - True - Congenital malformation in a case of diabetes can be due to variety of reasons like, Dutta Obs. 6/e, p 287 Genetic susceptibility Hyperglycemia - It is seen that good glycemic control indicated by HbAIC levels < %.9 can significantly lower the risk of fetal malformation. Arachidonic acid deficiency Ketone body formation Free Radical injury Somatomedin inhibition. Option b–6 to 10% cases are associated with major congenital abnormality Here we will have to read the option very carefully - the option is talking about Major congenital anomalities and not all anomalies. Dutta Obs 6/e, p- 287 says overall incidence of congenital Malformations is 6-10%. Williams 24/e p 1128. “The incidence of major malformations in women with type I diabetes is ~ 5%” Hence option b is incorrect Option c - 1 - 2% of newborns are associated with single umbilical artery This option can be taken in + /– status because no where the incidence of single umbilical artery in a case of diabetes has been mentioned separately. Whatever little information we have is from Dutta Obs. 6/e, p 220 Single umbilical artery: It is present in 1-2% cases (overall) May be due to failure of development of artery or due to its atrophy in later months It is seen in case of i. Twins ii. Babies born to diabetic mothers iii. In polyhydramines Single umbilical artery has been associated with congenital malformations of the fetus in 10-20% cases viz- Renal & Genital anomalies and fetal Trisomy There is increased incidence of abortions, prematurity, IUGR and increased perinatal mortality. Option d- insulin can be given There is no doubt as far as this option is concerned as insulin is the TOC for controlling hyperglycemia in case of diabetes in pregnancy. So from above discussion it is clear that option ‘b’ is absolutely incorrect, so we are opting it out." "This is a real-world medical entrance exam question, please give the true answer based on the question and selection. Topic name: None, Subject name: Biochemistry","Phenylbutyrate is used in management of urea cycle disorders. What is its role? Select and explain. A. Activates enzymes of urea cycle, B. Excretion of products of urea cycle, C. Maintains energy production, D. Scavenges nitrogen",D: Scavenges nitrogen,"Answer: D: Scavenges nitrogen Explanation: Answer- D. Scavenges nitrogenPhenylbutyrate is used to treat urea cycle disorders, because its metabolites offer an alternative pathway to the urea cycle to allow excretion of excess nitrogen. Urea cycle disorders result in the accumulation of precursors of urea, principally ammonia and glutamine. Phenylbutyrate provides an alternate means of detoxfication of glutamine acetylation, which bypasses the urea cycle." "This is a real-world medical entrance exam question, please give the true answer based on the question and selection. Topic name: None, Subject name: Medicine","A 60 years old patient having cardiorespiratory arrest in hospital ward, for oxygenation what should be done immediately? Select and explain. A. Nasal intubation, B. Oral intubation, C. Cricothyroid membrane puncture, D. Tracheostomy",B: Oral intubation,"Answer: B: Oral intubation Explanation: None" "This is a real-world medical entrance exam question, please give the true answer based on the question and selection. Topic name: None, Subject name: Medicine","Which of the following doesn't have any effect on pancreatic secretion? Select and explain. A. Cck, B. Gastrin, C. Secretin, D. Gastric inhibitory polypeptide",D: Gastric inhibitory polypeptide,"Answer: D: Gastric inhibitory polypeptide Explanation: Answer- D. Gastric inhibitory polypeptideCCK- As chyme floods into the small intestine, cholecystokinin is released into blood and binds to receptors on pancreatic acinar cells, ordering them to secrete large quantities of digestive enzymes.Secretin-The predominant effect of secretin on the pancreas is to stimulate duct cells to secrete water and bicarbonate.Gastrin- Stimulate acid secretion by the parietal cell, gastrin stimulates pancreatic acinar cells to secrete digestive enzymes." "This is a real-world medical entrance exam question, please give the true answer based on the question and selection. Topic name: None, Subject name: Anatomy","If there is absence of precursor cell of an organ with the subsequent non development of the organ, what is the condition is called as? Select and explain. A. Agenesis, B. Aplasia, C. Atresia, D. Atrophy",A: Agenesis,"Answer: A: Agenesis Explanation: Aplasia: - Failure of organ to develop with only rudiment of organ present. It occurs, when precursor cells are there but they do not differentiate into the organ.  Agenesis: - Complete failure of organ to develop, i.e. e complete absence of the organ, with no rudiment. It occurs when there is not precursor cell of an organ.  Atresia: - Ducts in the body or their openings or any orifice or passage is abnormally closed or absent.  Atrophy: - Shrinkage in the size of an organ by a decrease in cell size and number." "This is a real-world medical entrance exam question, please give the true answer based on the question and selection. Topic name: None, Subject name: Dental","Habit developed under psychological cause is: Select and explain. A. Non Compulsive habits., B. Unintentional habits., C. Useful habits., D. Intentional habits.",D: Intentional habits.,"Answer: D: Intentional habits. Explanation: None" "This is a real-world medical entrance exam question, please give the true answer based on the question and selection. Topic name: None, Subject name: Surgery","Treatment of fibrous dysplasia is: Select and explain. A. Radical resection of lesion, B. Radiation therapy, C. If the Lesion is small, dissection is done, if Lesion is large cosmetic surgery has to be carried out, D. Cryosurgery","C: If the Lesion is small, dissection is done, if Lesion is large cosmetic surgery has to be carried out","Answer: C: If the Lesion is small, dissection is done, if Lesion is large cosmetic surgery has to be carried out Explanation: None" "This is a real-world medical entrance exam question, please give the true answer based on the question and selection. Topic name: None, Subject name: Gynaecology & Obstetrics","A 10-year-old girl presents with a mass in lower abdomen involving umbilical and the hypogastrium. On examination it is cystic and mobile and the examiner is unable to insinuate fingers between the mass and the pelvic bone. What is the likely diagnosis? Select and explain. A. Duplication of small intestine, B. Omental cyst, C. Ovarian cyst, D. Mesenteric cyst",C: Ovarian cyst,"Answer: C: Ovarian cyst Explanation: Ans: C. Ovarian cyst(Ref Shaw 16/e p83, 448, 15/e p79, 385)* Accoording to examination, swelling is typically arising from pelvis & hand cannot be insinuated between mass & pelvic bone.* Best option is only ovarian cyst arises from pelvis.* Swellings arising from pelvis - Identified by abdominal palpation. - Sensitive ulnar border of left hand - Used from above downwards to palpate swellings arising from pelvis. - Upper & lateral margins felt. - Lower border unreached, i.e. the hand cannot be insinuated between mass & pelvis." "This is a real-world medical entrance exam question, please give the true answer based on the question and selection. Topic name: AIIMS 2019, Subject name: Social & Preventive Medicine","Improving Quality of Labour room is covered under which program? Select and explain. A. LaQshya, B. Improving care of newborn, C. Ayushman Bharat Scheme, D. JSSK",A: LaQshya,"Answer: A: LaQshya Explanation: 'LaQshya' programme of the Ministry of Health and Family Welfare (Launched in 2017) under the umbrella of NHM aims at improving quality of care in labour room and maternity Operation Theatre (OT). GOAL : Reduce preventable maternal and newborn moality. Morbidity and stillbihs associated with the care around delivery in the labour room and maternity OT. Ensure respectful maternity care." "This is a real-world medical entrance exam question, please give the true answer based on the question and selection. Topic name: None, Subject name: Pathology","A 26 years old female presented with pallor and hemoglobin of 9.5 mg/dl, PCV 30 mm Hg and RBC count of 2 million/mm. What is the most likely diagnosis? Select and explain. A. Iron deficiency anemia, B. Sideroblastic anemia, C. Thalassemia, D. Folic acid deficiency",D: Folic acid deficiency,"Answer: D: Folic acid deficiency Explanation: Ans: D. Folic acid deficiency(Ref Robbins 9/e p631, 648)High MCV, MCH & MCHC - Suggestive of macrocytic, hyperchromic anemia.Typically seen in folic acid deficiency.Folic acid (pteroylmono- glutamic acid) deficiency a results in megaloblastic anemia.Same pathologic features as vitamin B12 deficiency.Interpretation:In question, Hb= 9.5 mg/dL, PCV = 30, RBC count = 2 millions/mm3Hematocrit or PCV = MCV x RBC concentrationMCV PCV/RBC count = 30/0.2 = 150 (increased/Macrocytic)MCH = Hb/ RBC count =9.5/0.2 = 47.5 (increased/Hyperchromic)MCHC = Hb/MCV = 31 (normal)Macrocytic, hyperchromic anemia is typically seen in folic acid deficiency." "This is a real-world medical entrance exam question, please give the true answer based on the question and selection. Topic name: None, Subject name: Anatomy","The cribriform plate connects: Select and explain. A. Nasal cavity and anterior cranial fossa, B. Nasal cavity and orbit, C. Nasal cavity and oral cavity, D. Anterior cranial fossa and orbit",A: Nasal cavity and anterior cranial fossa,"Answer: A: Nasal cavity and anterior cranial fossa Explanation: The cribriform plate of the ethmoid bone separates the anterior cranial fossa from the nasal cavity. It is a horizontal perforated bony lamina, occupying ethmoidal notch of frontal bone. The cribriform plate contains foramina for olfactory nerve rootlets" "This is a real-world medical entrance exam question, please give the true answer based on the question and selection. Topic name: None, Subject name: Pharmacology","Major mechanism of transport of drugs across biological membranes is by ? Select and explain. A. Passive diffusion, B. Facilitated diffusion, C. Active transport, D. Endocytosis",A: Passive diffusion,"Answer: A: Passive diffusion Explanation: None" "This is a real-world medical entrance exam question, please give the true answer based on the question and selection. Topic name: None, Subject name: Skin","A child presented with complaint of severe itching over the web of fingers, more at night. Examination revealed burrows. Most probable diagnosis is:September 2009 Select and explain. A. Tinea cruris, B. Scabies, C. Infantile eczema, D. Papular uicaria",B: Scabies,"Answer: B: Scabies Explanation: Ans. B: ScabiesScabies is essentially a disease of the children. The itching appears a few days after infestation. It may occur within a few hours if the mite is caught a second time.The itch is characteristically more severe at night and affects the trunk and limbs.It does not usually affect the scalp.Burrow is the pathognomic lesion of scabies.Scabies burrows appear as tiny grey irregular tracks between the fingers and on the wrists.They may also be found in armpits, buttocks, on the penis, insteps and backs of the heels. Microscopic examination of the contents of a burrow may reveal mites, eggs or mite faeces (scybala)." "This is a real-world medical entrance exam question, please give the true answer based on the question and selection. Topic name: None, Subject name: Anatomy","Middle constrictor of pharynx has attachment from: Select and explain. A. Body of hyoid bone, B. Mandible, C. Pterygomandibular raphae, D. Cricoid cartilage",A: Body of hyoid bone,"Answer: A: Body of hyoid bone Explanation: None" "This is a real-world medical entrance exam question, please give the true answer based on the question and selection. Topic name: None, Subject name: Social & Preventive Medicine","If we know the value of one variable in an individual and wish to know the value of another variable, we calculate - Select and explain. A. Coefficient of correlation, B. Coefficient of regression, C. SE of mean, D. Geometric mean",B: Coefficient of regression,"Answer: B: Coefficient of regression Explanation: REGRESSION: Is change in measurements of a variable Provides structure of relationship between 2 quantitative variables Regression Coefficient (b): Measure of change of one dependent variable (y) with change in independent variable (x) or variables (x1, x2, x3……) Equations of regression, y = a + b (x) y = a + b (x1) + c (x2) + d (x3), where y is a dependent variable and x, x1, x2, x3 are independent variables; a is a constant and b, c, d are regression coefficients Types of regressions: – Simple linear regression: Only one dependent variable and one independent variable – Multiple linear regression: Only one dependent variable and more than one independent variable – Simple curvilinear regression: Only one dependent variable and one independent variable, with some power of independent variable – Multiple curvilinear regression: Only one dependent variable and more than one independent variables, with some power of independent variables). Types of regression equations:" "This is a real-world medical entrance exam question, please give the true answer based on the question and selection. Topic name: None, Subject name: Medicine","Which of the following infections causes a non-centrally distributed rash? Select and explain. A. Epidemic typhus, B. Measles, C. Secondary syphilis, D. Typhoid",A: Epidemic typhus,"Answer: A: Epidemic typhus Explanation: Ans: A. Epidemic typhus(Ref: Harrison p1158, 1051, 1297, 1135)Centrifugal rash:Stas from face or trunk centrally a moves to extremities peripherally.Centripetal rash:Stas from extremities peripherally a moves to trunk & face centrally.Non-centripetally distributed (centrifugal) rash:Seen in epidemic typhus.Epidemic typhus:Rash begins on upper trunk on 50th day.Later becomes generalized - (Involves entire body except face, palms & soles).Initially a macular rash.Progresses to maculopapular, petechial & confluent." "This is a real-world medical entrance exam question, please give the true answer based on the question and selection. Topic name: None, Subject name: Biochemistry","Hepcidin decreases iron absorption by inhibition of - Select and explain. A. Hephaestin, B. Ferropoin, C. Divalent metal ion transpoer, D. Transferrin",B: Ferropoin,"Answer: B: Ferropoin Explanation: Ans - b). FerropoinIron leaves the mucosal cell a transpo protein ferropoin, but only if there is free transferrin in plasma to bind to. Once transferrin is saturated with iron, any that has accumulated in the mucosal cells is lost when the cells are shed. Expression of the ferropoin gene is downregulated by hepcidin, a peptide secreted by the liver when body iron reserves are adequate. In response to hypoxia, anemia, or hemorrhage, the synthesis of hepcidin is reduced, leading to increased synthesis of ferropoin and increased iron absorption. As a result of this mucosal barrier, only ~10% of dietary iron is absorbed, and only 1 to 5% from many plant foods." "This is a real-world medical entrance exam question, please give the true answer based on the question and selection. Topic name: None, Subject name: Physiology","Detachment of myosin head from actin is caused by? Select and explain. A. Entry of calcium into sarcoplasmic reticulum, B. Change in troponin C configuration, C. Binding of ATP, D. Release of ADP and Pi",C: Binding of ATP,"Answer: C: Binding of ATP Explanation: Ans.C. Binding of ATPBinding of ATP on the free site of myosin leads to the detachment of myosin head from thin filament." "This is a real-world medical entrance exam question, please give the true answer based on the question and selection. Topic name: None, Subject name: Dental","Integration between blade vent implant and bone Is Select and explain. A. Fibto-oeseous integration, B. Osseo-integration, C. Osseo vent integration, D. Fibrocartilaginous",A: Fibto-oeseous integration,"Answer: A: Fibto-oeseous integration Explanation: None" "This is a real-world medical entrance exam question, please give the true answer based on the question and selection. Topic name: None, Subject name: Physiology","Characteristic feature true about outer membrane of peripheral nerve Select and explain. A. Relative permeable to sodium ions, B. Relative permeable to potassium ions, C. Permeable to chloride ions, D. Impermeable to potassium ions",B: Relative permeable to potassium ions,"Answer: B: Relative permeable to potassium ions Explanation: None" "This is a real-world medical entrance exam question, please give the true answer based on the question and selection. Topic name: None, Subject name: Medicine","A 14 years old male child presented with abnormal body movements with MRI showing signal changes in corpus striatum, thalami, pons, medulla, centrum semiovale and asymmetric diffuse white matter involvement. Most likely diagnosis is: Select and explain. A. Parkinsonism, B. Nigrostriatal degeneration, C. Wilson's disease, D. Hallervorden-Spatz disease",C: Wilson's disease,"Answer: C: Wilson's disease Explanation: Answer- C. Wilson's diseaseWilson disease:Wilson disease usually presents with tremors, rigidity, dementia, and pseudobulbar features and has an autosomal recessive mode of inheritance.Slit-lamp examination of the eyes may reveal a Kayser-Fleischer ring.MRI in Wilson disease exhibits the characteristic changes consisting of high-intensity lesions in the basal ganglia, thalami, and mid brain on T2-weighted images. The normal low intensity of red nuclei and SN surrounded by abnormal high-signal intensity in the tegmentum of the mid brain gives rise to the typical 'face-of-the-giant-panda' sign.Best diagnostic clue is symmetrical T2 hyperintensity." "This is a real-world medical entrance exam question, please give the true answer based on the question and selection. Topic name: None, Subject name: Pathology","Which of the following is the most likely (among them) to turn malignant? Select and explain. A. Intradermal nevus, B. Junctional nevus, C. Lichen planus, D. Papilloma",B: Junctional nevus,"Answer: B: Junctional nevus Explanation: None" "This is a real-world medical entrance exam question, please give the true answer based on the question and selection. Topic name: None, Subject name: Surgery","In case of Condylar fracture dysarthrosis refers to: Select and explain. A. Disturbance of anatomy but function is alright., B. Disturbance of both anatomy as well as function., C. Disturbance of function only., D. All of the above.",B: Disturbance of both anatomy as well as function.,"Answer: B: Disturbance of both anatomy as well as function. Explanation: option A is called as Metarthrosis." "This is a real-world medical entrance exam question, please give the true answer based on the question and selection. Topic name: None, Subject name: Dental","Gingival deformities are seen in: Select and explain. A. ANUG, B. Internal resorption, C. Cementomas, D. Periapical cysts",A: ANUG,"Answer: A: ANUG Explanation: None" "This is a real-world medical entrance exam question, please give the true answer based on the question and selection. Topic name: AIIMS 2019, Subject name: Social & Preventive Medicine","Numerator in Perinatal moality is Select and explain. A. Post neonate death with weight 2.5 kg, B. Early neonatal with weight 1000 grams, C. Aboion of < 500 gram foetus, D. Still bih of fetus > 500 grams",B: Early neonatal with weight 1000 grams,"Answer: B: Early neonatal with weight 1000 grams Explanation: - Perinatal perioda 28 weeks period of gestation till 7 days post delivery - Perinatal Deaths Still bihs Early Neonatal death - Cut- off / Indicators for perinatal moality: - Bihweighta > 1000 grams Period of Gestationa > 28 weeks Bih lengtha >35cms - Perinatal Moality Rate (PNMR) = x1000 ( LB = Live bih ; PND = Perinatal death) - PNMR in India= 23 per 1000 live bihs." "This is a real-world medical entrance exam question, please give the true answer based on the question and selection. Topic name: None, Subject name: Dental","Most commonly missing primary teeth are: Select and explain. A. Maxillary central incisors, B. Mandibular central incisors, C. Maxillary lateral incisors, D. Maxillary canines",D: Maxillary canines,"Answer: D: Maxillary canines Explanation: Developmental agenesis of primary teeth is relatively rare. When several primary teeth fail to develop, other ectodermal deficiencies are usually evident. Most commonly missing primary teeth are maxillary canines. Reference: McDONALD AND AVERY’S DENTISTRY for the CHILD and ADOLESCENT, 10th ed page no 64" "This is a real-world medical entrance exam question, please give the true answer based on the question and selection. Topic name: None, Subject name: Surgery","In a patient with dehydration, which of the following color intravenous cannula will you place for rapid fluid resuscitation? Select and explain. A. Grey, B. Blue, C. Pink, D. Green",A: Grey,"Answer: A: Grey Explanation: Ans:. A. Grey GaugeColor codeExternal DiameterLengthFlow Rate14GOrange2.1 mm45 mm240 ml/min16GGrey1.8 mm45 mm180 ml/min18GGreen1.3 mm32/45 mm90 ml/min20GPink1.1 mm32 mm60 ml/min22GBlue0.9 mm25 mm36 ml/min24GYellow0.7 mm19 mm20 ml/min26GViolet0.6 mm19 mm13 ml/min" "This is a real-world medical entrance exam question, please give the true answer based on the question and selection. Topic name: None, Subject name: Biochemistry","A girl complaints of acute abdominal pain on and off with tingling sensation of limbs. She had a history of eating paint from the wall of newly built house. Which of the following enzyme deficiency will be the cause of her condition? Select and explain. A. ALA dehydratase, B. ALA synthase, C. Coproporphyrinogen synthase, D. Heme synthase",A: ALA dehydratase,"Answer: A: ALA dehydratase Explanation: Answer- A. ALA dehydrataseALA dehydratase is a zinc-containing enzyme and is sensitive to inhibition by lead (present in the paints). High levels of lead can affect heme metabolism by combining with SH groups in enzymes such as ferrochelatase and ALA dehydratase.The most common presentation of lead poisoning is an encephalopathy.Laboratory investigation can reveal a microcytic hypochromic anemia with basophilic stippling of erythrocytes, an elevated serum lead level, and an elevated serum coproporplryrin level. A 24-h urine collection demonstrates elevated levels of lead excretion.The neuropathy is characterized by an insidious and progressive onset of weakness usually beginning in the arms, in paicular involving the wrist and finger extensors." "This is a real-world medical entrance exam question, please give the true answer based on the question and selection. Topic name: AIIMS 2018, Subject name: Surgery","Where is the second step of damage control resuscitation carried out? Select and explain. A. In emergency, B. In ICU, C. In OT, D. Prehospital resuscitation",B: In ICU,"Answer: B: In ICU Explanation: Phases of Damage control Surgery Phase (Initial exploration) Phase II (Secondary Resuscitation) Phase III (Definitive Operation) Consists of an initial operative exploration to attain rapid control of active hemorrhage & contamination. Abdomen is entered a middle incision and if exsanguinating hemorrhage is encountered four quadrant packing should be performed Any violation of GI tract should be treated with suture closure or segmental stapled resection External drains are placed to control any major pancreatic or biliary injuries. Following completion of the initial exploration the critically ill patients is transferred to the ICU Invasive monitoring & complete ventilator suppo are often needed. This phase focusses on secondary resuscitation to correct hypothermia, coagulopathy & acidosis It consists of planned re-exploration & definitive repair of injuries. This phase typically occurs 48 & 72 hours following initial and after successful secondary resuscitation Abdomen should be closed primarily if possible Risky GI anastomoses or complex reconstruction should be avoided STAGES OF DCS Stage I Patient selection Stage II Operative control of haemorrhage & Contamination Stage III ICU resuscitation Stage IV Definitive surgery Stage V Abdominal closure" "This is a real-world medical entrance exam question, please give the true answer based on the question and selection. Topic name: None, Subject name: Gynaecology & Obstetrics","A primigravida came to the labor room at 40 weeks + 5 days gestation for induction of labor. On per vaginal examination, the cervix is 1 cm dilated and 30% effaced. The veex is at --1 station and the cervix is soft and posterior. What will be the modified bishop score for this lady? Select and explain. A. 0, B. 3, C. 5, D. 8",C: 5,"Answer: C: 5 Explanation: Ans: C. 5(Ref: Williams 24Ie p525-526; Dutta 8Ie p600, 7/e p722)Cervical Station: -1 = 2; Cervical Dilatation: 1 cm = 1; Effacement: 30% = 0 ; Cervix Position: Posterior = 0; Consistency: Soft = 2.Hence, Bishop Score = 5.Bishop Scoring System Used for Assessment of InducibilityCervical FactorDilatation (cm)Effacement (%)Station (-3 to +2)ConsistencyPositionClosed0-30-3FirmPosterior1-240-50-2MediumMidposition3-460-70-1SoftAnterior>_5?80+1, +2--" "This is a real-world medical entrance exam question, please give the true answer based on the question and selection. Topic name: None, Subject name: Anatomy","Sensory nerve supply of capsule of TMJ is? Select and explain. A. Auriculotemporal nerve, B. Facial nerve, C. Massetric nerve, D. Auricular nerve",A: Auriculotemporal nerve,"Answer: A: Auriculotemporal nerve Explanation: None" "This is a real-world medical entrance exam question, please give the true answer based on the question and selection. Topic name: None, Subject name: Dental","Fishman's index is used in relation with Select and explain. A. Population, B. Hand wrist radiographs, C. Cephalograms, D. Periodontal diseases",B: Hand wrist radiographs,"Answer: B: Hand wrist radiographs Explanation: None" "This is a real-world medical entrance exam question, please give the true answer based on the question and selection. Topic name: None, Subject name: Dental","Arch space for eruption of 2nd & 3rd molar created by: Select and explain. A. Apposition of Hamular processes, B. Resorption of anterior border of ramus, C. Resorption of posterior border of ramus, D. Apposition of lower border of mandible",B: Resorption of anterior border of ramus,"Answer: B: Resorption of anterior border of ramus Explanation: None" "This is a real-world medical entrance exam question, please give the true answer based on the question and selection. Topic name: None, Subject name: Gynaecology & Obstetrics","All of the following are used in the treatment of postpartum hemorrhage except: Select and explain. A. Misoprostol, B. Mifepristone, C. Carboprost, D. Methyl ergometrine",B: Mifepristone,"Answer: B: Mifepristone Explanation: None" "This is a real-world medical entrance exam question, please give the true answer based on the question and selection. Topic name: None, Subject name: Dental","IOTN is not used for which malocclusion? Select and explain. A. Open bite, B. CLP, C. Bimaxillary protrusion, D. Crowding",C: Bimaxillary protrusion,"Answer: C: Bimaxillary protrusion Explanation: Index of Orthognathic Functional Treatment Need  This index applies to those malocclusions that are not amenable to orthodontic treatment alone, due to skeletal deformity, and will ordinarily apply to those patients who will have completed facial growth prior to surgery (commonly 18 years of age and older) It relates only to the functional need for treatment and should be used in combination with appropriate psychological and other clinical indicators." "This is a real-world medical entrance exam question, please give the true answer based on the question and selection. Topic name: None, Subject name: Medicine","A girl comes with symptoms of involuntary movements. Sydenham's chorea and acute rheumatic fever is suspected. Other major criteria of rheumatic fever (ahritis, skin rashes, subcutaneous nodules and carditis) were absent. No evidence of sore throat. Best investigation to prove rheumatic etiology is: Select and explain. A. Antistreptolysin S, B. Antistreptolysin O, C. Throat culture, D. PCR for M protein",B: Antistreptolysin O,"Answer: B: Antistreptolysin O Explanation: Ans: B. Antistreptolysin OAntistreptolysin O(ASLO):Marker for recent streptococcal infection.Best investigation to prove rheumatic etiology. Suppoing evidence:Preceding streptococcal infection within last 45 days.Elevated or rising anti-streptolysin O/other streptococcal antibodies.A positive throat culture.Rapid antigens test for group A streptococcus.Recent scarlet fever.""Revised Jones"" criteria do not include recent scarlet fever as suppoing evidence of recent streptococcal." "This is a real-world medical entrance exam question, please give the true answer based on the question and selection. Topic name: None, Subject name: Gynaecology & Obstetrics","In which of the following heart diseases maternal mortality is found to be highest ? Select and explain. A. Eisenmenger's complex, B. Coarctation of aorta, C. Mitral stenosis, D. Aortic stenosis",A: Eisenmenger's complex,"Answer: A: Eisenmenger's complex Explanation: Eisenmenger’s syndrome is the presence of secondary pulmonary hypertension that develops from any cardiac lesion. The syndrome develops when increased pulmonary blood flow due to left to right shunt produces a right side pressure more than left side and hence reversal of shunt occurs and subsequently cyanosis develops. It is the heart disease with the worst prognosis during pregnancy with a maternal mortality of 50%.Q Hence, pregnancy is contraindicated is patients of eisenmengers. If diagnosis of Eisenmenger is made in the first trimester, termination of pregnancy is advised. Most common cause of death in Eisenmenger’s syndrome is right ventricular failure with cardiogenic shock." "This is a real-world medical entrance exam question, please give the true answer based on the question and selection. Topic name: None, Subject name: Forensic Medicine","A young patient presented to casualty with a history of some substance abuse. His pulse was 130 beats per minute and respiratory rate was 30 per minute. Blood gas analysis revealed metabolic acidosis and his urea was 100 mg/dL and creatinine was 4 mg/dL. Urinalysis revealed calcium oxalate crystals. He improved symptomatically after management with intravenous fluids, gastric lavage, sodium bicarbonate, calcium gluconate and 4-methylpyrazole administration. What is the most likely substance that he consumed? Select and explain. A. Formaldehyde, B. Methyl alcohol, C. Ethylene glycol, D. Paraldehyde",C: Ethylene glycol,"Answer: C: Ethylene glycol Explanation: Answer- C. Ethylene glycolInitial symptoms are vomiting lethargy, ataxia, inebriation, convulsions, and coma. In 12 to 24 hours tachycardia, tachypnoea and circulatory collapse, electrolyte imbalance and metabolic acidosis occur. In one to three days hypocalcemia, oliguria, tubular necrosis, and renal failure occur. Urine contains crystals of calcium oxalate.Ethylene glycol: Clear, colorless, odorless, non-volatile liquid with a bitter-sweet taste.Mainly used as an antifreeze agent; Not absorbed through the skin.Metabolized to glycolaldehyde, glycolic acid & oxalic acid and inhibits oxidative phosphorylationFatal dose: 100-200 ml; Fatal period:3 days" "This is a real-world medical entrance exam question, please give the true answer based on the question and selection. Topic name: None, Subject name: Dental","3 year old girl having delayed milestones, playing with herself, difficulty in learning is likely to be suffering from Select and explain. A. Mental retardation, B. Autism, C. Specific Learning Disorder, D. ADHD",B: Autism,"Answer: B: Autism Explanation: None" "This is a real-world medical entrance exam question, please give the true answer based on the question and selection. Topic name: AIIMS 2019, Subject name: Anatomy","Post parotidectomy, patient feels numb while shaving. Which nerve was involved ? Select and explain. A. Facial, B. Mandibular, C. Auriculotemporal, D. Greater auricular",D: Greater auricular,"Answer: D: Greater auricular Explanation: Greater auricular nerve - supplies sensory innervation for skin over parotid & mastoid process(both surfaces of outer ear) | when damaged after parotidectomy, | causes hyperesthesia / hypoesthesia" "This is a real-world medical entrance exam question, please give the true answer based on the question and selection. Topic name: None, Subject name: Dental","Articular disc of TMJ is: Select and explain. A. Thickest posteriorly, B. Narrow in the centre, C. Strengthened by lateral pterygoid, D. All of the above",D: All of the above,"Answer: D: All of the above Explanation: None" "This is a real-world medical entrance exam question, please give the true answer based on the question and selection. Topic name: None, Subject name: Medicine","All of these are true about microalbuminuria Select and explain. A. Urine protein levels range from 20 mg/d to 200 mg/d, B. It is an independent risk factor for cardiovascular morbidity in diabetic patients, C. It is the earliest marker of diabetic nephropathy, D. It is not detected by routine dipstick method",A: Urine protein levels range from 20 mg/d to 200 mg/d,"Answer: A: Urine protein levels range from 20 mg/d to 200 mg/d Explanation: Ans: A. Urine protein levels range from 20 mg/d to 200 mg/d Refs Harrison 19/e p2425, 1582, 1813 18/e p2982, 29(SMicroalbuminuria:Defined as 30-299 mg/d in a 24-h collection or 30-299 mcg/mg creatinine in a spot collection.The American Diabetes Association (ADA) recently suggested that the terms previously used to refer to increased urinary protein (microalbuminuria as defined as 30-299 mg/d in a 24-h collection or 30-299ug/mg creatinine in a spot collection or macroalbuminuria as defined as >300 mg/24 h) be replaced by the phrases ""persistent albuminuria (30-299 mg/24 lz)"" and ""persistent albuminuria (>300 mg/24 h)"" to better reflect the continuous nature of albumin excretion in the urine as risk factor for nephropathy and cardiovascular disease (CVD)." "This is a real-world medical entrance exam question, please give the true answer based on the question and selection. Topic name: None, Subject name: Microbiology","Phase-contrast microscopy is based on the principle of: Select and explain. A. Different refractive indices of object, B. Different reflective indices of object, C. Light scattering, D. Light attenuation",A: Different refractive indices of object,"Answer: A: Different refractive indices of object Explanation: Phase-contrast microscopy takes advantage of minute refractive index differences within cellular components and between unstained cells and their surrounding aqueous medium to produce contrast in these and similar transparent specimens." "This is a real-world medical entrance exam question, please give the true answer based on the question and selection. Topic name: None, Subject name: Dental","Before an arbitrary face bow transfer record, the dentist must determine: Select and explain. A. Physiologic rest position, B. Inclination of each condyle, C. Axial centre of rotation of condyle, D. Kinematic axis of movement of condyle",C: Axial centre of rotation of condyle,"Answer: C: Axial centre of rotation of condyle Explanation: None" "This is a real-world medical entrance exam question, please give the true answer based on the question and selection. Topic name: None, Subject name: Pediatrics","In a 4 year old child with ASOM the infecting of organism is likely to be – Select and explain. A. Pneumococcus, B. H.influenza, C. Streptococcus, D. Staphylococcus",A: Pneumococcus,"Answer: A: Pneumococcus Explanation: Three most common organisms causing ASOM -4 S. pneumoniae (40%), H. influenzae (25-30%), Moraxella, catarrhalis (10-15%) Note - H.influenzae causing ASOM is nontypable. The incidence of H.influenzae type 'b' has decreased because of widespread use of Hib vaccine." "This is a real-world medical entrance exam question, please give the true answer based on the question and selection. Topic name: None, Subject name: Gynaecology & Obstetrics","Increased calories required during pregnancy:September 2012 Select and explain. A. 300, B. 400, C. 550, D. 800",A: 300,"Answer: A: 300 Explanation: Ans. A i.e. 300The increased calorie requirement is to the extent of 300 over the non pregnancy state during second half of pregnancy." "This is a real-world medical entrance exam question, please give the true answer based on the question and selection. Topic name: AIIMS 2018, Subject name: Biochemistry","Klenow fragment is formed by loss of fragment having which activity: Select and explain. A. 5'- 3' polymerase, B. 3'- 5' exonuclease, C. 5'- 3' exonuclease, D. 3'- 5' polymerase",C: 5'- 3' exonuclease,"Answer: C: 5'- 3' exonuclease Explanation: Klenow fragment Large fragment produced by Subtilisin mediated proteolytic cleavage of E.Coli DNA polymerase I. Proteolysis removes the 5' -->3' exonuclease activity from N-terminal. Klenow fragment - Functions : Remove 3' overhang Fills 5' overhangs Synthesis of double-stranded DNA from single-stranded templates Preparation of radioactive DNA probes Was used in PCR" "This is a real-world medical entrance exam question, please give the true answer based on the question and selection. Topic name: None, Subject name: Surgery","A 45 years old female presented with a history of pain!less breast lump of size 6 x 5 cm in left upper quadrant with no axillary lymph nodes. A true-cut biopsy was suggestive of ductal carcinoma in situ. She undergoes surgery with resection of all tumor tissue with adequate margins and postoperative HPE showing DCIS with high grade necrosis with 4 mm clearance on margins. Which of the following is needed? Select and explain. A. Adjuvant chemotherapy, B. Adjuvant chemoradiotherapy, C. Adjuvant radiotherapy, D. No additional treatment",C: Adjuvant radiotherapy,"Answer: C: Adjuvant radiotherapy Explanation: Answer- C. Adjuvant radiotherapyIn this case, patient is 45 years old (Score 2) with size 5 cm (Score 3), margin of 4 mm (Score 2), necrotic features withouthigh grade (Score 2), the total score is 9. Usually for score 7-9, if local excision is done radiotherapy should be given,If simple mastectomy is done, there is no need of adjuvant trcatment In this patient next best step would be adjuvant radiotherapy.DCIS is predominantly seen in the female breast (accounts for 5% of male breast cancers)DCIS carries a high risk for progression to an invasive cancer.DCIS is classified on the basis of nuclear grade & presence of necrosis" "This is a real-world medical entrance exam question, please give the true answer based on the question and selection. Topic name: None, Subject name: Microbiology","A newly introduced vaccines include all, except: Select and explain. A. Rota virus, B. Influenza, C. Dengue, D. Malaria vaccine",B: Influenza,"Answer: B: Influenza Explanation: Ans. b. InfluenzaVaccination Update 2016-2018Rota virusIt has been approved by the government of India for inclusioninto the UIP with the phase 1 launch of the vaccine in 4 states(Himachal Pradesh, Odisha, Andhra Pradesh and Haryana) inFebruary 2016.It is administered at age 6-15 weeks.Maximum age for administering the last dose of either vaccine should be 32 weeks.Dengue Vaccine It is known as Dengvaxia (CYD-TDV).It is live recombinant as a 3-dose series on a 0/6/12 monthsschedule in Phase-II clinical studies.It has been registered for use in individuals 9-45 years of ageliving endemic areas.April 2016, WHO Strategic Advisory Group of Expes (SAGE) on immunization recommended introduction of the vaccine only in geographic settings (national or subnational) with endemicity. Malaria Vaccine It is known as S, S/AS01It is the first malaria vaccine to have completed phase 3 testing. S, S is a vaccine against plasmodium falciparum.It offers no protection against P. vivax malariaThe vaccine is being considered as a complementary malaria control tool in Africa." "This is a real-world medical entrance exam question, please give the true answer based on the question and selection. Topic name: None, Subject name: Pathology","A man who had suffered from displaced fracture of mandible complains of pain in-old fracture site near mental foramen. There is a movable tender mass on palpation in the area. The best preoperative diagnosis is: Select and explain. A. Neuroleiomyoma, B. Neurofibroma, C. Trigeminal neuralgia, D. Traumatic neuroma",D: Traumatic neuroma,"Answer: D: Traumatic neuroma Explanation: None" "This is a real-world medical entrance exam question, please give the true answer based on the question and selection. Topic name: None, Subject name: Social & Preventive Medicine","Standard deviation of means measures - Select and explain. A. Non-sampling errors, B. Sampling error, C. Random errors, D. Conceptual errors",B: Sampling error,"Answer: B: Sampling error Explanation: None" "This is a real-world medical entrance exam question, please give the true answer based on the question and selection. Topic name: None, Subject name: Medicine","Addison's disease is due to: Select and explain. A. Chronic insufficiency of adrenal cortex, B. Chronic insufficiency of adrenal medulla, C. Insufficiency of pancreas, D. Hypofunction of thyroid gland",A: Chronic insufficiency of adrenal cortex,"Answer: A: Chronic insufficiency of adrenal cortex Explanation: None" "This is a real-world medical entrance exam question, please give the true answer based on the question and selection. Topic name: None, Subject name: Social & Preventive Medicine","Which is the best distribution to study the daily admission of head injury patients in a trauma care centre Select and explain. A. Normal distribution, B. Binomial distribution, C. Uniform distribution, D. Poisson distribution",D: Poisson distribution,"Answer: D: Poisson distribution Explanation: POISSON DISTRIBUTION: Is a ‘discrete probability distribution’ that expresses the ‘probability of a number of events occurring in a fixed period of time’ (if these events occur with a known average rate and independently of the time since the last event) It can also be used for the number of events in other specified intervals such as distance, area or volume Is generally used to model the number of events occurring within a given time interval Is a discrete distribution which takes on the values X = 0, 1, 2, 3,…. In the given question, one has to study the daily admission of head injury patients in a trauma care centre, Since, it describes the no. of events in time (no. of head injury patients admitted per day, Therefore, it is a Poisson distribution." "This is a real-world medical entrance exam question, please give the true answer based on the question and selection. Topic name: None, Subject name: Dental","In Inlay and 3 fourth crowns main purpose of bevel is Select and explain. A. To remove unsupported enamel rods, B. Increase retention, C. Provide secondary resistance, D. None of above",A: To remove unsupported enamel rods,"Answer: A: To remove unsupported enamel rods Explanation: None" "This is a real-world medical entrance exam question, please give the true answer based on the question and selection. Topic name: None, Subject name: Gynaecology & Obstetrics","35 years old female with choriocarcinoma treatment of choice is: Select and explain. A. Dilatation and evacuation, B. Radiotherapy, C. Hysterectomy, D. Chemotherapy",D: Chemotherapy,"Answer: D: Chemotherapy Explanation: None" "This is a real-world medical entrance exam question, please give the true answer based on the question and selection. Topic name: None, Subject name: Surgery","In post splenectomy patient, chances of Infection with which of these Increases: Select and explain. A. Encapsulated bacteria, B. Non capsulated bacteria, C. Anaerobic and gram positive bacilli, D. Anaerobic and grain negative bacilli.",A: Encapsulated bacteria,"Answer: A: Encapsulated bacteria Explanation: None" "This is a real-world medical entrance exam question, please give the true answer based on the question and selection. Topic name: AIIMS 2017, Subject name: Pharmacology","Which of the following instructions should be given to a lactating mother regarding drug usage? Select and explain. A. No advice is required as most of the drugs are secreted negligibly in the milk, B. Take drugs with longer half-life, C. Tell her to feed the baby just before next dose, D. Do not feed the baby if you are consuming any drug",C: Tell her to feed the baby just before next dose,"Answer: C: Tell her to feed the baby just before next dose Explanation: Just before the next dose, the plasma concentration of drug is minimum; so it is relatively safer to feed at that time. Some drugs are secreted significantly in milk. There is no relation between half-life of a drug and its milk secretion ; So, long half life drugs may also come in milk. Not all the drugs are harmful or significantly secreted in milk so feed should not be stopped. Drugs C/I during lactation: Lithium Aspirin Levetiracetam Atenolol Sulfonamide Metronidazole Methotrexate Metals(Cu)." "This is a real-world medical entrance exam question, please give the true answer based on the question and selection. Topic name: None, Subject name: Pharmacology","Which of the following gives the rate of drug absorption in plasma concentration graph? Select and explain. A. Tmax and Cmax, B. Area under the curve, C. Tmax alone, D. Cmax alone",A: Tmax and Cmax,"Answer: A: Tmax and Cmax Explanation: Ans: A. Tmax and CmaxRef: Sharma & Sharma's Principles of Pharmacolog, j'd ed., pg. 30-31 and hup:// whrw. e a r op e a n r ev i ew. o r g/wp /wp - c o n t e nt/ up I o a d s/6. p df .From the plasma concentration graph, we obtain three impoant parametersCmax is the peak plasma concentrationTmax i.e. time to attain the peak plasma concentration or CmaxAUC (Area under the curve) of plasma concentration time graphThe 1st two parameters i.e. Cmax and the Tmax are the indicators of the rate of absorption." "This is a real-world medical entrance exam question, please give the true answer based on the question and selection. Topic name: None, Subject name: Pediatrics","2 years female has been successfully treated for urinary tract infection. Next step in management is : Select and explain. A. Do nothing, B. Ultrasound, C. Ultrasound + DMSA scan, D. 6 monthly culture sensitivity",C: Ultrasound + DMSA scan,"Answer: C: Ultrasound + DMSA scan Explanation: The primary concerns following the diagnosis of UTI are : The presence of an underlying urological anomaly (obstruction or VUR). Involvement of kidney (Pyelonephritis). Recommended radiological evaluation includes a combination of - USG. Micturating cystourethrogram (MCU). Radionuclide scan (DMSA).   Indications of detailed radiological investigations are - Child below 3 yr old with first UTI. Symptoms of pyelonephritis. Recurrent UTI. Abnormal voiding (dribbling, straining, abnormal urinary stream). Persistently distended bladder. Family history of UTI or hypertension Coming back to question As this child is below 3 year (2 yrs) with first UTI, detailed radiological investigations are recommonded." "This is a real-world medical entrance exam question, please give the true answer based on the question and selection. Topic name: None, Subject name: Pathology","A 4-year-old child has a normal complement of primary teeth, but they are grey and exhibit extensive occlusal and incisal wear. Radiographic examination indicates extensive deposits of secondary dentin in these teeth. Most likely this condition is: Select and explain. A. Neonatal hypoplasia, B. Amelogenesis imperfecta, C. Cleidocranial dysplasia, D. Dentinogenesis imperfecta",D: Dentinogenesis imperfecta,"Answer: D: Dentinogenesis imperfecta Explanation: None" "This is a real-world medical entrance exam question, please give the true answer based on the question and selection. Topic name: None, Subject name: Dental","Thermal conductivity of which material is close to dentin: Select and explain. A. Amalgam, B. Silicate, C. Composite resin, D. Cavity varnish",B: Silicate,"Answer: B: Silicate Explanation: None" "This is a real-world medical entrance exam question, please give the true answer based on the question and selection. Topic name: None, Subject name: Anatomy","Sella tursica lies above: Select and explain. A. Pons, B. Frontal sinus, C. Foramen ovale, D. Sphenoidal sinus",D: Sphenoidal sinus,"Answer: D: Sphenoidal sinus Explanation: None" "This is a real-world medical entrance exam question, please give the true answer based on the question and selection. Topic name: None, Subject name: Pathology","In genomic imprinting. DNA is modified by: Select and explain. A. Acetylation, B. Methylation, C. Phosphorylation, D. Deamination",B: Methylation,"Answer: B: Methylation Explanation: Ans: B. Methylation(Ref Robbins 9/e p180, 8/e pl)Genomic imprinting:DNA modified by methylation.An epigenetic process resulting in differential inactivation of either maternal or paternal alleles of ceain genes.Mechanism:DNA methylation at CG nucleotide.Histone H4 deacetylation.Methylation." "This is a real-world medical entrance exam question, please give the true answer based on the question and selection. Topic name: None, Subject name: Dental","Numbness and tingling sensation felt in the anterior one third of the tongue can be due to: Select and explain. A. Over extended lingual borders, B. Sharp and rough lingual borders, C. Allergy to residual monomer of acrylic, D. Excessive vertical dimension",A: Over extended lingual borders,"Answer: A: Over extended lingual borders Explanation: None" "This is a real-world medical entrance exam question, please give the true answer based on the question and selection. Topic name: None, Subject name: Dental","Innervation of pulp is from Select and explain. A. Trigeminal nerve, B. Facial nerve, C. Glossopharyngeal nerve, D. Hypoglossal nerve",A: Trigeminal nerve,"Answer: A: Trigeminal nerve Explanation: None" "This is a real-world medical entrance exam question, please give the true answer based on the question and selection. Topic name: None, Subject name: Medicine","A 62 years old woman presented with acute onset confusion and bumping into th ings. On examination, she was ale, oriented with fluent speech and normal comprehension. Fuher examination revealed impaired writing (acalculia), right-left confusion, impaired arithmetic abilities, difficulty in finger identification. MRI demonstrated foci of coical and sub coical increase T2 signals and areas of leptomeningeal enhancement. Most likely diagnosis: Select and explain. A. Gerstmann's syndrome, B. Millard-Gubler syndrome, C. Anton syndrome, D. Korsakoff's psychosis",A: Gerstmann's syndrome,"Answer: A: Gerstmann's syndrome Explanation: Answer- A (Gerstmann's syndrome)The combination of acalculia (impairment of simple arithmetic), dysgraphia (impaired writing)' finger anomia (an inability to name individual fingers such as the index and thumb), and right-teft confusion (an inability to tell whether a hand, foot, or arm of the patient or examiner is on the right or left side ofthe body) is known as Gerstmann's syndrome. When Gerstmann's syndrome is seen in isolation, it is commonly associated with damage to the inferior parietal lobule (especially the angular gyrus) in the left hemisphere." "This is a real-world medical entrance exam question, please give the true answer based on the question and selection. Topic name: None, Subject name: Pharmacology","Which of the following statements about tedizolid is true? Select and explain. A. It causes peripheral neuropathy as an adverse effect, B. It is active against gram positive organisms, C. It has poor oral bioavailability, D. It is active against anaerobes",B: It is active against gram positive organisms,"Answer: B: It is active against gram positive organisms Explanation: Ans. B. It is active against gram positive organisms* Tedizolid Phosphate is an oxazolidinone-class antibiotic prodrug indicated in adults for the treatment of acute bacterial skin and skin structure infections (ABSSSI) caused by susceptible isolates of several Gram-positive bacteria.* Following administration oral or intravenous route, tedizolid phosphate prodrug is conveed by plasma phosphatases to its active moiety, tedizolid.* The active metabolite of tedizolid binds to the 50S bacterial ribosomal subunit, inhibiting protein synthesis and causing its bacteriostatic effects.* Tedizolid is an effective and potent alternative to linezolid for the treatment of patients with Gram-positive ABSSSI due to MRSA or MSSA.* Increased potency allows for once daily dosing with reduced total dosages, improving the side effect profile of this drug.* Of note, the minimum inhibitory concentrations of tedizolid appear to be largely unaffected by the chloramphenicol-florfenicol resistance (cfr) gene, which has been implicated in a number of published linezolid-resistant organism outbreaks.* OPTIC NEUROPATHY,PERIPHERAL NEUROPATHY ARE LESS SEEN WITH TEDIZOLID THAN OTHERS IN SAME CLASS" "This is a real-world medical entrance exam question, please give the true answer based on the question and selection. Topic name: None, Subject name: Dental","Which component of zinc oxide-eugenol cement gives its strength? Select and explain. A. Rosin, B. Zinc oxide, C. Zinc acetate, D. Oil of cloves",B: Zinc oxide,"Answer: B: Zinc oxide Explanation: None" "This is a real-world medical entrance exam question, please give the true answer based on the question and selection. Topic name: None, Subject name: Pharmacology","A healthy volunteer was taken for a blood experiment. A history was taken from the volunteer before the experiment regarding exposure of NSAlDs, which he specifically denied. But on testing, the BT was found to be increased All of the following can be causative agent except: Select and explain. A. Theophylline, B. Cephalosporin, C. Anti-depressants, D. Multivitamins containing Vitamin K",D: Multivitamins containing Vitamin K,"Answer: D: Multivitamins containing Vitamin K Explanation: Ans: D. Multivitamins containing Vitamin K(Ref. Goodman Gilman 12/e p1043, 1499; Katzung 13/e p1145)Routine use of multivitamins with Vitamin K doesn't cause thrombocytopenia.Vitamin K:Used as Warfarin antidote (anticoagulant) excess.Drug-induced immune thrombocytopenia (DITP):In acute thrombocytopenia patients.Most Common Causes of DITPAntiplateletAbciximab, Eptifibatide, TirofibanAnalgesicsAcetaminophen, Ibuprofen, NaproxenAntibioticAmpicillin, Piperacillin, Ethambutol, CephalosporinsDrugs acting on CNSCarbamazepine, Haloperidol, Phenytoin, AntidepressantsAnti-CancerIrinotecan, Oxaliplatin (selective thrombocytopenia apa from pancytopenia)" "This is a real-world medical entrance exam question, please give the true answer based on the question and selection. Topic name: None, Subject name: Surgery","A patient came with complaints of lower limb weakness. Examiner places one hand under the patient's heel and patient is asked to raise his other leg against downward resistance. What is the name of this test? Select and explain. A. Hoover test, B. Waddell's test, C. O'Donoghue test, D. McBride test",A: Hoover test,"Answer: A: Hoover test Explanation: Ans. a. Hoover testIn Hoover test, the subject relaxes in a supine position on the table while the examiner places both of the subject's heels into the palm of the examiners hands.Test positioning: The subject relaxes in a supine position on the table while the examiner places both of the subject's heels into the palm of the examiners hands.Action: The subject is asked to perform a unilateral straight leg raisePositive finding: Inability to lift the leg may reflect a neuromuscular weakness. A positive finding is also noted when the examiner does not feel increased pressure in the palm that underlies the resting leg" "This is a real-world medical entrance exam question, please give the true answer based on the question and selection. Topic name: AIIMS 2017, Subject name: Pharmacology","Which of the following is an example of placebo? Select and explain. A. Herbal medication with no known effect, B. Physiotherapy, C. Sham surgery, D. Cognitive behavioral therapy",C: Sham surgery,"Answer: C: Sham surgery Explanation: * In sham surgery, surgery is done without any purpose; we just open the abdomen in one person and close it, in other person appendix is removed; this is done to see whether appendectomy has any advantage.* Placebos (fake drug/dummy medicine) are used in clinical trials to compare the two treatments; it can't produce any effect. * Herbal medication can produce some effects.* Physiotherapy can also produce effects." "This is a real-world medical entrance exam question, please give the true answer based on the question and selection. Topic name: None, Subject name: Medicine","Dietary goals for avoiding Coronary Artery Disease include Select and explain. A. LDL cholesterol less than 100 mg/dl, B. Saturated fat <7% of total calories., C. Alcohol consumption of 30 ml/day, D. All of the above",D: All of the above,"Answer: D: All of the above Explanation: None" "This is a real-world medical entrance exam question, please give the true answer based on the question and selection. Topic name: None, Subject name: Dental","Tooth eruption is due to Select and explain. A. Osteoclastic activity, B. Proliferation of cells at crypt, C. Exfoliation of primary tooth, D. Eruptive forces",D: Eruptive forces,"Answer: D: Eruptive forces Explanation: None" "This is a real-world medical entrance exam question, please give the true answer based on the question and selection. Topic name: None, Subject name: Pharmacology","Steroids do not have a role in management of which of these tumors? Select and explain. A. Kaposi sarcoma, B. Chronic lymphoid leukemia, C. Hodgkin's lymphoma, D. Multiple myeloma Gram-stain, oropharynx",A: Kaposi sarcoma,"Answer: A: Kaposi sarcoma Explanation: Ans: A. Kaposi sarcoma(Ref Harrison 19/e p1270, 716; Goodman Gilman I2/e p1755; Katzung 13/e p954-955, 12/e p706: KDT 7/e p284-285, 6/e p285)Steroids - Therapeutic uses in cancer:Used as cytotoxic agents in treatment of acute leukemia in children & malignant lymphoma in children & adults.Component of curative regimens for Hodgkin's & non-Hodgkin's lymphoma, multiple myeloma & CLL.Glucocoicoids - Extremely helpful in controlling autoimmune hemolytic anemia & thrombocytopenia associated with CLL.Dexamethasone - Used in conjunction with radiotherapy - Reduces edema related to tumors in critical areas such as superior mediastinum, brain & spinal cords." "This is a real-world medical entrance exam question, please give the true answer based on the question and selection. Topic name: None, Subject name: Surgery","During 3rd molar surgical extraction, distal incision is curved buccally to prevent injury to: Select and explain. A. Buccal artery, B. Lingual nerve, C. Temporalis fibers, D. BFP",B: Lingual nerve,"Answer: B: Lingual nerve Explanation: None" "This is a real-world medical entrance exam question, please give the true answer based on the question and selection. Topic name: None, Subject name: Pediatrics","Infant of diabetic mother with weight 3.8 Kg presented with seizures after 16 hours of birth.What is the cause – Select and explain. A. Hypoglycemia, B. Hypocalcemia, C. Birth asphyxia, D. Intraventricular hemorrhage",A: Hypoglycemia,"Answer: A: Hypoglycemia Explanation: Electrolyte disturbances, including hypoglycemia and hypocalcemia, are commonly seen in association with infants of diabetic mother and may result in early-onset seizures. Sorry friends, I could not find any reference which has directly mentioned that hypoglycemia is a more common cause of seizure than hypocalcemia in infants of diabetic mother. I was just able to find following two statements : - ""Infants of diabetic mother are most at risk for hypoglycemia which can result in seizure"". ""Infant of diabetic mothers with seizures, that does not respond to glucose should have their serum calcium measured"" From these two statements, it seems to me that hypoglycemia is a more common cause of seizures in infants of diabetic mother." "This is a real-world medical entrance exam question, please give the true answer based on the question and selection. Topic name: None, Subject name: Dental","A 12 year old child complains of recurrent pain over right maxillary posterior tooth but clinically no caries can be found. He also reports of cold and fever spells. What should be advised to the patient? Select and explain. A. Extract the primary retained tooth i.e., 54, 55, B. Extract the tooth 16, 54, 55, C. Refer to physician for the evaluation of maxillary sinus, D. Do nothing",C: Refer to physician for the evaluation of maxillary sinus,"Answer: C: Refer to physician for the evaluation of maxillary sinus Explanation: None" "This is a real-world medical entrance exam question, please give the true answer based on the question and selection. Topic name: None, Subject name: Dental","All of the following device work on traction principle except: Select and explain. A. Ferrier double bow separator., B. Elliot separator., C. Perry separator., D. None",B: Elliot separator.,"Answer: B: Elliot separator. Explanation: Elliot separator work on wedge principle." "This is a real-world medical entrance exam question, please give the true answer based on the question and selection. Topic name: None, Subject name: Dental","Extension of odontoblast processes into enamel Select and explain. A. Enamel spindle, B. Cracks, C. Primary dentin, D. Secondary Dentine",A: Enamel spindle,"Answer: A: Enamel spindle Explanation: Enamel spindle- Diameter (2p) Extended odontoblast processes in enamel Perpendicular (right angle) to dentin DDT (Dark - Dehydrated - Transmitted light)" "This is a real-world medical entrance exam question, please give the true answer based on the question and selection. Topic name: None, Subject name: Dental","Short sharp pain of tooth transmitted to brain by Select and explain. A. A delta fibers, B. C fibers, C. Beta fibers, D. Gamma fibers",A: A delta fibers,"Answer: A: A delta fibers Explanation: None" "This is a real-world medical entrance exam question, please give the true answer based on the question and selection. Topic name: None, Subject name: Surgery","Which of these is the most reliable method for monitoring fluid resuscitation? Select and explain. A. Urine output, B. CVP, C. Pulse rate, D. Blood pressure",A: Urine output,"Answer: A: Urine output Explanation: Ans: A. Urine outputGoal of treatment:Restore cellular and organ perfusion.Hence, monitoring of organ perfusion should guide the management of shock.The best measures of organ perfusion and the best monitor of the adequacy of shock therapy remain the urine output." "This is a real-world medical entrance exam question, please give the true answer based on the question and selection. Topic name: AIIMS 2017, Subject name: Microbiology","Lipoarabinomannan (LAM) assay in urine is used for screening of? Select and explain. A. Mycobacterium tuberculosis, B. Pneumocystis jirovecii, C. Histoplasm capsulatum, D. Cryptococcus neoformans",A: Mycobacterium tuberculosis,"Answer: A: Mycobacterium tuberculosis Explanation: Lipoarabinomannan (LAM) is a virulence factor for Mycobacterium tuberculosis. It prevents phagolysosomal fusion. It is secreted in urine so can be used for screening purposes." "This is a real-world medical entrance exam question, please give the true answer based on the question and selection. Topic name: AIIMS 2020, Subject name: Pathology","Which of the following is not a cross-linking fixative? Select and explain. A. Osmium tetroxide, B. Glutaraldehyde, C. Methanol, D. Formaldehyde",C: Methanol,"Answer: C: Methanol Explanation: Chemical fixation This utilizes organic or non-organic solutions to maintain adequate morphological preservation. Chemical fixatives can be considered as members of three major categories: coagulant, cross-linking, and compound. Coagulant fixatives: Both organic and non-organic solutions may coagulate proteins making them insoluble. Cellular architecture in vivo is maintained primarily by lipoproteins and fibrous proteins such as collagen. Coagulating these proteins maintains tissue histomorphology at the light microscope level. Unfounately, because coagulant fixatives result in cytoplasmic flocculation and poor preservation of mitochondria and secretory granules, these fixatives are not useful in ultrastructural analysis. Examples: Ethanol, methanol, acetone, picric acid and trichloroacetic acid Cross-linking fixatives: Several chemicals were selected as fixatives secondary to their potential actions of forming cross-links both within and between proteins and nucleic acids. Examples include formaldehyde, glutaraldehyde and other aldehydes, e.g. chloral hydrate and glyoxal, as well as metal salts, e.g. mercuric and zinc chloride, and other metallic compounds, e.g. osmium tetroxide. useful for electron microscopic examination - Glutraldehyde (best one)/formaldehyde/osmium tetraoxide Compound fixatives: Alcoholic formalin" "This is a real-world medical entrance exam question, please give the true answer based on the question and selection. Topic name: None, Subject name: Pharmacology","Cholinomimetic is not used in which of the following? Select and explain. A. Bradycardia, B. Glaucoma, C. Myasthenia gravis, D. Post-surgical atony or ileus",A: Bradycardia,"Answer: A: Bradycardia Explanation: Ans. a. BradycardiaCholinomimetic is used for open-angle glaucoma, cobra bite, and myasthenia gravis but not in bradycardia. Uses of CholinomimeticDiseases of eye Glaucoma and accommodative esotropiaAtropine overdosesGastrointestinal and urinary tract Alzheimer's diseasePostoperative atony, neurogenic bladderCobra bite: Neostigmine and edrophoniumHea: Ceain atrial arrhythmiasNeuromuscular junction: Myasthenia gravis, Curare induced neuromuscular paralysis" "This is a real-world medical entrance exam question, please give the true answer based on the question and selection. Topic name: None, Subject name: Pathology","Gene which inhibits cell cycle is - Select and explain. A. p53, B. RB, C. p16, D. Notch receptor",A: p53,"Answer: A: p53 Explanation: Answer-A. p53Cell cycle inhibitorsCIP/KIP family - p21, p27, p57INK4a/ARF family - p161 NK4a, pl4ARFA G1 arrest can result simply by the p53 induced expression of p21 WAF1/CIP1/Sdi1 P53 also down regulates the expression of cyclin A, providing a secondary break on cell cycle progression into the through the S phase." "This is a real-world medical entrance exam question, please give the true answer based on the question and selection. Topic name: None, Subject name: Physiology","Cell membrane of a cell is freely permeable to three ions X, Y and Z. At resting membrane potential, the respective equilibrium potentials of Xand Y are -30mV and -40 mV.What will happen to the membrane potential if a drug is infused which can block the permeability of Z? Select and explain. A. Depolarisation, B. Hyperpolarization, C. No change, D. Hyperpolarization following repolarization",A: Depolarisation,"Answer: A: Depolarisation Explanation: Ans. A. DepolarisationRef: Guyton & Hall 13' ed., ch-5, pg. 61-63.In this question if Z has a reversal potential or original RMP was more negative (e.g., -70 or -90 mV) then closure of C ion channels would shift the RMP closer to reversal potential of A, B thus depolarising the cell" "This is a real-world medical entrance exam question, please give the true answer based on the question and selection. Topic name: None, Subject name: Dental","A patient complains of looseness of dentures within two to four hours of insertion. The most likely cause is: Select and explain. A. Psychological, B. Adaptive unresponsiveness, C. Deflective occlusion, D. Unretentive denture",C: Deflective occlusion,"Answer: C: Deflective occlusion Explanation: None" "This is a real-world medical entrance exam question, please give the true answer based on the question and selection. Topic name: None, Subject name: Dental","Split impedence based apex locator is: Select and explain. A. Generation 1, B. Generation 2, C. Generation 3, D. Generation 4",D: Generation 4,"Answer: D: Generation 4 Explanation: None" "This is a real-world medical entrance exam question, please give the true answer based on the question and selection. Topic name: None, Subject name: Pediatrics","A mother comes with her 3 months' child asking the physician if she can give cereals to her child. What problems can this lead to her child? Select and explain. A. Allergy due to the food content, B. Risk of gastrointestinal infection, C. Retarded oro-motor development, D. Contaminated food leading to reflux",A: Allergy due to the food content,"Answer: A: Allergy due to the food content Explanation: Ans: A: Allergy due to the food contentExplanation:(Ref: Nelson 20/e p1139; Ghni 8/e p90)Babies should receive only breast milk or infant formula for the first 6 months of life.Most impoant reason for this is allergy due to the food content.Exclusive breastfeeding for the first 4-6 months of life may reduce allergic disorders in the first few years of life. Potentially allergenic foods (eggs, milk, wheat, soy, peanut and fish) should be introduced after this period of exclusive breastfeeding to decrease chances of food allergy." "This is a real-world medical entrance exam question, please give the true answer based on the question and selection. Topic name: None, Subject name: Dental","Which of the following tooth is not having 5 cusps? Select and explain. A. Mandibular 2nd Molar, B. Mandibular 1st Molar, C. Mandibular 3rd Molar, D. Maxillary 1st Molar",A: Mandibular 2nd Molar,"Answer: A: Mandibular 2nd Molar Explanation: None" "This is a real-world medical entrance exam question, please give the true answer based on the question and selection. Topic name: None, Subject name: Dental","A case presented with lower natural teeth and 7 maxillary implants placed, having space of 15 mm for the restoration. What would be the ideal treatment plan for the patient? Select and explain. A. Single crown and bridge screw retained, B. Single crown and bridge cement retained, C. Overdenture, D. Hybrid denture",D: Hybrid denture,"Answer: D: Hybrid denture Explanation: The ideal restorative space for an implant supported hybrid denture is 12-15 mm (measured from the crest of alveolar ridge to occlusal plane) as accounted below: Abutment: 3.0 mm Hybrid Bar: 3.0 mm Space: 1.0 mm Acrylic: 4.0 mm Teeth: 4.0 mm." "This is a real-world medical entrance exam question, please give the true answer based on the question and selection. Topic name: None, Subject name: Gynaecology & Obstetrics","In non immune hydrops which of the following is NOT seen: Select and explain. A. Skin oedema, B. Ascites, C. Large placenta, D. Cardiomegaly",D: Cardiomegaly,"Answer: D: Cardiomegaly Explanation: “Hydrops is characterized by excess fluid in two or more body areas such as thorax, abdomen or skin. It is often associated with hydraminos and a hydropic thickened placenta”. It is characterised by: Increased skin thickness (> 5 mm) / skin oedema (first sign seen on USG). Placental enlargement Pleural effusion Ascites The fetus is in Buddha position with a halo around the head." "This is a real-world medical entrance exam question, please give the true answer based on the question and selection. Topic name: None, Subject name: Radiology","For a maxillary premolar in a radiograph, the facial root appears distal to the palatal root, the film was exposed with: Select and explain. A. Increased vertical angulation, B. Decreased vertical angulation, C. From mesial side of the tooth, D. From distal side of the tooth",C: From mesial side of the tooth,"Answer: C: From mesial side of the tooth Explanation: If the tube is shifted mesially and the object in question appears to move distally, it lies on the buccal aspect of the reference object. These relationships can be easily remembered by the acronym SLOB: same lingual, opposite buccal.  Thus, If the object in question appears to move in the same direction with respect to the reference structures as does the X-ray tube, it is on the lingual aspect of the reference object. If it appears to move in the opposite direction as the X-ray tube, it is on the buccal aspect. If it does not move with respect to the reference object, it lies at the same depth (in the same vertical plane) as the reference object. Ref: ORAL RADIOLOGY Principles and Interpretation, Stuart C. White, Michael J. Pharoah Edition 7 page no 88" "This is a real-world medical entrance exam question, please give the true answer based on the question and selection. Topic name: None, Subject name: Dental","Lalloo, a 13-year old child has a severe thumb-sucking habit. On examination he has a Class-II malocclusion, anterior open bite with an over-jet of 12mm. His cephalogram will show: Select and explain. A. Normal anterior and posterior facial heights, B. Increased anterior facial height and normal posterior facial height, C. Increased posterior facial height and normal anterior facial height, D. Increased posterior facial height and increased anterior facial height",B: Increased anterior facial height and normal posterior facial height,"Answer: B: Increased anterior facial height and normal posterior facial height Explanation: None" "This is a real-world medical entrance exam question, please give the true answer based on the question and selection. Topic name: None, Subject name: Dental","During implant placement maximum torque should not be above? Select and explain. A. 100 N, B. 50 N, C. 60 N, D. 70 N",D: 70 N,"Answer: D: 70 N Explanation: Once the final osteotomy diameter and depth is prepared, a bone tap is used in a low-torque hand (i.e., 30rpm, 70 N-cm) to form the threads within the bone for the implant" "This is a real-world medical entrance exam question, please give the true answer based on the question and selection. Topic name: None, Subject name: Anatomy","Lateral part of middle cranial fossa and posterior cranial fossa are divided by: Select and explain. A. Petrous temporal bone, B. Crista galli, C. Transverse groove, D. Sphenoid bone",A: Petrous temporal bone,"Answer: A: Petrous temporal bone Explanation: The interior of base of skull is naturally divided into 3 fossae Anterior cranial fossa Middle cranial fossa Posterior cranial fossa Lesser wing of sphenoid with anterior clinoid process divides anterior and middle cranial fossa. Petrous temporal bone separates the lateral part of middle cranial fossa from posterior cranial fossa. Dorsum sellae of sphenoid bone with posterior clinoid process separates the median part of middle cranial fossa from posterior cranial fossa." "This is a real-world medical entrance exam question, please give the true answer based on the question and selection. Topic name: None, Subject name: Gynaecology & Obstetrics","A 27 year primigravida presents with pregnancy induced hypertension with blood pressure of 150/100 mm of Hg at 32 weeks of gestation with no other complications. Subsequently, her blood pressure is controlled on treatment. If there are no complications, the pregnancy should be terminated at: Select and explain. A. 40 completed weeks, B. 37 completed weeks, C. 35 completed weeks, D. 34 completed weeks",B: 37 completed weeks,"Answer: B: 37 completed weeks Explanation: The patient in the question has BP = 150/100 mm of Hg i.e. mild hypertension (severe hypertension is when systolic BP is > 160 mm or diastolic BP > 110 mm of Hg) and has no other complications. Her BP is controlled on treatment i.e. she is being managed expectantly. In such patients pregnancy should be terminated at 37 weeks. “If Pregnancy is beyond 37 completed weeks termination is to be considered without delay.” ... Dutta Obs. 7/e, p229 Management of Mild Preeclampsia: Fernando Arias 3/e, p 419 Detailed examination for symptoms indicative of severe preeclampsia should be done daily." "This is a real-world medical entrance exam question, please give the true answer based on the question and selection. Topic name: None, Subject name: Surgery","A 1.5 years old child was brought to emergency with history of burn by hot water on both hands and palms. The lesion was pink, oozing and painful to air and touch. Which of the following is the best management for this patient? Select and explain. A. Paraffin gauze and dressing, B. Collagen dressing, C. Excision and grafting, D. Apply 1% silver sulfasalazine ointment and keep the wound open",A: Paraffin gauze and dressing,"Answer: A: Paraffin gauze and dressing Explanation: Answer- A. Paraffin gauze and dressingParaffin gauze and dressing - Collagenase is the most commonly employed enzymatic debriding agent currently commercially available in the United States.It is slow acting, requiring several days to one week to adequately debride or heal intermediate-depth paial thickness wounds.Superficial BurnsClean the burn wound & remove the roof of all blistersqExpose superficial burns of face but apply sterile liquid paraffin to reduce crusting.For burns of perineum, clean & expose but apply silver sulfadiazine cream. Nurse the patient without dressings on a sterile sheet on air or water-bed.Cover superficial burns of other areas with two layers of paraffin gauze & a bulky absorptive dressing" "This is a real-world medical entrance exam question, please give the true answer based on the question and selection. Topic name: None, Subject name: Dental","Growth of the maxilla in the vertical direction is due to: Select and explain. A. Growth of the alveolus, B. Growth at sutures, C. Growth of the cranial base, D. Growth of the synchondrosis",A: Growth of the alveolus,"Answer: A: Growth of the alveolus Explanation: None" "This is a real-world medical entrance exam question, please give the true answer based on the question and selection. Topic name: None, Subject name: Dental","Although standarize technique is simple and rapid but not used frequently because: Select and explain. A. Inaccurate technique., B. Relies on inherent shape of file to give shape of canal., C. Accessory canals are not removed efficiently., D. All of the above.",B: Relies on inherent shape of file to give shape of canal.,"Answer: B: Relies on inherent shape of file to give shape of canal. Explanation: None" "This is a real-world medical entrance exam question, please give the true answer based on the question and selection. Topic name: None, Subject name: Microbiology","clostridium difficile diarrhoea associated with: Select and explain. A. Aminopenicillins, B. Carbapenems, C. Macrolide, D. Fluoroquinolones",D: Fluoroquinolones,"Answer: D: Fluoroquinolones Explanation: Answer-D-fluoroquinolones,Clostridium difficile colitis results from a disturbance of the normal bacterial flora of the colon, colonization by C difficile, and the release of toxins that cause mucosal inflammation and damage.It is a spore-forming bacillus that is responsible for the development of antibiotic-associated diarrhea and colitis.The antibiotics that most often lead to C. difficile infections include fluoroquinolones, cephalosporins,penicillins and clindamycin. Once established, C. difficile can produce toxins that attack the lining of the intestine" "This is a real-world medical entrance exam question, please give the true answer based on the question and selection. Topic name: None, Subject name: Microbiology","Which of the following is the most common systemic symptom during migration of larval phase of Helminths like Ancylostoma, Strongyloides and Ascaris? Select and explain. A. Asymptomatic, B. Pneumonitis, C. Liver failure, D. Larva migrans",A: Asymptomatic,"Answer: A: Asymptomatic Explanation: Ans: A. Asymptomatic(Ref Paniker's 7/e p162, 6/e pg180, 210)Larvae of Ascaris, Hookworm and Strongyloides migrate through lung & various other tissues during their lifecycle.Larva migrans:Sometimes larvae appear to lose way & wander around aimlessly.Generally seen when human infection occurs with nonhuman species of nematodes.Ascaris:Pathogenic effects of larval migration due to allergic reaction & not larvae presence.Hence larvae's initial exposure usually asymptomatic.Except on very heavy larval load.Symptomatic conditions:Visceral larvae migrans is produced by infection with non-human nematodes.Le. Nematodes frequently infecting dogs & cats.Cause local alveolar hemorrhages.Symptoms during migration due to allergic reaction & significant only in massive infections.Clinical pneumonitis seen only in massive infections.Hence mostly asymptomatic." "This is a real-world medical entrance exam question, please give the true answer based on the question and selection. Topic name: None, Subject name: Surgery","The police has brought an unresponsive patient to you. What is the first thing you will do? Select and explain. A. Sta chest compressions immediately, B. Check carotid pulse, C. Check for response and call help, D. Sta rescue breaths",B: Check carotid pulse,"Answer: B: Check carotid pulse Explanation: Ans: B. Check carotid pulse(Ref BLS/ACLS Guidelines: hups://www.resus.org.uk/resuscitation guidelines /adult-advanced-life-suppo; Harrison 19/e p1768; Braunwald's 10/e p844-845)According to Adult Basic Life Suppo (BLS) algorithm in an unresponsive patient, next step is to check carotid pulse." "This is a real-world medical entrance exam question, please give the true answer based on the question and selection. Topic name: None, Subject name: Anatomy","Arrange the following layers of epidermis from surface to deep layer - CorneocytesMerkel cellsMelanocytesLangerhans cells Select and explain. A. Corneocytes>merkel cell>melanocytes>langerhans cells, B. Merkel cells>corneocytes>melanocytes>langerhans cells, C. Melanocytes >merkel cells>corneocytes>langerhans cells, D. corneocytes>>langerhans cells>melanocytes > merkel cells",D: corneocytes>>langerhans cells>melanocytes > merkel cells,"Answer: D: corneocytes>>langerhans cells>melanocytes > merkel cells Explanation: Ans. DThe epidermis is composed of 4 or 5 layers, depending on the region of skin being considered.Those layers in descending order are:Cornified layer (stratum corneum)Composed of 10 to 30 layers of polyhedral, anucleated corneocytes (final step of keratinocyte differentiation), with the palms and soles having the most layers.Clear/translucent layer (stratum lucidum, only in palms and soles)This narrow layer is found only on the palms and soles. The epidermis of these two areas is known as ""thick skin"" because with this extra layer, the skin has 5 epidermal layers instead of 4Granular layer (stratum granulosum)Keratinocytes lose their nuclei and their cytoplasm appears granular. Lipids, contained into those keratinocytes within lamellar bodies, are released into the extracellular space through exocytosis to form a lipid barrier. Those polar lipids are then conveed into non-polar lipids and arranged parallel to the cell surface.Spinous layer (stratum spinosum)Keratinocytes become connected through desmosomes and sta produce lamellar bodies, from within the Golgi, enriched in polar lipids, glycosphingolipids, free sterols, phospholipids and catabolic enzymes.Langerhans cells, immunologically active cells, are located in the middle of this layer.Basal/germinal layer (stratum basale/germinativum).Composed mainly of proliferating and non-proliferating keratinocytes, attached to the basement membrane by hemidesmosomes.Melanocytes are present, connected to numerous keratinocytes in this and other strata through dendrites.Merkel cells are also found in the stratum basale with large numbers in touch-sensitive sites such as the fingeips and lips." "This is a real-world medical entrance exam question, please give the true answer based on the question and selection. Topic name: None, Subject name: Dental","A material turns to liquid on applying critical shear stress. This type of flow is called as? Select and explain. A. Newtonian body, B. Bingham body, C. Dilatant, D. Pseudoplastic",B: Bingham body,"Answer: B: Bingham body Explanation: None" "This is a real-world medical entrance exam question, please give the true answer based on the question and selection. Topic name: None, Subject name: Dental","The setting time of zinc phosphate may be retarded by? Select and explain. A. Increase in the ratio of powder to liquid, B. Diluting the liquid with water, C. Increase the addition of powder to liquid, D. Decrease the addition of powder to liquid",D: Decrease the addition of powder to liquid,"Answer: D: Decrease the addition of powder to liquid Explanation: None" "This is a real-world medical entrance exam question, please give the true answer based on the question and selection. Topic name: None, Subject name: Medicine","Features of cardiac rest pain are all, except Select and explain. A. Compressive., B. Radiating, C. Excruciating, D. None of the above",D: None of the above,"Answer: D: None of the above Explanation: None" "This is a real-world medical entrance exam question, please give the true answer based on the question and selection. Topic name: None, Subject name: Radiology","All of the following statements about an intraoral dental X-ray film are true except: Select and explain. A. It is composed of a cellulose acetate base coated with a silver bromide emulsion, B. It has a thin sheet of lead foil that lies in front of the film to prevent overexposure, C. It has an embossed dot on it which should be orientated towards the source of X-rays, D. It is exposed by the direct action of X-rays on the emulsion",B: It has a thin sheet of lead foil that lies in front of the film to prevent overexposure,"Answer: B: It has a thin sheet of lead foil that lies in front of the film to prevent overexposure Explanation: None" "This is a real-world medical entrance exam question, please give the true answer based on the question and selection. Topic name: None, Subject name: Dental","Only tooth in which buccal surface is made of 2 lobes Select and explain. A. Upper 1st molar, B. Lower 1st molar, C. Upper 2nd molar, D. Lower 2nd molar",A: Upper 1st molar,"Answer: A: Upper 1st molar Explanation: None" "This is a real-world medical entrance exam question, please give the true answer based on the question and selection. Topic name: None, Subject name: Biochemistry","Niacin & riboflavin help in: Select and explain. A. Redox reactions, B. Transamination reaction, C. Methyl group transfer, D. Amine group transfer",A: Redox reactions,"Answer: A: Redox reactions Explanation: None" "This is a real-world medical entrance exam question, please give the true answer based on the question and selection. Topic name: None, Subject name: Anatomy","The middle meningeal artery is associated with which foramen: Select and explain. A. Ovale, B. Rotundum, C. Stylomastoid, D. Spinosum",D: Spinosum,"Answer: D: Spinosum Explanation: None" "This is a real-world medical entrance exam question, please give the true answer based on the question and selection. Topic name: None, Subject name: Anatomy","Asymmetrical union of sutures resuting in twisted skull known as: Select and explain. A. Plagiocephaly., B. Anencephaly., C. Scaphocephaly., D. Solicocephaly.",A: Plagiocephaly.,"Answer: A: Plagiocephaly. Explanation: None" "This is a real-world medical entrance exam question, please give the true answer based on the question and selection. Topic name: None, Subject name: Dental","Surgery of the cleft lip should be carried out at: Select and explain. A. 20 -30 weeks, B. 2 - 12 months, C. 3 - 6 months, D. 5 years",C: 3 - 6 months,"Answer: C: 3 - 6 months Explanation: None" "This is a real-world medical entrance exam question, please give the true answer based on the question and selection. Topic name: None, Subject name: Dental","Excessive forces of occlusion within physislogical limit will: Select and explain. A. Increase width of PDL, B. Decrease width of PDL, C. Width will remain same, D. Trauma from occlusion",A: Increase width of PDL,"Answer: A: Increase width of PDL Explanation: None" "This is a real-world medical entrance exam question, please give the true answer based on the question and selection. Topic name: None, Subject name: Pathology","Which of the following is the most consistent feature of rapidly progressing glomerulonephritis (RPGN)? Select and explain. A. Crescent formation, B. Mesangial cell proliferation, C. IgAdeposition, D. Loss offoot processes",A: Crescent formation,"Answer: A: Crescent formation Explanation: Ans: A. Crescent formationRapidly Progressive (Crescentic) Glomerulonephritis:Most common histologic - Presence of crescents in most glomeruli (crescentic glomerulonephritis).Crescents:Produced by proliferation of parietal cells & infiltration of monocytes & macrophages." "This is a real-world medical entrance exam question, please give the true answer based on the question and selection. Topic name: None, Subject name: Pathology","Which of the following is not a risk factor for malignancy Select and explain. A. DNA aneuploidy, B. Male gender, C. Invasive candidiasis, D. Leukoplakia in non smokers",B: Male gender,"Answer: B: Male gender Explanation: None" "This is a real-world medical entrance exam question, please give the true answer based on the question and selection. Topic name: None, Subject name: Surgery","A person met with road traffic accident and came to casualty with contusion on anterior chest wall with Pulse rate-90/minute, BP-120/80 mm Hg, respiratory rate-16/minute. Normal hea sounds are heard but breath sounds were decreased on the left side and trachea was deted towards right. Which of the following is the first line management? Select and explain. A. Needle thoracostomy, B. Pericardiocentesis, C. Chest tube inseion and drainage, D. Immediate exploratory thoracotomy",C: Chest tube inseion and drainage,"Answer: C: Chest tube inseion and drainage Explanation: Answer- C. Chest tube inseion and drainageThis case is a classical description of post RM traumutic pneumothorax of left hemithorax, with decreased breath sound and trachea shifted to the right. Presence of hea sounds indicstes absence of cardiac tamponade. Tension pneumothoro-r is ruled out, as the patient is not having hypotension (BP-120/80 mm Hg). So the first line management is chest tube inseion and drainage." "This is a real-world medical entrance exam question, please give the true answer based on the question and selection. Topic name: None, Subject name: Ophthalmology","Universal marker of limbal epithelial stem cells: Select and explain. A. Elastin, B. Keratin, C. Collagen, D. ABCG2",D: ABCG2,"Answer: D: ABCG2 Explanation: Ans: D. ABCG2(Ref Indian J Med Res. 2008 Aug:128(2):149-56. PM1D:19001678; http://wwwstembook.org/node/588).ABCG2:Universal marker of limbal epithelial stem cells.Limbal Stem Cell Marker.Keratin (K3-K13) = Corneal Stem Cell Marker" "This is a real-world medical entrance exam question, please give the true answer based on the question and selection. Topic name: None, Subject name: Social & Preventive Medicine","A 10-year boy with dog bite unprovoked comes to you. Appropriate action is: Select and explain. A. Give cell culture derived vaccine, B. Withhold vaccine and observe dog for 10 days, C. Kill dog and send brain for biopsy, D. No fuher action is necessary",A: Give cell culture derived vaccine,"Answer: A: Give cell culture derived vaccine Explanation: Ans: A. Give cell culture derived vaccine(Ref Park's 24/e p297, 23/e p279, 22/e p253)Most appropriate action - Give post-exposure prophylaxis with cell-culture derived vaccine.Post-Exposure Prophylaxis:Aim: To neutralize inoculated virus before entry to nervous system. Prevention:Effective & safe in preventing rabies:Concentrated & purified cell-culture vaccine (CCV).Embryonated egg-based vaccine (EEV).Uses:Intended for pre-exposure & post-exposure prophylaxis.Discontinuation point:Discontinued if suspected animal is proved by appropriate examination is rabies-free.Domestic dogs, cats or ferrets remain healthy throughout 10-days observation period staing from date of bite. Categories of contact with suspected Rabid animalPost-exposure prophylaxis measuresI. Touching or feeding animalsLicks on intact skinNoneII. Nibbling of uncovered skinMinor scratches or abrasions without bleedingImmediate vaccination & local treatment of wound.III. Single or multiple transdermal bites or scratchesLicks on broken skinContamination of mucous membrane with saliva from licksContact with batsImmediate vaccination and administration of rabies immunoglobulin and local treatment of the wound." "This is a real-world medical entrance exam question, please give the true answer based on the question and selection. Topic name: None, Subject name: Dental","Office personnel who are at higher risk of NO toxicity, to prevent this, the most sensitive test for nitrous oxide detection is Select and explain. A. Laminar flow, B. Dosimeter, C. Infrared spectrophotometer, D. High suction speed",C: Infrared spectrophotometer,"Answer: C: Infrared spectrophotometer Explanation: Periodic (i.e. semi-annual) sampling is done of dental personnel, especially chair-side personnel exposed to nitrous oxide (e.g. with a diffusive sampler, such as a dosimeter or infrared spectrophotometer) Office personnel who are at higher risk of NO toxicity, to prevent this most sensitive test for nitrous oxide detection is Infrared spectrophotometer. Reference: Pediatric Dentistry by Nikhil Marwah 3rd ed, page no 250" "This is a real-world medical entrance exam question, please give the true answer based on the question and selection. Topic name: None, Subject name: Dental","At 12 year age, 2nd mandibular molar will have? Select and explain. A. Crown complete with 1/3rd root, open apex, B. Crown complete with 3/4th root,, open apex, C. 3/4th crown completed, D. Crown complete with 1/4th root, open apex","B: Crown complete with 3/4th root,, open apex","Answer: B: Crown complete with 3/4th root,, open apex Explanation: None" "This is a real-world medical entrance exam question, please give the true answer based on the question and selection. Topic name: None, Subject name: Dental","Which of the following movements are performed by a non — working condyle? Select and explain. A. Straightward, B. Down wards forwards and lateral, C. Down wards forwards and medial, D. Down wards Back wards and medial",C: Down wards forwards and medial,"Answer: C: Down wards forwards and medial Explanation: None" "This is a real-world medical entrance exam question, please give the true answer based on the question and selection. Topic name: None, Subject name: Pathology","A rhabdomyoma is a tumour originating from Select and explain. A. Nerve tissue, B. Smooth muscle, C. Striated muscle, D. Vascular endothelium",C: Striated muscle,"Answer: C: Striated muscle Explanation: None" "This is a real-world medical entrance exam question, please give the true answer based on the question and selection. Topic name: None, Subject name: Dental","Free way space of less than normal, in complete denture has the following adverse result: Select and explain. A. Mucosal irritation, B. Irresistible alveolar ridge resorption, C. Nonspecific ulcerations, D. None of the above",B: Irresistible alveolar ridge resorption,"Answer: B: Irresistible alveolar ridge resorption Explanation: None" "This is a real-world medical entrance exam question, please give the true answer based on the question and selection. Topic name: None, Subject name: Dental","Which of the following best describes the working time of impression material? Select and explain. A. After the start of appearing elastic properties of impression material, B. Just after the start of appearing elastic properties of impression material, C. The time from start of mixing till just before the start of appearing elastic properties of impression material, D. Loss of lusture of impression material",C: The time from start of mixing till just before the start of appearing elastic properties of impression material,"Answer: C: The time from start of mixing till just before the start of appearing elastic properties of impression material Explanation: None" "This is a real-world medical entrance exam question, please give the true answer based on the question and selection. Topic name: None, Subject name: Microbiology","Purpose of sterilization is to eliminate Select and explain. A. Bacteria, B. Viruses, C. Spore formers, D. Fungus",C: Spore formers,"Answer: C: Spore formers Explanation: None" "This is a real-world medical entrance exam question, please give the true answer based on the question and selection. Topic name: None, Subject name: Dental","Lower density usually means Select and explain. A. High strength, B. Low strength, C. Low melting temperature, D. Lighter weight",D: Lighter weight,"Answer: D: Lighter weight Explanation: None" "This is a real-world medical entrance exam question, please give the true answer based on the question and selection. Topic name: None, Subject name: Pathology","The most common complication following rheumatoid arthritis of the TMJ is: Select and explain. A. Ankylosis, B. Synovial chondromatosis, C. Subluxation, D. Osteoarthritis",A: Ankylosis,"Answer: A: Ankylosis Explanation: None" "This is a real-world medical entrance exam question, please give the true answer based on the question and selection. Topic name: None, Subject name: Dental","In RVG bit depth is seen of? Select and explain. A. 8 bit, B. 6 bit, C. 12 bit, D. 4 bit",A: 8 bit,"Answer: A: 8 bit Explanation: None" "This is a real-world medical entrance exam question, please give the true answer based on the question and selection. Topic name: None, Subject name: Medicine","In pregnancy cardiac output increases 30 to 40% during? Select and explain. A. 20th week of pregnancy, B. 22 weeks of pregnancy, C. 24 weeks of pregnancy, D. 28 weeks of gestation",D: 28 weeks of gestation,"Answer: D: 28 weeks of gestation Explanation: None" "This is a real-world medical entrance exam question, please give the true answer based on the question and selection. Topic name: None, Subject name: Dental","Enamel hatchet is differentiated from chisel by all except: Select and explain. A. Curved in one plane only, B. Blade is larger, C. Blade is perpendicular to the long axis of handle, D. Blade is heavier",C: Blade is perpendicular to the long axis of handle,"Answer: C: Blade is perpendicular to the long axis of handle Explanation: None" "This is a real-world medical entrance exam question, please give the true answer based on the question and selection. Topic name: None, Subject name: Gynaecology & Obstetrics","In the pelvic inlet, which is the shoest anteroposterior diameter? Select and explain. A. True conjugate, B. Obstetric conjugate, C. Anatomical conjugate, D. Bispinous diameter",B: Obstetric conjugate,"Answer: B: Obstetric conjugate Explanation: Ans: B. Obstetric conjugate In pelvic inlet shoest anteroposterior diameter - Ostetric conjugate (10 cm).Pelvic inlet:Four diameters:Anteroposterior, transverse & two oblique diameters.Anteroposterior diameter:Distinct with specific landmarks.Most cephalad - Hence, true conjugate.Extends from upper- most margin ofthe symphysis pubis to sacral promontory.Clinically impoant obstetrical conjugate is shoest distance between the sacral promontory and the symphysis pubis. Normally 10 cm or more." "This is a real-world medical entrance exam question, please give the true answer based on the question and selection. Topic name: None, Subject name: Medicine","A patient was found to have splenomegaly, anemia and jaundice. All of the following is true about this condition except: Select and explain. A. Increased urobilinogen, B. Increased LDH, C. Decreased haptoglobin, D. Low reticulocyte count",D: Low reticulocyte count,"Answer: D: Low reticulocyte count Explanation: Ans: D. Low reticulocyte count(Ref: Robbins 9/e p 631, 632)A reactive increase in reticulocyte count occurs in hemolytic anemias due to increased erythropoiesis.Hemolytic anemia:Anemia & lowered tissue oxygen tension a Triggers erythropoietin production a Stimulates erythroid differentiation a Appearance of increased numbers erythroid precursors (normoblasts) in marrow. Compensatory increases in erythropoiesis a Resutls in reticulocytosis in peripheral blood." "This is a real-world medical entrance exam question, please give the true answer based on the question and selection. Topic name: None, Subject name: Anatomy","Dorsal part of 2nd pouch give rise to: Select and explain. A. Tonsils., B. Thymus., C. Tubotympanic recess., D. Tympanic antrum.",C: Tubotympanic recess.,"Answer: C: Tubotympanic recess. Explanation: None" "This is a real-world medical entrance exam question, please give the true answer based on the question and selection. Topic name: None, Subject name: Physiology","leksell efferents go to Select and explain. A. Interneurons, B. Spindle, C. Golgi tendon organ, D. Golgi bottle cells",B: Spindle,"Answer: B: Spindle Explanation: None" "This is a real-world medical entrance exam question, please give the true answer based on the question and selection. Topic name: None, Subject name: Pediatrics","A newborn with eyes closed 6 hrs after birth lustily crying, no chest retraction and movements of all four limbs. Neonatal behavioral response grading – Select and explain. A. State 1, B. State 3, C. State 5, D. State 6",D: State 6,"Answer: D: State 6 Explanation: Neonatal behavioral assessment scale Neonatal Behavioural Assessment Scale (NBAS) is used to support parents in the neonatal unit, especially around discharge home. The aim is to increase parents' confidence in understanding their baby. An infant's reaction are state related. So it is extremely important that observations of state be considered as a starting point from which all other observations are made. The ideal time to administer the examination is when the infant is asleep and midway between feeding cycles. Scores Sleep states State 1 : Sleep with regular breathing, eyes closed, no spontaneous activity except startles or jerky movements at quite regular intervals. a State 2 : Sleep with eyes closed; rapid eye movements can often be observed under closed lids; low activity level with random movements and startles. Awake states State 3 : Drowsy or semidrowsy; eyes may be open but dull and heavy lidded, or closed, eyelids fluttering; activity level minimal, Movements are usually smooth, although there may be startles. Some infants may also show fuss/cry vocalizations in this state. What distinguishes state 3 from state 5 when both are accompanied by fuss/cry vocalizations is the minimal movement in state 3 and considerable movement in state 5. State 4 : Alert, eyes open with bright look. Motor activity is minimal; there can be a glazed look that is easily changed into a brighter look with appropriate stimulation. State 5 : Eyes likely to be open; considerable motor activity, with thrusting movements of the extremities, and even a few spontaneous startles; reactive to external stimulation with increase in startles or motor activity. State 6 : Crying : characterized by intense, loud, rhythmic, and sustained cry vocalizations that are difficult to break through with stimulation; motor activity is high. Coming to the question According to above staging, the newborn in our question comes under state 6 (crying lustily, and moving all 4 limbs). If you have any confusion, read the following facts on crying (state 6) :- ""it is important to distinguish between crying as a state (stage 6) from the fuss/ay vocalization that can occur in state 5 and even state 3. Some infants show repeated episodes of fuss /cry vocalization in state 5 but may not reach state 6. This may also be a maturational issue, because some preterm infants may not have energy reserves to sustain state 6. In general, state 6 Can be distinguished from state 5 by the intensity and sustained quality of the crying (at least 15 seconds) and unavailability of infant in state 6. Repeated brief episodes of cry /fuss in state 5 do not mean that the infant has moved into state 6. Examiners need to give the infant the opportunity to show state 6"" So, crying lustily with all limbs moving (child in question), straight away goes in state 6." "This is a real-world medical entrance exam question, please give the true answer based on the question and selection. Topic name: None, Subject name: Pathology","Langerhan’s histiocytosis was previously known as Select and explain. A. Eosinophilic granuloma, B. Pyogenic granuloma, C. Letterer siwe disease, D. Histiocytosis x",D: Histiocytosis x,"Answer: D: Histiocytosis x Explanation: Langerhan’s histiocytosis was previously known as Histiocytosis X , it includes Eosinophilic granuloma Letterer siwe disease Hand Schuller Christian disease" "This is a real-world medical entrance exam question, please give the true answer based on the question and selection. Topic name: None, Subject name: Dental","In which of the following case Hamman's sign is seen? Select and explain. A. Hematoma, B. Accidental NaOCl, C. Pneumomediastinitis, D. Oral carcinoma",C: Pneumomediastinitis,"Answer: C: Pneumomediastinitis Explanation: None" "This is a real-world medical entrance exam question, please give the true answer based on the question and selection. Topic name: None, Subject name: Pathology","Magenta tongue and cracks at corner of mouth are seen in deficiency of: Select and explain. A. Vitamin B1, B. Niacin, C. Riboflavin, D. Pantothenic acid",C: Riboflavin,"Answer: C: Riboflavin Explanation: None" "This is a real-world medical entrance exam question, please give the true answer based on the question and selection. Topic name: None, Subject name: Social & Preventive Medicine","Height of group of 20 boys aged 10 years was 140 ± 13 cm and 20 girls of same age was 135 cm ± 7 cm. To test the statistical significance of difference in height, test applicable is - Select and explain. A. X2, B. Z, C. t, D. F",C: t,"Answer: C: t Explanation: Student’s t-test: – Paired Student’s t-test: Comparing means (± SD) in paired data (in same group of individuals before and after an intervention) – Unpaired Student’s t-test: Comparing means (± SD) in two different group of individuals – Z-test: Is a variant of student’s t-test which is used when sample size is > 30. In the given question, mean ± SD of 20 boys (140 ± 13 cm) and 20 girls (135 cm ± 7cm) of the same age are compared, Thus most appropriate statistical test of significance would be Unpaired Student’s t-test." "This is a real-world medical entrance exam question, please give the true answer based on the question and selection. Topic name: None, Subject name: Physiology","Adrenaline used for controlling of bleeding during surgery may result in: Select and explain. A. Syncope, B. Cardiac arrhythmias, C. Drastic fall in blood pressure, D. Precipitation of allergic reaction",B: Cardiac arrhythmias,"Answer: B: Cardiac arrhythmias Explanation: None" "This is a real-world medical entrance exam question, please give the true answer based on the question and selection. Topic name: None, Subject name: Dental","After removal of periodontal pack after a week of surgery, you see a large round elevated granulation tissue. It is most likely due to: Select and explain. A. Allergy to components in the pack, B. Normal finding, C. Incomplete calculus removal, D. Mechanical irritation due to periodontal pack",C: Incomplete calculus removal,"Answer: C: Incomplete calculus removal Explanation: None" "This is a real-world medical entrance exam question, please give the true answer based on the question and selection. Topic name: AIIMS 2019, Subject name: Physiology","Which centre first gets the input from neural control of CVS? Select and explain. A. RVLM, B. NTS, C. Nucleus ambiguous, D. Raphe Nucleus",B: NTS,"Answer: B: NTS Explanation: Nucleus tractus solitarius - Baroreceptors are spray type nerve endings that lie in the walls of the aeries & are stimulated when stretched. Most abundantly they are found in: 1. Wall of each Internal carotid aery (slightly above the carotid bifurcation) 2. Wall of aoic arch. Signal from the carotid baroreceptor are transmitted through hering's nerve to the glossopharyngeal nerve, in the high neck & then to the NTS in the medulla. Signal from the aoic baroreceptor in the arch of aoa are transmitted through vagus nerve to the same NTS to medulla. RVLM: - Rostral ventral lateral medulla. - Its vasoconstrictor area. - In hypotension, NTS receives the impulse when blood pressure falls & it needs to be increased. | RVLM will be stimulated. The axons of nerve cell bodies of RVLM, ends on each segment b/w T1 & L2. T1 - L2 contain intermediolateral horn, & there are sympathetic nerve cell bodies in them. Nucleus ambiguous: - Has cardio inhibitory area. Raphe nucleus: - Lies in midline of the PONS & medulla. Secrete serotonin. Not related to neural control of CVS" "This is a real-world medical entrance exam question, please give the true answer based on the question and selection. Topic name: None, Subject name: Pharmacology","Anti-hypeensive drug contraindicated in pregnancy is:September 2005, 2010 March 2007, March 2013 Select and explain. A. Hydralazine, B. Methyldopa, C. Enalapril, D. Amlodopine",C: Enalapril,"Answer: C: Enalapril Explanation: Ans. C: EnalaprilAntihypeensive used in pregnancyAlpha methyldopaLabetololFelodipineNot usually initiated in pregnancy due to volume depletion but may be continued:NifedipineHydralazineACE inhibitors, Thiazide diuretics, Angiotensin antagonists (losaan), Furosemide, propranolol and nitroprusside areunsafe/safety unceain in pregnancy.Foetal growth retardation, hypoplasia of organs and foetal death may occur if ACE inhibitors are given during later half of pregnancy." "This is a real-world medical entrance exam question, please give the true answer based on the question and selection. Topic name: None, Subject name: Surgery","What is the most common sequelae of traumatic shoulder dislocation in young adults? Select and explain. A. Rotator cuff tear, B. Recurrent shoulder dislocation, C. Adhesive capsulitis, D. Subscapular tendinitis",B: Recurrent shoulder dislocation,"Answer: B: Recurrent shoulder dislocation Explanation: Answer-B. Recurrent shoulder dislocationMost common sequelae of traumatic shoulder dislocation in young adults are recurrent shoulder dislocation. Anteriortlislocation Type I is the most common type of traumutic shoulder dislocation and its mosl common sequelae in youngadults is recurent dislocation in around 1/3rd patients. In fact, more than 9094 patients < 20 years have some shoulderinstability as sequelae." "This is a real-world medical entrance exam question, please give the true answer based on the question and selection. Topic name: AIIMS 2019, Subject name: Psychiatry","Which of the following best describes paradoxical suicide? Select and explain. A. Suicide after taking low dose of drug, B. Suicide occurring at the time when the pt. stas to recover, C. Suicidal tendency increase as the patient improves, D. Accidental completion of suicide",B: Suicide occurring at the time when the pt. stas to recover,"Answer: B: Suicide occurring at the time when the pt. stas to recover Explanation: Paradoxical suicide Occurs after the patient's improvement from a depression episode. Despite having suicidal thoughts there is no action taken on those thoughts but after taking medication the patient acts on it. Also seen in patients with schizophrenia." "This is a real-world medical entrance exam question, please give the true answer based on the question and selection. Topic name: None, Subject name: Dental","Decrease in creep occurs in: Select and explain. A. Under trituration or over trituration of amalgam, B. Decreases with condensation pressure, C. Increases with condensation pressure, D. Cannot be predictable",C: Increases with condensation pressure,"Answer: C: Increases with condensation pressure Explanation: None" "This is a real-world medical entrance exam question, please give the true answer based on the question and selection. Topic name: None, Subject name: Pathology","Burning tongue might not be associated with Select and explain. A. Ranula, B. Diabetes mellitus, C. Pernicious anaemia, D. Local irritation",A: Ranula,"Answer: A: Ranula Explanation: None" "This is a real-world medical entrance exam question, please give the true answer based on the question and selection. Topic name: AIIMS 2018, Subject name: Orthopaedics","10 year old present with ankle pain. X-ray has Lytic Lesion with sclerotic rim at calcaneum. Following HPE findings were seen. What is your diagnosis: Select and explain. A. Eumycosis, B. PVNS, C. Ochronosis, D. Hemophilic pseudotumar",D: Hemophilic pseudotumar,"Answer: D: Hemophilic pseudotumar Explanation: X-ray shows multiple lytic lesion. Histopathology shows cells with brown-color deposits, hemosiderin. * Ochronosis: Usually presents with blackening of urine.* Pigmented villo-nodular synovitis (PVNS): is an aggressive disorder arising from synol joints predominantly in the knee. It shows area of synol proliferation with histology showing hemosiderin stained giant cells.* Eumycosis: Commonly found in foot. Presents with sinuses. It shows Dot in circle sign. X-rays shows multiple bone cavities and progressive bone destruction.* Hemophilic pseudotumor: Usually found in ankle joint. Histopathology shows hemosiderin deposit due to haemorrhage." "This is a real-world medical entrance exam question, please give the true answer based on the question and selection. Topic name: None, Subject name: ENT","Olympian brown and rhagdes are seen in- Select and explain. A. CMV inclusion disease, B. Ectodermal dysplasia, C. Congenital syphilis, D. Hyper IgE syndrome",C: Congenital syphilis,"Answer: C: Congenital syphilis Explanation: Ans. c. Congenital syphilisClinical Presentation of SyphilisEarly Congenital Syphilis:Snuffles (rhinitis)Q is earliest feature.Lesions are vesicobullousQ, and snail track ulcers on mucosaClinical Presentation of SyphilisLate Congenital Syphilis:Characterized by Hutchinson's triad (interstitial keratitis + 8th nerve deafness + Hutchinson's teeth i.e. peggedcentral upper incisors)QSaddle nose, sabre tibia, mulberry molarsQBull dog's jaw (protrusion of jaw)Rhagadesdeg (linear fissure at mouth, nares)Frontal bossing, hot cross bun deformity of skullClutton's jointdeg (painless swelling of joints, most commonly both knee)Palatal perforationdegHigaumenakis sign (periostitis leads to unilateral enlargement of sterna end of clavicle)Primary Syphilis:Painless, indurated, nonbleeding, usually single punched out ulcer (hard chancre)QPainless, rubbery shotty lymphadenopathySecondary Syphilis:Bilateral symmetrical asymptomatic localized or diffuse mucocutaneous lesiondeg (macule, papule, paulosquamousand rarely pustule)Non-tender generalized lymphadenopathyQHighly infectious condylomata late, in warm moist interiginous areasMoth eaten alopecia, ahritis, proteinuriaQTeiary Syphilis:Gumma, neurosyphilis/tabes dorsalisQOstitis, periostitisAoitis, aoic insufficiency, coronary stenosis and nocturnal anginaQ" "This is a real-world medical entrance exam question, please give the true answer based on the question and selection. Topic name: None, Subject name: Anatomy","Soft palate is made up of: Select and explain. A. Palatoglossus and palatopharyngeus, B. Uvula, C. Mucous membrane and muscles, D. All of the above",D: All of the above,"Answer: D: All of the above Explanation: None" "This is a real-world medical entrance exam question, please give the true answer based on the question and selection. Topic name: None, Subject name: Biochemistry","Which of the following statements regarding proteoglycans is true? Select and explain. A. consist of a core protein with GAG chains attached., B. GAG contain oxidized acid sugars, C. negative charges cause to radiate out from the protein, D. All of these",D: All of these,"Answer: D: All of these Explanation: Ans-D. All of these* Proteoglycans contain many long unbranched polysaccharide chains attached to a core protein.* The polysaccharide chains, called glycosaminoglycans, are composed of repeating disaccharide units containing oxidized acid sugars (such as glucuronic acid), sulfated sugars, and N-acetylated amino sugars.* The large number of negative charges causes the glycosaminoglycan chains to radiate out from the protein so that the overall structure resembles a bottlebrush.The proteoglycans are essential pas of the extracellular matrix, the aqueous humor of the eye, secretions of mucus-producing cells, and cailage." "This is a real-world medical entrance exam question, please give the true answer based on the question and selection. Topic name: None, Subject name: Pathology","GNAS-1 gene mutation is associated with: Select and explain. A. Fibrous dysplasia, B. Osteogenesis imperfecta, C. Amelogenesis imperfecta, D. Cleidocranial dysplasia",A: Fibrous dysplasia,"Answer: A: Fibrous dysplasia Explanation: None" "This is a real-world medical entrance exam question, please give the true answer based on the question and selection. Topic name: None, Subject name: Microbiology","Hydatid cyst is: Select and explain. A. Parasitic in nature, B. Fungal, C. Congenital, D. Viral",A: Parasitic in nature,"Answer: A: Parasitic in nature Explanation: None" "This is a real-world medical entrance exam question, please give the true answer based on the question and selection. Topic name: None, Subject name: Pharmacology","Which of these anticonvulsants causes contraction of visual field? Select and explain. A. Levetiracetam, B. Phenytoin, C. Vigabatrin, D. Ethosuximide",C: Vigabatrin,"Answer: C: Vigabatrin Explanation: Ans: C. VigabatrinVigabatrin:Causes irreversible diffuse atrophy of the retinal nerve fiber layer.Most effect on the outer area (as opposed to the macular, or central area) of the retina, leading to the contraction of the visual field.Retinal toxicity attributed to taurine depletion." "This is a real-world medical entrance exam question, please give the true answer based on the question and selection. Topic name: None, Subject name: Dental","A 7-years child having very deep carious lesions in multiple teeth with no pain what should he the treatment plan: Select and explain. A. Indirect pulp capping, B. pulpotomy of all teeth, C. RCT of all teeth, D. Extraction of teeth",A: Indirect pulp capping,"Answer: A: Indirect pulp capping Explanation: Indirect pulp treatment is a procedure performed in a tooth with a deep carious lesion approximating the pulp but without signs or symptoms of pulp degeneration. The caries surrounding the pulp is left in place to avoid pulp exposure and is covered with a biocompatible material. Indirect pulp capping has been shown to have a higher success rate than pulpotomy in long term studies. It also allows for a normal exfoliation time." "This is a real-world medical entrance exam question, please give the true answer based on the question and selection. Topic name: None, Subject name: Forensic Medicine","Doctor or nursing disclosing the identity of rape, victim is punishable under the following section of IPC? Select and explain. A. Section 228A, B. Section 222A, C. Section 224A, D. Section 226A",A: Section 228A,"Answer: A: Section 228A Explanation: Answer- A. Section 228A 'Section 228A of the IPC a little known section introduced by an amendment to the law by the Parliament in 1983, deals with the disclosure of the identity of a victim of ceain offences and prevents it in rape cases. The section states that anyone who prints or publishes the name or only matter that may reveal the identity of a victim of rape shall be punished With fine and up to two years in jail. The law specifically lays down that under section 376 or 376A, 376 B, 376 C or 376 D, any victim of rape has to have her identity protected from public." "This is a real-world medical entrance exam question, please give the true answer based on the question and selection. Topic name: AIIMS 2017, Subject name: Anatomy","Wrong statement regarding grey rami communicates is Select and explain. A. Connected to spinal nerve, B. Present medial to white rami communicantes, C. Carries pre-ganglionic fibres, D. Fibres are non-myelinated",C: Carries pre-ganglionic fibres,"Answer: C: Carries pre-ganglionic fibres Explanation: Gray ramus means - Non myelinated fibers, so answer can be picked up easily as all pre-ganglionic fibers are Myelinated - white ramus Lateral horn cell gives sympathetic Neuron fibers which passes through ventral root of the spinal cord & enters spinal Nerve; passes through distal white ramus & enter Splanchnic N & then synapse in preveebral ganglion which controls the effectors sending post- ganglionic Nerves to viscera (eg. If sent to stomach, it decreases the peristalsis of stomach &| ses stomach distension). Lateral Horn cell sends fibers through ventral root of spinal cord |enter Spinal N - enter sympathetic chain & synapses here (distal white ramus), which gives - Post ganglionic fiber (non-Myelinated, hence gray ramus) - reenter spinal N - go & supply sweat glands in the skin. Gray Ramus is proximal & medial to distal white ramus. Pain of the skin is carried by sensory nerve, enter spinal N from here goes to dorsal root ganglion - enter dorsal root - synapse in posterior Horn cell - crosses midline & continue as lateral spinothalamic tract Thus, Dorsal root ganglionic not only receives visceral sensations, it is also receives cutaneous sensations. Distal white ramus carries pre-ganglionic fiber, not gray ramus." "This is a real-world medical entrance exam question, please give the true answer based on the question and selection. Topic name: None, Subject name: Dental","A 8-years-old patient with a supernumerary tooth and crowding in the arch comes to you; what will be your line of treatment: Select and explain. A. Wait and watch, B. Extraction and orthodontic treatment, C. Serial extraction, D. Immediate intervention with removal appliance",A: Wait and watch,"Answer: A: Wait and watch Explanation: None" "This is a real-world medical entrance exam question, please give the true answer based on the question and selection. Topic name: None, Subject name: Unknown","Consumption of which of the following poison produce bluish discoloration of stomach during postmortem examination. Select and explain. A. Sodium amytal, B. Soneryl, C. Oxalic acid, D. Arsenic",A: Sodium amytal,"Answer: A: Sodium amytal Explanation: Ref. Textbook of forensic medicine. Krishnan Vij. Page. 586   Barbiturate poisoning They are rapidly absorbed from the gastrointestinal tract including the rectum. They are concentrated in the liver for a short time and then distributed into the body tissues and fluids Long-acting, 4–7gm; intermediate-acting, 2–3gm; short- and ultrashort-acting, 1.5–2 gm. Plasma levels of 3.5 mg/dl for short- acting and 10 mg/dl for long-acting barbiturates are indicative of serious toxicity. Cyanosis is usually present. Postmortem staining may be promi- nent. In a few cases, there may be skin blisters, the so-called barbiturate blisters. They are commonly found at the sites where pressure has been exerted between the skin surfaces, such as buttocks, backs of thighs, calves and forearms. Inner surface of stomach has bluish and whitish barbiturate particles" "This is a real-world medical entrance exam question, please give the true answer based on the question and selection. Topic name: None, Subject name: Dental","The classification is given by: Select and explain. A. Ackerman, B. Proffit, C. Dewey, D. Both 1 and 2",D: Both 1 and 2,"Answer: D: Both 1 and 2 Explanation: Figure shows a Venn diagram; Ackerman and Proffit represented the five major characteristics of malocclusion via a Venn diagram. The sequential description of the major characteristics, not their graphic representation, is the key to this classification system, but the interaction of the tooth and jaw relationships with facial appearance must be kept in mind. Note that for each characteristic, the items to be evaluated are listed within the box or circle, with a spectrum of potential problems within that area represented by  opposing terms (spacing ,crowding, symmetry and asymmetry) and items to be evaluated for dentofacial appearance appear separately in the lower part of that field. The circle for each plane of space represents not only the position, but also the orientation of jaws and teeth in that plane of space, and the overlaps between the circles representing the three planes of space are labeled for the orientation problem that this interaction could represent." "This is a real-world medical entrance exam question, please give the true answer based on the question and selection. Topic name: AIIMS 2018, Subject name: Medicine","Which drug is given to prevent acute mountain sickness? Select and explain. A. Acetazolamide, B. Dexamethasone, C. Digoxin, D. Diltiazem",A: Acetazolamide,"Answer: A: Acetazolamide Explanation: Acute mountain Sickness: Acetazolamide causes diuresis Acclimatization 1000 m/day High altitude pulmonary edema High altitude cerebral edema" "This is a real-world medical entrance exam question, please give the true answer based on the question and selection. Topic name: None, Subject name: Gynaecology & Obstetrics","An episiotomy is to be performed in a primigravida in labor. Which of these is an advantage of mediolateral episiotomy over midline episiotomy? Select and explain. A. Less chance of extension, B. Can be repaired at ease, C. Fewer breakdown, D. Lesser blood loss",A: Less chance of extension,"Answer: A: Less chance of extension Explanation: Ans: A. Less chance of extension (Ref: Williams 24/e p551).A mediolateral episiotomy is preferred because it has a much lesser chance of extension through the perineum till anal sphincter, though there is increased risk of blood loss and it is difficult to repair." "This is a real-world medical entrance exam question, please give the true answer based on the question and selection. Topic name: None, Subject name: Anatomy","Normally seen in lumbar spine? Select and explain. A. Scoliosis, B. Lordosis, C. Kyphosis, D. Kyphosis",B: Lordosis,"Answer: B: Lordosis Explanation: Ans. B. LordosisAbnormal Curvatures of the Veebral ColumnLordosis (swayback or saddle back)An abnormally increased lumbar spine's (backward) curvature resulting from trunk muscular weakness or osteomalacia.ScoliosisA condition of lateral detion resulting from unequal growth of the veebral column, pathologic erosion of veebral bodies,or asymmetric paralysis or weakness of veebral muscles. Kyphosis (hunchback or humpback)An abnormally increased thoracic (forward) curvature resulting from osteoporosis.MeningoceleIt is a protrusion of the meninges through the unfused arch of the veebra.Spina bifida occultaIt is failure of the veebral arch to fuse (bony defect only).MeningomyeloceleIt is a protrusion of the spinal cord and the meninges." "This is a real-world medical entrance exam question, please give the true answer based on the question and selection. Topic name: None, Subject name: Dental","Mother of a 10 month old child came with a complaint of non-eruption of teeth, what will be the treatment protocol? Select and explain. A. Wait and watch, B. You will do OPG which shows developing tooth buds, C. PA skull view which shows developing tooth buds, D. Paediatric consultation",A: Wait and watch,"Answer: A: Wait and watch Explanation: Delay in eruption by around 6 months for deciduous dentition is considered normal. The age of the child in question is 10 months. So, wait and watch for further few months is the best method." "This is a real-world medical entrance exam question, please give the true answer based on the question and selection. Topic name: None, Subject name: Biochemistry","A 3- year old child has hepatosplenomegaly. On examination of the bone marrow, large cells are seen with crumpled paper appearance. Which of the following must have accumulated in these cells? Select and explain. A. Spingomyelins, B. Gulcocerebrosides, C. Ceramides, D. Sulphatides",B: Gulcocerebrosides,"Answer: B: Gulcocerebrosides Explanation: Ans. b. Glucocerebroside (Ref. Robbins 9/ p153, 8/e p153)A child who presents with hepatosplenomegaly and pancytopenia, on bone marrow biopsy, large cells with crumpled tissue paper appearance is seen. The clinical picture is suggestive of Gaucher's disease, caused by deposition of Glucocerebroside." "This is a real-world medical entrance exam question, please give the true answer based on the question and selection. Topic name: None, Subject name: Pathology","Which of the following is a negative acute phase reactant? Select and explain. A. Ferritin, B. Haptoglobin, C. Albumin, D. C-reactive protein",C: Albumin,"Answer: C: Albumin Explanation: Answer- C. AlbuminAlbumin is a negative acute phase reactant whereas ferritin, CRP and haptoglobin are positive phase reactants.""The serum levels of most proteins either increase or decrease during the acute phase response. Serum proteins that decrease levels during inflammation are called negative acute phase reactants.C-reactive protein, fibrinogen, protein S, and fibronectin are examples of positive acute phase reactants.Positive Acute Phase ReactantsC-reactive protein (CRP)Serum amyloid AHaptoglobinCeruloplasminalpha 2-Macroglobulinalpha l-Acid glycoproteinFibrinogenComplement (C3, C4)" "This is a real-world medical entrance exam question, please give the true answer based on the question and selection. Topic name: None, Subject name: Dental","Which of the following will lead to spread of communicable disease during disaster, except Select and explain. A. Diarrhoea, B. Acute Respiratory infection, C. Measles, D. Malnutrition",D: Malnutrition,"Answer: D: Malnutrition Explanation: None" "This is a real-world medical entrance exam question, please give the true answer based on the question and selection. Topic name: AIIMS 2018, Subject name: Pharmacology","Which of the following drug must be sold only on production of a prescription by a registered medical practitioner? Select and explain. A. Schedule H, B. Schedule G, C. Schedule X, D. Schedule M",A: Schedule H,"Answer: A: Schedule H Explanation: Schedule H: Drugs which can be sold only with prescription. On the strip of drug a long red line is present. Rx is written on the top right corner. NRx- is written on Narcotic (habit forming) drugs. XRx- not available for general market, only given under special conditions where health care provider can give the drug. OTC- Over The Counter drugs can be given without prescription." "This is a real-world medical entrance exam question, please give the true answer based on the question and selection. Topic name: None, Subject name: Pathology","40 year old female came with fullness in upper right quadrant of abdomen with diabetes type II, hyperlipidemia, biopsy shows Select and explain. A. Biliary cirrhosis, B. Luminal thrombosis, C. Non-alcoholic steatohepatitis, D. Autoimmune hepatitis",C: Non-alcoholic steatohepatitis,"Answer: C: Non-alcoholic steatohepatitis Explanation: None" "This is a real-world medical entrance exam question, please give the true answer based on the question and selection. Topic name: None, Subject name: Dental","Minimum number of lobes required to form a permanent tooth Select and explain. A. 1, B. 2, C. 3, D. 4",D: 4,"Answer: D: 4 Explanation: For permanent dentition" "This is a real-world medical entrance exam question, please give the true answer based on the question and selection. Topic name: None, Subject name: Gynaecology & Obstetrics","Comprehensive emergency obstetric care does not include: Select and explain. A. Manual removal of placenta, B. Hysterectomy, C. Blood transfusion, D. Cesarean section",B: Hysterectomy,"Answer: B: Hysterectomy Explanation: Setting standards of emergency obstetrics and newborn care: Basic emergency obstetric and newborn care provided in health centres, large or small include the facilities for: Administration of antibiotics, oxytocics and anticonvulsants. Manual removal of the placenta. Removal of retained products following miscarriage or abortion. Assisted vaginal delivery preferably with vacuum extractor. Comprehensive emergency obstetric and newborn care, typically delivered in district hospital, includes all basic functions above, plus cesarean section, safe blood transfusion and care to sick and low birth weight newborns including resuscitation. It is recommended that for every 5,00,000 people there should be 4 facilities offering comprehensive essential obstetric care." "This is a real-world medical entrance exam question, please give the true answer based on the question and selection. Topic name: None, Subject name: Dental","All of the following is true about dentinal tubules except: Select and explain. A. Inner pulpal layer contains more dentinal tubules than outer dentin layer, B. The diameter of dentin tubules ranges from 5 to 7μm, C. The diameter of dentinal tubule is more at pulpal layer than outer layer, D. The bonding is more difficult on the inner Layer of dentin compared to outer surface",B: The diameter of dentin tubules ranges from 5 to 7μm,"Answer: B: The diameter of dentin tubules ranges from 5 to 7μm Explanation: None" "This is a real-world medical entrance exam question, please give the true answer based on the question and selection. Topic name: None, Subject name: Medicine","A person is HBsAg positive, but Anti- HBc Ab is negative. What should he the next step? Select and explain. A. Repeat test after 6 months, B. Check HBV DNA load, C. Check HBeAg, if positive sta interferon, D. Reassure patient that he does not have any disease",B: Check HBV DNA load,"Answer: B: Check HBV DNA load Explanation: Ans: B. Check HBV DNA load(Ref Harrison 19/e p2032, 18/e p2551)Chronic Hepatitis B - Parameters:HBsAg positive; Anti- HBc Ab negative.Presence or absence of serum hepatitis Be antigen (HBeAg).HBeAg-reactive & HbeAg negative.HBV DNA level correlate with liver injury level & has progression risk.Treatment pre-requisites:HBV-DNA load should be done before initiating treatment." "This is a real-world medical entrance exam question, please give the true answer based on the question and selection. Topic name: None, Subject name: Gynaecology & Obstetrics","A 34-year-old primigravida at 11 weeks gestation presents to her obstetrics clinic with chief complain of exposure to a rash. Her husband is HIV+ve and has broken out on a rash in his left buttock which consists of a grouped vesicles on a maculopapular base, 4 days back. She has got her HIV testing done which is negative. Her P/R is 86/min, B/P = 100/60 mm of hg, resp rate 10/min and temp = 98.7F.FHS is heard via Doppler.What is the next step in the management: Select and explain. A. Administer high dose acyclovir to the infant at birth., B. Administer high dose acyclovir to the patient now., C. Administer varicella immunoglobulin to the infant at birth., D. Administer varicella immunoglobulin to the patient",D: Administer varicella immunoglobulin to the patient,"Answer: D: Administer varicella immunoglobulin to the patient Explanation: DOC for treatment of pregnant mothers infected with chickenpox is i/v acyclovir The pregnant woman is exposed to chickenpox rash, she does not have chickenpox…so obviously we will not treat her or her baby with acyclovir. Now since the female herself does not have chickenpox so why to give VZIG to the infant, rather this female should be given prophylactic VZIG so that she does not acquire chickenpox. Varicella prophylaxis: Exposed pregnant women who are susceptible should be given Varicella IG within 96 hrs of exposure to prevent or attenuate varicella infection." "This is a real-world medical entrance exam question, please give the true answer based on the question and selection. Topic name: None, Subject name: Dental","Which of the following is most appropriate for pseudopocket ? Select and explain. A. Is a feature of periodontitis, B. Is associated with attachment loss, C. Is associated with trans-septal fibres loss, D. May have a depth of more than 3 mm",D: May have a depth of more than 3 mm,"Answer: D: May have a depth of more than 3 mm Explanation: None" "This is a real-world medical entrance exam question, please give the true answer based on the question and selection. Topic name: None, Subject name: Dental","It is necessary to remove a disto-occlusal inlay from maxillary premolar. The procedure of choice is: Select and explain. A. Cut through the isthmus and remove the inlay in 2 pieces, B. Use a chisel and mallet, C. Try to tease the inlay out with help of an elevator, D. Remove the inlay in one piece",A: Cut through the isthmus and remove the inlay in 2 pieces,"Answer: A: Cut through the isthmus and remove the inlay in 2 pieces Explanation: None" "This is a real-world medical entrance exam question, please give the true answer based on the question and selection. Topic name: None, Subject name: Dental","Occlusal isthmus of a M.O. dental amalgam is resistant to fracture if: Select and explain. A. Pulpal floor depth is 1mm, B. Dove tail is present, C. Axiopulpal line angle is rounded, D. Unsupported enamel at the gingival cavosurface margin is planed",C: Axiopulpal line angle is rounded,"Answer: C: Axiopulpal line angle is rounded Explanation: None" "This is a real-world medical entrance exam question, please give the true answer based on the question and selection. Topic name: None, Subject name: Gynaecology & Obstetrics","Best test for fetal maturity in a diabetic mother is : Select and explain. A. L:S ratio, B. Lecithin-cephalin ratio, C. Phosphatidyl choline, D. Phosphatidyl glycerol",D: Phosphatidyl glycerol,"Answer: D: Phosphatidyl glycerol Explanation: None" "This is a real-world medical entrance exam question, please give the true answer based on the question and selection. Topic name: None, Subject name: Pediatrics","Which one of the following is the earliest manifestation of Cushing's syndrome : Select and explain. A. Loss of diurnal variation, B. Increased ACTH, C. Increased plasma cortisol, D. Increased urinary metabolites of cortisol",A: Loss of diurnal variation,"Answer: A: Loss of diurnal variation Explanation: Nelson writes : ""Laboratoiy .findings (for Cushing Syndrome)"" Cortisol levels in blood are normally elevated at 8 AM and decrease to less than 50% by midnight except in infants and young children in whom a diurnal rhythm is not always established. In patients with cushing syndrome this circadium rhythm is lost, and cortisol levels at midnight and 8 AM are usually comparable. Obtaining diurnal blood samples present logistical difficulties as part of an outpatient evaluation, but cortisol can be measured in saliva samples, which can be obtained at home at the appropriate times of day. Night time salivary cortisol levels are elevated and may be a screening test in obese children." "This is a real-world medical entrance exam question, please give the true answer based on the question and selection. Topic name: None, Subject name: Social & Preventive Medicine","What is under-5 moality' rate in world by 2010? Select and explain. A. 6 million, B. 8 million, C. 10 million, D. 12 million",B: 8 million,"Answer: B: 8 million Explanation: Answer- B. 8 millionIn 2012, 6.6 million, 2011, 6.9 million children underfive died, down from 7.6 million in, 8.1 million in 2009 and 12.4 million in 1990." "This is a real-world medical entrance exam question, please give the true answer based on the question and selection. Topic name: None, Subject name: Dental","Position and movement of articular disc of TMJ is controlled by fibers of: Select and explain. A. Lower head of lateral pterygoid muscle, B. Sphenomandibular ligament, C. Superficial head of medial pterygoid muscle, D. Upper head of lateral pterygoid muscle",D: Upper head of lateral pterygoid muscle,"Answer: D: Upper head of lateral pterygoid muscle Explanation: None" "This is a real-world medical entrance exam question, please give the true answer based on the question and selection. Topic name: AIIMS 2019, Subject name: Social & Preventive Medicine","Which of the following should be considered a 'High risk infant'? Select and explain. A. Mal-presentation, B. Folic acid tablet not consumed, C. Working mother, D. Antenatal preeclampsia",C: Working mother,"Answer: C: Working mother Explanation: - High risk infant includes, Bih weight < 2.5 kg Bih order > 5 Twin delivery Aificial feeding Weight < 70% of expected Failure to thrive Diarrhea/Protein energy malnutrition Working Mother /single parent" "This is a real-world medical entrance exam question, please give the true answer based on the question and selection. Topic name: None, Subject name: Social & Preventive Medicine","The number of patients required in a clinical trial to treat a specify disease increases as Select and explain. A. The drop-out rate increases, B. The significance level increases, C. The size of the expected treatment effect increased, D. The incidence of the disease decreases",A: The drop-out rate increases,"Answer: A: The drop-out rate increases Explanation: None" "This is a real-world medical entrance exam question, please give the true answer based on the question and selection. Topic name: AIIMS 2019, Subject name: Physiology","Myosin head detachment from actin is triggered by:- Select and explain. A. Change in troponin c configuration, B. Entry of Ca into sarcoplasmic reticulum, C. Binding of ATP to head, D. Hydrolysis of ATP to ADP and Pi",C: Binding of ATP to head,"Answer: C: Binding of ATP to head Explanation: At rest, myosin heads are bound=adenosine diphosphate(are said to be in a ""cocked"" position)in relation to the thin filament = does not have Ca 2+ bound to = the troponin--tropomyosin complex. Ca 2+ bound to = troponin--tropomyosin complex induces = conformational change in the thin filament = that allows for myosin heads to cross-bridge with thin filament actin. Myosin heads rotate = move the attached actin and shoen the muscle fiber = forming power stroke. At the end of power stroke = ATP binds = exposed site = causes = detachment = from actin filament. ATP is hydrolyzed = into ADP and inorganic phosphate (P i ). This chemical energy is used to ""re-cock"" the myosin head." "This is a real-world medical entrance exam question, please give the true answer based on the question and selection. Topic name: None, Subject name: Social & Preventive Medicine","Immunization that could help post-disaster includes Select and explain. A. Measles, B. Cholera, C. Typhoid, D. All",A: Measles,"Answer: A: Measles Explanation: Ans. A. MeaslesVaccines recommended in disastersFollowing vaccines are recommendedChildren < 10 years:- DPT, inactivated polio (IPV), H.influenzae type b (Hib), hepatitis B, pneumococcal conjugate vaccine (PCV), measles-mumps-rubella (MMR), varicella vaccine, influenza, hepatitis A and rotavirus.Children and adolescents (11-18 years):- Tetanus, diphtheria, peussis, meningococcal conjugate vaccine (MCV), Influenza.Adults (>18 years):- Tetanus, diphtheria, peussis, pneumococcal polysaccharide vaccine (PPSV23), and influenza.Vaccination against typhoid and cholera is not recommended." "This is a real-world medical entrance exam question, please give the true answer based on the question and selection. Topic name: None, Subject name: Anatomy","Sphenoidal air sinus is supplied by which nerve: Select and explain. A. Posterior ethmoidal, B. Posterior superior, C. Sphenoidal, D. Infratemporal",A: Posterior ethmoidal,"Answer: A: Posterior ethmoidal Explanation: Posterior ethmoidal nerve enters the posterior ethmoidal foramen and supplies the ethmoidal and sphenoidal air sinuses." "This is a real-world medical entrance exam question, please give the true answer based on the question and selection. Topic name: None, Subject name: Anatomy","Lower two pas of sternal body is fused by: Select and explain. A. 8 years, B. 10 years, C. 12 years, D. 14 years",D: 14 years,"Answer: D: 14 years Explanation: Ans: D. 14 yearsLower two pas of sternal body is fused by 14 years." "This is a real-world medical entrance exam question, please give the true answer based on the question and selection. Topic name: None, Subject name: Anatomy","Sensory nerve fibres to posterior one third of the tongue is supplied by: Select and explain. A. XII cranial nerve, B. IX cranial nerve, C. X cranial nerve, D. VII cranial nerve",B: IX cranial nerve,"Answer: B: IX cranial nerve Explanation: None" "This is a real-world medical entrance exam question, please give the true answer based on the question and selection. Topic name: None, Subject name: Medicine","Nobel prize for medicine/physiology in 2018 for the discovery of: Select and explain. A. Apoptotic pathway, B. Crispr-Cas9, C. Negative immune regulation, D. Molecular mechanisms controlling circadian rhythm",C: Negative immune regulation,"Answer: C: Negative immune regulation Explanation: Ans. C. Negative immune regulationNobel Prize in Physiology or Medicine jointly, has been awarded to James P. Allison/Tasuku Honjo for their discovery of cancer therapy by inhibition of negative immune regulation.James P. Allison studied a known protein that functions as a brake on the immune system. He realized the potential of releasing the brake and thereby unleashing our immune cells to attack tumors.Cytotoxic T-Lymphocyte-associated Antigen 4 (CTLA-4) and Programmed Death 1 (PD-1) immune checkpoints are negative regulators of T-cell immune function. Inhibition of these targets, resulting in increased activation of the immune system, has led to new immunotherapies for melanoma, non-small cell lung cancer, and other cancers.For several types of cancer, including lung cancer, renal cancer, lymphoma and melanoma checkpoint therapy against PD-1 has proven more effective and positive results found.But recent clinical studies indicate that for melanoma the combination therapy of both CTLA-4 and PD-1 is more effective.Thus, Allison and Honjo combine different strategies to release the brakes on the immune system with the aim of eliminating tumor cells even more efficiently.Their paper formed the basis of current cancer immunotherapy.But the original concept of cancer immunotherapy was discovered by William Bradley Coley (1872)." "This is a real-world medical entrance exam question, please give the true answer based on the question and selection. Topic name: None, Subject name: Pediatrics","A child with fever and sore throat developed acute cervical lymphadenopathy most likely investigation to be done is : Select and explain. A. Open biopsy of node, B. Radical neck dissection, C. Neck X–ray, D. Complete hemogram",D: Complete hemogram,"Answer: D: Complete hemogram Explanation: Cervical lymphadenopathy with signs of infection such as fever and sore throat points towards any infectious etiology. In this case the most likely investigation to be done should be complete hemogram. Complete hemogram can provide useful datas for diagnosis of : Pyogeizic infections. Viruses such as EBV CMV & HIV. Leukemias (acute or chronic). Biopsy should be done if the patients history and physical findings suggest malignancy. These features are : Solitary, hard non-tender cervical nodes in an older patient. Neck X-ray does not provide much information except for giving some idea about swollen soft tissues of neck. Radical neck dissection is absurd option." "This is a real-world medical entrance exam question, please give the true answer based on the question and selection. Topic name: None, Subject name: Gynaecology & Obstetrics","Dose of dexamethasone given to mother in anticipated preterm delivery: Select and explain. A. 12 mg 12 hourly 2 doses, B. 12 mg 24 hourly 4 doses, C. 6 mg 24 hourly 2 doses, D. 6 mg 12 hourly 4 doses",D: 6 mg 12 hourly 4 doses,"Answer: D: 6 mg 12 hourly 4 doses Explanation: Ans: D. 6 mg 12 hourly 4 doses(Ref Dutta 8/e p367, 7/e p316: Nelson 20/e p 852)Dose of dexamethasone given to anticipated preterm delivery mother - 6 mg 12 hourly 4 doses.Antenatal coicosteroids:Single course recommended for 24-34 weeks gestation with preterm delivery risk.Drugs & dosage:Dexamethasone (6 mg, 12 hourly, 4 doses).Betamethasone (12 mg, 2 doses, 24 hours apa)." "This is a real-world medical entrance exam question, please give the true answer based on the question and selection. Topic name: None, Subject name: Anatomy","Not a part of ethmoid bone is Select and explain. A. Inferior turbinate, B. Agar nasi cells, C. Uncinate process, D. Crista galli",A: Inferior turbinate,"Answer: A: Inferior turbinate Explanation: The ethmoid bone is cuboidal in overall shape and contains the ethmoidal cells (ethmoidal sinuses). It is composed of Two rectangular box-shaped ethmoidal labyrinths, one on each side. They are united superiorly across the midline by a perforated sheet of bone - the cribriform plate. The perpendicular plate - A second sheet of bone descends vertically in the median sagittal plane from the cribriform plate to form part of the nasal septum. Ethmoidal labyrinth: Each ethmoidal labyrinth is composed of two delicate sheets of bone, which sandwich between them the ethmoidal cells. ■ The lateral sheet of bone (the orbital plate) is flat and forms part of the medial wall of the orbit. ■ The medial sheet of bone forms the upper part of the lateral wall of the nasal cavity and is characterized by two processes and a swelling— The two processes are curved shelves of bone (the superior and middle conchae), which project across the nasal cavity and curve downward ending in free medial margins, while inferior to the origin of the middle concha, the middle ethmoidal cells form a prominent bulge (the ethmoidal bulla), on the medial wall of the labyrinth. Agger nasi - runs forward and downwards from upper end of anterior border of middle concha. TO NOTE: Inferior concha/ inferior nasal turbinate is an independant bone, not a projection of ethmoidal labyrinth. Ethmoidal infundibulum:- Extending anterosuperiorly from just under the bulla is a groove (the ethmoidal infundibulum), which continues upward, and narrows to form a channel that penetrates the ethmoidal labyrinth and opens into the frontal sinus. This channel is for the frontonasal duct, which drains the frontal sinus. Uncinate process:- A delicate irregularly shaped projection (the uncinate process) on the anterior aspect of the inferior surface of the ethmoidal labyrinth. Cribriform plate: The cribriform plate is at the apex of the nasal cavities and fills the ethmoidal notch in the frontal bone and separates the nasal cavities below from the cranial cavity above. Small perforations in the bone allow the fibers of the olfactory nerve to pass between the two regions. A large triangular process (the crista galli) at the midline on the superior surface of the cribriform plate anchors a fold (falx cerebri) of dura mater in the cranial cavity. Perpendicular plate of ethmoid: The perpendicular plate of the ethmoid bone is quadrangular in shape, descends in the midline from the cribriform plate, and forms the upper part of the median nasal septum." "This is a real-world medical entrance exam question, please give the true answer based on the question and selection. Topic name: None, Subject name: Biochemistry","Anaerobic glycolysis of which of these produces 3 ATPs per unit glucose consumed? Select and explain. A. Amino acid, B. Fructose, C. Galactose, D. Glycogen",D: Glycogen,"Answer: D: Glycogen Explanation: Answer- D. GlycogenAnaerobic glycolysis of glycogen produces 3 ATPs per unit glucose consumed.Consumption of ATP at the level of hexokinase is not required when we sta from glycogen as a substrate. As there is no glucose-6- phosphatase in muscle, glucose-6-phosphate directly enters into glycolysis. Hence, net ATPs are 4-1 = 3 ATPs." "This is a real-world medical entrance exam question, please give the true answer based on the question and selection. Topic name: AIIMS 2019, Subject name: Biochemistry","In which of the inheritance, if father is affected no offspring is affected, but if mother affected, all offspring affected? Select and explain. A. Mitochondrial, B. X linked recessive, C. Autosomal dominant, D. Autosomal recessive",A: Mitochondrial,"Answer: A: Mitochondrial Explanation: Mitochondrial Inheritence Mitochondria is derived only from mother Mitochondrial disease has high incidence as : - Mitochondrial DNA repair cannot occur - Introns are not present - Continuous exposure to the oxygen free radicals Therefore have more chances of mutations in mitochondrial DNA Mitochondrial Inheritence Affected mother will transmit disease to all offsprings X linked disease No male to male transmission Y linked disease Affected father will transmit disease to all son Autosomal disease Equal frequency of these disease in male and female" "This is a real-world medical entrance exam question, please give the true answer based on the question and selection. Topic name: AIIMS 2017, Subject name: Pharmacology","A patient on anti-tubercular drug therapy developed tingling sensation on lower limb. Which of the following drug should be used for the treatment? Select and explain. A. Thiamine, B. Folic acid, C. Pyridoxine, D. Vitamin B-12",C: Pyridoxine,"Answer: C: Pyridoxine Explanation: Patient is developing tingling sensation (peripheral neuropathy) which is a side effect of Isoniazid (INH) a first line antitubercular drug. INH is extensively metabolized in liver; most impoant pathway being N-acetylation by NAT2. The acetylated metabolite is excreted in urine. The rate of INH acetylation shows genetic variation. There are either: Fast acetylators: (30-40% of Indians) t1/2 of INH 1 hr. Slow acetylators: (60-70% of Indians) t1/2 of INH 3 hr. Slow acetylators will not be able to metabolize the drug quickly and there will be accumulation of isoniazid. Isoniazid inhibits pyridoxal phosphokinase enzyme and hence causes peripheral neuritis, paraesthesias, numbness etc. Pyridoxine given prophylactically (10mg/day) prevents this neurotoxicity. On the other hand, fast acetylators are more prone to develop." "This is a real-world medical entrance exam question, please give the true answer based on the question and selection. Topic name: None, Subject name: Microbiology","DPT Vaccine is not contraindicated in? Select and explain. A. Progressive neurological disorder, B. Severely immunocompromised patient, C. High allergic response to 1st dose, D. Severe convulsions in a patient",B: Severely immunocompromised patient,"Answer: B: Severely immunocompromised patient Explanation: None" "This is a real-world medical entrance exam question, please give the true answer based on the question and selection. Topic name: None, Subject name: Anatomy","A non synovial joint with connective tissue in between is: Select and explain. A. Syndesmosis, B. Synchondrosis, C. Saddle, D. Suture",A: Syndesmosis,"Answer: A: Syndesmosis Explanation: None" "This is a real-world medical entrance exam question, please give the true answer based on the question and selection. Topic name: None, Subject name: Dental","According to WHO guidelines for excess sugar control by all except Select and explain. A. Fiscal policies, B. Chair side council, C. Marketing, D. Food Labelling",B: Chair side council,"Answer: B: Chair side council Explanation: None" "This is a real-world medical entrance exam question, please give the true answer based on the question and selection. Topic name: AIIMS 2017, Subject name: Pathology","All of the following decrease in iron deficiency anemia except: Select and explain. A. Ferritin, B. TIBC, C. Serum iron, D. Transferrin saturation",B: TIBC,"Answer: B: TIBC Explanation: The diagnosis of iron deficiency anemia ultimately rests on laboratory studies. Both the hemoglobin and hematocrit are depressed, usually to a moderate degree, in association with hypochromia, microcytosis, and modest poikilocytosis. The serum iron and ferritin are low The total plasma iron-binding capacity (reflecting elevated transferrin levels) is high. Low serum iron with increased iron binding capacity results in a reduction of transferrin saturation to below 15%. Reduced iron stores inhibit hepcidin synthesis, and its serum levels fall. In uncomplicated iron deficiency, oral iron supplementation produces an increase in reticulocytes in about 5 to 7 days that is followed by a steady increase in blood counts and the normalization of red cell indices Earliest parameter to be affected in the Iron deficiency anemia patient - Serum ferritin. Gold standard for making the diagnosis of Iron deficiency anemia - Bone marrow examination (storage iron in the form of ferritin or protein hemosiderin can be detected with the help of stain Prussian blue.)" "This is a real-world medical entrance exam question, please give the true answer based on the question and selection. Topic name: None, Subject name: Anaesthesia","What is the ratio of chest compressions and breaths when a lone person is giving cardiopulmonary resuscitation? Select and explain. A. 10:01, B. 15:01, C. 30:01:00, D. 30:02:00",D: 30:02:00,"Answer: D: 30:02:00 Explanation: Ans: D. 30:2(Ref AHA 2015 CPR Guidelines "" wp-content/uploads/2015/10/2015-AHA!Guidclinet.-Hilivhtv-En,cgish ficIf.)Ratio of chest compressions to rescue breath in all adults (Even with 1 or 2 rescuers) = 30:2." "This is a real-world medical entrance exam question, please give the true answer based on the question and selection. Topic name: None, Subject name: Microbiology","Hydrolysis of IgG with papain will lead to formation of following fragments: Select and explain. A. 1 Fc and 2 Fab fragments, B. 2 Fc and 1 Fab fragment, C. 1 variable chain and 1 constant chain, D. 1 Fab and 1 hypervariable region",A: 1 Fc and 2 Fab fragments,"Answer: A: 1 Fc and 2 Fab fragments Explanation: Answer- A. 1 Fc and 2 Fab fragmentsHydrolysis of IgG with papain will lead to the formation of 1 Fc and 2 Fab fragments. Papain hydrolyzes IgG at the hinge region, which lyses IgG into one constant Fc region and two Fab fragments.When studying the Ig molecule structure, it was identified experimentally thot an antibody molecule, such as IgG, can be split into two fragments by the proteolytic enzyme, papain. When this happens, the peptide bonds in the hinge region are broken.The antigen-binding activity is associated with one of these fragments, the Fab poion- The second fragment is the Fc poion that is involved in the placental transfer, complement fixation, attachment to various cells, and other biologic activities." "This is a real-world medical entrance exam question, please give the true answer based on the question and selection. Topic name: None, Subject name: Gynaecology & Obstetrics","Fetal adrenals release which hormone predominantly:March 2009, September 2010 Select and explain. A. Oestrogen, B. Testosterone, C. Aldosterone, D. Coisone",D: Coisone,"Answer: D: Coisone Explanation: Ans. D: CoisoneFetal adrenals shows hyperophy of the reticular zone (fetal zone), which is the site of synthesis of oestriol precursor, coisol and dehydroepiandrosterone.After 12 weeks of gestation, the activity of 3 p-hydroxysteroid dehydrogenase (HSD3B) decreases in fetal adrenal and sulfokinase activity increases.At that time (during the mid-gestation, 12 to 22 weeks) the major steroid products are DHEA and DHEA Sulfate (DHEA-S).During the mid-gestation (12 to 22 weeks), the aromatase activity and sulfokinase activity increases. So Placenta itself utilizes fetal DHEA and DHEA-S as substrate for estrone and estradiol as precursors." "This is a real-world medical entrance exam question, please give the true answer based on the question and selection. Topic name: None, Subject name: Dental","Fear developed in a patient due to previous experience, events, objects: Select and explain. A. Innate fear., B. Subjective fear., C. Objective fear., D. None.",C: Objective fear.,"Answer: C: Objective fear. Explanation: Types of fear:" "This is a real-world medical entrance exam question, please give the true answer based on the question and selection. Topic name: None, Subject name: Dental","Sterilization of hand piece will cause: Select and explain. A. Rust of hand piece, B. Turbine wear, C. Fibre loss, D. Loss of torque",C: Fibre loss,"Answer: C: Fibre loss Explanation: None" "This is a real-world medical entrance exam question, please give the true answer based on the question and selection. Topic name: None, Subject name: Gynaecology & Obstetrics","A pregnant lady develops chickenpox. During which part of her pregnancy will it lead to highest chance of neonatal infection: Select and explain. A. Last 5 days, B. 12-16 week, C. 8-12 week, D. 16-20 week",A: Last 5 days,"Answer: A: Last 5 days Explanation: None" "This is a real-world medical entrance exam question, please give the true answer based on the question and selection. Topic name: None, Subject name: Pediatrics","A male child with coarse facial features, macroglossia, thick lips presents with copious mucous discharge from nose at 10 months of age. The child was absolutely normal at bih. On examination he was found to have enlarged Liver and Spleen. Diagnosis is: Select and explain. A. Hurler's syndrome, B. Beckwith-Weidman syndrome, C. Hypothyroidism, D. Proteus syndrome",A: Hurler's syndrome,"Answer: A: Hurler's syndrome Explanation: Ans: A. Hurler's syndrome(Ref Nelson 20/e p739)Likely diagnosis = Hurler's syndrome.Hurler's syndrome:Type 1 mucopolysacharidoses.AR disorder.Characterized by alpha L-iduronidase deficiency resulting in accumulation of dermatan > heparin-sulfate.Clinical features:Presents with Goesque gargoyle facies:Coarse and heavy face.Enlarged head - Due to hydrocephalus caused by meningeal deposits.Low forehead & ears.Eyes wide set.Wide nosePoorly formed & widespread teeth.Open mouth, enlarged tongue & eveed lips.Hepatosplenomegaly.Sho neck.Thoracolumbur kyphosis.Gibbus deformity with motor delays.Flexion contracture of joints.Sho stature.Genu valgum.Fiat feet.Broad-sho hand, radially curved little finger.Carpal tunnel syndrome in children." "This is a real-world medical entrance exam question, please give the true answer based on the question and selection. Topic name: None, Subject name: Gynaecology & Obstetrics","G3 with previous second trimester abortion presents with 22 week of gestation, abdominal pain, USG shows funneling of internal os. What is the ideal management? Select and explain. A. Dinoprost and bed rest, B. Misoprost and bed rest, C. Fothergills stitch, D. Mc Donald stitch",D: Mc Donald stitch,"Answer: D: Mc Donald stitch Explanation: Patient presenting at 22 weeks with: Funnelling of cervix on ultrasound examination and history of second trimester abortions indicating cervical incompetence as the cause of preterm labor. In this case Mc Donald stitch will be the ideal treatment as it will prevent preterm labor." "This is a real-world medical entrance exam question, please give the true answer based on the question and selection. Topic name: None, Subject name: Dental","The neutralization of saliva is due to Select and explain. A. Mucin, B. Ammonia, C. Amino a cids, D. Bicarbonates",D: Bicarbonates,"Answer: D: Bicarbonates Explanation: None" "This is a real-world medical entrance exam question, please give the true answer based on the question and selection. Topic name: None, Subject name: Surgery","A 45 y ears old patient presented to you with ongoing massive hematemesis. The patient is ale and hemodynamically stable. What will be the first step in management? Select and explain. A. Do an urgent upper GI endoscopy, B. Put the patient in recovery position and secure airway, C. Inse a cannula and sta IV fluids, D. Send for blood transfusion",B: Put the patient in recovery position and secure airway,"Answer: B: Put the patient in recovery position and secure airway Explanation: Ans: B. Put the patient in recovery position and secure airway(Ref BLS/ACLS Guidelines: resuscitation guidelines /adult- aclvanced-lik-suppo: Harrison 19/e p1768: Bratinwald's 10/e p844-845)BLS algorithm:Sta airway management in any patient who is collapsed/expected to collapse.Patient here is hemodynamically stable.Airway - Most impoant component at risk in this patient & requires immediate attention.Put patient in recovery position.i.e. left lateral decubitus position to prevent the risk of aspiration." "This is a real-world medical entrance exam question, please give the true answer based on the question and selection. Topic name: None, Subject name: Surgery","Extraction of a tooth during acute infection: Select and explain. A. Can cause extensive spread of infection, B. Helps drainage and relieves pain if proper antibiotic is given and its adequate blood level is reached, C. Can cause sudden death due to pulmonary embolism, D. Can Lead to trigeminal neuralgia in post operative period",B: Helps drainage and relieves pain if proper antibiotic is given and its adequate blood level is reached,"Answer: B: Helps drainage and relieves pain if proper antibiotic is given and its adequate blood level is reached Explanation: None" "This is a real-world medical entrance exam question, please give the true answer based on the question and selection. Topic name: None, Subject name: ENT","Kamla 4 yrs of age presented in emergency with mild respiratory ""stress. On laryngoscopy she was diagnosed to have multiple juvenile papilomatosis of the larynx. Next line of management is - Select and explain. A. Tracheostomy, B. Microlaryngoscopy, C. Steroid, D. Antibiotics",B: Microlaryngoscopy,"Answer: B: Microlaryngoscopy Explanation: The patient (a 4 years girl) in the question is presenting with mild respiratory distress due to multiple Juvenile papillomatosis of larynx. The management in such a case is microlarygoscopic surgery using CO2 laser to ablate the lesion. Steroids and antibiotics have no role. Tracheostomy is reserved for those patients who have severe respiratory distress." "This is a real-world medical entrance exam question, please give the true answer based on the question and selection. Topic name: None, Subject name: Pediatrics","Unconjugated bilirubin is increased in all, except – Select and explain. A. Criggler Najjar syndrome, B. Dubin Johnson syndrome, C. Gilbert syndrome, D. Hemolytic anaemia",B: Dubin Johnson syndrome,"Answer: B: Dubin Johnson syndrome Explanation: Dubin Jhonson syndrome is a cause of congenital conjugated hyperbilirubinemia. Congenital conjugated hyperbilirubinemia 1)Dubin Jhonson syndrome DJS is a type of congenital conjugated hyperbilirubinemia. It is autosomal recessive. Conjugated bilirubin is increased because of defective biliary excretion of bilirubin glucuronides due to mutation in canalicular multidrug resistance protein 2 (MRP 2). Liver function tests are normal A cardinal feature of DJS is the accumulation in the lysosome of centrilobular hepatocytes of dark, coarsely granular pigment. As a result, the liver is black in appearance. This pigment is thought to be derived from epinephrine metabolites that are not excreted normally. There is increased urinary excretion of coproporphyrine I (normal coproporphyrine III is more excreted), but total coproporphyrine level is normal. Gall bladder is not visualized on oral cholecystography. After iv administration, there is reflux of conjugated sulfobromophthalein (Bromsulphalein, BSP) from liver to circulation. 2)Rotor syndrome Rotor syndrome is a type of congenital conjugated hyperbilimbinemia. It is autosomal recessive. It is due to decreased biliary excretion of conjugated bilirubin and also due to decrease hepatic uptake & storage of bilirubin. Differentiating features of rotor syndrome (from DJS) - Liver is not pigment Coproporphyrine I is increased in urine but total coproporphyrine level is also increased. Gall bladder is visualized There is no reflux of conjugated BSP." "This is a real-world medical entrance exam question, please give the true answer based on the question and selection. Topic name: None, Subject name: Biochemistry","Which of these is not a cofactor for glycogen phosphorylase, an impoant enzyme of the glycogenolysis pathway? Select and explain. A. Calmodulin, B. c-AMP, C. Protein Kinase A, D. Glycogenin",D: Glycogenin,"Answer: D: Glycogenin Explanation: Ans: D. Glycogenin(Ref Harper 30/c p 181)Glycogenin:An enzyme involved in glucose conversion to glycogen.Acts as a primer - By polymerizing first few glucose molecules a enzymes take over.Involved in glycogen synthesis pathway rather than glycogenolysis." "This is a real-world medical entrance exam question, please give the true answer based on the question and selection. Topic name: None, Subject name: Dental","In which type of lesion 'eburnation' of the dentine is seen? Select and explain. A. Acute caries, B. Chronic caries, C. Arrested caries, D. Root Caries",C: Arrested caries,"Answer: C: Arrested caries Explanation: None" "This is a real-world medical entrance exam question, please give the true answer based on the question and selection. Topic name: None, Subject name: Dental","In case of a deep carious lesion incomplete debridement is done to: Select and explain. A. Allow secondary dentin formation first, B. To seal the cavity and create aseptic field, C. Prevent pulp exposure and allow a thin layer of a dentin to remain, D. Avoid microleakage",C: Prevent pulp exposure and allow a thin layer of a dentin to remain,"Answer: C: Prevent pulp exposure and allow a thin layer of a dentin to remain Explanation: None" "This is a real-world medical entrance exam question, please give the true answer based on the question and selection. Topic name: AIIMS 2019, Subject name: Microbiology","In chronic allergy which lg are more persistent in the body? Select and explain. A. lg G, B. lg E, C. lg A, D. lg M",B: lg E,"Answer: B: lg E Explanation: IgE is highly potent and mediates type I hypersensitivity reactions by binding to the mast cells causing its degranulation. lgE response is seen in various allergic (both acute and chronic) conditions, such as asthma, anaphylaxis, hay fever, etc. lgE has : lowest serum concentration, Shoest half-life, minimum daily production, only heat labile antibody (inactivated at 56degC in one hour), has affinity for the surface of tissue cells (mainly mast cells) of the same species (homocytotropism)." "This is a real-world medical entrance exam question, please give the true answer based on the question and selection. Topic name: None, Subject name: Gynaecology & Obstetrics","Which of the following drug is not used for medical management of ectopic pregnancy: Select and explain. A. Potassium Chloride, B. Methotrexate, C. Actinomycin D, D. Misoprostol",D: Misoprostol,"Answer: D: Misoprostol Explanation: A number of chemotherapeutic drugs have been used either systemically or directly (surgically administered medical management - SAM under sonographic or laparoscopic guidance) for the medical management of ectopic pregnancy. Drugs commonly used for medical management:" "This is a real-world medical entrance exam question, please give the true answer based on the question and selection. Topic name: None, Subject name: Microbiology","The following is characteristic feature of staphylococcus food poisoning except: Select and explain. A. Optimum temperature for toxin production is 370C, B. Intradietic toxin are responsible for intestinal symptoms, C. Toxin can be destroyed by boiling for 30 minutes, D. Incubation period is 1-6 hours",C: Toxin can be destroyed by boiling for 30 minutes,"Answer: C: Toxin can be destroyed by boiling for 30 minutes Explanation: Staphylococcal food poisoning: Staphylococcal food poisoning is due to heat-stable preformed toxin mostly after consuming milk products. Toxin is produced optimally at 350C to 370C.  Mechanism - Acts by stimulating vagus nerve and vomiting centre of brain. No antibiotics are given for Staphylococcal food poisoning." "This is a real-world medical entrance exam question, please give the true answer based on the question and selection. Topic name: None, Subject name: Pathology","A 22-year-old female gives the history of recurrent joint pains. She has now developed petechial hemorrhages. She is most likely to have: Select and explain. A. Megakaryocytic thrombocytopenia, B. Amegakaryocytic thrombocytopenia, C. Platelet function defects, D. Acquired factor VIII inhibitors",D: Acquired factor VIII inhibitors,"Answer: D: Acquired factor VIII inhibitors Explanation: Answer:. d. Acquired factor VIII inhibitors (Wintrobes p1442-144-.'74;Harrison19/ep734-735, 740, 18/e p982).The clinical presentation in a young female of recurrent joint pains with petechial hemorrhage is suggestive of an autoimmune disease.A female patient is unlike to have hemophilia, as it is an X-linked disorder. However, she can have autoantibodies against factor VIII." "This is a real-world medical entrance exam question, please give the true answer based on the question and selection. Topic name: None, Subject name: Dental","The red zones of lips have: Select and explain. A. 30% sebaceous glands, B. 75% sebaceous glands, C. very small number of sebaceous glands, if any, D. absolutely no sebaceous glands","C: very small number of sebaceous glands, if any","Answer: C: very small number of sebaceous glands, if any Explanation: None" "This is a real-world medical entrance exam question, please give the true answer based on the question and selection. Topic name: None, Subject name: Pediatrics","A 2-year-old child was brought to the emergency depament at 3 AM. The child presented with fever and cough. On examination, respiratory rate was 36/ min, temperature was 39degC and saturation on pulse oximetry was 96%. The child had barking cough and stridor only on crying. Otherwise, the child was hydrated, able to drink and consolable. The nest step in management would be: Select and explain. A. Racemic epinephrine nebulization, B. Single dose of dexamethasone, C. Complete blood count and culture, D. Nasal washings for Respiratory Syncytial Virus (RSV) and influenza",B: Single dose of dexamethasone,"Answer: B: Single dose of dexamethasone Explanation: Answer- B (Single dose of dexamethasone)Coicosteroids decrease the edema in the laryngeal mucosa through their anti-inflammatory action.Oral steroids are beneficial, even in mild croup, as measured by reduced hospitalization, shoer duration of hospitalization, and reduced need for subsequent intimations such as epinephrine administration." "This is a real-world medical entrance exam question, please give the true answer based on the question and selection. Topic name: None, Subject name: Microbiology","The virus HIV is produced and propagated in: Select and explain. A. Dendritic cells of the lymphnode germinal epithelium, B. Circulating T4 lymphocytes, C. B- lymphocytes, D. Epithelial cells of the sex organs",B: Circulating T4 lymphocytes,"Answer: B: Circulating T4 lymphocytes Explanation: None" "This is a real-world medical entrance exam question, please give the true answer based on the question and selection. Topic name: None, Subject name: Biochemistry","Deficiency of nicotinic acid results in: Select and explain. A. Beri-beri, B. Pellagra, C. Night blindness, D. Pernicious anemia",B: Pellagra,"Answer: B: Pellagra Explanation: None" "This is a real-world medical entrance exam question, please give the true answer based on the question and selection. Topic name: None, Subject name: Gynaecology & Obstetrics","At 28 weeks gestation, amniocentesis reveals a AOD 450 of 0.20 which is at the top of third zone of the liley curve. The most appropriate management of such a case is: Select and explain. A. Immediate delivery, B. Intrauterine transfusion, C. Repeat Amniocentesis after 1 week, D. Plasmapheresis",B: Intrauterine transfusion,"Answer: B: Intrauterine transfusion Explanation: Management of Rh negative females depends on whether the female is immunized/ nonimmunized. The question which says- what should be done in case of the paitent with 28 weeks pregnancy if ∆ OD lies at the top of Zone 3." "This is a real-world medical entrance exam question, please give the true answer based on the question and selection. Topic name: None, Subject name: Surgery","After a midline laparotomy, you have been asked to suture the incision. What length of suture material will you choose? Select and explain. A. 2x incision length, B. 4x incision length, C. 6x incision length, D. 8x incision length",B: 4x incision length,"Answer: B: 4x incision length Explanation: Ans: B. 4x incision length (Rel. Bailey 27/e p1041, 26/e p965, 25/e p234)According to Jenkins' rule, optimal ratio of suture length to wound length = 4:1.If less length used than ratio a suture bites are too far apa or too tight.Converse applies if more length than ratio used.""" "This is a real-world medical entrance exam question, please give the true answer based on the question and selection. Topic name: None, Subject name: Dental","In response to traumatic occlusal forces, alveolar bone has Select and explain. A. Osteoblasts in areas of tension and osteoclasts in areas of pressure, B. Osteoblasts in areas of tension and osteoblasts in areas of pressure, C. Osteoid in areas of tension and osteoblasts in areas of pressure, D. Afunctional atrophy",A: Osteoblasts in areas of tension and osteoclasts in areas of pressure,"Answer: A: Osteoblasts in areas of tension and osteoclasts in areas of pressure Explanation: None" "This is a real-world medical entrance exam question, please give the true answer based on the question and selection. Topic name: AIIMS 2018, Subject name: Surgery","Class 3 hemorrhagic shock refers to: Select and explain. A. Blood loss less than 15 %, B. Blood loss between 15 % - 30%, C. Blood loss between 30% - 40%, D. Blood loss more than 40%",C: Blood loss between 30% - 40%,"Answer: C: Blood loss between 30% - 40% Explanation: Four classes of Haemorrhagic Shock (According to the ATLS course ) Class Parameter I II III IV Blood loss (%) 0-15% 15-30% 30-40% >40% CNS Slightly anxious Mildly anxious Anxious or confused Confused or lethargic Pulse (Beats/min) < 100 >100 >120 >140 Blood pressure Normal Normal Decreased Decreased Pulse pressure Normal Decreased Decreased Decreased Respiratory rate 14-20/min 20-30 / min 30-40/min > 35/ min Urine (mL/hr) >30 20-30 5-15 Negligible Fluid Crystalloid Crystalloid Crystalloid + Blood Crystalloid + blood Base deficit 0 to -2 mEq/L -2 to -6 mEq/L -6 to -10mEq/L -10mEq/L or less" "This is a real-world medical entrance exam question, please give the true answer based on the question and selection. Topic name: AIIMS 2017, Subject name: Surgery","In a school bus accident, which of the following victim you will attend first? Select and explain. A. A child with Airway obstruction, B. A child with shock, C. A child with flail chest, D. A child with Severe head injury",A: A child with Airway obstruction,"Answer: A: A child with Airway obstruction Explanation: Initial assessment, also termed the primary survey, follows the mnemonic ABCDE: Airway and cervical spine protection, Breathing, Circulation, Disability or neurologic condition, Exposure and environmental control." "This is a real-world medical entrance exam question, please give the true answer based on the question and selection. Topic name: None, Subject name: Dental","Supplementary retention on the abutment tooth is made possible by: Select and explain. A. Greater bulk of metal, B. Proper cementation technique, C. Shoulder preparation, D. Proper use of pins and grooves",D: Proper use of pins and grooves,"Answer: D: Proper use of pins and grooves Explanation: None" "This is a real-world medical entrance exam question, please give the true answer based on the question and selection. Topic name: None, Subject name: Social & Preventive Medicine","According to WHO definition, what is the criteria for considering a high endemic area for meningococcal meningitis? Select and explain. A. <2 per 1,00,000 population, B. 2-10 per 1,00,000 population, C. >10 per 1,00,000 population, D. >100 per 1,00,000 population","C: >10 per 1,00,000 population","Answer: C: >10 per 1,00,000 population Explanation: Ans. c. >10 per 1,00,000 population (Ref: Park 23/e p165, 22/e pl56)WHO defines highly endemic areas where incidence of meningococcal meningitis as number of cases per 1,00,000 population is >10 cases." "This is a real-world medical entrance exam question, please give the true answer based on the question and selection. Topic name: None, Subject name: Surgery","Augmented genioplasty is done by: Select and explain. A. Sliding horizontal osteotomy, B. Bone grafts, C. Silicone implants, D. None of the above",A: Sliding horizontal osteotomy,"Answer: A: Sliding horizontal osteotomy Explanation: None" "This is a real-world medical entrance exam question, please give the true answer based on the question and selection. Topic name: None, Subject name: Pediatrics","Which is not a feature of wilson\'s disease in a child – Select and explain. A. Fanconi syndrome, B. Sensory changes, C. Hemolytic anemia, D. Chronic active hepatitis",B: Sensory changes,"Answer: B: Sensory changes Explanation: Clinical features of wilson disease in children Acute or chronic liver disease                                                                    o KF ring Psychiatric disturbances                                                                            o Hemolytic anemia Neurological --> Rigidity, tremer, Parkinsonism, cerebellar ataxia.        o Fanconi syndrome" "This is a real-world medical entrance exam question, please give the true answer based on the question and selection. Topic name: None, Subject name: Dental","Which of the following zones is used for melting alloy during casting: Select and explain. A. Reducing flame, B. Oxidizing flame, C. Green flame, D. All of the above",A: Reducing flame,"Answer: A: Reducing flame Explanation: None" "This is a real-world medical entrance exam question, please give the true answer based on the question and selection. Topic name: None, Subject name: Pediatrics","Most important prognostic factor in congenital diaphragmatic hernia – Select and explain. A. Pulmonary hypertension, B. Size of hernia, C. Timing of surgery, D. Gestational age",A: Pulmonary hypertension,"Answer: A: Pulmonary hypertension Explanation: ""Compression of the lung results in pulmonary hypoplasia involving both lungs, with the ipsilateral lung being the most affected. In addition to the abnormal airway development, the pulmonary vasculature is distinctly abnormal in that the medial muscular thickness of the arterioles is excessive and extremely sensitive to the multiple local and systemic factors lcnown to trigger vasospasm. Thus, the two main factors that affect morbidity and mortality are pulmonary hypoplasia and pulmonary hypertension.""-Sabiston 18th/e p2073" "This is a real-world medical entrance exam question, please give the true answer based on the question and selection. Topic name: None, Subject name: Dental","A 10 years old child has intrusion of permanent maxillary central incisor. The choice of treatment is Select and explain. A. Put tooth in its place and splint it, B. Allow tooth to erupt on its own (spontaneous eruption), C. Treat it orthodontically, D. Do nothing",B: Allow tooth to erupt on its own (spontaneous eruption),"Answer: B: Allow tooth to erupt on its own (spontaneous eruption) Explanation: None" "This is a real-world medical entrance exam question, please give the true answer based on the question and selection. Topic name: None, Subject name: Gynaecology & Obstetrics","True about diabetic mother is:a) Hyperglycemia occurs in all infants of diabetic mothersb) High incidence of congenital heart anomalies is commonc) Small babyd) Beta agonist drugs are CI during delivery Select and explain. A. c, B. d, C. ab, D. bd",D: bd,"Answer: D: bd Explanation: None" "This is a real-world medical entrance exam question, please give the true answer based on the question and selection. Topic name: None, Subject name: Dental","The advantage of metal base denture is: Select and explain. A. Increase in tissue tolerance, B. Easy laboratory procedure, C. Rebasing and relining are quite easy, D. Increase in restorative cost",A: Increase in tissue tolerance,"Answer: A: Increase in tissue tolerance Explanation: None" "This is a real-world medical entrance exam question, please give the true answer based on the question and selection. Topic name: None, Subject name: Gynaecology & Obstetrics","Anti D prophylaxis is administered in all except? Select and explain. A. MTP at 63 days, B. Amniocentesis at 16 weeks, C. Manual removal of placenta, D. Intra-uterine transfusion 28 weeks",D: Intra-uterine transfusion 28 weeks,"Answer: D: Intra-uterine transfusion 28 weeks Explanation: Ans. D. Intra uterine transfusionAnti D prophylaxis is not given after intrauterine transfusion.Intrauterine transfusion is done to prevent the fetus from dying.If the hydropic fetus is too immature for early delivery, intrauterine transfusion is done.Transfusion is done through intraperitoneal and intravascular routes.This is performed using O-negative, cytomegalovirus negative, washed irradiated packed red cell.The volume to be transfused is roughly calculated by the formula: (weeks of gestation-20) multiplied by 10.Transfusion is repeated whenever fetal hemoglobin levels falls below 10gm/dl." "This is a real-world medical entrance exam question, please give the true answer based on the question and selection. Topic name: None, Subject name: Dental","8 year-old child had fractured his maxillary central incisor 10-months ago. The pulp shows no response. There is no periapical lesion in the radiograph. The treatment of choice is: Select and explain. A. Ca(OH)2 pulp capping, B. Formocresol pulpotomy, C. Conventional root canal treatment, D. Complete debridement and apexification",D: Complete debridement and apexification,"Answer: D: Complete debridement and apexification Explanation: Apexification Definition “Apexification is defined as chemically induced root formation by calcium hydroxide or CMCP in nonvital immature, blunderbuss canals of young permanent teeth.” APEXIFICATION  It is a method of inducing apical closure by formation of mineralized tissue in the apical region of a nonvital permanent tooth with an incompletely formed root apex.  It is defined as a method to induce development of the root apex of an immature pulpless tooth by formation of osteocementum/bone-like tissue (Cohen).  Apexification is a method of inducing apical closure through the formation of mineralized tissue in the apical pulp region of a nonvital tooth with an incompletely formed root and an open apex (Morse et al. 1990)." "This is a real-world medical entrance exam question, please give the true answer based on the question and selection. Topic name: None, Subject name: Pathology","Marked reduction in amount of dentin, widening of predentin layer, presence of large area of interglobular dentin and irregular pattern of dentin is seen in Select and explain. A. Hypocalcified dentin, B. Odonto dysplasia, C. Dentin dysplasia, D. Dentinogenesis imperfecta",B: Odonto dysplasia,"Answer: B: Odonto dysplasia Explanation: None" "This is a real-world medical entrance exam question, please give the true answer based on the question and selection. Topic name: None, Subject name: Dental","One common radiographic finding in clinical gingivitis is Select and explain. A. Vertical bone loss, B. Normal bone pattern, C. Horizontal bone loss, D. Increase in bone density",B: Normal bone pattern,"Answer: B: Normal bone pattern Explanation: None" "This is a real-world medical entrance exam question, please give the true answer based on the question and selection. Topic name: None, Subject name: Dental","An 11-year-old boy complains of spacing between maxillary incisors. The appropriate treatment is with: Select and explain. A. Hawley's appliance, B. Fixed appliance, C. No treatment, D. Oral screen",C: No treatment,"Answer: C: No treatment Explanation: None" "This is a real-world medical entrance exam question, please give the true answer based on the question and selection. Topic name: None, Subject name: Dental","A traumatized tooth nonvital with or without crown fracture is classified as Select and explain. A. Class-III, B. Class-IV, C. Class -V, D. Class-VII",B: Class-IV,"Answer: B: Class-IV Explanation: None" "This is a real-world medical entrance exam question, please give the true answer based on the question and selection. Topic name: None, Subject name: Dental","Habits are classified as Intentional and Unintentional by: Select and explain. A. William James, B. Kingsley, C. Earnest Klein, D. Finn and Sim",C: Earnest Klein,"Answer: C: Earnest Klein Explanation: None" "This is a real-world medical entrance exam question, please give the true answer based on the question and selection. Topic name: None, Subject name: Pathology","Gingiva is most commonly affected by deficiency of: Select and explain. A. Vitamin A, B. Vitamin D, C. Vitamin C, D. Vitamin B",C: Vitamin C,"Answer: C: Vitamin C Explanation: None" "This is a real-world medical entrance exam question, please give the true answer based on the question and selection. Topic name: None, Subject name: Pediatrics","Injection of Glucagon is effective for management of persistent hypoglycemia in all EXCEPT – Select and explain. A. Large for date baby, B. Galactosemia, C. Infant of diabetic mother, D. Nesidioblastosis",B: Galactosemia,"Answer: B: Galactosemia Explanation: Hypoglycemia in hyperinsulinmic state require IM glucagon as treatment. Glucagon increases glucose level in blood by Glycogenolysis and Gluconeogenesis. But in galactosemia - Liver is the first organ to be affected, so glucagon will unable to increase glucose as glycogenolysis and gluconeogensis can not occur ion already damaged liver. Nesidioblastosis : It is persistent hyperinsulinemic hypoglycemia occurs due to 0 - cell hypertrophy in infant of diabetic mother due to continuous maternal hyperglycemia. Glucagon is effective in controlling such a hypoglycemic condition." "This is a real-world medical entrance exam question, please give the true answer based on the question and selection. Topic name: None, Subject name: Dental","Which of the following show chemical bond with enamel (calcified tissues)? Select and explain. A. Composites, B. Direct filling resins, C. Polycarboxylate cements, D. BIS-GMA resins in pit and fissure sealants",C: Polycarboxylate cements,"Answer: C: Polycarboxylate cements Explanation: When the powder and liquid are mixed, the surface of powder particles are attacked by the acid, releasing zinc, magnesium and tin ions. These ions bind to the polymer chain via the carboxyl groups. They also react with carboxyl groups of adjacent polyacid chains to form cross-linked salts. Structure of set cement the hardened cement consists of an amorphous gel matrix of zinc polyacrylate in which unreacted powder particles are dispersed. Key concept:- Zinc polycarboxylate cement was the first dental cement to exhibit chemical bonding to teeth, marking an improvement over the mechanical bonding of zinc phosphate cement. Zinc polycarboxylate cement is not used for restorative purposes because the cement is opaque. Ref: Phillips Ed 12th P: 318" "This is a real-world medical entrance exam question, please give the true answer based on the question and selection. Topic name: AIIMS 2018, Subject name: Pharmacology","'Store in a cool place' is written on a drug label. It means drug should be stored at a temperature of: Select and explain. A. -2 degree Celsius, B. 0 degree Celsius, C. 2-8 degree Celsius, D. 8-15 degree Celsius",D: 8-15 degree Celsius,"Answer: D: 8-15 degree Celsius Explanation: STORAGE TEMPRATURES: LABEL WHERE TO KEEP TEMPRATURE Keep frozen Inside freezer -20 degree celsius Keep cold Inside refrigerator; outside freezer 2-8 degree celsius Keep cool Outside refrigerator 8-15 degree Celsius(in USA); 8-25 degree Celsius (in India)" "This is a real-world medical entrance exam question, please give the true answer based on the question and selection. Topic name: None, Subject name: Pathology","During autopsy of a patient died due to suspected myocardial infarction, the hea was stained with triphenyltetrazolium tetrachloride dye. What will be the color of the ble myocardium? Select and explain. A. White, B. Red, C. Blue, D. Dark Brown",B: Red,"Answer: B: Red Explanation: Ans: B(Ref: Robbins 9/e p544)Triphenyltetrazolium chloride (TTC) stain:Impas a brick-red color to intact, non-infracted myocardium where dehydrogenase enzymes are preserved.Acute Ml - Early morphologic recognition:Infarct preceding death by 2 to 3 hours:Highlighting area of necrosis by immersion of tissue slices in a solution of triphenyltetrazolium chloride.Gross histochemical stain impas brick-red color to intact, non-infarcted myocardium where lactate dehydrogenase activity is preserved.Dehydrogenases leaks out damaged membranes of dead cells.Hence, an infarct appears as an unstained pale zone.By 12 to 24 hours after infarction:MI identified grossly as a reddish-blue area of discoloration.Caused by stagnated, trapped blood.By 2 days to 10 days:Infarct becomes progressively more sharply defined, yellow-tan & soft.By 10 days to 2 weeks:Rimmed by a hyperemic zone of highly vascularised granulation tissue.Over succeeding weeks:Fibrous scar" "This is a real-world medical entrance exam question, please give the true answer based on the question and selection. Topic name: None, Subject name: Dental","ADA specification for orthodontic wires is Select and explain. A. 32, B. 30, C. 27, D. 25",A: 32,"Answer: A: 32 Explanation: 26 - Dental X-ray Equipment  27 - Resin-Based Filling Materials (Direct filling materials)  28 - Endodontic Files and Reamers (Hand Use)  30 - Dental Zinc Oxide Eugenol and Zinc Oxide Non-Eugenol Cements  31 - Exposure time designation for timers of dental X-ray machines 32 - Orthodontic Wires 33 - Dental Terminology / Vocabulary" "This is a real-world medical entrance exam question, please give the true answer based on the question and selection. Topic name: None, Subject name: Dental","Frankfort - horizontal is a reference plane constructed by joining which of the following landmarks? Select and explain. A. Nasion and Sella, B. Porion and Sella, C. Porion and Nasion, D. Porion and Orbitale",D: Porion and Orbitale,"Answer: D: Porion and Orbitale Explanation: None" "This is a real-world medical entrance exam question, please give the true answer based on the question and selection. Topic name: None, Subject name: Dental","Gum pads are divided into following segments: Select and explain. A. 3 in each quadrant, B. 2 in each jaw, C. 5 in each quadrant, D. 2 in each quadrant",C: 5 in each quadrant,"Answer: C: 5 in each quadrant Explanation: None" "This is a real-world medical entrance exam question, please give the true answer based on the question and selection. Topic name: None, Subject name: Dental","A youthful feminine smile have all the characteristics except? Select and explain. A. Golden proportion is followed, B. Closure of incisor embrasure, C. Rounding of incisal line angles, D. Softened facial line angles",B: Closure of incisor embrasure,"Answer: B: Closure of incisor embrasure Explanation: None" "This is a real-world medical entrance exam question, please give the true answer based on the question and selection. Topic name: None, Subject name: Physiology","Release of synaptic vesicle from presynaptic terminal is inhibited by Select and explain. A. Inhibition of conduction of nerve impulse, B. Prevention of sodium influx, C. Prevention of calcium influx, D. Prevention of depolarization of nerve terminal",C: Prevention of calcium influx,"Answer: C: Prevention of calcium influx Explanation: Answer- C (Prevention of calcium influx)-The release of the synaptic vesicles from the presynaptic terminal is inhibited by the prevention of Ca2+ influx." "This is a real-world medical entrance exam question, please give the true answer based on the question and selection. Topic name: None, Subject name: Social & Preventive Medicine","What will be the 95% confidence interval (CI) for an estimated prevalence of 10% and a sample size of 100? Select and explain. A. 18-Feb, B. 16-Apr, C. 13-Jul, D. Given data inadequate for calculation of class intervals.",B: 16-Apr,"Answer: B: 16-Apr Explanation: Ans: B. 4-16" "This is a real-world medical entrance exam question, please give the true answer based on the question and selection. Topic name: None, Subject name: Anatomy","Greatest movement is seen in which type of joint: Select and explain. A. Amphiarthroses., B. Diarthroses., C. Synarthroses., D. None.",B: Diarthroses.,"Answer: B: Diarthroses. Explanation: None" "This is a real-world medical entrance exam question, please give the true answer based on the question and selection. Topic name: None, Subject name: Anatomy","Extra-embryonic mesoderm is derived from: Select and explain. A. Epiblast, B. Primary yolk sac, C. Secondary yolk sac, D. Hypoblast",B: Primary yolk sac,"Answer: B: Primary yolk sac Explanation: Ans: B. Primary yolk sac(Ref Gray's. 41/e p169, 40/e p167, 173, 186-189; Langmuir S 13/e p45,'Pe p54-55)Extra-embryonic mesoderm is derived from primary yolk sac.Formed by delamination of yolk sae cells - later by migration of cells through primitive streak during gastrulation." "This is a real-world medical entrance exam question, please give the true answer based on the question and selection. Topic name: None, Subject name: Microbiology","Not true about vibrio cholera is Select and explain. A. Increase water and sodium loss by debilitating diarrhoea, B. Toxic protein is cholera toxin, C. Activate adenylatecyclase, D. Gram positive rod",D: Gram positive rod,"Answer: D: Gram positive rod Explanation: None" "This is a real-world medical entrance exam question, please give the true answer based on the question and selection. Topic name: None, Subject name: Dental","A 7-years girl with single tooth crossbite came to clinic what should be treatment plan: Select and explain. A. Wait till all permanent teeth erupt, B. Removable appliance therapy, C. Immediate intervention therapy, D. Extraction of tooth",C: Immediate intervention therapy,"Answer: C: Immediate intervention therapy Explanation: None" "This is a real-world medical entrance exam question, please give the true answer based on the question and selection. Topic name: None, Subject name: Dental","Most prominent lingual fossa in Incisors is seen in Select and explain. A. Upper Lateral incisor, B. Lower Lateral Incisor, C. Upper Central Incisor, D. Lower Central Incisor",A: Upper Lateral incisor,"Answer: A: Upper Lateral incisor Explanation: None" "This is a real-world medical entrance exam question, please give the true answer based on the question and selection. Topic name: None, Subject name: Anatomy","Finger with two dorsal interossei attached? Select and explain. A. Little, B. Index, C. Middle, D. Ring",C: Middle,"Answer: C: Middle Explanation: Ans.C. Middle1st: largest; lateral side of index finger2nd and 3rd: both attach to either side of 3rd (middle) finger4th: medial side of 4th (ring) finger" "This is a real-world medical entrance exam question, please give the true answer based on the question and selection. Topic name: None, Subject name: Dental","Most common malignant bone tumor in child: Select and explain. A. Osteosarcoma, B. Ewing sarcoma, C. Metastatic carcinoma, D. Osteoma",A: Osteosarcoma,"Answer: A: Osteosarcoma Explanation: None" "This is a real-world medical entrance exam question, please give the true answer based on the question and selection. Topic name: None, Subject name: Medicine","A 29-year-old, 4 months pregnant primigravida has history of juvenile myoclonic epilepsy. She has been regularly taking sodium valproate and now presents requesting for an opinion for a change in her anti-epileptic treatment. What would you sugeets her? Select and explain. A. Immediately taper off valproate and sta lamotrigine, B. Switch to carbamazepine, C. Continue valproate and serial drug monitoring in blood, D. Add lamotrigine to valproate",C: Continue valproate and serial drug monitoring in blood,"Answer: C: Continue valproate and serial drug monitoring in blood Explanation: Answer: c. Continue valproate and serial drug monitoring in blood (Ref Harrison I9/e p2544, 18/e p3269)Continue valproate with serial drug monitoring in the blood of the 4-month old patient having history of juvenile myoclonic epilepsy." "This is a real-world medical entrance exam question, please give the true answer based on the question and selection. Topic name: None, Subject name: Surgery","Which of the following would you consider ohostatic hypotension? Select and explain. A. SBP fall by 20 mm hg, DBP fall by 10 mm hg within 3 minutes, B. SBP fall by 20 mm hg, DBP fall by 10 mm Hg within 6 minutes, C. SBP fall by 30 mm hg, DBP fall by 20 mm Hg within 3 minutes, D. SBP fall by 30 mm hg, DBP fall by 20 mm hg within 6 minutes","A: SBP fall by 20 mm hg, DBP fall by 10 mm hg within 3 minutes","Answer: A: SBP fall by 20 mm hg, DBP fall by 10 mm hg within 3 minutes Explanation: Answer- A. SBP fall by 20 mm hg, DBP fall by 10 mm hg within 3 minutesOhostatic hypotension is defined as a decrease in systolic blood pressure of 20 mm Hg or a decrease in diastolic blood pressure of 10 mm Hg within three minutes of standing when compared with blood pressure from the sitting or supine position." "This is a real-world medical entrance exam question, please give the true answer based on the question and selection. Topic name: None, Subject name: Pediatrics","What is NOT a major criteria for Rheumatic heart disease – Select and explain. A. Arthritis, B. Carditis, C. Subcutaneous nodule, D. Raised ASLO titer",D: Raised ASLO titer,"Answer: D: Raised ASLO titer Explanation: None" "This is a real-world medical entrance exam question, please give the true answer based on the question and selection. Topic name: None, Subject name: Dental","Which of the following is not used as an antiflux? Select and explain. A. Graphite, B. Boric acid, C. Iron oxide, D. Calcium carbonate with alcohol",B: Boric acid,"Answer: B: Boric acid Explanation: The flow of the flux and hence the solder can be limited by the use of an antiflux, which should be applied to the surface before the flux is applied. Zinc oxides in alcohol, rouge (iron oxide) in chloroform, whiting (calcium carbonate) in alcohol and graphite suspension are the commonly used antifluxes. Key comment : The boric acid/borax acts as flux." "This is a real-world medical entrance exam question, please give the true answer based on the question and selection. Topic name: None, Subject name: Medicine","Cerebellar lesion can produce all of the following except: Select and explain. A. Nystagmus, B. Past pointing, C. Resting tremor, D. Ataxic gait",C: Resting tremor,"Answer: C: Resting tremor Explanation: Ans: C. Resting tremor(Ref Harrison 19/e p2618, I8/e p3238)Intention tremors - Cerebellar lesions.Resting tremors - Basal ganglia diseases (Parkinsonism).Parkinson disease:Characterized by resting tremor, essential tremor (ET) by a postural tremor (trying to sustain a posture).Cerebellar disease:Tremors by intention or kinetic tremor (on reaching to touch a target)." "This is a real-world medical entrance exam question, please give the true answer based on the question and selection. Topic name: None, Subject name: Medicine","Enzymes elevated in the plasma in MI Select and explain. A. CPK-MB, B. SGOT (AST), C. LDH, D. All of the above",D: All of the above,"Answer: D: All of the above Explanation: None" "This is a real-world medical entrance exam question, please give the true answer based on the question and selection. Topic name: None, Subject name: Gynaecology & Obstetrics","An 18 years old girl presents with primary amenorrhea. On evaluation, she was having a karyotype of 45X0 and infantile uterus. What should he done next? Select and explain. A. H to induce pubey, B. Vaginoplasty, C. Clitoroplasty, D. Bilateral gonadectomy",D: Bilateral gonadectomy,"Answer: D: Bilateral gonadectomy Explanation: Ans. d. Bilateral gonadectomyHistory of primary amenorrhea, karyotype of 45X0 & infantile uterus is suggestive of Turner's syndrome. Approximately 5% of women with Turner's syndrome have a karyotype with Y chromosome (45X/46XY). It is impoant to identify' a Y chromosome because affected individuals are at significant risk of gonadoblastoma (20 to 30%). Therefore, prophylactic gonadectomy should be performed." "This is a real-world medical entrance exam question, please give the true answer based on the question and selection. Topic name: None, Subject name: Anatomy","Palatoglossus develops from: Select and explain. A. Occipital myotomes, B. Lingual swelling of first arch, C. Third arch, D. Mesoderm of sixth arch",D: Mesoderm of sixth arch,"Answer: D: Mesoderm of sixth arch Explanation: Muscles of tongue develop from occipital myotomes, so the cranial nerve XII (hypoglossal nerve) supplies all intrinsic and three extrinsic muscles. Only palatoglossus is supplied by cranial root of accessory through pharyngeal plexus and is developed from mesoderm of sixth arch." "This is a real-world medical entrance exam question, please give the true answer based on the question and selection. Topic name: None, Subject name: Pharmacology","Benzodiazepine antagonist is Select and explain. A. Flumazenil, B. Furazolidone, C. Naloxone, D. Naltrexone",A: Flumazenil,"Answer: A: Flumazenil Explanation: None" "This is a real-world medical entrance exam question, please give the true answer based on the question and selection. Topic name: None, Subject name: Medicine","In which of the following tumor MRI is better than CT Scan.. Select and explain. A. Brochogenic Carcinoma, B. SCC of lung, C. Mesothelioma, D. Pancost tumor",D: Pancost tumor,"Answer: D: Pancost tumor Explanation: Answer- D. Pancost tumorMRI: An at uncovering the extent to which a tumor has invaded other structures" "This is a real-world medical entrance exam question, please give the true answer based on the question and selection. Topic name: None, Subject name: Physiology","Glucocorticoids decrease inflammatory reaction by reducing: Select and explain. A. Activity of histamine, B. Release of histamine, C. Fibroblastic activity, D. Neutrophils",B: Release of histamine,"Answer: B: Release of histamine Explanation: None" "This is a real-world medical entrance exam question, please give the true answer based on the question and selection. Topic name: None, Subject name: Anatomy","Inca bone or Goethe's ossicles is a sutural bone present in? Select and explain. A. Bregma, B. Lambda, C. Pterion, D. Asterion",B: Lambda,"Answer: B: Lambda Explanation: Wormian bones, also known as intra sutural bones, are extra bone pieces that occur within a suture in the cranium. These are irregular isolated bones that appear in addition to the usual centers of ossification of the cranium. An isolated wormian bone at the lambda is referred as an Inca bone or Goethe's ossicle. Inca bones are also named as interparietal bones as they lie in between the two limbs of the lambdoid suture of the parietal bones. Wormian bones are a marker for some diseases and important in the primary diagnosis of brittle bone disease: osteogenesis imperfecta. Wormian bones may also be seen in: Pycnodysostosis Osteogenesis imperfecta Rickets ""Kinky-hair: Menke's syndrome Cleidocranial dysostosis Hypoparathyroidism and hypophosphatasia Otopalatodigital syndrome Primary acro-osteolysis Down syndrome" "This is a real-world medical entrance exam question, please give the true answer based on the question and selection. Topic name: AIIMS 2018, Subject name: Pathology","Anti-centromere antibodies are seen in which of the following conditions? Select and explain. A. Drug-induced lupus, B. SLE, C. Sjogren syndrome, D. Scleroderma",D: Scleroderma,"Answer: D: Scleroderma Explanation: It is seen in Systemic Sclerosis (Scleroderma). Limited form of scleroderma known as CREST syndrome is characterised by presence of anti-centromere antibodies positive and generalized form shows anti-topoisomerase antibodies Anti-histone antibodies are positive in drug-induced lupus. Anti-SS-A (RO) antibodies and anti-SS-B(LA) antibodies are positive in Sjogren's Syndrome. Most specific in SLE: anti ds-DNA and Anti-Smith antibodies" "This is a real-world medical entrance exam question, please give the true answer based on the question and selection. Topic name: None, Subject name: Pediatrics","You have been called to declare a brain dead l2-year-ord chitd in pICU, all ofthe given are signs of brain death except? Select and explain. A. Normal BP without pharmacological suppo, B. Positive spinal reflexes on stimulation, C. Sweating and tachycardia, D. Decoicate and decerebrate posturing",D: Decoicate and decerebrate posturing,"Answer: D: Decoicate and decerebrate posturing Explanation: Ans: D. Decoicate and decerebrate posturingRef Dhanwate AD. Brainstem death: A comprehensive review in Indian perspective' [ndian Journal o/ Critical Care Medicine : Peer-reviev,ed, o.fficiat pul.tlicalion of Indian Society of Critical Care Medicine. 2014;18(9):596-605. doi:10.4103/0972-5229.140151. Goila AK, Pawar M. The diagnosis of brain death. Indian Jrurnal of Critical Care Medicine; Peer-reviewed, o.fficial publication of Indian Society of Critical Corc Madicine. 2009;I 3(I ):7-11. doi: I 0.4103/0972-5229.53 I0t.Diagnosis of brain death in India:Who should diagnose:Team of four medical expes includingMedical Administrator In charge of the hospital.AuthorizedSpecialistAuthorized Neurologist/Neuro-SurgeonMedical Officer treating the patient." "This is a real-world medical entrance exam question, please give the true answer based on the question and selection. Topic name: None, Subject name: Anatomy","All of the following are derived from mesonephros except: Select and explain. A. Paroophoron, B. Vas deferens, C. Epididymis, D. Glomerulus",D: Glomerulus,"Answer: D: Glomerulus Explanation: Ans: D. GlomerulusMesonephric / Wolffian duct (Main genital duct of males):I. Structure formed: ""PUT A DEEP semen""Posterior wall of prostate urethra, cranial to openings of ejaculatory duct.Ureteric buds forming ureter, pelvis, calyces & collecting tubule.Trigone of bladder.Appendix of epididymis (not testis).Ductus deferens.Epididymis.Ejaculatory ducts.Prostate (mesodermal pa).Seminal vesicles.II. Remnants: ""SIP"":Superior aberrant ductile (epigenital tubule)Inferior aberrant ductile.Paradidymis (para-genital tubule)." "This is a real-world medical entrance exam question, please give the true answer based on the question and selection. Topic name: None, Subject name: Physiology","A phenomenon by which a sustance appears different under varying conditions of colour is called as? Select and explain. A. Mesomerism, B. Metamerism, C. Fluorescence, D. Refractive index",B: Metamerism,"Answer: B: Metamerism Explanation: Metamerism: Objects that appear to be color matched, under one type of light may appear different when viewed under another source of light. This phenomenon is called as metamerism. if possible the color matching, therefore, should be done two different sources of light, one of them preferably should be daylight. Phenomenon in which porcelain appears different under varying light condition is metamerism Fluorescence: Natural tooth sometimes absorbsthe light of a shorter wavelength in addition to the wavelengths of the normal visible light. The energy the tooth absorbs by ultraviolet radiation is converted into the visible light of larger wavelengths. In such cases, the tooth itself becomes a source of light and this phenomenon is called fluorescence. Fluorescence contributes to the brightness and a vital appearance of the tooth." "This is a real-world medical entrance exam question, please give the true answer based on the question and selection. Topic name: None, Subject name: Dental","Which of the following does not occur in High Copper Amalgam? Select and explain. A. Electrochemical Corrosion, B. Chemical Corrosion, C. Penetrating Corrosion, D. Corrosion does not occur at all",C: Penetrating Corrosion,"Answer: C: Penetrating Corrosion Explanation: None" "This is a real-world medical entrance exam question, please give the true answer based on the question and selection. Topic name: None, Subject name: Anatomy","Tendon of Gracilis, Sartorius and semitendenous muscles insert on tibia to form Pesanserinus. Similar structure is seen in Select and explain. A. TMJ, B. Parotid, C. Submandibular gland, D. Cheek",B: Parotid,"Answer: B: Parotid Explanation: None" "This is a real-world medical entrance exam question, please give the true answer based on the question and selection. Topic name: None, Subject name: Dental","Technique of root coverage called as: Select and explain. A. Langer and langer technique, B. Pouch and tunnel technique., C. Azis technique., D. Bruno technique",B: Pouch and tunnel technique.,"Answer: B: Pouch and tunnel technique. Explanation: None" "This is a real-world medical entrance exam question, please give the true answer based on the question and selection. Topic name: None, Subject name: Gynaecology & Obstetrics","All of these can be used for post-coital contraception except: Select and explain. A. Desogestrel, B. Copper-T, C. Levonorgestrel, D. OCP",A: Desogestrel,"Answer: A: Desogestrel Explanation: Ans: A. Desogestrel(Ref Williams 24/e p714).Desogestrel is not used as post-coital contraceptive.Drugs used for Emergency ContraceptionDrugDoseLevonorgestrel0.75 mg stat and after 12 hoursdegEthinyl estradiol 50 pg + Norgestrel 0.25 mg2 tab stat and 2 after 12 hoursdegConjugated estrogen15 mg BD x 5 daysEthinyl estradiol2.5 mg BD x 5 daysMifepristoneInseion of an IUCD within maximum period of 5-7 days after accidental unprotected exposure.It prevents implantation but is not suitable for women with multiple sex paners and for rape victimsCentchroman2 tablets (60 mg) to be taken twice at an in interval of 12 hours within 24 hours of intercourse.UlipristalInseion of an IUCD within maximum period of 5-7 days after accidental unprotected exposure.It prevents implantation but is not suitable for women with multiple sex paners and for rape victims." "This is a real-world medical entrance exam question, please give the true answer based on the question and selection. Topic name: None, Subject name: Dental","Which of the following is known as corner stone of behavior management: Select and explain. A. Modelling., B. Tell Show Do., C. Contigency management., D. Communication.",B: Tell Show Do.,"Answer: B: Tell Show Do. Explanation: None" "This is a real-world medical entrance exam question, please give the true answer based on the question and selection. Topic name: None, Subject name: Surgery","Medial dislocation of fractured condyle in subcondylar fracture is caused by Select and explain. A. Lateral pterygoid, B. Medial pterygoid, C. Masseter, D. Temporalis",A: Lateral pterygoid,"Answer: A: Lateral pterygoid Explanation: None" "This is a real-world medical entrance exam question, please give the true answer based on the question and selection. Topic name: None, Subject name: Medicine","Unstable Angina, True is Select and explain. A. Chest pain occurs at rest., B. Pain lasts for 10 minutes., C. Pain subsides on rest., D. All of the above.",A: Chest pain occurs at rest.,"Answer: A: Chest pain occurs at rest. Explanation: Unstable Angina Chest Pain at rest Pain > 10 min Pain at rest." "This is a real-world medical entrance exam question, please give the true answer based on the question and selection. Topic name: None, Subject name: Medicine","A known 60-year-old male Diabetic and hypeensive patient was found unconscious in the morning. On examination pulse- 120, bp- 180/100 and there was positive extensor plantar. What is next step to be done? Select and explain. A. Order CT scan, B. Immediately reduces bp with antihypeensives, C. Give I/v mannitol, D. Check blood glucose",D: Check blood glucose,"Answer: D: Check blood glucose Explanation: Ans: D. Check blood glucoseQuick assessment of Blood sugar should be done to rule out hypo or hyperglycemia is needed.In hypoglycemic coma, the eyeball tension is normal, there is profuse sweating and the plantar responses are extensor always either normal or subnormal in uncomplicated cases of either condition.It is well to remember that although a comatose patient is a known diabetic, the diagnosis is not always one of 'hyper- or hypo-glycaemia." "This is a real-world medical entrance exam question, please give the true answer based on the question and selection. Topic name: None, Subject name: Dental","Which of the following movement(s) is /are involved in the opening of the mouth? Select and explain. A. Only hinge (rotation), B. Only translation, C. Hinge followed by translation, D. Translation followed by hinge",C: Hinge followed by translation,"Answer: C: Hinge followed by translation Explanation: None" "This is a real-world medical entrance exam question, please give the true answer based on the question and selection. Topic name: None, Subject name: Dental","Dental plaque is formed by reaction of enzymes on: Select and explain. A. Sucrose and lipids, B. Glucose and proteins, C. Glucose and lipids, D. Sucrose and saliva",D: Sucrose and saliva,"Answer: D: Sucrose and saliva Explanation: None" "This is a real-world medical entrance exam question, please give the true answer based on the question and selection. Topic name: AIIMS 2018, Subject name: Gynaecology & Obstetrics","A 76 - year - old female presented with non - healing ulcer on labia majora for 6 months measuring 2 x 3 cm with no palpable lymphadenopathy. Biopsy shows Squamous cell carcinoma. Management of this patient includes? Select and explain. A. Radical vulvectomy with sentinal LN biopsy, B. Wide excision, C. Simple vulvectomy, D. Chemoradiation with resection",A: Radical vulvectomy with sentinal LN biopsy,"Answer: A: Radical vulvectomy with sentinal LN biopsy Explanation: Non- Healing Ulcer, 2x3 cm, no palpable lymphadenopathy concludes to stage Ib of Ca Vulva. Staging of Ca vulva Stage I Limited to vulva IA IB Size - < 2 cm, Invasion - < 1 mm Size - < 2 cm, Invasion - > 1 mm Stage II Adiacent organ femoral LN involvement Lower 1/3rdvagina Lower 1/3rdof urethra, anus Stage III Inguinal femoral LN involvement IIIAi Aii One LN - > 5mm One or Two LN - < 5m IIIBi Bii Two LN - > 5mm More than 3 LN - < 5mm III C LN involvement ?, with extra capsular spread Stage IV IV Ai Aii Upper urethra, upper vagina, rectal involvement Growth stuck to pelvic bone Fixed or ulcerated LN IV B Distant metastasis Pelvic LN Treatment: Stage IA- Wide excision Stage I & II- Radical vulvectomy with sentinal LN biopsy If negative- Radical vulvectomy alone If positive- Radical vulvectomy + LN removal Stage III & IV- Chemoradiation with resection" "This is a real-world medical entrance exam question, please give the true answer based on the question and selection. Topic name: None, Subject name: Dental","The most appropriate method of testing the statistical significance of the differences between means of two groups is Select and explain. A. Chi square test, B. Multiple regression analysis, C. Correlation coefficient analysis, D. Student test",D: Student test,"Answer: D: Student test Explanation: None" "This is a real-world medical entrance exam question, please give the true answer based on the question and selection. Topic name: AIIMS 2019, Subject name: Surgery","A 4 years old child was brought to the hospital with right impalpable testsis. During diagnostic laparoscopy for undescended testis, there are blind testicular vessels. What should be done next? Select and explain. A. Abdominal exploration, B. Nothing is to be done, C. Inguinal exploration, D. Scrotal exploration",B: Nothing is to be done,"Answer: B: Nothing is to be done Explanation: Undescended test (UDT) Palpable UDT Inguinal Exploration Non- Palpable UDT Diagnostic Laparoscopy" "This is a real-world medical entrance exam question, please give the true answer based on the question and selection. Topic name: None, Subject name: Biochemistry","A lady presented with fatigue and tingling sensation in both hands and legs. On examination, she is found to have a fissured red tongue, lesions at angle of mouth and peripheral neuropathy with a decreased RBC glutathione reductase activity. What is the likely deficient vitamin? Select and explain. A. Riboflavin, B. Vitamin B12, C. Thiamine, D. Vitamin B6",A: Riboflavin,"Answer: A: Riboflavin Explanation: Answer- A. RiboflavinClinical Manifestations of Riboflavin deficiency:Cheilosis, glossitis, lingual desquamation seborrheic dermatitisKeratitis, conjunctivitis, corneal vascularisation" "This is a real-world medical entrance exam question, please give the true answer based on the question and selection. Topic name: None, Subject name: Dental","Minimum dosage of tetracycline which will show tooth discoloration is Select and explain. A. 5 mg/kg body weight, B. 20 mg/kg body weight, C. 50 mg/kg body weight, D. 80 mg/kg body weight",B: 20 mg/kg body weight,"Answer: B: 20 mg/kg body weight Explanation: None" "This is a real-world medical entrance exam question, please give the true answer based on the question and selection. Topic name: None, Subject name: Social & Preventive Medicine","Mass Drug Administration is helpful is used/effective for all except Select and explain. A. Lymphatic Filariasis, B. Vit A Deficiency, C. Worm Infestation, D. Scabies",D: Scabies,"Answer: D: Scabies Explanation: Answer. D. Scabies * Mass chemoprophylaxis, i.e, chemoprophylaxis given to a large number of people, is recommended in yaws, pinta, bejel, trachoma,Vit A Deficiency, Worm Infestation, malaria and filaria." "This is a real-world medical entrance exam question, please give the true answer based on the question and selection. Topic name: None, Subject name: Forensic Medicine","A person of eonism derives pleasure from. Select and explain. A. Wearing clothes of opposite sex, B. Fondling female body pas, C. Rubbing genitalia against body of other person, D. Seeing the opposite paner nude",A: Wearing clothes of opposite sex,"Answer: A: Wearing clothes of opposite sex Explanation: Ans. a. Wearing clothes of opposite sexIt is a person whose personality is dominated by the desire to be identified with the opposite sex It is usuallyfound in males who obtain sexual pleasure by wearingfemale dress." "This is a real-world medical entrance exam question, please give the true answer based on the question and selection. Topic name: None, Subject name: Dental","Normal growth of maxilla occurs by Select and explain. A. Displacement and drift, B. Drift only, C. Apposition, D. Replacement resorption",A: Displacement and drift,"Answer: A: Displacement and drift Explanation: Growth of the nasomaxillary complex: The nasomaxillary complex consists of the bones and cartilages of the nose and maxilla. Growth of the nasomaxillary complex occurs by primary growth, secondary displacement and surface remodelling. The maxilla displaces downward and forward due to sutural growth at its posterior margins, while surface resorptive changes lead to deepening of its facial surface. Drift is brought about by deposition on one side and resorption on the opposite side of the same cortical plate. The classical example of cortical drift is provided by growth of the facial surface of the maxilla." "This is a real-world medical entrance exam question, please give the true answer based on the question and selection. Topic name: AIIMS 2017, Subject name: Orthopaedics","All are true about PCL except? Select and explain. A. It's a extra synol structure, B. Primary restrain for post dislocation of knee joint, C. Primary restrain for internal rotation around knee joint, D. Attached to medical femoral condyle",C: Primary restrain for internal rotation around knee joint,"Answer: C: Primary restrain for internal rotation around knee joint Explanation: * Both ACL and PCL are intra-capsular, extra synol structures.* PCL prevents posterior dislocation of knee joint* PCL is attached to medial femoral condyle * It is a restraint for external rotation of the leg hence, PCL is tested by external rotation on Dial test" "This is a real-world medical entrance exam question, please give the true answer based on the question and selection. Topic name: None, Subject name: Social & Preventive Medicine","The strength of association between the risk factor and disease is measured by Select and explain. A. Attributable risk, B. Absolute risk of the variable, C. Odds ratio/Relative risk, D. P-value",C: Odds ratio/Relative risk,"Answer: C: Odds ratio/Relative risk Explanation: Ans: C. Odds ratio/Relative riskThe strength of association between the risk factor and disease is measured by the Odds ratio/Relative risk.The estimation of disease risk associated with exposure is obtained by an index known as relative risk (RR) or risk ratio, which is defined as the ratio between the incidence of disease among exposed persons and incidence among non-exposed.Odds ratio (Cross product ratio):From a case-control study, we can derive what is known as the odds ratio (OR) which is a measure of the strength of the association between risk factor and outcome.Odds ratio is closely related to relative risk." "This is a real-world medical entrance exam question, please give the true answer based on the question and selection. Topic name: None, Subject name: Social & Preventive Medicine","A researcher wants to (to a study of blood levels of lipids among people who smoke and those who do not. But he is now concerned that the smokers might differ from non-smokers in their diet, exercise, etc as well. This concern is known as: Select and explain. A. Recall bias, B. Information bias, C. Selection bias, D. Interviewer bias",C: Selection bias,"Answer: C: Selection bias Explanation: Ans: C. Selection bias(Ref Park 24/e p78, 88, 23/e p73, 22/e p71; en.wikipedia.org/wiki/Bias_(statistics); http://sphweb. hume.hu.echdollt/mph-modules)Selection bias:Groups to be compared are differentially susceptible to outcome of interest, even before experimental maneuver is performed.Also referred as ""Susceptibility bias"".In the given question, researcher is concerned that the smokers might differ from non-smokers in their diet, exercise, etc.Hence, this concern is known as selection bias." "This is a real-world medical entrance exam question, please give the true answer based on the question and selection. Topic name: None, Subject name: Gynaecology & Obstetrics","All of the following are components of manning score/Biophysical score except: Select and explain. A. Non stress test, B. Oxytocin challenge test, C. Fetal body movement, D. Respiratory activity of child",B: Oxytocin challenge test,"Answer: B: Oxytocin challenge test Explanation: None" "This is a real-world medical entrance exam question, please give the true answer based on the question and selection. Topic name: None, Subject name: Pathology","Carbamazepine has been utilized to successfully diminish attacks in trigeminal neuralgia. During this therapy, which of the following is indicated? Select and explain. A. Clinical observation only, B. Clinical observation and complete blood and platelet counts prior to and at frequent intervals during therapy, C. No monitoring, D. Complete blood investigation, only if adverse symptoms arise",B: Clinical observation and complete blood and platelet counts prior to and at frequent intervals during therapy,"Answer: B: Clinical observation and complete blood and platelet counts prior to and at frequent intervals during therapy Explanation: Aplastic anemia and leucopenia are rare complications of carbamazepine therapy. So, the patients receiving carbamazepine must have periodic hematological laboratory investigations." "This is a real-world medical entrance exam question, please give the true answer based on the question and selection. Topic name: None, Subject name: ENT","Patient underwent surgery at the lateral pa of the skull. Postoperatively patient had aspirations without voice change Select and explain. A. Vagus, B. RLN, C. Glossopharyngeal, D. SLN",D: SLN,"Answer: D: SLN Explanation: Ans. D (Superior Laryngeal nerve)* Vagus, RLN involvement leads to hoarseness of the voice* SLN divide into internal and external laryngeal* External laryngeal nerve supplies only one muscle- cricothyroid - tensor* Internal laryngeal nerve- safety nerve of larynx- prevents aspiration and supplies supraglottic pa of larynx." "This is a real-world medical entrance exam question, please give the true answer based on the question and selection. Topic name: None, Subject name: Dental","Touidine blue is to: Select and explain. A. Differentiate between malignant transformation, B. Differentiate lichen planus from Leukoplakia, C. Differentiate between pemphigus and lichen planus, D. Differentiate between candidiasis & leukoplakia",A: Differentiate between malignant transformation,"Answer: A: Differentiate between malignant transformation Explanation: None" "This is a real-world medical entrance exam question, please give the true answer based on the question and selection. Topic name: None, Subject name: Radiology","Investigation of choice for a lesion of the temporal bone is Select and explain. A. X-ray, B. USG, C. CT, D. MRI",C: CT,"Answer: C: CT Explanation: None" "This is a real-world medical entrance exam question, please give the true answer based on the question and selection. Topic name: AIIMS 2018, Subject name: Anatomy","Ligamentum flavum consists of which fibres:- Select and explain. A. Type-I collagen, B. Type-II collagen, C. Reticular, D. Elastic",D: Elastic,"Answer: D: Elastic Explanation: Ligamentum flavum pass b/w the laminae of adjacent veebrae. They consist of predominantly elastic tissue form pa of the posterior surface of the veebral canal. Function: Resist separation of the laminae in flexion & assist in extension back to anatomical position. ." "This is a real-world medical entrance exam question, please give the true answer based on the question and selection. Topic name: None, Subject name: Pathology","Acanthosis is Select and explain. A. Increase in mitotic division, B. Increase in thickness of superficial layer, C. Increase in thickness of spinous Layer, D. Disruption of basal lamina",C: Increase in thickness of spinous Layer,"Answer: C: Increase in thickness of spinous Layer Explanation: None" "This is a real-world medical entrance exam question, please give the true answer based on the question and selection. Topic name: None, Subject name: Gynaecology & Obstetrics","A G3P2, pregnant comes to your clinic at 18 weeks of gestation for genetic counselling. She has a history of two kids born with thalassemia major. Which test would you recommend now? Select and explain. A. Amniocentesis, B. Chorionic villus sampling, C. Cordocentesis, D. Non-invasive prenatal testing",C: Cordocentesis,"Answer: C: Cordocentesis Explanation: Ans: C. Cordocentesis(Ref Williams 24/e p300; (Thai 8/e p341-344)Recommended test - Cordocentesis.As patient is presenting at 18 weeks - Quick method diagnosing thalassemia antenatally needed.Note:Legal age of aboion is only till 20 weeks.Fetal blood karyotyping accomplished within 24 to 48 hours.Significantly quicker than (7- to 10-day turnaround time with amniocentesis or CVS)." "This is a real-world medical entrance exam question, please give the true answer based on the question and selection. Topic name: None, Subject name: Surgery","Excision of the submandibular gland for calculus or tumours is done by incision below angle of the jaw. Special care should be taken to avoid which nerve? Select and explain. A. Ansa cervicalis, B. Mandibular branch of facial nerve, C. Posterior auricular nerve, D. Submandibular ganglion",B: Mandibular branch of facial nerve,"Answer: B: Mandibular branch of facial nerve Explanation: None" "This is a real-world medical entrance exam question, please give the true answer based on the question and selection. Topic name: None, Subject name: Physiology","The most common problem with blood transfusion is: Select and explain. A. Hemolytic reaction, B. Remission of disease, C. Hypokalemia, D. Metabolic acidosis",A: Hemolytic reaction,"Answer: A: Hemolytic reaction Explanation: None" "This is a real-world medical entrance exam question, please give the true answer based on the question and selection. Topic name: None, Subject name: Surgery","A man has 1x1.5cm pedunculated lesion on the soft palate which has a rough, ""warty"" surface but is the same colour as adjacent mucosa. Appropriate management of this lesion is to: Select and explain. A. Perform an incisional biopsy, B. Perform excisional biopsy, C. Scrape for exfoliative cytology, D. Observe for two weeks",B: Perform excisional biopsy,"Answer: B: Perform excisional biopsy Explanation: None" "This is a real-world medical entrance exam question, please give the true answer based on the question and selection. Topic name: None, Subject name: Pathology","A lady complains of headache, nausea and tenderness in temporal region with migraine. On microscopic investigation what will be seen Select and explain. A. Temporal aneurysm, B. Giant cell arteritis, C. Granulomatous giant cell lesions, D. Luminal thrombosis",B: Giant cell arteritis,"Answer: B: Giant cell arteritis Explanation: None" "This is a real-world medical entrance exam question, please give the true answer based on the question and selection. Topic name: None, Subject name: Dental","Chlorhexidine mouth rinsing: Select and explain. A. Specifically inhibits gram-negative bacteria, B. Can inhibit subgingival plaque, C. May disturb taste sensation, D. Can only be obtained on prescription",C: May disturb taste sensation,"Answer: C: May disturb taste sensation Explanation: None" "This is a real-world medical entrance exam question, please give the true answer based on the question and selection. Topic name: AIIMS 2017, Subject name: Forensic Medicine","Black foot is seen in which poisoning? Select and explain. A. Arsenic, B. Lead, C. Mercury, D. Phosphorus",A: Arsenic,"Answer: A: Arsenic Explanation: Black foot disease is caused by arsenic poisoning since it causes thrombosis of vessels and may cause peripheral gangrene. Mnemonic Arsenic poisoning A - Anemia / Aldrich Mee's line / Arsenophagist (Tolerate upto 300mg ) R - Rain drop pigmentation / Reinsch test / Red velvety mucosa S - Sub endocardial hemorrhages / Sensory neuropathy E - Eruptions N - NAA I - Imbibition of arsenic ( Arsenic imbibed from surrounding soils after death ) / before treated iron oxide C - Cumulative poison / Cholera like symptom / Chelation for treatment" "This is a real-world medical entrance exam question, please give the true answer based on the question and selection. Topic name: None, Subject name: Surgery","All the following are true about imaging in primary survey of a trauma patient except: Select and explain. A. Cervical X-ray is not mandatory, B. Chest X-ray and pelvic X-ray are taken as a pa of primary survey, C. Hemodynamically unstable patients should not be sent for CT scan, D. All patients should have chest X-ray-PA view only",D: All patients should have chest X-ray-PA view only,"Answer: D: All patients should have chest X-ray-PA view only Explanation: Answer- D. All patients should have chest X-ray-PA view onlyChest X-ray and X-ray pelvis are taken in AP view in the adjuncts to the primary survey of a trauma patient.The steps in the ATLS (Advanced Trauma Life Suppo):Primary survey with simultaneous resuscitation - identify and treat what is killing the patientSecondary survey - proceed to identify all other injuriesDefinitive care - develop a definitive management planABCDE of trauma care:Airway assessment and protection (maintain cervical spine stabilization when appropriate)Breathing and ventilation assessment (maintain adequate oxygenation)Circulation assessment (control hemorrhage and maintain adequate end-organ perfusion)Disability assessment (perform basic neurologic evaluation)Exposure, with environmental control (undress patient and search everywhere for possible injury, while preventing hypothermia)Adjuncts to the primary survey:Blood - FBC, urea and electrolytes, clotting screen, glucose, toxicology, cross-matchECGTwo wide-bore 18G cannulae for intravenous fluidsUrinary and gastric cathetersRadiographs of the cervical spine and chestLateral cervical spine, chest AP view, and pelvis AP view are done when necessary.Other more specialized forms of imaging, such as ultrasound, computerized tomography, angiography, and diagnostic peritoneal lavage, should be considered after the secondary survey and only when the patient is stable after initial resuscitation." "This is a real-world medical entrance exam question, please give the true answer based on the question and selection. Topic name: None, Subject name: Pathology","Radiographs of a 40 year old female revealed radiolucent areas around several of her mandibular teeth, all which tested vital on the electric pulp tester. These areas represent Select and explain. A. Multiple granulomas, B. Periapical osteofibroses, C. Chronic periapical abscesses, D. Bone hypoplasias associated with opalescent teeth",B: Periapical osteofibroses,"Answer: B: Periapical osteofibroses Explanation: None" "This is a real-world medical entrance exam question, please give the true answer based on the question and selection. Topic name: None, Subject name: Dental","Tooth brush abrasions are more common on the Select and explain. A. Mandibular teeth, on the right half of the arch, B. Mandibular teeth, on the left half of the arch, C. Maxillary teeth, on the right half of the arch, D. Maxillary teeth, on the left half of the arch","D: Maxillary teeth, on the left half of the arch","Answer: D: Maxillary teeth, on the left half of the arch Explanation: None" "This is a real-world medical entrance exam question, please give the true answer based on the question and selection. Topic name: None, Subject name: Anaesthesia","A patient is undergoing MRND for laryngeal malignancy; while dissecting the venous tributaries the surgeon elevated the internal jugular vein for ligation. Suddenly the patients EtCO2 dropped from 3g mmHg to 12 mmHg and the patient developed hypotension along with cardiac arrhythmia. Which of the following is most likely cause?? Select and explain. A. Sympathetic overactivity, B. Vagal stimulation, C. Venous air embolism, D. Carotid body stimulation",C: Venous air embolism,"Answer: C: Venous air embolism Explanation: Ans: C: Venous air embolismRef Millers anesthesia &h ed-, pg. 2170.Diagnosis standard of care is precordial doppler (left or right parastemal, between 2nd and 3rd ribs) + ETC02 monitoring although this is not the most sensitive test - TEE is most sensitive.Pulmonary aery pressure will rise, and CO2 will fall alter VAE." "This is a real-world medical entrance exam question, please give the true answer based on the question and selection. Topic name: None, Subject name: Dental","In amalgam filling there is a wide buccolingual space, the wedge technique used should be (OR) If you are going to restore lower molar with amalgam which is having cavity with proximal box wide buccolingually which wedging technique will you use Select and explain. A. Double wedge, B. Wedge wedge, C. Single wedge, D. No wedge",A: Double wedge,"Answer: A: Double wedge Explanation: None" "This is a real-world medical entrance exam question, please give the true answer based on the question and selection. Topic name: None, Subject name: Surgery","Hypertonic salt solutions are used in all, except Select and explain. A. Burn, B. Brain injury patient, C. Prolonged bowel surgery, D. Cardiac type IV shock",D: Cardiac type IV shock,"Answer: D: Cardiac type IV shock Explanation: None" "This is a real-world medical entrance exam question, please give the true answer based on the question and selection. Topic name: None, Subject name: Orthopaedics","Flexion, adduction and internal rotation is characteristic posture in - Select and explain. A. Anterior dislocation of hip joint, B. Posterior dislocation of hip joint, C. Fracture of femoral head, D. Fracture shaft of femur",B: Posterior dislocation of hip joint,"Answer: B: Posterior dislocation of hip joint Explanation: None" "This is a real-world medical entrance exam question, please give the true answer based on the question and selection. Topic name: None, Subject name: Dental","Ratio of upper anterior facial height to lower anterior facial height is: Select and explain. A. 55:45:00, B. 60:40:00, C. 40:60, D. 45:55:00",D: 45:55:00,"Answer: D: 45:55:00 Explanation: The ratio of upper anterior facial height and lower anterior facial height ratio is 45:55. It decreases with age. N-Sn occupying 45% and  Sn-Gn 55% of the total height." "This is a real-world medical entrance exam question, please give the true answer based on the question and selection. Topic name: None, Subject name: Dental","Saliva which is formed in salivary glands, when passes from salivary gland to duct orifices, it undergoes numerous ion exchanges and as a result saliva become ____________ as compared to plasma: Select and explain. A. Hypertonic sometimes, B. Hypotonic, C. Isotonic, D. Hypertonic always",B: Hypotonic,"Answer: B: Hypotonic Explanation: None" "This is a real-world medical entrance exam question, please give the true answer based on the question and selection. Topic name: None, Subject name: Anatomy","Palatine process began to fuse at Select and explain. A. 10 weeks post fertilisation, B. 8 weeks post fertilisation, C. 6 weeks post fertilisation, D. 12 weeks post fertilisation",A: 10 weeks post fertilisation,"Answer: A: 10 weeks post fertilisation Explanation: None" "This is a real-world medical entrance exam question, please give the true answer based on the question and selection. Topic name: None, Subject name: Dental","Mandibular Molar is Select and explain. A. Tritubercular teeth, B. Quadritubercular teeth, C. Haplodont, D. Triconodont",B: Quadritubercular teeth,"Answer: B: Quadritubercular teeth Explanation: None" "This is a real-world medical entrance exam question, please give the true answer based on the question and selection. Topic name: None, Subject name: Biochemistry","Which of the following statements about tRNA molecules is false? Select and explain. A. The enzyme that attaches an amino acid to a tRNA is aminoacyl-tRNA synthetase, B. Although composed of a single strand of RNA, each molecule contains several sho, double-helical regions, C. The amino acid attachment is always to 3' end of the molecule., D. There is at least one tRNA for each of the 2 amino acids",D: There is at least one tRNA for each of the 2 amino acids,"Answer: D: There is at least one tRNA for each of the 2 amino acids Explanation: Answer-D. There is at least one tRNA for each of the 2 amino acids* There is at least one tRNA for each of the 2 amino acids RNA which transfer amino acid from the cytoplasm to the ribosomal protein synthesizing machinery* Clover leaf shape in the secondary structure.* L-shaped teiary structure, Single tRNA contains 74-95 nucleotides.* Cytoplasmic translation system Possess 31 tRNA species* Mitochondrial system Possess 22 tRNAs'Contain significant propoion of nucleosides with unusual bases.* Dihydro uridine (contain Dihydrouracil)* Pseudouridine* Inosine (contain Hypoxanthine)* Ribothymidine" "This is a real-world medical entrance exam question, please give the true answer based on the question and selection. Topic name: None, Subject name: Forensic Medicine","20-year-old female had abdominal pain previous week, USG and clinically normal. Again she presented with loss of vision, examination is normal. Diagnosis is:AIIMS 13 Select and explain. A. Malingering, B. Masochist, C. Hypochondriac, D. Anxiety disorder",A: Malingering,"Answer: A: Malingering Explanation: Ans. MalingeringMalingering is fabricating or exaggerating the symptoms of mental or physical disorders for a variety of 'secondarygain' motives, which may include financial compensation (often tied to fraud); avoiding school, work or military service;obtaining drugs; getting lighter criminal sentences; or simply to attract attention or sympathy." "This is a real-world medical entrance exam question, please give the true answer based on the question and selection. Topic name: None, Subject name: Pediatrics","An 8–day old breast–fed baby presents with vomiting, poor feeding and loose stools. On examination the heart rate is 190/minute, blood pressure 50/30 mmHg, respiratory rate 72 breaths/minute and capillary refill time of 4 seconds. Investigations show hemoglobin level of 15 g/dl. Na 120 mEq/l, K 6.8 mEq/l, Cl 81 meq/l, bicarbonate 15 mEq/l, urea 30 mg/dl and creatinine 0.6 mg/dl. the most likely diagnosis is – Select and explain. A. Congenital adrenal hyperplasia, B. Acute tubular necrosis, C. Congenital hypertrophic pyloric stenosis, D. Galactosemia",B: Acute tubular necrosis,"Answer: B: Acute tubular necrosis Explanation: Both the serum creatinine and serum urea are abnormal in this neonate. They may be normal for an adult, but for an 8 day old infant they are elevated. First the serum creatinine level   The serum creatinine level is high at birth reflecting the maternal value but it falls rapidly to 0.4 mg/dl by the end of the first week. According to Nelson, creatinine level in various age groups are Cord blood     ______ >      0.6 —1.2 mg/di New born _______ >       0.3 — 1.0 mg/dl Infant       ______ >                           0.2 - 0.4 mg/di Child       ______ >                           0.3 - 0.7 mg/di Adolescent ______ >       0.5 — 1.0 mg/di Do not get confused by the creatinine level of the newborn i.e., 0.3 — 1.0 mg/dl. This high level reflects the maternal value and it comes down to 0.2 to 0.4 mg/dl by 5th day So, Creatinine level of 0.6 mg/dl is abnormally high in an 8 day old infant. Now, the serum urea level The serum urea level in cord blood is 21-40 mg/dl, but it falls rapidly to 3-12 mg/dl by the 4th or 5th day. So urea level of 30 mg/dl is abnormally high in an 8 day old infant. According to Nelson Serum urea level in various age group Cord blood   ______ >                     21-40 ing/dl Premature      ______ >                    3-25 mg/dl Newborn   ______ >                         3-12 mg/dl Infant/child _____ >       5-18 ing/d1 Now, the capillary refill time Capillary refill time is also prolonged here (N ---> < 3 seconds) Prolonged capillary refill time indicates loss offluid and indicates shock, heart failure, Sympathetic stimulation In acute renal failure due to fluid loss cappillary refill time is increased All these findings plus characteristic h/o vomiting, poor feeding and loose stools confirms the diagnosis of acute renal failure (acute tubular necrosis) Electrolyte abnormalities in ARF --->             1) Hyponatremia,        2) Hyperkalemia,       3) Loss of Bicarbonate About other options In other three options serum urea and creatinine level will be normal." "This is a real-world medical entrance exam question, please give the true answer based on the question and selection. Topic name: None, Subject name: Gynaecology & Obstetrics","Which of the following can be used to delay preterm contractions of uterus with best neurological outcome for the fetus? Select and explain. A. Mgso4, B. Nifidipine, C. Ritodrine, D. Isoxprine",A: Mgso4,"Answer: A: Mgso4 Explanation: Ans. A.MgSo4Antenatal magnesium sulfate for both tocolysis and fetal neuroprotection in premature rupture of the membranes before 32 weeks' gestation. Other tocolytic drugs:Ritodrine, salbutamol and magnesium sulphate are tocolytic drugs used to terminate preterm labour and delivery.Other tocolytic drugs are isoxsuprine, indomethacin, calcium channel blockers, glyceryl trinitrate, atosiban and glyceryl trinitrate." "This is a real-world medical entrance exam question, please give the true answer based on the question and selection. Topic name: None, Subject name: Dental","Which of the following instrument does not have 4 digits? Select and explain. A. GMT, B. Excavators, C. Enamel hatchet, D. None",C: Enamel hatchet,"Answer: C: Enamel hatchet Explanation: Three-Number Formula It is used for the instruments in which cutting edge is at right angle to the long axis of the blade. Eg: Enamel hatchet Four-Number Formula It is used for the instruments in which primary cutting edge is not at right angle to long axis of the blade. Eg: GMT, excavators" "This is a real-world medical entrance exam question, please give the true answer based on the question and selection. Topic name: None, Subject name: Pathology","Direct Coombs test is positive in all the following except: Select and explain. A. Hemolytic anemia due to transfusion, B. Hemolytic anemia due to transfusion, C. Aplastic anemia, D. Drug-induced AIHA",C: Aplastic anemia,"Answer: C: Aplastic anemia Explanation: Ans: C. Aplastic anemia(Ref: Robbins 9/e p643).Direct Coomb's test is negative in aplastic anemia.Direct Coomb's testAlso known as direct antiglobulin testdegDetect antibodies of complement bound to RBC surfaceantigens in vivodeg.It is used for:Immune-mediated hemolytic anemia degHemolytic disease of the newborndegRh D-hemolytic disease of the newborn degABO hemolytic disease of the newborndegDrug-induced immune-mediated hemolysisdegMismatch transfusion reactiondeg." "This is a real-world medical entrance exam question, please give the true answer based on the question and selection. Topic name: None, Subject name: Pediatrics","A child presented at 2 years of age with delayed motor development, mental retardation and finger biting. He was normal at bih. He subsequently develops cerebral palsy ahritis and dies due to renal failure at age of 25 years. What is the likely enzyme deficiency implicated? Select and explain. A. Hexosaminidase deficiency, B. Adenosine deaminase deficiency, C. HGP deficiency, D. Ornithine transcarbamoylase deficiency",C: HGP deficiency,"Answer: C: HGP deficiency Explanation: Ans: C. HGP deficiency(Ref: Nelson 20/e p746-747)Lesch-Nyhan syndrome:Complete HGP deficiency characterized by hyperuricemia, self-mutilative behavior, choreoathetosis, spasticity & mental retardation.HP gene:Located on X chromosome.Affected males - Hemizygous for mutant gene.Carrier females - Asymptomatic.Paial HP deficiency:Kelley!Seegmiller syndrome associated with hyperuricemia without CNS manifestations.Hyperuricemia - Due to urate overproduction a Causing uric acid crystalluria, nephrolithiasis, obstructive uropathy & gouty ahritis.Early diagnosis and appropriate therapy with allopurinol can prevent or eliminate all the problems attributable to hyperuricemia without affecting behavioral or neurologic abnormalities." "This is a real-world medical entrance exam question, please give the true answer based on the question and selection. Topic name: None, Subject name: ENT","Eustachian tube function is best assessed by: Select and explain. A. Tympanometry, B. VEMP, C. Rhinomanometry, D. Politzer test",A: Tympanometry,"Answer: A: Tympanometry Explanation: Ans: A. Tympanometry(Ref Dhingra 7/e p62-63, 6/e p59)Assessment of eustachian tube function:Tympanometry & Politzer test.Tympanometry better than Politzer test.Tympanometry:Positive & negative pressures created in external ear canal.Patient swallows repeatedly.Normal tubal function:Ability of tube to equilibrate positive & negative pressures to ambient pressure.Done both in patients with perforated or intact tympanic membrane." "This is a real-world medical entrance exam question, please give the true answer based on the question and selection. Topic name: None, Subject name: Surgery","Most metabolically active layer in the bone is: Select and explain. A. Periosteum, B. Endosteum, C. Cancellous bone, D. Coical bone",B: Endosteum,"Answer: B: Endosteum Explanation: Ans: B. Endosteum (Ref Osteoporosis and Bone Densitomeny Measurements edited by Giuseppe Guglielmi/ p72)The most metabolically active component of bone is the endosteal surface of coex with trabecula bone being next most common active area .Metabolically active layer in bone:Are in immediate contact with bone marrowEndosteum of coex - Most active layer in bone (overall).Total area of endosteal surface approximately 0.5 m2.Higher remodelling activity than periosteal surface -Due to greater biomechanical strain or greater cytokine exposure from adjacent hone marrow compament.Order:Trabecular bone - Next most metabolically active area.In children = Periosteum is very active." "This is a real-world medical entrance exam question, please give the true answer based on the question and selection. Topic name: AIIMS 2019, Subject name: Gynaecology & Obstetrics","Criteria for infant at risk? Select and explain. A. Working mothers, B. Have not taken 100 days folic acid, C. Preclampsia is pregnancy, D. Malpresentation during bih",C: Preclampsia is pregnancy,"Answer: C: Preclampsia is pregnancy Explanation: Infant safe Infant risk Working mothers Have not taken 100 days folic acid Malpresentation during bih Preclampsia in pregnancy" "This is a real-world medical entrance exam question, please give the true answer based on the question and selection. Topic name: None, Subject name: Biochemistry","Which of the following vitamin is synthesized in vivo, in the body by humans? Select and explain. A. Niacin, B. Pantothenic acid, C. Cyanocobalamin, D. Folic acid",A: Niacin,"Answer: A: Niacin Explanation: Ans: A. Niacin(Ref: Harper 30/e p547, 556)Niacin:Not strictly a vitamin.Can be synthesized in body from essential amino acid tryptophan.Two compounds, nicotinic acid & nicotinamide - Similar biologic activity of niacin.Metabolic function is as the nicotinamide ring of coenzymes NAD & NADP in oxidation-reduction reactions." "This is a real-world medical entrance exam question, please give the true answer based on the question and selection. Topic name: None, Subject name: Forensic Medicine","Embalming without issuing death ceificate is punishable under section: Select and explain. A. IPC 201, B. IPC 297, C. IPC 299, D. IPC 498",A: IPC 201,"Answer: A: IPC 201 Explanation: Answer- A. IPC 201Embalming without issuing death ceiJicate is punishable under section IPC 201.'ln a medico legal case, condueting embalming before autopsy invites liabilities, under section 201 IPC (causing disappearance of evidence of offence, or giving false information to screen offender)." "This is a real-world medical entrance exam question, please give the true answer based on the question and selection. Topic name: None, Subject name: Pathology","Which of the following anticoagulant is used for electrolyte estimation? Select and explain. A. EDTA, B. Citrate, C. Sodium fluoride, D. Lithium heparin",D: Lithium heparin,"Answer: D: Lithium heparin Explanation: Ans. d. Lithium heparin Lithium heparin is the anticoagulant used for electrolyte estimation.Lithium-heparin is the preferred anticoagulant for hematology in non-mammalians because EDTA causes in vitro hemolysis in some amphibian, reptile, and fish species; moreover, plasma harvested from blood anticoagulated with lithium-heparin can be used for routine chemistry/electrolyte analysis, which is especially advantageous with small sample volumes.""Commonly used anticoagulants are heparin, EDTA, oxalates, citrate and fluoride. Of these, lithium heparin is best suited for most of the biochemical estimations. All other anticoagulants the/ate calcium and hence unsuitable for calcium estimation. The possibility of enzyme inhibition especially creatine kinase, ALP, ACP, amylase and LDH are observed with several of these anticoagulants. Oxalates are unsuitable for estimation of sodium and potassium also.""-" "This is a real-world medical entrance exam question, please give the true answer based on the question and selection. Topic name: None, Subject name: Social & Preventive Medicine","Calculate the neonatal moality rate for a population according to the data of 2012. Number of neonatal deaths- 450, number of stillbihs-212, total number of live bihs-12,450. Select and explain. A. 36, B. 15, C. 90, D. 56",A: 36,"Answer: A: 36 Explanation: Ans. a. 36Neonatal moality rate=Number of deaths of children under 28 years of age in a year (neonatal deaths) x 1000/ Total live bihs in the same year450 x 1000/ 12450= 36 per 1000 live bihs" "This is a real-world medical entrance exam question, please give the true answer based on the question and selection. Topic name: None, Subject name: Dental","Lingual plate is indicated when Select and explain. A. There is need for addition of one or more Anterior teeth, B. Deep Lingual sulcus, C. Good ridge support, D. Narrow lingual sulcus",A: There is need for addition of one or more Anterior teeth,"Answer: A: There is need for addition of one or more Anterior teeth Explanation: None" "This is a real-world medical entrance exam question, please give the true answer based on the question and selection. Topic name: None, Subject name: Surgery","A 48 years old female presents with seizure, recurrent gross hematuria and left flank abdominal pain. Abdominal CT reveals left perinephric hematoma with 3 cm angiomyolipoma along with multiple right renal angiomyolipoma measuring 1.5 to 6.5 cm. What would be the most probable diagnosis? Select and explain. A. VHL syndrome, B. Autosomal dominant polycystic kidney disease, C. Tuberous sclerosis, D. Hereditary angiolipoma",C: Tuberous sclerosis,"Answer: C: Tuberous sclerosis Explanation: Answer- C (Tuberous sclerosis)Tuberous sclerosis complex (TSC) is a rare multisystem autosomal dominant genetic disease that causes non-cancerous tumours.A combination of symptoms may include seizures, intellectual disability, developmental delay, behavioral problems, skin abnormalities, lung disease, and kidney disease.Three types of brain tumours are associated with TSC:Giant cell astrocytomaCoical tubersSubependymal nodulesPeople with TSC are frequently also diagnosed psychiatric disorders: autism spectrum disorder (ASD), attention deficit hyperactivity disorder (ADHD), anxiety disorder and depressive disorder.TSC patients have benign tumors of the kidneys called angiomyolipomas causing hematuria." "This is a real-world medical entrance exam question, please give the true answer based on the question and selection. Topic name: None, Subject name: Physiology","Which of the following rightly describes the mechanism of ""Pressor Escape"" in SIADH? Select and explain. A. Characterized by sudden increase in urine volume with decrease in urine osmolality independent of circulating vasopressin levels., B. Characterized by sudden increase in urine volume with decrease in urine osmolality dependent of circulating vasopressin levels., C. Characterized by sudden decrease in urine volume with increase in urine osmolality independent of circulating vasopressin levels., D. Characterized by sudden decrease in urine volume with increase in urine osmolality dependent of circulating vasopressin levels.=",A: Characterized by sudden increase in urine volume with decrease in urine osmolality independent of circulating vasopressin levels.,"Answer: A: Characterized by sudden increase in urine volume with decrease in urine osmolality independent of circulating vasopressin levels. Explanation: Ans. A. Characterized by sudden increase in urine volume with decrease in urine osmolality independent of circulating vasopressin levels. * Also referred as ""Vasopressin Escape"".* Characterized by sudden increase in urine volume with decrease in urine osmolality independent of circulating vasopressin levels." "This is a real-world medical entrance exam question, please give the true answer based on the question and selection. Topic name: None, Subject name: Surgery","Distraction osteogenesis is done in pediatric patient which of the following is considered Select and explain. A. Increase in rate, B. Increase rhythm, C. Consolidation time is increased, D. Time of fixation is increased",A: Increase in rate,"Answer: A: Increase in rate Explanation: None" "This is a real-world medical entrance exam question, please give the true answer based on the question and selection. Topic name: AIIMS 2018, Subject name: Pathology","Malignant peripheral nerve sheath tumour showing which of the following differentiation is termed as ""Triton tumour""? Select and explain. A. Glandular, B. Rhabdomyoblastic, C. Cailaginous, D. Osseous",B: Rhabdomyoblastic,"Answer: B: Rhabdomyoblastic Explanation: An interesting phenomenon observed in malignant peripheral nerve sheath tumour (MPNST) is described as ""divergent differentiation."" This term refers to the presence of focal areas that exhibit other lines of differentiation, including glandular, cailaginous, osseous (bone), or rhabdomyoblastic (striated muscle) morphology. MPNST exhibiting the rhabdomyoblastic differentiation is referred to as Triton tumour. Most MPNSTs (approximately 85%) are high-grade tumors. Most are associated with larger peripheral nerves in the chest, abdomen, pelvis, neck, or limb-girdle. Typical cases show a fasciculated arrangement of spindle shaped tumour cells." "This is a real-world medical entrance exam question, please give the true answer based on the question and selection. Topic name: None, Subject name: Dental","Prevalence of pit & fissure caries is measured with: Select and explain. A. ICDAS, B. DMF, C. def/dmf, D. Transillumination",D: Transillumination,"Answer: D: Transillumination Explanation: Prevalence of pit & fissure caries is measured with transillumination." "This is a real-world medical entrance exam question, please give the true answer based on the question and selection. Topic name: None, Subject name: Pediatrics","A child can walk upstairs one step at a time, can ride cycle but can't jump can also speak sentences, can tell his/her name gender but finds difficult to narrate the story.What is her actual developmental age Select and explain. A. 1 yrs, B. 2, C. 3, D. 4",B: 2,"Answer: B: 2 Explanation: Ans. b) 2yearsGROSS MOTOR MILESTONES: AgeMilestone3 monthsNeck holding 5 monthsRolls over6monthsSitting suppoed8monthsSitting without suppo9monthsStands with suppo12monthsStands without suppo, Walks but falls15monthsWalks alone, Creeps upstairs18monthsRuns, explores drawers2 yearsWalks upstairs (baby steps), Jumps3 yearsWalks upstairs (alternate feet), rides tricycle4 yearsHops on one foot, walks downstairs (alternate feet)" "This is a real-world medical entrance exam question, please give the true answer based on the question and selection. Topic name: None, Subject name: Pharmacology","Which of the following sites is least commonly preferred for insulin injection? Select and explain. A. Anterior thigh, B. Lateral thigh, C. Dorsum of arm, D. Around umbilicus",B: Lateral thigh,"Answer: B: Lateral thigh Explanation: Ans: B. Lateral thigh(Ref Goodman Gilman 12/e p1252)Insulin injection:Usually injected into subcutaneous tissues of abdomen, buttock, anterior thigh, or dorsal arm.Absorption most rapid from abdominal wall, followed by arm, buttock & thigh." "This is a real-world medical entrance exam question, please give the true answer based on the question and selection. Topic name: None, Subject name: Dental","The access cavity for mandibular first molar typically is Select and explain. A. Trapezoid, B. Triangular, C. Oval, D. Round",A: Trapezoid,"Answer: A: Trapezoid Explanation: None" "This is a real-world medical entrance exam question, please give the true answer based on the question and selection. Topic name: None, Subject name: Surgery","During laparoscopic inguinal hernia repair a tacker was accidently placed below and lateral to the ilio-pubic tract. Postoperatively the patient complained of pain and soreness in the thigh. This is due to the involvement of: Select and explain. A. Lateral cutaneous nerve of thigh, B. llioinguinal nerve, C. Genital branch of genitofemoral nerve, D. Obturator nerve",A: Lateral cutaneous nerve of thigh,"Answer: A: Lateral cutaneous nerve of thigh Explanation: Ans: A. Lateral cutaneous nerve of thigh(Ref Sabiston 19/e pink Schwaz 10/e p-1515, 9/e p1313-1315; Sehackelford 7/e p562-565)Neuropathic groin pain:Caused by damage to a nerve in the groin region and may he due to paial or complete division, stretching, contusion, crushing, suturing, or electrocautery.Nerves involved:Ilioinguinal nerve, iliohypogastric nerve, both the genital and femoral branches of the genitofemoral nerve, and the lateral femoral-1 cutaneous nerve of the thigh.1st two more prone to injury during an open herniorrhaphy.Latter (i.e. Lateral cutaneous nerve of thigh) are more likely damaged during laparoscopy.The genital and femoral branches of the genitofemoral nerve and the lateral cutaneous nerve of the thigh are most at risk when the surgeon staples below the iliopubic tract when lateral to the internal .spermatic vessels.A burning, tingling pain along the lateral aspect of the thigh in the distribution of the lateral femoral cutaneous nerve is known as meralgia parestheticaDue to entrapment of that nerve.Affected skin area:Hyperaesthetic and/or pruritic.Complain of tactile hallucination of a sensation of small insects creeping under the skin (formication)." "This is a real-world medical entrance exam question, please give the true answer based on the question and selection. Topic name: None, Subject name: Anatomy","Efferent tracts from cerebellum arises from: Select and explain. A. Purkinje cells, B. Deep cerebellar nuclei, C. Cerebellar cortex, D. Vermis of cerebellum",A: Purkinje cells,"Answer: A: Purkinje cells Explanation: The deep nuclei provide the only output for the spinocerebellum, & neocerebellum. Purkinje cell axons are the only output from the cerebellar cortex, generally pass to deep nuclei." "This is a real-world medical entrance exam question, please give the true answer based on the question and selection. Topic name: None, Subject name: Pediatrics","Which of the following is the most appropriate method for obtaining a urine specimen for culture in an 8 month old girl – Select and explain. A. Suprapubic aspiration, B. Indwelling catheter sample, C. Clean catch void, D. Urinary bag sample",A: Suprapubic aspiration,"Answer: A: Suprapubic aspiration Explanation: Diagnosis of urinary tract infection A UTI may be suspected based on the symptoms or finding on urinalysis or both, but a urine culture is necessary for confirmation and appropriate therapy. Thus the diagnosis of UTI depends on having the proper sample of urine. There are several ways to obtain a urine sample, some are more accurate than others. These methods are — Midstream urine sample — It is satisfactory in toilet trained children and in circumcised boys. But in young infants and boys who cannot ratract their foreskin, it is not reliable. Such samples will usually reflect periurethral and prepucial organisms and cells. Collection in an adhesive sealed sterile collection bag after disinfection of the genitals Is a useful technique in infants, but potential contamination from genitals and perianal area can occur. 3. Urine specimen through catheterization or indwelling catheter — Urine specimen through a catheter is less likely to be contaminated than the voided specimens as described above. It is also less invasive than suprapubic aspiration, but it is not as sensitive as suprapubic aspiration. A catherized specimen is reliable if the first portion of the urine that may contain urethral organisms is discarded and specimen is taken from later flow through the catheter, but it has the disadvantage of being traumatic and of potentially introducing urethral organisms into sterile bladder Suprapubic aspiration - Suprapubic aspiration is best method to collect urine specimen for culture. It is the most reliable method It can be performed safely in children and in premature infants by 21 or 22 gauge needle. This procedure avoids any sort of contamination, however it is invasive. So it is used only in infants and selected patients. Note: The sensitivity of suprapubic aspiration is 99 % The sensitivity of catherization is 95%" "This is a real-world medical entrance exam question, please give the true answer based on the question and selection. Topic name: None, Subject name: Dental","Which one of the following is not an Indication for antibiotic therapy In patients with acute abscess? Select and explain. A. Fever, B. Deep inaccessible pocket, C. Horizontal bone loss, D. Regional lymphadenopathy",C: Horizontal bone loss,"Answer: C: Horizontal bone loss Explanation: None" "This is a real-world medical entrance exam question, please give the true answer based on the question and selection. Topic name: AIIMS 2018, Subject name: Anatomy","Which of the following sensation is NOT perceived by spinal nucleus of trigeminal nerve Select and explain. A. Touch, B. Pain, C. Temperature, D. Proprioception",D: Proprioception,"Answer: D: Proprioception Explanation: Trigeminal nerve nuclei - one Motor nucleus (in pons) two sensory nuclei Motor nucleus: - In pons Controls muscles of mastication (1st pharyngeal arch) Sensory nuclei: 1. Main sensory nucleus: In pons Receives fine touch & vibration senses 2. Mesencephalic sensory nucleus: - In mid brain Receives proprioception from eyeball, mandible & tongue 3. Spinal sensory nucleus: In medulla oblongata & spinal cord Receives pain, temperature, crude touch" "This is a real-world medical entrance exam question, please give the true answer based on the question and selection. Topic name: None, Subject name: Dental","The condylar cartilage at birth looks like Select and explain. A. A, B. B, C. C, D. Any of the above",C: C,"Answer: C: C Explanation: Figure shows; the condylar cartilage (blue) develops initially as a separate area of condensation from that of  the body of the mandible and only later is incorporated within it. A, Separate areas of mesenchymal  condensation at 8 weeks. B, Fusion of the cartilage with the mandibular body at 4 months. C, Situation at birth (reduced to scale)." "This is a real-world medical entrance exam question, please give the true answer based on the question and selection. Topic name: None, Subject name: Medicine","A patient of disseminated malignancy comes to the palliative care clinic with nausea, vomiting and altered sensorium. Hypercalcemia is detected on investigations. What will be the first line of management? Select and explain. A. Intravenous steroids, B. Thiazides, C. Intravenous fluids, D. Intravenous bisphosphonates",C: Intravenous fluids,"Answer: C: Intravenous fluids Explanation: Ans: C. Intravenous fluids(Ref: Harrison 19/e p314; 18Ie p361)Management of Significant, Symptomatic Hypercalcemia:Initial therapy:Volume expansion (hypercalcemia leads to dehydration) with 4-6 L of IV saline over first 24 h, keeping in mind that underlying comorbidities (e.g., congestive hea failure) may require use of loop diuretics to enhance Na+ & Ca+ excretion.Increased calcium mobilization from bone (in malignancy or severe hyperparathyroidism).Drugs that inhibit bone resorption should be considered.Zoledronic acid, pamidronate & ibandronate - In treatment of hypercalcemia of malignancy in adults.Onset of action is within 1-3 days, with normalization of serum calcium levels occurring in 60-90% of patients." "This is a real-world medical entrance exam question, please give the true answer based on the question and selection. Topic name: None, Subject name: Dental","Space between Apical constriction and Apical foamen is Select and explain. A. Funnel shaped, B. Hyper bolic, C. Morning glory, D. All of the above",D: All of the above,"Answer: D: All of the above Explanation: None" "This is a real-world medical entrance exam question, please give the true answer based on the question and selection. Topic name: None, Subject name: Social & Preventive Medicine","Which of the following statements is true regarding for influenza vaccine Select and explain. A. Secondary attack rate 5 to 15%, B. Virus shedding present before the pt presents with symptoms, C. 1 to 5 years is high risk age group, D. Aquatic birds are reservoirs",D: Aquatic birds are reservoirs,"Answer: D: Aquatic birds are reservoirs Explanation: Answer.D. Aquatic birds are reservoirs* RESERVOIR: Humans are the principle reservoir of human influenza A viruses. The an reservoir of influenza A viruses is wild birds, predominantly ducks, geese, and shorebirds.* Seasonal influenza tends to target children younger than 12 months of age and adults 65 years old or older.* Secondary attack rates were estimated at 10.3% (95% CI 6.8-14.7) for secondary cases with influenza-like illness and 20.2% (95% CI 15.4-25.6) for secondary cases with acute respiratory illness." "This is a real-world medical entrance exam question, please give the true answer based on the question and selection. Topic name: None, Subject name: Gynaecology & Obstetrics","A 2s-year-old women underwent induced ovulation.on uSG, ovary showed 8 follicles. Serum estradol level was 800 pg/ml.what is the next step in the management of this patient? Select and explain. A. Retrieve follicles, B. Give cabergoline, C. cancel cycle, D. withhold HCG",A: Retrieve follicles,"Answer: A: Retrieve follicles Explanation: Ans: A. Retrieve folliclesRef: Berek and Novak's GynaecologtDay of analysis of above measurements and size of fbllictes are not specified in the question. So, presuming these parameters to be treasured on the mid cycle, next step according to the options would be to retrieve follicles." "This is a real-world medical entrance exam question, please give the true answer based on the question and selection. Topic name: None, Subject name: Dental","The periodontal ligament Select and explain. A. Derives its blood supply primarily from the branches of vessels entering the pulp, B. Has a slow rate of turnover, C. Contains epithelial cells of malassez, D. Comprises primarily of type 2 collagen",C: Contains epithelial cells of malassez,"Answer: C: Contains epithelial cells of malassez Explanation: None" "This is a real-world medical entrance exam question, please give the true answer based on the question and selection. Topic name: AIIMS 2019, Subject name: Medicine","A patient presents with palpitations. O/E the patient was conscious, pulse was fast with BP of 80/50 mmHg. He was subjected to Vagal maneuver and was given intravenous adenosine. The rhythm remains refractory to maximum dose of adenosine. What is the next step in management of this patient? Select and explain. A. Repeat adenosine, B. Amiodarone, C. Defibrillation, D. Cardioversion",D: Cardioversion,"Answer: D: Cardioversion Explanation: Question about Hemodynamic instability. Vagal maneuver and IV adenosine hints about PSVT/ AVN. And Pt. is refractory to Adenosine as max. dose is already given. So, next best step would be repeat adenosine according to new ACC guidelines, Only for those patients who are hemodynamically Stable > 90 mm Hg- brains tends to switch on auto regulation Question mentions about unstable patient therefore rules out repeat adenosine. Amiodarone -mainly used in treatment of broad complex VT - H/o AVN, narrow QRS complex Therefore rules out Amiodarone Defibrillation: Done for patient with ventricular fibrillation. It is basically a non-synchronized DC shock. It is given in: a. PVT Broad QRS b. Ventricular fibrillation Therefore, ruled out. Cardioversion is synchronized DC shock used for management of patients with any tachyarrhythmia for hemodynamic unstability. RR interval constant HR =300/min Narrow QRS Global ST segment depression in all leads Subendocardial ischemia, which is usually demonstrated in patients of PSVT. In both Harrisons and ACC guidelines" "This is a real-world medical entrance exam question, please give the true answer based on the question and selection. Topic name: None, Subject name: Microbiology","Autoinfection can be caused by all the following helminths except: Select and explain. A. Enterobius vermicularis, B. Hymenolepis nana, C. Taenia solium, D. Ascaris lumbricoides",D: Ascaris lumbricoides,"Answer: D: Ascaris lumbricoides Explanation: Ans: D. Ascaris lumbricoidesAutoinfection:Infection of a primary host with a parasite, paicularly a helminth, in such a way that the complete life cycle of the parasite happens in a single organism, without the involvement of another host.Primary host is at the same time the secondary host of parasite.Autoinfection is seen in (CHEST)Cryptosporidium parvumdegHymenolepis nanadegEnterobius vermicularisQStrongyloides stercoralisdegTaenia soliumdeg" "This is a real-world medical entrance exam question, please give the true answer based on the question and selection. Topic name: AIIMS 2019, Subject name: Surgery","What is the name of this technique for palpation of thyroid. A thumb is placed on the lateral side of trachea and patient is swallowing? Select and explain. A. Lahey's method, B. Kocher's test, C. Pizzilo method, D. Crile method",D: Crile method,"Answer: D: Crile method Explanation: Thyroid palpation- methods Lahey's method- In this method,the examination of the thyroid is performed from the front.Normal lobe of thyroid is pushed to the side of nodule so that solitary nodule can be made prominent and palpated well. KOCHER'S TEST : Slight compression of the lateral lobe of thyroid gland produces stridor. If the test is positive , it signifies that the patient has an obstructed trachea. Fore finger of examiner should palpate each thyroid lobe. Pizillo's method - Fingers are clasped against occiput - Makes inspection of thyroid easier Crile method In this method,place thumb on thyroid gland and ask the patient to swallow. For palpation of smaller nodules within the thyroid gland this test is performed." "This is a real-world medical entrance exam question, please give the true answer based on the question and selection. Topic name: None, Subject name: Dental","Gelation temperature of agar is between Select and explain. A. 20°C to 25°C, B. 27°C to 33°C, C. 37°C to 50°C, D. 55°C to 60°C",C: 37°C to 50°C,"Answer: C: 37°C to 50°C Explanation: None" "This is a real-world medical entrance exam question, please give the true answer based on the question and selection. Topic name: None, Subject name: Gynaecology & Obstetrics","A 26 years old healthy female got pregnant for 1st time and LSCS was done for fetal distress. Mild hypeension was present during pregnancy. Two days after delivery she had headache and seizures but proteinuria was not seen. CT scan shows 2 x 3 cm parasagittal hematoma. Diagnosis is: Select and explain. A. Eclampsia, B. Hypeensive intracranial hemorrhage, C. Sagittal sinus thrombosis, D. Pituitary apoplexy",C: Sagittal sinus thrombosis,"Answer: C: Sagittal sinus thrombosis Explanation: Answer- C. Sagittal sinus thrombosisVenous sinus thrombosis of the lateral or sagittal sinus or of small coical veins (coical vein thrombosis) occurs as a complication of oral contraceptive use, pregnancy and the postpaum period, inflammatory bowel disease, intracranial infections (meningitis), and dehydration.Patients present with headache and may also have focal neurologic signs (especially paraparesis) and seizures." "This is a real-world medical entrance exam question, please give the true answer based on the question and selection. Topic name: None, Subject name: Dental","Which is the typical facies of Down syndrome? Select and explain. A. Hypoplastic maxilla, B. Prognathic maxilla, C. Retrognathic mandible, D. Prognathic mandible",A: Hypoplastic maxilla,"Answer: A: Hypoplastic maxilla Explanation: Skeletal findings are an underdeveloped midface, creating  a  prognathic  occlusal  relationship.  Oral  findings include  mouth  breathing,  open  bite,  appearance  of macroglossia, fissured lips and tongue, angular cheilitis, delayed eruption times, missing and malformed teeth, oligodontia, small roots, microdontia, crowding, and a low level of caries. Children with Down syndrome experience a high incidence of rapid, destructive periodontal disease, which may be related to local factors such as tooth morphology, bruxism, malocclusion, and poor oral hygiene. Reference: McDONALD AND AVERY’S DENTISTRY for the CHILD and ADOLESCENT, 10th ed page no 527" "This is a real-world medical entrance exam question, please give the true answer based on the question and selection. Topic name: None, Subject name: Gynaecology & Obstetrics","Exact number of weeks between last menstrual period and expected date of delivery :March 2005 Select and explain. A. 38 weeks, B. 39 weeks, C. 40 weeks, D. 41 weeks",C: 40 weeks,"Answer: C: 40 weeks Explanation: Ans. C: 40 weeksChildbih usually occurs about 38 weeks after conception; i.e., approximately 40 weeks from the last normal menstrual period (LNMP).The World Health Organization defines normal term for delivery as between 37 weeks and 42 weeksEDD is calculated by Naegele's ruleAdd 7 days to the first day of the last period and subtract 3 monthsNaegele's rule is based on 28 days regular cycle.If the cycle is shoer or longer than 28 days, EDD will be corrected and written as corrected EDD.Examples:40 days cycle regularly, to get corrected EDD, add 12 days (40-28) with the EDD calculated from LMP.21 days cycle regularly, to get corrected EDD, subtract 7 days (28-21) with the EDD calculated from LMP." "This is a real-world medical entrance exam question, please give the true answer based on the question and selection. Topic name: None, Subject name: Surgery","A patient is not responding even after undergoing multiple platelet transfusion. Which of the following will be the best treatment for him? Select and explain. A. Random platelet donor, B. Single platelet donor, C. HLA matched platelet, D. Irradiated platelets",C: HLA matched platelet,"Answer: C: HLA matched platelet Explanation: None" "This is a real-world medical entrance exam question, please give the true answer based on the question and selection. Topic name: None, Subject name: Surgery","An adult patient with panfacial trauma, treatment should start from Select and explain. A. Mandible, B. Zygomatic arch, C. NOE & frontal bar, D. Any but end at lefort 1",D: Any but end at lefort 1,"Answer: D: Any but end at lefort 1 Explanation: None" "This is a real-world medical entrance exam question, please give the true answer based on the question and selection. Topic name: AIIMS 2019, Subject name: Biochemistry","Which of the following doesn't occur in 5' to 3' direction? Select and explain. A. DNA repair, B. DNA replication, C. RNA editing, D. Transcription",C: RNA editing,"Answer: C: RNA editing Explanation: The 5' is upstream; the 3' is downstream. DNA and RNA are synthesized in the 5' to 3' direction In DNA Replication whenever synthesis occurs new nucleotide is added to 3' end In transcription, RNA is getting synthesized in 5' to 3' direction RNA editing It occurs in 3'-5' direction Post transcriptional modification in which differential RNA processing occurs. Also called chemical modification of RNA, in which mostly Cytosine gets conveed to Uracil. DNA Replication ,Proof reading DNA Repair 3'-5' exonuclease activity Mostly endonuclease activity, but sometimes it is 5'-3' exonuclease activity." "This is a real-world medical entrance exam question, please give the true answer based on the question and selection. Topic name: AIIMS 2019, Subject name: Orthopaedics","A 9 year old boy presenting with lytic lesion on midshaft tibia. On HPE, there are grooved nuclei, giant cells eosinophilic cytoplasm and fibroblastic proliferation. Xray is as shown below. What is the likely diagnosis? Select and explain. A. Langerhans cell histiocytosis, B. Giant cell tumor, C. Chondroblastoma, D. Osteoid osteoma",A: Langerhans cell histiocytosis,"Answer: A: Langerhans cell histiocytosis Explanation: The grooved nuclei, lytic lesion in first decade suggests Langerhan's cell histiocytosis. LANGERHANS CELL HISTIOCYTOSIS (LCH): DE!STRUCTION DUE TO HISTIOCYTES, IDIOPATHIC1. Letterer-Siwe disease: a fulminant systemic disease, age group < 3 years, fatal2. Hand-Schuller-Christian disease: Triad of skull lesions (lytic lesion), Exophthalmos and diabetes insipidus. 3. Eosinophilic Granuloma: Solitary lesion of bone or lung (Pulmonary histiocytosis X) 1st decade of life. Skull is the most common site in skeletal system - Bevelled edge lytic lesion is seen in skull (double contour).Biopsy: Gold standard (cells with Birbeck's granules (tennis racket appearance) under election microscopy) grooved nuclei are seen. TreatmentSpontaneous resolution. Highly radiosensitive and Excision + Curettage for resistant cases." "This is a real-world medical entrance exam question, please give the true answer based on the question and selection. Topic name: AIIMS 2017, Subject name: Anatomy","Dense irregular connective tissue is found in Select and explain. A. Dermis, B. Lamina propria, C. Tendon, D. Ligament",A: Dermis,"Answer: A: Dermis Explanation: *Connective Tissue: - Loose Areolar Connective Tissue: Have less fibers Example 1. Sub epithelial Lamina propria 2. Superficial papillary layer of dermis - Dense Connective Tissue: Have more fibers *Dense irregular CT- Collagen fibers are arranged in irregular pattern. Example 1. Periosteum 2. Capsules (liver, spleen capsules etc) 3. Perichondrium 4. Deep reticular layer of dermis *Dense Regular Connective Tissue: *Fibers are arranged regularly in parallel fashion - to pull bone Example 1. Ligament 2. Tendon 3. Aponeurosis Given options (a) Dermis - maximum is deep reticular layer - Dense irregular CT (b) Lamina propria - Loose areolar CT" "This is a real-world medical entrance exam question, please give the true answer based on the question and selection. Topic name: None, Subject name: Pharmacology","Antibiotic that interferes with DNA function is: Select and explain. A. Metronidazole, B. Nystatin, C. Tetracycline, D. Sulphonamides",A: Metronidazole,"Answer: A: Metronidazole Explanation: Mechanism of Action Metronidazole is a prodrug requiring reductive activation of the nitro group by susceptible organisms.  Unlike their aerobic counterparts, anaerobic and microaerophilic pathogens contain electron transport components that have a sufficiently negative redox potential to donate electrons to metronidazole.  The single-electron transfer forms a highly reactive nitro radical anion that kills susceptible organisms by radical-mediated mechanisms that target DNA. Reference: Goodman & Gilman’s THE PHARMACOLOGICAL BASIS OF THERAPEUTICS THIRTEENTH EDITION Page no 991" "This is a real-world medical entrance exam question, please give the true answer based on the question and selection. Topic name: None, Subject name: Anatomy","Muscles spared by complete transection of cranial part of accessory nerve: Select and explain. A. Cricopharyngeus, B. Palatopharyngeus, C. Stylopharyngeus, D. Salpingopharyngeus",C: Stylopharyngeus,"Answer: C: Stylopharyngeus Explanation: None" "This is a real-world medical entrance exam question, please give the true answer based on the question and selection. Topic name: None, Subject name: Gynaecology & Obstetrics","Chicken pox infection in mother most dangerous for the baby when it occurs at - Select and explain. A. 14-28 weeks, B. 3 -9 weeks, C. 28-37weeks, D. 8-14weeks",C: 28-37weeks,"Answer: C: 28-37weeks Explanation: Ans. C. 28-37weeksBefore 28 weeks pregnant: there's no evidence you are at increased risk of suffering a miscarriage. However, there's a small risk baby could develop foetal varicella syndrome (FVS). FVS can damage the baby's skin, eyes, legs, arms, brain, bladder or bowel.Between weeks 28 and 36 of pregnancy: the virus stays in the baby's body but doesn't cause any symptoms. However, it may become active again in the first few years of the baby's life, causing shingles.After 36 weeks of pregnancy: baby may be infected and could be born with chickenpox." "This is a real-world medical entrance exam question, please give the true answer based on the question and selection. Topic name: None, Subject name: Medicine","Site of lesion in endocarditis of RHD is: Select and explain. A. Along line of closure of valves, B. Both side of valves, C. Valve cusps, D. Free margins of valves",A: Along line of closure of valves,"Answer: A: Along line of closure of valves Explanation: None" "This is a real-world medical entrance exam question, please give the true answer based on the question and selection. Topic name: None, Subject name: Radiology","In an internal derangement of TMJ fracture, panoramic view has been taken for the patient. To confirm our diagnosis, we may take the help of Select and explain. A. MRI, B. CT scan, C. Arthroscopy, D. Bone scan",A: MRI,"Answer: A: MRI Explanation: Soft tissues of the joint (articular disk) can be imaged with MRI or Arthrography. MRI produces superb images of the soft tissues in internal derangement of the disk. Arthrography is invasive and has the risk of infection and allergic reaction due to the contrast agent." "This is a real-world medical entrance exam question, please give the true answer based on the question and selection. Topic name: None, Subject name: Dental","The key ridge useful in assessing malocclusion is present on: Select and explain. A. Maxilla, B. Zygomatic buttress, C. Sphenoid bone, D. Mandible",B: Zygomatic buttress,"Answer: B: Zygomatic buttress Explanation: None" "This is a real-world medical entrance exam question, please give the true answer based on the question and selection. Topic name: None, Subject name: Dental","Local irritating factors in gingiva is most likely give rise to: Select and explain. A. Pyogenic granuloma, B. Generalized fibrous hyperplasia of gingiva, C. Mucosal cobblestoning, D. Periapical abscess",A: Pyogenic granuloma,"Answer: A: Pyogenic granuloma Explanation: None" "This is a real-world medical entrance exam question, please give the true answer based on the question and selection. Topic name: AIIMS 2019, Subject name: Pathology","Which of the following is activated by intrinsic or extrinsic pathway's? Select and explain. A. Necroptosis, B. Apoptosis, C. Necrosis, D. Pyroptosis",B: Apoptosis,"Answer: B: Apoptosis Explanation: Apoptosis results from the activation of enzymes called caspases (so named because they are cysteine proteases that cleave proteins after aspaic residues). Two distinct pathways converge on caspase activation: the mitochondrial pathway (Intrinsic pathway) and the death receptor pathway( Extrinsic Pathway)." "This is a real-world medical entrance exam question, please give the true answer based on the question and selection. Topic name: None, Subject name: Microbiology","About Bacteroids. True are all except Select and explain. A. Susceptible to many antibiotics, B. Present mainly in mixed infections, C. Most common species is B fragilis, D. Bile resistant anerobes",A: Susceptible to many antibiotics,"Answer: A: Susceptible to many antibiotics Explanation: None" "This is a real-world medical entrance exam question, please give the true answer based on the question and selection. Topic name: None, Subject name: Pediatrics","All of the following are the complications in the new born of a diabetic mother except – Select and explain. A. Hyperbilirubinemia, B. Hyperglycemia, C. Hypocalcemia, D. Hypomagnesemia",B: Hyperglycemia,"Answer: B: Hyperglycemia Explanation: None" "This is a real-world medical entrance exam question, please give the true answer based on the question and selection. Topic name: None, Subject name: Gynaecology & Obstetrics","Vasopressor of choice in pregnancy is: Select and explain. A. Ephedrine, B. Phenylephrine, C. Methoxamine, D. Mephentermine",A: Ephedrine,"Answer: A: Ephedrine Explanation: Vasopressor of choice in pregnancy is ephedrine." "This is a real-world medical entrance exam question, please give the true answer based on the question and selection. Topic name: None, Subject name: Microbiology","Which of the following malignancy is not caused by Human Papilloma Virus (HPV) infection? Select and explain. A. Carcinoma base of tongue, B. Cervical carcinoma, C. Tonsillar carcinoma, D. Nasopharyngeal carcinoma",D: Nasopharyngeal carcinoma,"Answer: D: Nasopharyngeal carcinoma Explanation: Ans: D. Nasopharyngeal carcinoma(Ref Ananthanaravan 10/e p557, 9/e p553)Human infections caused by HPVDiseasesSerotypeSkin was (Plantar wa, common wa, flat wa &Epidermodysplasia verruciformis)1, 2, 43,Papilloma (Laryngeal, Oral)6, 11Condyloma acuminatum (genital wa)6, 11Oral squamous cell carcinoma16, 18Cervical intraepithelial neoplasia (CIN)6, 11Carcinoma cervix16, 18, 31, 33, 35, 42-44 Infectious AgentLymphoid MalignancyHIVDiffuse large B cell lymphomadegBurkitt's lymphomadeg" "This is a real-world medical entrance exam question, please give the true answer based on the question and selection. Topic name: None, Subject name: Dental","Stimulated saliva as compared to unstimulated saliva has more Select and explain. A. Bicarbonate, B. Cl, C. Na, D. H",A: Bicarbonate,"Answer: A: Bicarbonate Explanation: None" "This is a real-world medical entrance exam question, please give the true answer based on the question and selection. Topic name: None, Subject name: Surgery","All of the following are regarding osteoporosis except. Select and explain. A. Calcitonin decreases pain, B. Bisphophonates are work horse for treatment, C. T-score <1.5 in osteoporosis, D. PTH is used in severe osteoporosis",C: T-score <1.5 in osteoporosis,"Answer: C: T-score <1.5 in osteoporosis Explanation: Answer- C. T-score <1.5 in osteoporosisTLscore -2.5 or less is seen in osteoporosis.T-scoreInterpretation-1 or more-NormalBetween -l and -2.5Osteopenia-2.5 or less-Osteoporosis-2.5 or less with fragility fracture - Severe osteoporosis" "This is a real-world medical entrance exam question, please give the true answer based on the question and selection. Topic name: None, Subject name: Dental","Periapical bone resorption occurs due to: Select and explain. A. IL-1 alpha, B. TNF alpha, C. Both, D. None",C: Both,"Answer: C: Both Explanation: TNF-α, IL-1, and PGE2 also promote osteoclast activity, particularly in case of inflammatory osteolysis such as those found in periodontitis and peri-apical pathology. Textbook of Endodontics Nisha Garg 3rd Ed" "This is a real-world medical entrance exam question, please give the true answer based on the question and selection. Topic name: None, Subject name: Pathology","A non-painful, slowly enlarging benign neoplasm appears as a submucosaL lu nip and exhibits pseudoepitheliomatous hyperplasia in the overlying epithelium, is most probably: Select and explain. A. Fibroma, B. Rhabdomyoma, C. Granular cell tumour, D. Papilloma",C: Granular cell tumour,"Answer: C: Granular cell tumour Explanation: None" "This is a real-world medical entrance exam question, please give the true answer based on the question and selection. Topic name: None, Subject name: Forensic Medicine","Lateral traction test is done in unnatural sexual offences in: Select and explain. A. Habitual active agent, B. Pedophilia active agent, C. Bestiality active agent, D. Habitual passive agent",D: Habitual passive agent,"Answer: D: Habitual passive agent Explanation: Answer- D. Habitual passive agentLateral traction test is done in unnatural sexual offences in habitual passive agent.The useful guide as to the patient's habituation to anal intercourse is lateral buttock traction test.In lateral buttock traction test, the thumb is placed on the cheeks of the buttock on either side of the anus and gentle lateral traction is applied.In patients who are not accustomed to anal penetration (penile or instrumental or any other) the traction results in reflex constriction of anal sphincter.The patients, who are used to anal penetration, react to the lateral traction test by relaxation of the sphincter." "This is a real-world medical entrance exam question, please give the true answer based on the question and selection. Topic name: None, Subject name: Medicine","Valvular lesion most often resulting from myocardial infarction is: Select and explain. A. Aortic stenosis, B. Mitral stenosis, C. Mitral regurgitation, D. Pulmonary stenosis",C: Mitral regurgitation,"Answer: C: Mitral regurgitation Explanation: None" "This is a real-world medical entrance exam question, please give the true answer based on the question and selection. Topic name: AIIMS 2017, Subject name: Surgery","Balanced resuscitation in trauma management is:- Select and explain. A. Giving colloids and crystalloids ratio of 1:1, B. Maintaining pH by ensuring acid base are balanced, C. Maintaining permissible hypotension to avoid bleeding, D. Maintaining airway breathing and circulation simultaneously",C: Maintaining permissible hypotension to avoid bleeding,"Answer: C: Maintaining permissible hypotension to avoid bleeding Explanation: Components of Damage Control Resuscitation Permissive hypotension Minimizing crystalloid-based resuscitation Immediate release & administration of pre-defined blood products (red blood cells, plasma & platelets) in ratios similar to those of whole blood" "This is a real-world medical entrance exam question, please give the true answer based on the question and selection. Topic name: AIIMS 2017, Subject name: Medicine","A patient comes to ED with fever and headache. On examination he has neck stiffness. CSF analysis was done:- Parameter Finding Normal range Glucose (mg/dl) 55 50-80 Protein (g/L) 0.50 0.18-0.45 ICT (cmH2O) 35 5-20 WBC 25 (predominantly lymphocytes) <5 Most likely diagnosis is- Select and explain. A. TB, B. N. Gonorrhea, C. Cryptococcus, D. Coxsackie",D: Coxsackie,"Answer: D: Coxsackie Explanation: Elevated lymphocytes with normal protein & sugar point towards viral cause- Coxsackie infection Option A- TB meningitis should have >100 finding of proteins with low sugar. Option B- Sugar normal so bact. meningitis is ruled out as in bact. meningitis there is reduced (|) sugar. Option C- No information of AIDS, CD4 count less is not mentioned in the question and there should be normal CSF so ruled out." "This is a real-world medical entrance exam question, please give the true answer based on the question and selection. Topic name: None, Subject name: Dental","Estimate the dental age from the radiograph: Select and explain. A. 5 years, B. 10 years, C. 15 years, D. 25 years",C: 15 years,"Answer: C: 15 years Explanation: None" "This is a real-world medical entrance exam question, please give the true answer based on the question and selection. Topic name: None, Subject name: Pathology","A girl suffering from browning of teeth, wearing of enamel but not cavitatory. OPG shows obliteration of pulp with narrowing of canal & deposition of secondary dentin, she gave the history that out of her 4 brothers, 2 are suffering from the same disease. She is suffering from Select and explain. A. Amelogenesis imperfecta, B. Dentinogenesis imperfecta, C. Fluorosis, D. Odontodysplasia",B: Dentinogenesis imperfecta,"Answer: B: Dentinogenesis imperfecta Explanation: None" "This is a real-world medical entrance exam question, please give the true answer based on the question and selection. Topic name: None, Subject name: Pharmacology","When broad-spectrum antibiotics are administered with coumarin anticoagulants, the anticoagulant action may be: Select and explain. A. Reduced because of enhanced hepatic drug metabolism, B. Reduced because of increased protein-binding, C. Increased because of reduction of vitamin 'K' sources, D. Increased because of decreased renal excretion of the anticoagulant",C: Increased because of reduction of vitamin 'K' sources,"Answer: C: Increased because of reduction of vitamin 'K' sources Explanation: None" "This is a real-world medical entrance exam question, please give the true answer based on the question and selection. Topic name: None, Subject name: Pathology","In hemorrhage lost plasma is replaced by: Select and explain. A. Bone marrow, B. Kidney, C. Spleen, D. Muscle",B: Kidney,"Answer: B: Kidney Explanation: None" "This is a real-world medical entrance exam question, please give the true answer based on the question and selection. Topic name: None, Subject name: Biochemistry","In a reaction substrate is available in a concentration that is l000times the Km value of the enzyme. After 9 minutes of reaction, l7o substrate is conveed to product (12 microgram/ml). If the concentration of the enzyme is changed to 1/3 and concentration of substrate is doubled. What is the time taken to conve the substrate into the same amount of product, i.e. I 2microgram/ml? Select and explain. A. 9 minutes, B. 4.5 minutes, C. 27 minutes, D. 13.5 minutes",C: 27 minutes,"Answer: C: 27 minutes Explanation: Ans: C. 27 minutesRef: Lehninger principles of biochemistry, 6t' ed., pg. 204During derivation of Michaelis-Menten equation,If total enzyme concentration is reduced by 1/3 and is very high then Vmax also become l/3.So in this question same amount of product will be fbrmed in 27 rninutes (3 x 9min)" "This is a real-world medical entrance exam question, please give the true answer based on the question and selection. Topic name: AIIMS 2018, Subject name: Surgery","Which is the most conspicuous sign in breast cancer? Select and explain. A. Nipple retraction, B. Peau d'orange, C. Puckering, D. Cancer en-cuirasse",B: Peau d'orange,"Answer: B: Peau d'orange Explanation: Peau-d-Orange Peau-d-orange is due to cutaneous lymphatic edema, where the infiltrated skin is tethered by sweat ducts, it cannot swell, leading to an appearance like orange skin. Due to obstruction of subdermal lymphatics (lymphatic permeation by tumor cells) Seen in advanced breast cancer (may be seen in chronic abscess)" "This is a real-world medical entrance exam question, please give the true answer based on the question and selection. Topic name: None, Subject name: Pediatrics","Most common sequelae due to periventricular leukomalacia – Select and explain. A. Spastic diplegia, B. Spastic quadriplegia, C. Mental retardation, D. Seizures",A: Spastic diplegia,"Answer: A: Spastic diplegia Explanation: Spastic diplegia is commoner in preten-n babies and is associated with periventricular leukomalacia. Periventricular leukomalacia Periventricular leukomalacia is a form of brain injury characterized by the death of white matter near the cerebral ventricles due to damage and softening of brain tissue. Premature infants are at the greatest risk of developing periventricular leulcomalacia. Affected individuals develop : - i)    Motor control problems ii) Other developmental delay iii) Cerebral palsy (especially spastic diplegia) iv) Epilepsy" "This is a real-world medical entrance exam question, please give the true answer based on the question and selection. Topic name: None, Subject name: Dental","What should be the position of child during asthmatic attack: Select and explain. A. Supine, B. Semi erect, C. Erect, D. Trendelenburg",B: Semi erect,"Answer: B: Semi erect Explanation: Advocated patient positions: Unconscious patient:; supine position. Foreign body aspiration;: Trendelenburg position. Asthmatic patient;: semi erect position." "This is a real-world medical entrance exam question, please give the true answer based on the question and selection. Topic name: None, Subject name: Medicine","A patient on amphotericin B develops hypokalemia of 2.3 meq/l. K+ supplementation required? Select and explain. A. 40 mEq over 24 hours, B. 60 mEq over 24 hours, C. 80 mEq over 24 hours, D. 120-160 mEq over 24 hours",D: 120-160 mEq over 24 hours,"Answer: D: 120-160 mEq over 24 hours Explanation: Ans. d. 120-160 mEq over 24 hoursExcept in unusual circumstances, the total amount of potassium administered daily should not exceed 200 mEq.Potassium SupplementationA prudent protocol to follow is to add potassium chloride to IV solutions at a final concentration of 40-60 mEq/L and to administer no more than 10-20 mEq/L of potassium per hourExcept in unusual circumstances, the total amount of potassium administered daily should not exceed 200 mEq.IV potassium replacement is indicated for patients with severe hypokalemia and for those who can not take oral supplementation.For severe deficiency, potassium may be given through a peripheral IV line in a concentration that should exceed 40 mEq/L, at the rates upto 40 mEq/L/Hour.Continuous ECG monitoring is indicated and the serum potassium levels should be checked every 3-6 hours.For the initial administration, avoid glucose containing fluid to prevent fuher shift of potassium intothe cells. .Magnesium deficiency also needs to be corrected at the same time, paicularly in refractory hypokalemia" "This is a real-world medical entrance exam question, please give the true answer based on the question and selection. Topic name: None, Subject name: Dental","If a patient has gingival recession, how far should the gutta percha be trimmed back before sealing the coronat area? Select and explain. A. 1 mm below the level of the recession, B. 1 mm below the ACJ, C. 1 mm below the canal opening, D. 1 mm below the roof of pulpal chamber",A: 1 mm below the level of the recession,"Answer: A: 1 mm below the level of the recession Explanation: None" "This is a real-world medical entrance exam question, please give the true answer based on the question and selection. Topic name: None, Subject name: Pharmacology","A 50-year-old male has fever for the past l-week with abdominal ilistention and loss of appetite. It is not responding to antibiotics and antimatarials. widal test is negative but RK39 dipstick test is positive.Which of the following drugs can be used in the above patient? Select and explain. A. Bedaquiline, B. Linezolid, C. Fluconazole, D. Liposomal AmPhotericin B",D: Liposomal AmPhotericin B,"Answer: D: Liposomal AmPhotericin B Explanation: Ans: D. Liposomal AmPhotericin BRef Katzung, I3'h ed, Pg. 901-902RK39 dipstick test is a rapid immunochromatographic test being widely used in the diagnosis of visceral leishmaniasis.Out of the given options Liposomal Amphotericin B is the only drug which is used to treat visceral leishmaniasis." "This is a real-world medical entrance exam question, please give the true answer based on the question and selection. Topic name: None, Subject name: Dental","Which of the following orthodontic wire has least modulus of elasticity? Select and explain. A. NiTi, B. Beta titanium, C. Stainless steel, D. Co-Cr-Ni",A: NiTi,"Answer: A: NiTi Explanation: None" "This is a real-world medical entrance exam question, please give the true answer based on the question and selection. Topic name: None, Subject name: Surgery","Which of the following is common to all types of shock? Select and explain. A. Vasoconstriction, B. Hypotension, C. Sepsis, D. Bradycardia",B: Hypotension,"Answer: B: Hypotension Explanation: None" "This is a real-world medical entrance exam question, please give the true answer based on the question and selection. Topic name: None, Subject name: Gynaecology & Obstetrics","A 28 years old female presented with the history of recurrent aboions, pain in calves for 4 years. Patient is suffering from congenital deficiency of: Select and explain. A. Protein C, B. Thrombin, C. Plasmin, D. Factor XIII",A: Protein C,"Answer: A: Protein C Explanation: Answer- A (Protein C)'Inherited thrombophilia causes both early and late miscarriages due to intravascular thrombosis. Protein C resistance (factor V Leiden mutation) is the most common causeProtein C is the natural inhibitor or coagulation - Dutta" "This is a real-world medical entrance exam question, please give the true answer based on the question and selection. Topic name: AIIMS 2018, Subject name: Pharmacology","Sodium nitroprusside is metabolized to form an active metabolite. This active metabolite of sodium nitroprusside act activation of:- Select and explain. A. Phospholipase A, B. Phospholipase C, C. Guanylate cyclase, D. Protein kinase C",C: Guanylate cyclase,"Answer: C: Guanylate cyclase Explanation: Nitoprusside and nitrates act by production of NO (nitric oxide). NO stimulates guanylate cyclase in smooth muscles which leads to formation of cGMP. cGMP act on smooth muscles to cause vasodilation." "This is a real-world medical entrance exam question, please give the true answer based on the question and selection. Topic name: AIIMS 2018, Subject name: Physiology","Baroreceptor is which type of feedback? Select and explain. A. Negative feedback, B. Positive feedback, C. Both negative and positive, D. Feed forward control",A: Negative feedback,"Answer: A: Negative feedback Explanation: Feedforward mechanism comes into action even before the change has occurred In feedback mechanisms, change occurs in controlled variable & the change is fedback to controller & the controller then takes the action. In negative feedback, the controller does the opposite, the change is negated or error is minimized. In positive feedback mechanism, controller does the same in direction of error & error is amplified. If the BP is increased, baroreceptor acts in a way as to decrease it or vice- versa. Increased pressure in blood vessels containing baroreceptor causes increased impulse firing. Baroreceptor signal enters the nucleus tractus solitarius, inhibit the vasoconstrictor center of the medulla, excite the vagal center. The net effects are inhibition of sympathetic activity, which causes vasodilation of veins & aerioles, decreased hea rate & strength of hea contraction. Baroreceptor maintain aerial pressure at a relatively constant level during changes in body posture and other daily activites." "This is a real-world medical entrance exam question, please give the true answer based on the question and selection. Topic name: None, Subject name: Microbiology","A patient in an ICCU is on CVP line. His blood culture shows growth of Gram +ve cocci which are catalase positive and coagulase negative. The most likely etiological agent is: Select and explain. A. Staph. aureus, B. Staph. epidermidis, C. Streptococcus pyogenes, D. Enterococcus faecalis",B: Staph. epidermidis,"Answer: B: Staph. epidermidis Explanation: None" "This is a real-world medical entrance exam question, please give the true answer based on the question and selection. Topic name: None, Subject name: Surgery","A 55 years old male patient presents with 4 cm x 5 cm lump in right neck. FNAC assessment revealed it to be a squamous cell carcinoma. Clinical assessment of the oral cavity, pharynx, hypopharynx and larynx did not yield any tumor. Whole body PET scan did not show any increased uptake except for the neck mass. A diagnosis of unknown primary was made. According to AJCC system of classification, the TNM status of the tumor would be: Select and explain. A. T1N2M0, B. TON2aMO, C. T1N2aMO, D. TxN2aMx",D: TxN2aMx,"Answer: D: TxN2aMx Explanation: Answer- D (TxN2aMx)Tx: Unknown primary (T status cannot be assessed)N2a: Metastasis in single ipsilateral LN, >3 cm but 56 cm in greatest diinension.Mx: Unknown presence or absence of metastasis" "This is a real-world medical entrance exam question, please give the true answer based on the question and selection. Topic name: None, Subject name: Surgery","A patient has an infectious process associated with maxillary right Lateral incisor that tooth is deeply carious and non-restorable. His temperature is 102°F. The patient has not been able to chew for the past 24 hours. The right side of the face is swollen. The patient cannot open his right eye, on palpation the area is soft, painful to touch and tissue rebound when palpated; treatment for the patient should be Select and explain. A. Aspiration, B. Antibiotics of Heat only, C. Incision and Drainage, D. Antibiotics, heat and fluid",A: Aspiration,"Answer: A: Aspiration Explanation: None" "This is a real-world medical entrance exam question, please give the true answer based on the question and selection. Topic name: None, Subject name: Ophthalmology","True statements regarding Direct Ophthalmoscopy are all except: Select and explain. A. Image is viual and erect, B. 2 disk diameter field of vision, C. Magnification is 5 times, D. Self-illuminated device",C: Magnification is 5 times,"Answer: C: Magnification is 5 times Explanation: Ans: C. Magnification is 5 times(Ref Parson's 22/e p137-139. 20/c p126-133).In direct ophthalmoscopy magnification = 15 times.In indirect ophthalmoscopy, magnification = 4-5 times.FeaturesDirect ophthalmoscopyIndirect OphthalmoscopyCondensing lensNot requiredRequired (Convex Examination distanceAs close to patient's eye as possibleAt an arm's lengthImageViual, erectdegReal, inveedMagnificationAbout 15 timesdeg4-5 timesdegIlluminationNot so bright, so not useful in hazy Bright, so useful for hazy Area of field in focusAbout 2 disc dioptersdegAbout 8 disc dioptersdegStereopsisAbsentPresentAccessible fundus viewSlightly beyond equatorUp to Ora serrata i.e. PeripheralretinadegExamination through hazy Not possiblePossibledegPatient positionSittingSupineEaseEasy procedure for visualization ofposterior pole of retinaDifficult, require training" "This is a real-world medical entrance exam question, please give the true answer based on the question and selection. Topic name: None, Subject name: Gynaecology & Obstetrics","All of the following puy NiUiugiLai changes are seen in pregnancy except: Select and explain. A. Increased stroke volume, B. Increased cardiac output, C. Increased intravascular volume, D. Increased peripheral resistance",D: Increased peripheral resistance,"Answer: D: Increased peripheral resistance Explanation: Ans. d. Increased peripheral resistance(REF-Dutta6/e pg 53,51)During the physiological course of pregnancy, maternal peripheral vascular resistance decreases.""Pregnancy is a state of hypervolemia. There is active retention of sodium (900 mmol), potassium (350 mmol) and water.""- Dutta 6/e p51" "This is a real-world medical entrance exam question, please give the true answer based on the question and selection. Topic name: None, Subject name: Dental","About acid etching of dentin all are true, except: Select and explain. A. It removes smear layer, B. It expose collagen, C. It opens dentinal tubules, D. It increase surface tension",D: It increase surface tension,"Answer: D: It increase surface tension Explanation: None" "This is a real-world medical entrance exam question, please give the true answer based on the question and selection. Topic name: None, Subject name: Anatomy","Pterion is: Select and explain. A. Is a point of articulation of four skull bones, B. It is a point where 'bregma' and 'lamba' meet, C. It is region of the postero-lateral fontanelle merge, D. Lies deep to the zygomatic arch",A: Is a point of articulation of four skull bones,"Answer: A: Is a point of articulation of four skull bones Explanation: Pterion is the area in the temporal fossa where 4 bones (frontal, parietal, temporal and sphenoid) adjoin each other forming an H-shaped suture.’" "This is a real-world medical entrance exam question, please give the true answer based on the question and selection. Topic name: None, Subject name: Dental","Which of the following instruments are more efficient? Select and explain. A. Stainless steel, B. Carbon steel, C. Both have equal efficacy, D. None",B: Carbon steel,"Answer: B: Carbon steel Explanation: Carbon Steel Carbon steel alloy contains 0.5 to 1.5 percent carbon in iron. Instruments made from carbon steel are known for their hardness and sharpness. Disadvantages with these instruments are their susceptibility to corrosion and fracture. They are of two types: 1. Soft steel: It contains <0.5% carbon 2. Hard steel: It contains 0.5 to 1.5% carbon Stainless Steel Stainless steel alloy contains 72 to 85 percent iron, 15 to 25 percent chromium and 1 to 2 percent carbon. Instruments made from stainless steel remain shiny bright because of deposition of chromium oxide layer on the surface of the metal which reduces the tendency to tarnish and corrosion. Problem with stainless steel instruments is that they tend to lose their sharpness with repeated use, so they need to be sharpened again and again. ​Nisha Garg, Amit Garg. Textbook of Operative Dentistry.  Edition 3. Page: 80" "This is a real-world medical entrance exam question, please give the true answer based on the question and selection. Topic name: None, Subject name: Gynaecology & Obstetrics","Transmission of HIV from mother to child is prevented by all the following except: Select and explain. A. Oral zidovudine to mother at 3rd trimester along with oral zidovudine to infant for 6 weeks, B. Vitamin A prophylaxis to mother, C. Vaginal delivery, D. Stopping breast feed",C: Vaginal delivery,"Answer: C: Vaginal delivery Explanation: None" "This is a real-world medical entrance exam question, please give the true answer based on the question and selection. Topic name: None, Subject name: Pathology","Persons with the greatest risk of oral cancer Select and explain. A. Have a poor oro-dental hygiene, nutritional deficiencies and are chronic alcoholics, B. Are middle aged, have a poor oro-dental hygiene and regularly use tobacco, C. Are old, Chronic alcoholics and regularly use tobacco, D. Are young, have poor oro-dental hygiene and are heavy smokers","C: Are old, Chronic alcoholics and regularly use tobacco","Answer: C: Are old, Chronic alcoholics and regularly use tobacco Explanation: None" "This is a real-world medical entrance exam question, please give the true answer based on the question and selection. Topic name: AIIMS 2019, Subject name: Radiology","A 10 year old presented to ohopaedic OPD with a Curved Back, AP and Lateral Radiographs are obtained, what is the most likely cause of such deformity? Select and explain. A. Congenital, B. Neurofibromatosis, C. Idiopathic, D. Neuromuscular",A: Congenital,"Answer: A: Congenital Explanation: - In the given A-P view X-ray we could see the curvature of the spine and the presence of Hemiveebrae indicating it as a case of Scoliosis with congenital origin 3D CT image & Radiograph image showing Hemiveebrae SCOLIOSIS:* Scoliosis is defined as a lateral spinal curvature with a cobb angle of 10o or more* A curve with a Cobb angle of less than 10o is asymptomatic and does not progress; this state is known as spinal asymmetry, not Scoliosis * Cobb's angleo The angle formed by meeting of the two parallel tangential lines drawn from End Veebrae is known as Cobb's angleo Apex veebra - Fahest veebra from the spineo End veebra - Veebra that is tilted the mosto Neutral veebra - Veebra that is tilted the leasto Stable veebra - Veebra that is in line with the central sacral line * Levoscoliosis: Curvature towards the left* Dextroscoliosis: Curvature towards the right* Major/Main curvature: Curvature formed by the deformity* Minor curvature: It is a compensatory curvature Main Class and Subtype Demographic and Clinical Characteristics Idiopathic Infantile Occurs in the first 3 years of life; male preponderance; levoscoliosis is more common than dextroscoliosis Juvenile Occurs at age 4-10 years; female preponderance; dextroscoliosis is more common than levoscoliosis Adolescent Occurs at age 10-18 years; female preponderance; dextroscoliosis is more common than levoscoliosis Congenital Osteogenic Wedge - shaped veebrae, hemiveebrae, fused veebrae, unilateral bar Neuropathic Tethered cord, syringomyelia, Chiari malformation, (myelo) meningocele, diastematomyelia Developmental Skeletal dysplasia Achondroplasia Skeletal dyostosis Neurofibromatosis, osteogenesis imperfecta Neuromuscular Neuropathic (acquired) Cerebral palsy, spinocerebellar, degeneration, poliomyelitis Myopathic Muscular dystrophy of various types (eg, Duchenne dystrophy) Tumor - associated Osseous Osteoid osteoma, osteoblastoma Extraosseous Extramedullary (eg, neurofibroma) or intramedullary (eg, astrocytoma) tumor" "This is a real-world medical entrance exam question, please give the true answer based on the question and selection. Topic name: None, Subject name: Biochemistry","Essential fatty acid present in breast milk which is required for normal growth of child is: Select and explain. A. Lenoleic acid, B. Palmitic acid, C. Docosahexanoic acid, D. EPA",C: Docosahexanoic acid,"Answer: C: Docosahexanoic acid Explanation: DOCOSA HEXAENOIC ACID (DHA) Sources: Human milk, Fish liver oils, Algal oils Synthesized in the body from α Linolenic acid. Highest  concentration  of  DHA  found  in  retina,  cerebral  cortex, sperms. Functions: Needed for the development of fetal brain and retina DHA is supplied transplacentally and through breast milk. Clinical significance: Low DHA is associated with increased risk of Retinitis Pigmentosa." "This is a real-world medical entrance exam question, please give the true answer based on the question and selection. Topic name: None, Subject name: Microbiology","In which of the following ways is CLED medium better than MacConkey agar? Select and explain. A. It prevents proteus swarming, B. Inhibits growth of other commensals, C. It allows Staphylococcus and Candida to grow, D. It differentiates lactose fermenters from non-fermenters",C: It allows Staphylococcus and Candida to grow,"Answer: C: It allows Staphylococcus and Candida to grow Explanation: Ans: C. It allows Staphylococcus and Candida to growRef: Ananthanarayan 10/e p39, 10/e p39, 81e p667)Advantage of CLED medium over MacConkey agar is that it allows Staphylococcus and Candida to growTypical colony morphology on CLED AgarEscherichia coliOpaque yellow colonies with a slightly deeper yellow centerdegKlebsiella sppYellow to whitish-blue colonies, extremely mucoiddegProteus sppTranslucent blue coloniesdegPseudomonas aeruginosaGreen colonies with typical matted surface & rough peripherydegEnterococciSmall yellow colonies, about 0.5 mm in diameterdegStaphylococcus aureusDeep yellow colonies, uniform in colordegCoagulase negative StaphylococciPale yellow colonies, more opaque than Enterococcus faecalisa" "This is a real-world medical entrance exam question, please give the true answer based on the question and selection. Topic name: None, Subject name: Social & Preventive Medicine","The haemoglobin levels of pregnant females in a community was found to have a mean of 10.6 g/d1, and a standard detion of 2 g/dL. What is the minimum haemoglobin level below which 5% of the pregnant females will have their haemoglobin levels? Select and explain. A. 7.32, B. 8.64, C. 6.68, D. 8.96",C: 6.68,"Answer: C: 6.68 Explanation: Ans. c. 6.68" "This is a real-world medical entrance exam question, please give the true answer based on the question and selection. Topic name: None, Subject name: Pathology","Fibrous histiocytoma involves most commonly: Select and explain. A. Eyelid, B. Epibular, C. Intraocular, D. Orbit",D: Orbit,"Answer: D: Orbit Explanation: Fibrous histiocytoma is the most common primary mesenchymal orbital tumor in adults.  It may involve ocular structures such as the orbit, lids, conjunctiva, and ocular limbus.  Most fibrous histiocytomas are benign, but some are locally aggressive and malignant." "This is a real-world medical entrance exam question, please give the true answer based on the question and selection. Topic name: None, Subject name: Dental","Treatment of acute herpetic gingivostomatitis include : Select and explain. A. Corticosteroids to reduce inflammation, B. Penicillin, C. Local antibiotic application, D. None of the above",D: None of the above,"Answer: D: None of the above Explanation: None" "This is a real-world medical entrance exam question, please give the true answer based on the question and selection. Topic name: None, Subject name: Anatomy","Primary cartilaginous joint is called as: Select and explain. A. Symphyses., B. Synchondrosis., C. Syndesmosis., D. Synarthroses.",B: Synchondrosis.,"Answer: B: Synchondrosis. Explanation: None" "This is a real-world medical entrance exam question, please give the true answer based on the question and selection. Topic name: None, Subject name: Pathology","Biopsy from an eight-year-old child with leg swelling showed smallround blue tumor cells consistent with diagnosis of Ewing's sarcoma. What will be the best method to detect translocation t(ll;22) in this malignancy? Select and explain. A. Conventional karyotyping, B. Next generation sequencing, C. FISH, D. PCR",C: FISH,"Answer: C: FISH Explanation: Ans. C. FISHFISH is a molecular cytogenetic technique that uses fluorescent probes that bind to only those pas of the chromosome with a high degree of sequence complementarity." "This is a real-world medical entrance exam question, please give the true answer based on the question and selection. Topic name: None, Subject name: Surgery","A 7 years old child comes with fever and tibial swelling exhibits on X- ray exhibits periosteal reaction. Laboratory results show raised ESR and TLC. What is the next step in diagnosis of the patient? Select and explain. A. MRI, B. Pus culture, C. Bone biopsy, D. Blood culture",A: MRI,"Answer: A: MRI Explanation: Answer- A. MRIEwingts sarcoma:MRI is essential to elucidate soft-tissue involvement in Ewing's sarcoma, because the tumor has low signal intensity on Tl-weighted images compared with the normal high signal intensity of the bone marow.Osteomyelitis:MRI allows early detection of osteomyelitis and assessment of the extent of involvement and the activity of the disease in cases of chronic bone infection.It is considered the most useful imaging technique to evaluate suspected osteomyelitis because of its ability to demonstratechanges in the water content ofbone marrow with an excellent structural definition and spatial resolution." "This is a real-world medical entrance exam question, please give the true answer based on the question and selection. Topic name: None, Subject name: Dental","Which of the following organisms is NOT implicated in the etiology of Periodontal disease Select and explain. A. Bacteroides, B. Wolinella, C. Neisseria, D. Eikenella",C: Neisseria,"Answer: C: Neisseria Explanation: None" "This is a real-world medical entrance exam question, please give the true answer based on the question and selection. Topic name: None, Subject name: Medicine","Which of the following is not true about the salivary gland output in Sjogren's syndrome? Select and explain. A. Increase in sodium concentration, B. Increase in phosphate concentration, C. Decreased output of salivary glands <0.5 ml/min, D. Increase in IgA concentration",B: Increase in phosphate concentration,"Answer: B: Increase in phosphate concentration Explanation: Ans: B. Increase in phosphate concentration(Ref Harrison 19/e p2166; Oxford Textbook of Rheumatology 4/e p1049, Textbook of Oral & Maxillofacial Surgery (Elsevier)/402)Phosphate concentration of saliva is decreased in Sjogren's syndrome.Sjogren's syndrome:Increased sodium & chloride concentration.Decreased phosphate concentrations.Increased IgA.Decreased salivary flow rate." "This is a real-world medical entrance exam question, please give the true answer based on the question and selection. Topic name: None, Subject name: Medicine","While measuring blood pressure of a cardiac tamponade patient, what advice will you give him: Select and explain. A. Hold breath, B. Breathe normally, C. I.ong and deep breathing, D. Sho and shallow breaths",B: Breathe normally,"Answer: B: Breathe normally Explanation: Ans. b. Breath normallyParadoxus requires the patient to breathe normally and does not require any special maneuver.BP is best measured in the seated position with the arm at the level of thehea, using an appropriately sized cuff, after 5-10 minutes of relaxation.In Supine position: The arm should be raised to bring it to level of mid-tight atrium." "This is a real-world medical entrance exam question, please give the true answer based on the question and selection. Topic name: AIIMS 2017, Subject name: Pediatrics","Hutchinson's triad includes? Select and explain. A. Interstitial keratitis, malformed molars, conductive deafness, B. Interstitial keratitis, malformed molars, sensorineural deafness, C. Conductive deafness, malformed incisors, interstitial keratitis, D. Interstitial keratitis, malformed molars, conductive deafness","B: Interstitial keratitis, malformed molars, sensorineural deafness","Answer: B: Interstitial keratitis, malformed molars, sensorineural deafness Explanation: Congenital syphilis is characterized by Hutchinson's triad which includes- 1) Interstitial keratitis 2) Malformed teeth 3) SNHL" "This is a real-world medical entrance exam question, please give the true answer based on the question and selection. Topic name: None, Subject name: Surgery","Check point for ZMC fracture reduction is: Select and explain. A. Intraorbital margin, B. Fronto-zygomatic suture region, C. Zygomatico-maxillary buttress region, D. Symmetry of arch",A: Intraorbital margin,"Answer: A: Intraorbital margin Explanation: None" "This is a real-world medical entrance exam question, please give the true answer based on the question and selection. Topic name: None, Subject name: Pharmacology","Stoppage of which of the following drugs can result in anxiety and insomnia causing discontinuation syndrome? Select and explain. A. Venlafaxine, B. ImiPramine, C. Valproate, D. OlanzaPine",A: Venlafaxine,"Answer: A: Venlafaxine Explanation: Ans: A. VenlafaxineRef: Katzung, I3'h ed., pg. 524-525 and Goodman & Gilman, l3'h ed-, pg. 27372.Sudden withdrawal of Shoer acting antidepressants paicularly SSRIs (paroxetine and seraline) and SNRI (venlafaxine) can lead to discontinuation syndrome and troublesome side effects.These symptoms may include dizziness, headache, nervousness, nausea, anxiety and insomnia.Out of the options provided Venlafaxine is the best answer." "This is a real-world medical entrance exam question, please give the true answer based on the question and selection. Topic name: None, Subject name: Dental","Migration of odontoblasts into dentinal tubules occurs during __________ : Select and explain. A. Cavity preparation, B. Dehydration, C. Water irrigation, D. None of the above",A: Cavity preparation,"Answer: A: Cavity preparation Explanation: None" "This is a real-world medical entrance exam question, please give the true answer based on the question and selection. Topic name: None, Subject name: Dental","A patient complains of sensitivity in a Mandibular premolar. A well condensed class V amalgam restoration was placed in that tooth 5 months ago, with no discomfort for the first 4 months. Since then it has become increasingly sensitive. The problem probably relates to: Select and explain. A. Marginal leakage, B. Fractured restoration, C. Irreversible pulpal damage, D. Tooth brush abrasion",C: Irreversible pulpal damage,"Answer: C: Irreversible pulpal damage Explanation: None" "This is a real-world medical entrance exam question, please give the true answer based on the question and selection. Topic name: None, Subject name: Dental","The function of reciprocal clasp arm is: Select and explain. A. a) To balance the denture, B. b) To act as a direct retainer, C. c) To counteract the movement of denture which is caused during engagement of retentive arm, D. d) None of the above",C: c) To counteract the movement of denture which is caused during engagement of retentive arm,"Answer: C: c) To counteract the movement of denture which is caused during engagement of retentive arm Explanation: None" "This is a real-world medical entrance exam question, please give the true answer based on the question and selection. Topic name: None, Subject name: Surgery","Which score is used for wound infection Select and explain. A. Glasgow coma scale, B. Southampton score, C. Apgar score, D. SIRS score",B: Southampton score,"Answer: B: Southampton score Explanation: Answer- B. Southampton scoreSouthampton score and ASEPSIS criteria are used for wound infection.There are scoring systems for the severity of wound infection, which are paicularly useful in surveillance and research.Examples are the Southampton and ASEPSIS systemsGradeAppearance0Normal healingINormal healing with mild bruising or erythemaIaSome bruisingIbConsiderable bruisingIcMild erythemaIIErythema plus other signs of inflammationIIaAt one pointIIbAround suturesIIcAlong woundIIdAround woundIIIClear or haemoserous dischargeIIIaAt one point only (IIIbAlong wound (>2 cm)IIIcLarge-volumeIIIdProlonged (>3 days) Major complicationsIVPusIVaAt one point only (IVbAlong wound (>2 cm)VDeep or severe wound infection with or without tissuebreakdown; hematoma requiring aspiration" "This is a real-world medical entrance exam question, please give the true answer based on the question and selection. Topic name: None, Subject name: Surgery","Hypertonic solution is used in all except Select and explain. A. Prolonged bowel surgery, B. Cardiac type IV shock, C. Brain injury, D. Burn",B: Cardiac type IV shock,"Answer: B: Cardiac type IV shock Explanation: None" "This is a real-world medical entrance exam question, please give the true answer based on the question and selection. Topic name: None, Subject name: Dental","Vulcanization refers to the setting of: Select and explain. A. Reversible hydrocolloid, B. Mercaptan impression material, C. Zinc phosphate cement, D. Zinc oxide-eugenol",B: Mercaptan impression material,"Answer: B: Mercaptan impression material Explanation: None" "This is a real-world medical entrance exam question, please give the true answer based on the question and selection. Topic name: None, Subject name: Pathology","Diabetes insipidus, bone lesions and exophthalmos is seen in Select and explain. A. Sickle cell anemia, B. Niemen pick disease, C. Littere Siewe Disease, D. Hand-Schuller-Christian disease",D: Hand-Schuller-Christian disease,"Answer: D: Hand-Schuller-Christian disease Explanation: None" "This is a real-world medical entrance exam question, please give the true answer based on the question and selection. Topic name: None, Subject name: Pharmacology","What are the appropriate instructions to be given while prescribing bisphosphonates to a patient: Select and explain. A. To be given empty stomach with a glass of water, B. Taken along with food, C. Stop if features of gastritis develop, D. Stop if bone pains occur.",A: To be given empty stomach with a glass of water,"Answer: A: To be given empty stomach with a glass of water Explanation: Ans: A. To be given empty stomach with a glass of water(Ref: Gilman 12/e p1296; Katzung 13/e p754. 12/e p776; KDT 7/e p344, 6/e p334; 19/e p2.Esophageal irritation:Minimized by taking the drug with full glass of water and remaining upright for 30 minutes or by using the intravenous forms of these compounds.GI side effects:Oral bisphosphonates (including alendronate, ibandronate & risedronate) - Cause heaburn, esophageal irritation /esophagitis.Also cause abdominal pain & diarrhea.Symptoms often abate when patients take the medication after an overnight fast, with tap or filtered water (not mineral water), and remain upright." "This is a real-world medical entrance exam question, please give the true answer based on the question and selection. Topic name: None, Subject name: Dental","Which tooth is known as enigma to endodontist Select and explain. A. Mandibular canine, B. Maxillary 1st premolar, C. Mandibular 2nd premolar, D. Mandibular 1st premolar",D: Mandibular 1st premolar,"Answer: D: Mandibular 1st premolar Explanation: None" "This is a real-world medical entrance exam question, please give the true answer based on the question and selection. Topic name: None, Subject name: Radiology","Which of the following are most sensitive to X-ray? Select and explain. A. Tooth buds and salivary glands, B. Nerve and muscle tissue, C. Hair and nails, D. Cartilage",A: Tooth buds and salivary glands,"Answer: A: Tooth buds and salivary glands Explanation: None" "This is a real-world medical entrance exam question, please give the true answer based on the question and selection. Topic name: None, Subject name: Microbiology","A type of immediately occurring reaction in which antigens combine with antibodies already attached to the surface of mast cells and basophils is called: Select and explain. A. Type I hypersensitivity, B. Type II hypersensitivity, C. Type III hypersensitivity, D. Type IV hypersensitivity",A: Type I hypersensitivity,"Answer: A: Type I hypersensitivity Explanation: None" "This is a real-world medical entrance exam question, please give the true answer based on the question and selection. Topic name: None, Subject name: Microbiology","Toxic shock syndrome is due to the following virulence factor: Select and explain. A. M protein, B. Pyrogenic exotoxin, C. Streptolysin 0, D. Carbohydrate cell wall",B: Pyrogenic exotoxin,"Answer: B: Pyrogenic exotoxin Explanation: None" "This is a real-world medical entrance exam question, please give the true answer based on the question and selection. Topic name: None, Subject name: Dental","Which surface of tooth has most of plaque? Select and explain. A. Gingival third of tooth surface, B. Incisal one third of tooth surface, C. Middle third of tooth surface, D. Uniform on all surfaces",A: Gingival third of tooth surface,"Answer: A: Gingival third of tooth surface Explanation: None" "This is a real-world medical entrance exam question, please give the true answer based on the question and selection. Topic name: None, Subject name: Anatomy","Epidural venous plexus is located in Select and explain. A. Basal ganglia adjacent to pons, B. At the junction of middle and posterior cranial fossa, C. In vertebral canal below duramater, D. In vertebral canal above duramater",D: In vertebral canal above duramater,"Answer: D: In vertebral canal above duramater Explanation: None" "This is a real-world medical entrance exam question, please give the true answer based on the question and selection. Topic name: AIIMS 2019, Subject name: Orthopaedics","A young marathon runner is paicipating in a marathon competition. After running for 100 m, he develops at the anteromedial aspect of tibia which was mild to sta with, but increased on fuher running. X ray was normal. The doctor ordered a bonescan. What is the likely diagnosis? Select and explain. A. Lisfranc fracture, B. Jones fracture, C. Shin splint, D. Nutcracker fracture",C: Shin splint,"Answer: C: Shin splint Explanation: Shin splints (stress fracture) are seen in runners, army recruits or any vigorous physical activity. STRESS/FATIGUE FRACTUREStress fracture is due to imbalance between load and resistance of bone. It is of 2 types:1. Fatigue Fracture: caused by application of abnormal stress on normal bone.2. Insufficiency Fracture: caused by normal activity on weak bone. Sites of Stress FracturesLower Extremity* March fracture is a stress fatigue fracture of second metatarsal neck > 3rd metatarsal neck.* The most common site is metatarsal neck followed by tibia (proximal third in children, middle third in athlete and lower third in elderly).* Tibia stress fracture = Shin splints* Femoral neck (inferomedial compression side in young and superior tension side in older patients).* Rarely fibula lower end (runners fracture). Upper Extremity* Olecranon is most common site of upper limb stress fractures. Pelvis and SpinePars inter aicularis of 5th lumbar veebral (causing spondylolysis) is commonest in spine. Clinical Presentation* Load related pain often bilateral* The hallmark physical finding is tenderness with palpation and stress. InvestigationMRI provide excellent sensitivity and superior specificity compared to bone scan in differentiating from infections or tumors.Bone scan is preferable for bilateral cases due to feasibility, also bilateral cases go in our of stress fracture as compared to Infection or tumor and also can scan the whole body.* Treatment is symptomatic with cast and cessation of activity." "This is a real-world medical entrance exam question, please give the true answer based on the question and selection. Topic name: None, Subject name: Dental","In which tooth all the 8 types of Vertucci seen Select and explain. A. 1st maxillary premolar, B. 2nd maxillary premolar, C. 1st mandibular premolar, D. 2nd mandibular premolar",B: 2nd maxillary premolar,"Answer: B: 2nd maxillary premolar Explanation: None" "This is a real-world medical entrance exam question, please give the true answer based on the question and selection. Topic name: None, Subject name: Skin","A man presents it n rashes on face and also complains of decreased mental function. He is also having few macular lesions on his skin. On CT scan, intracranial calcification was seen. His wife is normal. His 10-year old daughter is also normal but his 6-year old son is also having similar skin lesions. What would be the most likely diagnosis? Select and explain. A. Neurofibrornatosis-1, B. Neurofibromatosis-2, C. Xeroderma pigmentosum, D. Autosomal dominant inheritance",D: Autosomal dominant inheritance,"Answer: D: Autosomal dominant inheritance Explanation: Answer- D. Autosomal dominant inheritanceClinical findings like rashes on face, decreased mental function with macular lesions on the skin and intracranial calcification on CT scan is highly suggestive of tuberous sclerosis, which has an autosomal dominant condition." "This is a real-world medical entrance exam question, please give the true answer based on the question and selection. Topic name: None, Subject name: Pharmacology","What causes malignant hyperpyrexia? Select and explain. A. Thiopentone, B. Cisatracurium, C. Propofol, D. Suxamethonium",D: Suxamethonium,"Answer: D: Suxamethonium Explanation: None" "This is a real-world medical entrance exam question, please give the true answer based on the question and selection. Topic name: None, Subject name: Dental","Force generated by 90° rotation of finger spring in pendulum appliance with distalisation: Select and explain. A. 75 gms, B. 125 gms, C. 150 gms, D. 200 gms",D: 200 gms,"Answer: D: 200 gms Explanation: None" "This is a real-world medical entrance exam question, please give the true answer based on the question and selection. Topic name: None, Subject name: Dental","All of the following are true about bacteriocin except: Select and explain. A. It has no role in adherence of plaque to the tooth surface, B. It can be used in therapy of dental caries, C. It is a protein, D. It enhances the growth of other bacteria",D: It enhances the growth of other bacteria,"Answer: D: It enhances the growth of other bacteria Explanation: None" "This is a real-world medical entrance exam question, please give the true answer based on the question and selection. Topic name: None, Subject name: Pathology","White rough pedunculated lesion on palate is most likely: Select and explain. A. Pleomorphic adenoma, B. Papilloma, C. Nevus, D. Fibroma",B: Papilloma,"Answer: B: Papilloma Explanation: None" "This is a real-world medical entrance exam question, please give the true answer based on the question and selection. Topic name: None, Subject name: Dental","Advantage of wide palatal bar over narrow bar is: Select and explain. A. Less discomfort, B. Less bulk, C. More tough, D. More tissue coverage (area of basal seat)",B: Less bulk,"Answer: B: Less bulk Explanation: None" "This is a real-world medical entrance exam question, please give the true answer based on the question and selection. Topic name: None, Subject name: Surgery","Which of these is pierced during posterior superior alveolar nerve Block? Select and explain. A. Medial Pterygoid, B. Lateral Pterygoid, C. Buccinator, D. Masseter",C: Buccinator,"Answer: C: Buccinator Explanation: None" "This is a real-world medical entrance exam question, please give the true answer based on the question and selection. Topic name: AIIMS 2017, Subject name: Anaesthesia","Which of the following is used in the induction of anesthesia:- Select and explain. A. Bupivacaine, B. Dexmediatomidine, C. Lorazepam, D. Neostigimine",C: Lorazepam,"Answer: C: Lorazepam Explanation: Bupivacaine - Local anesthetic Dexmedetomidine - Anesthesia adjuvant Lorazepam - Anesthetic agent for induction Neostigmine - Reversal agent" "This is a real-world medical entrance exam question, please give the true answer based on the question and selection. Topic name: None, Subject name: Dental","A suitable fixative for a routine biopsy specimen is Select and explain. A. 10% formalin, B. 40% formalin, C. 20% formalin, D. 10% alcohol",A: 10% formalin,"Answer: A: 10% formalin Explanation: None" "This is a real-world medical entrance exam question, please give the true answer based on the question and selection. Topic name: None, Subject name: Gynaecology & Obstetrics","Pre eclampsia with torsades de pointes, stable vitals. The next best management will be- Select and explain. A. DC shock, B. MgSo4, C. IV Calcium, D. None",B: MgSo4,"Answer: B: MgSo4 Explanation: Ans. B. MgSo4MgSo4 is Indicated to prevent seizures associated with pre-eclampsia, and for control of seizures with eclampsia4-5 g (diluted in 250 mL NS/D5W) IV in combination with either (a) up to 10 g (20 mL of undiluted 50% solution) divided and administered IM into each buttock or (b) after initial IV dose, 1-2 g/hr IV; (c) May administer q. 4hr as necessary." "This is a real-world medical entrance exam question, please give the true answer based on the question and selection. Topic name: None, Subject name: Surgery","SCC present on lateral margin of tongue of staging T2N0, options of treatment is Select and explain. A. SOHND (supraomohyoid neck dissections), B. Remove Lymph node I-II along with, C. Remove Lymph node I-V along with, D. Remove Lymph node I-IV along with",D: Remove Lymph node I-IV along with,"Answer: D: Remove Lymph node I-IV along with Explanation: None" "This is a real-world medical entrance exam question, please give the true answer based on the question and selection. Topic name: None, Subject name: Anatomy","All of the following are branches of external carotid aery except? Select and explain. A. Superior thyroid aery, B. Transverse cervical aery, C. Ascending pharyngeal aery, D. Superficial temporal aery",B: Transverse cervical aery,"Answer: B: Transverse cervical aery Explanation: Ans: B Transverse cervical aeryBranches of External Carotid AnteriorSuperior thyroid aerydegLingual aeryFacial aeryPosteriorOccipital aerydegPosterior auriculardegMedialAscending pharyngeal aerydegTerminalMaxillary aerydegSuperficial temporal aeryTransverse cervical aery is a branch of thyrocervical trunk." "This is a real-world medical entrance exam question, please give the true answer based on the question and selection. Topic name: None, Subject name: Ophthalmology","A patient gives a history of chuna falling into his eyes. All of the following would be a pa of his immediate management except: Select and explain. A. Irritation of both eyes with copious amounts of normal saline, B. Frequent sodium citrate drops, C. Thorough slit-lamp examination on presentation, D. Removal of chuna paicles by double eversion of upper eyelids",C: Thorough slit-lamp examination on presentation,"Answer: C: Thorough slit-lamp examination on presentation Explanation: Ans. c. Thorough slit-lamp examination on presentationImmediate Mananement of Chemical (Chuna-Powder) Injury: Irrigation of both eyes with copious amounts of normal saline Removal of chuna paicles by double eversion of upper eyelidsLids and fornices should be examined with double eversion of the lids and lime paicles should be removed with forceps; a cotton tipped applicator soaked in EDTA 1%deg.""Chuna (strong alkali material) is used with tobacco. Bursting of chuna packet is one of the most common modes of ocular injury. Thorough slit lamp examination is not possible at presentation because of severe pain and blepharospasm. Thorough irrigation with saline and removal of chemical is most impoant. Irrigation of the cornea/conjunctiva/fornix with the lids held open with a speculum should be done until pH of tears shows neutrality. Lids and fornices should be examined with double eversion of the lids and lime paicles should be removed with forceps; a cotton tipped applicator soaked in EDTA 1%.""- Yanoff and Ducker 's Ophthalmology; 3rd/e p349Chemical (Chuna-Powder) InjuryChuna (strong alkali material) is used with tobacco.Bursting of chuna packet is one of the most common modes of ocular injuryQ.Alkalis are more injurious to eye in comparison to acids as acids cause protein coagulation, which prevent fuher penetration of the chemical.Thorough slit lamp examination is not possible at presentation because of severe pain and blepharospasmQ.Thorough irrigation with saline and removal of chemical is most impoantdeg.Emergency Management of Chemical injury:Since duration of contact with chuna/chemical determines the extent of subsequent injury and prognosis, immediate copious irrigation is of the most impoanceQ.Irrigation of the cornea/conjunctiva/fornix with the lids held open with a speculum should be done until pH of tears shows neutralityQ.Lids and fornices should be examined with double eversion of the lids and lime paicles should be removed with forceps; a cotton tipped applicator soaked in EDTA 1%Q.If an acid burn is suspected, a base should not be used for irrigation in an effo to neutralize the acid.After copious irrigation, necrotic corneal epithelium should be debrided to promote re-epithelializationQ" "This is a real-world medical entrance exam question, please give the true answer based on the question and selection. Topic name: None, Subject name: Dental","A 8-years-old patient comes to you with malocclusion what will be your line of treatment: Select and explain. A. Wait and watch till canine erupts, B. Extraction a and orthodontic treatment, C. Serial extraction, D. Immediate intervention with removal appliance",A: Wait and watch till canine erupts,"Answer: A: Wait and watch till canine erupts Explanation: None" "This is a real-world medical entrance exam question, please give the true answer based on the question and selection. Topic name: None, Subject name: Dental","Which of the following zone is characterized by macrophages and osteoclasts? Select and explain. A. Zone of infection, B. Zone of contamination, C. Zone of irritation, D. Zone of stimulation",C: Zone of irritation,"Answer: C: Zone of irritation Explanation: The reaction of the periradicular tissues to noxious products of tissue necrosis, bacterial products and antigenic agents from the root canal has been described by Fish. The four zones  Zone of infection Zone of contamination Zone of irritation Zone of stimulation Zone of Infection: This is present in the center of the lesion. It is characterized by PMNL's. Zone of Contamination: It is characterized by round cell infiltration. It demonstrates cellular destruction due to toxins from the central zone or zone of infection. Because of autolysis and death of bone cells, the lacunae were empty. Lymphocytes are prevalent everywhere. Zone of Irritation: This zone is characterized by macrophages and osteoclasts. The collagen framework was digested by phagocytic cells, the macrophages, while osteoclasts attacked the bone tissue. This opens a gap in the bone all around the center of lesion. That space becomes filled with PMNL's. It contains cholesterol crystals. This zone demonstrates much activity preparatory to repair. Zone of Stimulation: (Peripheral zone) This zone is characterized by fibroblasts and osteoblasts. In this zone, the effects of toxins were mild enough to be stimulant which results in laying down of collagen fibers around the zone of irritation that acts both as a wall of defense and as a scaffolding on which the osteoblasts built new bone. This new bone was built in an irregular fashion." "This is a real-world medical entrance exam question, please give the true answer based on the question and selection. Topic name: None, Subject name: Medicine","Not a feature of infective endocarditis: Select and explain. A. Myocardial abscess., B. Vegetation along cusps., C. Thrombus in left atria., D. Perforation of cusp.",C: Thrombus in left atria.,"Answer: C: Thrombus in left atria. Explanation: None" "This is a real-world medical entrance exam question, please give the true answer based on the question and selection. Topic name: None, Subject name: Dental","Basically TMJ is a: Select and explain. A. Ball and socket joint, B. Hinge and axis joint, C. Synovial joint, D. Diarthrodal joint",D: Diarthrodal joint,"Answer: D: Diarthrodal joint Explanation: None" "This is a real-world medical entrance exam question, please give the true answer based on the question and selection. Topic name: None, Subject name: Gynaecology & Obstetrics","A pregnant lady with persistent late, variable deceleration with cervical dilatation of 6 cm shifted to OT for surgery. Which of the following is not done in Mmn: Select and explain. A. Supine position, B. O2 inhalation, C. I.V. fluid, D. Subcutaneous terbutaline",A: Supine position,"Answer: A: Supine position Explanation: “Deceleration is defined as a decrease in fetal heart rate below the base line of 15 beats per minute or more.” Variable deceleration is seen in case of cord-compression/prolapse. In case of cord-compression/prolapse patient should not be allowed to rest in supine position as it will lead to more pressure on the cord. In cord prolapse the patient is allowed to rest in exaggerated elevated Sims position with a pillow under the hip. Management of Non-reassuring fetal status (Fetal Distress) Lateral positioning avoids compression of vena cava and aorta by the gravid uterus. This increases cardiac output and uteroplacental perfusion. Oxygen is administered to the mother with mask to improve fetal SaO2. Correction of dehydration by IV fluids (crystalloids) improves intravascular volume and uterine perfusion. Correction of maternal hypotension (following epidural analgesia) with immediate infusion of 1 litre of Crystalloid (Ringer’s solution). Stoppage of oxytocin to improve fetal oxygenation. Fetal hypoxia may be due to strong and sustained uterine contractions. With reassuring FHR and in absence of fetal acidemia, oxytocin may be restarted. Tocolytic (Injection terbutaline 0.25 mg S.C.) is given when uterus is hypertonic and there is nonreassuring FHR. Amnioinfusion is the process to increase the intrauterine fluid volume with warm normal saline (500 ml). Indications are: – Oligohydramnios and cord compression – To dilute or to wash out meconium – To improve variable or prolonged decelerations. Advantages : Reduces cord compression, meconium aspiration, and improves Apgar score." "This is a real-world medical entrance exam question, please give the true answer based on the question and selection. Topic name: None, Subject name: Dental","Principle secretory cells found in the single layer outlining the pulp (Ourtermost zone of pulp) Select and explain. A. Fibroblasts, B. Odontoblast, C. Ameloblast, D. Undifferentiated cells",B: Odontoblast,"Answer: B: Odontoblast Explanation: The outermost odontoblast layer,(Single layer outlining pulp ) contains odontoblasts as principal secretory cells for Dentin while they also secrete Collagen Type 1 and type 5 collagen fibers" "This is a real-world medical entrance exam question, please give the true answer based on the question and selection. Topic name: None, Subject name: Surgery","A dentist planning to remove an impacted tooth will need which of the following radiographs: Select and explain. A. A periapical film will be all that is required, B. A bite-wing film will be of little or no use, C. A lateral view will be all that is required, D. An anteroposterior view is all that is required",B: A bite-wing film will be of little or no use,"Answer: B: A bite-wing film will be of little or no use Explanation: None" "This is a real-world medical entrance exam question, please give the true answer based on the question and selection. Topic name: None, Subject name: Pharmacology","Chloroquine related eye disease risk is Select and explain. A. HIV protease, B. HIV reverse transcriptase, C. HIV integrase, D. HIV entry into the cell",B: HIV reverse transcriptase,"Answer: B: HIV reverse transcriptase Explanation: Answer: . B. HIV reverse transcriptase >2 years (Ref Goodman and Gilman 12/e p14051Chloroquine related eye disease risk is high if duration of treatment is more than 5 yearsHigh Risk for Chloroquine Age >60 yearsdegDuration of treatment >5 yearsdegDaily dose >6.5 mg/kg/day of3.0 mg/kg/day of High body fatdegLiver diseasedegRenal diseasedegConcomitant retinal diseasedeg" "This is a real-world medical entrance exam question, please give the true answer based on the question and selection. Topic name: None, Subject name: Dental","Cervical resorption after bleaching is prevented by: Select and explain. A. 80% alcohol drying immediately after bleaching, B. Alpha tocopherol applied immediately after bleaching, C. Sodium perborate + sodium hypochlorite, D. Sodium perborate + calcium hydroxide",D: Sodium perborate + calcium hydroxide,"Answer: D: Sodium perborate + calcium hydroxide Explanation: Cervical resorption is usually painless, until the resorption exposes the pulp, necessitating endodontic therapy. Intracanal dressings of calcium hydroxide are often successful in halting further tooth resorption.  Severe external root resorption often necessitates extraction of the tooth.  Moderate root resorption can be treated by orthodontically extruding the tooth and restoring it with a post-retained crown, but the prognosis of this treatment can be doubtful.  Mild cervical resorption can be treated by surgical access, curettage, and placement of a restoration." "This is a real-world medical entrance exam question, please give the true answer based on the question and selection. Topic name: None, Subject name: Gynaecology & Obstetrics","Which of the following is done for screening of Down's syndrome in first trimester? Select and explain. A. Beta HCG and PAPP-A, B. Unconjugated estradiol and PAPPA, C. AFP and Inhibin A, D. AFP and Beta HCG",A: Beta HCG and PAPP-A,"Answer: A: Beta HCG and PAPP-A Explanation: Ans: A. Beta HCG and PAPP-A1st trimester aneuploidy screening:Human chorionic gonadotropin (either intact or free (b-hCG).Pregnancy-associated plasma protein A (PAPP-A).Fetal Down syndrome in 1st trimester:Higher serum free beta-hCG level.Lower PAPP-A levels.Trisomy 18 & 13:Lowered levels of both HCG PAPPP-A.2nd trimester analytes:Serum integrated screening.Accuracy of aneuploidy detection:Greater on combination with,Sonographic NT measurement." "This is a real-world medical entrance exam question, please give the true answer based on the question and selection. Topic name: None, Subject name: Anatomy","Muscles of mastication are supplied by: Select and explain. A. Second part of maxillary artery, B. Third part of maxillary artery, C. First part of maxillary artery, D. Facial artery",A: Second part of maxillary artery,"Answer: A: Second part of maxillary artery Explanation: None" "This is a real-world medical entrance exam question, please give the true answer based on the question and selection. Topic name: None, Subject name: Dental","Thumb sucking is normal during: Select and explain. A. The first year of life., B. First 2 years of life., C. First 3 years of life., D. First 4 years of life.",B: First 2 years of life.,"Answer: B: First 2 years of life. Explanation: None" "This is a real-world medical entrance exam question, please give the true answer based on the question and selection. Topic name: None, Subject name: Pharmacology","The most recent oral direct thrombin inhibitor (DTI) for acute stroke prevention is: Select and explain. A. Ximelagatran, B. Idraparinux, C. Dabigatran, D. Wasfarean",C: Dabigatran,"Answer: C: Dabigatran Explanation: Ans. c. Dabigatran (Ref Katzung 11 /e p594)The most recent oral direct thrombin inhibitor (DTI) for acute stroke prevention is Dabigatran.""In August 2011, Dabigatran was also licensed for prevention of stroke and systemic embolism in patients with atrial fibrillation by the European authorities, after approval for this indication was received in October 2010 in USA.-" "This is a real-world medical entrance exam question, please give the true answer based on the question and selection. Topic name: None, Subject name: Dental","According to Weerheijm et al, prevalence of MIH (Molar-Incisor hypomineralisation) : Select and explain. A. 1.2% to 25.6%, B. 2.4% to 40.2%, C. 6.2% to 48.8%, D. 9.7% to 62.1%",D: 9.7% to 62.1%,"Answer: D: 9.7% to 62.1% Explanation: None" "This is a real-world medical entrance exam question, please give the true answer based on the question and selection. Topic name: None, Subject name: Anatomy","The nerve supply for motor action of buccinator: Select and explain. A. Facial nerve, B. Mandibular nerve, C. Maxillary nerve, D. Auriculo temporal nerve",A: Facial nerve,"Answer: A: Facial nerve Explanation: None" "This is a real-world medical entrance exam question, please give the true answer based on the question and selection. Topic name: None, Subject name: Microbiology","Most common single cause of pyrexia of unknown origin Select and explain. A. Mycobacterium tuberculosis, B. Salmonella paratyphi, C. Brucella, D. Salmonella typhi",A: Mycobacterium tuberculosis,"Answer: A: Mycobacterium tuberculosis Explanation: None" "This is a real-world medical entrance exam question, please give the true answer based on the question and selection. Topic name: None, Subject name: Anatomy","Tongue movement has its primary effect on Select and explain. A. Palatoglossus arch, B. Facial musculature, C. Lips, D. Cheeks",A: Palatoglossus arch,"Answer: A: Palatoglossus arch Explanation: None" "This is a real-world medical entrance exam question, please give the true answer based on the question and selection. Topic name: None, Subject name: Dental","Edge angle is the angle between: Select and explain. A. Rake face and radial line., B. Rake face and clearance face., C. Clearance face and tooth structure., D. Radial line and clearance face",B: Rake face and clearance face.,"Answer: B: Rake face and clearance face. Explanation: None" "This is a real-world medical entrance exam question, please give the true answer based on the question and selection. Topic name: None, Subject name: Pediatrics","Absent thumb, radial deviation of wrist, bowing of forearm with thrombocytopenia which investigation need not to be done ? Select and explain. A. Echocardiography, B. Bone marrow examination, C. Platelet count, D. Karyotyping",A: Echocardiography,"Answer: A: Echocardiography Explanation: Characteristic features such as absent thumb and thrombocytopenia suggests Fanconis anemia. FEATURES OFFANCONISANEMIA Skin pigment change        --> Darkened areas of the skin, cafe-au-lait spots, vitiligo Upper limb                          --> Absent or hypoplastic thumb, absent or hypoplastic radii, dysplastic ulnae, clinodactyly Gonads                                —> Abnormal or atrophic testis Head and face                    -p Microcephaly, sprengels deformity, spina bifida Kidneys                                —+ Ectopic or pelvic, horseshoe, hypoplastic or dysplastic Abnormal eyes/eyelid Hematological abnormalities Pancytopenia+ thrombocytopenia and neutropenia these are the most common and most serious symptoms of Fanconis anemia. one marrow failure            ---> Its a major complication of Fanconis anemia Acute myeloid leukemia ---> Fanconis anemia have elevated risks of developing acute myeloid leukemia. Myelodysplastic syndromes —> Increased risk of myelodysplastic syndrome in Fanconis anemia. Common tests for Fanconis anemia include : Complete blood count (CBC) o Bone marrow biopsy                   o Chromosomal breakage analysis HLA tissue typing                      o Hearing test                                  o Hand X-ray and other imaging studies Developmental test                   o Ultrasound of the kidneys       Amniocentesis or chorionic vinous crimpling" "This is a real-world medical entrance exam question, please give the true answer based on the question and selection. Topic name: AIIMS 2018, Subject name: Gynaecology & Obstetrics","Placenta grade 3, 35+3 weeks pregnancy, and absent end diastolic flow Doppler; next ,management is:- Select and explain. A. Dexamethasone and terminate after 48 hours, B. Terminate after 37 weeks, C. Talk with pediatrician and termination, D. Monitor",C: Talk with pediatrician and termination,"Answer: C: Talk with pediatrician and termination Explanation: * Absent end diastolic flow : fetal compromise ( hence terminate pregnancy by caesarean section if > 34 weeks)* Reverse end diastolic flow : impending doom of death ( terminate pregnancy irrespective of gestational age)* Steroids are given if pregnancy < 34 weeks" "This is a real-world medical entrance exam question, please give the true answer based on the question and selection. Topic name: None, Subject name: Dental","In mandibular first molar, extra distal root is known as Select and explain. A. Radix paramolaris, B. Radix entomolaris, C. Tomt’s root, D. None of the above",B: Radix entomolaris,"Answer: B: Radix entomolaris Explanation: Extra mesial root is known as radix paramolaris" "This is a real-world medical entrance exam question, please give the true answer based on the question and selection. Topic name: None, Subject name: Dental","Girdle like elevation on the lingual surface of anteriors is Select and explain. A. Cingulam, B. Mamelons, C. Incisal edge, D. Incisal ridge",A: Cingulam,"Answer: A: Cingulam Explanation: None" "This is a real-world medical entrance exam question, please give the true answer based on the question and selection. Topic name: None, Subject name: Dental","Good clinical practice is observed in all phases of clinical trials, EXCEPT? Select and explain. A. Preclinical, B. Phase I, C. Phase II, D. Phase V",A: Preclinical,"Answer: A: Preclinical Explanation: None" "This is a real-world medical entrance exam question, please give the true answer based on the question and selection. Topic name: None, Subject name: Dental","Chroma is; Select and explain. A. Intensity of colour, B. Translucency of enamel, C. Translucency of enamel & dentin, D. None of the above",A: Intensity of colour,"Answer: A: Intensity of colour Explanation: None" "This is a real-world medical entrance exam question, please give the true answer based on the question and selection. Topic name: None, Subject name: ENT","Angio fibroma is classified as what stage if it extends to one or more paranasal sinuses Select and explain. A. Stage I, B. Stage II, C. Stage III, D. Stage IV",A: Stage I,"Answer: A: Stage I Explanation: Staging of angiofibromaRadkowski Classifications:I ALimited to nose and/or nasopharyngeal vaultBExtension into 1 paranasal sinusIIAMinimal extension through sphenopalatine foramen intosphenopalatine/pterygomaxillary fossaBFills sphenopalatine fossa bowing the posterior wall of themaxillary antrum anteriorly or extending into the orbit theinferior orbital fissure.CExtends beyond sphenopalatine fossa into infratemporal fossaIllAErosion of skull base-minimal intracranial extensionBErosion of skull base-Extensive intracranial extension with orwithout cavernous sinus invasion." "This is a real-world medical entrance exam question, please give the true answer based on the question and selection. Topic name: None, Subject name: Gynaecology & Obstetrics","A lady underwent vaginal hysterectomy for Carci!noma cervix. Following the surgery after her urethral catheter was removed, she complained of urinary in!continence. On examination she had normal voiding as well as continuous incontinence. Methylene blue dye was instilled in her bladder through her urethra and she was given oral Phenazopyridine dye. After some time her pads were checked and it showed yellow staining at the top most pad, while the middle or bottom pads were unstained. She is likely to have: Select and explain. A. Vesicovaginal fistula, B. Ureterovaginal fistula, C. Urethrovaginal fistula, D. Vesicouterine fistula",B: Ureterovaginal fistula,"Answer: B: Ureterovaginal fistula Explanation: Ans: B. Ureterovaginal fistula(Ref Shaws 16/e p223-224, 15/e p184; William Gynue lst/e p573)Pad showing yellow staining at top poion, but not middle or bottom poions - Likely to have Ureterovaginal fistula.Interpretations of Methylene Blue 3 Swab Test:ObservationInterpretationUpper most swabs soaked with urine but unstained with dye.Clear ureter (unstained).Via fistula a reachs vagina.Uppermost cotton swab will be wet with urine.No discoloration seen - As dye is in bladder & not in ureter.Ureterovaginal fistulaUpper and lower swab remain dry but the middle swab soaked with dyeVesicovaginal fistulaThe upper two swab remain dry but lower one soaked with dyeUrethrovaginal fistula" "This is a real-world medical entrance exam question, please give the true answer based on the question and selection. Topic name: None, Subject name: Dental","Smallest mesiodistal diameter is seen in Select and explain. A. Deciduous lower central incisor, B. Permanent lower central incisor, C. Deciduous upper lateral incisor, D. Permanent upper lateral incisor",A: Deciduous lower central incisor,"Answer: A: Deciduous lower central incisor Explanation: None" "This is a real-world medical entrance exam question, please give the true answer based on the question and selection. Topic name: None, Subject name: Dental","Anchoring fibres of lamina densa contains: Select and explain. A. Type 2 collagen., B. Type 4 collagen., C. Type 6 collagen., D. Type 7 collagen.",D: Type 7 collagen.,"Answer: D: Type 7 collagen. Explanation: None" "This is a real-world medical entrance exam question, please give the true answer based on the question and selection. Topic name: None, Subject name: Dental","Occurrence of malocclusion in given population is best described as: Select and explain. A. Prevalence rate, B. Incidence rate, C. Morbidity, D. Probability",A: Prevalence rate,"Answer: A: Prevalence rate Explanation: None" "This is a real-world medical entrance exam question, please give the true answer based on the question and selection. Topic name: AIIMS 2019, Subject name: Gynaecology & Obstetrics","Anti D is required in all cases except Select and explain. A. Aboion at 63 days, B. Manual removal of placenta, C. Amniocentesis of 16 Weeks, D. Intrauterine transfusion at 28 weeks",D: Intrauterine transfusion at 28 weeks,"Answer: D: Intrauterine transfusion at 28 weeks Explanation: Anti D prophylaxis administered in : Invasive prenatal diagnosis - eg, amniocentesis, chronic villus biopsy Antepaum hemorrhage External cephalic version of the fetus Ectopic pregnancy Evacuation of molar pregnancy Intrauterine death and stillbih Intrauterine procedures (inseion of shunts, embryo reduction) Therapeutic termination of pregnancy Delivery - Normal, instrumental or caesarean section" "This is a real-world medical entrance exam question, please give the true answer based on the question and selection. Topic name: None, Subject name: Surgery","CSF leakage is diagnosed by all except Select and explain. A. Tram Line, B. β 2- transferrin level, C. increase in glucose, D. increase in proteins",C: increase in glucose,"Answer: C: increase in glucose Explanation: None" "This is a real-world medical entrance exam question, please give the true answer based on the question and selection. Topic name: None, Subject name: Medicine","Lead cause following except Select and explain. A. Uroporphyrinuria, B. Sideroblastic anemia, C. Basophilic stippling, D. Macrocytic anemia",D: Macrocytic anemia,"Answer: D: Macrocytic anemia Explanation: None" "This is a real-world medical entrance exam question, please give the true answer based on the question and selection. Topic name: None, Subject name: Dental","Which among the following is used for sharpening of both stainless steel and carbon steel instrument: Select and explain. A. Arkansas., B. Silicon carbide., C. Alumina., D. Diamond.",D: Diamond.,"Answer: D: Diamond. Explanation: None" "This is a real-world medical entrance exam question, please give the true answer based on the question and selection. Topic name: None, Subject name: Dental","The initial pulpal response to any insult is: Select and explain. A. Necrosis, B. Ulceration, C. Calcification, D. Inflammation",D: Inflammation,"Answer: D: Inflammation Explanation: None" "This is a real-world medical entrance exam question, please give the true answer based on the question and selection. Topic name: None, Subject name: Dental","Which of the following is ture about afferent nerve fibers of pulp Select and explain. A. Parasympathetic post ganglionic, B. Sympathetic post ganglionic, C. Parasympathetic pre ganglionic, D. Sympathetic preganglionic",B: Sympathetic post ganglionic,"Answer: B: Sympathetic post ganglionic Explanation: None" "This is a real-world medical entrance exam question, please give the true answer based on the question and selection. Topic name: None, Subject name: Dental","Calcification in enamel starts at : Select and explain. A. Enamel matrix, B. Root, C. CEJ, D. DEJ",D: DEJ,"Answer: D: DEJ Explanation: Calcification of enamel proceeds from cusps or incisal edges towards cervical line and from DEJ it proceeds towards surface." "This is a real-world medical entrance exam question, please give the true answer based on the question and selection. Topic name: None, Subject name: Dental","Mc Spadden compactor is of __________ shape: Select and explain. A. Reverse blade of K-file, B. Reverse blade of H-file, C. Endosonic instrument, D. None of the above",B: Reverse blade of H-file,"Answer: B: Reverse blade of H-file Explanation: None" "This is a real-world medical entrance exam question, please give the true answer based on the question and selection. Topic name: None, Subject name: Dental","Tubercle of zukerkandle is seen on Select and explain. A. MB on upper and lower primary 1st molar, B. MB on upper and lower permanent 1st molar, C. MB on upper Primary 1st molar only, D. MB on lower primary 1st molar only",A: MB on upper and lower primary 1st molar,"Answer: A: MB on upper and lower primary 1st molar Explanation: None" "This is a real-world medical entrance exam question, please give the true answer based on the question and selection. Topic name: None, Subject name: Medicine","A 25 year old lady on treatment for rheumatoid ahritis has following lab findings: Hb-9gm/dl, MCV- 55fl, serum iron-30microgm/d1, ferritin 200 ng/ml, TIBC- 298 micrograms/dl. What is the most probable diagnosis? Select and explain. A. Thalassemia minor, B. Thalassemia major, C. Anemia of chronic disease, D. Iron deficiency anemia",C: Anemia of chronic disease,"Answer: C: Anemia of chronic disease Explanation: Answer C. Anemia of chronic diseaseE aicular manifestations of Rheumatoid Ahritis:* Subcutaneous nodules * Sjogren's syndrome* Pericarditis, Mitral regurgitation * Pleuritis, Interstitial Lung disease * Vasculitis * Felty's syndrome in the late stages of RA ; most common hematologic abnormality in RA- normochromic normocytic anemia.* Lymphoma" "This is a real-world medical entrance exam question, please give the true answer based on the question and selection. Topic name: None, Subject name: Dental","All of the following are synthetic sharpening stone except: Select and explain. A. Silicon carbide, B. Alumina, C. Arkansas, D. Diamond",C: Arkansas,"Answer: C: Arkansas Explanation: Stone type can come in natural or synthetic form: Natural-Arkansas (preferred) Synthetic: India stone Ceramic stone Composition stone Diamond Alumina Silicon carbide Nisha Garg, Amit Garg. Textbook of Operative Dentistry.  Edition 3. Page:97" "This is a real-world medical entrance exam question, please give the true answer based on the question and selection. Topic name: None, Subject name: Gynaecology & Obstetrics","Least complication in outlet forceps is: Select and explain. A. Complete perinea! tear, B. Vulval hematoma, C. Extension of episiotomy, D. Cervical tear",D: Cervical tear,"Answer: D: Cervical tear Explanation: Complication of forceps delivery:" "This is a real-world medical entrance exam question, please give the true answer based on the question and selection. Topic name: None, Subject name: Medicine","A lady developed breathlessness, pruritus, uicaria 1 hour after eating NSAIDS for headache. Chest examination was remarkable for rales with BP-80/50 mm Hg. All of following can be used for initial management except: Select and explain. A. Crystalloids infusion, B. Steroids, C. Early respiratory suppo and oxygenation, D. Adrenaline",B: Steroids,"Answer: B: Steroids Explanation: Answer- B. SteroidsMild symptoms such as pruritus and uicaria can be controlled by administration of 0.3 to 0.5 mL of l: 1000 (l mg/mL) epinephrine SC or IM.An IV infusion should be initiated to provide a route for administration of 2.5 mL epinephrine, diluted l: 10,000, at 5- to l0-min intervals, volume expanders such as normal saline, and vasopressor agents such as dopamine if intractable hypotension occurs.Oxygen alone a nasal catheter or with nebulized albuterol may be helpful, but either endotracheal intubation or a tracheostomy is mandatory for oxygen delivery if progressive hypoxia develops." "This is a real-world medical entrance exam question, please give the true answer based on the question and selection. Topic name: None, Subject name: Dental","To remove working side interference reduce: Select and explain. A. Guiding cusps, B. Supporting cusps, C. Both of the above., D. None.",A: Guiding cusps,"Answer: A: Guiding cusps Explanation: None" "This is a real-world medical entrance exam question, please give the true answer based on the question and selection. Topic name: AIIMS 2020, Subject name: Medicine","All are used in management of overdosage of Fibrinolytics except? Select and explain. A. Epsilon amino Caproic Acid, B. Tranexamic Acid, C. DDAVP, D. FFP",C: DDAVP,"Answer: C: DDAVP Explanation: Reversal of Fibrinolytic-Induced Bleeding Minor external bleeding Manual pressure Significant internal bleeding Immediate cessation of fibrinolytic agent, antiplatelet agent, and/or heparin Reversal of heparin with protamine, as discussed earlier Typed and cross-matched blood ordered with verification of activated PTT, CBC, thrombin clotting time, and fibrinogen level Volume replacement with crystalloid and packed red blood cells as needed Major bleeding or hemodynamic compromise All measures listed for significant internal bleeding Fibrinogen concentrate 70 milligrams/kg IV, and recheck fibrinogen level; if fibrinogen level <100 milligrams/dL, repeat fibrinogen concentrate dose. If bleeding persists after fibrinogen concentrate or despite fibrinogen level >100 milligrams/dL, administer FFP 2 units IV. If bleeding continues after FFP, administer an antifibrinolytic such as aminocaproic acid 5 grams IV over 60 min followed by 1 gram/h continuous IV infusion for 8 h or until bleeding stops, or tranexamic acid 10 milligrams/kg IV every 6-8 h. Consider platelet transfusion. Intracranial hemorrhage All measures listed for significant internal and major bleeding with hemodynamic compromise Immediate neurosurgery consultation" "This is a real-world medical entrance exam question, please give the true answer based on the question and selection. Topic name: None, Subject name: Dental","IQ range 90-109 is: Select and explain. A. Superior., B. Average., C. Low average., D. Borderline.",B: Average.,"Answer: B: Average. Explanation: IQ Classification guide:" "This is a real-world medical entrance exam question, please give the true answer based on the question and selection. Topic name: None, Subject name: Biochemistry","What is the codon for selenocysteine? Select and explain. A. UAG, B. UGA, C. UAA, D. GUA",B: UGA,"Answer: B: UGA Explanation: Ans: B. UGA(Ref Harper 30/e p18, 286)Selenocysteine:Commonly termed ""21st amino acid"".Present in biological proteins.Not coded directly in genetic code.Instead encoded by a UGA codon.UGA codon - Normally a stop codon.This mechanism called as|""translational recoding""." "This is a real-world medical entrance exam question, please give the true answer based on the question and selection. Topic name: None, Subject name: Dental","All are true about trauma from occlusion except Select and explain. A. Trauma to Marginalized gingiva, B. Reversible, C. Reduction in height of alveolar crest, D. No pocket",A: Trauma to Marginalized gingiva,"Answer: A: Trauma to Marginalized gingiva Explanation: None" "This is a real-world medical entrance exam question, please give the true answer based on the question and selection. Topic name: None, Subject name: Anatomy","All of the following muscles are grouped together as ""muscles of mastication"" except: Select and explain. A. Buccinator, B. Masseter, C. Temporalis, D. Pterygoids",A: Buccinator,"Answer: A: Buccinator Explanation: None" "This is a real-world medical entrance exam question, please give the true answer based on the question and selection. Topic name: None, Subject name: Dental","Mid palatal split following Le-Fort 1 osteotomy is done by moving the maxilla in which direction? Select and explain. A. Superior, B. Inferior, C. Antero-posterior, D. Transverse",D: Transverse,"Answer: D: Transverse Explanation: None" "This is a real-world medical entrance exam question, please give the true answer based on the question and selection. Topic name: None, Subject name: Dental","Which of the following gingival retraction cord is used in hypertensive patient? Select and explain. A. Plain retraction cord, B. Retraction cord with 2% racemic epinephrine, C. Retraction cord with 8% aluminium chloride, D. None of the above",C: Retraction cord with 8% aluminium chloride,"Answer: C: Retraction cord with 8% aluminium chloride Explanation: None" "This is a real-world medical entrance exam question, please give the true answer based on the question and selection. Topic name: None, Subject name: Forensic Medicine","Two bodies are found outside the car following an accident. The doctor conducting the autopsy was able to decide who was driver based on all of these features except: Select and explain. A. Whiplash injury, B. Steering wheel imprint, C. Sparrow foot marks, D. Seat belt abrasion over left shoulder",C: Sparrow foot marks,"Answer: C: Sparrow foot marks Explanation: Ans: C.Sparrow foot marks(Ref Reddy 34/e p263, 33/e p281)Sparrow footmarks:* Occur in both driver as well as passengers.* Does not differentiate between driver & passenger.* Multiple punctuate lacerations of lace produced. - Due to shattering of windscreen glass into multiple small cubical fragments with relatively blunt edges. - Hence, also windshield glass injury. Motor Vehicle Injuries:Car (Four-wheeler) Injuries:Whiplash injuryAcute hyperflexion followed by hyperextension of neck due to violent acceleration or deceleration force.Applied to (front seat) passenger (passenger >Driver)Steering wheel impact type injuryOccurs due to sudden stop of car, throwing forward the driverThroat crushed across top of steering wheel or horn ring (Occurs inSeat belt syndromeContusion,laceration,perforation & avulsions of small intestine, omentum, & mesenteryThere may be rupture of abdominal aoa, spleen, liver, pancreas, Hinge fractureSkull base fracture running along the petrous ridges through sella turcica, i.e.Ladder tearsMultiple transverse intimal tears in aoa, adjacent to main rupture." "This is a real-world medical entrance exam question, please give the true answer based on the question and selection. Topic name: None, Subject name: Dental","Tissue conditioners are not used for long as they Select and explain. A. Harden in mouth, B. Are expensive, C. Are difficult to fabricate, D. Cause ridge resorption",A: Harden in mouth,"Answer: A: Harden in mouth Explanation: None" "This is a real-world medical entrance exam question, please give the true answer based on the question and selection. Topic name: AIIMS 2018, Subject name: Anatomy","Type I collagen is present in all EXCEPT Select and explain. A. Ligament, B. Aponeurosis, C. Bone, D. Cailage",D: Cailage,"Answer: D: Cailage Explanation: COLLAGEN TYPE DISTRIBUTION Type 1 Tendons, ligament, fascia, bones, aponeurosis, derives, meninges (90% of total collagen), seen in old scars They form the fibrous basics of bones & fibrocailage Type 2 Hyaline & elastic cailage, vitreous body Type 3 Reticular fibers of skin, ligaments, blood vessels & internal organs, healing Type 4 Basement membrane, lens, capsule PATHOLOGIES: Collagenopathies Type I: Osteogenesis imperfecta Type IV: Alpo syndrome" "This is a real-world medical entrance exam question, please give the true answer based on the question and selection. Topic name: None, Subject name: Dental","Absorption of fluoride in child takes place mainly in Select and explain. A. Intestine, B. Lungs, C. Oral cavity, D. Gingiva",A: Intestine,"Answer: A: Intestine Explanation: None" "This is a real-world medical entrance exam question, please give the true answer based on the question and selection. Topic name: AIIMS 2019, Subject name: Biochemistry","In CRISPR-Cas 9 system, which repair mechanism is used for genome editing? Select and explain. A. Non homologous end repair, B. Homologous repair, C. Mismatch repair, D. Nucleotide excision repair",A: Non homologous end repair,"Answer: A: Non homologous end repair Explanation: * CRISPR-Cas-9 endonuclease system introduces double stranded breaks in DNA at the specific target site. Now cell tries to repair this double stranded breaks by two mechanism: 1. Non-homologous end repair 2. Homologous repair * Non-homologous repair mechanism is always a possibility as it does not require homologous DNA. * The homologous repair will only work if the DNA homologous to the cut DNA is present but homologous DNA is not always present or available. So, if both options are given, best option is Non-homologous repair." "This is a real-world medical entrance exam question, please give the true answer based on the question and selection. Topic name: None, Subject name: Pathology","Mucocutaneous circumoral pigmentation is found in: Select and explain. A. Peutz-Jeghers syndrome, B. Plummer-Vinson syndrome, C. Lead poisoning, D. Bechet's syndrome",A: Peutz-Jeghers syndrome,"Answer: A: Peutz-Jeghers syndrome Explanation: None" "This is a real-world medical entrance exam question, please give the true answer based on the question and selection. Topic name: AIIMS 2018, Subject name: Biochemistry","Biomarker of alcoholic hepatitis: Select and explain. A. ALP, B. AST, C. LDH, D. GGT",D: GGT,"Answer: D: GGT Explanation: Markers for alcoholism: y-Glutamyl transpeptidase / transferase (GGT) : It has EC number 2. This enzyme is present in liver. When damage occurs to liver cells this enzyme comes to blood. lt is a sensitive diagnostic marker for the detection of alcoholism. GGT is also increased in infective hepatitis and obstructive jaundice. CDT -carbohydrate deficient transferrin (transferrin is a protein which is responsible for the transpo of Iron.) This is a glycoprotein CDT is also the marker for alcoholism." "This is a real-world medical entrance exam question, please give the true answer based on the question and selection. Topic name: None, Subject name: Pathology","A 9 year old child's mother comes to dental clinic with the complaint of oral ulceration, fever and shedding of skin of palms and soles; she is giving history of premature shedding of teeth and increased sweating, she is also giving one month history of using any new teething gel available in market. The child is suffering from. Select and explain. A. Acrodynia, B. Pemphigus vulgaris, C. Epidermolysis Bullosa, D. Erosive lichen planus",A: Acrodynia,"Answer: A: Acrodynia Explanation: None" "This is a real-world medical entrance exam question, please give the true answer based on the question and selection. Topic name: None, Subject name: Dental","The diameter of the tip of a periodontal probe is: Select and explain. A. 0.25 mm, B. 0.75 mm, C. 0.5 mm, D. 1 mm",C: 0.5 mm,"Answer: C: 0.5 mm Explanation: None" "This is a real-world medical entrance exam question, please give the true answer based on the question and selection. Topic name: None, Subject name: Surgery","Polyglycolic acid suture material (""vicryl/Dexon"") is an: Select and explain. A. Absorbable natural suture, B. Absorbable synthetic suture, C. Non-absorbable natural suture, D. Non absorbable synthetic suture",B: Absorbable synthetic suture,"Answer: B: Absorbable synthetic suture Explanation: None" "This is a real-world medical entrance exam question, please give the true answer based on the question and selection. Topic name: AIIMS 2019, Subject name: Medicine","All are true about instrument given except: Select and explain. A. Platelets <40,000 cu/mm is contraindication, B. Can be done in both prone or lateral position, C. Infiltrative and granulomatous diseases are indications, D. No breath holding needed","A: Platelets <40,000 cu/mm is contraindication","Answer: A: Platelets <40,000 cu/mm is contraindication Explanation: It is an image of bone marrow biopsy needle. Thrombocytopenia is not a contraindication for BM biopsy. BM biopsy can be done in PSIS/ Iliac crest It is done for infiltrative and granulomatous diseases & myeloproliferative diseases of bone marrow. Breath holding is needed for liver disease." "This is a real-world medical entrance exam question, please give the true answer based on the question and selection. Topic name: None, Subject name: ENT","All are characteristic features of superior semicircular canal dehiscence except? Select and explain. A. Third window phenomenon, B. SNHL, C. Tullio's phenomenon, D. Vestibular evoked myogenic potential is useful investigation.",B: SNHL,"Answer: B: SNHL Explanation: Ans. B. SNHLsymptoms of SSDC include* Tullio phenomenon - identifiable symptoms leading to a positive SCD diagnosis is sound-induced loss of balance. Patients showing this symptom may experience a loss of equilibrium, a feeling of motion sickness or even actual nausea, triggered by normal everyday sounds.* Vestibular evoked myogenic potentials (VEMP): In SSCD, loud tones evoke a sho-latency relaxation potential in the ipsilateral sternocleidomastoid muscle. Patients with SSCD typically have a lower-than-normal threshold for the VEMP response, and the amplitude of the VEMP waveform in an SSCD ear is greater for comparable stimulus intensities than in an ear without dehiscence. The VEMP test plays an impoant role in the evaluation of patients with suspected SSCD* Low-frequency conductive hearing loss is present in many patients with SCDS and is explained by the dehiscence acting as a ""third window phenomenon""" "This is a real-world medical entrance exam question, please give the true answer based on the question and selection. Topic name: None, Subject name: Pathology","RBC should be transfused: Select and explain. A. With a 18-20 G needle within 4 hours of receiving at the patient's side, B. With a 18-20 G needle within 4 hours of receiving at the patient's side, C. With a 20-22 G needle within 4 hours of issue from the blood bank, D. With a 20-22 G needle within 4 hours of receiving at the patient's side",C: With a 20-22 G needle within 4 hours of issue from the blood bank,"Answer: C: With a 20-22 G needle within 4 hours of issue from the blood bank Explanation: Ans: C(Ref: WHO Clinical Transfusion Guidelines/p22).RBCs should he transfused with 18-20 G needle within 4 hours of issue from the blood bank.Sta infusion within 30 min of removing pack from refrigerator and complete the infusion within 4 hours.An 18-gauge needle is standard, but a needle or catheter as small as 23-gauge can be used for transfusion if necessary within 4 hours of issue from the blood bank." "This is a real-world medical entrance exam question, please give the true answer based on the question and selection. Topic name: None, Subject name: Dental","Step back technique is consist of: Select and explain. A. 2 phases., B. 3 phases., C. 4 phases., D. None.",A: 2 phases.,"Answer: A: 2 phases. Explanation: step back technique has 2 phases. Phase 1 is apical preparation phase. Phase 2 stepping back phase." "This is a real-world medical entrance exam question, please give the true answer based on the question and selection. Topic name: None, Subject name: Anatomy","All of the following structures lie in the relation to the left ureter except: Select and explain. A. Mesentery of sigmoid colon, B. Bifurcation of common iliac aery, C. Quadratus lumborum, D. Gonadal vessels",C: Quadratus lumborum,"Answer: C: Quadratus lumborum Explanation: Ans: C. Quadratus lumborum(Ref Grays 41/e p1251, 40/e p1239)Left ureter is related posteriorly to psoas major muscle (not quadratus lumborum).Relations of Right UreterAnterior:Duodenum,Terminal pa of the ileum,Right colic vessels,Iliocolic vessels,Right testicular or ovarian vessels, and theRoot of the mesentery.Posterior:Right psoas muscle,Bifurcation of the right common iliac aeryRelations of Left UreterAnterior:Sigmoid colon,Sigmoid mesocolon,Left colic vessels,Left testicular or ovarian vesselsPosterior:Left psoas muscle,Bifurcation of left common iliac aery" "This is a real-world medical entrance exam question, please give the true answer based on the question and selection. Topic name: None, Subject name: Dental","The odontoblastic layer histologically Select and explain. A. Appears 3-5 layer thick., B. Is single layer outlining pulp, C. Is psuedostratified palisade layer, D. All of the above",D: All of the above,"Answer: D: All of the above Explanation: None" "This is a real-world medical entrance exam question, please give the true answer based on the question and selection. Topic name: AIIMS 2017, Subject name: Pharmacology","Treatment of choice for a patient with gonococcal as well as non-gonococcal urethritis is: Select and explain. A. Ceftriaxone 250 mg IM single dose, B. Cefixime 400 mg oral single dose, C. Ciprofloxacin 500 mg oral single dose, D. Azithromycin 2 g oral single dose",D: Azithromycin 2 g oral single dose,"Answer: D: Azithromycin 2 g oral single dose Explanation: Ceftriaxone is DOC for gonococci but is not effective against non gonococcal cause of urethritis like mycoplasma and chlamydia. Azithromycin single dose of 2 g is used for both gonococcal and non gonococcal urethritis. In non gonococcal urethritis, doxycycline can also be used." "This is a real-world medical entrance exam question, please give the true answer based on the question and selection. Topic name: None, Subject name: Pathology","In OKC which is more aggressive and has more recurrence potential Select and explain. A. Orthokeratinized, B. Parakeratinized, C. Non keratinized, D. None",B: Parakeratinized,"Answer: B: Parakeratinized Explanation: 60% OKCs have Primordial origin (Dental lamina/cell rests of Serees). these are called primordial origin OKCs. They are Parakeratinized with thin friable lining and daughter/satellite cysts in their CT wall, hence great tendency to recur (1. multiplicative nature of primordial cells, thin lining that tears, 3. satellite cysts). rest of the 40% OKCs are Dentigerous origin OKCs (arise from Reduced enamel epithelium like dentigeous cysts). Since the proliferative potential of REE is lost, they dont have tendency to recur when their Orthokeratinized epithelial lining is left behind." "This is a real-world medical entrance exam question, please give the true answer based on the question and selection. Topic name: None, Subject name: Dental","In anteriors labial surface is formed from Select and explain. A. 1 lobe, B. 2 lobes, C. 3 lobes, D. 4 lobes",C: 3 lobes,"Answer: C: 3 lobes Explanation: None" "This is a real-world medical entrance exam question, please give the true answer based on the question and selection. Topic name: AIIMS 2017, Subject name: Pediatrics","4 day old breastfed neonate, otherwise well, term neonate presented with jaundice, on testing the bilirubin level was found to be 18 mg/dl. Which of the following is the best step of management? Select and explain. A. Sta phototherapy and continue breast feeding, B. Sta iv fluids and given phototherapy, C. Initiate exchange transfusion, D. Stop breast feeding and do phototherapy",A: Sta phototherapy and continue breast feeding,"Answer: A: Sta phototherapy and continue breast feeding Explanation: Use total bilirubin. Do not subtract direct reacting or conjugated bilirubin. Risk factors = isoimmune hemolytic disease, G6PD deficiency, asphyxia, significant lethargy, Temperature instability, sepsis, acidosis, or albumin <3.0 g/d (if measured) For well infants 35-47 6/7 wk can adjust TSB levels for intervention around the medium risk line. It is an option to intervene at lower TSB levels for infatns closer to 35 weeks and at higher TSB levels for those closer to 37 6/7 wk. It is an option to provide conventional phototherapy in hospital or at home at TSB levels 2-3 mg/dL (35 - 50 mmol/L) below those shown but home phototherapy should not be used in any infant with risk factors. Cut off for phototherapy in 4-day old neonate is 19-20 mg/dl The dashed lines for the first 24 hours indicate unceainly due to a wide range of clinical circumstances and a range of responses to phototherapy. Immediate exchange transfusion is recommended if infant shows signs of acute bilirubin encephalopathy (hypeonia, arching, retrocollis, opisthotonos, fever, high pitched cry) or IF TSB is > 5 mg/dL (85mmol/L) abvoe these lines. Risk factors - isoimmune hemolytic disease, GPD deficiency, asphyxia, significant lethargy, temperature instability, sepsis, acidosis. Measure serum albumin and calculate B/A ratio (see legend) Use total bilirubin. Do not subtract direct reacting or conjugated bilirubin. It infant is well and 36-37 6/7 wk (median risk) can individualize TSB levels for exchange based on actual gestational age. Cut off for exchange transfusion in 4-day old neonate is 25mg/dl For otherwise well term neonate- Age Phototherapy cut off Exchange transfusion cut off 24 to 48 hrs. >15 mg/dl >20 mg/dl 48 to 72 hrs. >18 mg/dl >25mg/dl >72 hrs. >20mg/dl >25mg/dl" "This is a real-world medical entrance exam question, please give the true answer based on the question and selection. Topic name: None, Subject name: Dental","After planning an implant in lower molar region, patient complains of inability to chew by other teeth on same side. What should be your first step in treating this patient Select and explain. A. Reduce the centric fossa, B. Reduce the supporting cusp, C. Check the occlusion in centric, D. None",C: Check the occlusion in centric,"Answer: C: Check the occlusion in centric Explanation: It’s a universal rule: To remove any occlusal prematurity, 1st step is to check the occlusion in centric." "This is a real-world medical entrance exam question, please give the true answer based on the question and selection. Topic name: None, Subject name: Social & Preventive Medicine","In a group of 100 people, the average GFR is 85 ml/ min with a standard detion of 25. What is the range for 90% confidence interval? Select and explain. A. 81-89, B. 80-90, C. 75-95, D. 70-100",A: 81-89,"Answer: A: 81-89 Explanation: Ans: A. 81-89(Ref: Park 24/e p888, 22/e p793; Essentials of Biostatistics 2/e pI70)Range for 90% confidence interval in the given question is 81-89.Confidence intervals:Lower limit & upper limit estimates for statistic given by:Lower Limit: statistic - C x SE (statistic)Upper Limit: statistic + C x SE.Confidence coefficient:C = Confidence coefficient = 1.65 for 90% confidence intervalC= 1.96 for 95% confidence intervalC= 2.58 for 99% confidence intervalC = 3.29 for 99.9% confidence intervalNow for 90% confidence interval:Upper limit = 85 + (1.65 x 2.5) = 85 + 4 = 89.Lower limit = 85 + (1.65 x 2.5) = 85 - 4 = 81.Hence, for 90% confidence interval will 81-89." "This is a real-world medical entrance exam question, please give the true answer based on the question and selection. Topic name: None, Subject name: Dental","All of the following are true about polymerization shrinkage of composite resin except: Select and explain. A. The more surface area of bonded structure causes LESS shrinkage compare to surface area of unbounded structure restoration., B. Class-V composite restoration causes. Contraction more than gold restoration, C. Etching and priming reduces shrinkages, D. One third (1/3rd) of the enamel involves causes more shrinkage",A: The more surface area of bonded structure causes LESS shrinkage compare to surface area of unbounded structure restoration.,"Answer: A: The more surface area of bonded structure causes LESS shrinkage compare to surface area of unbounded structure restoration. Explanation: None" "This is a real-world medical entrance exam question, please give the true answer based on the question and selection. Topic name: None, Subject name: Dental","Chi-square test is used to measure the degree of: Select and explain. A. Causal relationships between exposure and effect, B. Association between two variables, C. Correlation between two variables, D. Agreement between two observations",B: Association between two variables,"Answer: B: Association between two variables Explanation: None" "This is a real-world medical entrance exam question, please give the true answer based on the question and selection. Topic name: None, Subject name: Microbiology","Cytopathic effect is shown by: Select and explain. A. Virus, B. Bacteria, C. Spirochete, D. Anaerobe",A: Virus,"Answer: A: Virus Explanation: None" "This is a real-world medical entrance exam question, please give the true answer based on the question and selection. Topic name: None, Subject name: Dental","The cause of bone destruction in juvenile periodontitis is: Select and explain. A. Phagocytosis are reduced, B. Reduced neutrophilic chemotaxis, C. Decreased host resistance, D. Highly virulent microorganisms",B: Reduced neutrophilic chemotaxis,"Answer: B: Reduced neutrophilic chemotaxis Explanation: None" "This is a real-world medical entrance exam question, please give the true answer based on the question and selection. Topic name: None, Subject name: Medicine","Infective endocarditis due to pseudomonas is most commonly seen with: Select and explain. A. Intravenous drug abuse of pentazocin., B. HIV patient., C. Chronic steroid therapy., D. Elderly with community acquired pneumonia.",A: Intravenous drug abuse of pentazocin.,"Answer: A: Intravenous drug abuse of pentazocin. Explanation: None" "This is a real-world medical entrance exam question, please give the true answer based on the question and selection. Topic name: None, Subject name: Dental","Alginate fillers derived from Select and explain. A. Calcium sulphate, B. Sodium phosphate, C. Potassium alginate, D. Diatomaceous earth",D: Diatomaceous earth,"Answer: D: Diatomaceous earth Explanation: None" "This is a real-world medical entrance exam question, please give the true answer based on the question and selection. Topic name: None, Subject name: Dental","Given instrument is known as: Select and explain. A. Perry separator, B. Ferrier double bow separator, C. Elliot separator, D. Wood ward separator",C: Elliot separator,"Answer: C: Elliot separator Explanation: None" "This is a real-world medical entrance exam question, please give the true answer based on the question and selection. Topic name: None, Subject name: Pathology","Garre's osteomyelitis all statements are true except: Select and explain. A. Common in children and young, B. Occurs due to staphylococcus aureus, C. Moth eaten appearance in radiography, D. Prevalent in immune-competent patients",C: Moth eaten appearance in radiography,"Answer: C: Moth eaten appearance in radiography Explanation: None" "This is a real-world medical entrance exam question, please give the true answer based on the question and selection. Topic name: None, Subject name: Physiology","Best index to measure cardiac afterload is: Select and explain. A. Mean aerial pressure, B. LV end diastolic pressure, C. LV mean systolic pressure, D. Total peripheral resistance",D: Total peripheral resistance,"Answer: D: Total peripheral resistance Explanation: Ans: D. Total peripheral resistance(Ref: Ganong 25Ie p544, 24/e p546)Cardiac output depends on both preload & afterload.The preload is defined by the venous return & left ventricular end-diastolic volume while the afterload is defined by the mean aerial pressure, which then fither depends on total peripheral resistance." "This is a real-world medical entrance exam question, please give the true answer based on the question and selection. Topic name: None, Subject name: Dental","Apical migration of the epithelial attachment with corresponding recession of the marginal gingiva results in? Select and explain. A. A shallow sulcus, B. Gingival pocket formation, C. Infrabony pocket formation, D. Periodontal pocket formation",A: A shallow sulcus,"Answer: A: A shallow sulcus Explanation: None" "This is a real-world medical entrance exam question, please give the true answer based on the question and selection. Topic name: None, Subject name: Pathology","Which of the following may be a feature of acromegaty? Select and explain. A. Large tongue, B. Micrognathia, C. Hypoglycemia, D. Crowded teeth",A: Large tongue,"Answer: A: Large tongue Explanation: None" "This is a real-world medical entrance exam question, please give the true answer based on the question and selection. Topic name: AIIMS 2018, Subject name: Pathology","Immunofluorescence staining pattern from a kidney biopsy from a 35-year-old patient presenting with proteinuria has been shown below. What is the most probable cause? Select and explain. A. FSGS, B. PSGN, C. Lupus Nephritis, D. Goodpasture syndrome",C: Lupus Nephritis,"Answer: C: Lupus Nephritis Explanation: FSGS- Focal segmental glomerulosclerosis is associated with focal involvement of kidneys and deposition of IgM,C3 ,C1q PSGN- It is an immunocomplex disease and granular pattern is observed on immunofluorescence. There is deposition of C3,C1q and IgG in granular pattern Goodpasture syndrome- Auto antibodies against basement membrane ,linear pattern on immunofluorescence Lupus Nephritis- seen in SLE." "This is a real-world medical entrance exam question, please give the true answer based on the question and selection. Topic name: None, Subject name: Surgery","Which of the following is the most common symptom of aooiliac occlusive disease? Select and explain. A. Calf claudication, B. Gluteal claudication, C. Impotence, D. Symptomless",B: Gluteal claudication,"Answer: B: Gluteal claudication Explanation: Ans: B. Gluteal claudication(Ref Bailey 27/e p943-944, 261e 037, 879, 25/e p901; Sabiston 20/e p1739; Schwaz 10/e p874; Harrison 19/c p1643)Most common symptom of aooiliac disease = Gluteal claudication.Site of claudication:Distal to location of occlusive lesion.Eg: Buttock, hip, thigh & calf discomfo occurs in patients with aooiliac disease.Calf claudication develops in patients with femoral!popliteal disease." "This is a real-world medical entrance exam question, please give the true answer based on the question and selection. Topic name: None, Subject name: Dental","In mechanical trituration of amalgam, what is adversely affected ? Select and explain. A. Hardness of filling, B. Tarnish resistance, C. Working time, D. Final gloss of the filling",C: Working time,"Answer: C: Working time Explanation: None" "This is a real-world medical entrance exam question, please give the true answer based on the question and selection. Topic name: None, Subject name: Microbiology","Prions true is: Select and explain. A. Readily inactivated by autoclave at 121oC, B. Evokes strong immunogenic reaction, C. Have short incubation periods, D. Contains DNA/RNA",A: Readily inactivated by autoclave at 121oC,"Answer: A: Readily inactivated by autoclave at 121oC Explanation: Prions are infectious particles, which can transmit disease. These prions are composed of chiefly a protein without any detectable nucleic acid. This is a new name widely accepted for the old name slow viruses. The term prion was coined by Prusiner for proteinaceous infectious particles.  Unlike conventional viruses, the prions apparently have no virion structure or genomes and evoke no immune response in the infected host. These are extremely resistant to inactivation by heat, disinfectants, and radiation. The prions are causative agents of slow viral infections, such as subacute spongiform encephalopathy. After long incubation period of years, they produce a progressive disease that causes damage to the central nervous system, leading to subacute spongiform encephalopathy.  They are readily inactivated by autoclave at 121oC and resistant to antiseptics and disinfectants. Parija SC. Textbook of Microbiology & Immunology. Elsevier Health Sciences; 2014. Page:438-39" "This is a real-world medical entrance exam question, please give the true answer based on the question and selection. Topic name: None, Subject name: Dental","Millennium development goal was adopted in which year? Select and explain. A. 1977, B. 2000, C. 1984, D. 2004",B: 2000,"Answer: B: 2000 Explanation: None" "This is a real-world medical entrance exam question, please give the true answer based on the question and selection. Topic name: None, Subject name: Dental","Direction of placement of retentive pin holes should be: Select and explain. A. Perpendicular to external tooth surface, B. Parallel to external tooth surface, C. Parallel to other pins, D. Never beneath cusps to be restored",B: Parallel to external tooth surface,"Answer: B: Parallel to external tooth surface Explanation: None" "This is a real-world medical entrance exam question, please give the true answer based on the question and selection. Topic name: None, Subject name: Pathology","Which of the following produces osteoblastic secondaries Select and explain. A. Carcinoma Lung, B. Carcinoma breast, C. Carcinoma urinary bladder, D. Carcinoma prostate",D: Carcinoma prostate,"Answer: D: Carcinoma prostate Explanation: None" "This is a real-world medical entrance exam question, please give the true answer based on the question and selection. Topic name: None, Subject name: Pediatrics","A 10–month old baby previously normal, suddenly becomes distressed in his crib. The external appearance of genitalia was normal, except hyperpigmentation. lood glucose showed a level of 30 mg%. What is the most probable diagnosis ? Select and explain. A. 21 hydroxylase deficiency, B. Hyperinsulinism, C. Familial glucocorticoid deficiency, D. Cushing's syndrome",C: Familial glucocorticoid deficiency,"Answer: C: Familial glucocorticoid deficiency Explanation: Familial glucocorticoid deficiency : Familial glucocorticoid deficiency is a rare autosomal recessive condition. It is characterized by adrenal insufficiency. The pathological examination of the adrenal gland reveals that :- Zona glomerulosa ---> well preserved. Zona fasciculata ---> atrophied. Zona reticularis  ---> atrophied. This causes low cortisol concentration because the zona fasciculata is primarily responsible for glucocorticoid production. Because the zona glomerulosa is well preserved mineralocorticoid action is usually unaffected. Low circulating serum cortisol results in lack of feedback inhibition to the hypothalamus which results in increased ACTH secretion from pituitary. Clinical features : Patients with familial glucocorticoid deficiency generally presents with signs and symptoms of adrenal insufficiency with the important distinction that mineralocorticoid production is always normal. The most common initial presenting sign is deep hyperpigmentation of the skin, mucous membrane or both as a result of the action of adrenocorticotrophic hormone (ACTH) on cutaneous melanocyte stimulating hormone (MSH) receptors. The symptoms are compatible with glucocorticoid deficiency. Many patients presents with recurrent hypoglycemia or severe infections. In the neonatal period, frequent presenting signs include - feeding problems, failure to thrive, regurgitation and hypoglycemia manifesting as seizures." "This is a real-world medical entrance exam question, please give the true answer based on the question and selection. Topic name: None, Subject name: Anatomy","The largest commissure of the brain: Select and explain. A. Commissure of Ganser, B. Commissure of Gudden, C. Corpus callosum, D. Both (A) & (C)",C: Corpus callosum,"Answer: C: Corpus callosum Explanation: The corpus callosum is the largest commissure of the brain. It connects the two cerebral hemispheres. Since it is the neopallial commissure, it attains enormous size in man (10 cm long). Parts of brain connected The corpus callosum connects all parts of the cerebral cortex of the two sides, except the lower and anterior parts of the temporal lobes which are connected by the anterior commissure." "This is a real-world medical entrance exam question, please give the true answer based on the question and selection. Topic name: None, Subject name: Gynaecology & Obstetrics","A 32 weeks pregnant diabetic female with NST non reactive. What should be done next? Select and explain. A. Induction of labour, B. CS, C. Do NST after 1hr, D. Proceed to biophysical profile",D: Proceed to biophysical profile,"Answer: D: Proceed to biophysical profile Explanation: Non Stress Test" "This is a real-world medical entrance exam question, please give the true answer based on the question and selection. Topic name: None, Subject name: Gynaecology & Obstetrics","Renal agenesis is associated with: Select and explain. A. Hydramnios, B. Anencephaly, C. Tracheo-oesophageal fistula, D. Oligohydramnios",D: Oligohydramnios,"Answer: D: Oligohydramnios Explanation: Fetal urine is the main contributor of Amniotic fluid beyond 20 weeks therefore. In case of Renal agenesis → decrease/no urine → oligohydramnios." "This is a real-world medical entrance exam question, please give the true answer based on the question and selection. Topic name: None, Subject name: Psychiatry","A man comes with history of abnormal excessive blinking and grunting. He says he has no control over his symptoms, which have risen in frequency of late. This has staed affecting his social life making him depressed. Which of the following medications should be used in him? Select and explain. A. Risperidone, B. Imipramine, C. Carbamazepine, D. Methylphenidate",A: Risperidone,"Answer: A: Risperidone Explanation: Ans: A. Risperidone(Ref Kaplan and Sadock 11/e p1091; Niraj Ahuja 7/e p168).* This case suggests motor & vocal tics associated with Tourette's disorder.Treatment:* Pharmacotherapy - treatment of choice - Antipsychotics (haloperidol & resperidone). - DOC - Haloperidol.Other useful drugs:* Resperidone, Olanzapine, Aripiprazole.* SSRIs - Fluoxetine for co-morbid obsessive-compulsive symptoms.* Methylphenidate & Imipramine - Used in ADHD child.* Adjunct - Behavior therapy." "This is a real-world medical entrance exam question, please give the true answer based on the question and selection. Topic name: None, Subject name: Social & Preventive Medicine","Pick the right answer when 2 screening tests are done in series: Select and explain. A. Increased sensitivity and decreased specificity, B. Increased specificity and decreased sensitivity, C. Increased sensitivity and increased specificity, D. Decreased sensitivity and decreased specificity",B: Increased specificity and decreased sensitivity,"Answer: B: Increased specificity and decreased sensitivity Explanation: Ans. b. Increased specificity and decreased sensitivitySensitivity: Ability of a screening test to identify correctly all those who have the disease (Cases).Specificity: Ability of a screening test to identify correctly all those who don't have the disease (healthy).Positive predictive value (PPV): Ability of a screening test to identify correctly all those who have the disease, out of all those who test positive on a screening test.Negative predictive value (NPV): Ability of a screening test to identify correctly all those who don't have the disease, out of all those who test negative on a screening test.Screening tests used in the series: where the result of the first test determines whether to run the second test or not.Screening tests used in parallel: where the second test is independently performed regardless of what the first test result is. And, both are done at the same time.If multiple tests are applied in series, the specificity & PPV will increase, the sensitivity & NPV will decrease.If multiple tests are applied in Parallel, the sensitivity & NPV will increase, the specificity & PPV will decrease." "This is a real-world medical entrance exam question, please give the true answer based on the question and selection. Topic name: None, Subject name: Pathology","Scaphocephaly is caused by premature fusion of: Select and explain. A. Corona] suture, B. Sagittal suture, C. Metopic suture, D. Lambdoid suture",B: Sagittal suture,"Answer: B: Sagittal suture Explanation: None" "This is a real-world medical entrance exam question, please give the true answer based on the question and selection. Topic name: AIIMS 2019, Subject name: Pediatrics","Complete the missing step in the flow cha shown below: Select and explain. A. Intubate, spO2, ECG monitor, B. CPAP and ECG monitor, C. PPV, spO2 and ECG monitor, D. spO2 and ECG monitor only","C: PPV, spO2 and ECG monitor","Answer: C: PPV, spO2 and ECG monitor Explanation: PPV, spO2 and ECG monitor" "This is a real-world medical entrance exam question, please give the true answer based on the question and selection. Topic name: None, Subject name: Microbiology","A patient comes to your clinic with a complaint of multiple episodes of loose watery stool for 3 days. On probing, you discover that these episodes sta after he had ingested shellfish at a local restaurant 3 days back and other people who had food from that restaurant had similar symptoms. What is the most common cause of viral diarrhoea in adults? Select and explain. A. Calicivirus, B. Rotavirus, C. Adenovirus, D. Norovirus",D: Norovirus,"Answer: D: Norovirus Explanation: Ans: D. NorovirusRef Harrison 19/e p1285, 1286; Jawetz 27/e p537History of multiple episodes of loose watery stool for 3 days & history of ingestion shellfish with similar symptoms in other patients who consumed the shellfish is suggestive of acute viral gastroenteritis.Most likely organism responsible - Norovirus.Noroviruses:Most common infectious agents of mild gastroenteritis in the community and affect all age groups, whereas sapoviruses primarily cause gastroenteritis in children." "This is a real-world medical entrance exam question, please give the true answer based on the question and selection. Topic name: None, Subject name: Medicine","In a patient with dengue hemorrhagic fever, which of the following is most impoant to monitor? Select and explain. A. Hemoglobin, B. TLC, C. Platelet count, D. Hematocrit",D: Hematocrit,"Answer: D: Hematocrit Explanation: Ans: D. Hematocrit(Ref: Harrison 19/e p1319, 1322)In a patient with dengue hemorrhagic fever (DHF), hematocrit is most impoant parameter to monitor. Diagnosis of DHF needs the presence of rise in hematocrit 420% or more i.e. Hemoconcentration.Despite the name, the critical feature that distinguishes DHF from dengue fever is not hemorrhaging, but rather plasma leakage resulting from increased vascular permeability" "This is a real-world medical entrance exam question, please give the true answer based on the question and selection. Topic name: AIIMS 2018, Subject name: Anatomy","TRUE statement regarding nerve supply of adrenal gland Select and explain. A. Preganglionic fibres from lower thoracic & upper lumbar spinal segments sympathetic chain, B. Adrenal coex has no nerve supply, C. Adrenal medulla has no nerve supply, D. Release of catecholamines is not affected by nerve supply",A: Preganglionic fibres from lower thoracic & upper lumbar spinal segments sympathetic chain,"Answer: A: Preganglionic fibres from lower thoracic & upper lumbar spinal segments sympathetic chain Explanation: Preganglionic fibers from L thoracic & upper lumbar segment sympathetic. The suprarenal gland has a rich supply of nerves - myelinated preganglionic sympathetic fibers (thoracolumbar). the chromaffin cells in it are considered homologous to post ganglionic sympathetic fibers. Adrenal coex also receives some sympathetic innervation, controlling the release of coisol etc. (besides being under hormonal control) * Thoracolumbar sympathetic neurons arise from the intermedio - lateral horn cells of the spinal cord * These preganglionic nerves bypass 2 ganglion chains - the preveebral & paraveebral ganglionic chains" "This is a real-world medical entrance exam question, please give the true answer based on the question and selection. Topic name: None, Subject name: Ophthalmology","Maximum contribution to the refractive power of the eye is by which pa of the eye? Select and explain. A. Anterior surface of cornea, B. Posterior surface of cornea, C. Anterior surface of lens, D. Posterior surface of lens",A: Anterior surface of cornea,"Answer: A: Anterior surface of cornea Explanation: Ans: A. Anterior surface of cornea Maximum contribution to refractive power of eye - By anterior surface of cornea.Cornea's anterior surface is approximately spherical with a radius of curvature that is typically 8 mm.This surface is responsible for about two-thirds of the eye's refractive power." "This is a real-world medical entrance exam question, please give the true answer based on the question and selection. Topic name: None, Subject name: Dental","If the interocclusal distance is increased beyond physiologic limits, the patients chief complaint may result from: Select and explain. A. A muscular imbalance, B. An occlusal disharmony in centric, C. A displacement of the mandibular denture, D. A displacement of the maxillary denture while yawning",A: A muscular imbalance,"Answer: A: A muscular imbalance Explanation: None" "This is a real-world medical entrance exam question, please give the true answer based on the question and selection. Topic name: None, Subject name: Radiology","The best radiograph to obtain a view of maxillary sinus is: Select and explain. A. Lateral oblique maxilla, B. Submentovertex, C. Waters view, D. Lateral skull",C: Waters view,"Answer: C: Waters view Explanation: None" "This is a real-world medical entrance exam question, please give the true answer based on the question and selection. Topic name: None, Subject name: Gynaecology & Obstetrics","When would you do trans-vaginal sonography in post-menopausal bleeding if endometrial thickness is? Select and explain. A. 5mm, B. 7mm, C. 4mm, D. 9mm",A: 5mm,"Answer: A: 5mm Explanation: Ans. A. 5mmRef: RCOG guidelines 2012" "This is a real-world medical entrance exam question, please give the true answer based on the question and selection. Topic name: None, Subject name: Dental","Fracture of gold restoration primarily depends on Select and explain. A. Restoration design, B. Type of alloy, C. Noble metal & base metal ratio, D. All of above",A: Restoration design,"Answer: A: Restoration design Explanation: None" "This is a real-world medical entrance exam question, please give the true answer based on the question and selection. Topic name: None, Subject name: Pharmacology","Manufaeturer of a drug company labels that the drug contains 500 mg of paracetamol. On analysis' if was found to contain only 200 mg of drug. Which type of drug it is known as? Select and explain. A. Counterfeit drug, B. Adulterant drug, C. Spurious drug, D. Unethical drug",A: Counterfeit drug,"Answer: A: Counterfeit drug Explanation: Ans: A. Counterfeit drugRef: and hxp://www.globalforumljdorg/sites/default/files/docs/tibrary/lllflO-Evolution%20of/o20Definitions%20Definition.pdfand to the US FDA Counterfeit medicine is fake medicine.It may be contaminated or contain the wrong or no active ingredient.They could have the right active ingredient but at the wrong dose.Counterfeit drugs are illegal and may be harmful to your health." "This is a real-world medical entrance exam question, please give the true answer based on the question and selection. Topic name: None, Subject name: Pediatrics","A neonate presents with jaundice and clay coloured stools. Liver biopsy shows giant cells diagnosis is – Select and explain. A. Neonatal hepatitis with physiological jaundice, B. Neonatal hepatitis with extrahepatic biliary atresia, C. Physiological jaundice, D. Physiological jaundice with extrahepatic biliary atresia",B: Neonatal hepatitis with extrahepatic biliary atresia,"Answer: B: Neonatal hepatitis with extrahepatic biliary atresia Explanation: Jaundice with clay coloured stools suggests cholestasis. Both neonatal hepatitis and extrahepatic biliary atresia cause cholestasis jaundice. ""Giant cells"" are characteristics of both neonatal hepatitis and Extrahepatic Biliary Atresia. Therefore the answer is Neonatal hepatitis with extrahepatic Biliary Atresia. Idiopathic neonatal hepatitis It is also referred to as giant cell hepatitis. This type of cholestatic jaundice of unknown cause presents with features of cholestasis and a typical liver biopsy appearance. It is characterized by - i) Obstructive jaundice (cholestasis) ii) Giant cell transformation in liver                 Ili) Patent bile ducts Presenting features are suggestive of intrahepatic cholestasis - Acholic stools                          o Emesis                                o IUGR Poor feeding                            o Prematurity                       o Poor growth" "This is a real-world medical entrance exam question, please give the true answer based on the question and selection. Topic name: None, Subject name: Surgery","'Potts puffy' tumour is: Select and explain. A. A malignant tumour, B. An extradural abscess, C. A tumour or pituitary gland, D. An adrenal gland tumour",B: An extradural abscess,"Answer: B: An extradural abscess Explanation: None" "This is a real-world medical entrance exam question, please give the true answer based on the question and selection. Topic name: None, Subject name: Pathology","Event of early acquisition of malignant phenotype of epithelium is an expression of: Select and explain. A. Cyclin a, B. Cyclin b, C. Cyclin c, D. Cyclin d",D: Cyclin d,"Answer: D: Cyclin d Explanation: Conclusion of this study was Deregulation of the p16/ pRb/cyclin DI pathway is an early event in acquisition of dysplasia, but deregulation of both pRb and p53 pathway is associated with malignant transformation and adverse prognosis in oral tumorigenesis." "This is a real-world medical entrance exam question, please give the true answer based on the question and selection. Topic name: None, Subject name: Dental","In constructing a fixed partial denture for a patient, the dentist will use a hygienic pontic. Which of the following will primarily determine the faciolingual dimension of the occlusal portion of this pontic? Select and explain. A. The length of the pontic, B. The masticatory, force of the patient, C. The position of the opposing contact areas, D. The width and crestaL position of the edentulous ridge",C: The position of the opposing contact areas,"Answer: C: The position of the opposing contact areas Explanation: None" "This is a real-world medical entrance exam question, please give the true answer based on the question and selection. Topic name: None, Subject name: Anatomy","All of the following are diarthroses type of joint except: Select and explain. A. Plane, B. Symphyses., C. Condyloid., D. Saddle.",B: Symphyses.,"Answer: B: Symphyses. Explanation: None" "This is a real-world medical entrance exam question, please give the true answer based on the question and selection. Topic name: None, Subject name: ENT","Which of the following is premalignant - Select and explain. A. Chronic glossitis, B. Submucous fibrosis, C. Hypertrophic glossitis, D. Aphthous stomatitis",B: Submucous fibrosis,"Answer: B: Submucous fibrosis Explanation: Lesions and conditions of the oral mucosa associated with an increased risk of malignancy." "This is a real-world medical entrance exam question, please give the true answer based on the question and selection. Topic name: None, Subject name: Dental","In treating a tooth with a nonvital pulp and a sinus, the sinus should be treated as follows: Select and explain. A. No special treatment, B. Cauterisation, C. Curettage of sinus, D. Use of cautery to eliminate the sinus",A: No special treatment,"Answer: A: No special treatment Explanation: None" "This is a real-world medical entrance exam question, please give the true answer based on the question and selection. Topic name: None, Subject name: Physiology","What is the rise of blood flow if the radius of blood vessels is increased by 50%? Select and explain. A. 5 times, B. 10 times, C. 20 times, D. 100 times",A: 5 times,"Answer: A: 5 times Explanation: Ans: A. 5 times. Poiseuille's law:* Also referred as ""Hagen-Poiseuille law"".* Poiseuille's equation states, - Q = P1 - P2 * { (P r4) / (8 e L)} - Q - Flow rate - (P1 - P2) - Pressure difference across vessel (provided P1 > P2). - e - Blood viscosity. - r - Radius. - L - Tube length.* If parameter values remain constant, - Blood flow is directly propoional to 4th power of radius.* Resistance of vessel to blood flow can be calculated by combining Ohm's law with Poiseuille's equation. - By substituting values of Q from Poiseuille's law in Ohm's law. - Implying, resistance is mainly affected by, 1. Blood vessel radius, 2. Vasodilatation/vasoconstriction.* Thus ultimately, if parameter values remain constant, - Resistance to blood flow is inversely propoional to the 4th power of radius." "This is a real-world medical entrance exam question, please give the true answer based on the question and selection. Topic name: None, Subject name: Dental","Pain and sensitivity in a tooth with onlay after 4 months is due to: Select and explain. A. Supra occlusion, B. Gingivitis, C. Infra occlusion, D. None of the above",A: Supra occlusion,"Answer: A: Supra occlusion Explanation: None" "This is a real-world medical entrance exam question, please give the true answer based on the question and selection. Topic name: None, Subject name: Pathology","Which cell releases vasoactive amine so as to increase vascular permeability? Select and explain. A. Leukocyte, B. Macrophage, C. Mast cell, D. Fibroblast",C: Mast cell,"Answer: C: Mast cell Explanation: None" "This is a real-world medical entrance exam question, please give the true answer based on the question and selection. Topic name: None, Subject name: Dental","The only tissue - borne functional appliance is Select and explain. A. Activator, B. Bionator, C. Twin block, D. Frankel",D: Frankel,"Answer: D: Frankel Explanation: None" "This is a real-world medical entrance exam question, please give the true answer based on the question and selection. Topic name: None, Subject name: Dental","Incomplete casting is the result of: Select and explain. A. More porous investment in the mould, B. Narrow sprue diameter, C. Hollow sprue pin, D. Large reservoir",B: Narrow sprue diameter,"Answer: B: Narrow sprue diameter Explanation: None" "This is a real-world medical entrance exam question, please give the true answer based on the question and selection. Topic name: None, Subject name: Biochemistry","Restriction endonuclease cuts at Select and explain. A. AAGGAA, B. AAGAAG, C. AAGTTC, D. AAGCTT",D: AAGCTT,"Answer: D: AAGCTT Explanation: Answer-D. AAGCTT* AAGCTT is the only palindrome among the choices. Because the sequence of only one DNA strand is given, one must determine the base sequence of the complementary strand. To be a palindrome, both strands must have the* The vast majority of restriction endonucleases recognize palindromes, andsame sequence when read in the 5'-3' direction. Thus, the complement of 5'-AAGCTT-3' is also 5'-AAGCTT-3'." "This is a real-world medical entrance exam question, please give the true answer based on the question and selection. Topic name: None, Subject name: Dental","The flap technique for pocket elimination and to increase in width of attached gingiva is Select and explain. A. Coronally repositioned flap, B. Apically repositioned flap, C. Lateral pedicel flap, D. Modified widman flap",B: Apically repositioned flap,"Answer: B: Apically repositioned flap Explanation: None" "This is a real-world medical entrance exam question, please give the true answer based on the question and selection. Topic name: None, Subject name: Medicine","Hepatitis B occurs after an incubation period of: Select and explain. A. 1 week, B. 3 weeks, C. 3 months, D. 6 months",C: 3 months,"Answer: C: 3 months Explanation: None" "This is a real-world medical entrance exam question, please give the true answer based on the question and selection. Topic name: None, Subject name: Dental","A tooth can be made to appear shorter by Select and explain. A. Positioning gingival height of contour gingivally, B. Positioning gingival height of contour incisally, C. Positioning mesial and distal line angles closer, D. Positioning developmental depressions further apart",B: Positioning gingival height of contour incisally,"Answer: B: Positioning gingival height of contour incisally Explanation: None" "This is a real-world medical entrance exam question, please give the true answer based on the question and selection. Topic name: None, Subject name: Gynaecology & Obstetrics","True about placenta accreta is: Select and explain. A. Seen in cesarean scar, B. Removal should be done under GA in piecemeal, C. Chorionic villi invade serosa, D. It is an etiological factor for amniotic fluid embolism",A: Seen in cesarean scar,"Answer: A: Seen in cesarean scar Explanation: None" "This is a real-world medical entrance exam question, please give the true answer based on the question and selection. Topic name: None, Subject name: Dental","Standardized technique of cleaning and shaping was given by: Select and explain. A. Schilder, B. Ingle, C. Weine, D. Grossman",B: Ingle,"Answer: B: Ingle Explanation: None" "This is a real-world medical entrance exam question, please give the true answer based on the question and selection. Topic name: None, Subject name: Anatomy","In which of the following microvilli are not present? Select and explain. A. Gallbladder, B. Duodenum, C. Collecting duct, D. Proximal convoluted tubule",C: Collecting duct,"Answer: C: Collecting duct Explanation: Ans: C. Collecting duct Microvilli - Finger-like cell surface extensions.Usually 0.1 mm in diameter & up to 2 mm long.Absorptive surfaces of epithelial enterocytes of small intestine - Arranged in regular parallel series & constitute striated border.In gallbladder epithelium & proximal kidney tubules - Are less regular constitute ""brush border""." "This is a real-world medical entrance exam question, please give the true answer based on the question and selection. Topic name: None, Subject name: Medicine","In a patient with post-tuberculosis bronchiectasis, which of the following will you observe on auscultation? Select and explain. A. Late inspiratory crackles, B. Bibasilar crepts, C. Both early and late inspiratory crackles, D. Tubular breath sounds",C: Both early and late inspiratory crackles,"Answer: C: Both early and late inspiratory crackles Explanation: Ans: C. Both early and late inspiratory crackles(Ref: Harrison 19/e p1661-1665, 18/e p2143)Post-tuberculous bronchiectasis affect one lung only & produce coarse crackles, which are usually biphasic.Bihasilar crepitations:Typical of fluid overload (due to pulmonary edema).Occur in both inspiration and expiration. Combination of coarse and fine crackles.Feature of bronchiectasis.Related to a combination of secretions and increased compliance of the walls in larger airways.""Late inspiratory fine crackles - In COPD or pneumonia.Tubular breathing - In Lung consolidation." "This is a real-world medical entrance exam question, please give the true answer based on the question and selection. Topic name: None, Subject name: Dental","Radiograph of a periodontal ligament of a tooth which has lost its antagonist shows: Select and explain. A. Widening of the PDL space, B. Narrowing of PDL space, C. Increased density, D. Sclerotic change",B: Narrowing of PDL space,"Answer: B: Narrowing of PDL space Explanation: None" "This is a real-world medical entrance exam question, please give the true answer based on the question and selection. Topic name: None, Subject name: Dental","The property of amalgam that makes it undesirable to bevel occlusal margins of an amalgam cavity preparation is its: Select and explain. A. Flow, B. Ductility, C. Brittleness, D. Malleability",C: Brittleness,"Answer: C: Brittleness Explanation: None" "This is a real-world medical entrance exam question, please give the true answer based on the question and selection. Topic name: None, Subject name: Pediatrics","4 year old boy presented with recurrent chest infections. Sweat chloride test was done, showed values of 36 and 42. What is the next best investigation to confirm the diagnosis ? Select and explain. A. 72 hour fecal fat estimation, B. CT chest, C. Transepithelial nasal potential difference, D. DNA analysis of delta F 508 mutation",C: Transepithelial nasal potential difference,"Answer: C: Transepithelial nasal potential difference Explanation: Diagnosis of cystic fibrosis 1.   Sweat chloride testing The sweat test is the standard approach to diagnosis. The diagnosis is made by elevated sodium and chloride level in the sweat > 60 meq/1. Two test on different days are required for accurate diagnosis. A normal sweat chloride dose not exclude the diagnosis. Genotyping and other tests such as measurement of nasal membrane potential difference, pancreatic function should be done if there is high clinical suspicion of cystic fibrosis. 2.   Nasal potential difference Measurement of nasal transepithelial potential difference in vivo can be useful adjunct in the diagnosis of cystic fibrosis. Individuals with cystic fibrosis demonstrate a significantly more negative baseline nasal potential difference, with the topical application of amiloride there is loss of this potential difference. 3.   Genetic analysis Cystic fibrosis is an autosomal recessive disorder. It is caused due to defect in CFTR (Cystic fibrosis transmembrane conductance regulator) protein. Cystic fibrosis is associated with large number of mutations. More than 1500 CFTR polymorphisms are associated with cystic fibrosis syndrome. The most prevalent mutation of CFTR is the deletion of single phenylalanine residue at amino acid A508. This mutation is responsible for high incidence of cystic fibrosis in northern European populations. Approximately 50% of individuals with CF who are of northern European ancestry are homozygous for A508 and >70% carry at least one A508 gene. The remainder of patients has an extensive array of mutation, none of which has prevalance of more than several percent. Testing for cystic fibrosis mutation was not possible because of the large no. of mutations associated with the disease. Now days commercial laboratories test for 30-80 of the most common CFTR mutations. This testing identifies >90% individuals who carry 2CF mutations. No where it is mentioned in the texts that testing only for A508 is enough for diagnosis. Detection of atleast 2 CF mutations are necessary for making the diagnosis of cystic fibrosis." "This is a real-world medical entrance exam question, please give the true answer based on the question and selection. Topic name: None, Subject name: Surgery","Moality associated with emergency abdominal aoic surgery is: Select and explain. A. 10%, B. 20%, C. 40%, D. >50%",C: 40%,"Answer: C: 40% Explanation: Answer- c. 40%With careful preopetative csrdiac evaluation and postoperative care, the operative moality rate approximates 1-2%.After acute rupture, the moality rate of emergent operation is 45-50%. Endovascular repair with stent placement is an emerging approach but at the current time is associated with a moality rate of approximately 40%." "This is a real-world medical entrance exam question, please give the true answer based on the question and selection. Topic name: None, Subject name: Dental","Without supervision, allowed to give dentures Select and explain. A. Denturist, B. Hygienist, C. School Dental nurse, D. Dental therapist",A: Denturist,"Answer: A: Denturist Explanation: None" "This is a real-world medical entrance exam question, please give the true answer based on the question and selection. Topic name: None, Subject name: Dental","An apically repositioned flap Select and explain. A. Does not preserve the attached gingiva, B. Does not increase the length of clinical crown, C. Is the procedure of choice for palatal pockets, D. Is a pocket elimination procedure",D: Is a pocket elimination procedure,"Answer: D: Is a pocket elimination procedure Explanation: None" "This is a real-world medical entrance exam question, please give the true answer based on the question and selection. Topic name: None, Subject name: Dental","Which of the following cells are most increased in number in the connective tissue wall of a periodontal pocket? Select and explain. A. Plasma cells, B. Leukocytes, C. Lymphocytes, D. Erythrocytes",A: Plasma cells,"Answer: A: Plasma cells Explanation: None" "This is a real-world medical entrance exam question, please give the true answer based on the question and selection. Topic name: None, Subject name: Dental","Which of the following classification uses the ""canine law"" malocclusion in a sagittal direction Select and explain. A. Bonnet's classification, B. Simon classification, C. Dewey classification, D. Lischer's classification",B: Simon classification,"Answer: B: Simon classification Explanation: None" "This is a real-world medical entrance exam question, please give the true answer based on the question and selection. Topic name: None, Subject name: Dental","Stiffness is? Select and explain. A. Resistance to elastic deformation, B. Degree of elastic deformation, C. Resistance to plastic deformation, D. Degree of plastic deformation",A: Resistance to elastic deformation,"Answer: A: Resistance to elastic deformation Explanation: None" "This is a real-world medical entrance exam question, please give the true answer based on the question and selection. Topic name: AIIMS 2018, Subject name: Biochemistry","Which of the following type of collagen is present in healing and granulation tissue? Select and explain. A. Type I, B. Type II, C. Type III, D. Type IV",C: Type III,"Answer: C: Type III Explanation: COLLAGEN TYPE TYPE DISTRIBUTION I Skin Most abundant II Connective tissue cailage and vitreous humor III Aeries and CVS Healing and Granulation tissue IV Basement membrane Defect lead to Alpo syndrome Gene defect - COL4A3-COL4A6 AUTOSOMAL and X linked Hematuria + OCULER Problem + hearing loss VII Junction of dermal and epidermal Defect lead to Epidermolysis bullosa Gene defect - COL7A1" "This is a real-world medical entrance exam question, please give the true answer based on the question and selection. Topic name: None, Subject name: Pharmacology","Which one of the following drugs are not used in hea failure? Select and explain. A. Metoprolol, B. Nesiritide, C. Trimetazidine, D. Sacubitril",C: Trimetazidine,"Answer: C: Trimetazidine Explanation: Ans. C.Trimetazidine* Trimetazidine is not used in hea failure.* Trimetazidine is a newer antianginal drug that is a pFOX inhibitor which paially inhibits the fatty acid oxidation pathway in the myocardium. This decreases the oxygen requirement of the hea." "This is a real-world medical entrance exam question, please give the true answer based on the question and selection. Topic name: None, Subject name: Dental","Which of the following hematological disease is associated with periodontal disease? Select and explain. A. AIDS, B. Hypophosphatesia, C. Wegener's granulomatosis, D. Histocytosis",A: AIDS,"Answer: A: AIDS Explanation: None" "This is a real-world medical entrance exam question, please give the true answer based on the question and selection. Topic name: None, Subject name: Pathology","Which among the following laboratory investigation is best to reveal bleeding in Disseminated Intravascular Coagulation (DtC.? Select and explain. A. Increased PT, B. Increased aPTT, C. Decreased fibrinogen, D. Increased FDPs",D: Increased FDPs,"Answer: D: Increased FDPs Explanation: Ans. D. Increased FDPsRef: Essentials of Haematology by Shirish Kawathalkar, 2""d ed., pg. 442-147Two types of DIC are acute (decompensated and chronic cornpensated).Acute DIC:Findings in acute DIC are low platelet, prolonged PT & APTT, low fibrinogen, and increased FDP and D-dimer.Chronic DlC:Findings are normal platelet count, normal PT & APTT.However, FDP and D-dimer are increased.So in acute DIC, all the four options are correct, while if we consider the question as chronic DIC, then best possible answer is increased FDP." "This is a real-world medical entrance exam question, please give the true answer based on the question and selection. Topic name: None, Subject name: Dental","All of the following conditions are true regarding false positive results on EPT testing except? Select and explain. A. Immature apex, B. Improper Isolation, C. Partial Pulpal Necrosis, D. Anxiety",A: Immature apex,"Answer: A: Immature apex Explanation: None" "This is a real-world medical entrance exam question, please give the true answer based on the question and selection. Topic name: None, Subject name: Dental","Creep rate decreases with: Select and explain. A. Increase force of condensation, B. Decrease force of condensation, C. Under or overtrituration, D. Dealy in time between trituration and condensation",A: Increase force of condensation,"Answer: A: Increase force of condensation Explanation: None" "This is a real-world medical entrance exam question, please give the true answer based on the question and selection. Topic name: None, Subject name: Dental","How much Is the connective tissue width around an Implant Select and explain. A. 2-3 mm, B. 3-3.65 mm, C. 4-4.5 mm, D. 4-5 mm",A: 2-3 mm,"Answer: A: 2-3 mm Explanation: None" "This is a real-world medical entrance exam question, please give the true answer based on the question and selection. Topic name: None, Subject name: Medicine","Monitoring and diagnosis after completion of H. pylori treatment is done through Select and explain. A. Gastric acid test, B. urea breath test, C. bacterial culture of stool, D. sodium chloride test",B: urea breath test,"Answer: B: urea breath test Explanation: None" "This is a real-world medical entrance exam question, please give the true answer based on the question and selection. Topic name: None, Subject name: Medicine","ECG with ""mean"" axis of 90deg. In which lead there would be the maximum voltage of R wave? Select and explain. A. III, B. I, C. aVF, D. aVL",C: aVF,"Answer: C: aVF Explanation: Ans. C. aVF90deg cardiac axis corresponds straight to lead aVF and hence as such maximum 'It' wave amplitude would be seen in that lead only." "This is a real-world medical entrance exam question, please give the true answer based on the question and selection. Topic name: None, Subject name: Surgery","In arthroscopy, for TMJ adhesion lysis, the laser used is: Select and explain. A. Ho-Yag, B. Er-Yag, C. He-Ne, D. Argon",A: Ho-Yag,"Answer: A: Ho-Yag Explanation: None" "This is a real-world medical entrance exam question, please give the true answer based on the question and selection. Topic name: None, Subject name: Pediatrics","Which one of the following is the most common tumor associated with type I neurofibromatosis – Select and explain. A. Optic nerve glioma, B. Meningioma, C. Acoustic schwannoma, D. Low grade astrocytoma",A: Optic nerve glioma,"Answer: A: Optic nerve glioma Explanation: Neurofthromatosis type (Von-Reeltdinghewsen disease) NF-1 is diagnosed when any two of the following seven signs are present. 1. Six or more cafe-au-gait macules > 5 mm in prepupertal individuals > 15 mm in postpubertal individuals Cafe-au-lait spots are the hallmark of neurofibromatosis and are present in almost 100% of the patient. 2. Axillary or inguinal frecking 3. Two or more Lisch nodules. Lisch nodules are hamartomas located within the iris. 4. Two or more neurofibroma or one plexiform neurofibroma. Typically involve the skin, but may be situated along peripheral nerves and blood vessels. They are small, rubbery lesions with a slight purplish discoloration of the overlying skin. 5. A distinctive osseous lesion. Sphenoid dysplasia or cortical thinning of long bones. 6. Optic glioma 7. A first degree relative with NF-1 Other findings are : - Pseudoarthrosis of tibia. Scoliosis is the most common orthopaedic problem in NF-1, but is not specific enough to be included as a diagnostic criterian. Short stature Mental retardation, epilepsy Hypertension Aqueductal stenosis with hydrocephalus Meningiomas, ependynomas, Astrocytomas, pheochromocytomas. NF-1 is caused by mutation in NF-1 gene on chromosome 17 which encodes protein neurofibromin-1. Neurofibromatosis type -2 NF-2 may be diagnosed when one of the following two features are present. Bilateral ocoustic neuroma --> Most distinctive feature A parent, sibling or child with NF-2 and either unilateral eighth nerve masses or any two of the following Neurofibroma, meningioma, glioma, Schwannoma or juvenile post subcapsular cataract. NF-2 is cause by mutation in NF-2 gene on chromosome 22 that encodes for protein neurofibromin 2, Schwannomin or merlin." "This is a real-world medical entrance exam question, please give the true answer based on the question and selection. Topic name: None, Subject name: Dental","Etching depth after 30 sec for orthodontic bracket with orthophosphoric acid is: Select and explain. A. 10 μm — 20 μm, B. 3 μm — 9 μm, C. 50 μm — 110 μm, D. 200 μm — 250 μm",A: 10 μm — 20 μm,"Answer: A: 10 μm — 20 μm Explanation: None" "This is a real-world medical entrance exam question, please give the true answer based on the question and selection. Topic name: None, Subject name: Pediatrics","A 1.5 kg child born at 32 weeks by LSCS presents with moderate respiratory difficulty (RR 70/ minutes). Which of the following is the appropriate management – Select and explain. A. CPAP, B. Mechanical ventilation, C. Warm oxygen, D. Surfactant and ventilation",A: CPAP,"Answer: A: CPAP Explanation: Specific treatment for HMD is intratracheal surfactant therapy. This therapy requires endotracheal intubation, which also may be necessary to achieve adequate ventilation and oxygenation. Less premature infants (those > 1 kg or > 28-30 weeks gestation) and those with lower 02 requirements (Fi02 < 40 - 50%) may respond well to supplemental 02 alone or to treatment with nasal continuous positive airway pressure (CPAP)." "This is a real-world medical entrance exam question, please give the true answer based on the question and selection. Topic name: AIIMS 2019, Subject name: Pharmacology","A nurse got accidental prick from the HIV infected needle. Which of the following statements is false regarding the management of this nurse? Select and explain. A. Zidovudine is used as monotherapy for post-exposure prophylaxis, B. Washing hands with soap and water is advised, C. Baseline viral markers of health care personnel should be done at the time of presentation, D. Follow up viral markers of health care personnel should be measured at 6 weeks",A: Zidovudine is used as monotherapy for post-exposure prophylaxis,"Answer: A: Zidovudine is used as monotherapy for post-exposure prophylaxis Explanation: Post exposure prophylaxisTreatment should be staed within 72 hours of exposure, preferably as early as possible At first the wound is washed with soap and water. Secondly before initiating drug therapy blood samples are taken to measure the baseline viral markers Then three drug therapy is given for 28 days. The drugs include 2 NI PLUS a protease inhibitor (or alternative drug). Commonly used therapy include Tenofovir + Lamivudine + Atazanavir/r After 6 weeks again viral markers are measured to check for the infection. Post-exposure prophylaxis of HIV infection For adults and adolescents Preferred 2 NI: Tenofovir (300 mg) + Emtricitabine (200 mg) daily Preferred PI: Lopinavir/r (400 + 100 mg) or Atazanavir/r (300 + 100 mg) daily Alternative 3rd drug: Darunavir/r (600 + 100 mg BD) or Raltegravir/r (400 mg BD) or Eirenz (600 mg daily) For children <= 10 years Preferred 2 NI: Zidovudine + Lamivudine Alternative 3rd drug: Atazanavir/r or Darunavir/r or Eirenz or Raltegravir Preferred PI: Lopinavir Alternative 3rd drug: Atazanavir/r or Darunavir/r or Eirenz or Raltegravir Duration of regimen: 4 weeks" "This is a real-world medical entrance exam question, please give the true answer based on the question and selection. Topic name: None, Subject name: Dental","Which type of prefabricated post preparation system in mandibular molar is preferred? Select and explain. A. Tapered, screwed in, B. Parallel, screwed in, C. Tapered, passively cemented, D. Parallel, partially cemented","D: Parallel, partially cemented","Answer: D: Parallel, partially cemented Explanation: None" "This is a real-world medical entrance exam question, please give the true answer based on the question and selection. Topic name: None, Subject name: Physiology","All of the following increases calcium absorption from the gut except Select and explain. A. Phytates, B. Vitamin D, C. Alkaline pH in the gut, D. Protein in diet",C: Alkaline pH in the gut,"Answer: C: Alkaline pH in the gut Explanation: Ans: C. Alkaline pH in gut Factors increasing Calcium AbsorptionFactors decreasing Calcium AbsorptionVitamin DdegParathormonedegAcidic pHdegLactosedegAmino acidsdeg (protein rich diet)OxalatesdegPhytatesdegAlkaline pHdegHigh phosphatedegHigh Mg2+CaffeinesdegDietary fibersdeg" "This is a real-world medical entrance exam question, please give the true answer based on the question and selection. Topic name: None, Subject name: Dental","A screening test is used in the same way in two similar populations but the proportion of false positive results in population A is lower than that in population B. what is the likely explanation? Select and explain. A. The specificity of the test is lower in population A, B. The prevalence of the disease is lower in population A, C. The prevalence of the disease is higher in population A, D. The specificity of test is higher in population A",C: The prevalence of the disease is higher in population A,"Answer: C: The prevalence of the disease is higher in population A Explanation: None" "This is a real-world medical entrance exam question, please give the true answer based on the question and selection. Topic name: None, Subject name: Gynaecology & Obstetrics","If untreated, percentage of mother to child transmission of HIV during delivery without intervention in a non-breast fed child is: Select and explain. A. 40-50%, B. 10-15%, C. 15-30%, D. 5%",C: 15-30%,"Answer: C: 15-30% Explanation: Answer- C. 15-30%'In the absence of any intervention, an estimated 15-30% of mothers with HIY infection will transmit the infection duringprcgnsncy and delivery. and 10-20% will transmit the infection, through breast feeding. Veical transmission of IIIV-Loccurs mostly during the intrapaum period (50-70%).'- COGDT l0/e p692" "This is a real-world medical entrance exam question, please give the true answer based on the question and selection. Topic name: None, Subject name: Dental","Two palatal roots are found in what percentage of maxillary 1st molars Select and explain. A. 1-4%, B. 2-10%, C. 0-1 %, D. Two palatal roots are never found",A: 1-4%,"Answer: A: 1-4% Explanation: None" "This is a real-world medical entrance exam question, please give the true answer based on the question and selection. Topic name: None, Subject name: Physiology","The cell junctions allowing exchange of cytoplasmic molecules between two cells are called: Select and explain. A. Gap junctions, B. Tight junctions, C. Anchoring junctions, D. Focal junctions",A: Gap junctions,"Answer: A: Gap junctions Explanation: None" "This is a real-world medical entrance exam question, please give the true answer based on the question and selection. Topic name: None, Subject name: Surgery","A middle aged male patient presents with fever and diarrhea for 1 week and acute onset pain abdomen for 6 hours. An erect abdominal X-ray was taken as shown. What is the likely diagnosis? Select and explain. A. Pseudomembranous colitis, B. Adenocarcinoma colon, C. Pneumatosis intestinalis, D. Toxic megacolon",D: Toxic megacolon,"Answer: D: Toxic megacolon Explanation: Answer- D. Toxic megacolonToxic megacolon is a serious life-threatening condition that can occur in patients with ulcerative colitis, Crohn'scolitis, and infectious colitides such as pseudomembranous colitisThis decompensation results in a necrotic thin-walled bowel in which pneumatosisa can often be seen radiographically.Diagnosis:Plain abdominal radiographs are critical for diagnosing toxic megacolon and for following its course.Transverse colon is usually the most dilated >6 cm on supine films.Mutiple air-fluid levels in the colon are common; normal colonic haustlal pattern is either absent or severely disturbed." "This is a real-world medical entrance exam question, please give the true answer based on the question and selection. Topic name: None, Subject name: Dental","Difference between the skin and mucosa of cheek is: Select and explain. A. Thin lamina and non keratinized mucosa of cheek, B. Keratinized mucosa of cheek, C. Rete pegs, D. Thick lamina propria on skin of cheek",A: Thin lamina and non keratinized mucosa of cheek,"Answer: A: Thin lamina and non keratinized mucosa of cheek Explanation: None" "This is a real-world medical entrance exam question, please give the true answer based on the question and selection. Topic name: None, Subject name: Surgery","Scaphoid fracture at waist with retrograde blood supply. Which segment is most susceptible to avascular necrosis? Select and explain. A. Proximal, B. Distal, C. Middle, D. Scaphoid tubercle",A: Proximal,"Answer: A: Proximal Explanation: Answer- A. ProximalMost common site of scaphoid fracture is Waist.Fractures can occur essentially anywhere along the scaphoid, but distribution is not even:waist of scaphoid: 70-80%proximal pole: 20%distal pole (or so-called scaphoid tubercle): 10%" "This is a real-world medical entrance exam question, please give the true answer based on the question and selection. Topic name: None, Subject name: Skin","A 12 years old boy came with complaints of 4 hypopigmented patches on back and on left arm. The patches had loss of sensation. Which of the following is the treatment for this case? Select and explain. A. Rifampicin (450 mg) + Dapsone (50 mg) + Clofazimine (150 mg) monthly and 50 mg daily, B. Rifampicin (600 mg) + Dapsone (150 mg) only, C. Rifampicin (450 mg) + Dapsone (50 mg) + Clofazimine (150 mg) monthly and 50 mg alternate days, D. Rifampicin (600 mg) + Dapsone (150 mg) + Clofazimine (300 mg) monthly and 50 mg daily",C: Rifampicin (450 mg) + Dapsone (50 mg) + Clofazimine (150 mg) monthly and 50 mg alternate days,"Answer: C: Rifampicin (450 mg) + Dapsone (50 mg) + Clofazimine (150 mg) monthly and 50 mg alternate days Explanation: Answer- C. Rifampicin (450 mg) + Dapsone (50 mg) + Clofazimine (150 mg) monthly and 50 mg alternate daysThe clinical history fits the patient into multibacillary leprosy and requires extensive multidrug therapy for 12 months.Since the patient is aged 12 years, doses are Rifompicin (450 mg) * Dapsone (50 mg) + Clofazimine (150 mg) monthly and 50 mg alternate days." "This is a real-world medical entrance exam question, please give the true answer based on the question and selection. Topic name: None, Subject name: Pharmacology","Uses of atropine are A/E - Select and explain. A. Organophoshorus poisoning, B. Mushroom poisoning, C. Arrhythmia, D. Miotic",D: Miotic,"Answer: D: Miotic Explanation: Ans. is 'd' i.e., MioticCLINICAL USES OFATROPINE Remember - ATROPAA As mydriatic - cycloplegicT --> 'Traveller's diarrhoeaR --> Rapid (early) onset mushroom poisoning0 --> Organophosphate poisoningP --> Preanaesthetic medicationA Arrhythmias (brady-arrhythmias)Atropine is also used with neostigmine in mysthenia gravis to decrease anti,nuscarinic side effects of neostigmine --> As atropine blocks muscarinic receptors, use of atropine prevents muscarinic side effects of neostigmine, while neostigmine retains its benficial effects in mysthenics which are due to nicotinic receptors." "This is a real-world medical entrance exam question, please give the true answer based on the question and selection. Topic name: None, Subject name: Gynaecology & Obstetrics","What are the cut-off values in 2 hours oral glucose tolerance test for fasting and at 1 hour and 2 hours after meals respectively? Select and explain. A. 92, 182, 155, B. 92, 180, 153, C. 95, 180, 155, D. 92, 180, 155","B: 92, 180, 153","Answer: B: 92, 180, 153 Explanation: Ans: B. 92, 180, 153(Ref Williams 24/e p1137)Cut-off values in 2 hours oral glucose tolerance test for fasting and at 1 hour & 2 hours after meals respectively.Diagnosis of Gestational Diabetes by Oral Glucose Tolerance TestingTime 75-gm GlucoseFasting92 mg/dL5.1 mmol/L1-hour180 mg/dL10.0 mmol/L2-hours153 mg/dL8.5 mmol/L" "This is a real-world medical entrance exam question, please give the true answer based on the question and selection. Topic name: None, Subject name: Medicine","You have performed a pleural tap in a patient with suspected TB. You will send the sample for all of the following studies except: Select and explain. A. Gene Xpe, B. ADA, C. LDH, D. Albumin",D: Albumin,"Answer: D: Albumin Explanation: Ans: D. Albumin(Ref: Harrison 19/c p1715-1716)The most impoant diagnostic tests in pleural fluid specimen in a patient with suspected TB are GeneXpe (PCR) and ADA levels.Total protein levels and LDH levels are required to differentiate between transudative and exudative effusion according to Light's Criteria.So doing a pleural fluid albumin level separately will not be very useful.Total protein estimation should be done instead." "This is a real-world medical entrance exam question, please give the true answer based on the question and selection. Topic name: None, Subject name: Pathology","HOX gene is responsible for which malformation Select and explain. A. Polysyndactyly, B. Holoprosencephaly, C. Mayer Rokitansky syndrome, D. Gorlin syndrome",A: Polysyndactyly,"Answer: A: Polysyndactyly Explanation: None" "This is a real-world medical entrance exam question, please give the true answer based on the question and selection. Topic name: AIIMS 2018, Subject name: Gynaecology & Obstetrics","Possible conversion to choriocarcinoma after hydatidiform mole is denoted by all of the following, except:- Select and explain. A. Rising hCG, B. More Theca lutein cysts, C. Increase uterus size, D. Sub urethral nodule",D: Sub urethral nodule,"Answer: D: Sub urethral nodule Explanation: Pre-disposing factors for development of choriocarcinoma after molar pregnancy - Rising hCG >100,000 Increased uterus size >6 cm Theca-lutein cysts * Sub urethral nodule indicates vaginal metastasis from choriocarcinoma ,it is not a predisposing factor. * Do not try to biopsy the sub urethral nodule it will result in torrential hemorrhage." "This is a real-world medical entrance exam question, please give the true answer based on the question and selection. Topic name: None, Subject name: Surgery","Lateral pharyngeal space is not connected directly by: Select and explain. A. Buccal space, B. Sublingual space, C. Submandibular space, D. Retropharyngeal space",A: Buccal space,"Answer: A: Buccal space Explanation: None" "This is a real-world medical entrance exam question, please give the true answer based on the question and selection. Topic name: None, Subject name: Surgery","On abdominal ultrasound gallbladder shows diffuse wall thickening with hyperechoic nodules at neck and comet tail aifacts. The most likely diagnosis will be: Select and explain. A. Adenomyomatosis, B. Adenocarcinoma of gallbladder, C. Xanthogranulomatous cholecystitis, D. Cholesterol crystals",A: Adenomyomatosis,"Answer: A: Adenomyomatosis Explanation: Ans.A. AdenomyomatosisAdenomyomatosis is a benign condition characterized by hyperplastic changes of unknown etiology involving the GB wall.It causes overgrowth of the mucosa, thickening of muscular wall, and formation of intramural diveicula or sinus tracts termed as Aschoff-Rokitansky sinuses.Investigations-USG- The presence of cholesterol crystals in these sinuses can result in ""diamond ring sign"", ""V-shaped"", or ""comet-tail"" aifacts on USG.Treatment- Cholecystectomy is indicated in symptomatic adenomyomatosis or when cholelithiasis is present." "This is a real-world medical entrance exam question, please give the true answer based on the question and selection. Topic name: None, Subject name: Dental","p' value is 0.00, then Select and explain. A. Results are true of 99.9% of population, B. Test is not significant, C. 1% of significance, D. There is no co-relation",A: Results are true of 99.9% of population,"Answer: A: Results are true of 99.9% of population Explanation: None" "This is a real-world medical entrance exam question, please give the true answer based on the question and selection. Topic name: None, Subject name: Biochemistry","The amino acid, which is used in the estimation of collagen is: Select and explain. A. Hydroxyproline, B. Proline, C. Lysine, D. Glycine",A: Hydroxyproline,"Answer: A: Hydroxyproline Explanation: None" "This is a real-world medical entrance exam question, please give the true answer based on the question and selection. Topic name: None, Subject name: Pathology","A child with Down's syndrome has Moon facies, retarded mentally and which of the facial characteristics: Select and explain. A. Maxillary prognathism, B. Mandibular retrognathia, C. Mandibular prognathism, D. Maxillary hypoplasia",D: Maxillary hypoplasia,"Answer: D: Maxillary hypoplasia Explanation: None" "This is a real-world medical entrance exam question, please give the true answer based on the question and selection. Topic name: None, Subject name: Gynaecology & Obstetrics","Best Age to sta bone mineral density test in female. Select and explain. A. After 50 years, B. After 55 years, C. After 60 years, D. After 65 years",D: After 65 years,"Answer: D: After 65 years Explanation: Ans.d.After 65 yearsNOF recommends:Bone density test :In woman age 65 or older.In man age 70 or older." "This is a real-world medical entrance exam question, please give the true answer based on the question and selection. Topic name: None, Subject name: Social & Preventive Medicine","Haddon matrix is related to: Select and explain. A. Injury prevention, B. Communicable diseases, C. Maternal and child moality, D. Hypeensive disorders",B: Communicable diseases,"Answer: B: Communicable diseases Explanation: Ans: B. Communicable diseasesHaddon matrix:Related to injury prevention.Most commonly used paradigm in injury prevention field." "This is a real-world medical entrance exam question, please give the true answer based on the question and selection. Topic name: AIIMS 2017, Subject name: ENT","Which of the following marked arrow represent lateral semicircular canal during coical mastoidectomy: Select and explain. A. A, B. B, C. C, D. D",C: C,"Answer: C: C Explanation: A - Incus, B - Facial nerve, C - lateral semicircular canal, D - Chorda tympani." "This is a real-world medical entrance exam question, please give the true answer based on the question and selection. Topic name: None, Subject name: Surgery","Expected surgical procedure most frequently indicated for odontogenic cysts is: Select and explain. A. Incision and drainage, B. Sclerosing solution, C. Marsupialisation, D. Enucleation",D: Enucleation,"Answer: D: Enucleation Explanation: None" "This is a real-world medical entrance exam question, please give the true answer based on the question and selection. Topic name: None, Subject name: Pathology","A boy complains of bleeding gums, swollen, Joints with hemorrhage into joints. His paternal and maternal uncle complains of same problem. It is due to deficiency of factor: Select and explain. A. VIII, B. IX, C. X, D. VI",A: VIII,"Answer: A: VIII Explanation: None" "This is a real-world medical entrance exam question, please give the true answer based on the question and selection. Topic name: None, Subject name: Anatomy","Deepest layer of deep cervical fascia is Select and explain. A. Prevertebral, B. Carotid sheath, C. Pretracheal, D. Temporal",A: Prevertebral,"Answer: A: Prevertebral Explanation: Deep cervical fascia, also known as fascia colli is condensed to form 6 layers. Investing layer. Pretracheal layer. Prevertebral layer. Carotid sheath. Buccopharyngeal fascia. Pharyngobasilar fascia. Prevertebral fascia: It is the deepest layer of deep cervical fascia. It lies in front of prevertebral muscles and forms the floor of posterior triangle of neck. It is separated from pharynx and buccopharyngeal fascia by retropharyngeal space." "This is a real-world medical entrance exam question, please give the true answer based on the question and selection. Topic name: None, Subject name: Physiology","In the formula for urea clearance, C = U x V/P, what does U stands for: Select and explain. A. Urinary concentration in g/24 hour, B. Urinary concentration in mg/ml, C. Urine osmolarity, D. Urine volume per minute",B: Urinary concentration in mg/ml,"Answer: B: Urinary concentration in mg/ml Explanation: Ans: B. Urinary concentration in mg/ml(Ref: Ganong 25/p p676. 21/c p678)In given formula for measuring GFR:C = Clearance of the substanceU = Urinary concentration of the substance in mg/mlP = Plasma concentration of the substance in mg/mlV = Volume of urine" "This is a real-world medical entrance exam question, please give the true answer based on the question and selection. Topic name: None, Subject name: Dental","Trisodium phosphate in alginate acts as: Select and explain. A. Retarder, B. Reactor, C. Accelerator, D. Plasticizer",A: Retarder,"Answer: A: Retarder Explanation: None" "This is a real-world medical entrance exam question, please give the true answer based on the question and selection. Topic name: None, Subject name: Dental","For measuring hardness of Co-Cr alloy, which hardness test is used? Select and explain. A. Vicat apparatus, B. Cold bend, C. Heat and cold bend, D. Rockwell hardness test",D: Rockwell hardness test,"Answer: D: Rockwell hardness test Explanation: The Vicat apparatus is commonly used to measure the initial setting time of gypsum products.  A method for the measurement of ductility is known as the cold bend test. Rockwell Hardness Number (RHN) Like the Brinell hardness test, a steel ball or a conical diamond point is used. However, instead of measuring the diameter of the impression, the depth is measured directly by a dial gauge on the instrument.  Application: The Rockwell test has a wider application for materials, since Brinell test is unsuitable for brittle materials as well as plastic materials. Ref- Mannapallil BASIC DENTAL MATERIALS  P:24" "This is a real-world medical entrance exam question, please give the true answer based on the question and selection. Topic name: AIIMS 2018, Subject name: Pathology","A patient presented with intermittent fever, no weight loss, no anorexia, but with a retroperitoneal mass. Peripheral smear findings were normal. Gross & microscopy of the mass is given. What's the diagnosis? Select and explain. A. NHL, B. Castleman disease, C. Angiolymphoid hyperplasia, D. Ig G4 disease",B: Castleman disease,"Answer: B: Castleman disease Explanation: Onion skin appearance of lymph node is characteristic for Castleman disease It is alympho proliferative disorder. It has 2 variants: Unicentric Has 2 variants: 1)Highly vasculara More common , patients are asymptomatic Histology: -Twinning- Presence of two germinal centers -Onion skin pattern-Lymphocytes causes thickening of mantle zone -Lollipop lesions- Sclerosed blood vessel that infiltrate lymph node germinal center Multicentric ?Associated with immune suppression ?Presents as : Hepatomegaly Splenomegaly Night sweats Fever 2)Plasma cell varianta Less common, patients are symptomatic: - Weight loss -Night sweats -Fever Histology: Plasma cells in interfollicular areas causing hyperplasia of germinal center Lymph node architecture is distoed but LN is preserved in unicentric variant" "This is a real-world medical entrance exam question, please give the true answer based on the question and selection. Topic name: None, Subject name: Surgery","One day after complete mouth extraction, blue black spots are seen on the neck of the patients. These spots indicate: Select and explain. A. Thrombocytopenic purpura, B. Postoperative ecchymosis, C. Impaired blood circulation, D. Cellulitis",B: Postoperative ecchymosis,"Answer: B: Postoperative ecchymosis Explanation: None" "This is a real-world medical entrance exam question, please give the true answer based on the question and selection. Topic name: None, Subject name: Pathology","painful crater like 1.5 cm ulcer develops within one week on the hard palate mucosa of a 40 year old female. The most likely diagnosis is: Select and explain. A. Actinomycosis, B. Squamous cell carcinoma, C. Pleomorphic adenoma, D. Necrotizing sialometaplasia",D: Necrotizing sialometaplasia,"Answer: D: Necrotizing sialometaplasia Explanation: None" "This is a real-world medical entrance exam question, please give the true answer based on the question and selection. Topic name: AIIMS 2017, Subject name: Medicine","Most common cause of lobar hemorrhage in elderly age group ? Select and explain. A. Hypeension, B. Vascular malformation, C. Coagulopathy, D. Amyloid angiopathy",D: Amyloid angiopathy,"Answer: D: Amyloid angiopathy Explanation: Though hypeension is the most common cause of intracerebral hemorrhage, the usual site for it is putamen, thalamus, pons and cerebellum. But lobar hemorrhage in elderly is most commonly due to cerebral amyloid angiopathy, where there is amyloid deposition in the walls of cerebral aeries following aeriolar degeneration." "This is a real-world medical entrance exam question, please give the true answer based on the question and selection. Topic name: None, Subject name: Medicine","Which of the following is least likely to cause infective endocarditis: Select and explain. A. Staphylococcus albus., B. Streptococcus faecalis., C. Salmonella typhi., D. Pseudomonas aeruginosa.",C: Salmonella typhi.,"Answer: C: Salmonella typhi. Explanation: None" "This is a real-world medical entrance exam question, please give the true answer based on the question and selection. Topic name: None, Subject name: Pathology","Platelet derived growth factor is released in vessels from: Select and explain. A. Endothelial cells, B. Fibroblasts, C. Macrophages, D. Alpha granules",D: Alpha granules,"Answer: D: Alpha granules Explanation: Platelet activation:   The adherent platelets get activated, undergo a shape change and degranulate. The granules in the platelets can be Alpha  granules  having  P-selectin,  fibrinogen,  fibronectin,  factors  V  and  VIII, platelet  factor  4,  platelet-derived  growth  factor,  and  transforming  growth factor-β. Delta  granules  or  delta  bodies  having  ADP,  ATP,  ionized  calcium,  histamine, serotonin, and epinephrine." "This is a real-world medical entrance exam question, please give the true answer based on the question and selection. Topic name: None, Subject name: Pathology","Down syndrome is caused by all except: Select and explain. A. Trisomy of 21 chromosome, B. Mosaicism of 21 chromosome, C. Robertsonian translocation of 21.21, 21.18, D. Deletion of 21",D: Deletion of 21,"Answer: D: Deletion of 21 Explanation: Three cytogenetic variants cause Down syndrome:  Trisomy 21 Chromosomal translocation Mosaicism Trisomy 21 accounts for nearly 95% of all patients with Down syndrome. It is now generally accepted that there are at least three forms of Down syndrome:  One in which there is the typical trisomy 21 with 47 chromosomes (accounting for about 95% of cases) Another termed the translocation type, in which there appear to be only 46 chromosomes, although the extra chromosome material of number 21 is translocated to another chromosome of G or D group, either 21/22 translocation or 21/21 translocation (about 3% of cases) Another that is the result of chromosomal mosaicism (about 2%) Rajendran R. Shafer's textbook of oral pathology. Edition 7. Page 729" "This is a real-world medical entrance exam question, please give the true answer based on the question and selection. Topic name: None, Subject name: Dental","Which of the following is the most common organism found in retreatment cases of periapical abscess? Select and explain. A. E. faecalis, B. Trepenoma denticola, C. S. mutans, D. Actinomyces",A: E. faecalis,"Answer: A: E. faecalis Explanation: PERSISTENT APICAL PERIODONTITIS It is post-treatment apical periodontitis in an endodontically treated tooth (re-treatment case). Enterococcus faecalis is found the most consistently reported organism in persistent apical periodontitis." "This is a real-world medical entrance exam question, please give the true answer based on the question and selection. Topic name: None, Subject name: Surgery","Maximum chances of fracture are present when which of these forces are present on a bone: Select and explain. A. Tension, B. Torsion, C. Compression, D. Either of the three",A: Tension,"Answer: A: Tension Explanation: None" "This is a real-world medical entrance exam question, please give the true answer based on the question and selection. Topic name: None, Subject name: Anatomy","Phrenic nerve chiefly supplies? Select and explain. A. Stomach, B. Diaphragm, C. Oesophagus, D. Ileum",B: Diaphragm,"Answer: B: Diaphragm Explanation: None" "This is a real-world medical entrance exam question, please give the true answer based on the question and selection. Topic name: None, Subject name: Pathology","Regional Odontodysplasia is most common in Select and explain. A. Mandibular premolar, B. Mandibular canine, C. Mandibular third molar, D. Maxillary central incisor",D: Maxillary central incisor,"Answer: D: Maxillary central incisor Explanation: None" "This is a real-world medical entrance exam question, please give the true answer based on the question and selection. Topic name: None, Subject name: Surgery","During surgical excision of the parotid gland the following structures may be damaged: Select and explain. A. Lesser occipital nerve, hypoglossal nerve, chorda tympani, B. Facial nerve and auriculotemporal nerve, C. Submandibular duct, D. Cervical fascia",B: Facial nerve and auriculotemporal nerve,"Answer: B: Facial nerve and auriculotemporal nerve Explanation: None" "This is a real-world medical entrance exam question, please give the true answer based on the question and selection. Topic name: None, Subject name: Surgery","A dye injected in to space to see the joint is known as: Select and explain. A. Arthrography, B. Arthroscopy, C. Arthroplasty, D. Orthopantogram",A: Arthrography,"Answer: A: Arthrography Explanation: None" "This is a real-world medical entrance exam question, please give the true answer based on the question and selection. Topic name: None, Subject name: ENT","An elderly male presents with T3NO laryngeal carcinoma. What would be the management? Select and explain. A. Neoadjuvant chemotherapy followed by radiotherapy, B. Concurrent chemoradiotherapy, C. Radical radiotherapy followed by chemotherapy, D. Radical radiotherapy without chemotherapy",B: Concurrent chemoradiotherapy,"Answer: B: Concurrent chemoradiotherapy Explanation: Ans. b. Concurrent chemoradiotherapyAn elderly male presents with T3N0 laryngeal carcinoma. Treatment is concunent chemoradiotherapy.'Advanced laryngeal cancers are ofien treated by combining radiation with concurrent chemotherapy for larynxpresemation and total laryngectomy for bulky T4 disease or salvage.''Concurrent radiation therapy plus Cisplatin resulted in a statistically higher percentage ofpatients with an intact larynxat I0 years.'" "This is a real-world medical entrance exam question, please give the true answer based on the question and selection. Topic name: None, Subject name: Dental","Which is the only anterior teeth with Mesiodistal size more than Cervicoincisal dimension Select and explain. A. Deciduous lower central incisor, B. Permanent lower central incisor, C. Deciduous upper lateral incisor, D. Permanent upper lateral incisor",C: Deciduous upper lateral incisor,"Answer: C: Deciduous upper lateral incisor Explanation: None" "This is a real-world medical entrance exam question, please give the true answer based on the question and selection. Topic name: None, Subject name: Gynaecology & Obstetrics","Which of the following abnormalities is commonly seen in a fetus with congenital CMV infection: Select and explain. A. Colitis, B. Myocarditis, C. Blood dyscrasias, D. Pulmonary cyst",C: Blood dyscrasias,"Answer: C: Blood dyscrasias Explanation: Manifestation of congenital CMV infection: Still birth Microcephaly Choroidoretinitis  Deafness Hemolytic anemia (Blood dyscrasias simply stands for any hematological disorder) Pneumonitis Thrombocytopenia with petechiae and purpura IUGR Hepatosplenomegaly Icterus Mental retardation Have intracranial calcifications" "This is a real-world medical entrance exam question, please give the true answer based on the question and selection. Topic name: None, Subject name: Pathology","White radiating Lines can be observed in case of Lesions of Select and explain. A. Lichen planus, B. Erythema multiforme, C. Pemphigus, D. Leukoplakia",A: Lichen planus,"Answer: A: Lichen planus Explanation: None" "This is a real-world medical entrance exam question, please give the true answer based on the question and selection. Topic name: None, Subject name: Surgery","Which among the following is not true about ankylosing spondylitis? Select and explain. A. Enthesis is the primary site of pathology, B. More common in Females, C. TNF plays an impoant role in the pathogenesis of ankylosing spondylitis, D. Sacroiliitis is the earliest manifestation of ankylosing spondylitis",B: More common in Females,"Answer: B: More common in Females Explanation: Ans.B. More common in FemalesAnkylosing spondylitis is more common in males.(M:F is between 2:1 to 3:1).There is preferential involvement of entheses ( Site of attachment of ligament or tendon to the bone).TNF a plays a major role in immunopathogenesis.Sacroiliitis is often the first manifestation of ankylosing spondylitis. Synovitis, pannus, subchondral granulation tissue and marrow edema, enthesitis and chondroid differentiation are also found." "This is a real-world medical entrance exam question, please give the true answer based on the question and selection. Topic name: None, Subject name: Pharmacology","Cephalosporin active against pseudomonas aeruginosa Select and explain. A. Cefoperazone, B. Cefaclor, C. Ceftriaxone, D. Cefotaxime",A: Cefoperazone,"Answer: A: Cefoperazone Explanation: None" "This is a real-world medical entrance exam question, please give the true answer based on the question and selection. Topic name: None, Subject name: Dental","The least effect on the prognosis of a periodontal disease is Select and explain. A. Suppuration from the pocket, B. Furcation involvement, C. Alveolar bone loss, D. Co-operation and motivation of the patient",A: Suppuration from the pocket,"Answer: A: Suppuration from the pocket Explanation: None" "This is a real-world medical entrance exam question, please give the true answer based on the question and selection. Topic name: None, Subject name: Anatomy","Occulomotor nerve passes through Select and explain. A. Foramen rotundum, B. Superior orbital fissure, C. Inferior orbital fissure, D. Foramen ovale",B: Superior orbital fissure,"Answer: B: Superior orbital fissure Explanation: None" "This is a real-world medical entrance exam question, please give the true answer based on the question and selection. Topic name: None, Subject name: Pediatrics","A child presents to the emergency with a history of ingestion of button battery, on X-ray it was found in the stomach or duodenum. What is the next step? Select and explain. A. Endoscopic removal of battery, B. Wait and watch, C. Repeat X-ray after 5 days, D. Immediate laparotomy",B: Wait and watch,"Answer: B: Wait and watch Explanation: Ans: B. Wait and watchRef: NBIH Button bsttery ingestion triage and treatment guideline.The management of a Button battery ingestion depends upon the anatomical site of the impacted battery.Button batteries that have cleared the stomach usually pass through the gastrointestinal tract within one week without complications.Follow-up radiographs should be performed in asymptomatic patients who have not passed the battery by 10 to l4 days, regardless ofsize or earlier if patient becomes symptomatic." "This is a real-world medical entrance exam question, please give the true answer based on the question and selection. Topic name: None, Subject name: Pediatrics","A child with tetralogy of fallot uses which of the following positions – Select and explain. A. Supine, B. Prone, C. Squatting, D. Leaning forwards",C: Squatting,"Answer: C: Squatting Explanation: The patients of TOF assume a sitting posture (squatting) as soon as they get dyspneic. TOF is the commonest congenital lesion in which squatting is noted." "This is a real-world medical entrance exam question, please give the true answer based on the question and selection. Topic name: None, Subject name: Dental","True about Arrangement of hydroxyapatite crystals in Enamel rods? Select and explain. A. In center orientation of hydroxyapatite crystal is 0 degree, B. Orientation of hydroxyapatite crystals at periphery is 45 degree, C. Both, D. None",C: Both,"Answer: C: Both Explanation: None" "This is a real-world medical entrance exam question, please give the true answer based on the question and selection. Topic name: None, Subject name: Dental","Predisposing clinical condition for endodontic flare up? Select and explain. A. Acute periapical abscess, B. Acute apical periodontitis, C. Asymptomatic necrotic pulp with periapical lesion, D. Pain and swelling since the treatment",C: Asymptomatic necrotic pulp with periapical lesion,"Answer: C: Asymptomatic necrotic pulp with periapical lesion Explanation: None" "This is a real-world medical entrance exam question, please give the true answer based on the question and selection. Topic name: None, Subject name: Pediatrics","Regarding ASO titre all are seen except – Select and explain. A. ASO can be increased in school children, B. May be negative in post streptococcal glomerulonephritis, C. ASO titre included in major criteria in jones criteria, D. May not be elevated in 20% cases of carditis",C: ASO titre included in major criteria in jones criteria,"Answer: C: ASO titre included in major criteria in jones criteria Explanation: Evidence of antecedent group 'A' streptococcal infection Positive throat cultures or rapid streptococcal antigen tests for group A streptococcoci are less reliable for antecedent infection capable of producing rheumatic fever because they do not distinguish between recent infection and chronic pharyngeal carriage (as many people are carrier of this bacteria). Antibody tests are the most reliable laboratory evidence of antecedent streptococcal infection capable of producing        g acute rheumatic fever. The onset of clinical manifestations of acute rheumatic fever coincides with the peak of the streptococcal antibody response. The antibodies used commonly for serological tests are antistreptolysin 0 (ASO), antideoxyribonuclease (Anti-DNAse) and antihyaluronidase. ASO titre is well standardized and therefore is most widely used test. It has following features. Elevated in 80% of patients with acute rheumatic fever (So, 20% patients do not show elevated titre) Sensitivity is 80%. It is elevated in 20% of normal individuals perticularly in healthy school children of elementary school age (20% false positive). ASO titers of at least 333 Todd units in children and 250 Todd units in adults are considered elevated. It is included in minor criteria of jones criteria (not in major criteria). Anti DNAse B titers of 240 Todd units or greater in children and 1200 Todd units or greater in adults are considered elevated. If three antibody tests (ASO, anti-DNAse and anti-hyaluronidase) are used simultaneously, a titer for at least one antibody test is elevated in 95% of cases —> Sensitivity of combined three antibody tests is 95%. The streptozyme test is a simple slide agglutination test for extra-cellular streptococcal antigen absorbed to red blood cells (passive hemagglutination) test. It is almost 100% sensitive but specificity is very low and it is less standardized and less reproducible than the other antibody tests. Therefore, it should not be used as a diagnostic test for evidence of antecedent group A streptococcal infection. About option 'b' ASO titre may not be elevated in PSGN if PSGN develops secondary to skin infection." "This is a real-world medical entrance exam question, please give the true answer based on the question and selection. Topic name: None, Subject name: Gynaecology & Obstetrics","28 yr with infeility lapro tube uterus healthy ovary diagnosis - Select and explain. A. PCOS, B. Ovarian cyst, C. Fibroid, D. Endometriosis",D: Endometriosis,"Answer: D: Endometriosis Explanation: Ans.D. EndometriosisIn endometriosis, cause of infeility isImmobility of tubesAnovulationTubal blockMale factor: 30%Tubal, uterine & peritoneal factor: 25%Ovarian factor: 25%Cervical factor: 10%Unexplained factor: 10%" "This is a real-world medical entrance exam question, please give the true answer based on the question and selection. Topic name: AIIMS 2017, Subject name: ENT","Which of the following is not the branch of external carotid aery in Kiesselbach's plexus? Select and explain. A. Anterior Ethmoidal aery, B. Sphenopalatine aery, C. Greater palatine aery, D. Septal branch of superior labial aery",A: Anterior Ethmoidal aery,"Answer: A: Anterior Ethmoidal aery Explanation: NASAL SEPTUM Internal Carotid System 1. Anterior ethmoidal aery 2. Posterior ethmoidal aery External Carotid System Branches of ophthalmic aery 1. Sphenopalatine aery (branch of maxillary aery) gives Nasopalatine and posterior medial nasal branches. 2. Septal branch of greater palatine aery (branch of maxillary aery). 3. Septal branch of superior labial aery (branch of facial aery). LATERAL WALL Internal carotid System 1. Anterior ethmoidal 2. Posterior ethmoidal External Carotid System Branches of ophthalmic aery 1. Posterior lateral nasal branches 2. Greater palatine aery 3. Nasal branch of anterior superior dental 4. Branches of facial aery To nasal vestibule From sphenopalatine aery From maxillary aery From infraorbital branch of maxillary aery" "This is a real-world medical entrance exam question, please give the true answer based on the question and selection. Topic name: None, Subject name: Surgery","Excess of plasma level of lignocaine can cause cardiovascular collapse due to: Select and explain. A. Myocardial depression, B. Vagal stimulation, C. Syncope, D. CN5 excitability",A: Myocardial depression,"Answer: A: Myocardial depression Explanation: None" "This is a real-world medical entrance exam question, please give the true answer based on the question and selection. Topic name: None, Subject name: Radiology","Best noninvasive investigation to check for bility of myocardium is: Select and explain. A. FDG-18 PET CT, B. MIBG scintigraphy, C. Echocardiogram, D. Thallium scintigraphy",A: FDG-18 PET CT,"Answer: A: FDG-18 PET CT Explanation: Ans: A. FDG-18 PET CT(Ref Harrison 19/e p270 e4, 18/e p1846).Best noninvasive investigation for myocardium bility check = FDG-18 PET CTViable myocardium:Non-contracting ischemic myocardium at rest.Has full potential to recover its function (On self or on revascularization).PET:Gold standard for assessment of myocardium bility.Identifies ischemic or hibernating myocardium in 10-20% of fibrotic or infracted regions.Uses thallium & technetium labeling.Positron-emitting tracer F-18 fluorodeoxyglucose (FDG) assesses myocardial glucose metabolism - Hence an indicator of myocardial bility.Evaluation methods of myocardial bility:Mainly in ischemic cardiomyopathy patients.Myocardial perfusion imaging (with SPECT or PET) combined with metabolic imaging (i.e., fluorodeoxyglucose I FDGJ PET).Excellent alternative:Thallium-201 SPECT imaging (hospitals lacking PET scanning)." "This is a real-world medical entrance exam question, please give the true answer based on the question and selection. Topic name: None, Subject name: Social & Preventive Medicine","Regarding Japanese encephalitis vaccine, what is not true? Select and explain. A. Not given for infants less than 6 months, B. Two primary doses given to children in the one to three year age group, C. Booster doses are given after I year and repeated every 3 years, D. In endemic areas vaccination is given to cover children between one to nine years age group.",D: In endemic areas vaccination is given to cover children between one to nine years age group.,"Answer: D: In endemic areas vaccination is given to cover children between one to nine years age group. Explanation: Ans: D. In endemic areas vaccination is given to cover children between one to nine years age group.(Ref Park 24/e p303, p261; Indian Journal of Pediatrics,. Vol. 51, October 15, 2015 p785).Japanese Encephalitis Vaccine:In endemic areas, vaccine is given to cover children between 1-15 years (Not 1 to 9 yrs).2 primary doses 4 weeks apa, booster after 1 year and 3 years until the age of 10-15 years.Booster after 1 year and then repeated every 3 years.Minimum age: According to US-FDA-2 months." "This is a real-world medical entrance exam question, please give the true answer based on the question and selection. Topic name: None, Subject name: Dental","Role of Negative reinforcement: Select and explain. A. Makes negative to positive, B. Makes positive to negative, C. Makes negative to more negative, D. None of the above",C: Makes negative to more negative,"Answer: C: Makes negative to more negative Explanation: Negative reinforcer is withdrawal of thing which increases the frequency of positive behavior. It makes negative to positive. Negative reinforcement makes negative to more negative, means if the patient shows tamper tantrum, you send the patient back without treatment, this will be negative reinforcement for him to escape from situation." "This is a real-world medical entrance exam question, please give the true answer based on the question and selection. Topic name: None, Subject name: Pathology","All are features of reversible cell injury EXCEPT Select and explain. A. ER swelling, B. Dens deposition of mitochondria, C. Bleb, D. Detachment of ribosome",B: Dens deposition of mitochondria,"Answer: B: Dens deposition of mitochondria Explanation: Answer- B. DENS DEPOSITION OF MITOCHONDRIAThe ultrastructural changes (seen on electron microscopy) are:-Plasma membrane alterations - Blebbing blunting loss of microvilli.Mitochondrial changes - Swelling, small amorphous densities.Dilatation of ER and detachment of ribosomeNuclear alterations" "This is a real-world medical entrance exam question, please give the true answer based on the question and selection. Topic name: None, Subject name: Dental","Which of the following is urethane dimethacrylate (UDMA) resin based endosealer? Select and explain. A. Endorez, B. Real seal, C. Raeko sealer, D. Tubli seal",A: Endorez,"Answer: A: Endorez Explanation: None" "This is a real-world medical entrance exam question, please give the true answer based on the question and selection. Topic name: None, Subject name: Pathology","A patient is diagnosed of oral cancer of stage T3N2M0 Select and explain. A. Surgery, B. Surgery + radiotherapy, C. Chemotherapy alone, D. Surgery + chemotherapy",B: Surgery + radiotherapy,"Answer: B: Surgery + radiotherapy Explanation: None" "This is a real-world medical entrance exam question, please give the true answer based on the question and selection. Topic name: AIIMS 2018, Subject name: Physiology","GFR is increased by all except? Select and explain. A. Increased renal blood flow, B. Efferent aeriole constriction, C. Renal stone in ureter, D. Decreased oncotic pressure",C: Renal stone in ureter,"Answer: C: Renal stone in ureter Explanation: Increased blood flow to glomerular capillaries causes increased capillary hydrostatic pressure causing increased GFR. The efferent aeriole constriction also causes increased capillary hydrostatic pressure causing increased GFR. Plasma colloid oncotic pressure is the force that opposes the filtration; when it is decreased, GFR increased. In case of nephrolithiasis or renal stone, there will be decreased flow of tubular fluid which causes fluid accumulation in Bowman's capsule leading to increased hydrostatic pressure in Bowman's capsule which opposes filtration causing decreased GFR Net filtration pressure = PG -PB - pG PG: glomerular hydrostatic pressure PB: hydrostatic pressure in Bowman's capsule PG: Colloid osmotic pressure of glomerular capillary plasma proteins GFR = KFX net filtration pressure KF = capillary filtration coefficient" "This is a real-world medical entrance exam question, please give the true answer based on the question and selection. Topic name: None, Subject name: Anatomy","Muscle that attaches to zygomatic process of maxilla: Select and explain. A. Masseter, B. Buccinator, C. Middle temporal, D. Medial pterygoid",A: Masseter,"Answer: A: Masseter Explanation: None" "This is a real-world medical entrance exam question, please give the true answer based on the question and selection. Topic name: None, Subject name: Dental","Patient complaint of loose immediate dentures today morning, and also told that this happened 2-3 days back also. What could be the possible reason Select and explain. A. Patient forgot how to insert the denture, B. Continuous healing of defect, C. Due to shrinkage of resin, D. Due to reduction of diameter of defect",B: Continuous healing of defect,"Answer: B: Continuous healing of defect Explanation: None" "This is a real-world medical entrance exam question, please give the true answer based on the question and selection. Topic name: AIIMS 2019, Subject name: Pathology","All of the following are true about the given instrument except Select and explain. A. Can be done in prone or lateral position, B. To find out infiltrative and granulomatous disorders, C. Breath holding not necessary, D. Platelet count of 40000 is a contraindication",D: Platelet count of 40000 is a contraindication,"Answer: D: Platelet count of 40000 is a contraindication Explanation: Given instrument is Bone marrow biopsy needle Bone marrow biopsy can be done in prone or lateral position. Most common site is posterior iliac crest. In obese person, preferred site is anterior superior iliac spine In newly born, preferred site is superficial bone i.e. tibia. To find out infiltrative and granulomatous disorders. Breath holding is not necessary in bone marrow biopsy. During biopsy of other organs like liver , breath holding is necessary. During any bleeding disorder, RBC or WBC disorder or hypo activity of bone marrow, bone marrow biopsy is done . Even in severe thrombocytopenia , Bone marrow biopsy is done." "This is a real-world medical entrance exam question, please give the true answer based on the question and selection. Topic name: None, Subject name: Gynaecology & Obstetrics","A 22 years old gravida 3 para 2 lady delivers a normal child followed by delivery of an intact placenta. Following delivery, the lady develops severe per vaginal bleeding after 30 minutes. On table sonogram revealed retained placental tissue. What is the suspected type of placenta? Select and explain. A. Membranous placenta, B. Placenta fenestrae, C. Placenta accreta, D. Placenta succenturiata",D: Placenta succenturiata,"Answer: D: Placenta succenturiata Explanation: Ans: D. Placenta succenturiataSuccenturiate lobeSmall accessory lobes develop at a small distance from the main placentadeg.These lobes have vessels that course through the membranesdeg.If these vessels overlie the cervix to create a vasa pre, they can cause dangerous fetal hemorrhage if torndeg.An accessory lobe may be retained in uterus after delivery & cause postpaum uterine" "This is a real-world medical entrance exam question, please give the true answer based on the question and selection. Topic name: None, Subject name: Dental","In ALL patient; at which stage can the orthodontic treatment be done: Select and explain. A. During chemotherapy, B. Before chemotherapy, C. 2 years after remission, D. Any time as not affected by ALL",A: During chemotherapy,"Answer: A: During chemotherapy Explanation: None" "This is a real-world medical entrance exam question, please give the true answer based on the question and selection. Topic name: None, Subject name: Pathology","Which diseases are infectious but not communicable? Select and explain. A. Measles, B. Mumps, C. Scarlet fever, D. Tetanus",D: Tetanus,"Answer: D: Tetanus Explanation: None" "This is a real-world medical entrance exam question, please give the true answer based on the question and selection. Topic name: None, Subject name: Dental","Deepening the pulpal floor during cavity preparation provides for Select and explain. A. Outline form, B. Retention form, C. convenience form, D. Resistance form",D: Resistance form,"Answer: D: Resistance form Explanation: None" "This is a real-world medical entrance exam question, please give the true answer based on the question and selection. Topic name: None, Subject name: Dental","Clinically acceptable limit of run out: Select and explain. A. 0.012, B. 0.023, C. 0.15, D. 0.032.",B: 0.023,"Answer: B: 0.023 Explanation: None" "This is a real-world medical entrance exam question, please give the true answer based on the question and selection. Topic name: None, Subject name: Medicine","A 30-year-old gentleman has exce' cep at work attributed to sleep discomfo at night. He also has recent history of falling while paying with friends. What are the other features that can be seenassociated with his condition? Select and explain. A. Paralysis during sleep-wake transition with hallucinations, B. Snoring with witnessed sleep apnea, C. Pain in the legs before going to sleep, D. Generalized seizures in the wake state",A: Paralysis during sleep-wake transition with hallucinations,"Answer: A: Paralysis during sleep-wake transition with hallucinations Explanation: Answer: a. Paralysis during sleep-wake transition with hallucinations (Ref. Harrivon 19/c p189, 18/e p265History of fall is highly suggestive of atonia/paralysis and may be due to Narcolepsy." "This is a real-world medical entrance exam question, please give the true answer based on the question and selection. Topic name: None, Subject name: Dental","Improved Drinking water source protect from Select and explain. A. Fecal matter, B. Teratogenic, C. Inorganic, D. Not Recalled",A: Fecal matter,"Answer: A: Fecal matter Explanation: None" "This is a real-world medical entrance exam question, please give the true answer based on the question and selection. Topic name: None, Subject name: Dental","Most constant and valuable trait to differentiate among maxillary 1st , 2nd and 3rd molars is Select and explain. A. Relative position of DL groove, B. Relative position of ML groove, C. ML cusp size, D. Oblique ridge size",A: Relative position of DL groove,"Answer: A: Relative position of DL groove Explanation: Relative position of DL groove is most constant and valuable trait. As DL cusp becomes smaller as we move distally from 1st molar to 3rd molar." "This is a real-world medical entrance exam question, please give the true answer based on the question and selection. Topic name: None, Subject name: Dental","The odontoblasts killed during cavity preparation are derived from: Select and explain. A. Unaffected odontoblasts from the pulp, B. Undifferentiated cells, C. Histocytes, D. Osteoblasts",B: Undifferentiated cells,"Answer: B: Undifferentiated cells Explanation: None" "This is a real-world medical entrance exam question, please give the true answer based on the question and selection. Topic name: None, Subject name: Gynaecology & Obstetrics","All are true about aneuploidy except: Select and explain. A. 30% of trisomy 21 fetus die in utero, B. 80% of trisomy 18 fetus die in uero, C. Occurrence of aneuploidy has no relation with the progression of mother's age, D. The recurrence risk for nondysjunctional aneuploidy is 1% higher",C: Occurrence of aneuploidy has no relation with the progression of mother's age,"Answer: C: Occurrence of aneuploidy has no relation with the progression of mother's age Explanation: Aneuploidy Each human cell consists of 23 pair of chromosomes i.e., the normal chromosome number is 46 Aneuploidy is a deviation from the normal number of 46 chromosome. Which could be be 47 or 45 chromosomes or it can be defined as state of having chromosome number that is not multiple of 23. Nondisjunction Nondisjunction is failure of paired homologus chromosomes to separate during the first meiotic division that leads to the production of gametes (ova and spermatozoa). Thus, some gametes receive two and other receive none of the involved pair. After the second meiotic division the resulting gametes will have 24 and 22 chromosomes respectively. Such gametes are aneuploid. Anaphase lag In anaphase lag, one homologus chromosome in meiosis or one chromatid in mitosis lags behind and is left out of the cell nucleus. This results in one normal cell and one cell with monosomy. Survival of Aneuploidy Monosomy or trisomy involving the sex chromosome are compatible with life and are usually associated with variable degree of phenotypic abnormalities. Monosomy involving an autosome generally represents loss of too much genetic information to permit live births or  even embryogenesis. With the exception of trisomy 21 all others will produce severely handicapped infants who almost invariably die at an early age. Estimated abortion rate in early pregnancy" "This is a real-world medical entrance exam question, please give the true answer based on the question and selection. Topic name: None, Subject name: Microbiology","The following is the ovum of a helminth. What is true about the helminth? Select and explain. A. Transmission is through ingestion of infected pork, B. Both adult and larval stages are seen in humans, C. The helminth causes a transient self-resolving infection in humans, D. Drug of choice for this condition is albendazole",B: Both adult and larval stages are seen in humans,"Answer: B: Both adult and larval stages are seen in humans Explanation: Ans: b. Both adult and larval stages are seen in humans(Ref: Paniker's 7/e p135, 6/e p156)The given image is the egg of Hymenolepis nano showing characteristic polar filaments, polar knobs, yolk granules and,6 hooklets (hexacanth) in the oncosphere (embryo).Both adult and larval stages of this species are seen in humans.It causes a transient infection in humans but does not resolve on its own and has to be treated." "This is a real-world medical entrance exam question, please give the true answer based on the question and selection. Topic name: None, Subject name: Medicine","Progenitor hematopoietic stem cells originate in Select and explain. A. Bone marrow, B. Thymus, C. Lymph Node, D. Spleen",A: Bone marrow,"Answer: A: Bone marrow Explanation: None" "This is a real-world medical entrance exam question, please give the true answer based on the question and selection. Topic name: None, Subject name: Dental","Long face syndrome patient with increased lower facial height, the palatal plane will be: Select and explain. A. Posteriorly downward, B. Upward posteriorly, C. Downward anteriorly, D. No change",A: Posteriorly downward,"Answer: A: Posteriorly downward Explanation: Vertical type 1 is suggestive of ‘long face syndrome’ or ‘steep mandibular plane angle’ type of class II. The functional occlusal plane is also steep associated with a palatal plane that is tipped downward. In long face individuals, who have excessive lower anterior face height, the palatal plane rotates down posteriorly, often creating a negative rather than the normal positive inclination to the true horizontal. The mandible shows an opposite, backward rotation, with an increase in the mandibular plane angle. Ref: Orthodontics: Diagnosis and Management of Malocclusion and Dentofacial Deformities, 3e, Om P. Kharbanda pdf no 2526" "This is a real-world medical entrance exam question, please give the true answer based on the question and selection. Topic name: None, Subject name: Anaesthesia","Which of these is most commonly used as pre-anesthetic medication? Select and explain. A. Atropine, B. Promethazine, C. Scopolamine, D. Glycopyrrolate",D: Glycopyrrolate,"Answer: D: Glycopyrrolate Explanation: Ans: D. Glycopyrrolate(Ref: Miller 7/e p293; KDT 366, 117)Glycopyrrolate is most commonly used as pre-anesthetic medication.An anlicholinergic drug used for reducing secretions in the mouth, throat, airway, and stomach before surgery.Used before and during surgery to block ceain reflexes and to protect against ceain side effects of some medicines." "This is a real-world medical entrance exam question, please give the true answer based on the question and selection. Topic name: None, Subject name: Dental","If a permanent first molar is lost, the permanent second molar drifts to the: Select and explain. A. Buccal side, B. Distal side, C. Mesial side, D. Lingual side",C: Mesial side,"Answer: C: Mesial side Explanation: None" "This is a real-world medical entrance exam question, please give the true answer based on the question and selection. Topic name: None, Subject name: Gynaecology & Obstetrics","The best marker for neural tube defect is: Select and explain. A. Acetyl glucosonidase, B. Acetyl cholinesterase, C. Alpha fetoprotein, D. Chorionic gonadotrophin",B: Acetyl cholinesterase,"Answer: B: Acetyl cholinesterase Explanation: None" "This is a real-world medical entrance exam question, please give the true answer based on the question and selection. Topic name: AIIMS 2019, Subject name: Microbiology","Transmits which disease Select and explain. A. Kyasanur forest disease, B. Scrub typhus, C. Japanese encephalitis, D. Leptospirosis",B: Scrub typhus,"Answer: B: Scrub typhus Explanation: Trombiculid mite is a vector of chiggerosis or Scrub typhus which is caused by Orientia Tsutsugamushi. Scrub typhus causes fever, myalgia and black Eschar. Kyasanur forest disease is transmitted to humans through the bite of infected hard ticks. Japanese encephalitisvirus is transmitted to humans through the bite of infected Culex species mosquitoes, paicularlyCulex tritaeniorhynchus. Leptospirosis transmitted through rodents" "This is a real-world medical entrance exam question, please give the true answer based on the question and selection. Topic name: None, Subject name: Gynaecology & Obstetrics","A female come to gynaeoPD for preconceptual counseling, with history of two second trimester aboions. What is the next investigation you will advice Select and explain. A. TVS, B. hysteroscopy, C. Endometrial biopsy, D. chromosomal abnormalities",A: TVS,"Answer: A: TVS Explanation: Ans: A. TVSRef: Williams Obstetrics 24h ed""Most common cause of second trimester aboion is cervicouterine abnormalities.Next step would be to do an ultrasound and look for any structural uterine anomaly.Chromosomal abnormalities are common cause of aboftions in first trimester." "This is a real-world medical entrance exam question, please give the true answer based on the question and selection. Topic name: None, Subject name: Dental","The pattern of change of disease of mortality and morbidity where the pandemics of infection are replaced by degenerative and manmade disorders as the main cause of morbidity and mortality is known as? Select and explain. A. Epidemiological transition, B. Demographic transition, C. Paradoxical transition, D. Reversal of transition",A: Epidemiological transition,"Answer: A: Epidemiological transition Explanation: None" "This is a real-world medical entrance exam question, please give the true answer based on the question and selection. Topic name: None, Subject name: Gynaecology & Obstetrics","IV loading dose of MgSo4 prophylaxis in pre-eclampsia? Select and explain. A. 8ml MgSo4+10ml of NS, B. 10ml MgSo4+10ml of NS, C. 8ml MgSo4+12ml of NS, D. 12ml MgSo4+8 ml of NS",C: 8ml MgSo4+12ml of NS,"Answer: C: 8ml MgSo4+12ml of NS Explanation: Ans. C.8ml MgSo4+12ml of NSDose: 4-5 g (diluted in 250 mL NS/D5W) IV in combination with either :Up to 10 g (10 mL of undiluted 50% solution) divided and administered IM into each buttock orAfter initial IV dose, 1-3 g/hr IV.MgSO4 is continued 24 hours after delivery to prevent postpaum eclampsia" "This is a real-world medical entrance exam question, please give the true answer based on the question and selection. Topic name: None, Subject name: Dental","Bond found in GIC is: Select and explain. A. Covalent bond, B. Ionic bond, C. Hydrogen bond, D. Metallic bond",B: Ionic bond,"Answer: B: Ionic bond Explanation: None" "This is a real-world medical entrance exam question, please give the true answer based on the question and selection. Topic name: AIIMS 2018, Subject name: Biochemistry","An adolescent male patient came with pain in calf muscles on exercise. On biopsy excessive amount of glycogen present was found to be present in the muscle. What is the most likely enzyme deficiency? Select and explain. A. Muscle debranching enzyme, B. Phosphofructokinase I, C. Glucose 6 phosphatase, D. Phosphorylase enzyme",D: Phosphorylase enzyme,"Answer: D: Phosphorylase enzyme Explanation: Muscle cramps on exercise is a diagnostic feature of McArdle's disease. In this disease, due to deficiency of muscle glycogen phosphorylase, glycogen stored in muscle cannot be utilized for glucose production causing Muscle cramps and pain on doing strenuous work or exercise. Refer to the table for other symptoms. Type Name Enzyme deficiency Organ Clinical features I Von Gierke's disease Glucose 6-phosphatase Liver , Kidney Glycogen accumulation in liver and kidney Hypoglycemia lactic acidemia ketosis Hyperlipidemia II Pompe's disease Lysosomal alpha-1,4 glucosidase (acid maltase) All organs glycogen accumulation in lysosome in all organ hea is mostly affected* death occur at early age due to hea failure III Cori's disease Amylo alpha-1,6-glucosidase (debranching enzyme) Liver muscles and Hea fasting hypoglycemia hepatomegaly in infancy accumulation of characteristics branched polysaccharides (limit dextrin) muscles weakness IV Anderson's disease Glucosyl 4-6 transferase(branching enzyme) Most tissue A rare disease, glycogen with only few branches accumulate; cirrhosis of liver Death before 5 years of age dur to hea failure or liver failure V McArdle's disease Muscle glycogen phosphorylase Skeletal tissue Muscles glycogen is very high but cannot be used during exercise Cannot perform strenuous exercise Muscles cramps Blood lactate and pyruvate do not increase after exercise VI Her's disease Liver glycogen phosphorylase Liver Liver is increased in size because liver glycogen cannot conve to glucose Mild hypoglycemia and ketosis Good prognosis VII Tarui's disease Phosphofructokinase Skeletal muscles and RBC Muscles cramps due to exercise Blood lactate is elevated Hemolysis occur" "This is a real-world medical entrance exam question, please give the true answer based on the question and selection. Topic name: None, Subject name: Surgery","A 20-year-old male presents with hard painless testicular swellingon investigation AFP is 3080. No paraaoic or iliac nodes as well as no mediastinal lymph nodes found. usG shows uniform echotexture and small areas of necrosis surrounding structures are normal.What is the next best steP: Select and explain. A. FNAC, B. Trucut bioPsY, C. High inguinal orchidectomy, D. PET-CT",C: High inguinal orchidectomy,"Answer: C: High inguinal orchidectomy Explanation: Ans. C. High inguinal orchidectomyRe.f: Sabiston textbook of surgery,2AI' ed., ch-72, pg. 2102-2104Initial treatment of suspected testicular tumour is radical inguinal orchiectomy, which involves removal of the testicle and sperrnatic cord at the level of the inguinal ring.Because of the characteristic and well-described lymph drainage of the testicle there is no role or trans-scrotal biopsy or orchiectomy." "This is a real-world medical entrance exam question, please give the true answer based on the question and selection. Topic name: None, Subject name: Gynaecology & Obstetrics","What would be the ideal management of a woman with infeilty who is detected to have bilateral cornual block on hysterosalpingography? Select and explain. A. Tuboplasty, B. In-vitro feilization, C. Hydrotubation, D. Hysteroscopy and laparoscopy",D: Hysteroscopy and laparoscopy,"Answer: D: Hysteroscopy and laparoscopy Explanation: Ans. d. Hysteroscopy and laparoscopy (Ref The Infeility Manual/p266-267; Practical approach to infeility management by aiilr Rcensal/p33:Hysteroscopy and laparoscopy would be the ideal management of a woman with infeility who is detected to have bilateral cornual block on hysterosalpingography." "This is a real-world medical entrance exam question, please give the true answer based on the question and selection. Topic name: None, Subject name: Gynaecology & Obstetrics","Following renal disorder is associated with worst pregnancy outcome: Select and explain. A. Systemic lupus erythromatosus, B. IgA nephropathy, C. Autosomal dominant polycystic kidney disease, D. Scleroderma",D: Scleroderma,"Answer: D: Scleroderma Explanation: Prognostic indicators in renal disease and pregnancy: Most reliable prognostic indicator of the outcome of pregnancy is the presence of hypertension. The fetal prognosis for women with chronic renal disease is favorable as long as they do not develop superimposed preeclampsia. Second to hypertension, the most valuable prognostic index for patients with chronic renal disease during pregnancy is the degree of renal function impairment: – In patients with normal or only midly impaired renal function, pregnancy does not accelerate renal damage – In patients with moderate renal insufficiency (serum creatinine of 1.4 mg/dL or greater before pregnancy or creatinine clearance <30 ml/min. there is a decline in renal function during pregnancy). Another important prognostic sign is the presence or absence of proteinuria. As a general rule, if the patient has 2+ or more protein in qualitative tests or 3 g or more in 24 hours urine collections at the beginning of pregnancy, the tendency will be toward increased protein losses and development of nephritic syndrome during pregnancy. The histologic characteristics of the renal lesion also have prognostic value." "This is a real-world medical entrance exam question, please give the true answer based on the question and selection. Topic name: None, Subject name: Dental","Shape of 1st Maxillary molar Select and explain. A. Rhomboid, B. Rounded triangular, C. Trapezoid, D. Square",A: Rhomboid,"Answer: A: Rhomboid Explanation: None" "This is a real-world medical entrance exam question, please give the true answer based on the question and selection. Topic name: AIIMS 2019, Subject name: Dental","Topical steroids is most effective in: Select and explain. A. Dermal atrophy, B. Eczematous dermatitis, C. Bullous lesions due to HSV, D. Herpes Zoster",B: Eczematous dermatitis,"Answer: B: Eczematous dermatitis Explanation: Acute Phase of Eczematous dermatitis - Spongiosis (oozing out lesion) - Coicosteroids ( Topical+systemic) Chronic phase of Eczematous dermatitis - Lichenification Rx - Coicosteroids & topical calcineurin inhibitor Systemic : Azathioprine" "This is a real-world medical entrance exam question, please give the true answer based on the question and selection. Topic name: None, Subject name: Dental","Stable element in Ti6Al4V in alpha phase is? Select and explain. A. V, B. Al, C. Ti, D. Al, V",B: Al,"Answer: B: Al Explanation: Alpha titanium wires are manufactured from grade V titanium alloy containing 6% aluminum and 4% vanadium. These wires have good formability and stiffness equal to that of stainless steel. These wires possess poor spring back, and hence their use in orthodontics is limited for applying torque at the finishing stage. Aluminum is used to stabilize the alloy in alpha phase. GRADE V TITANIUM ALLOY (Ti6Al4V, OR Ti6-4)  6% Aluminum 4% Vanadium 0.25% Iron 0.2% Oxygen  Textbook of ORTHODONTICS Sridhar Premkumar" "This is a real-world medical entrance exam question, please give the true answer based on the question and selection. Topic name: None, Subject name: Anatomy","All of the following carry proprioception from head and neck except: Select and explain. A. Facial nerve, B. Trigeminal nerve, C. Glossopharyngeal nerve, D. Cranial accessory nerve",D: Cranial accessory nerve,"Answer: D: Cranial accessory nerve Explanation: None" "This is a real-world medical entrance exam question, please give the true answer based on the question and selection. Topic name: None, Subject name: Gynaecology & Obstetrics","Which of the following is not true about latent phase of labour? Select and explain. A. According to ACOG it stas after 3-4 cm cervical dilatation but they are planning to increase it to 5 cm, B. Begins at the end of active phase and is a pa of 1st stage of labour, C. Patient may present with false labour due to mild cramps, D. Stas with contractions of the uterus",C: Patient may present with false labour due to mild cramps,"Answer: C: Patient may present with false labour due to mild cramps Explanation: Answer- C. Patient may present with false labour due to mild crampsIt stas at the point at which mother perceives true labour pains and ends when cervix is 3cm dilated and 1.5 cm/hour for parous cervix.Duration in nulliparous is 6-8 hours and 5.3 hours in multiparous (average 4-6 hours).Mainly concerned with cervical effacement" "This is a real-world medical entrance exam question, please give the true answer based on the question and selection. Topic name: None, Subject name: Surgery","A patient with a history of fall after hypoglycemic episode presents with pain in bilateral preauricular region and deviation of the mandible to right side on mouth opening, probable diagnosis will be Select and explain. A. Fracture of right condyle, B. Fracture of left condyle, C. Dislocation of left condyle, D. Dislocation of both condyles",A: Fracture of right condyle,"Answer: A: Fracture of right condyle Explanation: None" "This is a real-world medical entrance exam question, please give the true answer based on the question and selection. Topic name: None, Subject name: Dental","The instrument shown in fig is used for Select and explain. A. Rapid separation, B. Slow separation, C. Not used for separation, D. Depends on force for separation",B: Slow separation,"Answer: B: Slow separation Explanation: The image shows Orthodontic brass ligature wire which brings about slow separation of teeth not more than 0.5mm." "This is a real-world medical entrance exam question, please give the true answer based on the question and selection. Topic name: None, Subject name: Anatomy","Tongue develops from all of the following except Select and explain. A. Tuberculum impar, B. Hypobranchial eminence, C. Lingual swellings, D. Arytenoid swellings",D: Arytenoid swellings,"Answer: D: Arytenoid swellings Explanation: None" "This is a real-world medical entrance exam question, please give the true answer based on the question and selection. Topic name: None, Subject name: Dental","Polishing of composite is problematic due to Select and explain. A. Soft matrix and hard filler particles, B. Hard filler particles, C. Hardness of matrix and filler particles, D. None of the above",A: Soft matrix and hard filler particles,"Answer: A: Soft matrix and hard filler particles Explanation: None" "This is a real-world medical entrance exam question, please give the true answer based on the question and selection. Topic name: None, Subject name: Pharmacology","Lithium should be stopped how many days before anaesthesia Select and explain. A. 1 day, B. 2 days, C. 3 days, D. 4 days",C: 3 days,"Answer: C: 3 days Explanation: None" "This is a real-world medical entrance exam question, please give the true answer based on the question and selection. Topic name: None, Subject name: Pediatrics","A 10 year old boy presents with midline swelling arising from cerebellum the diagnosis is – Select and explain. A. Astrocytoma, B. Glioblastoma multiforme, C. Ependymoma, D. Medulloblastoma",D: Medulloblastoma,"Answer: D: Medulloblastoma Explanation: Midline swelling arising from cerebellum in a child favour the diagnosis of medulloblastoma. Robbin's states ""In children medulloblastomas are located in midline but in adults they are found in lateral locations"". Note - Astrocytoma is also a posterior fossa tumor, but it does not commonly present as midline mass." "This is a real-world medical entrance exam question, please give the true answer based on the question and selection. Topic name: None, Subject name: Dental","Mast cells are Select and explain. A. Absent in both inflamed and normal pulp., B. Present in normal pulp, C. Present in inflamed pulp only., D. Absent in inflamed pulp only.",C: Present in inflamed pulp only.,"Answer: C: Present in inflamed pulp only. Explanation: None" "This is a real-world medical entrance exam question, please give the true answer based on the question and selection. Topic name: None, Subject name: Pathology","Which of the following is associated with a low concentration of ionized calcium in the serum? Select and explain. A. Hypothyroidism, B. Osteogenesis imperfecta, C. Paget's disease of the bone, D. Tetany",D: Tetany,"Answer: D: Tetany Explanation: None" "This is a real-world medical entrance exam question, please give the true answer based on the question and selection. Topic name: AIIMS 2019, Subject name: Social & Preventive Medicine","All of the following are done to remove Confounding except Select and explain. A. Randomization, B. Random Selection, C. Matching, D. Blinding",B: Random Selection,"Answer: B: Random Selection Explanation: Confounding factor - related to both the exposure and the outcome It leads to mistaken outcomes in the study, which leads to error in the study. Endemic Goitre is usually found in high altitudes, showing thereby an association between the two. However, we know that the goiter is not because of altitude but because of environmental deficiency of Iodine. Methods used to control confounding: Methods used to control confounding: METHOD UTILITY IN CONTROLLING CONFOUNDING 1. Randomization Most ideal method 2. Restriction Limiting study to people who have paicular characteristics 3. Matching Mostly useful in case control studies 4. Stratification Useful for larger studies 5. Statistical Modeling When many confounding variables exist simultaneously 6. Blinding Bias is also a type of confounding" "This is a real-world medical entrance exam question, please give the true answer based on the question and selection. Topic name: None, Subject name: Pharmacology","Asseion: In a patient admitted to hospital for community acquired pneumonia, combination therapy of beta lactams and azithromycin is given.Reason : This combination covers gram positive organisms and anaerobes. Select and explain. A. Both reason and asseion are true, B. Asseion is true but reason is false., C. Asseion is true but reason is paially true for asseion, D. Both asseion & reason are not true.",B: Asseion is true but reason is false.,"Answer: B: Asseion is true but reason is false. Explanation: Ans. B. Asseion is true but reason is false.* Combination is given to cover atypical bacteria.* The CDC and others recommend outpatient oral empirical antibiotics with a macrolide, doxycycline, or an oral betalactam (amoxicillin, cefuroxime , or amoxicillin/clavulanate ) or inpatient treatment with an intravenous betalactam (cefuroxime, ceftriaxone , cefotaxime ) or a combination of ampicillin/sulbactam (Unasyn) with a macrolide" "This is a real-world medical entrance exam question, please give the true answer based on the question and selection. Topic name: None, Subject name: Dental","Heart of controlled trial? Select and explain. A. Blinding, B. Experiment, C. Randomization, D. Intervention",C: Randomization,"Answer: C: Randomization Explanation: None" "This is a real-world medical entrance exam question, please give the true answer based on the question and selection. Topic name: None, Subject name: Dental","Definitive treatment for the condition shown in radiograph below is? Select and explain. A. Apexogenesis, B. Revascularisation, C. Calcium Hydroxide apexification, D. MTA barrier formation",D: MTA barrier formation,"Answer: D: MTA barrier formation Explanation: None" "This is a real-world medical entrance exam question, please give the true answer based on the question and selection. Topic name: AIIMS 2018, Subject name: Ophthalmology","A patient is taking drugs for rheumatoid ahritis and has a history of cataract surgery 1 year back, the patient presented with sudden painless loss of vision, probable diagnosis is? Select and explain. A. CME, B. Macularhole, C. Chloroquine toxicity, D. Chronic choroiditis",C: Chloroquine toxicity,"Answer: C: Chloroquine toxicity Explanation: Patient of rheumatoid ahritis are given hydroxychloroquine which cause painless loss of vision due to bull's eye maculopathy characterized by a foveolar island of pigment surrounded by a depigmented zone of RPE atrophy, which is itself encircled by a hyperpigmented ring (A). There is moderate to severe reduction in VA (6/36-6/60). A more substantial macular lesion follows, with widespread RPE atrophy surrounding the fovea (B). Later retinal aerioles may become attenuated, and pigment clumps can form in the peripheral retina.(C)" "This is a real-world medical entrance exam question, please give the true answer based on the question and selection. Topic name: AIIMS 2017, Subject name: Pathology","Tumor cells in chronic lymphocytic leukemia or small lymphoblastic lymphoma (CLL/SLL) arisefrom which of the following? Select and explain. A. Mature B cell, B. Naive B cell, C. Centrocytes of germinal center, D. Progenitor B-cell",B: Naive B cell,"Answer: B: Naive B cell Explanation: DNA sequencing has revealed that the Ig genes of some CLL/SLL are somatically hypermutated whereas others are not, suggesting that the cell of origin may be either a post germinal center memory B cell or a naive B cell. For unclear reasons, tumors with unmutated Ig segments (those putatively of naive B-cell origin) pursue a more aggressive course. Impoant points about CLL: CLL is the most common leukemia of adults in the Western world. CLL is the cancer which is associated with presence of Autoimmune Hemolytic Anemia. CLL is the cancer which is not associated with Radiation exposure. CLL patient blood sample contains large numbers of small round lymphocytes with scant cytoplasm. Some of these cells are usually disrupted in the process of making smears, producing so-called Smudge cells. These patients are best identified with the help of Tumor cells exhibiting specific type of CD markers paicularly -CD19, CD20, CD23, CD5." "This is a real-world medical entrance exam question, please give the true answer based on the question and selection. Topic name: None, Subject name: Orthopaedics","Positive pivot shift test in knee is because of injury to - Select and explain. A. Posterior cruciate ligament, B. Anterior cruciate ligament, C. Medial meniscus, D. Lateral meniscus",B: Anterior cruciate ligament,"Answer: B: Anterior cruciate ligament Explanation: COLLATERAL LIGAMENT INJURY The most common mechanism of ligament disruption of knee is abduction (valgus), flexion and internal rotation of femur on tibia which usually occur in sports in which the foot is planted solidly on the ground and leg is twisted by rotating body. The medial structures medial (tibial) collateral ligament (MCL) and medial capsular ligament are first to fail, followed by ACL tear, if the force is of sufficient magnitude. The medial meniscus may be trapped between condyles and have a peripheral tear, thus producing unhappy triad of 0’ Donoghue. Main test for MCL (medial collateral ligament) is valgus (abduction) stress in 30° of knee flexion. (Because in full extension it is indicative of combined MCL, posterior oblique ligament injury and posterior cruciate ligament injury). Varus (Adduction) stress test in 30° flexion (removes the lateral stabilizing effect of iliotibial band so that the lateral collateral ligament can exclusively be examined). Apleys distraction test is used for collateral ligaments." "This is a real-world medical entrance exam question, please give the true answer based on the question and selection. Topic name: None, Subject name: Dental","An apical radiolucent area present in central incisor after 4 months of RCT is due to: Select and explain. A. Inadequate obturation & leakage from main canal, B. Leakage from accessory canal, C. Leakage from gingival crevice, D. Leakage from access opening",A: Inadequate obturation & leakage from main canal,"Answer: A: Inadequate obturation & leakage from main canal Explanation: None" "This is a real-world medical entrance exam question, please give the true answer based on the question and selection. Topic name: AIIMS 2018, Subject name: Pharmacology","Low apparent volume of distribution of drug indicates that: Select and explain. A. Drug has low half life, B. Drug has low bioavailability, C. Drug has low efficacy, D. Drug is not extensively distributed to tissue",D: Drug is not extensively distributed to tissue,"Answer: D: Drug is not extensively distributed to tissue Explanation: Vd means ""the plasma volume that would accommodate all the drug in the body, if the concentration throughout was same as in plasma"". Drugs with high volume of distribution are more distributed in body and vice-versa. Low volume of distribution tells that most of the drug is retained in plasma." "This is a real-world medical entrance exam question, please give the true answer based on the question and selection. Topic name: None, Subject name: Anaesthesia","All of the following are true about lumbar puncture except: Select and explain. A. Level of needle inseion should be L I -L2 veebral junction, B. The bevel end of needle should face up, C. Needle should be inseed in a slightly cephalad direction, D. Legs should be straightened for CSF pressure measurement",A: Level of needle inseion should be L I -L2 veebral junction,"Answer: A: Level of needle inseion should be L I -L2 veebral junction Explanation: Ans: A. Level of needle inseion should be L I -L2 veebral junction(Ref: Harrison 19Ie p443-e2)The spinal cord ends at L3 veebrae in children and L1 in adults.With a safe margin, lumbar puncture should be performed at L3-L4 or L4-L5 interspace.A useful anatomic guide is a line drawn between the posterior superior iliac crests, which corresponds closely to the level of the L3--L4 interspace.The interspace is chosen following gentle palpation to identify the spinous processes at each lumbar level.The LP needle (typically 20- to 22-gauge) is inseed in the midline, midway between two spinous processes, and slowly advanced.The bevel of the needle should be maintained in a horizontal position, parallel to the direction of the dural fibers and with the flat poion of the bevel pointed upward; this minimizes injury to the fibers as the dura is penetrated." "This is a real-world medical entrance exam question, please give the true answer based on the question and selection. Topic name: AIIMS 2019, Subject name: Gynaecology & Obstetrics","Hormone Replacement therapy is not indicated in: Select and explain. A. Hot flashes, B. Prevention of CAD, C. Osteoporosis, D. Vaginal atrophy",B: Prevention of CAD,"Answer: B: Prevention of CAD Explanation: H indicated in menopausal women to overcome the sho term and long term consequences of estrogen deficiency. Indications of hormone replacement therapy in a menopausal woman Relief of vasomotor symptoms: hot flushes Prevention and treatment of osteoporosis Urogenital atrophy Premature Menopause Hormone Replacement Therapy is NOT indicated for prevention or treatment of cardiovascular disease. Natural estrogens produced in the body till menopause are cardioprotective, however Hormone Replacement Therapy (containing exogenous estrogens and progesterones)is not cardioprotective, n fact it is detrimental to hea when used for long term." "This is a real-world medical entrance exam question, please give the true answer based on the question and selection. Topic name: None, Subject name: Physiology","If sodium channel inactivation is prolonged then which property of nerve conduction is hampered? Select and explain. A. Decreased relative refractory period, B. Increased conduction blockade, C. Increased upward stroke velocity, D. Decreased downward stroke velocity of refractory period.",C: Increased upward stroke velocity,"Answer: C: Increased upward stroke velocity Explanation: None" "This is a real-world medical entrance exam question, please give the true answer based on the question and selection. Topic name: None, Subject name: Dental","Largest cusp is Select and explain. A. Mesiolingual cusp of upper 1st molar, B. Mesiolingual cusp of Upper 2nd molar, C. Buccolingual cusp of Upper 1st molar, D. Buccolingual cusp of lower 2nd molar",A: Mesiolingual cusp of upper 1st molar,"Answer: A: Mesiolingual cusp of upper 1st molar Explanation: None" "This is a real-world medical entrance exam question, please give the true answer based on the question and selection. Topic name: None, Subject name: Physiology","Which of following causes lysis of clot : Select and explain. A. Fibrin, B. Plasmin, C. Hyaluronidase, D. Coagulase",B: Plasmin,"Answer: B: Plasmin Explanation: None" "This is a real-world medical entrance exam question, please give the true answer based on the question and selection. Topic name: None, Subject name: Pathology","The oral findings in erythroblastosis fetal's include Select and explain. A. Dentinal dysplasia, B. Hypoplastic teeth, C. Pigmented teeth, D. All of the above",C: Pigmented teeth,"Answer: C: Pigmented teeth Explanation: None" "This is a real-world medical entrance exam question, please give the true answer based on the question and selection. Topic name: None, Subject name: Pediatrics","A neonate develops encephalitis without any skin lesions. Most probable causative organisms is – Select and explain. A. HSV I, B. HSV II, C. Meningococci, D. Streptococci",B: HSV II,"Answer: B: HSV II Explanation: Herpes simplex viruses Herpes simplex is an important cause of encephalitis in the neonate, the infection acquired during delivery from the vaginal tract. But it cause vesicular, ulcerative skin lesions Even though it causes skin lesions, its our best option, as Nelson writes - "" The hallmark of neonatal HSV infection- the vescicular ulcerative skin lesions are presnt in only 30 to 43% of children at presentation; one third will never manifest skin lesions."" Out of the.two types of Herpes; Neonatal herpes is mainly caused by type II IISV (75 to 80%). About other options Meningpcocci Its a rare cause of infection in neonates as neonates have antibodies (from mother) against meningiococcus which protects them for the initial 3 to 6 months of life. Streptococci Though streptococcus B is a common infection of neonates, it does not cause encephalitis. RAIN TUMORS You will frequently read the terms --> Infratentorial, supratentorial, Posterior cronial fossa in relation of Brain tumours, so I am giving very brief introduction Supratentorial region of the brain is located above the tentorium cerebelli; and contains the cerebrum. The infratentorial region of the brain is located below the tentorium cerebelli and contains the cerebellum and brainstem. Brain stem and cerebellum are contained in the posterior cranial fossil. As brain stem and cerebellum are infratentorial in location and contained in posterior cranial fossa -› Their tumors are infratentorial posterior fossa tumors, e.g., cerebellar astrocytoma, brainstem glioma, medulloblastoma (cerebeller)." "This is a real-world medical entrance exam question, please give the true answer based on the question and selection. Topic name: None, Subject name: Dental","Which one of the following is most elastic? Select and explain. A. α titanium, B. β titanium, C. Chrome-cobalt-nickel, D. Nickel titanium",D: Nickel titanium,"Answer: D: Nickel titanium Explanation: None" "This is a real-world medical entrance exam question, please give the true answer based on the question and selection. Topic name: None, Subject name: Surgery","Latent period of distraction osteogenesis in 8 months old child is Select and explain. A. 0-2 days, B. 5-7 days, C. 4-6 weeks, D. 31-40 days",A: 0-2 days,"Answer: A: 0-2 days Explanation: None" "This is a real-world medical entrance exam question, please give the true answer based on the question and selection. Topic name: None, Subject name: Physiology","Which of the following methods is used for calculation of anatomical dead space? Select and explain. A. Xenon dilution technique, B. Bohler's method, C. Spirometry, D. Single breath nitrogen test",D: Single breath nitrogen test,"Answer: D: Single breath nitrogen test Explanation: Ans: D. Single breath nitrogen test(Ref: Ganong 25/e p633, 634, 24/e p633, 634)Anatomical dead space - Calculation:By Bohr's equation - Uses single breath nitrogen inhalation technique.Xenon/Helium dilution technique:Used to measure functional residual capacity of lung.Spirometry:Cannot measure residual or dead space volumes." "This is a real-world medical entrance exam question, please give the true answer based on the question and selection. Topic name: None, Subject name: Microbiology","The virulence factors of Neisseria gonorrhea include all of the following except: Select and explain. A. Outer membrane proteins, B. IgA Protease, C. M-Proteins, D. Pili",C: M-Proteins,"Answer: C: M-Proteins Explanation: None" "This is a real-world medical entrance exam question, please give the true answer based on the question and selection. Topic name: None, Subject name: Pathology","Hard swelling at the angle of mandible with numerous draining sinuses is most likely: Select and explain. A. Actinomycosis, B. Ludwig's angina, C. Mucormycosis, D. Cellulitis",A: Actinomycosis,"Answer: A: Actinomycosis Explanation: None" "This is a real-world medical entrance exam question, please give the true answer based on the question and selection. Topic name: AIIMS 2018, Subject name: Orthopaedics","Which fracture results in the given deformity? Select and explain. A. Supracondylar fracture of humerus, B. Lateral condylar fracture, C. Olecranon fracture, D. Radial head fracture",A: Supracondylar fracture of humerus,"Answer: A: Supracondylar fracture of humerus Explanation: The image shows cubitus varus. Supracondylar fracture of humerus causes cubitus varus and Gunstock deformity. Lateral condylar fracture causes cubitus valgus" "This is a real-world medical entrance exam question, please give the true answer based on the question and selection. Topic name: None, Subject name: Gynaecology & Obstetrics","Which does not cross placenta: Select and explain. A. Heparin, B. Morphine, C. Naloxone, D. Warfarin",A: Heparin,"Answer: A: Heparin Explanation: Heparin does not cross placenta and is safe during pregnancy. It is the drug of choice for the management and prophylaxis of venous thromboembolism during pregnancy." "This is a real-world medical entrance exam question, please give the true answer based on the question and selection. Topic name: None, Subject name: Radiology","A middle aged female presents with slowly progressive weakness of lower limbs, spasticity and recent onset hesitancy of Micturition. On neurological examination there is evidence of dorsal myelopathy. MRI scan of spine reveals middorsal intradural contrast enhancing mass lesion. Diagnosis is: Select and explain. A. Intradural lipoma, B. Dermoid cyst, C. Meningioma, D. Epidermoid cyst",C: Meningioma,"Answer: C: Meningioma Explanation: Ans. c. Meningioma (Ref. Harrison 19/e p602, I 8/e p3388; Chapman 4/e p431; Sahicton /9/e p 1888- 1889; Schwa/17 9/e p1540- I 541: Bailey 26/e p614, 25/e p633)Diagnosis in a middle aged female with slowly progressive weakness of lower limbs, spasticity and recent onset hesitancy of micturition with evidence of dorsal myelopathy and middorsal intradural contrast enhancing mass lesion on MRI is meningioma." "This is a real-world medical entrance exam question, please give the true answer based on the question and selection. Topic name: None, Subject name: Surgery","Sodium bicarbonate when given with local anaesthetics has which of the following effect? Select and explain. A. Increases speed and quality of anaesthesia, B. Decreases diffusion of the anaesthetic drug, C. Causes rapid elimination of the Local anesthetic, D. Decreases speed and quality of anaesthesia",A: Increases speed and quality of anaesthesia,"Answer: A: Increases speed and quality of anaesthesia Explanation: None" "This is a real-world medical entrance exam question, please give the true answer based on the question and selection. Topic name: None, Subject name: Pediatrics","12 years old Shyam presented with gross hematuria with 80% dysmorphic RBC's 2 days after a attack of upper respiratory tract infection diagnosis is – Select and explain. A. Microangiopathic thrombotic anaemia, B. IgA Nephropathy, C. PSGN, D. H.S. purpura",B: IgA Nephropathy,"Answer: B: IgA Nephropathy Explanation: The patient is having glomerulo nephritis (gross hematuria and dysmorphic RBC's) 2 days after upper respiratory tract infection. Three conditions can manifest like this —> IgA nephropathy, Post streptococcal glomerulonephritis, H.S. Purpura." "This is a real-world medical entrance exam question, please give the true answer based on the question and selection. Topic name: None, Subject name: Dental","Metapex is a combination of which of these? Select and explain. A. Calcium hydroxide +ZoE, B. ZoE + Iodoform, C. Calcium hydroxide + iodoform, D. Calcium hydroxide + GIC",C: Calcium hydroxide + iodoform,"Answer: C: Calcium hydroxide + iodoform Explanation: None" "This is a real-world medical entrance exam question, please give the true answer based on the question and selection. Topic name: None, Subject name: Dental","Fibres of periodontal ligament embedded in the bone are Select and explain. A. Sharpey's fibres, B. Tomes fibres, C. Elastic fibres, D. Ray's fibres",A: Sharpey's fibres,"Answer: A: Sharpey's fibres Explanation: None" "This is a real-world medical entrance exam question, please give the true answer based on the question and selection. Topic name: AIIMS 2018, Subject name: Pathology","A patient presented with intermittent fever, no weight loss, no anorexia, but with a retroperitoneal mass. Peripheral smear findings were normal. Microscopy of the mass is given. What's the diagnosis? Select and explain. A. NHL, B. Castleman disease, C. Angiolymphoid hyperplasia, D. Ig G4 disease",B: Castleman disease,"Answer: B: Castleman disease Explanation: Onion skin appearance of lymph node is characteristic for Castleman disease It is alympho proliferative disorder. It has 2 variants: Unicentric Has 2 variants: 1)Highly vasculara More common , patients are asymptomatic Histology: -Twinning- Presence of two germinal centers -Onion skin pattern-Lymphocytes causes thickening of mantle zone -Lollipop lesions- Sclerosed blood vessel that infiltrate lymph node germinal center Multicentric ?Associated with immune suppression ?Presents as : Hepatomegaly Splenomegaly Night sweats Fever 2)Plasma cell varianta Less common, patients are symptomatic: - Weight loss -Night sweats -Fever Histology: Plasma cells in interfollicular areas causing hyperplasia of germinal center Lymph node architecture is distoed but LN is preserved in unicentric variant" "This is a real-world medical entrance exam question, please give the true answer based on the question and selection. Topic name: None, Subject name: Medicine","A healthy middle-aged man was arguing with his brother and got emotionally upset due to the arguments with his brother, he suddenly developed chest pain and collapsed. When brought to the hospital' hewas declared dead.What is the diagnosis? Select and explain. A. Takotsubo cardiomyopathy, B. Dilated cardiomyopathy, C. Arhmogenic right ventricle dysplasia, D. Chronic ischemic cardiomyopathy",A: Takotsubo cardiomyopathy,"Answer: A: Takotsubo cardiomyopathy Explanation: Ans: A. Takotsubo cardiomyopathyRef Harrison, 18't' ed., pg, 1964* Takotsubo cardiomyopathy, also known as stress cardiomyopathy, is a type of non-ischemic cardiomyopathy in which there is a sudden temporary weakening of the muscular poion of the hea.* This weakening may be triggered by emotional stress, such as the death of a loved one, a break-up, rejection from a paner or constant anxiety. - ""Broken hea syndrome"".* Stress cardiomyopathy is now a well-recognized cause of acute hea failure, lethal ventricular arrhythmias, and ventricular rupture." "This is a real-world medical entrance exam question, please give the true answer based on the question and selection. Topic name: None, Subject name: Pathology","Out of Syphilitic glossitis, Plummer Vinson syndrome, Mikulicis syndrome and hepatitis A; which of these predispose to squamous cell carcinoma Select and explain. A. Syphilitic glossitis and plummer Vinson syndrome, B. Syphilitic glossitis and Mikulicz's syndrome, C. Plummer Vinson disease and hepatitis A, D. hepatitis A and Mikulicz's Syndrome",A: Syphilitic glossitis and plummer Vinson syndrome,"Answer: A: Syphilitic glossitis and plummer Vinson syndrome Explanation: None" "This is a real-world medical entrance exam question, please give the true answer based on the question and selection. Topic name: None, Subject name: Dental","Calcification of third molar begins at: Select and explain. A. 8 months, B. 18 months, C. 8 years, D. 16 years",C: 8 years,"Answer: C: 8 years Explanation: None" "This is a real-world medical entrance exam question, please give the true answer based on the question and selection. Topic name: None, Subject name: Anaesthesia","Allergy in immediate perioperative period is due to: Select and explain. A. Opioids, B. LA agents, C. Induction agents, D. Neuromuscular blockers",D: Neuromuscular blockers,"Answer: D: Neuromuscular blockers Explanation: Ans: D. Neuromuscular blockers(Ref: Miller 7/e p884)Most common cause of perioperative anaphylaxis is muscle relaxants and antibiotics followed by opioids and intravenous anesthesia.Most Common Drugs Involved in Perioperative AnaphylaxisSubstanceMost commonly associatedMuscle relaxantsSuccinylcholine, rocuronium,atracuriumdegNatural rubber latexLatex gloves, tourniquets, Foley cathetersAntibioticsPenicillin & other beta-lactamsdegHypnoticsPropofol, thiopentaldegColloidsDextran, gelatindegOpioidsMorphine, meperidinedegOther substancesParacetamol, aprotinin,chymopapain, protamine,bupivacaine" "This is a real-world medical entrance exam question, please give the true answer based on the question and selection. Topic name: AIIMS 2020, Subject name: Pediatrics","A child transfers objects from one hand to other. What does it imply? Select and explain. A. Visual motor co-ordination, B. Explores small object, C. Object release, D. Comparison of objects",D: Comparison of objects,"Answer: D: Comparison of objects Explanation: GROSS MOTOR MILESTONE DEVELOPMENTAL IMPLICATION Holds head steady while sitting Allows more visual interaction Sits without suppo Increasing exploration Walks alone Exploration, controls proximity to parents FINE MOTOR MILESTONE DEVELOPMENTAL IMPLICATION Hand regard Self discovery of hands Reaches for objects Visuomotor coordination Palmar grasp gone Voluntary release Transfers object hand to hand Comparison of others Thumb -finger grasp Able to explore small objects Scribbles Visuomotor coordination Builds tower of 2 cubes Uses objects in combination SOCIAL MILESTONE DEVELOPMENTAL IMPLICATION Social smile More active social paicipation Follows 1 step command with gestures Non -verbal communication Points to objects Interactive communication Uncovers toy(after seeing it hidden) object permanence Pretends to drink from cup Symbolic thought Uses stick to reach toy Links actions to solve problems LANGUAGE MILESTONE DEVELOPMENTAL IMPLICATION Monosyllabic babble Experimentation with sound 1-step command without gesture Verbal receptive language Says ""mama ""or ""dada"" Expressive language Speaks first real world Beginning of labelling Speaks 4-6 words Acquisition of object and personal names Speaks 2-word sentences Beginning grammatization" "This is a real-world medical entrance exam question, please give the true answer based on the question and selection. Topic name: None, Subject name: Pharmacology","Heparin acts which of the following adjuvants? Select and explain. A. Antithrombin III, B. Protein C, C. Protein S, D. Thrombomodulin",A: Antithrombin III,"Answer: A: Antithrombin III Explanation: Ans: A. Antithrombin IIIRef: Goodman & Gilman, 13th ed, pg. 588Heparin binds to antithrombin III (AT-lll) and accelerate the rate at which it inhibits various coagulation proteases." "This is a real-world medical entrance exam question, please give the true answer based on the question and selection. Topic name: None, Subject name: Surgery","In submandibular surgery incision is given 2 cm below the border of mandible to preserve which nerve? Select and explain. A. Marginal mandibular nerve, B. Mental Nerve, C. Long buccal nerve branch of facial, D. Inferior alveolar nerve",A: Marginal mandibular nerve,"Answer: A: Marginal mandibular nerve Explanation: The mandibular branches of the facial nerve run parallel to the lower border of the mandible, mostly below it. Of them, the marginal mandibular nerve follows almost exactly the lower border. Hence, to avoid sectioning of this nerve and causing loss of motor supply to lower lip and mentalis, we give the submandibular incision at lest 2 cm of 2-finger width below the lower border of mandible." "This is a real-world medical entrance exam question, please give the true answer based on the question and selection. Topic name: None, Subject name: Pharmacology","What drug is used for prophylaxis against Pneumocystis jirovecii in patients on chemotherapy? Select and explain. A. Cotrimoxazole, B. Amoxicillin, C. Dexamethasone, D. Cephalosporin",A: Cotrimoxazole,"Answer: A: Cotrimoxazole Explanation: Answer- A. CotrimoxazoleDrug of choice for prophylaxis and treatment of pneumocystis infection in both immunocompetent as well as immunocompromised is cotrimoxazole.Cyclospora, Isospora & Pneumocystis jiroveci - Cotrimoxazole" "This is a real-world medical entrance exam question, please give the true answer based on the question and selection. Topic name: None, Subject name: Medicine","Which disease affects neurons only Select and explain. A. Spinocerebellar ataxia, B. Supranuclear palsy, C. Corticobasillar degeneration, D. Multiple system atrophy",A: Spinocerebellar ataxia,"Answer: A: Spinocerebellar ataxia Explanation: None" "This is a real-world medical entrance exam question, please give the true answer based on the question and selection. Topic name: None, Subject name: Dental","Electrical conductivity speed of A α fibres ? Select and explain. A. 70-120, B. 50-70, C. 30-51, D. 20-Oct",A: 70-120,"Answer: A: 70-120 Explanation: None" "This is a real-world medical entrance exam question, please give the true answer based on the question and selection. Topic name: None, Subject name: Medicine","Raised serum alkaline phosphatase is seen in all except: Select and explain. A. Pagets disease, B. Multiple myeloma, C. Osteomalacia, D. Hyperthyroidism",B: Multiple myeloma,"Answer: B: Multiple myeloma Explanation: None" "This is a real-world medical entrance exam question, please give the true answer based on the question and selection. Topic name: None, Subject name: Gynaecology & Obstetrics","A 24 years old female presented with amenorrhea for 3 months. LH and FSH levels are elevated three times the normal value. What is the next best step? Select and explain. A. Urinary HCG level, B. Check serum estradiol levels, C. Progesterone challenge test and look for withdrawal bleeding, D. Ultrasound of abdomen and pelvis",B: Check serum estradiol levels,"Answer: B: Check serum estradiol levels Explanation: Answer- B. Check serum estradiol levelsLH and FSH levels by the third montlt of pregnancy are steady st a low level. HCG takes over tlte function of LH after feilization and inhibils production on LH . Inhibin A produced by fetal trophoblasts suppresses maternal FSH secretion. Hence, 3 times elevation of LH and FSH points towards premature ovariun failure. Serum estradiol levels should be measured to confirm the diognosis" "This is a real-world medical entrance exam question, please give the true answer based on the question and selection. Topic name: None, Subject name: Medicine","A patient with tubercular meningitis was taking ATT regularly. At end of 1 month of regular intake of drugs deterioration in sensorium is noted in condition of the patient. Which of the following investions is not required on emergency evaluation ?. Select and explain. A. MRI, B. NCCT, C. CSF examination, D. Liver function tests",D: Liver function tests,"Answer: D: Liver function tests Explanation: Ans. D (Ref Harrison 19/e pi 111, ThIe p3414)When bacterial meningitis is suspected, blood cultures should be immediately obtained and empirical antimicrobial and adjunctive dexamethasone therapy initiated without delay.The diagnosis of bacterial meningitis is made by examination of the CSF. The need to obtain neuroimaging studies (CT or MRI) prior to LP requires clinical judgment." "This is a real-world medical entrance exam question, please give the true answer based on the question and selection. Topic name: None, Subject name: Pathology","A 10-year old boy was presented with a mass in abdomen. On imaging, the para-aoic lymph nodes were enlarged. On biopsy, starry sky appearance was seen. What is the underlying abnormality? Select and explain. A. p53 gene mutation, B. Rb gene mutation, C. Translocation involving BCR-ABL gene, D. Translocation involving Myc gene",D: Translocation involving Myc gene,"Answer: D: Translocation involving Myc gene Explanation: Ans: D. Translocation involving Myc geneStarry eye pattern on biopsy - Highly suggestive of Burkitt lymphoma.All forms of Burkitt lymphoma associated with translocations of c-MYC gene on chromosome 8.Burkitt lymphoma:Tumor exhibiting high mitotic index & contains numerous apoptotic cells.Nuclear remnants of apoptopic cells are phagocytosed by interspersed benign macrophages.These phagocytes with abundant clear cytoplasm, creating a characteristic ""starry sky"" pattern.""Etiology:All forms of Burkitt lymphoma are associated with translocations of c-MYC gene on chromosome S.Genetic mutation & associated conditions:TranslocationGene (Chromosome)Malignancy(9;22) (q34;q11)ABL-BCRChronic myeloid leukemia(11;14)(q13;q32)BCL1-IgHMantle cell lymphoma(8;21)RUNX1-RUNX1T1 (15;17)PML-RARAAcute myeloid leukemia(16;16)CBFB-MYH11 (11;22)(q24;q12)FLII-EWSEwing's sarcoma(8;14)(q24;q32)MYC-IgHBurkitt's lymphoma B cell acute lymphocytic leukemiaInv (2p13;p11.2-14)REL-NRGNon-Hodgkin's lymphoma(1;3)(p34;p21)TAL1-TCTAQAcute T cell leukemia(Ref: Robbins 9/e p597)" "This is a real-world medical entrance exam question, please give the true answer based on the question and selection. Topic name: None, Subject name: Dental","A child is brought to the clinic with complaint of irregular teeth. The maxillary central incisor is rotated in an otherwise normal occlusion. What should the next step be? Select and explain. A. Check for supernumerary teeth, B. Resection of supracrestal fibers, C. Exert a couple on tooth, D. Fixed orthodontic appliances given",A: Check for supernumerary teeth,"Answer: A: Check for supernumerary teeth Explanation: None" "This is a real-world medical entrance exam question, please give the true answer based on the question and selection. Topic name: None, Subject name: Gynaecology & Obstetrics","A pregnant female presents with fever. On lab investigation her Hb was decreased (7 mg%), TLC was normal and platelet count was also decreased. Peripheral smear shows fragmented RBCs. Which is least probable diagnosis? Select and explain. A. DIC, B. TTP, C. HELLP syndrome, D. Evans syndrome",D: Evans syndrome,"Answer: D: Evans syndrome Explanation: The clinical scenario of the patient shows the following signs and symptoms: Fever Anemia Thrombocytopenia Normal total leukocyte count Fragmented RBCs (Schistocytes) on peripheral smear. Now let us review each option one by one Option (a): DIC Harrison 20/e, p 979 DIC may present with sudden onset of fever (as the M/c cause of D/c is sepsis) Excessive bleeding may lead to anemia Platelet consumption may lead to thrombocytopenia Leukocyte count is not affected Intravascular microangiopathic hemolysis can lead to schistocytes on peripheral smear. Williams Obs 23/e, p 786 Option (b): TTP i.e Thrombotic thrombocytopenic purpura. TTP presents with a pentad of: Fever Microangiopathic haemolytic anemia, leading to anemia and fragmentation of RBCs Thrombocytopenia Neurologic symptoms Renal failure. Option (c): HELLP syndrome HELLP syndrome presents with the combination of: Hemolysis because of which fragmented RBC’s may be seen Elevated liver enzymes and Low platelet count Fever may or may not be present. Oprion (d): Evans syndrome Hoffman: Hematology: Basic Principle and Practice, 5/e Evans syndrome is an autoimmune disease in which an individual’s antibodies attack their own red blood cells and platelets. Its overall pathology resembles a combination of autoimmune haemolytic anemia and idiopathic thrombocytopenic purpura. Autoimmune hemolysis leads to the formation of spherocytes and not schistocytes. Schistocytes are fragmented RBCs that are the result of microangiopathic hemolysis. Autoiminune destruction of RBCs leads to the formation of spherocytes. Hence, Evans syndrome is the least likely possibility in this clinical scenario." "This is a real-world medical entrance exam question, please give the true answer based on the question and selection. Topic name: None, Subject name: Dental","Coefficient of thermal expansion of which of following is most similar to that of tooth? Select and explain. A. Gold inlay, B. Acrylic resin, C. Silicate cement, D. Gold foil",C: Silicate cement,"Answer: C: Silicate cement Explanation: None" "This is a real-world medical entrance exam question, please give the true answer based on the question and selection. Topic name: None, Subject name: Pediatrics","A baby is born with meconium stained liquor which of the following is taken account of in terming a baby vigorous except – Select and explain. A. Tone, B. Colour, C. HR, D. Respiration",B: Colour,"Answer: B: Colour Explanation: Resuscitation of neonate born through meconium-stained liquer (MSL) When baby passes meconium in utero, there is a chance that the mecomium will be aspirated into infant's mouth and potentially into the trachea and lungs. Appropriate steps must be taken immediately after delivery to reduce the risk of serious consequences resulting from aspiration of meconium. Intrapartum nasopharyngeal suctioning just after the delivery of head is no longer recommended as it does not reduce the risk of meconium aspiration syndrome and, on rare occasions, may cause nasophagngeal trauma or a cardiac arrhythmia. The first step after delivery is to identify whether the newborn is vigrous or non-vigrous : - A) Vigrous newborn A newborn is classified as vigrous, if he has all the three signs are present : - Strong respiratory effort Good muscle tone Heart rate greater than 100 The vigrous child does not require any tracheal suctioning and the usual initial steps of resuscitation are provided, i.e., provide warniith, positioning, suctioning of mouth and nose (not tracheal suctioning); Dry, stimulate and 02 if necessary. B) Non-vigrous newborn If any of the above three signs is present, the newborn is classified as non-vigrous. For non-vigrous child, the initial steps are modified : - Place the baby under radiant wanner and postpone suctioning to prevent stimulation of posterior pharyngeal wall that can cause bradycardia. Residual meconium in the mouth and posterior pharynx should be removed by suctioning under direct vision using a laryngoscope. The trachea should then be intubated and mechonium suctioned from the lower airway. Tracheal suctioning is best done by applying suction directly to the endotracheal tube. After providing initial steps, the further management is same as with resucitation for other conditions." "This is a real-world medical entrance exam question, please give the true answer based on the question and selection. Topic name: None, Subject name: Dental","Guiding planes help in: Select and explain. A. For better clasp prediction, B. Vertical to occlusal plane, C. Balanced occlusion, D. Avoid precision attachment",A: For better clasp prediction,"Answer: A: For better clasp prediction Explanation: None" "This is a real-world medical entrance exam question, please give the true answer based on the question and selection. Topic name: None, Subject name: Anatomy","Which of the following arteries does not supply the circle of Willis? Select and explain. A. Anterior cerebral, B. Middle cerebral, C. Posterior-inferior cerebral, D. Posterior communicating",C: Posterior-inferior cerebral,"Answer: C: Posterior-inferior cerebral Explanation: None" "This is a real-world medical entrance exam question, please give the true answer based on the question and selection. Topic name: AIIMS 2018, Subject name: Anatomy","Which of the following muscle do NOT work for inversion of foot? Select and explain. A. Extensor hallucis longus, B. Tibialis anterior, C. Tibialis posterior, D. Peroneus longus",D: Peroneus longus,"Answer: D: Peroneus longus Explanation: Movement Muscle Accessory muscle INVERSION Tibialis anterior Tibialis posterior Extensor hallucis longus Flexor digitorum longus Flexor hallucis longus EVERSION Peroneus longus Peroneus brevis Peroneus teius" "This is a real-world medical entrance exam question, please give the true answer based on the question and selection. Topic name: None, Subject name: Physiology","X,Y,Z are the ee ions pet meaule..\\ and V=-30. If at resting membrane potential (RMP), when there is no net electro genic transfer, what is the value of Z? Select and explain. A. 20, B. -20, C. 80, D. -80",C: 80,"Answer: C: 80 Explanation: Ans. c. +80 Resting membrane potential (RMP) is the static state of a membrane, where the net transmembrane electric flux is zero. Non-electrogenic transfer at RMP means X+Y+Z = 0 . Since X = -50 and Y= -30, then Z must be +80 since (-80 +80 = 0)" "This is a real-world medical entrance exam question, please give the true answer based on the question and selection. Topic name: AIIMS 2017, Subject name: Pediatrics","A child presented at 18 months of age who has never been vaccinated before. Which vaccines will you administer? Select and explain. A. DPT, OPV and MMR, B. Pentavalent vaccine alone, C. BCG and OPV, D. MMR, OPV, Rotavirus","A: DPT, OPV and MMR","Answer: A: DPT, OPV and MMR Explanation: Age Vaccine At bih BCG, bOPV-0, Hep B-0 6 weeks bOPV-1, Pentavalent-1, Rotavirus-1*, fIPV-1, PCV-1* 10 weeks bOPV-2, Pentavalent-2, Rotavirus-2* 14 weeks bOPV-3, Pentavalent-3, Rotavirus-3*, fIPV-2, PCV-2* 9 months MR-1*, JE-1*, PCV-3* 16-24 months DPT-B1, bOPV-B, JE-2*, MR-2* 5-6 years DPT-B2 11-13 yrs HPV-1*, HPV-2* BCG and Pentavalent can be given only up to 1st yr. of age, as per national immunization schedule" "This is a real-world medical entrance exam question, please give the true answer based on the question and selection. Topic name: None, Subject name: Surgery","For propofol all are true except Select and explain. A. Has a rapid recovery rate, B. Used for induction & maintenance of anesthesia, C. Causes vomiting after use, D. Causes sedation",C: Causes vomiting after use,"Answer: C: Causes vomiting after use Explanation: None" "This is a real-world medical entrance exam question, please give the true answer based on the question and selection. Topic name: None, Subject name: Surgery","Following a blunt trauma abdomen, a patient had renal laceration and urinoma. Even after 12 days, urinoma persisted, but the patient was stable and there was no fever. Next step in management would be: Select and explain. A. Percutaneous exploration and repair, B. Wait and watch, C. J-shaped urinary stent, D. Percutaneous nephrostomy",C: J-shaped urinary stent,"Answer: C: J-shaped urinary stent Explanation: Ans. c. J-shaped urinary stentManagement of urinoma is by endoscopic intervention, with cystoscopy, retrograde pyelography, placement of a ureteral stent, urethral catheter drainage, and intravenous antibiotics.""Although most post-traumatic urinomas are asymptomatic and have a spontaneous resolution rate approaching 85%, urinomas will occasionally persist.A small amount of urinary extravasation is usually not significant as long as they do not become infected.Initially, we treat the trauma by ureteric stentingOnly for large urinoma, we drain by percutaneous drainage." "This is a real-world medical entrance exam question, please give the true answer based on the question and selection. Topic name: None, Subject name: Pathology","Which of the following is unlikely to cause enamel hypoplasia? Select and explain. A. Rickets, B. Fluoride, C. Congenital syphilis, D. Cleidocranial dysostosis",D: Cleidocranial dysostosis,"Answer: D: Cleidocranial dysostosis Explanation: None" "This is a real-world medical entrance exam question, please give the true answer based on the question and selection. Topic name: None, Subject name: Biochemistry","Thiamine deficiency decreases cellular metabolism because: Select and explain. A. Thiamine is a coenzyme for pyruvate dehydrogenase and alfa-ketoglutarate dehydrogenases, B. Activity of transketolase is inhibited, C. It is required for the process of transamination, D. It is a cofactor in oxidative reduction",A: Thiamine is a coenzyme for pyruvate dehydrogenase and alfa-ketoglutarate dehydrogenases,"Answer: A: Thiamine is a coenzyme for pyruvate dehydrogenase and alfa-ketoglutarate dehydrogenases Explanation: Thiamine deficiency results in decreased energy production because TPP interferes with its coenzyme for pyruvate and alphaketoglutarate dehydrogenase.Chronic peripheral Neuritis, Beriberi & Wemicke Encephalopathy with Korsakoff's Psychosis.Thiamin requirements increase in excess intake of carbohydrates and its deficiency leads to decreased energy production" "This is a real-world medical entrance exam question, please give the true answer based on the question and selection. Topic name: None, Subject name: Surgery","In a bus accident, which patient is given more priority to shift to the hospital? Select and explain. A. Severe haemorrhage with leg fracture, B. Head injury, C. Circulation shock, D. Airway obstruction",A: Severe haemorrhage with leg fracture,"Answer: A: Severe haemorrhage with leg fracture Explanation: None" "This is a real-world medical entrance exam question, please give the true answer based on the question and selection. Topic name: None, Subject name: Surgery","Excessive bleeding during oral surgery can be decreased in which position? Select and explain. A. Head up, B. Head down, C. Prone, D. Supine",A: Head up,"Answer: A: Head up Explanation: None" "This is a real-world medical entrance exam question, please give the true answer based on the question and selection. Topic name: None, Subject name: Dental","Incidence of 2 canals in mandibular incisors is Select and explain. A. 3 - 12%, B. 12 - 20%, C. 20 - 41%, D. Less than 3%",C: 20 - 41%,"Answer: C: 20 - 41% Explanation: None" "This is a real-world medical entrance exam question, please give the true answer based on the question and selection. Topic name: AIIMS 2019, Subject name: Orthopaedics","What is the use of the given instrument: Select and explain. A. To hold bone and plate, B. To hold bone fragments with traction, C. To cut the bone, D. For creating a gap in wound",A: To hold bone and plate,"Answer: A: To hold bone and plate Explanation: For creating a gap in wound Bone Cutter Bone Nibbler Double Action Bone Holding Forceps Bone Plate Holding Forceps Fergusson Bone Holding Forceps Lane Bone Holding Forceps Dynamic Compression Plate (DCP) Locking Compression Plate (LCP) Limited Contact Dynamic CompressionPlate (LCDCP) Screws Osteotome Bone Curette" "This is a real-world medical entrance exam question, please give the true answer based on the question and selection. Topic name: None, Subject name: Pediatrics","Asphyxial injury in a term baby is characterized by all except – Select and explain. A. Seizures, B. Differential hypotonia (lower limbs > upper limbs), C. Altered sensorium, D. Difficulty in clearing oral secretions",B: Differential hypotonia (lower limbs > upper limbs),"Answer: B: Differential hypotonia (lower limbs > upper limbs) Explanation: Effects ofAsphaxia : Central nervous system-                                                            Cardiovascular Hypoxic ischemic encephalopathy                                        o Myocardial ischemia Infarction                                                                                    o Poor contractility Intracranial hemorrhage                                                           o Tricuspid insufficiency Seizures                                                                                        o Hypotension Cerebral edema                                                                    o Pulmonary Hypotonia                                                                                   o Pulmonary hypertension Hypertonia                                                                                  o Pulmonary hemorrhage                                                                                              o Respiratory distress syndrome   Renal                            —> Acute tubular or cortical necrosis Adrenal                        —> Adrenal hemorrhage Metabolic                    --> SIADH, Hyponatremia, Hypoglycemia, Hypocalcemia, Myoglobinuria Integument                  -+ Subcutaneous fat necrosis Hematology                -p DIC Gastrointestinal          —> Perforation, Ulceration with hemorrhage, Necrosis During asphyxia the infant may remain hypotonic or change from hypotonia to extreme hypertonia or their tone may appear normal. These changes are seen simultaneously and the change in muscle tone is also of the same degree in both the limbs." "This is a real-world medical entrance exam question, please give the true answer based on the question and selection. Topic name: None, Subject name: Surgery","Which of the following is not an etiological factor for pancreatitis? Select and explain. A. Abdominal trauma, B. Hyperlipidemia, C. Islet cell hyperplasia, D. Germline mutations in the cationic trypsinogen gene",C: Islet cell hyperplasia,"Answer: C: Islet cell hyperplasia Explanation: Ans. c. Islet cell hyperplasiaGallstones including microlithiasis (MC). Alcohol (2""d MC). Hyperiglyceridemia. ERCPO. Blunt abdominal trauma" "This is a real-world medical entrance exam question, please give the true answer based on the question and selection. Topic name: None, Subject name: Surgery","Percentage of SSI rate in patients with a clean contaminated wound? Select and explain. A. 1-2%, B. <10%, C. 10- 20 %, D. 20- 30 %",B: <10%,"Answer: B: <10% Explanation: Answer- B. <10%Clean wound- 1- 2%Clean contaminated- 7- 10%Contaminated- 10- 20%Diy- >20%class I (clean):2%class II (clean contaminated):5-15%class III (contaminated):>15%class IV (diy):>30%" "This is a real-world medical entrance exam question, please give the true answer based on the question and selection. Topic name: None, Subject name: Surgery","All of the following are seen in MEN-1 syndrome except: Select and explain. A. Posterior pituitary tumors, B. Pancreatic neuroendocrine tumors, C. Foregut carcinoid, D. Parathyroid hyperplasia",A: Posterior pituitary tumors,"Answer: A: Posterior pituitary tumors Explanation: Ans: A. Posterior pituitary tumors(Ref Bailey 27/e p856-858, 26/e p795-797; Sabiston 20/e p993, 1003,- Harrison 19/e p2335)MEN type I (MEN 1):Also referred ""Wermer's syndrome"".Characterized by triad of tumors involving parathyroids, pancreatic islets & anterior pituitary (not the posterior pituitary tumors).Additionally adrenal coical tumors, carcinoid tumors usually of foregut, meningiomnas, facial angiofibromas, collagenomas, and lipomas also occur.Affects all age groups (age range 5 to 81 years).Clinical & biochemical manifestations developing mainly in 5th decade.In absence of treatment, endocrine tumors are associated with an earlier moality.Cause of death - Malignant tumor (pancreatic neuroendocrine tumor (NET) or foregut carcinoid)." "This is a real-world medical entrance exam question, please give the true answer based on the question and selection. Topic name: None, Subject name: Pharmacology","Storage of drug in the tissues is suggested by: Select and explain. A. Large volume of distribution, B. Small volume of distribution, C. Excretion in urine, D. Excretion in saliva",A: Large volume of distribution,"Answer: A: Large volume of distribution Explanation: Answer- A. Large volume of distributionVolume of distribution (VD) is the theoretical volume that would be necessary to contain the total amount of an administered drug at the same concentration, which is observed in the blood plasma.The VD of a drug represents the degree to which a drug is distributed in body tissue rather than the plasma.VD is directly correlated with the amount of drug distributed into tissue; a higher VD indicates a greater amount of tissue distribution.A VD greater than the total volume of body water (approximately 42 liters in humans) is possible, and would indicate that the drug is highly distributed into tissue" "This is a real-world medical entrance exam question, please give the true answer based on the question and selection. Topic name: None, Subject name: Surgery","Which one of the following does not classify as locally advanced breast cancer? Select and explain. A. Tumour more than 4cm, B. Inflammatory breast cancer, C. Chest wall involvement, D. Skin involvement",A: Tumour more than 4cm,"Answer: A: Tumour more than 4cm Explanation: Answer- A. Tumour more than 4cmLocally advanced breast cancer-T4 N2 N3 M0" "This is a real-world medical entrance exam question, please give the true answer based on the question and selection. Topic name: None, Subject name: Pathology","Robinson's classification of ameloblastoma does not include: Select and explain. A. Multicentric, B. Non-functional, C. Anatomically benign, D. Clinically persistent",A: Multicentric,"Answer: A: Multicentric Explanation: None" "This is a real-world medical entrance exam question, please give the true answer based on the question and selection. Topic name: None, Subject name: Dental","In Parkinsonism patient, teeth arrangement is Select and explain. A. Anatomic, B. Non anatomic, C. Semi anatomic, D. Saucer teeth",B: Non anatomic,"Answer: B: Non anatomic Explanation: None" "This is a real-world medical entrance exam question, please give the true answer based on the question and selection. Topic name: AIIMS 2018, Subject name: Gynaecology & Obstetrics","Most useful test in vesicovaginal fistula? Select and explain. A. Cystoscopy, B. Three swab test, C. Urine culture, D. Intravenous pyelogram",A: Cystoscopy,"Answer: A: Cystoscopy Explanation: Best investigation for VVF : Cystoscopy Cystoscopy tells about Site of fistula Size of fistula No. of fistula Ureteric opening Tests done for diagnosis of VVF: Clinical examination for site of fistula and type of lak Three swab test: methylene blue dye test Cystoscopy Cystography VCUG: voiding cystourethrography CT, urine c/s, IVP, USG Retrograde pyelography; may be done to rule out other causes of urine incontinence" "This is a real-world medical entrance exam question, please give the true answer based on the question and selection. Topic name: None, Subject name: Forensic Medicine","""First hand knowledge"" refers to Select and explain. A. Opinion of a doctor in cou, B. Handwriting expe, C. Common witness, D. Fingerprint expe",C: Common witness,"Answer: C: Common witness Explanation: Ans: C. Common witness""Except common witness all other options are expe witnesses"".Only the common witness can be considered as having first-hand knowledge in the cou of law.In the cou procedures, this phrase refers to the personal understanding of circumstances and events, which is re-told and explained by a witness in a couroom, during a cou hearing on the concrete lawsuit.In other words, the witness has been an eye-witness of these circumstances or events and so has accepted directly. Therefore they have clear and detailed personal knowledge of it. An example is a statement in affidavits." "This is a real-world medical entrance exam question, please give the true answer based on the question and selection. Topic name: None, Subject name: Dental","The first radiographic sign of chronic pulpal infection of primary molars is: Select and explain. A. Periapical bony changes, B. Root resorption, C. Changes in bony furcation area, D. Widening of apical periodontal ligament",C: Changes in bony furcation area,"Answer: C: Changes in bony furcation area Explanation: None" "This is a real-world medical entrance exam question, please give the true answer based on the question and selection. Topic name: AIIMS 2017, Subject name: Medicine","First line therapy in anaphylactic shock is: Select and explain. A. Adenosine 12 mg intravenously, B. Atropine 3 mg intravenously, C. Epinephrine .5 ml of 1:1000 IM, D. Epinephrine 1 ml of 1:10000 intravenously",C: Epinephrine .5 ml of 1:1000 IM,"Answer: C: Epinephrine .5 ml of 1:1000 IM Explanation: Cause of death in anaphylactic shock is laryngeal edema. Undiluted adrenaline / epinephrine IM 0.5 ml of 1:1000 IM in thigh Option A- For PSVT Option B- For bradycardia Option D- For cardiac arrest" "This is a real-world medical entrance exam question, please give the true answer based on the question and selection. Topic name: None, Subject name: Dental","Which of the following headgear is preferred while treating a class II div. 1 malocclusion with a horizontal growth trend? Select and explain. A. High pull head gear, B. Medium pull head gear, C. Cervical head gear, D. Reverse pull head gear",C: Cervical head gear,"Answer: C: Cervical head gear Explanation: Cervical headgears cause extrusion of maxillary molars and thus increase the lower facial height. So these are generally indicated in low mandibular angle cases (horizontal growers) as an increase in lower facial height would be beneficial in these patients." "This is a real-world medical entrance exam question, please give the true answer based on the question and selection. Topic name: AIIMS 2019, Subject name: Medicine","What is the cause of delirium tremens in alcoholics? Select and explain. A. Fatty liver, B. Abrupt cessation of heavy and prolonged consumption of alcohol, C. Gradual withdrawal of alcohol, D. Small doses of consumption",B: Abrupt cessation of heavy and prolonged consumption of alcohol,"Answer: B: Abrupt cessation of heavy and prolonged consumption of alcohol Explanation: Alcohol Withdrawal Syndrome and Delirium Tremens Alcohol withdrawal syndrome typically occurs within 1-2 days of alcohol abstinence and is characterized by autonomic hyperactivity, tremors, insomnia, nausea and vomiting, transient hallucinations, psychomotor agitation, anxiety, and tonic-clonic seizures. Symptoms may be mild for many patients, but if left untreated, it can lead to delirium tremens, an uncommon but severe and life-threatening manifestation of withdrawal marked by severe hallucinations, disorientation, and autonomic overactivity. Symptoms and signs include the following: Profoundly delirious state associated with tremulousness and agitation. Excessive motor activity (most notable as a tremor that affects the face, tongue, and extremities but that may also involve speech) and purposeless activity such as picking at the bedclothes. Hallucinations, classically visual rather than auditory, are a prominent feature. Autonomic nervous system hyperactivity: tachycardia, dilated pupils, fever, and hyperhidrosis. Loss of orientation as to time and place. Such patients are often oblivious to the most obvious features of the surrounding environment. Treatment Note temperature, pulse, and blood pressure, and record results twice hourly to monitor for hyperpyrexia and hypotension. Consider lumbar puncture to rule out meningitis if fever or meningismus is present. 1. FLUIDS Fluid requirements on the first day of treatment may be as if profound dehydration is present. Intravenous fluid should contain glucose to prevent hypoglycemia. 2. THIAMINE Thiamine, 100 mg/d, prevents Wernicke encephalopathy. 3. BENZODIAZEPINES Benzodiazepines are the primary form of treatment. They may also prevent patients with impending delirium tremens from developing a full-blown case.Drug of choice-Chlordiazepoxide a - Fatty liver- irrelevant b - Its abrupt withdrawal not gradual d - Small doses of alcohol consumption will not lead to raving. Therefore, a,b,d ruled out." "This is a real-world medical entrance exam question, please give the true answer based on the question and selection. Topic name: None, Subject name: Dental","Which of the following material is not applied for root conditioning after placement of MTA? Select and explain. A. Tetracycline, B. Citric acid, C. Polyacrylic acid, D. EDTA",D: EDTA,"Answer: D: EDTA Explanation: Based on periodontal research, it would appear that if a root surface conditioning agent were to be used during periradicular surgery, EDTA might be the most appropriate solution. However, the manufacturer (personnel communication, Dr.  Torabinejad)  has  advised  against  the  use  of  EDTA  when mineral trioxide aggregate (MTA) is used as a root-end filling material,  because  it  may  interfere  with  the  hard  tissue–producing effect of MTA. Reference: Cohen Pathways of pulp 11th ed page no 416" "This is a real-world medical entrance exam question, please give the true answer based on the question and selection. Topic name: None, Subject name: Medicine","HbA2 is raised in: Select and explain. A. Beta thalassemic trait, B. Sickle cell anemia, C. Hereditary spherocytosis, D. None",A: Beta thalassemic trait,"Answer: A: Beta thalassemic trait Explanation: Failure to synthesise beta chains (beta-thalassaemia) is the most common type of thalassaemia, most prevalent in the Mediterranean area. Heterozygotes have thalassaemia minor, a condition in which there is usually mild microcytic anaemia and little or no clinical disability, which may be detected only when iron therapy for a mild microcytic anaemia fails. Homozygotes (thalassemia major) either are unable to synthesise haemoglobin A or, at best, produce very little; after the first 4–6 months of life, they develop profound transfusion-dependent hypochromic anaemia. Key Concept:  In beta thalassemia, HbA2 quantity is raised. Ref: Davidson Ed 23 Pg 47" "This is a real-world medical entrance exam question, please give the true answer based on the question and selection. Topic name: AIIMS 2017, Subject name: Psychiatry","A young male patient is on 5 mg haloperidol for many days, recently for last 4 days of duration he has inner restlessness and urges to move. Diagnosis is? Select and explain. A. Akathisia, B. Tardive dyskinesia, C. Rabbit syndrome, D. Acute Dystonia",A: Akathisia,"Answer: A: Akathisia Explanation: Antipsychotic drugs Extrapyramidal symptoms- dur to D2 blockade in limbic system Extrapyramidal symptoms Clinical features Special DOC Acute dystonia Oculogyric crisis ocular muscles spasm and opisthotonos Toicollis Protrusion of tongue leading to laryngospasm grimacing Earliest to develop (days) Anticholinergic drugs Acute akathisia Constant purposeless involuntary movement from one place to another Most common EPS Days to week Propranolol Tardive dyskinesia chewing and sucking movements grimacing choreoathetoid movements-akathisia Seen after long(years) use of anti-psychotic Tetrabenazine Malignant neuroleptic syndrome Fluctuating level of consciousness hypehermia -muscles rigidity increase level of CPK Increase level of liver enzyme confusion - Diaphoresis Most common cause of death in this syndrome is acute renal failure Most serious side effect dantrolene DRUG induced parkinsonism Few weeks anticholinergics" "This is a real-world medical entrance exam question, please give the true answer based on the question and selection. Topic name: None, Subject name: Dental","1st dental visit of the child should be at the age of: Select and explain. A. 6 months, B. 1 year, C. 2 years, D. 3 months",B: 1 year,"Answer: B: 1 year Explanation: None" "This is a real-world medical entrance exam question, please give the true answer based on the question and selection. Topic name: None, Subject name: Biochemistry","During exercise, the most rapid way to synthesize ATP is: Select and explain. A. Glycogenolysis, B. Glycolysis, C. TCA cycle, D. Creatine phosphate",D: Creatine phosphate,"Answer: D: Creatine phosphate Explanation: Ans. d. Creatine phosphate.The high group transfer potential of ATP enables it to act as a donor of high-energy phosphate compounds. There are three major sources of high phosphate compounds taking pa in energy conservation i.e. oxidative phosphorylation, glycolysis, and citric acid cycle.Phosphagens act as storage forms of group transfer potential and include creatine phosphate, which occurs in veebrate skeletal muscle, hea, spermatozoa, and brain, and arginine phosphate, which occurs in the inveebrate muscle." "This is a real-world medical entrance exam question, please give the true answer based on the question and selection. Topic name: None, Subject name: Dental","Mandibular molars show how many patterns of Accessory Canals Select and explain. A. 1, B. 2, C. 3, D. 4",C: 3,"Answer: C: 3 Explanation: None" "This is a real-world medical entrance exam question, please give the true answer based on the question and selection. Topic name: None, Subject name: Gynaecology & Obstetrics","In endometriotic lesions, histology represents its: Select and explain. A. High estrogen, B. Low insulin, C. High levels of prolactin, D. High cholesterol",A: High estrogen,"Answer: A: High estrogen Explanation: Answer- A. High estrogenDiagnosis:Laparoscopy is gold standard fbr diagnosis of endometriosis.Powder burn or matchstick spots are seell on laparoscopy.CL-125 >35 U/mL may be used as evidence of recurrence. Laparoscopic FindingsUnless disease is visible in the vagina or elsewhere, laparoscopy is the standard technique for visual inspection of pelvis and establishment of a definitive diagnosisCharacteristic findings include typical ""Powder burn or gun shot"" lesions on the serosal surface of peritoneum.In the presence of ovarian endometrioma >3cm in diameter and deeply infiltrative disease, histology should beobtained to identify endometriosis and to exclude rare instance of malignancy" "This is a real-world medical entrance exam question, please give the true answer based on the question and selection. Topic name: None, Subject name: Social & Preventive Medicine","'There has been a gradual increase in number of non-communicable disease cases as compared to previous years. This trend is called: Select and explain. A. Seasonal, B. Cyclical, C. Periodical, D. Secular",D: Secular,"Answer: D: Secular Explanation: Ans: D. Secular(Ref Parks 24/e p69, 23ie p66, 22/c p63)Secular:Trend of gradual increase in number of non- communicable disease cases as compared to previous years.Implies changes in disease occurrence over long time period (several years or decades).Maybe progressive increase/decrease in disease occurrence.Hence, long-term trend.Eg:CHD, lung cancer & DM (non-communicable disease) - Consistent upward trend in developed countries.TB, typhoid fever, diphtheria & polio - Declined trend in developed countries." "This is a real-world medical entrance exam question, please give the true answer based on the question and selection. Topic name: None, Subject name: Pathology","Hypercementosis of whole dentition is seen in Select and explain. A. Hypophosphatasia, B. Achondroplasia, C. Pagets disease, D. Fibrous dysplasia polyostotic type",C: Pagets disease,"Answer: C: Pagets disease Explanation: None" "This is a real-world medical entrance exam question, please give the true answer based on the question and selection. Topic name: None, Subject name: Dental","Acid dissolution is most common in which part of rod Select and explain. A. Periphery of head, B. Head region, C. Rod tails, D. equally",B: Head region,"Answer: B: Head region Explanation: None" "This is a real-world medical entrance exam question, please give the true answer based on the question and selection. Topic name: None, Subject name: Pathology","Intra nuclear inclusions detected during the course of herpes simplex virus infection are ratted Select and explain. A. Bacteriophages, B. Lipschutz bodies, C. Negri bodies, D. Donavan bodies",B: Lipschutz bodies,"Answer: B: Lipschutz bodies Explanation: None" "This is a real-world medical entrance exam question, please give the true answer based on the question and selection. Topic name: AIIMS 2017, Subject name: Pharmacology","A patient presented with pain in the right lower quadrant of abdomen. He has history of renal stones in right kidney. He was prescribed an opioid which is agonist at kappa receptors and antagonist at mu receptors. The likely drug given was: Select and explain. A. Pentazocine, B. Buprenorphine, C. Tramadol, D. Fentanyl",A: Pentazocine,"Answer: A: Pentazocine Explanation: Kappa agonist and mu antagonist are:Pentazocine, nalbuphine, butarphanol.They are responsible for causing dysphoria as a side effect.Buprenorphine is paial mu agonist and kappa antagonist.Tramadol and fentanyl are synthetic opioids." "This is a real-world medical entrance exam question, please give the true answer based on the question and selection. Topic name: None, Subject name: Skin","After sequential arrangement of blistering disorder staing from superficial to deep blisters, which is the deepest among these? Select and explain. A. IgA pemphigus, B. EBD, C. PV, D. Bullous pemphigoid+",C: PV,"Answer: C: PV Explanation: Ans. C. PVPemphigus vulgaris (PV) is a rare and serious (potentially life-threatening) condition that causes painful blisters to develop on the skin and lining of the mouth, nose, throat and genitals.The blisters are fragile and can easily burst open, leaving areas of raw unhealed skin that are very painful and can put you at risk of infection.H&E: Subepidermal bullaeDIF: linear IgG and C3 deposition at the basement membrane zone" "This is a real-world medical entrance exam question, please give the true answer based on the question and selection. Topic name: None, Subject name: Surgery","Specific for Ankylosing spondylitis is true? Select and explain. A. HLA, B. B27, C. Sacroileitis, D. Raised ESR",C: Sacroileitis,"Answer: C: Sacroileitis Explanation: Answer- C. Sacroileitis" "This is a real-world medical entrance exam question, please give the true answer based on the question and selection. Topic name: None, Subject name: Dental","The retrocuspid papilla is found approx 1mm below the free gingiva on the attached gingiva lingual to: Select and explain. A. Mandibular canine., B. Maxillary canine., C. Mandibular premolars, D. Maxillary premolars.",A: Mandibular canine.,"Answer: A: Mandibular canine. Explanation: None" "This is a real-world medical entrance exam question, please give the true answer based on the question and selection. Topic name: None, Subject name: Dental","Concentration of triple antibiotic paste (TAP) in treatment of revascularisation is? Select and explain. A. 1 mg, B. 0.1 mg, C. 100 mg, D. 10 mg",A: 1 mg,"Answer: A: 1 mg Explanation: None" "This is a real-world medical entrance exam question, please give the true answer based on the question and selection. Topic name: None, Subject name: Pathology","Antigen-presenting cells are all except: Select and explain. A. M-cells, B. Macrophages, C. Langerhans cells, D. Thymocytes",D: Thymocytes,"Answer: D: Thymocytes Explanation: Ans: D. Thymocytes(Ref Robbins 9/e p625, 788)Immature T lymphocytes of thymus are called thymocytes. Thymic epithelial cells are antigen presenting cells (Non!professional) but not thymocytes.""M-cells or microfold cells are a pa of GALT (gut associated lymphoid tissue) and an antigen presenting cells in intestine.""Classical APCs include dendritic cells, macrophages, Langerhans cells and B cells." "This is a real-world medical entrance exam question, please give the true answer based on the question and selection. Topic name: None, Subject name: Medicine","A child underwent a tonsillectomy at 6 years of age with no complications. He underwent a preoperative screening for bleeding at the age of 12 years before an elective laparotomy, and was found to have a prolonged partial thromboplastin time, but normal prothrombin time. There was no family history of bleeding. The patient is likely to have acquired Vitamin K deficiency: Select and explain. A. Acquired Vitamin K deficiency, B. Acquired liver disease, C. Factor XII deficiency, D. Mild hemophilia A",C: Factor XII deficiency,"Answer: C: Factor XII deficiency Explanation: None" "This is a real-world medical entrance exam question, please give the true answer based on the question and selection. Topic name: AIIMS 2019, Subject name: Ophthalmology","All are drugs which lower IOP except Select and explain. A. Clonidine, B. Mannitol, C. Dexamethasone, D. Acetazolamide",C: Dexamethasone,"Answer: C: Dexamethasone Explanation: Drugs which lower Intra Ocular presuure are : 1 CLONIDINE 2 MANNITOL Anti glaucoma drugs 3 ACETAZOLAMIDE But , DEXAMETHASONE is a topical steroid and raises the intra ocular pressure." "This is a real-world medical entrance exam question, please give the true answer based on the question and selection. Topic name: None, Subject name: Anatomy","False about innervation of parotid gland: Select and explain. A. Postganglionic parasympathetic fibre secretomotor, B. Preganglionic parasympathetic fibre relay in Otic ganglion, C. Preganglionic parasympathetic nerve begin in inferior petrosal nucleus, D. Sympathetic nerve are vasomotor",C: Preganglionic parasympathetic nerve begin in inferior petrosal nucleus,"Answer: C: Preganglionic parasympathetic nerve begin in inferior petrosal nucleus Explanation: Ans. C. Preganglionic parasympathetic nerve begin in inferior petrosal nucleusNerve supply:PARASYMPATHETIC:auriculo temporal nerveSYMPATHETIC SUPPLY- plexus around the external carotid aery.SENSORY NERVES: auriculotemporal nerve, except for parotid fascia & overlying skin which are innervated by Great auricular nerve (C2, C3)." "This is a real-world medical entrance exam question, please give the true answer based on the question and selection. Topic name: None, Subject name: Microbiology","A patient presented to the hospital with severe hydrophobia. You suspect rabies, obtained corneal scrapings from the patient. What test should be done on this specimen for a diagnosis of rabies? Select and explain. A. Negri bodies, B. Antibodies to rabies virus, C. -PCR for rabies virus, D. Indirect immunofluorescence",C: -PCR for rabies virus,"Answer: C: -PCR for rabies virus Explanation: Ans: C. -PCR for rabies virusRef: Ananthanarayan 10/e p536, 8/e p529: Harrison 19/e p1302).Detection of rabies virus:-PCR with genetic sequencing:Highly sensitive & specific.Accurately identify rabies genome (viral RNA) in CSF, fresh saliva samples, corneal scrapings or urine.Distinguish among rabies virus variants.Permits identification of probable infection source.Reverse transcription-polymerase chain reaction testing:Used to amplify rabies virus genome pas from fixed or unfixed brain tissue or saliva.Sequencing of amplified products allows identification of infecting virus strain." "This is a real-world medical entrance exam question, please give the true answer based on the question and selection. Topic name: None, Subject name: Pediatrics","Percentage of lactose in human milk is – Select and explain. A. 7.2gm, B. 4.5gm, C. 8.0gm, D. 6.7gm",A: 7.2gm,"Answer: A: 7.2gm Explanation: It is not exactly 7.2% but it is the closest choice." "This is a real-world medical entrance exam question, please give the true answer based on the question and selection. Topic name: None, Subject name: Dental","Juvenile periodontitis is a Select and explain. A. Definite genetic disease, B. Immunological defect, C. Neutrophil chemotactic defect, D. Neutrophil adhesion defect",C: Neutrophil chemotactic defect,"Answer: C: Neutrophil chemotactic defect Explanation: None" "This is a real-world medical entrance exam question, please give the true answer based on the question and selection. Topic name: None, Subject name: Medicine","In a post operative intensive care unit, five patients developed post-operative wound infection on the same wound. The best method to prevent cross infection occurring in other patients in the same ward is to: Select and explain. A. Give antibiotics to all other patients in the ward, B. Fumigate the ward, C. Disinfect the ward with sodium hypochlorite, D. Practice proper hand washing",D: Practice proper hand washing,"Answer: D: Practice proper hand washing Explanation: None" "This is a real-world medical entrance exam question, please give the true answer based on the question and selection. Topic name: None, Subject name: Dental","Caries tetralogy by Newburn includes the fourth factor, which is: Select and explain. A. Temperature., B. Time., C. Frequency., D. Substrate.",B: Time.,"Answer: B: Time. Explanation: None" "This is a real-world medical entrance exam question, please give the true answer based on the question and selection. Topic name: None, Subject name: Dental","According to Frankel's behavior rating scale, a positive child: Select and explain. A. May become uncooperative during treatment., B. Reluctant to accept treatment., C. Good rapport with the dentist., D. Show slight negativism",A: May become uncooperative during treatment.,"Answer: A: May become uncooperative during treatment. Explanation: None" "This is a real-world medical entrance exam question, please give the true answer based on the question and selection. Topic name: None, Subject name: Dental","Masseter is a strong muscle due to: Select and explain. A. Multipennate arrangement of fibers, B. Bipennate arrangement of fibers, C. Random arrangement of fibers, D. Circumpennate arrangement of fibers",A: Multipennate arrangement of fibers,"Answer: A: Multipennate arrangement of fibers Explanation: Multipennate: Having the fibers arranged at multiple angles in relation to the axis of force generation. E.g., Masseter." "This is a real-world medical entrance exam question, please give the true answer based on the question and selection. Topic name: None, Subject name: Dental","Which of the following cephalometric analysis does not reveal the severity of anteroposterior jaw dysplasia? Select and explain. A. Down's analysis, B. Steiner analysis, C. Tweed's analysis, D. Wit's analysis",C: Tweed's analysis,"Answer: C: Tweed's analysis Explanation: Down's analysis provides information by which we can determine whether the individual’s pattern shows comparatively harmonious relations or not and whether dysplasia present in a person is in the facial skeleton, the dentition or in both. Steiner concluded that it might not be possible to correct dentition according to ideal norms in non-growing patients whose skeletal relations cannot be altered. The imperative here is to come up with an acceptable compromise for the dentition, one that will mask the underlying skeletal deformity as much as possible. This can be achieved using Steiner’s sticks, which allow calculations to be carried out for a particular ANB value. In addition to commonly used sagittal discrepancy indicators, that is angle ANB, sagittal maxilla mandibular relationship should be reconfirmed with other variables which include WITS appraisal, A–B plane angle and beta angle.  Key Concept: Tweed’s analysis (originally undertaken in 1954) is primarily based on inclination of the mandibular incisors to the basal bone and the latter’s association with the vertical relation of the mandible to the cranium. Tweed’s analysis is clinically oriented. It does not directly assess the jaws.   Reference: Orthodontics: Diagnosis and Management of Malocclusion and Dentofacial Deformities 3rd O P Kharbanda, pdf no 1016,997,981,2463" "This is a real-world medical entrance exam question, please give the true answer based on the question and selection. Topic name: None, Subject name: Anatomy","All of the following muscles are elevators of the mandible EXCEPT: Select and explain. A. Digastric, B. Masseter, C. Medial pterygoid, D. Temporalis",A: Digastric,"Answer: A: Digastric Explanation: None" "This is a real-world medical entrance exam question, please give the true answer based on the question and selection. Topic name: None, Subject name: Surgery","In transfacial proportions, the face Is divided into how many segments: Select and explain. A. 3, B. 5, C. 6, D. 7",B: 5,"Answer: B: 5 Explanation: None" "This is a real-world medical entrance exam question, please give the true answer based on the question and selection. Topic name: None, Subject name: Pathology","Which of the following Glomerulonephritis has Nephrotic syndrome except - Select and explain. A. FSGS, B. Post-infectious Glomerulonephritis, C. MPGN, D. Minimal Change Disease",B: Post-infectious Glomerulonephritis,"Answer: B: Post-infectious Glomerulonephritis Explanation: Answer- B. Post-infectious GlomerulonephritisFocal segmental glomerulosclerosis is characterised by a sclerosis of segments of some glomerules. It is likely to present as a nephrotic syndrome.Membranous glomerulonephritis may cause either nephrotic or a nephritic syndrome.Post-infectious glomerulonephritis- present with malaise, a slight fever, nausea and a mild nephritic syndrome of moderately increased blood pressure, gross haematuria, and smoky-brown urine.Membranoproliferative GN (MPGN), also known as mesangiocapillary glomerulonephritis- present with the nephritic syndrome, hypocomplementemia." "This is a real-world medical entrance exam question, please give the true answer based on the question and selection. Topic name: AIIMS 2019, Subject name: Microbiology","Slow growing alveolar like tumor in liver Select and explain. A. E. granulosus, B. E. multilocularis, C. Cysticercus cellulosae, D. Amoebic liver abscess",B: E. multilocularis,"Answer: B: E. multilocularis Explanation: Echinococcus multilocularis causes malignant hydatid disease in which Tumour are slow-growing And are alveoli like which is ill-defined ,slow-growing and invasive .It causes alveolar hydatidosis. Echinococcus granulosus causes hydatid disease in liver or cystic echinococcosis. Amoebic liver abscess is an extra intenstinal manifestation of Entamoeba histolytica. Cysticercus cellulosae is the larval stage of Taenia solium." "This is a real-world medical entrance exam question, please give the true answer based on the question and selection. Topic name: None, Subject name: Radiology","The usual radiographic appearance of an osteosarcoma is: Select and explain. A. Discrete radiolucency with regular borders, B. Multicystic radiolucency with a soap bubble appearance with an irregular peripheral border, C. Sunburst pattern with radiopaque strands extending from the cortical plates, D. Cotton wool appearance with an irregular peripheral border",C: Sunburst pattern with radiopaque strands extending from the cortical plates,"Answer: C: Sunburst pattern with radiopaque strands extending from the cortical plates Explanation: None" "This is a real-world medical entrance exam question, please give the true answer based on the question and selection. Topic name: None, Subject name: Dental","Direction of force by class III elastic in face mask? Select and explain. A. 15-20 degree downward from occlusion, B. 15-20 degree upward from occlusion, C. 35-40 deg downward from occlusion, D. 5-10 deg downword from occlusion",A: 15-20 degree downward from occlusion,"Answer: A: 15-20 degree downward from occlusion Explanation: Biomechanical Considerations The maxilla can be advanced 2-4 mm forward over a period of 8-12 months. The amount of maxillary movement is influenced by a number of factors like: Amount of force: Successful maxillary protraction can be brought about by 300-500 gm of force per side in the primary or mixed dentition. Direction of force: According to most authors, a 15- 20° downward pull to the occlusal plane is required to produce forward maxillary movement. In most cases of maxillary deficiency, maxilla is deficient in the vertical plane as well, therefore, a slight downward, direction of force is usually desirable. The line of force passes below the center of resistance of the maxilla producing a counter-clockwise moment on the maxilla and dentition. This results in a possible extrusion of maxillary posterior teeth leading to a downward and backward rotation of the mandible. However, in patients with increased anterior facial height, downward pull is contradicted. Duration of force: A review of literature shows duration to vary between 3 and 16 months. On an average at least 8-12 months of wear is required to produce the desired effect. Frequency of use: 12-14 hrs/day. Key Concept: According to most authors, a 15- 20° downward pull to the occlusal plane is required to produce forward maxillary movement. Reference: Textbook of orthodontics 2nd ed Gurkeerat Singh page no 502" "This is a real-world medical entrance exam question, please give the true answer based on the question and selection. Topic name: None, Subject name: Gynaecology & Obstetrics","All are done in management of shoulder dystocia except: Select and explain. A. Fundal pressure, B. Mc Roberts manoeuvre, C. Suprapubic pressure, D. Woods manoeuvre",A: Fundal pressure,"Answer: A: Fundal pressure Explanation: Shoulder Dystocia: The term shoulder dystocia is used to define a wide range of difficulties encountered in the delivery of the shoulders (A head to body delivery time exceeding 60 secs defines shoulder dystocia). Risk factors: Shoulder dystocia can occur in all those conditions where fetus is too big or in case of mismanaged labour. D-Maternal diabetes O-Maternal obesity and fetal obesity, i.e macrosomia P-Post-term pregnancy A-Anencephaly, Fetal ascites Management of shoulder dystocia: Shoulder dystocia should be managed as quickly as possible as interval of time from delivery of head to delivery of body is of great importance as far as survival of baby is concerned. Management follows a sequence of steps together called as Shoulder Dystocia Drill. First line measures: Immediately after recognition of shoulder dystocia extra help should be called, in the form of midwifery assistance, an obstetrician, a paediatric resuscitation team and an anaesthetist. Maternal pushing should be discouraged, as this may lead to further impaction of the shoulder, thereby exacerbating the situation. Liberal episiotomy should be given to provide more space posteriorly. Fundal pressure should not be employed. As it is associated with an unacceptably high neonatal complication rate and may result in uterine rupture. Moderate suprapubic pressure can be applied by the assistant. McRoberts’ manoeuvre is the single most effective intervention and should the first maneouvre to be performed. The McRobert’s manoeuvre is flexion and abduction of the maternal hips, positioning the maternal thighs on her abdomen. Second line measures: They should be done only, if first line measures fail: Wood’s manoeuvre: It is progressively rotating the posterior shoulder by 180°. So that the impacted anterior shoulder is released. Third line measures—They should be done only, if second line measures fail: Cleidotomy: Fracturing the clavicle bone of the fetus Symphiostomy: Dividing the pubic symphysis of the mother Zavanelli manoeuvre: Replacing the head of the baby back into the pelvis followed by cesarean section." "This is a real-world medical entrance exam question, please give the true answer based on the question and selection. Topic name: None, Subject name: Dental","Deciduous teeth do not show fluorosis because: Select and explain. A. Placenta acts as a barrier, B. Fluoride is taken up by the fast growing skeletal tissue, C. The calcification time of deciduous teeth is less, D. All of the above",D: All of the above,"Answer: D: All of the above Explanation: Option 1: Placenta acts as a barrier. Some authors have said that the placenta acts as a complete barrier to fluoride, others have said that it is only partial. Some have said that the placenta only acts as a barrier, when there is a sudden increase in the maternal plasma fluoride level. Option 2: Fluoride is taken up by the fast-growing skeletal tissue. Fluoride’s actions on bone cells predominate as anabolic effects both in vitro and in vivo. More recently, fluoride has been shown to induce osteoclastogenesis. Option 3: The calcification time of deciduous teeth is less." "This is a real-world medical entrance exam question, please give the true answer based on the question and selection. Topic name: None, Subject name: Physiology","The temperature of body is controlled by: Select and explain. A. Medulla, B. Precentral gyrus, C. Diencephalon, D. Hypothalamus",D: Hypothalamus,"Answer: D: Hypothalamus Explanation: None" "This is a real-world medical entrance exam question, please give the true answer based on the question and selection. Topic name: None, Subject name: Dental","Among anteriors highest incidence of bifurcated root is seen in Select and explain. A. Mandibular canine, B. Maxilary canine, C. Mandibular Lateral incisor, D. Mandibular central incisor",A: Mandibular canine,"Answer: A: Mandibular canine Explanation: None" "This is a real-world medical entrance exam question, please give the true answer based on the question and selection. Topic name: None, Subject name: Dental","The tooth most commonly having bifurcated roots is the: Select and explain. A. Maxillary central incisor, B. Mandibular lateral incisor, C. Mand. central incisor, D. Mandibular premolar",D: Mandibular premolar,"Answer: D: Mandibular premolar Explanation: None" "This is a real-world medical entrance exam question, please give the true answer based on the question and selection. Topic name: None, Subject name: Anatomy","Which of the following vessels supply the anal canal? Select and explain. A. Superior rectal aery, B. Middle rectal aery, C. Inferior rectal aery, D. all of the following",D: all of the following,"Answer: D: all of the following Explanation: Ans: D. all of the above aeries supply rectal canal(Ref: Gray's 41/e p1058, 40/e p1155-1159; Manual of Total Mesorectal Excision by Moran, Richard John Heald (2013)/p40)Anal canal - Supply:By superior rectal, middle rectal, inferior rectal branches of pudendal aery.Occasionally by median sacral aery." "This is a real-world medical entrance exam question, please give the true answer based on the question and selection. Topic name: None, Subject name: Pathology","Saliva buffer system having low importance in stimulated saliva Select and explain. A. Amino acids, B. Carbonic acids, C. Phosphates, D. None",A: Amino acids,"Answer: A: Amino acids Explanation: None" "This is a real-world medical entrance exam question, please give the true answer based on the question and selection. Topic name: None, Subject name: Biochemistry","Which of the following amino acids is quickly converted to tyrosine? Select and explain. A. Arginine, B. Glycine, C. Phenylalanine, D. Leucine",C: Phenylalanine,"Answer: C: Phenylalanine Explanation: None" "This is a real-world medical entrance exam question, please give the true answer based on the question and selection. Topic name: None, Subject name: Pathology","Which of the following statement is not true about glomerular basement membrane? Select and explain. A. Type III collagen is present, B. Glomerular basement membrane is stained with PAS, C. Glomerular basement membrane acts as filtration barrier, D. Glomerular basement membrane is involved in charge dependent filtration",A: Type III collagen is present,"Answer: A: Type III collagen is present Explanation: Ans. a. Type III collagen is present Type IV collagens (not the type HI collagen) are the main components of the basement membrane, together with laminin.""The basement membrane is synthesized by contributions from the overlying epithelium and underlying mesenchymal cells, forming a flat lamellar ""chicken wire"" mesh (although labeled as a membrane; it is quite porous). The major constituents are amorphous nonfibrillar type IV collagen and lantinin."" '""Type IV collagens have long but interrupted triple-helical domains and form sheets instead of fibrils; they are the main components of the basement membrane, together with laminin" "This is a real-world medical entrance exam question, please give the true answer based on the question and selection. Topic name: None, Subject name: Biochemistry","After a point mutation, glutamic acid is replaced by valine, which leads to formation of sickle cell hemoglobin. The mobility of HbS as compared with normal hemoglobin on gel electrophoresis will be: Select and explain. A. Decreased, B. Increased, C. Dependent on HbS concentration, D. Unchanged.",A: Decreased,"Answer: A: Decreased Explanation: HbS mobility on gel electrophoresis:Decreased mobility - Compared to normal hemoglobin.Electrophoresis of hemoglobin:Obtained from lysed red blood cells.Used in sickle cell trait & disease diagnosis.Sequence of Movement: HbA2 < HbC < HhS < HbF < HbA" "This is a real-world medical entrance exam question, please give the true answer based on the question and selection. Topic name: None, Subject name: Radiology","All of the following form radiolucent stones except: Select and explain. A. Xanthine, B. Cysteine, C. Allopurinol, D. Orotic acid",B: Cysteine,"Answer: B: Cysteine Explanation: Plain radiograph  Calcium-containing stones are radiopaque:  Calcium oxalate +/- calcium phosphate  Struvite (triple phosphate) - usually opaque, but variable  Pure calcium phosphate  Lucent stones include:  Uric acid  Cystine Medication (indinavir is best known) stones  Pure matrix stones (although may have a radiodense rim or centre)  Key Point: Cysteine and Cystine are different amino acids." "This is a real-world medical entrance exam question, please give the true answer based on the question and selection. Topic name: None, Subject name: Dental","Chromium is used in stainless steel for which purpose? Select and explain. A. Corrosion resistance, B. Increases the lustre of stainless steel, C. Stabilization of the alloy, D. Increases the hardness of steel",A: Corrosion resistance,"Answer: A: Corrosion resistance Explanation: None" "This is a real-world medical entrance exam question, please give the true answer based on the question and selection. Topic name: None, Subject name: Medicine","In alpha thalassemia Select and explain. A. Excess alpha chain, B. No alpha chain, C. Excess beta chain, D. No beta chain",B: No alpha chain,"Answer: B: No alpha chain Explanation: None" "This is a real-world medical entrance exam question, please give the true answer based on the question and selection. Topic name: None, Subject name: Pediatrics","A 3 days old baby is admitted with intraventricular hemorrhage. Baby develops abdominal distention. The X-ray abdomen showed pneumatosis poalis. Stage the necrotizing enterocolitis: Select and explain. A. lb, B. 2a, C. 2b, D. 3a",C: 2b,"Answer: C: 2b Explanation: Answer- C. 2bThe severity, radiology and management of Necrotizing enterocolitis is best exemplified by the 'Modified Bell's stagingcrileria'- According to 'Modified Bell's staging criteria'pneumatosis poalis (Presinie of gas in pofiai vein) is suggestiveof stage 2b." "This is a real-world medical entrance exam question, please give the true answer based on the question and selection. Topic name: None, Subject name: Forensic Medicine","Aggravated penetrative sexual assault under POCSO 2012 are all except: Select and explain. A. Threatening, B. Gang, C. During communal or secular violence, D. By police officer",A: Threatening,"Answer: A: Threatening Explanation: Ans. A. ThreateningRef: KSN Reddy's The Essentials of Forensic Medicine and Toxicology 33rd edn; Page no. 36As per Protection of Children from Sexual Offences (POCSO) Act, an offence is treated as ""Aggravated"" when committed by a person in a position of trust or authority of child, such as member of security forces, police officers, public servant, etc.The Protection of Children from Sexual Offences (POSCO) Act, 2012It is applicable to the whole India.It defines a child as any person below the age of 18 years and provides protection to all children <18 years from sexual abuse.Offences against children covered under this Act:Penetrative and aggravated penetrative sexual assault.Sexual and aggravated sexual assault.Sexual harassment.Using a child for prolonged purposes.It is now mandatory for police to register an FIR in all cases of child abuse.The child's medical examination can be conducted even prior to registration of an FIR. The I0 has to get the child medically examined in a government hospital or local hospital within 24 hours of receiving information about the offence with consent of the child or parent or a competent person whom the child trusts and in their presence.Some child-friendly procedure envisaged:At night no child to be detained in the police station.The statement of the child to be recorded as spoken by the child.Frequent breaks for the child during trial.Child not to be called repeatedly to testify.PunishmentOffencePunishmentPenetrative sexual assault7 years imprisonmentAggravated penetrative sexual assault10 years imprisonmentSexual assault3-5 years imprisonmentAggravated sexual assault5-7 years imprisonmentSexual harassment3 years imprisonmentUsing a child for prolonged purposes3-5 years imprisonment. For subsequent offence: 7 years imprisonment" "This is a real-world medical entrance exam question, please give the true answer based on the question and selection. Topic name: None, Subject name: Pharmacology","A 70 years old hypeensive patient with stage 5 chronic kidney disease was diagnosed recently with Type 2 diabetes mellitus. He doesn't want to take injectable insulin. Which of the following oral hypoglycemic agents will be preferred in this patient, which won't require any renal dose modification? Select and explain. A. Linagliptin, B. Repaglinide, C. Vildagliptin, D. Glimepiride",A: Linagliptin,"Answer: A: Linagliptin Explanation: Answer- A. Linagliptinlinagliptin has the ability to be safely dosed in chronic kidney disease patients. Chronic kidney disease is a major complication in type 2 diabetesLinagliptin .No dose required linagliptin for patients with renal impairment" "This is a real-world medical entrance exam question, please give the true answer based on the question and selection. Topic name: None, Subject name: Medicine","First time causative agent of AIDS was discovered in: Select and explain. A. 1976, B. 1983, C. 1994, D. 1969",B: 1983,"Answer: B: 1983 Explanation: None" "This is a real-world medical entrance exam question, please give the true answer based on the question and selection. Topic name: None, Subject name: Dental","For the calculation of the positive predictive value of a screening test, the denominator is comprised of: Select and explain. A. True+ves + False-ves, B. False+ves + True-ves, C. True+ves + False+ves, D. True+ves + True-ves",C: True+ves + False+ves,"Answer: C: True+ves + False+ves Explanation: None" "This is a real-world medical entrance exam question, please give the true answer based on the question and selection. Topic name: None, Subject name: Microbiology","Culture media of candida is: Select and explain. A. Methylene blue dextrose agar, B. Saboraud's medium, C. Pingolevin, D. All of the above",B: Saboraud's medium,"Answer: B: Saboraud's medium Explanation: None" "This is a real-world medical entrance exam question, please give the true answer based on the question and selection. Topic name: None, Subject name: Surgery","A tender swelling in submandibular triangle is mostly likely diagnosed as Select and explain. A. Lymphadenopathy, B. Ludwig's angina, C. Phlegmon, D. None of the above",A: Lymphadenopathy,"Answer: A: Lymphadenopathy Explanation: None" "This is a real-world medical entrance exam question, please give the true answer based on the question and selection. Topic name: None, Subject name: Medicine","A 25 years old unidentified male from roadside was brought by police to emergency room with disorientation, altered sensorium and vomiting. He had a BP of 90/70 mm Hg, hea rate of 110/min, temperature -36.4degC and respiratory rate of 11/min. On examination, he had bilateral pin-point pupils. What is the most probable diagnosis? Select and explain. A. Pontine hemorrhage, B. Hypothermia, C. Dhatura poisonine, D. Opioid poisoning",D: Opioid poisoning,"Answer: D: Opioid poisoning Explanation: Ans: D. Opioid poisoning(Ref: Harrison 19/e p 1774, 2583, 18/e p3295)Findings in question represents hallmark sign of opiate overdose.Opioid overdose triad:Decreased level of consciousness + Pinpoint pupils + Respiratory depression. .Other Signs & symptoms:Hypotension, Seizures & muscle spasms.All secretions are suspended except sweat.Treatment:Specific opioid antagonist - Naloxone.Effective at reversing the cause, rather than just the symptoms, of an opioid overdose)" "This is a real-world medical entrance exam question, please give the true answer based on the question and selection. Topic name: None, Subject name: Pathology","All the following markers are expressed on the surface of T-cells at some stage of development except: Select and explain. A. CDIa, B. PAX5, C. CD 34, D. Tdt",B: PAX5,"Answer: B: PAX5 Explanation: Answer- B. PAX5PAXS is a B-cell transcription factor, essential in normal B-cell lymphopoiesis. T-cell never express PAXS throughout its development CD-1a, CD34 and Tdt are expressed on T-cells.T cell - CDl, CD2, CD3, CD4, CD5, CD7, CD8, cCD3, lymphoid TdT." "This is a real-world medical entrance exam question, please give the true answer based on the question and selection. Topic name: None, Subject name: ENT","Which of the following vessel is not ligated in case of epistaxis control? Select and explain. A. Maxillary aery, B. Anterior ethmoidal aery, C. Internal carotid aery, D. External carotid aery",C: Internal carotid aery,"Answer: C: Internal carotid aery Explanation: Answer- C. Internal carotid aerySurgical ligation of vessels or embolization is the last reso to control epistaxis when all other methods fail. Internal carotid aery is not ligated in case of epistaxis control, as it is the msinvessel supplying the central nervous system.""Endoscopy identifies the source of posterior epistaxis in over 80 percent of the cases. Ligation should be performed as close as possible to the likely bleeding point. Thus the hierarchy of ligation is: Sphenopalatine aery; internal maxillary aery; external carotid aery; anterior/posterior ethmoidal aery." "This is a real-world medical entrance exam question, please give the true answer based on the question and selection. Topic name: None, Subject name: Ophthalmology","A young adult presents 2 days after trauma to the eye with proptosis and pain in the right eye. On examination, hi is found to have a bruise on the right eye and forehead. The most likely diagnosis is: Select and explain. A. Fracture sphenoid bone, B. Cavernous sinus thrombosis, C. Internal carotid aery aneurysm, D. Carotico-cavernous fistula",D: Carotico-cavernous fistula,"Answer: D: Carotico-cavernous fistula Explanation: Ans. d. Carotico-cavernous fistula (Ref KANSKI7/E PG94; PARSON 20/E P455,462)The most likely diagnosis in a young adult with history of trauma to the eye with proptosis and pain in the right eye and bruise on the right eye and forehead is Carotico-cavernous fistula" "This is a real-world medical entrance exam question, please give the true answer based on the question and selection. Topic name: None, Subject name: Gynaecology & Obstetrics","What is the drug of choice for precocious pubey in girls? Select and explain. A. GnRH analogues, B. Cyproterone acetate, C. Danazol, D. Medroxyprogesterone acetate",A: GnRH analogues,"Answer: A: GnRH analogues Explanation: Ans: A. GnRH analogues(Ref- .Shaw 16/e p59).Precocious pubey - Treatment:Depends on cause.Primary treatment goal:Enable child to grow to a normal adult height.GnRH & Medroxy progesterone acetate.GnRH agonists:Doc regardless of cause.DOC for halting premature sexual development.Effective in children with organic brain lesions causing central precocious pubey.Effective in identification of hypothalamic hamaoma (precocious pubey - only manifestation)" "This is a real-world medical entrance exam question, please give the true answer based on the question and selection. Topic name: AIIMS 2019, Subject name: Medicine","True about Postural Hypotension: Select and explain. A. Decreases in systolic blood pressure 20 mmHg within 6 mins, B. Decreases in systolic blood pressure 20 mmHg within 3 mins, C. Decreases in diastolic blood pressure 20 mmHg within 6 mins, D. Decreases in diastolic blood pressure 20 mmHg within 3 mins",B: Decreases in systolic blood pressure 20 mmHg within 3 mins,"Answer: B: Decreases in systolic blood pressure 20 mmHg within 3 mins Explanation: Postural Hypotension: If SBP | - >20mm Hg within 3 mins DBP| - >10mmHg when supine to standing If fall in SBP>20, DBP>10 in 3 min duration. Mx: - Salt intake | - Volume expansion - MIDODRINE (DOC) - alpha1 agonist - vasoconstriction" "This is a real-world medical entrance exam question, please give the true answer based on the question and selection. Topic name: None, Subject name: Ophthalmology","Intraocular pressure in pregnancy Select and explain. A. Increases, B. Decreases, C. Remains same, D. None",B: Decreases,"Answer: B: Decreases Explanation: Answer: B. DecreasesPregnancy, paicularly the second half, is associated with decreased IOP in healthy eyes." "This is a real-world medical entrance exam question, please give the true answer based on the question and selection. Topic name: None, Subject name: Anatomy","The action of digastric muscle is: Select and explain. A. Depression of mandible, B. Protrusion of mandible, C. Side-to-side movement of mandible, D. Depressing the floor of the mouth",A: Depression of mandible,"Answer: A: Depression of mandible Explanation: None" "This is a real-world medical entrance exam question, please give the true answer based on the question and selection. Topic name: None, Subject name: Dental","True about bicuspidization: Select and explain. A. Separation of mandibular molar mesial and distal roots with their respective crown portions, B. Separation or removal of half root with their respective crown portion in mandibular molar, C. Separation or removal of half root without their respective crown portion in mandibular molar, D. Separation or removal of half-crown without their respective root portion in mandibular molar",A: Separation of mandibular molar mesial and distal roots with their respective crown portions,"Answer: A: Separation of mandibular molar mesial and distal roots with their respective crown portions Explanation: None" "This is a real-world medical entrance exam question, please give the true answer based on the question and selection. Topic name: None, Subject name: Dental","Size of finishing diamond bur ranges from: Select and explain. A. 38-44micron., B. 20-30micron., C. 10-38 micron., D. All of the above.",C: 10-38 micron.,"Answer: C: 10-38 micron. Explanation: None" "This is a real-world medical entrance exam question, please give the true answer based on the question and selection. Topic name: None, Subject name: Dental","To remove centric interference, reduce: Select and explain. A. Supporting cusps, B. Central fossa, C. Both of the above, D. None",B: Central fossa,"Answer: B: Central fossa Explanation: In case of centric interference, supporting cusps should not be reduced as non working and working contacts are compromised." "This is a real-world medical entrance exam question, please give the true answer based on the question and selection. Topic name: AIIMS 2018, Subject name: Pharmacology","Morphine should not be used in the treatment of:- Select and explain. A. Ischemic pain, B. Biliary colic, C. Cancer pain, D. Post operative pain",B: Biliary colic,"Answer: B: Biliary colic Explanation: Morphine (an opioid) is a strong analgesic. It is used in treatment of all types of pain like Crush injury, Fracture, MI, cancer pain, post-operative pain etc. However in Biliary colic (e.g. due to any stone blocking bile duct), morphine constricts sphincter of Oddi and increase the intrabiliary pressure. This increases the chances of rupture of bile duct. Therefore opioids like morphine are contra-indicated in biliary colic." "This is a real-world medical entrance exam question, please give the true answer based on the question and selection. Topic name: None, Subject name: Social & Preventive Medicine","A study is to be conducted with regards to the fat content in the expressed breast milk of pre-term infants as compared to term infants. Which study design is best suited? Select and explain. A. Case control, B. Prospective coho, C. Longitudinal study, D. Ambispective",A: Case control,"Answer: A: Case control Explanation: Ans. A. Since there is a comparison between fat content of EBM in preterm vs term infant the best study design will be, Case control. Prospective Coho study will be appropriate if we want to see the change in the fat content of EBM in preterm infants." "This is a real-world medical entrance exam question, please give the true answer based on the question and selection. Topic name: AIIMS 2017, Subject name: Pediatrics","APGAR acronym stands for? Select and explain. A. Activity, pulse pressure, grimace, appearance, rate of respiration, B. Appearance, pressure, grimace, MAP, hea rate, C. Appearance, pressure, grimace, appearance, rate of hea beat, D. Appearance, pulse, grimace, activity, respiration","D: Appearance, pulse, grimace, activity, respiration","Answer: D: Appearance, pulse, grimace, activity, respiration Explanation: 0 (Points) 1 2 Appearance Blue or pale all over Blue extremities, but torso pink Pink all over Pulse None <100 >100 Grimace No response Weak grimace when stimulated Cries or pulls away when stimulated Activity None Some flexion of arms Arms flexed, legs resist extension Respirations None Weak, irregular or gasping Strong cry 0-3 Critically low, 4-6 Fairly Low, 7-10 Generally Normal" "This is a real-world medical entrance exam question, please give the true answer based on the question and selection. Topic name: None, Subject name: Pharmacology","Most commonly implicated drug for acute liver failure is - Select and explain. A. Paracetamol, B. Valproate, C. Warfarin, D. Tetracyclines",A: Paracetamol,"Answer: A: Paracetamol Explanation: Ans. A. Paracetamol Acute liver failure after administration of paracetamol at the maximum recommended daily dose in adults.Paracetamol is the most commonly used analgesic and antipyretic in the world; it can be bought without prescription in most countries despite being the commonest cause of acute liver failure in western Europe.Prescribing information suggests that it is safe to use in adults in divided doses that total 4 g daily.Malnutrition, starvation, chronic alcohol misuse, and concomitant use of drugs that induce cytochrome P450 enzymes increase the risk of hepatotoxicity induced by paracetamol." "This is a real-world medical entrance exam question, please give the true answer based on the question and selection. Topic name: None, Subject name: Pediatrics","A 9 year old boy has steroid dependent nephrotic syndrome for the last 5 year. The patient is markedly cushingoid with blood pressure of 120/86 mmHg and small subcapsular cataracts. The most appropriate therapy of choice is – Select and explain. A. Longterm frusemide with enalapril, B. Cyclophosphamide, C. Intravenous immunoglobulin, D. Intravenous pulse corticosteroids",B: Cyclophosphamide,"Answer: B: Cyclophosphamide Explanation: If a steroid dependent patient develops severe corticosteroid toxicity (Cushingoid toxicity, hypertension cataracts and or growth failure), then alternatives available are a) Cyclophosphamide         b) Cyclosporine                                                                                                                                                                                                                         c) Tacrolimus             d) Mycophenolate  e) Levamisole Treatment of Nephrotic syndrome" "This is a real-world medical entrance exam question, please give the true answer based on the question and selection. Topic name: None, Subject name: Pathology","Most common type of Non-Hodgkin's lymphoma in the orbit: Select and explain. A. B-cell, B. T-cell, C. NK-cell, D. Plasma cell",A: B-cell,"Answer: A: B-cell Explanation: Answer- A (B- cell)MC type of cancer of the orbit in adultsUsually a form of B-cell non-Hodgkin's lymphomaIt may show up as a nodule in the eyelid or around the eye, or it may cause the eye to be pushed out.This type of eye cancer usually does not cause pain."